You are on page 1of 1856

JC2 Prelim Paper

2016
H2 Chemistry
1 ANGLO CHINESE JUNIOR COLLEGE
2 ANDERSON JUNIOR COLLEGE
3 CATHOLIC JUNIOR COLLEGE
4 DUNMAN HIGH SCHOOL
5 HWA CHONG INSTITUTION
6 INNOVA JUNIOR COLLEGE
7 JURONG JUNIOR COLLEGE
8 MILLENNIA INSTITUTE
9 MERIDIAN JUNIOR COLLEGE
10 NATIONAL JUNIOR COLLEGE
11 NANYANG JUNIOR COLLEGE
12 PIONEER JUNIOR COLLEGE
13 RAFFLES INSTITUTION
14 RIVER VALLEY HIGH SCHOOL
15 ST. ANDREWS JUNIOR COLLEGE
16 SERANGOON JUNIOR COLLEGE
17 TAMPINES JUNIOR COLLEGE
18 TEMASEK
19 VICTORIA JUNIOR COLLEGE
20 YISHUN JUNIOR COLLEGE

trendyline
1 trendyline
1

ANGLO-CHINESE JUNIOR COLLEGE


DEPARTMENT OF CHEMISTRY
Preliminary Examination

CHEMISTRY 9647/01
Higher 2

Paper 1 Multiple Choice


29 August 2016
1 hour
Additional Materials: Multiple Choice Answer Sheet
Data Booklet

READ THESE INSTRUCTIONS FIRST

Write in soft pencil.


Do not use staples, paper clips, glue or correction fluids.
Write your name, index number and tutorial class on the Answer Sheet in the spaces provided unless this
has been done for you.

There are forty questions on this paper. Answer all questions. For each question there are four possible
answers A, B, C and D.
Choose the one you consider correct and record your choice in soft pencil on the separate Answer Sheet.

Read the instructions on the Answer Sheet very carefully.

Each correct answer will score one mark. A mark will not be deducted for a wrong answer.
Any rough working should be done in this booklet.
The use of an approved scientific calculator is expected, where appropriate.

9647/01/Prelim/16
ACJC 2016
trendyline
This
is document consists of 17 printed pages and
Thi
ANGLO CHINESE JUNIOR
ANGLO-CHINESE NIO COLLEGE
Department of Chemistry
a 1 blank page.
pag
pa

[Turn over

ACJC 2016 9647/01/Prelim/2016 [Turn over


2 trendyline
2

Section A

For each question there are four possible answers, A, B, C, and D. Choose the one you consider
to be correct.

1 3.00 g of an impure sample of calcium carbonate was heated strongly until there was no
change in mass. 200 cm3 of carbon dioxide, measured at room conditions, was obtained.

What is the percentage purity of calcium carbonate?

A 27.8 % B 29.8 % C 30.4 % D 35.8 %

2 Use of the Data Booklet is relevant to this question.

Which underlined element in the following species has the +6 oxidation number?

A ReOCl4
B H2P2O72
C TiCl4
D SbCl52

3 Use of the Data Booklet is relevant to this question.

Deuterium, 21D, is an isotope of hydrogen.

Which of the following species has more electrons than protons, and more neutrons than
protons?

A D3O+ B ND4+
C OD D AlD4

4 In which of the following pairs does the first compound have a higher melting point than the
second compound?

first compound second compound

A Mg3N2 Al2O3

B NH2OH H2O

C CH3OCH3 (CH3)2NH

D
tren
trendyline
tr nd
dy
dyylin
n
ACJC 2016 9647/01/Prelim/2016 [Turn over
3 trendyline
3

5 Which of the following bonds is not present in the following molecule?

escitalopram

A a bond formed by sp3sp2 overlap between two C atoms


B a bond formed by spsp overlap between C and N atoms
C a bond formed by sp2sp2 overlap between two C atoms
D a bond formed by sp3sp2 overlap between C and N atoms

6 Which of the following sets of compounds and ions is arranged in increasing bond angles?

A NO2+ < CO32 < SO42


B IF4 < N2O4 < N3
C ICl2 < H2S < NO2
D SCl2 < C2H2 < CH2Cl2

7 0.50 g of a volatile liquid was placed in a syringe with a frictionless piston. The syringe was
slowly warmed up till 91.0 oC at atmospheric pressure. The volume occupied in the syringe
was 200.0 cm3 when all the liquid was vaporised.

Assuming the vapour behaves as an ideal gas, what is the relative molecular mass of the
liquid?

A 18.7 B 74.9 C 93.5 D 149.8

trendyline
ACJC 2016 9647/01/Prelim/2016 [Turn over
4 trendyline
4

8 The enthalpy change when solid calcium chloride dissolves in water is 120 kJ mol1.
250 g of water is placed in an insulated cup containing 11.1 g of calcium chloride.

If the solution has the same specific heat capacity as liquid water, what is the rise in
temperature of the solution?

11.1120 11.1 120103


A K B K
2504.18 111 250 4.18
250 4.18 11.1120
C 3 K D K
11.1 12010 111 2504.18

9 Use of the Data Booklet is relevant to this question.

The standard half-cells of four metals Mg, P, Q and R and their respective cations in
solution were connected in pairs and the potential difference recorded.

The results obtained are as shown.

positive electrode negative electrode e.m.f. /V

P Mg +1.60

Q Mg +2.22

Q R +2.55

Which of the following is the decreasing order of reducing power for the four metals?

A R > Mg > P > Q


B R > Mg > Q > P
C Q > P > Mg > R
D P > Q > Mg > R

10 Which of the following statements is always true for a chemical reaction that has reached
chemical equilibrium?

A The yield of the product(s) is greater than 50%.


B The rate of the forward reaction is greater than the backward reaction.
C The amounts of reactants and products do not change.
D Both forward and backward reactions have stopped.

trendyline
ACJC 2016 9647/01/Prelim/2016 [Turn over
5 trendyline
5

11 The graph shows the change in pH when 0.10 mol dm3 acid is gradually added to V cm3 of
0.10 mol dm3 alkali.

Which of the following combinations could have given these results?

V alkali acid

A 10 Sr(OH)2 CH3CO2H

B 20 Ba(OH)2 HCl

C 10 Ca(OH)2 H2SO4

D 20 NaOH HO2CCO2H

12 Nitrogen oxide reacts with hydrogen gas as shown in the equation below.

2NO(g) + 2H2(g) N2(g) + 2H2O(g)

The reaction was determined to be second order with respect to NO and first order with
respect to H2. In an experiment, 2.0 mol dm3 of excess NO was used to react with H2, the
concentration of H2 decreased to 6.25% of its original value in 24 minutes.

How many minutes will it take for the concentration of H2 to decrease to 6.25% of its original
value if the experiment was repeated using an excess of 4.0 mol dm3 of NO?

A 1.5 B 4.5 C 6.0 D 9.0

trendyline
ACJC 2016 9647/01/Prelim/2016 [Turn over
6 trendyline
6

13 Figure 1 shows the melting points of eight consecutive elements whereas Figure 2 shows
the third ionisation energies of another set of eight consecutive elements, inclusive of
element Z in Figure 1.

Which of the options in Figure 2 correspond to element Z in Figure 1?

third IE / kJ mol1
melting point / OC

B
A

Z C

elements elements

Figure 1 Figure 2

14 An equimolar amount of MgCl2, PCl5 and KCl were separately added to three beakers,
each containing 50 cm3 of water.

Which option correctly shows the resultant solutions arranged in increasing pH?

A MgCl2, PCl5, KCl


B PCl5, MgCl2, KCl
C KCl, MgCl2, PCl5
D PCl5, KCl, MgCl2

15 Which of the following properties would not be expected for Group II elements or their
compounds?

A Barium sulfate has a higher temperature of decomposition than magnesium sulfate.


B Barium oxide in water produces a solution which reacts with dilute hydrochloric acid to
give a solution with a pH of approximately 7.
C Beryllium forms compounds with the least covalent character.
D Upon heating the nitrates of Group II elements, the volume of nitrogen dioxide
evolved is four times as great as the volume of oxygen.

trendyline
ACJC 2016 9647/01/Prelim/2016 [Turn over
7 trendyline
7

16 Interhalogen compounds contain atoms of two or more different halogens. The interhalogen
ICl was reacted with excess aqueous sodium hydroxide for a period of time.

What is the relative proportion of each ion present in the solution after this reaction?

Cl I IO IO3

A 1 1 0 0

B 1 0 1 0

C 1 2 3 1

D 3 2 0 1

17 Which statement about hydrogen halides is incorrect?

A The acidity of hydrogen halides increases down the group.


B The reducing power of hydrogen halides increases down the group.
C The halogens, with the exception of fluorine, react with hydrogen to form hydrogen
halides which have permanent dipole permanent dipole interactions.
D The thermal stability of hydrogen halides increases down the group due to a larger
electron cloud that is more polarisable.

18 Use of the Data Booklet is relevant to this question.

Which of the following is not true about the first row transition metals and their compounds?

A Fe(CN)63 does not oxidise I.


B Fe2(CO3)3 can be prepared by reacting FeCl3(aq) with Na2CO3(aq).
C On addition of H2O2(aq) to hot acidified K2Cr2O7(aq), a green solution is formed.
D On addition of Co(NO3)3 to water, a gas that relights a glowing splint is produced.

trendyline
ACJC 2016 9647/01/Prelim/2016 [Turn over
8 trendyline
8

19 Aklavinone is a tetracycline antibiotic.

aklavinone

Which combination of the number of chiral centres and of the number of sp2 and sp3
hybridised carbon atoms does it possess?

number of chiral centres number of sp2 hybridised number of sp3 hybridised


carbon carbon

A 3 14 8

B 3 15 7

C 5 14 8

D 5 15 7

20 Methylcyclopentane reacts with bromine in the presence of UV light.

Which statement is true about this reaction?

A A possible rate determining step for this reaction is

B There are three possible monosubstituted products.


C Methylcyclopentane will react slower with chlorine than bromine in the presence of UV
light.
D A by-product formed in this reaction is hydrogen.

trendyline
ACJC 2016 9647/01/Prelim/2016 [Turn over
9 trendyline
9

21 Which is the main product formed when one mole of 2methylbuta1,3diene is allowed to
react with one mole of HBr at room temperature for a prolonged period of time?

A B

C D

22 Which of the following could be the reagents and conditions for the three steps used in the
synthesis shown below.

step 1 step 2 step 3

concentrated HNO3 and


A CH3Cl and AlCl3 KMnO4 and H2SO4, heat
concentrated H2SO4, heat

concentrated HNO3 and


B CH3(CH2)2Cl and AlCl3 K2Cr2O7 and H2SO4, heat
concentrated H2SO4, heat

concentrated HNO3 and


C CH3Cl and AlCl3 K2Cr2O7 and H2SO4, heat
concentrated H2SO4, heat

concentrated HNO3 and


D CH3(CH2)2Cl and AlCl3 KMnO4 and H2SO4, heat
concentrated H2SO4, heat

23 A student investigated four different fuels. Each fuel was used separately to raise the
temperature of 1 dm3 of water from 20 oC to 100 oC. Each fuel undergoes complete
combustion. All other conditions were kept the same in each experiment.

Which fuel will produce the smallest amount of carbon dioxide in these experiments?

standard enthalpy change


fuel
of combustion/ kJ mol1

trendyline
n n
A ethane 1560

B ethanol 1367

C methanol
th 715
15

D propane 2220

ACJC 2016 9647/01/Prelim/2016 [Turn over


10 trendyline
10

24 The main reason that phenol is a better Brnsted acid than cyclohexanol is

A that it is a better proton donor.


B the cyclohexyl group is an electron donating group by induction, which destabilises
the anion formed during the dissociation.
C phenol is able to stabilise the anion formed during the dissociation by resonance,
where the electrons are delocalised in the phenyl ring.
D the phenyl group is an electron withdrawing group by induction, which stabilises the
anion formed during the dissociation.

25 Propanone reacts with methylamine to form an imine as shown in the equation.

Similarly, propanone also reacts with hydroxylamine, NH2OH, to form an oxime.

Which of the following best describes the type of reaction that has occurred and the
structural formula of the oxime?

type of reaction structural formula of the oxime

A nucleophilic addition

B nucleophilic addition

C condensation

trendyline
n yli e
condensation

ACJC 2016 9647/01/Prelim/2016 [Turn over


11 trendyline
11

26 Compound A is an example of a prostaglandin, which belongs to a group of physiologically


active lipid compounds having diverse hormone-like effects in animals.

compound A compound B

Which reagent will convert compound A to compound B efficiently?

A NaBH4 B LiAlH4/dry ether


C H2/Pt D Sn/concentrated HCl

27 Lactic acid, CH3CH(OH)CO2H, is found in sour milk.

Which of the following equations represent a possible reaction with lactic acid?

A CH3CH(OH)CO2H + CH3OH o CH3CH(OCH3)CO2H + H2O


B CH3CH(OH)CO2H + CH3CO2H o CH3CH(OCOCH3)CO2H
C CH3CH(OH)CO2H + PCl5 o CH3CH(Cl)CO2H + POCl3 + HCl
D CH3CH(OH)CO2H + 4I2 + 7NaOH o (CO2Na+)2 + CHI3 + 5NaI + 6H2O

28

trendyline
The tetrapeptide
ide
de above is a fragment of SNAP-25,
SNAP
SNA -25, a substrate
subst of Botulinum toxin (Botox).

How many molesoles of NaOH will react with


wit 1 mole of the tetrapeptide when it is heated under
reflux with aqueous
ueous sodium hydroxide
hydroxid until
until no further reaction occurs?
occurs?

A 3 B 5 C 6 D 7

ACJC 2016 9647/01/Prelim/2016 [Turn over


12 trendyline
12

29 The demand for natural shampoos and detergents has led to the development of more
biodegradable detergents such as sorbitan monolaurate, which is made from plants.

sorbitan monolaurate

Which of the following statements about sorbitan monolaurate is correct?

A It is optically inactive.
B It can react with concentrated sulfuric acid on heating.
C There will be an orange precipitate formed upon adding 2,4dinitrophenylhydrazine to
the compound.
D There will be no colour change on heating the compound with acidified potassium
dichromate(VI) solution.

30 Hydrocortisone is a steroid hormone produced by the adrenal gland and is released in


response to stress. It is commonly used as an active ingredient in anti-inflammatory
creams.

hydrocortisone

Which of the following statements about hydrocortisone is correct?

A When treated with an excess of hot concentrated acidified KMnO 4, it forms a


compound containing seven carbonyl groups.
B When warmed with aqueous alkaline iodine, a yellow precipitate is observed.
C When treated with cold dilute KMnO4, it forms a compound containing two additional

trendyline
hydroxyy groups.
g
D When treated with NaBH4 in the presence of methanol, it for
forms a compound
containing
ng seven hydroxy groups.

ACJC 2016 9647/01/Prelim/2016 [Turn over


13 trendyline
13

Section B

For each of the questions in this section, one or more of the three numbered statements 1 to 3
may be correct.

Decide whether each of the statements is or is not correct (you may find it helpful to put a tick
against the statements that you consider to be correct).

The responses A to D should be selected on the basis of

A B C D

1, 2 and 3 1 and 2 2 and 3 1 only


are only are only are is
correct correct correct correct

No other combination of statements is used as a correct response.

31 Epsom salt is a sedative for the nervous system. Its active ingredient is magnesium sulfate
which is soluble in water. The corresponding barium sulfate is insoluble in water.

What factors account for the difference in solubility between magnesium sulfate and barium
sulfate?

1 Magnesium sulfate has a higher solubility product than barium sulfate.


2 Magnesium ions have a higher enthalpy of hydration than barium ions.
3 Magnesium sulfate has a larger magnitude of lattice energy than barium sulfate.

32 The diagram shows a set-up used to find the transition temperature at which white tin and
grey tin are in equilibrium. This was found to be 15 C. Above 15 C, grey tin from
electrode I dissolves and is deposited on electrode II as white tin.

V
grey tin white tin
electrode I
electrode II

thermostat tank

(NH4)2SnCl6 (aq)

Which of the following


ollowing statements about the
e set-up
set-up are
re true?
ru

1 At 15 C,
C,, no current flows.
2 Below 15
5 C, electrons flow through the external circuit from I to II.
II
3 The conversion of white tin to grey tin is an endothermic process.

ACJC 2016 9647/01/Prelim/2016 [Turn over


14 trendyline
14

The responses A to D should be selected on the basis of

A B C D

1, 2 and 3 1 and 2 2 and 3 1 only


are only are only are is
correct correct correct correct

No other combination of statements is used as a correct response.

33 The pKa values of some ammonium ions at 298 K are given below.

ammonium ion pKa

CH3NH3+ 10.66

(CH3)3NH+ 9.81

Based on the data given, which of the following statements are correct?

1 The pH of 0.1 mol dm3 of (CH3)3NH+Cl is 5.4.


2 (CH3)3N is more basic than CH3NH2 due to the presence of more electron donating
alkyl groups.
3 1 mol of (CH3)3N requires more HCl for complete neutralisation than 1 mol of CH3NH2.

34 Curve Z represents the distribution of energies of the gaseous molecules for an uncatalysed
reaction.

Curve W
Curve Z

P Q

Which of the following statements are false?

1 Curve W will be the distribution curve obtained when the reaction is catalysed and Q

trendyline
nts the most probable energy of the molecules.
represents molecule
2 Curve W will be the e distribution curve obtained at a higher temperature
te and S
nts the most probable energy of the molecules.
represents
3 Curve Z will be the distribution curve obtained
taine when the reaction is uncatalysed
u and P
represents the most probable energy off the molecules.

ACJC 2016 9647/01/Prelim/2016 [Turn over


15 trendyline
15

The responses A to D should be selected on the basis of

A B C D

1, 2 and 3 1 and 2 2 and 3 1 only


are only are only are is
correct correct correct correct

No other combination of statements is used as a correct response.

35 Sulfur burns in chlorine gas to form disulfur dichloride, S2Cl2. In excess chlorine gas, the
following equilibrium is established.

S2Cl2 + Cl2 2SCl2

Which statements regarding S2Cl2 and its equilibrium with SCl2 are correct?

1 S2Cl2 dissolves in water to give a solution with pH greater than 7.


2 S2Cl2 is a liquid at room temperature.
3 The sulfur atoms have a bent geometry in both S2Cl2 and SCl2.

36 Use of the Data Booklet is relevant to this question.

Which of the following statements are true for green aqueous solution of V2(SO4)3?

1 Zn metal can reduce V3+(aq) to V2+(aq).


2 The solution appears green as the d-orbitals are split by the presence of the sulfate
ions.
3 V3+(aq) can serve as a homogeneous catalyst in the reaction between iodide ions and
peroxodisulfate ions.

trendyline
ACJC 2016 9647/01/Prelim/2016 [Turn over
16 trendyline
16

The responses A to D should be selected on the basis of

A B C D

1, 2 and 3 1 and 2 2 and 3 1 only


are only are only are is
correct correct correct correct

No other combination of statements is used as a correct response.

37 Unripe fruits often contain polycarboxylic acids. These are acids with more than one
carboxylic acid group in their molecule. One such acid is citric acid shown below.

Another polycarboxylic acid present in unripe fruit is a colourless crystalline solid, W, which
has molecular formula of C4H6O5.

A sample of W of mass 1.97 g was dissolved in water and the resulting solution titrated with
1.00 mol dm3 NaOH. 29.4 cm3 of 1.00 mol dm3 NaOH was required for complete
neutralisation.

What could W be?

38 Which of the following statements are always true of the isoelectronic point, pI, of an amino
acid?
1
2
3 trendyline
It is the pH where the zwitterionic form of the amino acid is most dominant.
do
dom
It is the pH where the amino acid does not migrate when placed in an
It is the average of the first two pKa values.
a electric field.

ACJC 2016 9647/01/Prelim/2016 [Turn over


17 trendyline
17

The responses A to D should be selected on the basis of

A B C D

1, 2 and 3 1 and 2 2 and 3 1 only


are only are only are is
correct correct correct correct

No other combination of statements is used as a correct response.

39 The rod cells in the retina at the back of the eye contain an alcohol called retinol which is
responsible for their sensitivity to light. Retinol is oxidised by an enzyme-catalysed reaction
to retinal with the following structure:

retinal

Which of the following statements about retinal are correct?


1 It can react with NH2NH2.
2 The molecular formula of retinal is C20H28O.
3 Oxidative cleavage of retinal gives two different organic products.

40 Benzocaine and menthol are local anaesthetics used in sunburn ointments and skin lotions.
Their structures are shown below:

benzocaine menthol

Which of the following reagents and conditions can be used to distinguish between the two
compounds?

trendyline
1 aqueouss b
bromine, dark
2 iodine in
n aqueous sodium hydroxide, heat
3 acidified
d aqueous potassium manganate(
mangana VII),
VII
manganate(VII),), heat

ACJC 2016 9647/01/Prelim/2016 [Turn over


18 trendyline
18

BLANK PAGE

Answers:

1 2 3 4 5 6 7 8 9 10
A A D B D B B B A C
11 12 13 14 15 16 17 18 19 20
A C A B C D D B B A
21 22 23 24 25 26 27 28 29 30
A D A C D B D D B C
31 32 33 34 35 36 37 38 39 40
C D D B C D C B A B

trendyline
ACJC 2016 9647/01/Prelim/2016 [Turn over
19 trendyline
Index No. Name Form Tutorial Subject
Class Class Tutor

ANGLO-CHINESE JUNIOR COLLEGE


DEPARTMENT OF CHEMISTRY
Preliminary Examination

CHEMISTRY 9647/02
Higher 2
Paper 2 Structured Questions 16 August 2016
2 hours
Candidates answer on the Question Paper
Additional Materials: Data Booklet

READ THESE INSTRUCTIONS FIRST

Write your name, index number, form class, tutorial class and subject tutors name on all the work you
hand in.
Write in dark blue or black pen.
You may use a soft pencil for any diagrams, graphs or rough working.
Do not use staples, paper clips, glue or correction fluid.

Answer all questions.


A Data Booklet is provided.

The number of marks is given in brackets [ ] at the end of each question or part question.
At the end of the examination, fasten all your work securely together.

For Examiner's Use


Question no. Marks
1

trendyline
dyline
li TOTAL

This document consists of 15 printed


inte pages and 1 blank page
page.

9647/02/Prelim/16 ANGLO-CHINESE JUNIOR COLLEGE


ACJC 2016 Department of Chemistry [Turn over

20 trendyline
2

1 Planning (P)

The solubility of a metal hydroxide, M(OH)2 at a particular temperature is defined as the


mass of metal hydroxide that dissolve in 100 g of water.

A saturated solution of the metal hydroxide is one in which no more solid can dissolve at
this fixed temperature. At 25oC, the solubility of metal hydroxide is approximately less than
2g /100g of water.

You are to plan an experiment using the method of gravimetric analysis to achieve these
objectives:
1. determine the solubility product, Ksp of the metal hydroxide accurately;
2. determine how the solubility of the metal hydroxide in sodium hydroxide is affected
by the concentration of hydroxide ions.

(a) (i) Write an equation with state symbols to show the dissolution of the metal
hydroxide in water.

. [1]

(ii) Suggest and explain how the solubility of the metal hydroxide varies with the
concentration of hydroxide ions.

...

. [1]

(b) You are provided with:


x pure solid metal hydroxide
x 2.0 mol dm-3 of sodium hydroxide
x the apparatus and chemicals normally found in a school or college laboratory.

Your plan should include details of:


x the preparation of a saturated metal hydroxide solution
x the preparation of a suitable range of diluted sodium hydroxide solutions
x an outline of how the results could be used to calculate the solubility of metal
hydroxide and its Ksp and show how the solubility of metal hydroxide in sodium
hydroxide varies with the concentration of hydroxide ions
x the risks involved and safety precautions taken

...

...

...

trendyline


...



...



...

...

ACJC 2016 9647/02/Prelim/2016


21
[Turn over
trendyline
3

...

...

...

...

...

...

...

...

...

...

...

...

...

...

...

...

...

...

...

...

...

...

...

...

...



...



...



...

...[8]

ACJC 2016 9647/02/Prelim/2016


22
[Turn over
trendyline
4

(c) Suggest another experimental method to determine how the solubility of metal
hydroxide changes with the presence of sodium hydroxide. Suggest the results to be
collected.

...

...

...[2]

[Total: 12]

2 A, B and C are three organic molecules which share the same molecular formula,
C8H9NO2.

You are to make use of the data about the reactions of A, B and C given in the table
below to deduce their identities.

reaction reagent used A B C


1 KMnO4(aq), purple colour purple MnO4 purple MnO4
H2SO4(aq) remains decolourised decolourised
heat
2 excess Br2 (aq) white solid no visible white solid
formed with Mr reaction formed with Mr
= 308.8 = 308.8
3 NaOH (aq) at dissolves to give dissolves to does not
room a colourless give a dissolve at all
temperature solution colourless
solution
4 Na2CO3 (aq) no visible effervescence no visible
reaction seen reaction
5 HCl (aq) at does not dissolves to dissolves to
room dissolve at all give a give a
temperature colourless colourless
solution solution

(a) (i) Deduce the molecular formula of the white solid formed from A in reaction 2.

[1]

(ii) What type of reaction takes place in reaction 2 for A?

[1]

(iii) Hence list the possible functional groups present in A based on reaction 2
alone.

(b) (i)
trendyline


List
[2]

st the functional groups that are being tested for in reaction 3.

[2]
3

ACJC 2016 9647/02/Prelim/2016


23
[Turn over
trendyline
5

(ii) Identify the gas that is being produced in reaction 4 for B and how you would
confirm its identity.

[2]

(iii) Which of the functional groups you have named in b(i) is confirmed by
reaction 4?

[1]

ACJC 2016 9647/02/Prelim/2016


24
[Turn over
trendyline
6

(c) You now have enough information to determine the identities of A, B and C. Below
are three possible structures of A, B and C (not in order).

Draw the structural formula of the three compounds in the boxes below.

trendyline
e y [3]

ACJC 2016 9647/02/Prelim/2016


25
[Turn over
trendyline
7

(d) Hydroxydanaidal is an insect pheromone synthesised by male tiger moths from


heliotrine in plants. It is an isomer of the three organic molecules above and is made
up of two five-membered rings.

In order for a ring to be aromatic, it has to fulfil the following criteria:

x The ring must have 4n+2 electrons in the delocalised porbital cloud
(where n is a positive integer).
x The ring must be planar.
x Every atom in the ring must be able to participate in delocalising the
electrons.

(i) Given that the N atom is sp2 hybridised, what is the number of p electrons in
the right ring of hydroxydanaidal?

[1]

(ii) Hence state if the right ring is aromatic.

[1]

(iii) Suggest reagents and conditions for a reaction that that will distinguish
hydroxydanaidal from the three isomers A, B and C.

[1]
[Total: 15]

trendyline
ACJC 2016 9647/02/Prelim/2016
26
[Turn over
trendyline
8

3 Group VII elements can form oxoanions of XOn which are strong oxidising agents. One
such example is bromate(V) anion, BrO3 which can oxidise iodide to form iodine, and
itself reduced to bromide.

(a) Write a balanced ionic equation for the reaction between bromate(V) and iodide.

... [1]

(b) A student wished to analyse the amount of iodine and the bromide ions produced
from the above reaction and hence he carried out the following procedures.

1) Add 25.0 cm3 of 0.150 mol dm-3 potassium bromate(V) solution to excess
potassium iodide (3 grams) and 5 cm 3 of 1 mol dm-3 H2SO4 in a 250 cm3 conical
flask.

2) Add sufficient chloroform, CHCl3, to the above solution immediately and transfer
the mixture to a separating funnel.

3) Separate the organic and aqueous layers and add barium nitrate solution to the
aqueous layer in a beaker.

4) Filter the above mixture and add silver nitrate to the filtrate in the conical flask.

5) To the resultant mixture in the conical flask in step 4, add sufficient concentrated
ammonia and filter once again.

(i) What is the purpose of adding chloroform in step 2?

...

[1]

(ii) What is the reason for adding barium nitrate in step 3?

...

[1]

(iii) A mixture of two precipitates will be formed when silver nitrate is added into
step 4. Suggest the identities of the precipitates.

[1]

(iv) Describe clearly what the student will observe in step 5 after he added in
concentrated ammonia and performed a filtration. Explain the chemistry
behind his method.

...

trendyline



...



...

[3]

ACJC 2016 9647/02/Prelim/2016


27
[Turn over
trendyline
9

(c) Explain why fluorine does not form oxoanions such as FO3.

[1]

[Total: 8]

4 Compound X and compound Y are oxides formed from period 3 elements.

When water is added to both oxides, only X dissolves but not Y. However, Y will dissolve
when it is added to the aqueous solution of X and NaOH (aq) separately.

It is known that aqueous solution of X can react with solid potassium hydroxide in a ratio
of 1:3.

Identify compound X and Y, and explain the above reactions with the aid of balanced
equations.

...

...

...

...

...

...

...

...

[Total: 7]

trendyline
ACJC 2016 9647/02/Prelim/2016
28
[Turn over
trendyline
10

5 Chymotrypsin is a digestive enzyme found in the small intestines that can hydrolyse
peptide bonds within proteins. It is also used as a form of treatment for sports injuries to
reduce pain and inflammation. The three main amino acids involved in the catalytic activity
are histidine, aspartic acid and serine.

(a) The graph shows the results of an investigation of the initial rate of hydrolysis of
peptide D by the enzyme chymotrypsin. In the experiments, the initial concentration
of peptide D was varied but that of chymotrypsin was kept constant.

Rate

[Peptide D]

Explain the difference in the rate of hydrolysis at high and low concentrations of
peptide D.

...

...

......

... [2]

(b) The first step of the mechanism of the action of chymotrypsin is shown:

(i) In an aqueous external environment, suggest and explain whether the three
amino acids are orientated inward or outward of the compact structure.

trendyline



...




... [2]

ACJC 2016 9647/02/Prelim/2016


29
[Turn over
trendyline
11

(ii) State the roles of histidine in the mechanism shown above.

... [1]

(iii) Explain with reference to the mechanism, how a low pH might affect the
enzymatic activity of chymotrypsin.

...

... [2]

(iv) Draw the structural formula of a tripeptide with the sequence ser-his-asp,
showing the form which it would exist at pH 4.5 given the pKa of the R-group
in his = 6.04 and pKa of the R-group in asp = 3.90.

[2]

[Total: 9]

trendyline
ACJC 2016 9647/02/Prelim/2016
30
[Turn over
trendyline
12

6 Three unknown cations J2+, K2+ and L3+ are analysed according to the flowchart below.

J, K and L can be any one of the following: Al, Mn, Fe, Co, Ni, Pb, Zn, Sc.

(a) Identify the three cations and the three unknowns A, B and C. [6]

cations
J2+:

K2+:

L3+:

unknowns
A:

B:

C:

trendyline
ACJC 2016 9647/02/Prelim/2016 [Turn over
31 trendyline
13

(b) Vanadium(II) chloride, VCl2 is prepared by the thermal decomposition of


vanadium(III) chloride, VCl3.

(i) Write the electronic configuration of a vanadium atom.

[1]

(ii) Hence explain why the third ionisation energy of vanadium is higher than the
second ionisation energy.

...

...

... [1]

(iii) Predict, with reason, which of the chlorides of vanadium, VCl2 or VCl3 will
have a higher melting point.

...

...

... [1]

[Total: 9]

7 Nitryl fluoride, NO2F is a colourless gas which acts as a strong oxidising agent and
fluorinating agent.

(a) Draw a dot-and-cross diagram of NO2F.

[1]

(b) It is known that NO2F can be produced from nitrogen dioxide and fluorine according
to the equation below.

2NO2(g) + F2(g) 2NO2F(g)

The rate equation for the reaction was also found to be Rate = k[NO2][F2].

trendyline
(i) Propose
ropose
op a two-step mechanism
m consistent
consi
c stent
nt with the above equation,
equ indicating
thee rate determining step
step.




...

... [2]

ACJC 2016 9647/02/Prelim/2016 [Turn over


32 trendyline
14

(ii) Given that the reaction is exothermic, sketch a labelled energy profile
diagram for the complete reaction path including the formation of the
intermediate species.

[3]

(c) When NO2 and excess F2 were mixed together, [NO2F] varies with time, as shown in
the table below. The initial [NO2] used is 0.400 mol dm3.

>12)@
t / min [NO2F] / mol dm3 / mol dm3 min1 [NO2] / mol dm3
W

0.25 0.150 0.60 0.250


0.50 0.245
0.75 0.300
1.00 0.340
1.25 0.360
1.50 0.375 0.06 0.025

(i) Explain why it is necessary to use excess fluorine in this experiment to


confirm the order with respect to nitrogen dioxide.

...

... [1]

trendyline
ACJC 2016 9647/02/Prelim/2016 [Turn over
33 trendyline
15

(ii) The rate at a particular time can be approximated using:


>12 )@
rate = W

Complete the table above by calculating >12)@ (to 2 decimal places) and
W
the [NO2] left (to 3 decimal places) at the respective timings. [2]

(iii) Hence, plot a graph of >12)@ against [NO2] to confirm and explain that the
W
reaction is first order with respect to NO2.

trendyline
[3]


[Total: 12]

ACJC 2016 9647/02/Prelim/2016 [Turn over


34 trendyline
16

BLANK PAGE

trendyline
e
ACJC 2016 9647/02/Prelim/2016 [Turn over
35 trendyline
1 2 3 4 5 6 7 8 9 10
A A D B D B B B A C
11 12 13 14 15 16 17 18 19 20
A C A B C D D B B A
21 22 23 24 25 26 27 28 29 30
A D A C D B D D B C
31 32 33 34 35 36 37 38 39 40
C D D B C D C B A B

trendyline
36 trendyline
Index No. Name Form Tutorial Subject
Class Class Tutor

ANGLO-CHINESE JUNIOR COLLEGE


DEPARTMENT OF CHEMISTRY
Preliminary Examination

CHEMISTRY 9647/02
Higher 2
Paper 2 Structured Questions 16 August 2016
2 hours
Candidates answer on the Question Paper
Additional Materials: Data Booklet

READ THESE INSTRUCTIONS FIRST

Write your name, index number, form class, tutorial class and subject tutors name on all the work you
hand in.
Write in dark blue or black pen.
You may use a soft pencil for any diagrams, graphs or rough working.
Do not use staples, paper clips, glue or correction fluid.

Answer all questions.


A Data Booklet is provided.

The number of marks is given in brackets [ ] at the end of each question or part question.
At the end of the examination, fasten all your work securely together.

For Examiner's Use


Question no. Marks
1

trendyline
dyline
li TOTAL

This document consistss of 12 printed pages.


pages
9647/02/Prelim/16 ANGLO-CHINESE JUNIOR COLLEGE
ACJC 2016 Department of Chemistry [Turn over

37 trendyline
2

1 Planning (P)

The solubility of a metal hydroxide, M(OH)2 at a particular temperature is defined as the


mass of metal hydroxide that dissolve in 100 g of water.

A saturated solution of the metal hydroxide is one in which no more solid can dissolve at
this fixed temperature. At 25oC, the solubility of metal hydroxide is approximately less than
2g /100g of water.

You are to plan an experiment using the method of gravimetric analysis to achieve these
objectives:
1. determine the solubility product, Ksp of the metal hydroxide accurately;
2. determine how the solubility of the metal hydroxide in sodium hydroxide is affected
by the concentration of hydroxide ions.

(a) (i) Write an equation with state symbols to show the dissolution of metal
hydroxide in water. [1]

M(OH)2(s) + aq M2+(aq) + 2OH(aq) [1]

(ii) Suggest and explain how the solubility of the metal hydroxide varies with the
concentration of hydroxide ions. [1]

The solubility of metal hydroxide decreases when the concentration of sodium


hydroxide increases as there is common ion effect or explanation of eqm
shifting to the left when concentration of hydroxide increases (LCP).

Examiners Comments:
Students wrote full arrow instead of half arrows. Metal hydroxide is a sparingly
soluble salt hence the solid exists in equilibrium with its aqueous ions.

(b) You are provided with:


pure solid metal hydroxide
2.0 mol dm-3 of sodium hydroxide
the apparatus and chemicals normally found in a school or college laboratory.

Your plan should include details of:


the preparation of a saturated metal hydroxide solution
the preparation of a suitable range of diluted sodium hydroxide solutions
an outline of how the results could be used to calculate the solubility of metal
hydroxide and its Ksp and show how the solubility of metal hydroxide in sodium
hydroxide varies with the concentration of hydroxide ions
the risks involved and safety precautions taken [8]

ttrendyline
Method 1::

1. U sing a 50 cm3 burette/ 100 cm3 measuring cylinder/ 50 cm3 pipette, place
Using
100 cm3 of distilled water into a pre-weighted 250 cm3 dry and clean conical
aker (M1 g) placed in a thermostated
flask/beaker most water bath contro
controlled at a fixed
temperature. Weigh again with the distilled water inside to obtain the total mass of
the distilled water and beaker, M2 g.

ACJC2016 9647/JC2 Prelim Exam/2016


38
[Turn over
trendyline
3

2. Add about 2-4g (> solubility and depending on volume of solution used) mass of
solid metal hydroxide to the solution and stir with a glass rod. Stir the mixture
continuously until no more solid dissolves to ensure a saturated solution is obtained.
Let the mass of metal hydroxide be M3 g.

3. Allow reaction mixture to stand/ leave for a long time to reach equilibrium.

4. Pre-weigh the dry filter paper, M4 g. Filter the mixture with the use of a filter paper
and filter funnel into a beaker. Discard the filtrate. Weigh the mass of the filter paper
and solid, M5 g.

5. Wash the residue with some cold water and dry the filter paper with residue in an
oven/IR lamp. Temperature should be pre-set to 60oC (lower than the decomposition
temperature and to prevent the filter paper from burning). Let the filter paper cool
down in a dessicator. Weigh the filter paper and its contents.

6. Repeat the process of heating, cooling and weighing till constant mass of the filter
paper and its contents is obtained.

Mass of dissolved solute = M3 (M5 M4) = X g


(mass of solid M(OH)2 added - mass of residue)
Mass of solvent = M2 M1 = Y g
Solubility of metal hydroxide in 100g of water = X/ Y x 100 (g/100g of water)

OR

Method 2:
1. Measure about 100 cm3 (any volume) of water using a measuring cylinder and
place it in a dry and clean beaker placed in a themostated water bath.

2. Add about 2g (> solubility and depending on volume of solution used) mass of
solid metal hydroxide to the solution and stir with a glass rod. Stir the mixture
continuously until no more solid dissolves to ensure a saturated solution is
obtained.

3. Allow reaction mixture to stand/ leave for a long time to reach equilibrium.

4. Using an electronic weighing balance, measure and record the mass of an


empty, dry crucible/beaker. This is recorded as M1 g.

5. Using a dry filter funnel and filter paper, filter the suspension and collect the
saturated filtrate in the crucible. Measure the mass of the filtrate and crucible.
The mass obtained is M2 g.

g).
g a Bunsen burner, heat the solution until all the liquid is evap
6. Using

trendyline
tant
eva
evaporated off and
ant mass is obtained, cool it and re-weigh the beaker with its contents (M3
constant

Mass of dissolved solute = (M3 M1) = Xg


Mass of solvent = (M2 M1) = Yg
Solubility of metal hydroxide in 100g of water = 100 x X/Y
ACJC2016 9647/JC2 Prelim Exam/2016
39
[Turn over
trendyline
4

6. Repeat the experiment with another mass of metal hydroxide to get reliable
results.

7. Conversion of solubility in g/ 100 g of water to mol dm-3 assume density of water =


1g cm-3

Calculate the Ksp of metal hydroxide as Ksp = [M2+][OH-]2.

Results:
M(OH)2(s) M2+(aq) + 2OH (aq)

Equilibrium concentration x 2x
(mol dm-3)

In a saturated solution,
Ksp = [x][2x]2 = 4x3

Preparation of standard solutions of sodium hydroxide

Volume of Initial concentration Final concentration


Volume of NaOH
3
water / cm3 of NaOH of NaOH
/ cm
(mol dm-3) (mol dm-3)
50 50 2 1
50 50 1 0.5

7. Prepare at least three standard solutions of sodium hydroxide of different


concentration (2 mol dm-3, 1 mol dm-3 and 0.5 mol dm-3).
8. Use of a burette to transfer 50 cm3 of the 2 mol dm-3 sodium hydroxide solution
into a 100 cm3 volumetric flask. Measuring cylinder is not accepted.
9. Top up to the mark with distilled water. Stopper, invert and shake to obtain a
homogeneous solution of 1 mol dm-3 sodium hydroxide.
10. Repeat steps 1-3 using 50 cm3 of the 1 mol dm-3 sodium hydroxide to obtain a
stock solution of 0.5 mol dm-3 sodium hydroxide.
OR vary the volume of water and sodium hydroxide used (keeping total volume
constant), use the same concentration of sodium hydroxide to obtain diluted
solutions

To investigate how the solubility of metal hydroxide varies with the


concentration of sodium hydroxide:
11. Repeat steps 1- 6 using the other three standard solutions of sodium hydroxide
of concentration (2 mol dm-3, 1 mol dm-3 and 0.5 mol dm-3).
12. Calculate the solubility of metal hydroxide for the three concentrations of sodium
hydroxide and tabulate how the solubility of metal hydroxide change with

trendyline
concentration
entration
ntration of sodium hydroxide.
Solubility
bility of metal hydroxide in sodium hydroxide = g / 100g of so
solvent
13. Observe
rve the trend that when concentrations of sodium h
erve hydroxide used
eases,
ases, the solubility of metal hydroxide increase
decreases, increases.

Risk and safety precaution:


Metal hydroxide and sodium hydroxide is corrosive and toxic in nature. Minimise
contact by using gloves and goggles.
ACJC2016 9647/JC2 Prelim Exam/2016
40
[Turn over
trendyline
5

OR
Use oven/heat resistant gloves or tongs to handle the hot beaker/crucible.
OR Cool hot crucible before handling OR wear gloves when heating.
Examiners Comments:
1. Students need to pay more attention to details such as choosing appropriate
apparatus and suitable capacity for the apparatus.
2. Only some students were able to show more attention to details of how the
experimental data should be collected. For example, the mass of the solid
metal hydroxide should be known accurately to 2 or 3 decimal places.
Another example: to weigh the residue of the filtration, the mass of the dry
filter paper must be measured so that its mass can be deducted from the
combined mass of the dried residue and filter paper.
3. When determining how the solubility of metal hydroxide in sodium hydroxide
varies with the concentration of hydroxide ions, a known mass of solid metal
hydroxide must be dissolved into a fixed volume of known concentration of
NaOH (aq). This is not the same as adding NaOH (aq) to a saturated solution
of metal hydroxide.
4. If students suggest drawing a graph of mass of residue collected against
concentration of NaOH (aq), it is also necessary to state the expected shape
and gradient of the graph obtained. (Refer to N2012/III/1planning)
5. Very few students were able to apply their knowledge that the group II metal
hydroxide M(OH)2 undergoes thermal decomposition. In acknowledgement of
the likely decomposition reaction, the filtered M(OH)2 should not be dried at
high temperature i.e. over a Bunsen flame. Instead, drying should be carried
out under an infrared lamp or in a desiccator to prevent thermal
decomposition.
6. Students need to prepare at least 2-3 solutions of sodium hydroxide of
different concentrations to give a range of diluted solutions.
7. Students also need to recognise the fact that solubility is dependent on
temperature and to prepare a saturated solution in a thermostat controlled
water bath.
8.

(c) Suggest another experimental method to determine how the solubility of metal
hydroxide changes with the presence of sodium hydroxide. Suggest the results to be
collected. [2]

Titrimetric method. Prepare a saturated solution of metal hydroxide in sodium


hydroxide of known concentration and titrate the solution with standard HCl solution.

Results to be collected are: Initial burette reading/ final burette reading / titre volume
Examiners Comments:
Some students wrote that initial volume and final volume collected which is wrong.
[Total: 12]

C8H9NO2.

You are to make


trendyline
2 A, B and C are three organic molecules which share the same mol

ake use of the data about the reactions


below to deduce their identities.
eac
molecular formula,

of A, B and C given
giv in the table

reaction reagent used A B C


ACJC2016 9647/JC2 Prelim Exam/2016
41
[Turn over
trendyline
6

1 KMnO4(aq), purple colour purple MnO4 purple MnO4


H2SO4(aq) remains decolourised decolourised
heat
2 excess Br2 (aq) white solid no visible white solid
formed with Mr reaction formed with Mr
= 308.8 = 308.8
3 NaOH (aq) at dissolves to give
dissolves to does not
room a colourless give a dissolve at all
temperature solution colourless
solution
4 Na2CO3 (aq) no visible effervescence no visible
reaction seen reaction
5 HCl (aq) at room does not dissolves to dissolves to
temperature dissolve at all give a give a
colourless colourless
solution solution
(a) (i) Deduce the molecular formula of the white solid formed from A in reaction 2.
[1]

C8H7NO2Br2
(ii) What type of reaction takes place in reaction 2 for A? [1]

Electrophilic substitution

(iii) Hence, list the possible functional groups present in A based on reaction 2
alone. [2]

Phenol, phenylamine

Examiners Comments:

(b) (i) List the functional groups that are being tested for in reaction 3. [2]

phenol and carboxylic acid


.

(ii) Identify the gas that is being produced in reaction 4 for B and how you would
confirm its identity. [2]

CO2.
Pass the gas through a solution of Ca(OH)2. It should form a white precipitate.

(iii) Which of the functional groups you have named in b(i) is confirmed by
reaction 4? [1]

Carboxylic acid

Examiners
rs Comments:
ers

ACJC2016 9647/JC2 Prelim Exam/2016


42
[Turn over
trendyline
7

(c) You now have enough information to determine the identities of A, B and C. Below
are three possible structures of A, B and C (not in order).

Draw the structural formula of the three compounds in the boxes below. [3]

trendyline
tre
endyline 1m for each structure

ACJC2016 9647/JC2 Prelim Exam/2016


43
[Turn over
trendyline
8

Examiners Comments:
1. Chemical structures must be drawn using a PEN in examinations.

(d) Hydroxydanaidal is an insect pheromone synthesised by male tiger moths from


heliotrine in plants. It is an isomer of the three organic molecules above and is made
up of two five-membered rings.

In order for a ring to be aromatic, it has to fulfil the following criteria:

The ring must have 4n+2 electrons in the delocalised porbital cloud
(where n is a positive integer)
The ring must be planar
Every atom in the ring must be able to participate in delocalising the
electrons.

(i) Given that the N atom is sp2 hybridised, what is the number of p electrons in
the right ring of hydroxydanaidal? [1]

(ii) Hence, state if the right ring is aromatic. [1]

Yes.

(iii) Suggest reagents and conditions for a reaction that that will distinguish
hydroxydanaidal from the three isomers A, B and C. [1]

Add Tollens reagent and warm/heat.


Add 2,4-DNPH or Bradys Reagent.
If student stated in (ii) that it is not aromatic then accept:
Add Fehlings solution and warm/heat as ecf

trendyline
Examiners
er s Comments:

[Total: 15]

3 Group VII elements can form oxoanions of XOn which are strong oxidising agents. One
such example is bromate(V) anion, BrO3 which can oxidise iodide to form iodine, and
ACJC2016 9647/JC2 Prelim Exam/2016
44
[Turn over
trendyline
9

itself reduced to bromide.

(a) Write a balanced ionic equation for the reaction between bromate(V) and iodide. [1]

BrO3 + 6I + 6H+ Br + 3I2 + 3H2O [1]

Examiners Comments:

1. Students should use the information provided in the question (identity of products)
to construct the balanced redox reaction and not propose their own products.

(b) A student wished to analyse the amount of iodine and the bromide ions produced
from the above reaction and hence he carried out the following procedures.

1) Add 25.0 cm3 of 0.150 mol dm-3 potassium bromate(V) solution to excess
potassium iodide (3 grams) and 5 cm 3 of 1 mol dm-3 H2SO4 in a 250 cm3 conical
flask.

2) Add sufficient chloroform, CHCl3, to the above solution immediately and transfer
the mixture to a separating funnel.

3) Separate the organic and aqueous layers and add barium nitrate solution to the
aqueous layer in a beaker.

4) Filter the above mixture and add silver nitrate to the filtrate in the conical flask.

5) To the resultant mixture in the conical flask in step 4, add sufficient concentrated
ammonia and filter once again.

(i) What is the purpose of adding chloroform in step 2? [1]

It is to extract iodine into the organic layer as iodine is more soluble in CHCl3
due to the formation of effective interaction (induced dipole - induced dipole).

Examiners comments:

1. Students need to be clear in their reasoning that the removal of iodine will
allow for easy identification of the precipitates formed later since I2 forms an
equilibrium with I that results in a brown solution (I3).
(ii) What is the reason for adding barium nitrate in step 3?
[1]

It is to remove sulfate as barium sulfate so that it will not interfere with the
reaction when silver nitrate is added.

Examiners comments:
1. To state that BaNO3 is needed for reaction with H2SO4 is vague as it

trendyline
equires
quires the examiner to infer the product of the reaction. Also,
requires Als H2SO4 is an
acid
cid so such an answer brings up a possibility of a misconc
misconception whereby
BaNO
aNO
N 3 reacts with an acid.

(iii) A mixture of two precipitates will be formed when silver nitrat


nitra
nitrate is added into
ep 4
step 4. Suggest the identities of the
e precipitates.
p
precipitates [1]

AgBr and AgI


ACJC2016 9647/JC2 Prelim Exam/2016
45
[Turn over
trendyline
10

(iv) Describe clearly what the student will observe in step 5 after he added in
concentrated ammonia and performed a filtration. Explain the chemistry
behind his method. [3]

Cream ppt of AgBr dissolves to give a colourless filtrate and yellow ppt of AgI
is obtained as the residue.
Ag+(aq) + 2NH3(aq) [Ag(NH3)2]+(aq) ----- (1)

When concentrated NH3 is added, the soluble complex of [Ag(NH3)2]+ is


formed, lowering the concentration of Ag+.

Hence the eqm (2) position shifts to the RHS, causing AgBr to dissolve.

AgBr (s) Ag+(aq) + Br-(aq) ----- (2)


OR
The ionic product [Ag+][Br] become smaller than Ksp.

However the Ksp values of AgI(s) is extremely low such that its ionic product
[Ag+][I] still exceed its Ksp [1], hence AgI remains insoluble even when
concentrated NH3 is added.

Examiners Comments:

1. This question has two parts: one on observations and the other on the theory
behind.

2. Students need to be clear in describing the observations (colour of solutions or


precipitates, effervescence of gas and the corresponding test to confirm its identity,
dissolution of solids etc)

3. Account for the observations based on the knowledge of factors affecting


solubility and the difference in Ksp values of the silver halides.

4. Some students do not know the reaction of Ag+ with NH3.

(c) Explain why fluorine does not form oxoanions such as FO3. [1]

1. F does not have 3d orbitals that are energetically accessible for electrons to
occupy / for expansion of octet to occur.

2. Fluorine is too / highly electronegative to have a positive oxidation number OR


cannot form dative bonds (if student include answer 1)

trendyline
ne
e is a small atom such that 3 oxygen atoms around it would result in too
3. Fluorine
much electronic
ectronic repulsion / steric hindrance.
hindrance

[Total: 8]
Examinersrs Comments:
ers
1. The fluorine
uorine here is an atom though it should
sho be noted the entire sp
species is a
polyatomic ion. Hence ideas of charge density / polarising power of F that
destabilise the ion are not correct.
ACJC2016 9647/JC2 Prelim Exam/2016
46
[Turn over
trendyline
11

2. Students should pay attention to the usage of terms such as atomic or ionic
radius of F and what such terms imply of their description of F.

4 Compound X and compound Y are oxides formed from period 3 elements.

When water is added to both oxides, only X dissolves but not Y. However, Y will dissolve
when it is added to the aqueous solution of X and NaOH (aq) separately.

It is known that aqueous solution of X can react with solid potassium hydroxide in a ratio
of 1:3.

Identify compound X and Y, and explain the above reactions with the aid of balanced
equations.

Compound X: P4O10 / phosphorus (V) oxide / phosphorus pentoxide


Compound Y: Al2O3 / aluminium oxide

Only X is soluble in water but not Y:


P4O10 (s) + 6H2O (l) 4H3PO4 (aq)
ECF from identity of compound X for P2O5

Al2O3 has high lattice energy / strong ionic bonds and hence is not able to dissolve in
water.

Y will dissolve when added to the aqueous solution of X and NaOH (aq) separately:
Al2O3 is amphoteric and will react with both acids and alkalis.
Al2O3 (s) + 2H3PO4 (aq) 2AlPO4 (aq) + 3H2O (l)
OR
Al2O3 + 6H+ Al3+ + 3H2O

Al2O3 (s) + 2NaOH (aq) + 3H2O (l) 2NaAl(OH)4 (aq)


OR
Al2O3 (s) + 2OH (aq) + 3H2O (l) 2Al(OH)4

Reactions of solutions of X with potassium hydroxide:


H3PO4 (aq) + 3KOH (s) K3PO4(aq) + 3H2O(l)
OR
3H+ (aq) + 3KOH (s) 3K+ (aq) + 3H2O(l)

Examiners Comments:

trendyline
rmula
mula should not be used as an identity of su
1. Empirical formula substances.

[Total: 7]

ACJC2016 9647/JC2 Prelim Exam/2016


47
[Turn over
trendyline
12

5 Chymotrypsin is a digestive enzyme found in the small intestines that can hydrolyse
peptide bonds within proteins. It is also used as a form of treatment for sports injuries to
reduce pain and inflammation. The three main amino acids involved in the catalytic activity
are histidine, aspartic acid and serine.

(a) The graph shows the results of an investigation of the initial rate of hydrolysis of
peptide D by the enzyme chymotrypsin. In the experiments, the initial concentration
of peptide D was varied but that of chymotrypsin was kept constant.

Rate

[Peptide D]

Explain the difference in the rate of hydrolysis at high and low concentrations of
peptide D. [2]

At low concentration of peptide D, substrates do not saturate or occupy all the


available active sites. Rate of reaction will be directly proportional to [substrate] or
first order reaction with respect to substrate.

At high concentration of peptide D, substrates saturate or occupy all the available


active sites. Rate of reaction will be independent of [substrate] Or zero order
reaction with respect to substrate.

(b) The first step of the mechanism of the action of chymotrypsin is shown:

(i) In an aqueous external environment, suggest and explain whether the three
amino acids are orientated inward or outward of the compact structure.

trendyline
[2]]

Three
hree
ree amino acids are orientated outward of the compact st structure as they
contain
ntain the polar hydroxyl and amine group which are able to form hydrogen
ontain
bonds/
nds/ ion-dipole with water molecules/ catalytic activity as there
onds/ the is binding to
th
substrate
ubstrate .

Examiners comments:
ACJC2016 9647/JC2 Prelim Exam/2016
48
[Turn over
trendyline
13

Some students did not recognise that hydrophobic groups tend to face inward
and hydrophilic groups tend to face outward.

(ii) State the roles of histidine in the mechanism shown above.


[1]

It is both an acid (proton donor) and a base (proton acceptor). Or protonate


and deprotonate.

Examiners comments
1. Do not confuse between acid-base and redox reactions! The particle
that they give out are different!
Donor of H+ ion Bronsted-Lowry acid
Acceptor of H+ ion Bronsted-Lowry base
Loss of H atom Reducing agent
Gain of H atom Oxidising agent

(iii) Explain with reference to the mechanism, how a low pH might affect the
enzymatic activity of chymotrypsin. [2]

At low pH, excess H+ will react with the basic amine group of histidine OR it
will react with the COO- group of asp OR ionic bonds are disrupted, hence
this will lead to changes in enzyme shape and cause denaturation/ lose
enzymatic activity/ mechanism will not work.

(iv) Draw the structural formula of a tripeptide with the sequence ser-his-asp,
showing the form which it would exist at pH 4.5 given the pKa of the R-group
of his = 6.04 and pKa of the R-group of asp = 3.90. [2]

Examiners Comments:
1. The side chain of ser cannot be deprotonated, since alcohols are weaker
acids that water. (The effect of this relative acid strength is that alcohols will
not deprotonate in an aqueous medium.)
2. All protein / peptide sequences, in this case ser-his-asp, are shown with the
N-terminal is on the left and the C-terminal is on the right.

trendyline
3. The
he alpha-carboxy group for asp is also deprotonated
depro as it is more acidic
han the R group for asp as the COO- group is closer to the el
than e
electron
+
withdrawing
ithdrawing NH3 group.
4. The
he nitrogen atom on the left (Nx) is
s less basic compared to Ny as the lone
pair of electrons on Nx is unavailable
ble for protonation due to the overlap of p
orbitals on Nx atom with the p orbital of the neighbouring C atom. The lone
pair of electrons on Ny is found in a different plane hence it is available for
ACJC2016 9647/JC2 Prelim Exam/2016
49
[Turn over
trendyline
14

protonation.

[Total: 9]

trendyline
ACJC2016 9647/JC2 Prelim Exam/2016
50
[Turn over
trendyline
12

6 Three unknown cations J2+, K2+ and L3+ are analysed according to the flowchart below.

J, K and L can be any one of the following: Al, Mn, Fe, Co, Ni, Pb, Zn, Sc.

(a) Identify the three cations and the three unknowns A, B and C. [6]

Cations
J2+ : Pb2+ or [Pb(H2O)6]2+

K2+ : Zn2+ or [Zn(H2O)6]2+

L3+ : Fe3+ or [Fe(H2O)6]3+

Unknowns
A : [Fe(SCN)(H2O)5]2+

B : PbSO4

C : Zn(OH)2

trendyline
Identity
y of J and K can be switched.

ers
rs Comments:
Examiners

(b) Vanadium m (II) chloride, VCl2 is prepared by the thermal decomposition


decompositi of vanadium
(III) chloride, VCl3.

ACJC2016 9647/JC2 Prelim Exam/2016 [Turn over


51 trendyline
12
(i) Write the electronic configuration of the vanadium atom. [1]

V: 1s2 2s2 2p6 3s2 3p6 3d3 4s2

Examiners comments
1. Always write the full electronic configuration. It is not the candidates
(aka students) prerogative to use the shortform [Ar].

(ii) Hence, explain why the third ionisation energy of vanadium is higher than the
second ionisation. [1]

The second electron removed is from the 4s orbital whereas the third electron
is removed from the 3d orbital. The third electron removed is from an inner
principal quantum shell, which experiences greater nuclear attraction and
hence, requires more energy to remove.

(iii) Predict, with reason, which of the chlorides of vanadium, VCl2 or VCl3 will
have a higher melting point. [1]

Lattice energy q+ q-
r+ + r-

VCl3 will have a higher melting point. As V3+ has a smaller ionic radius and a
higher charge, VCl3 will have a more exothermic lattice energy requiring more
energy to melt. [1]

Examiners Comments:

1. Thermal decomposition is not the same as melting as the former involves a


chemical change in the substance while the latter involves a physical process with
no change in the chemical identity of the substance.
[Total: 9]

7 Nitryl fluoride, NO2F is a colourless gas which acts as a strong oxidising agent and
fluorinating agent.

(a) Draw a dot-and-cross diagram of NO2F. [1]

F
N
O O

Examiners comments
1. Students are reminded that the least electronegative atom should be the
central atom.

trendyline
2. Students should check their own answers to ensure that they did not attempt
to
o assign an expanded octet to the N atom, a Period 2 element.
elemen

(b) It is known
wn that NO2F can be produced from nitrogen dioxide and flu
fluorine according
quation below.
to the equation

2NO2 (g) + F2 (g) 2NO2F (g)


ACJC2016 9647/JC2 Prelim Exam/2016 [Turn over
52 trendyline
12

The rate equation for the reaction was also found to be Rate = k[NO2][F2].

(i) Propose a two-step mechanism consistent with the above equation, indicating
the rate determining step. [2]

NO2 (g) + F2 (g) NO2F (g) + F (g) (slow; rate determining step)
NO2 (g) + F (g) NO2F (g)
Examiners comments
1. Students are reminded that the power of concentrations in the rate
equation must correspond to the stoichiometry of the slow step and
2. that the sum of the elementary steps should match the overall
balanced equation.
3. Charge conservation must also be observed in each step. i.e make
sure that charges are balanced in each step.

(ii) Given that the reaction is exothermic, sketch a labelled energy profile
diagram for the complete reaction path including the formation of the
intermediate species. [3]

Energy

Ea (Step 2)
Ea (Step 1)
NO2F +
F + NO2

2NO2 + F2
H = -ve

2NO2F (g)

Progress of reaction
Examiners Comments:
1. Number of elementary steps in the mechanism must match the number of
peaks in the energy profile diagram.
2. The first Ea should be higher to show that the first step is the slow step. Ea is
the difference between the energy of the reactants and transition state or
intermediate and transition state.

trendyline
3. The energygy level of the intermediate is never lower than that of the reactants
or products!

ACJC2016 9647/JC2 Prelim Exam/2016 [Turn over


53 trendyline
12
(c) When NO2 and excess F2 were mixed together, [NO2F] varies with time, as shown in
the table below. The initial [NO2] used is 0.400 mol dm3.

t / min [NO2F] / mol dm3 [NO 2F] / mol dm3 min1 [NO2] / mol dm3
t
0.25 0.150 0.60 0.250
0.50 0.245 0.38 0.155
0.75 0.300 0.22 0.100
1.00 0.340 0.16 0.060
1.25 0.360 0.08 0.040
1.50 0.375 0.06 0.025

Examiners comments
1. [NO2] is calculated by Initial [NO2F] Instantaneous [NO2F]. (Refer to mole
ratio of NO2 and NO2F in the overall equation.)
2. [NO 2F] is calculated using ([NO2F]t [NO2F]t-1) (timet timet-1)
t
e.g. At = 0.75 min, [NO 2F] = 0.300 0.245 0.75 0.50 = 0.22
t

(i) Explain why it is necessary to use excess fluorine in this experiment to


confirm the order with respect to nitrogen dioxide. [1]

This is to keep the concentration of fluorine relatively constant during the


experiment so that the rate depends only on the concentration of NO2. /
pseudo first order wrt NO2.

(ii) [NO 2F]


The rate at a particular time can be approximated using rate = t .
Complete the table above by calculating [NO 2F] (to 2 decimal places) and
t
the [NO2] left (to 3 decimal places) at the respective timings. [2]

(iii) Hence, plot a graph of [NO 2F] against [NO2] to confirm and explain that the
t
reaction is first order with respect to NO2. [3]

trendyline
ACJC2016 9647/JC2 Prelim Exam/2016 [Turn over
54 trendyline
12

trendyline
ACJC2016 9647/JC2 Prelim Exam/2016 [Turn over
55 trendyline
12
[NO 2F] / mol dm-3 min-1
t
0.70

0.60

0.50

0.40

0.30

0.20

0.10

0.00 [NO2] / mol dm-3


0.000 0.050 0.100 0.150 0.200 0.250 0.300

As a straight line graph passing through the origin is obtained, OR


the rate is directly proportional to the concentration of NO2, hence the
reaction is first order with respect to NO2.

Examiners Comments:
1. The graph of rate of reaction against time MUST pass through the origin. It
indicates that the reaction does not occur when concentration is zero.
2. Rate of reaction and [NO2] have a directly proportional relationship; not
proportional.
3. The relationship of rate and [NO2] must be described as a general trend (i.e.
directly proportional) and should not only be described using the comparison
of two data points.

[Total: 12]

trendyline
ACJC2016 9647/JC2 Prelim Exam/2016 [Turn over
56 trendyline
2016 ACJC H2 Chem Prelim Paper 3
2

1 Besides reducing carbon dioxide emission, CO2 levels in the atmosphere can be lowered
through a process known as carbon capture and sequestration (CCS). One form of
naturally occurring CCS is the conversion of CO2 into plant biomass during
photosynthesis.

Scientists have also devised chemical methods of CCS. The processes for isolating
carbon dioxide and for storing it has been the focus of some research.

(a) Amine gas treating has been studied as a reaction for the first part of the CCS
process. This method uses aqueous solutions of various alkylamines to capture
carbon dioxide from the waste gases of industrial productions.

(i) Monoethanolamine (MEA) and dimethylethanolamine (DMEA) are two


alkylamines that are commonly used in amine gas treating.

base pKb
HOCH2CH2N(CH3)2 (DMEA) 3.73
HOCH2CH2NH2 (MEA) 4.52
C6H5NH2 9.13

Explain the difference in pKb values in the table. [3]

(ii) The aqueous solution containing 32% MEA by weight can neutralise up to
0.5 mol CO2 per mole of the amine. Assume that density of the solution is
1 kg dm3.

Calculate the pH of 32% MEA solution. [3]

ACJC2016 9647/JC2 Prelim Exam/2016 [Turn over


3

(b) (i) Another alkylamine, dimethylpiperazine (DMP), has the molecular formula
C6H14N2.

1 mole of DMP can be formed from 2 moles of N-methyl-2-chloroethylamine


under suitable conditions.

1. State the conditions needed for this reaction.

2. Write the balance equation for the conversion of N-methyl-2-


chloroethylamine into DMP, showing the structural formulae of N-
methyl-2-chloroethylamine and DMP.
[2]

(ii) Under room conditions, N-methyl-2-chloroethylamine reacts with

1. ClCH2CO2H and

2. CH3COCl

Name the type of reaction that occur and suggest the structure of the
compound formed in each reaction.
[3]

ACJC2016 9647/JC2 Prelim Exam/2016 [Turn over


4

(c) CO2 that is captured from amine gas treating needs to be sequestered so that it does
not return into the atmosphere.

Carbon sequestration has been attempted using mineral carbonation. In this


method, liquid CO2 is buried under a volcanic rock known as basalt. Over a period of
time, CO2 reacts with metal oxides in the rock to form MgCO3 and CaCO3. This
method allows a large amount of CO2 in the atmosphere to be trapped as solid thus
reducing its effect as a greenhouse gas.

(i) In volcanic areas, underground temperatures rises with increasing depth. Two
types of carbonates found in these areas are rhodochrosite MnCO3 and calcite
CaCO3.

By quoting suitable data from the Data Booklet, explain whether MnCO3 or
CaCO3 decomposes more readily in the presence of geothermal heat. [3]

(ii) Magnesite, MgCO3, forms an equilibrium with CO2 when it is trapped


underground in high temperature.

Reaction 1 MgCO3(s) MgO(s) + CO2(g) Hr = +117 kJ mol1

Suggest how mineral carbonation can be maintained despite high temperature


and justify your answer. [2]

(iii) Fosterite, Mg2SiO4, is a component of basalt that reacts with CO2 in the
following reaction.

Reaction 2 Mg2SiO4(s) + 2CO2(g) 2MgCO3(s) + SiO2(s) Hr < 0

ACJC2016 9647/JC2 Prelim Exam/2016 [Turn over


5

enthalpy change of combustion of Mg(s) = 607 kJ mol1


enthalpy change of formation of Mg2SiO4(s) = 59.5 kJ mol1
enthalpy change of formation of SiO2(s) = 911 kJ mol1

Using the information above and relevant data from the Data Booklet, draw an
energy level diagram and hence calculate the enthalpy change of reaction 2.
[4]

[Total: 20]

ACJC2016 9647/JC2 Prelim Exam/2016 [Turn over


2 (a) The primary fermentation of grape juice produces wine and carbon dioxide.
Sometimes, a secondary fermentation takes place a few months after the primary
fermentation is finished and the wine has been bottled. This secondary fermentation
converts 2-hydroxybutanedioic acid (malic acid) in the wine into 2-hydroxypropanoic
acid (lactic acid). There are two benefits: the carbon dioxide produced can be used
to make the wine sparkling; the conversion of the malic acid into lactic acid
decreases the acidity of the wine.

HOOCCH2CH(OH)COOH CH3CH(OH)COOH + CO2

A typical concentration of malic acid in wine is 0.05 mol dm3.

(i) Calculate the volume of carbon dioxide produced at room temperature and
pressure conditions when 1.0 dm3 of wine undergoes secondary fermentation.
[1]

(ii) A typical wine bottle has a capacity of 1.5 dm3. Using the ideal gas equation,
calculate the pressure the carbon dioxide produced would exert inside the air
gap of the bottle containing 1.3 dm3 of wine at room temperature.
[1]

(iii) In fact, the pressure inside the bottle is much less under these conditions.
Explain why this differs from the pressure you calculated in (a)(ii).
[1]

ACJC2016 9647/03/Prelim/2016 [Turn over


(b) Another way by which the acidity in wine can be decreased is by acid-catalysed
ester formation:

ethanol + acid ester + water

Wine consists of ethanol and water. A 1.0 dm3 of wine typically contains 10% of
ethanol by mass and a 0.10 mol dm3 concentration organic acid, assuming density
of wine is 1 g cm3.

(i) Calculate the value for the equilibrium constant Kc for the ester formation,
given that 17.4% of acid is converted to ester. [3]

(ii) Maturation is the process where the wine evolves to a state of readiness for
bottling or drinking. Before the wine is ready to be kept in a dark and cool
environment at an ambient temperature below 20 oC in barrels for storage,
they will equilibrate to form ester. Suggest two reasons why wine needs
several months to mature before this reaction takes place significantly.
[2]

(c) Describe a laboratory test to distinguish between 2-hydroxybutanedioic acid (malic


acid) and 2-hydroxypropanoic acid (lactic acid). State the reagents and conditions
you would use and any observations that you would make for each compound. Write
equation for any reaction that occurs.
[3]

[1]

ACJC2016 9647/03/Prelim/2016 [Turn over


(d) Typically, esters can be hydrolysed in dilute acid or alkali.

The mechanism of both hydrolysis methods are shown as follow:

alkaline hydrolysis

acidic hydrolysis

In alkaline hydrolysis, the hydroxide anion attacks the electron deficient carboxyl
carbon.

In acid hydrolysis, the acid protonates the carboxyl oxygen to render the carboxyl
carbon more electrophilic towards nucleophilic attack by water.

(i) What is the role of

1. OH in step 1 of alkaline hydrolysis


2. H+ in acidic hydrolysis respectively? [2]

ACJC2016 9647/03/Prelim/2016 [Turn over


(ii) What is the role of water in acid hydrolysis? [1]

(iii) Outline the two advantages of alkaline hydrolysis. [2]

(e) Claisen condensation is a carboncarbon bond forming reaction that can occur
between two esters in the presence of a strong base, resulting in a keto ester. In
the mixture of methyl ethanoate and ethyl methanoate, the following Claisen
condensation products are formed.

and

(Deuterium, 21D, is an isotope of hydrogen.)

Predict the structures of keto esters formed when ethyl ethanoate is mixed with
methyl propanoate under suitable reaction conditions.

ethyl ethanoate methyl propanoate [4]

ACJC2016 9647/03/Prelim/2016 [Turn over


[Total:20]

ACJC2016 9647/03/Prelim/2016 [Turn over


3 (a) The characteristic properties of transition metals include coloured ions, complex
formation, variable oxidation states and catalytic activity.

Transition metals are used extensively as heterogeneous catalysts in industrial


processes. An example is the use of iron as a catalyst in the Haber process to
manufacture ammonia.

(i) Explain what is meant by the term heterogeneous? [1]

(ii) State, in the correct order, three essential steps for the mechanism of
heterogeneous catalysis. [2]

(iii) The strength of the adsorption of reactants and products onto the surface of a
transition metal helps to determine its efficiency as a heterogeneous catalyst.

Suggest a reason why transition metals which adsorb weakly are usually not
good heterogeneous catalysts.
[1]

(iv) A catalyst containing molybdenum(VI) was analysed for its molybdenum


content by reducing a sample to the molybdenum(III) state.

When titrated with acidified potassium manganate(VII), the molybdenum(III)


was oxidised back to molybdenum(VI).

A 0.330 g sample of the catalyst, after reduction, required 27.50 cm3 of


0.0200 mol dm3 potassium manganate(VII) to convert the molybdenum(III)
into molybdenum(VI).

Calculate the percentage by mass of molybdenum in the catalyst. [2]

ACJC2016 9647/03/Prelim/2016 [Turn over


(b) (i) The chemistry of the transition metal, tungsten, has certain similarities to that
of sulfur. Both elements reach their maximum +6 oxidation states when
combined with the electronegative element, oxygen.

Most tungsten occurs naturally in the tungsten anion, WO42-, analogous to the
sulfate ion, SO42-. A common mineral is Scheelite which is calcium tungstate,
CaWO4.

Draw the structure showing the bonding in the WO42 ion and give the OWO
bond angle. [2]

(ii) Tungsten(VI) oxide, WO3, is used in electrochromic windows which change


colour when an electrical voltage is applied.

In electrochromic windows, a voltage is applied between a transparent layer of


tungsten(VI) oxide and a source of ions such as a lithium salt, and the
following reaction takes place during which some of the lithium ions are
incorporated into the structure of the oxide:

WO3 + xLi+ + xe LixWO3

The product, LixWO3, is known as a tungsten bronze and its colour depends on
the value of x. The value of x can vary from 0 to 1; a typical value giving blue-
black colour is when x = 0.3.

Calculate the average oxidation state of the tungsten when x = 0.3. [1]

ACJC2016 9647/03/Prelim/2016 [Turn over


(iii) A complex of tungsten, W(CO)6, has zero oxidation state for tungsten. It reacts
with CH3Li as shown by the following equation:

CH3Li + W(CO)6 Li[W(CO)5(COCH3)]

The reaction is nucleophilic addition where CH3 is a nucleophile.

Illustrate the shape of [W(CO)5(COCH3)] by drawing the displayed formula of


the anionic complex.
[1]

(c) CoF63 and Co(NH3)63+ are two octahedral complexes which are blue and yellow in
colour respectively.

The colours of the two complexes are different due to different ligands which cause
splitting of the five 3d orbitals by different amount.

The following diagram shows how the five d-orbitals are split in an octahedral
environment.

ACJC2016 9647/03/Prelim/2016 [Turn over


(i) Draw two diagrams like the one above to show the electronic configurations of
cobalt ions in CoF63 and Co(NH3)63+ respectively. [2]

Examiners comments
Students who got this wrong often miscalculated the number of d electrons.

ACJC2016 9647/03/Prelim/2016 [Turn over


(ii) Hence, explain which of the two complexes will have a larger energy gap, E.
[1]

(iii) Explain why electrons usually prefer to occupy orbitals singly, rather than in
pairs.
[1]

(d) Salbutamol is an anti-asthma drug which is prepared from aspirin, which in turn is
prepared from salicylic acid.

Salbutamol

The following reaction scheme shows the synthesis of Salbutamol from salicylic acid.

ACJC2016 9647/03/Prelim/2016 [Turn over


(i) Draw the structures of aspirin, product Z and reagent Y. [3]

(ii) A carbonyl compound with a hydrogen atom on its carbon rapidly equilibrates
with its corresponding enol form as shown.

ACJC2016 9647/03/Prelim/2016 [Turn over


This rapid inter-conversion between the keto and enol form is a special kind of
isomerism known as tautomerism.
In the presence of strong bases, the hydrogen atom on its carbon is
abstracted to form a resonance-stabilised enolate anion as shown below:

Resonance structures differ only in the position of their electrons.

The mechanism for the reaction from product W to form product X using Br2
and NaOH involves the intermediate, enolate anion. The two steps are:

The first step involves an acid-base reaction between the hydroxide ion
and the methyl group to give the enolate anion.

Using its resonance structure with the C=C bond, the enolate anion
undergoes electrophilic attack by Br2 to give product X.

Based on the information given, describe the mechanism for the reaction to
form product X from product W. In your answer, show any relevant charges,
lone pairs of electrons and arrows to show movement of electrons.
[3]

ACJC2016 9647/03/Prelim/2016 [Turn over


[Total: 20]

ACJC2016 9647/03/Prelim/2016 [Turn over


4 This question concerns ascorbic acid (vitamin C, C6H8O6) and its chemistry. The structure
of ascorbic acid is provided below.

ascorbic acid

(a) Ascorbic acid can be used in the synthesis of other compounds, with the use of
Si(CH3)3Cl as a protecting group for the alcohol groups.

(i) In water, Si(CH3)3Cl hydrolyses at a much faster rate than C(CH3)3Cl.

Explain the difference in the ease of hydrolysis. [2]

(ii) State and describe the mechanism for the hydrolysis of C(CH3)3Cl in water.
You should include in your answer:
Dipoles of relevant covalent bonds
Curly arrows
Label for the slow step [3]

ACJC2016 9647/03/Prelim/2016 [Turn over


(b) Dehydroascorbic acid (DHA), C6H6O6, with its structure shown below, is the oxidised
product of ascorbic acid.

(i) State the oxidation number of:


the carbon labelled C1 in ascorbic acid.
the carbon labelled C2 in DHA. [2]

(ii) Write a balanced half equation, showing their structural formulae, for the
reduction of DHA to ascorbic acid. [1]

(iii) The electrode potential of ascorbic acid at pH 2 is +0.281 V. Explain how the
electrode potential value changes as the pH increases.
[2]

(c) Electrolysis of an aqueous solution of Fe(ascorbate)2 buffered at pH 2 was carried

ACJC2016 9647/03/Prelim/2016 [Turn over


out, with 22.4 cm3 of hydrogen gas obtained at room temperature and pressure,
Ascorbate is the conjugate base of ascorbic acid.

(i) Using information from the Data Booklet, determine the minimum voltage
required to run this electrolysis. [2]

(ii) Calculate the mass of the product obtained at the anode. [2]

(d) To 50 cm3 of an aqueous solution of ascorbic acid, 200 cm3 of ethanol was added.
Some FeCl3 was dissolved in this solution and the solid, Fe(ascorbate)2X with an
Mr of 422.8, was filtered off. The equation is as follows:

(i) Suggest the role of ethanol in the preparation of the precipitate. [1]

ACJC2016 9647/03/Prelim/2016 [Turn over


(ii) Determine the identity of the ion [1]

(iii) The formation of Fe(ascorbate)2X from Fe(H2O)63+ involves a ligand


substitution between H2O and the ascorbate ion as shown above.

Explain the sign of the entropy change for the formation of Fe(ascorbate)2X.

Hence given that the formation is exothermic, deduce the sign of the standard
Gibbs free energy change. [4]

[Total: 20]

5 The use of the Data Booklet is relevant to this question.

(a) 2methylbuta1,3diene, also known as isoprene, is produced by many plants and


serves as a building block for monoterpenes when two isoprene units combine without
any rearrangement of the carbon skeleton. Monoterpenes all have molecular formula
of C10H16.

isoprene

Compounds A and B are geometric isomers of a monoterpene commonly used in


perfumes for their sweet herbal scent. Compounds A and B react with cold dilute
alkaline potassium manganate(VII) to produce stereoisomers of C10H22O6.

Compounds A and B react with hot concentrated acidified potassium manganate(VII)


to produce compounds C, C3H4O3, D, C3H4O4, E, C3H6O and a gas F. Both C and E
give a yellow precipitate with aqueous alkaline iodine. A 1.04 g sample of D reacted
with excess sodium metal to produce 240 cm3 of gas.

Use the information to suggest, with explanation, identities of A to F. [9]

ACJC2016 9647/03/Prelim/2016 [Turn over


ACJC2016 9647/03/Prelim/2016 [Turn over
ACJC2016 9647/03/Prelim/2016 [Turn over
(b) Fehlings solution is often used as a distinguishing test in the qualitatively analyses of
organic compounds. It is prepared by mixing two separate solutions, known as
Fehlings A and Fehlings B. Fehlings A is aqueous copper(II) sulfate while Fehlings
B is an aqueous solution of potassium sodium tartrate made in sodium hydroxide.

Tartaric acid, H4T, can exist as a tetraanion, T4, in a strongly alkaline medium. In
Fehlings B, it exists as the H2T2 anion.

The standard electrode potentials, E , of several species are shown in the table
below.

Electrode reaction E /V
1 Cu2O + H2O + 2e 2Cu + 2OH 0.36
2 2Cu2+ + H2O + 2e Cu2O + 2H+ +0.20
3 [Cu(HT)2]4 + 2H2O + 2e Cu + 2H2T2 + 2OH +0.25
4 HCO2H + 2H+ + 2e HCHO + H2O 0.03
5 CO2 + 4H+ + 4e HCHO + H2O 0.07
6 CO2 + 2H+ + 2e HCO2H 0.11

Adding Fehlings solution to methanal causes a reaction to take place.

(i) By selecting appropriate electrode reactions from the Data Booklet or the table
above, construct a balanced equation for this reaction. [1]

ACJC2016 9647/03/Prelim/2016 [Turn over



(ii) Use your answer in part (i) to calculate the Ecell for this reaction. [1]

(iii) Describe the observations for this reaction. [1]

(c) Electrode potential values are often quoted with reference to the standard hydrogen
electrode (SHE). It is however, seldom used because it is difficult to regulate the
pressure of hydrogen gas at 1 atm.

One of the more commonly used reference electrodes in electrochemistry is the


silver/silver chloride electrode (SSCE). It is made of a silver wire that is coated with a
thin layer of silver(I) chloride, immersed in a potassium chloride solution saturated
with silver(I) chloride.

Ag
AgCl

KCl and
saturated AgCl
ACJC2016 solution
9647/03/Prelim/2016 [Turn over

porous contact
schematic of a silver/silver chloride electrode

One of the advantages of using the SSCE is that it can be directly immersed into the
other half-cell, thereby negating the use of a salt bridge.

(i) Suggest the purpose of having the porous contact. [1]

The solution in the SSCE is colourless in spite of the presence of a dblock element.

(ii) Explain how the colour in transition metal compounds arise. [2]

(iii) Write the full electronic configuration of Ag+ and hence explain why it is not
coloured. [2]

ACJC2016 9647/03/Prelim/2016 [Turn over


(d) Adding a reagent that selectively brings one of the ions out of a solution as a
precipitate while leaving the other ion in the solution is a technique known as selective
precipitation.

Data about the solubilities in water and solubility products of silver(I) hydroxide and
copper(II) hydroxide at 298 K are given below. Only the numerical values of the
solubility products are given.

hydroxide solubility/mol dm3 solubility product

AgOH 1.23 104 1.52 108

Cu(OH)2 1.77 107 2.20 1020

As a general standard, two ions are selectively separated when a maximum 0.01% of
the precipitated compound still remains as the aqueous ions.

Determine if selective precipitation can be performed on a solution containing


0.200 mol dm3 Ag+ and 0.200 mol dm3 Cu2+ with the addition of solid potassium
hydroxide. [3]

ACJC2016 9647/03/Prelim/2016 [Turn over


2

1 Besides reducing carbon dioxide emission, CO2 levels in the atmosphere can be lowered
through a process known as carbon capture and sequestration (CCS). One form of
naturally occurring CCS is the conversion of CO2 into plant biomass during
photosynthesis.

Scientists have also devised chemical methods of CCS. The processes for isolating
carbon dioxide and for storing it has been the focus of some research.

(a) Amine gas treating has been studied as a reaction for the first part of the CCS
process. This method uses aqueous solutions of various alkylamines to capture
carbon dioxide from the waste gases of industrial productions.

(i) Monoethanolamine (MEA) and dimethylethanolamine (DMEA) are two


alkylamines that are commonly used in amine gas treating.

base pKb
HOCH2CH2N(CH3)2 (DMEA) 3.73
HOCH2CH2NH2 (MEA) 4.52
C6H5NH2 9.13

Explain the difference in pKb values in the table. [3]

Basic strength:
C6H5NH2 < HOCH2CH2NH2 (MEA) < HOCH2CH2N(CH3)2 (DMEA)

C6H5NH2 is the weakest base as the lone pair of electron on the N atom is
delocalised into the phenyl group. This makes the lone pair less available
to accept H+.

DMEA is a stronger base than MEA because it has two additional electron
donating methyl groups which increases the electron density on the N
atom and improves is ability to accept proton.

Examiners comments
Common wrong usage of term would be that the alkyl electron donating groups
increases the negative charge on N atom, instead of electron density.

(ii) The aqueous solution containing 32% MEA by weight can neutralise up to
0.5 mol CO2 per mole of the amine. Assume that density of the solution is
1 kg dm3.

Calculate the pH of 32% MEA solution. [3]

n(MEA) in 1 dm3 of 32% solution


= 320 [16.0 + 14.0 + 2(12.0) + 7(1.0)]
= 5.246 mol
Concentration MEA solution = 5.246 mol dm-3

trendyline
tren
Kb = 10-4.52 = 3.162 10-5 mol dm-3

ett [OH-] be y mol d


Let dm-33
[ ][[ ]
Kb =
[ ]

3.162 10-55 =
.

ACJC2016 9647/JC2 Prelim Exam/2016 [Turn over


57 trendyline
3

y = 0.01288 mol dm-3

pOH = 1.89
pH = 12.1 (correct to 1 d.p.)

Examiners comments
Many students could not interpret what 32% by weight of the solution meant.

(b) (i) Another alkylamine, dimethylpiperazine (DMP), has the molecular formula
C6H14N2.

1 mole of DMP can be formed from 2 moles of N-methyl-2-chloroethylamine


under suitable conditions.

1. State the conditions needed for this reaction.

2. Write the balance equation for the conversion of N-methyl-2-


chloroethylamine into DMP, showing the structural formulae of N-
methyl-2-chloroethylamine and DMP.
[2]

1. Ethanol, heat under pressure / in a sealed tube

2.

(ii) Under room conditions, N-methyl-2-chloroethylamine reacts with

1. ClCH2CO2H and

2. CH3COCl

Name the type of reaction that occur and suggest the structure of the
compound formed in each reaction.
[3]

1. Acid-base reaction

2. Condensation reaction / Nucleophilic substitution

trendyline
yl i
Examiners
xaminers comments
Students are not able to draw correct structure of the nomenclature given.

ACJC2016 9647/JC2 Prelim Exam/2016 [Turn over


58 trendyline
4

Many also failed to see that reaction 1 was a simple acid-base reaction and
proceeded to form an amide with the secondary amine and carboxylic acid.

(c) CO2 that is captured from amine gas treating needs to be sequestered so that it does
not return into the atmosphere.

Carbon sequestration has been attempted using mineral carbonation. In this


method, liquid CO2 is buried under a volcanic rock known as basalt. Over a period of
time, CO2 reacts with metal oxides in the rock to form MgCO3 and CaCO3. This
method allows a large amount of CO2 in the atmosphere to be trapped as solid thus
reducing its effect as a greenhouse gas.

(i) In volcanic areas, underground temperatures rises with increasing depth. Two
types of carbonates found in these areas are rhodochrosite MnCO3 and calcite
CaCO3.

By quoting suitable data from the Data Booklet, explain whether MnCO3 or
CaCO3 decomposes more readily in the presence of geothermal heat. [3]

Ion Radii / nm
Ca2+ 0.099
Mn2+ 0.080 (Quote both values with units)

MnCO3 is expected to decompose more readily than CaCO3.

This is because Mn2+ has smaller ionic radius than Ca2+. This causes the
charge density and polarising power of Mn2+ to be greater than that of
Ca2+ thus polarising the C-O bond / distorting the electron cloud in the C
O bond of the anion in MnCO3 to a greater extent.

* Polarising effect is on the covalent bond in the anion; it is not sufficient to


comment on polarising effect on the anion. (Refer to examiners report for
question N2015/III/5cii)

Examiners comments
Some students had mistakenly discussed the question based on Mg2+ instead.
Students who did not include units to the quoted values were not awarded
marks as quantities without units are meaningless.

(ii) Magnesite, MgCO3, forms an equilibrium with CO2 when it is trapped


underground in high temperature.

Reaction 1 MgCO3(s) MgO(s) + CO2(g) Hr = +117 kJ mol1

Suggest how mineral carbonation can be maintained despite high temperature


and justify your answer. [2]

High partial pressure of CO2 must be maintained so that the backward


reaction is favoured to lower the number of moles of gaseous particles.

trendyline
Examiners
xaminers
am comments

(iii) Fosterite,
osterite,
sterite, Mg2SiO4, is a component of basalt that reacts w
with CO2 in the
llowing
owing reaction.
following

Reaction 2 Mg2SiO4(s) + 2CO2(g) 2MgCO3(s) + SiO2(s) Hr < 0

ACJC2016 9647/JC2 Prelim Exam/2016 [Turn over


59 trendyline
5

enthalpy change of combustion of Mg(s) = 607 kJ mol1


enthalpy change of formation of Mg2SiO4(s) = 59.5 kJ mol1
enthalpy change of formation of SiO2(s) = 911 kJ mol1

Using the information above and relevant data from the Data Booklet, draw an
energy level diagram and hence calculate the enthalpy change of reaction 2.
[4]

Enthalpy / kJ mol1

2Mg(s) + 2O2(g) + Si(s) + 2CO2(g)


0
59.5 Mg2SiO4(s) + 2CO2(g)

2(607)

2MgO(s) + O2(g) + Si(s) + 2CO2(g) Hr

-911

2MgO(s) + SiO2(s) + 2CO2(g)


2(+117) 2MgCO3(s) + SiO2(s)

Hr = 2(117) + 2(607) + 59.5 911


Hr = 2299.5
Hr = 2300 kJ mol-1 (correct to 3 s.f.)

Check that each step has the correct chemical formulae and state
symbols, multiple for the enthalpy values or symbols and direction of
arrows.

Examiners comments
This question was poorly done. Many students were not able to balance the
correct processes required.

[Total: 20]

trendyline
ACJC2016 9647/JC2 Prelim Exam/2016 [Turn over
60 trendyline
2 (a) The primary fermentation of grape juice produces wine and carbon dioxide.
Sometimes, a secondary fermentation takes place a few months after the primary
fermentation is finished and the wine has been bottled. This secondary fermentation
converts 2-hydroxybutanedioic acid (malic acid) in the wine into 2-hydroxypropanoic
acid (lactic acid). There are two benefits: the carbon dioxide produced can be used
to make the wine sparkling; the conversion of the malic acid into lactic acid
decreases the acidity of the wine.

HOOCCH2CH(OH)COOH CH3CH(OH)COOH + CO2

A typical concentration of malic acid in wine is 0.05 mol dm3.

(i) Calculate the volume of carbon dioxide produced at room temperature and
pressure conditions when 1.0 dm3 of wine undergoes secondary fermentation.
[1]

Amount of CO2 = amount of malic acid = 0.05 x 1.0 = 0.05 mol

Volume of carbon dioxide produced = 0.05 x 24 = 1.20 dm3 (correct to 3


s.f.)

Examiners comments
Simple question which was well answered. Some students had mistakenly
used 22.4 dm3 as the molar volume of a gas at r.t.p. when that is the value at
s.t.p.

(ii) A typical wine bottle has a capacity of 1.5 dm3. Using the ideal gas equation,
calculate the pressure the carbon dioxide produced would exert inside the air
gap of the bottle containing 1.3 dm3 of wine at room temperature.
[1]
PV = nRT
P = (0.065)(8.31)(298)/(0.2x10-3) = 8.05 x 105 Pa (correct to 3 s.f.)

Examiners comments
Most students could not appreciate that the total volume of the bottle (1.5 dm3),
subtracting the volume of the wine (1.3 dm3), would have given a gas volume
of 0.20 dm3.

(iii) In fact, the pressure inside the bottle is much less under these conditions.
Explain why this differs from the pressure you calculated in (a)(ii).
[1]

This is due to the fact that CO2 is not an ideal gas and as such, it will not
obey the ideal gas equation. CO2 is a relatively massive non-polar
molecule, thus its molecular volume is not considered to be negligible as
compared to the total volume of the gas at such a high pressure.

OR

ttrendyline
d li
Intermolecular
termolecular
rmolecular interactions between CO2 molecules
m are n
not negligible at
such
uch a high pressure too. Thus, C
CO2 deviate more from ide
id
ideality.

OR
R

Carbon
arbon dioxide can dissolve in
n aqueous
a solution. Thus less CO2 will
occupy in the empty space.
ACJC2016 9647/03/Prelim/2016 [Turn over
61 trendyline
Examiners comments

(b) Another way by which the acidity in wine can be decreased is by acid-catalysed
ester formation:

ethanol + acid ester + water

Wine consists of ethanol and water. A 1.0 dm3 of wine typically contains 10% of
ethanol by mass and a 0.10 mol dm3 concentration organic acid, assuming density
of wine is 1 g cm3.

(i) Calculate the value for the equilibrium constant Kc for the ester formation,
given that 17.4% of acid is converted to ester. [3]

ethanol + acid ester + water


I/mol 100/46 0.1 0
C -0.0174 -0.0174 +0.0174
E/mol 2.1565 0.0826 0.0174

Kc = 0.0977

Examiners comments

(ii) Maturation is the process where the wine evolves to a state of readiness for
bottling or drinking. Before the wine is ready to be kept in a dark and cool
environment at an ambient temperature below 20 oC in barrels for storage,
they will equilibrate to form ester. Suggest two reasons why wine needs
several months to mature before this reaction takes place significantly.
[2]
The reaction is slow due to low temperature.

Concentration of acid is low thus leading to slow rate of reaction.

Examiners comments

(c) Describe a laboratory test to distinguish between 2-hydroxybutanedioic acid (malic


acid) and 2-hydroxypropanoic acid (lactic acid). State the reagents and conditions
you would use and any observations that you would make for each compound. Write
equation for any reaction that occurs.
[3]
Reagents & Conditions: aqueous I2, NaOH (aq), warm to 60 oC
Observations: 2-hydroxypropanoic acid will give yellow ppt. while 2-
hydroxybutanedoic acid will not.

Equation:

ttrendyline
r dyy nee [1]

ACJC2016 9647/03/Prelim/2016 [Turn over


62 trendyline
Examiners comments
Some students had forgotten to remove the proton from the carboxylic acid group by
reacting with an extra mole of OH.

(d) Typically, esters can be hydrolysed in dilute acid or alkali.

The mechanism of both hydrolysis methods are shown as follow:

alkaline hydrolysis

acidic hydrolysis

In alkaline hydrolysis, the hydroxide anion attacks the electron deficient carboxyl
carbon.

In acid hydrolysis, the acid protonates the carboxyl oxygen to render the carboxyl
carbon more electrophilic towards nucleophilic attack by water.

(i) What is the role of

trendyline
1
1. OH in step 1 of alkaline
e hydrolysis
+
2. H in acidic hydrolysis respectively? [2]

H- is acting as a nucleophile.
OH
+
H is acting as a catalyst.
catalys

Examiners comments
ACJC2016 9647/03/Prelim/2016 [Turn over
63 trendyline
(ii) What is the role of water in acid hydrolysis? [1]

Water is acting as a nucleophile.

Examiners comments

(iii) Outline the two advantages of alkaline hydrolysis. [2]

The reaction is irreversible or complete which means that there will be


high yield.

The products are easier to separate as one of the products is ionic


(aqueous layer) and will be found in a different phase from the organic
reactants.

Examiners comments

(e) Claisen condensation is a carboncarbon bond forming reaction that can occur
between two esters in the presence of a strong base, resulting in a keto ester. In
the mixture of methyl ethanoate and ethyl methanoate, the following Claisen
condensation products are formed.

and

(Deuterium, 21D, is an isotope of hydrogen.)

Predict the structures of keto esters formed when ethyl ethanoate is mixed with
methyl propanoate under suitable reaction conditions.

trendyline
y ethyl
hyl ethanoate methyl propanoa
propanoate [4]

ACJC2016 9647/03/Prelim/2016 [Turn over


64 trendyline
Examiners comments
Many students who attempted this question only gave two out of four possible
structures. They did not realise that one particular ester could also react with
another molecule of the same ester.

[Total:20]

trendyline
ACJC2016 9647/03/Prelim/2016 [Turn over
65 trendyline
3 (a) The characteristic properties of transition metals include coloured ions, complex
formation, variable oxidation states and catalytic activity.

Transition metals are used extensively as heterogeneous catalysts in industrial


processes. An example is the use of iron as a catalyst in the Haber process to
manufacture ammonia.

(i) Explain what is meant by the term heterogeneous? [1]

Reactants and catalyst are in different phase.

Examiners comments

(ii) State, in the correct order, three essential steps for the mechanism of
heterogeneous catalysis. [2]

1) Surface adsorption/adsorption of reactant molecules on surface of


catalyst
2) Reaction between reactant molecules Allow :bonds in reactants
weakened
3) Products desorbed from the surface

Examiners comments

(iii) The strength of the adsorption of reactants and products onto the surface of a
transition metal helps to determine its efficiency as a heterogeneous catalyst.

Suggest a reason why transition metals which adsorb weakly are usually not
good heterogeneous catalysts. [1]

Reactants not brought together or


No increase in reactant concentration on catalyst surface or
Reactants not held long enough for a reaction to occur or
Reactant bonds not weakened

Examiners comments

(iv) A catalyst containing molybdenum(VI) was analysed for its molybdenum


content by reducing a sample to the molybdenum(III) state.

When titrated with acidified potassium manganate(VII), the molybdenum(III)


was oxidised back to molybdenum(VI).

A 0.330 g sample of the catalyst, after reduction, required 27.50 cm3 of


0.0200 mol dm3 potassium manganate(VII) to convert the molybdenum(III)
into molybdenum(VI).

trendyline
y
alculate
lculate the percentage by mass of molybdenum
Calculate molybden in the cataly
catalyst. [2]

1 mole of molybdenum(IIIII) compound


ompound loses 3 moles of ele
electrons on
oxidation
xidation manganate(VII
idation while 1 mole of manganate II) accept 5 moles o
of electrons in
cidic medium (recognise mole ratio)
acidic ratio

Amount of MnO4- in 27.50 cm3 = 27.5 x 103 x 0.02 = 5.50 x 104 mol
ACJC2016 9647/03/Prelim/2016 [Turn over
66 trendyline
Amount of Mo = 5/3 x 5.50 x 104 = 9.17 x 10-4 mol

Mass of Mo = 95.9 x 9.17 x 104 = 8.80 x 10-2 g

% Mo = (8.80 x 10-2/0.330) x 100 = 26.7 % (correct to 3 s.f.)

Examiners comments
1) Students do not know how to work out the mole ratio between
molybdenum(III) and manganate(VII)

(b) (i) The chemistry of the transition metal, tungsten, has certain similarities to that
of sulfur. Both elements reach their maximum +6 oxidation states when
combined with the electronegative element, oxygen.

Most tungsten occurs naturally in the tungsten anion, WO42-, analogous to the
sulfate ion, SO42-. A common mineral is Scheelite which is calcium tungstate,
CaWO4.

Draw the structure showing the bonding in the WO42 ion and give the OWO
bond angle. [2]

2
O O

W O or W O
O O O O

Bond angle:109.5o
(Do not accept range of values as tungstate is symmetrical.)

Examiners comments
1) Many students gave the dot-cross diagram instead of the structural
formula
2) Charge of ion is not shown.
3) Structural formula requires students to show the bonds between atoms,
hence type of covalent bonds is necessary.

(ii) Tungsten(VI) oxide, WO3, is used in electrochromic windows which change


colour when an electrical voltage is applied.

In electrochromic windows, a voltage is applied between a transparent layer of


tungsten(VI) oxide and a source of ions such as a lithium salt, and the
following reaction takes place during which some of the lithium ions are
incorporated into the structure of the oxide:

WO3 + xLi+ + xe LixWO3

trendyline
The
he product, LixWO3, is known as a tungsten bronze and its co

black
ck colour is when x = 0.3.
ack

Calculate
ccolour depends on
e value of x. The value of x can vary from 0 to 1; a typical vvalue giving blue-
the

ate of the tungsten when x = 0.3.


alculate the average oxidation state [1]

ACJC2016 9647/03/Prelim/2016 [Turn over


67 trendyline
Let oxidation state of the tungsten be m
0.3 + m + (-2)3 = 0
m = +5.7

Examiners comments
1) Students need to show the working as they were told to calculate.

(iii) A complex of tungsten, W(CO)6, has zero oxidation state for tungsten. It reacts
with CH3Li as shown by the following equation:

CH3Li + W(CO)6 Li[W(CO)5(COCH3)]

The reaction is nucleophilic addition where CH3 is a nucleophile.

Illustrate the shape of [W(CO)5(COCH3)] by drawing the displayed formula of


the anionic complex.
[1]

Note: all covalent bonds must be shown as it is a displayed formula.


Octahedral shape illustrated with wedge and dash bonds.

Examiners comments
1) Many students did not know that carbon-oxygen bond is a triple bond
(carbon must form 4 bonds)
2) Students cannot relate what they learn in organic chemistry to this
inorganic complex even though question state that it is nucleophilic
addition.
3) Many students bond CH3 to oxygen instead of a C-C bond.
4) Students did not display all the bonds in structure. Students showed
lack of understanding of the term displayed formula
5) Students should use wedge and dash so as to illustrate shape in 3-
dimension.

(c) CoF63 and Co(NH3)63+ are two octahedral complexes which are blue and yellow in
colour respectively.

The colours of the two complexes are different due to different ligands which cause
splitting of the five 3d orbitals by different amount.

trendyline
lowing
wing diagram shows how the five d
The following orb
orbitals are split in an octahedral
d-orbitals
me
ment.
environment.

ACJC2016 9647/03/Prelim/2016 [Turn over


68 trendyline
of cobalt ion
mplex

In the high spin state, the electrons occupy all the d-orbitals singly, before starting
to pair up in the lower energy d-orbitals.

In the low spin state, the lower energy d-orbitals are filled first, by pairing up if
necessary before the higher energy d-orbitals are used.

CoF63 is a high spin complex while Co(NH3)63+ is a low spin complex.

(i) Draw two diagrams like the one above to show the electronic configurations of
cobalt ions in CoF63 and Co(NH3)63+ respectively. [2]

trendyline
Examiners
xaminers comments
Students who got this wrong often miscalculated the number of d electrons.

ACJC2016 9647/03/Prelim/2016 [Turn over


69 trendyline
(ii) Hence, explain which of the two complexes will have a larger energy gap, E.
[1]

Co(NH3)63+ since E is large so pairing of electrons at lower level is


preferred rather than promotion of electrons to a higher level.

Examiners comments
1) Students are expected to use c(i) to answer this question and hence
cannot answer question based on colour.

(iii) Explain why electrons usually prefer to occupy orbitals singly, rather than in
pairs. [1]

There will be electrostatic repulsion between electrons if they are paired


up.

Examiners comments
1) This is very well answered.

(d) Salbutamol is an anti-asthma drug which is prepared from aspirin, which in turn is
prepared from salicylic acid.

Salbutamol

The following reaction scheme shows the synthesis of Salbutamol from salicylic acid.

trendyline
ACJC2016 9647/03/Prelim/2016 [Turn over
70 trendyline
(i) Draw the structures of aspirin, product Z and reagent Y. [3]

aspirin product Z reagent Y

Examiners comments
1) Many students missed out giving structure of aspirin.

t d li
trendyline
(ii) A carbonyl
arbonyl compound with a hydrogen atom on its carbon rapidly equilibrates
with
th its corresponding enol form as shown.

ACJC2016 9647/03/Prelim/2016 [Turn over


71 trendyline
This rapid inter-conversion between the keto and enol form is a special kind of
isomerism known as tautomerism.
In the presence of strong bases, the hydrogen atom on its carbon is
abstracted to form a resonance-stabilised enolate anion as shown below:

Resonance structures differ only in the position of their electrons.

The mechanism for the reaction from product W to form product X using Br2
and NaOH involves the intermediate, enolate anion. The two steps are:

The first step involves an acid-base reaction between the hydroxide ion
and the methyl group to give the enolate anion.

Using its resonance structure with the C=C bond, the enolate anion
undergoes electrophilic attack by Br2 to give product X.

Based on the information given, describe the mechanism for the reaction to
form product X from product W. In your answer, show any relevant charges,
lone pairs of electrons and arrows to show movement of electrons.
[3]

trendyline
ACJC2016 9647/03/Prelim/2016 [Turn over
72 trendyline
O O
H H
R C R C + H2O
H OH H
H

O
O
H Br
R C + Br Br R C + Br
H
H
H

Note: Need to show the key to rep R or draw out the structure in full
+ and - on Br2

Examiners comments
1) Students need to understand the use of arrows to show the transfer of
a pair of electrons either from a bond or lone pair to form another bond
pair or lone pair.
2) Many students did not do this question d(ii)
3) Students need to explain any symbol used, for example R.

[Total: 20]

ACJC2016 9647/03/Prelim/2016 [Turn over


73 trendyline
4 This question concerns ascorbic acid (vitamin C, C6H8O6) and its chemistry. The structure
of ascorbic acid is provided below.

ascorbic acid

(a) Ascorbic acid can be used in the synthesis of other compounds, with the use of
Si(CH3)3Cl as a protecting group for the alcohol groups.

(i) In water, Si(CH3)3Cl hydrolyses at a much faster rate than C(CH3)3Cl.

Explain the difference in the ease of hydrolysis. [2]

Si has empty 3d orbitals that can accept a lone pair of electrons from
water while there are no empty orbitals available to accept lone pair for
C, requiring the formation of a carbocation prior to nucleophilic attack by
water.

Examiners comments
1) Some students failed to realise that compound is covalent in nature and
cannot mention the charge density of Si4+.

2) Many students discussed the difference in the bond strength of SiCl


and CCl but did not appreciate the fact that the mechanism of
hydrolysis was different and therefore the bonds broken in the slow
step would differ. Also, the bond strength of Si Cl is greater than that
of CCl.

(ii) State and describe the mechanism for the hydrolysis of C(CH3)3Cl in water.
You should include in your answer:
Dipoles of relevant covalent bonds
Curly arrows
Label for the slow step [3]

SN1 / unimolecular nucleophilic substitution

+ -

trre
rend
endyliine
ne
ACJC2016 9647/03/Prelim/2016 [Turn over
74 trendyline
Examiners comments
1) Students failed to read question properly and omitted stating the type of
mechanism.
2)

(b) Dehydroascorbic acid (DHA), C6H6O6, with its structure shown below, is the oxidised
product of ascorbic acid.

(i) State the oxidation number of:


the carbon labelled C1 in ascorbic acid.
the carbon labelled C2 in DHA. [2]

Ascorbic acid: +1
DHA: +2

Examiners comments

(ii) Write a balanced half equation, showing their structural formulae, for the
reduction of DHA to ascorbic acid. [1]

Examiners comments
1) Some students wrote the electrons on the right hand side of the
equation which represents the process of oxidation!

(iii) The electrode potential of ascorbic acid at pH 2 is +0.281 V. Explain how the
electrode potential value changes as the pH increases. [2]

With an increase in pH, there is a decrease in the concentration of H+.


Thus the position of equilibrium for the reaction in (ii) shifts to the left,
favouring the tendency for oxidation of ascorbic acid. Hence the
electrode potential becomes less positive / more negative with a rise in
pH.

(c)
ttrendyline
dyli
Examiners
xaminers comments
1) Many students confused electrode potentials and how h their values
change with changes in equilibrium with the behaviour of ascorbic acid
ns which
under differing pH conditions w co
are two different concepts.
Electrolysis of an aqueous solution of Fe(ascorbate)2 buffered at pH 2 was carried

ACJC2016 9647/03/Prelim/2016 [Turn over


75 trendyline
out, with 22.4 cm3 of hydrogen gas obtained at room temperature and pressure,
Ascorbate is the conjugate base of ascorbic acid.

(i) Using information from the Data Booklet, determine the minimum voltage
required to run this electrolysis. [2]

Cathode
Fe2+ + 2e Fe 0.44V
2H+ + 2e H2 +0.00V

Anode
DHA + 2H+ + 2e ascorbic acid +0.281V
O2 + 4H+ + 4e 2H2O +1.23V

Ecell = 0.00 (+0.281) = 0.281V

A minimum of 0.281V is required to run the electrolysis.

Examiners comments
1) Many students could not identify the correct electrode potential
occurring at the cathode.
(ii) Calculate the mass of the product obtained at the anode. [2]

The oxidation of ascorbic acid is easier at the anode than water.


Identifies DHA as the product at the anode in words or working
22.4
Amount of H2 = = 9.333 x 104 mol
24000
2e H2
DHA 2e

Mass of DHA = 9.333 x 104 x 174 = 0.162 g (correct to 3 s.f.)

Examiners comments

(d) To 50 cm3 of an aqueous solution of ascorbic acid, 200 cm3 of ethanol was added.
Some FeCl3 was dissolved in this solution and the solid, Fe(ascorbate)2X with an
Mr of 422.8, was filtered off. The equation is as follows:

(i)
trendyline
Suggest
uggest
ggest the role of ethanol in the preparation of the precipitate

It serves to reduce the solubility


encourages precipitation.
ubil
precipitate.

of the compound in solution /


[1]

ACJC2016 9647/03/Prelim/2016 [Turn over


76 trendyline
Examiners comments

(ii) Determine the identity of the ion X. [1]

422.8 55.8 2(175) = 17

Hydroxide / OH

Examiners comments
1) Some answers mentioned Cl. However, this ion is not shown in the
equation. Students should make use of information provided in the question.
(iii) The formation of Fe(ascorbate)2X from Fe(H2O)63+ involves a ligand
substitution between H2O and the ascorbate ion as shown above.

Explain the sign of the entropy change for the formation of Fe(ascorbate)2X.

Hence given that the formation is exothermic, deduce the sign of the standard
Gibbs free energy change. [4]

Being bidentate, each ascorbate ion replaces 2 water molecules


coordinated to Fe3+ and increases the number of species/particles. There
is an increase in the entropy during the formation of Fe(ascorbate)2X.
Hence S > 0.

As G = H - TS and H < 0,
G will have a negative sign.

Examiners comments

[Total: 20]

5 The use of the Data Booklet is relevant to this question.

(a) 2methylbuta1,3diene, also known as isoprene, is produced by many plants and


serves as a building block for monoterpenes when two isoprene units combine without
any rearrangement of the carbon skeleton. Monoterpenes all have molecular formula
of C10H16.

isoprene

Compounds A and B are geometric isomers of a monoterpene commonly used in


perfumes for their sweet herbal scent. Compounds A and B react with cold dilute
alkaline potassium manganate(VII) to produce stereoisomers of C10H22O6.

trendyline
Compounds
unds A and B react with hot concentrated ac acidified potassium manganate(VII)
uce compound
to produce compounds C, C3H4O3, D D,, C3H4O4, E,
E, C3H6O and a gasga F. Both C and E
give a yellow precipitate with aqueous alkaline iodine. A 1.04 g sasample of D reacted
ess sodium metal to produce 240 cm3 of gas.
cess
with excess gas

Use the information to suggest, with explanation,


lana identities of A to F. [9]

ACJC2016 9647/03/Prelim/2016 [Turn over


77 trendyline
Statements Explanations/Inferences/Deductions
1. A and B react A and B underwent mild oxidation.
with cold dil.
KMnO4/OH to An increase of 6 OH groups suggests 3 alkenes.
produce C10H22O6.

2. A and B react A and B underwent vigorous oxidation.


with hot conc.
KMnO4/H+ to F is CO2.
produce C, C3H4O3, a terminal alkene is present on A and B.
D, C3H4O4, E, C3H6O
and a gas F.

3a. C and E give C and E contain CH3CO group.


yellow ppt. with (Do not accept CH3CH(OH) since these are products
I2(aq)OH. of vigorous oxidation.)

C is 2oxopropanoic acid.

E is propanone.

3b. 1.04 g of D (D) = 1.04 104 = 0.0100 mol


produces 240 cm3 of Gas is H2, (H2) = 240 24000 = 0.0100 mol
gas with Na.
Since 1 D 1 H2 D has 2 acidic protons.
i.e. 2 carboxylic acid groups.

D is propane1,3dioic acid.

4. Alternative to Since C, D and E are oxidised products of alkene


deducing C, D and cleavage, they either possess ketone and/or
E. carboxylic acid functional groups.

Based on molecular formula of C, the 3 O atoms can


be attributed to (1) 3 ketones OR (2) 1 carboxylic acid
and 1 ketone. Since there are only 3 C atoms, option
(1) is invalid.

C is a ketoacid 2oxopropanoic acid.

Based on molecular formula of D, the 4 O atoms can


be attributed to (1) 2 carboxylic acid groups OR (2) 1
carboxylic acid group and 2 ketones OR (3) 4
ketones. Since there are only 3 C atoms, options (2)
and (3) are invalid.

D has to be a 3 C dicarboxylic acid.


propane1,3dioic acid.

trendyline
y
Based on molecular formula
formu of E, the 1 O atom can
only be a ketone functional group.

E is propanone.

C D E

ACJC2016 9647/03/Prelim/2016 [Turn over


78 trendyline
Deducing A and B

With the structures of C to F, we get the fragments below.

W from C, X from D, Y from E and Z from F.

Fragments Y and Z have to be on the ends of the molecule as they were


cleaved from one end of an alkene. W has two different ends so they have
been labelled W1 and W2 for ease of connection; X is symmetrical so it
doesnt matter which way it connects.

This leaves us with 4 combinations of:


(1) Y W1 W2 X Z (2) Z W1 W2 X Y

(3) Y X W1 W2 Z (4) Z X W1 W2 Y

N.B. that all options can undergo geometric isomerism as well.

However, options (1) and (4) are invalid as they are not made from the
combination of two isoprene units.

Therefore, acceptable structure for A and B would be the cis/trans isomers of


either (2) OR (3).

(2) Z W1 W2 X Y

trendyline &

ACJC2016 9647/03/Prelim/2016 [Turn over


79 trendyline
OR

(3) Y X W1 W2 Z

&

Examiners comments
1) Many students wrote the structural formulae of A to F in pencil. They
should take note that only answers in ink will be marked unless it is
graph plotting or the drawing of diagrams.
2) Many students failed to use data given to do simple mole calculations
and hence made the deduction based on mole ratio of reactants.
3) Some students proposed the wrong structures for A and B because they
had forgotten to include the given fact that they were made up of two
isoprene units, without rearrangement to the carbon framework.
(b) Fehlings solution is often used as a distinguishing test in the qualitatively analyses of
organic compounds. It is prepared by mixing two separate solutions, known as
Fehlings A and Fehlings B. Fehlings A is aqueous copper(II) sulfate while Fehlings
B is an aqueous solution of potassium sodium tartrate made in sodium hydroxide.

Tartaric acid, H4T, can exist as a tetraanion, T4, in a strongly alkaline medium. In
Fehlings B, it exists as the H2T2 anion.

The standard electrode potentials, E , of several species are shown in the table
below.


Electrode reaction E /V
1 Cu2O + H2O + 2e 2Cu + 2OH 0.36
2 2Cu2+ + H2O + 2e Cu2O + 2H+ +0.20
3 [Cu(HT)2]4 + 2H2O + 2e Cu + 2H2T2 + 2OH +0.25
4 HCO2H + 2H+ + 2e HCHO + H2O 0.03
5 CO2 + 4H+ + 4e HCHO + H2O 0.07
6 CO2 + 2H+ + 2e HCO2H 0.11

Adding Fehlings solution to methanal causes a reaction to take place.

(i) By selecting appropriate electrode reactions from the Data Booklet or the table
above, construct a balanced equation for this reaction. [1]

trendyline
alance equations 3 and 5 in acidic OR alkaline
Balance alkali medium.
[Cu(HT)2] + HCHO + H2O 2Cu + 4H2T2 + CO2
2[Cu(HT) 4

ORR
Cu(H 2]4 + HCHO + 2OH 2Cu
[Cu(HT)
2[Cu(HT) Cu + 4H2T22
+ CO32

Balance equations 3 and 4 in acidic


ic or alkaline medium.
medium
4 2
[Cu(HT)2] + HCHO + H2O Cu + 2H2T + HCO2H
ACJC2016 9647/03/Prelim/2016 [Turn over
80 trendyline
OR
[Cu(HT)2]4 + HCHO + OH Cu + 2H2T2 + HCO2

Examiners comments
A large number of students had chosen equation 2 as the reduction half
equation. This is not correct because the question had already provided
sufficient evidence for the incorporation of the hydrogen tartrate anion as a
ligand to Cu2+. In addition, it has been covered before that methanal will
produce a different observation for the Fehlings test by producing a copper
mirror instead of the Cu2O precipitate.

(ii) Use your answer in part (i) to calculate the Ecell for this reaction. [1]

Ecell = Ecat Ean
= +0.25 (0.07)
= +0.32 V

OR

Ecell = Ecat Ean
= +0.25 (0.03)
= +0.28 V

Examiners comments

Most students were able to demonstrate their ability to calculate the Ecell for this
reaction.

(iii) Describe the observations for this reaction. [1]

Blue coloured solution decolourised with the formation of a reddishbrown/ pink


coloured mirror/solid/precipitate.

Examiners comments
Many students were able to identify the colour of the precipitate formed but
failed to describe the colour change in the solution.

Effervescence to produce
CO2 was not accepted. Even though the Ecell could be balanced in acidic
medium as the equations were quoted in acidic medium, the experiment was
performed in an alkaline medium during to the pH of Fehlings solution.

(c) Electrode potential values are often quoted with reference to the standard hydrogen
electrode (SHE). It is however, seldom used because it is difficult to regulate the
pressure of hydrogen gas at 1 atm.

One of the more commonly used reference electrodes in electrochemistry is the


silver/silver chloride electrode (SSCE). It is made of a silver wire that is coated with a
thin layer of silver(I) chloride, immersed in a potassium chloride solution saturated
with silver(I) chloride.

trendyline
ndy
ndy Ag
Ag
AgCl

KCl and
saturated AgCl
ACJC2016 solution
9647/03/Prelim/2016 [Turn over
81 trendyline
porous contact
schematic of a silver/silver chloride electrode

One of the advantages of using the SSCE is that it can be directly immersed into the
other half-cell, thereby negating the use of a salt bridge.

(i) Suggest the purpose of having the porous contact. [1]

The porous contact allows for the exchange of ions/ close the
circuit/maintain electrical neutrality/acts as salt bridge/maintain osmotic
potential between the two halfcells.

Examiners comments:
Generally well done. Most students were able to show understanding that the
porous contact served as the salt bridge to maintain electrical neutrality in the
absence of an actual salt bridge.

The solution in the SSCE is colourless in spite of the presence of a dblock element.

(ii) Explain how the colour in transition metal compounds arise. [2]

When ligands approach transition metal atom/ion, the degenerate orbitals in


the d-subshell split into two nondegenerate energy levels. When an
electron from the lower energy level absorbs energy from visible/white light
and gets promoted to the higher energy level, colour seen is complementary
to the wavelength of light absorbed.

Examiners comments
Generally well done. Some students did not demonstrate awareness that the
approaching ligands cause the original degenerate d orbitals to split into two
different levels. The most common misconception was the absorbance of
visible light that caused the orbitals to split into two different levels

(iii) Write the full electronic configuration of Ag+ and hence explain why it is not
coloured. [2]

Ag+: 1s2 2s2 2p6 3s2 3p6 3d10 4s2 4p6 4d10

As Ag+ has a full d subshell/lacks vacant d orbitals, an electron is unable to


promote from the lower energy level to the higher energy level OR no energy
corresponding to a wavelength in the visible light spectrum is absorbed and all
are transmitted, making Ag+ colourless.

Examiners comments
The electronic configuration was generally well answered. Some students did
not read the question carefully and used the noble gas configuration of [Kr] to

trendyline
runcate
uncate the configuration. These answers were
truncate w accepte as the question
not accepted
ad already specified for the full configuration.
had configuration

ome students described Ag+ as not having a partially filled


Some fille d-subshell. This
an imply that the d-subshell iss either
can e t
totally empty or totally filled. Such
answers did not score the full mark.

ACJC2016 9647/03/Prelim/2016 [Turn over


82 trendyline
(d) Adding a reagent that selectively brings one of the ions out of a solution as a
precipitate while leaving the other ion in the solution is a technique known as selective
precipitation.

Data about the solubilities in water and solubility products of silver(I) hydroxide and
copper(II) hydroxide at 298 K are given below. Only the numerical values of the
solubility products are given.

hydroxide solubility/mol dm3 solubility product

AgOH 1.23 104 1.52 108

Cu(OH)2 1.77 107 2.20 1020

As a general standard, two ions are selectively separated when a maximum 0.01% of
the precipitated compound still remains as the aqueous ions.

Determine if selective precipitation can be performed on a solution containing


0.200 mol dm3 Ag+ and 0.200 mol dm3 Cu2+ with the addition of solid potassium
hydroxide. [3]

Since solubility of Cu(OH)2 is lower than AgOH, it will be precipitated first.

To maximally precipitate Cu(OH)2, AgOH has to be saturated.

8
1.52 10
[OH] when AgOH is sat. = = 7.60 108 mol dm3
0.200

20
2+ 2.20 10
[Cu ] when AgOH is sat. = 2 = 3.81 106 mol dm3
7.60 10-8

6
2+ 3.81 10
%[Cu ] remaining = 100% = 0.001905% 0.00191% < 0.01%
0.200

Therefore Cu2+ can be selectively precipitated from Ag+.

Examiners comments
The question was poorly attempted. Most candidates scored 1 mark at best for the
calculation of [OH] when [Ag+]=0.200 mol dm3, even though they had not fully
appreciated why that value of [Ag+] was used.

The question had asked for students to "Determine" if selective precipitation can be
performed. Determine often implies that the quantity concerned cannot be measured
directly but is obtained via calculation, substituting measured or known values of other
quantities into a standard formula. As such, students have to utilise the value of 0.01%
to determine the percentage of [Cu2+] remaining, just before AgOH starts precipitate.

trendyline
ACJC2016 9647/03/Prelim/2016 [Turn over
83 trendyline
ANDERSON JUNIOR COLLEGE
2016 JC2 PRELIMINARY EXAMINATIONS

CHEMISTRY 9647/01
Higher 2 22 September 2016
Paper 1 Multiple Choice 1 hour

Additional Materials: Multiple Choice Answer Sheet


Data Booklet

READ THESE INSTRUCTIONS FIRST

Write in soft pencil.


Do not use staples, paper clips, glue or correction fluid.

There are forty questions on this paper. Answer all questions. For each question there are four
possible answers A, B, C and D.
Choose the one you consider correct and record your choice in soft pencil on the Multiple
Choice Answer Sheet.

Each correct answer will score one mark. A mark will not be deducted for a wrong answer.
Any rough working should be done in this booklet.
The use of an approved scientific calculator is expected, where appropriate.

Multiple Choice Answer Sheet

Write your name, PDG and NRIC / FIN number, including the reference letter.

Shade the NRIC / FIN number.

Exam Title: JC2 Prelim

Exam Details: H2 Chemistry / Paper 1

Date: 22/09/2016

trendyline
y This document consists of 18 printed pages.

AJC JC2 Prelim 2016 9647/01/H2 [Turn over


84 trendyline
2

Section A

For each question there are four possible answers, A, B, C and D. Choose the one you consider to
be correct.

1 Antimony (Sb) can be produced in a twostage process from its ore stibnite, Sb 2 S 3 .

The ore is first roasted in oxygen, producing Sb 4 O 6 and SO 2 .

The Sb 4 O 6 is then reduced by carbon, producing Sb and CO 2 .

What volume of CO 2 , measured at room temperature and pressure, is produced from


20 moles of Sb 2 S 3 ?

A 360 dm3 B 670 dm3 C 720 dm3 D 1440 dm3

2 Alpha particles, 4He2+, are commonly emitted by large radioactive nuclei.

The path of a mixture of 16O2+ ions, alpha particles and two unknown ions, F and G, upon
entering an electric field is shown below.

Source

16
alpha F O2+ G
particles

Which of the following represent F and G?

F G
1
A H+ 9
Be2+
27
B 12
C2+ Al3+
9
C Be2+ 14
N3+
12
D C+ 1
H+

trendyline
AJC JC2 Prelim 2016 9647/01/H2 [Turn over
85 trendyline
3

3 In a research on the atomic nucleus, scientists have been comparing the stability of isotopes
with the same neutron : proton ratio.

Which isotope has the same neutron : proton ratio as 10B?


32 32 40 40
A P B S C Ar D K

4 Two glass vessels X and Y are connected by a closed valve.

X Y

X contains helium at 20 C at a pressure of 1 x 105 Pa. Y has been evacuated, and has three
times the volume of X. In an experiment, the valve is opened and the temperature of the
whole apparatus is raised to 100 C.

What is the final pressure in the system?

A 3.18 x 104 Pa
B 4.24 x 104 Pa
C 1.25 x 105 Pa
D 5.09 x 105 Pa

5 The interhalogen compound BrF 3 is a volatile liquid which autoionises.

2BrF 3 BrF 2 + + BrF 4

The electrical conductivity of BrF 3 decreases with increasing temperature.

Which statement is correct?

A The autoionisation process is endothermic and the shape of the cation is linear.
B The autoionisation process is endothermic and the shape of the cation is nonlinear.
C The autoionisation process is exothermic and the shape of the cation is linear.
D The autoionisation process is exothermic and the shape of the cation is nonlinear.

trendyline
AJC JC2 Prelim 2016 9647/01/H2 [Turn over
86 trendyline
4

6 In an experiment, it was found that complete combustion of 10.0 g of an alcohol, J, raised the
temperature of 100 g of water in a container from 25 oC to 75 oC.

Given that the process is only 65% efficient and the specific heat capacity of water is
4.18 J g1 K1, what is the total heat energy released per gram of J burnt?

A 2.09 kJ g1 B 3.22 kJ g1 C 13.5 kJ g1 D 20.8 kJ g1

7 Limestone is a sedimentary rock largely made up of different crystal forms of calcium


carbonate. At high temperatures, calcium carbonate decomposes to form calcium oxide and
carbon dioxide.

CaCO 3 (s) CaO(s) + CO 2 (g)

What are the correct signs of H and S for this decomposition?

H S
A
B +
C +
D + +

8 The circuit shown in the diagram was set up.

Which electrode reactions will occur on closing the switch?

anode reaction cathode reaction


A neither nickel nor copper is dissolved hydrogen is evolved
B copper is dissolved preferentially copper is deposited
C nickel is dissolved preferentially hydrogen is evolved
D nickel and copper are both dissolved copper is deposited

trendyline
AJC JC2 Prelim 2016 9647/01/H2 [Turn over
87 trendyline
5

9 The reaction of hydrogen peroxide with iodide ions in acidic solution can be monitored by an
initial rate method.

H 2 O 2 (aq) + 2H+(aq) + 2I(aq) 2H 2 O(l) + I2 (aq)

The following results were obtained for a series of experiments with different volumes of each
reagent used.

experiment volume of volume of volume of volume of time


number H 2 O 2 (aq) / cm3 H+(aq) / cm3 I(aq) / cm3 water / cm3 taken / s
1 40 40 20 0 33
2 20 40 20 20 66
3 40 20 30 10 22
4 20 40 30 10 44

What could be the mechanism of this reaction?

A H 2 O 2 + H+ H 2 O + OH+ (fast)
OH+ + 2I + H+ H 2 O + I2 (slow)
B H 2 O 2 + I H 2 O + IO (slow)
+
H + IO
HIO (fast)
HIO + H + I+
I2 + H 2 O (fast)

C 2H+ + 2I 2HI (fast)


2HI + H 2 O 2 I 2 + 2H 2 O (slow)
D H 2 O 2 + I + H+ H 2 O + HIO (fast)
HIO + I I 2 + OH (slow)
OH + H+ H2O (fast)

10 The same amount of electricity is passed through two electrolytic cells connected in series for
30 minutes. Cell 1 and Cell 2 contain aqueous solutions of Mn(NO 3 ) 2 and Au(NO 3 ) 3
respectively.

Given that 1.3 g of manganese is deposited on the cathode for Cell 1, what is the current used
for electrolysis and the mass of the gold deposited on the cathode for Cell 2?

current mass of gold


A 1.3 A 3.1 g
B 1.3 A 4.7 g
C 2.5 A 3.1 g
D 2.5 A 4.7 g

AJC JC2 Prelim 2016 9647/01/H2 [Turn over


88 trendyline
6

11 Cobalt forms many coloured complexes with ligands such as H 2 O and SCN.

A 100 cm3 solution of Co2+(aq) turns from pink to bright blue when 10 cm3 of KSCN(aq) is
added to the solution.

[Co(H 2 O) 6 ]2+(aq) + 4SCN(aq) [Co(SCN) 4 ]2(aq) + 6H 2 O(l)


pale pink bright blue

At equilibrium, [Co(H 2 O) 6 ]2+ and [Co(SCN) 4 ]2 are found to be present in a mole ratio of 1:10.

Given that lg K c for the equilibrium is 3.00, which of the statements is incorrect?

A The units of K c is mol4 dm12.


B At equilibrium, [SCN ] = 3.16 x 101 mol dm3.
C [Co(SCN) 4 ]2 is more stable than [Co(H 2 O) 6 ]2+.
D Dilution of the reaction mixture decreases the proportion of [Co(H 2 O) 6 ]2+ at equilibrium.

12 In which reaction is the underlined substance acting as a base?

A HNO 3 + H 2 SO 4 H 2 NO 3 + + HSO 4
B HSiO 3 + HCN CN + H 2 O + SiO 2
C HNO 2 + HCO 3 H 2 O + CO 2 + NO 2
D C 6 H 5 O + CH 2 ClCO 2 H C 6 H 5 OH + CH 2 ClCO 2

trendyline
AJC JC2 Prelim 2016 9647/01/H2 [Turn over
89 trendyline
7

13 The change of colour for an indicator occurs over a limited range of pH that falls between
1.00 of the pK a value of the indicator.

Which is the most suitable indicator that can be used to determine the end point of the
corresponding titration?

alizarin yellow
A
(pK a = 11.0)

bromothymol blue
B
(pK a = 7.1)

methyl yellow
C
(pK a = 3.3)

thymol blue
D
(pK a = 1.6)

trendyline
AJC JC2 Prelim 2016 9647/01/H2 [Turn over
90 trendyline
8

14 The bar chart shows the melting points of a series of consecutive elements arranged in order
of increasing atomic number. The elements sodium to chlorine form part of this series.

Which bar represents sodium?

melting C
point / K

atomic number

15 Which oxide does not react with dilute sodium hydroxide to produce a salt?

A Al 2 O 3 B P 4 O 10 C SO 2 D SiO 2

16 The three minerals below are obtained from mines around the world. Each one behaves as a
mixture of two carbonate compounds. They can be used as fire retardants because they
decompose in the heat, producing CO 2 . This gas smothers the fire.

Barytocite BaCa(CO 3 ) 2

Dolomite CaMg(CO 3 ) 2

Huntite Mg 3 Ca(CO 3 ) 4

What is the order of effectiveness as fire retardant, from best to worst?

best worst
A dolomite barytocite huntite
B dolomite huntite barytocite
C huntite barytocite dolomite
D huntite dolomite barytocite

trendyline
AJC JC2 Prelim 2016 9647/01/H2 [Turn over
91 trendyline
9

17 Iodine is far less soluble in water than it is in aqueous potassium iodide, where it forms the
complex ion I 3 . For this reason, reactions involving aqueous iodine are often carried out in
potassium iodide solution.

Which equation describes the quantitative determination of iodine in the presence of excess
potassium iodide?

A I 2 + 2SO 4 2 2I + S 2 O 8 2
B 2I + 2S 2 O 3 2 I 2 + S 4 O 6 2
C I 3 + 2SO 4 2 3I + S 2 O 8 2
D I 3 + 2S 2 O 3 2 3I + S 4 O 6 2

18 Astatine, At, is below iodine in Group VII of the Periodic Table.

Which statement is most likely to be correct?

A AgAt(s) reacts with excess dilute aqueous ammonia to form a solution of a soluble
complex.
B At 2 and KCl(aq) react to form KAt(aq) and Cl 2 .
C KAt(aq) and dilute sulfuric acid react to form white fumes of HAt(g).
D NaAt(s) and concentrated sulfuric acid react to form At 2 .

19 In 1869 Ladenburg suggested a structure for benzene, C 6 H 6 , in which one hydrogen atom is
attached to each carbon atom.

Ladenburg structure

A compound C 6 H 4 Cl 2 could be formed with the same carbon skeleton as the Ladenburg
structure.

How many structural isomers would this compound have?

A 3 B 4 C 5 D 6

trendyline
AJC JC2 Prelim 2016 9647/01/H2 [Turn over
92 trendyline
10

20 Highenergy irradiation in the stratosphere produces radicals from chlorofluoroalkanes,


commonly known as CFCs.

Which radical could result from this irradiation of CHFClCF 2 Cl?


A CHFClCFCl

B CHClCF 2 Cl

C CHFCF 2 Cl

D CFClCF 2 Cl

21 Compound L has the following structure.

OH OH

What is the total number of geometrical isomers that can be formed from the product of the
reaction of compound L with excess concentrated sulfuric acid at 170 oC?

A 8 B 7 C 4 D 3

22 1,1dichloropropane reacts with excess hot aqueous sodium hydroxide in a series of steps to
give propanal.

NaOH(aq)
CH3CH2CHCl2 CH3CH2CHO

Which term describes the second step of this reaction?

A addition
B elimination
C oxidation
D substitution

trendyline
AJC JC2 Prelim 2016 9647/01/H2 [Turn over
93 trendyline
11

23 4chloroaniline is a pale yellow solid which is an important building block used for the
production of pesticides, drugs and dyestuffs.

Cl NH2

4chloroaniline

Which of the following shows a suitable starting compound and sequence of steps to produce
a good yield of 4chloroaniline?

starting compound step 1 step 2

conc. HNO 3 , Sn, conc. HCl


A Cl
conc. H 2 SO 4 followed by NaOH(aq)

B Cl dil. HNO 3 LiAlH 4

Sn, conc. HCl


C NO2 Cl 2 , AlCl 3
followed by NaOH(aq)

D NO2 LiAlH 4 Cl 2 in CCl 4

24 Why does hydrogen cyanide add to propanone but not to propene?

A Propanone is more susceptible to H+ attack than propene.


B Propanone is more susceptible to CN attack than propene.
C The C=C bond in propene is stronger than the C=O bond in propanone.
D The two methyl groups in propanone exert a stronger electron donating effect than the
single methyl group in propene.

25 Equal amounts of compounds P, Q, R and S are separately shaken with 100 cm3 of water.
The pH of each resultant solution is then measured.

CH 3 CH 2 CO 2 H CH 3 CH 2 COCl ClCH 2 CH 2 CO 2 H BrCH 2 CH 2 CO 2 H


P Q R S

Which of the following shows the correct order of decreasing pH of the solutions formed?

trendyline
A P, R, S, Q
B P, S, R, Q
C Q, P, S, R
D Q, S, R, P

AJC JC2 Prelim 2016 9647/01/H2 [Turn over


94 trendyline
12

26 Saccharin is an artificial sweetening agent which can be synthesised from methylbenzene in


the laboratory.

O
CH3 CH3 CH3 CO2H C NH
SO2Cl SO2NH2 SO2NH2 SO2

saccharin

What type of reaction is not shown in this reaction scheme?

A electrophilic substitution
B elimination
C nucleophilic substitution
D oxidation

27 Which pair of organic compounds cannot be distinguished by a chemical test?

O O
C C
A O and NH

B and

CH2OH CH2OH

C
and
HO CO2H HO CO2H

D
and

trendyline
AJC JC2 Prelim 2016 9647/01/H2 [Turn over
95 trendyline
13

28 Cortisol is a steroid hormone which is released in response to stress and low blood glucose.
Its structure is shown in the diagram below.

OH
O

HO OH

Cortisol is reduced with hydrogen in the presence of a platinum catalyst, and then oxidised by
heating with acidified KMnO 4 . The product formed is further reacted with excess sodium to
give an organic ion.

What is the charge on the organic ion produced?

A 1 B 2 C 3 D 5

29 Consider the four species below.

I II

NH3 Cl NH3

III IV

CH2NH3 NH2CH3

Which of the following shows the given species arranged in order of decreasing pK a values?

A III, IV, I, II
B III, IV, II, I
C IV, III, I, II
D IV, II, III, I

trendyline
AJC JC2 Prelim 2016 9647/01/H2 [Turn over
96 trendyline
14

30 The amino acids glutamine and glutamic acid can react with each other to form amide
linkages.

NH2 O O O

O OH HO OH
NH2 NH2

glutamine glutamic acid

What is the maximum number of different compounds, each containing one amide linkage,
that can be formed from one molecule of glutamine and one molecule of glutamic acid?

A 2
B 3
C 4
D 5

trendyline
AJC JC2 Prelim 2016 9647/01/H2 [Turn over
97 trendyline
15

Section B

For each of the questions in this section, one or more of the three numbered statements 1 to 3 may
be correct.

Decide whether each of the statements is or is not correct (you may find it helpful to put a tick
against the statements that you consider to be correct.)

The responses A to D should be selected on the basis of

A B C D
1, 2 and 3 1 and 2 2 and 3 1 only
are correct only are correct only are correct is correct

No other combination of statements is used as a correct response.

31 Which of the following ions has more electrons than protons and more protons than neutrons?
[ H = 11H ; D = 21H ; C = 126 C ; O = 168 O ]

1 OH
2 HCO 3
3 DCO 3

32 An energy level diagram is shown below for the case of dissolving lithium chloride in water.

Li+(g) + Cl(g)

energy

H1 H2

LiCl(s)

Li+(aq) + Cl(aq)

Which of the following about the enthalpy change of solution of lithium chloride is correct?

1 It is equal to H 1 + H 2 .
2
3
trendyline
It implies that dissolving lithium
ected
thium chloride
cted to be less exothermic
It is expected
loride is accompanied by an
a increas
exothermic than that of silver chloride
increase
chloride.
ncrea in temperature.

AJC JC2 Prelim 2016 9647/01/H2 [Turn over


98 trendyline
16

The responses A to D should be selected on the basis of

A B C D
1, 2 and 3 1 and 2 2 and 3 1 only
are correct only are correct only are correct is correct

No other combination of statements is used as a correct response.

33 Why does raising the pressure of a fixed mass of gaseous reactants at a constant
temperature cause an increase in the rate of reaction?

1 More collisions occur per second when the pressure is increased.


2 More molecules have energy greater than the activation energy at the higher pressure.
3 Raising the pressure lowers the activation energy.

34 Use of the Data Booklet is relevant to this question.

Which ions contain one or more unpaired electrons?

1 Cu2+
2 Mn3+
3 V3+

35 When a hot glass rod is placed in a gas jar of hydrogen iodide, there is an immediate reaction
as the hydrogen iodide decomposes.

Which statements about this reaction are correct?

1 Brown hydrogen iodide decolourises.


2 The hot rod provides the activation energy.
3 Purple fumes are observed.

36 Which of the following statements are correct about compound M?

z O
H2C C CH C H
y
x
CN

compou M
compound

1 d angles in compound
The bond compou M increase
increa in the order
orde x < y < zz..
2 There is only one sp hybridised carbon atom
om in compound M.
3 Compound M is planar.
AJC JC2 Prelim 2016 9647/01/H2 [Turn over
99 trendyline
17

37 A reaction pathway diagram is shown.

energy

reaction progress

Which reactions would have this profile?

1 (CH 3 ) 3 CBr + NaOH (CH 3 ) 3 COH + NaBr


2 CH 3 CH 2 Br + NaOH CH 3 CH 2 OH + NaBr
3 (CH 3 ) 3 CCH 2 CH 2 Cl + 2NH 3 (CH 3 ) 3 CCH 2 CH 2 NH 2 + NH 4 Cl

38 Which of the following reactions will produce a racemic mixture?

1 CH 3 CH 2 C(CH 3 ) 2 Cl heated under reflux with aqueous KOH.


2 CH 3 CHO with HCN and a small amount of KOH.
3 CH 3 CH=CHCH 3 with HBr dissolved in CCl 4 .

39 Compound N can react with alkaline aqueous iodine to form a yellow precipitate. When
compound N reacts with hot acidified KMnO 4 , two products are obtained. Both products can
also form a yellow precipitate with alkaline aqueous iodine.

What could be the identity of compound N?

1
OH

O
2
OH O

O
O
3

trendyline
O

AJC JC2 Prelim 2016 9647/01/H2 [Turn over


100 trendyline
18

The responses A to D should be selected on the basis of

A B C D
1, 2 and 3 1 and 2 2 and 3 1 only
are correct only are correct only are correct is correct

No other combination of statements is used as a correct response.

40 Haemoglobin is a type of protein found in red blood cells. It transports oxygen from the lungs
to body tissues. A haemoglobin molecule is a tetramer made up of four globular protein
subunits. Each subunit is composed of a protein chain tightly associated with a nonprotein
haem group.

The diagram below shows the structure of a haem group.

CH2
CH3
CH2
H3C
N N
II
Fe

N N
H3C CH3

HO OH
OH O

Which of the following is correct about haemoglobin?

1 Haemoglobin has higher affinity for carbon monoxide than oxygen.


2 Each haemoglobin molecule can carry four oxygen molecules.
3 The polypeptide chains in haemoglobin are held together by hydrogen bonding only.

trendyline
AJC JC2 Prelim 2016 9647/01/H2 [Turn over
101 trendyline
H2 Chemistry 9647 2016 JC2 Prelim P1 Worked Solutions

1 2Sb 2 S 3 + 9O 2 Sb 4 O 6 + 6SO 2 C

Sb 4 O 6 + 3C 4Sb + 3CO 2

2Sb 2 S 3 3CO 2
20Sb 2 S 3 30CO 2

Volume of CO 2 produced at r.t.p. = 30 x 24 = 720 dm3

2 ionic charge B
Angle of deflection, (i.e. z/m ratio)
mass
Arranging the species in order of decreasing angle of deflection, :

27
species 1
H+ 4
He2+ 9
Be2+ 14
N3+ 12
C2+ 16
O2+ Al3+ C+
12

z/m 1.00 0.500 0.222 0.214 0.167 0.125 0.111 0.0833


identity F F F G G

3 no. of neutrons no. of protons ratio B


10
B 5 5 1:1
32
P 17 15
32
S 16 16 1:1
40
Ar 22 18

4 p1V1 p 2 V2 A
Using
T1 T2
p VT
p2 1 1 2
V2 T1
1 x 10 5 x V1 x 373

(3V1 V1 ) x 293
1 x 10 5 x 1 x 373

4 x 293
3.18 x 10 4 Pa

5 2BrF 3 BrF 2 + + BrF 4 D

Since the electrical conductivity of BrF 3 decreases (i.e. the position of equilibrium shifts
left) when temperature increases, the reverse endothermic process is favoured to
absorb the extra heat. Hence, the forward autoionisation process is exothermic.

F Br F

trendyline
Since there are atom, BrF 2 + has a
re 2 bond pairs and 2 lone pairs around the central Br atom
bent (nonlinear)
ear)) shape.
ear

2016AndersonJC/CHEM 1
102 trendyline
6 100 B
Total heat released = ( 100 4.18 50) = 32.2 kJ
65
1
Total heat released per gram = (32.154) = 3.22 kJ g1
10

7 Since n gas = +1 mol; S > 0. D


Since G < 0 only at high temperatures, H > 0.
(Since reaction involves breaking of CO bond during thermal decomposition, H > 0.)

8 At the positive electrode (anode): C

species Eo
Cl 2 /Cl +1.36
Cu2+/Cu +0.34
Ni2+/Ni 0.25

Since Eo(Ni2+/Ni) is the least positive, Ni will be oxidised to Ni2+.

At the negative electrode (cathode):

species Eo
H+/H 2 0.00

H+ will be reduced to H 2 .

9 Comparing expt. 1 and 2, B


When volume of H 2 O 2 is halved, time taken is doubled (i.e. rate is halved).
first order w.r.t. H 2 O 2

Comparing expt. 2 and 4,


When volume of I increased 1.5 times, time taken decreased 1.5 times (i.e. rate
increased 1.5 times).
first order w.r.t. I

Comparing expt. 3 and 4,


When volume of H 2 O 2 is halved and volume of H+ is doubled, time taken is doubled (i.e.
rate is halved).
Since reaction is first order w.r.t. H 2 O 2 , increasing [H+] has no effect on the time taken.
zero order w.r.t. H+

Rate equation: rate = k [H 2 O 2 ] [I]


one molecule of H 2 O 2 and one I ion are involved in the slow step (r.d.s) of the
mechanism

10 Mn2+ + 2e Mn C

n(Mn) = 1.3 / 54.9 = 0.0237 mol


n(e) = 2 x 0.0237 = 0.0474 mol

Using Q = n e F

trendyline
Q = 0.0474 x 96500 = 4574.1

Using Q = It, I = Q/t


I = 4574.1/ (30
4574.1 C

0 x 60) = 2.5
25A

2016AndersonJC/CHEM 2
103 trendyline
Au3+ + 3e Au

n(Au) = n(e) / 3 = 0.0474 / 3 = 0.0158 mol


mass of Au = 0.0158 x 197 = 3.1 g

11 [Co(H 2 O) 6 ]2+(aq) + 4SCN(aq) [Co(SCN) 4 ]2 (aq) + 6H 2 O(l) D

KC
[Co(SCN) ] ] 4
2

[Co(H O) ] ][SCN ]
2 6
2 4

mol dm-3
Units of K c
mol dm- 3 (mol dm- 3 )4

Units of K c = mol4 dm12 Hence, statement A is correct.

10
10 3
1 [SCN ] 4

[SCN] = 3.16 x 101 mol dm3 Hence, statement B is correct.

Since K c is more than 1, the position of equilibrium lies to the right. This indicates that
[Co(SCN) 4 ]2 is a more stable complex ion than [Co(H 2 O) 6 ]2+. Hence, statement C is
correct.

Dilution of the reaction mixture decreases the concentration of ions in the solution.
Hence, the position of equilibrium shifts to the left to increase the concentration of
ions. This increases the proportion of [Co(H 2 O) 6 ]2+. Hence, statement D is incorrect.

12 HNO 2 + HCO 3 H 2 O + CO 2 + NO 2 C

HCO 3 acts as a Bronsted base which accepts a proton from HNO 2 , a Bronsted acid.
The H 2 CO 3 formed is unstable and decomposes to H 2 O and CO 2 .

13 The working pH range of an indicator is between pK a 1.00 and pK a + 1.00. B

A suitable indicator will be one where the working pH range of the indicator lies within
the range of rapid pH change at the endpoint of the titration.

Hence, only the indicator in B fulfils the condition.

14 B
Si

Mg

e Na

ne
2016AndersonJC/CHEM 3
104 trendyline
15 reacts with D
oxide nature of oxide remarks
dil. NaOH
Al 2 O 3 amphoteric yes
P 4 O 10 acidic yes
SO 2 acidic yes
only reacts with hot and concentrated
SiO 2 acidic no
strong alkalis to form silicates

16 Each mineral behaves as a mixture of two carbonate compounds. D

mineral formula behaves as a mixture of


barytocite BaCa(CO 3 ) 2 1 mol BaCO 3 and 1 mol CaCO 3
dolomite CaMg(CO 3 ) 2 1 mol CaCO 3 and 1 mol MgCO 3
huntite Mg 3 Ca(CO 3 ) 4 3 mol MgCO 3 and 1 mol CaCO 3

The effectiveness of each of the three minerals as fire retardant is dependent on its
ease of thermal decomposition to produce CO 2 , which smothers the fire.

The ease of thermal decomposition of the minerals is dependent on the charge density
and hence the polarising power of the respective Group II metal ions (Ba2+, Ca2+ and
Mg2+).

The order of effectiveness as fire retardant, from best to worst, corresponds to the order
of decreasing polarising power of the Group II metal ions: Mg2+, Ca2+, Ba2+.

Huntite is a more effective fire retardant than dolomite as huntite contains more MgCO 3
and hence it produces more CO 2 upon complete thermal decomposition.

17 I 2 undergoes a redox reaction with S 2 O 3 2, forming I and S 4 O 6 2. (Recall from Group D


VII)

In the presence of excess KI, I 2 forms the complex ion I 3 . Hence, the overall equation
is as shown:
I 3 + 2S 2 O 3 2 3I + S 4 O 6 2

18 Since At is below I in Group VII, D


Similar to AgI, AgAt is likely to be insoluble in excess dilute aqueous NH 3
At 2 is a weaker oxidising agent than Cl 2 and hence, At 2 is unable to oxidise Cl to
form At and Cl 2
KAt and concentrated sulfuric acid react to form At 2 (HAt formed initially is oxidised
by the sulfuric acid)

19 The compound C 6 H 4 Cl 2 has the following three structural isomers: A

CCl CCl
CCl CCl

trendyline
endyy
e
CCl
Cl
ClC
Cl

2016AndersonJC/CHEM 4
105 trendyline
20 bond bond energy / kJ mol1 C
CF > 410
CH 410
CCl 340

Irradiation of CHFClCF 2 Cl results in homolytic fission of the weakest CCl bond,


forming the CHFCF 2 Cl radical.

21 OH OH D
excess conc. H2SO4

170 oC

The three geometrical isomers of the product are as follows:

trans trans trans cis cis cis

Note: The following two structures are equivalent.

trans cis cis trans

22 H B
NaOH(aq) - H2O
CH3CH2CHCl2 CH3CH2 C OH CH3CH2 C H
nucleophilic elimination
substitution OH O

23 conc HNO3 A
conc H2SO4
Cl O2N Cl

1. Sn, conc HCl


2. aq NaOH

trendyline
nd
dy
H2N Cl

Note that NO
O 2 is 3directing and NO2 cannot be reduced using
us LiAlH 4 .

2016AndersonJC/CHEM 5
106 trendyline
24 The CN nucleophile attacks the electron deficient carbon in propanone in the B
nucleophilic addition mechanism.

25 Q hydrolyses in water to produce a strongly acidic solution of CH 3 CO 2 H and HCl. B


Hence, the resulting solution formed has the lowest pH.

P, R and S are weak acids which ionise partially in water.


The presence of electronegative atoms (Cl and Br) on R and S exerts an electron
withdrawing effect on ClCH 2 CH 2 COO and BrCH 2 CH 2 COO respectively, which
disperses the negative charge and stabilises the anion. Hence, the dissociation of
CH 3 CH 2 COOH to give H+ ions is least favourable. The resulting solution from P has the
highest pH.

Since Cl is more electronegative than Br, it exerts a stronger electronwithdrawing


effect on ClCH 2 CH 2 COO and hence ClCH 2 CH 2 COOH is more acidic than
BrCH 2 CH 2 CO 2 H. Hence, the resulting solution from R has a lower pH than S.

26 CH3 CH3 CH3 CO2H B


SO2Cl SO2NH2 SO2NH2

oxidation
electrophilic
substitution
nucleophilic
substitution /
condensation

O
C NH
SO2

saccharin

27 A: aqueous KMnO 4 , dilute H 2 SO 4 , heat D


For the compound on the left, purple solution decolourises.
(hydrolysis of ester + oxidation of primary alcohol)
For the compound on the right, purple solution remains.
B: aqueous KMnO 4 , dilute H 2 SO 4 , heat
For the compound on the left, purple solution remains.
For the compound on the right, purple solution decolourises.
(sidechain oxidation on benzene ring)
C: aqueous Br 2 / neutral FeCl 3
For the compound on the left, orange solution remains / no violet colouration
observed.
For the compound on the right, orange solution decolourises / violet

trendyline
ation
tio observed.
colouration

2016AndersonJC/CHEM 6
107 trendyline
28 O B
COO- Na+

O O- Na+

29 The four species given are conjugate acids of their corresponding amines. A

A low pK a value for its conjugate acid will indicate a high pK b value for the amine.

A decreasing pK a value for the conjugate acids of the amines also indicates an
increasing pK b value for the amines.

Hence, the species can be arranged in the order of decreasing basicity of the
corresponding amine.

amine of I amine of II

NH2 Cl NH2

amine of III amine of IV

CH2NH2 NHCH3

Amine of III is the most basic as the electrondonating alkyl group (C 6 H 5 CH 2 )


increases the availability of lone pair on N.

Amines I, II and IV are less basic than that of III since the lone pair on N can be
delocalised into the electron cloud of the benzene ring, decreasing the availability of
the lone pair.

Amine of IV is more basic than that of I and II as it is a secondary amine. The presence
of the electrondonating alkyl group (CH 3 ) increases the availability of the lone pair as
compared to amines of I and II.

Amine of II is less basic than that of I as the presence of electronwithdrawing Cl


decreasing the availability of the lone pair on N to a greater extent.

trendyline
2016AndersonJC/CHEM 7
108 trendyline
30 Possible combinations: B

Glutamine
NH 2 CO 2 H
Glutamic acid

NH 2 3 different compounds can


be formed
CO 2 H (1)

CO 2 H (2)

31 no. of protons no. of electrons no. of neutrons B



OH 9 10 8
HCO 3 31 32 30
DCO 3 31 32 31

32 Li+(g) + Cl(g) B

energy

H1 H2

LiCl(s) Hsol
Only options 1 and 2 are correct: Li+(aq) + Cl(aq)
1. By Hess law, H sol (LiCl) = H 1 + H 2 .
2. From the energy level diagram, LiCl(s) Li+(aq) + Cl(aq) is an exothermic
process. Hence, the process is accompanied by an increase in temperature.
3. Silver chloride is sparingly soluble while lithium chloride is soluble. Hence,
H sol (LiCl) is expected to be MORE exothermic than H sol (AgCl).

33 1. When pressure is increased, the gaseous reactant particles become closer together. D
Hence, the frequency of effective collisions increases.
2. More molecules have energy greater than the E a at a higher temperature.
3. Raising the pressure has no effect on the E a . The E a is lowered only in the
presence of a catalyst.

34 electronic config. no. of unpaired electrons A


1. Cu2+ [Ar] 3d9 1
2. Mn3+ [Ar] 3d4 4
3. V3+ [Ar] 3d2 2

trendyline
35 2HI H 2 + I2 C

1. Gaseous hydrogen iodide appears as a white fumes NOT brown


brow
od
2. The hot rodd provides the activation energy to break the H
HI bond
HI
3. The I 2 formed
med appears as purple fumes (violet
ole black I 2 solids are also observed)

2016AndersonJC/CHEM 8
109 trendyline
36 D

37 The reaction pathway diagram illustrates a twostep reaction mechanism. D

1. Tertiary RX undergoes a twostep S N 1 mechanism


2. Primary RX undergoes a single step S N 2 mechanism
3. Same as option 2

38 The nucleophile (OH, CN, Br) can attack the carbocation / planar molecule from C
either side with equal probability in the S N 1 / nucleophilic addition / electrophilic addition
mechanism.

A racemic mixture will not be formed in (1) since the product is not even chiral.

39 Compound 1 will undergo oxidative cleavage and compound 2 will undergo hydrolysis B
when reacted with hot acidified KMnO 4 .

H O
will form a yellow ppt with
Compounds with the structure CH3 C or CH3 C
alkaline aqueous iodine.
OH

40 Option 1: Fe2+ in haemoglobin forms a stronger dative covalent bond with carbon B
monoxide than oxygen.
Option 2: Each haemoglobin molecule contains 4 haem groups. One molecule of
oxygen binds to the Fe2+ on each haem group.
Option 3: The polypeptide chains in haemoglobin are held together by R group
interactions.

trendyline
2016AndersonJC/CHEM 9
110 trendyline
ANDERSON JUNIOR COLLEGE
2016 JC2 PRELIMINARY EXAMINATIONS

NAME:______________________________ PDG:_____ /15 Register No: _____

CHEMISTRY 9647/02
Higher 2 16 September 2016
Paper 2 Structured Questions 2 hours

Candidates answer on the Question Paper.


Additional Materials: Data Booklet

READ THESE INSTRUCTIONS FIRST

Write your name, PDG and register number on all the work you hand in.
Write in dark blue or black pen.
You may use a pencil for any diagrams or graphs.
Do not use staples, paper clips, glue or correction fluid.

Answer all questions.


The use of an approved scientific calculator is expected, where appropriate.
A Data Booklet is provided.

At the end of the examination, fasten all your work securely together.
The number of marks is given in brackets [ ] at the end of each question or part question.

For Examiners Use

3
Paper 2

4
Total / 72
5

tren
trendyline
tr ndyline
dyline 6

This document consists of 20 printed pages.

AJC JC2 Prelim 2016 9647/02/H2 [Turn over


111 trendyline
2

1 Planning (P)

Group II nitrates decompose on heating to form metal oxide, nitrogen(IV) oxide and oxygen.

Group II nitrates and their oxides are solids that can be easily weighed to determine the amount
present. Nitrogen(IV) oxide is an acidic gas and can be absorbed by a known amount of excess
strong alkali such as sodium hydroxide.

2NaOH + 2NO 2 NaNO 2 + NaNO 3 + H 2 O

The amount of strong alkali remaining can then be determined by titrating against a standard
solution of strong acid.

The oxygen produced from the decomposition can be trapped by a simple gas collection setup to
measure the volume collected.

You are to plan an experiment to confirm that the molar quantities of magnesium oxide,
nitrogen(IV) oxide and oxygen produced agree with the equation for the thermal decomposition of
magnesium nitrate.

(a) Write an equation for the thermal decomposition of magnesium nitrate. You should include
state symbols in your equation.

.. [1]

(b) Suggest the capacity of the apparatus to be used to collect the oxygen gas.

Hence calculate the mass of magnesium nitrate to be heated to produce a stated volume of
oxygen gas appropriate for the apparatus.

You should assume that one mole of any gas occupies 24.0 dm3 under room conditions.

capacity of apparatus:

mass of magnesium nitrate to be heated: . [2]

trendyline
AJC JC2 Prelim 2016 9647/02/H2 [Turn over
112 trendyline
3

(c) Use your answer to (b) to calculate the minimum volume of 1.00 mol dm3 aqueous sodium
hydroxide needed to absorb the nitrogen(IV) oxide produced from the thermal decomposition
of magnesium nitrate.

volume of sodium hydroxide needed: ........ [1]

(d) You may assume that you are provided with:

solid magnesium nitrate


1.00 mol dm3 sodium hydroxide
0.20 mol dm3 hydrochloric acid
the apparatus and chemicals normally found in a school or college laboratory

Your plan should include:

a diagram of the assembled apparatus capable of absorbing the nitrogen(IV) oxide


and collecting the oxygen separately and in sequence;
an outline of how the results would be obtained, including the essential details of the
titration procedure;
measures to ensure reliability of results;
brief, but specific, details of how the results will be used to

determine the amounts in moles of reactant and all products at the end of
complete thermal decomposition
confirm that the decomposition had occurred according to the molar ratios in the
equation.

trendyline
AJC JC2 Prelim 2016 9647/02/H2 [Turn over
113 trendyline
4

.............

.............

.............

.............

.............

.............

.............

.............

.............

.............

.............

.............

.............

.............

.............

.............

.............

.............

.............

.............

.............

.............

.............

.............

.............



.............



.............

.............

.............
AJC JC2 Prelim 2016 9647/02/H2 [Turn over
114 trendyline
5

.............

.............

.............

.............

.............

.............

.............

.............

.............

.............

.............

.............

.............

.............

.............

.............

.............

.............

.............

.............

.............

.............

.............

.............

.............

trendyline


.............



.. [8]

[Total: 12]

AJC JC2 Prelim 2016 9647/02/H2 [Turn over


115 trendyline
6

2 (a) Ethoxyethane, CH 3 CH 2 OCH 2 CH 3 , may be regarded as a water molecule in which each of the
hydrogen atoms has been replaced by an ethyl group.

At room temperature, boron trifluoride is a gas and ethoxyethane is a liquid. When boron
trifluoride is mixed in a 1 : 1 molar ratio with ethoxyethane, a liquid with a boiling point of 127 oC
is formed.

(i) Suggest what type of bond is formed between boron trifluoride and ethoxyethane,
explaining your answer.

...

...

[2]

(ii) Draw a diagram to illustrate the shape of and bonding in the product.

(You may represent the CH 2 CH 3 group with R)

[1]

Often in the preparation of an organic compound the product is obtained as an aqueous solution.
The chemist needs to separate the organic compound from water by using solvent extraction
method. Organic compounds are more soluble in organic solvents than in water.

Ethoxyethane is a good solvent for many organic compounds. It is immiscible with water and has a
low boiling point (35 oC). When an aqueous solution containing an organic compound is shaken with
ethoxyethane in a separating funnel, the compound distributes itself between the two layers, in
accordance with its partition coefficient between ethoxyethane and water, until equilibrium is
reached.

The partition coefficient, K partition , of an organic compound, J, between ethoxyethane and water is
given by the following expression.

trendyline
d li
concentration of J in
i ethoxyetha
ethane
n (organic layer
layer)
K partition
p
concentrat
oncentration of J in water (aqueous
aqueous layer)

ration of J can be expressed in g cm3


where the concentration
tration
.

The partition coefficient


i i t iis a constant
t t att a constant
t t temperature.
t t

AJC JC2 Prelim 2016 9647/02/H2 [Turn over


116 trendyline
7

(b) When 20 cm3 of ethoxyethane were shaken with 75 cm3 of an aqueous solution containing
5.00 g of J, it was found that 2.14 g of J were extracted into the ethoxyethane.

(i) Calculate the partition coefficient, K partition , of J between ethoxyethane and water.

[1]

(ii) In another experiment

10 cm3 of ethoxyethane were shaken with 75 cm3 of an aqueous solution containing


5.00 g of J and the layers were separated.
The aqueous layer was shaken with a second 10 cm3 portion of ethoxyethane and the
layers were separated.
The two organic layers were combined.

Use the value of K partition you calculated in (b)(i) to calculate the total mass of J extracted
by this procedure.

[2]

(iii) Hence comment whether it is more efficient to use one 20 cm3 portion of ethoxyethane or
two successive portions of 10 cm3 ethoxyethane for extraction.

..... [1]

trendyline
(c) alcium
lcium chloride was then added to the combin
Anhydrous calcium laye obtained in (b)(ii).
combined organic layers

eason for the addition of anhydrous calcium chloride


Suggest the reason chloride.

.........................
......
........................................................... [1]

[Total: 8]

AJC JC2 Prelim 2016 9647/02/H2 [Turn over


117 trendyline
8

3 Nitrogen and phosphorus are both Group V elements. Nitrogen exists in its elemental form as
simple molecules, N 2 , while phosphorus occurs in one of the many forms, including white
phosphorus and red phosphorus. Some data about these two forms of phosphorus are shown in
Table 1.1.

Table 1.1

white phosphorus red phosphorus


appearance at room
creamy white solid red solid
temperature
melting point / oC 44 590

solubility in methylbenzene soluble insoluble

(a) Suggest the type of structure and bonding in red phosphorus. Explain your reasoning.

............

............

............

[2]

(b) The 2s and 2p orbitals of nitrogen atoms can hybridise in the same way as the 2s and 2p
orbitals of carbon atoms.

Suggest the type of hybridisation in N 2 and draw the arrangement of the hybrid orbital(s)
around each nitrogen atom.

type of hybridisation

[2]

trendyline
AJC JC2 Prelim 2016 9647/02/H2 [Turn over
118 trendyline
9

(c) The corresponding form of phosphorus, P 2 , is not stable under standard conditions. Instead,
white phosphorus consists of molecules with a tetrahedral structure as shown.

P P
P
white phosphorus

(i) Unlike N 2 , P 2 is not stable under standard conditions. Suggest a reason for this.

...

.. [1]

(ii) Using the data in Table 1.2, construct an appropriate energy cycle and use it to
determine the enthalpy change for the conversion of white phosphorus to the gaseous
diatomic molecule, P 2 .

P 4 (s) 2P 2 (g)

Table 1.2

PP bond energy 200 kJ mol1

PP bond energy 485 kJ mol1

enthalpy change of vaporisation of P 4 (s) +12 kJ mol1

trendyline [3]

AJC JC2 Prelim 2016 9647/02/H2 [Turn over


119 trendyline
10

(iii) The formation of P 2 (g) from white phosphorus is not spontaneous under standard
conditions.

Suggest how the conditions would need to be changed to make it spontaneous. Explain
your reasoning.

...

...

...

.. [2]

(d) Although nitrogen gas makes up about 79% of the atmosphere, it does not easily form
compounds.

(i) Explain why the conditions in a car engine lead to the production of oxides of nitrogen.

...

.. [1]

(ii) Give an equation for a reaction involved in the removal of nitrogen monoxide, NO, from
a cars exhaust gases, in the catalytic converter.

.. [1]

[Total: 12]

trendyline
AJC JC2 Prelim 2016 9647/02/H2 [Turn over
120 trendyline
11

4 (a) (i) What is meant by the term ligand in the context of transition element chemistry?

...

.... [1]

(ii) Although the five dorbitals are at the same energy in an isolated atom, when a
transition element ion is in an octahedral complex the orbitals are split into two groups,
as shown in the following diagram.

energy gap E

dorbitals of an isolated dorbitals of the transition element


transition element ion ion in an octahedral complex

Use this diagram as an aid in explaining the following.

Transition element complexes are often coloured.

...

...

...

...

...

...

The colour of a complex of a given transition element often changes when the
ligands around it are changed.

...

...

. [5]

trendyline
AJC JC2 Prelim 2016 9647/02/H2 [Turn over
121 trendyline
12

(b) Heating a solution containing potassium ethanedioate, iron(II) ethanedioate and hydrogen
peroxide produces the light green complex K 3 Fe(C 2 O 4 ) 3 , which contains the ion
[Fe(C 2 O 4 ) 3 ]3.

The structure of the ethanedioate ion is as follows.

-
O O-
C C
O O

(i) Determine the oxidation number of carbon in this ion.

.... [1]

(ii) Determine the oxidation number of iron in [Fe(C 2 O 4 ) 3 ]3.

.... [1]

(iii) The iron atom in the [Fe(C 2 O 4 ) 3 ]3 ion is surrounded octahedrally by six oxygen atoms.
Draw a threedimensional diagram to show the shape of this ion.

[2]

(c) Kidney stones are usually made up of an inorganic salt of calcium with ethanedioate. Thus,
people who are prone to kidney stones have to limit or avoid eating foods with high
ethanedioate content such as spinach, peanuts and sweet potatoes.

(i) Write the expression for the solubility product of calcium ethanedioate.

.... [1]

(ii) The value of K sp for calcium ethanedioate is 2.3 x 109. Calculate [C 2 O 4 2] in a


saturated solution of calcium ethanedioate.

trendyline
AJC JC2 Prelim 2016 9647/02/H2 [Turn over
122 trendyline
13
[1]

(iii) A solution W is saturated with both calcium ethanedioate and calcium chloride.

The concentration of ethanedioate ions in solution W is less than that calculated in


(c)(ii). Explain why this is so.

..........

........ [1]

(d) By quoting relevant data from the Data Booklet, explain the following observations:

compound pH of a 1.0 mol dm3 solution in water


NaCl 7.0
MgCl 2 6.5
AlCl 3 3.0

.. [2]

[Total: 15]

trendyline
AJC JC2 Prelim 2016 9647/02/H2 [Turn over
123 trendyline
14

5 (a) Kekul proposed the following structure for benzene.

The enthalpy change of hydrogenation of cyclohexene, as shown, is 121 kJ mol1.

+ H2

Based on this value for cyclohexene it is possible to calculate the enthalpy change of
hydrogenation of benzene, based on Kekuls structure, to be 363 kJ mol1.

Explain the difference between this calculated value and the actual value for the enthalpy
change of hydrogenation of benzene of 209 kJ mol1.

.....

........ [1]

(b) Benzene undergoes electrophilic substitution reactions.

Nitrobenzene, C 6 H 5 NO 2 , can be formed from benzene. Give the reagents and conditions
necessary for this process and identify the electrophile.

reagents ..

conditions ..............

electrophile ... [2]

trendyline
AJC JC2 Prelim 2016 9647/02/H2 [Turn over
124 trendyline
15

(c) Fig. 5.1 shows a reaction scheme starting from nitrobenzene.

1. Sn / conc. HCl 1. limited CH3CH2Br


C6H5NO2 C6H5NH2 X
2. excess NaOH 2. CH3COCl

Br2, FeBr3 Br2, CCl4


strong heat room temperature

BrC 6H4NO2 BrC 6H4NH2

Fig. 5.1

(i) Explain why the bromination of phenylamine, C 6 H 5 NH 2 , is possible with the milder
conditions shown in Fig. 5.1.

...

...

...

...... [2]

(ii) Write an equation for the reaction between nitrobenzene and the reducing mixture,
Sn / conc. HCl. Use [H] to represent the formula of the reducing agent in your equation.

[1]

(iii) Draw the structural formula of X.

[1]

trendyline
AJC JC2 Prelim 2016 9647/02/H2 [Turn over
125 trendyline
16

(d) When heated with chlorine, the hydrocarbon 2,2dimethylbutane produces compounds L, M
and N.

CH3 CH3 CH3 CH3


CH3CH2 C CH3 CH2CH2 C CH3 + CH3CH C CH3 + CH3CH2 C CH2
CH3 Cl CH3 Cl CH3 CH3 Cl

2,2-dimethylbutane L M N

It is observed that the rate of substitution of a hydrogen atom in 2,2dimethylbutane by a


chlorine atom is dependent on the type of hydrogen substituted primary, secondary or
tertiary.

The table below shows the relative rate of substitution by chlorine.

Reaction Type of hydrogen substituted Relative rate


RCH 3 RCH 2 Cl primary 1
R 2 CH 2 R 2 CHCl secondary 7
R 3 CH R 3 CCl tertiary 21

Using the information from the table and by considering the number of hydrogen atoms of
each type (primary, secondary or tertiary) within 2,2dimethylbutane, predict the ratio of the
three products L, M and N.

Explain how you arrived at your answer.

ratio of L to M to N: ..

explanation:

...........

.. [2]

[Total: 9]

trendyline
AJC JC2 Prelim 2016 9647/02/H2 [Turn over
126 trendyline
17

6 (a) The interhalogen compound ICl reacts with alkenes in an addition reaction. ICl reacts faster
with alkenes than the pure halogens.

(i) Describe the mechanism of the reaction between ICl and propene.

[3]

(ii) Suggest why ICl reacts with alkenes faster than the pure halogens, Cl 2 , Br 2 and I 2 .

.....

... [1]

Cyclohexa1,4diene behaves as a typical alkene.

cyclohexa1,4diene

(b) State the total numbers of bonds and bonds in a molecule of cyclohexa1,4diene.

number of bonds

number of bonds [1]

trendyline
AJC JC2 Prelim 2016 9647/02/H2 [Turn over
127 trendyline
18

(c) Cyclohexa1,4diene can be made by heating buta1,3diene with ethyne in the DielsAlder
reaction. The diagram below shows the movement of electron pairs, represented by curly
arrows, needed to generate the cyclohexa1,4diene in a single step.

heat

buta1,3diene ethyne cyclohexa1,4diene

(i) In a similar type of reaction, cyclohexene can be formed from buta1,3diene and
another alkene, Y.

+ Y
heat
buta1,3diene cyclohexene

Suggest the name of the alkene, Y, that would react with buta1,3diene to form
cyclohexene in this type of reaction.

... [1]

(ii) In another similar reaction, penta1,3diene reacts with propene to form two products
that are structural isomers.

Deduce the structures of these two isomers.

[2]

trendyline
AJC JC2 Prelim 2016 9647/02/H2 [Turn over
128 trendyline
19

(d) Chlorogenic acid occurs naturally in coffee and an edible species of bamboo.

OH

O O OH
OH
HO2C
OH
OH

chlorogenic acid

(i) Draw the structural formula of the compound formed when chlorogenic acid is treated
with an excess of sodium metal.

[1]

On heating with dilute acid, chlorogenic acid produces two compounds, B and C.

OH
O OH
dil. H+ (aq)
chlorogenic acid + C
heat
HO OH
OH
Br2(aq)
B in excess

conc. H2SO4
heat
E

trendyline
d D

AJC JC2 Prelim 2016 9647/02/H2 [Turn over


129 trendyline
20

(ii) State the type of reaction undergone by chlorogenic acid when B and C are formed.
Draw the displayed formula of compound C.

type of reaction

compound C
[2]

When compound B is heated with concentrated H 2 SO 4 , compound D, C 7 H 6 O 3 , is formed.

CO 2 is evolved when compound D is treated with Na 2 CO 3 (aq). Compound D decolourises


Br 2 (aq) giving a white precipitate, but does not react with cold, alkaline KMnO 4 .

When compound C is treated with an excess of Br 2 (aq), compound E is produced.

(iii) Identify the functional group that would have been shown to be present in D if the test
with cold, alkaline KMnO 4 had been positive.

. [1]

(iv) Name the functional groups in compound D that react with the following.

Na 2 CO 3 (aq) .. Br 2 (aq) .. [2]

(v) Suggest structures for compounds D and E.

trendyline
trendy
yline
D E
[2]

[Total: 16]

AJC JC2 Prelim 2016 9647/02/H2 [Turn over


130 trendyline
H2 Chemistry 9647 2016 JC2 Prelim P2 Suggested Solutions

1 (a) 2Mg(NO 3 ) 2 (s) 2MgO(s) + 4NO 2 (g) + O 2 (g) [1]

OR

Mg(NO 3 ) 2 (s) MgO(s) + 2NO 2 (g) + O 2 (g)

(b) Correctly stated volume, with units, of apparatus to be used for gas collection.
E.g. 100 ml or 100 cm3 gas syringe [1]

AND

Correctly calculated mass of magnesium nitrate to produce a stated volume of O 2 gas


that would be appropriate for the stated volume of apparatus.

E.g. for 60 cm3 of O 2 ,


n(O 2 ) = 60/24000 = 2.50 x 103 mol
n[Mg(NO 3 ) 2 ] = 2 x n(O 2 ) = 5.00 x 103 mol
mass of Mg(NO 3 ) 2 = 5.00 x 103 x [24.3 + 2(14.0) + 6(16.0)] = 0.742 g [1]

(c) 2Mg(NO 3 ) 2 (s) 2MgO(s) + 4NO 2 (g) + O 2 (g)


2NaOH + 2NO 2 NaNO 2 + NaNO 3 + H 2 O

n(NO 2 ) = 2 x n[Mg(NO 3 ) 2 ] = 2 x 5.00 x 103 = 0.0100 mol


n(NaOH) = n(NO 2 ) = 0.0100 mol
min. volume of 1.00 mol dm3 NaOH needed = 0.0100 / 1.00 x 1000 = 10.00 cm3 [1]

(d) A diagram of the assembled apparatus capable of absorbing the nitrogen(IV) oxide and [2]
collecting the oxygen separately and in sequence

Directly heated vessel (boiling tube) labelled magnesium nitrate with tube at exit
Gas stream led into a liquid labelled (1.00 mol dm3) NaOH which will absorb the
nitrogen(IV) oxide / NO 2
Collects a gas in a syringe or over a liquid, provided it is properly connected
All parts of the apparatus are connected and airtight AND nitrogen(IV) oxide
absorption precedes oxygen collection

trendyline
2016AndersonJC/CHEM 1
131 trendyline
An outline of how the results would be obtained and measures to ensure reliability of
results
[4]
1. Record the mass of the empty boiling tube.
2. Weigh accurately about 0.75 g of magnesium nitrate into a boiling tube.
3. Using a burette, add 20.00 cm3 of 1.00 mol dm3 sodium hydroxide into the
chamber collecting nitrogen(IV) oxide / NO 2 .
4. Set up the apparatus according to the diagram above.
5. After ensuring that all the connections are properly sealed, start heating the
magnesium nitrate in the boiling tube. Heat strongly for about 5 minutes, moving
the boiling tube to ensure even heating of the solid. Continue heating until no
further bubbling is seen in the sodium hydroxide solution and the syringe shows no
further change in volume of oxygen gas collected.
6. Remove the delivery tube from the alkali before taking the boiling tube off the
flame. (To prevent back flow of liquid into the boiling tube due to cooling of air)
7. Allow all the apparatus and chemicals to cool to room temperature before taking
measurements.
8. Weigh the boiling tube and residue. Subtract the mass of the empty boiling tube to
determine the mass of magnesium oxide produced.
9. Transfer all the sodium hydroxide solution into a 100 cm3 volumetric flask. Top up
to the mark with distilled water. Stopper the flask and shake well to obtain a
homogeneous solution.
10. Pipette 25.0 cm3 of the solution into a 250 cm3 conical flask for titration.
11. Add a few drops of methyl orange indicator. Titrate with the 0.20 mol dm3
hydrochloric acid from the 50.00 cm3 burette until the solution turns from yellow to
orange.
12. Repeat the titration until consistent results within 0.10 cm3 are obtained.
13. Record the volume of oxygen gas collected in the syringe.

Brief, but specific, details of how results will be used to determine the amounts in moles
of reactant and all products at the end of complete thermal decomposition AND confirm
that the decomposition had occurred according to the molar ratios in the equation

n(MgO) = mass of MgO / M r of MgO = x mol

n(NO 2 ) = n(NaOH that reacted with NO 2 )


= (20/1000 x 1) [(vol. of HCl/1000 x 0.20) x 100/25.0] = y mol [1]

n(O 2 ) = volume of O 2 /24000 = z mol

Compare each of the values x, y and z with n[Mg(NO 3 ) 2 ] XVHG 5.00 x 103 mol) to
check if the decomposition has occurred according to the molar ratios in the equation. [1]

[Total: 12]

trendyline
2016AndersonJC/CHEM 2
132 trendyline
2 (a) (i) B atom in BF 3 molecule is electrondeficient as it has only 6 electrons around it. [2]
Hence, the B atom can accept another 2 electrons from the lone pair of O atom in
ethoxyethane molecule. By forming a dative bond with the O atom of
CH 3 CH 2 OCH 2 CH 3 , the B atom achieves the octet configuration.

(ii) O [1]
F CH2CH3
B
CH2CH3
F
F

(b) (i) concentration of J in ethoxyethane (organic layer)


K partition
concentration of J in water (aqueous layer)
2.14 / 20
2.81 [1]
(5 2.14) / 75

(ii) 1st extraction [2]

2.81 = (x / 10) / [(5 x) / 75]


2.81 (5 x) = 7.5 x
x = 1.36 g

2nd extraction

Mass of J remaining in aqueous layer after 1st extraction = 5.00 1.36 = 3.64 g

2.81 = (y / 10) / [(3.64 y) / 75]


2.81 (3.64 y) = 7.5 y
y = 0.992 g

total mass of J extracted = 1.36 + 0.992 = 2.35 g

(iii) It is more efficient to use two successive portions of 10 cm3 ethoxyethane than [1]
one 20 cm3 portion of ethoxyethane since total mass of J extracted is greater
using two successive portions.

(c) Anhydrous calcium chloride (drying agent) was added to remove water present in the [1]
organic layer.

[Total: 8]

trendyline
2016AndersonJC/CHEM 3
133 trendyline
3 (a) Giant covalent structure with strong covalent bonds between P atoms. [1]

Relatively high melting point OR insolubility in organic solvent (methylbenzene) [1]


suggests strong bonding throughout the structure.

(b) type of hybridisation: sp [1]

[1]

(c) (i) Phosphorusphosphorus triple bonds are much weaker than nitrogennitrogen [1]
triple bonds as phosphorus atoms are too big to come close enough together to
form strong bonds.

OR

Orbitals of P are larger and more diffused hence overlap of orbitals is less
effective.

(ii) H [2]
P 4 (s) 2P 2 (g)

+12 2(485)

P 4 (g) 4P(g)
6(+200)

H = +12 + 6(+200) + 2(485)


= +242 kJ mol1 [1]

(iii) Increase temperature [1]

G = H TS
Since H > 0, S > 0 (as there is an increase in no. of moles of gas),
for G to be negative (i.e. spontaneous), magnitude of TS must be greater than
that of H. Hence, T must be large. [1]

(d) (i) High temperature (needed for reaction between N 2 and O 2 ) [1]

(ii) 2NO + 2CO N 2 + 2CO 2 [1]


OR 2NO + C N 2 + CO 2

[Total: 12]

trendyline
2016AndersonJC/CHEM 4
134 trendyline
4 (a) (i) A ligand is a neutral molecule or an anion which possesses at least one lone pair [1]
of electrons which can be used to form dative bonds with the central atom or ion.

(ii) Transition element complexes are often coloured:

In the presence of ligands, the d orbitals of transition metal ion are split into two
levels.

The energy gap E between the nondegenerate orbitals is small and corresponds [1]
to that in the visible light region.

Energy is absorbed from the visible region when an electron is promoted from [1]
a lower level d orbital to a vacant higher level d orbital (dd transition).

The colour of the transition metal complex is the complement of the colour [1]
absorbed.

The colour of a complex of a given transition element often changes when the
ligands around it are changed:

The size / magnitude of the energy gap E depends on the ligand. [1]

(Since E = hc / ),
When the ligand changes, E changes, (absorbed) also changes (hence colour of [1]
the complex changes).

(b) (i) Let O.N. of C in C 2 O 4 2 be x

2(x) + 4(2) = 2
x = +3 [1]

(ii) Let O.N. of Fe in [Fe(C 2 O 4 ) 3 ]3 be y

y + 3(2) = 3
y = +3 [1]

(iii) 3- [2]
O
O
O
O O O

Fe III
O
O O
O
O

trendyline
dy
O

(c) (i) K sp = [Ca ][ 2 O 4 22]


Ca2++][C [1]

(ii) [C 2 O 4 2] = (2.3 x 109)1/2 = 4.80 x 105 mol dm3 [1]

2016AndersonJC/CHEM 5
135 trendyline
(iii) CaC 2 O 4 (s) Ca2+(aq) + C 2 O 4 2(aq)

Increase in [Ca2+] due to the presence of the common ion Ca2+ (from CaCl 2 ) in
solution W shifts the position of the above equilibrium to the left. Hence, [C 2 O 4 2] [1]
in solution W is less than that calculated in (c)(i).

(d) cation ionic radius / nm


Na+ 0.095
Mg2+ 0.065
Al3+ 0.050 [1]

The charge density of the cation increases from Na+ to Mg2+ to Al3+. Thus, the polarising
power of the cation increases in the same order.

As a result, NaCl does not hydrolyse in water (pH = 7); MgCl 2 hydrolyses only to
a small extent (pH = 6.5) and AlCl 3 hydrolyses to a larger extent (pH = 3.0). [1]

[Total: 15]

5 (a) The actual value for the enthalpy of hydrogenation of benzene is less exothermic [1]
as more energy is needed to break the bonds in benzene due to delocalisation of the
electrons.

OR

Benzene is resonancestabilised and hence has a lower energy level.

(b) reagents: conc. HNO 3 + conc. H 2 SO 4 [1]


conditions: 55 60 C
electrophile: NO 2 + [1]

(c) (i) In phenylamine, the lone pair on the N atom is delocalised into the benzene ring, [1]
making the ring more electronrich.

Hence, the ring is more susceptible to (electrophilic) attack and hence does not [1]
require the strong electrophile Br+ generated by the reaction between Br 2 and
FeBr 3 .

(ii) C 6 H 5 NO 2 + 6[H] C 6 H 5 NH 2 + 2H 2 O [1]

(iii) O [1]
CH3 C N CH2CH3

2016AndersonJC/CHEM 6
136 trendyline
(d) 3 : 14 : 9 [1]

Relative rate suggests 1 : 7 : 1, but there are 3 primary to 2 secondary to 9 primary [1]
hydrogen atoms in L, M and N respectively. Hence, this ratio becomes 3 : 14 : 9.

[Total: 9]

6 (a) (i) H H H H H [3]


H slow fast
C C CH3 H C C CH3 H C C CH3
H
I I Cl
I Cl-
Cl

(ii) ICl is polar while pure halogens are not. [1]

(b) number of bonds: 14 [1]


number of bonds: 2

(c) (i) ethene [1]

(ii) [2]

(d) (i) [1]


O-Na+

O O O-Na+
O O-Na+

+Na-O
O-Na+
O-Na+

(ii) (acid) hydrolysis [1]

O
OH
HO
[1]

trendyline
OH

e OR C=C
(iii) alkene C [1]

2016AndersonJC/CHEM 7
137 trendyline
(iv) Na 2 CO 3 (aq): carboxylic acid [1]
Br 2 (aq): phenol [1]

(v) OH
OH [1]
D:
OR
HO2C HO2C

Br Br
HO2C OH
E:
Br [1]
Br OH
Br
[Total: 16]

trendyline
2016AndersonJC/CHEM 8
138 trendyline
ANDERSON JUNIOR COLLEGE

2016 JC2 PRELIMINARY EXAMINATIONS

CHEMISTRY 9647/03
Higher 2 21 September 2016
Paper 3 Free Response 2 hours

Candidates answer on separate paper.


Additional Materials: Writing Paper
Data Booklet
Graph Paper

READ THESE INSTRUCTIONS FIRST

Write your name, PDG and register number on all the work you hand in.
Write in dark blue or black pen on both sides of the paper.
You may use a pencil for any diagrams or graphs.
Do not use staples, paper clips, glue or correction fluid.

Answer any four questions.


Start each question on a fresh sheet of paper.

A Data Booklet is provided.


The use of an approved scientific calculator is expected, where appropriate.
You are reminded of the need for good English and clear presentation in your answers.

At the end of the examination, fasten all your work securely together.
The number of marks is given in brackets [ ] at the end of each question or part question.

trendyline
his document consists of 12 printed
This rinted pages
pages.

AJC JC2 Prelim 2016 9647/03/H2 [Turn over


139 trendyline
2

1 (a) Transition elements show typical properties that distinguish them from sblock elements
such as calcium. The following table gives data about some physical properties of the
elements calcium, iron and copper.

property calcium iron copper


relative atomic mass 40.1 55.8 63.5
atomic radius (metallic) / nm 0.197 0.126 0.128
ionic radius (2+) / nm 0.099 0.076 0.069
o
melting point / C 839 1535 1085
3
density / g cm 1.54 7.86 8.92
6 1
electrical conductivity / x 10 S cm 0.298 0.100 0.596

(i) Explain why the atomic radii of iron and copper are similar to each other. [2]

(ii) The melting point of iron is significantly higher than that of calcium. Explain this in
terms of the type and strength of bonding in each metal.
[3]

(iii) Using relevant data from the table, explain why the densities of iron and copper are
significantly greater than that of calcium. (No calculations are required.)
[2]

(b) Cu(I) ions are unstable and readily undergo disproportionation in aqueous solution.

Write an ionic equation showing the disproportionation of Cu+(aq) ions, and choose relevant
Eo values from the Data Booklet to calculate E cell o for the reaction.
[2]

(c) Cu(I) is stable in insoluble salts and in some complexes. Complexes of Cu(I) are colourless
whereas complexes of Cu(II) are coloured. Use these facts to explain the following
observations, writing equations for all reactions.

(i) The addition of KI(aq) to CuSO 4 (aq) produces a white precipitate, CuI, and a brown
solution. The brown solution turns colourless when aqueous sodium thiosulfate,
Na 2 S 2 O 3 , is added, and the white precipitate remains.
[2]

(ii) On boiling with copper foil, the colour of a solution of CuCl 2 in concentrated
hydrochloric acid changes from blue to colourless. The colourless solution contains a
linear complex ion of copper and chlorine.

After filtering off the excess copper metal and diluting the solution with water, a white
precipitate is formed, which contains 35.9 % of chlorine by mass.
[3]

trendyline
AJC JC2 Prelim 2016 9647/03/H2 [Turn over
140 trendyline
3

(d) Some copper compounds have found a use as reagents for unusual organic reactions. An
example is the use of a mixture of copper(II) ethanoate and lead(IV) ethanoate for the
conversion of carboxylic acids into alkenes containing one less carbon atom. This reaction is
known as oxidative decarboxylation.

R R
Cu(OCOCH3)2 , Pb(OCOCH3)4 R R
R C C H C C
R H
H CO2H

secondary tertiary
carboxylic acid alkene

When a secondary carboxylic acid G, C 9 H 16 O 2 , was treated in this way, alkene H, C 8 H 14 ,


was formed. On treatment with hot acidified concentrated KMnO 4 , H produced a single
compound J, C 8 H 14 O 3 .

Compound J evolved CO 2 with Na 2 CO 3 (aq), gave an orange precipitate with


2,4dinitrophenylhydrazine reagent, and gave a yellow precipitate with alkaline aqueous
iodine.

When the yellow precipitate had been filtered off, acidification of the aqueous solution
produced 3,3dimethylpentan1,5dioic acid, K.

HO 2 CCH 2 C(CH 3 ) 2 CH 2 CO 2 H
K

Use the information above to deduce the structures for compounds G, H and J, explaining
all the reactions involved.
[6]

[Total: 20]

AJC JC2 Prelim 2016 9647/03/H2 [Turn over


141 trendyline
4

2 (a) One frequently used method for preparing methyl esters is by reaction of carboxylic acids
with diazomethane, CH 2 N 2 .

RCO 2 H + CH 2 N 2 RCO 2 CH 3 + N 2

The reaction of a carboxylic acid with diazomethane occurs via a twostep mechanism.

Protonation of diazomethane by the carboxylic acid to yield methyldiazonium ion,


CH 3 N 2 +, and carboxylate ion in the first step.

Reaction of the carboxylate ion with CH 3 N 2 + to form N 2 in the second step.

(i) A and B are two possible resonance structures of diazomethane, CH 2 N 2 .

H H
C N N C N N
H H
A B

Suggest which resonance structure of diazomethane, A or B, is likely to be involved in


the first step of the mechanism.
[1]

(ii) Using your answer from (a)(i), suggest the mechanism for the reaction between
RCO 2 H and diazomethane. Show any relevant lone pairs, dipoles and charges, and
indicate the movement of electron pairs with curly arrows.
[3]

(b) (i) Write an equation for the complete combustion of methyl ethanoate, CH 3 CO 2 CH 3 .
[1]

(ii) Define the term standard enthalpy change of formation. [2]

(iii) Use the standard enthalpy changes of combustion, H c o in Table 2.1 to calculate the
standard enthalpy change of formation of methyl ethanoate.

Table 2.1

H c o / kJ mol1
carbon 393.5
hydrogen 285.8
methyl ethanoate 1592.1
[3]

trendyline
AJC JC2 Prelim 2016 9647/03/H2 [Turn over
142 trendyline
5

(c) Describe one simple chemical test to distinguish between HCO 2 CH 2 CH 3 and CH 3 CO 2 CH 3 .
State clearly how each compound behaves in the test.
[3]

(d) Consecutive elements W, X, Y and Z are in the third period of the Periodic Table. Element Y
has the highest first ionisation energy and the lowest melting point of these four elements.

(i) Describe what you would see when elements W and Z are separately burned in air or
oxygen.
[2]

(ii) The oxides of elements X and Y can be obtained when the elements are burned in
excess oxygen. The oxides of X and Y have melting points of 1720 oC and 580 oC
respectively.

Briefly relate these melting points to the structure of and bonding in each of these
oxides.
[2]

(iii) Describe the reactions, if any, of the oxides of elements X and Y with water.
Include the approximate pH value of any resulting solutions, and write equations for
any reactions that occur.
[3]

[Total: 20]

trendyline
AJC JC2 Prelim 2016 9647/03/H2 [Turn over
143 trendyline
6

3 (a) Nickelmetal hydride (NiMH) batteries are the most common rechargeable batteries used
for devices that require large amounts of energy such as digital cameras and MP3 players.
Most NiMH batteries use an alloy containing mainly lanthanum and nickel.

In one such battery, one of the electrodes is LaNi 5 H 6 and the other is NiO(OH). The
electrolyte is aqueous KOH. During discharge, an electrochemical reaction takes place to
produce LaNi 5 (s) and Ni(OH) 2 (s) and releases electrical energy.

(i) Construct the two halfequations for the reactions that take place at each electrode
during discharge. Indicate the polarity of the electrodes in your answer.
[2]

(ii) During recharge, an electrical potential is applied across the electrodes to reverse the
electrochemical reaction.

Using your answer in (a)(i), write the overall equation for the reaction that occurs
during recharge.
[1]

(iii) Overcharging the NiMH battery may result in the electrolyte being discharged at the
electrodes. A safety vent is thus incorporated in the battery to release the excess
pressure.

With reference to the Data Booklet, suggest a relevant halfequation for the formation
of one product at the electrode of the NiMH battery.
[1]

(b) Zinccerium battery is another type of rechargeable battery using a twoelectrolyte system.
The overall equation for the discharging process is given below.

Zn + 2Ce4+ Zn2+ + 2Ce3+

(i) Given that the typical cell voltage for the cell is 2.2 V, calculate the standard electrode
potential of the cerium halfcell, using relevant data from the Data Booklet.
[1]

(ii) Using relevant data from the Data Booklet, deduce if the Ce4+/Ce3+ halfcell can be
replaced with Br 2 /Br halfcell.
[1]

trendyline
AJC JC2 Prelim 2016 9647/03/H2 [Turn over
144 trendyline
7

(c) Amines can be made from a variety of compounds.

Suggest, in no more than three steps, how each of the following transformations can be
achieved.

For each transformation, draw the structures of the intermediate compounds and indicate
reagents and conditions for each of the steps.

(i) CH3 CH2CH2NH2

[3]

(ii) O NH2

[3]

(d) The buffer of 2amino2methylpropan1ol (AMP) has been shown to be well suited for
the determination of the activity of enzymes like alkaline phosphatase and lactate
dehydrogenase. It hydrolyses in water with a pK b value of 4.3.

HOCH 2 C(CH 3 ) 2 NH 2 + H 2 O HOCH 2 C(CH 3 ) 2 NH 3 + + OH

(i) A 100 cm3 solution containing 0.500 mol dm3 of HOCH 2 C(CH 3 ) 2 NH 2 is mixed with
a 100 cm3 solution containing 0.800 mol dm3 HOCH 2 C(CH 3 ) 2 NH 3 +.

Calculate the pH of the buffer solution formed.


[2]

[HOCH2CCH3 2 NH2 ]
(ii) State the ratio of when the pH of the buffer solution is 9.7.
[HOCH2CCH3 2 NH3 ]

Hence calculate the mass of solid NaOH to be added to the buffer solution in (d)(i) to
achieve this ratio.
[2]

trendyline
AJC JC2 Prelim 2016 9647/03/H2 [Turn over
145 trendyline
8

(e) Proteins are longchain molecules made by linking together amino acids.

Myoglobin is one such protein that stores and releases oxygen in vertebrate muscle cells.
The table below shows some of the amino acids in a molecule of myoglobin.

aspartic
amino acid serine alanine lysine
acid
abbreviation asp ser ala lys
formula of side
CH 2 CO 2 H CH 2 OH CH 3
chain (CH 2 ) 4 NH 2

(i) Given that asplysala is a particular section of the protein chain of myoglobin, draw
the displayed formula for this section at pH 7.
[1]

(ii) Describe two types of sidechain interaction that would occur between the amino
acids residues of myoglobin. Illustrate your answer with suitable pairs of amino acids
from the table above.
[2]

(iii) Casein is the predominant protein found in milk. The conversion of milk to cheese is a
denaturation process involving the addition of the Lactobacillus bacterium which
produces lactic acid.

Explain this phenomenon by considering the types of sidechain interaction that are
affected during the denaturation of casein.
[1]

[Total: 20]

trendyline
AJC JC2 Prelim 2016 9647/03/H2 [Turn over
146 trendyline
9

4 (a) 1,3dibromopropane is the starting material for a fourstep reaction sequence which produces
2,4dihydroxypentane1,5dioic acid. The four reactions are shown.

O O
reaction 1 reaction 2 reaction 3 reaction 4
Br Br E F G HO OH
H+/H2O
OH OH

1,3-dibromopropane 2,4-dihydroxypentane-1,5-dioic acid

Compound F has the molecular formula C 3 H 4 O 2 . On treatment with Tollens reagent, it gives a
silver mirror.

(i) Draw the structures of E and F. [2]

(ii) When preparing compound F from E it is important that the product is distilled off rather than
continually refluxed.

State which unwanted product is avoided by distilling off compound F.


[1]

(iii) In an experiment, 9.0 g of 1,3dibromopropane was converted into compound F with an


overall yield of 67 %. Calculate the mass of F that was obtained.
[2]

(iv) Compound G is converted into the final product by reaction with dilute acid. Suggest the
structure of G.
[1]

(b) The oxides of Group I and II metals have many uses. Beryllium oxide, BeO, is used in many high
performance semiconductor parts for applications such as radio equipment, while sodium oxide,
Na 2 O, is a significant component of glasses and windows.

(i) Using the given data and other relevant data from the Data Booklet, construct an energy level
diagram and use it to calculate the lattice energy of beryllium oxide. Label each energy level
in your diagram and draw arrows representing the energy terms involved. Use symbols or
numbers to represent these energy terms.

enthalpy change of formation of beryllium oxide 609 kJ mol1


enthalpy change of atomisation of beryllium +324 kJ mol1
first electron affinity of oxygen 141 kJ mol1
second electron affinity of oxygen +798 kJ mol1
[3]

(ii) The theoretical lattice energy of beryllium oxide is 4293 kJ mol1. Suggest a reason why this
value is different from your answer in (b)(i).
[1]

trendyline
(iii) Explain how
ow you would expect the numerical magnitude
magn of the lattice
latt energy of beryllium
oxide to compare with that of sodium oxide.
[2]

AJC JC2 Prelim 2016 9647/03/H2 [Turn over


147 trendyline
10

(iv) Explain the reaction(s) that occur(s) when chlorine gas is bubbled into a flask containing
water, at room temperature, to which sodium oxide has been added. Write equation(s) where
appropriate.
[2]

(c) Soluble chromates(VI) in the soil are toxic at low concentrations for plants.

In water treatment to remove the chromate(VI) ions, CrO 4 2(aq), from a sample of soil water,
aqueous acidified iron(II) sulfate is first added to the water sample to convert the chromate(VI) ions
to chromium(III) ions, Cr3+(aq). This is followed by adding aqueous sodium carbonate to the
resultant solution to produce effervescence and an insoluble compound.

(i) Construct an equation for the reaction between aqueous acidified iron(II) sulfate and
chromate(VI) ions.
[1]

(ii) Identify the insoluble compound formed upon adding aqueous sodium carbonate.

With the aid of an appropriate ionic equation, explain its formation. [2]

(d) Chromium(III) ions coordinate with both monodentate and polydentate ligands.

Consider the following complex formation reactions with thiocyanate, SCN, and ethylenediamine,
H 2 NCH 2 CH 2 NH 2 , ligands:

I [Cr(H 2 O) 6 ]3+(aq) + 6SCN(aq) [Cr(SCN) 6 ]3(aq) + 6H 2 O(l)

II [Cr(H 2 O) 6 ]3+(aq) + 3(en)(aq) [Cr(en) 3 ]3+(aq) + 6H 2 O(l)

where (en) is ethylenediamine

(i) Explain what is meant by entropy. [1]

(ii) Explain how \RXZRXOGH[SHFWWKHHQWURS\FKDQJHS r o, for reaction II to compare with that


for reaction I.
[2]

[Total: 20]

trendyline
AJC JC2 Prelim 2016 9647/03/H2 [Turn over
148 trendyline
11

5 Nitrogen forms numerous oxides, two of which are dinitrogen monoxide, N 2 O, and dinitrogen tetroxide,
N2O4.

(a) Dinitrogen monoxide, N 2 O, is a nontoxic gas that is used commercially as the propellant in cans of
whipped cream.

Given that N 2 O is a linear molecule, draw a dotandcross diagram showing the electrons (outer
shells only) in N 2 O.

You should distinguish carefully between electrons originating from each of the nitrogen atoms.
[1]

(b) At 1200 K and in the presence of a gold wire, N 2 O decomposes as follows.

2N 2 O(g) 2N 2 (g) + O 2 (g)

The kinetics of this reaction can be followed by measuring the total pressure, P, as it changes with
time, t. In one such experiment, the total pressure changes as follows.

total pressure, P / kPa 25.0 27.5 30.0 32.5 34.0 35.0


time, t / s 0 1030 2360 4230 5870 7420
partial pressure, p, of N 2 O / kPa 25.0

(i) What is the total pressure when the reaction is complete? [1]

(ii) Show that the partial pressure of N 2 O at any time, t, is equal to (75.0 2P) kPa. [2]

(iii) Hence calculate the partial pressures of N 2 O after 1030 s, 2360 s, 4230 s, 5870 s and
7420 s.
[1]

(iv) Using your answers from (b)(iii) and the data above, plot these data on suitable axes and use
your graph to determine the order of reaction with respect to N 2 O.

You should show all your working and draw clearly any construction lines on your graph.
[3]

(v) The rate of reaction is expressed in kPa s1. Write the rate equation for the reaction, and
calculate a value for the rate constant. Include units in your answer.
[3]

trendyline
AJC JC2 Prelim 2016 9647/03/H2 [Turn over
149 trendyline
12

At a temperature of 27 oC, gaseous N 2 O 4 and NO 2 are in dynamic equilibrium according to the following
equation.
N 2 O 4 (g) 2NO 2 (g)
pale yellow deep redbrown

(c) When 4.60 g of N 2 O 4 is placed in an evacuated 1.48 dm3 flask at 27 oC, the equilibrium pressure is
1.00 atm (101 kPa).

(i) Calculate the number of moles of N 2 O 4 at the start. [1]

(ii) Calculate the number of moles of gas at equilibrium, assuming the gases behave ideally. [1]

(iii) Hence calculate the percentage of the N 2 O 4 that has been dissociated. [2]

(iv) Write an expression for the equilibrium constant, K p , for the reaction and show it has a value
of 0.17 atm at 27 oC.
[3]

(d) When the equilibrium mixture from (c) at 27 oC is suddenly compressed in a gas syringe, the
mixture immediately darkens and then slowly becomes paler. Suggest why the mixture behaves in
this way.
[2]

[Total: 20]

trendyline
AJC JC2 Prelim 2016 9647/03/H2 [Turn over
150 trendyline
151 trendyline
H2 Chemistry 9647 2016 JC2 Prelim P3 Suggested Solutions

1 (a) (i) Nuclear charge / proton number increases from Fe to Cu.

However, additional electrons are being added to the inner 3d subshell. As such, [1]
there is an increase in shielding effect caused by the increasing number of
d electrons from Fe to Cu.

Hence, effective nuclear charge remains fairly constant from Fe to Cu. [1]

(ii) Fe uses both the 3d and 4s electrons while Ca only uses the 4s electrons for [1]
metallic bonding.

The ionic radius of Fe is also smaller than that of Ca, resulting in higher charge [1]
density.

This results in stronger metallic bonding between Fe ions and the valence [1]
electrons.

(iii) density = mass / volume


[1]
Fe and Cu have greater atomic mass

and smaller atomic radius. [1]

(b) 2Cu+(aq) Cu(s) + Cu2+(aq) [1]

State symbols not required

Eo(Cu+/Cu) = +0.52 V
Eo(Cu2+/Cu+) = +0.15 V

E cell o = +0.52 (+0.15) = +0.37 V [1]

(c) (i) I and Cu2+ undergo a redox reaction to produce the white precipitate copper(I)
iodide, CuI, and brown solution I2 .
[1]
2+
2I (aq) + Cu (aq) CuI(s) + I2 (aq)

S 2 O 3 2 then reduces I 2 to I (colourless solution), itself oxidises to S 4 O 6 2 (redox


reaction).
[1]
2 2
2S 2 O 3 (aq) + I 2 (aq) 2I (aq) + S 4 O 6 (aq)

State symbols not required

(ii) Cu and Cu2+ undergo a redox reaction (or comproportionation) to form a colourless
Cu(I) complex, [CuCl 2 ].

trendyline
dyy
[1]
Cu(s)) + Cu2+(aq) + 4Cl
4Cl(aq) uCl 2 ](aq)
2[CuCl
2
2[ Cu

2016AndersonJC/CHEM 1
152 trendyline
Element Cu Cl
% by mass 64.1 35.9
Ar 63.5 35.5
No. of moles 64.1/63.5 35.9/35.5
= 1.009 = 1.011 [1]
Simplest whole no. ratio 1 1

The white precipitate is CuCl.

[CuCl 2 ](aq) CuCl(s) + Cl(aq) [1]

State symbols not required

(d) Information Type of reaction Deduction(s) [6]


On treatment with hot Oxidative H contains a C=C / alkene.
acidified concentrated cleavage
KMnO 4 , H produced a Since a single compound is formed
single compound J, (without loss of C), H is a cycloalkene.
C 8 H 14 O 3 .
J evolved CO 2 with Acidbase J contains RCO 2 H / carboxylic acid.
Na 2 CO 3 (aq),
gave an orange precipitate Condensation J contains a ketone.
with 2,4DNPH, (J cannot contain an aldehyde since J
is produced from strong oxidation of H
with KMnO 4 )

and gave a yellow Oxidation / J has a CH 3 CO structure


precipitate with alkaline triiodomethane (J cannot have a CH 3 CH(OH)
aqueous iodine. reaction structure since J is produced from
strong oxidation of H with KMnO 4 )

O CH3
CH3 C CH2 C CH2 CO2H
CH3
CO2H

G H J

Do not accept for structure of G as G is a secondary carboxylic acid


CO2H

trendyline
[Total: 20]

2016AndersonJC/CHEM 2
153 trendyline
2 (a) (i) B [1]

(ii) O O O [3]
R C CH2 N N R C CH3 N N R C + N2
O H O O CH3

(b) (i) CH 3 CO 2 CH 3 + 7/2O 2 3CO 2 + 3H 2 O [1]

Accept equation multiplied through by 2

(ii) The amount of heat absorbed or evolved when one mole of substance is formed [2]
from its constituent elements in their standard states at 298 K and 1 atm.

(iii) H f o
3C(s) + 3H 2 (g) + O 2 (g) CH 3 CO 2 CH 3 (l)

H f o = H c o(reactants) H c o(products)
= [3(393.5) + 3(285.8) + 0] [ 1592.1)] = 446 kJ mol1 [3]

(c) Heat each compound separately with NaOH(aq), and then add I2 (aq) to the resulting [1]
solution.

A yellow precipitate (CHI3 ) is observed for HCO 2 CH 2 CH 3 . [1]


No (yellow) precipitate is observed for CH 3 CO 2 CH 3 . [1]

(d) (i) W (aluminium) burns in O 2 with an intense white flame to give a white solid of Al 2 O 3 . [1]

Z (sulfur) burns in O 2 on gentle heating with a pale blue flame to give a colourless [1]
gas of SO 2 .

(ii) The oxide of X (SiO 2 ) has a giant molecular structure. Large amount of energy is [1]
required to break the strong covalent bonds between Si and O atoms.

The oxide of Y (P 4 O 10 or P 4 O 6 ) has a simple molecular structure. Less energy is [1]


required to break the weak van der Waals forces of attraction between the
molecules.

(iii) The oxide of X (SiO 2 ) is insoluble in water. Hence, the pH of the resulting solution is [1]
7.

The oxide of Y (P 4 O 10 or P 4 O 6 ) reacts readily in water to produce a strongly acidic [1]


solution of pH 2.

P 4 O 10 + 6H 2 O 4H 3 PO 4 OR P 4 O 6 + 6H 2 O 4H 3 PO 3 [1]

[Total: 20]

trendyline
2016AndersonJC/CHEM 3
154 trendyline
3 (a) (i) At LaNiH 6 electrode: LaNi 5 H 6 + 6OH LaNi 5 + 6H 2 O + 6e [2]

At NiO(OH) electrode: NiO(OH) + H 2 O + e Ni(OH) 2 + OH


++

(ii) LaNi 5 + 6Ni(OH) 2 LaNi 5 H 6 + 6NiO(OH) [1]

(iii) 4OH O 2 + 2H 2 O + 4e [1]

OR

2H 2 O + 2e H 2 + 2OH

(b) (i) Eo(Zn2+/Zn) = 0.76 V

2.2 = Eo(Ce4+/Ce3+) (0.76)


Eo(Ce4+/Ce3+) = +1.44 V [1]

(ii) Eo(Br 2 /Br) = +1.07 V

Eo cell = +1.07 (0.76)


= +1.83 V > 0 (feasible)

Since it is feasible for the battery to discharge electricity, the Ce4+/Ce3+ halfcell [1]
can be replaced with Br 2 /Br halfcell.

(c) (i) CH3 CH2Cl CH2CN CH2CH2NH2 [3]


(limited) Cl2(g) NaCN in ethanol LiAlH4
uv light heat under reflux in dry ether

(ii) O OH Cl NH2 [3]


LiAlH4 excess conc. NH3
in dry ether SOCl2 heat in sealed tube

(d) (i) Since equal volume of HOCH 2 C(CH 3 ) 2 NH 2 and HOCH 2 C(CH 3 ) 2 NH 3 + are mixed,
concentration of both species are halved.

[HOCH2CCH3 2 NH3 ][OH- ]


Kb
[HOCH2CCH3 2 NH2 ]

(0.400)[OH ] [1]
10 -4.3
0.250

[OH] = 3.13 x 105 mol dm3

pOH
H=

ttrendyline
log(3.13 x 1055) = 4.5
log(3.13

pH = 9.5 [1]

2016AndersonJC/CHEM 4
155 trendyline
(ii) Since pK b (AMP) = 4.3, pK a (conjugate acid of AMP) = 9.7.

When pH of the buffer = 9.7, the buffer solution is at its maximum buffering
capacity.

[HOCH2CCH3 2 NH2 ] [1]


Hence = 1.
[HOCH2CCH3 2 NH3 ]

initial n(HOCH 2 C(CH 3 ) 2 NH 3 +) = 100/1000 x 0.800 = 0.0800 mol


initial n(HOCH 2 C(CH 3 ) 2 NH 2 ) = 100/1000 x 0.500 = 0.0500 mol

Let x be n(OH) added

HOCH 2 C(CH 3 ) 2 NH 3 + + OH HOCH 2 C(CH 3 ) 2 NH 2 + H 2 O


initial (mol) 0.0800 x 0.0500
change x x +x
final (mol) 0.0800 x 0 0.0500 + x

[HOCH2CCH3 2 NH2 ]
Since = 1,
[HOCH2CCH3 2 NH3 ]

0.0800 x = 0.0500 + x
x = 0.015 mol

mass of NaOH added = 0.015 x (23.0 + 16.0 + 1.0) = 0.600 g [1]

(e) (i) H O H O H O [1]

N C C N C C N C C

H H C H H H C H C
H H
C H C H H

O O
H C H

H C H

N
H
H H

(ii) Ionic bonding between charged R groups containing the CO 2 groups, NH 3 + [1]
groups.
Example: Between lysine (containing NH 3 +) and aspartic acid (containing

trendyline
tren dyli
y
C 2 )
CO

Hydrogen bonding between


etwee polar R groupsroup containing the carboxyl groups [1]
(
CO 2 H), amino groups (NH
(CO (NH 2 ) or hydroxyl (
(OH)
(OH) groups.
Example: Between aspartic acid and serine or between lysine and serine.

2016AndersonJC/CHEM 5
156 trendyline
(iii) Lactic acid dissociates to give H+ which may protonate the carboxylate group (in [1]
glutamic acid) or the amine group (in lysine), disrupting ionic bonds between the
charged R groups.

OR

disrupting the hydrogen bonding between the polar R groups (e.g. lysine and
serine, leading to denaturation.

[Total: 20]

trendyline
2016AndersonJC/CHEM 6
157 trendyline
4 (a) (i) O O [2]

HO OH H H

E F

(ii) O O O O [1]

HO OH OR H OH

Accept propanedioic acid Accept 3oxopropanoic acid

(iii) n(1,3dibromopropane) = 9.0 / 201.8 = 0.0446 mol [1]


mass of F = 0.67 x 0.0446 x 72.0 = 2.15 g [1]

(iv) OH OH [1]

NC CN

(b) (i) Energy [2]


Be2+(g) + O2(g)

(+900) + (+1760) + (141) + 798 (+3317)

Be(g) + O(g)
lattice energy of BeO(s)

(+324) + (496) (+572)

Be(s) + O2(g)



BeO(s)

trendyline
Lattice
ce
e energy of BeO(s)
= (3317
BeO(s
3317 + 572 + 609)
98 kJ mol11
= 4498
498 [1]

2016AndersonJC/CHEM 7
158 trendyline
(ii) Theoretical calculation assumes pure ionic character.
But Be2+ ion is small and highly charged, hence it has high polarising power
BeO exhibits covalent character [1]
Attraction is stronger, so observed value is more exothermic.

(iii) q q-
Given LE
r r-

ionic charge of Be2+ > ionic charge of Na+


ionic radius of Be2+ < ionic radius of Na+ [1]

Stronger attraction between oppositely charged ions in BeO


LE (BeO) > of LE (Na 2 O) in magnitude [1]

(iv) Na 2 O(s) + H 2 O(l) 2NaOH(aq)


Sodium oxide dissolves in water to produce sodium hydroxide [1]

Cl 2 (g) + 2NaOH(aq) NaCl(aq) + NaClO(aq) + H 2 O(l)


Chlorine undergoes disproportionation reaction when reacted with NaOH. [1]

State symbols not required

(c) (i) CrO 4 2 + 8H+ + 3e Cr3+ + 4H 2 O


Fe2+ Fe3+ + e

CrO 4 2 + 3Fe2+ + 8H+ Cr3+ + 3Fe3+ + 4H 2 O [1]

(ii) chromium(III) hydroxide OR Cr(OH) 3 (H 2 O) 3 OR Cr(OH) 3

2[Cr(H 2 O) 6 ]3+ + 3CO 3 2 2[Cr(OH) 3 (H 2 O) 3 ] + 3CO 2 + 3H 2 O [1]

Cr3+(aq) undergoes hydrolysis with water liberating H+(aq)


[1]
Undergoes acidbase reaction with Na 2 CO 3 (aq)

(d) (i) a state function that measures the degree of disorder / randomness in a system [1]
OR
gives a measure of the extent to which particles and energy are distributed
within the system OR
the greater the disorder / randomness, the larger is the entropy.

(ii) In reaction I, there is no change in number of particles in aqueous solution, [1]


whereas in reaction II, there is an increase of 3 particles.

Increase in number of ways of arrangement and energy distribution of the


particles
S r o is more positive [1]

[Total: 20]

trendyline
2016AndersonJC/CHEM 8
159 trendyline
5 (a) [1]
N N O

(b) (i) total pressure, P, when reaction is complete = 3/2 x 25 = 37.5 kPa [1]

(ii) 2N 2 O(g) 2N 2 (g) + O 2 (g)


initial / kPa 25 0 0
change / kPa x +x +1/2x
final at any time, t / kPa 25 x x 1/2x

25 x + x + 1/2x = P
25 + 1/2x = P
x = 2P 50 [1]

Hence, partial pressure of N 2 O at any time, t, is

25 x = 25 (2P 50)
= 25 2P + 50
= 75.0 2P (shown) [1]

(iii) P / kPa 27.5 30.0 32.5 34.0 35.0


t/s 1030 2360 4230 5870 7420
p of N 2 O / kPa 20 15 10 7 5 [1]

trendyline
2016AndersonJC/CHEM 9
160 trendyline
(iv) [2]

[1]
Since t 1/2 is constant at 3200 s, the order of reaction w.r.t. N 2 O is 1.

(v) rate k pN2 O [1]

ln 2
t1/2
k
ln 2
k
t1/2

trendyline
ren
ln 2 [1]
2.17 x 10
1 - 4 s-1 [1]
3200

2016AndersonJC/CHEM 10
161 trendyline
(c) (i) n(N 2 O 4 ) = 4.60 / 92.0 = 0.0500 mol [1]

(ii) pV = nRT
n = pV / RT
101 x 103 x 1.48 x 10 -3
= = 0.0600 mol [1]
8.31 x 300

(iii) N 2 O 4 (g) 2NO 2 (g)


initial / mol 0.05 0
change / mol x/2 +x
eqm / mol 0.05 x/2 x

0.05 x/2 + x = 0.06


x = 0.02 [1]

n(N 2 O 4 ) dissociated = x/2 = 0.02/2 = 0.01


% of N 2 O 4 dissociated = 0.01 / 0.05 x 100 = 20.0 % [1]

(iv) pNO2
2 [1]
Kp
pN2O4

p(NO 2 ) = 0.02/0.06 x 1.00


= 1/3 atm [1]

p(N 2 O 4 ) = 0.04/0.06 x 1.00


= 2/3 atm

( 1 )2
Kp 3 = 1/6 0.17 (shown)
2 [1]
3

(d) N 2 O 4 (g) 2NO 2 (g)


pale yellow deep redbrown

When the equilibrium mixture is suddenly compressed in a gas syringe, [N 2 O 4 ] and [NO 2 ] [1]
increase due to the reduced volume.

Compression of the gas also results in an increase in pressure. The position of the
above equilibrium shifts left to decrease pressure by producing fewer gas molecules. The [1]
mixture becomes paler as [N 2 O 4 ] increases.

[Total: 20]

trendyline
2016AndersonJC/CHEM 11
162 trendyline


&$7+2/,&-81,25&2//(*(
-&35(/,0,1$5<(;$0,1$7,216
+LJKHU

&+(0,675< 
3DSHU0XOWLSOH&KRLFH :HGQHVGD\$XJXVW
KRXU
$GGLWLRQDO0DWHULDOV 0XOWLSOH&KRLFH$QVZHU6KHHW
'DWD%RRNOHW

5($'7+(6(,16758&7,216),567

:ULWHLQVRIWSHQFLO
'RQRWXVHVWDSOHVSDSHUFOLSVJOXHRUFRUUHFWLRQIOXLG
:ULWH\RXUQDPHDQG+7JURXSRQWKH$QVZHU6KHHWLQWKHVSDFHVSURYLGHG

7KHUHDUHIRUW\TXHVWLRQVRQWKLVSDSHU$QVZHUDOOTXHVWLRQV)RUHDFKTXHVWLRQWKHUHDUHIRXU
SRVVLEOHDQVZHUV$%&DQG'
&KRRVHWKHRQH\RXFRQVLGHUFRUUHFWDQGUHFRUG\RXUFKRLFHLQVRIWSHQFLORQWKHVHSDUDWH$QVZHU
6KHHW

5HDGWKHLQVWUXFWLRQVRQWKH$QVZHU6KHHWYHU\FDUHIXOO\

(DFKFRUUHFWDQVZHUZLOOVFRUHRQHPDUN$PDUNZLOOQRWEHGHGXFWHGIRUDZURQJDQVZHU
$Q\URXJKZRUNLQJVKRXOGEHGRQHLQWKLVERRNOHW
7KHXVHRIDQDSSURYHGVFLHQWLILFFDOFXODWRULVH[SHFWHGZKHUHDSSURSULDWH

trendyline
7KLVGRFXPHQWFRQVLVWVRISULQWHGSDJHVDQGEODQNSDJH
LVGRFXPHQWFRQVLVWVRI
V GRFXPHQW FRQVLVWV RI 
SULQWHGSDJHVDQGEODQNSD
SULQWHGSDJHVDQGEODQNSD

 &-&-&3UHOLPLQDU\([DP >7XUQRYHU
163 trendyline


6HFWLRQ$

)RU HDFK TXHVWLRQ WKHUH DUH four SRVVLEOH DQVZHUV A B C DQG D &KRRVH WKH RQH \RX
FRQVLGHUWREHcorrectDQGUHFRUG\RXUFKRLFHLQVRIWSHQFLORQWKHseparate Answer Sheet
SURYLGHG

 Use of the Data Booklet is relevant to this question.

$UVHQLF ,,, VXOILGH$V6LVDEULJKW\HOORZVROLGZKLFKKDVEHHQXVHGDVDSLJPHQWLQ


SDLQWLQJV
:KHQDUVHQLF ,,, VXOILGHLVKHDWHGLQDLULWUHDFWVZLWKR[\JHQWRJLYHDUVHQLF ,,, R[LGH
DQGVXOIXU ,9 R[LGH
$V62$V262
:KDWZRXOGEHWKHPDVVRIDUVHQLF ,,, R[LGHSURGXFHGLIGPR[\JHQPHDVXUHG
DWURRPWHPSHUDWXUHDQGSUHVVXUHUHDFWHGZLWKDUVHQLF ,,, VXOILGH"

$ J % J & J ' J

 
 ,QFRPSOHWHFRPEXVWLRQRIFPRIDJDVHRXVK\GURFDUERQJDYHFDUERQGLR[LGHDQG
FDUERQPRQR[LGHLQDUDWLRDVZHOODVZDWHUYDSRXU,WZDVIRXQGWKDWWKHFDUERQ
GLR[LGH WRRN XS D YROXPH RI  FPZKHUHDV WKH ZDWHU YDSRXU WRRN XS D YROXPH RI
FP$OOJDVYROXPHVDUHPHDVXUHGDWWKHVDPHWHPSHUDWXUHDQGSUHVVXUH

:KDWLVWKHPROHFXODUIRUPXODRIWKHK\GURFDUERQ"

$ &+ % &+ & &+ ' &+

 3RO\WKLRQDWHVDUHDVHULHVRIVXOIXUR[RDQLRQVZLWKWKHJHQHUDOIRUPXOD6n2ZKHUH
n!$VLPSOHH[DPSOHLVWHWUDWKLRQDWHLRQ62

:KLFK RI WKH IROORZLQJ JUDSKV EHVW LOOXVWUDWHV WKH YDULDWLRQ LQ WKH DYHUDJH R[LGDWLRQ
QXPEHURIVXOIXULQ6n2ZLWKn"
 DYHUDJHR[LGDWLRQ
QXPEHURI6



 $

%


t en
trendyline
endy
ndyline
yline


&

 '
'

  QXPEHURI6DWRPVn
&-&-&3UHOLPLQDU\([DP
164 trendyline


 :KLFKRIWKHIROORZLQJVSHFLHVKDVWKHVPDOOHVWERQGDQJOHDURXQGWKHFHQWUDODWRP"

$ 1&l % 3&l & 62 ' %)


 :KLFKRIWKHIROORZLQJVKRZVWKHFRUUHFWERQGVSUHVHQWLQVROLG&+,"
$ &RYDOHQWERQGVRQO\
% ,RQLFERQGVRQO\
& &RYDOHQWERQGV WHPSRUDU\GLSROHLQGXFHGGLSROHIRUFHVRIDWWUDFWLRQ
' &RYDOHQWERQGV SHUPDQHQWGLSROHSHUPDQHQWGLSROHIRUFHVRIDWWUDFWLRQ


 3RO\PHULVDWLRQLVDSURFHVVLQZKLFKODUJHQXPEHURIVPDOOPROHFXOHV PRQRPHUV DUH
MRLQHGWRJHWKHUWRIRUPORQJFKDLQPROHFXOHV
&RQVLGHULQJWKDWSRO\PHULVDWLRQLVDVSRQWDQHRXVUHDFWLRQZKDWDUHWKHFRUUHFWVLJQV
RISDQGHIRUWKHSURFHVV"

 S H
$  
%  
&  
'  


 61 LV D WKHUPRFKURPLF VROLG ZKLFK FKDQJHV FRORXU ZLWK WHPSHUDWXUH 61 KDV D
FDJHVWUXFWXUHDVVKRZQLQWKHGLDJUDP


*LYHQ WKH IROORZLQJ GDWD ZKDW LV WKH DYHUDJH ERQG HQHUJ\ RI GLVVRFLDWLRQ RI 61 LQ
N-PRO"
+I 61  N-PRO 

+DW VXOIXU  N-PRO


+DW QLWURJHQ  N-PRO
66 ERQGHQHUJ\RIGLVVRFLDWLRQLQ61 N-PRO

$



trendyline %
% 


&
& 
 '
' 

 &-&-&3UHOLPLQDU\([DP >7XUQRYHU
165 trendyline


 $W  & D GHQWHG SLQJ SRQJ EDOO KDV DQ LQWHUQDO YROXPH RI  FP DQG LQWHUQDO
SUHVVXUH RI  N3D ,W LV WKHQ SODFHG LQ D ZDWHU EDWK PDLQWDLQHG DW  & ZKLFK
UHWXUQHGWKHEDOOWRLWVRULJLQDOVSKHULFDOVKDSHZLWKLQWHUQDOYROXPHRIFP
$VVXPLQJLGHDOJDVEHKDYLRXUZKDWLVWKHSUHVVXUHRIDLULQVLGHWKHEDOODW&LQLWV
RULJLQDOVSKHULFDOVKDSH"

$ N3D % N3D & N3D ' N3D

 *DV;GHFRPSRVHVWRWZRRWKHUJDVHV<DQG=DFFRUGLQJWRWKHIROORZLQJHTXDWLRQ
; J < J = J 
7KHJUDSKEHORZVKRZVWKHGHFRPSRVLWLRQRIPRORISXUHJDV;LQWKHSUHVHQFHRI
DFDWDO\VWDWYDULRXVWHPSHUDWXUHV





PROHVRI; 
UHPDLQLQJ

 &

&



WLPH 

:KLFKRQHRIWKHIROORZLQJVWDWHPHQWVDERXWWKHDERYHV\VWHPLVFRUUHFW"
$ 7KHGHFRPSRVLWLRQRI;LVHQGRWKHUPLF
% 7KHKpRIWKHV\VWHPGHFUHDVHVZLWKLQFUHDVLQJWHPSHUDWXUH
& 7KHSHUFHQWDJHGHFRPSRVLWLRQRI;LVDWR&
' 7KHV\VWHPEHFRPHVOHVVGLVRUGHUHGZKHQLWUHDFKHVHTXLOLEULXPVWDWH



trendyline
&-&-&3UHOLPLQDU\([DP
166 trendyline


 0DORQLFDFLG+2&&+&2+LVDZHDNGLEDVLFDFLG
0DORQLF DFLG LV WLWUDWHG DJDLQVW  FP RI VRGLXP K\GUR[LGH RI WKH VDPH
FRQFHQWUDWLRQWKHIROORZLQJS+YROXPHFXUYHLVREWDLQHG

$WZKLFKSRLQWRQWKHWLWUDWLRQFXUYHLVWKHPL[WXUHPRVWDEOHWRUHVLVWS+FKDQJHXSRQ
DGGLWLRQRIDVPDOODPRXQWRIDTXHRXVDFLGRUEDVH"

S+ 

 $
 ,

%



&

 '

 9RORI
PDORQLFDFLG
     FP

 7KHIROORZLQJLVWKHUHDFWLRQIRUFRPSRXQG;<
;< J ; J < J 
,I WKH UDWH FRQVWDQW IRU WKH UHDFWLRQ LV  [  V ZKDW LV WKH WLPH WDNHQ IRU WKH
FRQFHQWUDWLRQRIDVDPSOHRI;<WRGHFUHDVHIURPPROGPWRPROGP"
$ V % V & V ' V


 7KHUHDFWLRQEHWZHHQ&DQG'LVDVIROORZV
&'(
,QDQH[SHULPHQWWRLQYHVWLJDWHWKHHIIHFWRIFRQFHQWUDWLRQVRQWKHUDWHRIUHDFWLRQWKH
IROORZLQJUHVXOWVZHUHREWDLQHGDWFRQVWDQWWHPSHUDWXUH

([SHULPHQW >&@PROGP >'@PROGP ,QLWLDOUDWHPROGPV
   
   
   [
   


trendyline
:KDWLVWKHYDOXHRI["
HYDOXHRI
Y ["
$ 
 % 
 &  '
' 

 &-&-&3UHOLPLQDU\([DP >7XUQRYHU
167 trendyline


 7KHIROORZLQJJUDSKVKRZVKRZDSURSHUW\RIWKHHOHPHQWVLQ3HULRGIURP1DWR3
RUWKHLUFRPSRXQGVYDULHVZLWKSURWRQQXPEHU

SURWRQQXPEHU
    

:KDWSURSHUW\LVVKRZQE\WKHJUDSK"
$ 0HOWLQJSRLQWRIHOHPHQW
% 0HOWLQJSRLQWRIR[LGH 
& S+RIDTXHRXVFKORULGH
' (OHFWULFDOFRQGXFWLYLW\RIHOHPHQW


 :KLFKRIWKHIROORZLQJVWDWHPHQWVDERXWFDOFLXPVWURQWLXPDQGEDULXPLVFRUUHFW"
$ 7KHPDJQLWXGHRIWKHK\GUDWLRQHQHUJ\RIWKH0LRQLQFUHDVHVIURPFDOFLXPWR
EDULXP
% 7KH HQHUJ\ UHTXLUHG IRU WKH SURFHVV 0 J   0 J   H LQFUHDVHV IURP
FDOFLXPWREDULXP
& 7KHUHGXFLQJSRZHUGHFUHDVHVIURPFDOFLXPWREDULXP
' 7KHUHDFWLYLW\RIWKHHOHPHQWVZLWKZDWHULQFUHDVHVIURPFDOFLXPWREDULXP

 :KHQ  J RI HDFK RI WKH IROORZLQJ *URXS ,, PHWDOV LV DGGHG WR FROG ZDWHU DQG DQ\
JDVSURGXFHGLVFROOHFWHGDW&DQGDWPZKLFKVDPSOHZLOOPRVWOLNHO\SURGXFH
WKHODUJHVWYROXPHRIJDV"
$ 0J % &D & 6U ' %D


 :KLFKVWDWHPHQWDERXWWKHWUHQGVLQWKHSURSHUWLHVRIWKHKDORJHQVLVFRUUHFW"
$ 7KHYRODWLOLW\RIKDORJHQVLQFUHDVHVGRZQWKHJURXS
% 7KHHOHFWURQHJDWLYLW\RIKDORJHQVLQFUHDVHVGRZQWKHJURXS
& 7KHUHDFWLYLW\RIKDORJHQVZLWKK\GURJHQGHFUHDVHVGRZQWKHJURXS
' 7KHERQGGLVVRFLDWLRQHQHUJ\RIKDORJHQVLQFUHDVHVGRZQWKHJURXS
 

trendyline
&-&-&3UHOLPLQDU\([DP
168 trendyline


 %URPLQH WDEOHWV DUHXVHG DV GLVLQIHFWDQWV,W LV D VRXUFH RI +&l2 DT  DQG +%U2 DT 
ERWKRIZKLFKDUHR[LGLVLQJDJHQWV
+%U2GHVWUR\VEDFWHULDIXQJLDQGDOJDHYLDLWVR[LGLVLQJSURSHUW\LQWKHSURFHVV+&l2
LVDVWURQJHUR[LGLVLQJDJHQWWKDQ+%U2DQGKHOSVWRVXVWDLQWKHHIIHFWLYHQHVVRIWKH
EURPLQHWDEOHWV
:KDWLVWKHOLNHO\UROHRI+&l2LQUHODWLRQWR+%U2"
$ 2[LGLVHV%UWR%U2
% 2[LGLVHV%UWR%U
& 5HGXFHV%U2WR%U
' 5HGXFHV%U2WR%U


 :KHQ GURSV RI DTXHRXV DPPRQLD DUH DGGHG WR D VROXWLRQ RI &X62 D SDOH EOXH
SUHFLSLWDWH LV IRUPHG 7KLV SUHFLSLWDWH GLVVROYHG ZKHQ H[FHVV DTXHRXV DPPRQLD LV
DGGHGIRUPLQJDGHHSEOXHVROXWLRQ2QDGGLWLRQRIGLOXWHK\GURFKORULFDFLGWKHSDOH
EOXHSUHFLSLWDWHLVUHIRUPHG
:KLFKSURFHVVGRHVQRWRFFXULQWKHDERYH"
$ 'DWLYHERQGIRUPDWLRQ
% )RUPDWLRQRIDFRPSOH[LRQ
& 3UHFLSLWDWLRQRIFRSSHU ,, K\GUR[LGH
' 5HGXFWLRQRIFRSSHU ,, LRQV
 

 3URJHVWHURQHLVDQHQGRJHQRXVVWHURLGLQYROYHGLQWKHPHQVWUXDOF\FOHDQGSUHJQDQF\
RIKXPDQV,WKDVWKHVWUXFWXUHVKRZQEHORZ


SURJHVWHURQH

+RZPDQ\FKLUDOFHQWHUVGRHVSURJHVWHURQHFRQWDLQ"
$  %  &  ' 



trendyline
 &-&-&3UHOLPLQDU\([DP >7XUQRYHU
169 trendyline


 Use of the Data Booklet is relevant to this question.


$QRUJDQLFFRPSRXQG$LVFRPPRQO\XVHGLQVNLQFDUHSURGXFWV,WKDVWKHIROORZLQJ
IHDWXUHV
 ,WLVDPRQREDVLFDFLG
 2QHPROHRIFRPSRXQG$UHDFWVZLWK1DWRJLYHPROHRI+JDV
 ,W GLPHULVHV LQ WKH SUHVHQFH RI KRW FRQFHQWUDWHG +62 WR JLYH DQ RUJDQLF
FRPSRXQGRIUHODWLYHPROHFXODUPDVV

+RZPDQ\FDUERQDWRPVDUHLQRQHPROHFXOHRIWKLVRUJDQLFFRPSRXQG$"
$  %  &  ' 


 GLPHWK\OSHQWDQH &+UHDFWV ZLWK EURPLQH WR IRUP PRQREURPR FRPSRXQGV ZLWK
PROHFXODUIRUPXODRI&+%U



+RZPDQ\SRVVLEOHVWUXFWXUDOLVRPHUVHDFKZLWKPROHFXODUIRUPXOD&+%UFRXOGEH
SURGXFHGE\GLPHWK\OSHQWDQH"
$  %  &  ' 


 9LQ\OF\FOREXWDQHFDQUHDFWZLWK+&lWRJLYHDUHDUUDQJHGDON\OFKORULGH:LWKWKHDLGRI
WKH IORZ RI HOHFWURQV UHSUHVHQWHG E\ WKH FXUYHG DUURZV ZKDW LV WKH DON\O FKORULGH
SURGXFWREWDLQHG"


$  %   &  '

      

trendyline





&-&-&3UHOLPLQDU\([DP
170 trendyline


 <DQXFDPLGH % FDQ EH H[WUDFWHG IURP D PDULQH VSRQJH DQG KDV WKH VWUXFWXUH VKRZQ
EHORZ


<DQXFDPLGH%
:KLFKRIWKHIROORZLQJVHWRIUHDJHQWVDQGFRQGLWLRQVZLOOQRWUHVXOWLQDFRORXUFKDQJH"

$ %ULQ&&l
% QHXWUDO)H&l DT 
& /L$l+LQGU\HWKHU
' KRWDFLGLILHG.&U2


 ,QWKHIROORZLQJVHTXHQFHRIUHDFWLRQVZKDWLVWKHPHFKDQLVPRIVWHS"


 

$ 1XFOHRSKLOLF$GGLWLRQ
% 1XFOHRSKLOLF6XEVWLWXWLRQ
& (OHFWURSKLOLF$GGLWLRQ
' (OHFWURSKLOLF6XEVWLWXWLRQ


trendyline
 &-&-&3UHOLPLQDU\([DP >7XUQRYHU
171 trendyline


 EURPRSURSDQH XQGHUJRHV QXFOHRSKLOLF VXEVWLWXWLRQ ZLWK DTXHRXV 1D2+ YLD WKH
IROORZLQJPHFKDQLVP

+
+
VORZ +& &  %U
+& & %U
&+
&+

+ +
+& & 
IDVW
2+ +& & 2+
&+
&+
:KLFKRIWKHUHDFWLRQSDWKZD\GLDJUDPILWVWKHDERYHPHFKDQLVP"
$      % 
 HQHUJ\
 HQHUJ\






 UHDFWLRQSDWKZD\
 UHDFWLRQSDWKZD\

&      '

HQHUJ\ HQHUJ\








UHDFWLRQSDWKZD\ UHDFWLRQSDWKZD\
 


trendyline
&-&-&3UHOLPLQDU\([DP
172 trendyline


 &RPSRXQG;&+%UXQGHUJRHVWKHIROORZLQJUHDFWLRQV
 1D2+ DT  &U2 DT + DT 
&+%U &+2 &+2RQO\
 KHDWXQGHUUHIOX[ KHDWXQGHUUHIOX[
 ;

:KDWLV;OLNHO\WREH"
$ EURPREXWDQH
% EURPREXWDQH
& EURPRPHWK\OSURSDQH
' EURPRPHWK\OSURSDQH


 &RPSRXQG ;VKRZQ EHORZ LV DQ LQWHUPHGLDWH XVHG WR JHQHUDWH S\UUROHV ZKLFK DUH
HVVHQWLDOWRWKHSURGXFWLRQRIPDQ\GLIIHUHQWFKHPLFDOVLQWKHSKDUPDFHXWLFDOLQGXVWU\


 &RPSRXQG;

:KLFKVHQWHQFHLVFRUUHFWIRUFRPSRXQG;"
$ ,WJLYHVDVLOYHUPLUURUZLWK7ROOHQVUHDJHQW
% ,WGHFRORXULVHVDFLGLILHGSRWDVVLXPPDQJDQDWH 9,, 
& ,WGRHVQRWJLYHDQRUDQJHSUHFLSLWDWHZLWKGLQLWURSKHQ\OK\GUD]LQH
' ,WJLYHV\HOORZSUHFLSLWDWHZLWKDONDOLQHDTXHRXVLRGLQH


 FRXPDUR\O+6/LVDQHZW\SHRIEDFWHULDOTXRUXPVHQVLQJVLJQDOFRPSRXQG:KHQ
RQH PROH RI FRXPDUR\O+6/ LV KHDWHG XQGHU UHIOX[ ZLWK 1D2+ DT  XQWLO QR IXUWKHU
UHDFWLRQRFFXUVKRZPDQ\PROHVRI1D2+ZLOOUHDFW"



$  %  &  ' 

trendyline
 &-&-&3UHOLPLQDU\([DP >7XUQRYHU
173 trendyline


 6KLNLPLFDFLGLVIRXQGLQVWDUDQLVH7KHIROORZLQJFRPSRXQG<LVWKHLQWHUPHGLDWHWKDW
FDQEHXVHGWRIRUPVKLNLPLFDFLG:KHQ/L$l+LVDGGHGWRFRPSRXQG<ZKDWLVWKH
SURGXFWREWDLQHG"


&RPSRXQG<

 &RQVLGHUWKHIROORZLQJFRPSRXQGVEHORZ
, &+1+ ,, &+&21+ ,,, &+&+1+ ,9 &+&+ 1+

 :KLFKRIWKHIROORZLQJVKRZVWKHFRUUHFWRUGHURIGHFUHDVLQJSKEYDOXHVIRUWKHDERYH
FRPSRXQGV"

$ ,,,,,,,9
% ,,,,,,,9
& ,,,,9,,,

trendyline
' ,9,,,,,,
,,,
,,,,,

 

&-&-&3UHOLPLQDU\([DP
174 trendyline


6HFWLRQ%

)RUHDFKRIWKHTXHVWLRQVLQWKLVVHFWLRQRQHRUPRUHRIWKHWKUHHQXPEHUHGVWDWHPHQWVWR
PD\EHFRUUHFW

'HFLGHZKHWKHUHDFKRIWKHVWDWHPHQWVLVRULVQRWFRUUHFW \RXPD\ILQGLWKHOSIXOWRSXWD
WLFNDJDLQVWWKHVWDWHPHQWVWKDW\RXFRQVLGHUWREHFRUUHFW 

7KHUHVSRQVHV$WR'VKRXOGEHVHOHFWHGRQWKHEDVLVRI

$ % & '
DQGDUHFRUUHFW DQGRQO\DUH DQGRQO\DUH RQO\LVFRUUHFW
FRUUHFW FRUUHFW

1RRWKHUFRPELQDWLRQRIVWDWHPHQWVLVXVHGDVDFRUUHFWUHVSRQVH

 Use of the Data Booklet is relevant to this question.


:KLFKRIWKHIROORZLQJVSHFLHVKDVWKHHOHFWURQLFFRQILJXUDWLRQRI>$U@GV"
 &U
 0Q
 )H


 7KHYDOXHRIWKHLRQLFSURGXFWRIZDWHUKZYDULHVZLWKWHPSHUDWXUH

WHPSHUDWXUHR& KZPROGP

 [

 [

:KLFKRIWKHIROORZLQJVWDWHPHQWLVFRUUHFW"
 7KHLRQLFGLVVRFLDWLRQRIZDWHULVDQHQGRWKHUPLFSURFHVV
 :DWHULVDQHXWUDOOLTXLGDW&
 S+DW&


 :KLFK RI WKH IROORZLQJ KDV WKH VDPH YDOXH DV WKH VWDQGDUG HQWKDOS\ FKDQJH RI
IRUPDWLRQHIRRIFDUERQPRQR[LGH"
 HFRPEXVWLRQR & HFRPEXVWLRQR &2 

ttrendyline
 HIR &2
&2  HHFRPEXVWLRQR &2 
H
 HHIR &2 



 &-&-&3UHOLPLQDU\([DP >7XUQRYHU
175 trendyline


7KHUHVSRQVHV$WR'VKRXOGEHVHOHFWHGRQWKHEDVLVRI

$ % & '
DQGDUHFRUUHFW DQGRQO\DUH DQGRQO\DUH RQO\LVFRUUHFW
FRUUHFW FRUUHFW

1RRWKHUFRPELQDWLRQRIVWDWHPHQWVLVXVHGDVDFRUUHFWUHVSRQVH


 Use of the Data booklet is relevant to this question.
,QWKH'DQLHOOFHOOFRSSHUDQG]LQFHOHFWURGHVDUHLPPHUVHGLQDVROXWLRQRIFRSSHU ,, 
VXOIDWHDQG]LQFVXOIDWHUHVSHFWLYHO\
7KHWZRKDOIHTXDWLRQVIRUD'DQLHOOFHOODUHJLYHQEHORZ

&X DT H&X V 

=Q DT H=Q V 

:KLFKVWDWHPHQWVDERXWWKLVFHOODUHFRUUHFWZKHQLWLVXVHG"
 7KHHOHFWURQIORZIURPWKH]LQFHOHFWURGHWRWKHFRSSHUHOHFWURGHLQWKHH[WHUQDO
FLUFXLW
 7KHRYHUDOOFHOOUHDFWLRQLV&X=Q&X=Q
 7KHFHOOSRWHQWLDOLV9


 $ QHXWUDO UHG FRPSOH[ )H$ LV IRUPHG ZKHQ D ELGHQWDWH OLJDQG $ LV DGGHG WR DQ
DTXHRXVVROXWLRQRI)HLRQV


$

:KLFKRIWKHIROORZLQJVWDWHPHQWVLVFRUUHFW"
 7KHFRPSOH[KDVDFRRUGLQDWLRQQXPEHURI
 7KHFRPSOH[DEVRUEVUDGLDWLRQLQWKHEOXHJUHHQUHJLRQRIWKHYLVLEOHVSHFWUXP
 7KH FRPSOH[ KDV D VPDOOHU HQHUJ\ JDS EHWZHHQ GRUELWDOV FRPSDUHG WR \HOORZ
>)H +2 @

trendyline
&-&-&3UHOLPLQDU\([DP
176 trendyline


 7KHJUDSKVKRZVKRZDSURSHUW\IURP1DWR&lLQ3HULRGYDULHVZLWKSURWRQQXPEHU


      
SURWRQQXPEHU

:KDWLVWKLVSURSHUW\"
 ,RQLFUDGLXV
 )LUVWLRQLVDWLRQHQHUJ\
 (OHFWURQHJDWLYLW\


 :LOOLDPVRQ V\QWKHVLV LV RQH RI WKH EHWWHU PHWKRGV WR SUHSDUH HWKHUV 525 ,W
LQYROYHVD61GLVSODFHPHQWRIKDOLGHLRQLQKDORJHQRDONDQH 5; E\DQDONR[LGHLRQ
QXFOHRSKLOH 52 DVVKRZQ
525;525;
HWKHU

:KLFKRIWKHIROORZLQJVWDWHPHQWVDUHWUXH"
 7KHUDWHHTXDWLRQLQYROYHGLVUDWH N>52@>5;@
 7KHUHDFWLRQLQYROYLQJEURPRPHWKDQHZLOOEHIDVWHUFRPSDUHGWRFKORURPHWKDQH
 7KH UHDFWLRQ LQYROYLQJ EURPRSURSDQH ZLOO EH IDVWHU FRPSDUHG WR
EURPRPHWKDQH


 &DUV DUH ILWWHG ZLWK D FDWDO\WLF FRQYHUWHU :KDW UHDFWLRQV W\SLFDOO\ RFFXU LQ D FDWDO\WLF
FRQYHUWHU"
 12&21&2
[ \ \
 &[+\  2[&2 +2
  
 &212&212


trendyline
 &-&-&3UHOLPLQDU\([DP >7XUQRYHU
177 trendyline


7KHUHVSRQVHV$WR'VKRXOGEHVHOHFWHGRQWKHEDVLVRI

$ % & '
DQGDUHFRUUHFW DQGRQO\DUH DQGRQO\DUH RQO\LVFRUUHFW
FRUUHFW FRUUHFW

1RRWKHUFRPELQDWLRQRIVWDWHPHQWVLVXVHGDVDFRUUHFWUHVSRQVH


 $WRVLEDQDSURWHLQLVXVHGDVDQLQWUDYHQRXVPHGLFDWLRQDVDELUWKODERXUUHSUHVVDQW
WRKDOWSUHPDWXUHODERXU


$WRVLEDQ
:KLFKRIWKHIROORZLQJ5JURXSLQWHUDFWLRQVLQWKHSURWHLQDUHSRVVLEOH"
 +\GURJHQERQG
 'LVXOILGHERQG
 ,RQLFERQG



trendyline
&-&-&3UHOLPLQDU\([DP
178 trendyline


 7KH LVRPHULVDWLRQ RI HWKR[\HWKDQH WR SURGXFH WKH FRUUHVSRQGLQJ DOFRKRO LV VKRZQ
EHORZ


 
:KLFKRIWKHIROORZLQJVKRZVWKHFRUUHFWSURGXFWIRUPHGXQGHUWKHVDPHFRQGLWLRQV"



 




 
 





 




 

trendyline
 &-&-&3UHOLPLQDU\([DP >7XUQRYHU
179 trendyline


%/$1.3$*(

trendyline
&-&-&3UHOLPLQDU\([DP
180 trendyline


&$7+2/,&-81,25&2//(*(
-&35(/,0,1$5<(;$0,1$7,216
+LJKHU

&+(0,675< 
3DSHU0XOWLSOH&KRLFH :HGQHVGD\$XJXVW
KRXU
$GGLWLRQDO0DWHULDOV 0XOWLSOH&KRLFH$QVZHU6KHHW
'DWD%RRNOHW

5($'7+(6(,16758&7,216),567

:ULWHLQVRIWSHQFLO
'RQRWXVHVWDSOHVSDSHUFOLSVJOXHRUFRUUHFWLRQIOXLG
:ULWH\RXUQDPHDQG+7JURXSRQWKH$QVZHU6KHHWLQWKHVSDFHVSURYLGHG

7KHUHDUHIRUW\TXHVWLRQVRQWKLVSDSHU$QVZHUDOOTXHVWLRQV)RUHDFKTXHVWLRQWKHUHDUHIRXU
SRVVLEOHDQVZHUV$%&DQG'
&KRRVHWKHRQH\RXFRQVLGHUFRUUHFWDQGUHFRUG\RXUFKRLFHLQVRIWSHQFLORQWKHVHSDUDWH$QVZHU
6KHHW

5HDGWKHLQVWUXFWLRQVRQWKH$QVZHU6KHHWYHU\FDUHIXOO\

(DFKFRUUHFWDQVZHUZLOOVFRUHRQHPDUN$PDUNZLOOQRWEHGHGXFWHGIRUDZURQJDQVZHU
$Q\URXJKZRUNLQJVKRXOGEHGRQHLQWKLVERRNOHW
7KHXVHRIDQDSSURYHGVFLHQWLILFFDOFXODWRULVH[SHFWHGZKHUHDSSURSULDWH

:25.('62/87,216

trendyline
7KLVGRFXPHQWFRQVLVWVRISULQWHGSDJHV
7KLVGRFXPHQWFRQVLVWVRI
LVGRFXPHQWFRQVLVWVRI
 SULQWHGSDJHV
ULQWHG SDJHV

 &-&-&3UHOLPLQDU\([DP >7XUQRYHU
181 trendyline


6HFWLRQ$

)RU HDFK TXHVWLRQ WKHUH DUH four SRVVLEOH DQVZHUV A B C DQG D &KRRVH WKH RQH \RX
FRQVLGHUWREHcorrectDQGUHFRUG\RXUFKRLFHLQVRIWSHQFLORQWKHseparate Answer Sheet
SURYLGHG

 Use of the Data Booklet is relevant to this question.

$UVHQLF ,,, VXOILGH$V6LVDEULJKW\HOORZVROLGZKLFKKDVEHHQXVHGDVDSLJPHQWLQ


SDLQWLQJV
:KHQDUVHQLF ,,, VXOILGHLVKHDWHGLQDLULWUHDFWVZLWKR[\JHQWRJLYHDUVHQLF ,,, R[LGH
DQGVXOIXU ,9 R[LGH
$V62$V262
:KDWZRXOGEHWKHPDVVRIDUVHQLF ,,, R[LGHSURGXFHGLIGPR[\JHQPHDVXUHG
DWURRPWHPSHUDWXUHDQGSUHVVXUHUHDFWHGZLWKDUVHQLF ,,, VXOILGH"

$ J % J & J ' J

 $QVZHU$

 1XPEHURIPROHVRI2   PRO

 6LQFH2$V2

 1XPEHURIPROHVRI$V2   PRO

 0URI$V2  


 0DVVRI$V2  J
 
 ,QFRPSOHWHFRPEXVWLRQRIFPRIDJDVHRXVK\GURFDUERQJDYHFDUERQGLR[LGHDQG
FDUERQPRQR[LGHLQDUDWLRDVZHOODVZDWHUYDSRXU,WZDVIRXQGWKDWWKHFDUERQ
GLR[LGHWRRNXSDYROXPHRIFPZKHUHDVWKHZDWHUYDSRXUWRRNXSDYROXPHRI
FP$OOJDVYROXPHVDUHPHDVXUHGDWWKHVDPHWHPSHUDWXUHDQGSUHVVXUH

:KDWLVWKHPROHFXODUIRUPXODRIWKHK\GURFDUERQ"

$ &+ % &+ & &+ ' &+

 $QVZHU'

&[+\"2[&2[&2 +2


 5HGXFWLRQRIFP DIWHUSDVVLQJWKURXJKGHK\GUDWLQJDJHQW
 YROXPHRIZDWHUYDSRXUZDVFP

trendyline

 RIFP
RI
5HGXFWLRQRIFP DIWHUSDVVLQJWKURXJK.2+ DT 
DIWHUSDVVLQJWKURXJK.2+ DT 
PHRI&2ZDVFP
 YROXPHRI&2
 RI&[+\&2
0ROHUDWLRRI& &2 [ 



&-&-&3UHOLPLQDU\([DP
182 trendyline


     [ 

 0ROHUDWLRRI&[+\+2   



      

     \ 
 7KHUHIRUHWKHPROHFXODUIRUPXODRIWKHK\GURFDUERQLV&+

 3RO\WKLRQDWHVDUHDVHULHVRIVXOIXUR[RDQLRQVZLWKWKHJHQHUDOIRUPXOD6n2ZKHUH
n!$VLPSOHH[DPSOHLVWHWUDWKLRQDWHLRQ62

:KLFK RI WKH IROORZLQJ JUDSKV EHVW LOOXVWUDWHV WKH YDULDWLRQ LQ WKH DYHUDJH R[LGDWLRQ
QXPEHURIVXOIXULQ6n2ZLWKn"
 DYHUDJHR[LGDWLRQ
QXPEHURI6



 $

%



&

 '

   QXPEHURI6DWRPVn

 $QVZHU%
 /HWWKHDYHUDJHR[LGDWLRQQXPEHURI6EH\
 )RU6Q2   n\
   n\ 

   \  

 +HQFHWKHJUDSKLVDK\SHUEROLFJUDSKVWDUWLQJIURPWKHSRLQW  ZKLFK


FRUUHVSRQGVWRJUDSK%

 :KLFKRIWKHIROORZLQJVSHFLHVKDVWKHVPDOOHVWERQGDQJOHDURXQGWKHFHQWUDODWRP"

trendyline

$ 1&l %
% 3&l & 62 '
' %)




 &-&-&3UHOLPLQDU\([DP >7XUQRYHU
183 trendyline


 $QVZHU%

   

ESOS 
ESOS 
 ES ESOS
$QG3LVOHVV 
 OS
HOHFWURQHJDWLYHWKDQ
1VRHOHFWURQSDLUV
OHVVDWWUDFWHGWR3
KHQFHVPDOOHVWERQG
DQJOH

 :KLFKRIWKHIROORZLQJVKRZVWKHFRUUHFWERQGVSUHVHQWLQVROLG&+,"
$ &RYDOHQWERQGVRQO\
% ,RQLFERQGVRQO\
& &RYDOHQWERQGV WHPSRUDU\GLSROHLQGXFHGGLSROHIRUFHVRIDWWUDFWLRQ
' &RYDOHQWERQGV SHUPDQHQWGLSROHSHUPDQHQWGLSROHIRUFHVRIDWWUDFWLRQ

 $QVZHU'
&+, WULLRGRPHWKDQH LVDVLPSOHFRYDOHQWPROHFXOHZKLFKLVSRODU KHQFHLWFRQWDLQV
FRYDOHQW ERQGV EHWZHHQ DWRPV DQG SHUPDQHQW GLSROHSHUPDQHQW GLSROH IRUFHV RI
DWWUDFWLRQEHWZHHQPROHFXOHV

 3RO\PHULVDWLRQLVDSURFHVVLQZKLFKODUJHQXPEHURIVPDOOPROHFXOHV PRQRPHUV DUH
MRLQHGWRJHWKHUWRIRUPORQJFKDLQPROHFXOHV
&RQVLGHULQJWKDWSRO\PHULVDWLRQLVDVSRQWDQHRXVUHDFWLRQZKDWDUHWKHFRUUHFWVLJQV
RISDQGHIRUWKHSURFHVV"

 S H
$  
%  
&  
'  

 $QVZHU$
G QHJDWLYHEHFDXVHLWLVDVSRQWDQHRXVUHDFWLRQ
S QHJDWLYHEHFDXVHWKHGHJUHHRIGLVRUGHUQHVVGHFUHDVHVDVSRO\PHULVDWLRQIRUP
RUGHUO\ORQJFKDLQPROHFXOHV
6LQFHG = H - TSZLWKG QHJDWLYHDQG 7S = positive, H KDVWREHQHJDWLYH



trendyline




 61 LV D WKHUPRFKURPLF VROLG ZKLFK FKDQJHV FRORXU ZLWK WHPSHUDW
WHPSHUDWXUH
WHPSHUDWX 61 KDV D
FDJHVWUXFWXUHDVVKRZQLQWKHGLDJUDP
FWXUH DV VKRZQ LQ WKH GLDJUDP

&-&-&3UHOLPLQDU\([DP
184 trendyline


6 6

1 1
1 6 1
6
61 
*LYHQ WKH IROORZLQJ GDWD ZKDW LV WKH DYHUDJH ERQG HQHUJ\ RI GLVVRFLDWLRQ RI 61 LQ
N-PRO"
+I 61  N-PRO 

+DW VXOIXU  N-PRO


+DW QLWURJHQ  N-PRO
66 ERQGHQHUJ\RIGLVVRFLDWLRQLQ61 N-PRO

$  %  &  ' 


$QVZHU%
6 V 1 J  
 61 V 

  

   H


6 J 1 J 
 
H >    @ N-PRO
7RWDO%RQG(QHUJ\ N-PRO %( 61 %( 66 
%( 61     
%( 61   N-PRO

 $W  & D GHQWHG SLQJ SRQJ EDOO KDV DQ LQWHUQDO YROXPH RI  FP DQG LQWHUQDO
SUHVVXUH RI  N3D ,W LV WKHQ SODFHG LQ D ZDWHU EDWK PDLQWDLQHG DW  & ZKLFK
UHWXUQHGWKHEDOOWRLWVRULJLQDOVSKHULFDOVKDSHZLWKLQWHUQDOYROXPHRIFP
$VVXPLQJLGHDOJDVEHKDYLRXUZKDWLVWKHSUHVVXUHRIDLULQVLGHWKHEDOODW&LQLWV
RULJLQDOVSKHULFDOVKDSH"

$ N3D % N3D & N3D ' N3D

 trendyline
 $QVZHU%
%
%

 &-&-&3UHOLPLQDU\([DP >7XUQRYHU
185 trendyline


3 9 3 9
 
7 7
  3 
 
 

S N3D

 *DV;GHFRPSRVHVWRWZRRWKHUJDVHV<DQG=DFFRUGLQJWRWKHIROORZLQJHTXDWLRQ
; J < J = J 
7KHJUDSKEHORZVKRZVWKHGHFRPSRVLWLRQRIPRORISXUHJDV;LQWKHSUHVHQFHRI
DFDWDO\VWDWYDULRXVWHPSHUDWXUHV





PROHVRI; 
UHPDLQLQJ

 &

&



WLPH 

:KLFKRQHRIWKHIROORZLQJVWDWHPHQWVDERXWWKHDERYHV\VWHPLVFRUUHFW"
$ 7KHGHFRPSRVLWLRQRI;LVHQGRWKHUPLF
% 7KHKpRIWKHV\VWHPGHFUHDVHVZLWKLQFUHDVLQJWHPSHUDWXUH
& 7KHSHUFHQWDJHGHFRPSRVLWLRQRI;LVDWR&
' 7KHV\VWHPEHFRPHVOHVVGLVRUGHUHGZKHQLWUHDFKHVHTXLOLEULXPVWDWH

  $QVZHU%
$ $V7LQFUHDVHGDPRXQWRI;UHPDLQLQJDWHTXLOLEULXPLQFUHDVHG7KLVVKRZVWKDW
SRVLWLRQ RI HTXLOLEULXP VKLIWV OHIW KHQFH EDFNZDUG UHDFWLRQ LV HQGRWKHUPLF
)RUZDUGUHDFWLRQLVWKXVH[RWKHUPLFLQFRUUHFW
% 3RVLWLRQ RI HTXLOLEULXP VKLIWV OHIW ZLWK LQFUHDVLQJ 7 WR IDYRXU HQGRWKHUPLF UHDFWLRQ
VLQFHIRUZDUGUHDFWLRQLVH[RWKHUPLF 7KXVKSZLOOGHFUHDVHZLWKLQFUHDVLQJ7
FRUUHFW
&$W R&DPRXQWRI;UHPDLQLQJLVPRO+HQFHSHUFHQWDJHGHFRPSRVLWLRQRI
;LVLQVWHDGRILQFRUUHFW

trendyline
' 7KHUHLVDQLQFUHDVHLQWKHQXPEHURIPROHVRIJDVHRXVSDUWLFOHVUHVXOWLQJLQDQ
HLVDQLQFUHDVHLQWKHQXPEHURIPROHVRIJDVHRXVSDUWLFO
LVDQLQFUHDVHLQWKHQXPEHURIPROHVRIJ H
DVHLQGHJUHHRIUDQGRPQHVV
DVHLQGHJUHHRIUDQGRPQHVV LQFRUUHFW
LQFUHDVHLQGHJUHHRIUDQGRPQHVVLQFRUUHFW
QFRUUHFW


&-&-&3UHOLPLQDU\([DP
186 trendyline


 0DORQLFDFLG+2&&+&2+LVDZHDNGLEDVLFDFLG
0DORQLFDFLGLVWLWUDWHGDJDLQVWFPRIVRGLXPK\GUR[LGHRIWKHVDPHFRQFHQWUDWLRQ
WKHIROORZLQJS+YROXPHFXUYHLVREWDLQHG

$WZKLFKSRLQWRQWKHWLWUDWLRQFXUYHLVWKHPL[WXUHPRVWDEOHWRUHVLVWS+FKDQJHXSRQ
DGGLWLRQRIDVPDOODPRXQWRIDTXHRXVDFLGRUEDVH"

S+ 

 $
 ,

%



&

 '

 9RORI
PDORQLFDFLG
     FP

$QVZHU'
$1D2+ZLOOEHSUHVHQWLQH[FHVV1REXIIHUSUHVHQW
%1D2+VWLOOSUHVHQWLQH[FHVV

&)LUVWHTXLYDOHQFHSRLQW+2&+&+&2 DQGZDWHUSUHVHQW

'0D[LPXP EXIIHULQJ FDSDFLW\ ZKHUH +2&+&+&2  ZHDN DFLG  DQG

2&+&+&2 VDOW DUHSUHVHQW

 7KHIROORZLQJLVWKHUHDFWLRQIRUFRPSRXQG;<
;< J ; J < J 
,I WKH UDWH FRQVWDQW IRU WKH UHDFWLRQ LV  [  V ZKDW LV WKH WLPH WDNHQ IRU WKH
FRQFHQWUDWLRQRIDVDPSOHRI;<WRGHFUHDVHIURPPROGPWRPROGP"
$ V % V & V ' V

$QVZHU&

 W  


 

trendyline



 




 [ V
V
V


 &-&-&3UHOLPLQDU\([DP >7XUQRYHU
187 trendyline


 7KHUHDFWLRQEHWZHHQ&DQG'LVDVIROORZV
&'(
,QDQH[SHULPHQWWRLQYHVWLJDWHWKHHIIHFWRIFRQFHQWUDWLRQVRQWKHUDWHRIUHDFWLRQWKH
IROORZLQJUHVXOWVZHUHREWDLQHGDWFRQVWDQWWHPSHUDWXUH

([SHULPHQW >&@PROGP >'@PROGP ,QLWLDOUDWHPROGPV
   
   
   [
   

:KDWLVWKHYDOXHRI["
$  %  &  ' 

$QVZHU&
 &RPSDULQJH[SHULPHQWVDQGZKHQ>'@LVGRXEOHGUDWHGRXEOHV2UGHURIUHDFWLRQ
ZUW'LV
 &RPSDULQJH[SHULPHQWVDQGZKHQ>&@LVGRXEOHGUDWHGRXEOHV2UGHURIUHDFWLRQ
ZUW&LV
 5DWH N>&@>'@
 &RPSDULQJH[SHULPHQWVDQGZKHQ>'@WULSOHVUDWHVKRXOGWULSOH+HQFHYDOXHRI[
LV PROGPV

 7KHIROORZLQJJUDSKVKRZVKRZDSURSHUW\RIWKHHOHPHQWVLQ3HULRGIURP1DWR3
RUWKHLUFRPSRXQGVYDULHVZLWKSURWRQQXPEHU

SURWRQQXPEHU
    

:KDWSURSHUW\LVVKRZQE\WKHJUDSK"
$ 0HOWLQJSRLQWRIHOHPHQW
% 0HOWLQJSRLQWRIR[LGH 
& S+RIDTXHRXVFKORULGH
' &RQGXFWLYLW\RIHOHPHQW

trendyline






&-&-&3UHOLPLQDU\([DP
188 trendyline


$QVZHU'
(OHFWULFDOFRQGXFWLYLW\LQFUHDVHVIURP1DWR$OGXHWRWKHLQFUHDVHLQQXPEHURIPRELOH
GHORFDOLVHGHOHFWURQV(OHFWULFDOFRQGXFWLYLW\WKHQGHFUHDVHWR6LDV6LLVDVHPL
FRQGXFWRUDQGKDVSRRUFRQGXFWLYLW\XQGHUQRUPDOFRQGLWLRQV3GRHVQRWFRQGXFW
HOHFWULFLW\DVLWQHLWKHUKDYHGHORFDOLVHGHOHFWURQVQRUPRELOHFKDUJHFDUULHUV

 :KLFKRIWKHIROORZLQJVWDWHPHQWVDERXWFDOFLXPVWURQWLXPDQGEDULXPLVFRUUHFW"
$ 7KHPDJQLWXGHRIWKHK\GUDWLRQHQHUJ\RIWKH0LRQLQFUHDVHVIURPFDOFLXPWR
EDULXP
% 7KH HQHUJ\ UHTXLUHG IRU WKH SURFHVV 0 J   0 J   H LQFUHDVHV IURP
FDOFLXPWREDULXP
& 7KHUHGXFLQJSRZHUGHFUHDVHVIURPFDOFLXPWREDULXP
' 7KHUHDFWLYLW\RIWKHHOHPHQWVZLWKZDWHULQFUHDVHVIURPFDOFLXPWREDULXP

$QVZHU'

$  K\GUDWLRQ HQHUJ\  6LQFH VL]H LQFUHDVHV GRZQ WKH JURXS PDJQLWXGH RI

K\GUDWLRQHQHUJ\GHFUHDVHV
%,(GHFUHDVHVGRZQWKHJURXSDVYDOHQFHHOHFWURQVDUHOHVVVWURQJO\DWWUDFWHGWR
WKHPHWDOQXFOHXV
&  5HGXFLQJ SRZHU RU HDVH RI EHLQJ R[LGLVHG LH ORVV RI HOHFWURQV WR IRUP 0 
LQFUHDVHVGRZQWKHJURXS
'5HDFWLYLW\GRHVLQFUHDVHGRZQWKHJURXSGXHWRLQFUHDVHGUHGXFLQJSRZHUGRZQ
WKHJURXS

 :KHQ  J RI HDFK RI WKH IROORZLQJ *URXS ,, PHWDOV LV DGGHG WR FROG ZDWHU DQG DQ\
JDVSURGXFHGLVFROOHFWHGDW&DQGDWPZKLFKVDPSOHZLOOPRVWOLNHO\SURGXFH
WKHODUJHVWYROXPHRIJDV"
$ 0J % &D & 6U ' %D

$QVZHU%
*URXS,,PHWDOVUHDFWZLWKFROGZDWHUWRIRUPWKHLUK\GUR[LGHVDQGK\GURJHQJDV
   0 V +2 l 0 2+  DT + J 
 

$ 1RRIPRORI0J   PROEXWWKHUHZLOOEHDOPRVWQRUHDFWLRQLQFROG

ZDWHUGXHWRLQVROXEOH0J 2+ OD\HUSUHYHQWLQJIXUWKHUUHDFWLRQ

%1RRIPRORI&D   PRODQGDOWKRXJK&D 2+ LVRQO\SDUWLDOO\VROXEOH

WKHUHDFWLRQVKRXOGJRWRFRPSOHWLRQ/DUJHVWDPRXQW LQPROHV RIJDVSURGXFHG
KHQFHODUJHVWYROXPH

&1RRIPRORI6U   PRO  

e


I PRORI%D 
'1RRIPRORI%D  PRO  
PRO 


 :KLFKVWDWHPHQWDERXWWKHWUHQGVLQWKHSURSHUWLHVRIWKHKDORJHQVLVFRUUHFW"
WHPHQWDERXWWKHWUHQGVLQWKHSURSHUWLHVRIWKHKDORJ
PHQWDERXWWKHWUHQGVLQWKHSURSH HQVLVF
L

 &-&-&3UHOLPLQDU\([DP >7XUQRYHU
189 trendyline


$ 7KHYRODWLOLW\RIKDORJHQVLQFUHDVHVGRZQWKHJURXS
% 7KHHOHFWURQHJDWLYLW\RIKDORJHQVLQFUHDVHVGRZQWKHJURXS
& 7KHUHDFWLYLW\RIKDORJHQVZLWKK\GURJHQGHFUHDVHVGRZQWKHJURXS
' 7KHERQGGLVVRFLDWLRQHQHUJ\RIKDORJHQVLQFUHDVHVGRZQWKHJURXS

$QVZHU&
$  %RLOLQJSRLQW LQFUHDVHV GRZQ WKH JURXS JDV WR VROLG KHQFH YRODWLOLW\ GHFUHDVHV
GRZQWKHJURXS
%(OHFWURQHJDWLYLW\GHFUHDVHVGRZQWKHJURXSDVDWRPLFUDGLXVLQFUHDVHV
&5HDFWLYLW\GHFUHDVHVGRZQWKHJURXSDVVWUHQJWKRI+;ERQGGHFUHDVHV
' %RQGGLVVRFLDWLRQHQHUJ\GHFUHDVHVGRZQWKHJURXSDVVL]HRI DWRPVLQFUHDVH
UHGXFLQJHIIHFWLYHQHVVRIRUELWDORYHUODS

 %URPLQH WDEOHWV DUHXVHG DV GLVLQIHFWDQWV,W LV D VRXUFH RI +&l2 DT  DQG +%U2 DT 
ERWKRIZKLFKDUHR[LGLVLQJDJHQWV
+%U2GHVWUR\VEDFWHULDIXQJLDQGDOJDHYLDLWVR[LGLVLQJSURSHUW\LQWKHSURFHVV+&l2
LVDVWURQJHUR[LGLVLQJDJHQWWKDQ+%U2DQGKHOSVWRVXVWDLQWKHHIIHFWLYHQHVVRIWKH
EURPLQHWDEOHWV
:KDWLVWKHOLNHO\UROHRI+&l2LQUHODWLRQWR+%U2"
$ 2[LGLVHV%UWR%U2
% 2[LGLVHV%UWR%U
& 5HGXFHV%U2WR%U
' 5HGXFHV%U2WR%U

$QVZHU$
2[LGDWLRQVWDWHRI%ULQ+%U2LV:KHQ+%U2GHVWUR\VEDFWHULDWKURXJKLWVR[LGLVLQJ
SURSHUW\ LW JHWV UHGXFHG WR %U +RZHYHU VLQFH +&l2 LV D VWURQJHU R[LGLVLQJ DJHQW
WKDQ+%U2KHQFHLWZLOOR[LGLVH%UWR%U27KLVKHOSVWRVXVWDLQWKHHIIHFWLYHQHVVRI
WKHEURPLQHWDEOHWVE\UHJHQHUDWLQJ%U2

&l2+H&l+2 (R
%U2+H%U+2  (R
(R!(R

 :KHQ GURSV RI DTXHRXV DPPRQLD DUH DGGHG WR D VROXWLRQ RI &X62 D SDOH EOXH
SUHFLSLWDWH LV IRUPHG 7KLV SUHFLSLWDWH GLVVROYHG ZKHQ H[FHVV DTXHRXV DPPRQLD LV
DGGHGIRUPLQJDGHHSEOXHVROXWLRQ2QDGGLWLRQRIGLOXWHK\GURFKORULFDFLGWKHSDOH
EOXHSUHFLSLWDWHLVUHIRUPHG

trendyline
:KLFKSURFHVVGRHVQRWRFFXULQWKHDERYH"
FH
FHVVGRHV QRW RFFXULQWKHDERYH"
ER
$ 'DWLYHERQGIRUPDWLRQ
YHERQGIRUPDWLRQ
YHERQGIRUPDWLRQ
% )RUPDWLRQRIDFRPSOH[LRQ
PDWLRQRIDFRPSOH[LRQ
DWLRQRIDFRPSOH[LRQ
& 3UHFLSLWDWLRQRIFRSSHU ,, K\GUR[LGH
FLSLWDWLRQRIFRSSHU ,, K\GUR[LGH
SLWDWLRQ RI FRSSHU ,, K\GUR[LGH
' 5HGXFWLRQRIFRSSHU ,, LRQV
XFWLRQ RI FRSSHU ,, LRQV

&-&-&3UHOLPLQDU\([DP
190 trendyline


$QVZHU'  
 :KHQDIHZGURSVRIDTXHRXVDPPRQLDDUHDGGHGWRVROXWLRQRI&X62WKHIROORZLQJ
UHDFWLRQRFFXUVDQGDSDOHEOXHSSW&X 2+ LVIRUPHG
 &X DT 2+ DT &X 2+  V 
8SRQDGGLWLRQRIH[FHVVDPPRQLDWKHSDOHEOXHSSWGLVVROYHV$FRPSOH[&X 1+ @
LVIRUPHG
 &X 2+  V 1+ DT >&X 1+ @ DT 2+ DT   

'DWLYHERQGV


:KHQGLOXWHK\GURFKORULFDFLGLVDGGHGLWUHDFWVZLWKWKHDPPRQLDOLJDQGVDQG
FDXVHVHTXLOLEULXPSRVLWLRQ  WRVKLIWWRWKHOHIWIRUPLQJEDFN&X 2+ VHHQDVWKH
EOXHSSW
5HGXFWLRQRIFRSSHU ,, LRQVGRHVQRWRFFXU
 3URJHVWHURQHLVDQHQGRJHQRXVVWHURLGLQYROYHGLQWKHPHQVWUXDOF\FOHDQGSUHJQDQF\
RIKXPDQV,WKDVWKHVWUXFWXUHVKRZQEHORZ


SURJHVWHURQH

+RZPDQ\FKLUDOFHQWHUVGRHVSURJHVWHURQHFRQWDLQ"
$  %  &  ' 

$QVZHU& 

FKLUDOFHQWHUV

trendyline


 &-&-&3UHOLPLQDU\([DP >7XUQRYHU
191 trendyline


 Use of the Data Booklet is relevant to this question.


$QRUJDQLFFRPSRXQG$LVFRPPRQO\XVHGLQVNLQFDUHSURGXFWV,WKDVWKHIROORZLQJ
IHDWXUHV
 ,WLVDPRQREDVLFDFLG
 2QHPROHRIFRPSRXQG$UHDFWVZLWK1DWRJLYHPROHRI+JDV
 ,W GLPHULVHV LQ WKH SUHVHQFH RI KRW FRQFHQWUDWHG +62 WR JLYH DQ RUJDQLF
FRPSRXQGRIUHODWLYHPROHFXODUPDVV

+RZPDQ\FDUERQDWRPVDUHLQRQHPROHFXOHRIWKLVRUJDQLFFRPSRXQG$"
$  %  &  ' 

$QVZHU%
 ,WLVPRQREDVLF&2+IXQFWLRQDOJURXSSUHVHQW
 6LQFH52+1D+521DDQGPRORI$JLYHVPROHRI+JDV
2+IXQFWLRQDOJURXSVSUHVHQW
 ,W GLPHULVHV LQ WKH SUHVHQFH RI KRW FRQFHQWUDWHG +62 WR JLYH D FRPSRXQG RI
UHODWLYHPROHFXODUPDVVWKHDFLGKDV&2+JURXSDQG2+JURXS,QWKH
SUHVHQFHRI+62HVWHULILFDWLRQZLOORFFXU$F\FOLFSURGXFWLVIRUPHG

 
*LYHQWKH0UWKHUHDUHFDUERQVLQWKHSURGXFWVLQFHSURGXFWLVIRUPHGIURPWKH
GLPHUVLDWLRQRIFRPSRXQG$WKXVFRPSRXQG$KDVFDUERQV



 GLPHWK\OSHQWDQH &+UHDFWV ZLWK EURPLQH WR IRUP PRQREURPR FRPSRXQGV ZLWK
PROHFXODUIRUPXODRI&+%U



+RZPDQ\SRVVLEOHVWUXFWXUDOLVRPHUVHDFKZLWKPROHFXODUIRUPXOD&+%UFRXOGEH
SURGXFHGE\GLPHWK\OSHQWDQH"
$  %  &  ' 

$QVZHU$
GLPHWK\OSHQWDQH XQGHUJRHV IUHH UDGLFDO VXEVWLWXWLRQ LQ WKH SUHVHQFH RI VXQOLJKW
7KHUH DUH  GLIIHUHQW VHWV RI FKHPLFDOO\ HTXLYDOHQW K\GURJHQ DWRPV WKDW FDQ EH

trendyline
G WR JLYH  GLIIHUHQW PRQREURPR SURGXFWV
VXEVWLWXWHG SURGXF W
1RWH WKDW WKH PROHFXOH LV
FDO
FDO
V\PPHWULFDO

&-&-&3UHOLPLQDU\([DP
192 trendyline




 9LQ\OF\FOREXWDQHFDQUHDFWZLWK+&lWRJLYHDUHDUUDQJHGDON\OFKORULGH:LWKWKHDLGRI
WKH IORZ RI HOHFWURQV UHSUHVHQWHG E\ WKH FXUYHG DUURZV ZKDW LV WKH DON\O FKORULGH
SURGXFWREWDLQHG"


$  %   &  '

      
$QVZHU&

7KHGLDJUDPEHORZVKRZVWKHEUHDNGRZQRIWKHIORZRIHOHFWURQV+RZHYHUWKHILUVW
VWHSVVKRXOGEHFRQFHUWHG
+ &l
&l
 
     
      
&l  
 
 
    

7KHDONHQHIXQFWLRQDOJURXSZLOOXQGHUJRHOHFWURSKLOLFDGGLWLRQZLWK+&l
%\IROORZLQJWKHDUURZZKLFKUHSUHVHQWVWKHIORZRIHOHFWURQV
 WKHERQGEHWZHHQ&DQG&LVEURNHQ
 DQHZERQGLVIRUPHGEHWZHHQ&DQG&
 &ZLOOEHSRVLWLYHO\FKDUJHG
 $PHPEHUHGULQJLVIRUPHG

trendyline

&lZLOODWWDFNWKHFDUERFDWLRQRQ&
7KXVWKH&l ZLOODWWDFNWKHFDUERFDWLRQRQ&

 &-&-&3UHOLPLQDU\([DP >7XUQRYHU
193 trendyline


 <DQXFDPLGH % FDQ EH H[WUDFWHG IURP D PDULQH VSRQJH DQG KDV WKH VWUXFWXUH VKRZQ
EHORZ


<DQXFDPLGH%
:KLFKRIWKHIROORZLQJVHWRIUHDJHQWVDQGFRQGLWLRQVZLOOQRWUHVXOWLQDFRORXUFKDQJH"

$ %ULQ&&l
% QHXWUDO)H&l DT 
& /L$l+LQGU\HWKHU
' KRWDFLGLILHG.&U2

$QVZHU&
/L$l+UHDFWZLWK<DQXFDPLGH%E\UHGXFLQJWKHDPLGHDQGHVWHUIXQFWLRQDOJURXSV
KRZHYHUWKHUHLVQRFRORXUFKDQJH

9LROHWFRORXUDWLRQ
REVHUYHGZKHQ)H&l
LVUHDFWHGZLWKSKHQRO

tren
tren
%URZQ%Undyline
dyli
ylin
inee
%UGHFRORXULVH
GHFRORXULV
ZKHQUHDFWHGZLWKDONHQH
IXQFWLRQDOJURXS
KRWDF 
KRWDFLGLILHG. &U2
WXUQHGIURPRUDQJHWR
JUHHQZKHQUHDFWHGZLWK
DOFRKROIXQFWLRQDOJURXS

&-&-&3UHOLPLQDU\([DP
194 trendyline


 ,QWKHIROORZLQJVHTXHQFHRIUHDFWLRQVZKDWLVWKHPHFKDQLVPRIVWHS"


 
$ 1XFOHRSKLOLF$GGLWLRQ
% 1XFOHRSKLOLF6XEVWLWXWLRQ
& (OHFWURSKLOLF$GGLWLRQ
' (OHFWURSKLOLF6XEVWLWXWLRQ

$QVZHU$

trendyline
 &-&-&3UHOLPLQDU\([DP >7XUQRYHU
195 trendyline


 EURPRSURSDQH XQGHUJRHV QXFOHRSKLOLF VXEVWLWXWLRQ ZLWK DTXHRXV 1D2+ YLD WKH
IROORZLQJPHFKDQLVP

+
+
VORZ +& &  %U
+& & %U
&+
&+

+ +
+& & IDVW
2+ +& & 2+
&+
&+
:KLFKRIWKHUHDFWLRQSDWKZD\GLDJUDPILWVWKHDERYHPHFKDQLVP"
$      % 
 HQHUJ\
 HQHUJ\






 UHDFWLRQSDWKZD\
 UHDFWLRQSDWKZD\

&      '

HQHUJ\ HQHUJ\








UHDFWLRQSDWKZD\ UHDFWLRQSDWKZD\
 
$QVZHU&
6LQFH WKLV LV D VWHS UHDFWLRQ PHFKDQLVP DQG D FDUERFDWLRQ LQWHUPHGLDWH LV IRUPHG
UHDFWLRQ SURFHHGV YLD DQ 61 PHFKDQLVP ZKHUH WKH VORZ VWHS RI WKH PHFKDQLVP LV
XQLPROHFXODU7KHHQHUJ\SURILOHRID61UHDFWLRQLVDVVKRZQEHORZ

trendyline
&-&-&3UHOLPLQDU\([DP
196 trendyline


(D (D


6WHS  LV WKH VORZ UDWHGHWHUPLQLQJ VWHS WKXV LW ZRXOG KDYH WKH ODUJHVW DFWLYDWLRQ
HQHUJ\(D!(D7KXVRSWLRQ&LVWKHDQVZHU


 &RPSRXQG;&+%UXQGHUJRHVWKHIROORZLQJUHDFWLRQV
 1D2+ DT  &U2 DT + DT 
&+%U &+2 &+2RQO\
 KHDWXQGHUUHIOX[ KHDWXQGHUUHIOX[
 ;

:KDWLV;OLNHO\WREH"
$ EURPREXWDQH
% EURPREXWDQH
& EURPRPHWK\OSURSDQH
' EURPRPHWK\OSURSDQH

$QVZHU%
$LQFRUUHFWDVUHDFWLRQRIEURPREXWDQH SULPDU\5; ZLOOSURGXFHSULPDU\DOFRKRO
RI VXEVWLWXWLRQ DQG LQ WXUQ UHVXOW LQ FDUER[\OLF DFLG DV WKH ILQDO SURGXFW KDYLQJ
PROHFXODUIRUPXOD&+2




%FRUUHFW DV VXEVWLWXWLRQ RI K\GUR[\EXWDQH VHFRQGDU\ 5; JLYHV EXWDQRO


VHFRQGDU\ DOFRKRO  ZKLFK LQ WXUQ XQGHUJRHV R[LGDWLRQ WR \LHOG EXWDQRQH ZLWK

trendyline
PROHFXODUIRUPXOD&
FXODUIRUPXOD&
XOD +2


 &-&-&3UHOLPLQDU\([DP >7XUQRYHU
197 trendyline



 
&LQFRUUHFW DV UHDFWLRQ RI EURPRPHWK\OSURSDQH SULPDU\ 5;  ZLOO SURGXFH
SULPDU\ DOFRKRO RI VXEVWLWXWLRQ DQG LQ WXUQ UHVXOW LQ FDUER[\OLF DFLG RI PROHFXODU
IRUPXODRI&+2

KHDW 2+  %U
%U  2+ DT

2
KHDW
 >2@
2+ 2+


' LQFRUUHFWDVEURPRPHWK\OSURSDQH WHUWLDU\5; XQGHUJRHVVXEVWLWXWLRQWRJLYH
DWHUWLDU\DOFRKRO7HUWLDU\DOFRKROVGRQRWXQGHUJRR[LGDWLRQUHDFWLRQV0ROHFXODU
IRUPXODRIWHUWEXWDQROLV&+2

  

 &RPSRXQG ;VKRZQ EHORZ LV DQ LQWHUPHGLDWH XVHG WR JHQHUDWH S\UUROHV ZKLFK DUH
HVVHQWLDOWRWKHSURGXFWLRQRIPDQ\GLIIHUHQWFKHPLFDOVLQWKHSKDUPDFHXWLFDOLQGXVWU\


 &RPSRXQG;

:KLFKVHQWHQFHLVFRUUHFWIRUFRPSRXQG;"

trendyline
$ YHVDVLOYHUPLUURUZLWK7ROOHQVUHDJHQW
,WJLYHVDVLOYHUPLUURUZLWK7ROOHQVUHDJHQW
% ,WGHFRORXULVHVDFLGLILHGSRWDVVLXPPDQJDQDWH 9,, 
FRORXULVHVDFLGLILHGSRWDVVLXPPDQJDQDWH 9,, 
FRORXULVHVDFLGLILHGSRWDVVLXPPDQJDQDWH 9,, 
& ,WGRHVQRWJLYHDQRUDQJHSUHFLSLWDWHZLWKGLQLWURSKHQ\OK\GUD]LQH
HVQRWJLYHDQRUDQJHSUHFLSLWDWHZLWKGLQLWURSKHQ\OK\GUD
HVQRWJLYHDQRUDQJHSUHFLSLWDWHZLWKGLQLWURSKHQ\OK\GUD
' ,WJLYHV\HOORZSUHFLSLWDWHZLWKDONDOLQHDTXHRXVLRGLQH
YHV\HOORZSUHFLSLWDWHZLWKDONDOLQHDTXHRXVLRGLQH
HV\HOORZSUHFLSLWDWHZLWKDONDOLQHDTXHRXVLRGLQH


&-&-&3UHOLPLQDU\([DP
198 trendyline


$QVZHU'
&RPSRXQG;LVDGLNHWRQH
2SWLRQ$LVLQFRUUHFWVLQFHWKHUHLVQRDOGHK\GHIXQFWLRQDOJURXSLQFRPSRXQG;DQG
WKXVFDQQRWJLYHDVLOYHUPLUURUZLWK7ROOHQVUHDJHQW
2SWLRQ%LVLQFRUUHFWVLQFHNHWRQHFDQQRWXQGHUJRIXUWKHUR[LGDWLRQ
2SWLRQ&LVLQFRUUHFWDVNHWRQHFDQXQGHUJRFRQGHQVDWLRQZLWK'13+WRJLYHDQ
RUDQJHSUHFLSLWDWH

2SWLRQ'LVFRUUHFWVLQFHLWKDV JURXSDQGWKXVDEOHWRJLYHD\HOORZ
SUHFLSLWDWHZLWKDONDOLQHDTXHRXVLRGLQH

 FRXPDUR\O+6/LVDQHZW\SHRIEDFWHULDOTXRUXPVHQVLQJVLJQDOFRPSRXQG:KHQ
RQH PROH RI FRXPDUR\O+6/ LV KHDWHG XQGHU UHIOX[ ZLWK 1D2+ DT  XQWLO QR IXUWKHU
UHDFWLRQRFFXUVKRZPDQ\PROHVRI1D2+ZLOOUHDFW"



$  %  &  ' 

$QVZHU&
3KHQRODPLGHDQGHVWHUFDQXQGHUJRUHDFWLRQZLWK1D2+$OFRKROLVQHXWUDODQGZLOO
QRWXQGHUJRUHDFWLRQZLWK1D2+

)XQFWLRQDO 7\SHRIUHDFWLRQ (TXDWLRQ
JURXS
3KHQRO $FLGEDVHUHDFWLRQ 5&+2+1D2+5&+2+2
$PLGH %DVLFK\GURO\VLV 5&21+52+5&2251+
(VWHU %DVLFK\GURO\VLV 5&2252+5&2252+

2
2 2 2
2 2
2+
1 2 
+
+1
+2 2



trendyline


 &-&-&3UHOLPLQDU\([DP >7XUQRYHU
199 trendyline


 6KLNLPLFDFLGLVIRXQGLQVWDUDQLVH7KHIROORZLQJFRPSRXQG<LVWKHLQWHUPHGLDWHWKDW
FDQEHXVHGWRIRUPVKLNLPLFDFLG:KHQ/L$l+LVDGGHGWRFRPSRXQG<ZKDWLVWKH
SURGXFWREWDLQHG"


&RPSRXQG<


$QVZHU%
/L$l+LVDUHGXFLQJDJHQWWKDWFDQUHGXFHFDUER[\OLFDFLGDQGNHWRQHWRSULPDU\DQG
VHFRQGDU\DOFRKROVUHVSHFWLYHO\+RZHYHULWFDQQRWUHGXFHDONHQH






trendyline







&-&-&3UHOLPLQDU\([DP
200 trendyline


 &RQVLGHUWKHIROORZLQJFRPSRXQGVEHORZ
, &+1+ ,, &+&21+ ,,, &+&+1+ ,9 &+&+ 1+

 :KLFKRIWKHIROORZLQJVKRZVWKHFRUUHFWRUGHURIGHFUHDVLQJSKEYDOXHVIRUWKHDERYH
FRPSRXQGV"

$ ,,,,,,,9
% ,,,,,,,9
& ,,,,9,,,
' ,9,,,,,,

$QVZHU%
 7KHVWURQJHUWKHEDVHWKHORZHUWKHS.EYDOXH%DVLFLW\GHSHQGVRQWKHDYDLODELOLW\RI
ORQHSDLURIHOHFWURQVRQ1DWRPWRDWWUDFWDSURWRQ
 ,,LVDQDPLGHZKLFKLVQHXWUDOVRLWZRXOGKDYHWKHKLJKHVWS.EYDOXH
,LVWKHZHDNHVWEDVHDVWKHORQHSDLURIHOHFWURQVRQ1LVGHORFDOLVHGLQWRWKHEHQ]HQH
ULQJWKXVPDNLQJWKHORQHSDLURIHOHFWURQVRQ1DWRPOHVVDYDLODEOHWRDWWUDFWDSURWRQ
 ,,,LVDVWURQJHUEDVHWKDQ ,GXHWRWKHSUHVHQFHRIDQHOHFWURQGRQDWLQJHWK\OJURXS
WKDWLQFUHDVHVWKHDYDLODELOLW\RIORQHSDLURIHOHFWURQVRQ1DWRPWRDWWUDFWDSURWRQ
 ,9 LV WKH PRVW EDVLF DV LW KDV  HOHFWURQGRQDWLQJ HWK\O JURXSV ZKLFK LQFUHDVH WKH
DYDLODELOLW\RIORQHSDLURIHOHFWURQVRQ1DWRPWRDWWUDFWDSURWRQWRDODUJHUH[WHQW

 

trendyline
 &-&-&3UHOLPLQDU\([DP >7XUQRYHU
201 trendyline


6HFWLRQ%

)RUHDFKRIWKHTXHVWLRQVLQWKLVVHFWLRQRQHRUPRUHRIWKHWKUHHQXPEHUHGVWDWHPHQWVWR
PD\EHFRUUHFW

'HFLGHZKHWKHUHDFKRIWKHVWDWHPHQWVLVRULVQRWFRUUHFW \RXPD\ILQGLWKHOSIXOWRSXWD
WLFNDJDLQVWWKHVWDWHPHQWVWKDW\RXFRQVLGHUWREHFRUUHFW 

7KHUHVSRQVHV$WR'VKRXOGEHVHOHFWHGRQWKHEDVLVRI

$ % & '
DQGDUHFRUUHFW DQGRQO\DUH DQGRQO\DUH RQO\LVFRUUHFW
FRUUHFW FRUUHFW

1RRWKHUFRPELQDWLRQRIVWDWHPHQWVLVXVHGDVDFRUUHFWUHVSRQVH

 Use of the Data Booklet is relevant to this question.


:KLFKRIWKHIROORZLQJVSHFLHVKDVWKHHOHFWURQLFFRQILJXUDWLRQRI>$U@GV"
 &U
 0Q
 )H

$QVZHU% DQGRQO\ 
&U>$U@GVFRUUHFW
0Q>$U@GV
 7KXV0Q>$U@GVFRUUHFW
)H>$U@GV
 7KXV)H>$U@GLQFRUUHFW

 7KHYDOXHRIWKHLRQLFSURGXFWRIZDWHUKZYDULHVZLWKWHPSHUDWXUH

WHPSHUDWXUHR& KZPROGP

 [

 [

:KLFKRIWKHIROORZLQJVWDWHPHQWLVFRUUHFW"
 7KHLRQLFGLVVRFLDWLRQRIZDWHULVDQHQGRWKHUPLFSURFHVV
 :DWHULVDQHXWUDOOLTXLGDW&

trendyline
 DW &
S+DW &




&-&-&3UHOLPLQDU\([DP
202 trendyline


$QVZHU$ DQG 
 +2 l + DT 2+ DT 
KZ >+ DT @>2+ DT @

  :KHQ WHPSHUDWXUH LQFUHDVHV KZ LQFUHDVHV 3RVLWLRQ RI HTXLOLEULXP VKLIWV WR WKH
ULJKW7KHUHIRUHWKHIRUZDUGUHDFWLRQLVHQGRWKHUPLF
>+@ >2+@DVZDWHUGLVVRFLDWHVWRWKHVDPHH[WHQW
6LQFHZDWHULVQHXWUDO>+@ >2+@  [  [PROGP
 S+ ORJ [  

 :KLFK RI WKH IROORZLQJ KDV WKH VDPH YDOXH DV WKH VWDQGDUG HQWKDOS\ FKDQJH RI
IRUPDWLRQHIRRIFDUERQPRQR[LGH"
 HFRPEXVWLRQR & HFRPEXVWLRQR &2 
 HIR &2 HFRPEXVWLRQR &2 
 HIR &2 

$QVZHU%  RQO\ 
 
  R
Hf &2 J 
& V 2 J  &2 J 

 
Hc & V 
R
R
2 J   Hc &2 J 2 J 
 
  &2 J 

R R R
 Hf &2 J  + FRPEXVWLRQ & + FRPEXVWLRQ &2 

R R R
 Hf &2 J  + IRUPDWLRQ &2 + FRPEXVWLRQ &2 

1RWH
7KH VWDQGDUG HQWKDOS\ FKDQJH RI FRPEXVWLRQ RI D FRPSRXQG LV GHILQHG DV WKH
HQWKDOS\ FKDQJH ZKHQ  PRO RI WKH VXEVWDQFH LV EXUQW FRPSOHWHO\ LQ R[\JHQ XQGHU
VWDQGDUGFRQGLWLRQVRI.DQGDWP
R
 + FRPEXVWLRQ & & V 2 J &2 J 

R
 + FRPEXVWLRQ &2 &2 J 2 J &2 J 


7KH VWDQGDUG HQWKDOS\ FKDQJH RI IRUPDWLRQ RI D FRPSRXQG LV GHILQHG DV WKH
HQWKDOS\ FKDQJH ZKHQ  PRO RI WKH VXEVWDQFH LV IRUPHG IURP LWV HOHPHQWV XQGHU
VWDQGDUG FRQGLWLRQV RI  . DQG  DWP (OHPHQWV PXVW EH LQ PRVW VWDEOH SK\VLFDO
IRUP 

trendyline
R
 Hf &2 J & V 2
J & V 2 J  &2 J 
J &2
J J 
 
 7KXV+FRPEXVWLRQ
RPEXVWLRQ & DQG+IRUPDWLRQ &2 J DUHWKHVDPH
J DUHWKHVDPH
DUH

 &-&-&3UHOLPLQDU\([DP >7XUQRYHU
203 trendyline


 Use of the Data booklet is relevant to this question.


,QWKH'DQLHOOFHOOFRSSHUDQG]LQFHOHFWURGHVDUHLPPHUVHGLQDVROXWLRQRIFRSSHU ,, 
VXOIDWHDQG]LQFVXOIDWHUHVSHFWLYHO\
7KHWZRKDOIHTXDWLRQVIRUD'DQLHOOFHOODUHJLYHQEHORZ

&X DT H&X V 

=Q DT H=Q V 

:KLFKVWDWHPHQWVDERXWWKLVFHOODUHFRUUHFWZKHQLWLVXVHG"
 7KHHOHFWURQIORZIURPWKH]LQFHOHFWURGHWRWKHFRSSHUHOHFWURGHLQWKHH[WHUQDO
FLUFXLW
 7KHRYHUDOOFHOOUHDFWLRQLV&X=Q&X=Q
 7KHFHOOSRWHQWLDOLV9

$QVZHU$ DQG 
&X DT H&X V   (R 9

 
=Q DT H =Q V   (R 9

6LQFH (R &X&X LV PRUH SRVLWLYH WKDQ (R =Q=Q  WKXV UHGXFWLRQ RFFXUV DW FRSSHU
HOHFWURGH FDWKRGH ZKHUHDVR[LGDWLRQRFFXUVDW]LQFHOHFWURGH DQRGH 

2YHUDOOHTXDWLRQ&X=Q&X=Q
(FHOOR (UHGR(R[GR    9

+HQFHDOOWKHRSWLRQVDUHFRUUHFW
2SWLRQLVFRUUHFWDVHOHFWURQDOZD\VIORZIURPWKHDQRGHWRFDWKRGH
2SWLRQLVFRUUHFWVLQFH(FHOORLVSRVLWLYHWKXVUHDFWLRQLVIHDVLEOH
2SWLRQLVFRUUHFWDVVKRZQLQWKHFDOFXODWLRQDERYH












trendyline






&-&-&3UHOLPLQDU\([DP
204 trendyline


 $ QHXWUDO UHGFRPSOH[ )H$ LV IRUPHG ZKHQ D ELGHQWDWH OLJDQG $ LV DGGHG WR DQ
DTXHRXVVROXWLRQRI)HLRQV


$

:KLFKRIWKHIROORZLQJVWDWHPHQWLVFRUUHFW"
 7KHFRPSOH[KDVDFRRUGLQDWLRQQXPEHURI
 7KHFRPSOH[DEVRUEVUDGLDWLRQLQWKHEOXHJUHHQUHJLRQRIWKHYLVLEOHVSHFWUXP
 7KH FRPSOH[ KDV D VPDOOHU HQHUJ\ JDS EHWZHHQ GRUELWDOV FRPSDUHG WR \HOORZ
>)H +2 @

$QVZHU&  RQO\ 
  $ LVELGHQWDWHDVWKHFRPSOH[IRUPHGLVQHXWUDOKHQFHWKHUHPXVWEHOLJDQGV
IRUPLQJ  FRRUGLQDWH ERQGV HDFK WR EDODQFH RXW WKH )H IRUPLQJ DQ RFWDKHGUDO
FRPSOH[ FRRUGLQDWLRQQXPEHU 
 7KH FRORXU RI WKH FRPSOH[ LV WKH FRPSOHPHQWDU\ FRORXU WR WKH ZDYHOHQJWKV
DEVRUEHG
 
 2
 5<

 9*
%


$V VHHQ IURP WKH FRORXUV WKH UHG FRPSOH[ LV GXH WR EOXHJUHHQ OLJKW EHLQJ
DEVRUEHG ZLWK D ORZHU ZDYHOHQJWK RU KLJKHU HQHUJ\ WKDQ WKH YLROHW OLJKW WKDW LV
DEVRUEHG WR JLYH \HOORZ >)H +2 @ KHQFH WKH HQHUJ\ RI WKH OLJKW DEVRUEHG LV
ORZHULQWKHUHGFRPSOH[ZKLFKFRUUHVSRQGVWRDVPDOOHUHQHUJ\JDS

   +LJKHQHUJ\     /RZHQHUJ\
  YLROHWEOXHJUHHQ\HOORZRUDQJHUHG

         
     :DYHOHQJWK QP 




trendyline






 &-&-&3UHOLPLQDU\([DP >7XUQRYHU
205 trendyline


 7KHJUDSKVKRZVKRZDSURSHUW\IURP1DWR&lLQ3HULRGYDULHVZLWKSURWRQQXPEHU


      
SURWRQQXPEHU

:KDWLVWKLVSURSHUW\"
 ,RQLFUDGLXV
 )LUVWLRQLVDWLRQHQHUJ\
 (OHFWURQHJDWLYLW\

$QVZHU' RQO\ 
 )RU HDFK LVRHOHFWURQLFVHULHV DV ZH JR DFURVV D SHULRG QXFOHDU FKDUJH
LQFUHDVHV 7KH HOHFWURQV DUH SXOOHG FORVHU WR WKH QXFOHXV VR WKH LRQLF UDGLXV
GHFUHDVHV 7KXV WKH FDWLRQLF UDGLXV GHFUHDVHV IURP 1D! 0J!$l!6L DQG
DQLRQLFUDGLXVGHFUHDVHVIURP3!6!&l
7KHDQLRQVDUHPXFKODUJHUWKDQWKHFDWLRQVEHFDXVHDQLRQVKDYHRQHPRUHILOOHG
HOHFWURQVKHOOKHQFHWKHUHLVDODUJHUVKLHOGLQJHIIHFWUHVXOWLQJZHDNHUDWWUDFWLRQRI
WKHRXWHUPRVWHOHFWURQVIURPWKHQXFOHXV
 ,RQLVDWLRQ HQHUJ\ JHQHUDOO\ LQFUHDVHV DFURVV D SHULRG GXH WR WKH LQFUHDVH LQ
QXFOHDUFKDUJHDQGDGHFUHDVHLQDWRPLFUDGLL
 5HFDOO WKH GHILQLWLRQ RI HOHFWURQHJDWLYLW\ (OHFWURQHJDWLYLW\ LV D PHDVXUH RI WKH
DELOLW\RIDQDWRPLQDFRYDOHQWERQGWRDWWUDFWWKHERQGLQJHOHFWURQV
(OHFWURQHJDWLYLW\ LQFUHDVHVDFURVV WKH SHULRG )URP 1D WR &O DWRPLF UDGLXV
GHFUHDVHVDQGWKHQXFOHXVKDVPRUHDWWUDFWLRQWRERQGLQJHOHFWURQV

 :LOOLDPVRQ V\QWKHVLV LV RQH RI WKH EHWWHU PHWKRGV WR SUHSDUH HWKHUV 525 ,W
LQYROYHVD61GLVSODFHPHQWRIKDOLGHLRQLQKDORJHQRDONDQH 5; E\DQDONR[LGHLRQ
QXFOHRSKLOH 52 DVVKRZQ
525;525;
HWKHU

:KLFKRIWKHIROORZLQJVWDWHPHQWVDUHWUXH"
 7KHUDWHHTXDWLRQLQYROYHGLVUDWH N>52@>5;@

trendyline
 7KHUHDFWLRQLQYROYLQJEURPRPHWKDQHZLOOEHIDVWHUFRPSDUHGWRFKORURPHWKDQH
UHDFWLRQLQYROYLQJEURPRPHWKDQHZLOOEHIDVWHUFRPSDUHGWR
HDFWLRQLQYROYLQJEURPRPHWKDQHZLOOEHIDVW F
 7KH UHDFWLRQ LQYROYLQJ EURPRSURSDQH ZLOO EH IDVWHU FRPSDUHG WR
EURPRPHWKDQH
PRPHWKDQH
RPHWKDQH



&-&-&3UHOLPLQDU\([DP
206 trendyline


$QVZHU%  RQO\ 
  &RUUHFW DV D 61 PHFKDQLVP FRQVLVW ERWK KDORJHQRDONDQ DQG DONR[LGH LRQV
SUHVHQWLQWKHUDWHGHWHUPLQLQJVWHSRIWKHPHFKDQLVP
&RUUHFWVLQFH&%UERQGLVORQJHUWKDQ&&lERQGWKXV&%UERQG N-PRO LV
ZHDNHU WKDQ &&l  N- PRO DQG PRUH HDVLO\ EURNHQ WKXV WKH UHDFWLRQ
LQYROYLQJEURPRHWKDQHZLOOEHIDVWHU
,QFRUUHFW VLQFH SUHVHQFH RI  EXON\ PHWK\O JURXSV LQ EURPRSURSDQH KLQGHUV
DSSURDFK VWHULF KLQGUDQFH  RI WKH QXFOHRSKLOH WR WKH KDORJHQRDONDQH PDNLQJ
ERQGIRUPDWLRQGLIILFXOW+HQFHWKHUHDFWLRQLQYROYLQJEURPRSURSDQHVKRXOGEH
VORZHUWKDQEURPRPHWKDQH


 &DUV DUH ILWWHG ZLWK D FDWDO\WLF FRQYHUWHU :KDW UHDFWLRQV W\SLFDOO\ RFFXU LQ D FDWDO\WLF
FRQYHUWHU"
 12&21&2
[ \ \
 &[+\  2[&2 +2
  
 &212&212

$QVZHU' RQO\ 
&DWDO\WLFFRQYHUWHUVDUHXVHGWRUHPRYHSROOXWDQWVVXFKDV&212[DQGXQEXUQW
K\GURFDUERQVWRIRUPOHVVKDUPIXO&21DQG+27KXVRSWLRQVDQGDUH
LQFRUUHFWDV&2DQG12VKRXOGQRWEHIRUPHG



 $WRVLEDQDSURWHLQLVXVHGDVDQLQWUDYHQRXVPHGLFDWLRQDVDELUWKODERXUUHSUHVVDQW
WRKDOWSUHPDWXUHODERXU

t nd
tr
trendyline
dylin
ne
ne $WRVLEDQ


 &-&-&3UHOLPLQDU\([DP >7XUQRYHU
207 trendyline


:KLFKRIWKHIROORZLQJ5JURXSLQWHUDFWLRQVLQWKHSURWHLQDUHSRVVLEOH"
 +\GURJHQERQG
 'LVXOILGHERQG
 ,RQLFERQG


$QVZHU% DQG 


&RUUHFW DV D WKHUH DUH SRODU XQFKDUJHG 5 JURXSV ZKLFK FDQ IRUPLQJ K\GURJHQ
ERQGLQJ
&RUUHFWVLQFHWKHUHLVD66ERQGLQWKHSURWHLQ
,QFRUUHFWVLQFHWKHUHLVRQO\FKDUJHG5JURXSSUHVHQWLQWKHSURWHLQ DSRVLWLYHO\
FKDUJHG &+&+&+1+JURXS  )RU LRQLF ERQGLQJ WR IRUP WKHUH PXVW EH DW
OHDVWFKDUJHG5JURXSVDQGHDFKRIRSSRVLWHFKDUJH

 7KH LVRPHULVDWLRQ RI HWKR[\HWKDQH WR SURGXFH WKH FRUUHVSRQGLQJ DOFRKRO LV VKRZQ
EHORZ


 
:KLFKRIWKHIROORZLQJVKRZVWKHFRUUHFWSURGXFWIRUPHGXQGHUWKHVDPHFRQGLWLRQV"



 




ttrendyline
rendy
dyyli
ylin
inne
e


 
 



&-&-&3UHOLPLQDU\([DP
208 trendyline





  2


 2+



$QVZHU$ DQG 
 $FFRUGLQJWRWKHVFKHPHJLYHQ

 
7KHHWK\OJURXSQH[WWRWKH2LVVKLIWHGWRWKH& 

7KXVIROORZLQJWKLVSDWWHUQ







 

trendyline
 &-&-&3UHOLPLQDU\([DP >7XUQRYHU
209 trendyline


 $  &  $  %


 '  &  &  $
 %  '  &  %
 %  '  $  $
 '  %  &  &
 $  &  %  '
 %  $  '  %
 %  '  &  '
 %  &  %  %
 '  %  %  $



trendyline
&-&-&3UHOLPLQDU\([DP
210 trendyline
CATHOLIC JUNIOR COLLEGE
JC2PRELIMINARY EXAMINATIONS
Higher 2

CANDIDATE
NAME

CLASS 2T

CHEMISTRY 9647/02
Paper 2 Structured Questions Monday 22 August 2016
2 hours

Candidates answer on the Question Paper.

Additional Materials: Data Booklet

READ THESE INSTRUCTIONS FIRST

Write your name and class on all the work you hand in.
Write in dark blue or black pen.
You may use an HB pencil for any diagrams or graphs.
Do not use staples, paper clips, or correction fluid.

Answer all questions.


The use of an approved scientific For Examiners Use
calculator is expected, where appropriate.
A Data Booklet is provided. Paper 1 40
Q1 12
At the end of the examination, fasten all
your work securely together. Q2 15
The number of marks is given in brackets
[ ] at the end of each question or part Paper 2
Q3 15
question.
Q4 15
Q5 15 72
Q1 20
Q2 20
Paper 3 Q3 20
Q4 20
Q5 20 80

trendyline
yli
Total
ta 192

This document consists of 16 printed pages.

9647/02/CJC JC2 Preliminary Examination 2016


211 trendyline
2

1 Planning (P) For


Examiners
Use
One method of studying the kinetics of a chemical reaction, to find the order of reaction
with respect to a particular reactant, is to measure the initial rates, by using the clock
experiment.

To determine the initial rate, we can measure the time taken for a prominent visual change
to occur in the course of a reaction.

When aqueous sodium thiosulfate, Na 2 S 2 O 3 , is added to a solution containing dilute


hydrochloric acid, HCl, a fine, pale yellow precipitate of solid sulfur will be formed after a
while. This is the prominent visual change that can be clearly identified.

The overall equation for the reaction is represented as follows:

S 2 O 3 2-(aq) + 2H+(aq) S(s) + SO 2 (g) + H 2 O(l)

The initial rate of this reaction is determined by measuring the time taken for sufficient
precipitate of sulfur to be formed to just obscure a cross marked on a piece of white
paper below a reaction container.

A student carried out a series of preliminary experiments, using approximate volumes of


the two reactants and each reaction mixture was made up to the same total volume with
deionised water. The student found that the time taken for the pale yellow precipitate to
appear doubled when the volume of hydrochloric acid added was halved.

(a) (i) State the relationship between the initial rate of reaction and the time taken for
thepale yellow precipitate to appear.

..........................................................................................................................

...................................................................................................................... [1]

(ii) Explain why it is necessary to top up the reaction mixture with deionised water
to the same total volume.

..........................................................................................................................

..........................................................................................................................

..........................................................................................................................

..........................................................................................................................

...................................................................................................................... [1]

trendyline
9647/02/CJC JC2 Preliminary Examination 2016
212 trendyline
3

(b) Hence, based on the results of the students preliminary experiments, predict the For
Examiners
order of the reaction with respect to H+ ions. Use

...................................................................................................................................

...................................................................................................................................

...................................................................................................................................

...................................................................................................................................

............................................................................................................................... [1]

(c) You are to design an experiment to study the kinetics of the reaction between S 2 O 3 2-
ions and H+ ions.

The following data in tabulated form is shown below.

experiment volume of volume of volume of time taken for


1.00 mol dm-3 1.00 mol dm-3 deionised water the marked
Na 2 S 2 O 3 / cm3 HCl / cm3 / cm3 cross to be
obscured / s
1 10 20 20 t1
2 20 t2
3 10 t3

(i) Fill in the blanks in the table above, appropriate volumes of the reactants and
deionised water to be used in experiments 2 and 3 such that the order of
reaction with respect to S 2 O 3 2- and H+ ions can be determined and verified. [1]

(ii) Outline in a series of numbered steps, how experiment 1 could be carried out.
Your plan should include:
the apparatus used to measure the various volumes,
the order that the various solutions are mixed,
how the time to determine the rate of reaction is measured, and
other experimental details to ensure the consistency of the
experiment.

Based on the table in (c)(i), explain how the results of any two of the three
experiments can be used to determine the order of reaction with respect to
S 2 O 3 2- ions.

..........................................................................................................................

..........................................................................................................................

trendyline
..............................................................................................
...............................................................
..........................................................................................................................

..............................................................................................
.....................................................................................
..........................................................................................................................

..............................................................................................
....
..........................................................................................................................

9647/02/CJC JC2 Preliminary Examination 2016 [Turn over


213 trendyline
4
For
.......................................................................................................................... Examiners
Use

..........................................................................................................................

..........................................................................................................................

..........................................................................................................................

..........................................................................................................................

..........................................................................................................................

..........................................................................................................................

..........................................................................................................................

..........................................................................................................................

..........................................................................................................................

..........................................................................................................................

..........................................................................................................................

..........................................................................................................................

..........................................................................................................................

..........................................................................................................................

..........................................................................................................................

..........................................................................................................................

..........................................................................................................................

..........................................................................................................................

..........................................................................................................................

..........................................................................................................................

..........................................................................................................................

..........................................................................................................................

..........................................................................................................................

...................................................................................................................... [7]

(d) Identify one potential safety hazard in this experiment and state how you would

trendyline
ise
minimisese this risk.

..............................................................................................
.............................................................................................
..........................................................................................................................

......................................................................................................................
..............................................................................................
..... [1]

[Total: 12]
9647/02/CJC JC2 Preliminary Examination 2016

214 trendyline
5
For
2 Silver halides (silver chloride, silver bromide and silver iodide) are useful in qualitative Examiners
Use
analysis as they are insoluble in water. In qualitative analysis, silver nitrate is added to a
test solution to identify the presence of halide ions by forming a precipitate.

(a) (i) When aqueous silver nitrate was added to a test solution which was known to
contain a halide ion, a cream precipitate was formed.

Identify the precipitate and deduce the identity of the halide ion present in the
solution.

..........................................................................................................................

...................................................................................................................... [1]

(ii) To confirm the identity of the halide, aqueousammonia was added dropwise,
and the precipitate remained unchanged at first, but as more ammonia was
added, the precipitate dissolved.

With the aid of an equation, brieflyexplain why the precipitate dissolved.

..........................................................................................................................

..........................................................................................................................

...................................................................................................................... [2]

(iii) Draw the structure of the silver-containing product in solution after the
precipitate dissolved, displaying any lone pair of electrons where appropriate.

[1]

In photography, after an image is developed, sodium thiosulfate, Na 2 S 2 O 3 , is used


to dissolve any remaining silver halide away, to fix the image and ensure that it
does not change upon further exposure to light.

(iv) An equal volume of sodium thiosulfate was added to the solution


from (ii). State the type of reaction that occurred when sodium thiosulfate was
added.

...................................................................................................................... [1]

trendyline
(v) Suggest silver-containing
uggest the identity of the silve
silver ning product which has
containing h a coordination
number of 2.

......................................................................................................................
..............................................................................................
............................................................................................ [1]

9647/02/CJC JC2 Preliminary Examination 2016 [Turn over


215 trendyline
6

(b) The table below shows data about silver and two other metals, iron and copper. For
Examiners
Use

silver iron copper


10 1 6 2
electronic configuration [Kr] 4d 5s [Ar] 3d 4s [Ar] 3d104s1
conductivity at room 6.3 x 107 1.0 x 107 ?
temperature/ S m1

(i) Explain, in terms of structure and bonding, why the three metals have high
electrical conductivity.
..........................................................................................................................

..........................................................................................................................

..........................................................................................................................

...................................................................................................................... [2]

(ii) Silver and copper are known to be excellent metallic conductors at room
temperature. Noting the electronic configuration, suggest the value of the
conductivity of copper at room temperature.

......................................... S m-1[1]

(iii) Iron is used as a catalyst in the synthesis of ammonia in the Haber process.
State the type of catalysis and explain clearly how it carries out its function.

..........................................................................................................................

..........................................................................................................................

..........................................................................................................................

..........................................................................................................................

..........................................................................................................................

..........................................................................................................................

...................................................................................................................... [3]

trendyline
9647/02/CJC JC2 Preliminary Examination 2016

216 trendyline
7
For
(c) When a jar is filled with gaseous hydrogen iodide,HI, and a red-hot glass rod Examiners
Use
inserted into the jar, violet fumes are observed.

(i) State the identity of the violet fumes.

...................................................................................................................... [1]

(ii) When the same procedure is carried out with HBr instead of HI, no fumes are
observed, but on strong heating, reddish-brown vapour is obtained.

With reference to values in theData Booklet, explain the difference in the


behaviour of HBr and HI.

..........................................................................................................................

..........................................................................................................................

..........................................................................................................................

..........................................................................................................................

..........................................................................................................................

..........................................................................................................................

...................................................................................................................... [2]

[Total: 15]

trendyline
9647/02/CJC JC2 Preliminary Examination 2016 [Turn over
217 trendyline
8
For
3 Methanol is one of the alternative fuels proposed to replace fossil fuels. It has the Examiners
advantage of more efficient production and ease of storage as compared to other Use

alternatives.

Methanol can be formed from carbon monoxide and hydrogen as shown below.

CO(g) + 2H 2 (g) CH 3 OH(g)

The reaction was investigated by mixing 3.2 mol of CO and 3.2 mol of H 2 in a 10.0
dm3sealed vessel, and equilibrium was established at 120 C under a pressure of
1.60 x 106 Pa.

(a) (i) Assuming ideal gas behaviour, determine the total amount of gas in moles,at
equilibrium.

[2]

(ii) Hence, calculate the amounts of CO, H 2 and CH 3 OH in moles, present at


equilibrium.

[3]

(iii) Write an expression for the equilibrium constant, K c for the above reaction,
and use your answers in (a)(ii) to calculate its value under the stated
conditions.

trendyline [2]

9647/02/CJC JC2 Preliminary Examination 2016

218 trendyline
9

(b) Standard Gibbs free energy change,Gois related to K c by the following equation. For
Examiners
Use
o
G = RTln K c

where Gois in joules per mole, T is the temperature at which equilibrium is


established and R is the molar gas constant.

(i) Use this equation and your answer in (a) to calculate Gofor the formation of
methanol from carbon monoxide and hydrogen at 120 C.

[1]

(ii) With reference to theData Booklet, calculate the enthalpy change offormation
of methanol from carbon monoxide and hydrogen.Use
-1
1072 kJ mol for the bond energy of CO bond.

[2]

(iii) Hence, state briefly why higher temperature is not used industrially for the
formation of methanol from carbon monoxide and hydrogen.

..........................................................................................................................

..........................................................................................................................

...................................................................................................................... [2]

(iv) Use your answers in (b)(i) and (ii) to calculate the standard entropy change for
the reaction at 298K.

trendyline [1]

9647/02/CJC JC2 Preliminary Examination 2016 [Turn over


219 trendyline
10

For
(v) Explain the significance of the sign of the calculated entropy change and Examiners
comment if this is expected. Use

..........................................................................................................................

..........................................................................................................................

..........................................................................................................................

......................................................................................................................[2]

[Total:15]

trendyline
9647/02/CJC JC2 Preliminary Examination 2016

220 trendyline
11
For
4 (a) Acetanilide was the first aniline derivative found to possess analgesic and antipyretic Examiners
Use
properties.It was introduced into medical practice under the name of the drug,
Antifebrin.

Suggest the reagents and conditions that can be used to distinguish acetanilide from
compound X. State clearly the expected observations for each compound.
H O
N
N
O H
Acetanilide Compound X

Reagents and conditions ..

....

Observations .

....

....[2]

(b) It was later discovered that acetanilide was toxic and thus paracetamol, a derivative
of acetanilide, was produced.
H
N

O
HO

Paracetamol
(i) State the shape about N atom in paracetamol and hence identify the type
ofhybridisation involved.

Shape about N atom ..

Hybridisation of N .. [2]

trendyline
9647/02/CJC JC2 Preliminary Examination 2016 [Turn over
221 trendyline
12
For
(ii) A reaction scheme was proposed for the synthesis of paracetamol from phenol. Examiners
Fill in the boxes provided with the reagents, conditions and intermediates for Use

the synthesis.
OH OH

Step 1 Step 2
Intermediate

NH2

Step 3

OH

N
H
O
Paracetamol
Reagents and conditions:

Step 1 ..

Step 2 ..

Step 3 ..

Structure of intermediate:

[4]

(iii) In the synthesis of paracetamol from phenol, two possible side-products,


compound Y and Z, are formed. Given that only compound Y is a position
isomer of paracetamol, suggest the identities of compound Y and Z.

trendyline
trendy ine Compound Y Compound
Compo Z

[2]

9647/02/CJC JC2 Preliminary Examination 2016

222 trendyline
13
For
(iv) Predict, with explanation, how the boiling point of paracetamol might compare Examiners
to that of compound Y. Use

..........................................................................................................................

..........................................................................................................................

..........................................................................................................................

...................................................................................................................... [2]

(c) In a typicalpain relief tablet, there are500 mg of paracetamol.

(i) An adult can consume up to a maximum dosage of 4000 mg of paracetamol


within 24 hours. The average adult weight is 62 kg. Calculate the number of
tablets a childwho weighs 45 kg, can consume within 24 hours. (Maximum
dosage is dependent on weight of an individual.)

[2]

(ii) It was recommended to consume the tablets at 6 hours intervals. Using your
answer in (c)(i), calculate the recommended number of tablets the child should
take at each interval without exceeding dosage.

[1]

[Total: 15]

9647/02/CJC JC2 Preliminary Examination 2016 [Turn over


223 trendyline
14
For
5 The compound p-cresol is used in the production of antioxidants. It is also used in the Examiners
Use
fragrance and dye industries.
CH3

OH
-creso
p l

(a) Draw the displayed formula of a functional group isomer of p-cresol, and describe a
laboratory test that will distinguish these two compounds. You should state the
reagents used and the observations expected.

...................................................................................................................................

...................................................................................................................................

...................................................................................................................................

...................................................................................................................................

............................................................................................................................... [3]

(b) p-Cresol can be prepared industrially by a 2-step process as shown below.

CH3 CH3 CH3

+ +
concentrated I II
SO3
H2SO4

SO3H OH
- -
p toluenesulf onic acid p cresol

Reaction I involves sulfonation of methyl benzene to give p-toluenesulfonic acid.


The proposed mechanism of reactionI involves electrophilic substitution.

(i)

trendyline
The first step of the proposed mechanism of reaction I involves
reaction +
i
in
eaction in the generation of an electrophile, HSO 3 . Writ
how how the electrophile HSO 3 iss formed.
show +
acid-base
Write an equation to

......................................................................................................................[1]
9647/02/CJC JC2 Preliminary Examination 2016

224 trendyline
15

(ii) Complete the proposed mechanism to produce p-toluenesulfonic acid as For


Examiners
shown above. In the mechanism, show clearly the curly arrows to indicate the Use
movement of electrons and all charges.

[3]

(c) p-Cresolis used in the production of antioxidants such as butylated hydroxytoluene,


BHT, which is mainly used as an antioxidant in the food industry. The reaction
involves a dialkylation of p-cresol with 2-methylpropene in the presence of an
acidcatalystgiven below.

CH3 CH3

+
H
+

OH OH
-creso -
p l 2 methylpropene BHT

(i) Draw a labelled diagram to show how the orbitals overlap to form the C=C
bond in 2-methylpropene and state the type of hybridisation involved.
[1]

Type of hybridisation: ...................................................................................[1]

(ii) Explain why 2-methylpropene cannot exhibit geometric isomerism.

...

trendyline
..............................................................................................
.............................................................................................
...................................................................................................................... [1]

9647/02/CJC JC2 Preliminary Examination 2016 [Turn over


225 trendyline
16
For
(iii) In order for BHT to act as an antioxidant, it forms a very stable radical on the Examiners
Use
oxygen atom as shown below. Suggest an explanation why this stable BHT
radical is unable to react further with other molecules.
CH3

O
BHT radical

...

...................................................................................................................... [1]

(iv) Compound R, C 14 H 20 O 2 , is a metabolite derived from the biotransformation of


BHT. It is structurally related to BHT, C 15 H 24 O, and is also a cyclic compound.

State the type of reaction and deduction for the following observations.

I 1 mole of Compound R will react with 2 moles of 2, 4-DNPH.

Type of reaction:.......................................................................................

Deduction: ...........................................................................................[1]

II Compound R will not react with Tollens reagent.

Deduction: ........................................................................................... [1]

III 1 mole of compound R willdecolourise 2 moles of aqueous Br 2 but no


HBr is formed.

Type of reaction:.......................................................................................

Deduction: ...........................................................................................[1]

IV Deduce the structure of compound R.

trendyline [Total: 15]


[1]

9647/02/CJC JC2 Preliminary Examination 2016

226 trendyline
&$7+2/,&-81,25&2//(*(
-&35(/,0,1$5<(;$0,1$7,216
+LJKHU

&$1','$7(
1$0(

&/$66

&+(0,675< 
3DSHU6WUXFWXUHG4XHVWLRQV 0RQGD\$XJXVW
KRXUV

&DQGLGDWHVDQVZHURQWKH4XHVWLRQ3DSHU

$GGLWLRQDO0DWHULDOV'DWD%RRNOHW

5($'7+(6(,16758&7,216),567

:ULWH\RXUQDPHDQGFODVVRQDOOWKHZRUN\RXKDQGLQ
:ULWHLQGDUNEOXHRUEODFNSHQ
<RXPD\XVHDQ+%SHQFLOIRUDQ\GLDJUDPVRUJUDSKV
'RQRWXVHVWDSOHVSDSHUFOLSVRUFRUUHFWLRQIOXLG

$QVZHUDOOTXHVWLRQV
7KH XVH RI DQ DSSURYHG VFLHQWLILF )RU([DPLQHUV8VH
FDOFXODWRULVH[SHFWHGZKHUHDSSURSULDWH
$'DWD%RRNOHWLVSURYLGHG 3DSHU 
4 
$W WKH HQG RI WKH H[DPLQDWLRQ IDVWHQ DOO
\RXUZRUNVHFXUHO\WRJHWKHU 4 
7KHQXPEHURIPDUNVLVJLYHQLQEUDFNHWV
>  @ DW WKH HQG RI HDFK TXHVWLRQ RU SDUW 3DSHU
4 
TXHVWLRQ
4 
4  
4 
4 
3DSHU 4 

$16:(56&+(0( 4 


4  

trendyline
ylin
ne
ne
7RWDO
7RWD
7RWDO 

7KLVGRFXPHQWFRQVLVWVRISULQWHGSDJHV
I

&-&-&3UHOLPLQDU\([DPLQDWLRQ
227 trendyline


 3ODQQLQJ 3 

2QH PHWKRG RI VWXG\LQJ WKH NLQHWLFV RI D FKHPLFDO UHDFWLRQ WR ILQG WKH RUGHU RI UHDFWLRQ
ZLWK UHVSHFW WR D SDUWLFXODU UHDFWDQW LV WR PHDVXUH WKH LQLWLDO UDWHV E\ XVLQJ WKH FORFN
H[SHULPHQW

7RGHWHUPLQHWKHLQLWLDOUDWHZHFDQPHDVXUHWKHWLPHWDNHQIRUDSURPLQHQWYLVXDOFKDQJH
WRRFFXULQWKHFRXUVHRIDUHDFWLRQ

:KHQ DTXHRXV VRGLXP WKLRVXOIDWH 1D62 LV DGGHG WR D VROXWLRQ FRQWDLQLQJ GLOXWH
K\GURFKORULFDFLG+&lDILQHSDOH\HOORZSUHFLSLWDWHRIVROLGVXOIXUZLOOEHIRUPHGDIWHUD
ZKLOH7KLVLVWKHSURPLQHQWYLVXDOFKDQJHWKDWFDQEHFOHDUO\LGHQWLILHG

7KHRYHUDOOHTXDWLRQIRUWKHUHDFWLRQLVUHSUHVHQWHGDVIROORZV

62 DT + DT 6 V 62 J +2 O 

7KH LQLWLDO UDWH RI WKLV UHDFWLRQ LV GHWHUPLQHG E\ PHDVXULQJ WKH WLPH WDNHQ IRU VXIILFLHQW
SUHFLSLWDWH RI VXOIXU WR EH IRUPHG WR MXVW REVFXUH D FURVV PDUNHG RQ D SLHFH RI ZKLWH
SDSHUEHORZDUHDFWLRQFRQWDLQHU

$VWXGHQWFDUULHGRXWDVHULHVRISUHOLPLQDU\H[SHULPHQWVXVLQJDSSUR[LPDWHYROXPHVRI
WKHWZRUHDFWDQWVDQGHDFKUHDFWLRQPL[WXUHZDVPDGHXSWRWKHVDPHWRWDOYROXPHZLWK
GHLRQLVHG ZDWHU 7KH VWXGHQW IRXQG WKDW WKH WLPH WDNHQ IRU WKH SDOH \HOORZ SUHFLSLWDWH WR
DSSHDUGRXEOHGZKHQWKHYROXPHRIK\GURFKORULFDFLGDGGHGZDVKDOYHG

D  L  6WDWHWKHUHODWLRQVKLSEHWZHHQWKHLQLWLDOUDWHRIUHDFWLRQDQGWKHWLPHWDNHQIRU
WKHSDOH\HOORZSUHFLSLWDWHWRDSSHDU 

rate 25 
time

5DWHRIUHDFWLRQLVLQYHUVHO\SURSRUWLRQDOWRWLPHWKHWLPHWDNHQIRUWKH
SDOH\HOORZSUHFLSLWDWHWRDSSHDULVORZVKRUWLIWKHUDWHLVKLJK
>@


LL  ([SODLQZK\LWLVQHFHVVDU\WRWRSXSWKHUHDFWLRQPL[WXUHZLWKGHLRQLVHGZDWHU
WRWKHVDPHWRWDOYROXPH 

volume concentrat ion RU

7R HQVXUH WKDW WKH total volumeRI WKH UHVXOWDQW UHDFWLRQ PL[WXUH

UHPDLQV WKH same LQ DOO H[SHULPHQWV VR WKDW WKH YROXPH RI HDFK

UHDFWDQW LV SURSRUWLRQDO WR LWV FRQFHQWUDWLRQ LQ WKH UHVXOWDQWVROXWLRQ

:KHQWKHYROXPHRIWKDWUHDFWDQWDGGHGGRXEOHVLWVFRQFHQWUDWLRQLQ
WKHUHVXOWDQWVROXWLRQGRXEOHV
>@

trendyline








&-&-&3UHOLPLQDU\([DPLQDWLRQ
228 trendyline


E  +HQFH EDVHG RQ WKH UHVXOWV RI WKH VWXGHQWV SUHOLPLQDU\ H[SHULPHQWV SUHGLFW WKH
RUGHURIWKHUHDFWLRQZLWKUHVSHFWWR+LRQV 

 7KH WLPH WDNHQ IRU WKH SDOH \HOORZ SUHFLSLWDWH WR DSSHDU GRXEOHG ZKHQ WKH

YROXPHRIK\GURFKORULFDFLGDGGHGZDVKDOYHG


6LQFH  volume concentrat ion DQG rate  ZKHQ WKH FRQFHQWUDWLRQ RI
time

K\GURFKORULFDFLGDGGHGZDVKDOYHGWKHUDWHZDVKDOYHG

UDWHRIUHDFWLRQ>+@RUWKHUHDFWLRQLVILUVWRUGHUZLWKUHVSHFWWR+
>@


F  <RXDUHWRGHVLJQDQH[SHULPHQWWRVWXG\WKHNLQHWLFVRIWKHUHDFWLRQEHWZHHQ62
LRQVDQG+LRQV

7KHIROORZLQJGDWDLQWDEXODWHGIRUPLVVKRZQEHORZ

H[SHULPHQW YROXPHRI YROXPHRI YROXPHRI WLPHWDNHQIRU
PROGP PROGP GHLRQLVHGZDWHU WKHPDUNHG
1D62FP 
+&lFP FP FURVVWREH
REVFXUHGV
    W
    W
    W

L  )LOOLQWKHEODQNVLQWKHWDEOHDERYHDSSURSULDWHYROXPHVRIWKHUHDFWDQWVDQG
GHLRQLVHG ZDWHU WR EH XVHG LQ H[SHULPHQWV  DQG  VXFK WKDW WKH RUGHU RI
UHDFWLRQZLWKUHVSHFWWR62DQG+LRQVFDQEHGHWHUPLQHGDQGYHULILHG >@

$SSURSULDWHYROXPHVRIUHDFWDQWVDQGGHLRQLVHGZDWHUVWDWHGFRUUHFWO\LQ
H[SHULPHQWVDQG


LL  2XWOLQHLQDVHULHVRIQXPEHUHGVWHSVKRZH[SHULPHQWFRXOGEHFDUULHGRXW
<RXUSODQVKRXOGLQFOXGH
 WKHDSSDUDWXVXVHGWRPHDVXUHWKHYDULRXVYROXPHV
 WKHRUGHUWKDWWKHYDULRXVVROXWLRQVDUHPL[HG
 KRZWKHWLPHWRGHWHUPLQHWKHUDWHRIUHDFWLRQLVPHDVXUHGDQG
 RWKHUH[SHULPHQWDOGHWDLOVWRHQVXUHWKHFRQVLVWHQF\RIWKHH[SHULPHQW

%DVHG RQ WKH WDEOH LQ F L  H[SODLQ KRZ WKH UHVXOWV RI DQ\ WZR RI WKH WKUHH
H[SHULPHQWV FDQ EH XVHG WR GHWHUPLQH WKH RUGHU RI UHDFWLRQ ZLWK UHVSHFW WR
62LRQV

 

trendyline













&-&-&3UHOLPLQDU\([DPLQDWLRQ

229 trendyline


 8VLQJ D  FPPHDVXULQJ F\OLQGHU DGG  FP RI 1D62 DT  DQG

FPRIGHLRQLVHGZDWHULQWRDFPFRQLFDOIODVN0DUNDFURVV

RQDSLHFHRIZKLWHSDSHUDQGSODFHLWXQGHUWKHFRQLFDOIODVN

 8VLQJDsecondPHDVXULQJF\OLQGHUPHDVXUHFPRI+&l DT 
 3RXU WKH +&l DT  LQWR WKH FRQLFDO IODVN DQG LPPHGLDWHO\ VWDUW WKH

VWRSZDWFK

 *LYHWKHFRQLFDOIODVNRQHRUWZRVZLUO V WRPL[WKHFRQWHQWV

 6WRS WKH sWRSZDWFK ZKHQ WKH SDOH \HOORZ SUHFLSLWDWH IRUPHG MXVW

REVFXUH WKH FURVV PDUNHG RQ WKH SLHFH RI ZKLWH SDSHU EHORZ WKH
FRQLFDOIODVN5HFRUGWKHWLPH
























>@

)URP F L FRPSDULQJH[SHULPHQWV 


Inspection Method
7KH FRQFHQWUDWLRQ RI+&l DT  LV NHSW FRQVWDQW ZKLOH WKH FRQFHQWUDWLRQ RI
1D62 DT LVYDULHG

2UGHU ([SODQDWLRQ
,I t2 = t1, WKHQ WKH UDWHV IRU ERWK UHDFWLRQV UHPDLQ XQFKDQJHG
WKRXJK WKH FRQFHQWUDWLRQ RI 62 LV GRXEOHG 6R WKH UHDFWLRQ
0 UDWH LV LQGHSHQGHQW RI WKH FKDQJH LQ FRQFHQWUDWLRQ RI 62
ZeroRUGHU

ttrendyline
rend
d lin

,It2 is halved that of t1WKHUDWHIRUH[SHULPHQWL
,I WKHUDWHIRUH[SHULPHQWLVWZLFHWKDWRI
WKHUDWHIRUH[SHULPHQW
H[SHULPHQW6LQFHWKHUDWHLVGRXEOHGZKHQWKHFRQFHQWUDWLRQ
H[SHULPHQW6LQFHWKHUDWHLVGRXEOHGZKHQWKH
[SHULPHQW6LQFHWKHUDWHLVGRXEOHGZKHQWKH
1 RI 62
 LV GRXEOHG WKH UHDFWLRQ LVfirst
LVfirst
LV t RUGHU ZLWK
Z UHVSHFW WR

62 

2 ,It2 is four times less that of t1WKHUDWHIRUH[SHULPHQWLVIRXU
&-&-&3UHOLPLQDU\([DPLQDWLRQ

230 trendyline


WLPHV JUHDWHU WKDW RI H[SHULPHQW  6LQFH WKH UDWH LV IRXU WLPHV
JUHDWHUZKHQWKHFRQFHQWUDWLRQRI62LVGRXEOHGWKHUHDFWLRQ
LVsecondRUGHUZLWKUHVSHFWWR62


25
V x t method
2UGHU ([SODQDWLRQ
0 WLVFRQVWDQWW W
1 962 [W FRQVWDQWRU[W [W

2 962 [W FRQVWDQWRU[W [W

>@

G  ,GHQWLI\ RQH SRWHQWLDO VDIHW\ KD]DUG LQ WKLV H[SHULPHQW DQG VWDWH KRZ \RX ZRXOG
PLQLPLVHWKLVULVN 

+&l DT LVFRUURVLYHZHDUKDQGJORYHVWRSUHYHQWGLUHFWFRQWDFWZLWKLW


62 JDV OLEHUDWHG LV WR[LF ZHDU SURWHFWLYH PDVN RU SHUIRUP WKH
>@
H[SHULPHQWLQDZHOOYHQWLODWHGSODFHRUXVLQJDIXPHFXSERDUG
 
(LWKHURQH >7RWDO@

 

trendyline

&-&-&3UHOLPLQDU\([DPLQDWLRQ

231 trendyline


 6LOYHU KDOLGHV VLOYHU FKORULGH VLOYHU EURPLGH DQG VLOYHU LRGLGH  DUH XVHIXO LQ TXDOLWDWLYH
DQDO\VLVDVWKH\DUHLQVROXEOHLQZDWHU,QTXDOLWDWLYHDQDO\VLVVLOYHUQLWUDWHLVDGGHGWRD
WHVWVROXWLRQWRLGHQWLI\WKHSUHVHQFHRIKDOLGHLRQVE\IRUPLQJDSUHFLSLWDWH

D  L  :KHQDTXHRXVVLOYHUQLWUDWHZDVDGGHGWRDWHVWVROXWLRQZKLFKZDVNQRZQWR
FRQWDLQDKDOLGHLRQDFUHDPSUHFLSLWDWHZDVIRUPHG 

,GHQWLI\WKHSUHFLSLWDWHDQGGHGXFHWKHLGHQWLW\RIWKHKDOLGHLRQSUHVHQWLQWKH
VROXWLRQ 
7KH FUHDP SUHFLSLWDWH IRUPHG LV $J%U,KHQFH WKH VROXWLRQ FRQWDLQV


%U LRQV>@
>@

LL  7R FRQILUP WKH LGHQWLW\ RI WKH KDOLGH DTXHRXVDPPRQLDZDV DGGHGGURSZLVH


DQG WKH SUHFLSLWDWH UHPDLQHG XQFKDQJHG DW ILUVW EXW DV PRUH DPPRQLD ZDV
DGGHGWKHSUHFLSLWDWHGLVVROYHG

:LWKWKHDLGRIDQHTXDWLRQEULHIO\H[SODLQZK\WKHSUHFLSLWDWHGLVVROYHG 

$J%U V + 1+ DT >$J 1+ @ DT %U DT 


:KHQ H[FHVVFRQFHQWUDWHG DPPRQLD LV DGGHG D VROXEOH FRPSOH[ LRQ LV
>@
IRUPHG

LLL  'UDZ WKH VWUXFWXUH RI WKH VLOYHUFRQWDLQLQJ SURGXFW LQ VROXWLRQ DIWHU WKH
SUHFLSLWDWHGLVVROYHGGLVSOD\LQJDQ\ORQHSDLURIHOHFWURQVZKHUHDSSURSULDWH


>@

,QSKRWRJUDSK\DIWHUDQLPDJHLVGHYHORSHGVRGLXPWKLRVXOIDWH1D62LVXVHGWR
GLVVROYHDQ\UHPDLQLQJVLOYHUKDOLGHDZD\WRIL[WKHLPDJHDQGHQVXUHWKDWLWGRHV
QRWFKDQJHXSRQIXUWKHUH[SRVXUHWROLJKW

LY  $Q HTXDO YROXPH RI VRGLXP WKLRVXOIDWH ZDV DGGHG WR WKH VROXWLRQ
IURP LL 6WDWHWKHW\SHRIUHDFWLRQWKDWRFFXUUHGZKHQVRGLXPWKLRVXOIDWHZDV
DGGHG 
/LJDQGH[FKDQJHGLVSODFHPHQWUHDFWLRQ 
>@

Y  6XJJHVWWKHLGHQWLW\RIWKHVLOYHUFRQWDLQLQJSURGXFWZKLFKKDVDFRRUGLQDWLRQ
QXPEHURI 

trendyline
>$J 62 @ 



>@


&-&-&3UHOLPLQDU\([DPLQDWLRQ

232 trendyline


E  7KHWDEOHEHORZVKRZVGDWDDERXWVLOYHUDQGWZRRWKHUPHWDOVLURQDQGFRSSHU

 VLOYHU LURQ FRSSHU


HOHFWURQLFFRQILJXUDWLRQ >.U@GV >$U@GV >$U@GV
FRQGXFWLYLW\ DW URRP [ [ "
WHPSHUDWXUH6P


L  ([SODLQ LQ WHUPV RI VWUXFWXUH DQG ERQGLQJ ZK\ WKH WKUHH PHWDOV KDYH KLJK
HOHFWULFDOFRQGXFWLYLW\ 

7KH WKUHH WUDQVLWLRQ PHWDOV KDYH JLDQW PHWDOOLF VWUXFWXUHV ZLWK VWURQJ


PHWDOOLFERQGVEHWZHHQPHWDOFDWLRQVDQGDVHDRIGHORFDOL]HGHOHFWURQV


7KH\ KDYHPDQ\ G DQG V YDOHQFH HOHFWURQV ZKLFKFDQ EH GHORFDOL]HG DQG


KHQFHFDQFRQGXFWHOHFWULFLW\WKURXJKWKHPRYHPHQWRIWKHHOHFWURQV
>@

 
LL  6LOYHU DQG FRSSHU DUH NQRZQ WR EHH[FHOOHQW PHWDOOLF FRQGXFWRUV DW URRP
WHPSHUDWXUH 1RWLQJ WKH HOHFWURQLF FRQILJXUDWLRQ VXJJHVW WKH YDOXH RI WKH
FRQGXFWLYLW\RIFRSSHUDWURRPWHPSHUDWXUH
 
$Q\YDOXHEHWZHHQ[[
6P>@
$FWXDO YDOXH  [  6 P XQLWV DUH 6LHPHQV SHU PHWUH UHFLSURFDO RI

UHVLVWLYLW\ 

LLL  ,URQLVXVHGDVDFDWDO\VWLQWKHV\QWKHVLVRIDPPRQLDLQWKH +DEHUSURFHVV
6WDWHWKHW\SHRIFDWDO\VLVDQGH[SODLQFOHDUO\KRZLWFDUULHVRXWLWVIXQFWLRQ 

,URQFDQDFWDVDKHWHURJHQHRXVFDWDO\VWLQWKH+DEHUSURFHVV

,W KDV SDUWLDOO\ILOOHG G RUELWDOVZKLFK DOORZV WKH UHDG\ H[FKDQJH RI

HOHFWURQV WR DQG IURP UHDFWDQW PROHFXOHV 1 DQG +  5HDFWDQWV DUH

DGVRUEHG RQWR WKH VXUIDFH ZHDNHQLQJ WKH ERQGVDQG WKHUHE\ ORZHULQJ

WKHDFWLYDWLRQHQHUJ\DVZHOODVLQFUHDVLQJWKHVXUIDFHFRQFHQWUDWLRQRI

UHDFWDQWVWKXVLQFUHDVLQJWKHUDWHRIUHDFWLRQ


>@

trendyline 


&-&-&3UHOLPLQDU\([DPLQDWLRQ

233 trendyline


F  :KHQ D MDU LV ILOOHG ZLWK JDVHRXV K\GURJHQ LRGLGH+, DQG D UHGKRW JODVV URG
LQVHUWHGLQWRWKHMDUYLROHWIXPHVDUHREVHUYHG

L  6WDWHWKHLGHQWLW\RIWKHYLROHWIXPHV 
,
>@

LL  :KHQWKHVDPHSURFHGXUHLVFDUULHGRXWZLWK+%ULQVWHDGRI+,QRIXPHVDUH
REVHUYHGEXWRQVWURQJKHDWLQJUHGGLVKEURZQYDSRXULVREWDLQHG

:LWK UHIHUHQFH WR YDOXHV LQ WKHData Booklet H[SODLQ WKH GLIIHUHQFH LQ WKH
EHKDYLRXURI+%UDQG+, 
)URP'DWD%RRNOHW

%(RI+, N-PRO

%(RI+%U N-PRO



$V WKH ERQG HQHUJ\ RI LV +%U JUHDWHUVWURQJHU WKDQ +, GXH WR

VPDOOHU DWRPLF VL]HUDGLXV DQG VKRUWHU ERQG OHQJWK  +%U GRHV QRW

GHFRPSRVH ZKHQ KHDWHG E\ FRQWDFW ZLWK D UHGKRW JODVV URG EXW

UHTXLUHVVWURQJHUKHDWLQJPRUHHQHUJ\WRGHFRPSRVH
>@


>7RWDO@



 

trendyline
&-&-&3UHOLPLQDU\([DPLQDWLRQ

234 trendyline


 0HWKDQRO LV RQH RI WKH DOWHUQDWLYH IXHOV SURSRVHG WR UHSODFH IRVVLO IXHOV ,W KDV WKH
DGYDQWDJH RI PRUH HIILFLHQW SURGXFWLRQ DQG HDVH RI VWRUDJH DV FRPSDUHG WR RWKHU
DOWHUQDWLYHV
0HWKDQROFDQEHIRUPHGIURPFDUERQPRQR[LGHDQGK\GURJHQDVVKRZQEHORZ

&2 J + J  &+2+ J 

 7KH UHDFWLRQ ZDV LQYHVWLJDWHG E\ PL[LQJ  PRO RI &2 DQG  PRO RI + LQ D 
GPVHDOHG YHVVHO DQG HTXLOLEULXP ZDV HVWDEOLVKHG DW  & XQGHU D SUHVVXUH RI
[3D

D  L  $VVXPLQJ LGHDO JDV EHKDYLRXU GHWHUPLQH WKH WRWDO DPRXQW RI JDV LQ PROHVDW
HTXLOLEULXP
 S9 Q57
S9  
 Q 
57  


Q RUPRO


 >@

LL  +HQFH FDOFXODWH WKH DPRXQWV RI &2 + DQG &+2+ LQ PROHV SUHVHQW DW
HTXLOLEULXP
  &2 + &+2+ 
,QLWLDOPRO   
 
&KDQJHPRO [ [ [
 (TXLOLEULXPPRO [ [ [ 
Q  [[[
  [ 
Q &+2+  [ PRO
 

 Q &2   PRO  Q +     PRO

 >@

LLL  :ULWH DQ H[SUHVVLRQ IRU WKH HTXLOLEULXP FRQVWDQW Kc IRU WKH DERYH UHDFWLRQ DQG
XVH\RXUDQVZHUVLQ D LL WRFDOFXODWHLWVYDOXHXQGHUWKHVWDWHGFRQGLWLRQV
 >&+ 2+@ 
Kc  
>&2@>+ @
  
9 
Kc 
      


trendyline
y
9 9 
 
Kc PROGP
 

 >@


&-&-&3UHOLPLQDU\([DPLQDWLRQ

235 trendyline


E  6WDQGDUG *LEEV IUHH HQHUJ\ FKDQJH*RLV UHODWHG WR Kc E\ WKH IROORZLQJ HTXDWLRQ

Go RTOQKc

ZKHUHGoLV LQ MRXOHV SHU PROH T LV WKH WHPSHUDWXUH DW ZKLFK HTXLOLEULXP LV
HVWDEOLVKHGDQGRLVWKHPRODUJDVFRQVWDQW

L  8VHWKLVHTXDWLRQDQG\RXUDQVZHULQ D WRFDOFXODWHGoIRUWKHIRUPDWLRQRI
PHWKDQROIURPFDUERQPRQR[LGHDQGK\GURJHQDW&

 Go  (8.31)(273+120) OQ
Go -PRO
 

 >@

LL  :LWKUHIHUHQFHWRWKHData BookletFDOFXODWHWKHHQWKDOS\FKDQJHRIIRUPDWLRQ
RI PHWKDQRO IURP FDUERQ PRQR[LGH DQG K\GURJHQ8VH
N-PROIRUWKHERQGHQHUJ\RI&2ERQG

%RQGVEURNHQ +   %RQGVIRUPHG + 

 ++    &+   
  &2   &2 
     2+ 

 +R     


 N-PRO 


 o 1
>@

LLL  +HQFH VWDWH EULHIO\ ZK\ KLJKHU WHPSHUDWXUH LV QRW XVHG LQGXVWULDOO\ IRU WKH
IRUPDWLRQRIPHWKDQROIURPFDUERQPRQR[LGHDQGK\GURJHQ
7KHIRUZDUGUHDFWLRQLVH[RWKHUPLF+LJKHUWHPSHUDWXUHZLOOUHVXOWLQ

SRVLWLRQ RI HTXLOLEULXP WR VKLIW WR WKH OHIWUHVXOWLQJ LQ ORZHU \LHOG RI

SURGXFWKHQFHLWLVQRWSUHIHUUHG
>@


LY  8VH\RXUDQVZHUVLQ E L DQG LL WRFDOFXODWHWKHVWDQGDUGHQWURS\FKDQJHIRU
WKHUHDFWLRQDW.

 *R +R76R 

 2

trendyline
ren y
6 

  
 -.PRO


 

 >@

&-&-&3UHOLPLQDU\([DPLQDWLRQ

236 trendyline




Y  ([SODLQ WKH VLJQLILFDQFH RI WKH VLJQ RI WKH FDOFXODWHG HQWURS\ FKDQJH DQG
FRPPHQWLIWKLVLVH[SHFWHG
&DOFXODWHG HQWURS\ FKDQJH LV QHJDWLYH DV H[SHFWHG ZKLFK PHDQV WKH

HQWURS\ KDV GHFUHDVHG WKHUHIRUH WKH V\VWHP EHFRPHV OHVV

GLVRUGHUHGDV WKHUH LV D GHFUHDVH LQ QXPEHU RI PROHV RI JDV DIWHU WKH

UHDFWLRQ

>@

>7RWDO@


 


&-&-&3UHOLPLQDU\([DPLQDWLRQ

237 trendyline


 D  $FHWDQLOLGHZDVWKHILUVWDQLOLQHGHULYDWLYHIRXQGWRSRVVHVVDQDOJHVLFDQGDQWLS\UHWLF
SURSHUWLHV,W ZDV LQWURGXFHG LQWR PHGLFDO SUDFWLFH XQGHU WKH QDPH RI WKH GUXJ
$QWLIHEULQ

6XJJHVWWKHUHDJHQWVDQGFRQGLWLRQVWKDWFDQEHXVHGWRGLVWLQJXLVKDFHWDQLOLGHIURP
FRPSRXQG;6WDWHFOHDUO\WKHH[SHFWHGREVHUYDWLRQVIRUHDFKFRPSRXQG

  
$FHWDQLOLGH  &RPSRXQG;

6WHS1D2+ DT KHDW
5HDJHQWVDQGFRQGLWLRQV 

6WHS%U DT UWS 

2EVHUYDWLRQV
2UDQJHDTXHRXVEURPLQHGLVFKDUJHGDQGZKLWHSSWZDVIRUPHG
IRUDFHWDQLOLGH+RZHYHUQRVXFKGHFRORXULVDWLRQDQGSUHFLSLWDWLRQREVHUYHG
IRUFRPSRXQG;

25(WK\ODPLQHJDVUHOHDVHGE\FRPSRXQG;DIWHUEDVHK\GURO\VLVZKLFKWXUQV
PRLVWUHGOLWPXVSDSHUEOXH+RZHYHUJDVUHOHDVHGZLOOQRWWXUQPRLVWUHG
>@
OLWPXVSDSHUEOXHIRUDFHWDQLOLGH

E  ,WZDVODWHUGLVFRYHUHGWKDWDFHWDQLOLGHZDVWR[LFDQGWKXVSDUDFHWDPRODGHULYDWLYH
RIDFHWDQLOLGHZDVSURGXFHG


3DUDFHWDPRO

L  6WDWH WKH VKDSH DERXW 1 DWRP LQ SDUDFHWDPRO DQG KHQFH LGHQWLI\ WKH W\SH
RIK\EULGLVDWLRQLQYROYHG

7ULJRQDOSODQDU
6KDSHDERXW1DWRP 
+\EULGLVDWLRQRI1 VS
 >@


 

&-&-&3UHOLPLQDU\([DPLQDWLRQ

238 trendyline


LL  $UHDFWLRQVFKHPHZDVSURSRVHGIRUWKHV\QWKHVLVRISDUDFHWDPROIURPSKHQRO
)LOO LQ WKH ER[HV SURYLGHG ZLWK WKH UHDJHQWV FRQGLWLRQV DQG LQWHUPHGLDWHV IRU
WKHV\QWKHVLV


5HDJHQWVDQGFRQGLWLRQV
'LOXWH+12UW
6WHS 
  6QFRQF+&lKHDW 1D2+ DT UW
6WHS 
&+&2&l
6WHS 

6WUXFWXUHRILQWHUPHGLDWH


>@
LLL  ,Q WKH V\QWKHVLV RI SDUDFHWDPRO IURP SKHQRO WZR SRVVLEOH VLGHSURGXFWV
FRPSRXQG < DQG = DUH IRUPHG *LYHQ WKDW RQO\ FRPSRXQG < LV D SRVLWLRQ
LVRPHURISDUDFHWDPROVXJJHVWWKHLGHQWLWLHVRIFRPSRXQG<DQG=

trendyline
trendy ine

 
 25 DPLGHV DQ
DQG
 HVWHUV
&RPSRXQG
&RPSRXQG< &RPSRXQG=
&RPSR
 
>@


&-&-&3UHOLPLQDU\([DPLQDWLRQ

239 trendyline




LY  3UHGLFWZLWKH[SODQDWLRQKRZWKHERLOLQJSRLQWRISDUDFHWDPROPLJKWFRPSDUH
WRWKDWRIFRPSRXQG<

3DUDFHWDPROKDVDKLJKHUERLOLQJSRLQWWKDQFRPSRXQG<


3DUDFHWDPRO FDQ IRUP PRUH H[WHQVLYH LQWHUPROHFXODU K\GURJHQ


ERQGLQJ DV XQOLNH FRPSRXQG < SDUDFHWDPROGRHV QRW KDYH

LQWUDPROHFXODUK\GURJHQERQGLQJ
>@


F  ,QDW\SLFDOSDLQUHOLHIWDEOHWWKHUHDUHPJRISDUDFHWDPRO

L  $QDGXOWFDQFRQVXPHXSWRDPD[LPXPGRVDJHRIPJRISDUDFHWDPRO
ZLWKLQ  KRXUV 7KH DYHUDJH DGXOW ZHLJKW LV  NJ &DOFXODWH WKH QXPEHU RI
WDEOHWV D FKLOGZKR ZHLJKV  NJ FDQ FRQVXPH ZLWKLQ  KRXUV 0D[LPXP
GRVDJHLVGHSHQGHQWRQZHLJKWRIDQLQGLYLGXDO 
 0D[SDUDFHWDPRODFKLOGFDQFRQVXPH
 
  PJ 



 0D[QRRIWDEOHWVWKDWFDQEHFRQVXPHG
 

 WDEOHWV

 WDEOHWV 
>@

LL  ,W ZDV UHFRPPHQGHG WR FRQVXPH WKH WDEOHWV DW  KRXUV LQWHUYDOV 8VLQJ \RXU
DQVZHULQ F L FDOFXODWHWKHUHFRPPHQGHGQXPEHURIWDEOHWVWKHFKLOGVKRXOG
WDNHDWHDFKLQWHUYDOZLWKRXWH[FHHGLQJGRVDJH
 1RRIWLPHVWRFRQVXPHWDEOHWVLQK
 
  WLPHV 



 0D[QRRIWDEOHWVWKDWFDQEHFRQVXPHGHDFKWLPH
 
  WDEOHWV

 WDEOHW 
>@
>7RWDO@
 

trendyline
&-&-&3UHOLPLQDU\([DPLQDWLRQ

240 trendyline


 7KH FRPSRXQG pFUHVRO LV XVHG LQ WKH SURGXFWLRQ RI DQWLR[LGDQWV ,W LV DOVR XVHG LQ WKH
IUDJUDQFHDQGG\HLQGXVWULHV




D  'UDZWKHGLVSOD\HGIRUPXODRIDIXQFWLRQDOJURXSLVRPHURIpFUHVRODQGGHVFULEHD
ODERUDWRU\ WHVW WKDW ZLOO GLVWLQJXLVK WKHVH WZR FRPSRXQGV <RX VKRXOG VWDWH WKH
UHDJHQWVXVHGDQGWKHREVHUYDWLRQVH[SHFWHG 


 
 
7R VHSDUDWH WHVW WXEH FRQWDLQLQJ HDFK VDPSOH DGG QHXWUDO )H&O 9LROHW

FRORXUDWLRQZLOOEHIRUPHGZLWKpFUHVROZKHUHDVQRYLROHWFRORXUDWLRQIRULWV

LVRPHU25

7R VHSDUDWH WHVW WXEH FRQWDLQLQJ HDFK VDPSOH DGG DTXHRXV %U2UDQJH
EURPLQHVROXWLRQGHFRORXULVHDQGDZKLWHSUHFLSLWDWHLVIRUPHGZLWKpFUHVRO

ZKHUHDVQRGHFRORXULVDWLRQRIRUDQJHEURPLQHVROXWLRQZLWKLWVLVRPHU
>@


NOT ACCEPTED: PCl5

Note: Oxidation using KMnO4/H2SO4is not accepted for p-cresol and


phenylmethanol since both can undergo side-chain oxidation.

Note:

are not accepted because these are positional isomers of p-cresol. Positional
isomers have
ave functional groups.
ve the same carbon chain, but different positions of fu
Functional group isomers have different functional groups.


&-&-&3UHOLPLQDU\([DPLQDWLRQ

241 trendyline


E  p&UHVROFDQEHSUHSDUHGLQGXVWULDOO\E\DVWHSSURFHVVDVVKRZQEHORZ





5HDFWLRQ , LQYROYHV VXOIRQDWLRQ RI PHWK\O EHQ]HQH WR JLYH pWROXHQHVXOIRQLF DFLG
7KHSURSRVHGPHFKDQLVPRIUHDFWLRQ,LQYROYHVHOHFWURSKLOLFVXEVWLWXWLRQ

L  7KH ILUVW VWHS RI WKH SURSRVHG PHFKDQLVP RI UHDFWLRQ , LQYROYHV DFLGEDVH
UHDFWLRQLQWKHJHQHUDWLRQRIDQHOHFWURSKLOH+62:ULWHDQHTXDWLRQWRVKRZ
KRZWKHHOHFWURSKLOH+62LVIRUPHG 
  
62+62+62 +62 
>@


LL  &RPSOHWH WKH SURSRVHG PHFKDQLVP WR SURGXFH pWROXHQHVXOIRQLF DFLG DV
VKRZQDERYH,QWKHPHFKDQLVPVKRZFOHDUO\WKHFXUO\DUURZVWRLQGLFDWHWKH
PRYHPHQWRIHOHFWURQVDQGDOOFKDUJHV 





25


trendyline

>@
 



&-&-&3UHOLPLQDU\([DPLQDWLRQ

242 trendyline


F  p&UHVROLVXVHGLQWKHSURGXFWLRQRIDQWLR[LGDQWVVXFKDVEXW\ODWHGK\GUR[\WROXHQH
%+7 ZKLFK LV PDLQO\ XVHG DV DQ DQWLR[LGDQW LQ WKH IRRG LQGXVWU\ 7KH UHDFWLRQ
LQYROYHV D GLDON\ODWLRQ RI pFUHVRO ZLWK PHWK\OSURSHQH LQ WKH SUHVHQFH RI DQ
DFLGFDWDO\VWJLYHQEHORZ





L  'UDZ D ODEHOOHG GLDJUDP WR VKRZ KRZ WKH RUELWDOV RYHUODS WR IRUP WKH & &
ERQGLQPHWK\OSURSHQHDQGVWDWHWKHW\SHRIK\EULGLVDWLRQLQYROYHG 

VSK\EULGLVDWLRQ



>@

LL  ([SODLQZK\PHWK\OSURSHQHFDQQRWH[KLELWJHRPHWULFLVRPHULVP 

2QHFDUERQDWRPRIWKH& &GRXEOHERQGKDVWKHVDPHPHWK\OJURXS


DWWDFKHGWRLW
>@

 

trendyline

&-&-&3UHOLPLQDU\([DPLQDWLRQ

243 trendyline


LLL  ,QRUGHUIRU%+7WRDFWDVDQDQWLR[LGDQWLWIRUPVDYHU\VWDEOHUDGLFDORQWKH
R[\JHQ DWRP DV VKRZQ EHORZ 6XJJHVW DQ H[SODQDWLRQ ZK\ WKLV VWDEOH %+7
UDGLFDOLVXQDEOHWRUHDFWIXUWKHUZLWKRWKHUPROHFXOHV
  


7KH ODUJHEXON\ DON\O JURXSV SRVH  VWHULF KLQGUDQFH UHSHO LQFRPLQJ

PROHFXOHV WKDW
 SUHYHQW WKH UDGLFDO WR UHDFW ZLWK RWKHU PROHFXOHV
25 7KH UDGLFDO LV VWDEOH GXH WR WKH GHORFDOLVDWLRQ RI XQSDLUHG HOHFWURQ IURP
>@
R[\JHQ DWRP LQWR WKH EHQ]HQH ULQJ25 3UHVHQFH RI HOHFWURQGRQDWLQJ DON\O
 JURXSVZLOOVWDELOL]HWKHUDGLFDO

LY  &RPSRXQG5&+2LVDPHWDEROLWHGHULYHGIURPWKHELRWUDQVIRUPDWLRQRI
%+7,WLVVWUXFWXUDOO\UHODWHGWR%+7&+2DQGLVDOVRDF\FOLFFRPSRXQG

6WDWHWKHW\SHRIUHDFWLRQDQGGHGXFWLRQIRUWKHIROORZLQJREVHUYDWLRQV

, PROHRIFRPSRXQG5ZLOOUHDFWZLWKPROHVRI'13+

&RQGHQVDWLRQ
7\SHRIUHDFWLRQ
&RPSRXQG5LVD DOGHK\GHNHWRQH FDUERQ\OFRPSRXQG
'HGXFWLRQ>@

,, &RPSRXQG5ZLOOQRWUHDFWZLWK7ROOHQVUHDJHQW

'HGXFWLRQ&RPSRXQG 5 KDV QR DOGHK\GHSUHVHQW &RPSRXQG
>@
5LVDNHWRQHDQGQRWDQDOGHK\GH 
,,,  PROH RI FRPSRXQG 5 ZLOOGHFRORXULVH  PROHV RI DTXHRXV %U EXW QR
+%ULVIRUPHG

(OHFWURSKLOLFDGGLWLRQ
7\SHRIUHDFWLRQ

'HGXFWLRQ&RPSRXQG5KDVDONHQHIXQFWLRQDOJURXSV 
>@

,9 'HGXFHWKHVWUXFWXUHRIFRPSRXQG5
2

trendyline
d i 2


>@

>7RWDO@

&-&-&3UHOLPLQDU\([DPLQDWLRQ

244 trendyline
CATHOLIC JUNIOR COLLEGE
JC2PRELIMINARY EXAMINATIONS
Higher 2

CHEMISTRY 9647/03
Paper 3 Free Response Friday 26 August 2016
2 hours
Candidates answer on separate paper.

Additional Materials: Answer Paper


Data Booklet

READ THESE INSTRUCTIONS FIRST

Write your name and class on all the work you hand in.
Write in dark blue or black pen.
You may use an HB pencil for any diagrams or graphs.
Do not use staples, paper clips, glue or correction fluid.

Answer any four questions.


A Data Booklet is provided.
The use of an approved scientific calculator is expected, where appropriate.
You are reminded of the need for good English and clear presentation in your answers.

At the end of the examination, fasten all your work securely together.
The number of marks is given in brackets [ ] at the end of each question or part question.

ttrendyline
d liThis
his documents consists of 10 printed

9647/03/CJC JC2 Preliminary


rinted pages.
pages

nary Examination 2016 [Turn over

245 trendyline
2

Answer any four questions.

1 Use of the Data Booklet is relevant to this question.

(a) When potassium manganate(VII), KMnO 4 , is heated with concentrated sodium hydroxide,
NaOH, bubbles of oxygen are seen and a green solution of MnO 4 2- is obtained. The
addition of barium chloride, BaCl 2 , to this solution precipitated out a solid X with
composition by mass of 53.5% barium, 21.5% manganese and 25.0% oxygen.
(i) Calculate the empirical formula of X. [1]

(ii) When the green solution above is acidified, a brown precipitate in a purple solution
is formed.

Suggest the identities of the species formed upon acidification and hence state the
type of reaction that has occured. [3]

(b) A solution of acidified potassium manganate(VII), KMnO 4 , can be standardised through


titration against sodium ethanedioate, Na 2 C 2 O 4 . The ethanedioate ion is oxidised to
carbon dioxide in this reaction.

(i) State the oxidation numbers of C in the ethanedioate ion and in carbon dioxide. [1]

(ii) Write the half-equation for the oxidation of the ethanedioate ion. [1]

(iii) Hence, write the overall balanced equation for the reaction between acidified
manganate(VII) and ethanedioate ions. [1]

(iv) In one such standardisation procedure, 25.0 cm3 of 0.0150 mol dm3 Na 2 C 2 O 4
solution required 28.85 cm3 of acidified KMnO 4 to reach the end-point.

Determine the concentration of KMnO 4 in the solution. You may assume that
2MnO 4 & 2 O 4 2. [3]

(c) Myrcene, A and ocimene, B,are isomers with the molecular formula C 10 H 16 . When
subjected to hydrogen with platinum catalyst, both isomers give 2,6-dimethyloctane,
C 10 H 22 .

2,6-dimethyloctane

When treated with hot concentrated acidified KMnO 4 , A gives CO 2 and compound C,
C 7 H 10 O 4 ; B gives CO 2 and compounds D, C 5 H 8 O 3 , and E, C 3 H 4 O 3 .

Compounds C, D andE give one mole of CHI3 with alkaline aqueous iodine. On addition

trendyline
Na
of aqueous NaHCO 3 , compounds C, D andE
and
ndE produce
ce effervescence.
eff

Suggest structures
ctures for A-E,
uctures A-E, and explain the observations described abov
above. [10]

[Total: 20]

9647/03/CJC JC2 Preliminary


246 Examination 2016 trendyline
3

2 The table below lists the standard enthalpy change of formation of four compounds.

Compound H f o/kJ mol1


H 2 O(l) 286
HCl(g) 92.0
SiO 2 (s) 910
SiCl 4 (l) 640

(a) What is meant by the term, standard enthalpy change of formation of a compound? [1]

(b) SiCl 4 (l) undergoes hydrolysis to give SiO 2 (s).

(i) Write the balanced equation with state symbols for the hydrolysis reaction. [1]

(ii) Using the above data, calculate the standard enthalpy change for the hydrolysis
reaction. State one assumption made in your calculation. [1]

(iii) Does the hydrolysis have a positive, negative or zero entropy change? Explain your
answer. [1]

(iv) Hence, by the use of the Gibbs free energy, Go, explain why the hydrolysis of
SiCl 4 (l) is always a spontaneous process. [1]

(c) Consider reaction1and reaction 2 shown below.

anhydrous AlCl3
dimethylated benzenes
reacti on 1: + CH Cl
3 CH3 + an other ro ucts
d p d
Heat

anhydrous AlCl3
reacti on 2: + CH COCl
3 COCH3
Heat

(i) Both reaction 1 and reaction 2proceed via a similar mechanism. Name the type of
reaction undergone. [1]

(ii) Suggest why reaction 1 gives a mixture of organic products while reaction 2 gives
only one mono-substituted product. [2]

(d) The organic product in reaction 2, phenylethanone, COCH3, reacts with HCN to
produce the corresponding cyanohydrin under certain experimental conditions.

(i) State the experimental conditions. [1]

(ii)

trendyline
Describe
ibe the mechanism of reaction, of phenylethanone with HCN, showing clearly
the curly
ly arrows to indicate the movement of electrons and all charges.
rly char [3]

9647/03/CJC JC2 Preliminary


247 Examination 2016 [Turn
trendylineover
4

(iii) Hence, suggest the reagents and conditions used in the following conversion,
stating clearly the intermediate products in the process of synthesis. [2]

CONH2

COCH3 C CH3

Cl

(e) The rate of reaction between phenylethanone and iodine in acid medium to give
2-iodo-1-phenylethanone, COCH2I , is found to be independent of [ I 2 ], but

directly proportional to [H+] and directly proportional to [phenylethanone].

(i) Write the rate equation for this reaction and state the overall order and the
units of the rate constant. [2]

(ii) The reaction between phenylethanone and bromine proceeds by a similar


mechanism. How would you expect the rate of this reaction to compare with
that of the above reaction? Explain your answer. [1]

(iii) When a basic medium is used instead for the reaction between
phenylethanone and iodine, a different mechanism of reaction is followed and
different organic products are produced.

(I) State the type of reaction undergone. [1]

(II) Give the structures of all the organic products formed in this reaction. [2]

[Total: 20]

trendyline
9647/03/CJC JC2 Preliminary
248 Examination 2016 trendyline
5

3 Fireworks are a traditional part of many celebrations. Their colours can come from heating
ofGroup II metals and their salts. These salts often contain anions such as chlorates,
perchlorates, and nitrates which are oxidising agents.

(a) One of the Group II metals,M, when heated with oxygen under pressure, forms a metal
peroxide which can be used as anoxidising agent in fireworks to produce a vivid green
colour. The equation for the reaction is shown below:

2MO(s) + O 2 (g) 2MO 2 (s)

When equilibrium is attained, the mixture of oxides was found to have a mass of 1.60g,
where MO 2 has a percentage mass of 18.7%. When the mixture was treated with dilute
sulfuric acid, a white solid was precipitated and a solution of hydrogen peroxide was
produced.

An excess of acidified potassium iodide was added to the hydrogen peroxide solution.
The reaction between hydrogen peroxide and acidified potassium iodide is as follows.

H 2 O 2 + 2I+ 2H+ I 2 + 2H 2 O

The iodine liberated needed 11.80 cm3 of 0.300 moldm3 sodium thiosulfate, Na 2 S 2 O 3 , for
complete reaction.

I 2 + 2S 2 O 3 2 6 4 O 6 2 + 2I

(i) Write a balanced equation to represent the reaction between MO 2 and sulfuric
acid. [1]

(ii) By determining the amount of MO 2 that has reacted,calculate the relative atomic
mass of M and suggest its identity. [5]

(b) Strontium nitrate, Sr(NO 3 ) 2 is an oxidising agent and is commonly used as a red colouring
agent in fireworks.

(i) Write a balancedequation for the reaction that occurs when Sr(NO 3 ) 2
undergoesthermal decomposition. [1]

(ii) Predictwhether the decomposition temperature of Sr(NO 3 ) 2 will be higher or lower


than the decomposition temperature of Mg(NO 3 ) 2 . Explain your answer. [3]

(c) Anotheroxidising agent used in fireworks is potassium chlorate(V), KClO 3 .When KClO 3
undergoes thermal decomposition, two products are obtained, one of which is KCl.

(i) Write a balanced equation to show how KClO 3 is produced from Cl 2 gas and hot,
aqueous potassium hydroxide. [1]

(ii)

trendyline
Write a balanced equation for
suggest
st why KC
or the thermal decomposition of KClO 3 and hence
ClO 3 is an important ingredient
ingredien used in fireworks. [2]

9647/03/CJC JC2 Preliminary


249 Examination 2016 [Turn
trendylineover
6

(d) Fireworks which produce bright sparks, flashes and coloured flames also involve the
elements phosphorus, sulfur and chlorine.

The table below gives the melting points of these elements.

element phosphorus sulfur chlorine


melting point /K 317 392 172

In terms of structure and bonding, explain why sulfur has the highest melting point among
the three elements. [2]

(e) Sodium salicylate and sodium benzoate are used as whistle mixes in fireworks. Both are
white crystalline powder.
OH O O

+ +
O Na O Na

sodium salicylate sodium benzoate

(i) Suggest methods by which sodium salicylate and sodium benzoate can be
distinguished from each other by a simple chemical test. State the reagents and
conditions used and the expected observations for each compound. [2]

(ii) The pK a values of the two acidic groups in salicylic acid are pK a1 = 2.98 and
pK a2 =13.6.
OH O

OH

salicylic acid

Draw the displayed formula of the mono-anion produced after the dissociation of the
carboxylic acid group and use your formula to explain why the dissociation of the
phenol group is unlikely to occur. [3]

[Total: 20]

trendyline
9647/03/CJC JC2 Preliminary
250 Examination 2016 trendyline
7

4 Strecker synthesis is a method used for the synthesis of amino acids. It is often studied due to
the simplicity of the process.

(a) Alanine, an amino acid, can be formed using this method as shown below.

+
O NH2 Cl NH2
C
NH4Cl H3C C KCN
H3C C NaOH, heat
H3C Products
step 1 step 2 CN step 3
H H H

(i) Construct a balanced equation for step 1 and suggest the type of reaction that has
occurred. [2]

(ii) Give the structural formulae for allthe products formed in step 3. [2]

(b) Insulin is a hormonal protein which was first discovered in 1921. In 1959, it became the
first protein to have its primary structure determined. These led to the award of two Nobel
Prizes, in Physiology/Medicine and Chemistry respectively.

The basic unit of insulin consists of two polypeptides, A-chain and B-chain. A-chain has
21amino acid residues, and some of these are listed in the following table(You may use
the abbreviations for your answers).

formula of R group side chain number of amino acid


amino acid abbreviation
at pH 7.4 (physiological pH) residues in A-chain

cysteine cys CH 2 SH 4

glutamic acid glu CH 2 CH 2 CO2 2
glutamine gln CH 2 CH 2 CONH 2 2
glycine gly H 1
isoleucine ile CH(CH 3 )CH 2 CH 3 2
threonine thr CH(OH)CH 3 1
valine val CH(CH 3 )CH 3 1

In order to determine the sequence of the A-chain, partial hydrolysis was carried out with
two separate methods to obtain several polypeptide fragments. One method of hydrolysis
involves the use of 6 mol dm-3 HCl or NaOH, while the othermethod causes hydrolysis at
specific peptide bonds.

(i) State what may be used in the method for the hydrolysis at specific peptide bonds.[1]

(ii) subjecte for further hyd


One of the polypeptides was isolated and subjected hydrolysis giving the
following
ng smaller tri-peptide
tri--peptide fragments.
tri

cys-cys-thr,
cys
cys-
-cys ile-val-glu,
cy -thr, ile val--glu, glu-gln-cys,
ile-val glu n-cys, gly-ile-val
glu-gln gly
g -ile
e-val
val

Using the same abbreviations, write out the amino acid sequenceof the smallest
polypeptide that couldproduce the above fragments. [2]

9647/03/CJC JC2 Preliminary


251 Examination 2016 [Turn
trendylineover
8

(iii) Describe the tertiary structure of insulin A-chain, include the type of bondings or
interactions with reference to all the relevant amino acid residues listed in the above
table. [4]

(iv) In further confirmation of structures of insulin, chemicals can be used to affect the
structures. Suggest one type of bond or interaction from your answer in (b)(iii) that
can be broken by chemicals and state the chemicals that can be used. [2]

(v) Zinc has an important role in the the production, storage and use of insulin. Zinc
cation can bind to insulin through interactions with the amino acids. Suggest the
type of interaction and state an amino acid that may be involved. [1]

(c) Zinc compounds are often used in paints for various purposes. The properties of some of
these compounds are given below.

compound solubility product,K sp use in paints


17
Zn(OH) 2 3.00 x 10 White pigment
23
ZnS 3.21 x 10 Luminous pigment
ZnCO 3 1.46 x 1010 Fire Retardant

(i) Write an expression for K sp for Zn(OH) 2 and determine its solubility. Hence,
calculate the pH of a saturated solution of Zn(OH) 2 . [4]

(ii) In preparation of a water-based luminous paint mixture, all 3 of the above


compounds have to be precipitated. Given that concentration of Zn2+ is
2.00x 107 mol dm3, calculate the minimum amount of S2 that must be present in
250 cm3of this mixture. [2]

[Total: 20]

trendyline
9647/03/CJC JC2 Preliminary
252 Examination 2016 trendyline
9

5 Use of the Data Booklet is relevant to this question.

Aredoxflow batteryis a type of rechargeablebatterywhere rechargeability is provided by two


chemical components dissolved in liquids contained within the system. The two half-cells are
separated by a ion-exchange membrane instead of a salt bridge. A typical redox flow battery
uses vanadium ions and inert electrodes.

(a) Vanadium flow battery stores energy using vanadium redox couples. During the discharge
cycle, V2+ loses an electron to the external circuit to form V3+ in the negative half-cell and
acidified VO 2 + accepts an electron from the external circuit to form VO2+ in the positive
half-cell.

Proton
Exchange
Membrane
Inert electrode Inert electrode

V3+/V2+ VO2+/VO2+

- +

(i) Write out the two half-equations for the electrode processes that occur at the two
half-cells. [2]

(ii) Calculate the e.m.f. of the cell. [1]

(iii) Suggest whyvanadium ionscan beused in this redoxflow battery. [1]

trendyline
(iv) Suggest
st the role of a proton exchange membrane.
est [1]

9647/03/CJC JC2 Preliminary


253 Examination 2016 [Turn
trendylineover
10

(b) Calcium hydride is used in the synthesis of proton exchange membrane used in the
vanadium flow battery. Data about calcium and vanadium are given below.

Calcium Vanadium
Electronic configuration [Ar]4s2 [Ar]3d34s2
Relative atomic mass 40.1 50.9
Atomic radius/ nm 0.197 0.122
-3
Density/ gcm 1.54 6.07

(i) Using relevant data from the table, suggest why the density of vanadium is
significantly greater than that of calcium. [2]

(ii) Explain why vanadium compounds are usually coloured whereas calcium
compounds are usually colourless. [4]

(c) Compounds of calcium contain the Ca2+ ion. The corresponding ion of vanadium is V2+.

(i) Write an equation to show the third ionization energy for calcium. [1]

(ii) Hence explain why Ca3+ compounds do not exist whilst V3+ compounds do. [2]

(d) The following properties and reactionsare exhibited byCompound X, C 7 H 9 ON.

Compound X is insoluble in dilute NaOH, but dissolves in dilute HCl(aq).

On reaction with acidified potassium dichromate(VI)under certain condition,


compound X forms compound Y, C 7 H 7 ON.

On reaction with ethanoyl chloride, CH 3 COCl, X forms compound Z, C 11 H 13 NO 3 .

Deduce the structures of compounds X, Y and Z. Explain the chemistry of the


reactionsdescribed. Write an equation for the formation of compound Z. [6]

[Total: 20]

trendyline
9647/03/CJC JC2 Preliminary
254 Examination 2016 trendyline
CATHOLIC JUNIOR COLLEGE
JC2PRELIMINARY EXAMINATIONS
Higher 2

CHEMISTRY 9647/03
Paper 3 Free Response Friday 26 August 2016
2 hours
Candidates answer on separate paper.

Additional Materials: Answer Paper


Data Booklet

READ THESE INSTRUCTIONS FIRST

Write your name and class on all the work you hand in.
Write in dark blue or black pen.
You may use an HB pencil for any diagrams or graphs.
Do not use staples, paper clips, glue or correction fluid.

Answer any four questions.


A Data Booklet is provided.
The use of an approved scientific calculator is expected, where appropriate.
You are reminded of the need for good English and clear presentation in your answers.

At the end of the examination, fasten all your work securely together.
The number of marks is given in brackets [ ] at the end of each question or part question.

ANSWER SCHEME&
EXAMINERS
COMMENTS

ttrendyline
This
his
d li
is documents consists of 17printed
printed pages and 1 blank

9647/03/CJC JC2 Preliminary


lank page
pages.

nary Examination 2016 [Turn over

255 trendyline
2

Answer any four questions.

1 Use of the Data Booklet is relevant to this question.

(a) When potassium manganate(VII), KMnO 4 , is heated with concentrated sodium hydroxide,
NaOH, bubbles of oxygen are seen and a green solution of MnO 4 2- is obtained. The
addition of barium chloride, BaCl 2 , to this solution precipitated out a solid X with
composition by mass of 53.5% barium, 21.5% manganese and 25.0% oxygen.
(i) Calculate the empirical formula of X. [1]
Element Ba Mn O
% mass 53.5 21.5 25.0
Ar 137 54.9 16.0
No. of 0.391 0.392 1.56
moles
Mole ratio 1 1 4

Empirical formula of X is BaMnO 4

(ii) When the green solution above is acidified, a brown precipitate in a purple solution
is formed.
Suggest the identities of the species formed upon acidification and hence state the
type of reaction that has occured. [3]

Brown precipitate is MnO 2


Purple solution is MnO 4
Disproportionation

(b) A solution of acidified potassium manganate(VII), KMnO 4 , can be standardised through


titration against sodium ethanedioate, Na 2 C 2 O 4 . The ethanedioate ion is oxidised to
carbon dioxide in this reaction.

(i) State the oxidation numbers of C in the ethanedioate ion and in carbon dioxide. [1]

O.N. of C in C 2 O 4 2 = +3

O.N. of C in CO 2 = +4
(ii) Write the half-equation for the oxidation of the ethanedioate ion. [1]

C 2 O 4 2 2CO 2 + 2e

(iii) Hence, write the overall balanced equation for the reaction between acidified
manganate(VII) and ethanedioate ions. [1]

2MnO 4 + 16H+ + 5C 2 O 4 2 2Mn2+ + 10CO 2 + 8H 2 O

trendyline
9647/03/CJC JC2 Preliminary
256 Examination 2016 trendyline
3

(iv) In one such standardisation procedure, 25.0 cm3 of 0.0150 mol dm3 Na 2 C 2 O 4
solution required 28.85 cm3 of acidified KMnO 4 to reach the end-point.

Determine the concentration of KMnO 4 in the solution. You may assume that
2MnO 4 & 2 O 4 2. [3]

25.0
No. of moles of C 2 O 4 2 = 0.0150
1000

= 0.000375 mol

Since 2MnO 4 & 2 O 4 2,

2
No. of moles of MnO 4 = 0.000375
5

= 0.000150 mol

0.000150
Concentration of KMnO 4 = 1000
28.85

= 0.00520 mol dm3

(c) Myrcene, A and ocimene, B,are isomers with the molecular formula C 10 H 16 . When
subjected to hydrogen with platinum catalyst, both isomers give 2,6-dimethyloctane,
C 10 H 22 .

2,6-dimethyloctane

When treated with hot concentrated acidified KMnO 4 , A gives CO 2 and compound C,
C 7 H 10 O 4 ; B gives CO 2 and compounds D, C 5 H 8 O 3 , and E, C 3 H 4 O 3 .

Compounds C, D andE give one mole of CHI3 with alkaline aqueous iodine. On addition
of aqueous NaHCO 3 , compounds C, D andE produce effervescence.

Suggest structures for A-E, and explain the observations described above. [10]

trendyline
9647/03/CJC JC2 Preliminary
257 Examination 2016 [Turn
trendylineover
4

C10H22

H2/Pt
hydrogenation / reduction
=> 3 moles H added
2
=> 3 x C=C / alkene in A, B

A B
C10H16 C10H16

KMnO4/H , heat
+

oxidative cleavage
=> 3 C atoms lost as CO in A
C has 4 oxygen atoms => 3 terminal C=C in A
2
=> 1 x CH CO-
3
=> 1 x -CO H or
2 => 2 C atoms lost as CO in B
=> 1 x ketone without 2
adjacent -CH3 => 2 terminal C=C in B

C D E
+ + +
C7H10O4 CO2 C5H8O3 C3H4O3 CO2

KMnO4/H , heat
+

oxidative cleavage
=> C, D and E have ketone and/or carboxylic acid groups

Iodof orm/Triiodomethane test


=> C, D and E have CH CO- groups
3
NaHCO3(aq)
- -
Acid carbonate/acid base/neutralisation reaction
=> C, D and E produce CO
2
=> they have carboxylic acid groups

CO2
HO

O O CO2
Hence, A: CO2 O

HO

O CO2

CO2
Hence, B: O E O

ttrendyline
ine
ne
ne
O

D
HO
HO

[Total: 20]

9647/03/CJC JC2 Preliminary


258 Examination 2016 trendyline
5

2 The table below lists the standard enthalpy change of formation of four compounds.

Compound H f o/kJ mol1


H 2 O(l) 286
HCl(g) 92.0
SiO 2 (s) 910
SiCl 4 (l) 640

(a) What is meant by the term, standard enthalpy change of formation of a compound? [1]
The enthalpy change when 1 mol of a compound is formed from its elements(in its
most stable form), under standard conditions (25oC, 1 atm pressure).

(b) SiCl 4 (l) undergoes hydrolysis to give SiO 2 (s).


(i) Write the balanced equation with state symbols for the hydrolysis reaction. [1]
SiCl 4 (l) + 2H 2 O(l 6L2 2 (s) + 4HCl(g)

(ii) Using the above data, calculate the standard enthalpy change for the hydrolysis
reaction. State one assumption made in your calculation. [1]

H r o H f o(products) H f o(reactants)
= [ 910 + 4(-92.0)] [ 640 + 2( 286)]
= 66.0 kJ mol-1

Assumption:
- SiCl 4 (l) is in excess. The hydrolysis gives HCl(g) instead of HCl(aq).
- Hess Law is followed/obeyed.

(iii) Does the hydrolysis have a positive, negative or zero entropy change? Explain your
answer. [1]
Positive entropy change. 4 moles of gas (HCl) are produced. There is an
increase in disorder of the system as there are more ways of arranging the
partciples in the system.

(iv) Hence, by the use of the Gibbs free energy, Go, explain why the hydrolysis of
SiCl 4 (l) is always a spontaneous process. [1]
o o o
G = H - TS
Since Ho is negative and So is positive, Go is always negative. So the
process is spontaneous.

(c) Consider reaction 1 and reaction 2 shown below.

anhydrous AlCl3
dimethylated benzenes
reacti on 1: + CH Cl
3 CH3 + an other ro ucts
d p d
Heat

reacti on 2:
trendyline
re dyliine
re ne + CH
CH3COC
COCll
nhyd
anhydrous
drous
s AlCl

Heat
AllCl3
COCH3
CO

9647/03/CJC JC2 Preliminary


259 Examination 2016 [Turn
trendylineover
6

(i) Both reaction 1 and reaction 2proceed via a similar mechanism. Name the type of
reaction undergone. [1]
Electrophilic substitution.
(ii) Suggest why reaction 1 gives a mixture of organic products while reaction 2 gives
only one mono-substituted product. [2]

Reaction 1
The CH 3 is an electron donating or activating group. It activates the benzene
ring and so makes the delocalised electrons more susceptible/available to
electrophilic attack/electrophiles. This favours further substitution to give
multi-substituted products.

Reaction 2
The C=O is an electron withdrawing or deactivating group. It deactivates the
benzene ring and so makes the delocalised electrons less
susceptible/available to electrophilic attack/electrophiles. Further substitution
is less favourable.

(d) The organic product in reaction 2, phenylethanone, COCH3, reacts with HCN to
produce the corresponding cyanohydrin under certain experimental conditions.

(i) State the experimental conditions. [1]

Trace amount of NaOH/NaCN and 10 20oC

(ii) Describe the mechanism of reaction, of phenylethanone with HCN, showing clearly
the curly arrows to indicate the movement of electrons and all charges. [3]

Nucleophilic Addition Mechanism.

HCN H+ + :CN-


O O

+ slow
CH3 H CH2CH3
CN
CN
(intermediate)

O CH3
fast
+ H CN C OH + CN
-
CH3 +
CN CN

(iii) Hence, suggest the reagents and conditions used in the following conversion,

ttrendyline
tr nd
dyyli
stating
g clearly
y the intermediate p
products in the p
process of synthesis.
y [2]

CONH2

COCH
OCH3 C C 3
CH

Cl

9647/03/CJC JC2 Preliminary


260 Examination 2016 trendyline
7

HCN/
CN COOH
Trace NaOH H2SO4 (aq)
COCH3 - o C CH3 C CH3
10 20 C H t under reflux
ea
OH OH

PCl5

CONH2 COCl
NH3,
C CH3 room temperature C CH3
Cl Cl

(e) The rate of reaction between phenylethanone and iodine in acid medium to give
2-iodo-1-phenylethanone, COCH2I , is found to be independent of [ I 2 ], but

directly proportional to [H+] and directly proportional to [phenylethanone].

(i) Write the rate equation for this reaction and state the overall order and the
units of the rate constant. [2]

+
Rate = k [ COCH3 ][H ]

Overall order = 2, units: mol-1 dm3 s-1

(ii) The reaction between phenylethanone and bromine proceeds by a similar


mechanism. How would you expect the rate of this reaction to compare with
that of the above reaction? Explain your answer. [1]

The reaction rate is unchanged as it is also independent of [Br 2 ] like I 2 .

(iii) When a basic medium is used instead for the reaction between
phenylethanone and iodine, a different mechanism of reaction is followed and
different organic products are produced.

(I) State the type of reaction undergone. [1]

Triidomethane/iodoform reaction.

(II) Give the structures of all the organic products formed in this reaction. [2]

CO2
CHI 3
[Total: 20]

trendyline
9647/03/CJC JC2 Preliminary
261 Examination 2016 [Turn
trendylineover
8

3 Fireworks are a traditional part of many celebrations. Their colours can come from heating
ofGroup II metals and their salts. These salts often contain anions such as chlorates,
perchlorates, and nitrates which are oxidising agents.

(a) One of the Group II metals,M, when heated with oxygen under pressure, forms a metal
peroxide which can be used as anoxidising agent in fireworks to produce a vivid green
colour. The equation for the reaction is shown below:

2MO(s) + O 2 (g) 2MO 2 (s)

When equilibrium is attained, the mixture of oxides was found to have a mass of 1.60g,
where MO 2 has a percentage mass of 18.7%. When the mixture was treated with dilute
sulfuric acid, a white solid was precipitated and a solution of hydrogen peroxide was
produced.

An excess of acidified potassium iodide was added to the hydrogen peroxide solution.
The reaction between hydrogen peroxide and acidified potassium iodide is as follows.

H 2 O 2 + 2I+ 2H+ I 2 + 2H 2 O

The iodine liberated needed 11.80 cm3 of 0.300 moldm3 sodium thiosulfate, Na 2 S 2 O 3 ,for
complete reaction.

I 2 + 2S 2 O 3 2 6 4 O 6 2 + 2I

(i) Write a balanced equation to represent the reaction between MO 2 and sulfuric acid.[1]
MO 2 + H 2 SO 4 062 4 + H 2 O 2

(ii) By determining the amount of MO 2 that has reacted,calculate the relative atomic
mass of M and suggest its identity. [5]

H 2 O 2 + 2I + 2H+ I 2 + 2H 2 O
I 2 + 2S 2 O 3 2 6 4 O 6 2 + 2 I
MO 2 + 2 O 2 I 2 6 2 O 3 2
MO 2 6 2 O 3 2
Amount of S 2 O 3 2= 11.8 x 103 x 0.3
= 3.54 x 103mol
Amount of MO 2 = 0.5 x 3.54 x 103
= 1.77 x 103 mol
Let the relative atomic mass of M be x g
Mass of MO 2 = (x+ 2 x 16.0) x 1.77 x 103
18.7
1.60 =(x+216.0)1.7710-3
100

trendyline
=137
Identityty of M: Barium
Bariu

9647/03/CJC JC2 Preliminary


262 Examination 2016 trendyline
9

(b) Strontium nitrate, Sr(NO 3 ) 2 is an oxidising agent and is commonly used as a red colouring
agent in fireworks.

(i) Write a balancedequation for the reaction that occurs when Sr(NO 3 ) 2
undergoesthermal decomposition. [1]
2SrO(s) + 4NO 2 (g) + O 2 (g)
2Sr(NO 3 ) 2 (s)

(ii) Predictwhether the decomposition temperature of Sr(NO 3 ) 2 will be higher or lower


than the decomposition temperature of Mg(NO 3 ) 2 . Explain your answer. [3]

Higher.

cationic radius Sr2+ is larger than that of Mg2+ and charge density or polarising
power of Sr2+ is smaller.

electron cloud of NO 3 anion is less distorted 12 ERQG ZHDNHQHG WR D


smaller extent, does not break as easily.

Thus Sr(NO 3 ) 2 is thermally more stable than MgNO 3 , and decomposition


temperature is higher.

(c) Anotheroxidising agent used in fireworks is potassium chlorate(V), KClO 3 .When KClO 3
undergoes thermal decomposition, two products are obtained, one of which is KCl.

(i) Write a balanced equation to show how KClO 3 is produced from Cl 2 gas and hot,
aqueous potassium hydroxide. [1]

3Cl 2 (g) + 6KOH(aq) 5KCl(aq) + KClO 3 (aq) + 3H 2 O(l)

(ii) Write a balanced equation for the thermal decomposition of KClO 3 and hence
suggest why KClO 3 is an important ingredient used in fireworks. [2]

2KClO 3 (s) 2KCl(s) + 3O 2 (g)


O 2 gas is produced when heat is applied to potassium chlorate. O 2 gas
supports subsequent combustion of fireworks/explosives, therefore it is an
important ingredient.

(d) Fireworks which produce bright sparks, flashes and coloured flames also involve the
elements phosphorus, sulfur and chlorine.

The table below gives the melting points of these elements.

element phosphorus sulfur chlorine


melting point / K 317 392 172

In terms of structure and bonding, explain why sulfur has the highest melting point among
the three elements. [2]

trendyline
ndyy e
ents exist as simple covalent molecules
All 3 elements olecules with van
n de
d
der Waals forces
between the
e molecules.

S 8 has the
he
e largest number of electrons
electrons,, compared to t P4 a and Cl 2 ; it has
the strongest forces, hence
est van der Waals forces ce iit needs the largest amo
amount of energy to
overcome these attractions during melting.

9647/03/CJC JC2 Preliminary


263 Examination 2016 [Turn
trendylineover
10

(e) Sodium salicylate and sodium benzoate are used as whistle mixes in fireworks. Both are
white crystalline powder.
OH O O

+ +
O Na O Na

sodium salicylate sodium benzoate

(i) Suggest methods by which sodium salicylate and sodium benzoate can be
distinguished from each other by a simple chemical test. State the reagents and
conditions used and the expected observations for each compound. [2]
Method 1
To 2 separate test tubes containing sodium salicylate and sodium benzoate,
add neutral FeCl 3 (aq). A violet colouration will be observed in the test tube
containing sodium salicylate. No violet colouration is seen in the test tube
containing sodium benzoate.

OR

Method 2
To 2 separate test tubes containing sodium salicylate and sodium benzoate,
add aqueous Br 2 and shake. White precipitate formed in test tube
containing sodium salicylate and orange bromine water decolourises.
Orange aqueous bromine water remains orange in test tube containing
sodium benzoate.

(ii) The pK a values of the two acidic groups in salicylic acid are pK a1 = 2.98 and
pK a2 =13.6.

OH O

OH

salicylic acid

Draw the displayed formula of the mono-anion produced after the dissociation of the
carboxylic acid group and use your formula to explain why the dissociation of the
phenol group is unlikely to occur. [3]

H Intramolecular hydrogen bonding
- O O

trendyline
ttr
ren
e O

[Total: 20]

9647/03/CJC JC2 Preliminary


264 Examination 2016 trendyline
11

4 Strecker synthesis is a method used for the synthesis of amino acids. It is often studied due to
the simplicity of the process.

(a) Alanine, an amino acid, can be formed using this method as shown below.

+
O NH2 Cl NH2
C
NH4Cl H3C C KCN
H3C C NaOH, heat
H3C Products
step 1 step 2 CN step 3
H H
H

(i) Construct a balanced equation for step 1 and suggest the type of reaction that has
occurred. [2]
+
O NH2 Cl
H3C C + NH4Cl H3C C + H2O
H H
Condensation/Addition-Elimination

(ii) Give the structural formulae for allthe products formed in step 3. [2]

NH2 O N
+ H H
CH3 CH C O Na H

(b) Insulin is a hormonal protein which was first discovered in 1921. In 1959, it became the
first protein to have its primary structure determined. These led to the award of two Nobel
Prizes, in Physiology/Medicine and Chemistry respectively.

The basic unit of insulin consists of two polypeptides, A-chain and B-chain. A-chain has
21amino acid residues, and some of these are listed in the following table(You may use
the abbreviations for your answers).

formula of R group side chain number of amino acid


amino acid abbreviation
at pH 7.4 (physiological pH) residues in A-chain

cysteine cys CH 2 SH 4
glutamic acid glu CH 2 CH 2 CO
2 2
glutamine gln CH 2 CH 2 CONH 2 2
glycine gly H 1
isoleucine ile CH(CH 3 )CH 2 CH 3 2
threonine thr CH(OH)CH 3 1

trendyline
tren
ndylin
ne
valine val CH(CH
C 3 )CH
CH 3 1

In order to determine
deter
et mine the sequence of the A-chain,
A-cchain, partial hydrolysis was
wa carried out with
two separatee methods to obtain several polypeptide fragments.
fragmen
fragm e tss. One me
method of hydrolysis
involves the use of 6 mol dm-3 HCl or NaOH
NaOH,H while the othermethod cau causes hydrolysis at
specific peptide bonds.

9647/03/CJC JC2 Preliminary


265 Examination 2016 [Turn
trendylineover
12

(i) State what may be used in the method for the hydrolysis at specific peptide bonds.[1]
Enzyme

(ii) One of the polypeptides was isolated and subjected for further hydrolysis giving the
following smaller tri-peptide fragments.

cys-cys-thr, ile-val-glu, glu-gln-cys, gly-ile-val

Using the same abbreviations, write out the amino acid sequenceof the smallest
polypeptide that couldproduce the above fragments. [2]

gly-ile-val-glu-gln-cys-cys-thr

(iii) Describe the tertiary structure of insulin A-chain, include the type of bondings or
interactions with reference to all the relevant amino acid residues listed in the above
table. [4]
The tertiary structure of insulin A-chainis the overall 3-dimensional shape of
the protein resulting from folding or coiling of the chains due to various R-
group interactionswithin a polypeptide chain. These may be hydrogen
bonding between R-groups of gln and thrresidues, covalent disulfide linkages
between sulfur of cys residues and van der Waals between R-groups of gly,ile
and val residues.

(iv) In further confirmation of structures of insulin, chemicals can be used to affect the
structures. Suggest one type of bond or interaction from your answer in (b)(iii) that
can be broken by chemicals and state the chemicals that can be used. [2]
Disulfide linkage can be broken by heavy metal ions, such as Ag+ or Hg2+,
hence either AgNO 3 or Hg(NO 3 ) 2 can be used.

(v) Zinc has an important role in the the production, storage and use of insulin. Zinc
cation can bind to insulin through interactions with the amino acids. Suggest the
type of interaction and state an amino acid that may be involved. [1]
Ionic interactions with glutamic acid (glu);OR ion-dipole attractions with
threonine (thr) or glutamine (gln); OR Covalent dative bonds with glutamic
acid (glu), threonine (thr) or glutamin (gln).

trendyline
9647/03/CJC JC2 Preliminary
266 Examination 2016 trendyline
13

(c) Zinc compounds are often used in paints for various purposes. The properties of some of
these compounds are given below.

compound solubility product, K sp use in paints


Zn(OH) 2 3.00 x 1017 White pigment
ZnS 3.21 x 1023 Luminous pigment
10
ZnCO 3 1.46 x 10 Fire Retardant

(i) Write an expression for K sp for Zn(OH) 2 and determine its solubility. Hence,hence
calculate the pH of a saturated solution of Zn(OH) 2 . [4]
2+ -
Zn(OH) 2 (s) Zn (aq) + 2OH (aq)
K sp = [Zn2+][OH-]2

Let solubility of Zn(OH) 2 be xmol dm-3


K sp = (x)(2x)2
3.00 x 10-17 =4x3
3 -17
3.00 10
x= = 1.96 x 10-6 mol dm-3
4
[OH-] = 2x = 2(1.96 x 10-6) = 3.92 x 10-6mol dm-3

pOH = lg (3.92 x 10-6) = 5.41


pH = 14 5.41 = 8.59

(ii) In preparation of a water-based luminous paint mixture, all 3 of the above


compounds have to be precipitated. Given that concentration of Zn2+ is
2.00x 107 mol dm3, calculate the minimum amount of S2 that must be present in
250 cm3of this mixture. [2]

For precipitation, [Zn2+][S2-]>K sp


[S2-] > 3.21 x 10-23 / 2.00 x 10-7
[S2-] > 1.605 x 10-16

Minimum amount of S2- = 1.605 x 10-16 x 250/1000


= 4.0125 x 10-17 mol = 4.02 x 10-17 mol

[Total: 20]

trendyline
9647/03/CJC JC2 Preliminary
267 Examination 2016 [Turn
trendylineover
14

5 Use of the Data Booklet is relevant to this question.

A redox flow battery is a type of rechargeable battery where rechargeability is provided by two
chemical components dissolved in liquids contained within the system. The two half-cells are
separated by a ion-exchange membrane instead of a salt bridge. A typical redox flow battery
uses vanadium ions and inert electrodes.

(a) Vanadium flow battery stores energy using vanadium redox couples. During the discharge
cycle, V2+ loses an electron to the external circuit to form V3+ in the negative half-cell and
acidified VO 2 + accepts an electron from the external circuit to form VO2+ in the positive
half-cell.

Proton
Exchange
Membrane
Inert electrode Inert electrode

V3+/V2+ VO2+/VO2+

- +

(i) Write out the two half-equations for the electrode processes that occur at the two
half-cells. [2]
2+ 3+ -
Negative half-cell: V V + e
Positive half-cell: VO 2 + + 2H+ + e- VO2+ + H 2 O

(ii) Calculate the e.m.f. of the cell. [1]


Eo cell =Eo red Eo oxi =(+1.00) ( 0.26) = +1.26V

trendyline
(iii) est
Suggestst whyvanadium ionscan beused se in this redoxflow
redo flow battery.
redox [1]
y to exist in different oxidation states
Ability states

(iv) est
st the role of a proton
Suggest ro exchange membrane. [1]
To maintain
aintain electrical neutrality

9647/03/CJC JC2 Preliminary


268 Examination 2016 trendyline
15

(b) Calcium hydride is used in the synthesis of proton exchange membrane used in the
vanadium flow battery. Data about calcium and vanadium are given below.

Calcium Vanadium
Electronic configuration [Ar]4s2 [Ar]3d34s2
Relative atomic mass 40.1 50.9
Atomic radius/ nm 0.197 0.122
-3
Density/ g cm 1.54 6.07

(i) Using relevant data from the table, suggest why the density of vanadium is
significantly greater than that of calcium. [2]

Both vanadium and calcium have a metallic structure. The relative atomic
mass of vanadium is higher than that of calcium.The atomic volume
4
(V= r3) of vanadium is lower than that of calcium due to its smaller atomic
3
radius.

With the greater Ar of vanadium and its smaller atomic volume, the density of
vanadium is significantly higher since density = mass/volume.

(ii) Explain why vanadium compounds are usually coloured whereas calcium
compounds are usually colourless. [4]

Vanadium compounds are usually coloured because they have incompletely


filled d- orbitals.In the presence of ligands, the 3d orbitals become non-
degenerate and split into two groups with slightly different energy (d-d*
splitting). When a d-electron from the lower energy group is promoted to the
higher energy group (d-d* electronic transition), radiation in the visible region
of the electromagnetic spectrum is absorbed. Thecomplementary colour not
absorbed will be seen.

Calcium compounds are usually colourless because they have no d-electrons


and so, d-d* electronic transition cannot take place.

(c) Compounds of calcium contain the Ca2+ ion. The corresponding ion of vanadium is V2+.

(i) Write an equation to show the third ionization energy for calcium. [1]
Ca2+(g) Ca3+(g) + e-

(ii) Hence explain why Ca3+ compounds do not exist whilst V3+ compounds do. [2]

Formation of Ca3+ involves removing an electron from the inner principal

ttrendyline
yli
quantum
um shell(3p
shell(3p subshell) which
ch is more e strongly
str attracted by the nucleus
and this
his requires a lla
large electron. Hence Ca3+
rge amount of energy to remove the elec
compounds
ounds do not exist.
exis
3+
Formation
tion of V involves
ation involv remov
removal
al of electrons from the 3d orb
or
orbital which has a
arable energy with that of 4s orbital.
comparable orbital Therefore,
Therefore it isi less strongly
attracted by the nucleus and this requires much less energy to remove the
electron. Hence V3+ compounds can be formed.

9647/03/CJC JC2 Preliminary


269 Examination 2016 [Turn
trendylineover
16

(d) The following properties and reactionsare exhibited byCompound X, C 7 H 9 ON.

Compound X is insoluble in dilute NaOH, but dissolves in dilute HCl(aq).

On reaction with acidified potassium dichromate(VI)under certain condition,


compound X forms compound Y, C 7 H 7 ON.

On reaction with ethanoyl chloride, CH 3 COCl, X forms compound Z, C 11 H 13 NO 3 .

Deduce the structures of compounds X, Y and Z. Explain the chemistry of the


reactionsdescribed. Write an equation for the formation of compound Z. [6]

Structures of X, Y and Z are:

O
CH2OH CH2 O C CH3

NH2 NH2 NHCOCH3


X Y Z

Compound Xdissolves in HCl(aq) shows that it contains a basic group.

Compound Xdoes not dissolve in NaOH(aq), Hence, it does not contain a phenol group.

Compound Xundergo oxidation with acidified potassium dichromate (VI) and therefore, it is
a (primary/secondary) alcohol.

Compound Xundergo condensation/ acylation/ addition-eliminationwith ethanoyl chloride to give


compoundZ.

O
C CH3
CH2OH
CH2 O

+ 2 CH3COCl + 2 HCl

NH2
NHCOCH3
[Total: 20]

9647/03/CJC JC2 Preliminary


270 Examination 2016 trendyline
1

Name: Index Number: Class:

DUNMAN HIGH SCHOOL


Preliminary Examinations 2016
Year 6

H2 CHEMISTRY 9647/01
Paper 1 Multiple Choice 27 September2016
1 hour
Additional Materials: Optical Mark Sheet
Data Booklet

INSTRUCTIONS TO CANDIDATES
1 Write your name and class on this question paper.
2 There are forty questions on this paper. Answer all questions. For each question there are
four possible answers A, B, C and D.
Choose the one you consider correct and record your choice in soft pencil on the separate
Optical Mark Sheet.
3 Each correct answer will score one mark. A mark will not be deducted for wrong answer.
4 Any rough working should be done in this booklet.
5 You may use a calculator.

trendyline
This question paper consists of18 printed pages and 0blank page.
DHS 2016 [Turn over
271 trendyline
2

Section A

For each question, there are four possible answers,A, B, C, and D. Choose the one you consider
to be correct.

1 Ferrous sulfate (FeSO 4 ) tablets are commonly prescribed by doctors as 350 mg tablets and
serve as a dietary supplement for pregnant women. However, it was found that these tablets
may cause poisoning in young children if accidentally consumed. The lethal dosage for a
12.0 kg child is 590 mg of Fe2+.

What is the minimum number of tablets that would constitute a lethal dose to a 12.0 kg child?

A 1
B 2
C 4
D 5

2 25.0 cm3 of a solution of 0.0518 moldm3hydroxylammonium chloride, NH 3 OH+Cl was


added to a solution containing an excess of acidified Fe3+ ions and the mixture boiled. The
Fe2+ ions in the resultant solution was titrated with 25.90 cm3 of 0.02 moldm3 potassium
manganate (VII) solution.

Given the mole ratio MnO 4  )H2+, which of the following nitrogencontaining species is
formed in the reaction?

A NH 4 +
B N2O
C NO 2
D NO 3

3 Two elements, Y and Z, have the following properties.

Property 1: Y and Z form ionic compounds Na 3 Y and Na 3 Z respectively.


Property 2: Element Z forms ZCl 5 molecule whereas Y is unable to form YCl 5 .

Which pair of electronic configurations of Y and Z is correct?

Y Z
A [He] 2s2 2p2 [Ne] 3s2 3p3
B [He] 2s2 2p3 [Ne] 3s2 3p3

trendyline
C [He] 2s2 2p2 [He] 2s2 2p3
D [Ne] 3s2 3p2 [Ne] 3s2 3p3

DHS 2016 9647/01 [Turn over


272 trendyline
3

4 For one mole of an ideal gas, which plot produces a straight line graph passing through the
origin?

Yaxis Xaxis
A PV P at constant T (K)
B P V at constant T (K)
C P T (C) at constant V
D P 1/V at constant T (K)

5 Use of Data Booklet is relevant to this question.


Which substance is considered to have the largest covalent character?

A AlF 3 B Al 2 O 3 C Be 3 N 2 D BeO

6 Which substance does not contain an atom that has an unpaired electron?

A ClO 2 B NO C NO 2 D N2O

7 Ammonia is made via the Haber Process. The reactants are nitrogen and hydrogen.

N 2 (g) + 3H 2 (g) 2NH 3 J H = 92 kJ mol1

What will increase the rate of the forward reaction?

A Adding argon to the mixture but keeping the total volume constant.
B Decreasing the temperature.
C Increasing the total pressure by reducing the total volume at constant temperature.
D Removing nitrogen from the mixture but keeping the total volume of the mixture the
same.

8 How much water must be added to a 10 cm3solution of 0.05 moldm3 sulfuric acid in order to
increase its pH to 2.0?

A 10 cm3
B 50 cm3
C 90 cm3

trendyline
D 100 cm3

DHS 2016 9647/01 [Turn over


273 trendyline
4

9 Given that the K sp for magnesium hydroxide, Mg(OH) 2 , is 1.80 x 1011 mol3dm9 at 298 K,
calculate the pH of a saturated solution of Mg(OH) 2 .

A 3.48
B 3.78
C 10.2
D 10.5

10 The reaction of nitrogen monoxide and hydrogen gas

2NO(g) + 2H 2 (g) N 2 (g) + 2H 2 O(g)

is hypothesised to involve the following steps:

I NO + NO N 2 O 2 (fast)
II N 2 O 2 +H 2 H 2 O + N 2 O (slow)
III N 2 O +H 2 N 2 +H 2 O (fast)

Which of the following is true about the reaction?

A H 2 acts as a catalyst in this reaction.


B The overall order of the reaction is 2.
C There are 2 intermediates present in the reaction mechanism.
D Increasing the concentration of NO will increase the rate constant.

11 The radius and charge of each of the six ions are shown in the table.

ion J+ L+ M2+ X Y Z2
radius / nm 0.14 0.18 0.15 0.14 0.18 0.15

Which of the following pair shows the first compound having a smaller magnitude of lattice
energy than the second?

A JX, MZ
B JX, LX
C JY, LX
D MZ, LY

DHS 2016 9647/01 [Turn over


274 trendyline
5

12 The common rubber band has very interesting thermodynamic properties due to its
randomly coiled long polymeric molecular structure. When the rubber band is stretched, a
slight warming effect is felt.

:KDW DUH WKH FRUUHFW VLJQV RI 6 + DQG * LI WKH VWUHWFKHG UXEEHU EDQG LV UHOHDVed
quickly?

S H G

A + +

B + +

C +

D +

13 The nickelcadmium cell is a rechargeable battery which contains an alkaline electrolyte


such as aqueous KOH.

During the discharging process, Cd is oxidised to solid Cd(OH) 2 whileNiO(OH) is reduced to


solid Ni(OH) 2 .

Which statement is true about this rechargeable battery?

A Water is produced in the discharging process.


B The mass of cadmium remains unchanged.
C The alkaline electrolyte can be replaced by an acid.
D The concentration of the alkaline electrolyte remains unchanged in the discharging
process.

14 Which statement regarding the oxides across Period 3 is true?

A The covalent character decreasesfrom Na to S.

B The oxides of the elements changes from basic to neutral then to acidicfrom Na to S.

C The oxides formed are increasing soluble in water from Na to S.

D The standard entropy change of formation of the oxides becomes morenegativefrom


Na to Al.

trendyline
DHS 2016 9647/01 [Turn over
275 trendyline
6

15 The following information is about a Period 3 element L.

The oxide of L is a solid at room temperature.


The oxide of L, when added to water, gives a non-acidic solution.
The aqueous chloride of L gives a white precipitate with aqueous sodium hydroxide.

In which Group of the Periodic Table could L be found?

A Vonly
B I and III only
C II and III only
D II, III and IVonly

16 Which property shows an increasing trend down Group II from Mg to Ba?

A Polarising power of metal cation


B Second ionisation energy
C Reducing strength
D Melting point

17 Which statement regarding the halogens or their hydrides is correct?

A HCl has a higher melting point than HF due to its larger electron cloud.
B Iodine, when dissolved in hexane, gives a colour that is similar to its vapour.
C The halogens become less volatile from fluorine to iodine due to the weaker covalent
bonds.
D HF is less thermally stable than HI because of the larger electronegativity difference
between the H and F atoms.

trendyline
DHS 2016 9647/01 [Turn over
276 trendyline
7

18 Photolysis of a solution of Fe(CO) 5 in ethanoic acid produces Fe 2 (CO) 9 according to the


following equation.
O
CO C
OC CO
OC O
C
2 Fe CO
OC Fe Fe CO + CO
OC CO
CO OC C
O

What is the oxidation state of Fe in Fe(CO) 5 and the coordination number of Fe in


Fe 2 (CO) 9 ?

oxidation state of Fe coordination number of Fe


in Fe(CO) 5 in Fe 2 (CO) 9
A +5 9
B +5 6
C 0 9
D 0 6

19 The structure of 11-ketotestosterone, a sex hormone in fish, is shown below.

OH
O

Which statement about 11-ketotestosterone is correct?

A Its molecular formula is C 19 H 26 O 3 .


B It has a total of 27 stereoisomers.
C It has six sp2-hybridised carbon atoms.
D It has a tertiary alcohol functional group.

trendyline
DHS 2016 9647/01 [Turn over
277 trendyline
8

20 The following compounds have the same molecular formula.

Cl Cl Cl H

Cl Cl
H CH3 Cl CH3

X Y Z

Which of the following best describes the isomeric relationships between the compounds?

X and Y Y and Z
A chain optical
B positional functional group
C cistrans positional
D positional chain

21 Which of the following is a non-greenhouse gas that could be released from the catalytic
converter of a car exhaust?

A CO 2
B H2O
C CH 4
D N2

22 Which compound is not possibly formed when cyclopentane is reacted with excess bromine
gas in the presence of ultraviolet light?

Br
A Br B

Br

C D

Br Br

trendyline
DHS 2016 9647/01 [Turn over
278 trendyline
9

23 Carbonyl groups in aldehydes and ketones, C=O, undergo nucleophilic addition while alkene
groups, C=C, undergo electrophilic addition.

Which statement explains the above reactions?

A Oxygen is more reactive than carbon.


B The different lengths of the double bonds.
C The different strengths of the double bonds.
D The electronegativity difference between the carbon and oxygen atoms in the carbonyl
group.

24 The citrus flavour of lemons is due to the compound limonene, present in both the peel and
the juice.

limonene

Limonene is separately treated with


cold, dilute acidified KMnO 4 ,
hot, concentrated acidified KMnO 4 .

What is the change in the number of chiral carbon atoms in the molecule during each
reaction?

cold, dilute acidified KMnO 4 hot, concentrated acidified KMnO 4


A +3 0
B +3 1
C +4 0
D +4 1

trendyline
DHS 2016 9647/01 [Turn over
279 trendyline
10

25 The diagram shows the structure of a derivative of propofol.

When reacted with a dilute solution of Cl 2 , a chlorine atom may substitute a hydrogen atom
on the benzene ring but not for a hydrogen atom on the alkyl branches.

Given that any number of the benzene hydrogen atoms may be substituted, how many
possible products of the reaction are there?

A 3
B 4
C 5
D 6

26 In the body, cellular respiration produces energy from the oxidation of glucose.

The diagram shows the structure of glucose.

OH

H O OH
H
OH H
HO H
H OH

A new artificial sweetener has been produced by replacing all of the hydroxyl groups
attached directly to the ring carbon atoms in glucose with chlorine atoms.

What is the empirical formula of this chlorinated glucose?

A CH 2 Cl
B C 3 H 4 Cl 2 O
C C 6 H 7 Cl 5 O
D C 6 H 8 Cl 4 O 2

trendyline
DHS 2016 9647/01 [Turn over
280 trendyline
11

27 Tetrangomycinis one of the first member of the class of antibiotics under the angucycline
group. What is the maximum number of optical isomers that can be obtained when
1 mole of tetrangomycinfully reacts with NaBH 4 ?

O CH3
O
OH

OH O
T et rangomy ci n

A 2
B 8
C 16
D 32

28 The small hive beetle, which invades colonies of the honeybee, identifies these colonies by
detecting the bees own alarm signal, the pheromone 3-methylbutyl ethanoate.

How may this ester be made in the laboratory?

A CH 3 COCl + (CH 3 ) 2 CHCH 2 CH 2 OH ester + HCl


B CH 3 COCl + CH 3 CH 2 CH(CH 3 )CH 2 OH ester + HCl

conc. H2SO4
C (CH3)2CHCH2CO2H + CH3CH2OH ester + H2O
heat

conc. H2SO4
D (CH3)2CHCH2CH2CO2H + CH3CH2OH ester + H2O
heat

29 Why are amides, RCONH 2 , less basic than amines, RNH 2 ?

A Amides form a zwitterion in which the nitrogen atom carries a positive charge.
B Amides have a resonance structure involving the movement of a pair of electrons from
the nitrogen atom to the oxygen atom.
C

D trendyline
ns on the nitrogen atom of amides move
Electrons
haracter
aracter so that it is more difficult to break.
bond character
mide
ve on the C-N

de carbonyl group withdraws electrons from the -NH


The amide
en atoms acidic.
hydrogen acidic
break
C-
C-N bond giving

NH 2 g
giv it some double

group to make the

DHS 2016 9647/01 [Turn over


281 trendyline
12

30 3-aminoisobutyric acid was shown to prevent diet induced diabetes in mouse. It has the
structure shown below.

H2N OH

Which statement about 3-aminoisobutyric acid is not true?

A It migrates to the cathode at pH 10.


B It exists predominately as a zwitterion at pH 7.
C It reacts with ethanoyl chloride to form an amide.
D It exists as a crystalline solid at room temperature.

trendyline
DHS 2016 9647/01 [Turn over
282 trendyline
13

Section B
For each question, one or more of the three numbered statements 1 to 3 may be correct.

Decide whether each of the statements is or is not correct (you may find it helpful to put a tick
against the statements which you consider to be correct).

The responses A to D should be selected on the basis of


A B C D
1, 2 and 3 1 and 2 only 2 and 3 only 1 only
are correct are correct are correct is correct

No other combination of statements is used as a correct response.

31 Which statement about ethanoic acid can be deduced from the sketch below?

Ka of ethanoic
acid

T / C

1 The ionic dissociation of ethanoic acid is an endothermic process.


2 The pH of the ethanoic acid decreases with increasing temperature.
3 At T = 40 C, pH of ethanoic acid = 14 pOH

trendyline
DHS 2016 9647/01 [Turn over
283 trendyline
14

The responses A to D should be selected on the basis of


A B C D
1, 2 and 3 1 and 2 only 2 and 3 only 1 only
are correct are correct are correct is correct

No other combination of statements is used as a correct response.

32 The following graph represents how the solubility of a sparingly soluble salt lead(II) chloride,
PbCl 2 , changes upon addition of solid potassium chloride.

solubility of PbCl2
Z

mass of KCl added

Which statement is correct?

1 The K sp value decreases along XY and then increases along YZ.


2 The change in solubility along XY is caused by common ion effect.

3 The change in solubility along YZ is caused by the formation of a complex between


Pb2+ and Cl ions.

33 In a chemical reaction, P reacts with Q to form R. The rate equation is found to be


rate = k [P] [Q].

The energy profile diagram for the reaction is as shown.

energy

progress of reaction

Which is the likely overall equation of the above reaction?

1
2
3
trendyline
R
2P + QR
R
P + 2QR
R
P + QR

DHS 2016 9647/01 [Turn over


284 trendyline
15

The responses A to D should be selected on the basis of


A B C D
1, 2 and 3 1 and 2 only 2 and 3 only 1 only
are correct are correct are correct is correct

No other combination of statements is used as a correct response.

34 Use of the Data Booklet is relevant to this question.

The half-cells for four metals: Mg, X, Y and Z were in turn connected in pairs and the
potential difference was recorded at room temperature.

salt bridge
metal metal

1 moldm3 of the metal 1 moldm3 of the metal


cation solution cation solution

The results obtained are as shown in the table below.

positive electrode negative electrode


o
E /V
X Mg +2.10
Y Mg +2.72
Mg Z +0.33

Which of the following statements is true?

1 Yis likely to be copper metal.


2 X is stronger than Y in terms of reducing power.
3 Ease of oxidation of metals: Z<X<Y.

trendyline
DHS 2016 9647/01 [Turn over
285 trendyline
16

The responses A to D should be selected on the basis of


A B C D
1, 2 and 3 1 and 2 only 2 and 3 only 1 only
are correct are correct are correct is correct

No other combination of statements is used as a correct response.

35 Solid P dissolves in aqueous chlorine to give a solution Q which gives precipitate R on adding
excess aqueous sodium thiosulfate followed by aqueous lead(II) nitrate.

Which combination could agree with the procedure above?

Identity of P Colour of Q Colour of R


1 NaI brown mixture of white and bright yellow
2 NaBr orange white only
3 NaCl colourless white only

36 Consider the two reactions of Cu2+(aq) below.

[Cu(H 2 O) 6 ]2+ + EDTA4 [Cu(EDTA)]2 + 6H 2 O (equilibrium 1)

[Cu(H 2 O) 6 ]2+ + 4NH 3 [Cu(NH 3 ) 4 (H 2 O) 2 ]2+ + 4H 2 O (equilibrium 2)

What can you deduce from the equilibria above?

1 The entropy change of equilibrium 1 is more positive than that of equilibrium 2.


2 [Cu(NH 3 ) 4 (H 2 O) 2 ]2+ in equilibrium 2 has two possible isomers:

2+ 2+
H2O NH 3
H3N NH 3 H2O NH 3
Cu Cu
H3N NH 3 H2O NH 3

H2O NH 3

3 EDTA4 is a stronger ligand than NH 3.

trendyline
DHS 2016 9647/01 [Turn over
286 trendyline
17

The responses A to D should be selected on the basis of


A B C D
1, 2 and 3 1 and 2 only 2 and 3 only 1 only
are correct are correct are correct is correct

No other combination of statements is used as a correct response.

37 Compounds L and M are precursors to the synthesis of some analgesic drugs.

O CH3
OH
OH OH
CHO
O
H3C

L M

Which statement is true?

1 Only L reacts with KBr in concentrated H 2 SO 4 to give a bromine-based compound


under heat.
2 Both L and M react with HCN to form a product with at least one chiral carbon atom.

3 L is expected to be more acidic than M.

38 Which of the following will yield an organic compound containing deuterium? (D = 2H)

O DCl (D2O)
1 C NH2
heat
O O
dil D2SO4
2 C O C CH3
D2O

O
LiAlD4
3 C OH

trendyline
DHS 2016 9647/01 [Turn over
287 trendyline
18

The responses A to D should be selected on the basis of


A B C D
1, 2 and 3 1 and 2 only 2 and 3 only 1 only
are correct are correct are correct is correct

No other combination of statements is used as a correct response.

39 Which reagent can be used to distinguish catechol from benzene 1,2 dicarboxylic acid?

OH
OH

catechol

1 Sodium bicarbonate solution


2 Neutral iron(III) chloride solution
3 Aqueous bromine solution

40 Aspartame is an artificial sweetener used as a sugar substitute in some foods and


beverages. The structure is shown below:

O OCH3
N
H
OH NH2 O

Which deduction about the reactions of aspartame can be made from this structure?

1 It undergoes hydrolysis to form two -amino acids.


2 It reacts with 2,4-dinitrophenylhydrazine to give an orange precipitate.
3 The solution remains orange when heated with acidified potassium dichromate(VI).

trendyline
DHS 2016 9647/01 [Turn over
288 trendyline
2016 DHS H2 CHEMISTRY YEAR 6PRELIMS
Paper 1 MCQ Answers

1 2 3 4 5 6 7 8 9 10 11 12 13 14 15 16 17 18 19 20
D B B D C D C C D C A A D D C C B D A B

21 22 23 24 25 26 27 28 29 30 31 32 33 34 35 36 37 38 39 40
D C D A C B C A B A B C B B A B A A A D

trendylin
ne
DHS 2016 9647/01 [Turn over
289 trendyline
2

Name: Index Number: Class:

DUNMAN HIGH SCHOOL


Preliminary Examination 2016
Year 6

H2 CHEMISTRY 9647/02
Paper 2 Structured 16 September 2016
2 hours

Additional Materials: Data Booklet

INSTRUCTIONS TO CANDIDATES
1 Write your name, index number and class on this cover page.
2 Answer allquestions.
3 Write your answers in the spaces provided on the question paper.
4 A Data Booklet is provided.
5 The number of marks is given in brackets [ ] at the end of each question or part question.
6 You may use a calculator.

trendyline
tre
end
e
en
n
nd
ddylin
ylin
y lin
ne
ne
Question
1 2 3 4 5 Total %
No.

Marks
12 15
5 15 15
1 15
1 [[72]

This question paper consists of16 printed pages and 2blank pages.
DHS 2016 [Turn over
290 trendyline
2

Answer all questions in the spaces provided.

1 Planning (P)

The reaction between peroxodisulfate ions, S 2 O 8 2, and ethanedioate ions, C 2 O 4 2, is


slow and can be catalysed by Cu2+ ions.

S 2 O 8 2+ C 2 O 4 2 2SO 4 2 + 2CO 2

To determine the rate law of this reaction, it is necessary to selectively vary the
concentrations of S 2 O 8 2 and C 2 O 4 2 ions and determine how the rate of reaction
responds to these changes. The concentrations of the reactants are varied in such a
way that one is in excess compared to the other in each experiment.

A suitable end point (the point at which the final time reading is made) for the
experiments will be when the reaction produced the same volume of CO 2 gas. With
the measured reaction time, relative rate of the reaction can be determined.

You are required to write a plan to determine the rate law of the reaction between
S 2 O 8 2 and C 2 O 4 2 ions.

(a) Suggest an explanation why this reaction is slow when performed in the absence
of a catalyst.

[1]

(b) You may assume that you are provided with

1.00 moldm3peroxodisulfate ions


1.00 moldm3ethanedioate ions
10 cm3 aqueous copper (II) ions
the equipment and materials normally found in a school laboratory.

It can be assumed that a reactant is in excess if its volume is at least five times
the volume of the other reactant used.

Your plan should include the following:

quantities of reactants and condition you would use in four differentreaction


mixtures
the measurements you would take
an outline of how one of the reaction mixtures is prepared
brief, but specific details of how the results would then be used to determine
the rate law of reaction

trendyline
DHS 2016 9647/02 [Turn over
291 trendyline
3

trendyline
DHS 2016 9647/02 [Turn over
292 trendyline
4

[8]

(c) Temperature can also affect the rate of reaction and the activation energy, E a , of
this reaction can be found by repeating the experiments at several different
temperatures.

Using equation (1) and by means of a suitable graph, the activation energy of
the reaction can be determined.

Ea 1
ln (relative rate) = ( ) + ln (constant) (1)
R T

Briefly describe, with specific experimental details, of how the activation energy,
E a , of the reaction could be determined.

[3]

trendyline
[Total: 12]

DHS 2016 9647/02 [Turn over


293 trendyline
5

2 (a) A leadacid car battery is a reversible cell that consists of a lead anode and a
grid of lead coated with lead (IV) oxide as the cathode. The electrolyte is a 30%
solution of sulfuric acid. When the cell supplies electric current to power the car,
the process is known as discharging. The reversible process is thus known as
charging.
The overall reaction that takes place in the leadacid car battery is as follows:
discharging
Pb(s) + PbO2(s) + 2H2SO4(aq) 2PbSO4(s) + 2H2O(l)
charging

(i) Write the halfequations, with state symbols, for the reactions that take
place at the anode and the cathode during the discharging process.
Hence, state the polarity at the two electrodes.

Halfequations Polarity
Anode

Cathode

[3]

(ii) When the lead and lead(IV) oxide have been converted to lead(II) sulfate,
the cell can no longer give a current and the battery becomes flat. As the
car moves, the generator then charges the battery. When the leadacid
battery is fully charged, the sulfuric acid has a relative density of about
1.275.

Predict and explain what happens to the relative density of the sulfuric acid
during the discharging process.

[2]

(iii) Overcharging the leadacid battery with high voltages causes hydrogen
and one other gas to form. In addition, the electrolyte level is observed to
have dropped.
Explain briefly what happens during overcharging and hence, suggest the
identity of the other gas that is formed.

trendyline [2]

DHS 2016 9647/02 [Turn over


294 trendyline
6

(iv) A simple rechargeable cell similar to that of leadacid car battery may be
constructed by dipping two lead electrodes into aqueous lead(II) nitrate
and passing a current for a few minutes. During the charging process,
lead(IV) oxide is deposited on one of the electrodes. By reference to the
Data Booklet, calculate the value of E when the cell discharges.

[2]

(v) The voltage of a typical leadacid battery is 2.0V. Explain the difference in
the voltage and the E calculated in (a)(iv), based on the concentration of
ions.

[2]

(b) (i) Pb2+ ions are often used in qualitative analysis to identify halide ions as
they form insoluble precipitates. However, it is not an ideal test to
distinguish between Cl and Br ions as both ions form white precipitate
with Pb2+ ions.

Suggest another simple chemical test involving precipitation that could be


used to distinguish between Cl and Br ions, stating clearly the steps and
reagents involved and the expected observations.

[2]

trendyline
DHS 2016 9647/02 [Turn over
295 trendyline
7

(ii) With reference to your answer in (b)(i), explain the chemistry involved and
writing equations where appropriate.

[2]

[Total: 15]

3 Cobalt and vanadium are transition elements with more than one oxidation state and
many of its compounds are coloured.

(a) When a few drops of aqueous ammonia is added to a testtube containing


Co3+(aq), a brown precipitate, X is observed. The chemical formula of X does
not contain water molecules. The precipitate then dissolves in excess aqueous
ammonia to give a cationY which has a coordination number of 6.

(i) Complete the electronic configuration of cobalt (III) ions.

Co3+: 1s22s22p6
[1]

(ii) Suggest the identities of X and Y.

X:
Y:
[2]

DHS 2016 9647/02 [Turn over


296 trendyline
8

(iii) With the use of appropriate ionic equations, explain the formation of X and
Y.

[3]

(b) When concentrated hydrochloric acid is added to Co2+(aq), the colour changes
from pink to blue. When water is added to the blue solution, it returns to the pink
colour.

The chemistry of Co2+(aq) with concentrated hydrochloric acid closely resembles


that of Cu2+(aq).

Write a balanced ionic equation to account for the observations. In your answer,
state the change in coordination number of cobalt ion in this reaction (if any).

[2]

(c) Vanadium is also another commercially important transition element.

(i) Explain why V2+ is coloured.

[2]

DHS 2016 9647/02 [Turn over


297 trendyline
9

(ii) V 2 O 5 is used as a catalyst to speed up the conversion of SO 2 into SO 3 in


the contact process for making sulfuric acid.

2SO 2 (g) + O 2 (g) 2SO 3 J + 197 kJ mol1

For the reaction above, explain clearly how the catalyst works.

[2]

(iii) When a 2:1 ratio of sulfur dioxide and oxygen is passed over V 2 O 5 , the
catalyst in a fixed volume steel vessel at 430 C, the percentage
conversion of SO 2 to SO 3 is 95%.

Given that K p is 7200 atm1 at 430 C, calculate the initial total pressure, in
atm, in the steel vessel.

[2]

(iv) Comment on the effect on the rate of conversion if another unreactive but
toxic gas was accidentally added to V 2 O 5 .

[1]

[Total: 15]

trendyline
DHS 2016 9647/02 [Turn over
298 trendyline
10

4 One of the factors that establishes the quality of a water supply is its degree of
hardness. Water hardness is usually reported as parts per million (ppm) of calcium
carbonate in the water sample.

mass of CaCO3
ppm CaCO 3 = x 106
mass of water

Water Supply Classification


calcium carbonate (ppm) Hardness Level
043 soft
43150 slightly hard
150300 moderately hard
300450 hard
>450 very hard

Water hardness can be determined by titrating Ca2+ ions with


ethylenediaminetetraacetic acid (represented by H 4 Y).

HOOC CH2 CH2 COOH


N CH2CH2 N
HOOC CH2 CH2 COOH

H4Y
The titration is often carried out in a buffered basic medium, where H 4 Y exists mainly
as HY3 (buffered solution).

Ca2+(aq) + HY3 DT &D<2(aq) + H+(aq) where Y = C 10 H 12 N 2 O 8

(a) A 50 g sample of hard water was titrated against the buffered solution of
concentration 0.0149 moldm3. The volume of the buffered solution required was
25.55 cm3.

(i) Calculate the mass of CaCO 3 present in the water sample.

trendyline [1]

DHS 2016 9647/02 [Turn over


299 trendyline
11

(ii) Hence calculate the ppm of CaCO 3 in the water sample and determine its
hardness level.

[2]

There are two types of water hardness, temporary and permanent. The difference is
due to the presence of different dissolved calcium compounds in the water.

Temporary hardness is caused by the presence of dissolved calcium


hydrogencarbonate, Ca(HCO 3 ) 2 . This type of hardness is called temporary hardness
because the calcium ions can be removed from the water causing it to precipitate as
CaCO 3 .

Ca2+(aq) + 2HCO3 (aq) CaCO3(s) + H2O(l) + CO2(aq) + SRVLWLYH

(b) Using the given equilibrium, suggest and explain one way how the removal of
Ca2+ from the water sample can be enhanced.

[2]

A common detergent contains sodium dodecylbenzenesulfonate, C 18 H 29 SO 3 Na. In


hard water, it is ineffective as a detergent because it reacts with calcium ions to give a
precipitate.

(C 18 H 29 SO 3 ) 2 Ca(s) Ca2+ (aq) + 2C 18 H 29 SO 3 (aq)K sp = 1.2 x 1017 mol3dm9

A typical sample of hard water has a concentration of calcium ions of


2.5 x 104moldm3.

(c) (i) Calculate the maximum concentration of C 18 H 29 SO 3 in a solution of hard


water.

trendyline [1]

DHS 2016 9647/02 [Turn over


300 trendyline
12

The manufacturers claim that the detergent contains 17.4% by mass of


C 18 H 29 SO 3 Na [M r = 348]. In hard water, 1.0 g of the detergent should be used
per dm3 of hard water.

(ii) Calculate the number of moles of C 18 H 29 SO 3 Na added to 1 dm3 of hard


water.

[1]

(iii) Using your answer in (c)(i) and (c)(ii), show that 99% of C 18 H 29 SO 3 Na in
the detergent added will precipitate out of the hard water.

[2]

(d) In order for the detergent to be used in hard water, sodium tripolyphosphate,
Na 5 P 3 O 10 , is added as a water softening agent.

The sodium tripolyphosphate softens water by complexing with calcium ions.

Equilibrium 1: Ca2+ (aq) + P 3 O 10 5 (aq) CaP 3 O 10 3 (aq)


K c = 7.7 x 108mol1 dm3

(i) Explain qualitatively why the addition of Na 5 P 3 O 10 will allow the detergent
to be used in the hard water.

[2]

DHS 2016 9647/02 [Turn over


301 trendyline
13

(ii) What is the oxidation state of phosphorus in sodium tripolyphosphate?

[1]

(iii) Suggest a likely structure for the tripolyphosphate anion.

[1]

(iv) Magnesium ions can also cause water hardness. It forms a complex with
tripolyphosphate ions as well.

State the effect, if any, of adding magnesium ions on the equilibrium


position and value of K c of Equilibrium 1 at constant temperature. Explain
your answer.

[2]

[Total: 15]

trendyline
DHS 2016 9647/02 [Turn over
302 trendyline
14

5 The male silk moth secretes juvenile hormone, C 16 H 26 O 4 , which contains an


unknown number of epoxide rings. Epoxide rings are readily hydrolysed by cold
aqueous acid.

OH OH
1 O 3 1 3
R R H+ R R
C C C C
2 4 2 4
R R R R

The reactions that juvenile hormone and the resulting derivatives undergo are shown
in the table below.

Reactant Reagent Result


hydrogen in the presence
S, C 16 H 28 O 4 , formed.
of a palladium catalyst
Juvenile hormone T, C 16 H 30 O 6 , containing four
cold dilute hydrochloric
alcoholic groups and three
acid
chiral centres is formed.
warmed with aqueous
Methanol and U, C 15 H 28 O 6
T sodium hydroxide and then
are formed.
acidified

V, C 15 H 24 O 6 , which has two


U Mild oxidation
alcoholic groups, is formed.

hot concentrated acidified Two moles of carbon dioxide


V KMnO 4 and one mole of W,
C 13 H 22 O 5 are formed.

2,4
dinitrophenylhydrazine orange precipitate formed.
W reagent

alkaline aqueous iodine yellow precipitate formed.

In addition, it is known that juvenile hormone has a continuous 12carbon long


backbone. There are three methyl substituents on the skeletal backbone of juvenile
hormone and that each methyl substituent is five carbon atoms apart from the next
methyl substituent.

(a) Give the products that will be formed when methylethyl ether, CH 3 OCH 2 CH 3
undergoes hydrolysis with aqueous acid.

[1]

(b) State and explain the number of epoxide ring(s) in juvenile hormone.

trendyline
[2]

DHS 2016 9647/02 [Turn over


303 trendyline
15

(c) State the type of reaction that T has undergone and the functional group that it
contains other than the four alcoholic groups.

[1]

(d) Classify the two alcoholic groups in V into either primary, secondary and tertiary
alcohols. State the number of each alcohol and explain how you arrive at your
conclusion.

[2]

(e) V produced an intermediate upon reacting with hot concentrated acidified


KMnO 4 which eventually produced carbon dioxide. Draw the displayed formula
of this intermediate.

[1]

(f) State the type of reaction between W and 2,4 dinitrophenylhydrazine reagent.
State the deduction about W from the result of W reacting with aqueous alkaline
iodine.

[2]

DHS 2016 9647/02 [Turn over


304 trendyline
16

(g) Suggest the structures for compounds S to W and for juvenile hormone.

Compound Structure

Juvenile
hormone

[6]

trendyline
[Total: 15]

DHS 2016 9647/02 [Turn over


305 trendyline
17

BLANK PAGE

trendyline
DHS 2016 9647/02 [Turn over
306 trendyline
18

BLANK PAGE

trendyline
DHS 2016 9647/02 [Turn over
307 trendyline
Answer all questions in the spaces provided.

1 (a)
Repulsion between two negatively charged ions causes the activation energy to
be high.
[1]

(b)

Preparation of reaction mixture

1. Using a 50.0 cm3 measuring cylinder, add 25.0 cm3 of peroxodisulfate ions
into a 250 cm3 conical flask.
2. Using a 10.0 cm3 measuring cylinder, add 2.0 cm3 of aqueous copper (II)
ions into the same conical flask.
3. Using a 50.0 cm3 measuring cylinder, add 45.5 cm3 of deionised water into
the conical flask.
4. Using a 10.0 cm3 measuring cylinder, measure out 2.5 cm3 of ethanedioate
ions.
5. Transfer the ethanedioate ions into the conical flask and insert a rubber bung
into the conical flask. The rubber bung has a delivery tube connected to
rubber tubing with the other end of the tubing inserted into an inverted
burette filled with water.
6. Start the stopwatch immediately and gently swirl the conical flask
continuously.
7. Monitor the water level in the burette and stop the stopwatch when
40.00 cm3 of CO 2 is produced. Record the time taken.
8. Repeat step 1 to 7 using the following volumes of reactants, copper (II) ions
and deionised water shown in experiment 2 to 4, at the same temperature.

Experiment Volume of Volume of Volume of Volume of


S 2 O 8 2 / cm3 C 2 O 4 2 / cm3 Cu2+ / cm3 water / cm3
1 25.0 2.5 2.0 45.5
2 25.0 5.0 2.0 43.0
3 2.5 25.0 2.0 45.5
4 5.0 25.0 2.0 43.0

Treatment of results

1. Relative rates (= 1/t) of expt 1 to 4 are calculated. Since total volume of


mixture is constant in expt 1 to 4, concentration of reactant is proportional to
its volume used.
2. Compare the relative rates of expt 1 to 2 and 3 to 4, to find the order of
reaction with respect to (w.r.t) C 2 O 4 2 and S 2 O 8 2 respectively. If volume of
C 2 O 4 2 doubles and rate remains the same, it is zero order w.r.t C 2 O 4 2. If
volume of C 2 O 4 2 doubles, and rate remains doubles, it is first order w.r.t
C 2 O 4 2. If volume of C 2 O 4 2 doubles and rate quadruples, it is second order
w.r.t C 2 O 4 2. The same applies for S 2 O 8 2.
3. The rate law of reaction can then be found, rate = k[S 2 O 8 2]n[ C 2 O 4 2]m,

trendyline
where
here n and m are the orders of reaction r t S 2 O 8 2 and
w.r.t to
C 2 O 4 2respectively.

[8]

DHS 2016 9647/02 [Turn over


308 trendyline
2

(c)

1. Repeat one of the experiments (e.g. expt 2) at two (at least) different
temperatures, other than that for experiment 1 to 4 above. This can be done
by immersing conical flasks in water bath maintained at different constant
temperatures e.g. 40 C and 60 C.
2. Using the results in experiment 2 and that of the two further experiments,
relative rate, followed by ln (relative rate) for each expt is calculated.
1
3. A graph of ln (relative rate) against is plotted. The gradient of the best fit
T
E
line is then determined, where gradient = a . E a is given by gradient x
R
R.
[3]

[Total: 12]

2 (a) (i) Halfequations Polarity


2
Anode Pb(s) + SO 4 (aq) PbSO 4 (s) + 2e

PbO 2 (s) + 4H+(aq) + SO 4 2(aq) + 2e


Cathode +
PbSO 4 (s) + 2H 2 O(l)
[3]

(ii)
The relative density of the sulfuric acid will decrease.

During discharging, as sulfuric acid is used up to produce PbSO 4 , the


concentration of sulfuric acid will decrease, resulting in a decrease in
density.
[2]

(iii)

Overcharging will result in electrolysis of water.


Identity of other gas: Oxygen
[2]

(iv)
E = 1.47 (0.13) = +1.60 V

[2]

t d li
trendyline
(v)

Lead acid battery has a higher voltage as PbSO 4 form


Leadacid formed is insoluble,
which resulted in a lower concentration of Pb2++ in
n the electrolyte,
e thus
driving the forward reaction.
[2]

DHS 2016 9647/02 [Turn over


309 trendyline
3

(b) (i)

To 1 cm3 of halide ions, add AgNO 3 (aq) dropwise. Then add aqueous NH 3
in excess.

Cl ions form white ppt with AgNO 3 which is soluble in excess NH 3 (aq).

Br ions form cream ppt with AgNO 3 which is insoluble in excess NH 3 (aq)
[2]

(ii)

Ag+ (aq) + X (aq) AgX (s), where X is Cl and Br


Both halide ions form insoluble ppt with Ag+.

Ag+ (aq) + 2NH 3 (aq) [Ag(NH 3 ) 2 ]+ (aq)


Ag+ ions form soluble diammine complex which decreases the [Ag+],
resulting in a corresponding decrease in ionic product (i.e. I.P = [Ag+][X])
for both halide ions.
As K sp value of AgCl is much larger than that of AgBr, the ionic product of
AgCl will fall below its Ksp but not for AgBr.
[2]

[Total: 15]

3 (a) (i)

Co3+: 1s22s22p63s23p63d6
[1]

(ii) Suggest the identities of X and Y.

X: Co(OH) 3

Y: [Co(NH 3 ) 6 ]3+
[2]

(iii)

[Co(H 2 O) 6 ]3+ + 3OH Co(OH) 3 + 6H 2 O (1)

When aqueous ammonia is added in excess, a ligand exchange reaction


occurs. The stronger NH 3 ligands replace weaker H 2 O ligands in the
[Co(H 2 O) 6 ]3+ ions to form [Co(NH 3 ) 6 ]3+ complex.

[Co(H 2 O) 6 ]3+(aq) + 6NH 3 (aq) [Co(NH 3 ) 6 ]3+(aq) + 6H 2 O(l) (2)

trendyline
3+
This decreases the concentration of [Co(H [C 2 O) 6 ] in solution.
s By Le
Chateliers Principle, equilibrium (1) shifts left to increase ththe concentration
of [Co(H 2 O) 6 ]3+
3
(aq). Hence the brown precipitate of Co(OH
Co(OH) 3 dissolves [1].
[3]

DHS 2016 9647/02 [Turn over


310 trendyline
4

(b)
[Co(H 2 O) 6 ]2+ + 4Cl [CoCl 4 ]2 + 6H 2 O

There is a change of coordination number from 6 to 4.


[2]

(c) (i)
The partiallyfilled d orbitals of V2+ are split into two groups of different
energy levels by H 2 O ligands. When white light shines on the complex, a d
electron undergoes dd transition and is promoted to a higher energy d
orbital. During the transition, the d electron absorbs light from the yellow
region of the visible spectrum. The colour observed is the colour of
transmitted light, which is a mixture of remaining wavelengths that have
not been absorbed.
[2]

(ii)
The reactant molecules are physically adsorbed onto the catalyst surface.
This allows for formation of weak bonds between reactants and the
surface catalyst, thus weakening the intramolecular bonds in the reactants
and helps to catalyse the reaction. After reaction, the reactant molecules
desorb from the catalyst surface.
[2]

(iii)

2SO 2 (g) + O 2 (g) 2SO 3 (g)


Initial pressure
0.6667x 0.3333x 0
/atm
Change in
0.6333x 0.3167x +0.6333x
pressure /atm
Equilibrium
0.0334x 0.0166x 0.6333x
pressure /atm

Let the initial total pressure be x atm.

K p = (0.6333x)2 / (0.0166x)(0.0334x)2 = 7200


x = (0.6333)2 / 7200(0.0166)(0.0334)2
= 3.01 atm
[2]

(iv)
Rate of conversion will slow down/decrease due to poisoning of the
catalyst.
[1]

[Total: 15]

4 (a) (i)

trendyline
trendyline
No. off moles
N H 3 required = 0.0149 x
l of HY
25 5
25.55
1000

= 0.00038070 mol

DHS 2016 9647/02 [Turn over


311 trendyline
5

No. of moles of Ca2+ present = 0.00038070 mol


No. of moles of CaCO 3 = 0.00038070 mol
Mass of CaCO 3 = 0.00038070 x [40.1 + 12 + 3(16)]

= 0.0381g (3 sf)
[1]

(ii)

0.038107
ppm CaCO 3 = x 106 = 762 (3 sf)
50
The water is very hard.
[2]

(b)

Ca2+ can be removed by heating the water sample.

By Le Chateliers Principle, the position of equilibrium will shift right to remove


the heat supplied causing a decrease in the concentration of Ca2+ ions.

OR

Ca2+ can be removed by heating the water sample.

Heating the water sample removes CO 2 (g) hence decreasing the concentration
of CO 2 (aq).

By Le Chateliers Principle, the position of equilibrium will shift right to increase


the concentration of CO 2 (aq) causing a decrease in the concentration of Ca2+
ions.
[2]

(c) (i)

[C 18 H 29 SO 3 @  [17 / 2.5 x 104)


= 2.1908 x 107 moldm3
= 2.19 x 107 moldm3
[1]
(ii)

Mass of C 18 H 29 SO 3 Na added to 1.0 dm3 = 0.174 g

0.174
No of moles of C 18 H 29 SO 3 Na added to 1.0 dm3 = = 5.00 x 104 mol
348
[1]

ttrendyline
d li
(iii)
Maximum moles of C 18 H 29 SO 3 dissolved dm3 = 2.19
solved in 1.0 d 2.1908 x 107 mol
2.1

No of moles of C 18 H 29 SO 3 precipitated out = 5.00 x 1044 2.1908 x 107


=4 9978 x 1044 mol
4.9978

DHS 2016 9647/02 [Turn over


312 trendyline
6

4.9978 x 104
Percentage of C 18 H 29 SO 3 precipitated = x 100%
5.00 x 104

= 99% (shown)
[2]

(d) (i)
Ca2+(aq) + P 3 O 10 5(aq) CaP 3 O 10 3(aq) (1)

Ca2+ (aq) + 2C 18 H 29 SO 3 (aq) (C 18 H 29 SO 3 ) 2 Ca (s) (2)

P 3 O 10 5 form complexes with Ca2+ causing a decrease in the concentration


of Ca2+.

By Le Chaterliers Principle, the equilibrium position of (2) shifts left to


increase the concentration of Ca2+. Hence reducing the amount of
(C 18 H 29 SO 3 ) 2 Ca formed.

Hence, preventing the formation of precipitate when detergent is added


into hard water.
[2]

(ii)

+5
[1]

(iii)

[1]

(iv)
Concentration of P 3 O 10 5 decreases as it complexes with the added Mg2+
to form MgP 3 O 10 3.

By Le Chateliers Principle, the equilibrium position of (1) will shift left to


increase the concentration of P 3 O 10 5.

There is no change to the value of K c as temperature is kept constant.


[2]

trendyline
[Total: 15]

5 (a)
CH 3 OH an
and CH 3 CH 2 OH.
O
[1]

DHS 2016 9647/02 [Turn over


313 trendyline
7

(b)
Two epoxide rings.

Two alcoholic groups are produced for every epoxide ring that will be
hydrolysed. Since there are four alcoholic groups formed after juvenile
hormone undergoes hydrolysis, this implies that there are two epoxide rings in
juvenile hormone.
[2]

(c) State the type of reaction that T has undergone and the functional group that it
contains other than the four alcoholic groups.
T undergoes basic hydrolysis to give methanol and U. T is likely to contain an
ester.
[1]
(d)
Two tertiary alcohol groups.

T has four alcohol groups which were retained in U even after basic hydrolysis.
Since V has only two alcohol groups left after mild oxidation, this implies that the
two alcohol group must be tertiary in order to resist oxidation.
[2]

(e)
O O
C C
H O O H
[1]

(f)

OH
O
H3C C CH3
C
CH3
Condensation. W contains eitherH3C or H

[2]

trendyline
DHS 2016 9647/02 [Turn over
314 trendyline
8

(g)

O
Juvenile hormone
O O

O
S
O O O

O
T
HO OH O
OH OH

O
U
HO OH OH
OH OH

O
V
HO OH OH
O O

W
HO OH O
O O

[6]
[Total: 15]

trendyline
DHS 2016 9647/02 [Turn over
315 trendyline
Name: Index Number: Class:

DUNMAN HIGH SCHOOL


Preliminary Examination 2016
Year 6

H2 CHEMISTRY 9647/03
Paper 3 Free Response 20 September 2016
2 hours

Additional Materials: Data Booklet


Writing Papers
Graph Paper

INSTRUCTIONS TO CANDIDATES
Write your name, index number and class on this question paper and on the Cover Sheet provided.
Write in dark blue or black pen on both sides of the paper.
You may use a soft pencil for any diagrams, graphs or rough working.
Do not use staples, paper clips, highlighters, glue or correction fluid.

Answer any four questions.


Start each question on a fresh sheet of paper.
A Data Booklet is provided.
You are reminded of the need for good English and clear presentation in your answers.

The number of marks is given in brackets [ ] at the end of each question or part question.
At the end of the examination:
Fasten all work securely together with the Cover Sheet on top.
Hand in the question paper separately.
The total marks for this paper is 80 marks.

trendyline
This question paper consists of15 printed pages and 1blank page.
DHS 2016 [Turn over
316 trendyline
2

Answer any four questions.

1 Blue #1 (Na 2 C 37 H 34 N 2 S 3 O 9 ) is a synthetic food colouring often found in ice cream.

Na O S SO 3 Na
3

+
N N
H 5C 2 C 2H 5

SO 3

(a) One of the main compounds used to produce Blue #1 is benzenesulfonic acid, which is
produced by heating benzene under reflux with concentrated sulfuric acid for several
hours.

SO 3H

+ +
H2SO4 H2O

(i) The first step of the mechanism involves the protonation of one molecule of sulfuric
acid by another and the loss of a molecule of water.
Name the type of reaction and describe the mechanism. In your answer, show
relevant lone pairs and charges, and use curly arrows to indicate the movement of
electrons.
[4]

(ii) Benzenesulfonic acid is a useful starting material as it can be easily converted to


phenol by heating with NaOH(aq) at 300C followed by acidification. It also has
similar chemical reactivity as nitrobenzene.

SO 3H
ONa O H

NaOH (aq)
HCl (aq)
3000C

trendyline
DHS 2016 9647/03 [Turn over
317 trendyline
3

Compound A can be synthesised from either benzenesulfonic acid or


ethylbenzene in not more than 4 steps.

The first step of the synthesis using either benzenesulfonic acid or ethyl benzene is
as shown below in Scheme 1 and 2 respectively:

Scheme 1:
SO H SO 3
H
3

CH2CH3
CH3CH2Cl / AlCl3

Scheme 2:
CH2CH3
CH2CH3
SO 3 H

conc. H2SO4
heat

Identify the correct scheme that will give compound A and copy the synthetic
scheme in your answer.

Using information in (a)(ii), suggest the next three steps to synthesise compound
A. Show all intermediates, reagents and conditions clearly in your synthetic route.

[4]

trendyline
DHS 2016 9647/03 [Turn over
318 trendyline
4

(b) Blue #1 can be oxidised by household bleach to form colourless products, as


represented by the equation below.
Na 2 C 37 H 34 N 2 S 3 O 9 + bleach  products
blue colourless colourless

To study the kinetics of the reaction, a student used an UV spectrophotometer to study


the absorbance of Blue #1 over time during the bleaching process.

In experiment 1, the student mixes 3.0 cm3 of 0.1 moldm3 Blue #1, 1.0 cm3 of water and
0.5 cm3 of 3.5 moldm3 bleach together. The resultsof experiment 1 are as shown below.

Time (s) 15 30 45 60 75 90 105 120 135


Absorbance 0.495 0.424 0.371 0.328 0.285 0.251 0.218 0.191 0.170

(i) Explain why bleach is used in large excess as compared to that of Blue #1.
[1]

(ii) Given that absorbance measured varies linearly with the concentration of
Blue #1, use a suitable graphical method to determine the order of reaction
with respect to the blue food colouring, Blue #1.
[3]

(iii) The student subsequently conducted a few more experiments using other
volumes of Blue #1 and bleach. The table below shows the results obtained.
Experiment Volume of Volume of Volume of
Rate
Blue #1 distilled water bleach
constant
/ cm3 / cm3 / cm3
1 3.0 1.0 0.5
2 4.0 0.0 0.5 0.00882
3 3.0 0.5 1.0 0.01698

Using results from (b)(ii), determine the rate constant for experiment 1. Hence,
deduce the overall order of reaction, showing your reasoning clearly.
[3]

(c) (i) Household bleach is also often known as chlorine water. The active chemical
responsible for the bleaching action in household bleach is the anion, OCl,
which is easily formed by the disproportionation of chlorine in water.
Write a balanced equation between chlorine and water, and hence, suggest
the identity of the other compound formed.
[1]

(ii) Chlorine reacts with Period 3 elements, Mg and Si, to give chlorides. The two
chlorides differ in their reaction with water. Describe their differences, and
explain them in terms of the different structures and types of chemical

trendyline
bonding. Write an equation for the reaction,
reaction if any.
[4]

[Total: 20]

DHS 2016 9647/03 [Turn over


319 trendyline
5

2 Vanillin and vanillyl alcohol (derived from vanillin) are both used as flavouring, usually in
sweet foods such as chocolate. Vanillyl alcohol is commonly synthesised via the reduction of
vanillin using sodium borohydride, as shown below.

O H OH

4 + NaBH4 (aq) + 4H2O (l) 4 + H3BO3 (aq) + NaOH (aq)


O O
OH OH
vanillin vanillyl alcohol

(a) (i) Draw a dotandcross diagram to show the bonding in sodium borohydride.
[1]

(ii) Suggest why sodium borohydride is used in this reduction instead of lithium aluminium
hydride.
[1]

(iii) Given that the melting points of vanillin and vanillyl alcohol are 355 K and
388 K respectively, predict and explain the sign of the entropy change of reaction at
360 K.
[1]

(iv) A higher temperature leads to a faster rate of reaction. However, thermodynamic factors
must also be taken into consideration when considering appropriate conditions for a
reaction.

Explainhow higher temperatures will affect thethermodynamic outcome.


[2]

trendyline
DHS 2016 9647/03 [Turn over
320 trendyline
6

(b) The procedure for the reduction of vanillin to vanillyl alcohol is as follows:

1. Place 2 g of vanillin in a 25 cm3 round bottom flask followed by 4 cm3 ethanol. Add
a magnetic stirrer, clamp the flask above a stir plate and commence stirring at room
temperature to dissolve vanillin.
2. After the vanillin dissolves, add an ice bath under the flask to cool the solution.

3. In a separate reaction vial, dissolve 0.5 g of NaBH 4 in 3.8 cm3 of


1 moldm3NaOH solution.
4. Using a glass pipette, slowly add the NaBH 4 solution dropwise to the vanillin
solution over a period of 10 minutes.
5. Only after the addition is complete, remove the ice bath. This is because NaBH 4
decomposes at high temperatures.
6. Stir the resulting mixture for 10 minutes at room temperature.
7. Add 6 moldm3HCldropwiseuntil the evolution of hydrogen gas stops.
8. Stir to allow the product to precipitate from solution.

(i) State a reason why sodium borohydride must be added dropwise to the reaction mixture
(in Step 4).
[1]

(ii) In Step 7, hydrochloric acid was added to react with excess sodium borohydride present
in the reaction mixture. Write an equation for this reaction, given that boric acid (H 3 BO 3 )
is one of the products formed.
[1]

(iii) State one way you could modify the procedure to increase the isolated yield.
[1]

(iv) A preliminary way to determine whether the reaction has gone to completion is to draw
out aliquots of the reaction mixture and test for the presence of vanillin in it.

Describe a simple chemical test to test for the presence of vanillin. Give the expected
observations and write a balanced equation for the reaction with vanillin.
[2]

trendyline
DHS 2016 9647/03 [Turn over
321 trendyline
7

(c) Compound W, an isomer of V, can also be used as a starting reagent to synthesise vanillic
acid and other organic compounds shown in the reaction scheme below.

W Step 1 Step 2 Br2(aq) Y


X (Mr = 339.8)
(C10H12O2) HBr(g)
O
hot OH
acidified
KMnO4

O OH O O

Step 3 Step 4
Z
O O
OH HO
vanillic acid

(i) Draw the structures of compounds W, X and Y, and state the reagents and conditions
for Steps 2 and 4.
[4]

(ii) Vanillic acid (4hydroxy3methoxybenzoic acid) has an isomer,


2hydroxy3methoxybenzoic acid.

State which of the two isomers has a higher boiling point. Explain your answer.
[2]

(d) Organometallic compounds, usually a metal attached to an R group, can be used to convert
carbonyl compounds to alcohols.

A common type of organometallic compound are Grignard reagents. They have the formula
RMgX, where X is a halogen and R is an alkyl or aryl group.

O OMgX OH
dry ether H+
+ R'-MgX
R H R H R H
R' R'
carbonyl compound intermediate alcohol

trendyline
Draw the structure
ructure
ucture of the intermediate formed when vanillin reacts w
with ethylmagnesium
bromide.
[1]

DHS 2016 9647/03 [Turn over


322 trendyline
8

(e) The chemistry of lithium and its compounds differs significantly from the rest of the Group I
metals. In fact, lithium shows many similarities to magnesium exemplifying the socalled
diagonal relationship because of their positions in the periodic table.

In an experiment, a sample of solid magnesium amide, Mg(NH 2 ) 2 and lithium amide, LiNH 2
are heated separately.

Magnesium amide decomposes when heated to give magnesium nitride, Mg 3 N 2 and


ammonia gas.

(i) Write a balanced chemical equation for the decomposition of lithium amide.
[1]

(ii) With reference to Data Booklet, predict the decomposition temperature of LiNH 2 when
compared to Mg(NH 2 ) 2 , given that the ionic radius of Li+ is 0.060 nm. Explain your
answer.
[2]

[Total: 20]

trendyline
DHS 2016 9647/03 [Turn over
323 trendyline
9

3 (a) Methanal undergoes reaction with water to yield 1,1diol. The reaction is reversible and
the 1,1diol can eliminate water to regenerate methanal.

O OH
C + H2O C
H H H OH
H

(i) State the type of reaction for the above reaction.


[1]

(ii) The reaction is slow but the rate is increased by the addition of a small amount of
NaOH as catalyst. Suggest why NaOH is needed to initiate the reaction in the
mechanism.
[1]

The oxygen in water is primarily 99.8% 16O but water enriched with the heavy
isotope, 18O, is also available. When methanal is dissolved in 18Oenriched water, the
isotopic label becomes incorporated into the 1,1diol which regenerates methanal.

(iii) Using your answer to (a)(ii), suggest how 18O is incorporated into the 1,1diol.

*OH

C
H OH
H where *O = 18O
18
O-enriched 1,1-diol

[1]

(iv) The addition of water to aldehydes proceeds more rapidly than it does to ketones.
Suggest a reason why.
[1]

(b) Methanal also reacts with HCN to form 2hydroxyacetonitrile, HCH(OH)CN.


The 2hydroxyacetonitrile formed can be further converted to 2hydroxyethanoic acid,
as shown below.
HCH(OH)CN HCH(OH)COOH

(i) Explain whether 2hydroxyethanoic acid has a larger or smaller K a than ethanoic
acid.
[2]

trendyline
Phenol an
and 22hydroxyethanoic
hydroxyethanoic acid are weak Br
B n
nsted acids with pK a values of 9.80
and 3.86
6 respectively.

re of 20.0 cm3 of phenol and 2


ure
A mixture 2hydroxyethanoic
hydroxyethanoic acid was
wa titrated against a
3
solution
n of 1 moldm of sodium hydroxide.
xide Only 1 to 2 drops of p
phenolphthalein and
bromothymol blue indicators are used in this titration.

DHS 2016 9647/03 [Turn over


324 trendyline
10

It was found that the first colour change occurs at 17.20 cm3 and the second colour
change occurs at 26.80 cm3 of sodium hydroxide used.

A sketch (not drawn to scale) of the pH titration curve is shown below.

pH

volume of NaOH
8.60 added / cm3

(ii) Both phenolphthalein and bromothymol blue are weak acids. State the effect on
the volume of sodium hydroxide used for titration, if both indicators are added in
larger amount.
[1]

(iii) State the value of point X.


[1]

(iv) Write an equation to illustrate how the species found in point Y can maintain the
pH of a solution when a small amount of H+(aq) is added.
[1]

(v) Calculate the concentration of phenol and 2hydroxyethanoic acid in the original
mixture.
[2]

(c) The Strecker synthesis is a route to preparing amino acids. Methanal can be used to
synthesise glycine, 2aminoethanoic acid, in this way.

O NH NH2 H2N O
NH3 KCN, H2O H3O+
H
H2O H H
H H H H CN H OH
imine

The amino acid shown below is glutamic acid.

trendyline
dyy H2N
H
C
CH
COO
COOH
H2CH2COOH

DHS 2016 9647/03 [Turn over


325 trendyline
11

(i) Identify the aldehydecontaining compound and the imine intermediate which are
needed to synthesise glutamic acid via the Strecker synthesis.
[2]

(ii) Suggest, with a reason, if there is any difference in the optical activity of a sample
of glutamic acid synthesised by Streckers method and that of natural occurring
glutamic acid.
[1]

(d) Glycine and glutamic acid are among the many amino acids which constitute the long
amino acid sequence of protein kinase.Protein kinaseis an enzyme whichplays a major
role in protein regulation. It modifies other proteins by chemically adding phosphate
groups to them. This process is called phosphorylation.

(i) In phosphorylation, a phosphate group (PO 4 3) from adenosine triphosphate


(ATP) is transferred to and replaces a free hydroxyl group of amino acid. Shown
below are the structures of three amino acids and phosphate group.

serine (ser) glycine (gly) glutamic acid (glu)


H H H
H2N C COOH H2N C COOH H2N C COOH
CH2OH H CH2CH2COOH

P
O O
O
phosphate group (PO 4 3):

Draw the structure of the tripeptide, serglyglu, after phosphorylation.


[2]

(ii) The secondary structure of protein kinase consists of segments of -helix.


Describe the bonding which holds the -helix in place.
[2]

(iii) The protein kinase is destroyed during the process of denaturation. Explain how
this can occur by heating.
[2]

[Total: 20]

trendyline
DHS 2016 9647/03 [Turn over
326 trendyline
12

4 In 1849, the German chemist Adolf Kolbe reported his investigations into the electrolysis of
aqueous solutions of some carboxylate anions, using inert platinum electrodes. The Kolbe

electrolysis of trifluoroacetate (CF 3 CO 2 ), for example, is known to produce trifluoromethyl
radical (CF 3 , M r = 69) and a gas at the anode.

(a) Suggest the identity of the gas and hence construct a half-equation for the reaction at
the anode.
[2]

When the Kolbe electrolysis of trifluoroacetate was carried out in the presence of alkenes
such as CH 2 =CHCO 2 CH 3 , the radical intermediate X was formed which further dimerised to
yield product Y shown below.

CO 2CH3

CF3COO
F 3C F 3C
CO 2CH3 CO 2CH3 CF 3
electrolysis
X
CO 2CH3
Y

(b) State the oxidation number of the carbonyl carbon atom in CH 2 =CHCO 2 CH 3 .
[1]

(c) Suggest a two-step mechanism for the reaction above to form Y, starting from the
reaction between trifluoromethyl radical (CF 3 ) and the alkene. Use curly arrowsto
indicate the movement of single electrons and indicate any unpaired electrons by a dot
().
[3]

The Kolbe electrolysis in (c) was performed by continuous introduction of a water/CH 3 CN


solution containing trifluoroacetate and the alkene into an electrochemical microreactor at
constant current and room temperature.

The electrochemical microreactor has a flow channel sandwiched between two platinum
electrodes as shown in the figure below.

(d) (i) Given that the flow channel has a volume of 23 L and reaction mixture was
injected at a rate of 20 L min1, calculate the time (in seconds) in which the
current was passed through the solution.

trendyline
= 103 cm3)
(1 L
[1]

(ii) The
hee purified product
produc Y ((M
M r = 310) was obtained as a colourless
colou solid weighing
401
01 g. Using your answers from
0.401 fro (
(a) d (d)(i),
and (d ca
calculate the current
n ampere) that would have been applied
(in ap to the microreactor
microreactor.
[2]

DHS 2016 9647/03 [Turn over


327 trendyline
13

(e) (i) How many stereoisomers does Y have?


[1]

(ii) Draw the isomer of Y that is not optically active, showing clearly the
stereochemistry of the molecule using wedged and dashed bonds .
[1]

(f) When the Kolbe electrolysis of trifluoroacetatewas carried out in the presence of another
alkene, CH 2 =CHCONH 2 ( M r = 71), a non-dimer product ( M r = 209) was formed instead.
Suggest the structure of the product obtained.
[1]

(g) The trifluoromethyl (CF 3 ) group is regarded as an important structural feature in many
pharmaceutically relevant molecules because it is known to enhance chemical
inertness. Suggest an explanation for its property.
[1]

(h) (i) Deduce the structure of the radical intermediate formed when Kolbe electrolysis of
difluoroacetate, CHF 2 CO 2 , was carried out in the presence of the alkene,
CH 2 =C(CH 3 )CO 2 CH 3 .
[1]

(ii) The yield of the dimer product for this electrolysis was much poorer than that for
trifluoroacetate and CH 2 =CHCO 2 CH 3 . Use your answer in (h)(i) to suggest a
reason for the difference in yield.
[2]

Some carboxylate anions, L, are also known to form 1:1 complexes with divalent metal ions,
M2+, according to the following equilibrium.

M2+ + L ML+

(i) The table below gives the values of log K for some 1:1 metal-carboxylate complexes.

metal ion acetate (CH 3 COO) chloroacetate (CH 2 ClCOO)


Cd2+ 1.30 0.99
Cu2+ 1.76 1.07
Ni2+ 0.72 0.23

Using relevant data from the table, deduce

I the relative stabilities of the metal complexes formed with the acetate ligand.
[1]

II the relative magnitude of the log Kvalues of 1:1 Cu2+-CH 3 COO and

trendyline
Cu HCOO complexes and suggest
u2+-HCOO uggest a reason for your answer.
answe
[3]

[Total: 20]

DHS 2016 9647/03 [Turn over


328 trendyline
14

5 This question is about Period 3 elements and the uses of its compounds.

(a) A sample of magnesium oxideis suspected to be contaminated with a substantial


amount of aluminium oxide.Describe a simple gravimetric method to prove that the
contamination has taken place. Include equation(s)for the chemical reaction(s) involved,
if any.
[3]

(b) Another Period 3 oxide, sulfur trioxide dissolves in water to form sulfuric acid, H 2 SO 4
which can be converted into peroxodisulfuric acid, H 2 S 2 O 8 via the two-step process
below, with chlorosulfonic acid, HSO 3 Cl, as an intermediate:

H 2 SO 4 + HCl HSO 3 Cl + H 2 O
H 2 O 2 + 2HSO 3 Cl H 2 S 2 O 8 + 2HCl

(i) The structure of HSO 3 Clis given below.

O
S
HO O
Cl

The boiling point of H 2 SO 4 is 290 C whereas that of HSO 3 Cl is 152 C. By


making reference to the structures of the two compounds, explain the difference in
the boiling points.
[2]

(ii) Draw the structure of a H 2 S 2 O 8 molecule, showing the spatial arrangement and
estimated bond angle around any one sulfur atom.The H 2 S 2 O 8 molecule contains
a OO bond.
[2]

(c) Chlorine forms an oxide, Cl 2 O 7 , with the structure O 3 ClOClO 3 and a boiling point of
82C. Its standard enthalpy change of formation is 546 kJ mol1.

(i) Write the equation which corresponds to the standard enthalpy change of
formation of Cl 2 O 7 .
[1]

(ii) Given that the bond energy of ClO bond is 269 kJ mol1 and using relevant data
from the Data Booklet, estimate the average bond energy of the Cl=O bond.
[2]

(iii) The bond energy calculated in (c)(ii) could have been more accurate if the value
of an additional enthalpy change is known. State what process this enthalpy

trendyline
g corresponds
change p to.
[1]

(iv) iven
ven that the
Given th value of the enthalpy change in (c)(iii)
(c (iii)
(c) + 5 kkJmol1, construct an
i) iss +45
nergy
ergy level diagram to calculate the enthalpy change of formation
energy ffo of gaseous
Cll 2 O 7 .
[3]

DHS 2016 9647/03 [Turn over


329 trendyline
15

(d) Aluminium oxide is used extensively in organic synthesis as a dehydrating agent.

An organic compound A, C 12 H 18 O, contains a benzene ring with 2 substituents on the


1 and 4 positions on the ring. A is optically active. When it is treated with Al 2 O 3 ,
compound B is formed. Bcan be converted into another optically active compound C by
reacting with dry gaseous HCl. When C is heated under reflux with acidified KMnO 4 ,
compound D, C 11 H 14 O 2 , is produced. Treatment of D with PCl 5 produces compound E.
When E is heated with gaseous NH 3 , a neutral compound F is formed.

Identify the six compounds A F.


[6]

[Total: 20]

trendyline
DHS 2016 9647/03 [Turn over
330 trendyline
16

BLANK PAGE

trendyline
e
DHS 2016 9647/03 [Turn over
331 trendyline
2016 Y6 H2 Chemistry Preliminary Examination Paper 3
(Answer Scheme)

1 (a) (i) Electrophilic substitution

(ii) Choose scheme 2


CH2CH3 CH2CH3
CH2CH3
+
SO 3 H O Na

conc. H2SO4 NaOH (aq)

heat 3000C

Cl2, uv

CH2CH2Cl
+
O Na

heat

(b) (i) To ensure that [bleach] remains almost constant throughout the reaction so
that the rate can be measured with respect to Blue #1 in a pseudo-order
reaction.

DHS 2016 9647/03


332 trendyline
(ii)

From graph hand-plotted,


when absorbance decreases from 0.40 to 0.20, t 78s
when absorbance decreases from 0.50 to 0.25, t 76s
Since almost constant t 77s is observed, the order of reaction wrt to [Blue#1] is 1.
OR
Reaction is first order with respect to[Blue#1].

(iii) Using t = ln 2, find value of k for experiment 1.


= 277 = 0.00900 s-1

Compare experiment 1 and 3, when the volume of bleach doubles, rate


constant almost doubles (i.e. 0.014980.00900 = 1.67 2).

The reaction is first order with respect to [bleach].

Hence, overall order of reaction is 2.

(c) (i) Cl 2 + H 2 2+&l + HOCl

(ii) MgCl 2 is an ionic compound which ionises in water to form hydrated Mg2+ and
Cl ions.
As Mg2+ has a high charge and small ionic radius / has a high charge density,
it hydrolyses slightly in water to form a weakly acidic solution of pH 6.5

[Mg(H 2 O) 6 ]2+ + H 2 O [Mg(H 2 O) 5 OH]+ + H 3 O+

SiCl 4 is a covalent compound which hydrolyses in water to form an acidic

trendyline
s
solution of pH 2.

SiC 4 + H 2 26L 2+
SiCl 26L 2+ 4 + 4HCl
4HCl

DHS 2016 9647/03


333 trendyline
2(a) (i)
H -
+ x
Na H x
B x
H
H
(ii) Sodium borohydride is a milder reducing agent and hence is less reactive
and will not react as violently with water present.

OR

Lithium aluminium hydride is a very strong reducing agent and hence is


highly reactive and will react violently with water present.

(iii)
At 360 K, vanillin is a liquid while vanillyl alcohol is a solid. There is a
decrease in the degree of disorderliness, hence the entropy change of
reaction at 360 K is negative.

(iv) Ho and So will change as temperature increases.


Go may become more/less negative as temperature increases.

(b) (i) The reaction is exothermic / large amounts of heat will be produced. If it was
added all at once and this may cause NaBH 4 to decompose.

(ii) NaBH4 + 3H2O + HCl 1D&l + H3BO3 + 4H2

(iii) Use excess NaBH 4 as it may decompose at high temperatures.


Allow reactants to stir for a longer period.

(iv) To the aliquot drawn out, add a small amount of 2,4DNPH.

An orange precipitate will be seen if vanillin is present.

H O H2N H N NH NO 2
C NH C
NO 2
+ + H2O
OCH3 OCH3 NO 2
OH NO 2 OH

trendyline
DHS 2016 9647/03
334 trendyline
(c) (i) W X Y

Br HO Br
Br OH

or
Br O Br O
O O OH OH
OH OH

Step 2: alcoholic KOH, heat under reflux


Step 4: phenol, room temperature

(ii) Vanillic acid has a higher boiling point.


Due to proximity of OH and COOH groups, 2hydroxy3
methoxybenzoic acid is capable of forming intramolecular hydrogen
bonding, thus reduces the extent of intermolecular hydrogen bonding
formed.
More energy is needed to overcome the more extensive
intermolecular hydrogen bonds in vanillic acid.

(d)
BrMgO

O
OH

(e) (i) 3LiNH 2 Li 3 N + 2NH 3

(ii)
1 2
Charge density: Li+ ( ) << Mg2+ ( )
0.06 0.065
Polarising power: Li+<< Mg2+ OR
Distortion of electron cloud of NH 2 by both cations: Li+<< Mg2+
Therefore LiNH 2 will have a higher decomposition temperature than
Mg(NH 2 ) 2 .

trendyline
DHS 2016 9647/03
335 trendyline
3 (a) (i) Nucleophilic addition

(ii) NaOH provides the OH ions which is a stronger nucleophile for the nucleophilic
attack on the carbonyl carbon.

(iii)
OH + H2O* H2O + *OH
The OH will deprotonate the 18Oenriched water to form the *OH. *OH then
attacks the methanal via nucleophilic addition reaction.

(iv) The carbonyl carbon in aldehyde is less sterically hindered than that in ketones,
hence more easily attacked by the nucleophile.
OR
The carbonyl carbon in aldehyde is more electron deficient (+) than that of
ketones as ketones have an additional electron donating alkyl group.

(b) (i) 2hydroxyethanoic acid has a largerK a as it is a stronger acid.

HCH(OH)COOH HCH(OH)COO + H+ - (1)

CH 3 COOH CH 3 COO + H+ - (2)

The electronwithdrawing OH group disperses the negative charge on the


conjugate base of 2hydroxyethanoic acid / 2hydroxyethanoate ion hence
stabilising the conjugate base/ 2hydroxyethanoate ion. The equilibrium
position of reaction (1) lies more to the right, producing more H+ ions, resulting
in 2hydroxyethanoic acid being a stronger acid.

OR

Intramolecular hydrogen bond can be formed in the 2hydroxyethanoate ion


between the carboxylate ion and the H of the alcohol group hence stabilising
the conjugate base/ 2hydroxyethanoate ion. The equilibrium position of
reaction (1) lies more to the right, producing more H+ ions resulting in 2
hydroxyethanoic acid being a stronger acid.

(ii) If both indicators are used in larger amount, a larger volume of sodium
hydroxide will be used to reach the endpoint to deprotonate the indicators
which are also weak acids.

(iii) X = 3.86

X corresponds to the pH of the buffer at maximum buffering capacity.


At maximum buffer capacity, pH = pK a

(iv)

trendyline
nd
dylin
ne
ne O

+ H+
H
OH
O

DHS 2016 9647/03


336 trendyline
(v) First endpoint corresponds to the neutralisation of 2hydroxyethanoic acid.
17.20 20
Concentration of 2hydroxyethanoic acid = ( x1)
1000 1000

= 0.860 mol dm3

Second endpoint corresponds to the neutralisation of phenol.

Volume of NaOH used for reaction with phenol = 26.80 17.20 = 9.60 cm3

9.60 20
Concentration of phenol = ( x1)
1000 1000

= 0.480 mol dm3

(c) (i) O NH
C C
H CH 2CH 2COOH H CH 2CH 2COOH

(ii) Streckers synthesis produces a racemic mixture and hence, do not display any
optical activity while naturally occurring glutamic acid is present as one of the
enantiomers and will rotate plane polarised light.

(d) (i) H O O H O O

H2N C C N CH2 C N C C O P O

CH2 H H CH2 O

O CH2

O P O C O

O O

O P O

(ii) The alpha helix is held in place due to hydrogen bonding.


Hydrogen bonding occurs between the peptide C=O group of the nth amino
acid and the peptide NH group of the (n+4)th amino acid which is in the
adjacent turn.

(iii) Heat increases thermal vibrations of the protein molecule, disrupting van der

trendyline
Waals interactions formed between uncharged R Rgroups in the
tertiary/quaternary structure.

The hydrogen bonds between polar R groups in tertiary structure are also
broken.

DHS 2016 9647/03


337 trendyline
4 (a) CO 2
CF 3 CO 2 CF 3 + CO 2 + e

(b) +3

(c)
F 3C
CO 2CH3 CO 2CH3
F 3C
X

CO 2CH3
CO 2CH3
F 3C F 3C
CF 3
CF 3
CO 2CH3
CO 2CH3
Y

(d) (i) time = 23/20 60 = 69 s

(ii) n(Y) = 0.401/310 = 0.0012935 mol


Since 2e 2CF 3 2X Y,
n(e) = 0.0025870 mol

Q = (0.0025870)(96500) = 249.65 C
I = 249.65/69 = 3.62 A

(e) (i) 3

(ii) CO 2CH 3 CO 2CH 3


H H

CH 2CF 3 CH 2CF 3

(f) CF 3

F 3C
CONH2

(g) The CF bonds in the CF 3 group are relatively strong and hence inert to chemical
reactions.

(h) (i) CH3


CHF 2
CO 2CH3

(ii) The presence of the methyl group at the C atom with the unpaired electron

trendyline
makes it more sterically hindered. It is more
mo difficult for two
tw bulkier radicals to
collide effectively to form the dimer.

(i) I CuL+> Cd
CuL L+> NiL
CdL NiL+

II log K (Cu2+-CH 3 COO) > log K (Cu2+-HCOO)

DHS 2016 9647/03


338 trendyline
From the table, the stability of the complexes is in the same order as the
availability of the lone pair of electrons on the negatively charged oxygen of
the carboxylate anions for dative bonding with the metal ion. Since the lone
pair of electrons on the negatively charged oxygen of CH 3 COO is more
available than that of HCOO for dative bonding with Cu2+, the
Cu2+-CH 3 COO complex is expected to be more stable than the Cu2+-HCOO
complex.

5 (a) 1. Weigh the original sample first.


2. Add aqueous sodium hydroxide in excess.
3. Only aluminum oxide will dissolve to form a soluble complex:
Al 2 O 3 + 2NaOH + 3H 2 O 2NaAl(OH) 4
4. The mixture is then filtered.
5. Dry the residue and reweigh the sample again.
6. If there is contamination, then there should be a difference in mass.

(b) (i) The molecular formulae suggest that the structure of chlorosulfonic acid is
derived from that of sulfuric acid, with O atom being replaced by Cl atom.
Hence the extent of hydrogen bonding is lesser between chlorosulfonic acid
molecules and hence requires less energy to overcome, giving it a lower
boiling point.

(ii) O
HO 109.5o

O O S
S O O
HO
O

(c) (i) Cl 2 (g) + 7/2 O 2 (g) Cl 2 O 7 (l)

(ii) Bond energy of ClCl = 244 kJ mol1


Bond energy of O=O = 496 kJ mol1
546 = 244 + 7/2(496) [2269 + 6(Cl=O)]
Bond energy of Cl=O = +331 kJ mol1

(iii) Vaporisation of Cl 2 O 7 (l)

(iv) Energy

Cl2(g) + 7/2 O2(g)


0

HfCl2O7(g)

trendyline
r ndylil
546 Cll2O7(g)
C

+45
45
Cll2O7(l)
C (l

DHS 2016 9647/03


339 trendyline
By Hess Law,
H fCl2O7(g) = +45 + (546)
= 501 kJ mol1

(d)
CH3 CH3 CH3 CH3 CH3
H3C C CH3 H3C C CH3 H3C C CH3 H3C C CH3 H3C C CH3

H C CH3 C H C CH3 C C
H CH2 HO O Cl O
OH Cl
A B C D E

CH3
H3C C CH3

C
H2N O
F

trendyline
DHS 2016 9647/03
340 trendyline
HWA CHONG INSTITUTION
C2 Preliminary Examinations
Higher 2

CANDIDATE
CT GROUP 15S
NAME

CENTRE INDEX
NUMBER NUMBER

CHEMISTRY 9647/01
Paper 1 Multiple Choice 21 September 2016
1 hour
Additional Materials: Optical Mark Sheet (OMS)
Data Booklet

READ THESE INSTRUCTIONS FIRST

Write in soft pencil.


Do not use staples, paper clips, glue or correction fluid.
Complete the information on the optical mark sheet (OMS) as shown below.

There are forty questions on this paper. Answer all questions. For each question, there are four
possible answers A, B, C and D.
Choose the one you consider correct and record your choice in soft pencil on the OMS.

trendyline
Each correct answer
err will score one mark. A mark will not be deducted
ded for a wron
wrong answer.
Any rough working should be done in this booklet.
The use of an approved
oved scientific calculator is expected, where appropriate.
roved

This document consists of 17 printed pages.


341 trendyline
2
SECTION A

For each question, there are four possible answers A, B, C and D. Choose the one you consider
to be correct.

1 Methyl mercaptan, CH3SH, is one of the substances responsible for bad breath and is
often used to impart a smell to natural gas in a pipeline. When a sample of 10 cm3 of
CH 3 SH is exploded with 60 cm3 of oxygen, the final volume of gas after burning is
passed into an excess of aqueous alkali. The chemical equation for the combustion
process is shown below.
CH3SH + 3O 2 CO 2 + SO 2 + 2H 2 O

What percentage of the final volume of gas after burning dissolves in the alkali? All
gaseous volumes are measured at room temperature and pressure.

A 40.0% B 28.5% C 20.0% D 14.3%

2 Figure 1 shows the first six ionisation energies of an element Q while Figure 2 shows the
second ionisation energies of eight consecutive elements, including Q, in the Periodic
Table.

Figure 1 Figure
se 2
co
io ndi
ni oni
sa sat
tio ion
n en
en erg
er y/ B
gy kJ A
/ mo
kJ l1 C
m
ol
1
D
1 2 3 4 5 6
proton number
number of electrons removed

Which of the elements A, B, C or D could be the element Q?

3 Which statement is not always true when an ionic solid dissolves in water?

A Energy is absorbed to separate the ions in the solid.

trendyline
B Energy
gy is
s absorbed
abso bed to break
b ea tthe
e hydrogen bonding
yd oge bo between
d g bet ee water
ate molecules.

C Energy
gy is released when the ions are hydrated
hydrated.

D Energy
gyy is released when the ionic solid dissolves completely.

Hwa Chong Institution 2016 9647 / 01 / C2 Prelim 2016


342 trendyline
3
4 Which of the following explains the non-ideal behavior of the gases present in the
reaction chamber in the Haber process?

A the presence of a catalyst

B the high temperature of 450 C

C the high pressure of 150 atm

D the strong bonds between the atoms in the nitrogen molecules

5 Which enthalpy change would best indicate the relative strengths of the intermolecular
forces in liquid hydrogen halides?

A bond dissociation energy

B enthalpy change of formation

C enthalpy change of solution

D enthalpy change of vaporisation

6 CH 3 COCH 3 and I 2 react in the presence of an acid catalyst. The rate of reaction is
independent of [I 2 ] but directly proportional to [H+] and to [CH 3 COCH 3 ].

Which graph is correct?

A [I 2 ] B rate

0 time 0 [H+][CH 3 COCH 3 ]

C [CH 3 COCH 3 ] D [H+][CH 3 COCH 3 ]

0 rate 0 time

trendyline
Hwa Chong Institution 2016 9647 / 01 / C2 Prelim 2016
343 trendyline
4
7 When the system X 2 (g) + 3Y 2 (g) 2XY 3 (g) is at equilibrium at 500 K and 1 atm, the
value of the equilibrium constant, K p , is 100. What is the value of K p at a pressure of
2 atm at the same temperature?

A 25 B 100 C 200 D 400

8 The numerical values of the solubility product of BaSO 4 , BaCO 3 and Ba(IO 3 ) 2 at 25 C
are given in the table below.

Compound Solubility product


BaSO 4 1.1 1010
BaCO 3 2.6 109
Ba(IO 3 ) 2 4.0 109

An aqueous solution of BaCl 2 was added slowly, until in excess, to a solution


containing 0.5 mol dm3 Na 2 SO 4 , 1.0 mol dm3 Na 2 CO 3 and 1.5 mol dm3 NaIO 3 at 25
C.

What is the correct order of precipitation of the three barium salts?

First to precipitate Last to precipitate

A BaSO 4 BaCO 3 Ba(IO 3 ) 2

B Ba(IO 3 ) 2 BaCO 3 BaSO 4

C BaSO 4 Ba(IO 3 ) 2 BaCO 3

D BaCO 3 Ba(IO 3 ) 2 BaSO 4

9 What is the pH of the final solution formed when Vdm3of dilute hydrochloric acid of
pH 1.0 is mixed with Vdm3of dilute sulfuric acid of pH 1.0, followed by the addition of
2V dm3 of water?

A 1.1 B 1.3 C 2.0 D 4.0

10 Stomach juices have a pH of 1.0.

Aspirin is a monobasic acid represented by HA (K a = 104 mol dm3) which dissociates


into H+ and A ions.

What are the relative concentrations of H+, A and HA when aspirin from a tablet enters
the stomach?

A [H+] > [HA] > [A] B [H+] > [A] > [HA]

trendyline
C A]
[HA] > [H+] = [A D A] > [HA]
H+] = [A
[H

Hwa Chong Institution 2016 9647 / 01 / C2 Prelim 2016


344 trendyline
5
11 In the construction of heart pacemakers, a tiny magnesium electrode is used which
creates an electrochemical cell with the inhaled oxygen. The relevant half-equations are
as follows.

Mg2+ + 2e- Mg Eo = -2.38 V


O 2 + 2H+ + 2e- H 2 O Eo = +1.23 V

Under standard conditions, the cell e.m.f. would be 3.61 V. However, in the body, a
potential of 3.25 V is more usual.

Which of the following is an explanation for this lower e.m.f. in the body?

A the pH of about 7.4 of the body fluid surrounding the electrodes

B the low concentration of Mg2+ ions surrounding the electrodes

C the small size of the magnesium electrode

D the large amount of oxygen inhaled

12 Ethanol fuel cells are more practical than hydrogen fuel cells since ethanol is easier to
store and transport than hydrogen. Ethanol is converted to carbon dioxide when the fuel
cell is operated.
wire connecting a pair of
graphite electrodes

ethanol air

H2SO4 (aq)
Which statement is correct about ethanol fuel cells?

A Ethanol fuel cells can work indefinitely as long as there is a supply of ethanol
and air.

B For every 1 mol of ethanol oxidised, 6 mol of electrons are produced.

C H+ ions react with air to produce water at the anode.

D The acid needs to be replenished as hydrogen ions are being used up.

trendyline
Hwa Chong Institution 2016 9647 / 01 / C2 Prelim 2016
345 trendyline
6
13 How many structural isomers of C 3 H 4 Br 2 contain C=C and how many do not contain
C=C?

structural isomers with C=C structural isomers without C=C


A 4 0
B 4 2
C 5 0
D 5 2

14 2-methylbuta-1,3-diene, CH 2 =C(CH 3 )CH=CH 2 , is used as a monomer in the


manufacture of synthetic rubbers. Which compound would not produce this monomer on
treatment with excess concentrated sulfuric acid at 170C?

A OH B

HO

OH
OH

C OH D
OH
HO
HO

15 Sorbitol is a naturally-occurring compound with a sweet taste. It is often used as a


substitute for sucrose by the food industry.

OH OH OH OH OH OH

H C C C C C C H

H H H H H H

sorbitol

How many chiral centres are present in sorbitol?

A 3 B 4 C 5 D 6

trendyline
Hwa Chong Institution 2016 9647 / 01 / C2 Prelim 2016
346 trendyline
7
16 Deuterium, D, is a heavy isotope of hydrogen. Deuteriobenzene is reacted with chlorine
and AlCl 3 under controlled condition so that only monochlorination takes place.

Assuming that the carbon-deuterium bond is broken as easily as a carbon-hydrogen


bond, which proportion of the chlorinated products will be 2-chlorodeuteriobenzene?

D D

Cl

-
deuteriobenzene 2 chlorodeuteriobenzene

A 16% B 20% C 33% D 40%

17 What is the correct set of reagents and conditions for the conversion of methylbenzene to
2-nitromethylbenzene?

A dilute nitric acid, 100 C

B concentrated HNO 3 , 30 C

C concentrated HNO 3 and concentrated H 2 SO 4 , 30 C

D concentrated HNO 3 and concentrated H 2 SO 4 , 100 C

18 Which reaction gives the best yield of products? [(alc) indicates an alcoholic solution.]

A Cl
uv light
(alc) + Cl2 (aq) + HCl

B Br NH2
heat in
(alc) + NH3 (alc) + HBr
sealed tube

C Br
heat
(alc) + CH3CH2ONa (alc) + NaBr + CH3CH2OH

D CO2H COCl
r.t.p
(l) + PCl5 (aq) + POCl3 + HCl

trendyline
19 Chlorofluorocarbons (CFCs) have been widely used in aerosol sprays, refrigerators and
Hwa Chong Institution 2016 9647 / 01 / C2 Prelim 2016
347 trendyline
8
in making foamed plastics, but are now known to destroy ozone in the upper atmosphere.

Which compound will not destroy ozone, and therefore can be used as a replacement for
CFCs?

A CHBr 3

B CCl 3 CBr 3

C CHClFCClF 2

D CH 3 CH 2 CH 2 CH 3

20 Compound X has the following properties.

It changes the colour of acidified sodium dichromate(VI) from orange to green.


It gives yellow precipitate with alkaline aqueous iodine.

Which compound could be X?

A O OH B O
CH3 C C CH2I CH3 C OCH2CH2OH
CH3

C O O D I
H C C CH2I CH3 C CH2OH
OH

21 -naphthol is a crystalline solid that is widely used in the production of dyes.

OH

-
naphthol

Which of the following is likely to be a property of -naphthol?

A Its pK a is lower than that of phenol.

B It gives white fumes with SOCl 2 .

C It reacts with carboxylic acid to form ester.

D It undergoes nucleophilic substitution in the reaction below.

tre
trendyline
en
n ne
ndyl
OH
+ ++N
N2 SO3 + OH + H2O
N
N SO3
OH

22 Cinnamaldehyde, gallic acid and vanillin are naturally occurring organic compounds

Hwa Chong Institution 2016 9647 / 01 / C2 Prelim 2016


348 trendyline
9
found in plants.

OH OH
O
OCH3

HO OH
CO2H O
cinnamaldehyde gallic acid vanillin

The CH 3 O group is inert and can be disregarded in this question.

Which set of reagents can be used to distinguish these three compounds?

A Aqueous bromine and aqueous sodium hydrogencarbonate

B Aqueous bromine and Tollens reagent

C Fehlings solution and neutral iron(III) chloride

D Fehlings solution and hot acidified potassium manganate(VII)

23 Two esters can react via Claisen condensation in the presence of a strong base. The two
steps of this reaction are shown below.

OR' OR'
R B R + BH+
Step 1:
O = O
(B base)
R
OR' OR' R'O
R + R R + R'O
Step 2:
O O O O
product of
Claisen condensation

Which compound will be formed when CH 3 CH 2 OCOCH 3 undergoes Claisen


condensation?

A O H O B O O
CH3 C C O C CH3 CH3 C CH2 C O CH2CH3
CH3

C O H O D O O
CH3O C C C CH2CH3 CH3 C CH2 CH2 C OCH3

trendyline
CH3

Hwa Chong Institution 2016 9647 / 01 / C2 Prelim 2016


349 trendyline
10
24 Ergometrine is a drug used in obstetrics to treat heavy bleeding after childbirth.

OH
O NH

N
H ergometrine

Which statement about ergometrine is correct?

A pK b of the labelled three nitrogen atoms in increasing order is N < N < N .

B It reacts with both dilute NaOH and dilute HCl at room temperature.

C It forms orange precipitate with 2,4-dinitrophenylhydrazine.

D One mole of ergometrine reacts with 3 moles of CH 3 COCl.

trendyline
Hwa Chong Institution 2016 9647 / 01 / C2 Prelim 2016
350 trendyline
11
25 The graph represents the change in mass that occurs when 1.0 g of powdered calcium
carbonate, CaCO 3 , is heated at a temperature, T.
Mass / g

1.0

0.5

Time / min
0
Which graph would be obtained by heating 1.0 g of powdered magnesium carbonate,
MgCO 3 , at the same temperature,T?

A Mass / g B Mass / g

1.0 1.0

0.5 0.5

Time / min Time / min


0 0

C Mass / g D Mass / g

1.0 1.0

0.5 0.5

Time / min Time / min


0 0

26 What chlorine-containing products are formed when chlorine is bubbled into hot aqueous
sodium hydroxide?

A NaClO only

B NaClO 3 only

C NaCl and NaClO

trendyline
D NaCl an
and NaClO
NaClO 3

Hwa Chong Institution 2016 9647 / 01 / C2 Prelim 2016


351 trendyline
12
27 Which statement concerning the Period 3 elements, sodium to chlorine, is correct?

A Chlorine has the largest anion.

B Aluminium has the highest melting point.

C Chlorine exhibits the most number of different oxidation states.

D Phosphorus reacts most vigorously with water.

28 Element X is one of the first five elements in Period 3 of the Periodic Table. The following
four statements were made about the properties of element X or its compounds.

Three statements are correct descriptions and one is false.

Which statement does not fit with the other three?

A The oxide of X dissolves in excess dilute NaOH(aq).

B X exhibits only one possible oxidation number in its chloride, which is not a
solid at room temperature.

C The oxide of X has a very high melting point.

D The chloride of X reacts with water to give an acidic solution with pH 1.

29 Which property of the first row transition elements doesnot remain relatively constant?

A atomic radius B melting point

C first ionisation energy D ionic radius of the +2 ions

30 Aqueous platinum(IV) chloride reacts with ammonia to form compounds in which the
coordination number of platinum is 6. When dissolved in water,1 mol of such compound
gave 3 mol of ions.

What is the formula of this compound?

A Pt(NH 3 ) 2 Cl 4 B Pt(NH 3 ) 3 Cl 4

C Pt(NH 3 ) 4 Cl 4 D Pt(NH 3 ) 6 Cl 4

trendyline
Hwa Chong Institution 2016 9647 / 01 / C2 Prelim 2016
352 trendyline
13
SECTION B

For each of the questions in this section, one or more of the three numbered statements 1 to 3
may be correct.

Decide whether each of the statements is or is not correct (you may find it helpful to put a tick
against the statements that you consider to be correct).

The responses A to D should be selected on the basis of

A B C D
1, 2 and 3 are 1 and 2 only 2 and 3 only 1 only is
correct are correct are correct correct

No other combination of statements is used as a correct response.

31 Which statements about one mole of a metal are always correct?

1 It contains the same number of atoms as one mole of hydrogen atoms.

2 It contains the same number of atoms as that in 12 grams of 12C.

3 It is liberated by one mole of electrons.

32 Beryllium is the first member of Group II and forms covalent compounds which are said to
be electron deficient. In many ways, beryllium resembles aluminium.

Which of the following are possible?

1 F F

Be Be

F F

2
F F
Be Be

F F n

3 2

ttrendyline
tr
re
F F
Be

F F

Hwa Chong Institution 2016 9647 / 01 / C2 Prelim 2016


353 trendyline
14
The responses A to D should be selected on the basis of

A B C D
1, 2 and 3 are 1 and 2 only 2 and 3 only 1 only is
correct are correct are correct correct

No other combination of statements is used as a correct response.

33 When 25.0 cm3 of a 0.100 mol dm3 strong monobasic acid was titrated against
0.200 mol dm3 of a weak base X, 12.50 cm3 of the base was required to completely
neutralise the acid.

Which statement is correct for this experiment?

1 Bromocresol green (pK In = 4.7) is a suitable indicator for this titration.

2 1 mole of X reacts with 1 mole of the monobasic acid.

3 A solution of maximum buffering capacity was formed when 6.25 cm3 of the
base was added.

34 The use of the Data Booklet is relevant to this question.

Electrolysis of a solution of Mn+(aq) led to the deposition of M atoms at the cathode.


5.00 103 mol of M is deposited when 965 C of electricity are passed through the
solution.

What could Mn+be?

1 Sn2+

2 Cr2+

3 Ag+

Hwa Chong Institution 2016 9647 / 01 / C2 Prelim 2016


354 trendyline
15
35 Humulene can be extracted from carnation flowers.

CH3

CH3

CH3
CH3
humulene

Which products are obtained from the reaction of humulene with hot acidified
concentrated KMnO 4 ?

1 CH 3 COCH 2 CH 2 CO 2 H

2 CH 3 COCH 2 CO 2 H

3 HO 2 CCH 2 C(CH 3 ) 2 CO 2 H

36 Consider the following reaction scheme.

Cl O Step I
O OH
M

Step IV
Cl

Cl

Which of the following statements about this reaction scheme are correct?

1 Step I requires a catalyst.

2 Compound M is a planar molecule.

3 Step IV is a substitution reaction.

trendyline
Hwa Chong Institution 2016 9647 / 01 / C2 Prelim 2016
355 trendyline
16
The responses A to D should be selected on the basis of

A B C D
1, 2 and 3 are 1 and 2 only 2 and 3 only 1 only is
correct are correct are correct correct

No other combination of statements is used as a correct response.

37 PGE 2 is a prostaglandin that has pharmacological activity.

O
CO2H

CH3

HO OH
PGE2

What is the correct number of hydrogen atoms incorporated per molecule of PGE 2 when
PGE 2 is reacted with each of the following reducing agents?

Number of hydrogen atoms


Reducing agent
incorporated per molecule of PGE 2
1 H 2 / Ni 6

2 LiAlH 4 in dry ether 4

3 NaBH 4 in ethanol 4

trendyline
Hwa Chong Institution 2016 9647 / 01 / C2 Prelim 2016
356 trendyline
17
38 The cyclic peptide gramicidin S can act as an antibiotic.

NH2

O O
H H
N N
N N
H H
N O O
O
OO O N
H H
N N
N N
H H
O O

H2N
gramicidin S

Which statements about gramicidin S are correct?

1 It is made up of 5 different amino acids.

2 1 mole of gramicidin S reacts with 10 moles of hot dilute HCl.

3 Its three-dimensional structure can be stabilised by ionic interaction.

39 Which of the following are trends across Period 3 of the Periodic Table?

1 The electronegativity of the elements increases.

2 The oxides of the elements change from ionic to covalent.

3 The electrical conductivity of the elements decreases.

40 Prussian blue, Fe 4 [Fe(CN) 6 ] 3 , is a dark blue pigment often used in painting and dyeing.

It can be synthesised in the laboratory by mixing iron(III) chloride, FeCl 3 , with potassium
ferrocyanide, K 4 [Fe(CN) 6 ].

Based on the information given above, which conclusions can be made about Prussian
blue?

1 The percentage by mass of iron in Prussian blue is 45.5%.

2 A redox reaction has taken place in the formation of Prussian blue.

trendyline
Prussian

dy
sian blue absorbs blue light strongly.
strongl

The End

Hwa Chong Institution 2016 9647 / 01 / C2 Prelim 2016


357 trendyline
HWA CHONG INSTITUTION
2016 C2 H2 CHEMISTRY PRELIMINARY EXAM
SUGGESTED SOLUTIONS

Paper 1

1 2 3 4 5 6 7 8 9 10
A D D C D C B C B A
11 12 13 14 15 16 17 18 19 20
A A D B B C C C D C
21 22 23 24 25 26 27 28 29 30
A D B A C D C A B C
31 32 33 34 35 36 37 38 39 40
B C B D A D B D B D

trendyline
2016 HCI C2 H2 Chemistry Preliminary Exam / Paper 1
358 trendyline
HWA CHONG INSTITUTION
C2 Preliminary Examinations
Higher 2

CANDIDATE
CT GROUP 15S
NAME

CENTRE INDEX
NUMBER NUMBER

CHEMISTRY 9647/02
Paper 2 Structured Questions 30 August 2016
2 hours
Candidates answer on the Question Paper
Additional Materials: Data Booklet

INSTRUCTIONS TO CANDIDATES
1) Write your name, CT group, centre number and index number clearly in the spaces at
the top of this page.
2) Answer all questions in the spaces provided in this Question Paper.

INFORMATION FOR CANDIDATES


The number of marks is given in brackets [ ] at the end of each question or part question.
A Data Booklet is provided.
Do not use staples, paper clips, highlighters, glue or correction fluid.
You may use a calculator.
You are reminded of the need for good English and clear presentation in your answers.

For Examiners Use

1 / 12

2 / 17

3 / 14

4 / 13

5 /16

Deductions

Calculator Model:
trendyline
tren ne Total / 72

This document consists


359 of 18 printed pages. trendyline
2

1 Planning (P) For


Examiner's
use
An electrochemical cell is a device that produces an electric current as a result of redox
reactions. Any spontaneous redox reaction can be harnessed to produce electrical energy
under the right conditions. Electrons are generated at one electrode and driven towards the
other by electromotive force, also known as the cell potential, measured in volts.

The Gibbs Free Energy equation is given as G = H- TS. G may be obtained from the
cell potential of an electrochemical cell by the relationship:

G = -nFE

where n = number of moles of electrons transferred in the reaction


F = Faradays constant
E = cell potential

By constructing an electrochemical cell, the cell potential can be measured over a range of
temperatures. The entropy change of a reaction can then be determined by plotting a suitable
linear graph using the data obtained.

(a) (i) Write an ionic equation for the reaction between Cu and Fe3+. You should include
state symbols in your equation.

[1]

(ii) Predict the sign of Sfor the above reaction. Explain your answer.

[1]

(b) Using the information given above, you are required to write a plan for determining the
entropy change for the redox reaction between Cu and Fe3+. You will measure the cell
potential over a 30C range of temperature.

You are also required to explain how the data you obtain from this experiment may be
used to determine the entropy change of the reaction.

You may assume that the values of H and S are constant over this measured
temperature change.

You may also assume that you are provided with:

0.20 mol dm-3 aqueous FeCl 3


0.10 mol dm-3 aqueous CuSO 4
0.50 mol dm-3 aqueous H 2 SO 4
solid hydrated iron(II) sulfate, FeSO 4 .7H 2 O
common electrodes
the apparatus
pparatus normally found in a school or colleg
college laboratory.

Hwa Chong Institution 2016 9647 / 02 360


/ C2 Prelim 2016 trendyline
3

Your plan should include: For


Examiner's
use
a fully-labelled diagram of the set-up of the electrochemical cell you would use to
monitor how cell potential varies with temperature;
details for the preparation of 100 cm3 of 0.20 mol dm3 aqueous acidified FeSO 4 ;
details of the appropriate quantities and concentrations of solutions used;
the temperatures at which the experiments would be carried out;
an outline of how the results would be obtained;
a sketch of the graph you would expect to obtain;
how the data obtained from the graph would be used to determine the entropy
change for the reaction.


1

2

trendyline
7


8


9

10

Hwa Chong Institution 2016 9647 / 02 361


/ C2 Prelim 2016 trendyline
4

For
Examiner's
use

trendyline


Hwa Chong Institution 2016 9647 / 02 362
/ C2 Prelim 2016 trendyline
5

For
Examiner's
use

trendyline

...[10]

[Total: 12]

Hwa Chong Institution 2016 9647 / 02 363


/ C2 Prelim 2016 trendyline
6

2 (a) The transition metals form many complexes. Two examples of complexes formed by Fe3+ For
Examiner's
are [Fe(H 2 O) 5 (SCN)]2+ and [Fe(CN) 6 ]3. use

These complexes contain the ligands, H 2 O, SCN, and CN, in which the "donor atom"
for dative bonding are O, S and C respectively.

(i) Explain why the atomic radius of oxygen (0.066 nm) is smaller than that of carbon
(0.077 nm).

[2]

(ii) Explain why the atomic radius of sulfur (0.104 nm) is larger than those of oxygen
and carbon.

[1]

trendyline
Hwa Chong Institution 2016 9647 / 02 364
/ C2 Prelim 2016 trendyline
7

(b) When water ligands in an aqua complex are replaced by other ligands, the equilibrium For
Examiners
constant for the reaction is called the stability constant, K stab , of the new complex. use

For example,
[Fe(H 2 O) 6 ]3+(aq) + 6CN(aq) [Fe(CN) 6 ]3(aq) + 6H 2 O(l)

[Fe(CN) 6 3]
K stab =
[Fe(H 2 O) 6 3+][CN]6

The K stab value is an indication of the stability of a complex ion with reference to the aqua
complex. The table below lists the K stab values of some iron complexes.

complex K stab
[Fe(H 2 O) 5 (SCN)]2+ 1 102
[Fe(CN) 6 ]3 1 1031
[Fe(CN) 6 ]4 1 1024
[Fe(edta)] 1 1025
[Fe(edta)]2 2 1014
edta = (O 2 CCH 2 ) 2 NCH 2 CH 2 N(CH 2 CO 2 ) 2

(i) State the colour of the solution obtained when aqueous potassium thiocyanate,
KSCN, is added to aqueous Fe3+.

[1]

(ii) Suggest why the colour you stated in (b)(i) is not obtained when aqueous KSCN
is added to aqueous [Fe(CN) 6 ]3.

[1]

(iii) Predict whether edta would make the reduction potential for the Fe(III)/Fe(II)
system higher or lower than +0.77 V. Explain your answer.

[Fe(H 2 O) 6 ]3+ + e [Fe(H 2 O) 6 ]2+ E = +0.77 V

[1]

trendyline
(c) A series of tests was conducted as follow: For
Examiners

Hwa Chong Institution 2016 9647 / 02 365


/ C2 Prelim 2016 trendyline
8

I When concentrated hydrochloric acid wasadded to dissolve black copper(II) use

oxide, a yellow solution wasobtained.

II Dilution of this yellow solution with water changed its colour to green and then
blue.

III When aqueous ammonia was then added, a pale blue precipitate formed which
dissolved in excess aqueous ammonia to give a dark blue solution.

(i) Identify the copper-containing species formed in each test.

Test Identity of copper-containing species


I
Yellow solution:
....
II
Blue solution:
III
Pale blue precipitate:
...

Dark blue solution:


...
[2]

(ii) Write equations to illustratethe ligand exchange reactions in forming

the blue solution inTest II:

the dark blue solution in Test III:

...... [2]

(d) Aluminium is extracted from molten aluminium oxide, Al 2 O 3 , by electrolysis.

(i) With reference to E data from the Data Booklet, explain why an aqueous
solution of Al3+ (from a soluble aluminium salt) would be unsuitable as the
electrolyte.

[1]

trendyline
Hwa Chong Institution 2016 9647 / 02 366
/ C2 Prelim 2016 trendyline
9

In the industry, Al 2 O 3 is dissolved in molten cryolite, Na 3 AlF 6 to lower the melting point of For
Examiners
Al 2 O 3 . The molten mixture is then electrolysed using graphite electrodes. use

(ii) The electrode reactions are very complicated. In one simplified study, some
scientists say that, in molten cryolite, Al 2 O 3 ionises into Al3+ ions and AlO 2 ions.

At the cathode, Al3+ is converted to Al metal: Al3+ + 3e Al


At the anode, AlO 2 is converted to Al 2 O 3 and oxygen gas.

Write an equation for the anode reaction.

... [1]

(iii) A typical cell operates at 104 A. Calculate the time, in hours, needed to produce
1 tonne of aluminium. (1 tonne = 1000 kg)

[2]

(e) A saturated solution of XeF 2 (M r = 169) in water at 0 C contains 25.0 g dm3 of XeF 2 .
The solution is a strong oxidising agent, quantitatively oxidising another solution
containing 0.150 mol dm3 of a Cr(III) salt. XeF 2 is reduced to Xe in the reaction.

In an experiment, 15.20 cm3 of the XeF 2 solution is required to completely react with
10.0 cm3 of the Cr(III) solution.

Determine the oxidation state of Cr after the reaction.

trendyline [3]

[Total: 17]
Hwa Chong Institution 2016 9647 / 02 367
/ C2 Prelim 2016 trendyline
10

3 The hydrogenation of but-2-ene is shown in the equation below For


Examiner's
use
CH 3 CHCHCH 3 (g) + H 2 (g) CH 3 CH 2 CH 2 CH 3 (l)

(a) Describe the changes in bond angle and the type of hybridisation involved at the site of
the hydrogenation reaction.

.....

.....

.....

.....[2]

(b) Under certain conditions,but-2-ene gives alcohol A, C 4 H 10 O, which exists as a liquid at


room temperature.

(i) State the reagents and conditions required in this reaction. Draw the displayed
formula of alcohol A formed in this reaction.

Reagents and conditions:

.
A:

[2]

(ii) Explain, in terms of structure and bonding, the difference in the physical state of
but-2-ene and alcohol A at room temperature. Draw a diagram to illustrate your
answer for alcohol A.

trendyline


Diagram:
agram

Hwa Chong Institution 2016 9647 / 02 368


/ C2 Prelim 2016 trendyline
11

[3]
(iii) Ester C is used in synthetic fruit flavours. Alcohol A reacts with compound B at For
Examiners
room temperatureto give a good yield of ester C with relative molecular mass use
of 116.

Draw the structures of compounds B and C.

B: C:

[2]

(c) (i) In a separate reaction, butane is treated with a small quantity of bromine in the
presence of ultraviolet light to give 1-bromobutane and 2-bromobutane. It is
found experimentally that the hydrogen atom is substituted by bromine at
different rates, as shown in the table below.

Type of H atom Relative rate of substitution


Primary hydrogen 1
Secondary hydrogen 4.5

Using this information, and considering the number and types of hydrogen within
butane, determine the mole ratio of the two monobrominated products.

..

..

..

..[2]

trendyline
Hwa Chong Institution 2016 9647 / 02 369
/ C2 Prelim 2016 trendyline
12

(ii) Name and outline the mechanism for the formation of 2-bromobutane in this For
Examiner's
reaction. use

trendyline
[3]

[Total: 14]

Hwa Chong Institution 2016 9647 / 02 370


/ C2 Prelim 2016 trendyline
13

4 (a) Using relevant data from the Data Booklet, describe and explain how the thermal stability For
Examiner's
of the hydrogen halides varies down Group VII. use

.....

.....

.....

.....

.....[2]

(b) In the laboratory, three bottles each containing one of the following reagents were mixed
up:
NaI(aq), KBr(aq) and KCl(aq).

Describe chemical test(s), which does not involve silver nitrate, that could distinguish and
identify the contents of each bottle.

.....

.....

.....

.....

.....

.....

.....[3]

(c) HF behaves as a medium-strength acid and dissociates only partially in aqueous


solutions as shown below. K 1 and K 2 representthe numerical values of the respective
equilibrium constants.

HF + H 2 O H 3 O+ + F K 1 = 1.1 103
HF + F HF 2 K 2 = 2.6 101

(i) Write the overall equation for the reaction of liquid HF with H 2 O to form HF 2 and
calculate the numerical value of the equilibrium constant for this reaction.

trendyline
[2]
(ii) The dissociation equilibrium of a solute in a solvent can be significantly shifted by For

Hwa Chong Institution 2016 9647 / 02 371


/ C2 Prelim 2016 trendyline
14

the addition of a suitable substance into the solution. Suggest an inorganic Examiner's
use
reagent which can be added to increase the dissociation of HF in water. Explain
your choice.

[2]

(d) Similar to water, pure liquid hydrogen fluoride also undergoes auto-ionisation according
to the following equilibrium:

3HF H 2 F+ + HF 2

The ionic product of HF is 8.0 1012 mol2 dm.

(i) Draw the structures of both the cationic and anionic species in liquid HF. Label
the types of bonding within the anionic species.

[2]

(ii) Calculate the fraction of H 2 F+ in liquid HF (density= 1.002 g cm3), supposing that
only these three species are present in the system.

trendyline [2]

[Total: 13]
5 The following reaction scheme is proposed for the synthesis of the amino acid proline. For

Hwa Chong Institution 2016 9647 / 02 372


/ C2 Prelim 2016 trendyline
15

Examiner's
use
multiple O
O
O HCl steps
Cl N
Cl H2N H
H2N step I
Cl
Cl

enzyme
amidase
N N N N
H H H H
step III step II
CO2H CONH2 CN Cl
proline ro
p linamide

(a) (i) Draw structures of two possible intermediates formed in step I. Hence, explain
why the yield of the product in step I is low.

....

....

....[2]

(ii) With the aid of a sketch of the Boltzmann distribution, explain how an increase in
temperature affects the rate of reaction in step II.

....

trendyline
..
....

....
..

....[3]

(iii) It was found that only 50% of prolinamide synthesised by the method above could For
Examiners
Hwa Chong Institution 2016 9647 / 02 373
/ C2 Prelim 2016 trendyline
16

be converted to proline by enzyme amidase in step III. Suggest an explanation for use
this observation.

....

....

....[1]

(b) The enzyme amidase used in step III contains amino acids listed in the following table.

name abbreviation structural formula

aspartic acid asp HO 2 CCH 2 CH(NH 2 )CO 2 H


asparagine asn H 2 NCOCH 2 CH(NH 2 )CO 2 H
cysteine cys HSCH 2 CH(NH 2 )CO 2 H

glycine gly CH 2 (NH 2 )CO 2 H

isoleucine ile CH 3 CH 2 CH(CH 3 )CH(NH 2 )CO 2 H

lysine lys H 2 N(CH 2 ) 4 CH(NH 2 )CO 2 H

phenylalanine phe C 6 H 5 CH 2 CH(NH 2 )CO 2 H

serine ser HOCH 2 CH(NH 2 )CO 2 H

(i) There are three pK a values associated with aspartic acid: 2.1, 3.7 and 9.8. In the
boxes below, draw the structures of the major species present in solutions of
aspartic acid at pH 3 and pH 8.

pH 3 pH 8
[2]

trendyline For
(ii) Complete the following table by indicating the type of interaction between each
Examiners
pair of amino acid residues that stabilises the tertiary structure of enzyme use
Hwa Chong Institution 2016 9647 / 02 374
/ C2 Prelim 2016 trendyline
17

amidase at pH 7.

amino acid residue 1 amino acid residue 2 type of interaction

aspartic acid lysine

isoleucine phenylalanine

serine asparagine

[2]

(iii) Explain how mercury ion, Hg2+(aq), affects the activity of the enzyme amidase
through denaturation.

.......

....

....[2]

(c) A section of the polypeptide chain in enzyme amidase was partially hydrolysed and the
following peptide fragments are produced.

lys-gly, asp-lys-ser, cys-asn-phe, gly-cys-asn, ser-phe-lys

State the smallest possible sequence of amino acids in this section of the polypeptide
chain using the 3-letter abbreviations.

.....[1]

(d) A scientist extracted a new compound Dfrom


f microorganisms. He proposed the following For
Examiners
structure for D. use

Hwa Chong Institution 2016 9647 / 02 375


/ C2 Prelim 2016 trendyline
18

O
S OH
O N
H2N N
O
H

(i) D was postulated to be synthesised from amino acids.

-amino acids that could be used in the synthesis.


Identify from the list in (b), two

....[1]

(ii) Assuming that the SC bond is inert, draw the organic products if D was heated in
aqueous H 2 SO 4 .

[2]

[Total: 16]

trendyline
Hwa Chong Institution 2016 9647 / 02 376
/ C2 Prelim 2016 trendyline
Paper 2

1 (a) (i) Cu(s) + 2Fe3+(aq) Cu2+(aq) + 2Fe2+(aq) [1]

(ii) Positive. More aqueous ions produced and thus more ways that
energy can be distributed in the system through the motion of ions.
[1]

1 (b)

M1 [1/2] voltmeter and salt bridge

[1/2] a single water bath or hot plate (not separate)

M2 Correct Fe3+/Fe2+ half cell:


[1/2] electrode + [1/2] solutions (with correct concentration)

M3 Correct Cu2+/Cu half cell:


[1/2] electrode + [1/2] solution (with correct concentration)

M4 [1/2] 5.56 g of solid

[1/2] details for weighing electronic balance, weighing bottle/


beaker, reweigh or tare and rinse out all solid

M5 Preparation of acidified FeSO 4 solution:


[1/2] adding acid before topping up to 100 cm3 mark

trend
trendyline
d
dy
[1/2] 100 cm3 volumetric flask
[1 f

M6 Preparation of FeSO 4 solution:


lution:
Transfer solid to beaker
Dissolve solid in beaker
ke
Transfer solution to volumetric flask
7
377 trendyline
Transfer washings to volumetric flask
Top up volumetric flask
Shake

[1/2] any 3 5 points


[1] all 6 points

M7 [1/2] measure volumes of solutions used for both half-cells


(minimum volume of electrolyte in each half-cell: 20 cm3)

[1/2] use appropriate apparatus e.g. measuring cylinder for


measurement of solutions for both half-cells

M8 [1/2] measure temperature using thermometer or set temperature


of thermostatically controlled water bath

[1/2] range of temperature used at least 30 C should have at


least 5 readings with at least 5 C intervals (lowest temperature
should not be below 20 C)

M9 Sketch of graph:
[1/2] Correct choice of axes E vs T (ignore units)

[1/2] Linear graph with positive gradient

$OORZDSORWRI*YV T. In this case, a linear graph with negative


gradient would be obtained. Candidate needs to explain clearly
KRZ*FDQEHREWDLQHG

M10 [1] explain how entropy change could be obtained from the graph
HJ*UDGLHQW S/ nF (for E vs T graph)

Gradient = 6 IRU*YV7JUDSK

Sample procedure
1. Using an electronic weighing balance, weigh accurately 5.56g of
FeSO 4 .7H 2 O in a weighing bottle.
2. Transfer this solid into a 100 cm3 beaker. Reweigh the weighing
bottle to account for any residual solid. Record the mass of the solid
used.
3. Dissolve the solid usingg about 20 cm3of 0.50 moldm3H 2 SO 4 (aq).

trendyline
4. Transfer this solution into a 100 cm3 volumetric flas
beaker
eaker thoroughly and transfer all washin
he
e mark with deionised water/
the water/H
solution. Label
homogeneous solution
washing
washings
H 2 SO
el this
flask.
flask Rinse the
s into the flask. Top up to
O 4 (aq) and shake well to obtain a
th solution as FA1 FA1.

2016 HCI C2 H2 Chemistry Preliminary Exam / Paper 2


378 trendyline
5. Transfer 15 cm3 of FA1 and 15 cm3 of 0.20 mol dm-3 FeCl 3 into a
100 cm3 beaker using separate burettes.
6. Transfer 30 cm3 of 0.10 mol dm-3 CuSO 4 into a 100 cm3 beaker
using a burette.
7. Set up the apparatus as shown in the diagram above.
8. Ensure that the water bath is maintained at 25oC by measuring the
temperature with a thermometer.
9. Record the cell potential using a voltmeter.
10. Record the cell potential at different temperatures: 30oC, 35oC,
40oC, 45oC, 50oC, 55oC.

Plot a graph of E against T.


E

S = gradient of the graph x F x n

2 (a) (i) m each:


O atom has more protons (or higher nuclear charge than C atom)
shielding effect is similar (or almost the same)
O has higher effective nuclear charge
its outer electrons are pulled (or attracted) more closely to the
nucleus

(ii) 1m: S atom has 1 more occupied quantum shell than O and C.

2 (b) (i) 1m: blood red

(ii) m each:
[Fe(CN) 6 ]3 has (much) higher K stab than [Fe(H 2 O) 5 (SCN)]2+
SCN (and H 2 O) unable to replace CN as the ligands attached to
Fe3+ i.e. no ligand exchange

(iii) 1m:

So
OR
trendyline
m:
e(edta has (much) highe
e(edta)]
[Fe(edta)] han [Fe(edta)]2.
higher K stab than
o [Fe(edta)] is less likely to be reduced to [Fe(edta)
[Fe(edta)]2.
edt )] + e [Fe(edta)]22 lies more to
R reduction equilibrium [Fe(edta)]
[Fe(edta
the left.
8
379 trendyline
OR edta ligand stabilises +3 oxidation state of Fe relative to +2 state.
reduction potential for Fe(III)/Fe(II) with edta as ligands or for
[Fe(edta)]/[Fe(edta)]2 would be lower than +0.77 V.

2 (c) (i) m each:


yellow solution: [CuCl 4 ]2
blue solution: [Cu(H 2 O) 6 ]2+
pale blue ppt: Cu(OH) 2 or [Cu(OH) 2 (H 2 O) 4 ]
dark blue solution: [Cu(NH 3 ) 4 (H 2 O) 2 ]2+

(ii) 1m each:
Test II [CuCl 4 ]2 + 6H 2 O [Cu(H 2 O) 6 ]2+ + 4Cl
Test III [Cu(H 2 O) 6 ]2+ + 4NH 3 [Cu(NH 3 ) 4 (H 2 O) 2 ]2+ + 4H 2 O
Accept

2 (d) (i) m each:


Eo(H 2 O/H 2 ) = 0.83 V and Eo(Al3+/Al) = 1.66 V
or Eo(H+/H 2 ) = 0 V and Eo(Al3+/Al) = 1.66 V
Eo(H 2 O/H 2 ) is higher or less negative, so H 2 O would be
preferentially reduced to H 2 , Al3+ would not be reduced to give Al.
orEo(H+/H 2 ) is higher or less negative, so H+ would be
preferentially reduced to H 2 , Al3+ would not be reduced to give Al.

(ii) 1m: 2AlO 2 Al 2 O 3 + O 2 + 2e


or 4AlO 2 2Al 2 O 3 + O 2 + 4e

(iii) 1m: calculates no. of moles of electrons


1m: calculates time in hours

1000103
Amount of Al deposited = = 3.704 104mol
27
Amount of electrons passed = 3 3.704104 = 1.111 105mol

Q = It = 1.111105 96500 = 1.072 1010 C


1.0721010
t= 4 = 1.072 106 s = 298 h (or 297.8 h)
10

trendyline
2 (e) 1m: concludes
ncludes
c +6
1m: calculates
culates 0.00225 mol XeF 2 and 0.00150 mol Cr(III)
1m: explains
lains how to conclude +6, e.g. 1 mol Cr(III) loses 3 mol electrons
plains

Sample
e working:

2016 HCI C2 H2 Chemistry Preliminary Exam / Paper 2


380 trendyline
15.2 25
Amount of XeF 2 = = 0.002249 mol
1000 169
XeF 2 reduced to Xe, oxidation state of Xe changes from +2 to 0
Amount of electrons transferred = 2 0.002249 = 0.004497 mol

10
Amount of Cr(III) = 0.150 = 0.00150 mol
1000
0.004497
= 3 mol of electrons lost from 1 mol of Cr(III)
0.00150
oxidation state of Cr increases by 3 units
oxidation state of Cr is +6 after reaction

3 (a) The carbon atom of the reactant (but-2-ene) in the CC bond is


sp2hybridised [1/2] and the bond angle around the carbon atom is 120o
[1/2].

The carbon atom of the product (butane) in the CC bond is sp3hybridised


[1/2], and the bond angle around the carbon atom is 109.5o. (accept 109o)
[1/2]

(b) (i) Steam, 300C, 70 atm (or 60 atm), (conc) H 3 PO 4 [1]


ORconc H 2 SO 4 followed by warming with water.

H H H H
H C C C C H
H H H
O

A: H [1]

(ii) Both but-2-ene and alcohol are simple covalent compounds /simple,
discrete molecules /have simple molecular structure. [1/2]

For but-2-ene, there are weak dispersion forces between its


molecules. For alcohol A (butan-2-ol) with polar OH bond, there are
strong intermolecular hydrogen bonding between its molecules. [1/2]

More energy is required to overcome the stronger intermolecular

trendyline
ydrogen
drogen bonding in alcohol
hydrogen alcoho A thanan the weak di disp
dispersion forces in
ut-2-
ut -2-ene
ene which is therefore in gaseous state. [1/2]
but-2-ene
ORR
thee energy provided by room temperature is insuffic
insufficie to overcome
insufficient
thee strong hydrogen bonds between
tw alcohol A molecules,
molecu

9
381 trendyline
Hence alcohol A exists as a liquid at room temperature and but-2-
ene exists as gas. [1/2]

Diagram [1]
o showing correct hydrogen
bond (must be oxygen atom
and the H atom from the OH
group)
o labelling hydrogen bond
o showing lone pair of electrons
on oxygen
o labelling+ and - for the OH
group forming H bond.

Each mistake minus 1/2 mark

(iii) O O

B: Cl [1] C: O [1]

3 (c) (i) Ratio of 1-bromobutane : 2-bromobutane = 1 : 3 [1]

In 1-bromobutane: there are 6 possible primary (1o) H atoms for


substitution and in 2-bromobutane: there are only 4 secondary (2o) H
atoms for substitution [1/2]
Relative rate of substitution suggests that
the mole ratio of 1-bromobutane : 2-bromobutane
= 1 x 6 : 4.5 x 4 = 6 : 18 [1/2] for working

(ii) Type of mechanism: Free radical substitution [1]

uv light
Initiation: BrBr 2 Br [1/2]

Propagation: [1]

CH 3 CH 2 CH 2 CH 3 + Br CH 3 CH 2 CHCH 3 + HBr

CH 3 CH 2 CHCH 3 + Br 2 CH 3 CH 2 CH(Br)CH 3 + Br

t endy
trendyline
y
Termination [[1/2]]

Br + Br Br 2

Br + CH 3 CH 2 CHCH 3 CH 3 CH 2 CH(Br)
CH(Br)CH 3

2016 HCI C2 H2 Chemistry Preliminary Exam / Paper 2


382 trendyline

CH 3 CH 2 CHCH 3 + CH 3 CH 2 CHCH 3
CH 3 CH 2 CH(CH 3 )CH(CH 3 )CH 2 CH 3

4 (a) The thermal stability of the hydrogen halidesdecreases down the group
[1/2] as the bond energy of HX decreases accordingly from +562 (HF)
to +431 (HCl) to +366 (HBr) to +299 kJ mol (HI) [1/2].
Down the group, less energy is required to break the bond [1/2] as the
bond strength decreases [1/2].

(b) Step 1: To 1 cm3 of each reagent in separate test tubes, add a few drops
of aqueous lead(II) nitrate and shake. [1/2]
If yellow ppt (of PbI 2 ) is obtained, the test tube contains NaI. If white ppt is
obtained, the test tube contains KBr or KCl. [1]

Step 2: To 1 cm3 of each reagent that gave white ppt in step 1, add a few
drops of acidified sodium chlorate(I) OR Cl 2 (aq) and shake. [1/2]
If the solution turned yellow-orange, the test tube contains KBr. If the
solution remained colourless, the test tube contains KCl. [1]

OR
Step 1: To 1 cm3 of each reagent in separate test tubes, add a few drops
of acidified sodium chlorate(I) OR Cl 2 (aq), followed by 1 cm3 of
hexane.Shake well and leave to stand. [1]
If both layers remained colourless, the test tube contains KCl.
If the aqueous layer turned yellow-orange, while the organic layer turned
reddish-brown, the test tube contains KBr.
If the aqueous layer turned brown, while the organic layer turned violet,
the test tube contains KI. [3obs correct:2m; 2 obs correct:1m]

4 (c) (i) 2HF + H 2 O H 3 O+ + HF 2 [1]


K K1 K 2 (1.1 10 3 )(2.6 10 1 ) 2.86 10 4 [1]

(ii) NaOH, Na 2 CO 3 , etc. Base reacts with acid, lowers [H 3 O+] and shifts
position of equilibrium forward. [2]

(d) (i) hydrogen bonding




H F H F H F

trendyline
tr
re dy i
re
R
OR

H F H F H F
hydrogen bonding
en
[1] x 2

10
383 trendyline
(ii) [H F+] = [HF ] = (8.0 10 12 ) 2.83 10 6 mol dm3
2 2 [1]
1.002 10
3
[HF] = 50.1mol dm3
1.0 19.0

The auto-dissociation causes a negligible change in [HF], hence the


[H2F ] 2.83 106
requested fraction is 5.65 108
[HF] 50.1 [1]

5 (a) (i) O
Cl
H2N
First intermediate: [1/2]
O
Cl
Second intermediate: H2N [1/2]

Product formed is from the first intermediate which is less stable than
second intermediate. [1/2]

This is because the positive charge is intensified by electron


withdrawing chlorine through inductive effect. [1/2]

(ii) Boltzmann distribution diagram: [1]

No. of Proportion of molecules with KE Ea at T1


T1
molecules
/proportion T2
of + Proportion of molecules with KE Ea at T2
molecules /
percentage T2 > T1
of
molecules

0 energy
Ea

At higher temperatures, the proportion of molecules with kinetic


energy greater than or equal to activation energy increases [1].

Thus, frequency of effective collision increases [0.5], reaction rate


increases [0.5]

(iii) The prolinamide is synthesised as a racemic mixture containing

trendyline
qual
ual proportions of two enantiomers. [1
equal [1/2]

ence
nce only 50% of the synthesisedprolinamide can fit into / would
Hence
ossess
ssess the correct 3D shape to bind to the ac
possess active site of the
nzyme. [1/2]
enzyme.

2016 HCI C2 H2 Chemistry Preliminary Exam / Paper 2


384 trendyline
5 (b) (i) O O
+
H3N CHC O H3N CHC O
CH2 CH2
C O C O
pH 3: OH [1] pH 8: O [1]

(ii)
aspartic acid lysine Ionic interaction
isoleucine phenylalanine dispersion forces / van der
Waals interaction
serine asparigine Hydrogen bond
3 correct = [2], 2 correct = [1], 1 correct = [1/2]

(iii) Mercury ion disrupts the disulfide bridge by binding tightly with SH
of cysteine residue
OR
Mercury ion disrupts the ionic interaction by forming ionic
interactionwith CO 2 side chain of aspartic acid. [1]

The tertiary structure / shape of the enzyme is altered, and hence its
function is lost. [1]

5 (c) Asp-lys-ser-phe-lys-gly-cys-asn-phe [1]

(d) (i) Glycine [1/2], cysteine [1/2]

(d) (ii) O O
S OH
H3N OH
NH2
H3N OH
O [1] each

trendyline
11
385 trendyline
HWA CHONG INSTITUTION
C2 Preliminary Examinations
Higher 2

CANDIDATE
CT GROUP 15S
NAME

CENTRE INDEX
NUMBER NUMBER

CHEMISTRY 9647/03
Paper 3Free Response 19September 2016
2 hours
Candidates answer on separate paper.
Additional Materials: Answer Paper
Data Booklet

INSTRUCTIONS TO CANDIDATES
Write your name and class on all the work you hand in.
Write in dark blue or black pen.
You may use a soft pencil for any diagrams, graphs or rough working.
Do not use staples, paper clips, highlighters, glue or correction fluid.

Answer any fourquestions.

Begin each question on a newpiece of paper.

A Data Booklet is provided.


You are reminded of the need for good English and clear presentation in your answers.

The number of marks is given in brackets [ ] at the end of each question or part question.
At the end of the examination, fasten all your work securely together.

Write down the question numbers for the questions attempted on the cover page
provided.

trendyline
d li
This document consists of 11 printed pages.

386 trendyline
2

Answer any four questions.

1 (a) Suggest and explain how the boiling points of 1-chlorobutane and 1-bromobutane
differ from each other. [2]

(b) (i) Define the term standard enthalpy change of combustion. [1]

(ii) Use of the Data Booklet is relevant to this question.

In an experiment to determine the enthalpy change of combustion of


bromobutane, a quantity of the fuel was burnt underneath a copper can
containing 200 g of water. It was found that the temperature rose by 45.0 C after
2.35 g of bromobutane had been burnt. The heat transfer was known to be only
80% efficient.

Calculate the enthalpy change of combustion of the bromobutane. Ignore the heat
capacity of the copper can. [2]

(c) An experiment was set up to study the reaction between 2-bromo-3-methylbutane and
sodium hydroxide. The reagents used were FA1,containing 0.0100 mol dm3 of
2-bromo-3-methylbutane and FA2, containing 0.0100 mol dm3 NaOH. In each
experiment, the reagents were dissolved in a suitable solvent to ensure good mixing.
The results are recorded in the table below.

Expt Volume of FA1 Volume of FA2 Volume of solvent Rate / mol dm3 s1
/ cm3 / cm3 / cm3
1 10.0 30.0 60.0 7.40 x 10-7
2 5.0 20.0 25.0 9.86 x 10-7
3 10.0 5.0 35.0 4.93 x 10-7

(i) Use the data in the table to deduce the order of reaction with respect to
2-bromo-3-methylbutane and sodium hydroxide. Hence, write a rate equation for
this reaction. [3]

(ii) With reference to your answer to (c)(i), describe the mechanism for the reaction
between 2-bromo-3-methylbutane and sodium hydroxide. [3]

(iii) Calculate the rate constant for the reaction, stating the units. [2]

trendyline
Hw
Hwa
a Chong Institution 2016 9647 / 03 / C2 Prelim 2016
387 trendyline
3

(d) In a separate experiment, a student performed the following qualitative analysis tests
on three different bromoalkanes: 1-bromobutane, 2-bromobutane and 2-bromo-2-
methylpropane. Different solvents were used for Test 1 and Test 2 and it was
observed that the choice of solvents did have an effect on the type of reaction
mechanisms undergone by the bromoalkanes.

Test 1:

When silver nitrate was added to the bromoalkanes using ethanol as the solvent,
silver bromide was precipitatedfor all three bromoalkanes at different rates.

AgNO 3 + RBr + C 2 H 5 OH ROC 2 H 5 + HNO 3 + AgBr

Test 2:

When sodium iodide in propanone solvent was added to the bromoalkanes,


sodium bromide was precipitated for 1-bromobutane and 2-bromobutane at different
rates, but not 2-bromo-2-methylpropane.

NaI + RBr RI + NaBr

The observations for each test were recorded in the following table.

Time taken for precipitate to appear


Test 1 Test 2
1-bromobutane 5 min Almost immediately
2-bromobutane 3 min 5 min
2-bromo-2-methylpropane Almost immediately No precipitate

(i) Different nucleophilic substitution mechanisms have been proposed for the
reactions in the two tests.

Considering the information provided, suggest the predominant mechanism


undergone in each test. [1]

(ii) Considering the structure of the bromoalkane, explain why the test results support
your answer in (d)(i). [2]

(iii) Considering the interactions involved with the solvent, explain why the suggested
mechanism for Test 1 occurs predominantly. [1]

(e) 2-methylbenzonitrile can be used as a starting reagent to form compound A. Propose


a synthesis route for the formation of compound A in no more than 3 steps, showing
clearly the reagents, conditions and structures of the intermediates formed.

CH3

NH

trendyline
d li
N
CN
C

2-methylbenzonitrile
benzonitrile Compound A
[3]

[Total: 20]

Hw
Hwa
a Chong Institution 2016 9647 / 03 / C2 Prelim 2016
388 trendyline
4

2 Ethanoic acid behaves as a weak acid in water, with K a = 1.8 x 105 mol dm3.
In contrast, ethanoate ion behaves as a weak base in water, withK b = 5.6 x 1010 mol dm3.

(a) (i) Write the expressions for K a and K b of ethanoic acid and ethanoate ion
respectively, and use these expressions to show that the product K a K b has a
constant value at a fixed temperature. [2]

(ii) Compare the relative strength of ethanoate ion and water as bases, and explain
with the aid of an equation, whether a solution of sodium ethanoate is acidic,
alkaline, or neutral. [2]

(iii) Calculate the pH of a solution which contains 0.50 mol dm3 ethanoic acid and
0.50 mol dm3 sodium ethanoate. [1]

(b) The Strecker synthesis is a route to prepare amino acids. Glycine


(2-aminoethanoic acid) can be prepared from methanal as shown in the reaction
scheme below.
elimination NH2
O of H2O NH
NH3 HCN
C A C H C C N
reaction I reaction II reaction III
H H H H H

reaction IV

NH2
O
H C C
OH
H
glycine

(i) Suggest a structure for compound A. [1]

(ii) The product of reaction II is an imine, which has a C=N bond. Methanal
undergoes reaction with another compound B to form an orange solid C which
also has a
C=N bond. Name compound B and draw the structure of the orange solid C. [2]

(iii) What type of reaction is occurring duringreaction III? [1]

(iv) State the reagents and conditions for reaction IV. [1]

(v) Compound D is an isomer of glycine. Upon warming D with dilute sodium


hydroxide, a gas which turns litmus blue is evolved. Draw the structure of D. [1]

(vi) A carbonyl compound E can be used as the starting material to prepare another
amino acid, alanine (2-aminopropanoic acid) using the Strecker synthesis. Draw
the structure of E. [1]

trendyline
te
State
i (b)(vi)
(vii) The synthesis of alanine in (b)(vi produces a mixture of two optical isomers.
e which one of the reactions, I to IV
yourr choice briefly.
V, gives rise to this mix
IV, mixture, and explain
[2]

Hw
Hwa
a Chong Institution 2016 9647 / 03 / C2 Prelim 2016
389 trendyline
5

(c) In organic chemistry, the number of bonds that a given carbon atom has with
electronegative atoms like O or N is called the functional group level of that carbon
atom. Some examples are shown below, but there are many other possibilities.

Functional group level


1 2 3

R R
O N O O
C
C C C C
R
OH
R R H R R R OH R OCR3

(R = alkyl or aryl group or H atom)

(i) State the functional group level of the carbon atom in methane and in carbon
dioxide. [1]

Keeping track of the functional group level can be used to understand a large number
of organic reactions.
During oxidation reactions, the functional group level increases as CH bonds are
converted into new CO bonds.
During reduction reactions, the functional group level decreases as CO bonds are
converted into new CH bonds.
During hydrolysis reactions, the functional group level of every carbon remains the
same.
In each of the reactions in (c)(ii) and (c)(iii), no carbon-carbon bonds are broken or
formed.

(ii) Ethyl butanoate undergoes the following reaction to form two products. State the
functional group level of the circled carbon atom at the start and end of the
reaction, and hence deduce whether the reaction is an oxidation, a reduction, or a
hydrolysis reaction.

O
C2H5OH
OH
OC2H5
ethyl butanoate [2]

(iii) Gyromitrin, a toxin found in fungi, undergoes hydrolysis to give three different
products, F, G, and H. Suggest structures for these products using their relative
molecular masses (shown in brackets) and the functional group levels of the
carbon atoms.

O
N F G H
N H

trendyline
r
(58) (46) (46)

(114)
114
gyromitrin [3]

[Total: 20]

Hw
Hwa
a Chong Institution 2016 9647 / 03 / C2 Prelim 2016
390 trendyline
6

3 Use of the Data Booklet is relevant to this question.

In 2016, 4 new elements were added to the Periodic Table. Part of the Periodic Table
showing information about the 4 new elements is shown below.
Key
relative atomic mass
Nh Mc Ts Og atomic symbol
nihonium moscovium tennessine oganesson name
113 115 117 118 proton number

Mc, Ts, and Og were synthesised by accelerating calcium ions into atoms of heavier
elements. Once in every few billion collisions, the nuclei of the 2 particles would fuse and
give rise to the new element. For example, Ts was synthesised using berkelium ( 97 Bk).

(a) (i) State the heavy element used in the synthesis of Mc. [1]

(ii) Based on the relative position of Ts in the Periodic Table, predict the physical
state of Ts at room temperature. Give a brief explanation for your answer. [2]

(iii) Scientists have predicted the existence of superheavy elements above proton
number 120 that are stable under standard conditions. One such element, if
successfully synthesised in large quantities, would give rapid effervescence of a
colourless gas in water, and its oxide would be basic in nature. The sulfate
compound of the element would be insoluble in water.

State, with reasons, the group of the Periodic Table that this element is likely to
belong. [2]

(b) Scientists are already working on the next element of proton number 119 using
berkelium and titanium.

(i) Explain, using relevant electronic configurations, why titanium is considered a


transition element but not zinc. [2]

(ii) Tetrahydrofuran(THF), M r = 72.1, is a simple covalent molecule that behaves as


a monodentate ligand. Titanium forms a complex with zero net charge with THF
and chloride ligands. A 1.00 g sample of the complex was completely hydrolysed
in water and filtered. The residue contained 2.99 103 mol of TiO 2 . The filtrate
yields 1.72 g of silver chloride after treating with aqueous silver nitrate. Determine
the coordination number of the central titanium of the complex. [3]

trendyline
Hw
Hwa
a Chong Institution 2016 9647 / 03 / C2 Prelim 2016
391 trendyline
7

Calcium and titanium are both metals but differ in their chemical and physical properties.

(c) (i) Compounds of calcium contain the Ca2+ ion. The corresponding ion of titanium is
Ti3+. Use relevant data from the Data Booklet to explain why Ca3+ compounds do
not exist and Ti3+ compounds do. [3]

(ii) Draw labelled diagrams to show the shapes of the orbitals in the outermost
occupied quantum shell of a Ca2+ ion. [2]

(iii) Explain why the melting point of titanium is higher than that of calcium. [2]

(iv) Describe the observations when calcium is burned in air, and excess water is
added to the resulting oxide. Write equations where appropriate and suggest the
pH value of the aqueous solution formed.
[3]

[Total: 20]

trendyline
Hw
Hwa
a Chong Institution 2016 9647 / 03 / C2 Prelim 2016
392 trendyline
8

4 Transition metal catalysed reactions have emerged in recent years as powerful tools for
preparing organic compounds from previously unavailable routes. Ruthenium-based
catalysts have been at the heart of these advances.

(a) The ruthenium(II) based catalyst RuH 2 (PPh 3 ) 4 , where PPh 3 = P(C 6 H 5 ) 3 , is able to
catalyse the formation of an amide under suitable conditions, as shown in the scheme
below.
O
RuH2(PPh3)4
NH2 OH
N
H

(i) Draw the structure of the product for the reaction below.

RuH2(PPh3)4
OH
H2N C4H7NO [1]

(ii) RuH 2 (PPh 3 ) 4 also catalyses the formation of the amide shown in the reaction
scheme below. Draw the structures of compounds A and B.

CH3
RuH2(PPh3)4 O
A+B
N
H
[2]

(iii) Propose a synthetic route for the formation of compounds A and B, using only
methylbenzene as the starting compound in each case. [3]

(b) The catalyst RuH 2 (PPh 3 ) 4 exhibits geometric isomerism. In the cis configuration, the
H-Ru-H angle is 90, while in the trans configuration, the H-Ru-H angle is 180.

Draw labelled diagrams showing the structures of the two isomers. [2]

(c) In addition to RuH 2 (PPh 3 ) 4 , other ruthenium complexes like the one shown below,
also proved to be effective in organic synthesis.

H
N N
Ru
P P
Ph Ph Ph Ph

State the oxidation number of ruthenium in the complex above. [1]

trendyline
Hw
Hwa
a Chong Institution 2016 9647 / 03 / C2 Prelim 2016
393 trendyline
9

(d) The complex ion tris(bipyridine)ruthenium(II) is a catalyst which has received much
attention because of its ability to exhibit optical isomerism. Two of its isomers are
shown below.
2+ 2+

N N
N N N N
Ru Ru
N N N N
N N

(i) Explain why the complex above exhibits optical isomerism. [1]

(ii) Tris(bipyridine)ruthenium(II) cation contains three bipyridine rings, which can be


represented by the symbol, bipy. The formation of tris(bipyridine)ruthenium(II)
from aqueous ruthenium(II) ions is shown in the equation below. The enthalpy
change for this reaction is approximately zero.

[Ru(H 2 O) 6 ]2+(aq) + 3bipy(aq) [Ru(bipy) 3 ]2+(aq) + 6H 2 O(l)

Deduce the sign of the entropy change,S, and hence the sign of the free energy
change, G, in this reaction. [3]

Iron is directly above ruthenium in the transition metal block of the Periodic Table.

(e) Ligands are able to modify the difference in energy between the non-degenerate
d-orbitals in an octahedral crystal field.

The following table lists the different colours of visible light and their corresponding
wavelengths. The energy of light is inversely proportional to its wavelength.

Colour violet blue green yellow red


Wavelength / nm 400 450 500 600 650

The equation below represents the hydrolysis of Fe(III) ions in water. The pale yellow
colour of Fe(III) solutions observed commonly is actually due to the presence of the
complex ion [Fe(H 2 O) 5 (OH)]2+, while unhydrolysed [Fe(H 2 O) 6 ]3+ is pale purple in color.

[Fe(H 2 O) 6 ]3+(aq) + H 2 O(l) [Fe(H 2 O) 5 (OH)]2+(aq) + H 3 O+(aq)


pale purple yellow

(i) With reference to the colours of the two complexes above, explain which complex
ion has a larger energy gap between their d-orbitals. [2]

(ii) Electrons in both [Fe(H 2 O) 6 ]3+ and [Fe(H 2 O) 5 (OH)]2+ exhibit a high spin state,
where electrons occupy the d-orbitals singly before starting to pair up in the lower
energy d-orbitals.

trendyline
Draww a suitable diagram to show the electron distribution in th
Fe3++ ion in a high sp
rbital
bital has been promoted to a higher energy d-
d-orbital
certain
in wavelength.
ain wavelength
spin excited
d-orbital
the 3d subshell of a
dstate, where an electron from a lower energy
excitedstate,
bital by ab
d-orbital absorbing light of a

[1]

Hw
Hwa
a Chong Institution 2016 9647 / 03 / C2 Prelim 2016
394 trendyline
10

(f) K 2 FeO 4 is a strong oxidising reagent which is dark red in colour.

FeO 4 2(aq) + 8H+(aq) + 3e Fe3+(aq) + 4H 2 O(l)E = +2.20 V

Using relevant data from the Data Booklet, describe and explain the observations
when the following solutions are mixed together in I and in II respectively.Calculate
the E cell andwrite a balanced equation for any reaction occured.

I K 2 FeO 4 and H 2 O 2
II KMnO 4 and Fe(NO 3 ) 3 [4]

[Total: 20]

Hw
Hwa
a Chong Institution 2016 9647 / 03 / C2 Prelim 2016
395 trendyline
11

5 (a) Ag 2 CO 3 is a sparingly soluble salt. A 100 cm3 saturated solution of Ag 2 CO 3 in water


was found to contain 2.57 10mol dm3 of Ag+ ions.

(i) Calculate the K sp of Ag 2 CO 3 and state its units. [2]

(ii) Hence, calculate the solubility of Ag 2 CO 3 when 2.50 g of Na 2 CO 3 solid was


added to the solution above. [2]

(b) The table below shows the pK a values of benzoic acid and its derivatives.

Acid benzoic acid 2-hydroxybenzoic acid 4-hydroxybenzoic acid


OH

Structure CO2H HO CO2H


CO2H

pK a 4.20 2.98 4.58

Rank the three compounds in order of increasing acid strength and account for the
trend. [3]

(c) Compounds P and Q can be synthesised from 4-hydroxybenzoic acid.

Cl OH
O O
OH OH

HO Cl
P Q

Suggest the reagents and conditions for a reaction that could be used to distinguish
between P and Q. You need to state how each of P and Q react.
[2]

(d) Heating compound A, C 13 H 14 O 3 , under reflux with dilute H 2 SO 4 produces compound


B, C 8 H 8 O, and compound C, C 5 H 8 O 3 .

B reacts with aqueous bromine to give compound D, C 8 H 7 O 2 Br 3 . On heating B with


acidified KMnO 4 , 4-hydroxybenzoic acid and carbon dioxide gas are formed.

When C is heated with acidified KMnO 4 , compound E is the only organic product
formed. Both C and E gave yellow precipitate when warmed with alkaline aqueous
iodine. Treatment of E with aqueous Na 2 CO 3 liberates carbon dioxide gas.

trendyline
(i) p
When vaporised pp , 0.219 g of E occupied
in a suitable apparatus, p a volume of 70
cm3 at 150 C
C and a pressure of 125 kPa.

Calculate
ulate the relative molecular mass of E and hence suggest
culate sugg a molecular
mula
ula
formula for E,
E, given that it contains 3 carbon atoms.
[2]

(ii) Suggest structures for A to E, explain your reasoning. [9]

Hw
Hwa
a Chong Institution 2016 9647 / 03 / C2 Prelim 2016
396 trendyline
12

[Total: 20]

trendyline
Hw
Hwa
a Chong Institution 2016 9647 / 03 / C2 Prelim 2016
397 trendyline
Paper 3

1 (a) 1-bromobutane has a higher boiling than 1-chlorobutane. Both


1-bromobutane and 1-chlorobutane are simple molecular. Since
1-bromobutane has a larger electron cloud size[1], there is stronger
dispersion forces between its molecules as compared to
1-chlorobutane. Hence, more energy[1] is needed to break the stronger
dispersion forces for 1-bromobutane.

(b) (i) Standard enthalpy change of combustion is the heat


evolvedwhenone mole of the substance is completely burnt in
excess oxygen at 298 K and 1 atm. [1]

(ii) Heat absorbed by water q= (200)(4.18)(45.0) = 37620 J


= 37.6 kJ [1]

Enthalpy change of combustion = [100/80 37.62] / (2.35/136.9)


= 2.74 103 kJ mol1[1]

(c) (i) Comparing Expt 1 and 2,


(Total volume in Expt 2 is half that of Expt 1.)
Concentration of 2-bromo-3-methylbutane is constant, when
concentration of NaOHincreases 4/3 times, rate increases 4/3 times.
Hence, the reaction is 1st order with respect to NaOH[1]

Comparing Expt 2 & 3,


(Total volume in Expt 2 and 3 are the same)
When concentration of 2-bromo-3-methylbutane increases 2 times
and concentration of NaOHdecreases 4 times, the rate decreases 2
times. Since the reaction is first order with respect to NaOH, the
reaction is thus 1st order with respect to 2-bromo-3-methylbutane[1]

[Calculation method also accepted]

Rate = k [2-bromo-3-methylpentane][NaOH] [1] ecf

(ii) S N 2 Nucleophilic substitution [1]

trendyline
2016 HCI C2 H2 Chemistry Preliminary Exam / Paper 3
398 trendyline
CH(CH3)2 CH(CH3)2 CH(CH3)2

HO C Br HO C Br HO C Br
+
H H
H CH3 CH3
CH3
2m for mechanism

(iii) Rate = k [2-bromo-3-methylbutane][NaOH]

To obtain the numerical value for k, substitute the rate and the
respective concentrations into the rate equation.

10 30
(0.0100) (0.0100)
7.40 10-7 = k 1000
10.0+30.0+60.0 1000
10.0+30.0+60.0

1000 1000

k = 0.247 [1] Calculation

To derive the units,


mol dm3 s1 = k (mol dm3)2
k = mol1 dm3 s1[1] Units

(d) (i) Test 1: S N 1 mechanism


Test 2: S N 2 mechanism [1]

(ii) In Test 1, the rate of reaction is the fastestfor thetertiary


bromoalkane. The carbocation formed from the tertiary bromoalkane
is stabilised by three electron-donating alkyl groups, hence it
undergoes S N 1 rapidly. [1]

In Test 2, the rate of reaction is the fastestfor theprimary


bromoalkane asthere is least steric hindrance when the nucleophile
attacks the electron deficient carbon atom of the primary
bromoalkane, hence it undergoes S N 2 rapidly. [1]

(iii) The carbocation formation is favoured as ethanol can form ion-dipole

t
trendyline
interactions with the carbocation intermediate therebyy stabilising it
].
[1].

1
399 trendyline
(e)
CH3 CH2Cl
LiAlH4 in
Cl2(g), UV dry NH
ether
[1] [1]
CN CN
[1]

For the second step, accept H 2 , Ni, high temp and high pressure. Also
accept if split up last step to show reduction followed by nucleophilic
substitution with heat for last step.

2 (a) (i) [CH 3 CO 2 ][H+]


Ka =
[CH 3 CO 2 H]
[CH 3 CO 2 H][OH]
Kb =
[CH 3 CO 2 ]
[1]

[3 2 ][ + ] [3 2 [] ]
K a K b = x
[3 2 ] [3 2 ]
= [H+] [OH]
= K w (ionic product of water), which is a constant. [1]

(ii) CH 3 CO 2 H is a weak acid so its conjugate base CH 3 CO 2 is a


stronger base than water.
So CH 3 CO 2 undergoes hydrolysis forming OH ions, forming an
alkaline solution.[1]
CH 3 CO 2 (aq) + H 2 O (l) CH 3 CO 2 H (aq) + OH (aq) [1]

(iii) As [CH 3 CO 2 H] = [CH 3 CO 2 ], this is a maximum capacity buffer;


so pH = pK a
ORJ [5 )
= 4.74 [1]

(b) (i) A[1]

OH
NH2
C

ttrendyline
H H

(ii) B = 2,4
2,4-dinitrophenylhydrazine
2,4--dinitrophenylhydrazine
nitrophenylhydrazine [1]

2016 HCI C2 H2 Chemistry Preliminary Exam / Paper 3


400 trendyline
C [1]
NO2
H H
C N N NO2
H

(iii) addition [1]

(iv) Dilute H 2 SO 4 / HNO 3 / HCl, heat [1]

(v) D [1]
OH
O
H C C
NH2
H

(vi) E [1]
O

C
CH3 H

(vii) Reaction III[1]


The imine is trigonal planar about the imine carbon atom. The
nucleophile can thus attack this + carbon atom in equal probability
from the top as well as the bottom[1], resulting in forming equal
proportions of the two optical isomers / racemic mixture.

(c) (i) Functional group level of C in CH 4 = 0; in CO 2 = 4


[1]

(ii) Functional group level of circled C at the start = 3; at the end= 1


[1]
As the functional group level decreases, the reaction is reduction. [1]

(iii) H
O
N H
H N
O HO H
CH3

trendyline
F G/H
1m
m each

2
401 trendyline
3 (a) (i) [1] Americium (Am)

(ii) Ts lies below astatine in Group VII, following the trend of increasing
melting point down the group[1], it should be a solid [1] like iodine or
astatine.

(iii) [1] Group II.

[0.5]: Effervescence of hydrogen gas with water => Group I or II


metal
or
Basic oxide hydrolyses to give metal hydroxide in water => Group I or
II metal
[0.5]: Sulfate insoluble => Group II metal as Group I sulfates all
soluble. Or Group II sulfate solubility decreases down the group.

(b) (i) Titanium is able to form ions with incomplete d-subshell, Ti2+:
1s22s22p63s23p63d2 or Ti3+: 1s22s22p63s23p63d1[1]
but
zinc forms only the Zn2+ ion which has 1s22s22p63s23p63d10
configurationwith a fully filled d-subshell [1].

(ii) No. of moles of titanium = no. of moles of TiO 2 = 2.99 103 mol
No. of moles of chloride = 1.72 / 143.5 = 0.0120 mol
Ratio of chloride ions to titanium = 0.0120 / 2.99 103 = 4.01 = 4 [1]
1st mark can be awarded if student is able to provide an answer that
clearly recognises from TiO 2 that the complex is formed from a Ti(IV)
center [0.5] and since complex is electrically neutral, there must be 4
chloride ions [0.5]

Mass of titanium and chloride in 1 g sample = 2.99 103 189.9 =


0.568 g
Mass of coordinated THF molecules in 1 g sample = 0.432 g
0.432 / (72.1 n) = 2.99 103
No. of THF molecules coordinated = 0.432 / (2.99 103 72.1) =
2.00 = 2 [1](working forno. of THF ligands must be shown)

Coordination number = 6 [1](this answer must be based on an actual


no. of chloride and THF ligands stated earlier)

Allow ecf from errors in number or chloride and THF ligands


calculated, rounded up to whole numbers.

trendyline
tren
rendyline
ylin
y
(c) (i) he 3rd ionization energy of Ca (+4940
The (+4940 mo 1) is much
4940 kJ mol m higher than
nd
d 1
1 rd
d
the
e 2 ((+1150
1150 kJ mol )as the 3 electronlectron in Ca is removed
re from an
inner
ner
er quantum sh shell
shell[1]
[1] which
whic requires a lot of eneenergy. Thus, Ca3+
compounds
mpounds do not exist.

2016 HCI C2 H2 Chemistry Preliminary Exam / Paper 3


402 trendyline
For Ti, the 2nd and 3rd ionisation energies (+1310, +2720 kJ mol1) do
not differ greatly as 4s and 3d electrons are close in energy[1]. Thus,
Ti3+ can be formed.

[1]: some comparison/discussion of ionisation energies for Ca and Ti


pertaining to the high 3rd ionisation energy for Ca compared to Ti.

(ii)
z
y

x
3px
3s 3px 3py
3py 3pz
3s 3pz

[2]

(iii) In calcium, only the 4s electrons are donated to the sea of


delocalised electrons, while in titanium, both the 4s and 3d electrons
are involved (idea of more electrons contributed towards metallic
bonding). OR Titanium cations have higher charge/ smaller ionic
radius / greater charge density than calcium cations. [1]

This leads to titanium having stronger metallic bond strength and


more energy is needed to overcome the metallic bonds[1] and hence
a higher melting point.

(iv) Burns slowly with brick-red flame , leaving a white residue [1]
2Ca(s) + O 2 (g) 2CaO(s)
CaO(s) + H 2 O(l) Ca(OH) 2 (aq) [1]
Ca(OH) 2 dissolves / gives a colorless solution in water to give an
alkaline solution of pH = 12[1] (Accept any value above 7)

4 (a) (i) O

HN
[1]

(ii) CH2OH

[1] [1]

NH2

trendyline
3
403 trendyline
.
(iii) 1 Sn, conc HCl, heat
conc HNO3 .
2 NaOH (aq)
conc H2SO4
30C [1]
. NO2
[0 5] NH2

CO2H LiAiH4 CH2OH


KMnO4 (aq) dry ether
H2SO4 (aq)
. [1]
heat [0 5]

accept Cl 2 (g), UV or heat, followed NaOH (aq), heat to get


phenylmethanol.

(b) H H
PPh3 H PPh3 PPh3
Ru Ru
PPh3 PPh3 PPh3 PPh3
PPh3 H
cis trans [1] x 2

(c) +1 [1]

(d) (i) The complex is chiral as the two optical isomers are non-
superimposable mirror images of each other. [1]

(ii) S is positive[0.5] as the number of product particles is greater than


the number of reactant particlestherefore the number of ways to
distribute energy in the system increases.[1]
G H 7S.
$VH DQGG = 7S[1]7KXVG is negative [0.5]

(e) (i) The wavelength of light (complementary of yellow) absorbed by


[Fe(H 2 O) 5 (OH)]2+ is of a lower wavelength than that for
[Fe(H 2 O) 6 ]3+[1]. Therefore, the energy gap between the d-orbitals
must be larger in [Fe(H 2 O) 5 (OH)]2+[1].

(ii) Energy

trend
trendyline
[1]
2016 HCI C2 H2 Chemistry Preliminary Exam / Paper 3
404 trendyline
(f) (I)
O 2 + 2H+ + 2e H 2 O 2 +0.68 V
E cell = +2.20 (0.68)
= +1.52 V > 0 reaction is feasible
2
2FeO 4 + 10H + 3H 2 O 2 2Fe3+ + 8H 2 O + 3O 2
+

Observations: Red solution turns pale yellow, as FeO 4 2is reduced to


Fe3+.Effervescence of O 2 OR effervescence of a colourless and odourless
gas that rekindles a glowing splint. [3]

(II)
MnO 4 + 8H+ + 5e Mn2+ + 4H 2 O +1.52 V
E cell = +1.52 (2.20) <0 reaction is not feasible
Solution remains purple [1]

5 (a) (i) K sp = [Ag+]2[CO 3 2]


= (2.57104)2(2.57104/2) = 8.49 1012 mol3 dm9

[1m for numerical answer; 1m for units]

(ii) K sp = [Ag+]2[CO 3 2]
8.49 1012 = [Ag+]2 (2.50 (23 2 + 12 + 16 3) (100/1000))
[Ag+] = 6.00 106 mol dm-3[1]
Solubility of Ag 2 CO 3 = 6.00 106 = 3.00 106 mol dm-3[1]

(b) 4-hydroxybenzoic acid, benzoic acid, 2-hydroxybenzoic acid [1]

4-hydroxybenzoic acid is a weaker acid than benzoic acid because the


OH group attached to the benzene ring is electron-donating (lone pair of
electrons on O atom is delocalised into the benzene ring), hence
intensifying the negative charge on the carboxylate anion and reducing its
stability.[1]

2-hydroxybenzoic acid is a stronger acid than benzoic acid because its


conjugate base can be stabilised by intramolecular hydrogen bonding/ion-
dipole interactions. [1]
(c) Add AgNO 3 (aq) at room temperature [1]
Q : White ppt of AgCl formed immediately
P: No white ppt formed[1]

trendyline
OR

Add K 2 Cr 2 O 7 , dilute H 2 SO
O 4 , heat [1]
[1]
nge K 2 Cr 2 O 7 turns
Q : Orange urns green
P: Orange
nge K 2 Cr 2 O 7 remains orange e [1]
[1

4
405 trendyline
OR

Add water [1]


Q: Dense white fumes evolved
P: No white fumes evolved[1]

(d) (i) PV = nRT


Mr = (mass R T) / PV
(0.219 8.31 423) / (125 x 103 70 10-6) = 88.0[1]
C 3 H 4 O 3 [1]

(ii) A, C 13 H 14 O 3 , and B, C 8 H 8 O have high C:H ratioBenzene ring


present in A and B

A undergoes acidic hydrolysis with dilute H 2 SO 4 to form B and CA


contains an ester group

B undergoes electrophilic substitution and electrophilic addition with


aqueous bromine to form D, C 8 H 7 O 2 Br 3 B contains phenol and
alkene respectively.

B undergoes oxidative cleavage with acidified KMnO 4 to form 4-


hydroxybenzoic acid and CO 2 in a 1:1 mole ratio B contains a
terminal alkene

C undergoes oxidative cleavage with acidified KMnO 4 to form E,


C 3 H 4 O 3 , and carbon dioxide gas in a 1:2 mole ratio Ccontains
alkene and possibly forms ethanedioic acid (further oxidised to form 2
CO 2 ) upon oxidation

C and E undergo oxidation / forms CHI 3 yellow ppt / gave positive


iodoform test with warm aqueous alkaline iodine
C contains CO(CH3) or CH(CH 3 )(OH) group or E contains
CO(CH3) group

E undergoes acid-base reaction with Na 2 CO 3


E contains a carboxylic acid group
O

OH
O

ttrendyline
r dy
dyliine
OH

HO
O

A: B: C: OH

2016 HCI C2 H2 Chemistry Preliminary Exam / Paper 3


406 trendyline
OH

Br Br

HO
Br
D: HO E: O

4m max for explanations


1m for each structure
[9m]

trendyline
5
407 trendyline
INNOVA JUNIOR COLLEGE
JC 2 PRELIMINARY EXAMINATION
in preparation for General Certificate of Education Advanced Level
Higher 2

CANDIDATE
NAME

CLASS INDEX NUMBER

CHEMISTRY 9647/01
Paper 1 Multiple Choice 30 August 2016
1 hour
Additional Materials: Data Booklet
Multiple Choice Answer Sheet

READ THESE INSTRUCTIONS FIRST

Write your name and class on all the work you hand in.
Write in soft pencil.
Do not use staples, paper clips, highlighters, glue or correction fluid.

There are forty questions on this paper. Answer all questions. For each question there are four
possible answers A, B, C and D.
Choose the one you consider correct and record your choice in soft pencil on the separate
Answer Sheet.

Read the instructions on the Answer Sheet very carefully.

Each correct answer will score one mark. A mark will not be deducted for a wrong answer.
Any rough working should be done in this booklet.

trendyline
This
y
Thi document
d t consists
i t off 17 printed
i t d pages and
d 1 blank
bl k page.

InnovaJunior College [Turn over


408 trendyline
2
Section A

For each question there are four possible answers, A, B, C, and D. Choose the one you consider
to be correct.

1 Carbon disulfide, CS 2 , is a volatile liquid used in the production of cellophane which is used
for food packaging.

On combustion, CS 2 is oxidised as follows:

CS 2 (g) + 3O 2 (g) CO 2 (g) + 2SO 2 (g)

A 20 cm3 sample of carbon disulfide vapour is ignited with 100 cm3 of oxygen. The final
volume of gas after burning is treated with an excess of aqueous alkali.

Which percentage of this final volume dissolves in alkali? (All volumes are measured at
room temperature and pressure.)

A 20%
B 40%
C 60%
D 80%

2 Consider the following half-equations

C 2 O 4 2 2CO 2 + 2e
Fe2+ Fe3+ + e
MnO 4 + 8H+ + 5e

Mn2+ + 4H 2 O

What volume of 0.01 mol dm3 potassium manganate(VII) is needed to completely oxidise
25.0 cm3 of an acidified solution of 0.01 mol dm3 FeC 2 O 4 ?

A 5 cm3
B 7.5 cm3
C 10 cm3
D 15 cm3

3 Which electronic configuration represents an element that forms a simple ion with a
chargeof 3?

A 1s22s22p63s23p1
B 1s22s22p63s23p3

trendyline
C 1s22s22p
p63s23p63d14s2
D 1s22s22p
p63s
s23p63d34s2

IJC 2016 Prelim/9647/01 [Turn over


409 trendyline
3

4 Carbon dioxide is a gas at room temperature while silicon dioxide is a solidbecause

A carbon dioxide contains double covalent bonds while silicon dioxide contains single
covalent bonds.
B instantaneous dipole induced dipole attractions are weaker than permanent dipole
permanent dipole attractions.
C carbon-oxygen bonds are less polar than silicon-oxygen bonds.
D van der Waals forces are much weaker than covalent bonds.

5 Under which conditions will the behaviour of a gas be most ideal?


pressure temperature
A high high
B high low
C low high
D low low

6 The enthalpy changes involving some oxides of nitrogen are given below:
N 2 (g) + O 2 J 12 J H = +180 kJ mol1
1
2NO 2 (g)+ O 2 J 1 2 O 5 (g) H = 55 kJ mol1
2
5
N 2 (g) + O 2 J 1 2 O 5 (g) H = +11 kJ mol1
2

What is the enthalpy change, in kJ mol1, of the following reaction?

2NO(g) + O 2 J 12 2 (g)

A 114 B +114 C 136 D +136

7 A typical protein forms hundreds of hydrogen bonds and thousands of van der Waals forces
in folding from primary to tertiary structures.
Which of the following thermodynamic state functions of the protein best represents the
folding process?

G / kJ mol1 H / kJ mol1 S / J K1 mol1

trendyline
e y e
B + +

C +

D + +

IJC 2016 Prelim/9647/01 [Turn over


410 trendyline
4
8 When 1 mole of carbon dioxide gas solidifies as dry ice, 25.2 kJ of heat energy is
evolved.The sublimation temperature of carbon dioxide is 78.5 C.

What is the entropy change when 132 g of carbon dioxide gas solidifies at this
temperature?

A +130 J K1
B 130 J K1
C +389J K1
D 389 J K1

9 A student set up the hydrogen electrode shown in the diagram below.

What would have to be changed to make this a standard hydrogen electrode?

A the acid solution used


B the temperature of the gas and of the acid solution
C the pressure of the gas
D the metal comprising the electrode

trendyline
IJC 2016 Prelim/9647/01 [Turn over
411 trendyline
5

10 Use of the Data Booklet is relevant to this question.

Using inert electrodes, a current was passed through two beakers containing aqueous
silver sulfate and aqueous copper(II) nitrate, connected in series under standard
conditions.
A

At 25oC, 1 atm

1.00 mol dm- 0.50 mol dm-


3 3
Cu(NO3)2(aq) Ag2SO4(aq)

What is the ratio of the mass of silver to copper deposited after the current was passed for
t minutes?

A 0.59 B 0.85

C 1.70 D 3.40

11 A nitrogen-hydrogen mixture, initially in the mole ratio of 1:3, reached equilibrium with
ammonia when 50% of the nitrogen had reacted. The total equilibrium pressure was p.

N 2 (g) + 3H 2 (g) 2NH 3 (g)

What was the partial pressure of ammonia in the equilibrium mixture?

A p B p C p D 2p
3 4 6 7

12 Solid calcium hypochlorite pellets, Ca(ClO) 2 (s), are added to swimming pools to form
HClO(aq), which kills disease-causing bacteria and algae.


Ca(ClO) 2 (s) Ca2+(aq) + 2ClO (aq)


ClO (aq) + H 2 O(l) HClO(aq) + OH (aq)

What is the effect on the solubility of calcium hypochlorite and bacterial growth when pH
decreases?

solubility of calcium hypochlorite effect on bacterial growth

C
ttrendyline
trend
dyline
li
decreases
ecrease
decreases

creases
decreases

increases
more favourable
more favourab

ess favourable
less

more favourable

IJC 2016 Prelim/9647/01 [Turn over


412 trendyline
6

D increases less favourable


13
The dissociation constant, K w , for the ionisation of water, H 2 O H+ + OH , at different
temperatures is given below.

temperature / C Kw
0 1.15 x 10-15
25 1.00 x 10-14
50 5.50 x 10-14

What can be deduced from this information?

A Only at 25 C are [H+]and [OH-] equal.


B The equilibrium lies furthest to the right at 0C.
C The forward reaction is exothermic.
D The pH of water decreases as temperature increases.

14 A sparingly soluble salt, Mg 3 (PO 4 ) 2 dissociates in aqueous solution as follows:


Mg 3 (PO 4 ) 2 3 Mg2+ + 2 PO 4 3-
Given that the solubility product, K sp of Mg 3 (PO 4 ) 2 is Q, what is the value of [Mg2+] in a
saturated solution?

9 1 1 1 9 1
A ( )5 B ( )3 C ( )5 D ( )3
4 27 108 64

trendyline
IJC 2016 Prelim/9647/01 [Turn over
413 trendyline
7

15. The reaction of hydrogen peroxide with iodide ions in an acidic solution is first order with
respect to hydrogen peroxide as well as iodide ions, and zero order with respect to
hydrogen ions.
H 2 O 2 (aq) + 2H+(aq) + 2I(aq) 2H 2 O(l) + I 2 (aq)
Two experiments were carried out. Which pair of diagrams represents the variation of
[H 2 O 2 ] and [H+] with time?

Experiment 1: In the presence of Experiment 2: In the presence of


excess I excess I and H 2 O 2
[H2O2] [H+]

0 time 0 time

[H2O2]
[H+]

0 time
0 time

[H2O2]
[H+]

0 time
0 time

[H2O2] [H+]

0 time 0 time

trendyline
IJC 2016 Prelim/9647/01 [Turn over
414 trendyline
8

16 Aluminium is an element in the third period, Na to Ar, of the Periodic Table.

What is true for aluminium?

A Aluminium has the highest melting point of the elements in this period.
B Aluminium is the only element in this period whose chloride reacts with water to form
an acidic solution.
C Aluminium is the only element in this period which can exist, at room temperature
and pressure, as solid and conduct electricity.
D Aluminium is the only element in this period whose oxide reacts with both acids and
bases.

17 Which of the following statements regarding Group II elements or their compounds is


correct?

A Magnesium chloride has a higher melting point than barium chloride.


B Magnesium is a stronger reducing agent than strontium.
C Calcium hydroxide is more thermally stable than barium hydroxide.
D Strontium reacts more readily with oxygen than radium.

18 Which of the following statements about astatine, the element below iodine in Group VII
of the periodic table, is incorrect?

A Silver astatide is insoluble in aqueous ammonia.


B Hydrogen astatide is less stable to heat than hydrogen iodide.
C Sodium astatide and hot concentrated sulfuric acid react to form astatine.
D Astatine and aqueous potassium chloride react to form aqueous potassium astatide
and chlorine.

trendyline
IJC 2016 Prelim/9647/01 [Turn over
415 trendyline
9

19 When copper(II) chloride is dissolved in water it gives a blue solution. When this solution is
treated with an excess of concentrated hydrochloric acid it turns yellow.

What are the formulae of the copper species in the blue and yellow solution?

blue yellow

A CuCl 2 [CuCl 4 ]2-

B CuCl 2 (H 2 O) 4 [CuCl 6 ]4-

C Cu(OH) 2 CuCl 2 (H 2 O) 4

D [Cu(H 2 O) 6 ]2+ [CuCl 4 ]2-

20 Four carbon-carbon bonds are labelled in the diagram.


w
x y z
CH CH C CH

Which bonds are made up of an sp2sp2 overlap?

A w and y only
B w, x and y only
C w, x, y and z
D x and y only

21 When heated with chlorine, the hydrocarbon 3,3-dimethylpentane undergoes free radical
substitution. In the propagation step, the free radical X is formed by the loss of one
hydrogen atom.

CH3
CH3CH2 C CH2CH3 Cl X HCl

CH3

How many different forms of Xare theoretically possible?


A 2 B 3 C 4 D 5

trendyline
IJC 2016 Prelim/9647/01 [Turn over
416 trendyline
10

22 Bromoethene, CH 2 =CHBr, is unreactive to nucleophiles whereas 3-bromopropene,


CH 2 =CHCH 2 Br is very reactive by comparison.

What explains the lack of reactivity of CH 2 =CHBr?

A Substituted alkenes undergo only electrophilic addition.


B The C=C double bond and the Br atom are both electron withdrawing which
stabilises CH 2 =CHBr.
C The electrons on the Br atom delocalise into the bond.
D The presence of the bond prevents free rotation of the C-Br bond this decreasing
the reactivity.

23 Phosgene, COCl 2 ,is a colourless gas that gained infamy as a chemical weapon during
World War I. It is formed when trichloromethane, CHCl 3 , is left exposed to the atmosphere.
Below is a reaction scheme that involves trichloromethane and phosgene:

CHCl 3 COCl 2 COCl(OH) CO 2 + HCl

Which of the following type of reaction is not being observed in the reaction scheme?

A oxidation
B elimination
C electrophilic addition
D nucleophilic substitution

24 A chlorine-containing organic compound, X, undergoes an elimination reaction when treated


with hot ethanolic potassium hydroxide solution.

What is X?

A CH 2 Cl 2
B C 2 Cl 6
C (CH 3 ) 2 CHCH 2 Cl
D (CH 3 ) 3 CCH 2 Cl

trendyline
IJC 2016 Prelim/9647/01 [Turn over
417 trendyline
11
25 Dopamine is an important organic chemical that helps control the brains reward and pleasure
centres. It also helps regulate movement and emotional responses.

NH2
HO CH2CH
CO2H
HO
dopamine

Which of the following statements about dopamine isincorrect?

A It is able to rotate plane-polarized light.


B It is soluble in water due to zwitterion formation.
C It migrates to the negative electrode of an electrolytic cell at pH 14.
D The acidity of dopamine will increase when aqueous bromine is added.

26 Two bottles containing the following reagents had their labels removed.

CH2CH2OH CH2COOH

Cl Cl Cl CH3

Which one of the following reagents can be used to distinguish between them?

A sodium metal
B sodium hydroxide
C hot acidified potassium manganate(VII)
D hot acidified potassium dichromate

trendyline
IJC 2016 Prelim/9647/01 [Turn over
418 trendyline
12

27 Marijuana (cannabis) is the most widely used illegal drug in many developed countries.
Medical studies have shown that the active ingredient in marijuana, tetrahydrocannabinol,
might provide medical benefits to some patients.

OH

O
tetrahydrocannabinol

In the following reactions, the O remains unaltered.

Which reaction will tetrahydrocannabinol undergo?

A It reacts with aqueous bromine to incorporate up to 2 atoms of bromine in each


molecule.
B It reacts with hot acidified potassium dichromate(VI) to form a green solution.
C It reacts with dilute nitric acid to form a substitution product.
D It reacts with ethanoic acid to form a sweet-smellingproduct.

28 A student carried out an experiment to study the ease of hydrolysis of a series of chlorinated
compounds, and recorded the observations based on the addition of acidified silver nitrate
solution.

Which of the following gives the expected results?

time taken for precipitate to appear

shortest longest

A C 6 H 5 Cl CH 2 ClCONH 2 CH 3 COCl COCl 2

B CH 2 ClCONH 2 C 6 H 5 Cl COCl 2 CH 3 COCl

C CH 3 COCl COCl 2 C 6 H 5 Cl CH 2 ClCONH 2

D COCl 2 CH 3 COCl CH 2 ClCONH 2 C 6 H 5 Cl

trendyline
IJC 2016 Prelim/9647/01 [Turn over
419 trendyline
13
29 In the following two-step synthesis, Y a solid which is insoluble in water, is one of the
components in the final reaction mixture.

COOH

PCl5, warm NH3


CH2OH X Y
then heat

What is Y?

A CO C CONH2
NH

CH2 CH2OH

+
B COO D CO2 NH4

+
CH2NH3 CH2NH2

30 Benzylamine has the formula C 6 H 5 CH 2 NH 2 and is a common precursor in organic


synthesis.

Which of the following statements about benzylamine is correct?

A It decolourises aqueous Br 2 .
B It can be produced by reduction of C 6 H 5 CH 2 CN.
C It reacts with CH 3 COOH to form C 6 H 5 CH 2 NHCOCH 3 .
D It reacts with excess CH 3 CH 2 Cl to form the compound, C 13 H 22 NCl.

trendyline
IJC 2016 Prelim/9647/01 [Turn over
420 trendyline
14

Section B

For each of the questions in this section, one or more of the three numbered statements 1 to 3 may
be correct.

Decide whether each of the statements is or is not correct (you may find it helpful to put a tick
against the statements that you consider to be correct).

The responses A to D should be selected on the basis of

A B C D

1, 2 and 3 1 and 2 2 and 3 1 only


are only are onlyare is
correct correct correct correct

No other combination of statements is used as a correct response.

31 Gaseous particle Y has a proton(atomic) number n and a charge of +1.

Gaseous particle Z has a proton(atomic) number of n+1 and is isoelectronic with Y.

Which statement correctly describes Y and Z?

1 Y has a larger radius than Z.


2 Y requires more energy than Z when a further electron is removed from each particle.
3 Y releases more energy than Z when an electron is added to each particles.

32 Which of the following molecules will form a hydrogen bond with another of its own
molecules?

1 CH 3 NH 2
2 CH 3 CHO
3 CH 3 CH 2 F

trendyline
IJC 2016 Prelim/9647/01 [Turn over
421 trendyline
15

33 The graph below shows the results of an investigation of the initial rate of hydrolysis of
maltose by the enzyme amylase. In the experiments, the initial concentration of maltose
was varied but that of amylase was kept constant.

Initial rate of
reaction

[maltose]

Which conclusions can be deduced from these results?

1 When [maltose] is low, the rate is first order with respect to[maltose].
2 When [maltose] is high, the rate is independent of[maltose].
3 When [maltose] is high, the rate is independent of[amylase].

34
With reference to the Data Booklet and the data below, deduce which of the following
statements are correct.

Half-equation E/V

VO 2 +(aq) + 2H+(aq) + e- VO2+(aq) + H 2 O(l) +1.00

VO2+(aq) + 2H+(aq) + e- V3+(aq) + H 2 O(l) +0.34

V3+(aq) + e- V2+(aq) + H 2 O(l) -0.26


Xx
1 An excess of zinc is capable of reducing VO 2 +(aq) to V2+(aq) in acidic conditions.
2 Fe3+(aq) is capable of oxidising V2+(aq) to VO 2 + (aq) in acidic conditions.
3 Zn2+(aq) ions will oxidise V2+(aq) ions to V3+(aq) ions.

trendyline
IJC 2016 Prelim/9647/01 [Turn over
422 trendyline
16

35 The working range and colourchange of chlorophenol red is given below.

colour change
indicator working pH range
Acid Alkali

chlorophenol red 4.8 6.4 Yellow Red

Two drops of this indicator are added to each of the three aqueous solutions listed below.

Which solution has its colour correctly stated?

solution colour

1 aluminium oxide added to water Red

2 0.1 mol dm-3 of CH 3 COOH (K a = 1.8 x 10-5 mol dm-3) Yellow

solution of sodium ethanoate and ethanoic acid in


3 Orange
1:2 proportion [pK a of ethanoic acid = 4.7]

36 Use of the Data Booklet is relevant to this question.

Which statements about the trends in properties of the hydrogen halides are correct?

1 Volatility decreases in the order: HF > HCl> HBr > HI.


2 Thermal stability increases in the order: HI< HBr < HCl< HF.
3 Ease of oxidation increases in the order: HF < HCl< HBr < HI.

37 Nickel is purified by a method called the Mond process. The equation of the first step of the
purification is shown below.

Ni(s) + 4CO(g) Ni(CO) 4 (l)

Which of the following statements are correct?

1 CO is a monodentate ligand.
2 Ni in Ni(CO) 4 has a co-ordination number of 4.

trendyline
3 The oxidation
dation
a number of Ni in Ni(CO) 4 is zero.

IJC 2016 Prelim/9647/01 [Turn over


423 trendyline
17

38 Compound X is a synthetic precursor of meloscine, an alkaloid isolated from the New


Caledonian plant Melodinus Scandens Forst.

compoundX

Which of the following statements about its reactions are correct?

1 1 mol of X reacts with 2 mol of HBr(g) when heated.


2 1 mol of X reacts with 4 mol of Br 2 (aq) at room temperature.
3 1 mol of X reacts with 2 mol of CH 3 COCl at room temperature.

39 Which of the following reactions give products that does not rotate the plane of polarised
light?

1 1bromobutane refluxed with aqueous KOH


2 butanone reacted with HCN, trace amount of NaOH
3 bXWHQHUHDFWHGZLWKEURPLQHGLVVROYHGLQRUJDQLFVROYHQW

40 Which of the following could result in the loss of the tertiary structure of a protein?

1 addition of weak acid


2 addition of alcohol
3 addition of AgNO 3 solution

trendyline
ne
IJC 2016 Prelim/9647/01 [Turn over
424 trendyline
INNOVA JUNIOR COLLEGE
JC 2 PRELIMINARY EXAMINATION 2
in preparation for General Certificate of Education Advanced Level
Higher 2

CANDIDATE
NAME

CLASS INDEX NUMBER

CHEMISTRY 9647/01
Paper 1 Multiple Choice 30 August 2016
1 hour
Additional Materials: Data Booklet
Multiple Choice Answer Sheet

READ THESE INSTRUCTIONS FIRST

Write your name and class on all the work you hand in.
Write in soft pencil.
Do not use staples, paper clips, highlighters, glue or correction fluid.

There are forty questions on this paper. Answer all questions. For each question there are four
possible answers A, B, C and D.
Choose the one you consider correct and record your choice in soft pencil on the separate
Answer Sheet.

Read the instructions on the Answer Sheet very carefully.

Each correct answer will score one mark. A mark will not be deducted for a wrong answer.
Any rough working should be done in this booklet.

trendyline
This
y
Thi document
d t consists
i t off 16 printed
i t d pages and
d 0 blank
bl k page.

InnovaJunior College [Turn over


425 trendyline
2
Section A

For each question there are four possible answers, A, B, C, and D. Choose the one you consider
to be correct.

1 Carbon disulfide, CS 2 , is a volatile liquid used in the production of cellophane which is used
for food packaging.

On combustion, CS 2 is oxidised as follows.

CS 2 (g) + 3O 2 (g) CO 2 (g) + 2SO 2 (g)

A 20 cm3 sample of carbon disulfide vapour is ignited with 100 cm3 of oxygen. The final
volume of gas after burning is treated with an excess of aqueous alkali.

Which percentage of this final volume dissolves in alkali? (All volumes are measured at
room temperature of pressure.)

A 20%
B 40%
C 60%
D 80%
Answer: C
1 mole of CS 2 reacts with 60cm3 of O 2 , remaining volume of O 2 = 40 cm3
produces 1 mole of CO 2
20 cm3 of CS2 produces 20 cm3 of CO 2 and 40 cm3 of SO 2
Total volume of gases at the end of reaction = 40 + 20 + 40 = 100 cm3
Both CO 2 and SO 2 are acidic gases which will react with NaOH
Hence % of final volume dissolved in alkali 60/100 x 100% = 60%

2 Consider the following half-equations

C 2 O 4 2 2CO 2 + 2e
Fe2+ Fe3+ + e
MnO 4 + 8H+ + 5e

Mn2+ + 4H 2 O

What volume of 0.01 mol dm3 potassium manganate(VII) is needed to completely oxidise
25.0 cm3 of an acidified solution of 0.01 mol dm3 FeC 2 O 4 ?

A 5 cm3
B 7.5 cm3

trendyline
C 10 cm3
D 15 cm3
Answer: D
qns:
Combining eqns:
FeC 2 O 4 Fe3++ 2CO 2 + 3ex5

IJC 2016Prelim 2/9647/01[Turn over


426 trendyline
3
MnO 4 + 8H+ + 5e Mn2+ + 4H 2 O x3
Overall equation: 3MnO 4 + 5FeC 2 O 4 10CO 2 + 5Fe3+ + 3Mn2+ + 12H 2 O

Amount of FeC 2 O 4 = 25/100 x 0.01 = 0.00025 mol


Mole ratio of FeC 2 O 4 : MnO 4 = 5 : 3
Amount of MnO 4 = 3/5 x 0.00025 = 0.00015 mol
Volume of MnO 4 = 0.00015 / 0.01 = 0.015 dm3 = 15 cm3

3 Which electronic configuration represents an element that forms a simple ion with a charge
of 3?
A 1s22s22p63s23p1
B 1s22s22p63s23p3
C 1s22s22p63s23p63d14s2
D 1s22s22p63s23p63d34s2
Answer:B
A : Group III element : + 3 ion
B: Group V : 3 ion
C and D are transition elements forms positively charged ions

4 Carbon dioxide is a gas at room temperature while silicon dioxide is a solidbecause

A carbon dioxide contains double covalent bonds while silicon dioxide contains single
covalent bonds.
B instantaneous dipole induced dipole attractions are weaker than permanent dipole
permanent dipole attractions.
C carbon-oxygen bonds are less polar than silicon-oxygen bonds.
D van der Waals forces are much weaker than covalent bonds.

Answer D
Carbon dioxide is a simple molecules with weak can der waals forces of attraction
between molecules while silicon dioxide is a giant covalent structure with strong covalent
bonds. Thus, less amt of energy is required to overcome the weaker VDW forces of
attraction between CO 2 molecules, resulting in lower boiling point.

trendyline
IJC 2016Prelim 2/9647/01[Turn over
427 trendyline
4
5 Under which conditions will the behaviour of a gas be most ideal?
pressure temperature
A high high
B high low
C Low high
D Low low

Answer: C
Gas behave most ideally at high temperatures and low pressures.

At high temperatures, particles have sufficient energy to overcome the intermolecular forces of
attraction.

At low pressures, particles are very far apart. The volume of the particles is negligible to the
volume of container which they moves in; intermolecular forces of attraction is negligible.

6 The enthalpy changes involving some oxides of nitrogen are given below:
N 2 (g) + O 2 J 12 J H = +180 kJ mol1
1
2NO 2 (g)+ O 2 J 1 2 O 5 (g) H = 55 kJ mol1
2
5
N 2 (g) + O 2 J 1 2 O 5 (g) H = +11 kJ mol1
2

What is the enthalpy change, in kJ mol1, of the following reaction?

2NO(g) + O 2 J 12 2 (g)

A 114 B +114 C 136 D +136

Answer: A
5HYHUVH(TXDWLRQ12 J 1 2 (g) + O 2 J H = 180 kJ mol1
1
Reverse Equation 2: N 2 O 5 J 12 2 (g)+ O 2 J H = +55 kJ mol1
2
5
Equation 3 remains: N 2 (g) + O 2 J 1 2 O 5 (g) H = +11 kJ mol1
2

Sum up these 3 equations:2NO(g) + O 2 J 12 2 J H = 180+55+11


= 114 kJ mol1

IJC 2016Prelim 2/9647/01[Turn over


428 trendyline
5

7 A typical protein forms hundreds of hydrogen bonds and thousands of van der Waals forces
in folding from primary to tertiary structures.
Which of the following thermodynamic state functions of the protein best represents the
folding process?

G / kJ mol1 H / kJ mol1 S / J K1 mol1

B + +

C +

D + +
Answer: A

When hydrogen bonds and van der Waals forces are formed, heat will be released. Thus H
is negative.

When the protein is folded from primary to tertiary structures, the system becomes less
disordered. Thus S is negative.

The process of protein folding is spontaneous. Thus, G is negative.

8 When 1 mole of carbon dioxide gas solidifies as dry ice, 25.2 kJ of heat energy is
evolved.The sublimation temperature of carbon dioxide is 78.5 C.

What is the entropy change when 132 g of carbon dioxide gas solidifies at this
temperature?

A +130 J K1
B 130 J K1
C +389 J K1
D 389 J K1
Answer: D
This is a phase change reaction.G = 0
G = H TS

S =

25200 0
S =
78.5+273

ttrendyline
25200
0
S =
194.5
6 J K-1
S = 129.56
132
Swhen 132 g of carbon dioxide gas solidifies = 129.56
129 56 x = 389 J K-1
44

IJC 2016Prelim 2/9647/01[Turn over


429 trendyline
6

9 A student set up the hydrogen electrode shown in the diagram below.

What would have to be changed to make this a standard hydrogen electrode?

A the acid solution used


B the temperature of the gas and of the acid solution
C the pressure of the gas
D the metal comprising the electrode
Answer:A
In 1.0 mol dm3 sulfuric acid, [H+] = 2.0 mol dm3. To make the electrode a standard
hydrogen electrode, either change the acid to a 1.0 mol dm3 monoprotic acid (option A),
or halve the concentration of sulfuric acid used.

10 Use of the Data Booklet is relevant to this question.

Using inert electrodes, a current was passed through two beakers containing aqueous
silver sulfate and aqueous copper(II) nitrate, connected in series under standard
conditions.

At 25oC, 1 atm

1.00 mol dm- 0.50 mol dm-


3 3
Cu(NO3)2(aq) Ag2SO4(aq)

t minutes?

A
trendyline
What is the ratio
atio of the mass of silver to copper deposited after the curren

0.59
59 B 0.85
0 85
current was passed for

IJC 2016Prelim 2/9647/01[Turn over


430 trendyline
7

C 1.70 D 3.40
Answer: D
It = n e F
It
transferred =
F
Cu2+ + 2e Cu
It
Cu =
2F
Ag+ + e Ag
It
Ag =
F
It
mass of Ag formed Ar x no. of moles of Ag formed 108 x F
= = It =3.40
mass of Cu formed Ar x no. of moles of Cu formed 63.5 x
2F

11 A nitrogen-hydrogen mixture, initially in the mole ratio of 1:3, reached equilibrium with
ammonia when 50% of the nitrogen had reacted. The total equilibrium pressure was p.

N 2 (g) + 3H 2 (g) 2NH 3 (g)

What was the partial pressure of ammonia in the equilibrium mixture?

A p B p C p D 2p
3 4 6 7
Ans A

N 2 (g) + 3H 2 (g) 2NH 3 (g)


Initial amt x 3x
Change x/2 3x/2 x
Eqm amt x/2 3x-3x/2 x

Total final pressure = sum of eqm partial pressure = x/2 + (3x-3x/2) + x = p


3x = p x = p/3

12 Solid calcium hypochlorite pellets, Ca(ClO) 2 (s), are added to swimming pools to form
HClO(aq), which kills disease-causing bacteria and algae.


Ca(ClO) 2 (s) Ca2+(aq) + 2ClO (aq)


ClO (aq) + H 2 O(l) HClO(aq) + OH (aq)

What is the effect on the solubility of calcium hypochlorite and bacterial growth when pH
decreases?

B
ttrendyline
trend
dyline
bility

li
ility of calcium hypochlorite
Solubility

crease
decreases

decreases
Effect on bacterial growth

ore favourable
more

less favourable

IJC 2016Prelim 2/9647/01[Turn over


431 trendyline
8

C increases more favourable

D increases less favourable


Answer: D

When pH decreases, [H+] increases and [OH] decreases, equilibrium position of second
equation shift to the right and [HClO] increases. Bacterial growth will be less favourable.

When [ClO] decreases, equilibrium position of first equation shift to the right
and solubility of calcium hypochlorite increases.

13
The dissociation constant, K w , for the ionisation of water, H 2 O H+ + OH , at different
temperatures is given below.

temperature / C Kw
0 1.15 x 10-15
25 1.00 x 10-14
50 5.50 x 10-14

What can be deduced from this information?

A Only at 25 C are [H+]and [OH-] equal.


B The equilibrium lies furthest to the right at 0C.
C The forward reaction is exothermic.
D The pH of water decreases as temperature increases.
Answer: D
Option A is wrong as [H+]and [OH-] are equal at all temperature for pure water.
Option B is wrong as the equilibrium lies furthest to the right at 50C (as shown by K w
value).
Option C is wrong as the forward reaction is endothermic. When temperature increases,
K w increases. This shows that equilibrium lies to the right.
Option D is correct as the forward reaction is endothermic. When temperature increases,
K w increases. This shows that equilibrium lies to the right.[H+] increases and this leads to
decrease in pH.

trendyline
IJC 2016Prelim 2/9647/01[Turn over
432 trendyline
9

14 A sparingly soluble salt, Mg 3 (PO 4 ) 2 dissociates in aqueous solution as follows:


Mg 3 (PO 4 ) 2 3 Mg2+ + 2 PO 4 3-
Given that the solubility product, K sp of Mg 3 (PO 4 ) 2 is Q, what is the value of [Mg2+] in a
saturated solution?

9 1 1 1 9 1
A ( )5 B ( )3 C ( )5 D ( )3
4 27 108 64

Answer: A
Mg 3 (PO 4 ) 2 3 Mg2+ + 2 PO 4 3-
Change x + 3x + 2x
Equilibrium 3x 2x
K sp = [Mg2+]3 [PO 4 3-]2
Q = (3x)3 (2x)2
Q = (27x3) (4x
1

x=( ) 5
108
1

[Mg2+] = 3 x ( )5
108
1
243 5
=( )
108
1
9 5
=( )
4

trendyline
IJC 2016Prelim 2/9647/01[Turn over
433 trendyline
10

15. The reaction of hydrogen peroxide with iodide ions in an acidic solution is first order with
respect to hydrogen peroxide as well as iodide ions, and zero order with respect to
hydrogen ions.
H 2 O 2 (aq) + 2H+(aq) + 2I(aq) 2H 2 O(l) + I 2 (aq)
Two experiments were carried out. Which pair of diagrams represents the variation of
[H 2 O 2 ] and [H+] with time?

Experiment 1: In the presence of Experiment 2: In the presence of


excess I excess I and H 2 O 2
[H2O2] [H+]

0 time 0 time

[H2O2]
[H+]

0 time
0 time

[H2O2]
[H+]

0 time
0 time

[H2O2] [H+]

trendyline
tre
e yline
n
D

0 time
tim 0 time

Ans B

IJC 2016Prelim 2/9647/01[Turn over


434 trendyline
11

Given in question: Rate = k[H 2 O 2 ][I],


For expt 1: [I-] is first order
The graph of [H 2 O 2 ] against time is a downward sloping curve with decreasing gradient.
Rate = k[H 2 O 2 ] where k = k[I]
For expt 2:
[H+] is zero order. The graph of [H+] against time is a decreasing straight line with a constant
gradient.
Rate = k where k = k[I][H 2 O 2 ]

16 Aluminium is an element in the third period, Na to Ar, of the Periodic Table.

What is true for aluminium?

A Aluminium has the highest melting point of the elements in this period.
B Aluminium is the only element in this period whose chloride reacts with water to form
an acidic solution
C Aluminium is the only element in this period which can exist, at room temperature
and pressure, as solid and conduct electricity.
D Aluminium is the only element in this period whose oxide reacts with both acids and
bases.
Answer: D
Period 3 trends
Aluminium is the only amphoteric oxide in third period.

17 Which of the following statements regarding Group II elements or their compounds


is correct?

A Magnesium chloride has a higher melting point than barium chloride.


B Magnesium is a stronger reducing agent than strontium.
C Calcium hydroxide is more thermally stable than barium hydroxide.
D Strontium reacts more readily with oxygen than radium.
Answer:
MgCl 2 has a larger magnitude of lattice energy than BaCl 2 since ionic radius of Mg2+ is
smaller than that of Ba2+ ion.
Mg is a weaker reducing agent or undergoes oxidation less readily than Sr since

trendyline
Eo(Mg2+/Mg) is less negative than Eo(Sr2+/Sr).
/Sr)
Ca(OH) 2 is less
ess Ba(O 2 as Ca2+
ss thermally stable than Ba(OH) 2
has a higher charge
cha density than
Ba2+ (due to the
he smaller ionic radius of Ca2+. Ca2+ polarises
larises tthe electron ccloud of OH- ions
O bond in the hydroxide ions to a greater extent, thus less energy is
and weakens the O-H

IJC 2016Prelim 2/9647/01[Turn over


435 trendyline
12
required for the decomposition of Ca(OH) 2.
The reactivity of Group II metals increases down the group due to increasing reducing
power of the metals down the Group. Since Sr is above Ra in Group II, Sr is expected to
react less readily with oxygen than Ra.

18 Which of the following statements about astatine, the element below iodine in Group VII
of the periodic table, is incorrect?

A Silver astatide is insoluble in aqueous ammonia.


B Hydrogen astatide is less stable to heat than hydrogen iodide.
C Sodium astatide and hot concentrated sulfuric acid react to form astatine.
D Astatine and aqueous potassium chloride react to form aqueous potassium astatide
and chlorine.
Answer:
AgI is insoluble in both dilute and concentrated NH 3 and since Ksp of AgX decreases
down the group, AgAt is expected to be less soluble in both dilute and concentrated NH 3
than AgI.
H-At bond is weaker than H-I bond since At atom is bigger and more diffuse leading to
less effective orbital overlap in H-At bond.
Since reducing power of the halides increases down the group and NaI reacts with conc
H 2 SO 4 to give I 2 , NaAt is similarly expected to react with conc H 2 SO 4 to give I 2.
At 2 cannot undergo displacement reaction with KCl since At 2 is a weaker oxidising agent
(or less likely to undergo reduction) than Cl 2 as oxidising power of halogen decreases
down the group.

19 When copper(II) chloride is dissolved in water it gives a blue solution. When this solution is
treated with an excess of concentrated hydrochloric acid it turns yellow.

What are the formulae of the copper species in the blue and yellow solution?

blue yellow

A CuCl 2 [CuCl 4 ]2-

B CuCl 2 (H 2 O) 4 [CuCl 6 ]4-

C Cu(OH) 2 CuCl 2 (H 2 O) 4

trendyline
tre
endyyline
D (H 2 O)) 6 ]2+
[Cu(H [[CuCll 4 ]2-
[CuC
Answer: D

hloride
oride dissolves in water, it dissociates into C
When copper(II) chloride Cu2+ and
d Cl- iions. The Cu2+ ions
2+
exist in the hydrated
d form,
form [Cu(H 2 O) 6 ] , which is blue
ue in colour.
colour

IJC 2016Prelim 2/9647/01[Turn over


436 trendyline
13

When excess conc HCl is added, ligand exchange occurs, where the Cl- ligand displace the H 2O
ligands to form [CuCl 4 ]2-, according to the following equation.

[Cu(H 2 O) 6 ]2+ + 4Cl- [CuCl 4 ]2- + 6 H2O

20 Four carbon-carbon bonds are labelled in the diagram.


w
x y z
CH CH C CH

Which bonds are made up of an sp2sp2 overlap?

A w and y only
B w, x and y only
C w, x, y and z
D x and y only
Answer: B
$OOWKHFDUERQVDUHVSH[FHSWWKHERQGIRU]ZKHUHWKHFDUERQLQ&&LVVSK\EULGL]HG

21 When heated with chlorine, the hydrocarbon 3,3-dimethylpentane undergoes free radical
substitution. In the propagation step, the free radical X is formed by the loss of one
hydrogen atom.

CH3
CH3CH2 C CH2CH3 Cl X HCl

CH3

How many different forms of Xare theoretically possible?


A 2 B 3 C 4 D 5
Answer: B
There are 3 types of Hydrogen atoms that can be lost.

trendyline
i
IJC 2016Prelim 2/9647/01[Turn over
437 trendyline
14

22 Bromoethene, CH 2 =CHBr, is unreactive to nucleophiles whereas 3-bromopropene,


CH 2 =CHCH 2 Br is very reactive by comparison.

What explains the lack of reactivity of CH 2 =CHBr?

A Substituted alkenes undergo only electrophilic addition.


B The C=C double bond and the Br atom are both electron withdrawing which
stabilises CH 2 =CHBr.
C The electrons on the Br atom delocalise into the bond.
D The presence of the bond prevents free rotation of the C-Br bond this decreasing
the reactivity.
Answer: C
For halogen-containing organic compounds, reactivity to nucleophiles (such as OH-)
depends on the bond strength of C-X bonds. The unreactivity of bromoethene with
nucleophile possibly suggest stronger C-Br bonds, as a result of the p orbital of Br
overlapping with bonds, hence having partial double-bond character.

23 Phosgene, COCl 2 ,is a colourless gas that gained infamy as a chemical weapon during
World War I. It is formed when trichloromethane, CHCl 3 , is left exposed to the atmosphere.
Below is a reaction scheme that involves trichloromethane and phosgene:

CHCl 3 COCl 2 COCl(OH) CO 2 + HCl

Which of the following type of reaction is not being observed in the reaction scheme?

A Oxidation
B Elimination
C Electrophilic addition
D Nucleophilic substitution
Ans:C

CHCl 3 COCl 2 COCl(OH) CO 2 + HCl


Oxidation nucleophilic substitution elimination

trendyline
IJC 2016Prelim 2/9647/01[Turn over
438 trendyline
15

24 A chlorine-containing organic compound, X, undergoes an elimination reaction when treated


with hot ethanolic potassium hydroxide solution.

What is X?

A CH 2 Cl 2
B C 2 Cl 6
C (CH 3 ) 2 CHCH 2 Cl
D (CH 3 ) 3 CCH 2 Cl
Answer: C
To undergo elimination, there must be a hydrogen atom on the carbon atom adjacent to the
carbon atom bearing the halogen atom. Only option C fulfils this criterion.

25 Dopamine is an important organic chemical that helps control the brains reward and pleasure
centres. It also helps regulate movement and emotional responses.

NH2
HO CH2CH
CO2H
HO
Dopamine

Which of the following statements about dopamine isincorrect?

A It is able to rotate plane-polarized light.


B It is soluble in water due to zwitterion formation.
C It migrates to the negative electrode of an electrolytic cell at pH 14.
D The acidity of dopamine will increase when aqueous bromine is added.

trendyline
IJC 2016Prelim 2/9647/01[Turn over
439 trendyline
16
Ans: C

Option A: Dopamine indeed contains a chiral carbon and exhibits optical activity

Option B: Dopamine is able to exist as a zwitterion RCH(NH3+)(COO-) which is able to form


ion-dipole interactions with water.

Option C: At pH 14, the COOH group is deprotonated. The predominant form is the anionic
form, which should migrate to the positive electrode instead of the negative electrode.

Option D: When aqueous bromine is added, electrophilic substitution occurs with the
introduction of Br into the benzene ring. Br is electron withdrawing and is able to disperse the
negative charge on the oxygen atoms in the anion, thus increasing the acidity of the
compound.

26 Two bottles containing the following reagents had their labels removed.

CH2CH2OH CH2COOH

Cl Cl Cl CH3

Which one of the following reagents can be used to distinguish between them?

A sodium metal C hot acidified potassium


manganate(VII)
B sodium hydroxide D hot acidified potassium dichromate
Ans: D
Option A: Both compounds react with Na. Effervescence will be seen for both compounds.
Gas evolved will extinguish lighted splint with a pop sound.
CH2COOH CH2CH2OH

Cl CH3
Option B: can react with NaOH but not Cl Cl .
However, the reaction is not easily visible.
CH2CH2OH CH2COOH

trendyline
re
e li
Cll
C CH3
CH
oth C
Option C: Both Cll Cll and reacts with hot acidified
KMnO 4 via side
de
e chain oxidation. Hence purple KMnO 4 will decolourise for both compounds
and effervescence
cence will be seen for both. Gas evolved will form white ppt with
w Ca(OH) 2 .

IJC 2016Prelim 2/9647/01[Turn over


440 trendyline
17
CH2CH2OH

Option D: Cl Cl reacts with hot acidified K 2 Cr 2 O 7 via oxidation of primary


CH2COOH

Cl CH3
alcohol (orange solution turns green). does not undergo side chain
oxidation as K 2 Cr 2 O 7 is a weaker oxidizing agent than KMnO 4 .

27 Marijuana (cannabis) is the most widely used illegal drug in many developed countries.
Medical studies have shown that the active ingredient in marijuana, tetrahydrocannabinol,
might provide medical benefits to some patients.

OH

O
tetrahydrocannabinol

In the following reactions, the O remains unaltered.

Which reaction will tetrahydrocannabinol undergo?


A It reacts with aqueous bromine to incorporate up to 2 atoms of bromine in each
molecule.
B It reacts with hot acidified potassium dichromate(VI) to form a green solution.
C It reacts with dilute nitric acid to form a substitution product.
D It reacts with ethanoic acid to form a sweet-smelling product.
Ans: C
Option A is incorrect because both alkene group and phenol group will react with
aqueous Br 2 to incorporate 3 Br atoms, forming the following products:

OH Br
Br OH
OH OH
Br Br
OR

O O
Br Br

trendyline
(more major) (less major)

Option B iss incorrect. K 2 Cr 2 O 7 is not strong enough to oxidise


oxidis side-chain of
benzene andd alkene. Phenol is not oxidised as wewell.

Option C is correct. Dilute


t Dil t nitric
it i acidid reacts
t with
ith phenol
h l functional
f t group via
electrophilic substitution to form the following product:

IJC 2016Prelim 2/9647/01[Turn over


441 trendyline
18

OH OH
NO2 Or

O O
NO2
Option D is incorrect. Phenol functional group does not react readily with ethanoic
acid (even in the presence of conc. H 2 SO 4 as catalyst). Hence, no ester will be
formed. (To form the ester, the phenol should be treated with aq. NaOH to generate
a stronger phenoxide ion before added into ethanoyl chloride)

28 A student carried out an experiment to study the ease of hydrolysis of a series of chlorinated
compounds, and recorded the observations based on the addition of acidified silver nitrate
solution.

Which of the following gives the expected results?

time taken for precipitate to appear

shortest longest

A C 6 H 5 Cl CH 2 ClCONH 2 CH 3 COCl COCl 2

CH 3 COCl
B CH2ClCONH2 C6H5Cl COCl2

C
CH 3 COCl CH 2 ClCONH 2
COCl2 C6H5Cl

D
COCl 2
CH3COCl CH2ClCONH2 C6H5Cl

Ans: D
COCl 2 and CH 3 COCl more reactive as the highly electronegative O and Cl atoms that are
bonded to acyl carbon make it more electron deficient (or more + charged), thus attract
nucleophiles more readily. COCl 2 hydrolyses more rapidly than CH 3 COCl as the former has
its acyl carbon more electron deficient due to its additional Cl atom.

In addition, the acyl carbon atoms are sp2 hybridised, so they have trigonal planar
arrangement with respect to C, resulting in less steric hindrance for the approach of
nucleophiles compared to the sp3 C atom (that is bonded to Cl in CH 2 ClCONH 2 ).

CH 2 ClCONH 2 is the next most reactive as the C atom bonded to only one highly
electronegative Cl atom is less electron deficient (or less + charged) compared to COCl 2
and CH 3 COCl.

C 6 H 5 Cl is a halogenoarene (or aryl halide) where its Cl atom is bonded directly to benzene
ring. The p orbital of Cl overlaps with the electron cloud of benzene ring resulting its lone

trendyline
pair being delocalized
calized into benzene and double bond character
ocalized char in its C-Cl
C-Cl bond. Since its C-
Cl is most difficult
cult to break, nucleophilic substitution (or hydrolysis) cannot o
occur.

29 In the following
ng two-step synthesis, Y a solid which is insoluble in water,
id w wa is one of the
components in the final reaction mixture.

IJC 2016Prelim 2/9647/01[Turn over


442 trendyline
19
COOH

PCl5, warm NH3


CH2OH X Y
then heat

What is Y?

A CO C CONH2
NH

CH2 CH2OH

+
B COO D CO2 NH4

+
CH2NH3 CH2NH2

Ans: A
COOH COCl COCl
PCl5, warm NH3
CH2OH CH2Cl CH2NH2
then heat
X
heat

CO
NH
CH2
Y
COCl

CH2NH2
The reaction of X with NH 3 produces a mixture of products. is one of the
products which contains an amine group and an acyl chloride group. The amine group is
nucleophilic and undergo intramolecular reaction with the electron-deficient C of the acyl
chloride to form a cylic product.

Since reaction with PCl 5 converts COOH to COCl and CH 2 OH to CH 2 Cl (as shown in

trendyline
X), the product should not contain COO- or CH 2 OH g
group.
p Hence,, by elimination of
options, B, C and D are incorrect.

30 Benzylamine has the formula C 6 H 5 CH 2 NH


N 2 and is a common precursor
pre in organic
synthesis.

IJC 2016Prelim 2/9647/01[Turn over


443 trendyline
20
Which of the following statements about benzylamine is correct?

A It decolourises aqueous Br 2 .
B It can be produced by reduction of C 6 H 5 CH 2 CN.
C It reacts with CH 3 COOH to form C 6 H 5 CH 2 NHCOCH 3 .
D It reacts with excess CH 3 CH 2 Cl to form the compound, C 13 H 22 NCl.

(A) CH 2 NH 2 group is not strongly activating, hence benzylamine, C 6 H 5 CH 2 NH 2 , cannot


undergo easy electrophilic substitution with aqueous Br 2 . This is not to be confused
with phenylamine, C 6 H 5 NH 2 .
(B) When C 6 H 5 CH 2 CN undergoes reduction, C 6 H 5 CH 2 CH 2 NH 2 is produced, not
C 6 H 5 CH 2 NH 2 .
(C) C 6 H 5 CH 2 NH 2 undergoes acid base reaction, not condensation reaction, with
CH 3 COOH to give C 6 H 5 CH 2 NH 3 +CH 3 COO. An amide is formed from condensation
between a primary or secondary amine and acyl halide.
(D) A quaternary ammonium salt (shown below) is formed from successive nucleophilic
substitutions (polyalkylation).
CH2CH3
+
CH2N CH2CH3 Cl-
CH2CH3

Section B

For each of the questions in this section, one or more of the three numbered statements 1 to 3 may
be correct.

Decide whether each of the statements is or is not correct (you may find it helpful to put a tick
against the statements that you consider to be correct).

The responses A to D should be selected on the basis of

A B C D

1, 2 and 3 1 and 2 2 and 3 1 only


are only are onlyare is
correct correct correct correct

No other combination of statements is used as a correct response.

31 Gaseous particle Y has a proton(atomic) number n and a charge of +1.

Gaseous particle Z has a proton(atomic) number of n+1 and is isoelectronic with Y.

Which statement correctly describes Y and Z?

1 Y has a larger radius than Z.


2
3
Ans: D trendyline
Y requires
res more energy than Z when
ses
es more energy than
Y releases
hen a further electron is removed ffrom
fr each particle.
th Z when an electron is added to each particles.

Statement 1 (correct) Y (less protons) and isoelectronic


and has a larger radius.
oele
p
pa

with Z has a low


lower nuclear charge

IJC 2016Prelim 2/9647/01[Turn over


444 trendyline
21
Statement 2: (wrong) Z has a higher nuclear charge and hold electrons more
strongly(stronger attraction) and hence requires more energy.

32 Which of the following molecules will form a hydrogen bond with another of its own
molecules?

1 CH 3 NH 2
2 CH 3 CHO
3 CH 3 CH 2 F
Answer: D
Option 1: There is H-bonding between CH 3 NH 2 .
Option 2: No H-bonding as O is not directly bonded to H atom.
Option 3: No H-bonding as F is not directly bonded to H atom.

33 The graph below shows the results of an investigation of the initial rate of hydrolysis of
maltose by the enzyme amylase. In the experiments, the initial concentration of maltose
was varied but that of amylase was kept constant.

Initial rate of
reaction

[maltose]

Which conclusions can be deduced from these results?

1 When [maltose] is low, the rate is first order with respect to[maltose].
2 When [maltose] is high, the rate is independent of[maltose].
3 When [maltose] is high, the rate is independent of[amylase].

Answer: B
Answer: B

Option 1:
At low substrate concentration[maltose], not all of the active sites are occupied.
- rate [maltose] reaction is first order with respect to the maltose.

Option 2:
At high [maltose], all the active sites are occupied, i.e. the active sites of the enzyme [amylase]

trendyline
become saturated with
w maltose.
- Any increase in [maltose] will not have any effect on the reaction rate.
- The reaction is
s zero
ero order with respect to the maltose
maltose.

Option 3: If substrate
trate is not the limiting factor, increasing
reas enzyme amylase
concentration will increase theh rate off reaction,
i as there
h will be more active
ill b i sites
i of enzyme.

IJC 2016Prelim 2/9647/01[Turn over


445 trendyline
22
34
With reference to the Data Booklet and the data below, deduce which of the following
statements are correct.

Half-equation E/V

VO 2 +(aq) + 2H+(aq) + e- VO2+(aq) + H 2 O(l) +1.00

VO2+(aq) + 2H+(aq) + e- V3+(aq) + H 2 O(l) +0.34

V3+(aq) + e- V2+(aq) + H 2 O(l) -0.26


Xx
1 An excess of zinc is capable of reducing VO 2 +(aq) to V2+(aq) in acidic conditions.
2 Fe3+(aq) is capable of oxidising V2+(aq) to VO 2 + (aq) in acidic conditions.
3 Zn2+(aq) ions will oxidise V2+(aq) ions to V3+(aq) ions.

trendyline
IJC 2016Prelim 2/9647/01[Turn over
446 trendyline
23

Answer: D
Answer: D
Half-equation E/V

Zn2+ + 2e Zn 0.76

Option 1:
Excess Zn is the reducing agent which can reduce VO 2 +(aq) to VO2+(aq)
Ecell = +1.00 (0.76) =1.76V > 0
Excess Zn can further reduce VO2+(aq) to V3+(aq) :
Ecell = +0.34 (0.76) = 1.10V >0
Excess Zn can further reduce V3+(aq) + e- V2+(aq) :
Ecell: 0.26 (0.76) = 0.50V > 0

Quick thinking for MCQ:


Since Ecell = Ered Eoxi > 0 for reaction to proceed (Zn is oxidised),
Ered (0.76) > 0
Ered > 0.76 for reaction to proceed

Option 2: wrong
Quick thinking for MCQ:
Since Ecell = Ered Eoxi > 0 for reaction to proceed
(Fe3+ is reduced to Fe2+),
+0.77 Eoxi > 0
Eoxi < + 0.77 for reaction to proceed

Fe3+ can oxidise V2+ to VO2+ but not VO 2 +

VO 2 +(aq) + 2H+(aq) + e- VO2+(aq) + H 2 O(l) +1.00

Fe3+ + e Fe2+ +0.77


Ecell = +0.77 1.00 = 0.23v < 0
Another factor: Fe3+ is not in excess, successive oxidation of vandadium compounds
may not occur.

35 The working range and colourchange of Chlorophenol red is given below.

Colour change
Indicator Working pH range
Acid Alkali

trendyline
tren
ndyline
e
Chlorophenol
hlorophenol
o red 4.8 6.4
6 Yellow
Y Red
R

Two drops of this


his indicat
indicator
or are added to each of the three aqueous solutio
solutions listed below.

Which solution has its colour correctly stated?

IJC 2016Prelim 2/9647/01[Turn over


447 trendyline
24

Solution Colour

1 Aluminium oxide added to water Red

2 0.1 mol dm-3 of CH 3 COOH (K a = 1.8 x 10-5 mol dm-3) Yellow

Solution of sodium ethanoate and ethanoic acid in


3 Orange
1:2 proportion [pK a of ethanoic acid = 4.7]

Answer: B

Option 1 is correct.
Aluminium oxide does not dissolve in water and it gives a pH7 solution. According
to the working pH range of this indicator, the color is red.

Option 2 is correct.
[H+] = K a [acid]
= 1.8X10-5 [0.1]
= 1.342 10-3 mol dm-3

pH = - log 10 (1.342 10-3) = 2.87

According to the working pH range of this indicator, the color shown for a solution
of pH 2.87 is yellow.

Option 3 is incorrect.
[]
pH = pK a + log =4.7 + log 0.5 = 4.7 0.3 = 4.4
[]
According to the working pH range of this indicator, the color shown for a solution
of pH 4.4 is yellow and not orange.

36 Use of the Data Booklet is relevant to this question.

Which statements about the trends in properties of the hydrogen halides are correct?
1 Volatility decreases in the order: HF > HCl> HBr > HI.
2 Thermal stability increases in the order: HI< HBr < HCl < HF.
3 Ease of oxidation increases in the order: HF < HCl< HBr < HI.
Ans: C

Option 1:Volatility is inversely proportional to the boiling point. The higher the boiling point,
the less volatile the substance. HF has stronger intermolecular hydrogen bonding that
requires more energy to break while the strength of instantaneous dipole-induced dipole
interaction increases from HCl to HBr to HI due to increasing polarizability of electron cloud

trendyline
of the molecules. Thus the correct trend of volatilityy is HF < HI < HBr < HCl.

Option 2:Thermal strength of H-X. The


ermal stability of hydrogen halides depend on the bond stre
stronger the H-X
H--X bond, the more thermally stable the hydrogen halides. Do
Down the group, the
valence orbital overlap between the
al is more diffused, as there is less effective orbital ove
valence orbitals
als of hydrogen and halogen OR down
dow the group, as the atom
atomic size of halogen
increases, the H-X bond length increases. As such, the H-X bond strength is weaker, and

IJC 2016Prelim 2/9647/01[Turn over


448 trendyline
25

less thermally stable. Therefore, the trend of increasing thermal stability (HI< HBr < HCl <
HF) is correct.

Option 3:The ease of oxidation or reducing power of the halides increases down the group
due to the less positive Eo(X 2 /X-) down the group. Hence, the trend is correct.

37 Nickel is purified by a method called the Mond process. The equation of the first step of the
purification is shown below.

Ni(s) + 4CO(g) Ni(CO) 4 (l)

Which of the following statements are correct?

1 CO is a monodentate ligand.
2 Ni in Ni(CO) 4 has a co-ordination number of 4.
3 The oxidation number of Ni in Ni(CO) 4 is zero.
Ans: A
CO is a monodentate ligand as it forms one dative bond (using the lone pair of electron on
the carbon atom) per formula unit.
Ni in Ni(CO) 4 has 4 dative or coordinate bonds formed with 4 CO ligands, thus its
coordination number is 4.
Since the charges of the complex compound, Ni(CO) 4 and CO ligands are zero, the charge
of Ni is zero too.

38 Compound X is a synthetic precursor of meloscine, an alkaloid isolated from the New


Caledonian plant Melodinus Scandens Forst.

compoundX
Which of the following statements about its reactions are correct?

1 1 mol of X reacts with 2 mol of HBr(g) when heated.


2 1 mol of X reacts with 4 mol of Br 2 (aq) at room temperature.
3 1 mol of X reacts with 2 mol of CH 3 COCl at room temperature.
Ans: C
Option 1: Incorrect. 1 mol of X reacts with 3 mol of HBr.
Each C=C will undergo electrophilic addition with HBr. 1 alcohol will undergo nucleophilic
substitution with HBr, heat.

trendyline
Option 2: Correct
rect
Each C=C will undergo electrophilic additionadditio with h Br 2 (aq).
aq). Phenyla
Phenylamine
Phenylam will undergo
electrophilic substitution with 2 mol of Br 2 to form the di
d
di-substituted
-substituted
substituted produc
product.

rect
Option 3: Correctect
1 alcohol willl undergo condensation reaction with
wit CH 3 COCl to form an ester.
e Phenylamine
will undergo condensation reaction with 1 mol of CH 3 COCl to form amide.

IJC 2016Prelim 2/9647/01[Turn over


449 trendyline
26

39 Which of the following reactions give products that does notrotate the plane of polarised
light?

1 1bromobutane refluxed with aqueous KOH


2 Butanone reacted with HCN, trace amount of NaOH
3 %XWHQHUHDFWHGZLWKEURPLQHGLVVROYHGLQRUJDQLFVROYHQW
Ans: A
Option 1: 1-bromobutane is a 1 halogenoalkane hence it will react with aq KOH via S N 2
mechanism. Since the product does not contain a chiral carbon, it is unable to rotate plane
of polarised light.

Option 2: Butanone reacts with HCN via nucleophilic addition. As the nucleophiles can
attack the carbonyl carbon (trigonal planar in shape) either from the top or the bottom of the
plane with equal chance, it results in a racemic mixture Hence, even though the products
are chiral, they are unable to rotate plane of polarised light.

Option 3: But-1-ene reacts with bromine via electrophilic addition. As the electrophile can
attack the C atom of the carbocation intermediate is trigonal planar in shape, either from the
top or the bottom of the plane with equal chance, it results in a racemic mixture Hence, even
though the products are chiral, they are unable to rotate plane of polarised light.

40 Which of the following could result in the loss of the tertiary structure of a protein?

1 Addition of weak acid


2 Addition of alcohol
3 Addition of AgNO 3 solution
Ans: A
Note: The tertiary structure of proteins refers to the 3D arrangement of the protein due to the
folding of a polypeptide chain. Folding is due to R group (side chain) interactions.

Option 1: Addition of weak acid pH can change the ionic charges on amino acid residues
(i.e. groups containing acidic COOH or basic NH 2 groups) and hence disrupt the ionic
interactions.
In acidic conditions), protonation occurs, i.e. COO is converted to COOH.

Option 2: Alcohol can disrupt the original hydrogen bonds between polar R groups thereby
disrupting their normal folding.

Option 3: Metal ions (Ag+) are positively charged cations. They compete with positively
charged groups (e.g. NH 3 +) for attraction to negatively charged groups (e.g. -COO), hence
disrupting the original ionic interactions.

1 C 6 A 11 A 16 D 21 B 26 D 31 D 36 C
2 D 7 A 12 D 17 A 22 C 27 C 32 D 37 A

ttrendyline
tre
re
endy
e nd
dy
ylin
liin
in
nee
3 B 8 D 13 D 18 D 23
2 C 28 D 33 B 38 C
4 D 9 A 14 A 19 D 24 C 29 A 34 D 39 A
5 C 10 D 15
1 B 20
2 B 25
2 C 30 D 35 B 40 A

A = 10
B=6
C=9

IJC 2016Prelim 2/9647/01[Turn over


450 trendyline
27
D = 15

trendyline
IJC 2016Prelim 2/9647/01[Turn over
451 trendyline
INNOVA JUNIOR COLLEGE
JC 2 PRELIMINARY EXAMINATION
in preparation for General Certificate of Education Advanced Level
Higher 2

CANDIDATE
NAME

CLASS INDEX NUMBER

CHEMISTRY 9647/02
Paper 2 Structured Questions 18 August 2016
2 hours
Candidates answer on the Question Paper

Additional Materials: Data Booklet

READ THESE INSTRUCTIONS FIRST

Write your index number, name and civics group.


Write in dark blue or black pen.
You may use pencil for any diagrams, graphs or rough working.
Do not use staples, paper clips, highlighters, glue or correction fluid.

Answer all questions in the space provided.


A Data Booklet is provided. For Examiners Use

Section A
You are advised to show all working in calculations.
You are reminded of the need for good English and
clear presentation in your answers.
1 12
You are reminded of the need for good handwriting. 2
Your final answers should be in 3 significant figures. 17

You may use a calculator. 3


10
The number of marks is given in brackets [ ] at the end of each 4
question or part question. 18
5
At the end of the examination, fasten all your work 15
securely together. Significant
figures
Handwriting

Total
72

trendyline
This document consists of 17 printed
nted pages and 1 blank page.
page

Innova Junior College [Turn over

452 trendyline
For
2 Examiners
Use

Answer ALL questions on the spaces provided.

1 Planning

Using the information below, you are to write a plan for determining number of molecules
of water of crystalisation, n, in barium chloride crystals, BaCl 2 .nH 2 O where n = 1,2 or 3.
The basis of this investigation is a technique known as precipitation titration with the use of
silver nitrate solution, AgNO 3 (aq).

AgCl, Ag 2 CrO 4 and BaCrO 4 are sparing soluble salts and relevant information about these
salts are given in the table below.

compound colour numerical value of K sp at 25 O C


AgCl white 2.0 1010
Ag 2 CrO 4 red 2.0 1012
BaCrO 4 yellow 1.0 1010

Some barium chloride crystals, BaCl 2 .nH 2 O, is dissolved to make up 250.0 cm3 of
standard solution. Approximately 10 cm3 of 0.2 mol dm3 of reagent X solution is added to
a portion of this standard solution to precipitate the barium ions before a few drops of
K 2 CrO 4 indicator solution is added. A titration is then carried out on this portion of solution
against silver nitrate solution, AgNO 3 (aq). Ag 2 CrO 4 would just precipitate only when almost
all of the Cl ions have been precipitated as AgCl. The titration is repeated until a more
reliable average value, V cm3, can be determined.

(a) (i) Identify reagent X.

... [1]

(ii) A student suggested that BaCrO 4 is more soluble than Ag 2 CrO 4 as BaCrO 4
has larger numerical value of K sp than Ag 2 CrO 4 .

Suggest whether this students claim is valid.

........

........

........

........

... [1]

(iii) Suggest why reagent X is added before a few drops of K 2 CrO 4 indicator
solution is added.

trendyline


........

........

... [1]
IJC 2016 Prelim/9647/02 [Turn over
453 trendyline
For
3 Examiners
Use
(b) You may assume that you are provided with

0.10 mol dm3 silver nitrate

0.2 mol dm3 of reagent X solution

10 g of barium chloride crystals, BaCl 2 .nH 2 O

Potassium chromate solution, K 2 CrO 4 (aq)

The equipment and materials normally found in a school or college


laboratory.

Your plan should include the following

brief, but specific details of the apparatus you would use, bearing in mind the
levels of precision they offer

details, including quantities, for preparation of 250.0 cm3 of BaCl 2 solution


from barium chloride crystals, BaCl 2 .nH 2 O

essential details of the titration procedure

an outline of how the results obtained, including V cm3 , would be used to


determine n, in barium chloride crystals (BaCl 2 .nH 2 O)

....................

....................

....................

....................

....................

.........

....................

....................

....................

....................

....................

trendyline
.........

.........

.........

.........

IJC 2016 Prelim/9647/02 [Turn over


454 trendyline
For
4 Examiners
Use

.........

....................

....................

....................

....................

....................

.........

.........

.........

.........

.........

.........

.........

.........

.........

.........

.........

.........

.........

.........

.........

.........

.........

.........

.........

trendyline
.........

.........

.........

........ [8]
IJC 2016 Prelim/9647/02 [Turn over
455 trendyline
For
5 Examiners
Use

(c) Explain why this titration cannot be conducted under acidic or alkaline medium.

.........

.........

.........

........ [1]

[Total: 12]

trendyline
IJC 2016 Prelim/9647/02 [Turn over
456 trendyline
For
6 Examiners
Use
2 This question is about the oxides of nitrogen and its reactions. Oxides of nitrogen
constitute air pollutants originating from emission of car exhaust that cause acid rain
and photochemical smog.

The two gases of NO and N 2 O 4 slowly react to form the blue compound, N 2 O 3
according to the following equation.

2NO(g) + N 2 O 4 (g) 2N 2 O 3 (g)

(a) (i) Draw the dot and cross diagrams of the two molecules, NO and N 2 O 3 .
The N 2 O 3 molecule containVD11ERQG.

[2]

(ii) From your answer in (a)(i), suggest why the forward reaction is likely to
occur.

...

...

..... [1]

(b) In an experiment, a mixture containing NO and N 2 O 4 was introduced into a


1.48 dm3 evacuated vessel was allowed to reach equilibrium at 280 K. The
equilibrium pressure is 98.9 kPa.

Calculate the total number of moles of gases at equilibrium, assuming the gases
behave ideally.

trendyline [2]
IJC 2016 Prelim/9647/02 [Turn over
457 trendyline
For
7 Examiners
Use
(c) Two colourless gases NO and N 2 O 4 slowly react to form the blue compound,
N 2 O 3 according to the following equation.

2NO(g) + N 2 O 4 (g) 2N 2 O 3 (g)

Equimolar mixtures of NO and N 2 O 4 are mixed at varying pressure P but at two


different temperatures of 300 K and 330 K, and the variation in colour intensity
was monitored over a period of time.

The graphs below show the variation of the colour intensity with pressure at
temperatures of 300 K and 330 K.

colour
colour 300 K
intensit
y
intensity
330 K

0 pressure/ atm

(i) What is the significance of the colour intensity in this reaction?

...

..... [1]

(ii) Explain why colour intensity of the reaction mixture increases with increasing
pressure.

...

...

..... [1]

(iii) Using information from the graphs, state and explain whether the formation
of N 2 O 3 from NO and N 2 O 4 is an exothermic reaction.

...

...
...
...

...
...
...

...
...

..... [2]
IJC 2016 Prelim/9647/02 [Turn over
458 trendyline
For
8 Examiners
Use
(d) Nitrogen monoxide reacts with chlorine to form nitrosyl chloride, according to the
equation:

2NO(g) + Cl 2 (g) 2NOCl(g).

In an experiment, the amount of Cl 2 (g) was kept in large excess while the initial
partial pressure of NO(g) was varied at constant temperature of 500K.
The table below shows the experimental results obtained.

time / s P NO / atm (Rate / P NO ) (Rate / (P NO )2)


/ s1 / (atm  s1)
0 0.917 1.033 104 1.126 104

1000 0.827 9.312 105 1.126 104

2000 0.753 8.486 105 1.127 104

3000 0.691 7.788 105 1.127 104

4000 0.638 7.190 105 1.127 104

(i) Suggest why the amount of Cl 2 (g) was kept in large excess.

...

..... [1]

(ii) Using the data from the table above, deduce the order of reaction with
respect to NO(g).

...

..... [1]

(iii) In another experiment, the initial partial pressure of NO(g) was 4.2 atm and it
was reacted with Cl 2 (g) at a constant temperature of 500 K. The partial
pressure of Cl 2 (g) was recorded at time intervals of 30 s. The data obtained
are tabulated below.

time / s partial pressure time / s partial pressure


of Cl 2 (g) / atm of Cl 2 (g) / atm
0 0.78 300 0.49
30 0.76 330 0.46
60 0.72 360 0.44
90 0.70 390 0.42
120 0.66 420 0.39
150 0.63 450 0.38

ttrendyline
re
endyylin
ne
180 0.59 480
48 0.36
0
2
210 0.57 510
51 0.34
0
2
240 0.54 540
54 0.33
0
270 0.52
0 52 570 0.32
0

IJC 2016 Prelim/9647/02 [Turn over


459 trendyline
For
9 Examiners
Use
Using the data but without the plotting of any graph, deduce the order of
reaction with respect to Cl 2 (g).

...

...

..... [1]

(iv) Write the rate equation for this reaction.


Hence, calculate the rate constant, including its units

[3]

(v) One possible mechanism of the reaction is given below.

Step I NO(g) + Cl 2 (g) NOCl 2 (g) fast

Step II NOCl 2 (g) + NO(g) 2NOCl(g) slow

Explain whether it is consistent with the observed kinetics data in (d)(iv).

...

...

...

...

..... [2]

[Total: 17]

trendyline
IJC 2016 Prelim/9647/02 [Turn over
460 trendyline
For
10 Examiners
Use

3 Organic compounds can undergo combustion. Chemical companies produce containers


filled with butane for use by campers.

The enthalpy change of combustion of butane is 3000 kJ mol 1.

(a) (i) Define the term standard enthalpy of combustion of butane.

...

..... [1]

(ii) Calculate the mass of water at 25 C that could be brought to the boiling point
by the combustion of 1.2 dm3 of butane gas. Assume 75 % of the heat from
the butane is absorbed by the water.

[2]

(b) Explain in terms of structure and bonding, why butanone has a higher boiling point
than pentane.

...

...

...

...

...

trendyline
.....

[2]

IJC 2016 Prelim/9647/02 [Turn over


461 trendyline
For
11 Examiners
Use

(c) Organic compounds are also widely used for pharmaceutical purposes such as
ephedrine which is an anti-asthmatic and stimulant. Ephedrine can be converted
into ethylephedrine via a S N 2 reaction.

OH OH
N N
H

Ephedrine Ethylephedrine

(i) Suggest the reagent that can be used to perform this conversion.

....[1]

(ii) Explain why an S N 2 mechanism is favoured for this reaction.

.....

.[1]

(d) Complete hydrolysis of proteins produces individual units of amino acids, but partial
hydrolysis can break the protein down into dipeptide or tripeptide fragments.
Partial hydrolysis of a tetrapeptide (containing four amino acid residues) produces
the following three dipeptides, as well as the individual amino acids.
NH 2 CH 2 CONHCH(CH 3 )CO 2 H
NH 2 CH 2 CONHCH 2 CO 2 H
NH 2 CH(CH 3 )CONHCH(CH 3 )CO 2 H

(i)
Define the primary structure of a protein.

...

..... [1]

(ii) Deduce the order in which the amino acids are bonded together in the
tetrapeptide.

...

..... [1]

(iii) Suggest suitable reagents and conditions to hydrolyse the tetrapeptide into its

trendyline
ividual
vid
individual amino acids.

...
...

...
...
..... [1]

[Total: 10]
IJC 2016 Prelim/9647/02 [Turn over
462 trendyline
For
12 Examiners
Use

4 (a) A 0.031 mol dm-3 solution of a base, MOH, has a pH of 12.5. (M is a metal.)

25 cm3 of the solution of MOH was titrated with 0.025 mol dm-3 propanoic acid,
CH 3 CH 2 COOH, at 25 C. The pH of the solution was followed using a pH meter
and the following titration curve was obtained.

pK a of propanoic acid = 1.29 x 10-5 mol dm-3

pH

volume of propanoic acid


V added/cm3

(i) Calculate the concentration of hydroxide ions present in the sample of MOH
and use it to explain whether it is a strong or weak base.

...

...

......[1]

(ii) Calculate the value of V.

[1]

trendyline
IJC 2016 Prelim/9647/02 [Turn over
463 trendyline
For
13 Examiners
Use

(iii) Explain, with the aid of an equation, why the pH when V cm3 of propanoic
acid was added, is greater than 7.

...

...

...

..... [2]

(b) The colour of blackberries is due to a compound known as cyanidin. Cyanidin is a


weak organic acid which may be represented by CyH. In aqueous solution, CyH
dissociates slightly:

CyH (aq) Cy (aq) + H+ (aq)


red purple

The colours of CyH and Cy are indicated in the above equation.


Acid dissociation constant, K a , of CyH is 5.0 x 105 mol dm3 at 25 C.

A glass of blackberry juice has a pH of 3.00 at 25 C. Calculate the ratio of the red
to purple form in the juice, and hence predict its colour.

[3]

(c) When chlorine is bubbled through cold sodium hydroxide solution and acidified
silver nitrate solution, only half of the chlorine that has dissolved is precipitated as
silver chloride. When the sodium hydroxide is hot, up to fivesixth of the chlorine
can be precipitated. Explain the observations, giving balanced equations where
appropriate.

............

........

trendyline
........

........

........

IJC 2016 Prelim/9647/02 [Turn over


464 trendyline
For
14 Examiners
Use
...... [3]

(d) The table below gives data about some physical properties of the elements
calcium, nickel and copper.

calcium nickel copper


atomic radius / nm 0.197 0.124 0.128
electronic
[Ar]4s2 [Ar]3d84s2 [Ar]3d104s1
configuration

(i) What do you understand by the term transition element?

......

......

..... [1]

(ii) Although the nickel and copper atoms have more electrons than the calcium
atom, the atomic radii of nickel and copper are smaller than that of calcium.

Suggest an explanation for this.

...

...

...

...

...

...

...

...[3]

trendyline
IJC 2016 Prelim/9647/02 [Turn over
465 trendyline
For
15 Examiners
Use

(e) The following are some reactions involving copper(II) nitrate.

hydrated excess water KHCO (aq)


3
copper(II) Cu(NO3)2 (aq) blue ppt + CO
2
nitrate solid
excess NH (aq)
3

D
dark blue solution

excess KCN(aq)

E
violet solution

(i) Explain why carbon dioxide is evolved when KHCO 3 (aq) is added to
aqueous copper(II) nitrate. Include any relevant equations with state
symbols in your answer.

...

...

...

...

...

...[2]

(ii) Suggest a formula for the complex ion present in D.

.... [1]

(iii) The metal ion in complex E is known to have a coordination number of four.
Suggest a formula for complex ion E.

... [1]

[Total: 18]

trendyline
IJC 2016 Prelim/9647/02 [Turn over
466 trendyline
For
16 Examiners
Use
5 Benzene reacts with propanoyl chloride and 1-phenylpropan-1-one is formed in the
reaction.

(a) State the reagents and conditions needed to convert 1-phenylpropan-1-one into
compound Q. Show the structure of the intermediate in the box provided.

O OH
Step 1 Step 2
C CH2CH3 C COOH

CH2CH3
- - -
1 phenylpropan 1 one compound Q

Reagents and conditions

Step 1 .

Step 2 .
[3]

(b) Describe the mechanism in Step 1 in part (a). In your answer, you should show all
charges and lone pairs and show the movement of electrons by curly arrows.

[3]

(c) The 2,4-D weed killer, C 8 H 6 Cl 2 O 2, is widely used on cereal crops, pastures and
orchards. Compound R is an isomer of the weed killer under development to improve
its properties. It is an aromatic compound which contains three functional groups.

The following tests are carried out in the order given. In each case, state all
deductions about the compound R you can make at that stage. When identifying the
functional groups, your answers should be unambiguous.

trendyline
(i) Compound R is
When Compoun with
s treated with Na, C 8 H 5 C
Cll 2 O 2 Na is forme
formed.
form

deduction(s)..........................................................................................................
eduction(s)...............................................................................
duction(s)...............................................................................

..........................................................................................................................[1]
................................................................................................
...

IJC 2016 Prelim/9647/02 [Turn over


467 trendyline
For
17 Examiners
Use

(ii) When Fehlings solution is added to Compound R, red brick precipitate is


obtained.

deduction(s)..........................................................................................................

..........................................................................................................................[1]

(iii) When 1 mole of Compound R is boiled with ethanolic silver nitrate, 287 g of
white precipitate is formed.

deduction(s)..........................................................................................................

.............................................................................................................................

.............................................................................................................................

..........................................................................................................................[2]

(iv) When Compound R is treated with concentrated nitric acid, C 8 H 3 Cl 2 N 3 O 8 is


formed.

1. What type of reaction takes place?

..................................................................................................................
2. Name the functional group present in Compound R that is confirmed by
this reaction.

..................................................................................................................
[2]

(v) You now have enough information to determine the structural formula of R.

1. Draw the fully displayed structure of R.

2. Explain clearly why you have placed each of the aromatic substituent
groups in their particular positions.

.............................................................................................................................

trendyline
e
............................................................................................................................

.............................................................................................................................
..........................................................................................
.......................................................................................

.............................................................................................................................
..........................................................................................
........................................................................................
[3]
[Total: 15]

IJC 2016 Prelim/9647/02 [Turn over


468 trendyline
For
18 Examiners
Use

BLANK PAGE

trendyline
IJC 2016 Prelim/9647/02 [Turn over
469 trendyline
INNOVA JUNIOR COLLEGE
JC 2 PRELIMINARY EXAMINATION
in preparation for General Certificate of Education Advanced Level
Higher 2

CANDIDATE
NAME

CLASS INDEX NUMBER

CHEMISTRY 9647/02
Paper 2 Structured Questions 18 August 2016
2 hours
Candidates answer on the Question Paper

Additional Materials: Data Booklet

READ THESE INSTRUCTIONS FIRST

Write your index number, name and civics group.


Write in dark blue or black pen.
You may use pencil for any diagrams, graphs or rough working.
Do not use staples, paper clips, highlighters, glue or correction fluid.

Answer all questions in the space provided.


A Data Booklet is provided. For Examiners Use

Section A
You are advised to show all working in calculations.
You are reminded of the need for good English and
clear presentation in your answers.
1 12
You are reminded of the need for good handwriting. 2
Your final answers should be in 3 significant figures. 17

You may use a calculator. 3


10
The number of marks is given in brackets [ ] at the end of each 4
question or part question. 18
5
At the end of the examination, fasten all your work 15
securely together. Significant
figures
Handwriting

Total
72

trendyline
This document consists of 17 printed
nted pages and 1 blank page.
page

Innova Junior College [Turn over

470 trendyline
For
2 Examiners
Use
Answer ALL questions on the spaces provided.

1 Planning

Using the information below, you are to write a plan for determining number of molecules
of water of crystalisation, n, in barium chloride crystals, BaCl 2 .nH 2 O where n = 1,2 or 3.
The basis of this investigation is a technique known as precipitation titration with the use of
silver nitrate solution, AgNO 3 (aq).

AgCl, Ag 2 CrO 4 and BaCrO 4 are sparing soluble salts and relevant information about these
salts are given in the table below.

compound colour Numerical value of K sp at 25 O C


AgCl white 2.0 1010
Ag 2 CrO 4 red 2.0 1012
BaCrO 4 yellow 1.0 1010

Some barium chloride crystals, BaCl 2 .nH 2 O, is dissolved to make up 250.0 cm3 of
standard solution. Approximately 10 cm3 of 0.2 mol dm3 of reagent X solution is added to
a portion of this standard solution to precipitate the barium ions before a few drops of
K 2 CrO 4 indicator solution is added. A titration is then carried out on this portion of solution
against silver nitrate solution, AgNO 3 (aq). Ag 2 CrO 4 would just precipitate only when almost
all of the Cl ions have been precipitated as AgCl. The titration is repeated until a more
reliable average value, V cm3, can be determined.

(a) (i) Identify reagent X. [1]

Na 2 SO 4 (aq) or K 2 SO 4 (aq) [1]

(ii) A student suggested that BaCrO 4 is more soluble than Ag 2 CrO 4 as BaCrO 4
has larger numerical value of K sp than Ag 2 CrO 4 .

Suggest whether this students claim is valid. [1]

This claim is not true as both Ag 2 CrO 4 and BaCrO 4 are of different unit
formula or have different number of ions per formula unit. [1]

Alternatively, students can prove from calculations of solubility of the


two salts.

Solubility of BaCrO 4 = = 1.00 105 mol dm3

Solubility of Ag 2 CrO 4 = = 7.94 105 mol dm3

From the calculations above, Ag 2 CrO 4 has a higher solubility than

ttrendyline
tr
r d li
Ba
BaCrO 4 despite Ag 2 CrO 4 having
aving a smalle
smaller numerical value of K sp than
BaCrO 4 . [1
[1]]

(iii) Suggest why reagent


reage X iss adde
added before
befor a few drops of K 2 CrO 4 indicator
olution is added.
solution [1]

This is to prevent the precipitation of BaCrO 4 that uses up the CrO 4 2-


ions.
IJC 2016 Prelim/9647/02
471
[Turn over
trendyline
For
3 Examiners
Use
(b) You may assume that you are provided with
0.10 mol dm3 silver nitrate

0.2 mol dm3 of reagent X solution

10 g of barium chloride crystals, BaCl 2 .nH 2 O

Potassium chromate solution, K 2 CrO 4 (aq)

The equipment and materials normally found in a school or college


laboratory.

Your plan should include the following

brief, but specific details of the apparatus you would use, bearing in mind the
levels of precision they offer

details, including quantities, for preparation of 250.0 cm3 of BaCl 2 solution


from barium chloride crystals, BaCl 2 .nH 2 O

essential details of the titration procedure

an outline of how the results obtained, including V cm3 , would be used to


determine n, in barium chloride crystals (BaCl 2 .nH 2 O)

Assuming titre volume of AgNO 3 (aq) used is 25.00 cm3 and the volume of BaCl 2
solution pipetted is 25.0 cm3,

Ag+(aq) + Cl(aq) AgCl(s)

BaCl 2 (aq) Ba2+(aq) + 2Cl(aq)

Amt of Ag+(aq) = = 0.00250 mol = Amt of Cl(aq) in 25 cm3

Amt of Cl(aq) in 250 cm3 = 0.00250 = 0.0250 mol

Amt of BaCl 2 (aq) in 250 cm3 = 0.0250 mol 2 = 0.0125 mol

Conc of BaCl 2 (aq) = 0.0125 mol = 0.0500 mol dm3


Assuming n = 1,
Min mass of BaCl 2 .nH 2 O in 250 cm3 = 0.0125 226.0 = 2.825 g
OR
Assuming n = 3,
Max mass of BaCl 2 .nH 2 O in 250 cm3 = 0.0125 262.0 = 3.725 g

Thus the suitable mass of BaCl 2 .nH 2 O to be weighed ranges 2.825 g to 3.725 g.

ttrendyline
d
Preparation
at o of
o standard
sta da d solution
so ut o of
o BaCl
aCl 2

1. Weighgh 2.825g 3.275g g of BaC BaCll 2 ..nH


H 2 O crystals in
nH n a clean and dry weighing
bottlee using a weighing balanc
le balance.
2. Transfer
nsfer BaCll 2 .nH
sfer the BaC H 2 O crystals to a 10 cm3 beaker.
100 cm beak
3. Weighgh the emptied weighing bottle and nd tthe residual BaC
BaCll 2 .nH 2 O crystals to
determine the mass of BaCl 2 .nH 2 O crystals dissolved, m grams, by finding the
difference the weighings in (i) and (iii).
4. Add 100 cm3 of de-ionised water to the BaCl 2 .nH 2 O crystals in the 100 cm3
IJC 2016 Prelim/9647/02
472
[Turn over
trendyline
For
4 Examiners
Use
beaker.
5. Use a glass stirrer to stir the solution in the 100 cm3 beaker to ensure a
homogeneous solution is obtained.
6. Transfer the homogeneous solution carefully to a 250 cm3volumetric flask.
7. Using 50 cm3 of de-ionised water, rinse the beaker, the weighing bottle and the
stirrer and transfer the washings carefully to the solution in the 250 cm3volumetric
flask.
8. Top up to the mark with de-ionised water, stopper the bottle and shake this
solution to obtain a homogeneous solution. Label this solution obtained as FA 2.

Titration Procedures
1. Fill up the burette with AgNO 3 (aq), noting the initial volume reading.
2. Pipette 25.0 cm3 of FA 2 to a 250 cm3 conical flask.
3. Using a 10cm3 measuring cylinder, add 10 cm3 of 0.2 mol dm3 of Na 2 SO 4
solution to the solution in the conical flask. Shake the mixture and let it equilibrate
for about 15 minutes.
4. Add 5 drops of K 2 CrO 4 (aq) indicator solution to the mixture in the conical flask.
5. Carry out the titration until permanent pink colour is obtained. (Colour change is
from white ppt to pink due of 1 drop of AgNO 3 (aq) in excess to cause
precipitation of red Ag 2 CrO 4 )
6. Repeat titration until at least 2 consistent titre readings are obtained (titre volume
differ not more than 0.10 cm3). Use the average of these 2 consistent titres to
obtain volume, V cm3 for subsequent calculations.

Calculation Procedure

Amt of Cl ions in titration (25 cm3) = = 0.000100V mol


3
Amt of Cl ions in 250 cm = 0.000100V x 10 = 0.00100V mol
Amt of BaCl 2 = 0.00100V 2 = 0.000500V mol
Mass of BaCl 2 = 0.000500V 208.0 = 0.104 V g
Mass of water of crystalisation = (m  V ) g
Amt of water of crystalisation = ((m  V ) 18) mol

No of molecules of water of crystalisation, n


= Amt of water of crystalisation Amt of BaCl 2
= ((m  V ) 18) 0.0000500V

Marking points

1. use of appropriate apparatus with correct capacities i.e. weighing bottle


for measuring mass of BaCl 2. nH 2 O crystals, 250 cm3 volumetric flask to
prepare BaCl 2 solution, 10 cm3 measuring cylinder for measuring vol of
reagent X solution, pipette+burette+ 250 cm3 conical flask
2. Appropriate volumes of solutions pipetted being 20.00 25.00 cm3
based on appropriate choice of titre volume of 20.00 25.00 cm3 and max
of 5 drops of indicator to be used.
3. appropriate mass of BaCl 2. nH 2 O crystals used = 2.825g 3.275g
4. Quantitative transfer of BaCl 2 solution to volumetric flask or mass
difference of weighing bottle+ crystals and mass of emptied weighing

trendyli
trendyline
dylil e
bottle to determine mass of B BaCl
BaCl 2. nH 2 O crystals
cr
5. Repeat titration until two consistent readings (+ 0.10 cm3) are obtained.
6. Correct
Correc colour change (white to pink or red) red at end point
7. Derivation of mass of water of crystallisation
8. Derivation n being mole ratio of water
wa of crystallisation to BaCl 2
[8]
(c) Explain why this titration cannot be conducted under acidic or alkaline medium. [1]
IJC 2016 Prelim/9647/02
473
[Turn over
trendyline
For
5 Examiners
Use

Under acidic medium, CrO 4  ions will undergo condensation or acid-base


reaction to form Cr 2 O 7  ions, thus the indicator ions of CrO 4  will be absent and
the end-point of the titration cannot be detected. [1]

OR

Under alkaline medium, Ag+ ions will be precipitated as Ag 2 O resulting in a much


higher than accurate titre reading. [1]
[Total: 12]

trendyline
IJC 2016 Prelim/9647/02
474
[Turn over
trendyline
For
6 Examiners
Use

2 This question is about the oxides of nitrogen and its reactions. Oxides of nitrogen
constitute air pollutants originating from emission of car exhaust that cause acid rain
and photochemical smog.

The two gases of NO and N 2 O 4 slowly react to form the blue compound, N 2 O 3
according to the following equation.

2NO(g) + N 2 O 4 (g) 2N 2 O 3 (g)

(a) (i) Draw the dot and cross diagrams of the two molecules, NO and N 2 O 3 .
The N 2 O 3 molecule containVD11ERQG.
[2]
xx
x
O xO x
x x x
x x x
N N N
x x O
xx x x
x
x O
xx
x

N2O3 NO
(ii) From your answer in (a)(i), suggest why the forward reaction is likely to
occur.
[1]

The NO molecule contains an unpaired electron/lone electron>@ or The


NO molecule is a free radical.
Thus, it is highly reactive/unstable>@, and reacts to form N 2 O 3 molecule
(that has all atoms attaining noble gas configuration).

(b) In an experiment, a mixture containing NO and N 2 O 4 was introduced into a 1.48


dm3 evacuated vessel was allowed to reach equilibrium at 280 K. The equilibrium
pressure is 98.9 kPa.

Calculate the total number of moles of gases at equilibrium, assuming the gases
behave ideally.
[2]

39 Q57Q = 0.0629 mol [1]


[1] for working (correct unit conversion)

trendyline
(c) Two colourless gases NO and N 2 O 4 slowly react to form the blue compound,
IJC 2016 Prelim/9647/02
475
[Turn over
trendyline
For
7 Examiners
Use
N 2 O 3 according to the following equation.

2NO(g) + N 2 O 4 (g) 2N 2 O 3 (g)

Equimolar mixtures of NO and N 2 O 4 are mixed at varying pressure P but at two


different temperatures of 300 K and 330 K, and the variation in colour intensity
was monitored over a period of time.

The graphs below show the variation of the colour intensity with pressure at
temperatures of 300 K and 330 K.

colour 300 K
intensity
330 K

0 Pressure/ atm
(i) What is the significance of the colour intensity in this reaction? [1]

The colour intensity changes with changes in partial pressure/ concentration/


amount of N 2 O 3 produced. [1]

Accept any answers of similar meaning.

(ii) Explain why colour intensity of the reaction mixture increases with increasing
pressure. [1]

When the pressure is increased, the equilibrium position shifts right>@,


to decreasing the number of moles of gaseous molecules>@ present.

Or

An increase in pressure favours the forward reaction, causing more N 2 O 3


to be formed. Hence, the colour intensity increases.

(iii) Using information from the graphs, state and explain whether the formation
of N 2 O 3 from NO and N 2 O 4 is an exothermic reaction.
[2]

At the higher temperature of 330 K, the colour intensity is lower. This


implies less N 2 O 3 is produced [] and hence indicates that the backward

ttrendyline
d li
reaction
action is favoured at higher temperatures.
temperatures []

Byy Le Chateliers principle, when


en temperature of a system is increased,
the
he e system favours endothermic reaction in order to remove the
excess
xcess heat []
[
]
Since
i the
h bbackward
k d reaction
i iis ffavoured/
d/ lless products
d are formed,
the forward reaction is exothermic. []

IJC 2016 Prelim/9647/02


476
[Turn over
trendyline
For
8 Examiners
Use
Or

At the lower temperature of 300 K, the colour intensity is higher. This


implies more N 2 O 3 is produced [] and hence indicates that the forward
reaction is favoured at lower temperatures. []

By Le Chateliers principle, when temperature of a system is decreased,


the system favours exothermic reaction in order to produce heat [].

Since the forward reaction is favoured/ more products are formed,


the forward reaction is exothermic. []

(d) Nitrogen monoxide reacts with chlorine to form nitrosyl chloride, according to the
equation:

2NO(g) + Cl 2 (g) 2NOCl(g).

In an experiment, the amount of Cl 2 (g) was kept in large excess while the initial
partial pressure of NO(g) was varied at constant temperature of 500K.
The table below shows the experimental results obtained.

time / s P NO / atm (Rate / P NO ) (Rate / (P NO )2)


/ s1 / (atm  s1)
0 0.917 1.033 104 1.126 104
1000 0.827 9.312 105 1.126 104

2000 0.753 8.486 105 1.127 104

3000 0.691 7.788 105 1.127 104

4000 0.638 7.190 105 1.127 104

(i) Suggest why the amount of Cl 2 (g) was kept in large excess.
[1]

Since the amount of Cl 2 (g) was kept in large excess, the partial pressure
of Cl 2 (g) remains relatively constant [1] so that any change in rate is due
to the changes in the partial pressure of NO(g) only.

(ii) Using the data from the table above, deduce the order of reaction with
respect to NO(g).
[1]

trendyline
tren d li e
Sincec Rate / (P NO )2 is constantt throughout the
ince t reaction [] (and thus rate
off reaction is directly proportional to (P NO )2),, the
he reaction is s
second order
with
ith
th respect to NO
NO.. [
[]
]

(iii) In another experiment, the initial partial pressure of NO(g) wa


was 4.2 atm and it
was
as reacted with Cl 2 (g) at a constant
nst temp
temperature
erature of 500 K. The partial
pressure of Cl 2 (g) was recorded at time intervals of 30 s. The data obtained
are tabulated below.
IJC 2016 Prelim/9647/02
477
[Turn over
trendyline
For
9 Examiners
Use

Time / s Partial pressure Time / s Partial pressure


of Cl 2 (g) / atm of Cl 2 (g) / atm
0 0.78 300 0.49
30 0.76 330 0.46
60 0.72 360 0.44
90 0.70 390 0.42
120 0.66 420 0.39
150 0.63 450 0.38
180 0.59 480 0.36
210 0.57 510 0.34
240 0.54 540 0.33
270 0.52 570 0.32

Using the data but without the plotting of any graph, deduce the order of
reaction with respect to Cl 2 (g). [1]

1st t 1/2
= time taken for P Cl2 to decrease from 0.78 atm to 0.39 atm
= 420 s

2nd t 1/2
= time taken for P Cl2 to decrease from 0.72 atm to 0.36 atm (or 0.66 0.33)
= 480 60
= 420 s

Since t 1/2 is constant at 420s [], the reaction first order with respect to Cl 2 .
Hence order of reaction with respect to Cl 2 = 1 []

(iv) Write the rate equation for this reaction.


Hence, calculate the rate constant, including its units [3]

Rate = k (P NO )2(P Cl2 ) [1] , allow ecf based on answers in (ii) and (iii)
Rate = k (P Cl2 ) where k = k (P NO )2
t 1/2 = 420 s = =

k = 9.36 105 [1] atm2 s1 [1]

(v) One possible mechanism of the reaction is given below.

Step I NO(g) + Cl 2 (g) NOCl 2 (g) fast

trendyline
Step
ep II
tep g)
NOCl 2 (g) + NO(g) 2NOC
2NOCl(g)
NOCll(g) slow

Explain
plain whether it is consistent with the observed kinetics data
xplain d in (d)(iv).

[2]]

IJC 2016 Prelim/9647/02


478
[Turn over
trendyline
For
10 Examiners
Use
If Step II is the rds: rate = k (P NOCl2 )(P Cl2 )

Since rate equation should not contain NOCl 2 intermediate, from step 1:
PNOCl2
Kp
PNOPCl2
Hence, PNOCl2 K pPNOPCl2

Substitute into the rate equation:

rate = k (P NOCl2 )(P Cl2 ) = k K p (P NO )(P Cl2 )(P NO ) = k (P NO )2(P Cl2 ) [1]

Step II is the rate-determining step since its rate equation is consistent


with the experimentally determined rate law, rate = k (P NO )2(P Cl2 ). [1]
[Total: 17]

trendyline
IJC 2016 Prelim/9647/02
479
[Turn over
trendyline
For
11 Examiners
Use

3 Organic compounds can undergo combustion. Chemical companies produce containers


filled with butane for use by campers.

The enthalpy change of combustion of butane is 3000 kJ mol 1.

(a) (i) Define the term standard enthalpy of combustion of butane.[1]


It is enthalpy change when 1 mole of butane is burnt in excess oxygen under
standard conditions of 298K and 1 atm. [1]

(ii) Calculate the mass of water at 25 C that could be brought to the boiling point
by the combustion of 1.2 dm3 of butane gas. Assume 75% of the heat from the
butane is absorbed by the water. [2]

Heat = mcT
Moles of butane = (1.2) / 24 = 0.05 moles
Heat released = 3000 x 0.05 = 150 kJ
Heat absorbed = 0.75 x 150 = 112.5 kJ [1]
112.5 x 103 = m x 4.18 x (100 25)
m = 358 g [1]

(b) (i) Explain in terms of structure and bonding, why butanone has a higher boiling
point than pentane. [2]
Both butanone and pentane have simple molecular structures>@ with similar
Mr
Butanone has a higher boilng point as it requires more energy>@
to overcome its stronger p.d-pd interactions between its molecules
>@FRPSDUHGWRid-id interactions between butane molecules. >@

>@ 2m (accept answers along this line SBE, comparing strength)

(c) Organic compounds are also widely used for pharmaceutical purposes such as
ephedrine which is an anti-asthmatic and stimulant. Ephedrine can be converted into
ethylephedrine via a S N 2 reaction.

OH OH
N N
H

Ephedrine Ethylephedrine

(i) Suggest the reagent that can be used to perform this conversion.[1]
CH 3 CH 2 I [1]

(ii) Explain why an S N 2 mechanism is favoured for this reaction. [1]


There is only 1 bulky alkyl group bonded to the carbon of the CH 3 CH 2 I
that could possibly block the backside approach of the nucleophile. Thus

trendyline
d
there
ere is less steric hindrancece in thee halogenoalkane
ha and
a hence S N 2
mechanism favoured [1]
echanism is favoured.

(d) Completee hydrolysis of proteins produces individual units of amino acids, but partial
ete
hydrolysis
sis can break the protein down into
to dipeptide
d fragments.
or tripeptide frag
Partial hydrolysis of a tetrapeptide (containing four amino acid residues) produces
the following three dipeptides, as well as the individual amino acids.
IJC 2016 Prelim/9647/02
480
[Turn over
trendyline
For
12 Examiners
Use
NH 2 CH 2 CONHCH(CH 3 )CO 2 H
NH 2 CH 2 CONHCH 2 CO 2 H
NH 2 CH(CH 3 )CONHCH(CH 3 )CO 2 H

(i)
Define the primary structure of a protein [1]

Primary structure is the sequence of amino acids in a polypeptide chain[1]

(ii) Deduce the order in which the amino acids are bonded together in the
tetrapeptide. [1]
NH 2 CH 2 CONHCH 2 CONHCH(CH 3 )CONHCH(CH 3 )CO 2 H [1]

(iii) Suggest suitable reagents and conditions to hydrolyse the tetrapeptide into its
individual amino acids. [1]
Reagents : dilute H 2 SO 4 / HCl >@
Condition: prolonged heating (many hours) >@

[Total: 10]

4 (a) A 0.031 mol dm-3 solution of a base, MOH, has a pH of 12.5. (M is a metal.)

25 cm3 of the solution of MOH was titrated with 0.025 mol dm-3 propanoic acid,
CH 3 CH 2 COOH, at 25 C. The pH of the solution was followed using a pH meter and
the following titration curve was obtained.

Ka of propanoic acid = 1.29 x 10-5 mol dm-3

(i) Calculate the concentration of hydroxide ions present in the sample of MOH
and use it to explain whether it is a strong or weak base. [1]
pH = 12.5
pOH = 14 12.5 = 1.5
[OH-] = 10-1.5 = 0.0316 mol dm-3 []

(ii) trendyline
OH has dissociated completely, thus it is a strong base [].
MOH

alculate the value o


Calculate of V
V..
CO -M+ + H 2 O
H 3 CH 2 COOH + MOH CH 3 CH 2 COO
CH
[].

[1]

Amt of MOH = 0.031 x 25/1000


= 7.75 x 10-4 mol
IJC 2016 Prelim/9647/02
481
[Turn over
trendyline
For
13 Examiners
Use

Amt of CH 3 CH 2 COOH required = 7.75 x 10-4 mol []

Volume of CH 3 CH 2 COOH required = (7.75 x 10-4) / 0.025


= 0.031 dm3
= 31.0 cm3

V = 31.0 []

(iii) Explain, with the aid of an equation, why the pH when V cm3 of propanoic acid
was added, is greater than 7. [2]
When V cm3 of acid is added, the acid and base react completely to produce
a basic salt, which undergoes hydrolysis in water to produce hydroxide
ions / an alkaline solution. [1]

CH 3 CH 2 COO- (aq) + H 2 O (l) CH 3 CH 2 COOH (aq) + OH- (aq) [1]

(b) The colour of blackberries is due to a compound known as cyanidin. Cyanidin is a


weak organic acid which may be represented by CyH. In aqueous solution, CyH
dissociates slightly:

CyH (aq) Cy (aq) + H+ (aq)


red purple

The colours of CyH and Cy are indicated in the above equation.


Acid dissociation constant, K a , of CyH is 5.0 x 10-5 mol dm-3 at 25 C.

A glass of blackberry juice has a pH of 3.00 at 25 C. Calculate the ratio of the red to
purple form in the juice, and hence predict its colour. [3]
When a mix of CyH and Cy- is present, system is an acidic buffer.

pH = pK a + log 10
salt
acid

3.00 = - log 10 (5.0 x 10-5) + log 10


Cy - [1]
CyH
Cy - = 0.05
CyH
CyH = 20 : 1 [1]
Cy -
Colour of juice: red [1]

(c) When chlorine is bubbled through cold sodium hydroxide solution and acidified silver

trendyline
nitrate solution,
olution, only half of the chlorine that has d prec
dissolved is precipitated as silver
e. When the sodium hydroxide is hot, up to fivesixth
chloride. five
fivesixth
sixth of the chlorine can be
precipitated.
ted. Explain the observations, giving balanced equations wh
ated. where appropriate.
[3]

OH 2OH ((aq)) + Cl 2 ((aq)) Cl ((aq)) + ClO ((aq)) + H 2 O (l)


cold NaOH: [1]

IJC 2016 Prelim/9647/02


482
[Turn over
trendyline
For
14 Examiners
Use
hot NaOH: 6 OH (aq) + 3 Cl 2 (aq) 5 Cl (aq) + ClO 3 (aq) + 3 H 2 O (l) [1]

The free chloride ions will be precipitated out as silver chloride with silver nitrate.
When chlorine reacts with cold NaOH, mole ratio of Cl: ClO = 1 : 1 []
When chlorine reacts with hot NaOH, Cl : ClO 3 = 5 : 1 []

(d) The table below gives data about some physical properties of the elements calcium,
nickel and copper.

calcium nickel copper


atomic radius / nm 0.197 0.124 0.128
electronic [Ar]4s2 [Ar]3d84s2 [Ar]3d104s1
configuration

(i) What do you understand by the term transition element? [1]


A transition element is a d-block element that forms at least one stable ion
with a partially filled d-subshell. [1]

(ii) Although the nickel and copper atoms have more electrons than the calcium
atom, the atomic radii of nickel and copper are smaller than that of calcium.
Suggest an explanation for this. [3]
Nuclear charge of Ni and Cu are larger. [] However, shielding effect of 3d
electrons of Ni and Cu is poorer [1] than that of 3s and 3p electrons of Ca
because 3d orbitals are more diffused. Hence, Ni and Cu have larger
effective nuclear charge. [1] Valence electrons of Ni and Cu are attracted
more strongly to nucleus [], so have smaller atomic radii.

OR

Ni and Cu has higher nuclear charge than Ca. []


For Ni and Cu, electrons are added to the inner 3d subshell. These
inner 3d electrons shield the outer 4s electrons more effectively. []
For Ni and Cu, the effect of increasing nuclear charge is reduced by the
increasing shielding effect. / the increase of nuclear charge outweighs
the increase of shielding effect [1]
Thus effective nuclear charge of Ni and Cu is slightly higher than
Ca []
Valence electrons of Ni and Cu are attracted more strongly to
nucleus [], so have smaller atomic radii

trendyline
IJC 2016 Prelim/9647/02
483
[Turn over
trendyline
For
15 Examiners
Use
(e) The following are some reactions involving copper(II) nitrate.
hydrated excess water KHCO (aq)
3
copper(II) Cu(NO3)2 (aq) blue ppt + CO
2
nitrate solid
excess NH (aq)
3

D
dark blue solution

excess KCN(aq)

E
violet solution

(i) Explain why carbon dioxide is evolved when KHCO 3 (aq) is added to aqueous
copper(II) nitrate. Include any relevant equations with state symbols in your
answer. [2]
A solution of Cu2+ is acidic [] in nature due to the high charge density [] of
the cation and is able to polarise the electron cloud of water molecule.

[Cu(H 2 O) 6 ]2+(aq) + H 2 O(l) [Cu (H 2 O) 5 (OH)]+(aq) + H 3 O+(aq)


(good to have but not necessary)

2H+ + CO 3 2- CO 2 + H 2 O [1]

(ii) Suggest a formula for the complex ion present in D. [1]


[Cu(H 2 O) 2 (NH 3 ) 4 ]2+ OR [Cu(NH 3 ) 4 ]2+

(iii) The metal ion in complex E is known to have a coordination number of four.
Suggest a formula for complex ion E. [1]
[Cu(CN) 4 ]2-
[Total: 19]

5 Benzene reacts with propanoyl chloride and 1-phenylpropan-1-one is formed in the


reaction.

(a) State the reagents and conditions needed to convert 1-phenylpropan-1-one into
compound Q. Show the structure of the intermediate in the box provided.

O OH
Step 1 Step 2
C CH2CH3 C COOH

CH2CH3
- - -
1 phenylpropan 1 one compound Q

Reagents
nts

Step 1

Step 2
trendyline
s and condition
conditions

.
[3]

IJC 2016 Prelim/9647/02


484
[Turn over
trendyline
For
16 Examiners
Use

O OH OH
Step 1 Step 2
C CH2CH3 C CN C COOH

CH2CH3 CH2CH3

[1]

Step 1 HCN, trace amt of NaOH/NaCN, 10-20oC (can they use cold?) [1]

Step 2 Dilute or (aq) HCl/H 2 SO 4 , heat [1]

(b) Describe the mechanism in step 1 in part(a). In your answer, you should show all
charges and lone pairs and show the movement of electrons by curly arrows.
[3]
Mechanism: Nucleophilic Addition[1]

Step 1:
HCN H+ CN-
1D&11D+ + CN-

Step 2: Nu attack on butanone to form intermediate

Step 3: Protonation

Step 4: Regeneration of catalyst

Correct mechanism (include all important aspect)


-correct arrows
-partial charges
-correct intermediate [2]

(c) The 2,4-D weed killer, C 8 H 6 Cl 2 O 2, is widely used on cereal crops, pastures and
orchards. Compound R is an isomer of the weed killer under development to improve
its properties. It is an aromatic compound which contains three functional groups.

The following tests are carried out in the order given. In each case, state all
deductions about the compound R you can make at that stage. When identifying the
functional groups, your answers should be unambiguous.

(i) When Compound R is treated with Na, C 8 H 5 Cl 2 O 2 Na is formed.

deduction(s)..........................................................................................................

..........................................................................................................................[1]
Acid metal reaction.[@
Possible functional groups present: alcohol or phenol or carboxylic acid>@
>@ [1]
Must give both functional possible functional gps based on 2 O atoms.

(ii)

trendyline
When Fehlings solution is added to Compound R,
obtained.
btained.
tained
R, red br
brick precipitate is

deduction(s)..........................................................................................................
eduction(s)...............................................................................
duction(s).............................................................................

..........................................................................................................................[1]
Mild oxidation occurs. [@
IJC 2016 Prelim/9647/02
485
[Turn over
trendyline
For
17 Examiners
Use
Aliphatic aldehyde is present >@
2>@ - [1] [aldehyde alone is not accepted]

(iii) When 1 mole of Compound R is boiled with ethanolic silver nitrate, 287 g of
white precipitate is formed.

deduction(s)..........................................................................................................

.............................................................................................................................

.............................................................................................................................

..........................................................................................................................[2]

2 moles of AgCl is formed with 1 mole of compound R. This suggests the


presence of 2 aliphatic chloro substituted atoms undergone nucleophilic
substitution.
Alphatic halogeno compound is present or chlorine atoms are on the aliphatic
side chain and not bonded to the ring.

[1]: identify aliphatic halogeno compound


[1] : identify two Cl aliphatic atoms

trendyline
IJC 2016 Prelim/9647/02
486
[Turn over
trendyline
For
18 Examiners
Use

(iv) When Compound R is treated with concentrated nitric acid, C 8 H 3 Cl 2 N 3 O 8 is


formed.

1. What type of reaction takes place?

.........................................................

Electrophilic substitution[1]

2. Name the functional group present in Compound R that is confirmed by


this reaction.

........................................................
[2]
Phenol- [1]

aromatic compound not accepted as catalyst will be required

(v) You now have enough information to determine the structural formula of R.

1. Draw the fully displayed structure of R.


+
2

&O 2
& & +
&O side chain must be at position 3

2. Explain clearly why you have placed each of the aromatic substituent
groups in their particular positions.

[3]
As phenol is 2- and 4- directing, and trisubstitution took place, as seen by
3NO 2 substituent groups added in the electrophilic substitution reaction. Thus,
the side chain is not on the 2-, 4- or 6- position but on position 3.
[1m for explanation along the lines]
+
2
12
21

&O 2
& & +
12 &O

trendyline
&+&O12
[1m] tri-substitution
m] tri-
tri-substitution takes position 2,4,6
m] side chain on 3rd position
[1m] ositio
[Total:15]

IJC 2016 Prelim/9647/02


487
[Turn over
trendyline
INNOVA JUNIOR COLLEGE
JC 2PRELIMINARY EXAMINATION
in preparation for General Certificate of Education Advanced Level
Higher 2

CHEMISTRY 9647/03
Paper 3 Free Response 26 August 2016
2 hours
Candidates answer on separate paper.

Additional Materials: Writing Papers


Data Booklet
Cover Page

READ THESE INSTRUCTIONS FIRST

Write your name and class on all the work you hand in.
Write in dark blue or black pen on both sides of the paper.
You may use a soft pencil for any diagrams, graphs or rough working.
Do not use staples, paper clips, highlighters, glue or correction fluid.

Answer any four questions.


Begin each question on a fresh sheet of paper.

You are advised to show all working in calculations.


You are reminded of the need for good English and clear presentation in your answers.
You are reminded of the need for good handwriting.
Your final answers should be in 3 significant figures.

At the end of the examination, fasten all your work securely together.
The number of marks is given in the brackets [ ] at the end of each question or part question.

trendyline
y This document consists of 12printed pages
pages.

Innova Junior College [Turn over


488 trendyline
2

Answer any four questions.

1 Data concerning the elements of group II of the Periodic Table, at 298K, are given in the
table. Further data may be found in the Data Booklet. M refers to elements of group II.
st nd 2+
1 and 2 IE of M + atom of M + hydration of M E /V
-1 -1 -1
/ kJ mol / kJ mol / kJ mol
Ca 1740 178 1650 2.87
Sr 1608 164 1480 2.89

(a) The electrode reduction potential E of Group II elements gives an indication of the
ease with which the following reaction occurs:

M(s) M2+(aq) + 2e Reaction 1

The enthalpy change of reaction 1 can be related to the following factors:


first and second ionisation energy of the element
enthalpy change of atomisation and
enthalpy change of hydration of its gaseous ion.

(i) Construct an energy cycle relating the factors above, and use the values given
above to calculate the enthalpy change of Reaction 1 for calcium. [3]

(ii) By quoting appropriate data from Data Booklet, explain the difference in
enthalpy change of hydration of calcium and strontium ions. [2]

(b) Calcium oxide is the key ingredient for the process of making cement.

When calcium is burned in oxygen, calcium oxide is produced.


When 1.50g calcium is burned in air, a mixture of ionic solids calcium oxide and red
brown solid is formed.

The red brown solid has the following composition by mass: Ca, 81.1%; N,18.9%.

Adding water to the red brown solid produces calcium hydroxide and 19.2 cm3 of
ammonia gas at room temperature and pressure.

(i) Calculate the formula of the red brown solid. [1]

(ii) Write an equation for the reaction between red brown solid with water. [1]

(iii) Write equations for the reactions of calcium with air and use them to calculate
the mass of CaO formed when 1.5g of calcium is burnt in air. [3]

(c) Calcium oxide is normally made by heating calcium carbonate to a temperature above
825oC. A gas which turns lime water chalky is produced in the process.

trendyline
(i) Writete an equation for the decomposition of CaCO 3 .
[1]]

(ii) xplain
plain why barium carbonate decomposes at a higher temperature than
Explain
alcium
cium carbonate.
calcium [2]

IJC 2016Prelim/9647/03[Turn over 489 trendyline


3

(d) Methylcyclohexene reacts with hydrogen bromide to form alkyl bromides.

+%U &+%U

PHWK\OF\FORKH[HQH

(i) The above reaction could produce two isomers with molecular formula C 7 H 13 Br.
However, when the reaction is carried out, mainly one product is formed. Give
the structural formula of the major product, explaining your answer. [2]

(ii) Describe the mechanism of the reaction between hydrogen bromide and
methylcyclohexene, C 6 H 9 CH 3 , showing curly arrows, charges, dipoles and any
relevant lone pairs. [3]

(iii) Unsymmetrical dihalides reacts with alkene in a similar way as hydrogen halides.
Predict the structure of the product when IBr reacts with methylcyclohexene, and
explain your answer by considering the polarity within the molecule IBr.[2]

[Total: 20]

trendyline
IJC 2016Prelim/9647/03[Turn over 490 trendyline
4

2 Alcohols and carboxylic acids have many scientific, medical and industrial uses worldwide.

(a) Methanol is made from the following gaseous reaction.

CO(g) + 2H 2 (g) CH 3 OH(g)

A 1:2 mixture of CO and H 2 was pumped into a sealed reactor at 6 atm and then
heated to 227 oC and maintained at this temperature for some time. When equilibrium
was reached, the partial pressure of CH 3 OH was found to be 1.6 atm.

(i) Write an expression for K p for the reaction.Hence, determine the value of
K p stating its units.
[3]

(ii) Suggest the effect on the position of equilibrium and value of K p when methanol
is removed from the reaction vessel. Explain your answer. [2]

(b) Grignard reagent, RMgX, is an important reagent in organic synthesis. Grignard


reagents are prepared by the addition of activated magnesium on an alkyl halide or
aryl halide in non-polar solvents such as ether in anhydrous conditions.

- +
R CH2 X + Mg R CH2 MgX X = Cl or Br

An example of a Grignard reagent reacting with a ketone to form an alcohol via a two
stage process is given below:
R' R

Stage 1 : R CH2 MgCl + C O R CH2 C O MgCl

R'' R''

R' R'
H2O
Stage 2 : R CH2 C O MgCl R CH2 C OH + Mg(OH)Cl

R'' R''

R' and R'' could be alkyl / aryl group / H atoms

(i) Suggest the type of reaction occurred between the Grignard reagent and the
carbonyl compound in Stage 1 and 2. [2]

(ii) Suggest the structural formula of the final organic product formed when

CH2MgBr

t
trendyline
is reacted
ed with propanone,
propanone
propan , CH 3 COCH 3 , in a similar two-
tep
ep process.
step [1]

(iii) educe the structures of a suitable Grignard reagent, RMgBr,


Deduce RMg
RMgB and a suitable
arbonyl compound, RRCO, tto synthesize the following
carbonyl fo alcohol,
(CH 3 ) 2 CHCH 2 CH(OH)CH 2 CH 2 CH 3 .
[2]

IJC 2016Prelim/9647/03[Turn over 491 trendyline


5

(iv) The Grignard reagent CH 3 CH 2 CH 2 MgBr can be readily converted into a


carboxylic acid by using carbon dioxide.

Suggest the structural formula of the organic product formed. [1]

(v) The product formed by using Grignard reagent in (b) (iii)does not rotate the
plane polarised light. Explain your reasoning. [2]

(c) Describe and explain the relative acidities of phenol, benzoic acid and
phenylmethanol. [3]

(d) Lactic acid, glycolic acid and salicylic acid are three organic acids commonly used in
chemical peel.

CH3CHCO2H CH2CO2H OH

OH OH
COOH
lactic acid glycolic acid salicylic acid

Suggest a simple chemical test that can be used to distinguish each of the following
pairs.State what you would observe for each chemical test.

1. lactic acid from glycolic acid, and


2. glycolic acid from salicylic acid.

[4]

[Total: 20]

trendyline
IJC 2016Prelim/9647/03[Turn over 492 trendyline
6

3 Aluminium, is the third most abundant element (after oxygen and silicon) found in the
Earths crust. Aluminium reacts readily with carbon to form aluminium carbide, Al 4 C 3 .

(a) Aluminium carbide can react with protic reagents such as hydrochloric acid to produce
methane gas and aluminium(III) chloride.

(i) Write a balanced equation for the reaction between aluminium carbide and
hydrochloric acid. [1]

(ii) Using the equation in (a)(i), calculate the maximum volume of methane gas that
can be produced when 12.0 g of aluminium carbide reacts with 250 cm3 of2.0
mol dm-3 hydrochloric acid at standard temperature and pressure.
[3]

(b) Aluminium chloride is used as a catalyst in electrophilic substitution reactions. The


chlorination of benzene is represented by the following overall equation.

Cl
AlCl3
+ Cl + HCl
2

The reaction occurs in several steps.

The first step is the reaction between Cl 2 and AlCl 3 .

Cl 2 + AlCl 3 Cl + AlCl 4

The benzene ring is then attacked by the Clcation in the second step.

AlCl 3 reacts in a similar way with halogenoalkanes and acyl chlorides, producing a
carbocation that can then attack a benzene ring.

Using the reactions described above as parts of a synthesis route, a student


suggested the following route to form phenyl 4-phenylbutanoate, from benzene.
I
Cl Cl OH
with AlCl3

step 1 step 2

A
step 3

O OH

O Route B O
-
phenyl 4 phenylbutanoate 2 steps

(i)

(ii) trendyline
xplain why step 1 has to be carried out in an anhydrous condition.
Explain

raw
aw the carbocation formed in step 1.
Draw
cond [1]

[1]

IJC 2016Prelim/9647/03[Turn over 493 trendyline


7

(iii) A student suggested that A could have the following structure:

By quoting appropriate data in the Data Booklet, explain why the structure
suggested by the student is less likely to be formed. [2]

(iv) Give appropriate reagents and conditions necessary for steps 2 and 3. [2]

(v) Suggest the reagents and conditions for each of the two steps in route B. [2]

(vi) Predict the structure of the product of each of the following reactions. Both
compoundsEandFreact with 2,4-dinitrophenylhydrazine.[2]

AlCl3 E
Cl C9H8O
O
C
O
AlCl3 F
Cl C9H8O

(vii) Describe and explain the relative ease of hydrolysis of the two
chloro-compounds C and D. [2]

(c) By considering the structure and bonding, explain why

(i) the boiling point of aluminium fluoride is higher than that of aluminium chloride.
[2]

COOH COOH
OH
(ii) salicylic acid, is less soluble in water than benzoic acid, .

[2]

trendyline [Total: 20]

IJC 2016Prelim/9647/03[Turn over 494 trendyline


8

4 The Strecker synthesisis a term used for a series of chemical reactions that synthesize an
amino acid from an aldehyde or ketone. This reaction requires acid and HCN is supplied
from cyanide salts.

NH2
Stage 1 Stage 2 NH2
O
KCN H+ O
R R
R H NH4Cl N

OH

One example of the Strecker synthesis is a multikilogram scale synthesis of amino acids.

(a) Name the type of reaction occurring in Stage 2 and state the reagent and conditions
used in this reaction. [2]

(b) InStage 1, ammonia formed from the reaction between KCN and NH 4 Cl reacts with
the aldehyde.
Suggest an equation for the formation of ammonia from the reaction between KCN
and NH 4 Cl and describe the role of KCN in stage 1. [2]

(c) Explain why isobutyramide is less basic than 3-aminobutan-2-one.

NH2 O NH2
- - -
isobutyramide 3 aminobutan 2 one
[2]

(d) Predict the outcomes of the following reactions starting from isobutyramide, drawing
the structures of the intermediate Jand the productsKand L.

O
LiAlH4 excess CH3Cl
J K
C7H18NCl
NH2
isobutyramide

NaOH(aq),
reflux

trendyline
L + gas

[3]

IJC 2016Prelim/9647/03[Turn over 495 trendyline


9

(e) Compound P, C 9 H 12 O 2 , shows optical activity and does not react with aqueous
potassium carbonate. However, it is soluble in aqueous potassium hydroxide.
When P reacts with hot acidified potassium dichromate(VI), the solution turns from
orange to green. P also decolourises aqueous bromine to form a white precipitate,Q,
C 9 H 10 O 2 Br 2 .

Upon heating with concentrated sulfuric acid, P gives only compound R, C 9 H 10 O,


which displays geometric isomerism. R gives compound S and ethanoic acid on
reacting with hot acidified potassium manganate(VII) solution.

Suggest the structuresof P, Q, Rand S, explaining the reactions involved. [7]

(f) The structure of squaric acid is shown below.

O OH

O OH

(i) State the functional groups in squaric acid.[2]

(ii) Squaric acid is a stable molecule although it has high angle strain associated
with a four-membered ring. Itis a strong diprotic acid with a pK a values 1.5 and
3.4 respectively. Explain the high acidity of squaric acid. [2]

[Total: 20]

trendyline
IJC 2016Prelim/9647/03[Turn over 496 trendyline
10

5 Iron is, by mass, the most common element on Earth, forming much of Earth's outer and
inner core. Iron and its compounds have been used as catalysts in many reactions. For
example, aqueous iron(II) chloride can be used to catalyse the reaction between I and
S 2 O 8 2, to form I2 and SO 4 2ions.

(a) (i) Write an equation to represent the third ionisation energy of Fe. [1]

(ii) Write the electronic configuration for Fe2+. Hence, suggest why the third
ionisation energy of 26 Fe is lower than that of 25 Mn. [2]

(iii) State the type of catalysis for the reaction between I and S 2 O 8 2 that is
catalysed by aqueous iron(II) chloride. [1]

(iv) The reaction between I and S 2 O 8 2 can also be catalysed by aqueous cobalt(II)
chloride. Using relevant E values from the Data Booklet, explain why cobalt(II)
chloride can be a catalyst for this reaction. [2]

(b) A green solution of aqueous iron(II) chloride is reacted with acidified potassium
manganate(VII). The resultant solution D is treated with aqueous sodium carbonate to
form a reddish brown precipitate E, together with a colourless gas which forms white
precipitate in aqueous calcium hydroxide. When aqueous NH 4 SCN is added to D, a
blood red solution is formed.

(i) Explain why aqueous iron(II) chloride is coloured. [2]

(ii) Identify precipitate E. [1]

(iii) Suggest the type of reaction that has occurred for the formation of the blood red
solution. [1]

trendyline
IJC 2016Prelim/9647/03[Turn over 497 trendyline
11

(c) Iron is also used in making iron-air battery which is a low-cost and environmental
friendly rechargeable energy source. An iron-air battery comprises of iron electrode,
porous carbon-containing air electrode. The electrolyte used is aqueous potassium
hydroxide.
external circuit
e e

Fe3O4 OH O2

O2
Fe O2
OH
O2

KOH(aq) O2
iron electrode air electrode
separator

At the iron electrode, iron is oxidised to magnetite, Fe 3 O 4 , during discharging,


according to the following reaction:

3Fe + 8OH Fe 3 O 4 + 4H 2 O + 8e

At the air electrode, oxygen is reduced to form hydroxide ions.

(i) Write the half-equation for the reaction that occurred at the air electrode during
discharging. [1]

(ii) During discharging, the cell acts as a power source and the overall reaction
occurred is as follows:

3Fe + 2O 2 Fe 3 O 4

Suggest, with explanation, for the sign of the G for this reaction. [1]

(iii) The battery is capable of producing an e.m.f. of 1.28V. By using suitable data
from the Data Booklet suggest a value for the Eof the Fe 3 O 4 |Fe electrode
reaction.[1]

During charging, at the iron electrode, magnetite and water undergo the following
reaction:

Fe 3 O 4 + 4H 2 O + 8e 3Fe + 8OH

At the same electrode, a competing side-reaction occurs where water reacts to form

trendyline
y
hydrogenn gas according to the following reaction:
en

2H
2 2 O + 2e H 2 + 2OH

IJC 2016Prelim/9647/03[Turn over 498 trendyline


12

After a 1-hour charging, the volume of H 2 gas evolved, measured at r.t.p., is 0.652
dm3.

(iv) Calculate the amount of electrons that was required to produce H 2 gas. [1]

(v) A current of 9.5 A is supplied to the battery during the 1-hour charging.

Use this information and your answer in (c)(iv) to calculate the amount of iron
produced during the charging. [2]

(d) The separator used in the iron-air battery is an organic polymeric membrane, made
from compound X.

Compound X has a molecular formula C 3 H 3 N. On heating with dilute sulfuric acid, X


forms compound Y, C 3 H 4 O 2 . Both X and Ydecolourise aqueous bromine, however,
only Y reacts with sodium carbonate to form gas which forms white precipitate in
Ca(OH) 2 (aq).

Suggest structures for compounds X and Y, giving reasons for your answers. [4]

[Total: 20]

trendyline
IJC 2016Prelim/9647/03[Turn over 499 trendyline
TEACHERS COPY
INNOVA JUNIOR COLLEGE
JC 2PRELIMINARY EXAMINATION
in preparation for General Certificate of Education Advanced Level
Higher 2

CHEMISTRY 9647/03
Paper 3 Free Response 26 August 2016
2 hours
Candidates answer on separate paper.

Additional Materials: Writing Papers


Data Booklet
Cover Page

READ THESE INSTRUCTIONS FIRST

Write your name and class on all the work you hand in.
Write in dark blue or black pen on both sides of the paper.
You may use a soft pencil for any diagrams, graphs or rough working.
Do not use staples, paper clips, highlighters, glue or correction fluid.

Answer any four questions.


Begin each question on a fresh sheet of paper.

You are advised to show all working in calculations.


You are reminded of the need for good English and clear presentation in your answers.
You are reminded of the need for good handwriting.
Your final answers should be in 3 significant figures.

At the end of the examination, fasten all your work securely together.
The number of marks is given in the brackets [ ] at the end of each question or part question.

trendyline
y This document consists of 18printed pages.

Innova Junior College [Turn over


500 trendyline
2

Answer any four questions.

1 Data concerning the elements of group II of the Periodic Table, at 298K, are given in the
table. Further data may be found in the Data Booklet. M refers to elements of group II.
st nd 2+
1 and 2 IE of M + atom of M + hydration of M E /V
-1 -1 -1
/ kJ mol / kJ mol / kJ mol
Ca 1740 178 1650 2.87
Sr 1608 164 1480 2.89

(a) The electrode reduction potential E of Group II elements gives an indication of the
ease with which the following reaction occurs:

M(s) M2+(aq) + 2e reaction 1

The enthalpy change of reaction 1 can be related to the following factors:


first and second ionisation energy of the element
enthalpy change of atomisation and
enthalpy change of hydration of its gaseous ion.

(i) Construct an energy cycle relating the factors above, and use the values given
above to calculate the enthalpy change of reaction 1 for calcium. [3]

Reaction 1
2+
Ca(s) Ca (aq) + 2e

Enthalpy change of atomisation

Ca(g) 2+
Hhydration(Ca )
st
1 ionisation energy
+
-
Ca (g) + e
nd
2 ionisation energy
2+ -
Ca (g) + 2e
Cycle: all correct compounds with state symbols [2]
(deduct 1m for omission of state symbols,
Enthalpy change of reaction 1 = 178 + 1740 + (-1650) = +268 kJ mol-1 [1]

(ii) By quoting appropriate data from Data Booklet, explain the difference in
enthalpy change of hydration of calcium and strontium ions. [2]
Ca2+(0.099nm) has smaller ionic radius than Sr2+(0.113nm) -[1]

trendyline
ndyline
li
Since
nce they have the same charge, arge density of Ca2+ is larger,
charg the charge
2+
Thus Ca forms strongertronger ion
ion-dipole
dipole interactions with water molecules
m [1]
+
+ hydration
ra of Ca2+ is
s more exothermic than that of Sr2+.

PRELIMINARY EXAM INNOVA9647/03/2016 501


[Turn over trendyline
3

(c) Calcium oxide is the key ingredient for the process of making cement.

When calcium is burned in oxygen, calcium oxide is produced.


When 1.50g calcium is burned in air, a mixture of ionic solids calcium oxide and red
brown solid is formed.

The red brown solid has the following composition by mass: Ca, 81.1%; N,18.9%.

Adding water to the red brown solid produces calcium hydroxide and 19.2 cm3 of
ammonia gas at room temperature and pressure.

(i) Calculate the formula of the red brown solid. [1]

Ca N
Mass in 100g 81.1 18.9
Amt in 100g 2.0224 1.35
Simplest ratio 1.50 1
ratio 3 2

Empirical formula of solid: Ca 3 N 2

(ii) Write an equation for the reaction between red brown solid with water. [1]
Ca 3 N 2 + 6H 2 O 2NH 3 + 3Ca(OH) 2

(iii) Write equations for the reactions of calcium with air and use them to calculate
the mass of CaO formed when 1.5g of calcium is burnt in air. [3]
Ca + O 2 CaO
3Ca + N 2 Ca 3 N 2
2equations correct : [1]

No. of moles of ammonia produced = 19.2/24000 = 0.0008


No. of moles of Ca 3 N 2 produced = 0.0008/2 = 0.0004
No. of moles of calcium used to react with nitrogen= 0.0004 x 3 = 0.0012
mass of CaO formed = (1.5/40.1 0.0012)(40.1+16) = 2.03g

(d) Calcium oxide is normally made by heating calcium carbonate to a temperature above
825oC. A gas which turns lime water chalky is produced in the process.

(i) Write an equation for the decomposition of CaCO 3 .


[1]
CaCO 3 (s) CaO(s) + CO 2 (g)

(ii) Explain why barium carbonate decomposes at a higher temperature than


calcium carbonate. [2]
Ba2+ has larger cation size than Ca2+,thus has lower charge density. [1]
The polarising power of Ba2+ is lower, thus less able to polarize electron
cloud of carbonate anion, weakening covalent bonds (CO) in the anion to a
smaller extent [1]
More energy is needed to break down the anion and hence decomposition

ttrendyline
d
temperature for BaCO 3 is higher
higher.
g .

PRELIMINARY EXAM INNOVA9647/03/2016 502


[Turn over trendyline
4

(e) Methylcyclohexene reacts with hydrogen bromide to form alkyl bromides.

+%U &+%U

PHWK\OF\FORKH[HQH

(i) The above reaction could produce two isomers with molecular formula C 7 H 13 Br.
However, when the reaction is carried out, mainly one product is formed. Give
the structural formula of the major product, explaining your answer.
[2]

+
%U

[1]

Presence of an additional (more) electron donating alkyl group directly


+

bonded to the positively charged carbon in disperse the positive


+

charge to a greater extent thus is more stable, leading to the


formation of the major product. [1]

(ii) Describe the mechanism of the reaction between hydrogen bromide and
methylcyclohexene, C 6 H 9 CH 3 .
[3]

Electrophilic addition [1]


CH3 CH3
slow H
+ H Br + Br

CH3
CH3
Br
H H
+ Br f ast

2 marks for mechanism


(penalise 1 mark for 1-2 errors, penalise 2 marks for 3 errors or more)
slow step and balanced equation
arrows and lone pair

trendyline
polarity
correct intermediate

Unsymmetrical
metrical dihalides reacts with alkene in a similar way as hydro
hydrogen halides.

(iii) Predict the structure


di t th t t off th
the product
d t when IB reacts
h IBr t with
ith methylcyclohexene,
th and
explain your answer by considering the polarity within the molecule IBr.[2]

PRELIMINARY EXAM INNOVA9647/03/2016 503


[Turn over trendyline
5

Understanding that iodine is less EN than bromine:


Iodine is less electronegative (or electron-deficient or electrophile) than bromine
[]

Thus, iodine will be added across the C=C first. []


(or Brwill attack the carbocation formed)

Thus, product is
I
Br

product [1]

[Total: 20]

trendyline
PRELIMINARY EXAM INNOVA9647/03/2016 504
[Turn over trendyline
6

2 Alcohols and carboxylic acids have many scientific, medical and industrial uses worldwide.

(a) Methanol is made from the following gaseous reaction.


CO(g) + 2H 2 (g) CH 3 OH(g)

A 1:2 mixture of CO and H 2 was pumped into a sealed reactor at 6 atm and then
heated to 227 oC and maintained at this temperature for some time. When equilibrium
was reached, the partial pressure of CH 3 OH was found to be 1.6 atm.

(i) Write an expression for K p for the reaction.Hence, determine the value of
K p stating its units.
[3]
PCH3OH
Kp = [1]
PCOPH2 2
CO(g) + 2H 2 (g) CH 3 OH(g)
Initial / atm 2 4 0
Change / atm 1.6 3.2 +1.6
Equilibrium / atm 0.4 0.8 1.6
Correct equilibrium pressure for each reactant and product [1]

1.6
Kp= = 6.25 atm2[1] (correct units and values)
0.4 0.82
(ii) Suggest the effect on the position of equilibrium and value of K p when methanol
is removed from the reaction vessel. Explain your answer. [2]
When methanol is removed from the equilibrium, the partial pressure of
methanol decreases>@. By Le Chateliers Principle, the equilibrium position
would shift to the right to increase the partial pressure of methanol>@.

The value of K p would remain the same[1]as there is no change in


temperature. (K p is temperature dependent)

(b) Grignard reagent, RMgX, is an important reagent in organic synthesis. Grignard


reagents are prepared by the addition of activated magnesium on an alkyl halide or
aryl halide in non-polar solvents such as ether in anhydrous conditions.

- +
R CH2 X + Mg R CH2 MgX X = Cl or Br

An example of a Grignard reagent reacting with a ketone to form an alcohol via a two
stage process is given below:

trendyline
PRELIMINARY EXAM INNOVA9647/03/2016 505
[Turn over trendyline
7

R' R

Stage 1 : R CH2 MgCl + C O R CH2 C O MgCl

R'' R''

R' R'
H2O
Stage 2 : R CH2 C O MgCl R CH2 C OH + Mg(OH)Cl

R'' R''

R' and R'' could be alkyl / aryl group / H atoms

(i) Suggest the type of reaction occurringin Stages1 and 2.


[2]
Stage 1: Nucleophilic Addition[1]
Stage 2: Hydrolysis [1]

(ii) Suggest the structural formula of the final organic product formed when

CH2MgBr

is reacted with propanone, CH 3 COCH 3 , in a similar two-


step process. [1]
H OH

C C CH3

H CH3
[1]

(iii) Deduce the structures of a suitable Grignard reagent, RMgBr, and a suitable
carbonyl compound, RRCO, to synthesize the following alcohol.
(CH 3 ) 2 CHCH 2 CH(OH)CH 2 CH 2 CH 3 [2]
(CH 3 ) 2 CHCH 2 CHO [1] and CH 3 CH 2 CH 2 MgBr[1]

OR

CH 3 CH 2 CH 2 CHO[1] and(CH 3 ) 2 CHCH 2 MgBr [1]

(iv) The Grignard reagent CH 3 CH 2 CH 2 MgBr can be readily converted into a


carboxylic acid by using carbon dioxide.

Suggest the structural formula for the organic product formed. [1]
CH 3 CH 2 CH 2 COOH[1]

trendyline
yyline
(v) p
The product formed byy using g Grignard
g g
reagent ( ) ((iii)does
in (b) ) not rotate the
ane polarised light. Explain your reasoning.
plane [2]

As the Grignard reagents can n attack the sp2 hybridised carbonyl carbon (trigonal
planar shape)>@
shap >@
shape) @ either from the top or the bottom of the plane p with equal
chance>@,
e>@, it results in a racemic mixture
mixture>@
ure>@ of products being formed.
for The optical
activity of the two isomers cancel out each other>@and the resulting mixture does
not the rotate plane polarised light.

PRELIMINARY EXAM INNOVA9647/03/2016 506


[Turn over trendyline
8

(c) Describe and explain the relative acidities of phenol, benzoic acid and
phenylmethanol.
[3]
Acid strength: benzoic acid > phenol >phenylmethanol[1]

In benzoate ion, C 6 H 5 COO , the negative charge on oxygen is delocalised over


the two highly electronegative oxygen atoms.>@The C 6 H 5 COO anion is most
stable among the 3 anions and hence benzoic acid is the most acidic.

In the phenoxide ion of phenol, C 6 H 5 O-, the p-orbital of O overlaps with the -
electron cloud of the benzene ring so that the negative charge on O delocalises
into the benzene ring. >@This stabilization is not as great as that in the
C 6 H 5 COOion in which the negative charge is delocalized over 2 highly
electronegative O atoms. The phenoxide ion is less stable than benzoate ion and
hence phenol is less acidic than benzoic acid.

In the phenylmethoxide, C 6 H 5 CH 2 O-, the electron-donating alkyl/benzyl group


intensifies the negative charge on O atom.>@The C 6 H 5 CH 2 O- ion is, therefore, the
least stable and phenylmethanol is least acidic.
>@ comparing the stability of the 3 anions
Marking along the line of giving credit for the EDG/EWG/delocalisation +
stability of anions

(d) Lactic acid, glycolic acid and salicylic acid are three organic acids commonly used in
chemical peel.

CH3CHCO2H CH2CO2H OH

OH OH
COOH
lactic acid glycolic acid salicylic acid

Suggest a simple chemical test that can be used to distinguish each of the following
pairs.State what you would observe for each chemical test.

1) lactic acid from glycolic acid, and


2) glycolic acid from salicylic acid.
[4]
1) reagents and conditions: I 2 , NaOH (aq), warm[1]

ttrendyline
nd
dyylliin
dy
Yellow
llow
lo ppt of CHI 3 formed for lactic acid but not glycolic ac
acid[1]

OR
agents
gents and conditions:
reagents conditio KMn 4 (aq), H 2 SO
KMnO O 4 (aq), heat
heat[1]
[1]
1]
rple
ple KMnO 4 decolourises for
Purple fo lactic
actic acid and an glycolic acid.
a For glycolic
acid,
id effervescence of colourless s gas
g which formed white ppt with calcium
hydroxide was observed but not for lactic acid.[1]

PRELIMINARY EXAM INNOVA9647/03/2016 507


[Turn over trendyline
9

2) reagents and conditions: neutral FeCl 3 (aq)[1]


Violet complex formed for salicylic acid but not for glycolic acid.[1]
OR
reagents and conditions: Br 2 (aq)[1]
Observation with: Reddish brown Br 2 (aq) decolourised for salicylic acid but
not for glycolic acid.[1]

*NaOH cannot be used to distinguish these 2 compounds as there is no


observable change in both compounds although phenol reacts with NaOH.
[Total: 20]

trendyline
PRELIMINARY EXAM INNOVA9647/03/2016 508
[Turn over trendyline
10

3 Aluminium, is the third most abundant element (after oxygen and silicon) found in the
Earths crust. Aluminium reacts readily with carbon to form aluminium carbide, Al 4 C 3 .

(a) Aluminium carbide can react with protic reagents such as hydrochloric acid to produce
methane gas and aluminium(III) chloride.

(i) Write a balanced equation for the reaction between aluminium carbide and
hydrochloric acid. [1]
Al 4 C 3 (s) + 12HCl (aq) 4AlCl 3 (aq) + 3CH 4 (g) [1] ss not required

(ii) Using the equation in (a)(i), calculate the maximum volume of methane gas that
can be produced when 12.0 g of aluminium carbide reacts with 250 cm3 of
2.0 mol dm-3 hydrochloric acid at standard temperature and pressure.[3]

12.0
Amt of Al 4 C 3 = 0.08333 mol
4(27.0) 3(12.0)

Amt of HCl = 0.250 x 2.0 = 0.5 mol [1]


If Al 4 C 3 is limiting, amt of HCl required = 12 x 0.08333= 1.000 mol
So Al 4 C 3 is in excess. HCl is limiting reagent [1]
0.5
Amt of CH 4 produced = 3 0.125 mol
12
Volume of CH 4 obtained = 0.125 x 22.4= 2.80 dm3 [1]

(b) Aluminium chloride is used as a catalyst in electrophilic substitution reactions. The


chlorination of benzene is represented by the following overall equation.

Cl
AlCl3
+ Cl + HCl
2

The reaction occurs in several steps.

The first step is the reaction between Cl 2 and AlCl 3 .

Cl 2 + AlCl 3 Cl + AlCl 4

The benzene ring is then attacked by the Cl cation in the second step.

AlCl 3 reacts in a similar way with halogenoalkanes and acyl chlorides, producing a
carbocation that can then attack a benzene ring.

Using the reactions described above as parts of a synthesis route, a student


suggested the following route to form phenyl 4-phenylbutanoate, from benzene.

trendyline
PRELIMINARY EXAM INNOVA9647/03/2016 509
[Turn over trendyline
11
I
Cl Cl OH
with AlCl3

step 1 step 2

A
step 3

O OH

O Route B O
-
phenyl 4 phenylbutanoate 2 steps

(i) Explain why step 1 has to be carried out in an anhydrous condition. [1]
AlCl 3 undergoes hydrolysis when reacted with water. [1]
(Accept hydration)

(ii) Draw the carbocation formed in step 1. [1]

Cl

(iii) A student suggested that Acould have the following structure:

By quoting appropriate data in the Data Booklet, explain why the structure
suggested by the student is less likely to be formed. [2]

E(CI) = + 240 kJ mol1E(CCl) = + 340 kJ mol1 [1] for quoting data


Since E(C-I) is smaller than E(CCl) and thus CI bond is weaker than the C
Cl bond, [1] it is easier to break CI bond. Hence, F should contain the Cl atom,
rather than I atom.
(iv)
Give appropriate reagents and conditions necessary for steps 2 and 3. [2]

Step 2: KOH(aq) or NaOH(aq), heat [1]

Step 3: K 2 Cr 2 O 7 , dilute H 2 SO 4 , heat [1]

(v)

t d li
trendyline
Suggest
g the reagents
g and conditions for each of the two steps in route B. [2]
Route
oute B step 1:
1 PCl 5 (s)
((s),
), room
m temp OR PCl
PC 3 , room ttemp
tempOR
OR SOCl
S 2 , room temp
Route
oute B step 2:
2: NaOH(aq), phenol or phenoxide ion

PRELIMINARY EXAM INNOVA9647/03/2016 510


[Turn over trendyline
12

(vi)
Predict the structure of the product of each of the following reactions.
Both compounds E andFreact with 2,4-dinitrophenylhydrazine. [2]

AlCl3 E
Cl C9H8O
O
C
O
AlCl3 F
Cl C9H8O

E O
F
[1] for each structure

(vii) Describe and explain the relative ease of hydrolysis of the two chloro-
compoundsC and D. [2]
Compound D undergoes hydrolysis more readily than compound C. [1]

The C atom of the acyl groupin compound D is attached to two


electronegative atoms, Cl and O. Hence this carbon atom is more electron
deficient>@and thus more susceptible to nucleophilic attack >@FRPSDUHGWR
the carbon atom of C-Cl in compound C.

(c) By considering the structure and bonding, explain why


(i) the boiling point of aluminium fluoride is higher than that of aluminium chloride.
[2]

AlCl 3 hassimple molecular structure while AlF 3 has giant ionic structure.>@

More energy>@ is required to overcome the stronger electrostatic forces of


attraction between oppositely charged ions in AlF 3 >@than the weak van der
waals forces of attraction between AlCl 3 molecules>@. This results in the

trendyline
en
en
higher
g boiling
g point
p in AlF 3 .

COOH
COOH COOH
H
OH
O

salicylic acid, , is less soluble in water than benzoic acid,


.
[2]
Both compounds are simple covalent molecules and can form hydrogen
PRELIMINARY EXAM INNOVA9647/03/2016 511
[Turn over trendyline
13

bonding with water molecules.

However, intramolecular hydrogen bonding present in salicylic acid due to


the close proximity of the carboxyl group and hydroxyl group. [1]

Thus, the intermolecular hydrogen bonding between salicylic acid and


water is less extensive than the hydrogen bonding between benzoic acid and
water. [1]

This results in the lower solubility of salicylic acid in water.

[Total: 20]

trendyline
PRELIMINARY EXAM INNOVA9647/03/2016 512
[Turn over trendyline
14

4 The Strecker synthesis is a term used for a series of chemical reactions that synthesize an
amino acid from an aldehyde or ketone. This reaction requires acid and HCN is supplied
from cyanide salts.

NH2
Stage 1 Stage 2 NH2
O
KCN H+ O
R R
R H NH4Cl N

OH

One example of the Strecker synthesis is a multikilogram scale synthesis of amino acids.

(a) Name the type of reaction occurring in Stage 2 and state the reagent and
conditions used in this reaction.[2]
Acidic hydrolysis[1]
Reagents and conditions: dilute sulfuric acid , heat [1]

(b) InStage 1, ammonia formed from the reaction between KCN and NH 4 Cl reacts
with the aldehyde.

Suggest an equation for the formation of ammonia from the reaction between
KCN and NH 4 Cl and describe the role of KCN in stage 1. [2]

NH 4 Cl + KCN NH 3 + HCN + KCl [1]


Accept if above reaction is irreversible.
KCN supplies CN- which acts as a proton acceptor or base.[1]OR CN- acts a
nucleophile [1]

(c) Explain why isobutyramide is less basic than 3-aminobutan-2-one.

NH2 O NH2
- - -
isobutyramide 3 aminobutan 2 one
[2]
.

Isobutyramide is an amide and is neutral as lone pair of electrons on N atom


is delocalised into the pi electron cloud[1] of the adjacent C=O group making
the lone pair on N atom unable to accept a proton[] In 3-aminobutan-2-
one, the lone pair on N atom is available to accept a proton[].

trendyline
PRELIMINARY EXAM INNOVA9647/03/2016 513
[Turn over trendyline
15

(d) Predict the outcomes of the following reactions starting from isobutyramide, drawing
the structures of the intermediate J and the products Kand L.

O
LiAlH4 excess CH3Cl
J K
C7H18NCl
NH2
isobutyramide

NaOH(aq),
reflux

L + gas

[3]

Ans:

+ -
Cl
NH2 N

- +
O Na

1m each

(e) Compound P, C 9 H 12 O 2 , shows optical activity and does not react with aqueous
potassium carbonate. However, it is soluble in aqueous potassium hydroxide.
When P reacts with hot acidified potassium dichromate(VI), the solution turns from
orange to green. P also decolourises aqueous bromine to form a white precipitate, Q,
C 9 H 10 O 2 Br 2 .

Upon heating with concentrated sulfuric acid, P gives only compound R, C 9 H 10 O,

trendyline
isplays geometric isomerism. R gives
which displays s compound
com S and ethanoic acid on
g with hot acidified potassium manganate(VII) solution.
reacting solutio
Suggestt the structuresof
structureso PP,, Q
Q,, R and
a S S,, explaining the reactions involved.
invo
[7]

PRELIMINARY EXAM INNOVA9647/03/2016 514


[Turn over trendyline
16

OH

OH OH

C CH2CH3

C H
H3CH2C H
P: OH OR

OH

Br Br OH OH

Br C CH2CH3

C
H3CH2C H
Q: OH OR Br

OH

OH

H
C CHCH3

C
H CHCH3
R: OR

OH

OH

ttrendyline
r dyyl
COOH
COOH
CO

C
S: HO O OR

1m for each structure

PRELIMINARY EXAM INNOVA9647/03/2016 515


[Turn over trendyline
17

P shows optical activity P has chiral carbon>@

P does not react with aqueous No -COOH


potassium carbonate.

However, it is soluble in aqueous Undergoes acid-base reaction.


potassium hydroxide. Contains phenol >@

P reacts with hot acidified potassium P undergoes oxidation. >@


dichromate(VI), the solution turns from
orange to green Contains primary or secondary
alcohol >@

P also decolourises aqueous bromine to Pundergoes electrophilic


form a white precipitate, Q, substitution>@
C 9 H 10 O 2 Br 2 .
PcontainphenolJURXS>@

Br atoms added to 2 or 4 positions on


ring

Upon heating with concentrated sulfuric P undergoes elimination of H 2 O>@


acid, P gives only compound R,
C 9 H 10 O, which displays geometric R is an alkeneFRQWDLQV& &>@
isomerism.
R gives compound S and ethanoic acid R undergoes oxidative cleavage>@
on reacting with hot acidified potassium
manganate(VII) solution.

9 [] 3m
6-8 [] 2m
3-5 [] 1m
0-2 [] 0m

(f) The structure of squaric acid is shown below.

O OH

O OH

(i) State the functional groups in squaric acid.[2]

Ketone, alkene and alcohol


2m for all 3 functional groups ; 1m for 2 functional groups

trendyline
y
(ii) quaric
uaric acid is a stable molecule although it has high angle strain associated
Squaric
ith a four
with four--membered ring. It is a strong diprotic acid with a p
four-membered pK a values 1.5 and
3.44 respectively. Explain the high acidity
acidity of squaric acid. [2]
The
he e high acidity of the squaric acid is due to the formation of a very stable
anion[1]due
nion[1]due to negative charges rges being delocalised across the sp2
hybridised carbon[1].

[Total: 20]
PRELIMINARY EXAM INNOVA9647/03/2016 516
[Turn over trendyline
18

trendyline
PRELIMINARY EXAM INNOVA9647/03/2016 517
[Turn over trendyline
19

5 Iron is, by mass, the most common element on Earth, forming much of Earth's outer and
inner core. Iron and its compounds have been used as catalysts in many reactions. For
example, aqueous iron(II) chloride can be used to catalyse the reaction between between I
and S 2 O 8 2, to form I 2 and SO 4 2ions.

(a) (i) Write an equation to represent the third ionisation energy of Fe. [1]
Fe2+(g) Fe3+(g) + e [1, state symbols must be included]

(ii) Write the electronic configuration for Fe2+. Hence, suggest why the third
ionisation energy of 26 Fe is lower than that of 25 Mn.
[2]
Fe2+: 1s2 2s2 2p6 3s2 3p6 3d6 [1]
The lower 3rd IE of Fe is due to inter-electronic repulsion>@by the paired
electrons in the d orbital>@.

(iii) State the type of catalysis for the reaction between I and S 2 O 8 2 that is
catalysed by aqueous iron(II) chloride. [1]
Homogenous catalysis[1]

(iv) The reaction between I and S 2 O 8 2 can also be catalysed by aqueous cobalt(II)
chloride. Using relevant E values from the Data Booklet, explain why cobalt(II)
chloride can be a catalyst for this reaction. [2]

S 2 O 8 2 + 2Co2+ 2Co3+ + SO 4 2

Ecell = +2.01 (+1.82)


= +0.19 V (reaction is energetically feasible) [1]

2 I + 2Co3+ 2Co2+ + I 2
Ecell = +1.82 (+0.54)
= +1.28 V (reaction is energetically feasible)[1]

(b) A green solution of aqueous iron(II) chloride is reacted with acidified potassium
manganate(VII). The resultant solution D is treated with aqueous sodium carbonate to
form a reddish brown precipitate E, together with a colourless gas which forms white
precipitate in aqueous calcium hydroxide. When aqueous NH 4 SCN is added to D, a
blood red solution is formed.

(i) Explain why aqueous iron(II) chloride is coloured. [2]


FeCl 2 (aq) is coloured due to:
presence of incompletely filled 3d orbitalsin the Fe2+ ions.
In the presence of ligands, 3d orbitals split into two groups with small
energy gap >@
Some of the light energy is used to promote an electron from a 3d
orbital of lower energy into the partially filled 3d orbital of higher
energy.[1]
Colour seen is the complement of the colour absorbed in thevisible

t
regionof the spectrum
rregionof

(ii) entify precipitate E


Identify E.. [1]]
on(III) hydroxide [1]]
iron(III)
ron(III)

(iii) Suggest the type of reaction that has occurred for the formation of the blood red
solution. [1]

PRELIMINARY EXAM INNOVA9647/03/2016 518


[Turn over trendyline
20

Ligand exchange[1]

PRELIMINARY EXAM INNOVA9647/03/2016 519


[Turn over trendyline
21

(c) Iron is also used in making iron-air battery which is a low-cost and environmental
friendly rechargeable energy source. An iron-air battery comprises of iron electrode,
porous carbon-containing air electrode. The electrolyte used is aqueous potassium
hydroxide.
external circuit
e e

Fe3O4 OH O2

O2
Fe O2
OH
O2

KOH(aq) O2
iron electrode air electrode
separator

At the iron electrode, iron is oxidised to magnetite, Fe 3 O 4 , during discharging,


according to the following reaction:

3Fe + 8OH Fe 3 O 4 + 4H 2 O + 8e

At the air electrode, oxygen is reduced to form hydroxide ions.

(i) Write the half-equation for the reaction that occurred at the air electrode during
discharging. [1]
O 2 + 2H 2 O + 4e 4OH[1]

(ii) During discharging, the cell acts as a power source and the overall reaction
occurred is as follows:

3Fe + 2O 2 Fe 3 O 4

6XJJHVWZLWKH[SODQDWLRQIRUWKHVLJQRIWKH* for this reaction. [1]


Negative>@. The reaction that occurred during discharging should be a
spontaneous reaction>@.

(iii) The battery is capable of producing an e.m.f. of 1.28V. By using suitable data
from the Data Booklet suggest a value for the Eof the Fe 3 O 4 |Fe electrode
reaction.[1]
Ecell = Ered Eox
+1.28 = +0.40 E(Fe 3 O 4 |Fe)
E(Fe 3 O 4 |Fe) = 0.88 V[1]

During charging, at the iron electrode, magnetite and water undergo the following

trendyline
yli
reaction:
n:

Fe 3 O 4 + 4H 2 O + 8e Fe + 8OH
3Fe

At the same electrode, a competing side


side-reaction
e-re
reaction occurs where water
wa reacts to form
hydrogen gas according
di to the
h following
f ll i reaction:
i

2H 2 O + 2e H 2 + 2OH
PRELIMINARY EXAM INNOVA9647/03/2016 520
[Turn over trendyline
22

After a 1-hour charging, the volume of H 2 gas evolved, measured at r.t.p., is 0.652
dm3.

(iv) Calculate the amount of electrons that was required to produce H 2 gas.
[1]
Amount of H 2 evolved = 0.652/24 = 2.717 x 10-2 mol
Amount of electrons required = 5.434 x 10-2 mol[1]

(v) A current of 9.5 A is supplied to the battery during the 1-hour charging.

Use this information and your answer in (c)(iv) to calculate the amount of iron
produced during the charging. [2]
It = n e F
Ne = (9.5)(3600)/96500 = 0.3544 mol[1]

Amount of electrons reacted with Fe 3 O 4 = 0.3544 - 5.434 x 10-2 = 0.3001 mol

Amount of Fe produced = 0.3001 x 3/8 = 0.113 mol[1]

Ignore if student went on to calculate mass of Fe.


(d) The separator used in the iron-air battery is an organic polymeric membrane, made
from compound X.

Compound X has a molecular formula C 3 H 3 N. On heating with dilute sulfuric acid, X


forms compound Y, C 3 H 4 O 2 . Both X and Y decolourise aqueous bromine, however,
only Y reacts with sodium carbonate to form gas which forms white precipitate in
Ca(OH) 2 (aq).

Suggest structures for compounds X and Y, giving reasons for your answers. [4]

Evidence Deduction
Both X and Y decolourise Both X and Y undergoes electrophilic [1]
aqueous bromine addition.
X and Y contains alkene group.
Y reacts with sodium carbonate to Y undergoes acid-carbonate reaction [1]
form gas which forms white with sodium carbonate.
precipitate in Ca(OH) 2 (aq).
Y contains carboxylic acid group.
On heating with dilute sulfuric X undergoes acidic hydrolysis to form [1]
acid, X forms compound Y, Y.
C3H4O2. X contains nitrile group

[max 2m]

X: CH 2 =CHCN[1]Y: CH 2 =CHCOOH[1]

[Total: 20]

trendyline
PRELIMINARY EXAM INNOVA9647/03/2016 521
[Turn over trendyline
JURONG JUNIOR COLLEGE
2016 JC 2 PRELIMINARY EXAMINATION
Higher 2

CANDIDATE
NAME

EXAM INDEX
CLASS 16S
NUMBER

CHEMISTRY 9647/01
Paper 1 Multiple Choice 16 September 2016
1 hour

Candidates answer on separate paper.


Additional Materials: Multiple Choice Answer Sheet
Data Booklet

READ THESE INSTRUCTIONS FIRST

Write in soft pencil.


Do not use staples, paper clips, highlighters, glue or correction fluid.
Write your name, class and exam index number on the Answer Sheet in the spaces provided unless this has
been done for you.

There are forty questions on this paper. Answer all questions. For each question there are four possible
answers A, B, C or D.
Choose the one you consider correct and record your choice in soft pencil on the separate Answer Sheet.

Read the instructions on the Answer Sheet very carefully.

Each correct answer will score one mark. A mark will not be deducted for a wrong answer.
Any rough working should be done in this booklet.
The use of an approved scientific calculator is expected, where appropriate.

ttrendyline
d li e
ocument
cu
This document consists of 15 printed pages and 1 blank page.

[Turn over

522 trendyline
2

Section A
For each question there are four possible answers, A, B, C, and D. Choose the one you consider
to be correct.

1 Use of the Data Booklet is relevant to this question.

Which particle would, on losing an electron, have a configuration that corresponds to the
ground state of an element in Group V?

A Cl+ B Si C N D C+

2 In the following, the species in each pair contain the same number of electrons and the same
number of neutrons.

Identify the pair of species that does not fit the above description.

A HF F
B H2O OH
C NH 3 NH 4 +
D CH 4 CH 3 +

3 Four substances have the physical properties shown.

Which substance could be aluminium chloride?

electrical electrical
electrical
melting point conductivity conductivity
conductivity
/ oC of molten of aqueous
of solid
substance solution
A 98 poor poor poor
B 190 poor poor good
C 660 good good insoluble
D 800 poor good good

4 In which of the following pairs does the second compound have a higher boiling point than
the first?

A NaF NaI
B CaO Na 2 O
C CH 3 OH CH 3 OK
D CH 3 CH 2 OH CH 3 CH 2 SH

Jurong Junior College

trendyline
e
9647/01/J2
9647/0
647 1/J2
J2 PRELIMINARY EXAMINATION/2016
EXAMINATION
AMINAT /201
016

523 trendyline
3

5 The interhalogen compound BrF 3 is a volatile liquid which autoionises.

2BrF 3 BrF 2 + + BrF 4 H

The electrical conductivity of BrF 3 decreases with increasing temperature.

Which combination correctly represents the sign of H and the shape around bromine
in BrF 4 ?

H BrF 4
A + tetrahedral
B tetrahedral
C square planar
D + square planar

6 A quantity of ethanol was burned underneath a copper can containing 400 g of water at 30
o
C. The temperature of the water rose to 85 oC after the complete combustion of 5 g of
ethanol (M r = 46).
The efficiency of heat transfer to the water will not be 100% after taking into considerations
the heat capacity of the copper can.

Given that the specific heat capacity of water is 4.2 J g1 K1 and the enthalpy change of
combustion of ethanol is 1367 kJ mol1, what is the efficiency of heat transfer to the water?

A 7.4% B 62% C 63% D 96%

7 Clouds and rain form when air saturated with water vapour cools.

H 2 O(g) H 2 O(l)

What are the correct signs of H and S for this reaction?

H S
A
B + +
C +
D +

Jurong Junior College

trendyline
e
9647/01/J2
9647/0
647 1/J2
J2 PRELIMINARY EXAMINATION
EXAMINATION/2016
EXAMINAT /201
016 [Turn over

524 trendyline
4

8 Hydrazine, N 2 H 4 , is used as a rocket fuel because it reacts with oxygen as shown, producing
environmentally friendly gases.

N 2 H 4 (l) + O 2 J 1 2 (g) + 2H 2 O(g) H N-PRO1

Despite its use as a rocket fuel, hydrazine does not burn readily in oxygen.

Which statement explains why hydrazine does not burn readily?

A The activation energy is too high.


B Hydrazine is a liquid.
C The NN bond is very strong.
D G of the reaction is negative.

9 Use of the Data Booklet is relevant to this question.

During electrolysis under suitable conditions, 1.18 g of chromium is deposited on the cathode
when 4370 C of electricity is passed into a chromium-containing electrolyte.

Which of the following could have been the electrolyte?

A CrCl 2
B CrCl 3
C K 2 CrO 3
D K 2 Cr 2 O 7

10 A sample of hydrogen iodide gas is maintained at a constant temperature of 600 K.


A dynamic equilibrium is established with hydrogen, iodine and hydrogen iodide all present.

Which statement describes this situation?

A The amount of hydrogen iodide present is constantly changing at equilibrium.


B Half of the original amount of hydrogen iodide is present at equilibrium.
C At equilibrium, hydrogen iodide is being formed at the same rate as it decomposes.
D The amount of hydrogen iodide present at equilibrium is affected by the pressure.

Jurong Junior College

trendyline
e
9647/01/J2
9647/0
647 1/J2
J2 PRELIMINARY EXAMINATION/2016
EXAMINATION
AMINAT /201
016

525 trendyline
5

11 Metal sulfides are generally insoluble and some relevant K sp values are given in the table.

Salt Colour K sp / mol2dm6


CuS black 6.3 1036
ZnS white 1.6 1024

When aqueous copper sulfate is added to white solid of ZnS, black precipitate of CuS
is observed according to the following reaction.

ZnS(s) + Cu2+(aq) CuS(s) + Zn2+(aq)

What is the equilibrium constant for the reaction above?

A 1.0 1059
B 3.9 1012
C 5.0 105
D 2.5 1011

12 A solution was made by mixing 0.040 mol of lactic acid and 0.030 mol of sodium hydroxide.
Water was added until the volume of the solution was 2 dm3.

What is the pH of the solution?


[Lactic acid is a monobasic acid with K a = 8.4 104 mol dm3 ]

A 2.6 B 2.7 C 3.0 D 3.6

13 A piece of magnesium ribbon was added to 25 cm3 of dilute hydrochloric acid. The
magnesium was completely dissolved and the total volume of hydrogen gas evolved was
measured.

In a second experiment an identical piece of magnesium ribbon was used. This was added to
another 50 cm3 of the same dilute hydrochloric acid. The total volume of hydrogen evolved
was measured.

How will the initial rate of reaction and total volume of hydrogen evolved in the second
experiment compare to that of the first experiment?

initial rate of total volume of


reaction hydrogen evolved
A increase increase
B increase no change
C no change increase
D no change no change

Jurong Junior College

trendyline
e
9647/01/J2
9647/0
647 1/J2
J2 PRELIMINARY EXAMINATION
EXAMINATION/2016
EXAMINAT /201
016 [Turn over

526 trendyline
6
14 In acid solution BrO 3 ions will slowly oxidise Br ions to Br 2 .

BrO 3 + 5Br + 6H+ 3Br 2 + 3H 2 O

The variation for the initial rate of reaction with the concentrations of the reactants was
investigated in two experiments.

experiment initial [BrO 3 ] initial [Br ] initial [H+] initial rate


/mol dm3 /mol dm3 /mol dm3 /mol dm3 min1
1 0.10 0.15 0.08 3.5
2 0.20 0.30 y 3.5

The rate equation for this reaction is

Rate = k [BrO 3 ] [Br ] [H+] m

where k is the rate constant with units of mol3 dm9 min1.

What is the value of y in experiment 2?

A 0.02 B 0.04 C 0.16 D 0.32

15 Covalent chlorides like SiCl 4 and PCl 3 undergo hydrolysis to give an acidic solution. Formatted: Not Highlight
Formatted: Not Highlight
Which of the following chlorides is not expected to undergo hydrolysis?

A CCl 4 B GeCl 4 C AsCl 3 D SbCl 3 Formatted: Not Highlight


Formatted: Not Highlight

16 Which of the following is not a result of the high charge density of Al3+?

A AlCl 3 has a tendency to form Al 2 Cl 6 . Formatted: Not Highlight


Formatted: Not Highlight
B A solution of AlCl 3 has a lower pH than MgCl 2 .
Formatted: Not Highlight
C Al 2 O 3 is an amphoteric oxide. Formatted: Not Highlight
D Al(OH) 3 has a lower decomposition temperature than Ca(OH) 2 . Formatted: Not Highlight
Formatted: Not Highlight
Formatted: Not Highlight

17 Lime mortar is made from quicklime, water and sand. Over a period of time, lime mortar
changes into a much harder form. Both fresh and old lime mortar react with aqueous
hydrochloric acid but only the old lime mortar effervesces during the reaction.

Which equation describes the change from fresh to old lime mortar?

A CaO + CO 2 CaCO 3
B CaO + H 2 O Ca(OH) 2 Formatted: Not Highlight

trendyline
e
C Ca(OH) 2 CaO + H 2 O
D Ca(OH) 2 + CO
O 2 CaCO 3 + H 2 O

Jurong Junior College 9647/01/J2


9647/0
647 1/J2
J2 PRELIMINARY EXAMINATION/2016
EXAMINATION
AMINAT /201
016

527 trendyline
7
18 A sodium halide evolved thick white fumes when reacted with phosphoric acid. The white
fumes was then passed through AgNO 3 (aq) which gives a precipitate insoluble in
concentrated NH 3 .

What is the role of phosphoric acid in the reaction and the identity of the sodium halide?

Role of phosphoric acid Identity of sodium halide


A Acid Sodium bromide
B Acid Sodium iodide Formatted: Not Highlight

C Oxidising agent Sodium bromide


D Oxidising agent Sodium iodide

191 Which of the following is not a result of the small energy gap between 3d and 4s orbitals? Formatted: Not Highlight

A Transition metals have higher electrical conductivity than aluminium.


B Transition metals can exhibit variable oxidation states.
C Transition metals have higher melting point than calcium.
D Transition metals ions are usually coloured. Formatted: Not Highlight

20 Which compound cannot be readily prepared from 4-nitromethylbenzene?

A CH3 B CCl3 C COOH D CH3 Formatted: Not Highlight

Br

NH2 NO2 NO2 NO2

Jurong Junior College

trendyline
e
9647/01/J2
9647/0
647 1/J2
J2 PRELIMINARY EXAMINATION
EXAMINATION/2016
EXAMINAT /201
016 [Turn over

528 trendyline
8

21 Benzaldehye is the simplest aromatic aldehyde and one of the most industrially useful.

CHO

Benzaldehyde

Which of the following is true about the electrophilic reaction of benzaldehye?

A AlCl 3 (aq) can be used as a catalyst in its reaction with Cl 2 .


B It reacts with concentrated HNO 3 in the presence of concentrated H 2 SO 4 at 30 oC.
C A halogen carrier, FeCl 3 , can be used as a catalyst in its reaction with Br 2 . Formatted: Not Highlight
Formatted: Not Highlight
D It is more reactive towards Br 2 than alkene as it has delocalised electrons and is more
electron-rich.

22 Which of the following compounds, in its reaction with NaCN, is the least likely to form a
carbocation as an intermediate?

A C(CH 3 ) 3 Br
B CH 3 CHBrCH 2 CH 3
C CH 3 CH 2 CH 2 CH 2 Br Formatted: Not Highlight
Formatted: Not Highlight
D
Formatted: Not Highlight
CH2Br
Formatted: Not Highlight
Formatted: Not Highlight
Formatted: Not Highlight
Formatted: Not Highlight
Formatted: Not Highlight
23 1-bromopropane is converted to 2-bromopropane by a two-step process involving the
Formatted: Not Highlight
intermediate T.

step 1 step 2
CH 3 CH 2 CH 2 Br T CH 3 CHBrCH 3

What could be the reagent for step 1 and for step 2 to get the highest yield of
2-bromopropane?

step 1 step 2
A Concentrated H 2 SO 4 , heat SOBr 2 , heat
B NaOH(aq), heat HBr(aq)
C NaOH in ethanol, heat HBr(g) Formatted: Not Highlight

D NaCN in ethanol, heat PBr 3 , heat

Jurong Junior College

trendyline
e
9647/01/J2
9647/0
647 1/J2
J2 PRELIMINARY EXAMINATION/2016
EXAMINATION
AMINAT /201
016

529 trendyline
9
24 A solid compound Z dissolved readily in water to give a weakly alkaline solution.
On evaporation of the water, Z was recovered unchanged.

What could Z be?

A CH 3 NH 3 +Cl B CCl 3 CO 2 Na+ C C 6 H 5 O Na+ D H 2 NCH 2 CO 2 H Formatted: Not Highlight


Formatted: Not Highlight

I think I made a typo, was thinking of friedal crafts and testing them on the directing effect
of CH 3 and NO 2 .
252 N-Methyl-D-aspartic acid is a neuroprotective agent which is used in the treatment of
strokes, trauma and epilepsy. Its structure is shown below.

CO2H

CH2

CH3NHCHCO2H

Which of the following statements is correct?

A r solutionIt exists as an anion at pH 7.exists as an anion at pH 7. Formatted: Not Highlight


Formatted: Strikethrough, Not
B It reacts with phenol to form an ester. Highlight
C It cannot form intramolecular hydrogen bonding. Formatted: Not Highlight

D One mole of N-Methyl-D-aspartic acid reacts with Na metal to give two moles of H 2
gas.

26 Which of the following reagents is able to convert compound Q to compound R?

CH3CHBrCH(OH)COOH CH3CHBrCH(Br)COOH
Compound Q Compound R

A PBr 3 B HBr C SOBr 2 D NaBr Formatted: Not Highlight


Formatted: Not Highlight

Qn 7 was the original qn 2, I changed the option (a) so that it will not overlap with the pka
question. But I feel that the 2 assessment objectives are rather different. The original option
of it exists as an anion at pH 7 is to let them recognise the low pka values of the COOH and
high pKa value of NH 3 + (this was asked in N2014 P3). While the pka values require them to
assign pKa value to get the correct species at a certain pH.

Jurong Junior College

trendyline
e
9647/01/J2
9647/0
647 1/J2
J2 PRELIMINARY EXAMINATION
EXAMINATION/2016
EXAMINAT /201
016 [Turn over

530 trendyline
10

This question students have to think of S N 1 mechanism, so they would narrow it down to
option C and D. as there is only one answer, they have to compare the stability of the
carbocation and eliminate D

27 Which reagent will convert compound G to compound E efficiently?

O CN HO CN Formatted: Centered

Cl Cl
O O O O
Compound G Compound E

A H 2 /Ni B H 2 O/H+ C LiAlH 4 D NaBH 4 Formatted: Not Highlight


Formatted: Not Highlight
3 Amides and esters both undergo hydrolysis, however esters are more reactive towards
hydrolysis compared to amides. Formatted: Not Highlight

When comparing CH 3 COOCH 3 and CH 3 CONHCH 3 , which factor best contributes to this
difference in ease of hydrolysis?
A A lone pair of electrons on the nitrogen in NHCH 3 , is more readily delocalised into the
>C=O group than is the corresponding pair on the oxygen, OCH 3
B The nitrogen atom in CH 3 CONHCH 3 is less electronegative than the OCH 3 group in
CH 3 CO 2 CH 3.
C There is greater positive charge on the carbon atom in the >C=O group in
CH 3 CONHCH 3 .
D CH 3 COOCH 3 has a lower energy content than CH 3 CONHCH 3 .

28 A student added aqueous silver nitrate into test tubes containing compounds W, X, Y and Z.
The following observations were obtained.

Compound Observation
W precipitate formed immediately
X precipitate formed after 5 minutes

ttrendyline
tre
endylin
nd
d line
Y precipitate formed after 10 minutes
Z no precipitate
cipitat formed

Jurong Junior College 9647/01/J2


9647/0
647 1/J2
J2 PRELIMINARY EXAMINATION/2016
EXAMINATION
AMINAT /201
016

531 trendyline
11

What could be the identities of compounds W, X, Y and Z?

W X Y Z
A CH 3 COCl CH 3 CH 2 Cl CH 3 CH 2 Br C 6 H 5 Br
B CH 3 COCl CH 3 CH 2 Br CH 3 CH 2 Cl C 6 H 5 Br Formatted: Not Highlight
Formatted: Not Highlight
C CH 3 CH 2 Br CH 3 COCl C 6 H 5 Br CH 3 CH 2 Cl
Formatted: Not Highlight
D CH 3 CH 2 Br C 6 H 5 Br CH 3 COCl CH 3 CH 2 Cl Formatted: Not Highlight
Formatted: Not Highlight
Formatted: Not Highlight
Formatted: Not Highlight
Formatted: Not Highlight
29 Chlorogenic acid occurs naturally in both coffee and an edible species of bamboo.
Formatted: Not Highlight
Formatted: Not Highlight
O
Formatted: Not Highlight

HO Formatted: Not Highlight

OH Formatted: Not Highlight

O Formatted: Not Highlight


Formatted: Not Highlight

HO O

OH
OH

Chlorogenic acid OH

Which statement about chlorogenic acid is not correct? Formatted: Font: Bold

A It reacts with 4 mol of NaOH when heated with NaOH(aq). Formatted: Not Highlight

B It gives CO 2 (g) when heated with acidified potassium manganiate (VII). Formatted: Not Highlight
Formatted: Not Highlight
C It reacts with Br 2 (aq) to incorporate up to 5 atoms of bromine in each molecule.
Formatted: Not Highlight
D It reacts with a heated acidified solution of potassium dichromate (VI) to give a product
Formatted: Not Highlight
with 112 sp2 carbons.
Formatted: Font: (Default) Times
5 Non covalent chlorides like SiCl 4 and PCl 3 undergo hydrolysis to give an acidic solution. New Roman, Not Highlight

Which of the following chlorides is not expected to undergo hydrolysis? Formatted: Not Highlight
Formatted: Not Highlight
A CCl 4 B GeCl 4 C AsCl 3 D SbCl 3
Formatted: Not Highlight
6 Which compound cannot be prepared directly from 4-nitromethylbenzene?
Formatted: Not Highlight
A CH3 B CH3 O C COOH D CH3
Formatted: Not Highlight
Formatted: Font: (Default) Times
C
New Roman, Not Highlight
Cl
Formatted: Not Highlight

Br Formatted: Not Highlight

trendyline
e
Formatted: Not Highlight
NH2 NO2 NO2
NO2

Jurong Junior College 9647/01/J2


9647/0
647 1/J2
J2 PRELIMINARY EXAMINATION
EXAMINATION/2016
EXAMINAT /201
016 [Turn over

532 trendyline
12

7 Which reagent will convert compound G to compound E efficiently.


O CN HO CN

Cl Cl
O O O O
Compound G Compound E
+
A H 2 /Ni B H 2 O/H C NaBH 4 D LiAlH 4
8 A sodium halide was reacted with phosphoric acid which evolves thick white fumes. The
white fumes was then passed through AgNO 3 (aq) which gives a precipitate insoluble in
concentrated NH 3 .
What is the role of phosphoric acid in the reaction and the identity of the sodium halide?
Role of phosphoric acid Identity of sodium halide
A Acid Sodium bromide
B Acid Sodium iodide
C Oxidising agent Sodium bromide
D Oxidising agent Sodium iodide

9 When heated with bromine, the hydrocarbon 3-methylpropane undergoes free radical
substitution. In a propagation step, the free radical X is formed by the loss of one hydrogen
atom.

CH3

CH3CH2CHCH2CH3 + Cl X + HCl

Which X formed is the most stable?


A B
CH3 CH3

CH2CH2CHCH2CH3 CH3CHCHCH2CH3

Jurong Junior College

trendyline
e
9647/01/J2
9647/0
647 1/J2
J2 PRELIMINARY EXAMINATION/2016
EXAMINATION
AMINAT /201
016

533 trendyline
13
C D
CH3 CH2

CH3CH2CCH2CH3 CH3CH2CHCH2CH3

10 Histidine, an essential amino acid, is present in many proteins and enzymes and plays a vital
role in the structure and oxygen-binding function of haemoglobin.
Similar to Q2
O
Remove Q2

N
OH

NH2
HN
histidine
Histidine has pKa values of 1.8, 6.0 and 9.2.
What is the structure of the major species in a solution of histidine at pH 7?

trendyline
e
A B

Jurong Junior College 9647/01/J2


9647/0
647 1/J2
J2 PRELIMINARY EXAMINATION
EXAMINATION/2016
EXAMINAT /201
016 [Turn over

534 trendyline
14
COOH COO

+ +
H3N H3N

+ +
HN HN

NH NH

C D
COO COO

+
H3N H2N

N N

NH NH

11 1-bromopropane is converted to 2-bromopropane by a two-step process involving the


intermediate T.

step 1 step 2
CH 3 CH 2 CH 2 Br T CH 3 CHBrCH 3

What could be the reagent for step 1 and for step 2 to get the highest yield of 2-
bromopropane?
step 1 step 2
A Concentrated H 2 SO 4 , heat SOBr 2 , heat
B NaOH(aq), heat HBr(aq)
C NaOH in ethanol, heat HBr(g)
D NaCN in ethanol, heat PBr 3 , heat

302 The structure of 1-acetylpiperidine is shown below.

N C CH3

-
1 acetylpiperidine

Which pair of compounds would produce 1-acetylpiperidine when reacted together?

Jurong Junior College

trendyline
e
9647/01/J2
9647/0
647 1/J2
J2 PRELIMINARY EXAMINATION/2016
EXAMINATION
AMINAT /201
016

535 trendyline
15
A
N H + CH3COOH

B Formatted: Not Highlight


N H + CH3COCl Formatted: Not Highlight

C
N COOH + CH3OH

D
N COCl + CH3OH

13 A student added aqueous lead(II) nitrate into test tubes containing compounds W, X, Y and
Z. The and obtained the following observations were obtained.

Compound Observation
W White precipitate formed immediately
X White precipitate formed after 5 minutes
Y White precipitate formed after 10 minutes
Z No precipitate formed

What could be the identity of Compounds compounds W, X, Y and Z?


W X Y Z
A CH 3 COCl CH 3 CH 2 Cl CH 3 CH 2 Br C 6 H 5 Br
B CH 3 COCl CH 3 CH 2 Br CH 3 CH 2 Cl C 6 H 5 Br
C CH 3 CH 2 Br CH 3 COCl C 6 H 5 Br CH 3 CH 2 Cl
D CH 3 CH 2 Br C 6 H 5 Br CH 3 COCl CH 3 CH 2 Cl

Jurong Junior College

trendyline
e
9647/01/J2
9647/0
647 1/J2
J2 PRELIMINARY EXAMINATION
EXAMINATION/2016
EXAMINAT /201
016 [Turn over

536 trendyline
16

Section B
For each of the questions in this section, one or more of the three numbered statements 1 to 3 may
be correct.
Decide whether each of the statements is or is not correct (you may find it helpful to put a tick
against the statements that you consider to be correct).
The responses A to D should be selected on the basis of

A B C D
1, 2 and 3 1 and 2 2 and 3 1 only
are only are only are is
correct correct correct correct

No other combination of statements is used as a correct response.

31 Consider the reaction below:

2H+(aq) + 2NO 2 (aq) H 2 O(l) + NO(g) + NO 2 (g)

Which statements correctly describe the process?

1 The bond angle in NO 2 ion is larger than that in H 2 O.


2 This is a disproportionation reaction.
3 H+(aq) is oxidised by NO 2 (aq).

32 The boiling points of six consecutive elements, lettered T to Y, are given in the table below.
(Note: these letters are not the atomic symbols of the elements concerned.)

Element T U V W X Y
Atomic Number z z+1 z+2 z+3 z+4 z+5
Boiling Point / K 73 93 83 23 1163 1373

Which of the following statements are true?

1 Only X and Y have giant structures.


2 The atomic radius of W is smaller than that of X.
3 First ionisation energy of T is higher than that of U.

Jurong Junior College

trendyline
e
9647/01/J2
9647/0
647 1/J2
J2 PRELIMINARY EXAMINATION/2016
EXAMINATION
AMINAT /201
016

537 trendyline
17
33 The value of pV is plotted against p for 2 gases, A and B, where p is the pressure
and V is the volume of the gas.

pV

Which of the following could be the identities of the gases?

gas A gas B
1 0.5 mol of CH 4 at 25 C 0.5 mol of CH 4 at 50 C
2 0.5 mol of NH 3 at 25 C 1.0 mol of CH 4 at 25 C
3 0.5 mol of NH 3 at 25 C 0.5 mol of NH 3 at 323 C

34 The diagram shows apparatus that can be used to measure the E of a half-cell.

hydrogen Ga

H2SO4(aq)
gallium chloride
solution
platinum foil coated with
finely divided platinum

Which factors are essential for an accurate measurement of E for the following electrode
system?
Ga3+ + 3e Ga

1 The H 2 SO 4 (aq) should be at a concentration of 0.5 mol dm3.


2 The Ga3+(aq) and Cl (aq) should each be at a concentration of 1.0 mol dm3.
3 The hydrogen should be dry.

Jurong Junior College

trendyline
e
9647/01/J2
9647/0
647 1/J2
J2 PRELIMINARY EXAMINATION
EXAMINATION/2016
EXAMINAT /201
016 [Turn over

538 trendyline
18

The responses A to D should be selected on the basis of

A B C D
1, 2 and 3 1 and 2 2 and 3 1 only
are only are only are is
correct correct correct correct

No other combination of statements is used as a correct response.

35 Aqueous CuCl 2 undergoes the following reactions:

Step 1 Step 2 Cu(NH3)4(H2O)22+ Formatted: Font: Not Bold


CuCl2(aq) Cu(OH)2
Formatted: Font: Not Bold
Pale blue solution dilute ammonia pale blue pptexcess dilute deep blue solution
ammonia Formatted: Centered
Formatted: Font: Not Bold
Conc HCl Formatted: Font: Not Bold
A
CuCl4 CuCl2(a)
Yellow-green solutuion

Which of the following statements are correct?

1 CuCl 4 2 is a more stable complex than Cu(NH 3 ) 4 (H 2 O) 2 2+. Formatted: Not Highlight

2 2+ Formatted: Font: (Default) Times


2 CuCl 4 , Cu(NH 3 ) 4 (H 2 O) 2 and CuCl 2 (aq) have different colours due to the different New Roman, Not Highlight
ligands present. Ammonia acts as base to give a source of OH in step 1.
Formatted: Not Highlight
3 Ligand exchange takes place in step 2 as NH 3 is a stronger ligand than H 2 O. Formatted: Not Highlight
Formatted: Not Highlight

32 The halogens are oxidising agents which oxidises sodium thiosulfate, Na 2 S 2 O 3. Which of the Formatted: Not Highlight
following statements is true regarding the halogenthiosulfate reaction? Formatted: Not Highlight

1 The oxidising power of the halogens decreases down the group Formatted: Not Highlight
Formatted: Not Highlight
2 When Iodine is the oxidising agent, the average oxidation state of S changes from +2 to
+2.5 after the reaction. Formatted: Not Highlight
Formatted: Not Highlight
3 When bromine is the oxidising agent, one of the products gives a white precipitate with
Formatted: Not Highlight
Ba(NO 3 ) 2.
Formatted: Not Highlight
Formatted: Not Highlight
Potential question to replace question on which is true about alpha amino acid.
Formatted: Font: (Default) Times
New Roman, Not Highlight

3631 Consider the following reaction scheme. Formatted: Subscript

Jurong Junior College

trendyline
e
9647/01/J2
9647/0
647 1/J2
J2 PRELIMINARY EXAMINATION/2016
EXAMINATION
AMINAT /201
016

539 trendyline
19
OH O OCH3 OCH3

CH3Br NH2NH2
Na

CHO CHO CHO HC N NH2

Which of the following type of reactions is involved in the reaction scheme.?

1 Acid base reaction


2 Condensation Formatted: Not Highlight

3 Nucleophilic substitution Formatted: Not Highlight

32 Chlorogenic acid occurs naturally in coffee and an edible species of bamboo.


O

HO
OH
O

HO O

O
H OH

Chlorogenic acid OH
Which reaction will ajulmeic acid undergo?
1 It reacts with 4 mol of NaOH(aq) when heated with NaOH(aq).
2 It reacts with Br 2 (aq) to incorporate up to 5 atoms of bromine in each molecule.
3 It will turn a heated acidified solution of potassium dichromate (VI) from orange to
green.

3743 Chlorofluorocarbons (CFCs) have been widely used in aerosol sprays, refrigerators and in
making foamed plastics, but are now known to destroy ozone in the upper atmosphere.

Formatted: Not Highlight


What will not destroy ozone, and therefore can be used as a replacement for CFCs?
Formatted: Not Highlight
Formatted: Not Highlight
1 CH 2 FCH 2 F
Formatted: Not Highlight
2 CH 3 CH 2 CH 3 Formatted: Not Highlight
3 CHBr 3 Formatted: Not Highlight
Formatted: Not Highlight
Formatted: Not Highlight
34 Which of the following is true about an -amino acid?

t e
Formatted: Not Highlight
1 It is amphoteric.. Formatted: Not Highlight
Formatted: Not Highlight
Jurong Junior College 9647/01/J2
9647/0
647 1/J2
J2 PRELIMINARY EXAMINATION
EXAMINATION/2016
EXAMINAT /201
016 [Turn over

540 trendyline
20 Formatted ...
Formatted ...
2 It is soluble in water.
Formatted ...
3 It always exists as a zwitterion at pH 7.
Formatted ...
Formatted ...
Formatted ...
3855 Which aspect of the protein structure will be affected by strong acids at room temperature? Formatted ...
Formatted ...

1 Tertiary structure Formatted ...


Formatted ...
2 Secondary Structure
Formatted ...
3 Primary Structure Formatted ...
Formatted ...
OR Formatted ...
394 Which compounds will react to produce CH 3 CCH 2 HCH 2 CH 2 OH in good yield? Formatted ...
Formatted ...
Formatted ...
1 CH 3 CCH 2 HCH 2 OCOCH 2 CH 3 + NaOH(aq) with heat
Formatted ...
2 CH 3 CH=CH 2 + concentrated H 2 SO 4 followed by steam CH 3 COOH + D 2 , Ni with heat Formatted ...
3 CH 3 CH 2 COOH + H 2 , Ni with heat CH 3 CH=CH 2 + D 2 O with H+ Formatted ...
Formatted ...
Formatted ...
Formatted ...
40 Most of the chemistry of magnesium relates to its ionic compounds. However, magnesium
6 does form an important group of covalent compounds known as Grignard reagents. Formatted ...
Formatted ...
A typical example of the use of a Grignard reagent is the two-step reaction of C 2 H 5 MgBr Formatted ...
RMgBr with carbonyl compounds like propanone to form RC(CH 3 ) 2 (OH), CH 3 COCH 3 , to form Formatted ...
2-methylbutan-2-ol.
Formatted ...
Formatted ...
You may take R to represent an alkyl group.
Formatted ...
Formatted ...
OMgBr OH
Formatted ...
Step 1 Step 2
RMgBr + CH3COCH3 H3C C CH3 H3C C CH3 + Mg(OH)Br Formatted ...
H2O Formatted ...
R R Formatted ...
Formatted ...
Which of the following compounds could be the product of a Grignard reagent reacting with Formatted ...
an aldehyde?
Formatted ...
Formatted ...
1 CH 3 CH(OH)CH 3 Formatted ...
2 CH 3 CH(CH 3 )CH(OH)CH 3 Formatted ...
3 CH 3 CH 2 C(OH)(CH 3 )CH 2 CH 3 Formatted ...
Formatted ...
This is a pattern recognition question
Formatted ...
Formatted ...
Formatted ...
Formatted ...
Formatted ...

trendyline
e
Formatted ...
Formatted ...
Formatted ...
Jurong Junior College 9647/01/J2
9647/0
647 1/J2
J2 PRELIMINARY EXAMINATION/2016
EXAMINATION
AMINAT /201
016

541 trendyline
21

BLANK PAGE

Jurong Junior College 9647/01/J2


9647/0
647 1/J2
J2 PRELIMINARY EXAMINATION
EXAMINATION/2016
EXAMINAT /201
016

e
[Turn over

542 trendyline
Answer Key to 2016Jurong Junior College H2 Chemistry Prelim Exam Paper 1

1 A 11 D 21 C 31 B
2 D 12 D 22 C 32 A
3 B 13 D 23 C 33 C
4 C 14 B 24 C 34 D
5 C 15 A 25 A 35 C
6 B 16 A 26 B 36 C
7 A 17 D 27 D 37 B
8 A 18 B 28 B 38 D
9 A 19 D 29 D 39 D
10 C 20 D 30 B 40 B

trendyline
543 trendyline
-85
521*-81,25&2//( (*(
 /,0,1$5<(;$0,1$7
-&35(/ 7,21
+LJK
KHU

&$1','$7
7(
1$0(

(;$0,1'(;
&/$66
(5
180%(

&+(0,675< 
 
3DSHU6WWUXFWXUHG4XHVWLRQV $X
XJXVW

KRXUV
V

&DQGLGDWHVVDQVZHURQ QWKH4XHVWWLRQ3DSHU
$GGLWLRQDO0DWHULDOV 'DWD%
%RRNOHW

5($'7+(6(,16758&7,216),5
567

:ULWH\RXUQQDPHFODVVDQGH[DPLQ QGH[QXPEHURQDOOWKHZR RUN\RXKDQG


GLQ
:ULWHLQGDUUNEOXHRUEOD
DFNSHQ
<RXPD\XVVHD+%SHQFFLOIRUDQ\GLD DJUDPVRUJUUDSKV
'RQRWXVHVWDSOHVSDS KOLJKWHUVJOXHRUFRUUHFWLR
SHUFOLSVKLJK RQIOXLG

$QVZHUDOOWWKHTXHVWLRQV
7KHXVHRIDDQDSSURYHGGVFLHQWLILFFD
DOFXODWRULVH[[SHFWHGZKH
HUHDSSURSULD
DWH
$Data Boo GHG
okletLVSURYLG

$WWKHHQGR
RIWKHH[DPLQ
QDWLRQIDVWHQ
QDOO\RXUZRRUNVHFXUHO\WWRJHWKHU
7KHQXPEHURIPDUNVLVVJLYHQLQEUD
DFNHWV>@DWWWKHHQGRIHD
DFKTXHVWLRQRUSDUWTXHVVWLRQ

)RUU([DPLQHUV
V8VH
 

 

 

 

 

 
3HQ
QDOW\ GHOHWHDFFFRUGLQJO\
/DFNVILQILQDODQV 1$

trendyline
ne
ne
0LVVLQJZ ZURQJXQLWV
ZURQ
LQILQ
LQIL
LQILQ
QQDODQV
DO
DOD  1$
%RQGOOOLQNDJHV
%RQG LQND 1$
7R
7 R
RWDO
WD

7KLVGRFXPH
HQWFRQVLVWVRISULQWHG
GSDJHVDQGEODQNSDJH
H

544 >7XUQRYHU
trendyline U
 
$QVZHUDOOWKHTXHVWLRQV

 3ODQQLQJ 3   For
Examiners
Use
 :KHQ LRGLQH LV PL[HG ZLWK SURSDQRQH LQ WKH SUHVHQFH RI GLOXWH VXOIXULF DFLG RQH RI
WKHK\GURJHQDWRPVLQSURSDQRQHLVUHSODFHGE\DQLRGLQHDWRPDQGK\GURLRGLFDFLG
+,LVSURGXFHG
7KHUDWHHTXDWLRQIRUWKHUHDFWLRQLVGHWHUPLQHGWREH
UDWH k>&+&2&+@>+@
,QLWLDOO\WKHUHDFWLRQPL[WXUHLVEURZQGXHWRWKHLRGLQHWKDWLVSUHVHQW7KHUHDFWLRQLV
FRPSOHWHZKHQDOOWKHLRGLQHKDVUHDFWHGDQGWKHVROXWLRQEHFRPHVFRORXUOHVV
$WHDFKHUGHPRQVWUDWHVWKLVH[SHULPHQWDVIROORZV
6KHSUHSDUHVWZRVHSDUDWHVROXWLRQV;DQG<
6ROXWLRQ; FRQWDLQVJGPLRGLQHVROXWLRQ
6ROXWLRQ< LVDPL[WXUHRIFPRIPROGPSURSDQRQHDQGFPRI
PROGPGLOXWHVXOIXULFDFLG
6KH PL[HV  FP RI VROXWLRQ ; ZLWK  FP RI VROXWLRQ < DW URRP WHPSHUDWXUH RI
R&DQGDIWHUDERXWVHFRQGVWKHPL[WXUHGHFRORXULVHV 
'DWD%RLOLQJSRLQWRISURSDQRQHLVR& 

 D  &RQVWUXFWDEDODQFHGFKHPLFDOHTXDWLRQIRUWKLVUHDFWLRQ 

 
 >@

 E  &RQVLGHUWKHGHVFULSWLRQRIWKHH[SHULPHQWJLYHQDERYH
:ULWH D SODQ WR GHWHUPLQH KRZ WKH UDWH RI WKLV UHDFWLRQ FKDQJHV ZLWK
WHPSHUDWXUH
<RX PD\ ILQG LW XVHIXO WR XVH W WR UHSUHVHQW WKH UHDFWLRQ UDWH ZKHUH W LV WKH
WLPHWDNHQIRUWKHUHDFWLRQPL[WXUHWRGHFRORXULVH
,Q \RXU SODQ \RX VKRXOG XVH WKH VDPH YROXPHV RI VROXWLRQ ; DQG VROXWLRQ <
GHVFULEHGDERYH
<RXU SODQ VKRXOG LQFOXGH ILYH H[SHULPHQWV (QVXUH WKDW DW OHDVW RQH RI \RXU
H[SHULPHQWVZRXOGEHH[SHFWHGWRWDNHPRUHWKDQVHFRQGV
<RXPD\DVVXPHWKDW\RXDUHSURYLGHGZLWKWKHIROORZLQJ
 VROXWLRQ;FRQWDLQLQJJRILRGLQHLQGPRIZDWHU
 PROGPGLOXWHVXOIXULFDFLG
 PROGPSURSDQRQHVROXWLRQ
 GHLRQLVHGZDWHUKRWZDWHUDQGFUXVKHGLFH
 ERLOLQJWXEHVDQGRWKHUDSSDUDWXVQRUPDOO\IRXQGLQDVFKRRORUFROOHJH
ODERUDWRU\

<RXUSODQVKRXOGFRQWDLQWKHIROORZLQJ

trendyline
 GHWDLOV
HWDLOV IRU WKH SUHSDUDWLRQ RI PRO
 GP SU
SURSDQRQH VROXWLRQ
VROXWLR IURP WKH
SURSDQRQHVROXWLRQSURYLGHG
URSDQRQHVROXWLRQSURYLGHG
 GHWDLOVIRUWKHSUHSDUDWLRQRIVROXWLRQ<
HWDLOVIRUWKHSUHSDUDWLRQRIVROXWLRQ <
 WKHWHPSHUDWXUHVDWZKLFKWKHILYHH[SHULPHQWVZRXOGEHFDUULHGRXW
KHWHPSHUDWXUHVDWZKLFKWKHILYHH[SHULPHQWVZRXOGEHFDUULH
HWHPSHUDWXUHVDWZKLFKWKHILYHH[SHULPHQWVZRXOGEHFDUULH
 DOOPHDVXUHPHQWV\RXZRXOGPDNH
OOPHDVXUHPHQWV\RXZRXOGPDNH
PHDVXUHPHQWV\RXZRXOGPDNH
 DOOHVVHQWLDOH[SHULPHQWDOGHWDLOVWRHQVXUHDFFXUDWHUHVXOWV
OO HVVHQWLDO H[SHULPHQWDO GHWDLOV WR R HQVXUH
H DFFXUDWH UHVXOWV 


-XURQJ-XQLRU&ROOHJH -35(/,0,1$5<(;$0,1$7,21  
545 trendyline

 

 E  For
 Examiners
Use



















































trendyline








 


-XURQJ-XQLRU&ROOHJH -35(/,0,1$5<(;$0,1$7,21 >7XUQRYHU 
546 trendyline

 

 E  For
 Examiners
Use



















































trendyline








 >@


-XURQJ-XQLRU&ROOHJH -35(/,0,1$5<(;$0,1$7,21  
547 trendyline

 

 F  ,GHQWLI\RQHSRWHQWLDOVDIHW\KD]DUGLQWKLVH[SHULPHQWDQGVWDWHKRZ\RXZRXOG For
Examiners
PLQLPLVHWKLVULVN  Use

  





 >@

 G  7KHDFWLYDWLRQHQHUJ\EDIRUWKHUHDFWLRQFDQEHGHWHUPLQHGIURPWKHIROORZLQJ 
ArrheniusHTXDWLRQ
E
 D
k  $H 57 ZKHUHkLVWKHUDWHFRQVWDQW
$LVWKHArrheniusFRQVWDQW
EDLVWKHDFWLYDWLRQHQHUJ\
5LVWKHPRODUJDVFRQVWDQWDQG
7LVWKHWHPSHUDWXUHLQ.HOYLQV
7KHArrheniusHTXDWLRQPD\DOVREHH[SUHVVHGLQWKHIROORZLQJOLQHDUIRUP
E
OQk OQ$ D 
57 

  L  6WDWH WKH UHODWLRQVKLS EHWZHHQ k DQG W WKH WLPH WDNHQ IRU WKH UHDFWLRQ 
PL[WXUHWRGHFRORXULVH 

   
 >@

  LL  2XWOLQH KRZ WKH GDWD FROOHFWHG LQ \RXU H[SHULPHQW ZRXOG EH XVHG WR 
GHWHUPLQHWKHDFWLYDWLRQHQHUJ\EDIRUWKHUHDFWLRQ 

   







>@

 H  :KDW LV WKH HIIHFW RQ W WKH WLPH WDNHQ IRU WKH UHDFWLRQ PL[WXUH WR GHFRORXULVH 
LIWKHFRQFHQWUDWLRQRILRGLQHVROXWLRQXVHGLVKDOYHG"([SODLQ\RXUDQVZHU 

  


trendyline



 >@
    
   >7RWDO@ 

 

-XURQJ-XQLRU&ROOHJH -35(/,0,1$5<(;$0,1$7,21 >7XUQRYHU 


548 trendyline

 


 7KLVTXHVWLRQLVDERXWFKURPLXPFRPSRXQGVDQGWKHIROORZLQJGDWDDUHUHOHYDQW For
Examiners
Use

salt Colour Ksp value at 25 oC


%D&U2 RUDQJH 6ROXEOHLQZDWHU
%D&U2 \HOORZ [
$J&U2 UHG [

ions &U2 DT  &U2 DT  &U DT  &U DT 
Colour RUDQJH \HOORZ JUHHQ EOXH



 D  L  :ULWHDQH[SUHVVLRQIRUKVSRI$J&U2VWDWLQJLWVXQLWV  

   

>@

  LL  &DOFXODWHWKHVROXELOLW\RI$J&U2LQJGP  

   

>@

  LLL  6WDWHWKHH[SHFWHGREVHUYDWLRQVZKHQFPRI[PROGPRI 
.&U2 VROXWLRQ LV PL[HG ZLWK  FPRI DQ DTXHRXV VROXWLRQ FRQWDLQLQJ
PROGPRI$J12DQGPROGPRI%D 12 
-XVWLI\\RXUDQVZHUZLWKUHOHYDQWFDOFXODWLRQV 

    

   

>@

 

-XURQJ-XQLRU&ROOHJH -35(/,0,1$5<(;$0,1$7,21  
549 trendyline

 

 E  ,QDTXHRXVVROXWLRQ&U2LRQVH[LVWLQHTXLOLEULXPZLWK&U2LRQV For
Examiners
Use
&U2 DT +2 l &U2 DT + DT 
3UHGLFW DQG H[SODLQ WKH HIIHFW RI DGGLQJ DTXHRXV EDULXP FKORULGHWR DQ
HTXLOLEULXP PL[WXUH RI &U2 DQG&U2LRQV LQ WHUPV RI HTXLOLEULXP SRVLWLRQ
DQGS+ 

  



 >@

 F  :KHQ JUDQXODWHG ]LQF LV DGGHG WR DQ DFLGLILHG VROXWLRQ RI DTXHRXV
FKURPLXP ,,, VROXWLRQWKHVROXWLRQWXUQVIURPJUHHQWREOXH
:KHQ WKH H[FHVV ]LQF LV ILOWHUHG RII DQG WKH EOXH ILOWUDWH LV OHIW WR VWDQG IRU
VHYHUDO KRXUV LQ WKH DEVHQFH RI DLU LW VORZO\ WXUQHG EDFN WR JUHHQ DQG D
FRORXUOHVVJDVLVHYROYHG
8VLQJ WKH JLYHQ LQIRUPDWLRQ DQG UHOHYDQW EYDOXHV IURP WKH Data
BookletH[SODLQWKHDERYHREVHUYDWLRQV
:ULWHEDODQFHGHTXDWLRQVIRUDQ\UHDFWLRQVWKDWRFFXU 

    















>@

   >7RWDO@ 


trendyline


-XURQJ-XQLRU&ROOHJH -35(/,0,1$5<(;$0,1$7,21 >7XUQRYHU 


550 trendyline

 

 7KLVTXHVWLRQLVDERXW3HULRGDQGHOHPHQWVDQGWKHLUFRPSRXQGV  For
Examiners
Use
 D  %RWKPDJQHVLXPDQGOLWKLXPJLYHR[LGHV0J2DQG/L2UHVSHFWLYHO\ZKHUHDV
VRGLXPJLYHVDPL[WXUHRIR[LGHDQGSHUR[LGH1D2DQG1D2ZKHQEXUQHGLQ
R[\JHQ
([SODLQZK\OLWKLXPGRHVQRWJLYH/L2ZKHQEXUQHGLQR[\JHQ 

  



 >@

 E  ;<DQG=LQQRSDUWLFXODURUGHUDUH0J26L2DQG32 
6RPHLQIRUPDWLRQUHJDUGLQJWKHVHWKUHHR[LGHVDUHOLVWHGEHORZ

R[LGH PHOWLQJSRLQW. UHVXOWLQJS+ZKHQPL[HGZLWKZDWHU


;  
<  
=  



  L  6XJJHVWWKHLGHQWLWLHVRI;<DQG=  

   
;<= >@

  LL  :LWKUHIHUHQFHWRWKHVWUXFWXUHDQGERQGLQJH[SODLQZK\WKHUHVXOWLQJS+ 
LVZKHQ<LVPL[HGZLWKZDWHU 

   






 >@

  LLL  :ULWH HTXDWLRQV WR DFFRXQW IRU WKH GLIIHUHQFH LQ S+ ZKHQ ; DQG = DUH 
PL[HGZLWKZDWHU 

   

>@

trendyline
  LY  ZRRIWKHVHWKUHHR[LGHVDUHDFLGLFLQQDWXUH
ZRRIWKHVHWKUHHR[LGHVDUHDFLGLFLQQDWXUH:ULWHHTXDWLR
7ZRRIWKHVHWKUHHR[LGHVDUHDFLGLFLQQDWXUH:ULWHHTXDWLRQVWRVKRZ 
RZWKHVHWZRR[LGHVDFWDVDQDFLG
KRZWKHVHWZRR[LGHVDFWDVDQDFLG 

    


 >@

-XURQJ-XQLRU&ROOHJH -35(/,0,1$5<(;$0,1$7,21  
551 trendyline

 

 F  6RPHHQWKDOS\FKDQJHVDUHOLVWHGEHORZ For
Examiners
Use
/DWWLFHHQHUJ\RI&D 2+  N-PRO
(QWKDOS\FKDQJHRIK\GUDWLRQRI&D N-PRO
(QWKDOS\FKDQJHRIK\GUDWLRQRI2+ N-PRO
(QWKDOS\FKDQJHRIIRUPDWLRQRI+2 l  N-PRO
(QWKDOS\FKDQJHRIIRUPDWLRQRI&D 2+  V  N-PRO


  L  8VLQJ UHOHYDQW GDWD IURP WKH DERYH FDOFXODWH WKH HQWKDOS\ FKDQJH RI 
VROXWLRQRI&D 2+  

   

>@

  LL  &DOFLXP UHDFWV ZLWK ZDWHU WR IRUP DTXHRXV FDOFLXP K\GUR[LGH DQG 
K\GURJHQJDV
&D V +2 l &D 2+  DT + J Hr
8VLQJ UHOHYDQW GDWD IURP WKH DERYH DQG \RXU DQVZHU LQ F L  GUDZ DQ
HQHUJ\F\FOHDQGXVHLWWRGHWHUPLQHHU 

    













>@

   >7RWDO@ 


trendyline
 

-XURQJ-XQLRU&ROOHJH -35(/,0,1$5<(;$0,1$7,21 >7XUQRYHU 


552 trendyline

 

 D  1LWURJHQPRQR[LGHDQGIOXRULQHXQGHUJRWKHIROORZLQJJDVHRXVUHDFWLRQWRIRUP For
Examiners
QLWURV\OIOXRULGH21) Use

12 J ) J 21) J 


7KH UHDFWLRQ ZDV IROORZHG WZLFH ZLWK GLIIHUHQW FRQFHQWUDWLRQV RI ) ,Q HDFK
H[SHULPHQWWKHRYHUDOOFRQFHQWUDWLRQRI)UHPDLQHGYLUWXDOO\FRQVWDQW
7KHIROORZLQJUHVXOWVZHUHREWDLQHGIURPExperiment 1ZKHQWKHFRQFHQWUDWLRQ
RI)LVPROGP
7LPHPLQ >12@PROGP
 
 
 
 
 
 
 
7KHGDWDLQWKHWDEOHDERYHZDVSORWWHGRQVXLWDEOHD[HVDQGWKHJUDSKEHORZ
ZDVREWDLQHG

>12@PROGP















        

 WLPHPLQ 

  L  8VLQJ WKH JUDSK DERYH GHWHUPLQH WKH RUGHU RI UHDFWLRQ ZLWK UHVSHFW WR 
12 

    





 >@

 
-XURQJ-XQLRU&ROOHJH -35(/,0,1$5<(;$0,1$7,21  
553 trendyline

 

 D  LL  7KHFRQFHQWUDWLRQRI)IRUExperiment 2LVPROGP7KHWLPHWDNHQ For


Examiners
IRUWKHFRQFHQWUDWLRQRI12WRGURSWRKDOIRILWVRULJLQDOFRQFHQWUDWLRQLQ Use
Experiment 2LVPLQ
8VLQJ WKLV LQIRUPDWLRQ DQGVXLWDEOH GDWD IURP WKH JUDSKGHWHUPLQH WKH
RUGHURIUHDFWLRQZLWKUHVSHFWWR) 

   









 >@

  LLL  +HQFHGHWHUPLQHWKHUDWHHTXDWLRQIRUWKLVUHDFWLRQ  

   

>@

  LY  7KHPHFKDQLVPKDVWZRVWHSVRQHRIZKLFKSURGXFHV21)DQGWKHIUHH 
UDGLFDO)LQHTXLPRODUDPRXQWV6XJJHVWHTXDWLRQVIRUWKHWZRVWHSVRI
WKHPHFKDQLVPLQGLFDWLQJFOHDUO\WKHVORZVWHS 

   

>@

 E  1LWURJHQGLR[LGHDQGFDUERQGLR[LGHJDVHVEHKDYHOLNHLGHDOJDVHVXQGHUKLJK 
WHPSHUDWXUHDQGORZSUHVVXUH 

  L  'UDZGRWDQGFURVVGLDJUDPVIRU12DQG&2PROHFXOHV  

     
 
 
 
12 &2 >@

  LL  6XJJHVW D UHDVRQ ZK\ 12 JDV LV H[SHFWHG WR GHYLDWH PRUH IURP WKH 
LGHDOJDVEHKDYLRXUWKDQ&2JDV 

   


trendyline










 >@

   >7RWDO@ 

-XURQJ-XQLRU&ROOHJH -35(/,0,1$5<(;$0,1$7,21 >7XUQRYHU 


554 trendyline

 

 *DVROLQHLVDPL[WXUHRI&WR&DONDQHV:KHQWUHDWHGZLWKEURPLQHXQGHUIUHH For
Examiners
UDGLFDOFRQGLWLRQVLWSURGXFHVDPL[WXUHRIEURPLQDWHGFRPSRXQGV  Use

 D  &RPSRXQG$&+%ULVRQHRIWKHEURPLQDWHGFRPSRXQGV 
:KHQ$LVKHDWHGZLWKDTXHRXVSRWDVVLXPK\GUR[LGHFRPSRXQG%LVIRUPHG
%GRHVQRWUHDFWZLWKKRWDFLGLILHGSRWDVVLXPGLFKURPDWH 9,  

  L  6WDWHWKHIXQFWLRQDOJURXSSUHVHQWLQ%  

   
 >@

  LL  'UDZWKHGLVSOD\HGIRUPXODRI$DQG%  

      
  
  
  
 
 
$ % >@

  LLL  'HVFULEH WKH PHFKDQLVP WR VKRZ KRZ FRPSRXQG $ LV IRUPHG IURP LWV 
FRUUHVSRQGLQJDONDQHVKRZLQJVWUXFWXUDOIRUPXODRIDOOVSHFLHV 

    
















trendyline

 >@




-XURQJ-XQLRU&ROOHJH -35(/,0,1$5<(;$0,1$7,21  
555 trendyline

 

 E  &RPSRXQG&&+%ULVDQRWKHUEURPLQDWHGFRPSRXQG For
Examiners
Use
:KHQ&XQGHUJRHVGHK\GUREURPLQDWLRQFRPSRXQG'&+LVSURGXFHG
 PRO RI ' UHDFWV ZLWK KRW DFLGLILHG .0Q2 WR JLYH  PRO RI &2  PRO RI
SURSDQHGLRLFDFLG+2&&+&2+DQGPRORIFRPSRXQG(&+2
&RPSRXQG ( VKRZV D SRVLWLYH WHVW ZLWK DONDOLQH DTXHRXV LRGLQH DQG
GLQLWURSKHQ\OK\GUD]LQHUHVSHFWLYHO\ 

  L  6WDWHWKHIXQFWLRQDOJURXSVSRVLWLYHO\LGHQWLILHGE\ 
 DONDOLQHDTXHRXVLRGLQH



 GLQLWURSKHQ\OK\GUD]LQH


>@

  LL  ,GHQWLI\FRPSRXQG(DQGKHQFHGHGXFHWKHVWUXFWXUHRI'  
 
    
 
  

 
 

 
 
( ' >@


 

trendyline
-XURQJ-XQLRU&ROOHJH -35(/,0,1$5<(;$0,1$7,21 >7XUQRYHU 
556 trendyline

 

 F  EURPRPHWK\OKH[DQHLVDQRWKHUEURPLQDWHGFRPSRXQG For
Examiners
Use
8QGHU VXLWDEOH FRQGLWLRQV LW UHDFWV ZLWK SRWDVVLXP HWKR[LGH &+&+2 . WR
JLYHPHWK\OKH[HQH 

  L  7KH IROORZLQJ VKRZV D SRVVLEOH PHFKDQLVP IRU WKH IRUPDWLRQ RI 


PHWK\OKH[HQH
&RPSOHWH WKH PHFKDQLVP E\ GUDZLQJ DQ\ PLVVLQJ ORQH SDLU SDUWLDO
FKDUJHVDQGFXUO\DUURZV 

  6WHS 
%U&+&+

&+&+&&+&+&+&&+
&+&+%U 

&+&+ 

  
6WHS

&+&++&+&+&+&+


&&&+&+ 2&+&+&&&+&+2+

&++&++
>@

  LL  )ROORZLQJ WKH VDPH PHFKDQLVP WZR RWKHU VWUXFWXUDO LVRPHUV RI 


PHWK\OKH[HQH ZHUH IRUPHG 7KLV H[SODLQV ZK\ WKH \LHOG RI
PHWK\OKH[HQHLVSRRU
*LYHWKHVWUXFWXUDOIRUPXODHRIWKHVHWZRVWUXFWXUDOLVRPHUV 

    

>@

  LLL  $Q XQLQWHQGHG RUJDQLF SURGXFW FRPSRXQG ) PD\ EH IRUPHG IURP D 
GLIIHUHQWPHFKDQLVP
6XJJHVW WKH VWUXFWXUDO IRUPXOD RI FRPSRXQG ) DQG VWDWH WKH W\SH RI
UHDFWLRQLQYROYHG 

     

  
trendyline
trendy
y
\SH RI UHDFWLRQ
)

7\SHRIUHDFWLRQ
>@



-XURQJ-XQLRU&ROOHJH -35(/,0,1$5<(;$0,1$7,21  
557 trendyline

 

 F  LY  $QRWKHU EURPLQDWHG FRPSRXQGXQGHUJRHV D VLPLODU UHDFWLRQ WR JLYH For


Examiners
FRPSRXQG* Use

&+%U
+&&&+ &+%U
&+
*
:KHQ FRPSRXQG * LV KHDWHG ZLWK FRQFHQWUDWHG 1+ FRPSRXQG + ZLWK
PROHFXODUIRUPXOD&+%U1LVIRUPHG
*LYHWKHVWUXFWXUDOIRUPXODRIFRPSRXQG+DQGH[SODLQZK\RQO\RQHRI
WKHEURPLQHDWRPVLQFRPSRXQG*UHDFWVZLWK1+ 

      
  



+ 

   







 >@

  >7RWDO@ 

 

trendyline
-XURQJ-XQLRU&ROOHJH -35(/,0,1$5<(;$0,1$7,21 >7XUQRYHU 
558 trendyline

 

 D  (SLQHSKULQHDOVRNQRZQDVDGUHQDOLQHLVSULPDULO\DPHGLFDWLRQDQGKRUPRQH For
Examiners
Use
7KH IROORZLQJ V\QWKHVLV RI HSLQHSKULQH ZDV ILUVW SHUIRUPHG E\ )UHLGULFK 6WRO] LQ


  L  6WHSPXVWEHFDUULHGRXWXQGHUGU\FRQGLWLRQV([SODLQZK\LWLVVR  

   


 >@

  LL  6XJJHVWWKHUHDJHQWVDQGFRQGLWLRQVXVHGLQ6WHS  

   
 >@

  LLL  6WDWHWKHW\SHRIUHDFWLRQLQYROYHGLQ6WHS  

   
 >@

 

trendyline
-XURQJ-XQLRU&ROOHJH -35(/,0,1$5<(;$0,1$7,21  
559 trendyline

 

 E  &RUGDEULQHDQDVDOFRQJHVWDQWGUXJLVDQLVRPHURIHSLQHSKULQH For
Examiners
Use


FRUGDEULQH
,V HSLQHSKULQH H[SHFWHG WR EH D VWURQJHU RU ZHDNHU EDVH WKDQ FRUGDEULQH"
([SODLQ\RXUDQVZHU 

  











 >@

 F  (SLQHSKULQH LV DQ LPSRUWDQW QHXURWUDQVPLWWHU WKDW ZDV FRQYHUWHG IURP 


SKHQ\ODODQLQH IRXQG LQ DVSDUWDPH DQ DUWLILFLDO VZHHWHQHU $Q DVSDUWDPH
RYHUGRVH FDQ FDXVH SUREOHPV LQ WKH SK\VLRORJ\ RI WKH EUDLQ DQG SV\FKLDWULF
GLVRUGHUV

DVSDUWDPH

'UDZ WKH VWUXFWXUDO IRUPXODH RI WKH SURGXFWV IRUPHG ZKHQ DVSDUWDPH LV
KHDWHGZLWK+&l DT  

   






trendyline





>@

   >7RWDO@ 
 

-XURQJ-XQLRU&ROOHJH -35(/,0,1$5<(;$0,1$7,21 >7XUQRYHU 


560 trendyline

 


%/$1.3$*(


-XURQJ-XQLRU&ROOHJH -35(/,0,1$5<(;$0,1$7,21  
561 trendyline

2016 H2 Chemistry Prelim Exam Paper 2 Suggested Mark Schemes

1 (a) CH 3 COCH 3 + I 2 CH 3 COCH 2 I + HI [1]

(b)
1. Using a 50.00 cm3burette, run **50.00 cm3 of the 5.0
moldm3propanone solution into a *250 cm3 [1]prepn of 1.0 mol
-
volumetric/standard flask. Make up to the graduated dm
3
propanonesoution
mark with deionised water. Stopper and shake the flask to
obtain a homogeneous solution.

*any appropriate capacity e.g. 100 cm3, 150 cm3


**calculated volume based on chosen capacity of standard
flask

2. Using a 50 cm3 measuring cylinder/burette, add 50.0 cm3


[1]prepn of
of the diluted propanone solution prepared in step 1 into a solution Y
200 cm3 beaker.
Using another 50 cm3 measuring cylinder/burette, add
50.0 cm3 of1.0 mol dm dilute sulfuric acid into the
same beaker. Stir well and label the solution Y.
[1]prepare X and Y
3. Using a 10 cm3 measuring cylinder/pipette/burette, add in separate boiling
10.0 cm3of solution Y into a boiling tube (labelled Y). tubes
Using another 10 cm3 measuring cylinder/pipette/burette,
add 10.0 cm3 of solution X to a second boiling tube
(labelled X). [1]place X and Y in
water bath set at a
particular temp.
4. Place both boiling tubes X and Y in the water bath
maintained at 15 C which is prepared by mixing tap water
and ice water. [1]Mixing and
Place a 0.2 oC interval thermometer inside tubeX record both t
and leave both tubes inside the water bath for some & temp. of mixture
timeto allow both solutions in the boiling tubes to equilibrate.

5. Then quickly add solution Y from boiling tube Y to boiling [1]state 4 other
tube X, starting the stopwatch simultaneously. Stir well temps
o
with the thermometer. Record the time taken for the < 50 C
reaction mixture to completely decolourise and and state how the
the temperature of the reaction mixture. temp.of both warm
& cool water-
bathscan be
6. Repeat the experiment (steps 3-5) at 4 other different achieved
temperatures, *20 oC, 25 oC, 30 oC and 35oC.
For temperature above room temp., prepare the water
bath by mixing tap water and hot water.
For temperature below room temp., prepare the water

ttrendyline
d
bath by mixing tap water and ice
ic water
water..
Use a thermometer to monitor the temperature of the wate
water-
water
bath.

*One of the chosen temp. should be below 25 oC.


e be

562 trendyline
2

(c) As propanone is flammable, do not use direct/naked flame


from Bunsen burner for heating. Use hot water to prepare the [1]
water bath.

1 (d) (i) 1

k [1]
t

(ii) 1 E 1
ln = ln A a
t R T
1 1
By plotting ln against graph, astraight line graph
t T
( [1]
with gradient of D is obtained.
5

(e) Since therate is independent of [I 2 ], [1]


when [I 2 ] is halved, t is halved. [1]

[Total: 12]

2 (a) (i) K sp = [Ag+]2 [CrO 4 2][1]units:mol3 dm9[1]

(ii) 1.1 x 1012 = 4x 3


x = 6.50 x 105mol dm-3 [1]
Solubility of Ag 2 CrO 4 in g dm3 = 6.50 x 105 332 = 0.0216 [1]

(iii) [Ag+]2 [CrO 4 2] [Ba2+][CrO 4 2]


2
8.0 10 5 2.0 10 4 8.0 10 5 2.0 10 4
= =
2 2 2 2
= 1.60 x 1013<K sp (Ag 2 CrO 4 )[1] =4.00 x 109>K sp (BaCrO 4 )[1]

yellow ppt. of BaCrO 4 is observed. [1]

(b) Precipitation of BaCrO 4 [1]reduces [CrO 4 2] and causes


the equilibrium position to shiftto the right. [1]
Therefore, [H+] increases and hence pHof the solution decreases. [1]

(c) E cell = 0.41 (0.76) = +0.35 V > 0

ttrendyline
trend
r d
Equation: Z + 2Cr3+ 2Cr2++ Zn2+
n: Zn [1]
Hence Zn reduces green Cr3+(aq) to blue Cr2+(aq)
(aq).. [1]

E cell = 0.00 (0.41) = +


00 (0.41) +0.
+0.41
41
1V>0 [1]
2+ + 3+
Equation: n: 2Cr + 2H 2Cr + H 2 [1]
Hence, on standing, the H+ present in the solution oxidises blue
Cr2+(aq) back to green Cr3+(aq) and H 2 gas is produced.
Jurong Junior College 9647/02/J2 PRELIMINARY EXAMINATION/2016
563 trendyline
3

[Total: 14]

trendyline
Jurong Junior College 9647/02/J2 PRELIMINARY EXAMINATION/2016 [Turn over
564 trendyline
4

3 (a) Due to smaller size of Li+, Li+ has higher charge density.
Li+ polarises the large O 2 2 anion to a large extent, hence it forms [1]
Li 2 O instead of Li 2 O 2 .

(b) (i) X:P 4 O 6 Y:SiO 2 Z:MgO [1]

(ii) SiO 2 has giant molecular structure. Due to the strong covalent
bond between Si and O atoms throughout the whole giant
[1]
structure, it does not hydrolyse in water.

(iii) P 4 O 6 + 6H 2 O 4H 3 PO 3 [1]
MgO+ H 2 O Mg(OH) 2 Accept [1]

(iv) SiO 2 + 2OHSiO 3 2+ H 2 O [1]


P 4 O 6 + 8OH4HPO 3 2 + 2H 2 O [1]

(c) (i) H sol (Ca(OH) 2 ) = (2506) + (1579) + 2(460)


=+7 kJ mol1 [1]

(ii)
Hr
Ca(s) + 2H 2 O(l) Ca(OH) 2 (aq) + H 2 (g)

2(286) (+7) [1]

987)
(
Ca(s) + 2H 2 (g) + O 2 (g) Ca(OH) 2 (s) + H 2 (g)

By Hess Law,
H r = 2(286) + (987) + (+7) = -408 kJ mol1 [1]
(e.c.f. from ans(c)(i))

[Total: 10]

Jurong Junior College 9647/02/J2 PRELIMINARY EXAMINATION/2016


565 trendyline
5

4 (a)
[NO] / moldm3
0.14

0.12

0.1

0.08

0.06

0.04
0.03
0.02
(t1/2)1 = 5 (t1/2)2 = 5
0
0 2 4 6 8 10 12 14 16
5

(i) t = 5 min
(t ) 1 = 5 min, (t ) 2 = 5 min [1]

Sincehalf-lives are constant, the reaction isfirst orderwith [1]


respect to NO.

(ii) Let rate = k [NO][F 2 ]a


1
Since rate ,
t [1]

D
  

  


D
 
a = 1
 

Hence, order of reaction with respect to F 2 is1. [1]

(iii) rate = k[NO][F 2 ] [1]

(iv) Step 1: NO + F 2 ONF + F slow step [1]


Step 2: F + NO ONF [1]

(b) (i)




O O
N O O
C


[1]
[1]

trendyline
yi

(ii) Thepermanent
hepermanent
he ermanent dipole-permanent
dipole-permanent dipole attraction between
dipole-
NO
O 2 molecules is stronger than
han the van der Waa
Waals forces [1]
between
etween CO 2 molecules.

[Total: 10]

Jurong Junior College 9647/02/J2 PRELIMINARY EXAMINATION/2016 [Turn over


566 trendyline
6

5 (a) (i) Tertiary alcohol [1]

(ii) H H
H H H H
H H C H H H C H
H C CCC H H C CCC H
H H Br H H H O H
H
[1] [1]
A B

(iii) Initiating: H
uv
Br Br 2Br( 1 )

Propagating step:
(2)

CH 3 CH 2 CH(CH 3 ) 2 + Br CH 3 CH 2 C(CH 3 ) 2 + HBr( 3 )

CH 3 CH 2 C(CH 3 ) 2 + Br 2 CH 3 CH 2 C(CH 3 ) 2 + Br( 4 )
Br

Termination ste:

Br+ BrBr 2

CH 3 CH 2 C(CH 3 ) 2 + Br CH 3 CH 2 C(CH 3 ) 2
( 5 ) Br
( 6 )

CH 3 CH 2 C(CH 3 ) 2 + CH 3 CH 2 C(CH 3 ) 2 CH 3 CH 2 C(CH 3 ) 2
CH 3 CH 2 C(CH 3 ) 2

6 (
) [3]
45 () [2]
23 () [1]

trendyline
Jurong Junior College 9647/02/J2 PRELIMINARY EXAMINATION/2016
567 trendyline
7

5 (b) (i) alkaline aqueous iodine


[1]
CH 3 C=O and CH 3 CH(OH) groups

2,4-dinitrophenylhydrazine

Ketones and aldehydes [1]

(ii) O CH 2 =CHCH 2 CH=C CH=CH 2


CH 3 C
CH 3
COOH
or
CH 2 =CHCH 2 CH=C C=CH 2
E [1] H CH 3
[1]
D

(c) (i)

Step 1:
-
Br CH 3 CH 2
+
CH 3 CH 2 C CH 2 CH 2 CH 3 C + 2 CH 2 CH 3 + :Br
CH [1]
CH 3 CH 3

Step 2:

CH 3 CH 2 H CH 3 CH 2 CH 2 CH 3

+C CCH 2 CH 3 +
OCH 2 CH 3 C C + CH 3 CH 2 OH [1]
CH 3 H CH 3 H

(ii)
CH 2 =C CH 2 CH 2 CH 3 CH 3 CH=C CH 2 CH 2 CH 3
CH 2 CH 3 CH 3
[1] [1]

(iii) OCH 2 CH 3
CH 3 CH 2 C CH 2 CH 2 CH 3

trendyline
trendy
y
CH 3
[1
[1]
F

Type off reaction:


ti Nucleophilic
ucleophilic substitution [1
[1]

Jurong Junior College 9647/02/J2 PRELIMINARY EXAMINATION/2016 [Turn over


568 trendyline
8

5 (c) (iv) CH 2 NH 2
H 3 C C CH=CHBr
CH 3
H [1]

The p orbital of Br atom overlaps with the p orbital of carbon


atom of C=C double bond, forming partial double bond [1]
character in the CBr bond, thereby strengthening the CBr
bond.

[Total: 18]

6 (a) (i) POCl 3 and the acyl chloride group inchloroacetyl chloride can [1]
undergo hydrolysisif water is present.

(ii) Excess CH 3 NH 2 in ethanol, heat in sealed tube [1]

(iii) Reduction [1]

(b) Twoelectrondonating alkyl groups bonded to N atom in


epinephrine, as compared to only one alkyl group bonded to N atom in [1]
cordabrine, make the lone pair on N in epinephrine more available for
protonation.
So epinephrine is expected to be the stronger base. [1]

(c) O HO O

OH CH3OH
+
NH3 H3N
+
HO O

[1] for each correct hydrolysed product

[Total: 8]

trendyline
Jurong Junior College 9647/02/J2 PRELIMINARY EXAMINATION/2016
569 trendyline
JURONG JUNIOR COLLEGE
2016 JC 2PRELIMINARY EXAMINATION
Higher 2

CANDIDATE
NAME

CLASS 16S

CHEMISTRY 9647/03
Paper 3Free Response 14 September 2016
2 hours

Candidates answer on separate paper.


Additional Materials: Answer Paper
Data Booklet

READ THESE INSTRUCTIONS FIRST

Write your name, class and exam index number on all the work you hand in.
Write in dark blue or black pen.
You may use anHB pencil for any diagrams or graphs.
Do not use staples, paper clips, highlighters, glue or correction fluid.

Answer any four questions.


A Data Booklet is provided.
The use of an approved scientific calculator is expected, where appropriate.
You are reminded of the need for good English and clear presentation in your answers.

At the end of the examination, fasten all your work securely together.
The number of marks is given in brackets [ ] at the end of each question or part question.

trendyline
This document consists of 9 printed pages and 1blank page.

570 [Turn over


trendyline
2

1 Methane, the main component of natural gas, is converted into a combustible mixture of
gases by reaction with steam in the presence of a heated nickel catalyst.
CO(g) + 3H 2 (g)
reaction 1: CH 4 (g) + H 2 O(g)

(a) (i) The total bond energy in the carbon monoxide molecule is 1046 kJ mol1.
Suggest why this value is higher than either of the bond energies between
carbon and oxygen found in the Data Booklet. [1]

(ii) Using the given bond energy value for the carbon monoxide molecule and
other relevant data from the Data Booklet, calculate the enthalpy change of
reaction for reaction 1. [2]

(b) The standard Gibbs free energy change of a reaction can be calculated based on
its standard enthalpy as well as its standard entropy change.

(i) The G for the forward reaction in reaction 1 is +142 kJ mol1.


Using the given value for G and your answer in (a)(ii), calculateS for the
forward reaction in reaction 1 at 25 C. [2]

(ii) Calculate the temperature at which the reaction just becomes feasible. [1]

(c) When a 2:1 mole ratio of methane and steam at aninitialtotal pressure of 5atm is
passed over the catalyst at 450 C, the partial pressure of hydrogen at equilibrium
is found to be 1.8 atm.

(i) Calculate the partial pressures of methane, steam and carbon monoxide at
equilibrium. [1]

(ii) Write an expression for the equilibrium constant, K p , and calculate its value,
giving its units. [2]

(iii) Calculate the percentage yield of hydrogen gas at equilibrium under the
above given conditions. [1]

(iv) Suggest what conditions of temperature and pressure would increase the
yield of hydrogen gas. Explain your answer in each case. [3]

(d) The hydrogen gas from reaction 1 can be harnessed and used as a fuel in fuel
cells. Oxygen from air is pumped into the fuel cell at the cathode. An alkali such
as warm aqueous potassium hydroxide can be used as an electrolyte.

(i) Write the halfequations of the reactions which would take place at the
anode and the cathode of such a fuel cell. [2]

(ii) Using relevant E values from the Data Booklet, calculate the standard cell
potential of the above fuel cell. [1]

(iii) One refinement of the fuel cell design is to replace the alkaline electrolyte
solution with a film of solid OH ion-conducting polymer.
riefly
iefly explain why this would be an improvement.
Briefly improvem [1]

Jurong Junior College 9647/03/J2 PREIMINARY EXAMINATION/2016


571 trendyline
3
1 (e) The carbon monoxide attained from reaction 1is also useful for the production of
aromatic aldehydes from various aromatic compounds, including derivatives of
benzene.

One such example is the GattermannKoch reaction, in which carbon monoxide,


hydrochloric acid, and aluminium chloride are used to react with benzene to
produce benzaldehyde.

CO + HCl + AlCl3
O

Gattermann-Koch reaction

Using the Gattermann-Koch reaction as the first step of the synthesis, show how
compound Gcan be formed from methylbenzene in a three-step synthesis.
You should suggest the reagents and conditions to be used for the second and
third steps and identify the intermediates involved.

CH3 CH3

HO C COOH

CompoundG [3]

[Total: 20]

trendyline
Jurong Junior College 9647/03/J2 PREIMINARY EXAMINATION/2016 [Turn over
572 trendyline
4

2 (a) By passing dry chlorine over aluminium metal, aluminium chloride, AlCl 3 ,
a yellow-white solid that sublimes at low temperatures, can be prepared.

(i) Other than the formation of the yellow-white solid, describe another
observation that you can make between the reaction of aluminium and
chlorine. [1]

(ii) Describe and explain what you would see when a sample of solid AlCl 3 is
added to a solution of litmus in water.
Include relevantequation(s) in your answer. [3]

(iii) When NaOH(aq) is added dropwise to the resulting solution formed in


(a)(ii), a white precipitate forms, which is soluble on adding excess
NaOH(aq).
With the aid of suitable equations with state symbols, explain these
observations. [3]

(b) AlCl 3 is used to catalyse electrophilic substitution reactions such as the


chlorination of benzene.
Chlorination of benzene takes place in several steps.
The first step is the reaction between Cl 2 and AlCl 3 .
Cl 2 + AlCl 3 Cl+ + AlCl 4
The benzene ring is then attacked by the Cl+ cation in the second step.

Alkylation of benzene can also be catalysed by AlCl 3 in a similar way.


When benzene and propene react together in the presence of AlCl 3 and HCl,
cumene is formed in a high yield.

CH3
AlCl3
+ CH3CH=CH2 C CH3
HCl
propene H

cumene

(i) By considering the structure of cumene, write a balanced equation showing


the formation of the carbocation obtained from the reaction of CH 3 CH=CH 2 ,
AlCl 3 and HCl. [1]

(ii) Hence, using the above information provided, describe the mechanism for
the reaction between the carbocation and benzene to give
cumene. In your answer, show relevant charges and use curly arrows to
show the movements of electron pairs. [2]

(iii) In the above reaction between benzene and propene, propylbenzene is


obtained in small quantities. Suggest a reason for this.

trendyline
nd
dy
y CH2CH
C 2CH3

propylbenzene
ropy [1]

Jurong Junior College 9647/03/J2 PREIMINARY EXAMINATION/2016


573 trendyline
5

2 (b) (iv) When cumene is treated with a small quantity of bromine in the presence of
ultraviolet light, only two monobrominated compounds can be formed.
Suggest the structure of an isomer of cumene that will also give only two
monobrominated products under the same reaction conditions.
Assume no bromination takes place on the benzene ring. [1]

(v) Describe a simple chemical test to distinguish the isomer drawn in (b)(iv)
from cumene. [2]

(c) Phosphorus and sulfur are in the same Period as aluminium. In addition to
forming halides like aluminium, both phosphorus and sulfur form oxyhalides that
are commonly used as chlorinating agents in organic syntheses. These include
POCl 3 and SOCl 2 .

(i) Draw the shape for the SOCl 2 molecule. [1]

(ii) POCl 3 boils at 106 C while SOCl 2 boils at 75 C.


Account for the difference in boiling points between the two oxyhalides. [2]

(d) Phosphoric(V) acid, H 3 PO 4 , is one of the most well-known triprotic acids.


Apart from phosphoric(V) acid, there exists other phosphorus-containing acids
that dissociate to give more than three protons per molecule. One such acid is W,
which can be formed when H 3 PO 4 reacts with POCl 3 . HCl gasis also produced
during the reaction.
W has the following composition by mass: P, 34.8%; O: 62.9%; H, 2.3%.

(i) Calculate the empirical formula of W. [1]

(ii) Given that the empirical formula of W is the same as its molecular formula
and that the constituent atoms in W show their usual valencies, suggest the
structure of molecule W. [1]

(iii) Write an equation for the reaction between H 3 PO 4 and POCl 3 . [1]

[Total: 20]

trendyline
Jurong Junior College 9647/03/J2 PREIMINARY EXAMINATION/2016 [Turn over
574 trendyline
6

3 Thermogravimetry is an analytical method used to determine the composition of solids


which decompose on heating. The change in mass which is measured during the
heating process provides information about the composition of the mixture analysed.
A mixture of calcium oxalate, CaC 2 O 4 , and magnesium oxalate, MgC 2 O 4 , was heated
up to 900 C. During this process, the mass of the mixture was measured continuously.
It is known that the following two decomposition reactions take place at around 400 C.
MgC 2 O 4 (s) MgO(s) + CO(g) + CO 2 (g)
CaC 2 O 4 (s) CaCO 3 (s) + CO(g)
At 700 C, a third decomposition with the evolution of an acidic gas is observed.

(a) (i) Write an equation to illustrate the third decomposition reaction. [1]

(ii) At 500 C, the mass of the mixture obtained was 3.06 g.


At 900 C, the mass of the mixture obtained was 2.03 g.
moles of MgC2O4
Calculate the ratio prior to heating.
moles of CaC2O 4 [3]

(b) Other than bonding with Group II metal ions, the oxalate ion, C 2 O 4 2, is known to
act as a bidentate ligand to transition metal ions to form complex ions.
An example is the light green Fe(C 2 O 4 ) 3 3 complex ion.
The structure of the oxalate ion is as follows.
O O

O O
(i) What do you understand by the term, ligand? [1]

(ii) State the oxidation number of iron in Fe(C 2 O 4 ) 3 3. [1]

(iii) Draw the structure of the Fe(C 2 O 4 ) 3 3complex ion, showing the geometry
around Fe. [2]

(iv) Briefly explain how the light green colour of Fe(C 2 O 4 ) 3 3 arises. [3]

(v) Explain why iron forms complex ions while calcium does not. [1]

(c) Compound K has the molecular formula, C 4 H 7 NO 2 . When compound K was


warmed with aqueous sodium hydroxide, a colourless pungent gas was evolved.
Acidification of the resulting solution gives compound L, C 4 H 6 O 3 .
L gives an orange precipitate with 2,4-dinitrophenylhydrazine but has no reaction
with Fehlingssolution. Effervescence was observed when L was treated with
aqueous sodium hydrogencarbonate. When lithium aluminium hydride was added
to L, compound M, C 4 H 10 O 2 , was produced.
M can also be obtained from the reaction of coldacidified potassium
ate(VII) with compound N, C 4 H 8 .
manganate(
na
Deducee the structures of compounds K,
K, L,, M and
nd N and explain the reactions
ed
ed.
described.
[8]

[Total: 20]

Jurong Junior College 9647/03/J2 PREIMINARY EXAMINATION/2016


575 trendyline
7

4 This question concerns chemical reactions of halogens, halides and the organic
derivatives of halogens.

(a) Chlorine-containing bleaches are usually made by reacting chlorine gas with cold
aqueous sodium hydroxide.
State the type of reaction and the products formed in this reaction.
Write an equation to illustrate what happens when chlorine gas reacts with hot
aqueous sodium hydroxide instead. [3]

(b) Three unmarked jars contain HCl, HBr and HI respectively.


Describe how plunging a red hot glass rod into each of the jars in turn can help to
identify the gaseous hydrogen halide present.
Explain your answer in terms of relevant bond energies. Where relevant, include
equation(s) to support your answer. [3]

(c) Both halogenoalkanes and hydrogen halides undergo hydrolysis under


specific reagents and conditions.

(i) Suggest reasons why reaction I must be heated for some time for it to
occur, whereas reaction II takes place almost instantaneously at room
temperature.

ICH 3 CHBrCH 2 CH 3 (l) + NaOH(aq) CH 3 CH(OH)CH 2 CH 3 (aq) + NaBr(aq)

IIHBr(aq) + NaOH(aq) H 2 O(l) + NaBr(aq) [2]

(ii) How would the rate of the reaction between CH 3 CHClCH 2 CH 3 and NaOH
compare to that of reaction I? Explain your answer with the aid of the
Data Booklet. [2]

(iii) In reaction I,an inversion of the stereochemical configuration of the organic


product was observed.
Describe the mechanism for reaction I.Show relevant lone pairs and
dipoles, and use curly arrows to indicate the movement of electron pairs. [2]

(iv) Write the rate equation for reaction I.


With the help of your rate equation, estimate the effect on the rate of
reaction I of diluting the solution with an equal volume of solvent. Explain
your answer. [3]

(v) Rates of reactions can also be dependent on the temperature used.


Describe, and explain in molecular terms, how the rate of a reaction is
affected by a change in temperature. You should include a reference to the
Boltzmann distribution in your answer. [3]

(d) With the use of NaOH(aq) and another reagent, describe how you could
distinguish 1-bromopropane from 2-bromopropane. [2]

trendyline
[Total: 20]

Jurong Junior College 9647/03/J2 PREIMINARY EXAMINATION/2016 [Turn over


576 trendyline
8

5 Ovalbumin, OVA, is the predominant protein in egg white, making up approximately


54% of the total protein content. It is also a key reference protein used in protein
research, vaccination experiments and immunology.
Some of the predominant and significant amino acids found in OVA are listed in the
table below.

name abbreviation structural formula


H2N CH COOH
cysteine cys
CH2SH

H2N CH COOH
aspartic acid asp
CH2COOH

H2N CH COOH
leucine leu
CH2CH(CH3)2

H2N CH COOH
lysine lys
CH2CH2CH2CH2NH2

HN CH COOH

proline pro H2C CH2


CH2

H2N CH COOH
serine ser
CH2OH

H2N CH COOH
valine val
CH(CH3)2

(a) Research has shown that the C-terminal sequence of OVA is cys-val-ser-pro,
with proline being the last amino acid residue of the polypeptide chain.
Draw the structure of this C-terminal portion at pH 1. [2]

(b) When heat is applied, OVA undergoes denaturation. The structure of


OVA, when denatured by heat, exists mostly in the form of-pleated sheets.

(i) Define what a -pleated sheet is. Sketch a diagram to show how a
polypeptide chain is held in the shape of a -pleated sheet. [3]

(ii) Explain what is meant by the term denaturation.


Using suitable pairs of amino acids from the table above, draw labelled
diagrams to illustrate two different types of side-chain interactions that will
be affected by the heat-induced denaturation. [4]

trendyline
(iii) Aggregation
ggregation
gr of the -pleated sheets form formed during the heat-induced
denaturation
enaturation of OVA results in the formation of a white insoluble
insolubl solid.
From
om the given table, suggest
sugges three amino acidswhose side chains are on
the
e outside of the -pleated
pleated sheets such that the denatured OVA
OV is insoluble
O
in water
water. [1]

Jurong Junior College 9647/03/J2 PREIMINARY EXAMINATION/2016


577 trendyline
9

5 (c) OVA can be administered to treat suspected poisoning by heavy metal ions via
ingestion.
In this treatment, consumption of a small amount of OVA causes the heavy metal
ions in the gastrointestinal tract to be trapped by the protein, thus preventing the
ions from being absorbed by the body.
Suggest possible chemical interactionsbetween OVA and heavy metal ions in this
treatment. [2]

(d) Aspartic acid, which can be found in OVA, is a non-essential amino acid that can
be synthesised by the human body. It plays a vital role in the construction of
biochemicals in the citric acid cycle and assists the liver in removing excess
ammonia and other toxins from the bloodstream.
2 1
H2N CH COOH

CH2COOH
3

It is found that the length of the C 1 -C 2 bond is shorter than the C 2 -C 3 bond.
Suggest a reason for this difference. [1]

(e) There are three pK a values associated with aspartic acid: 2.1, 3.9 and 9.8.
Starting with the cationic form, aspartic acid undergoes the first two stages of acid
dissociation as shown below:
- +
- +
H3N CH COOH H H3N CH COO H H3N CH COO
. .
pK a1 = 2 1 pK a2 = 3 9
CH2COOH CH2COOH CH2COO

(i) Explain why the-COOH dissociates in preference to the


side-chain COOH in the first stage of dissociation. [2]

(ii) Ignoring the effect of pK a2 and pK a3 on the pH, calculate the pH of a


0.050 mol dm3 solution of aspartic acid. [1]

A solution of aspartic acid can resist pH changes when a small amount of acid or
base is added.

(iii) Draw the structures of the species responsible for this characteristic of the
solution at pH 9.8. [2]

(iv) With the aid of equations, show how this solution of aspartic acid can
maintain its pH at 9.8. [2]

[Total: 20]

trendyline
Jurong Junior College 9647/03/J2 PREIMINARY EXAMINATION/2016 [Turn over
578 trendyline
10

BLANK PAGE

trendyline
Jurong Junior College 9647/03/J2 PREIMINARY EXAMINATION/2016
579 trendyline
1

Suggested Answers
2016 H2 Chemistry Preliminary Examination Paper 3

1 (a) (i) The two bond energies of bonds between C and O in the Data Booklet are
E(C=O): +740 kJ mol1 and E(CO): +360 kJ mol1, which are double and
single bonds respectively. The bond between C and O in carbon
monoxide is however a triple bond which is stronger than both the
double and single carbon-oxygen bonds.[1m]

(ii) Energy due to bond formation


= E(CO) 3E(HH) = 1046 3(436) = 2354 kJ mol1
Energy due to bond breaking
= 4E(CH) + 2E(OH) = 4(410) + 2(460) = +2560 kJ mol1
Enthalpy change of reaction
=  BE(bonds broken) BE (bonds formed)
=2354 + 2560[1m]
= +206 kJ mol1[1m]

(b) (i) G = H TS; Since H = +206 kJ mol1, and G = +142 kJ mol1.


Hence at 298K, +142 = +206 (298)S[1m]
S = +0.215kJ mol1 K1 [1m]

(ii) When the reaction just becomes feasible, G= 0.


Assuming H and S do not change over a large temperature change,
H = TS , hence T = 958 K[1m] for correct ans + 3sf + units

(c) (i) CH 4 (g) + H 2 O (g) CO (g) + 3H 2 (g)


Initial pressure/ atm 10/3 5/3 00
Change in pressure/ atm 1.8/3=0.61.8/3=0.6 +1.8/3= 0.6+1.8
Eqm pressure/ atm3.330.6 = 2.731.670.6 = 1.070.6+1.8
[1m]

(ii)

3
PCO . PH2
Kp = [1m]
PCH4 .PH2O

K p = [0.6(1.8)3]/[2.73(1.07)]
= 1.20 atm2[1m]

trendyline
Jurong Junior College 9647/03/J2 PREIMINARY EXAMINATION/2016 [Turn over
580 trendyline
2

1 (c) (iii) Percentage yield of H 2 = (1.8/5) x 100 = 36% [1m]

(iv) The yield will be higher at a higher temperature.()


This is because higher temperature favours the endothermic reaction
asthe reaction that absorbs heat is favoured at higher temperature
and position of equilibrium shifts right. [1m]

The yield will be increased at a lower pressure. ()


This is because since the forward reaction produces more gaseous
molecules than the backward reaction, a lower pressure favours
the forward reaction that produces more gaseous molecules to
counteract the decrease in pressure so that position of equilibrium shifts
right, thus increasing the yield. [1m]
2(): [1m]
(d) (i) When an alkaline electrolyte (e.g. warm KOH) is used, the reactions are
as follows:
At the anode: H 2 (g) + 2OH(aq) 2H 2 O(l) + 2e [1m]ss not reqd
At the cathode: O 2 (g) + 2H 2 O(l) + 4e 4OH(aq) [1m]ss not reqd

(ii) E cell = (+0.40) ( 0.83) = +1.23V [1m]

(iii) Eradicate/Prevent/Avoid/Remove the possibility of alkali leakage from


the fuel cell. [1m]

(e)

CH3 CH3 CH3 CH3


HCN,
trace amount of
CO + HCl + AlCl3 NaCN / NaOH H2SO4(aq) or HCl (aq)

Gattermann-Koch heat under reflux


reaction

C H C OH
[1m] H C OH
H O
CN COOH

[1m] [1m]

2 (a) (i) Pale greenish-yellowcolour of Cl 2 gas fades


OR
Aluminium glows/ burns.[1m]

(ii) AlCl 3 dissolves in water to give[Al(H 2 O) 6 ]3+, which hydrolyses (to a large
extent) in water to give H+ ions / to give a weaklyacidic solution. [1m]

trendyline
ndy
dylin
nce, the solution turns red
enc
Hence, /pinkish
in -red
d.[1m]
[1m
red/pinkish-red.[1m]

AlCl
Cl 3 (s) + 6H 2 O( l)
O(l)
O(l [A H 2 O) 6 ]3+
[Al(H
Al(H 3
((aq) 3Cl(aq)
aq) + 3Cl (aq)
[1m]
l(H 2 O) 6 ]3+ (aq) [Al
[Al(H [Al(H )]2+(aq) + H+(aq)
A (H 2 O) 5 (OH)]

Jurong Junior College 9647/03/J2 PREIMINARY EXAMINATION/2016


581 trendyline
3

2 (a) (iii) On addition of a small quantity of NaOH(aq),[Al(H 2 O) 6 ]3+/ Al3+(aq) forms the
white Al(OH) 3 precipitate.
On addition ofexcess NaOH(aq), Al(OH) 3 /white precipitate forms a
soluble, colourless complex Al(OH) 4 . [1m]
OR
As Al(OH) 3 is an amphoteric hydroxide, so it can further react with
NaOH(aq).

[Al(H 2 O) 6 ]3+(aq) + 3OH(aq)


Al(OH) 3 (s) + 6H 2 O(l)[1m]ss are reqd
[Note: Accept Al3+(aq) + 3OH(aq) Al(OH) 3 (s)]
Al(OH) 3 (s) + OH(aq) [Al(OH) 4 ](aq)[1m]ss are reqd
H
(b) (i)
CH 3 CH=CH 2 + AlCl 3 + HCl CH3C CH2 + AlCl4 [1m]
+
H
(ii) H H
CH3
+ CH3C CH2 C
+
H
+ H
CH3

H
CH3 CH3
C + AlCl4 CH3
C
+ H
CH3

+ AlCl3 + HCl
[2m]
+ +
(iii) CH 3 CH 2 CH 2 carbocation is less stable than CH 3 CHCH 3
[1m]
asit has one less electron-donating alkyl group.

+
Hence, the positive charge on CH 3 CH 2 CH 2 is less dispersed and this
carbocation is formed in a smaller quantity.

(iv)
CH3

CH3

tr
trendyline
[1m]
C 3
CH

Jurong Junior College 9647/03/J2 PREIMINARY EXAMINATION/2016 [Turn over


582 trendyline
4

5 (c) (v) Test: Add acidified KMnO 4 to each compound separately and heat. Pass
any gas evolved through limewater. [1m]
Observations: Both cumene and the isomer in (iv) will decolourise purple
KMnO 4 but only the cumene will give CO 2 gas which gives a white ppt with
limewater. [1m]

(i)

S
[1m]
Cl O
Cl
(ii) POCl 3 has more electrons per molecule than SOCl 2 . [1m]
As a result, more energy is required to overcome the stronger van der
Waals forces / induced dipole-induced dipole attractions between POCl 3
molecules than the weaker van der Waals forces / induced dipole-
induced dipole attractions (and permanent dipole-permanent dipole
attractions) between SOCl 2 molecules.[1m]

(d) (i) P O H

% by mass 34.8 62.9 2.3

Amount/mol 1.12 3.93 2.3

Ratio 1 3.5 2

2 7 4

The empirical formula of W is P 2 O 7 H 4 . [1m]

(ii) O O

P P
O OH
OH [1m]
OH
OH
(iii) 5H 3 PO 4 + POCl 3 3P 2 O 7 H 4 + 3HCl[1m]

trendyline
Jurong Junior College 9647/03/J2 PREIMINARY EXAMINATION/2016
583 trendyline
5

3 (a) (i) CaCO 3 CaO + CO 2 [1m]

(ii) At 500 C, the solids present are MgO and CaCO 3 .


At 900 C, the solids present are MgO and CaO.
So, mass of CO 2 = 3.06 2.03 = 1.03 g
Since CaC 2 O 4 CaCO 3 CO 2 ,
1.03
Amount of CaC 2 O 4 = = 0.0234 mol[1m]
44.0

Mass of MgO = 3.06 mass of CaCO 3


= 3.06 (0.0234 100.1)
[1m]
= 0.718 g (or 0.717)
ORMass of MgO = 2.03 (0.0234 56.1) = 0.717 g
0.718
Hence, amount of MgC 2 O 4 = amount of MgO = =0.0178 mol
40.3
0.0178
So, the ratio is = 0.761(or 0.760) [1m]
0.0234

(b) (i) A ligand is an anion or molecule which has at least one lone pair of
electrons to form dative / co-ordinate bonds with metal ion or atom.[1m]

(ii) +3 [1m]

(iii) O 3
O

O

O
O O
Fe

O O O

O O
[2m]
O

(iv) The colour of the complex arises due to electron transition between the d
orbitals.The presence ofligands causes the 3d orbitals in Fe3+to split into
two different energy levels with relatively small energy difference.
Radiation from thevisible lightregion of the electromagnetic spectrum is
absorbedwhen an electron transits from a d-orbital of lower energy to
anunfilled / partially-filled d-orbital of higher energy.
The colour seen is the complement of the colour absorbed. [3m]

(v)

ttrendyline
trend
Fehas

rendyline
ren
ehas energetically accessible vacantorbitals that can accept
off electrons from ligandsto
onds,
nds, unlike calcium
bonds,
ligand
ligandsto
calciu . [1m]
calcium.
a
ac
o form dative covalent bond
lone pair
bonds / coordinate

Jurong Junior College 9647/03/J2 PREIMINARY EXAMINATION/2016 [Turn over


584 trendyline
6

3 (c) K undergoesalkaline hydrolysis with NaOH(aq) on warming to give anion of L


and NH 3 gas.
K is a primary amide.
L is carboxylic acid.
L undergoes condensation reaction with 2,4-DNPH to give positive test.
L is a carbonyl compound/ is either a ketone or an aldehyde
L has no reaction with Fehlings solution
L is not an aldehyde/L is a ketone
L undergoes acid-hydrogencarbonate reaction with NaHCO 3 (aq)
L is carboxylic acid.
L undergoes reduction reaction with LiAlH 4 .
M is a primary alcoholandsecondary alcohol.
N undergoes mild oxidation to give M.
M is a diol.
N is an alkene.

Explanation: [4m]

O O O O H H

CH3CH2C C NH2 CH3CH2C C OH CH3CH2C C H CH3CH2CH=CH2

K L N
OH OH
M
[1m] each
4 (a) Type of reaction: Disproportionation [1m]
Products formed: Cl(aq), ClO(aq) [or NaCl (aq), NaClO(aq)] [1m]
3Cl 2 (g) + 6OH(aq) 5Cl(aq) + ClO 3 (aq) + 3H 2 O(l) [1m]
[OR3Cl 2 (g) + 6NaOH(aq) 5NaCl (aq) + NaClO 3 (aq) + 3H 2 O(l)]
(ss not reqd)

(b) Bond energy and bond strengthof HX decreases down the Group. [1m]
Hence, down the Group, HX becomes less thermally stable.
Thermal decomposition of HX involves the reaction (for X = I and Br)
[1m]
2HX(g) H 2 (g) + X 2

With HI, copious violet fumes of I 2 are seen.


With HBr, somered-brown / orange fumes of Br 2 are seen. [1m]
With HCl, no fumes observed (or no observation).

Jurong Junior College 9647/03/J2 PREIMINARY EXAMINATION/2016


585 trendyline
7

4 (c) (i) Energy must be supplied to break the covalent bond between C & Br,
hence reactionI has high E a [1m] and must be heated.
In aqueous state, HBr andNaOHaredissociated into ionsORexist as ions)
as they are strong acid and strong base. Net reaction is the formation of bonds
between H+ and OH ions to give water. Since neutralisation
involves attraction ofoppositelycharged ionsH+ and OHwhichrequires low E a ,
rate of reaction II is faster. [1m]

(ii) Rate of reaction will be slower [1m].


The carbon-halogen bond is broken in the rate-determining step and
since CCl bond (340 kJ mol1) is stronger than CBr bond (280 kJ
mol1), more energy will be required to break the CCl bond and hence the
rate will be slower [1m].

(iii) Since the stereochemical configuration of the product is reversed, the reaction
follows the S N 2mechanism.

CH3 CH3 H CH3


H H
:OH
C Br HO C Br HO C + Br

CH2CH3 CH2CH3 CH2CH3

[2m]

(iv) Rate = k[CH 3 CHBrCH 2 CH 3 ][NaOH][1m]

Rate of reactiondecreases by 4 times[1m] as concentration of each


reactant is halved [1m].
OR
Diluting the solution with equal volume of solvent will lower the
concentrations of both reagents by half [1m] and hence, the rate will be
decreased by four times [1m].

trendyline
Jurong Junior College 9647/03/J2 PREIMINARY EXAMINATION/2016 [Turn over
586 trendyline
8

4 (c) (v) As the temperature is increased, more reactant particles / molecules


have(kinetic)energies greater than or equal to the activation energy[1m]
Hence thefrequency of effectivecollision between reactant particles/
moleculesincreases and reaction rate increases.[1m]
This is evident from the Boltzmann distribution curve of kinetic energies as
shown below:

Number of particles /Fraction of particles

No. of particles with energy Ea at T1


T1 No. of particles with energy Ea at T2

T2 > T1
T2 Ea: Activation energy

[1m]

0 Ea Energy

Effect of temperature on Boltzmann distribution of molecular energies

(d) Add NaOH(aq) (or alkaline), I 2 (aq), then heat.


bromopropanebut no precipitate formed for
Yellow precipitate formed for 2
1bromopropane. [1m]

5 (a) H H O H O H O H

N C C N C C N C C N C COOH

CH2SH H CH(CH3)2 H CH2OH


[2m]

(b) (i) A pleated sheet consists of adjacent polypeptide strands stabilised


by hydrogen bonds between the backbone C=O group of one strand and the
backbone NH group of the adjacent strand.[1m]

adjacent
C
strand

O

hydrogen
H
bonds

N C a strand of

trendyline
re
e
polypeptide
chain
O

H


N adjacent
strand
[2m]

Jurong Junior College 9647/03/J2 PREIMINARY EXAMINATION/2016


587 trendyline
9

5 (b) (ii) Denaturation involves the disruption of R-group interactions


/ side-chain interactions of a protein, [1m]
leading to possible destruction of the secondary, tertiary and quaternary
structures of the protein.
OR
which in turnalters the conformation/shape of the protein and causes the
protein to lose its ability to perform its specific function. [1m]

Category Diagram illustrating R group interaction (Any one of the suggested pairs)

hydrogen
bond
[1m]

van der
Waals
forces
[1m]

(b) (iii) Leucine, valine, proline, cysteine [1m]


(Any 3 of the 4 stated amino acid residues)

5 (c) Heavy metal ionshave a high affinity for sulfur and will bind tightly(to SH group
/ sulfur) group(of cysteineresidue / in OVA). [1m]
Heavy metal ionsform ionic interactions with COO. [1m]
These interactions between OVA and heavy metal ions may lead to precipitation
and prevent the metal ions from being absorbed by the body.

(d) The C 1 -C 2 bond is a sp2-sp3 overlap, whereas the C 2 -C 3 bond is a sp3-sp3


overlap.
OR
The C 1 -C 2 bond is formed by the overlap of the sp2 hybrid orbital of C 1 and
the sp3 hybrid orbital of C 2 . In contrast, the C 2 -C 3 is formed by the overlap of
the sp3 hybrid orbital of C 2 and the sp3 hybrid orbital of C 3 . [1m]

The sp2 orbital of C 1 contains greater s-character

trendyline
trendylin
n
ne
whereelectrons
lectrons nucleus, the bonding e
ectrons can penetrate closer to the nucleus electrons are
more strongly attracted to the nucleus of the C 1 atom atom.. He
Hence
H the bond
formed by the sp2-sp3 overlap is shorter.

Jurong Junior College 9647/03/J2 PREIMINARY EXAMINATION/2016 [Turn over


588 trendyline
10

5 (e) (i) The pK a YDOXHRIWKH-COOH is lower, hence the -COOH is a stronger


acid than side-chain COOH [1m]
because theelectron-withdrawing N / NH 2 / NH 3 + LV FORVHU WR WKH -
COOH group.
The negative charge RI -COO anion is more dispersed DQG WKH - [1m]
COOanion is more stable/stabilised to a larger extent.

(ii)
[H+] = K a1 0.050 = 10 2.1 0.050 = 0.0199 moldm3
Hence, pH = lg(0.0199) = 1.70 [1m]

(iii) The two species present in the buffer solution at pH 9.8:

[2m]

(iv)

[1m]

[1m]

trendyline
Jurong Junior College 9647/03/J2 PREIMINARY EXAMINATION/2016
589 trendyline
Class Adm No

Candidate Name:

2016 Preliminary Examination II


Pre-University 3

H2 CHEMISTRY 9647/01
Paper 1 Multiple Choice 21st Sept 2016
1 hour
Additional materials: Multiple Choice Answer Sheet
Data Booklet

READ THESE INSTRUCTIONS FIRST

Do not turn over this question paper until you are told to do so
Write in soft pencil.
Do not use staples, paper clips, glue or correction fluid.
Write your name, class and admission number in the spaces provided at the top of this page
and on the Multiple Choice Answer Sheet provided.

There are forty questions on this paper. Answer ALL questions. For each question there are
four possible answers A, B, C and D.
Choose the one you consider correct and record your choice in soft pencil on the Multiple
Choice Answer Sheet provided.

Read the instructions on the Multiple Choice Answer Sheet very carefully.

Each correct answer will score one mark. A mark will not be deducted for a wrong answer.
Any rough working should be done in this question paper.

trendyline
yline
li
The use of an approved
proved scientific calculator is expected, where
pproved whe appropriate.

FOR EXAMINERS
EXAMINER USE
TOTAL
OTAL (4
((40
40 marks)

This question paper consists of 23 printed pages and 1 blank page.


590 trendyline
2

Section A
For each question there are four possible answers, A, B, C, and D. Choose the one you consider to
be correct.

1 Gaseous hydrochloric acid was bubbled through a solution of sodium carbonate until the
solution turned blue litmus paper red.

How will the total number of ions present in solution in the reaction mixture vary?

A B

number number
of ions of ions

0 0
amount of amount of
acid added acid added

C D

number number
of ions of ions

0 0
amount of amount of
acid added acid added

2 In winter, ice forms only on the surface of lakes and rivers, allowing aquatic life to survive
below the surface.

Which of the following statements about ice does not explain this phenomenon?

A Ice has an open structure.


B The water molecules in ice are in an orderly tetrahedral arrangement.
C Water molecules can form intermolecular hydrogen bonding in ice.

trendyline
D ec
ecules
Water molecules have a much lower energy
ergy in ice than in water.

591 trendyline
3

3 An ion SF 3 n is T-shaped as shown below.

n
F
F S
F

What is the value of n?

A 2 B 1 C +1 D +2

4 What is the new pressure of a sample of gas, initially at 1 atm, when it is heated from 25 C to
75 C at constant volume?

A 0.3 atm B 0.9 atm C 1.2 atm D 3.0 atm

5 Diboron tetrachloride, B 2 Cl 4 , is a colourless liquid at room temperature. The combination of


gaseous atoms of boron and chlorine to form B 2 Cl 4 (g) is represented by the equation below.

2B(g) + 4Cl J % 2 Cl 4 (g + 2127 kJ mol-1

The bond energy of a BB bond is +303 kJ mol-1.

What is the bond dissociation energy of a BCl bond?

A 608 kJ mol-1 B 456 kJ mol-1 C +456 kJ mol-1 D +608 kJ mol-1

trendyline
592 trendyline
[Turn over
4

6 Use of the Data Booklet is relevant to this question.

A student carried out an experiment to determine the heat capacity of a copper can using the
setup below.

copper can water

spirit burner
methanol (Mr = 32.0)

The following data was recorded by the student:

Initial temperature of water / C 25 C


Final temperature of water / C 42 C
Mass of spirit burner before burning / g 220.00 g
Mass of spirit burner after burning / g 219.50 g
3
Volume of water / cm 150 cm3

Given that the enthalpy change of combustion of methanol is 715 kJ mol-1, determine the
heat capacity of the copper can.

A 30.2 J K-1 B 43.8 J K-1 C 626 J K-1 D 655 J K-1

7 The following reaction is thermodynamically feasible only at a high temperature.

CaCO 3 V &D2 V &2 2 (g)

What are the VLJQVRIH DQGS for the reaction?

H S

trendyline
A
B +
C +
D + +

593 trendyline
5

8 The mechanism involved in the formation of compound Z is as follows:

Step I: W + X Y (fast)
Step II: W + Y Z (slow)

What is the rate equation for this reaction?

A Rate = k[W][X]
B Rate = k[W][Y]
C Rate = k[W][Y]2
D Rate = k[W]2[X]

9 Alkenes in an acidic medium can isomerise in the presence of Al 2 O 3 catalyst.

Ealkene Zalkene

The energy profile diagram of the reaction is shown below.

Which of the following statements is correct?

A (E1 E2) corresponds to the enthalpy change of the forward reaction.

trendyline
B Leaving pure Ealkene to isomerise for a long
g duration yields
y mainlyy Zalkene.
C The isomerisation more Zalkene
erisation produces mor Zalkene at high temperatures and
an produces more
Ealkene att low temperatures.
temperatures
D There are 2 intermediates in the isomerisation process.

594 trendyline
[Turn over
6

10 At 313 K, iodine monochloride exists in equilibrium with iodine and chlorine.

I 2 (g) + Cl 2 (g) 2ICl(g)

When a mixture of iodine and chlorine in the mole ratio of 1:1 is heated at 313 K and
a constant pressure of 1 atm, the mole fraction of iodine monochloride present at equilibrium
is 0.23.

What is the equilibrium constant, K p ?

A 0.299 B 0.357 C 0.597 D 1.55

11 The concentration of Sr2+ ions in a 0.5 dm3 solution is 2.0 x 10-2 mol dm-3. Na 2 CO 3 solid was
added to the solution in excess until the concentration of CO 3 2- ions remained at
2.5 x 10-6 mol dm-3.

Given that the K sp of SrCO 3 is 1.6 x 10-9, what mass of SrCO 3 was precipitated?

A 0.0472 g
B 0.0944 g
C 1.43 g
D 2.86 g

12 Halogens T 2 , U 2 and V 2 were added to separate aqueous solutions containing T, U and V


ions and observations recorded in the table below.

T(aq) U(aq) V(aq)


T2 no reaction U 2 formed V 2 formed
U2 no reaction no reaction no reaction
V2 no reaction U 2 formed no reaction

In which sequence is reducing power of the halides arranged in increasing order?

trendyline
A T < V < U
B U < T < V
C U < V < T
D V < U < T

595 trendyline
7

13 It is known that lithium and magnesium have similar chemical properties.

Which property of the following lithium compounds is likely to be wrong?

A Li 2 CO 3 undergoes thermal decomposition to produce carbon dioxide as the only gas.


B LiNO 3 produces oxygen as the only gas upon heating.
C Li 2 O dissolves in water to form a solution of pH greater than 7.
D LiSO 4 is soluble in water.

14 The following Group II elements form sulfates with the following crystalline forms:

MgSO 4 .7H 2 O CaSO 4 .2H 2 O SrSO 4 BaSO 4

Which of the following explains the observed trend in number of moles of water of
crystallisation?

A Atomic radius of the elements increases down the group.


B Ionic character of sulfates increases down the group.
C Ionisation energy of the elements decreases down the group.
D Radius of the cations increases down the group.

15 What are the products formed when chlorine gas is bubbled into cold aqueous sodium
hydroxide?

A NaCl and NaClO


B NaCl and NaClO 3
C NaClO only
D NaClO 3 only

trendyline
596 trendyline
[Turn over
8

16 The graphs below show the variation of two properties of the halogen hydrides.

Graph I Graph II

HF HCl HBr HI HF HCl HBr HI

Which properties are illustrated in Graphs I and II?

Graph I Graph II
A Boiling point Thermal stability
B pH of solution Thermal stability
C Thermal stability Melting point
D Thermal stability pH of solution

17 When halogenomethanes are warmed in aqueous NaOH followed by acidified AgNO 3 , the
rate of formation of precipitate is observed to be in the order:

CH 3 F < CH 3 Cl < CH 3 Br < CH 3 I

Which of the following best explains this observation?

A Anionic radius increases from F to I.


B Bond energy decreases from CF to CI.
C Bond polarity decreases from CF to CI.
D Charge density decreases from F to I.

trendyline
597 trendyline
9

18 The Co2+ ion forms a blue complex with Cl ions. Various samples containing different
amounts of Co2+ and Cl were prepared, and their colour intensities measured using a
colorimeter. The following graph was obtained.

Which statement about the complex ion formed is correct?

A The complex ion absorbs blue light.


B The co-ordination number of the Co2+ ion is 4.
C The geometry of the complex ion is octahedral about Co2+.
D The overall charge of the complex ion is 2+.

19 A compound with the molecular formula C 5 H 10 reacts with hot acidified KMnO 4 to form
products that give a white precipitate with limewater and a yellow precipitate with aqueous
alkaline iodine.

What is the identity of the compound?

A CH 3 CH 2 CH 2 CH=CH 2
B CH 3 CH 2 CH=CHCH 3
C CH 3 CH(CH 3 )CH=CH 2

trendyline
D CH 3 CH 2 C(CH
(CH
C 3 )=CH 2

598 trendyline
[Turn over
10

20 The deep purple colour of elderberries originates from cyanidin 3-sambubioside.

OH
OH
Cl
HO O

OH OH
OH O OH
O
HO O
OH OH
OH

cyanidin 3-sambubioside

How many optical isomers does cyanidin 3-sambubioside have?

A 26 B 28 C 29 D 211

21 Which species could be an intermediate in an S N 1 substitution?

+ CH3
A CH2 B +
H

OH CH3

C Br C CH3 D HO C Br
-
O CH3 CH3

trendyline
599 trendyline
11

22 When heated with bromine, hydrocarbon Q undergoes free radical substitution. In the
propagation step, free radical R is formed.

Q + Br R + HBr

If there are exactly three possible structures of R, which of the following is a possible
structure of Q?

A B

C D

23 What is the major organic product formed when excess aqueous bromine is added to the
following compound?

OH

OH OH
Br Br
A B

OH Br
Br Br Br

ttrendyline
e yylliine
H
OH H
OH
O
Br Br Br Br
C D

Br OH
Br OH Br Br

600 trendyline
[Turn over
12

24 Which graph shows the amounts of gases evolved when 1 mol of compound M is subjected to
reagents I and II separately?

HO CHO

HO COOH
M

I: Na 2 CO 3 (s) in excess
II: Na(s) in excess

A B

C D

trendyline
601 trendyline
13

25 The mechanism of esterification with acyl derivatives in base is as follows, where ROH is the
alcohol and X is the leaving group.

acyl leaving
derivative group

The rate of esterification is known to increase with better leaving group ability, which is related
to basicity. The less basic the leaving group, the better its ability to leave.

Which of the following is the reactivity of acyl derivatives arranged in increasing order?

Acyl derivative
O O O O
A < <
R Cl R O R NH2

O O O O
B < <
R Cl R NH2 R O

O O O O
C < <
R NH2 R Cl R O

O O O O
D < <
R NH2 R O R Cl

trendyline
602 trendyline
[Turn over
14

26 Compounds K, L and M have the following structures.

H H
O O
N
O O O
O

K L M

Which pair of statements about the properties of K, L and M are correct?

gives a red precipitate with gives a yellow precipitate with hot


Fehlings solution alkaline aqueous iodine

A K, L and M L only
B L and M only L and M only
C L only L only
D L only K and L only

trendyline
603 trendyline
15

27 Serotonin is a neurotransmitter found in the central nervous system of humans.

H
N

HO

NH2

serotonin

Which of the following could be used to synthesise serotonin?

H
N
KCN
+
in ethanol H (aq)
A intermediate serotonin
HO
reflux heat

Br
H
N
LiAlH4
in dry ether NH3
B
HO OH intermediate serotonin
reflux
O

H
N
LiAlH4 ethanolic
Br
i dry ether
n NaOH
C HO intermediate serotonin
CN reflux

H
N
HBr(g) NH3
D
HO intermediate serotonin

trendyline
e
reflux

604 trendyline
[Turn over
16

28 +XFNHOV UXOH GLFWDWHV WKDW ZKHQ WKHUH DUH  GHORFDOLVHG -electrons, the compound has
additional stability and is considered to be aromatic. Benzene is one such aromatic
compound.

Pyrrole and pyridine are two nitrogen containing aromatic compounds with 6 delocalised
-electrons.

N H N

pyrrole pyridine

How many electrons do the nitrogen atoms in pyrrole and pyridine contribute to their
GHORFDOLVHG-electron cloud respectively?

pyrrole pyridine
A 1 1
B 1 2
C 2 1
D 2 2

trendyline
605 trendyline
17

29 A mixture of two amino acids, leucine and aspartic acid, were run on a gel electrophoresis
plate. The following results were obtained:

mixture of amino acids


Before

Cathode Anode (+)

After

&DWKRGH Anode (+)

pK a
Amino acid Side chain
COOH NH 3 + Side chain
Leucine CH 2 CH(CH 3 ) 2 2.3 9.7
Aspartic acid CH 2 COOH 2.0 9.9 3.9

What pH was the gel likely to be buffered at?

A 1.5 B 3.9 C 9.8 D 10.5

trendyline
606 trendyline
[Turn over
18

30 The Mitsunobu reaction is a procedure that allows for the conversion of primary and
secondary alcohols into esters with the addition of a carboxylic acid and a DEAD catalyst. The
reaction is highly useful as the stereochemistry at the chiral carbon is completely inverted.

O
OH O
+
DEAD O R3
R1 R2 R3 OH PPh3
R1 R2

Which pair of alcohol and carboxylic acid could give compound E?

O
O
O

O
Compound E

alcohol carboxylic acid

OH
O
A CH 3 CH 2 OH O

O
OH
O
B CH 3 CH 2 OH O

O
O
COOH
C
O
OH

ttrendyline
re dy
dy
O
COOH
COO
OOHH
D
O
H
OH

607 trendyline
19

Section B
For each of the questions in this section, one or more of the three numbered statements 1 to 3 may
be correct.
Decide whether each of the statements is or is not correct (you may find it helpful to put a tick
against the statements that you consider to be correct).

The responses A to D should be selected on the basis of


A B C D
1, 2 and 3 1 and 2 2 and 3 1 only
are only are only are is
correct correct correct correct

No other combination of statements is used as a correct response.

31 Which properties of beryllium chloride are related to its electron-deficient Be atom?

1 its tendency to polymerise


2 its covalent character
3 its acidity in aqueous solution

32 The following diagram illustrates the energy changes of a set of reactions.

H = 581 kJ mol1
X2(standard state) X(aq)

H = +351 kJ mol1

X(g) X(g)
H = 342 kJ mol1

Which of the following can be deduced from the diagram?

trendyline
1 m -1.
ectron affinity of X is 581 kJ mol
ec
The first electron
2 lpy
py change from X
The enthalpy J  X(aq)
X J X q) is exothermic
exothermic.
3 lpy
py change for the reaction X
The enthalpy X 2 VWDQGDUGVWDWH 
VWDQGDUGVWDWH X(g)
DQGDUGVWDWH  X(g) +112 kJ mol1.
g) is +1

608 trendyline
[Turn over
20

The responses A to D should be selected on the basis of


A B C D
1, 2 and 3 1 and 2 2 and 3 1 only
are only are only are is
correct correct correct correct

No other combination of statements is used as a correct response.

33 Use of Data Booklet is relevant to this question.

A voltaic cell is set up using a Zn2+(aq)/Zn(s) half-cell and a Cu2+(aq)/Cu(s) half-cell. What are
the features of the above set up?

1 Electrons flow in the external circuit from zinc to copper.


2 Reduction takes place at the copper terminal and the copper electrode increases in
mass over time.
3 The zinc electrode is the negative electrode.

34 Three elements H, I and J belong to the same period of the Periodic Table.

The oxide of H reacts with both strong acids and bases. The oxide of I gives a solution of
pH > 7 in water and the oxide of J gives a solution pH < 7 in water.

Which statements about elements H, I and J are always correct?

1 Element H has the highest electrical conductivity.


2 The ionic radius decreases in the order I, H, J.
3 The electronegativity of the elements decreases in the order I, H, J.

35 Samples of strontium and barium are burnt in air.

Which of the following flame colours are observed?

1
2
3
n
pale green
orange-red
lilac
d
trendyline
609 trendyline
21

36 Which of the following titanium compounds are likely to exist?

1 Ti 2 O 5
2 TiBr 4
3 K 2 Ti 2 O 3

37 Capsaicin is the chemical found in chillies that gives rise to the sensation of spiciness.

O
CH3
N
H
CH3
HO
OCH3

Capsaicin

Which of the following statements about capsaicin are correct?

1 Capsaicin decolourises cold alkaline potassium manganate(VII).


2 Capsaicin gives a positive FeCl 3 test.
3 Capsaicin gives an orange precipitate with 2,4-dinitrophenylhydrazine.

38 Bromobenzene is unreactive to nucleophiles while 3-bromopropene is very reactive in


comparison.

What could be reasons for the lack of reactivity of bromobenzene with nucleophiles?

1 The electrons repel incoming attacking nucleophiles.


2 The CBr bond in bromobenzene is stronger than the CBr bond in 3-bromopropene.
3 The CBr bond is unable to rotate freely.

trendyline
610 trendyline
[Turn over
22

The responses A to D should be selected on the basis of


A B C D
1, 2 and 3 1 and 2 2 and 3 1 only
are only are only are is
correct correct correct correct

No other combination of statements is used as a correct response.

39 The Claisen Condensation is a reaction that occurs between two esters, or between one ester
and another carbonyl compound.

O O O
2
O O

O O O O
+
H O H

Which of the following are examples of the Claisen Condensation reaction?

O O O
O
+
1 O H O
H O

O O O O

O +
2

O O
O
O
3
O

trendyline
611 trendyline
23

40 Hair contains chains of the protein keratin, which comprises amino acids forming interactions
with other keratin chains. Hair can be treated in many ways to achieve a desired hair style.

Which of the following treatments will produce the desired styles?

1 Heating hair with steam can help to elongate it.


2 The curling of hair requires both reducing and oxidising agents.
3 Usage of ammonia causes hair to soften and uncurl.

END OF PAPER

trendyline
612 trendyline
[Turn over
2016 PU3 H2 CHEM PRELIM 2 EXAM
ANSWER SHEET (H2)

Question Question
Key Key
Number Number
1 B 21 A
2 D 22 B
3 B 23 D
4 C 24 A
5 C 25 D

6 A 26 D
7 D 27 C
8 D 28 C
9 B 29 D
10 B 30 D

11 C 31 D
12 A 32 C
13 B 33 A
14 D 34 D
15 A 35 D

16 A 36 C
17 B 37 B
18 B 38 B
19 D 39 A
20 C 40 A

trendyline
613 trendyline
Class Adm No

Candidate Name:

2016 Preliminary Examination II


Pre-University 3

H2 CHEMISTRY 9647/02
Paper 2 Structured 15th Sept 2016
2 hours
Candidates answer on the Question Paper.
Additional materials: Data Booklet

READ THESE INSTRUCTIONS FIRST


Do not turn over this question paper until you are told to do so
Write your name, class and admission number on all the work you hand in.
Write in dark blue or black pen.
You may use an HB pencil for any diagrams or graphs.
Do not use staples, paper clips, glue or correction fluid.

Answer all the questions.


The use of an approved scientific calculator is expected, where appropriate.
A Data Booklet is provided.

At the end of the examination, fasten all your work securely together.
The number of marks is given in brackets [ ] at the end of each question or part question.

Question 1 2 3 4 5 6 Total

Marks

trendyline
tre
en
en
ndy
dyyli
ylin
liine
ine
12
2
e 12
1 7 16 11 14 72

This question paper consists of 21 printed pages and 1 blank page.


614 trendyline
2

Answer ALL questions on the spaces provided.

1 Planning For
examiners
A student found a bottle of solid transition metal complex, X, in the laboratory. The
use
label on the bottle showed that the molecular formula of X is Cr(H 2 O) 6 Br 3 and that
the cation of complex X is octahedral in shape. The label also showed that X is
soluble in water.
(a) State the oxidation state of Cr in complex X and hence the full electronic
configuration of the Cr ion in X.

.......[2]
The student researched online and found out that X could have any of the four
structural formulae below:
[Cr(H 2 O) 6 ]Br 3
[CrBr(H 2 O) 5 ]Br 2 H 2 O
[CrBr 2 (H 2 O) 4 ]Br 2H 2 O
[CrBr 3 (H 2 O) 3 ] 3H 2 O

The student wanted to determine the structural formula of X using precipitation and
gravimetric analysis. Gravimetric analysis involves the measurement of mass of the
precipitate collected after a suspension is filtered.

In order to perform the experiment, a 100 cm3 standard solution of 0.100 mol dm-3 of
Cr(H 2 O) 6 Br 3 needs to be prepared.
(b) Write a plan to prepare the standard solution. In your plan, provide details of
calculation of mass of solid X used. You may assume you are provided with:
solid X
distilled water
apparatus normally found in a school laboratory.

trendyline


.......

.......

615 trendyline
3

For
examiners
use
.......

.......

.......

.......

[3]

With the standard solution of X prepared, the student wanted to carry out an
experiment to determine the structural formula of X.
(c) You are to design an experiment to help the student determine the structural
formula of X using the method of precipitation of silver bromide followed by
gravimetric analysis.
You may assume that you are provided with:
standard solution prepared in (b)
aqueous silver nitrate of concentration 2.0 mol dm-3
apparatus normally found in a school laboratory.
Your plan should include:
the quantity of the reagents used
the procedure for the precipitation and gravimetric analysis
an outline of how the experimental data would be used to determine the
structural formula of X.

trendyline




.......

[Turn over
616 trendyline
4

....... For
examiners
use

.......

.......

.......

.......

.......

.......

.......

[5]

trendyline
617 trendyline
5

(d) In an experiment using a different sample of Cr(H 2 O) 6 Br 3 , it was found that the For
examiners
mole ratio of Cr(H 2 O) 6 Br 3 to AgBr formed is 1: 1. Given that the shape of the
use
cation of this sample is also octahedral and that it can exhibit geometric
isomerism, draw the structures of the two isomers of the cation.

[2]

[Total: 12]

trendyline
[Turn over
618 trendyline
6

2 (a) This question is about the chemistry of crown ethers. For


examiners
use
Crown ethers are cyclic chemical compounds that consist of a ring containing
several ether groups. The most common crown ethers are oligomers of ethylene
oxide, the repeating unit being ethyleneoxy, i.e. -CH 2 CH 2 O-. The term crown
refers to the resemblance between the structure of a crown ether bound to a
cation, and a crown sitting on a persons head.

The first number in a crown ethers name refers to the number of atoms in the
ring, and the second number refers to the number of oxygen atoms in the crown
ether.

Crown ethers strongly bind to certain cations, forming complexes. The oxygen
atoms are well situated to coordinate with a cation located at the interior of the
ring, whereas the exterior of the ring is hydrophobic.

Structures of common crown ethers are shown below.

12-crown-4 15-crown-5 Q

(i) Name the crown ether Q.

...[1]

(ii) Predict and explain whether crown ethers are soluble in water.

...

trendyline
...[2]

619 trendyline
7

Crown ethers are size-selective, as they form the most stable complexes with For
examiners
metal ions that fit best into the ring. The metal cation sits in the centre of the ring
use
and interacts with lone pairs from all of the oxygen atoms.

Q forms stable complexes with K+ metal ion.

(iii) Illustrate how K+ metal ion binds to Q with an aid of a diagram. State the
type of bonding between the crown ether and the metal ion.

[2]

(iv) Suggest the identity of a Group I cation which would be most suitable to bind
with 15-crown-5.

...[1]

(v) When propyl bromine is treated with KF in benzene, no reaction takes place.
But when the crown ether Q is added to the reaction mixture, the desired
propyl fluoride is produced.

Br KF
no reaction
benzene

Br KF F

benzene
O
O O

O O
O

[Turn over
620 trendyline
8

Explain why this is so. For


examiners
use
..........................................................................................................................

......................................................................................................................[2]

(b) Crown ethers are generally synthesised involving the Williamson ether syntheses
between excess dichloride derivatives of the appropriate ethylene glycols and the
appropriate bishydroxy benzenes. The synthesis scheme for the synthesis of
difunctionalised 32-crown-10 derivative is shown below.

Cl O O O Cl O O O O Cl
NaH O
+
Step F CH3O
OH O O O O Cl
O

CH3O OH
OH
CsF
O
OH
OCH3

O O O O O
O O O O O
O O
Step G
HO OH
CH3O OCH3
O O O O O
O O O O O

Step H

O O O O O

Br Br
O O O O O

trendyline
32-crown-10 derivative

(i) uggest
ggest the roles of NaH and CsF in the reaction scheme.
Suggest

...[1]
(ii) State the type of reaction occurring in steps F and G.

621 trendyline
9

...[1]
(iii) State the reagents and conditions required for step H. For
examiners
..
use
[2]
[Total: 12]

3 In the Periodic Table, there is said to be a diagonal relationship between elements of


the second and third periods of the Periodic Table exemplified by lithium and
magnesium.

An element R is related to aluminium by such a relationship and R also has the highest
first ionisation energy of its group.

(a) (i) Identify R and explain the similarity in polarising powers of the cations of R
and Al3+.

..........................................................................................................................

......................................................................................................................[2]

(ii) Predict and explain the pH of the solution formed when the chloride of R
dissolves in water.

..........................................................................................................................

......................................................................................................................[2]

(b) Another class of chlorides, acid chlorides, can react with alcohols to form esters
which are sweet-smelling. Esters have been used as flavourings in food and
perfumes. For example, ethyl propanoate is an ester with pineapple-like smell.

The kinetics of the hydrolysis of ethyl propanoate in alkaline medium at T K was


studied.

trendyline
eparate
parate experiments were carried out wi
Two separate with varying con
concentrations of
aqueouss sodium hydroxide solution. For each experiment, the co
concentration of
ropanoate
opanoate was determined
ethyl propanoate deter
dete mined
ined at regular time intervals.
interva
The graph
aph below shows the results of Experiment
xpe 2 mol dm-3.
1 with [NaOH] = 2.0 For
examiners
use

[Turn over
622 trendyline
10

[CH3CH2COOCH2CH3] / mol dm-3

Experiment 1

Time / min

trendyline
623 trendyline
11

The results of Experiment 2 with [NaOH] = 1.0 mol dm-3 are shown below. For
examiners
use
Time Experiment 2 with [NaOH] = 1.0 mol dm3
/ min [CH 3 CH 2 COOCH 2 CH 3 ] / mol dm3
0 0.0200
25 0.0152
50 0.0115
75 0.0088
100 0.0067
125 0.0051

(i) Plot the results of Experiment 2 on the same graph above. Label your graph
clearly. [1]

(ii) Use the two graphs to determine the order of reaction with respect to
[CH 3 CH 2 CO 2 CH 2 CH 3 ] and [NaOH], showing your working clearly.

[2]

[Total: 7]

trendyline
[Turn over
624 trendyline
12

4 2-propanol is a colourless, flammable liquid with a strong odour. It is widely used as a For
examiners
solvent in many industrial applications.
use

(a) 2-propanol can be used as a fuel in the fuel cell. At the anode of the cell,
2-propanol is oxidised to carbon dioxide. The electrons pass around the external
circuit to the cathode. The protons formed from the oxidation move through the
conducting polymer electrolyte to the cathode, where they react with oxygen to
produce water.

(i) Construct ion-electron equations for the reactions at the anode and cathode
respectively.

Anode:.........

Cathode:..[2]

(ii) Hence construct the equation for the overall reaction.

.....[1]

(iii) The cell is capable of producing an e.m.f. of 1.56 V. By using suitable data
from the Data Booklet, suggest a value for the Eo of the CO 2 /CH 3 CH(OH)CH 3
electrode reaction.

[2]

(iv) Suggest a possible advantage of using the 2-propanol fuel cell compared to a
hydrogen fuel cell.

........

trendyline


.....[1]

625 trendyline
13

(b) 2-propanol reacts with hot excess concentrated sulfuric acid to form propene. The For
examiners
reaction follows a 3-step mechanism as described below:
use
Step 1 The oxygen atom on 2-propanol is protonated by H 2 SO 4 forming
intermediate A and HSO 4 - in an exothermic reaction.
H
+
O H
CH3 C CH3
H

Intermediate A
Step 2 The C-O bond in intermediate A breaks, forming a carbocation B
and water. This step is endothermic and is the rate determining
step.
Step 3 The carbocation is then deprotonated in an exothermic reaction,
forming propene. H 2 SO 4 is regenerated.

(i) State the type of reaction undergone when 2-propanol reacts with hot excess
concentrated sulfuric acid.

.....[1]

(ii) Use the information given in the table above to draw out the full mechanism
for the reaction between 2-propanol and concentrated sulfuric acid, indicating
the movement of electrons using curly arrows and showing clearly the lone
pair of electrons involved in the mechanism. You are advised to use structural
formulae for all species, so that it is clear which bonds are broken and which
are formed.

trendyline [3]

[Turn over
626 trendyline
14

(iii) Using the information given in the table in (b), sketch a fully labelled energy For
examiners
profile diagram of the reaction between 2-propanol and concentrated sulfuric
use
acid.

[2]

(c) 2-propanol reacts with methanoic acid in the presence of concentrated sulfuric acid,
producing an ester, isopropyl formate, and water. The equilibrium below is set up
during the reaction:

O OH O
+ + H 2O
OH O
isopropyl formate

If 0.500 mol of methanoic acid and 0.300 mol of 2-propanol were mixed with a small
amount of concentrated sulfuric acid and allowed to reach equilibrium, the
equilibrium mass of isopropyl formate was found to be 16.64 g.

(i) Write an expression for the equilibrium constant, K c , for the reaction.

trendyline
[1]

627 trendyline
15

(ii) Calculate a value for K c . For


examiners
use

[2]

(iii) A molecular sieve is a crystalline substance with pores of molecular


dimensions which permits the passage of very small molecules such as water.
The use of molecular sieves in the reaction above increases the yield of
isopropyl formate. Suggest a reason for the increase in yield.

......

.....[1]

[Total: 16]

[Turn over
628 trendyline
16

5 Hydrastine is a nitrogen containing drug used in the treatment of memory-related For


examiners
disorders such as Alzheimers disease and schizophrenia and has the structure shown
use
below.

N
O CH3

O
O
OCH 3

Hydrastine

A chemist planned a synthetic route to synthesise Hydrastine from


1,2-dimethylbenzene through the intermediates, compounds W and X, as shown below.
O

N
CH3 CH 2 OH O
a few steps step A many steps
CH3
O
O
CH3 O
O
OH
O
Compound W Compound X O
OCH 3

Hydrastine

(a) Hydrastine can be hydrolysed in either


I acidic or
II basic conditions.

Draw the structures of the products obtained for each case. Assume that the ether
group (C-O-C) is inert. [2]

I II

trendyline
y
629 trendyline
17

(b) Devise a reaction scheme to help him synthesise compound X via compound W For
examiners
from 1,2-dimethylbenzene. Show clearly the reagents and conditions required for
use
each step.

[4]

(c) State the oxidation numbers of the carbon atoms, labelled with an *, in
1,2-dimethylbenzene and compound X.

*
*
CH 3 O

CH3 O

Oxidation number of C* in 1,2-dimethylbenzene:...

Oxidation number of C* in compound X: ........[2]

(iv) Compare the relative acidity of the alcohol and the carboxylic acid groups
present in compound W.

.........

.....

trendyline
.....[3]

[Total: 11]

[Turn over
630 trendyline
18

6 (a) Boiling raw eggs in hot spring water at pH = 9.2 produces black eggs (kuro tamago) For
examiners
through a chemical reaction with geothermal heat and volcanic gas as iron
use
properties ahere to the porous shells. The black colour of iron sulfide is a result of a
reaction with hydrogen sulfide, and it produces boiled eggs with black shells.

Iron sulfide is precipitated according to the following reaction.

reaction 1: Fe2+(aq) + S2(aq) FeS(s)

Hydrogen sulfide gas from volcanic systems is released into the hot spring water
and behaves as a dibasic (diprotic) weak acid.
H 2 S(g) + aq 2H+(aq) + S2(aq)

In a saturated solution of hydrogen sulfide,

[H+]2[S2-] = 1.0 x 10-23 mol3 dm-9

(i) Calculate the maximum concentration of sulfide ions present in hot spring
water.

[2]

(ii) Hence, calculate the minimum concentration of Fe2+ that must be present in
hot spring water in order for precipitation to occur.

(K sp of FeS = 4.9 1018 mol2 dm6)

trendyline [1]

631 trendyline
19

(iii) Use the following data, together with relevant data from the Data Booklet, to For
examiners
calculate for reaction 1 with the aid of an energy cycle.
use

standard enthalpy change of formation of FeS(s) 102 kJ mol1


standard enthalpy change of atomisation of Fe(s) +415 kJ mol1
standard enthalpy change of atomisation of S(s) +279 kJ mol1
sum of first two electron affinities of S(g) +337 kJ mol1
enthalpy change of hydration of Fe2+(g) 1981 kJ mol1
enthalpy change of hydration of S2(g) 1372 kJ mol1

[4]

(b) When a precipitate is formed, , in J mol1, is given by the following expression.

trendyline
li
(i) Use
se
e the data given in (a)(ii) to calculate , in kJ mol-1, for F
FeS.

[1]

[Turn over
632 trendyline
20

(ii) Use your answer in (a)(iii) and (b)(i) to calculate , in J mol-1 K-1, for the For
examiners
formation of the precipitate FeS(s) at 298 K.
use

[1]

(c) Geraniol (C 10 H 18 O) appears as a clear to pale-yellow oil that is insoluble in water


but soluble in most common organic solvents. It has a rose-like scent and is
commonly used in perfumes.

OH

Geraniol

Geraniol reacts with HBr(g) to form the two possible products:

I: Br Br OH

OH
II: Br Br

(i) State which of the products would be the major product. Explain your answer.

...

......

trendyline
.[3]

633 trendyline
21

(ii) Draw a diagram to illustrate the bonding in C=C bond in geraniol in terms of For
examiners
orbital overlap.
use

[2]

[Total: 14]

END OF PAPER

trendyline
[Turn over
634 trendyline
Class Adm No

Candidate Name:

2016 Preliminary Examination II


Pre-University 3

H2 CHEMISTRY 9647/03
Paper 3 Free Response 19th Sept 2016
2 hours
Candidates answer on separate paper.
Additional materials: Cover Page
Answer Paper
Data Booklet

READ THESE INSTRUCTIONS FIRST


Do not turn over this question paper until you are told to do so
Write your name, class and admission number on all the work you hand in.
Write in dark blue or black pen.
You may use an HB pencil for any diagrams or graphs.
Do not use staples, paper clips, glue or correction fluid.

Answer any four questions.


A Data Booklet is provided.
The use of an approved scientific calculator is expected, where appropriate.
You are reminded of the need for good English and clear presentation in your answers.

At the end of the examination, fasten all your work securely together.
The number of marks is given in brackets [ ] at the end of each question or part question.

trendyline
This question paper consists of 15
635printed pages and 1 blank page. trendyline
2

1 (a) At room temperature, the electrode potential E for the system


[MnO 4 -(aq) + 8H+(aq)], [Mn2+(aq) + 4H 2 O(l)] / Pt
is given by the equation
-
0.059 [MnO4 (aq)] [H+ (aq)] 8
E = E + log
5 [Mn2+ (aq)]

where E = +1.51 V. E values at various pH are given below when
[MnO 4 -(aq)] = 0.01 mol dm-3 and [Mn2+(aq)] = 0.0001 mol dm-3.

pH E/V
0 +1.52
3 +1.24
7 +0.83

(i) Define what the symbol Erefers to. [1]

(ii) Calculate the electrode potential of the cell at pH 1, when the


manganate(VII) ion concentration is 0.01 mol dm-3 and the manganese(II)
ion concentration is 0.001 mol dm-3. [2]

(iii) Predict at which pH value (0, 3 or 7) the solution is the most oxidising. [1]

The electrolytic conductivity ,, of a solution in a cell is given by the following


formula:
(c) (l)
 
a
where c is the conductance of the cell and a is the cross-sectional area of the
electrodes, separated by a distance l.

(iv) l
Often the term is replaced by a cell constant K. Calculate the cell
a
constant of a particular cell if it was found that the conductance of the cell
containing Mn2+(aq) of concentration 0.100 mol dm-3 LV-1 and the

trendyline
HOHFWURO\WLFFRQGXFWLYLW\ZDV-1 cm-1. State the units of K.
HOHFWURO\WLFFRQGXFWLYLW\ZDV [2]

636 trendyline
3

(b) A diaphragm cell consisting brine solution, concentrated NaCl, is used in the
commercial electrolytic production of chlorine and sodium hydroxide.

product

(i) Write the ion-electron equations for the reactions occurring at the cathode
and at the anode. [2]

(ii) The chlorine gas is collected separately at the anode to prevent a side
reaction that will occur with one of the products formed. Write a balanced
equation for this side reaction that occurs at room conditions. State the
oxidation numbers of chlorine in all the products formed. [2]

(iii) In an electrolytic experiment using this diaphragm cell, 1.0 mol of gas is
produced at the cathode. Calculate the mass of chlorine gas that is
produced in the same experiment. [1]

(c) Chlorine contains electrons which occupy the n = 2 principal quantum shell.
Draw all the possible shapes of orbitals at the energy level, n = 2. [2]

(d) Explain why aqueous manganese(II) ions are coloured. [3]

trendyline
637 trendyline
[Turn over
4

(e) Cocaine, C 17 H 21 NO 4 , was first used as a local anaesthetic. It is also a powerful


stimulant and its structure is shown below.
O CH3
O
O
O

N
CH3

Cocaine

Student X was asked to suggest a suitable synthetic route to prepare cocaine


from methylbenzene and compound A.
OH
O

HO

N
CH3

Compound A

Student X suggested the following steps:

Step 1 Methylbenzene is oxidised with hot, acidified KMnO 4 (aq) to give


benzoic acid.
Step 2 Benzoic acid is converted into benzoyl chloride by reacting the
resultant solution from step 1 with thionyl chloride.
Step 3 Benzoyl chloride is reacted with compound A at room temperature.
Step 4 The resultant compound from step 3 is reacted with methanol in the
presence of concentrated sulfuric acid.

(i) Give two reasons why the method suggested would not work. [2]

(ii)

trendyline
Draw the aromatic organic product formed when cocaine
hydrolysis.
acidic hydrolysis
cocain undergoes
[1]

638 trendyline
5

Cocaine is smuggled and inhaled as its protonated hydrochloride salt, known as


cocaine hydrochloride. Smoking cocaine is more stimulating than inhaling the
salt as it is absorbed quickly by the capillaries in the lung tissues. The salt is
hence converted back to cocaine before smoking. This is also done as the salt
will decompose before it vapourises.

(iii) Suggest a suitable reagent to convert cocaine hydrochloride back into


cocaine. [1]

[Total: 20]

trendyline
639 trendyline
[Turn over
6

2 Researchers at the Max Planck Institute of Chemistry conducted a study to determine


whether the chemicals emitted by human beings via their breath and their skin vary in
response to audiovisual stimulus. In one of the experiments carried out during the
screening of The Hunger Games 2 in a cinema, the researchers detected that there
were spikes of emission of a diene, isoprene, twice; when the heroines dress caught
fire and when the final battle began. Isoprene is associated with muscle contraction,
hence the spikes in emission could be due to unconscious muscle twitching as the
audience empathised with the heroine.

The structure of isoprene is shown below. In its pure form, it exists as a colourless
volatile liquid.

Isoprene

(a) Isoprene reacts with gaseous HBr at room temperature.

(i) Draw the structure of the major product of the reaction and hence state the
IUPAC name of the major product. [2]

(ii) Name and illustrate with a diagram the type of stereoisomerism exhibited by
the major product of the reaction. [2]

(b) Polyisoprene, a polymer of isoprene, is the primary chemical constituent of natural


rubber.

CH3 CH3
CH2 C CH2 C
n CH H < 0
CH
CH2 CH2
n
Isoprene Polyisoprene

(i) Predict, with a reason, the sign of the entropy change of the polymerisation

trendyline
process of isoprene. [1]

(ii) With reference to your answer in (b)(i)


(b)(
(b)(i),, state and explain if the
polymerisation is more spontaneous
ane spontane
or less spontaneous at higher
temperatures. [2]

640 trendyline
7

(iii) Natural rubber is insoluble in water. With reference to the structure of


polyisoprene, explain why. [2]

(c) In a Diels-Alder reaction, 2 moles of isoprene react to form 1 mole of limonene.


Limonene is a colourless liquid which possesses a strong smell of oranges. The
mechanism of the reaction proceeds as follow.

limonene

The enthalpy changes of vaporisation of isoprene and limonene are


+26.4 kJ mol-1 and +39.4 kJ mol-1 respectively.

Using the data given and relevant data from the Data Booklet, construct an energy
cycle to calculate the enthalpy change for the reaction below.

2
(l) (l)

Limonene

You may use C 5 H 8 and C 10 H 16 in your energy cycle to represent isoprene and
limonene respectively. [3]

trendyline
641 trendyline
[Turn over
8

(d) Compound P is an isomer of limonene. When P is oxidised by hot acidified


potassium manganate(VII), Q, C 3 H 4 O 3 , and R, C 7 H 12 O 3 , are formed.

Both Q and R give orange precipitate with 2,4-dinitrophenylhydrazine and give


effervescence when reacted with solid calcium carbonate. Only Q gives a yellow
precipitate with warm alkaline iodine.

When Q reacts with NaBH 4 in methanol, S is formed. 1 mole of S gives 1 mole of


H 2 (g) upon reaction with excess sodium metal.

R reacts with phosphorus pentachloride, giving white fumes and T, C 7 H 11 ClO 2 .


When T reacts with ethereal LiAlH 4 , U, as shown below is formed.
O
O

compound U

Deduce the structures for compounds P T, explaining the chemistry involved. [8]

[Total: 20]

trendyline
642 trendyline
9

3 Compound U reacts with aqueous hydroxide ions when heated under reflux.
Cl
Cl

Compound U

The product of the reaction between U and hot aqueous hydroxide ions does not have
optical activity.

(a) (i) Describe the full mechanism of the reaction. [3]

(ii) Explain why the product of the reaction does not exhibit optical activity. [2]

(b) When 0.100 mol dm-3 of hydroxide ions react with 0.00100 mol dm-3 of compound
U at 90 C, the half-life of compound U is 25 s.
(i) Define the term half-life. [1]

(ii) Using your answer in (a)(i), state the rate equation of the reaction. [1]

(iii) Calculate the rate constant at 90 &VWDWLQJLWVXQLWV [1]

(iv) If the reaction is repeated using 0.100 mol dm-3 of hydroxide ions and
0.00200 mol dm-3 of U at the same temperature, state the half-life of U in
the new experiment. [1]

(v) With the aid of a clearly labelled Boltzmann distribution curve, explain why
the rate of the reaction decreases when the temperature of the reaction
decreases. [2]

(c) 0.450 g of U was reacted completely with excess hot aqueous sodium hydroxide.
The reaction mixture was cooled and acidified with excess HNO 3 (aq). Calculate
the mass of silver chloride formed when an excess of AgNO 3 (aq) was added.
[2]

trendyline
643 trendyline
[Turn over
10

(d) Compound V can be formed from compound U in 3 steps.


O
N
Cl H

compound V

Propose a 3-step synthesis of V from U, indicating clearly all reagents and


conditions used. Draw the structures of all intermediates. [5]

(e) Compound W is an isomer of U.


Cl

Cl
compound W

When W is boiled with aqueous sodium hydroxide, it immediately loses water to


form an organic compound X which gives a yellow precipitate with a medicinal
smell when reacted with warm iodine in KOH(aq).

Suggest the structure of X and write a balanced chemical equation for the reaction
of X with warm iodine in KOH(aq). [2]

[Total: 20]

trendyline
644 trendyline
11

4 (a) Boron is a metalloid chemical element in Group III. However due to its smaller
size compared to other Group III elements, its charge over mass ratio is closer to
that of silicon. Hence it resembles silicon in many reactions.

(i) Predict the pH of the solution when BCl 3 is added into water. Hence write
an equation for the reaction. [2]

(ii) Explain why CCl 4 does not undergo the same reaction as BCl 3 when it is
added to water. [1]

(iii) The oxide of boron reacts with NaOH the same way SiO 2 reacts with
NaOH. One of the products of the reaction is Na 3 BO 3 . Propose a chemical
equation for the reaction between the oxide of boron and NaOH.
[1]

(b) The following equilibrium exists when BCl 3 (g) is mixed with NH 3 (g).
BCl 3 (g) + NH 3 (g) Cl 3 BNH 3 (s)

(i) Draw a dot-and-cross diagram of the Cl 3 BNH 3 molecule showing clearly its
co-ordinate (dative covalent) bonds. [1]

(ii) Suggest the shape and the bond angle of the BCl 3 molecule. [1]

(iii) Predict the effect of decreasing the temperature on the above equilibrium.
Explain your answer. [3]

(iv) At a total initial pressure of 3 atm, a 1 : 2 mixture of BCl 3 and NH 3 was


DOORZHG WR UHDFK HTXLOLEULXP DW  & ,W ZDV IRXQG WKDW WKH HTXLOLEULXP
partial pressure of BCl 3 was 0.68 atm.
Determine the value of the equilibrium constant, K p , for this reaction at
&VWDWLQJLWVXQLWV [2]

trendyline
645 trendyline
[Turn over
12

(c) Keratin is a fibrous protein that acts as the key structural component of hair. The
amino acid content of keratin in hair of human varies slightly with ethnicity.
The hair sample of an Asian was analysed. Eight of the amino acids in a keratin
molecule present in the hair are listed below.

Amino acid Formula of side chain Number of amino acid residues


(R in RCH(NH 2 )COOH) per molecule of keratin
cysteine -CH 2 SH 1200
glutamic acid -CH 2 CH 2 COOH 1026
serine -CH 2 OH 990
threonine -CH(OH)CH 3 572
leucine -CH 2 CH(CH 3 ) 2 520
aspartic acid -CH 2 COOH 478
glycine -H 464
valine -CH(CH 3 ) 2 456

(i) Use any three of the above amino acids to construct the displayed formula
of a possible section of the protein chain of keratin. [2]

(ii) Describe how a polypeptide chain of keratin is held in the shape of an


-helix. [2]

(iii) The cysteine residues in the keratin molecule can form disulfide bridges.
Illustrate this process by means of a balanced equation.

Assuming all the disulfide bridges are intramolecular, what is the maximum
number of bridges which could be formed within each keratin molecule? [2]

(iv) Apart from the disulfide bridges mentioned in (c)(iii), describe, by means of
diagrams, two other types of side-chain interaction, using suitable pairs of

trendyline
amino
mino acids from the table above. [2]

(v) Glutamic acid has three pK


pK a values: 2.1, 4.1 and 9.5.
9.5
Suggest the structure of the major
jor sspecies present in soluti
solutions of glutamic
acid at pH 7. [1]

646 trendyline
13

[Total: 20]

5 Fluorine is a highly toxic pale yellow diatomic gas under standard conditions. As the
most electronegative element, it is highly reactive.

Hydrogen fluoride is also a highly toxic gas which forms corrosive hydrofluoric acid
upon contact with water.

(a) State and explain which gas, fluorine or hydrogen fluoride, displays greater
deviation from ideal gas behaviour. [2]

(b) Fluorine reacts with hydrogen to form hydrogen fluoride.


Write an equation with state symbols for the reaction and describe how you would
expect the reaction of hydrogen with fluorine to differ from that of hydrogen with
chlorine. [2]

(c) Hydrofluoric acid is a weak acid. It forms a buffer solution with salts containing its
conjugate base.
Calculate the mass of solid sodium fluoride required to be added into a 250 cm3
solution of 0.00420 mol dm-3 of HF to form a buffer solution of pH 5.6.
[K a of HF = 7.20 10-4 mol dm-3]
[2]

(d) Hydrofluoric acid is a poison with greater hazards than strong acids even though it
is a weak acid. It can react with calcium in the blood leading to hypocalcemia and
potentially causing death through cardiac arrhythmia. This is due to the formation
of sparingly soluble calcium fluoride in the body.

(i) The solubility of calcium fluoride in water is 2.707 10-4 mol dm-3. Calculate
the solubility of calcium fluoride in a solution of 0.00200 mol dm-3 of aqueous
NaF. [2]

(ii) Explain briefly how the presence of NaF affects the solubility of calcium

trendyline
fluoride.
luoride.
uoride. [1]

647 trendyline
[Turn over
14

(e) HCl is a reagent often used in organic reactions.

(i) For each of the following types of reaction, write a balanced equation for a
reaction involving HCl as a reagent or as a catalyst. You may use any
organic compound as the starting material for each reaction.

I Hydrolysis

II Neutralisation

III Addition [3]

(ii) A student proposed the following reaction for the conversion of


2-methylphenol to 4-chloro-2-methylphenol using aqueous hydrochloric acid
as the reagent.

CH3 CH3

HCl(aq)
OH rtp Cl OH

2 methylphenol 4 chloro 2 methylphenol

Explain why the reaction he proposed is not likely to work, and suggest how
the student can improve the conversion. [3]

(iii) Suggest a simple chemical test to distinguish between 2-methylphenol and


phenylmethanol. State clearly the reagent and conditions used as well as the
observations expected. [2]
OH

trendyline
y methanol
p enyl
ph

648 trendyline
15

(iv) Compound Z can be formed from 2-methylphenol in two steps.


O
O

O
O
compound Z

Propose a two-step synthesis of Z from 2-methylphenol, stating the reagents


and conditions for each step and showing the structure of the intermediate.
Your synthesis should involve 2-methylphenol as the only organic
reactant. [3]

[Total: 20]

END OF PAPER

649 trendyline
[Turn over
2

Answer ALL questions on the spaces provided.

1 Planning For
examiners
A student found a bottle of solid transition metal complex, X, in the laboratory. The
use
label on the bottle showed that the molecular formula of X is Cr(H 2 O) 6 Br 3 and that
the cation of complex X is octahedral in shape. The label also showed that X is
soluble in water.
(a) State the oxidation state of Cr in complex X and hence the full electronic
configuration of the Cr ion in X. [2]
+3
1s2 2s2 2p6 3s2 3p6 3d3
The student researched online and found out that X could have any of the four
structural formulae below:
[Cr(H 2 O) 6 ]Br 3
[CrBr(H 2 O) 5 ]Br 2 H 2 O
[CrBr 2 (H 2 O) 4 ]Br 2H 2 O
[CrBr 3 (H 2 O) 3 ] 3H 2 O

The student wanted to determine the structural formula of X using precipitation and
gravimetric analysis. Gravimetric analysis involves the measurement of mass of the
precipitate collected after a suspension is filtered.

In order to perform the experiment, a 100 cm3 standard solution of 0.100 mol dm-3 of
Cr(H 2 O) 6 Br 3 needs to be prepared.
(b) Write a plan to prepare the standard solution. In your plan, provide details of
calculation of mass of solid X used. You may assume you are provided with:
solid X
distilled water
apparatus normally found in a school laboratory. [3]
Mass of X needed = (100/1000) 0.100 399.7 = 3.997g
Using a mass balance, weigh accurately 3.997g of solid X in a weighing
bottle.

trendyline
250 3 beaker,
Transfer the solid into a 250cm eaker, rinsing the weighing
we
weig bottle with
distilled water and pouring the washing into the beaker. D
Dissolve the solid
with distilled water.
wate
Transfer
T f the
th solution
l ti iinto 100cm3 volumetric
t a 10
100 l t i flask,
fl k making sure all

650 trendyline
3

washing goes into the volumetric flask.


Add distilled water into the volumetric flask and top up to the mark.
Stopper the volumetric flask and shake well to form a homogeneous
solution.
With the standard solution of X prepared, the student wanted to carry out an
experiment to determine the structural formula of X.
(c) You are to design an experiment to help the student determine the structural
formula of X using the method of precipitation of silver bromide followed by
gravimetric analysis.
You may assume that you are provided with:
standard solution prepared in (b)
aqueous silver nitrate of concentration 2.0 mol dm-3
apparatus normally found in a school laboratory.
Your plan should include:
the quantity of the reagents used
the procedure for the precipitation and gravimetric analysis
an outline of how the experimental data would be used to determine the
structural formula of X. [5]
Using a pipette, transfer 10.0 cm3 of the standard solution of X into a
beaker.
Add aqueous silver nitrate until in excess and stir with a glass rod. Stop
adding when the cream precipitate formed does not increase in mass (or at
least 3 times the amount of X in 10 cm3).
Using a filter funnel, filter the mixture to obtain the residue.
Transfer the residue to a pre-weighed dry evaporating dish and heat the
residue to dryness by placing the evaporating dish on top of a Bunsen
burner with tripod.
Cool and reweigh the evaporating dish to obtain mass of the residue.
Repeat the heating, cooling and weighing until a constant mass is
obtained.
Amount of X used = (10/1000) 0.100 = 0.00100 mol
Amount of residue formed = mass of residue / 187.9 = y

ttrendyline
rendyyline
Mol ratio of X to AgBr Structural
ctura formula
1:0 [CrBr
CrBr 3 (H
H 2 O) 3 ] 3
3H 2 O
1:1 [CrBr 2 (H
H 2 O) 4 ]Brr 2H 2 O
1:2 [CrBr(H
[ 2 O) 5 ]Br 2 H 2 O

1:3 [Cr(H 2 O) 6 ]Br 3

[Turn over
651 trendyline
4

(d) In an experiment using a different sample of Cr(H 2 O) 6 Br 3 , it was found that the
mole ratio of Cr(H 2 O) 6 Br 3 to AgBr formed is 1: 1. Given that the shape of the
cation of this sample is also octahedral and that it can exhibit geometric
isomerism, draw the structures of the two isomers of the cation.
[2]

[Total: 12]

2 (a) This question is about the chemistry of crown ethers.

Crown ethers are cyclic chemical compounds that consist of a ring containing several ether
groups. The most common crown ethers are oligomers of ethylene oxide, the repeating unit
being ethyleneoxy, i.e. CH 2 CH 2 O-. The term crown refers to the resemblance between
the structure of a crown ether bound to a cation, and a crown sitting on a persons head.

The first number in a crown ethers name refers to the number of atoms in the ring, and the
second number refers to the number of oxygen atoms in the crown ether.

Crown ethers strongly bind to certain cations, forming complexes. The oxygen atoms are
well situated to coordinate with a cation located at the interior of the ring, whereas the
exterior of the ring is hydrophobic.

Structures of common crown ethers are shown below.

trendyline
652 trendyline
5

12-crown-4 15-crown-5 Q
(i) Name the crown ether Q. [1]
18-crown-6
(ii) Predict and explain whether crown ethers are soluble in water. [2]
Crown ethers are generally insoluble in water because the energy released in
hydrogen bonds formed between crown ethers and water are not exothermic enough
to overcome the temporary dipole-induced dipole interactions between crown ether
molecules and hydrogen bonds between water molecules.

Crown ethers are size-selective, as they form the most stable complexes with metal ions
that fit best into the ring. The metal cation sits in the centre of the ring and interacts with lone
pairs from all of the oxygen atoms.

Q forms stable complexes with K+ metal ion.


(iii) Illustrate how K+ metal ion binds to Q with an aid of a diagram. State the type of
bonding between the crown ether and the metal ion. [2]

Coordinate / Dative bonding


(iv) Suggest the identity of a Group I cation which would be most suitable to bind with
15-crown-5. [1]
Na+
Remarks: Based on question context, as qns say the most stable complex are with

trendyline
trend
d li
gg crown ether bind to K+, a slightly smaller
the one that fits best into ring. Since a bigger
crown ether will bind to Na+.

(v) When
hen propyl bromine is treated with KF in benzene, no re
reaction takes place. But
when the crown ether Q is added to the reaction mixture, the desired propyl fluoride is

[Turn over
653 trendyline
6

produced.

Br KF
no reaction
benzene

Br KF F

benzene
O
O O

O O
O

Explain why this is so. [2]


KF is insoluble in benzene, so F- is unavailable to attack the propyl bromine.
When 18-crown-6 is added, the crown ether surrounds the K+ ion, allowing the F- ion
to be formed for nucleophilic substitution, so the reaction can proceed

Remarks: recognise that you need to ans according to the context. From earlier part,
K+ will react with crown ether, thus producing F-.
(b) Crown ethers are generally synthesised involving the Willaimsen ether syntheses between
excess dichloride derivatives of the appropriate ethylene glycols and the appropriate
bishydroxy benzenes. The synthesis scheme for the synthesis of difunctionalised
32-crown-10 derivative is shown below.

trendyline
654 trendyline
7

Cl O O O Cl O O O O Cl
NaH O
+
Step F CH3O
OH O O O O Cl
O

CH3O OH
OH
CsF
O
OH
OCH3

O O O O O
O O O O O
O O
Step G
HO OH
CH3O OCH3
O O O O O
O O O O O

Step H

O O O O O

Br Br
O O O O O

(i) Suggest the roles of NaH and CsF in the reaction scheme. [1]
They act as bases, abstracting a proton from phenol.
(ii) State the type of reaction occurring in steps F and G [2]
Step F: Nucleophilic Substitution (S N 2)
Step G : Reduction
(remarks: Calculate the change in OS of the carbon centre)
(iii) State the reagents and conditions required for Step H. [2]
PBr 3 / SOBr 2 , rtp
[Total: 12]

trendyline
[Turn over
655 trendyline
8

3 In the Periodic Table, there is said to be a diagonal relationship between elements of the
second and third periods of the Periodic Table exemplified by lithium and magnesium.

An element R is related to aluminium by such a relationship and R also has the highest first
ionisation energy of its group.

(a) (i) Identify R and explain the similarity in polarising powers of the cations of R and
Al3+. [2]
R is beryllium.
Polarising power Is directly proportional to charge density (charge/size).
Although Al3+ is of a higher charge than Be2+, Al3+ is also larger in size. The two
ions have similar charge densities and hence, similar polarising powers.
(ii) Predict and explain the pH of the solution formed when the chloride of R
dissolves in water. [2]
pH = 3 ( accept any value from 1 to 4 )
Due to the high charge density of Be2+, it undergoes hydrolysis whereby it
weakens and breaks the O-H bond in H 2 O, readily donating a proton. Hence the
solution becomes acidic.

(b) Another class of chlorides, acid chlorides, can react with alcohols to form esters which
are sweet-smelling. Esters have been used as flavourings in food and perfumes. For
example, ethyl propanoate is an ester with pineapple-like smell.

The kinetics of the hydrolysis of ethyl propanoate in alkaline medium at T K was


studied.

Two separate experiments were carried out with varying concentrations of aqueous
sodium hydroxide solution. For each experiment, the concentration of ethyl
propanoate was determined at regular time intervals as the reaction proceeded.

The graph below shows the results of Experiment 1 with [NaOH] = 2.0 mol dm-3. The
results of Experiment 2 with [NaOH] = 1.0 mol dm-3 are shown below.

trendyline
656 trendyline
9

[CH3CH2COOCH2CH3] / mol dm-3

Experiment 1

Time / min

trendyline
[Turn over
657 trendyline
10

Time Experiment 2 with [NaOH] = 1.0 mol dm3


/ min [CH 3 CH 2 COOCH 2 CH 3 ] / mol dm3
0 0.0200
25 0.0152
50 0.0115
75 0.0088
100 0.0067
125 0.0051
(i) Plot the results of Experiment 2 on the same graph above. Label your graph
clearly. [1]

(ii) Use the two graphs to determine the order of reaction with respect to
CH 3 CH 2 CO 2 CH 2 CH 3 and NaOH, showing your working clearly.
[2]

To find the order of reaction with respect to the ester.


Consider the graph for experiment 1.
1st t 1/2 = 31.0 min (with construction lines shown on the graph)
2nd t 1/2 = 32.0 min (with construction lines shown on the graph)
Since t 1/2 is almost constant, order of reaction with respect to ethyl propanoate =
1

Alternative answer
Can also use the graph for experiment 2 to find two t 1/2 values.
1st t 1/2 = 63.0 min (with construction lines shown on the graph)
2nd t 1/2 = 63.0 min (with construction lines shown on the graph)
Since t 1/2 is constant, order of reaction with respect to ethyl propanoate = 1

To find the order of reaction with respect to NaOH.


For experiment 1, initial rate = 0.010/24.0 = 4.17 x 104 mol dm3 min1
For experiment 2, initial rate = 0.010/48.0 = 2.08 x 104 mol dm3 min1
When [NaOH] is doubled, initial rate is also doubled (4.17x10-4/2.08x10-4 = 2).
Hence order of reaction with respect to NaOH = 1

Alternative answer
Consider the graph for experiment 2.
t 1/2 = 63.0 min
t 1/2 for experiment 2 = (2)(t 1/2 for experiment 1)
reaction rate is halved when [NaOH] is halved.
Hence order of reaction with respect to NaOH = 1

trendyline
[Total: 7]

658 trendyline
11

4 2-propanol is a colourless, flammable liquid with a strong odour. It is widely used as a


solvent in many industrial applications.
(a) 2-propanol can be used as a fuel in the fuel cell. At the anode of the cell,
2-propanol is oxidised to carbon dioxide. The electrons pass around the external
circuit to the cathode. The protons formed from the oxidation move through the
conducting polymer electrolyte to the cathode, where they react with oxygen to
produce water.
(i) Construct ion-electron equations for the reactions at the anode and cathode
respectively. [2]
+
Anode: CH 3 CH(OH)CH 3 + 5H 2 O &2 2 + 18H + 18e
Cathode: O 2 + 4H+ + 4e + 2 O
(ii) Hence construct the equation for the overall reaction. [1]
2CH 3 CH(OH)CH 3 + 9O 2 &2 2 + 8H 2 O
(iii) The cell is capable of producing an e.m.f. of 1.56 V. By using suitable data
from the Data Booklet, suggest a value for the Eo of the
CO 2 /CH 3 CH(OH)CH 3 electrode reaction.
[2]
1.56 = +1.23 Eo anode
Eo anode = -0.33V
(iv) Suggest a possible advantage of using the 2-propanol fuel cell compared to
a hydrogen fuel cell. [1]
2-propanol is a liquid but hydrogen is a gas at rtp. 2-propanol is easier to
store and transport.

(b) 2-propanol reacts with hot excess concentrated sulfuric acid to form propene. The
reaction follows a 3-step mechanism as described below:
Step 1 The oxygen atom on 2-propanol is protonated by H 2 SO 4 forming
intermediate A and HSO 4 - in an exothermic reaction.
H
+
O H
CH3 C CH3

trendyline
tr
rrendyline
endyline
H

Intermediate A
Step 2 he C-O
The C-O bond in intermediate A breaks, forming a ca
C- carbocation B
and water
water. This step is endothermic
nd and is the rate determining
step.

[Turn over
659 trendyline
12

Step 3 The carbocation is then deprotonated in an exothermic reaction,


forming propene. H 2 SO 4 is regenerated.

(i) State the type of reaction undergone when 2-propanol reacts with hot
excess concentrated sulfuric acid. [1]
Elimination
(ii) Use the information given in the table above to draw out the full mechanism
for the reaction between 2-propanol and concentrated sulfuric acid,
indicating the movement of electrons using curly arrows and showing
clearly the lone pair of electrons involved in the mechanism. You are
advised to use structural formulae for all species, so that it is clear which
bonds are broken and which are formed. [3]

H
H O O +
O
O H
CH3 C CH3 + H O S OH +
-
O S OH
CH3 C CH3
H O O
H

H
+
O H +
CH3 C CH3 + O
CH3 C CH3 H H
H
H

H O O
+ - = +
CH3 C C H + O S OH CH3CH CH2 HO S OH

O O
H H

(iii) Using the information given in the table above, sketch a fully labelled
energy profile diagram of the reaction between 2-propanol and
concentrated sulfuric acid. [2]

t
660 trendyline
13

(c) 2-propanol reacts with methanoic acid in the presence of concentrated sulfuric
acid, producing an ester, isopropyl formate, and water. The equilibrium below is
set up during the reaction:
O OH O
+ + H 2O
OH O
isopropyl formate

If 0.500 mol of methanoic acid and 0.300 mol of 2-propanol were mixed with a
small amount of concentrated sulfuric acid and allowed to reach equilibrium, the
equilibrium mass of isopropyl formate was found to be 16.64 g.
(i) Write the expression for KC.
[1]
[isopropyl formate][H O]
2
KC = [methanoic acid][2-propanol]

(ii) Calculate a value for KC.


[2]
Amount of isopropyl formate = 16.64 / 88 = 0.189 mol
acid + alcohol ester + water
Initial amount / mol 0.500 0.300 0 0
Change / mol -0.189 -0.189 +0.189 +0.189
Eqm amount / mol 0.311 0.111 0.189 0.189

. .

KC =
. . = 1.03

(iii) A molecular sieve is a crystalline substance with pores of molecular


dimensions which permits the passage of very small molecules such as
water. The use of molecular sieves in the reaction above increases the yield
of isopropyl formate. Suggest a reason for the increase in yield. [1]
The molecular sieve removes water, causing concentration of water to
decrease. By LCP, equilibrium will then shift to the right to replenish water,
hence increasing the yield.
[Total: 16]

trendyline
[Turn over
661 trendyline
14

5 (a) Hydrastine is a nitrogen containing drug used in the treatment of memory-related


disorders such as Alzheimers disease and schizophrenia and has the structure
shown below.

N
O CH3

O
O
OCH 3

Hydrastine
A chemist planned a synthetic route to synthesise Hydrastine from
1,2-dimethylbenzene through the intermediates, compounds W and X, as shown
below.

N
CH3 CH 2 OH O
a few steps step A many steps
CH3
O
O
CH3 O
O
OH
O
Compound W Compound X O
OCH 3

Hydrastine

(i) Hydrastine can be hydrolysed in either


I acidic or
II basic conditions.

Draw the structures of the products obtained for each case. Assume that the

trendyline
ther
he group (C-O-C)
ether (C-O-C) is inert. [2]

662 trendyline
15

O O

+ H
N N
O CH3 O CH3

OH OH
-
O OH O O
OCH 3 O OCH 3 O

Acidic conditions basic conditions


Remarks: In this case, since no specific acid or base is given, you may leave ans as
charged species. In cases where qns give a specific acid/base and asked for organic

compound formed, pls give compound in your answers (ie. COONa instead of COO for
example)

(ii) Devise a reaction scheme to help him synthesise compound X via compound W
from 1,2-dimethylbenzene. Show clearly the reagents and conditions required for
each step.
[4]
O

CH3 KMnO4,H2SO4, CH 2 OH
OH limited LiAlH4
heat O
O
CH3 dryether
OH
OH

Conc. H2SO 4, heat

(iii) State the oxidation numbers of the carbon atoms, labelled with an *, in
1,2-dimethylbenzene and compound X.

*
*
CH3
O

CH3 O

trendyline
[2]
In 1,2
1,2-dimethylbenzene,
dimethylbenzene, C = -3
-3
In compound X , C = -1

(iv) Compare
ompare
mpare the relative acidity of the alcohol and the carb
carbo
carboxylic acid groups
resent in compound W
present W.
The R-CH 2 OH is less acidic as compared to R-COOH; The electron donating R
group intensifies the negative charge on the oxygen of the conjugate base
[Turn over
663 trendyline
16

(alkoxide ion), making it less stable; The R-COOH is more acidic because the
negative charge can be delocalised across two electronegative oxygen which
are both electron withdrawing, thus dispersing the charge and stabilising the
conjugate base, making it more acidic.

[Total: 11]

6 (a) Boiling raw eggs in hot spring water at pH = 9.2 produces black eggs (kuro tamago)
through a chemical reaction with geothermal heat and volcanic gas as iron properties
ahere to the porous shells. The black colour of iron sulfide is a result of a reaction with
hydrogen sulfide, and it produces boiled eggs with black shells.

Iron sulfide is precipitated according to the following reaction.

reaction 1: Fe2+(aq) + S2(aq) FeS(s)

Hydrogen sulfide gas from volcanic systems is released into the hot spring water and
behaves as a dibasic (diprotic) weak acid.
H 2 S(g) + aq 2H+(aq) + S2(aq)

In a saturated solution of hydrogen sulfide,

[H+]2[S2-] = 1.0 x 10-23 mol3 dm-9

(i) Calculate the maximum concentration of sulfide ions present in hot spring water.
[2]
+ 9.2
[H ] = 10
= 6.31 1010 mol dm3
[S2] = = 2.51 105 mol dm3

(ii) Hence, calculate the minimum concentration of Fe2+ present in hot spring water
in order for precipitation to occur.

(K

trendyline
K sp
[1]]
of

K sp = [Fe2+][S2]
FeS = 4.9 1 18
10 18
mol2 dm6)

664 trendyline
17

[Fe2+] = = 1.95 1013 mol dm3

(iii) Use the following data, together with relevant data from the Data Booklet, to
calculate for reaction 1 with the aid of an energy cycle.

standard enthalpy change of formation of FeS(s) 102 kJ mol1


standard enthalpy change of atomisation of Fe(s) +415 kJ mol1
standard enthalpy change of atomisation of S(s) +279 kJ mol1
sum of first two electron affinities of S(g) +337 kJ mol1
enthalpy change of hydration of Fe2+(g) 1981 kJ mol1
enthalpy change of hydration of S2(g) 1372 kJ mol1
[4]

102 = 415 + 279 + 337 + 762 + 1560 1981 1372 +


= 102 kJ mol1
Remarks: Do reflect the magnitude for each enthalpy change by the size of the
gap between each energy level (the ans here failed to do so) when constructing
your level diagram.

trendyline
dylin
n
(b) When a precipitate is formed, , in J mol1, is giv
given by the follow
following expression.

(i) Use the data given in (a)(ii) to calculate , in kJ mol-1, for FeS. [1]

[Turn over
665 trendyline
18

= 2.303 8.31 298 lg(4.9 1018)


= 98.7 kJ mol1

(ii) Use your answer in (a)(iii) and (b)(i) to calculate , in J mol-1 K-1, for the
formation of the precipitate FeS(s) at 298 K. [1]

98.7 = 102 298


= = 11.1 J mol1 K1
(c) Geraniol (C 10 H 18 O) appears as a clear to pale-yellow oil that is insoluble in water but
soluble in most common organic solvents. It has a rose-like scent and is commonly
used in perfumes.

OH

Geraniol

Geraniol reacts with HBr(g) to form the two possible products:

I: Br Br OH

OH
II: Br Br

(i) State which of the products would be the major product. Explain your answer. [3]
I would be the major product. This is due to the fact that the tertiary carbocation
intermediate formed after the addition of H is more stable than that of II, which
forms a secondary carbocation, due to more electron donating alkyl groups
attached to the carbocation which stabilises the cation more by dispersing the
positive charge.
(ii) Draw a diagram to illustrate the bonding in any of the C=C bond in geraniol in
terms of orbital overlap. [2]

trendyline
666 trendyline
19

[Total: 14]

[Turn over
667 trendyline
2

1 (a) At room temperature, the electrode potential E for the system


[MnO 4 -(aq) + 8H+(aq)], [Mn2+(aq) + 4H 2 O(l)] / Pt
is given by the equation
-
0.059 [MnO4 (aq)] [H+ (aq)] 8
E = E + log
5 [Mn2+ (aq)]

where E = +1.51 V. E values at various pH are given below when


[MnO 4 -(aq)] = 0.01 mol dm-3 and [Mn2+(aq)] = 0.0001 mol dm-3

pH E/V
0 +1.52
3 +1.24
7 +0.83

(i) Define what the symbol E refers to. [1]


It refers to the standard electrode potential under standard conditions of
298K, 1 atm and 1 mol dm-3 electrolyte used.
(ii) Calculate the electrode potential of the cell at pH 1, when the
manganate(VII) ion concentration is 0.01 mol dm-3 and the manganese(II)
ion concentration is 0.001 mol dm-3. [2]
pH = 1
[H+] = 10-1 = 0.100 mol dm-3
0.059 (0.01) [0.1] 8
E = +1.51 + log = + 1.43 V
5 [0.001]

(iii) Predict at which pH value, from the table above, the solution is the most
oxidising. [1]
At pH=0

The electrolytic conductivity of a solution in a cell is given by the following


formula:
(c) (l)
 
a

trendyline
where c is the conductance of the
e cell and a is
s the cross-sectional
cross-secti area of the
odes,
des, separated by a distance l.
electrodes, l.
(iv) l
Often the term is replaced by a cell constant K. Calculate the cell constant
a
of a particular cell if it was found that the conductance of the cell containing

668 trendyline
3

Mn2+(aq) of concentration 0.100 mol dm-3 LV  -1 and the electrolytic
FRQGXFWLYLW\ZDV-1 cm-1. State the units of K. [2]
0.011 = K (0.022)
K = 0.500 cm-1
(b) A diaphragm cell consisting brine solution, concentrated NaCl, is used in the
commercial electrolytic production of chlorine and sodium hydroxide.

product

(i) Write the ion-electron equations for the reactions occurring at the cathode
and at the anode. [2]
Anode: 2Cl- Cl 2 + 2e-
Cathode: 2H 2 O + 2e- H 2 + OH-

(ii) The chlorine gas is collected separately at the anode to prevent a side
reaction that will occur with one of the products formed. Write a balanced
equation for this side reaction that occurs at room conditions. State the
oxidation numbers of chlorine in all the products formed. [2]
Cl 2 + 2OH- Cl- + ClO- + H 2 O
Oxidation no. of Cl in Cl- = -1 Oxidation no of Cl in ClO- = +1

(iii) In an electrolytic experiment using this diaphragm cell, 1.0 mol of gas is
produced at the cathode. Calculate the mass of chlorine gas that is produced
in the same experiment. [1]

trendyline
1 mol
m of H 2 SURGXFHGPRORI&O
R 2 produced
duce
Mass of Cl 2 produced = 1 x 71.0 = 71.0 g

669 trendyline
[Turn over
4

(c) Chlorine contains electrons which occupy the n=2 principal quantum shell. Draw all
the possible shapes of orbitals at the energy level, n=2. [2]

(d) Explain why aqueous manganese(II) ions are coloured. [3]


2+
Mn has partially-filled d-orbitals. When the ligands approach the central metal ion, splitting
of the d-orbitals occurs.
7KHenergy gap, E, between the non-degenerate orbitals corresponds to the wavelength
of light in the visible region of the electromagnetic spectrum.
:KHQDQHOHFWURQIURPDlower energy d orbital is promoted to a higher energy d orbital (d-
d electron transition), radiation corresponding to E is absorbed. The complementary colour
will be seen as the colour of the complex.

(e) Cocaine, C 17 H 21 NO 4 , was first used as a local anaesthetic. It is also a powerful


stimulant and its structure is shown below.
O O
O CH3
O

N
CH3

Cocaine

Student X was asked to suggest a suitable synthetic route to prepare cocaine from
methylbenzene and compound A.
O OH

HO

N
CH3

trendyline
l
nt X suggested the following steps:
Student s:
Compoun A
Compound

670 trendyline
5

Step 1 Methylbenzene is oxidised with hot, acidified KMnO 4 to give benzoic


acid.
Step 2 Benzoic acid is converted into benzoyl chloride by reacting the
resultant solution from step 1 with thionyl chloride.
Step 3 Benzoyl chloride is reacted with compound A at room temperature.
Step 4 The resultant compound from step 3 is reacted with methanol in the
presence of concentrated sulfuric acid.
(i) Give two reasons why the method suggested would not work. [2]
1. In step 2, thionyl chloride will hydrolyse in the presence of aqueous
solution
2. The final product will be the protonated form of cocaine since it is
prepared in an acidic medium.
3. Heat is required for step 4.
(ii) Draw the aromatic organic product formed when cocaine undergoes acidic
hydrolysis. [1]
O

OH

Cocaine is smuggled and inhaled as its protonated hydrochloride salt, known as


cocaine hydrochloride. Smoking cocaine is more stimulating than inhaling the salt
as it is absorbed quickly by the capillaries in the lung tissues. The salt is hence
converted back to cocaine before smoking. This is also done as the salt will
decompose before it vapourises.

(iii) Suggest a suitable reagent to convert cocaine hydrochloride back into


cocaine. [1]
Dilute NaOH
[Total: 20]

trendyline
671 trendyline
[Turn over
6

2 Researchers at the Max Planck Institute of Chemistry conducted a study to determine


whether the chemicals emitted by human beings via their breath and their skin vary in
response to audiovisual stimulus. In one of the experiments carried out during the
screening of The Hunger Games 2 in a cinema, the researchers detected that there
were spikes of emission of a diene, isoprene, twice; when the heroines dress catches
fire and when the final battle begins. Isoprene is associated with muscle contraction,
hence the spikes in emission could be due to unconscious muscle twitching as the
audience empathises with the heroine.

The structure of isoprene is shown below. In its pure form, it exists as a colourless
volatile liquid.

Isoprene

(a) Isoprene reacts with gaseous HBr at room temperature.


(i) Draw the structure of the major product of the reaction and hence state the
IUPAC name of the major product. [2]

Br
Br
2,3-dibromo-2-methylbutane
(ii) Name and illustrate with a diagram the type of stereoisomerism exhibited by
the major product of the reaction. [2]
Optical isomerism

CH3 CH3
CH3 Br CH3 Br
C C

C C
H Br Br
H
CH3 CH3

(b) Polyisoprene, a polymer of isoprene, is the primary chemical constituent of natural


rubber.

trendyline
672 trendyline
7

CH3 CH3
CH2 C CH2 C
n CH H < 0
CH
CH2 CH2
n
Isoprene Polyisoprene

(i) Predict, with a reason, the sign of the entropy change of the polymerisation
process of isoprene. [1]
Negative, because the system becomes less disorderly as the number of
molecules decrease, hence having less way to arrange
(ii) With reference to your answer in (b)(i), state and explain if the
polymerisation is more spontaneous or less spontaneous at higher
temperatures. [2]
G = H - TS
H and S are both negative. At high temperature, magnitude of TS is
larger than magnitude of H hence G is more positive.
Less spontaneous
(iii) Natural rubber is insoluble in water. With reference to the structure of
polyisoprene, explain why. [2]
Energy evolved from the formation of temporary dipoled-induced dipole
between polyisoprene and water is not sufficient to overcome the hydrogen
bonds between water molecules.
(c) In a Diels-Alder reaction, 2 moles of isoprene react to form 1 mole of limonene.
Limonene is a colourless liquid which possesses a strong smell of oranges. The
mechanism of the reaction proceeds as follow.

limonene

The enthalpy changes of vaporisation of isoprene and limonene are


+26.4 kJ mol-1 and +39.4 kJ mol-1 respectively.

cycle totrendyline
Using the data given and relevant data from the Data Booklet,
Booklet, cons
o calculate the enthalpy change for the reaction below.
construct an energy

673 trendyline
[Turn over
8

2
(l) (l)

Limonene

You may use C 5 H 8 and C 10 H 16 in your energy cycle to represent isoprene and
limonene respectively. [3]

;
Using bond energy,
H 1 = 2 BE(C=C) 4 BE(C-C) = 2(610) 4(350) = -180 kJ mol-1
By Hess Law,
H = 2(+26.4) +(-180) (+39.4) = --167 kJ mol-1
(d) Compound P is an isomer of limonene. When P is oxidised by hot acidified
potassium manganate(VII), Q, C 3 H 4 O 3 , and R, C 7 H 12 O 3 , are formed.

Both Q and R give orange precipitate with 2,4-dinitrophenylhydrazine and give


effervescence when reacted with solid calcium carbonate. Only Q gives a yellow
precipitate with warm alkaline iodine.

When Q reacts with NaBH 4 in methanol, S is formed. 1 mole of S gives 1 mole of


H 2 (g) upon reaction with excess sodium metal.

R reacts with phosphorus pentachloride, giving white fumes and T, C 7 H 11 ClO 2 .


When T reacts with ethereal LiAlH 4 , U, as shown below is formed.
O
O

trendyline
ompound U
co
com

Deduce
e the structures for compounds P T
T,, explaining the chemist
chemistry involved. [8]

674 trendyline
9

Information Deduction
Both Q and R give orange precipitate Q and R are ketones.
with 2,4-dinitrophenylhydrazine and Q and R has carboxylic acid group.
give effervescence when reacted with
solid calcium carbonate.
Only Q gives a yellow precipitate with Q has the structure CH 3 CO-
warm alkaline iodine.
When Q reacts with NaBH 4 in Reduction of ketone.
methanol, S is formed. S is secondary alcohol.
1 mole of S gives 1 mole of H 2 (g) Acid-metal reaction
upon reaction with excess sodium S has two OH groups
metal.
R reacts with phosphorus Substitution reaction
pentachloride, giving white fumes and T has acyl chloride
T, C 7 H 11 ClO 2 .
When T reacts with ethereal LiAlH 4 , Reduction of ketone to secondary
U is formed. alcohol
Followed by nucleophilic substitution
to form ester.

O OH
OH OH

O O
P Q S

O O
HO Cl

O O
R T

trendyline
Alternative

dy
ative answer for P:
Remarks:
P:
ks: Recognise that q is 8marks for 5 structures, and they asked for
chemistry
ry involved in each reaction, so the type of reaction for ea
very important, need to be stated clearly!
y!
each conversion is

[Total: 20]

675 trendyline
[Turn over
10

3 Compound U reacts with aqueous hydroxide ions when heated under reflux.
Cl
Cl

Compound U

The product of the reaction between U and hot aqueous hydroxide ions does not have
optical activity.
(a) (i) Describe the full mechanism of the reaction. [3]
Nucleophilic substitution (S N 1)


Cl
slow Cl + Cl
Cl + +

OH
-
Cl +
+ OH Cl

Remarks: clue of S N 1 from qns stem that the ion does not have optical
activity.

(ii) Explain why the product of the reaction does not exhibit optical activity. [2]
The intermediate formed in the first step is trigonal planar at the C with the
positive charge. The nucleophile can attack the carbon from the top or the
bottom of the plane, forming a racemic mixture where rotation of plane-
polarised light by one enantiomer is cancelled out by the other enantiomer.
(b) When 0.100 mol dm-3 of hydroxide ions react with 0.00100 mol dm-3 of compound
U DW&WKHKDOI-life of compound U is 25 s.
(i) Define the term half-life. [1]
Half-life is the time taken for the concentration of a reactant to reduce to half
its original value.
(ii) Using your answer in (a)(i), state the rate equation of the reaction. [1]
Cl
Rate = k[ Cl ]

(iii) Calculate the rate FRQVWDQWDW&VWDWLQJLWVXQLWV>@

trendyline
tre
ln 2 ln 2
k= = =0.0277
alf life 25
half
s-1

(iv) d -33 of hydr


Iff the reaction is repeated using 0.100 mol dm hydroxide ions and
dm-3 of U at the
002 moll d
0.002 e same temperature, state the half-life
h
half of U in the
new experiment. [1]
25 s

676 trendyline
11

(v) With the aid of a clearly labelled Boltzmann distribution curve, explain why
the rate of the reaction decreases when the temperature of the reaction
decreases. [2]

T 1 <T 2
At lower temperature, kinetic energy of molecules decreases, less
molecules have energy greater than or equal to activation energy.
Frequency of effective collision decreases, thus rate of reaction decreases.
(c) 0.450 g of U was reacted completely with excess hot aqueous sodium hydroxide.
The reaction mixture was cooled and acidified with excess HNO 3 . Calculate the
mass of silver chloride formed when an excess of AgNO 3 (aq) was added.
[2]
Amount of U = 0.450 / 175 = 0.002571 mol
Amount of AgCl = 0.002571 mol
Mass of AgCl = 0.002571 (108+35.5) = 0.369 g
(d) Compound V can be formed from compound U in 3 steps.
O
N
Cl H

compound V

Propose a 3-step synthesis of V from U, indicating clearly all reagents and


conditions used. Draw the structures of all intermediates. [5]
Cl CN
ethanolic KCN
Cl Cl
heat under reflux

trendyline
n
LiAllH4 in
LiAlH4
dry eth
heerr

CH2NH2
C6H5COC
COCll
CO
compound V Cl

677 trendyline
[Turn over
12

(e) Compound W is an isomer of U.


Cl

Cl
compound W

When W is boiled with aqueous sodium hydroxide, it immediately loses water to


form an organic compound X which gives a yellow precipitate with a medicinal
smell when reacted with warm iodine in KOH(aq).
Suggest the structure of X and write a balanced chemical equation for the reaction
of X with warm iodine in KOH(aq). [2]

O
-
O
+ 3I + 4OH + CHI + 3I + 3H O
2 3 2
O O

[Total: 20]

4 (a) Boron is a metalloid chemical element in Group III. However due to its smaller
size compared to other Group III elements, its charge over mass ratio is closer to
that of silicon. Hence it resembles silicon in many reactions.
(i) Predict the pH of the solution when BCl 3 is added into water. Hence write
an equation for the reaction. [2]
pH 2 (accept between 1 to 2)
BCl 3 + 3H 2 O B(OH) 3 + 3HCl
Also accept:
2BCl 3 + 3H 2 O B 2 O 3 + 6HCl
(ii) Explain why CCl 4 does not undergo the same reaction as BCl 3 when it is
added to water. [1]
B in BCl 3 has energetically accessible empty orbital to accept lone pair from
O of H 2 O but C in CCl 4 do not.
(iii) The oxide of boron reacts with NaOH the same way SiO 2 reacts with
NaOH. One of the products of the reaction is Na 3 BO 3 . Prop
Propose a chemical
equation for the reaction between the oxide of boron
equati boro and NaOH.
1]
[1]
B 2 O 3 + 6NaOH 2Na 3 BO 3 + 3
3H 2 O
(b) The following equilibrium exists when BCl 3 (g) is mixed with NH 3 (g).

678 trendyline
13

BCl 3 (g) + NH 3 (g) Cl 3 BNH 3 (s)


(i) Draw a dot-and-cross diagram of the Cl 3 BNH 3 molecule including its
co-ordinate (dative covalent) bonds. [1]

(ii) Suggest the shape and the bond angle of the BCl 3 molecule. [1]
WULJRQDOSODQDU
(iii) Predict the effect of decreasing the temperature on the above equilibrium.
Explain your answer. [3]
Reaction is exothermic since it involves formation of dative bonds
By LCP, position of equilibrium will shift to the right
This is to increase the temperature of the system by favouring the
exothermic reaction.
(iv) At a total initial pressure of 3 atm, a 1 : 2 mixture of BCl 3 and NH 3 was
DOORZHG WR UHDFK HTXLOLEULXP DW  & ,W ZDV IRXQG WKDW WKH HTXLOLEULXP
partial pressure of BCl 3 was 0.68 atm.
Determine the value of the equilibrium constant, K p , for this reaction at
&VWDWLQJLWVXQLWV [2]
BCl 3 (g) + NH 3 (g) Cl 3 BNH 3 (s)
Initial partial pressure 1 2 0
Change / atm -0.32 -0.32 +0.32
Eqm partial pressure 0.68 1.68 0.32

1
Kp = =0.875 atm-2
(.)(.)

(c) Keratin is a fibrous protein that acts as the key structural component of hair. The
amino acid content of keratin in hair of human varies slightly with ethnicity.
The hair sample of an Asian was analysed. Eight of the amino acids in a keratin
molecule present in the hair are listed below.

trendyline
tre
endyyline
mino
n acid
Amino Formula of side
e chain Number
Num of amino acid
ac residues
(R in RCH(NH 2 )COOH) per molecule of keratin
cysteine
ysteine
steine -CH
CH 2 SH 1200
glutamic
amic acid -CH 2 CH 2 COOH
H 1026
serine -CH 2 OH 990

679 trendyline
[Turn over
14

threonine -CH(OH)CH 3 572


leucine -CH 2 CH(CH 3 ) 2 520
aspartic acid -CH 2 COOH 478
glycine -H 464
valine -CH(CH 3 ) 2 456

(i) Use any three of the above amino acids to construct the displayed formula
of a possible section of the protein chain of keratin. [2]
H H O H H H O
N C C N C C N C C
H H O H C H
H C H O
S H
H
or any other combination

(ii) Describe how a polypeptide chain of keratin is held in the shape of an


-helix. [2]
The a-helix is held in place via hydrogen bonds formed between O of -C=O
of a peptide and H of NH of another peptide.
(iii) The cysteine residues in the keratin molecule can form disulfide bridges.
Illustrate this process by means of a balanced equation.

Assuming all the disulfide bridges are intramolecular, what is the maximum
number of bridges which could be formed within each keratin molecule? [2]
2 -CH 2 SH -CH 2 S-SCH 2 - + 2H
600
(iv) Apart from the disulfide bridges mentioned in (c)(iii), describe, by means of
diagrams, two other types of side-chain interaction, using suitable pairs of
amino acids from the table above. [2]

trendyline
680 trendyline
15

(v) Glutamic acid has three pK a values: 2.1, 4.1 and 9.5.
Suggest the structure of the major species present in solutions of glutamic
acid at pH 7. [1]

NH3
- -
O O
C C
O O

[Total: 20]

5 Fluorine is a highly toxic pale yellow diatomic gas under standard conditions. As the
most electronegative element, it is highly reactive.
Hydrogen fluoride is also a highly toxic gas which forms corrosive hydrofluoric acid
upon contact with water.
(a) State and explain which gas, fluorine or hydrogen fluoride, displays greater
deviation from ideal gas behaviour. [2]
Hydrogen fluoride is held by hydrogen bonds and fluorine is held by td-id.
Hydrogen fluoride has stronger intermolecular forces of attraction hence has
greater deviation from ideal gas behaviour.
(b) Fluorine reacts with hydrogen to form hydrogen fluoride.
Write an equation with state symbols for the reaction and describe how you would
expect the reaction of hydrogen with fluorine to differ from that of hydrogen with

trendyline
e.
chlorine. [2]
H 2 (g) + F 2 (g)
(g 2HF(g)
2HF(g
ne
e reacts explosively and more vigorously with hydrogen
Fluorine hydroge compared to
e with hydrogen.
chlorine hydrogen

681 trendyline
[Turn over
16

(c) Hydrofluoric acid is a weak acid. It forms a buffer solution with salts containing its
conjugate base.
Calculate the mass of solid sodium fluoride required to be added into a 250 cm3
solution of 0.00420 mol dm-3 of HF to form a buffer solution of pH 5.6.
[K a of HF = 7.20 10-4 mol dm-3]
[2]
pH = pK a + lg ([salt]/[acid])
5.6 = -lg(7.20 10-4) + lg ([NaF]/0.0042)
[NaF] = 1.204 mol dm-3
mass of NaF = 1.204 (250/1000) (23.0 + 19.0) = 12.6 g

(d) Hydrofluoric acid is a poison with greater hazards than strong acids even though it
is a weak acid. It can react with calcium in the blood leading to hypocalcemia and
potentially causing death through cardiac arrhythmia. This is due to the formation
of sparingly soluble calcium fluoride in the body.
(i) The solubility of calcium fluoride in water is 2.707 10-4 mol dm-3. Calculate
the solubility of calcium fluoride in a solution of 0.00200 mol dm-3 of aqueous
NaF. [2]
2+ -
CaF 2 (s) Ca (aq) + 2F (aq)
K sp = [Ca2+][F-]2 = (2.707 10-4)(2 2.707 10-4)2 = 7.93 10-11 mol3 dm-9
7.93 10-11 = y (2y + 0.002)2 \  2
y = 1.98 10-5 mol dm-3

(ii) Explain briefly how the presence of NaF affects the solubility of calcium
fluoride. [1]
CaF 2 (s) Ca2+(aq) + 2F-(aq)
The presence of NaF increases [F-], hence by LCP, shifting the equilibrium to
the left to partially offset the increase in F. Hence, solubility decreases.
Remarks: Pls rmb to write your eqm equation before explanation, otherwise,
0 mark.

(e) HCl is a reagent often used in organic reactions.


(i) For each of the following types of reaction, write a balanced equation for a

trendyline
reaction
eaction involving
involvi HCl
HCl ass a reagent or as a catalyst. You
Yo may use any
organic
rganic compound
compoun as the starting material for each
e reaction.
[3]
3]]
I Hydrolysis
any correct equation showing hydrolysis of ester, amide or nitrile

682 trendyline
17

II Neutralisation
any correct equation showing acid-base reaction of amine or
carboxylate salt
III Addition
any correct equation showing addition equation of alkene forming
alcohol (must balance equation with water)

(ii) A student proposed the following reaction for the conversion of


2-methylphenol to 4-chloro-2-methylphenol using aqueous hydrochloric acid
as the reagent.

CH3 CH3

HCl(aq)
OH rtp Cl OH

2 methylphenol 4 chloro 2 methylphenol

Explain why the reaction he proposed is not likely to work.


Hence suggest the correct reagent and conditions for the conversion. [3]
The reaction involves electrophilic substitution of benzene. Hence a
electrophile Cl+ is required for the reaction
However in HCl, since Cl is more electronegative, Cl+ is not likely to be
formed.
Cl 2 in CCl 4 at room temperature

(iii) Suggest a simple chemical test to distinguish between 2-methylphenol and


phenylmethanol. State clearly the reagent and conditions used as well as the
observations expected. [2]
OH

phenylmethanol

K 2 Cr 2 O 7 in dilute H 2 SO 4 , heat

trendyline
orange
range solution turns green for phenylmethanol
phenylmethan
range solution remains
orange rem
remains 2-methylphenol
ns orange for 2-
2-methylphenol
OR
R
Br 2 (aq) ;
orange solution decolourises for 2-methylphenol, forming white ppt

683 trendyline
[Turn over
18

orange solution remains orange for phenylmethanol, no ppt formed


OR
neutral FeCl 3 ;
violet colouration formed for 2-methylphenol
no violet colouration for phenylmethanol

(iv) Compound Z can be formed from 2-methylphenol in two steps.


O
O

O
O
compound Z

Propose a two-step synthesis of Z from 2-methylphenol, stating the reagents


and conditions for each step and showing the structure of the intermediate.
Your synthesis should involve 2-methylphenol as the only organic
reactant. [3]

CH3 COOH
KMnO4 dil H2SO4
heat
OH OH

PCl5

O
O

O
O

[Total: 20]

trendyline
684 trendyline
Calculator Model / No.

Name: ____________________ Class: 15S_________ Reg Number:


______

MERIDIAN JUNIOR COLLEGE


JC2 Preliminary Examination
Higher 2

____________________________________________________________________

Chemistry 9647/01
Paper 1 Multiple-Choice Questions 23 September 2016

1 hour
Additional Materials: Data Booklet
OMR Answer Sheet

INSTRUCTIONS TO CANDIDATES

Write your name, class and register number in the spaces provided at the top of this page.

There are forty questions in this section. Answer all questions. For each question, there are
four possible answers labelled A, B, C and D. Choose the one you consider correct and
record your choice in soft pencil on the OMR answer sheet.

Read very carefully the instructions on the OMR answer sheet.

You are advised to fill in the OMR Answer Sheet as you go along; no additional time will be
given for the transfer of answers once the examination has ended.

Use of OMR Answer Sheet

Ensure you have written your name, class register number and class on the OMR
Answer Sheet.
Use a 2B pencil to shade your answers on the OMR sheet; erase any mistakes cleanly.
Multiple shaded answers to a question will not be accepted.

For shading of class register number on the OMR sheet, please follow the given
examples:

trendyline
y
If your register
g number is 1, then shade
de 01 in the
e inde
index number column.
egister number is 21, then shade 2
If your register 21 in
n the index number column.
co

This document consists of 23 printed pages

MJC Prelim 2016


685 trendyline
Answer all questions in this section.

For each question there are four possible answers, A, B, C, and D. Choose the one
you consider to be correct.

1 Use of the Data Booklet is relevant to this question.

Sodium percarbonate, (Na 2 CO 3 ) x y(H 2 O 2 ), is an oxidising agent in some home and


laundry cleaning products.

20.0 cm3 of 0.0500 moldm3 sodium percarbonate releases 48.0 cm3 of carbon
dioxide at room conditions on acidification.

An identical sample, on titration with 0.0500 moldm3 K 2 Cr 2 O 7 , requires


20.0 cm3 before the first green colour appears. K 2 Cr 2 O 7 reacts with H 2 O 2 in the
mole ratio of 1 : 3.

y
What is the ratio ?
x

1 2 3
A B C D
3 3 1

2 Magnesium nitrate(V), Mg(NO 3 ) 2 decomposes on heating to form magnesium


oxide, nitrogen dioxide and oxygen, while sodium nitrate(V), NaNO 3 decomposes to
formNaNO 2 and oxygen only.

Mg(NO 3 ) 2
MgO + 2NO 2 + O 2

NaNO 3
NaNO 2 + O 2

A mixture of magnesium nitrate(V) and sodium nitrate(V) was heated until no more
gases were evolved. The water soluble part of the residue was used to prepare
1.00 dm3 of solution. 10.00 cm3 of this solutionwas reacted with 30.00 cm3 of
0.0200 moldm3 of acidified potassium manganate(VII) solution.

2MnO 4 + 6H+ + 5NO 2


2Mn2+ + 5NO 3 + 3H 2 O

The excess potassium manganate(VII) solution required 10.00 cm3 of


0.0500 moldm3ethanedioic acid solution for complete reaction

2MnO 4 + 5(COO) 2 2 + 16H+


2Mn2+ + 10CO 2 + 8H 2 O

trendyline
What is the
he mass of sodium nitrate(V) in the mixture?
Given M r of NaNO 3 = 85.0

A 85
5g
0.85 B 1 36
1. 6g
1.36 C 4.25
25 g
4.25 D 8
8.50 g
3 h D
Use of the t B
Data kl t iis relevant
Booklet l t to
t this
thi question.
ti

Dinitrogen tetraoxide, N 2 O 4 , has a melting point of 11 C and a boiling point of 21


MJC Prelim 2016 2 [Turn Over
686 trendyline
C.

A sample of 18.4 g of N 2 O 4 (s) is placed in a 24 dm3 vessel under an atmosphere of


helium gas chilled to 20 C at a pressure of 1.0 atm. When the vessel is warmed
to 25 C and the contents are allowed to reach equilibrium, 40 % of the N 2 O 4 (g)
has dissociated into NO 2 (g).

What is the total pressure of the gases in the vessel at 25 C?

A 0.28 atm B 1.28 atm C 1.38 atm D 1.46 atm

4 Use of the Data Booklet is relevant to this question.

The ion X4+ contains 46 electrons and 69 neutrons.

Which of the following statements about X4+ or X2+ is correct?

A X4+ is deflected to approximately twice the extent as Rb+ when subjected to


an electric field.
B X4+has the same electronic configuration as Sr2+.
C X2+ undergoes hydration in water to a greater extent than X4+.
D X2+ can be oxidised by Fe3+.

5 Use of the Data Booklet is relevant to this question.

Which particle contains a single unpaired electron?

A a molecule of N 2 O
B one of the particles formed after the heterolytic fission of a HBr molecule
C the vanadium ion in VO 2
D the chromium ion in Cr 2 O 3

trendyline
MJC Prelim 2016 3 [Turn Over
687 trendyline
6 Use of the Data Booklet is relevant to this question.

In which of the following pairs of compounds would the first compound have a lower
melting point than the second compound?

A CH 3 CH(NH 2 )COOH and CH 3 CH(OH)COOH


B CH 3 CH 2 CH 2 CH 3 and CH 3 CH(CH 3 )CH 3
C GaF 3 , GaCl 3
D Na 2 S, MgF 2

7 Compound X has the following properties.


It is very hard.
It is a lubricant.
It can conduct electricity in the solid state.

What is the most likely structure of compound X?

A simple molecular
B giant molecular
C giant metallic
D giant ionic

8 The MaxwellBoltzmann distribution curve shown below is for a chemical reaction in


living systems.
fraction of molecules
with a given energy

fraction of molecules with


energy > Ea at a particular
temperature

kinetic
0 Ea energy
(activation energy)

Which statement is not true?

A The total area under the curve is 1.

trendyline
B The activation energy will decrease with a decrease in temperature.
C When
hen temperature is increased, the peak of the curve becomes
become lower.
D The
e shaded area of the curve will decrease when enzymes are denatured.
9 ne
e reacts with iodine in the presence of an acid:
Propanone

CH 3 COCH 3 + I 2
H+
CH 3 COCH 2 I + HI
MJC Prelim 2016 4 [Turn Over
688 trendyline
The mechanism involves the following steps.

+

H
CH 3 COCH 3 CH 3 C(OH)CH 2 slow
CH 3 C(OH)CH 2 + I 2
CH 3 CI(OH)CH 2 I fast
CH 3 CI(OH)CH 2 I
CH 3 COCH 2 I + HI fast

Which graph would be obtained?

A [I2]
[reactant] / mol dm -3 B [I2]

0
time
time/s rate

C +
D [CH3COCH3][H ]
+
[CH3COCH3][H ]

time rate

10 The following data was obtained from the studies of the reaction between O 2 and
NO in a vessel at constant temperature.

Experiment 1 2 3
Initial total pressure of O 2 and NO/ atm 1.00 1.30 1.80
Initial partial pressure of O 2 / atm 0.60 0.90 1.20
Initial rate of reaction/ atm s1 1.08 1.62 4.86

Which of the following statements is correct regarding the above system?

A The order of reaction with respect to O 2 is zero.


B The rate constant k has units of atm1s1.
C The rate equation is rate = k [NO][O 2 ]2.
D The overall order of the reaction is three.
11 Aluminium m chloride exists in two different forms in the
th vapour state. When some
Al 2 Cl 6 was
as added into a reaction vessel, the following equilibrium is slowly
s set up.

Al 2 C
Cll 6 (g) 2AlCl
2Al
2AlCl 3 J 
J H
 H = +ve

At different times during the experiment, changes were made to the conditions in
the reaction vessel. At each time, there was only one change made to the
MJC Prelim 2016 5 [Turn Over
689 trendyline
conditions in the reaction vessel.

The change in the concentrations in the mixture with time is shown in the graph
below.
Concentration

Al2Cl6

AlCl3

t1t2t3 time

Which conclusion can be drawn from the graph?

A At t 1 , the pressure of the system was increased by increasing the volume.


B At t 2 , a catalyst was added to the system.
C The temperature of the mixture at t 2 was higher than that at t 1 min.
D The equilibrium constant, K p ,remains the same at t 1 and t 3 .

trendyline
MJC Prelim 2016 6 [Turn Over
690 trendyline
12 Citrate buffers are commonly used to control pH in household cleaners and
pharmaceuticals.

H3C6H5O7 + H2O H 2 C 6 H 5 O 7 + H 3 O+ pK a1 = 3.1

H2C6H5O7 + H2O HC 6 H 5 O 7 2 + H 3 O+ pK a2 = 4.8

HC 6 H 5 O 7 2 + H 2 O C 6 H 5 O 7 3 + H 3 O+ pK a3 = 6.4

A citrate buffer with pH 5.1 was prepared by dissolving solid NaH 2 C 6 H 5 O 7 and
Na 2 HC 6 H 5 O 7 . The total sodium concentration of the prepared buffer was 1.0
moldm3.

What are the concentrations of NaH 2 C 6 H 5 O 7 and Na 2 HC 6 H 5 O 7 in this buffer?

[NaH 2 C 6 H 5 O 7 ] / moldm3 [Na 2 HC 6 H 5 O 7 ] / moldm3


A 0.10 0.20
B 0.20 0.40
C 0.20 0.10
D 0.40 0.30

13 Use of the Data Booklet is relevant to this question.

Excess aqueous (COOH) 2 was added to an acidified solution of


potassium dichromate(VI) in a conical flask. Effervescence was observed until all
(COOH) 2 had completely reacted. The resulting solution was then left to stand in
air for a long time.

Given : 2CO 2 + 2H+ + 2e (COOH) 2 E= 0.44V

What is the colour of the final solution?

A Blue B Green
C Yellow D Orange

trendyline
MJC Prelim 2016 7 [Turn Over
691 trendyline
14 When heated, magnesium carbonate decomposes to form carbon dioxide and
magnesium oxide.

$JUDSKLFDOSORWRI*YHUVXV7GHVFULELQJWKHFKDQJHRIWKHGibbs free energy of


the decomposition of magnesium carbonate with respect to temperature, is shown
below.

*N-PRO-1

200

180

160

140

120

100

80

60

40

20

0
0 100 200 300 400 500 600 700 800 900 100011001200
T/K
Using the information from the graph, what is the value of 6ofor the
decomposition reaction?

A + 6.22 x 103J mol1 K1 B + 6.04 x 102 J mol1 K1


C + 1.61 x 102 J mol1 K1 D + 1.80 x 106 J mol1 K1

trendyline
MJC Prelim 2016 8 [Turn Over
692 trendyline
15 Use of the Data Booklet is relevant to this question.

An electrochemical cell is set up as shown:

T = 298K

Ni salt bridge Pt

Ni2+ Fe2+ Fe3+

[Ni2+] = 0.1moldm3 [Fe3+] = [Fe2+] = 0.1moldm3

Which statement about the e.m.f. of the above cell is correct?

A The e.m.f of the cell is +1.02 V.


B Addition of water to the Fe3+ / Fe2+ halfcell increases the e.m.f of the cell.
C Increasing the mass of the nickel electrode decreases the e.m.f of the cell.
D Addition of excess aqueous ammonia to the Ni2+/ Ni halfcell increases the
e.m.f of the cell.

16 In which row of the table are all statements comparing calcium and barium as well
as their hydroxides correct?

Solubility of
Melting point Flame Test
hydroxide

Calcium Barium Calcium Barium Ca(OH) 2 Ba(OH) 2

A higher lower green red lower higher

B higher lower red green lower higher

C higher lower white green higher lower

D lower higher red green higher lower

trendyline
MJC Prelim 2016 9 [Turn Over
693 trendyline
17 Element Z is in Period 3 of the Periodic Table. The following four statements were
made about the properties of element Z or its compounds.

Three statements are correct descriptions and one is false.

Which statement does not fit with the other three?

A Element Z burns in oxygen with a bright white flame.


B Element Z is a solid at room temperature which does not conduct electricity.
C The oxide ofelement Z reacts with water to form a solution that turns red on
addition of Universal Indicator.
D Adding NaOH (aq) to the solution resulting from the reaction of the chloride
of Z with water produces a white precipitate which is soluble in an excess of
NaOH (aq).

18 Use of the Data Booklet is relevant to this question.

Paramagnetism is the property of being attracted to a magnetic field. Many


transition compounds that contain unpaired electrons tend to be paramagnetic
while those that do not contain unpaired electrons tend to be diamagnetic. Some
compounds are more paramagnetic than others as they contain more
unpaired electrons.

A blue solution of VO2+ undergoes the following reactions.

MnO4/ H+
VO2+ VO2+

excess Sn2+ excess Sn

solutionY
Which of the following statements is incorrect?

A Solution Y contains [V(H 2 O) 6 ]3+.

B Both Sn2+ and Sn act as reducing agents.

C The reacting ratio of VO2+ and MnO 4 is 5 : 1.

D VO2+ is diamagnetic while VO 2 +is paramagnetic.

19

trendyline
udents, W
Three students,
operties
W,, Y and Z Z,, were asked to draw bar charts to represent
perties of the halogens
some properties

Their diagrams
re
halogens and their compounds differ in magnitude.

shown.
grams are shown
magnit
how

Student Y

MJC Prelim 2016 10 For H2 (g) + X2 (g)


2HX (g)Over
[Turn
694 trendyline
o
H
Student W
Ka of HX

Cl Br I
Student Z
For reaction of X2 and cold NaOH (aq)
Oxidation number of
X in XOn formed

0
Cl Br I
Which of the students diagrams are correct?

A both W and Z B both W and Y


C both Y and Z D none of the diagrams

20 How many isomers (both structural and stereoisomers) are there for a compound
with the formula C 4 H 10 O that can react with Na (s)?

A 3 B 4 C 5 D 6

trendyline
MJC Prelim 2016 11 [Turn Over
695 trendyline
21 Isopropylcyclohexane reacts with bromine gas to form different
monosubstituted products. Three of the products are shown below.

Br
Br

Limited Br2 (g) Br


uv

A B C

Given that the rate of abstraction of primary, secondary and tertiary


hydrogen is 2: 3: 6 respectively, what is the expected ratio of the mono
substituted products A: B : C formed?

A 6:2:3 B 1:1:1
C 1:2:2 D 1:6:4

22 Which of the following reactions will not incorporate deuterium (D) into any
2
of the organic products formed? (D= 1H, an isotope of hydrogen)

O
A NaOH, D2O
reflux
O

O
B D2SO4, D2O
reflux
O

O
NaOD, D2O
C reflux
O

O
D2SO4, D2O
D reflux

ttrendyline
r
O

23 t t iis commonly
Phenyl acetate l used
d iin th
h llaboratory
the b t as a precursor for organic

MJC Prelim 2016 12 [Turn Over


696 trendyline
synthesis.

O CH3

O
phenyl acetate

Which pair of compounds would produce phenyl acetate in high yield when reacted
together?

OH O
A
HO CH3

O-Na+ O
B
HO CH3

O Cl
C CH3OH
O

OH O
D
Cl CH3

Use of the following reaction scheme is relevant for Q24 and Q25.

NO2 NH2 N N Cl CN CH2NH2


HNO2, HCl CuCN
I II III IV

24
Which of the following statements is correct about the reaction scheme?

A The reagent for stage I is LiAlH 4 .


B Stage II is neutralisation reaction.
C Stage III is nucleophilic substitution reaction.
D The reagent for stage IV is hot aqueous NaOH.

25
trendyline
Stage IIis
with other
yes
useful dyes.
alt, which can undergo coupling
s theformation of the diazonium salt, cou
compounds
err organic compounds, such as phenol, to form azo compo
compou
reactions
which are

MJC Prelim 2016 13 [Turn Over


697 trendyline
N N Cl OH

+ N=N OH + HCl

azo compound
diazonium salt

What is the structure of the azo compound formed when P and Q react together in
the presence of HNO 2 and HCl?

CH3
HO3S NH2 N
CH3
P Q

A HO3S N=N OH

CH3
B HO3S N=N N
CH3

CH3 CH3 CH3


C N N N
CH3

CH3 CH3
D HO3S N N

trendyline
MJC Prelim 2016 14 [Turn Over
698 trendyline
26 Williamson ether synthesis is a very useful reaction in the formation of ethers via
the S N 2 mechanism shown below.

- -
RO R' X RO R' X

Which of the following compounds will be formed as a major product when


compound Y undergoes Williamson ether synthesis followed by acidification?

Br

OH OH
compound Y

O
O
A B
O
OH
O HO
C D O
OH

27 When a conjugated diene undergoes electrophilic addition with Br 2 , it forms two


products through the 1,2addition and the 1,4addition, which is shown in the
mechanism below.

- Br
Br Br Br
diene
Br Br

Br
Br
Br Br
, -a -
1 2 ddition 1,4 addition

Which of the following statements is not correct?

A The overall rate law is second order.

trendyline
ne
e
B The carbocation intermediates are resonance stabilised.

C The 1,2addition
1,2addition product formed when HCl
e 1,2 HCl is used is Cl .
Cll
C
D The
e 1,4addition
1,4
1,4 dition product formed when IICl
addition Cl is used is I.

MJC Prelim 2016 15 [Turn Over


699 trendyline
28 Which of the following shows the given species arranged in order of decreasing K b
values?

CH2O OCH3 NO2

O O
CH 3 CH(I)COO

I II III IV

A I, III, IV, II B I, IV, III, II


C II, III, IV, I D II, IV, III, I

29 The rod cells at the back of the eye contain a primary alcohol called retinol
which is responsible for their sensitivity to light. Retinol is oxidised by an
enzymecatalysed reaction that keeps the double bonds intact, to retinal, an
aldehyde.

OH

retinol

Which of the following is correct?

A The number of stereoisomers in retinol is 32.


B There are six bonds formed by 2sp22sp3 overlap in a molecule of
retinol.
C In laboratory preparations, retinal can be formed from heating retinol
under reflux with hot potassium dichromate(VI).
D One mole of retinol reacts with 4 moles of hydrogen gas in the
presence of palladium catalyst.

30
trendyline
b l
The table below i
gives i f i pertaining
some information i i to three
h i acids.
amino id

MJC Prelim 2016 16 [Turn Over


700 trendyline
isoelectric
amino acid structure
point
O
lysine H 2N CH C OH 9.6
(lys)
(CH2 )4 NH 2

O
serine H 2N CH C OH 5.7
(ser)
CH2 OH

O
aspartic acid
H2 N CH C OH 2.9
(asp)
CH 2COOH

A mixture of these three amino acids can be separated by electrophoresis.

Which of the following diagrams shows the result of the separation of the
amino acids mixture at pH 7.0?

anode cathode anode cathode


A B

ser asp lys asp ser lys

anode cathode anode cathode


C D

lys ser asp lys asp ser

trendyline
MJC Prelim 2016 17 [Turn Over
701 trendyline
For questions 31 to 40, one or more of the three numbered statements 1 to 3 may be
correct. Decide whether each of the statements is or is not correct (you may find it
helpful to put a tick against the statements which you consider to be correct.)

The responses A to D should be selected on the basis of


A B C D
1, 2 and 3 are 1 and 2 only 2 and 3 only
1 only is correct
correct are correct are correct
No other combination of statements is used as a correct response.

31 In a closed reaction vessel of 10 dm3 maintained at a temperature of 150 C,


gaseous hydrazine decomposes into nitrogen and hydrogen. The system reaches
equilibrium with a total pressure of 1 atm.

N 2 H 4 (g) N 2 (g) + 2H 2 (g)

The average M r of the equilibrium gas mixture in the 10 dm3 vessel is found to be
20.

Which of the following statements are correct?

1 In liquid form, both reactants and products flowing from a burette remain
undeflected when subjected to close vicinity to a charged rod.

2 The mass of the gaseous mixture inside the reaction vessel is 5.75 g.

3 The following graphs depicts the correct behaviour of the three gases under
standard conditions.

N2H4
S9
Q57
N2

H2
1.0 ideal gas

trendyline
The responsess A to
A
y
o D should be selected on the basis of
B C D

MJC Prelim 2016 18 [Turn Over


702 trendyline
1, 2 and 3 are 1 and 2 only 2 and 3 only
1 only is correct
correct are correct are correct

No other combination of statements is used as a correct response.

32 Metal thiocyanates have very different industrial uses. For example, calcium
thiocyanate,Ca(NCS) 2 , is used as a stiffening agent of fabrics.

The following data is provided:


H latt o (Ca(NCS) 2 ) = 2118 kJ mol1
o
H hyd (NCS ) = 310 kJ mol1
H hyd o (Ca2+) = 1577 kJ mol1

Which of the following statements are correct?

1 The magnitude of Ho soln of Ca(NCS) 2 is 79 kJ mol1.

2 Magnitude ofHo latt of Ba(NCS) 2 is smaller than that of Ca(NCS) 2 .

3 Go soln of Ca(NCS) 2 is negative at all temperatures.

33 The graph below shows the variation in the standard enthalpy change of
YDSRULVDWLRQ H(vaporisation), for eight consecutive elements in the Periodic
Table, all with atomic number, Z 20.
1
H (vaporisation) / kJ mol
Q
P

S V

R T
U
Which of the following statements are correct?
atomic number, Z
1 The chloride of element Q is acidic in aqueous solution.
2 The oxide of element R has a higher melting point than that of
element S.
3

trendyline
tre
end
The responsess A to
A
dylin
dy
Electrical

ne
o D should be selected on the basis of
B C
W..
ectrical conductivity decreases from element P to W

MJC Prelim 2016 19 [Turn Over


703 trendyline
1, 2 and 3 are 1 and 2 only 2 and 3 only
1 only is correct
correct are correct are correct

No other combination of statements is used as a correct response.

34 A lithiumion battery consists of: a cathode made from cobalt oxide, CoO 2 ; an
anode made from graphite with lithium atoms inserted between the layers.

During discharge, Li atoms at the anode give up electrons to become Li+ ions. The
electrons travel round the external circuit, and are picked up by the cathode. A Li+
ion from the electrolyte also moves to the cathode. This is illustrated in the following
diagram in which CCCCC is a simplified representation of a layer of carbon
atoms in graphite.

external circuit
e
e
Co O Co O C C C C C

O O
Li+
Co O Co O Li
C C C C C
O O
electrolyte
cathode anode

Co O Co O C C C C C

O O Li+
Li+
Co O Co O C C C C C

O O
electrolyte
cathode
Which of the following statements are true?

1 The mass of lithium metal that is deposited when a current of 3 A is passed


through the battery during the charging process in 1 h 20 min is 1.03 g.
2 The oxidation number of cobalt oxide cathode after discharge is +3.

3 The bonding between the lithium atoms and the layers of carbon atoms is
ionic in nature.

trendyline
MJC Prelim 2016 20 [Turn Over
704 trendyline
The responses A to D should be selected on the basis of
A B C D
1, 2 and 3 are 1 and 2 only 2 and 3 only
1 only is correct
correct are correct are correct
No other combination of statements is used as a correct response.

35 A chemist allowed a halogen, X 2 , to react completely with S 2 O 3 2 (aq). This


resultant acidic solution was treated with an excess of aqueous barium nitrate to
produce a white precipitate which is insoluble in dilute nitric acid.

Another halogen, Y2, gave a different sulfurcontaining product with


S 2 O 3 2 (aq).

Which of the following statements are correct with regards to the chemistry
described?

1 A possible identity for X 2 is Br 2 .

2 A possible identity for Y 2 is I 2 .

3 Y 2 is a stronger oxidising agent than X 2 .

36 Which statements concerning the first row transition metal ions are incorrect?

1 Addition of K 2 CO 3 (aq) to CrCl 3 (aq) produces a green precipitate of


Cr 2 (CO 3 ) 3 .
2 Addition of KI (aq) to Fe 2 (SO 4 ) 3 (aq) produces a brown precipitate of FeI 3 .

3 Addition of NaOH (aq) to K 2 CrO 4 (aq) produces an orange solution of


K 2 Cr 2 O 7 (aq).

37 A catalytic converter is part of the exhaust system of modern cars.

Which reactions occur in a catalytic converter?

y
1 2C x H y + (4x + y) NO
2x CO 2 + y H 2 O + (2x + ) N 2
2
2 2CO + 2NO
2CO 2 + N 2

trendyline
tre
end
dylin
dy ne
3 O 2 + NO
CO CO + NO 2
The responsess A to
o D should be selected on the basis of
A B C D

MJC Prelim 2016 21 [Turn Over


705 trendyline
1, 2 and 3 are 1 and 2 only 2 and 3 only
1 only is correct
correct are correct are correct

No other combination of statements is used as a correct response.

38 A non-cyclic organic compound has the molecular formula C 5 H 9 O 2 N.

Which pair of functional groups could be present in this molecule?

1 one ketone group and one amide group

2 one ester group and one nitrile group

3 one carboxylic acid group and one nitrile group

39 Doxorubicin is an anti-cancer drug used to treat a wide range of cancers


includingblood cancers, likeleukaemiaandlymphoma.

[The ether group, R-O-R is inert].


OH
O OH
O

OH

O O OH O NH2

O
OH

doxorubicin

Which of the following statements about doxorubicin are true?

1 Addition of sodium boron hydride causes six atoms of hydrogen to be


incorporated into the molecule.
2 On reacting with ethanoyl chloride, six moles of ethanoyl chloride is used up

trendyline
per mole of doxorubicin.
3 On
n reacting with thionyl
hionyl chloride, five moles of thionyl ch
chloride
chl is used
up per mole of doxorubicin.
doxorubicin

MJC Prelim 2016 22 [Turn Over


706 trendyline
The responses A to D should be selected on the basis of
A B C D
1, 2 and 3 are 1 and 2 only 2 and 3 only
1 only is correct
correct are correct are correct
No other combination of statements is used as a correct response.

40 A reaction pathway diagram is shown.

Which organic reactions could have such a profile?


H H
H H
C C + H SO H C C H
2 4
1
H H
H OSO3H

Br

2 + HBr
+ Br2

O O O
O O
C2H5 C2H5 base
+ H C2H5 C2H5
O O
O O
H H
3
HO

trendyline END OF PAPER


PAPER

MJC Prelim 2016 23 [Turn Over


707 trendyline
2016 MJC H2 Chemistry Prelim Paper 1 (9647/01)
Suggested Answers

Qn Ans Qn Ans Qn Ans Qn Ans


1 C 11 D 21 C 31 C
2 D 12 B 22 C 32 A
3 D 13 B 23 D 33 B
4 D 14 C 24 C 34 A
5 C 15 D 25 B 35 B

6 D 16 B 26 B 36 A
7 B 17 D 27 C 37 B
8 B 18 D 28 C 38 D
9 B 19 A 29 B 39 B
10 D 20 C 30 B 40 A

trendyline
708 trendyline
Calculator Model / No.

Name: ____________________ Class: 15S_________ Reg Number:


______

MERIDIAN JUNIOR COLLEGE


JC2 Preliminary Examination
Higher 2

__________________________________________________________________________

Chemistry 9647/02
Paper 2 Structured Questions 19September 2016

2 hours
Additional Materials: Data Booklet

INSTRUCTIONS TO CANDIDATES Examiners Use


Write your name, class and register number in the
spaces provided at the top of this page. Paper 1 MCQ / 40

Write your calculator brand and model/number in the Q1 / 12


box provided above.
Q2 / 17
Answer all questions in the spaces provided on the
question paper. Q3 / 15
All working must be shown clearly.
Paper 2 Q4 /9

INFORMATION FOR CANDIDATES Q5 /7

The number of marks is given in brackets [ ] at the Q6 / 12


end of each question or part question.
Total /72
You are reminded of the need for good English and
clear presentation in your answers. Paper 3 / 80

Total / 192

Percentage

trendyline
yline
e
Grade

This document consists of 28printed pages

MJC Prelim 2016


709 trendyline
1 Planning (P)

Lead iodide, PbI2 , is a sparingly soluble salt.

PbI2 (s) Pb2+ (aq) + 2I (aq)

One method of determining the relative solubility and an approximate value of


the solubility product of the salt, K sp , would be through direct observation. Two
standard solutions (one of a soluble lead(II) salt, and the other of a soluble
iodide salt) in different proportions are mixed and allowed to stand and reach
an equilibrium. In some mixtures, the ionic product of the particular solution
exceeds the K sp value and precipitation of PbI2 crystals will occur. In other
mixtures, the final concentrations of lead and iodide ions will be such that
precipitation does not occur.
The experimental ionic product can be determined for each solution. In the
experiment, the value of K sp lies between the ionic product values for solutions
with precipitate and those values for solutions without precipitate. The value for
K sp can be given as:

Maximum ionic product Minimum ionic product


<Ksp<
that does not gives ppt that givesppt

The value for K sp can be estimated by determining the mean between the
above two ionic products. In order for a more precise determination of K sp , the
concentration of the solution mixtures should be prepared in a manner that
would result in a relatively small range of ionic products.
In the experiment, two standard solutions of lead(II) nitrate and potassium
iodide are prepared and then diluted appropriately using relevant volumes of
the solutions and water. To avoid localised supersaturation that brings about
erroneous precipitation, water has to be mixed in first.
In order to predetermine the concentrations required, a student prepared two
solutions, A and B.
Solution A contains 2.50 cm3 of 0.0100 mol dm3 lead(II) nitrate, Pb(NO 3 ) 2
with 5.50 cm3 of deionised water added.
Solution B contains 2.00 cm3 of 0.0100 mol dm3 potassium iodide, KI.

Mixing these above solutions (at 25C) produced the yellow precipitate of PbI2 .
These solutions provide an ionic product which just exceeds the K sp of PbI2 .

trendyline
MJC Prelim 2016 2 [Turn Over
710 trendyline
(a) Calculate the ionic product of PbI2 in this given mixture to determine a
rough estimate of the K sp value of PbI 2 .
[1]

(b) Consider the description of the precipitation experiment given in this


question.

Write a plan for such a series of four experiments to determine a value for
the K sp of PbI2 . In your plan, you should use the same total volume as
described when solutions A and B are mixed.

Your plan should ensure that


at least two of your experiments do not produce a precipitate
the estimated range for the value of K sp is relatively small. (< 10%
deviation from the estimated ionic product in (a)).
You should use the quantities specified in (a) in one of your experiments.

You may assume that you are provided with


solid lead nitrate, Pb(NO 3 ) 2
0.0400 mol dm3 aqueous potassium iodide, KI,
deionised water,
the equipment and materials normally found in a school or college
laboratory.

Your plan should include the following.


details, including quantities, for the preparation of a 100 cm3 stock
solution of 0.0100 mol dm3 of Pb(NO 3 ) 2
details, including quantities for the preparation of a 100 cm3 stock
solution of 0.0100 mol dm3 of KI
suitable format summarising appropriate quantities of solutions to
be used in each experiment (for four experiments),

trendyline
the corresponding concentrations and calculated ionic
centrations an ion products
outline of all essential experimental procedures
brief, but specific details on how the K sp value can be estimated
from the experiment.
[8]

MJC Prelim 2016 3 [Turn Over


711 trendyline
trendyline
MJC Prelim 2016 4 [Turn Over
712 trendyline
trendyline
MJC Prelim 2016 5 [Turn Over
713 trendyline
(c) Explain how the solubility of PbI2 would change if PbI2 is dissolved in a
solution of hydroiodic acid, HI.
[1]

(d) Lead(II) salts are toxic and prolonged exposure affects the mental
development in children. Pb2+ ions can be removed from contaminated
water through precipitation by adding a soluble salt solution containing the
appropriate anion.
The solubility products of some sparingly soluble lead(II) salts, at 25 C
are given in the table below.

Salt K sp
PbCO 3 7.4 1014 mol2 dm6
Pb(IO 3 ) 2 3.7 1013 mol3 dm9
Pb 3 (PO 4 ) 2 7.9 1043 mol5 dm15

Calculate the solubility of the above lead(II) salts in mol dm-3at 25C.
Hence, state the most effective anion in removing Pb2+ from the
contaminated water sample.
[2]

trendyline [Total: 12]

MJC Prelim 2016 6 [Turn Over


714 trendyline
2 Amines are a class of compounds that are widely used in both the
pharmaceutical and agricultural industry. An aminecontaining drug, Tamiflu is
an effective drug against the H1N1 strain of influenza A virus. Another amine,
ethylamine, CH 3 CH 2 NH 2 , is widely used in the production of herbicides.
(a) In order to determine the standard enthalpy change of neutralisation,
60.0 cm3 of 0.370 mol dm3 aqueous ethylaminewas placed in a
polystyrene cup. Dilute HCl was added and after each addition, the
mixture was stirred and the temperature noted.
The highest temperature reached was recorded and plotted on the graph
below.

Graph of temperature of mixture against volume of HCl added


30

X
X
Temperatue of mixture / C

X
29 X
X
X
28 X

X
27

26
0 10 20 30 40 50
Volume of HCladded / cm3

(i) Calculate the initial pH of aqueous ethylaminein the polystyrene cup.


Base dissociation constant of ethylamine is 5.6 104 mol dm3.
[1]

trendyline
MJC Prelim 2016 7 [Turn Over
715 trendyline
(ii) Calculate the concentration of dilute HCl used in the experiment.
[1]

(iii) Calculate the enthalpy change of neutralisation for the reaction


between HCl and ethylamine.
(You may assume the density of the solutions is 1.00 g cm3 and
their specific heat capacity is 4.18 J g1 K1.)
[2]

(iv) With reference to your answer in (a)(iii), explain why the temperature
of the mixture was reported to be higher when the above experiment
was repeated using an aliquot of aqueous potassium hydroxide of
the same concentration as that of the ethylamine sample.
[1]

trendyline
MJC Prelim 2016 8 [Turn Over
716 trendyline
(b) In another experiment, a sample of 0.640 mol dm3 aqueous ethylamine
was added to 10.0 cm3 dilute HNO 3 in a conical flask. The equivalence
point can be detected by a distinct colour change brought about by the
use of a suitable indicator.
The graph below shows the change in pH of the reaction mixture against
the volume of ethylamine added.

pH

Volume of ethylamine added / cm3

(i) Suggest a suitable indicator for this acidbase titration and state the
colour change observed at the equivalence point.
[1]

Indicator:

Colour
change:

(ii) Circle clearly on the graph, the region within which the mixture is
acting as a buffer.
[1]

(iii) Write an equation to suggest how the solution in (b) (ii) acts a buffer
upon the addition of a small amount of aqueous sodium hydroxide.

trendyline
[1]

MJC Prelim 2016 9 [Turn Over


717 trendyline
(c) An agricultural chemistsynthesised ethylamine from ethanal in an
industrial laboratory via a twostep procedure as shown in the scheme
below.

I II
CH 3 CHO CH 3 CH=NOH CH 3 CH 2 NH 2
NH 2 OH, H+

(i) Suggest the type of reactions for both steps and the reagents and
conditions for step II.
[2]

Step Type of reaction Reagents and conditions

II

(ii) Suggest a chemical test that the chemist can perform to confirm
that theprocedure is complete.
[1]

trendyline
MJC Prelim 2016 10 [Turn Over
718 trendyline
(d) Use the following data and relevant data from the Data Booklet to
construct an energy cycle to calculate the enthalpy change of formation of
CH 3 CH 2 NH 2 (l).
Enthalpy change of vapourisation of CH 3 CH 2 NH 2 (l) = +29 kJ mol1
Enthalpy change of atomisation of C(s) = +715 kJ mol1
[3]

(e) The structure of Tamiflu is shown below.

O
O
O O

NH
NH2

(i) Name the two nitrogencontaining functional groups present in

trendyline
Tamiflu.
[1]

MJC Prelim 2016 11 [Turn Over


719 trendyline
(ii) Compound Q is a derivative of Tamiflu while compound R is an
aromatic compound with similar functional groups as Tamiflu.

O
O
O
O O O
O O

NH
NH
N
H CH3 NH2

compound Q compound R

Describe and explain the relative basicities of the following


compounds:

Compound Q and Tamiflu


[1]

Compound R and Tamiflu


[1]

trendyline [Total: 17]


MJC Prelim 2016 12 [Turn Over
720 trendyline
3 Halogens have a wide range of uses, such as in water purification and as
antiseptics. Halogens form compounds, such as halides and oxoanions which
are used in photography and as oxidising agents respectively.

(a) When potassium iodide reacts with concentrated sulfuric acid, violet
vapour is observedand a pungent gas is detected.

(i) Write achemical equationfor the reaction to account for the formation
of the violet vapour.
[1]

(ii) When potassium bromide reacts with concentrated sulfuric acid,


different observations are made.

By quoting suitable values from the Data Booklet, explain the


difference in the observations.
[2]

(iii) In order to prepare HBr as the only brominecontaining product from


KBr, concentrated H 3 PO 4 should be used instead of concentrated
H 2 SO 4 .

Suggest a plausible reason for this. Write a balanced equation for


this reaction.
[2]

trendyline
MJC Prelim 2016 13 [Turn Over
721 trendyline
(b) Precipitates are formed whenaqueous silver nitrate is added to separate
tubes containing aqueous potassium chloride, aqueous potassium
bromide and aqueous potassium iodide.

(i) Describe the effect of adding concentrated NH 3 to each of the


resulting mixtures.
[1]

Tube containing KCl

Tube containing KBr

Tube containing KI

(ii) Provide a qualitative explanation to substantiate your answer for


observations in the tube containing KI.
[2]

trendyline
MJC Prelim 2016 14 [Turn Over
722 trendyline
(c) 5.0104mol of a bromate salt containing theBrO 4 nanion was added to
20.0 cm3 of acidified KI present in excess to yield iodine and bromide
ions.
The remaining solution was made up to 250 cm3 with distilled water.
A 25.0 cm3 aliquot of the resultant solution required 40.0 cm3 of
0.010 mol dm3 sodium thiosulfate, Na 2 S 2 O 3 , for complete reaction.

Calculate the value of n.


[2]

MJC Prelim 2016 15 [Turn Over


723 trendyline
(d) When heated in chlorine, magnesium forms magnesium chloride.
Magnesium can also be heated in air to give magnesium oxide. Similarly,
aluminium and zinc can be vapourised and reacted with air to form the
respective oxides.

(i) Describe the reactions, if any, of magnesium chloride and


magnesium oxide with water, suggesting the pH of the resulting
solutions and writing equations where appropriate.
[3]

(ii) Zinc oxide has acidbase properties that are similar to aluminium
oxide. Write equations for two reactions which demonstrate these
properties of zinc oxide.
[2]

trendyline [Total: 15]

MJC Prelim 2016 16 [Turn Over


724 trendyline
4 The Williamson reaction is widely used in the laboratory and industrial
synthesis of ethers. It is the simplest and most popular method of preparing
ethers.

In a typical Williamson reaction, an alkoxide ion is prepared in situ and is


reacted with a halogenoalkane according to the general equation shown below.

RONa+ + RX
ROR + NaX

where R,R are alkyl groups and X is Br or Cl.

The following experimental procedure details the synthesis of


(3methylbutoxy)benzene using the Williamson reaction.

+
OH O Na O
Na (CH3)2CHCH2CH2Br

Stage 1 Stage 2

(3-methylbutoxy)benzene

PROCEDURE:

Preparation of (3-methylbutoxy)benzene

1 2.90 g of sodium metal is added to a dry round-bottom flask equipped with a


dropping funnel and a condenser. This apparatus set-up is shown below.
The set-up is protected from atmospheric moisture with the use of a calcium
chloride guard tube.

CaCl2 guard tube

dropping funnel

thermometer

trendyline
ndy
n
nd
dyylin
d condenser

round bottom flask ccontaining


reaction mixture

MJC Prelim 2016 17 [Turn Over


725 trendyline
2 Prepare an ethanolic solution containing 11.8 g of phenol. Add dropwise to the
reaction mixture using the dropping funnel. If the reaction becomes too vigorous,
cool the flask with a cold towel until the reaction is again under control.

3 Transfer 20.0 cm3 of 1-bromo-3-methylbutane to the dropping funnel and add it


to the flask slowly over 3 4minutes.
4 Boil the reaction mixture gently over a water bath for 30 minutes.

5 Evaporate as much of the ethanol solvent as possible.

Purification of (3-methylbutoxy)benzene

6 Add about 20 cm3 of water to the residue in the flask and separate the organic
layer using a separating funnel.

7 Wash the organic layer with sodium hydroxide.

8 Then, wash the organic layer with dilute sulfuric acid and water again.

9 Add anhydrous magnesium sulfateto the organic layer.


10 Distill the organic layer to purify (3-methylbutoxy)benzene at a suitable
temperature.

molar physical boiling


density / solubility in
substance formula mass / state at point /
g cm3 water
g mol1 r.t.p. C
ethanol C 2 H 5 OH 46 liquid 0.789 soluble 78
phenol C 6 H 5 OH 94 solid 1.07 insoluble 182
1bromo3methylbutane C 5 H 11 Br 151 liquid 1.21 insoluble 120
(3methylbutoxy)benzene C 11 H 16 O 164 liquid 0.91 insoluble 216
sodium bromide NaBr 103 solid 3.21 soluble
water H2O 18 liquid 1.00 100
Table 4: Physical Properties of the reagents and the organic product

(a) (i) Suggest why it is necessary to protect the apparatus setup from
moisture using the calcium chloride guard tube instep 1.
[1]

trendyline
MJC Prelim 2016 18 [Turn Over
726 trendyline
(ii) Determine the theoretical yield (in g) of(3methylbutoxy)benzene for
this reaction. You may assume that the reaction between sodium
and phenol to form phenoxide in stage 1 is complete.
[2]

+
OH O Na O
Na (CH3)2CHCH2CH2Br

Stage 1 Stage 2

(3-methylbutoxy)benzene

(iii) On the diagram of the separating funnel below, label the two layers
that will be observed after step 6.
[1]

ttrrrendyline
e
MJC Prelim 2016 19 [Turn Over
727 trendyline
(b) Other than the crude product, the organic layer (Step 6) also contains
several impurities. These impurities comprises of unreacted reactants,
byproducts or products from undesired reactions.
In the purification of the crude product (Steps 6 9), the impurities are
removed from the crude product with each subsequent addition of
chemicals.
(i) Suggest the impurity removed by each of the chemicals below.
[1]

1 sodium hydroxide

2 anhydrous magnesium sulfate

(ii) With reference to the data provided in Table 4, account for the
observed boiling points of ethanol and the product,
(3methylbutoxy)benzene.
In your answer, you should state the predominant type of
interactions present in the two compounds.
[2]

trendyline
MJC Prelim 2016 20 [Turn Over
728 trendyline
(iii) The final stage in the purification process is the distillation of the
crude product (Step 10).
Suggest why distillation is a suitable technique to obtain a pure
sample of (3methylbutoxy)benzene. Propose a suitable range of
temperature for the distillation setup.
[2]

[Total: 9]

trendyline
MJC Prelim 2016 21 [Turn Over
729 trendyline
5 On 21st April 2016, famed American pop artiste, Prince Rogers Nelson died of
an accidental overdose of the drug, fentanyl.
Fentanyl, an opioid analgesic was first synthesised in 1960. It was primarily
used for pain relief in clinical practice. Its efficacy is about 100 times that of
morphine.

HO

N N
N

O HO

Fentanyl Morphine

Fentanyl can be synthesised from 2bromoethylbenzene following the reaction


scheme below.

HN O
Reactant A

Br N N
Step 1 N O
Step 2

Step 3 NaBH4

Cl
N N Intermediate B
pyridine

O Step
ep 4
Fentanyl

MJC Prelim 2016 22 [Turn Over


730 trendyline
(a) Name and outline the mechanism for the reaction in step 1. Show relevant
lone pairs and dipoles, and use curly arrows to indicate the movement of
electron pairs.
[2]

(b) Suggest the displayed formula of reactant A in Step 2.


[1]

(c) Propose the structure of intermediate B.


[1]

MJC Prelim 2016 23 [Turn Over


731 trendyline
(d) Propanyl chloride is used in step 4 in the synthesis of fentanyl.
Explain why the use of propanoic acid as a replacement in step 4 is not
suitable.
[1]

(e) Suggest achemical test that can be used to distinguish between fentanyl
and morphine.
[2]

[Total: 7]

MJC Prelim 2016 24 [Turn Over


732 trendyline
6 Copper forms compounds with variable oxidation states and these compounds
are used in numerous industries such as in wastewater treatment and as
mineral supplements.

(a) The following scheme illustrates the reactions between different


compounds of copper.

pink solid

Cu2SO4 H2O,
white solid filter
Rochelle salt blue solution
CuSO4
D

excess CH3CH2NH2 (aq)

deep blue solution


C

(i) State the full electronic configuration of copper in the white solid.
[1]

(ii) With reference to relevant E data from the Data Booklet, describe
and explain the type of reaction undergone when the white solid
dissolves in water.
[2]

trendyline
MJC Prelim 2016 25 [Turn Over
733 trendyline
(iii) Identify the species in solution C that is responsible for the deep blue
colour and account for its formation from CuSO 4 . Include a relevant
equation in your answer.
[2]

(iv) Solution D is formed when Rochelle salt (an aqueous solution of


potassium sodium tartrate KNaC 4 H 4 O 6 ) is mixed with a solution of
CuSO 4 in basic medium.

Use the following data to determine the chemical formula of the


copper tartrate anion complex.
The composition by mass of the elements is
Cu: 17.76 % C: 26.85% H: 1.68% O: 53.71%

The chemical formula takes the form of [ML 2 ]n.


[2]

trendyline
MJC Prelim 2016 26 [Turn Over
734 trendyline
(b) Copper(II) ions are commonly used as homogeneous catalysts in the
removal of harmful cyanide ions, CN from wastewater by hydrogen
peroxide to form the less harmful cyanate CNO ions as illustrated in the
following reaction.

CN (aq) + H 2 O 2 (aq)
CNO (aq) + H 2 O (l)

By considering relevant Evalues from the Data Booklet, explain how


copper(II) ions act as homogeneous catalyst in this reaction, writing
balanced equations where appropriate.
[3]

Given: CNO + H 2 O + 2e CN + 2OH E= 0.97 V

trendyline
MJC Prelim 2016 27 [Turn Over
735 trendyline
(c) When aqueous glycine solution is added to a heated ethanolic solution of
copper(II) ethanoate, light blue needles of a planar, neutral
copperglycinate complex is obtained upon cooling.

H2N

O
-

O

glycinate ion

(i) Draw the threedimensional structure of the copperglycinate


complex ion.
[1]

(ii) State the type of isomerism exhibited by the copperglycinate


complex ion.
[1]

[Total: 12]

END OF PAPER

trendyline
MJC Prelim 2016 28 [Turn Over
736 trendyline
2016 MJC Prelim P2 Suggested Answers

1 Planning (P)

(a) Ionic product = (2.50 10-3) (2.00 10-3)2= 1.00 10-8 mol3dm9.Show working
to consider total volume is different.

(b) Preparation of 0.01 moldm3Pb(NO 3 ) 2 standard solution (100 cm3)


1) Weigh a clean, dry empty weighing bottle using a mass balance.
2) Weigh out approximately 0.331 g of solid Pb(NO 3 ) 2 .
3) Transfer the solid into a clean 100 cm3 beaker and reweigh the
weighing bottle with the residue to determine the exact mass of
Pb(NO 3 ) 2 used.
4) Dissolve the solid carefully with some deionised water and transfer it
into a 100 cm3 volumetric flask.
5) Top up the flask to the mark with deionsed water, stopper and mix
well to obtain a homogeneous solution.
Dilution of KIstandard solution (100 cm3)

1) Pipette 25.0 cm3 of given 0.0400 moldm3 PbI2 solution into a


100 cm3 volumetric flask.
2) Top of top the mark with deionised water, stopper and mix well.

Possible quantities and concentrations of solutions to be used(4 expt sets)

Vol. of Vol of Vol of


[Pb2+] (mixture) [I ] (mixture) Ionic product of
Expt Pb2+ I H2O
/ moldm3 / moldm3 PbI2
/ cm3 / cm3 / cm3

5 2.50 2.00 5.50 0.00250 0.00200 y 1 : 1.00 108

6 1.00 3.00 6.00 0.00100 0.00300 y 2 : 9.00 109

7 2.00 2.00 6.00 0.00200 0.00200 8.00 109

8 1.50 2.00 6.50 0.00150 0.00200 6.00 109

9 1.00 1.00 8.00 0.00100 0.00100 1.00 109

Experimental procedure
1) Fill three 50.00 cm3 burettes with the prepared 0.0100 moldm3Pb(NO 3 ) 2
stock solution, 0.0100 moldm3 KI stock solution and deionised water
respectively.

trendyline
2) Run out o 1.00 cm3 of deionised 5.00 cm3 of KI into a 100 cm3
d water and 5
conical
ical
cal flask / beaker.
3) Run out 4.00 cm3 off Pb(NO 3 ) 2 into the conical flask / beake
beaker and swirl the
flaskk well / stir with a glass rod
rod.
4) Allow w the flask / beaker to stand undisturbed
un 30 min to attain
for at least 3
equilibrium.

MJC Prelim 2016 1 chemistry unit @ mjc


737 trendyline
5) Note if any precipitate of yellow PbI 2 is present on standing.
6) Repeat the experiment using the various volumes shown in table
(Total volume = 10.00 cm3 / constant).
7) Estimated K sp = y1+y 2 where y 1 : minimum ionic product with precipitation
2
and y 2 :maximum ionic product without precipitation.

(c) Solubility would decrease.



PbI2 (s) Pb2+ (aq) + 2I (aq) (1)

HI (aq) H+ (aq) + 2I (aq) (2)

Due to presence ofcommon ion (I ), equilibrium position of (1) shifts left.

(d)
Salt Solubility
s2 = 7.4 1014
PbCO 3 Solubility = s = 2.72 10-7mol dm-3

4s3 = 3.7 1013


Pb(IO 3 ) 2 Solubility = s = 4.52 10-5mol dm-3

108s5 = 7.9 1043


Pb 3 (PO 4 ) 2
Solubility = s = 1.49 10-9mol dm-3

Most effective: PO 4 3-

4
2(ai) [OH] = 0.370 5.610 = 0.01439 moldm3
pOH= lg 0.01439 = 1.84 hence pH = 12.2

(ii) No. of mole of ethylamine = 0.370 60 = 0.0222


1000

[HCl] = 0.0222 = 1.11 moldm3


20
1000

(iii) Heat absorbed by solution =80 4.18 3.5 = 1170 J


60

trendyline
ylline
e
No. of moles
ol of H 2 O produced = 0.370
37 = 0.0222
0
1000
0
1170
170
Enthalpyy change of neutralisation
neutralisatio = = 52.7 kJ mol11
=
=
0.0222

(iv) Some energy


nergy evolved from the neutralisation
utra process is u
used to further
dissociate the weak base completely

MJC Prelim 2016 2 [Turn Over


738 trendyline
(bii)
pH

3
Volume of ethylamine added / cm

(bi) Methyl orange.Red to orange.

(iii)CH 3 CH 2 NH 3 + + OH
CH 3 CH 2 NH 2 + H 2 O

(ci)
Step Type of reaction Reagents and conditions

I condensation

II reduction LiAlH 4 in dry ether

(ii) Add 2,4-DNPH to sample and heat.Synthesis is complete when no orange ppt is
observed. Accept other possible tests.

(d)

CH 3 CH 2 NH 2 (l) CH 3 CH 2 NH 2 (g)

H f [CH 3 CH 2 NH 2 (l)]
7 1
2C(s) +2 H 2 (g) + 2
N 2 (g)
5(410)+350+305+2(390)
7 1
2(715)+2(436)+2(994)

2C(g) + 7H(g) + 2N(g)

H f [CH
H f [CH trendyline
nd
dy
H 3 CH 2 NH
H 2 ((l)]
H 3 CH 2 NH
l)] + 29+3485 = 3453
H 2 ((l)]=
l)]
)]= 0 kJ mol11
= 61.0
6 .0
61

(ei) amide, amine


MJC Prelim 2016 3 [Turn Over
739 trendyline
(ii) Compound Q ismore basic than Tamiflu.Theelectron-donating R group
increases the electron density of the lone pair on N atom hence more
available to accept a proton.Compound Risless basic. The lone pair of
electron on N atom is delocalised into the benzene ring hence less available
to accept a proton.

3(ai) 8HI + H 2 SO 4
4I2 + H 2 S + 4H 2 O

(ii) E = +0.54 Vand E = +1.07 V. As E Br2/Br- is more positive than E I2/I ,Br is a
weaker reducing agent than I.

(iii)Conc. H 3 PO 4 is a weaker oxidising agent .than conc. H 2 SO 4


KBr(s) + H 3 PO 4 (l) HBr (g) + KH 2 PO 4 (s)

(bi) KCl: pptdissolves


KBr: ppt dissolves
KI: ppt does not dissolve

(ii) NH 3 ligand combines with Ag+(aq) to form diamminesilver(I) complex or


[Ag(NH 3 ) 2 ]+.Ionic product (of [Ag+][I]easily)exceed the low K sp value
hencethus AgIdoes not dissolve.

1
(c) Number of mole of iodine in 25.0cm3 aliquot = (0.0400 0.010)
2
Number of mole of iodine in 250 cm3 = 2.00 103

nI 2 2 103
4
nBrO 4n 5 104

Oxidation: 2I I 2 + 2e

Mole ratio: BrO 4 nI 2 H

Oxidation number of Br in BrO 4 n= +7

+7 + 4(2) = n(1)

n=1

trendyline
2+
(di)MgCl 2 (s) + 6H 2 O (l) 2Cll (aq)
[Mg(H 2 O)) 6 ] (aq) + 2
2C
[Mg(H 2 O)) 6 ]2+(aq) (OH +(aq) + H+(aq) pH
[Mg(H 2 O) 5 (OH)] H = 6.5

MgO (s) + H 2 O ((l)


l)) Mg(OH) 2 (aq
(aq)) S+
+
(ii)ZnO + 2HCll
ZnCl 2 + H 2 O
ZnO + 2NaOH + H 2 O
Na 2 Zn(OH) 4
MJC Prelim 2016 4 [Turn Over
740 trendyline
4(ai) Prevent the moisturefrom reacting with the sodium metal.

(ii)No. of moles of phenol = (11.8 / 94) = 0.1255( same as phenoxide)

No. of moles of 1-bromo-3-methylbutane = (20.0 x 1.21) / 151

Mass of (3-methylbutoxy)benzene = 0.1255 x 164 = 20.6 g

(iii)

Organic layer

Aqueous layer

(bi) 1 (unreacted) phenol; 2water

(ii) (3-methylbutoxy)benzene has a higher boiling point.The extensive van der


Waals forces of attraction between3-methylbutoxy)benzene molecules is
stronger than the hydrogen bonding in ethanol.

(iii) Boiling point of the (3-methylbutoxy)benzene is significantly higher than the


other impuritiesTemperature range is from 214 oC - 218 oC.

trendyline
MJC Prelim 2016 5 [Turn Over
741 trendyline
5(a)S N 2

-
+ Br
+ - + - O N C
C Br O N C Br H
O N H
H H
H
H H H H

[1/2]
[1/2]
[1/2]

O N C + HBr
H
H
(b)
H

(c)

N N

(d)Carboxylic acid will undergo acid-base reaction with the amine groups instead.

(e) Add aqueousneutral iron (III) chloride. For morphine, a violet complex observed but
not for fentanyl. Accept other tests.

trendyline
MJC Prelim 2016 6 [Turn Over
742 trendyline
6(ai) 1s2 2s2 2p6 3s2 3p6 3d10

(ii)E cell = 0.52 0.15 = + 0.37 > 0. Hence, Cu+disproportionatesinto pink solid Cu
and blue solution of Cu2+.

(iii)The stronger CH 3 CH 2 NH 2 ligands replace H 2 O ligandsto form a deep blue


complex[Cu(CH 3 CH 2 NH 2 ) 4 (H 2 O) 2 ]2+.
[Cu(H 2 O) 6 ]2+ + 4 CH 3 CH 2 NH 2 [Cu(CH 3 CH 2 NH 2 ) 4 (H 2 O) 2 ]2+ + 4H 2 O
(iv)
Cu C H O
% by mass 17.76 26.85 1.68 53.71
No. of moles 17.76 26.85 1.68 53.71
63.5 12.0 1.0 16.0

Simplest ratio 1 8 6 12

Chemical formula of anion is [Cu(C 4 H 3 O 6 ) 2 ]4

(b) E cell = 0.15 (0.97) = + 1.12V > 0 hence reaction is feasible.

2Cu2+ + CN +2OH
2Cu+ + CNO+ H 2 O
E cell = 1.77 0.15 = + 1.62V > 0 hence reaction is feasible.

H 2 O 2 + 2H+ + 2Cu+
2H 2 O + 2Cu2+

(ci)

O O- O- O O O- H2N
+
Cu2 Cu2
+

OR
NH2 H2N NH2 O- O

(ii) Cis-transisomerism

trendyline
MJC Prelim 2016 7 [Turn Over
743 trendyline
&DOFXODWRU0RGHO1R

1DPH BBBBBBBBBBBBBBBBBBBB      &ODVV 6BBBBBBBBB      5HJ 1XPEHU


BBBBBB


0(5,',$1-81,25&2//(*(
 -&3UHOLPLQDU\([DPLQDWLRQ
 +LJKHU

BBBBBBBBBBBBBBBBBBBBBBBBBBBBBBBBBBBBBBBBBBBBBBBBBBBBBBBBBBBBBBBBBBBB

&KHPLVWU\ 
3DSHU)UHH5HVSRQVH 6HSWHPEHU

KRXUV
$GGLWLRQDO0DWHULDOV Data Booklet
Writing Paper

,16758&7,21672&$1','$7(6
:ULWH\RXUQDPHFODVVDQGUHJLVWHUQXPEHULQWKHVSDFHVSURYLGHGDWWKHWRSRIWKLVSDJH

$QVZHURXWRITXHVWLRQVLQWKLVSDSHU
%HJLQHDFKTXHVWLRQRQDIUHVKVKHHWRIZULWLQJSDSHU
)DVWHQWKHZULWLQJSDSHUVEHKLQGWKHJLYHQ&RYHU3DJHIRU4XHVWLRQV DQG&RYHU3DJH
IRU4XHVWLRQV UHVSHFWLYHO\
+DQGLQ4XHVWLRQV DQG VHSDUDWHO\
<RXDUHDGYLVHGWRVSHQGDERXWPLQXWHVSHUTXHVWLRQRQO\

,1)250$7,21)25&$1','$7(6
7KHQXPEHURIPDUNVLVJLYHQLQEUDFNHWV>@DWWKHHQGRIHDFKTXHVWLRQRUSDUWTXHVWLRQ
<RXDUHUHPLQGHGRIWKHQHHGIRUJRRG(QJOLVKDQGFOHDUSUHVHQWDWLRQLQ\RXUDQVZHUV

trendyline G L

7KLVGRFXPHQWFRQVLVWVRISULQWHGSDJHV
7KL I  L G  

MJC Prelim 2016


744 trendyline
























[Blank Page]

 

MJC Prelim 2016  >7XUQ2YHU


745 trendyline
$QVZHUDQ\RXWRITXHVWLRQVLQWKLVSDSHU
Begin each question on a fresh sheet of writing paper.

 %HQ]\OKDOLGHVDUHFRORXUOHVVOLTXLGVWKDWKDYHEHHQXVHGLQFKHPLFDOZDUIDUH
GXH WR WKHLU DELOLW\ WR FDXVH VHYHUH H\H UHVSLUDWRU\ DQG VNLQ LUULWDWLRQ %HQ]\O
KDOLGHV DUH DOVR XVHG LQRUJDQLF V\QWKHVLV IRU WKH LQWURGXFWLRQ RI WKH EHQ]\O
SURWHFWLQJJURXSIRUDOFRKROVDQGFDUER[\OLFDFLGV


EHQ]\OKDOLGH


D  %HQ]\OLRGLGH&+&+,FDQEHV\QWKHVLVHGIURPEHQ]HQHLQDIRXUVWHS
SDWKZD\LQYROYLQJ )ULHGHO&UDIWV DON\ODWLRQ)ULHGHO&UDIWV
DON\ODWLRQLQYROYHV WKH DON\ODWLRQ RI D EHQ]HQH ULQJ LQ WKH SUHVHQFH RI D
VXLWDEOHFDWDO\VWYLDHOHFWURSKLOLFVXEVWLWXWLRQ


2XWOLQHWKLVV\QWKHVLVURXWHVXJJHVWLQJUHOHYDQWUHDJHQWVFRQGLWLRQVDQG
LQWHUPHGLDWHSURGXFWV
>@

 E  $QRWKHUPHWKRGWRV\QWKHVLVHEHQ]\OLRGLGHLVYLDWKHFinkelstein reaction
ZKLFK LQYROYHV WKH FRQYHUVLRQ RI EHQ]\O EURPLGH WR EHQ]\O LRGLGH
XVLQJVWRLFKLRPHWULFDPRXQWVRISRWDVVLXPLRGLGHXQGHUUHIOX[FRQGLWLRQV


&+&+%U., &+&+,.%U Finkelstein reaction




7KHUDWHHTXDWLRQLVUDWH k>&+&+%U@


L  'UDZ GLDJUDPV WR LOOXVWUDWH WKH PHFKDQLVP IRU WKLV UHDFWLRQ 6KRZ
UHOHYDQWORQHSDLUVDQGGLSROHVDQGXVHFXUO\DUURZVWRLQGLFDWHWKH
PRYHPHQWRIHOHFWURQSDLUV
>@


LL  6XJJHVW D SODXVLEOH FKHPLFDO WHVW WKDW FDQ EH SHUIRUPHG RQ D


VDPSOH RI WKH UHDFWLRQ PL[WXUH WR WHVW ZKHWKHU WKH UHDFWLRQ LV
HIIHFWLYHO\FRPSOHWH
>@

trendyline


LLL  ([SODLQZK\EHQ]\OEURPLGHXQGHUJRHV
([SODLQZK\EHQ]\OEURPLGHXQGHUJRHVDXQLPROHFXOD
([SODLQZK\EHQ]\OEURPLGHXQGHUJRHVDXQLPROHFXODUUHDFWLRQZLWK
.,
., DT  ZKHUHDV EURPRHWK\O EHQ]HQH
EURPR
&+&++&+%UXQGHUJRHVDELPROHFX
%UXQGHUJRHVDELPROHFXODUUHDFWLRQZLWK., DT 
%UXQGHUJRHVDELPROHF ODUUHDFWLRQZLWK
DUUHDFWLRQZLWK
>@

MJC Prelim 2016  >7XUQ2YHU


746 trendyline
F  7KHFinkelstein reactionSURFHHGVDWDIDVWHUUDWHLQWKHSUHVHQFHRISRODU
VROYHQWVVXFKDVSURSDQRQH&+&2&+7KHLQFUHDVHLQUDWHOLHVLQWKH
GLIIHUHQFHLQVROXELOLW\EHWZHHQ.,DQG.%ULQSURSDQRQH


&+&+%U., &+&+,.%U Finkelstein reaction




 3RWDVVLXPKDOLGHV 6ROXELOLW\LQSURSDQRQH
., 9HU\VROXEOH

.%U $OPRVWLQVROXEOH

L  6XJJHVWZK\WKHFinkelstein reactionJRHVDOPRVWWRFRPSOHWLRQ
>@


LL  ,Q WKH VROYDWLRQ SURFHVV RQH SRWDVVLXP LRQ LQWHUDFWV ZLWK VL[
SURSDQRQHPROHFXOHV'UDZDGLDJUDPWRVKRZKRZWKLVLQWHUDFWLRQ
IRUPV DQ RFWDKHGUDO XQLW 6WDWH FOHDUO\ WKH W\SH RI LQWHUDFWLRQ
LQYROYHG
>@


LLL  6XJJHVW ZK\ SRWDVVLXP EURPLGH LV OHVV VROXEOH WKDQ SRWDVVLXP
LRGLGH LQ SURSDQRQH E\ FRQVLGHULQJ WKH ODWWLFH HQHUJ\ RI WKH
SRWDVVLXP KDOLGHV DQG HQWKDOS\ FKDQJH RI VROYDWLRQ RI WKH KDOLGHV
LRQV
>@


G  %HQ]\O LRGLGHV DUH ZLGHO\ XVHG LQ FRPSOH[ RUJDQLF V\QWKHVLV WR IRUP
EHQ]\O HWKHUV &+&+25  ZKLFK DUH JRRG EORFNLQJ JURXSV WKDW
SURWHFW DOFRKRO IXQFWLRQDO JURXSV IURP IXUWKHU UHDFWLRQV 7KHVH EORFNLQJ
JURXSV DUH LQHUW WR K\GURO\VLV XQGHU QRUPDO FRQGLWLRQV EXW WKH\ FDQ EH
UHPRYHG DW WKH HQG RI WKH UHDFWLRQ YLD WKH DGGLWLRQ RI DQ DSSURSULDWH
K\GURJHQ KDOLGH PROHFXOH WR REWDLQ WKH RULJLQDO IXQFWLRQDO JURXS DQG
UHJHQHUDWHWKHEHQ]\OLRGLGH


7KHIROORZLQJLOOXVWUDWHVDV\QWKHWLFVFKHPHIURPFRPSRXQG:LQZKLFKD
EHQ]\OHWKHULVDFWLQJDVWKHSURWHFWLQJJURXS


trendyline

MJC Prelim 2016  >7XUQ2YHU
747 trendyline



L  6WDWHWKHUROHRI&+&+&+&+ /L LQVWHS,
>@


LL 3UHGLFW ZLWK UHDVRQLQJ WKH UHODWLYH UDWH RI WKH UHDFWLRQ LQ 6WHS
,,ZKHQ WKH UHDFWDQW XVHG LV EHQ]\O EURPLGH &+&+%U LQVWHDG RI
EHQ]\O LRGLGH &+&+,4XRWH UHOHYDQW YDOXHV IURP WKH Data
BookletWRVXEVWDQWLDWH\RXUDQVZHU
>@


LLL  6WDWHWKHUHDJHQWVDQGFRQGLWLRQVQHHGHGIRUVWHSV,,,,9DQG9
>@


trendyline
LY  :ULWHDEDODQFHGHTXDWLRQIRUWKHUHDFWL
:ULWHDEDODQFHGHTXDWLRQIRUWKHUHDFWLRQEHWZHHQFR
:ULWHDEDODQFHGHTXDWLRQIRUWKHUHDFWLRQEHWZHHQFRPSRXQG;DQG
GLQLWURSKHQ\OK\GUD]LQH
GLQLWURSKHQ\OK\GUD]LQH
GLQLWURSKHQ\OK\GUD]LQH
>@

>7RWDO@


MJC Prelim 2016  >7XUQ2YHU
748 trendyline

 D  =LQFDLU EDWWHULHV KDYH UHFHLYHG UHYLYHG LQWHUHVW UHFHQWO\ GXH WR LWV KLJKHU
HQHUJ\GHQVLW\ODUJHUVWRUDJHFDSDFLW\DQGORZHUSURGXFWLRQFRVWDVFRPSDUHG
WRWKHFRPPRQO\XVHGFRQYHQWLRQDOOLWKLXPLRQEDWWHULHV


'XULQJ GLVFKDUJH R[\JHQ IURP WKH DLU R[LGLVHV ]LQF WR ]LQF R[LGH 7KH
HOHFWURO\WHXVHGLQWKHEDWWHU\LV.2+ DT 7KH]LQFDLUEDWWHU\KDVDVWDQGDUG
FHOOSRWHQWLDORI9


LQVXODWRU

 SRZGHUHGSDVWHRI=QLQ
 .2+ DT 


VHSDUDWRU

 SRURXVJUDSKLWHHOHFWURGH

 DLUDFFHVVKROH


L  &RQVWUXFW HTXDWLRQV IRU WKH UHDFWLRQV ZKLFK RFFXU DW WKH DQRGH DQG
FDWKRGH+HQFHFRQVWUXFWWKHRYHUDOOHTXDWLRQIRUWKHFHOOUHDFWLRQ
>@


LL  8VLQJ VXLWDEOH GDWD IURP WKH Data Booklet,FDOFXODWH WKH YDOXH IRU WKH
ERRIWKH=Q2=QHOHFWURGHUHDFWLRQ
>@


LLL  6XJJHVWDUHDVRQZK\WKHDLUDFFHVVKROHRIWKHVH]LQFDLUEDWWHULHVPXVW
EHZHOOVHDOHGGXULQJWKHSDFNDJLQJRIWKHVHEDWWHULHVIRUVWRUDJH
>@


LY  :LWKUHIHUHQFHWR\RXUDQVZHULQ D L H[SODLQIXOO\ZK\WKHRSHUDWLQJFHOO
SRWHQWLDO RI WKH ]LQFDLU EDWWHULHV EHFRPHV OHVV SRVLWLYH DW KLJK DOWLWXGHV
ZKHUHWKHDWPRVSKHULFSUHVVXUHLVORZHUHG
>@


Y  'HWHUPLQH WKH FXUUHQW RXWSXW LQ DPSHUHV  RI WKLV ]LQFDLU EDWWHU\ JLYHQ
WKDWWKHDLUDFFHVVKROHVDOORZDERXWGPRIDLUH[FKDQJHLQKRXUDW
URRPWHPSHUDWXUHDQGSUHVVXUH<RXPD\DVVXPHWKDWWKHSHUFHQWDJHRI

trendyline
2SUHVHQWLQDLULV
SUHVHQWLQDLULV
S
>@




MJC Prelim 2016  >7XUQ2YHU
749 trendyline

E  &DUERQGLR[LGHZKHQSUHVHQWLQVXIILFLHQWFRQFHQWUDWLRQLQDLUPD\UHVXOWLQWKH
IRUPDWLRQ RI LQVROXEOH ]LQF FDUERQDWH WKDW FORJV WKH SRURXV JUDSKLWH HOHFWURGH
7KLVKLQGHUVWKHQRUPDORSHUDWLRQRIWKHEDWWHU\


 :ULWHDQHTXDWLRQZLWKVWDWHV\PEROVIRUWKHWKHUPDOGHFRPSRVLWLRQRI]LQF
FDUERQDWH
>@


LL  =LQF FDUERQDWH DQG EDULXP FDUERQDWH GHFRPSRVH ZKHQ KHDWHG WR JLYH
VLPLODUSURGXFWV


%\ TXRWLQJ DSSURSULDWH GDWD IURP WKH Data Booklet GHGXFH ZKHWKHU ]LQF
FDUERQDWHZRXOGGHFRPSRVHDWDKLJKHURUORZHUWHPSHUDWXUHWKDQEDULXP
FDUERQDWH([SODLQ\RXUDQVZHU
>@


LLL  8VLQJ WKH IROORZLQJ GDWD DQG UHOHYDQW GDWD IURP WKH Data Booklet
FRQVWUXFW D %RUQ+DEHU F\FOH WR FDOFXODWH WKH ODWWLFH HQHUJ\ RI ]LQF
FDUERQDWH


VWDQGDUGHQWKDOS\FKDQJHRIDWRPLVDWLRQRI=Q V  N-PRO
VWDQGDUGHQWKDOS\FKDQJHRIIRUPDWLRQRI&2 J  N-PRO
VWDQGDUGHQWKDOS\FKDQJHRIIRUPDWLRQRI=Q&2 V  N-PRO
    >@

F  =LQF H[KLELWV VRPH SURSHUWLHV WKDW DUH VLPLODU WR WKRVH RI D WUDQVLWLRQ HOHPHQW
)RU H[DPSOH ]LQF PHWDO KDV GHPRQVWUDWHG LWV SRWHQWLDO DV D heterogeneous
FDWDO\VWIRUWKHDF\ODWLRQRIDODUJHYDULHW\RISKHQROVZLWKDF\OFKORULGHV






'HVFULEHIXOO\KRZ]LQFIXQFWLRQVDVDFDWDO\VWIRUWKLVUHDFWLRQ
>@

G  6LPLODU WR DOXPLQLXP ]LQF FDQ EH DQRGLVHG WR PDNH LW PRUH UHVLVWDQW WR
FRUURVLRQ'UDZDIXOO\ODEHOOHGGLDJUDPDQGZLWKWKHDLGRIHTXDWLRQ V H[SODLQ

trendyline
KRZ]LQFFDQEHDQRGLVHG
QFFDQEHDQRGLVHG
F
>@

>7RWDO@


MJC Prelim 2016  >7XUQ2YHU
750 trendyline

 Atorrvastatin KDV EHHQ D
D YHU\ SRSSXODU GUXJ VLQFH 
 ,W LV FFRQVXPHG WR UHGXFH
H
EORRGFKROHVWH HUROOHYHODQ
QGIRUSUHYYHQWLRQRIK
KHDUWGLVHDDVHV


2+
+ 2+ 2
2
1 2+
5 1+
1
5

) 
Atorvastatin R is a hydrocarb
bon group


D  $ VDPSOHH RI  J RI


R atorvasttatin ZDV FRPSOHWHO\
F \ EXUQW LQ H[FHVV R[
[\JHQ DQG
G
 R
FPP RI12J JDVZDVSUURGXFHG$ $OOYROXPHVVZHUHPHDVXUHGDW  &DQG G
DWP


L  &DOFFXODWH WKH MU RI atorrvastatin WR


W WKH QHD
DUHVW ZKROH
H QXPEHU XVLQJ WKHH
DER
RYHLQIRUPD DWLRQ
>@@


LL  7KH
H DFWXDO MU RI atorva
astatin LV 
 ([SODLQ ZK\ WKH
HUH LV D GLVFUHSDQF\\
EHWZ
ZHHQWKHFFDOFXODWHGM MULQ D L DQGWKHDFFWXDOMU
>@@


E  Atorvasta
atin LV VROG DOFLXP VDOWW LQ WDEOHW IRUP (DFFK WDEOHW FR
G DV WKH FD RQWDLQV 

PJRIWKH
HFDOFLXPVVDOW7KHP PD[LPXPG GRVDJHLV PJSHUUGD\IRUD DQDYHUDJH H
NJDGX
XOW















L  Use
e of informa
ation in the
e photograp
ph above is
i relevantt to this que
estion.

trendyline


&DOFFXODWH
&DOF XODWH WKH
WKH PLQLPXP QXPEHU QXPEHU RI
RI ERWWOHV
ERWWOH RI WDEOHWV
WDEOHWV D D SDWLHQWW QHHGV WR
R
VWRF
WR
RFNXSIRU
FFNXSIRUD
XSIRUD
D \HDU
D \HDUDVV
\HD DV
DVVVVXPLQJWKD
XPLQJWKD
XPLQJWK DWKHDGKHUHVVWULFWO\
DWKHDGKH UHVVWULFWO\\WRWKHGD
WR
WR LO\GRVDJHH
OLPLW
PLWWWDQGFRQV
PL DQGFRQV
DQG FRQVXPHVLWH
XPHVLWHY
XPHV LW HYYHU\GD\
YHU\GD\
HU\ GD
>@@



MJC Prelim 2016  >7XUQ2YHU
> U
751 trendyline

LL  8VLQJ VWUXFWXUH DQG ERQGLQJ H[SODLQ WKH GLIIHUHQFHV LQ VROXELOLW\RI
atorvastatinDQGatorvastatincalcium LQ&&l
>@

,Q SKDUPDFHXWLFDO FRPSDQLHV atorvastatin FDQ EH V\QWKHVLVHG E\ UHDFWLQJ
FRPSRXQG$ZLWKFRPSRXQG&


F  &RPSRXQG$LVPDGHIURPVPDOOHUPROHFXOHV7KHIROORZLQJUHDFWLRQVFKHPH
VKRZVWKHV\QWKHVLVRIFRPSRXQG$


2
2 2 2
EDVH 5 +  +
5 5 5
1 2 5HDFWLRQ  1 2 5HDFWLRQ  1 2
+ + +
5 2+

2 5HDFWLRQ 
5
1 2
+ 2
5 2
IOXRUREHQ]DOGHK\GH 5
1 2
+
5
)
FRPSRXQG $ 

5HDFWLRQ   DQG  DUH SDUW RI DQ DOGRO FRQGHQVDWLRQ UHDFWLRQ $ EULHI
GHVFULSWLRQRIHDFKUHDFWLRQLVJLYHQEHORZ


5HDFWLRQ 7KHFDUERQ\OFDUERQLVGHSURWRQDWHG


5HDFWLRQ 1XFOHRSKLOLFDGGLWLRQWDNHVSODFHRQWKHFDUERQ\OFRPSRXQG


5HDFWLRQ (OLPLQDWLRQRIWKHDOFRKROIRUPVDQDONHQHZKLFKLVFRQMXJDWHG
WRWKHNHWRQH


L  2XWOLQH WKH PHFKDQLVP IRU 5HDFWLRQ  6KRZ UHOHYDQW ORQH SDLUV DQG
GLSROHVDQGXVHFXUO\DUURZVWRLQGLFDWHWKHPRYHPHQWRIHOHFWURQSDLUV

trendyline
>@


MJC Prelim 2016  >7XUQ2YHU
752 trendyline

LL  8VLQJWKHLQIRUPDWLRQLQWKHUHDFWLRQVFKHPHJLYHQVXJJHVWWKHSURGXFWV
IRUPHG ' DQG ( ZKHQ WKH IROORZLQJ FDUERQ\O FRPSRXQGV XQGHUJR DOGRO
FRQGHQVDWLRQUHDFWLRQLQWKHSUHVHQFHRIDEDVH
>@

1 



2  



G  7KHIROORZLQJUHDFWLRQVFKHPHVKRZVWKHV\QWKHVLVRIFRPSRXQG&

2+ 2 2 2+ 2+ 2
1& 1&
2+ 5HDFWLRQ  2+
FRPSRXQG %
5HDFWLRQ 
2 2

+ 3G 2 2 2  &+2+
FRPSRXQG &
1&
&+12  UWS 2+

L  6WDWHWKHUHDJHQWDQGFRQGLWLRQIRU5HDFWLRQ
>@


LL  6XJJHVWWKHW\SHRIUHDFWLRQLQ5HDFWLRQ
>@


LLL  'UDZWKHVWUXFWXUHRIFRPSRXQG&
>@


LY  &RPSRXQG%IRUPVDF\FOLFSURGXFWZLWKWKHORVVRIRQHZDWHUPROHFXOH
ZKHQ LW LV KHDWHG ZLWK FRQFHQWUDWHG VXOIXULF DFLG 6XJJHVW WKHVWUXFWXUH RI

trendyline
WKHSURGXFWIRUPHG
KHSURGXFWIRUPHG
H
>@




MJC Prelim 2016  >7XUQ2YHU
753 trendyline

H  7KHIROORZLQJ UHDFWLRQ VFKHPH VKRZV WKH ILQDO VWHSV LQ WKH SURGXFWLRQ RI
atorvastatin


2+ 2+ 2
2
FRPSRXQG $ + 1 2+
5 1+
FRPSRXQG &
5

)
Ator vastatin 


AtorvastatinFRQWDLQVVHYHUDOIXQFWLRQDOJURXSV

L  ,WKDVWZRGLIIHUHQWDFLGLFIXQFWLRQDOJURXSV1DPHWKHPDQGH[SODLQZK\
RQHKDVDKLJKHUKDYDOXHWKDQWKHRWKHU
>@


LL  7KHQLWURJHQDWRPLQWKHILYHPHPEHUHGULQJLVEDVLF'UDZWKHVWUXFWXUHV
RI WKH SURGXFWV IRUPHG ZKHQ atorvastatin LV KHDWHG ZLWK DFLGLILHG
SRWDVVLXPGLFKURPDWH 9, 
>@

>7RWDO@
 

trendyline

MJC Prelim 2016  >7XUQ2YHU
754 trendyline

 $GLSLF DFLG LV DQ RUJDQLF FRPSRXQG ZLWK WKH IRUPXOD &+  &22+  ,W LV WKH PRVW
LPSRUWDQWRIWKHGLFDUER[\OLFDFLGVIURPWKHLQGXVWULDOSHUVSHFWLYH$SSUR[LPDWHO\
ELOOLRQ NLORJUDPV RI WKLV ZKLWH FU\VWDOOLQH SRZGHU DUH SURGXFHG DQQXDOO\
SUHGRPLQDQWO\DVDSUHFXUVRUIRUWKHSURGXFWLRQRIQ\ORQ


2
+2
2+
2
$GLSLF DFLG 


D  2QH PHWKRG WR V\QWKHVLV DGLSLF DFLG LV E\ WKH FDUERQ\ODWLRQ RI JDVHRXV
EXWDGLHQHZLWKFDUERQPRQR[LGHDQGVWHDP


&+ &+&+ &+ J &2 J +2 J 


+22& &+ &22+ J 


L  8VH UHOHYDQW ERQG HQHUJ\ GDWD IURP WKH Data Booklet WR FDOFXODWH WKH
HQWKDOS\FKDQJHIRUWKLVUHDFWLRQ


*LYHQWKDWERQGHQHUJ\RI&2LVN-PRO 
>@


LL  7KH HQWKDOS\ FKDQJH IRU WKH IROORZLQJ UHDFWLRQ WR IRUP DGLSLF DFLG LQ WKH
VROLGIRUPLVN-PRO


&+ &+&+ &+ J &2 J +2 J 


+22& &+ &22+ V 


$FFRXQWIRUWKHGLIIHUHQFHVLQWKHYDOXHVIRU D L DQG D LL 
>@


LLL  ,W LV DOVR SRVVLEOH WR V\QWKHVLVHDGLSLF DFLG E\ UHDFWLQJ DGLSR\O GLFKORULGH
WKURXJKDK\GURO\VLVUHDFWLRQZLWKZDWHU


2 2
&l  + 2 +2  +&l
&l 2+
2 2
$GLSR\O GLFKORULGH $GLSLF DFLG


'HVFULEH DQG H[SODLQ WKH UHODWLYH HDVH RI K\GURO\VLV RI DGLSR\O GLFKORULGH
DQGFRPSRXQG$VKRZQEHORZ


trendyline
re
end
e d ne
&l &l
&l &l
2
$GLSR\OO GLFKORULGH
$GL GL KO LG &RPSRXQG
&RPSR
RPSR $

>@

MJC Prelim 2016  >7XUQ2YHU
755 trendyline

E  $GLSLF DFLG DQG LWV FDOFLXP VDOW DUH XVHG DV D EXIIHU VROXWLRQ WR UHPRYH VXOIXU
GLR[LGH IURP H[KDXVWIOXH JDVHVRI IRVVLOIXHOSRZHU SODQWV &DOFLXP VXOIDWH
&D62LVIRUPHGDWWKHHQGRIWKHUHDFWLRQ


L  'UDZWKHGRWDQGFURVVGLDJUDPIRUWKHLRQ62
>@

 
LL  ,QSULQFLSOH62 FDQEHIRUPHGIURPWKHR[LGDWLRQRI62 


'UDZ WKH VWUXFWXUH RI62 JLYHQ WKDWWKHUH DUHIRXU FHQWUDO DWRPV DQG
WKDWWKHLRQLVV\PPHWULFDOZLWKDQ22ERQG


$FFRXQWIRULWVVKDSHZLWKUHVSHFWWRHDFKGLIIHUHQWFHQWUDODWRPXVLQJWKH
96(35WKHRU\
>@


LLL  $GLSLFDFLGLVDGLEDVLFDFLGZLWKSKaYDOXHVRIDQG


7KH RSWLPDO EXIIHU S+ IRU WKH DGLSLF DFLGFDOFLXP VDOW PL[WXUH LV 
&DOFXODWHWKHYROXPHRIPROGPRIFDOFLXPK\GUR[LGHWKDWQHHGVWREH
DGGHGWRFPRIPROGPRIDGLSLFDFLGWR\LHOGWKHRSWLPDOEXIIHU
VROXWLRQ
>@


F  &RPSRXQG % KDV WKH VDPH PROHFXODU IRUPXOD &+2 DV DGLSLF DFLG % LV D
QHXWUDO PROHFXOH EXW UHDFWV ZLWK KRW DTXHRXV VXOIXULF DFLG WR \LHOG WZR RUJDQLF
FRPSRXQGV&&+2DQG'LQDUDWLR


&RPSRXQG&FRQWDLQVFKLUDOFDUERQDWRPV&RPSRXQG&IRUPVQRSUHFLSLWDWH
ZLWKGLQLWURSKHQ\OK\GUD]LQHEXWUHDFWVZLWKKRWDOXPLQLXPR[LGHWRIRUPRQO\
WKUHH LVRPHUV ( ) DQG * ZLWK WKH PROHFXODU IRUPXOD RI &+2 2QO\ LVRPHU
*URWDWHVWKHSODQHRISRODULVHGOLJKW


&RPSRXQG ' UHDFWV ZLWK KRW DONDOLQH &X VROXWLRQ WR JLYH D UHG SUHFLSLWDWH ,W
DOVRUHDFWVZLWKKRWDFLGLILHG.0Q2WRIRUPHIIHUYHVFHQFHRIFDUERQGLR[LGH

L  6XJJHVW VWUXFWXUHV IRU FRPSRXQGV % & ' ( ) DQG * VKRZLQJ \RXU
UHDVRQLQJFOHDUO\
>@


trendyline
LL  :ULWH
ULWH D EDODQFHG FKHPLFDO HTXDWLRQ IRU WKH
W UHDFWLRQ EHWZHHQ
EH ' DQG KRW

DONDOLQH&X
ONDOLQH&X VROXWLRQ
VROXWLRQ
>@

>7RWDO@

MJC Prelim 2016  >7XUQ2YHU
756 trendyline

 HemocyaninDQGchymotrypsinDUHSURWHLQVXVHGIRUYDULRXVIXQFWLRQV


Hemocyanin LV D FRSSHUFRQWDLQLQJ R[\JHQ WUDQVSRUW SURWHLQ IRXQG LQ PDULQH


LQYHUWHEUDWHV VXFK DV VTXLGV ,WV PRGH RI WUDQVSRUW RI R[\JHQ LV VLPLODU WR WKDW RI
haemoglobin,Qhemocyanin, R[\JHQLVWUDQVSRUWHGLQWKHIRUPRIR[\KHPRF\DQLQ


D  'HVFULEHZKDWLVPHDQWE\WKH quaternaryVWUXFWXUHRIhemocyanin
>@


E  ,QVTXLGVRQHPROHFXOHRIhemocyanin +F ELQGVWRRQHPROHFXOHRIR[\JHQYLD
WKHR[\JHQDWLRQSURFHVVDFFRUGLQJWRWKHIROORZLQJHTXDWLRQ


+F DT 2 J  +F 2  DT 


L  ,Q VTXLG hemocyanin ZKHQ WKH SDUWLDO SUHVVXUH RI R[\JHQ JDV LV
DWPDWR&WKHR[\JHQELQGLQJVLWHVDUHVDWXUDWHG


&DOFXODWHWKHHTXLOLEULXPFRQVWDQWKFDWWKLVWHPSHUDWXUH
>@


LL  7KH *LEEV IUHH HQHUJ\ FKDQJH DW & ZKHQ hemocyaninLQWHUDFWV ZLWK
2 J LVUHODWHGWRWKHHTXLOLEULXPFRQVWDQWXVLQJWKHIROORZLQJHTXDWLRQ


G 57OQKF


 &DOFXODWHWKH*LEEVIUHHHQHUJ\FKDQJH
>@


LLL  ,W LV FODLPHG WKDWglobal warming would harm aquatic organisms such as
the squid populationWKURXJKDGYHUVHO\DIIHFWLQJWKHR[\JHQDWLRQRIVTXLG
hemocyanin


9HULI\ WKLV FODLP E\ FRQVLGHULQJ WKH VLJQV RI WKHH DQG S of WKH
R[\JHQDWLRQSURFHVV
>@


LY  7KH EORRG RI WKH VTXLG LV EOXH LQ FRORXU DV R[\JHQDWLRQ FDXVHV D FRORXU
FKDQJHEHWZHHQWKHFRORXUOHVV&X , GHR[\JHQDWHGIRUPRIhemocyaninWR
WKHEOXH&X ,, R[\JHQDWHGIRUP


 ([SODLQZK\WKHR[\JHQDWHG&X ,, IRUPRIhemocyaninLVEOXHLQFRORXU
>@





MJC Prelim 2016  >7XUQ2YHU
757 trendyline

Chymotrypsin LV DQ HQ]\PH SURGXFHG E\ WKH SDQFUHDV WKDW LV UHVSRQVLEOH IRU
FDWDO\VLQJWKHK\GURO\VLVRIFHUWDLQSURWHLQVLQWKHVPDOOLQWHVWLQHGXULQJWKHGLJHVWLYH
SURFHVV 3DUW RI WKH VWUXFWXUH RI chymotrypsin LV JLYHQ EHORZ DORQJ ZLWK WKH
DEEUHYLDWHGQDPHVRILWVFRQVWLWXHQWDPLQRDFLGVJO\OHXHWF




F  8VHWKHDEEUHYLDWHGQDPHVRIWKHDPLQRDFLGVWRVWDWHWKHSULPDU\VWUXFWXUHRI
WKHIUDJPHQWRIchymotrypsin JLYHQDERYH
>@


G  7KH ELRORJLFDO IXQFWLRQ RI chymotrypsin GHSHQGV RQ LWV WKUHHGLPHQVLRQDO


VKDSH'HVFULEHKRZWKHSDUWLFXODUDPLQRDFLGVLQchymotrypsin DUHOLNHO\WREH
LQYROYHGLQPDLQWDLQLQJLWVWKUHHGLPHQVLRQDOVKDSH
>@


H  7KH DFWLYLW\ RIchymotrypsin LV LQKLELWHG E\ SURORQJHG KHDWLQJ ZLWK VRGLXP
K\GUR[LGH


'UDZ WKH VWUXFWXUH RI WKH WULSHSWLGH REWDLQHG XQGHU WKHVH FRQGLWLRQV IURP WKH
SDUWRIchymotrypsinFRQWDLQLQJWKHDPLQRDFLGVLOHYDOJOX

trendyline
>@




MJC Prelim 2016  >7XUQ2YHU
758 trendyline

I  Chymotrypsin XQGHUJRHV GHQDWXUDWLRQ E\ VRGLXP K\GUR[LGH ZLWK WKH
PHFKDQLVPGHSHQGHQWRQS+$WKLJKS+WKHUHDFWLRQLVILUVWRUGHUZLWKUHVSHFW
WRERWKWKHchymotrypsinDQGK\GUR[LGHLRQV


 ,Q DQ H[SHULPHQW WKH GHQDWXUDWLRQ RI WKH HQ]\PH ZDV PRQLWRUHG LQ WKH
SUHVHQFHRIH[FHVVDONDOL


7KHSORWRIWKHWLPHFRXUVHRIWKHUHDFWLRQLVVKRZQEHORZ


(
[

(
[

(
[
OQ>FK\PRWU\SVLQ@

(
[

(
[

(
[

(
[

(
[

        

WLPHV

7KHUDWHODZFDQEHH[SUHVVHGLQWKHIRUPRI


OQ>$@ kWOQ>$@

ZKHUH>$@ FRQFHQWUDWLRQRIDUHDFWDQWDWWLPHW
>$@ LQLWLDOFRQFHQWUDWLRQRIWKDWUHDFWDQW
W WLPHVLQFHWKHUHDFWLRQVWDUWHG
 k UDWHFRQVWDQW
  

L  ([SODLQ KRZ WKH SORW RI WKH WLPH FRXUVH RI WKH UHDFWLRQ FRQILUPV WKDW WKH
GHQDWXUDWLRQ LV ILUVW RUGHU ZLWK UHVSHFW WR chymotrypsin DQG KRZ WKH
FRQGLWLRQVJLYHULVHWRRYHUDOOILUVWRUGHUNLQHWLFV

trendyline
>@


LL  8VLQJWKHSORWRIWKHWLPHFRXUVHRIWKHUHDFWLRQFDOFXODWHWKHYDOXHRIWKH
VLQJWKHSORWRIWKHWLPHFRXUVHRIWKHUHDFWLRQFDOFXODW
H[SHULPHQWDOO\GHWHUPLQHGUDWHFRQVWDQWRIWKLVGHQDWXUDWLRQ
SHULPHQWDOO\GHWHUPLQHGUDWHFRQVWDQWRIWKLVGHQDWXUD
>@


MJC Prelim 2016  >7XUQ2YHU
759 trendyline

LLL  +HQFH FDOFXODWH WKH YDOXH RI WKH WUXH UDWH FRQVWDQW JLYHQ WKDW
FRQFHQWUDWLRQRI1D2+XVHGLQWKHH[SHULPHQWZDVPROGP
>@


J  2QH RI WKH DPLQRDFLGV LQYROYHG LQ WKH FDWDO\WLF DFWLYLW\ RI chymotrypsin LV
DVSDUWLFDFLG




      $VSDUWLFDFLG


7KHUHDUHWKUHHSKDYDOXHVDVVRFLDWHGZLWKDVSDUWLFDFLGDQG


6XJJHVWWKHSUHGRPLQDQWVSHFLHVSUHVHQWLQVROXWLRQVRIDVSDUWLF DFLGZLWKWKH
IROORZLQJS+YDOXHVRI


L  


LL  
>@


>7RWDO@



(1'2)3$3(5 


MJC Prelim 2016  >7XUQ2YHU
760 trendyline






















trendyline

MJC Prelim 2016  >7XUQ2YHU
761 trendyline
2016 MJC Prelim Paper 3 Suggested Answers

1(a)
CH2I

CH3Cl, AlCl3 anhydrous PI3, heat


electrophilic substitution OR
.
NaI in conc H2SO4, heat
nucleophilic substitution
CH3
CH2OH

KMnO4, H2SO4 (aq) COOH .. .


heat LiAlH4 in dry ether, r t p
oxidation reduction

(b)

+ slow
Step 1: H C Br +
C
+ Br
~

H H H
Carbocation intermediate


f ast
Step 2: C
+ I H C I

H H H

(i) Add AgNO 3 (aq). Observation: No yellow precipitate observed.

(ii) The C 6 H 5 CH 2 + carbocation is resonance stabilised

(ci)

trendyline
KBr is
is precipitated out as a solid.
solid Position
Position of equilibrium shi
shifts
shif to the right.

MJC Prelim 2016 1 chemistry unit @ mjc


762 trendyline
(ii)


O


O O

+ -

K ion dipole interactions
O O

O

(iii) Enthalpy change of solution for KI in propanoneis more exothermic.

(di) As a base.
(ii) BE (CI) = 240 kJ mol1 ; BE ( CBr) = 280 kJ mol1
The CBr bond is stronger than CI bond hence rate is slower.

(iii) Step III: alkaline KMnO 4 (aq), heat

Step IV: alkaline aqI2 , heat

StepV: hydrogen iodide

(iv)

H 2N
NH NO2 N
NH NO 2
+ + H 2O
O

NO2
NO 2

trendyline
MJC Prelim 2016 2 chemistry unit @ mjc
763 trendyline
2(ai) Zn + 2 OH
ZnO + H 2 O + 2e

O 2 + 2H 2 O + 4e
4 OH
2 Zn + O 2
2 ZnO

(ii) +1.59 = 0.40 E o (ZnO / Zn)


E o (ZnO / Zn) = 1.19 V

(iii) Prevent entry of O 2 so to minimise the batterys discharging process to prolong the
shelf-life of the battery.

(iv) Partial pressure of O 2 hence the equilibrium position shifts left. Eo (O2/OH) will be
less positive hence E cell < 1.59 V.

I t I 60 60
(v) No. of moles of O 2 = =
n F 4 96500
Current output of battery = 0.188 A

(bi) ZnCO 3 (s)


CO 2 (g) + ZnO (s)
(ii) Zn2+ 0.074 nm;Ba2+: 0.135 nm.Charge density: Zn2+> Ba2+.Zn2+ distorts the anion
charge cloud to a greater extent, weakening and breaking the CO bond in
CO 3 2.Hence, decomposition temperature of ZnCO 3 is lower than BaCO 3 .

(iii)
Zn2+(g) + C(s) + 3/2 O 2 (g) + 2e
o
H f (CO 3 2(g))
2nd IE Zn2+(g) + CO 3 2 (g)___

Zn+(g) + C(s) + 3/2 O2 (g) + e H latt


st
1 IE
Zn(g) + C(s) + 3/2 O 2 (g)
o
H at (Zn)
0 Zn (s) + C(s) + 3/2 O 2 (g)
o
H f
ZnCO 3 (s)

By Hess Law,H
aw,
w,H
trendyline
H latt ZnCO 3 ) = 3266 kJ mol
l (ZnCO
1

MJC Prelim 2016 3 chemistry unit @ mjc


764 trendyline
(c) Zn is a heterogeneous catalyst because of the availability of 3d&4s electrons for
temporary bond formation with reactants.Reactant molecules are adsorbed onto
the Zn catalyst surface.This adsorption increases the surface concentration of
the reactants and weakens the covalent bonds lowering the activation energy
such that molecules are brought close for reaction to take place easily. Products
formed diffuse away from the surface of the catalyst.
(d)

Anode:zinc
object to be Graphite
coated with cathode
ZnO
dilute H2SO4

Anode :2H 2 O (l) O 2 (g) + 4H+ (aq) + 4e

Overall :2 Zn(s) + O 2 (g) 2 ZnO (s)

PV 1.01 105 103.7 10 6


3(ai) No of moles of NO 2 gas produced = =
RT 8.31 353

1
M r of atorvastatin = = 560
-3
1.785 10

(ii) NO 2 is a real gas with significant intermolecular forces of attraction.

(bi) No of tablets required in year = 2 x 365.


730
No of bottles needed = = 8.11 9
90
(ii) The weak van der Waals forces between the atorvastatin molecules is compatible
with that between CCl 4 molecules. The atorvastatin calcium cannot form ion

trendyline
dipole interaction
eraction
ra with the CCl
CCl 4 molecules,
cules, hence iti will not dissolve.
dissol

MJC Prelim 2016 4 chemistry unit @ mjc


765 trendyline
(i)
O
O O
R slow R
N O R H N O
H H
R O-

O O
R R
N O N O
H+
H H
R O- R OH

O H O

(ii)

(di) NaBH 4 in ethanol

(ii) Nucleophilic substitution

O O O

H2N OH
(iii)

O
NC
(iv) O

(ei) The carboxylic acid group has higher K a .The carboxylate anion is resonance
stabilised.The alcohol has lower K a. The electron donating alkyl group increases
the intensity of the negative charge on the oxygen atom of the alkoxide anion
hence destabilising it.

trendyline
MJC Prelim 2016 5 chemistry unit @ mjc
766 trendyline
(ii)

O O O
O H
N OH
HO
R
R NH3
F

4(ai) + rxn =2(1079) + 4(460) + 1(350) + 2(610) + 6(410) [ 5(350) + 2(740) + 2(360) +
2(460) + 8(410) = 122 kJ mol1

(ii) The difference is due to + fusion plusvaporisation of adipic acid.Accept other answers.

(iii) For adipoyl dichloride, the carbonyl C atom is bonded to electronegative atoms
hence the electron deficient carbonyl C atom is very susceptible to hydrolysis.
For compound A, the lone electron pair of the chlorine atom is delocalised with the
adjacent C=C double bond hence rendering partial double bond character.
..
..O
.. .. 2
-

..S O..
xx
.

O
. ..
.. .
x

..
x

xx
..

O
..

(bi)

(ii)
-
O O 2
O - O
- S O O or S O O
O O S O O S
O O
O O

trendyline
The shape
ape with respect to S is tetrahedral.
The shape
tetrahedral
ape with respect to O is bent

MJC Prelim 2016 6 chemistry unit @ mjc


767 trendyline
(iii) Let y dm3.=vol of base added.

No. of mole of HA = 0.2y

Final no. of mole of H 2 A = 5 x 103 0.2y

0.2/(0.5+)
4.6 = 4.4 + lg(5 x 103 0.2)/(0.5+)

y = 0.0153 dm3

(ci) B undergoes acidic hydrolysis B is an ester.


C does not undergo condensation with 2,4dinitrophenylhydrazine.
C does not containaldehyde or ketone
C undergoes elimination with hot Al 2 O 3 .E to G contains alkene
E and F are cis-trans isomersandG contains chiral carbonandis a structural isomer
ofE&F.
D undergoes oxidation with Fehlings D contains aliphatic aldehyde.
Disethanedioic acid and it undergoes further oxidation to form carbon dioxide.
H OH

HO C
C CH3

CH3 H
Structure of C is

H H

H OH C C
CH3 OH
C C C C HO C H
CH3 CH3 H H
CH3
CH3

E F G
E and F (can interchange)

C OH

trendyline
d
H C

Structure
tructure
ructure of D is O

MJC Prelim 2016 7 chemistry unit @ mjc


768 trendyline
O H OH

C O C
H C C CH3

O CH3 H
Structure of B is

(i)
O O

C OH C O- + Cu2O
+ 2 Cu2+ + 6OH- -
H C O C + 4H2O

O O

5(a) Quaternary structure is the combination of two or more polypeptide chains interacting
to form a more complex structure.The subunits in the structureinteract via van der
Waals forces, ionic bonds and hydrogen bonds.

(bi) [O 2 ] =
0.13 1.01 10 5

= = 5.30 x 103moldm3.Accept answer in mol m-3


8.31 298
1
K c = 0.33 = 62.3
5.30 10 3

(ii) G= (8.31) (298) ln (62.3)= 10.2 kJ mol1

(iii) S is negative since there is a decrease is disorder as there is a decrease in


number of moles of gas.H is negative .When temperature is high, IH I < I
TSI,hence G is positive hence oxygenation of squid is not supported leading to
death.

(iv) Cu(II) has partially-filled d orbitals. The d orbitals are split into two groups by
ligands. The d electron is promoted to the higher d orbital.During the transition, the
d electron absorbs red/orange/yellow wavelength light from the visible region of the
electromagnetic spectrum and the remaining wavelengths (blue) not absorbed will
appear as the colour observed.

(c) gly ile val glu gln cys cys ala ser val cys ser leu

(d) van der Waals


aals
als attraction between val
v andleu;
andleu
and leu;; h
hydrogen
ydrogen bond betw
between gln and ser;
disulfide bond
ond
nd between side chains of cys.
c

MJC Prelim 2016 8 chemistry unit @ mjc


769 trendyline
(e)
H O H O H O
-
H2N C C N C C N C C O

HC H HC H H2C
H2C CH3H3C CH3 CH2
CH3 C -
O O
ile val glu

(fi) The plot is a straight line graph.[NaOH] is kept constant hence the rate is
independent of [NaOH]

(ii) k = = 8.21 x 1010Allow range of 8.0 8.4 x 1010

(iii) 8.21 x 1010 = k [1.5]


k = 5.47 x 1010

(g) pH 2.4pH 7.0

O O
-
HO -
O -
O O
+ +
O NH3 O NH3
and

trendyline
MJC Prelim 2016 9 chemistry unit @ mjc
770 trendyline
1

H
NATIONAL JUNIOR COLLEGE
SH2 PRELIMINARY EXAMINATION
Higher 2

CANDIDATE
NAME

SUBJECT REGISTRATION
CLASS NUMBER

CHEMISTRY 9647/01
Paper 1 Multiple Choice
Friday16Sept 2016
Additional Materials: Multiple Choice Answer Sheet
1 hour
Data Booklet

READ THESE INSTRUCTIONS FIRST

Write in soft pencil.

Do not use staples, paper clips, highlighters, glue or correction fluid.

Write your name, subject class and registration number on the Answer Sheet in the spaces
provided unless this has been done for you.

There are 40 questions on this paper. Answer all questions. For each question there are four
possible answers A, B, C and D.

Choose the one you consider correct and record your choice in soft pencil on the separate
Answer Sheet.

Read the instructions on the Answer Sheet very carefully.

Each correct answer will score one mark. A mark will not be deducted for a wrong answer.
Any rough working should be done in this booklet.

trendyline
This paper consists
s of
o 20 printed
ted pages
p
pages..

NJC/H2 Chem/01/2016 [Turn over


771 trendyline
2

Instructions on how to fill in the Optical Answer Sheet

Example:
Shade the index number in a 5 digit format on the optical answer sheet:2nd digit and the last
4 digits of the Registration Number.

Student Example of Registration No. Shade

Tan Ah Teck 1505648 55648

trendyline
NJC/H2 Chem/01/2016 [Turn over
772 trendyline
3

Section A

For each question there are four possible answers A, B, C, and D. Choose the one you consider to
be correct.

1 3 g of hydrogen reacted with 160 g of bromine gas to give hydrogen bromide, HBr.

How many molecules are present at the end of the reaction?

A 6.02 1023 C 12.0 1023

B 9.03 1023 D 15.1 1023

2 In an experiment, 50 cm3 of a 0.1 mol dm3 solution of a metallic salt reacts exactly with
25 cm3 of 0.1 mol dm3 aqueous sodium sulfite.
The half-equation for the oxidation of sulfite ion is shown below:
SO 3 2(aq) + H 2 O(l) SO 4 2(aq) + 2H+(aq) + 2e
If the original oxidation number of the metal ion is +3, what is the new oxidation number of the
metal ion?

A 0 B 1 C 2 D 4

trendyline
NJC/H2 Chem/01/2016 [Turn over
773 trendyline
4

3 The graph below shows the variation in the third ionisation energies for the consecutive
elements M to U in the Periodic Table, all with proton number below 20.
The symbols M to U do not represent the actual elements.

8000

6000
3rdIE / kJ mol1

4000

2000

0
M N O P Q R S T U
Elements
Elements

What can be deduced from the above graph?


A M has a noble gas configuration.
B M reacts with R to form compound MR 2 .
C The atomic radius of T is smaller than that of S.
D The decrease in 3rd I.E from O to P is due to inter-electronic repulsion.

4 Which set of compounds includes a giant ionic structure, a giant covalent structure and a
simple covalent structure?

A C 60 , C 2 H 5 CO 2 NH 4 , SiO 2 C CO 2 , Na 2 O, SO 2

B Al 2 O 3 , PbO, SiO 2 D AlCl 3 , P 4 O 6 , KHF 2

5 Which pair of species has the same bond angle?

A IF 3 , NF 3

trendyline
B NO 2 ,SO
O2
C XeF 4 , CF
F4
D O 3 2
NO 3 , CO

NJC/H2 Chem/01/2016 [Turn over


774 trendyline
5

6 Which statement best explains why iodine is a solid while water is a liquid at room
temperature?

A Iodine has a much larger molar mass than water.


B Iodine has stronger and more extensive covalent bonds between its atoms in its
molecular structure.
C The van der Waals forces of attraction between iodine molecules are stronger than the
intermolecular hydrogen bonding in water.
D The intermolecular hydrogen bonding in water is stronger than the intermolecular van der
Waals forces of attraction in iodine.

7 How does the volume of a 1.0 dm3 sample of gas change when it is heated from 20 C to40
C at constant pressure?

A It increases to about 1.1 dm3.


B It increases to about 2.0 dm3.
C It decreases to about 0.9 dm3.
D It decreases to about 0.5 dm3.

8 The data below refer to gas phase reactions at constant pressure.

CH 3 CH 3 CH 3 CH 2 + H H= +412 kJ mol1

CH 3 CH 2 CH 2 =CH 2 + H H= +168 kJ mol1

What is the enthalpy change for the following reaction?

2CH 3 CH 2 CH 3 CH 3 + CH 2 =CH 2

A +244 kJ mol1
B 122 kJ mol1
C 244 kJ mol1
D 580 kJ mol1

trendyline
NJC/H2 Chem/01/2016 [Turn over
775 trendyline
6

9 Use of the Data Booklet is relevant to this question.


The energy level diagram shown belowinvolves magnesium and water.

Mg2+(g) + 2H+(aq) + 2OH(aq) + 2e


Energy
/ kJ 116
B
Mg2+(g) + 2H2O(l) + 2e-

1926

Mg2+(aq) + 2H+(aq) + 2OH(aq)+ 2e

Mg(g) + 2H2O(l)
+146
Mg(s) + 2H2O(l) 796

C
Mg2+(aq) + 2OH(aq) + H2(g)

Which statement is incorrect?

A Step A represents the first and second ionisation energies of magnesium.

B Step B represents 2 H f (H 2 O).

C The enthalpy change for step C is 274 kJ mol1.

D The enthalpy change of hydration of magnesium ion is 1926 kJ mol1.

trendyline
NJC/H2 Chem/01/2016 [Turn over
776 trendyline
7

10 Measured amounts of hydrogen and iodine are allowed to reach an equilibrium at 300 C in
a container of known volume.
H 2 (g)+ I2 (g) 2HI(g)
From which experimental method can the equilibrium constant, K c , be determined?

A measuring the total pressure in the container

B slow cooling to 25 oC, breaking open the container under aqueous potassium iodide,
and titrating the iodine present with aqueous sodium thiosulfate

C rapid cooling to 25 oC, breaking open the container under aqueous potassium iodide,
and titrating the iodine present with aqueous sodium thiosulfate

D withdrawal of a measured sample of the equilibrium mixture, followed by complete


decomposition of the hydrogen iodide present, and then titrating the total amount of
iodine with aqueous sodium thiosulfate

11 For the reaction:


2E(g) 3F(g)
the numerical values of the equilibrium constant, K p , are 1.50 107 at 500 K and 1.60 106
at600 K.
Which statement about the equilibrium is true?

A Decreasing the pressure increases the proportion of E in the equilibrium mixture.

B Adding a catalyst increases the proportion of F in the equilibrium mixture.

C The forward reaction is endothermic.

D The reaction is feasible at all temperatures.

12 The rate of removal of paracetamol, a pain-killing drug, from the body is a first order reaction
with a rate constant, k = 0.26 h1.
How long will it take to remove 75% of the paracetamol that a patient consumes?

trendyline
A 0.20 h B 0.26 h C 2.7 h D 5.3 h
5.

NJC/H2 Chem/01/2016 [Turn over


777 trendyline
8

13 Using a colorimeter, the following reaction is studied by finding the time taken for acoloured
reactant, A, to decolourise. The reaction is catalysed by Y.

A+B
Y
C + D
The following results are obtained:

Vol of solution added / cm3


Experiment Time taken / s
A B Y H2O
1 10 20 10 10 20
2 10 10 10 20 40
3 10 20 5 15 40
4 5 20 10 15 10

What is the rate equation for the reaction?

A rate = k[A][B] C rate = k[B][Y]

B rate = k[A][Y] D rate = k[A][B][Y]

14 Use of the Data Booklet is relevant to this question.


Which reagent can be used to convert Na 4 Fe(CN) 6 to Na 3 Fe(CN) 6 ?

A Cl(aq) B Cu2+(aq) C Fe3+(aq) D Mn2+(aq)

15 Water dissociates into ions to establish following equilibrium:


H 2 O(l) H+(aq) + OH(aq)
Which is true about the dissociation of water?

A Dissociation of water is an exothermic process.

B When temperature increases, pH becomes lower than pOH.

C Water is only neutral at room temperature.

trendyline
D ure
re water, the values of its dissociation constant
For pure const (K product (K w ) and [H+]
(K a ), ionic pr
increase
ase in the order:
order K a <
<KK w < [H+].

NJC/H2 Chem/01/2016 [Turn over


778 trendyline
9

16 Bromocresol green indicator has a pH range of 3.8 5.5. It is yellow at pH < 3.8 and blue at
pH > 5.5.
Which statement about bromocresol green is incorrect?

A The indicator is suitable for a strong acidweak base titration.

B The indicator is yellow in 0.10 mol dm3 of an acid with K a = 105 mol dm3.

C The indicator is green in a solution containing equimolar concentrations of


CH 3 CO 2 H and CH 3 CO 2 K. (pK b of CH 3 CO 2 K = 9.24)

D When an acid in the conical flask is titrated against a base, the indicator changes
from yellow to blue at the end-point.

17 Four compounds of some Period 3 elements are listed below.

Na 2 O SiCl 4 SO 2 NaCl

Water is added separately to each compound. Pairs of the resulting solutions are mixed
together.

Which pair of solutions would give a solution of pH 7?

A NaCl and Na 2 O C NaCl and SO 2

B Na 2 O and SiCl 4 D SiCl 4 and SO 2

18 In the preparation of silicon, silicon dioxide is heated with magnesium.

SiO 2 + 2Mg 2MgO + Si

The product mixture contains only MgO and Si only.

To separate the silicon from the product mixture, a student proposed the following methods.

1 Shake the mixture with aqueous hydrochloric acid and filter.


2 Heat the mixture gently and collect the evaporated silicon.

Which methods would work?

trendyline
A 1 only
y C 1 and 2

B 2 onlyy D neither
neither 1 or 2

NJC/H2 Chem/01/2016 [Turn over


779 trendyline
10

19 The high UHDFWLYLW\RIIOXRULQHLVODUJHO\GXHWRWKHORZHQHUJ\RIWKH)F bond.

Which statement best DFFRXQWVIRUWKHZHDN))ERQG"

A The FF bond is weak because of the repulsion between the non-bonding electrons.

B The FF bond is weak because of the short bond length.

C The FF bond is weak because of the small nuclear charge of fluorine atom.

D The FF bond is weak because of the small size of fluorine atom.

20 7KHPROHFXOHLVRSURS\OFKROHVWDQHZKLFKKDVEHHQLVRODWHGIURPDFODVVRIVSRQJHFDQ
serve as a biomarker and has determined the first revolutionary appearances from some
species.

LVRSURS\OFKROHVWDQH

Carbon atoms in a hydrocarbon molecule are classified as primary, secondary, tertiary or


quaternary, depending on whether they are directly bonded to one, two, three or four other
carbon atoms.

How many tertiary and chiral carbon atoms are presentin a molecule of
LVRSUopylcholestane?

number of number of
tertiary carbon atoms chiral carbon atoms
A 9 4

B 9 8

tren
ndylin
n
C 11 4

D 11
1 8

NJC/H2 Chem/01/2016 [Turn over


780 trendyline
11

21 A reaction pathway diagram is shown below.

Which reactions could have this profile?

I (CH 3 ) 3 CBr + NaOH (CH 3 ) 3 COH + NaBr

II CH 3 CH 2 Br + NaOH CH 3 CH 2 OH + NaBr

III C 6 H 5 CH 2 Cl + NH 3 C 6 H 5 CH 2 NH 2 + HCl

IV (CH 3 ) 3 CCH 2 CH 2 Cl + NH 3 (CH 3 ) 3 CCH 2 CH 2 NH 2 + HCl

A Iand IIonly

B I and III only

C II and IV only

D II, III and IV only

22 The alkanes used to be known as the paraffin hydrocarbons paraffin meaning lack of
affinity (i.e. unreactive).

Which statement is best explains the lack of affinity in alkanes?

A The atoms are arranged tetrahedrally around each carbon atom.

B The alkanes have van der Waals forces of interaction.

trendyline
C DUHQRVLJQLILFDQWGLSROHPRPHQWVLQ& +DQ
+DQG& &ERQGV
7KHUHDUHQRVLJQLILFDQWGLSROHPRPHQWVLQ&+DQG&&ERQGV

D LVIUHHURWDWLRQDERXW&&VLQJOHERQGV
LVIUHHURWDWLRQDERXW&&VLQJOHERQGV
7KHUHLVIUHHURWDWLRQDERXW&&VLQJOHERQGV

NJC/H2 Chem/01/2016 [Turn over


781 trendyline
12

23 A new industrial preparation of ethyl ethanoate has been developed using cheap sources of
ethanol.

Which type of reaction is involved at some stage of this reaction sequence?

A nucleophilic addition

B condensation

C disproportionation

D reduction

24 Which compound is least likely to be prepareddirectly from methylbenzene?


COOH

CH3

Br

CH2Br

CH3
NO2
D

trendyline
NJC/H2 Chem/01/2016 [Turn over
782 trendyline
13

25 Compound Y can be prepared from compound X in the following reaction sequence.

(CH2)4CH3
(CH2)4CH3

CH3
CH3

Step 1 O
OH C
C CH3
CH3 H2C
H2C

Compound X
Step 2

(CH2)4CH3

CH3

O(CH2)2CO2H
C
H2C CH3

Compound Y

Which reagents can be used to prepare compound Y?

Step I Step II

A Na CH 2 (OH)(CH 2 ) 2 CO 2 H

B NaOH CH 2 ClCH 2 CO 2 H

C NaOH CH 2 ClCH 2 CH 2 CO 2 H

D Na 2 CO 3 CH 2 (OH)CH 2 CO 2 H

NJC/H2 Chem/01/2016 [Turn over


783 trendyline
14

26 Compound Zis optically active and exists as zwitterions.

Which could be the structure ofZ?

A OH C CH(OH)CH3
CO2H
NO2

CH3
NH2

B H CH2CH3 D CH(NH2)CH3
H N C CO2H
CH2CH3
CO2H

27 Which isthe most feasiblesynthetic route?

A H H H H
aqueous NaOH acidified KMnO4
H C C H H C C H O C C O
heat heat
Br Br OH OH OH OH
B CH3 CH3 CO2H
Br Br
aqueous Br2 acidified KMnO4
FeBr3 heat

C OH OH OCOCH3
Br Br Br Br
aqueous Br2 CH3CO2H

conc. H2SO4, heat

Br Br
D O CH2I HO CH2I
CO2

NaBH4 I2

t nd e
trendyline
aqueous NaOH,
NaOH heat

NJC/H2 Chem/01/2016 [Turn over


784 trendyline
15

28 Compound Y gives a positive result when treated with [Ag(NH 3 ) 2 ]+and PCl 5 but a negative
result when treated with alkaline Cu(II) tartrate.

What could Y be?

A C
O O
OH
OH

B D
O O
OH

O O

29 What is the order of increasing pK a of the organic compounds?

A CCl 3 CH 2 OH, CHCl 2 CH 2 OH, CH 3 CO 2 H

B CH 3 CHClCO 2 H, CH 3 CH 2 CO 2 H, C 6 H 5 OH

C C 6 H 5 OH, CH 3 CHClCO 2 H, CH 3 CHFCO 2 H

D CH 2 ClCH 2 CO 2 H, CH 3 CHClCO 2 H, CH 3 CH 2 CO 2 H

trendyline
NJC/H2 Chem/01/2016 [Turn over
785 trendyline
16

30 The first stage in the synthesis of compound P, a drug used in reducing fever, is the reaction
between compound O and phenylhydrazine.

CH3COCH2CO2CH2CH3 + NHNH2 P

O phenylhydrazine

What is P?

A
CH3COCH2CONH

B CH3
N C
CH2CO2CH2CH3

C CH3
N N C
CH2CO2CH2CH3
H

D CH2CH3
N N C
CH2COCH3
H

trendyline
NJC/H2 Chem/01/2016 [Turn over
786 trendyline
17

Section B

For each of the questions in this section, one or more of the three numbered statements 1 to 3 may
be correct. Decide whether each of the statements is or is not correct (you may find it helpful to put
a tick against the statements that you consider to be correct.)

The responses A to D should be selected on the basis of

A B C D

1,2, and 3 are 1 and 2 only are 2 and 3 only are


1 only is correct
correct correct correct

No other combination of statements is used as a correct response.

31 For which pair does the first species contain more unpaired electrons than the second
species?

1 N, C+
2 Mn2+, Co2+
3 Mn4+, Co

32 Which reactions represent standard enthalpy changes?

1 NH 3 (g) + HCl(g) NH 4 Cl(s)


2 6C(s) + 6H(g) C6H6(l)
3
CH3OH(l) + O2(g) CO2(g) + 2H2O(g)

33 At 298 K, the numerical values for the dissociation constant of the aliphatic carboxylicacids,
RCO 2 H and R 1 CO 2 H in aqueous solution are 2.1 108 and 2.2 104 respectively.
Which statements can be inferred from the given information?

1 The pH of 1 mol dm3RCO 2 H is greater than 1 mol dm3R 1 CO 2 H.

2 The K b of RCO 2 is greater than that of R 1 CO 2 .

3 The acid strength of RCO 2 H is greater than that of R 1 CO 2 H.

trendyline
NJC/H2 Chem/01/2016 [Turn over
787 trendyline
18

The responses A to D should be selected on the basis of

A B C D

1,2, and 3 are 1 and 2 only are 2 and 3 only are


1 only is correct
correct correct correct

No other combination of statements is used as a correct response.

34 Which statements correctly describe the role of ammonia in the reactions below?

reaction role of ammonia

1 CuO + 6NH 3 [Cu(NH 3 ) 4 ]2+ + 2NH 2 + H 2 O Brnsted acid

2 2Na + 2NH 3 2NaNH 2 + H 2 oxidising agent

3 CH 3 Cl + NH 3 CH 3 NH 2 + HCl Brnsted base

35 Barium sulfate is opaque to X-rays and barium ions are toxic to humans. In hospitals, before
an X-ray examination is carried out, patients with digestive tract problems are sometimes
given a barium meal, consisting of a suspension of barium sulfate in water.

[Given: K sp of BaSO 4 = 1.3 1010 mol2 dm6; K sp of BaCO 3 = 5.5 1010 mol2 dm6]
Which statements regarding the barium meal are true?

1 Barium sulfate is used because it is insoluble in water and hence is not poisonous
when ingested by mouth.

2 Barium carbonate can be used since it is also insoluble in water.

3 Barium hydroxide can also be used since it is thermally stable in the human body.

NJC/H2 Chem/01/2016 [Turn over


788 trendyline
19

36 Deuterium, D, is the 21H isotope of hydrogen. DBr has the same chemical properties as HBr.

Which compounds could be made by the reaction of DBr with another compound in a single
reaction?

1 Br
D

2 CHBr 2 CHDBr

3 CH 3 CHBr(OD)

37 Which compounds form a single organic product when each is heated separately with
acidified potassium manganate(VII) solution?

1
CH3 CH3

CH3

3 CH 3 CH 2 CH 2 CH=CH 2

38 Ethyl propanoate, CH 3 CH 2 OCOCH 2 CH 3 undergoes acidic hydrolysis in the presence of


H 2 18O.

Which products are formed?

1 CH 3 CH 2 18OH

2 CH 3 CH 2 CO18OH

trendyline
3 CH
H 3 CH 2 OH

NJC/H2 Chem/01/2016 [Turn over


789 trendyline
20

The responses A to D should be selected on the basis of

A B C D

1,2, and 3 are 1 and 2 only are 2 and 3 only are


1 only is correct
correct correct correct

No other combination of statements is used as a correct response.

39 The structure of benzene, C 6 H 6 , is a regular hexagon in which the electrons are described
as delocalised.

Which statements support this structure?

1 Benzene does not undergo addition reactions.

2 The length of the C-C bonds in benzene are intermediate between CC bond in an
alkane and C=C bond in an alkene.

3 The hydrogenation of benzene is less exothermic than that predicted forcyclohexa-


1,3,5-triene.

40 Aigialomycin D, a fungal metabolite, has the structure shown below.

OH O

HO

OH
OH

Aigialomycin D

Which observations arecorrect?

1 It reacts with hot, acidified K 2 Cr 2 O 7 to give a diketone.

2 One mole of AigialomycinD reacts with Na metal to produce 2 moles ofhydrogen.

3 It reacts with cold, dilute acidified KMnO 4 to give a product that has 7 chiral
centres in each molecule.

trendyline End of Paper


Paperr

NJC/H2 Chem/01/2016 [Turn over


790 trendyline
6++3UHOLPLQDU\([DP3DSHU:RUNHG6ROXWLRQV

 '  '  %  %


 &  '  &  '
 %  &  $  %
 $  &  %  %
 '  '  %  '
 &  '  '  %
 $  %  '  $
 &  $  $  &
 %  $  %  $
 &  %  &  &

6HFWLRQ$
+%U+%U 

$PWRI+ PRO
$PWRI%U  PRO OLPLWLQJDJHQW 
+HQFHDPWRI+%UIRUPHG PRO
1RRIPROHFXOHVSUHVHQWDWWKHHQGRIUHDFWLRQ
  


$QV'
$PWRI62 [ PRO

$PWRIH [ PRO
$PWRIPHWDOOLFVDOW  PRO
+HQFHPROHUDWLRRIPHWDOOLFVDOWHLV
6LQFH RULJLQDO R[LGDWLRQ QXPEHU RI WKH PHWDO LQ WKH VDOW LV  DIWHU JDLQLQJ  H WKH QHZ
R[LGDWLRQQXPEHUZLOOEH
$QV&
; J ; J VW,(

; J ; J QG,(
; J ; J UG,(
UGLRQL]DWLRQHQHUJ\SHDNVDW*US,,HOHPHQWVZKHUHWKHUHPRYDORIWKHHOHFWURQLQYROYHVDQ
HOHFWURQLQWKHLQQHUTXDQWXPVKHOOKHQFHUHTXLULQJDORWPRUHHQHUJ\

trendyline
+HQFH HOHPHQW
HPHQW
P 0 LV LQ *US ,, (OHPHQW
W 5 LV LQ *US
*U 9,,
9  6R WKH FRP
9,, FRPSRXQG 05 LV IRUPHG
EHWZHHQHOHPHQWV0DQG5
HOHPHQWV0
HOHPHQWV 0DQG
DQG 5
2LVLQ*US
LVLQ*US,9 LVLQ*US 9 7KHUG,(RI
(OHPHQW2LVLQ*US,9DQGHOHPHQW3LVLQ*US97KH
DQGHOHPHQW 3LVLQ*US
,9DQGHOHPHQW ,(RI2LQYROYHVUHPRYLQJWKHV
(RI2
2LQY
KHUHDV IRU
HOHFWURQ ZKHUHDV IRU 3  LW LQYROYHV UHPRYLQJ WKH S HOHFWURQ 6LQFH HOHFWURQ
3 H LQ S VXEVKHOO
XFOHDU DWWUDFWLRQ WKDQ HOHFWURQ LQ V VXEVKHOO OHVV HQHUJ\ LV U
KDVOHVVQXFOHDUDWWUDFWLRQWKDQHOHFWURQLQVVXEVKHOOOHVVHQHUJ\LVUHTXLUHGWRUHPRYHLW
$QV%

791 trendyline
$ & VLPSOHFRYDOHQWVWUXFWXUH

 &+&221+ JLDQWLRQLFVWUXFWXUH
 6L2 JLDQWFRYDOHQWVWUXFWXUH
  
% &2 VLPSOHFRYDOHQWVWUXFWXUH
 1D2 JLDQWLRQLFVWUXFWXUH
 62 VLPSOHFRYDOHQWVWUXFWXUH
  
& $l2 JLDQWLRQLFVWUXFWXUH
 3E2 JLDQWLRQLFVWUXFWXUH
 6L2 JLDQWFRYDOHQWVWUXFWXUH
  
' $l&l VLPSOHFRYDOHQWVWUXFWXUH
 32 VLPSOHFRYDOHQWVWUXFWXUH
 .+) JLDQWLRQLFVWUXFWXUH

$QV$
$ ,) 1)


7ULJRQDOS\UDPLGDO
7VKDSH
    
% 12 62

EHQW EHQW
    
& ;H) &)


6TXDUHSODQDU
7HWUDKHGUDO
    
' 12 &2



7ULJRQDOSODQDU 7ULJRQDOSODQDU

1RWH 7KRXJK 12 DQG 62 KDYH WKH VDPH VKDSH WKH\ KDYH GLIIHUHQW ERQG DQJOHV GXH WR
GLIIHUHQWUHSXOVLRQEHWZHHQORQHSDLUERQGSDLUDQGORQHHOHFWURQERQGSDLU
$QV'



trendyline
7KHGRPLQDQWLQWHUPROHFXODUIRUFHVRIDWWUDFWLRQEHWZHHQLRGLQHLVWHPSRUDU\GLSROHLQGXFHG
QDQWLQWHUPROHFXODUIRUFHVRIDWWUDFWLRQEHWZHHQLRGLQH
DQWLQWHUPROHFXODUIRUFHVRIDWWUDFWLRQEHWZH LVWHP
GLSROH LQWHUDFWLRQV
HUDFWLRQV ZKLOH WKDW EHWZHHQ ZDWHU LV K\GURJHQ ERQGLQJ
ERQGLQJ 6LQFH
6 LRGLQH H[LVWV DV
VROLG ZKLOH
H ZDWHU H[LVWV DV OLTXLGDW UWS LW LPSOLHV WKDW WKH WHPSRUDU\ GLSROH LQGXFHG GLSROH
LQWHUDFWLRQV
V EHWZHHQ LRGLQH DUH VWURQJHU WKDQ WKH +ERQGV EHWZHHQ
QV EHWZHHQ ZDWHU UHTXLULQJ PRUH
HQHUJ\WREUHDNDQGHQHUJ\DWUWSLVLQVXIILFLHQWWREUHDNWKHPKHQFHWKH\H[LVWDVVROLG
EUHDNDQGHQHUJ\DWUWSLVLQVXIILFLHQWWREUHDNWKHPKHQFH
IILFLH
$QV&

792 trendyline
$WFRQVWDQWSUHVVXUH


  


  

9 GP DSSUR[LPDWHO\GP 

$QV$

 H """

&+&+&+&+&+ &+

H N-PROH N-PRO


&+ &++&+&++

%\+HVVVODZH  N-PRO



$QV&



2SWLRQ$0J J 0J J VW,(QG,(
2SWLRQ%6WDQGDUGHQWKDOS\FKDQJHRIIRUPDWLRQRIZDWHUUHIHUVWRWKHHQHUJ\UHOHDVHG
RUDEVRUEHGZKHQPROHRIZDWHULVIRUPHGIURPLWVFRQVWLWXHQWHOHPHQWVLQWKHLUVWDQGDUG
VWDWHVXQGHUVWDQGDUGFRQGLWLRQV
6WDQGDUG HQWKDOS\ FKDQJH RI QHXWUDOLVDWLRQ LV WKH HQHUJ\ UHOHDVHG ZKHQ RQH PROH RI
ZDWHULVIRUPHGIURP+DQG2+XQGHUVWDQGDUGFRQGLWLRQV

7KHUHIRUHVWHS%UHSUHVHQWVHQHXWUDOLVDWLRQ
2SWLRQ&%\+HVVVODZHIRUVWHS& 
 N-PRO
2SWLRQ'6WDQGDUGHQWKDOS\FKDQJHRIK\GUDWLRQLVWKHHQHUJ\UHOHDVHGZKHQRQHPROHRI
JDVHRXVLRQVLVK\GUDWHGLQWRDTXHRXVLRQVXQGHUVWDQGDUGFRQGLWLRQV
$QV%

trendyline
793 trendyline
 (TXLOLEULXP FRQVWDQW KF LV WHPSHUDWXUH GHSHQGHQW LH LW FKDQJHV RQO\ LI WHPSHUDWXUH
FKDQJHV
2SWLRQ$LVQRWWUXHDVWKHWRWDOSUHVVXUHRIWKHUHDFWLRQPL[WXUHUHPDLQVFRQVWDQWVLQFHWKH
WRWDOQXPEHURIJDVHRXVSDUWLFOHVUHPDLQHGXQFKDQJHG
2SWLRQ%LVQRWWUXHDVWKHSRVLWRQRIWKHHTXLOLEULXPKDVWLPHWRFKDQJHGXHWRVORZFRROLQJ
RIUHDFWLRQPL[WXUH7KHHTPFRQVWDQWGHWHUPLQHGLVOLNHO\RQHDWZHOOEHORZ&
2SWLRQ&LVWUXHDVWKHSRVLWRQRIWKHHTXLOLEULXPKDVQRRUOLWWOHWLPHWRFKDQJHGXHWRUDSLG
FRROLQJRIUHDFWLRQPL[WXUH7KHHTPFRQVWDQWGHWHUPLQHGLVOLNHO\WREHRQHDW&
2SWLRQ'LVQRWWUXHDVWKHPHWKRGJLYHVRQO\WKHLQLWLDODPRXQWRILRGLQHXVHG

$QV&


 2SWLRQ$LVQRWWUXHDVGHFUHDVLQJWKHSUHVVXUHIDYRXUVIRUZDUGUHDFWLRQWRSURGXFHPRUH)
OHVV(ZRXOGUHPDLQ
2SWLRQ%LVQRWWUXHDVDGGLQJDFDWDO\VWZRXOGQRWDIIHFWWKHSRVLWRQRIWKHHTXLOLEULXPEXW
DOORZVWKHHTXLOLEULXPWREHHVWDEOLVKHGIDVWHU
2SWLRQ & LV QRW WUXH DV WKH HTXLOLEULXP FRQVWDQW GHFUHDVHV ZLWK LQFUHDVLQJ WHPSHUDWXUH
ZKLFK LPSOLHV EDFNZDUG UHDFWLRQ LV IDYRXUHG DV WHPSHUDWXUH LQFUHDVHV 7KXV EDFNZDUG
UHDFWLRQPXVWEHHQGRWKHUPLF
2SWLRQ 'LVWUXHDVLWFDQEHGHGXFHGWKDW+DQG6! GXHWRDQLQFUHDVHLQQRRI
JDVHRXV SDUWLFOHV LQ WKH UHDFWLRQ  WKXV *  +  76   DW DOO WHPSHUDWXUHV UHDFWLRQ
VKRXOGEHIHDVLEOHDWDOOWHPSHUDWXUHV
$QV'


 OQ OQ


W     K
N 

:KHQRIWKHSDUDFHWDPROFRQVXPHGLVUHPRYHGLWPHDQV  RIWKHSDUDFHWDPRO
FRQVXPHGUHPDLQVDQGWPXVWKDYHODSVHG
W W K 
$QV'


 6LQFHWRWDOYRORIUHDFWLRQZDVNHSWFRQVWDQWLQDOOH[SWVFRQFHQWUDWLRQRIHDFKUHDFWDQW 
LWVYROXPHXVHG
&RPSDULQJ GDWD IURP H[SW   ZKHQ YRO RI % LV KDOYHG ZKLOH NHHSLQJ YRO RI $ DQG <
FRQVWDQW UDWHLVDOVRKDOYHGRUGHURIUHDFWLRQZUW%LV
&RPSDULQJ GDWD IURP H[SW   ZKHQ YRO RI < LV KDOYHG ZKLOH NHHSLQJ YRO RI $ DQG %
FRQVWDQW UDWHLVDOVRKDOYHGRUGHURIUHDFWLRQZUW<LV
&RPSDULQJJ GDWD IURP H[SW   ZKHQ YRO RI $ LV K NH
KDOYHG ZKLOH NHHSLQJ YRO RI % DQG <
UDWHUHPDLQVXQFKDQJHG RUGHURIUHDFWLRQZUW
FRQVWDQW UDWHUHPDLQVXQFKDQJHGRUGHURIUHDFWLRQZUW$LV
UDWHUHPDLQVXQFKDQJHG RUGHURIUHDFWLRQZUW$ $LV
$QV&



794 trendyline
 8VLQJ(RYDOXHVIURPData BookletRQO\)H DT FDQR[LGLVH1D)H &1 WR1D)H &1 DV
(RFHOO    9!DQGUHDFWLRQLVIHDVLEOH

2SWLRQ$LVQRWWUXHDV&l DT LVDUHGXFLQJDJHQW

2SWLRQ%LVQRWWUXHDV(RFHOO   LI&X&XRU(RFHOO   
LI&X&X
2SWLRQ'LVQRWWUXHDV(RFHOO   

$QV&


 2SWLRQ $LVQRW WUXHDVGLVVRFLDWLRQRIZDWHULQYROYHVEUHDNLQJRI2+ERQGVDQGPXVWEH


HQGRWKHUPLF
2SWLRQ % LV QRW WUXH DV GLVVRFLDWLRQ RI ZDWHU DOZD\V UHVXOWV LQ WKH VDPH DPRXQW RI + DT 
DQG2+ DT IRUPHGS+ S2+
2SWLRQ & LV QRW WUXH DV >+ DT @  >2+ DT @ LV DOZD\V WUXH LH ZDWHU LV QHXWUDO DW DOO
WHPSHUDWXUHV
.Z
2SWLRQ'LVWUXHDVYDOXHVRI.D  [.Z DQG>+@ 


$QV'


 2SWLRQ $ LV WUXH DQG KHQFH LV QRW WKH DQVZHU $W WKH HTXLYDOHQFH SRLQW RI D VWURQJ DFLG 
ZHDNEDVHWLWUDWLRQWKHVROXWLRQXVXDOO\KDVS+
2SWLRQ %LVWUXHDQGKHQFHLVQRWWKHDQVZHUS+RIPROGPDFLGZLWK.D PRO
GPLVDQGWKHLQGLFDWRULV\HOORZLQFRORXU
2SWLRQ&LVWUXHDQGKHQFHLVQRWWKHDQVZHUS+RIDVROXWLRQZLWK>&+&2+@ >&+&2.@
 S.D  EHWZHHQ VROXWLRQKDV>,Q@>+,Q@DQGLQGLFDWRUZLOOEHJUHHQLQWKH
VROXWLRQ
2SWLRQ'LVQRWWUXHDQGKHQFHLVWKHDQVZHU7KHHQGSRLQWRIDQDFLGEDVHWLWUDWLRQRFFXUV
ZKHQ>,Q@>+,Q@DQGFRORXURIWKHVROXWLRQVKRXOGEHJUHHQ GXHWRDERXWHTXDODPRXQWRI
ERWK\HOORZDQGEOXHVSHFLHV &RORXUFKDQJHDWHQGSRLQWVKRXOGEHIURP\HOORZWRJUHHQ
QRWEOXH
$QV'

trendyline
795 trendyline
 1D&lGLVVROYHVLQZDWHUWRIRUPDQHXWUDOVROXWLRQRIS+ 

1D&l V DT1D DT &l DT 

1DGRHVQRWK\GURO\VHLQZDWHUEHFDXVHRIORZFKDUJHGHQVLW\&lLVDZHDNFRQMXJDWH
EDVHRIWKHVWURQJDFLG+&lKHQFHGRHVQRWK\GURO\VHLQZDWHU

6L&lK\GURO\VHVLQZDWHUWRJLYHDQDFLGLFVROXWLRQRIS+ 

6L&l l +2 l 6L2+2 V +&l DT 


1D2GLVVROYHVLQZDWHUWRIRUPDVWURQJO\DONDOLQHVROXWLRQRI1D2+(pH = 13 or 14).

1D2 V +2 l 1D2+ DT 

62GLVVROYHVLQZDWHUWRIRUPDQDFLGLFVROXWLRQRI+62 DT 

62 J +2 l +62 DT 

$ 1D&lDQG1D2S+! & 1D&lDQG62S+

% 1D2DQG6L&lS+  ' 6L&lDQG62S+


+HQFHPL[LQJRI

$QV%


 6L LV LQVROXEOH LQ +&l ZKLOH 0J2 UHDFWV ZLWK +&lWR JLYH 0J&l DQG ZDWHU DFLGEDVH
UHDFWLRQ +HQFH6LUHPDLQVDVUHVLGXHZKLOHWKHILOWUDWHFRQWDLQV0J&l DT 

0J2DQG6LGRQRWHYDSRUDWHRQKHDWLQJGXHWRKLJKPHOWLQJERLOLQJSRLQW

$QV$

 ,QWHUHOHFWURQLF UHSXOVLRQ EHWZHHQ WKH ORQH SDLUV RQ ) DWRPV LQ D ) PROHFXOH FDXVHV WKH
))ERQGWREHORQJHUDQGZHDNHUWKDQH[SHFWHG
$QV$

7HUWLDU\FDUERQVFLUFOHGLQUHG

&KLUDOFDUERQVZLWK 




trendyline



$QV%


796 trendyline
5HDFWLRQSDWKZD\GLDJUDPVKRZV61PHFKDQLVPLHVWHSUHDFWLRQ

61LVIDYRXUHGZKHQWKHUHDFWDQWFDQIRUPDVWDEOHFDUERFDWLRQLQWHUPHGLDWHKHQFHXVXDOO\
LQYROYHVWHUWLDU\KDORJHQRDONDQHV LHRSWLRQ, DQGQRWSULPDU\KDORJHQRDONDQHV RSWLRQV,,
DQG ,9  (OHFWURQ GRQDWLQJ DON\O JURXSV FDQ KHOS WR HDVH WKH SRVLWLYH FKDUJH RQ WKH
FDUERFDWLRQ

)RU RSWLRQ ,,, &+&+&l FDQ IRUP D VWDEOH FDUERFDWLRQ &+&+ WKH FDUERFDWLRQ LV
VWDELOLVHGE\UHVRQDQFHDVWKHHOHFWURQGHILFLHQWFDUERQLVDGMDFHQWWRDEHQ]HQHULQJ
$QV%

'XHWRWKHODFNRIGLSROHPRPHQWVLQDONDQHPROHFXOHVWKHUHLVDODFNRIHOHFWURQULFKDQG

HOHFWURQGHILFLHQWFHQWUHVWRDWWUDFWQXFOHRSKLOHVDQGHOHFWURSKLOHVIRUUHDFWLRQ
$QV&

6WHSR[LGDWLRQRISULPDU\DOFRKROWRDOGHK\GH

6WHSQXFOHRSKLOLFDGGLWLRQRIDOGHK\GH
6WHSR[LGDWLRQ ORVVRIK\GURJHQ 
$QV$

2SWLRQ$R[LGDWLRQRIPHWK\OVLGHFKDLQRQ&+&+JLYHV&+&22+

2SWLRQ% HOHFWURSKLOLF VXEVWLWXWLRQ RI &+&+ JLYHV EURPRPHWK\OEHQ]HQH RU
EURPRPHWK\OEHQ]HQH DV WKH PDLQ SURGXFWV EURPRPHWK\OEHQ]HQH LV D PLQRU SURGXFW
$ON\OFKDLQRQEHQ]HQHLVGLUHFWLQJ
2SWLRQ&)UHHUDGLFDOVXEVWLWXWLRQRIPHWK\OVLGHFKDLQRQ&+&+JLYHV&+&+%U
2SWLRQ' HOHFWURSKLOLF VXEVWLWXWLRQ RI &+&+JLYHV LQ QLWURPHWK\OEHQ]HQH RU
QLWURPHWK\OEHQ]HQHDVWKHPDLQSURGXFWV
$QV%

,Q6WHS,SKHQROFDQRQO\EHGHSURWRQDWHGE\HLWKHU L 1DRU LL 1D2+EXWQRW1D&2

6WHS,,LVDQXFOHRSKLOLFDWWDFNRQDQHOHFWURQGHILFLHQWFDUERQ &ERQGHGWR&lLQVWHDGRI&
ERQGHGWR2+VLQFHWKH&&lERQGLVZHDNHU &KHFNWKDWWKHQXPEHURIFDUERQDWRPVDUH
FRUUHFW
$QV%

$]ZLWWHULRQLVDGLSRODUVSHFLHVZLWKQRQHWFKDUJHV ZKHUH&2+FDQEHGHSURWRQDWHGE\

WKH 1+ JURXS  6R RSWLRQV $% DQG ' KDYHSRVVLELOLW\ RI IRUPLQJ ]ZLWWHULRQV ;PXVW DOVR

trendyline
FKLUDOFHQWUH FDUERQERQGHGWRIRXUGLIIHUHQWJURXSV
KLUDOFHQWUH FDUERQERQGHGWRIRXUGLIIHUHQW VLQFHLW
FRQWDLQDFKLUDOFHQWUH FDUERQERQGHGWRIRXUGLIIHUHQWJURXSV VLQFHLWLVRSWLFDOO\DFWLYH
$QV'

797 trendyline
$ + + + +

DTXHRXV1D2+ DFLGLILHG.0Q2
+ & & + + & & + 2 & & 2
KHDW KHDW
%U %U 2+ 2+ 2+ 2+ 

&22+  ZLOO JHW R[LGLVHG WR &2 DQG +2 ZKHQ .0Q2 LV XVHG DV R[LGLVLQJ
DJHQW
% &+ &22+
&+
%U %U
DTXHRXV%U DFLGLILHG.0Q2

)H%U KHDW


7KH VWVWHS VKRXOG EH FDUULHG RXW XQGHU DQK\GURXV FRQGLWLRQV DV ZDWHU ZLOO
GHVWUR\WKHKDORJHQFDUULHU)H%U
& 2+ 2+ 2&2&+

DT%U %U %U %U %U
&+&2+

FRQF+62KHDW

%U %U


7KHQGVWHSFDQRQO\EHFDUULHGRXWZLWKDFLGFKORULGH LQWKHSUHVHQFHRIDT
1D2+  DV SKHQROV DUH WRR XQUHDFWLYH WR UHDFW ZLWK FDUER[\OLF DFLG LQ D
FRQGHQVDWLRQUHDFWLRQ
' 2 &+ , +2 &+ , 
&22

1D%+ ,

1D2+KHDW


7KHVWVWHSLVDUHGXFWLRQRINHWRQHWRDVHFRQGDU\DOFRKRO7KHQGVWHSLVDQ
LRGRIRUP R[LGDWLRQ UHDFWLRQGXHWRSUHVHQFHRI&+ 2+ &+,
1RWH&+ FDQ EH UHSODFHG E\ &+, RU &+, LQ WKH VWUXFWXUH %RWK VWHSV DUH
IHDVLEOH

$QV'

3RVLWLYHWHVWZLWK>$J 1+ @ 7ROOHQVUHDJHQW  !$OGHK\GHLVSUHVHQW

3RVLWLYHWHVWZLWK3&l !5&2+RU52+LVSUHVHQW QRWSKHQRO 
1HJDWLYHWHVWZLWKDONDOLQH&X ,, WDUWUDWH )HKOLQJVVROXWLRQ  !DOLSKDWLFDOGHK\GHLVDEVHQW
$QV$

trendyline
798 trendyline
$ &&l&+2+&+&l&+2+&+&22+

&+&22+LVWKHPRVWDFLGLFDVLWLVDFDUER[\OLFDFLGWKHILUVWWZRPROHFXOHV
DUHDOFRKROV
% &+&+&l&22+&+&+&22+&+2+

&+&+&l&22+DQG&+&+&22+DUHPRUHDFLGLFWKDQ&+2+DVWKH\DUH
FDUER[\OLFDFLGVZKLOH&+2+LVSKHQRO&+&+&l&22+LVPRVWDFLGLFDVWKH
HOHFWURQ ZLWKGUDZLQJ &l GLVSHUVHV WKH QHJDWLYH FKDUJH RQ WKH FRQMXJDWH EDVH
DQGKHQFHVWDELOLVHVLW
& &+2+&+&+&l&22+&+&+)&22+

3KHQROLVOHVVDFLGLFWKDQFDUER[\OLFDFLGV

' &+&l&+&22+&+&+&l&22+&+&+&22+

&+&+&l&22+ LV PRUH DFLGLF WKDQ &+&l&+&22+ GXH WR WKH FORVHU
SUR[LPLW\RIWKHHOHFWURQZLWKGUDZLQJ&lWRWKH&22+ZKLFKFDQEHWWHUKHOSWR
GLVSHUVHWKHFKDUJHRQWKHFRQMXJDWHEDVH

$QV%
 GLQLWURSKHQ\OK\GUD]LQH UHDFWV ZLWK FDUERQ\O FRPSRXQGV YLD FRQGHQVDWLRQ IRUPLQJ D

QHZ& 1ERQG3KHQ\OK\GUD]LQHUHDFWVLQVLPLODUZD\ZLWKWKHNHWRQHLQFRPSRXQG27KH
HVWHUIXQFWLRQDOJURXSGRHVQRWUHDFWZLWKSKHQ\OK\GUD]LQH
$QV&

6HFWLRQ%
 1 VVS XQSDLUHGH

 & VVS XQSDLUHGH
   
 0Q >$U@G XQSDLUHGH
 &R >$U@G XQSDLUHGH
   
 0Q >$U@G XQSDLUHGH
 &R >$U@GV XQSDLUHGH

$QV%
2SWLRQ  +\GURJHQ VKRXOG H[LVW DV PROHFXODU + J  LQVWHDG RI + J  XQGHU VWDQGDUG

FRQGLWLRQV

2SWLRQ+2VKRXOGH[LVWDVOLTXLGLQVWHDGRIJDVXQGHUVWDQGDUGFRQGLWLRQV

$QV'

trendyline
 VWUXHDV5&2+KDVDVPDOOHU.
2SWLRQLVWUXHDV5&2 KDVDVPDOOHU.DPHDQVLWLVDZHDNHUDFLG
PHDQVLWLVDZHDNHUDFLGDQGGLVVRFLDWHVLQDT
HDQVLWLVDZHDNHUDFL
JLYHDORZHU>+ DT @DQGWKXVKDVDKLJKHUS+
JLYHDORZHU>+
VROXWLRQWRJLYHDORZHU>+ DT @DQGWKXVKDVDKLJKHUS+
DT @DQGWKXVKDVDKLJKHUS+
V WUXH DV .D [ .E  .Z  
2SWLRQ  LV 
RO GP $Q DFLG ZLWK D OD
 PRO OODUJHU .D ZRXOG JLYH D
FRQMXJDWHEDVHZLWKDORZHU.
EDVHZLWKDORZHU.EYDOXHDQGYLFHYHUVD
YDOXHDQGYLFHYHUVD
YLFH
2SWLRQLVQRWWUXHDV5&2+KDVDODUJHU.DPHDQVLWLVWKHVWURQJHUDFLG
$QV%
799 trendyline
 2SWLRQLVWUXHDVHDFK1+GRQDWHV+ DFWDV%URQVWHGDFLG WRIRUP1+
2SWLRQLVWUXHDV1DLVR[LGLVHGWR1DLQ1D1+
2SWLRQLVQRWWUXHDV1+DFWVDQXFOHRSKLOHLQWKHUHDFWLRQ

$QV%

 2SWLRQ  LV WUXH DV %D62 ZLOO UHPDLQ LQVROXEOH DQG QRW GLVVRFLDWH WR JLYH SRLVRQRXV %D
LRQV
2SWLRQ LVQRWWUXHDV%D&2ZLOOUHDFWZLWK+&l DT LQWKHVWRPDFKWRUHOHDVHSRLVRQRXV
%DLRQV
2SWLRQLVQRWWUXHDV%D 2+ ZLOOUHDFWZLWK+&l DT LQWKHVWRPDFKWRUHOHDVHSRLVRQRXV
%DLRQV

$QV'

 %U
'
'%U
 

&+%U &+%U'%U&+%U&+'%U
 FDQQRWEHIRUPHGLQDVLQJOHVWHS

$QV%


 &+

&+&2&+&2+

&+

 2

2
&+
&+

 &+&+&+&+ &+&+&+&+&22+&2+2


$QV$

trendyline
800 trendyline
 &+&+2&2&+&++2&+&+2+&+&+&22+DVWKH+RI+2DGGVWRWKH
DOFRKRODQGWKH 2+RI+2DGGVWRWKHFDUER[\OLFDFLGLQK\GURO\VLV +2+ERQGLQZDWHU
EUHDNV 

$QV&

 %HQ]HQHGRHVQRWXQGHUJRDGGLWLRQUHDFWLRQV

7UXH $GGLWLRQ ZLOO UHVXOW LQ D ORVV RI DURPDWLF VWDELOLW\ RI EHQ]HQH ORVV RI
DOWHUQDWHVLQJOHDQGGRXEOHERQG 

 7KH OHQJWK RI WKH && ERQGV LQ EHQ]HQH DUH LQWHUPHGLDWH EHWZHHQ &&
ERQGLQDQDONDQHDQG& &ERQGLQDQDONHQH
7UXH7KHERQGRUGHURIDOO&&ERQGVLQEHQ]HQHDUHGXHWRUHVRQDQFH
ZKLFKDULVHVIURPWKHFRQWLQXRXVRYHUODSRIXQK\EULGLVHGSRUELWDOV

 7KH K\GURJHQDWLRQ RI EHQ]HQH LV OHVV H[RWKHUPLF WKDQ WKDW SUHGLFWHG
IRUF\FORKH[DWULHQH
7UXH%HQ]HQHLVPRUHVWDEOHWKDQF\FORKH[DWULHQHGXHWRUHVRQDQFH
7KXV LW H[LVWV DW D ORZHU HQHUJ\ OHYHO VHH EHORZ  +\GURJHQDWLRQ RI
EHQ]HQHJLYHVF\FORKH[DQH



$QV$

trendyline
801 trendyline
 ,WUHDFWVZLWKKRWDFLGLILHG.&U2WRJLYHDGLNHWRQH

)DOVH+RWDFLGZLOOK\GURO\VHWKHHVWHUWRJLYHDVHFRQGDU\DOFRKROZKLFKZLOO
WKHQEHR[LGLVHGWRJLYHDNHWRQH7KHRWKHUWZRVHFRQGDU\DOFRKROVZLOODOVR
EHR[LGLVHGWRJLYHNHWRQHV6RWKHSURGXFWZLOOEHDWULNHWRQH
 2QH PROH RI $LJLDORP\FLQ UHDFWV ZLWK 1D PHWDO WR SURGXFH  PROHV RI
K\GURJHQJDV
7UXH2QHPRORIK\GUR[\OJURXSUHDFWZLWK1DWRSURGXFHPROHRIJDV
6LQFH WKHUH LV  K\GUR[\ JURXSV SHU PRO RI $LJLDORP\FLQ  PROHV RI
K\GURJHQJDVDUHSURGXFHG
 ,WUHDFWVZLWKFROGGLOXWHDFLGLILHG.0Q2WRJLYHDSURGXFWWKDWKDVFKLUDO
FHQWUHV
7UXH$ONHQHXQGHUJRPLOGR[LGDWLRQWRJLYHGLRO3URGXFWLVVKRZQEHORZ
2+ 2 &+

2
2+
2+
+2
+2
+2 2+
2+ 

$QV&


trendyline
802 trendyline
H
NATIONAL JUNIOR COLLEGE
SH2 PRELIMINARY EXAMINATION
Higher 2

CANDIDATE
NAME

SUBJECT REGISTRATION
CLASS NUMBER

CHEMISTRY 9647/02
Paper 2 Planning Question
Wednesday 24Aug2016
Candidates answer on the Question Paper.

Additional Materials: Data Booklet

For Examiners Use


READ THESE INSTRUCTIONS FIRST

Write your subject class, registration number and name on all


1 /12
the work you hand in.
Write in dark blue or black pen.
You may use a soft pencil for any diagrams, graphs or rough
working.
Do not use paper clips, highlighters, glue or correction
fluid/tape.

Answers all questions.


A Data Booklet is provided.

The number of marks is given in brackets [ ] at the end of each


question or part question.

Total /12

trendylineThis document consists o


of 6printed
6printed pages
pages..

NJC PRELIM PLANNING QUESTION9647/02/16


803 trendyline
2 For
Examiners
Use
Answer the question.

1 Planning (P)

The solubility of silver chloride at a particular temperature, is defined as the mass of silver
chloride that can dissolve in and just saturate 100 g of a solvent at that temperature.

In a saturated solution in contact with undissolved solid, the following


equilibrium is established:

AgCl(s) + aq Ag+(aq) + Cl(aq) Ksp value = 1.8 1010

(a) A student was tasked to investigate the concentration of chloride ions in a sample of
sea water from the East Coast Park. Keeping in mind the low solubility of silver
chloride, she carried out her investigation through a method known as precipitation
titration, which is useful for reactions that yield ionic compounds of limited solubility.

In her investigation, solutions of chlorides were titrated with aqueous silver ions in the
presence of chromate ions. The end point of the titration occurred when almost all the
chloride ions have been precipitated and is signalled by the appearance of brick-red
silver chromate precipitate, Ag 2 CrO 4 .

The reactions are as follows:

Ag+(aq) + Cl(aq) AgCl(s)

2Ag+(aq) + CrO 4 2(aq) Ag 2 CrO 4 (s)

From the stoichiometry and amount of reactants used, the amount of chloride ions in
the sample of sea water can be determined.

Using the information given above, you are to plan an experiment to determine the
concentration of chloride ions in the sample of sea water.

You may assume that you are provided with:


a sample of 250 cm3 of sea water of approximate chloride concentration of
2.0 103 mol dm3
1.00 mol dm3 potassium chromate(VI) solution, K 2 CrO 4
0.1 mol dm3 silver nitrate solution
deionised water
the apparatus normally found in a school or college laboratory.

Your plan should include details of:


preliminary calculation to decide if the silver nitrate solution provided is suitable
for use in the titration
preparation of a standard silver nitrate solution from the 0.1 mol dm3 silver
nitrate solution provided, if necessary

trendyline
appropriate
ppropriate quantities of solutio
solutions
all essential experimental details
an outline of how the results would be used to determine th the concentration of
hloride ions in the sample of sea water
chloride wat

NJC/H2 Chem/02/2016[Turn over


804 trendyline
3 For
Examiners
Use

....

....

....

....

....

....

....

....

....

....

....

....

....

....

trendyline
....
..
..

....
.

.
NJC/H2 Chem/02/2016 [Turn over

805 trendyline
4 For
Examiners
Use
....

....

....

....

....

....

....

....

....

....

....

....

....

....

....

....

trendyline
....

....
.

....
..

[6]

NJC/H2 Chem/02/2016[Turn over


806 trendyline
5 For
Examiners
Use
(b) The student obtained a titre volume of 20.60 cm3from her experiment.
Determine the concentration of chloride ions remaining in the solution at the end point.
You may assume the following:

25.0 cm3 of sea water was used in the titration


1 cm3 of 1.00 mol dm3 potassium chromate(VI) solution, K 2 CrO 4 was used as
the indicator
K sp value of Ag 2 CrO 4 is 1.1 1012

[2]
(c) Subsequently, the student investigates the solubility of silver chloride in aqueous
hydrochloric acid.

By considering the ions present in a solution of silver chloride in aqueous hydrochloric


acid, predict and explain how the solubility of silver chloride will be affected by the
concentration of the acid.

Prediction:...................

Explanation:.........

....

....

trendyline
.

[2]

NJC/H2 Chem/02/2016 [Turn over

807 trendyline
6 For
Examiners
Use
(d) Identify the following variables in the investigation of solubility of silver chloride in
hydrochloric acid.

Independent variable: .....

Dependent
variable: ....

[1]

(e) Calculate the solubility of silver chloride in a 2 mol dm3 hydrochloric acid solution.

[1]

[Total: 12]

NJC/H2 Chem/02/2016[Turn over


808 trendyline


 1$7,21$/-81,25&2//(*(


6+35(/,0,1$5<(;$0,1$7,216

+LJKHU

&$1','$7( 
1$0(

68%-(&7  5(*,675$7,21 
&/$66 180%(5

 
&+(0,675<
3DSHU6WUXFWXUHG 

$QVZHURQWKH4XHVWLRQ3DSHU :HGQHVGD\$XJ

$GGLWLRQDO0DWHULDOV'DWD%RRNOHW KRXUV


5($'7+(,16758&7,216),567 )RU([DPLQHUV8VH
:ULWH\RXUQDPHVXEMHFWFODVVDQGUHJLVWUDWLRQQXPEHURQDOOWKH
ZRUN\RXKDQGLQ  

:ULWHLQGDUNEOXHRUEODFNLQNLQWKHVSDFHVSURYLGHG  

<RXPD\XVHDVRIWSHQFLOIRUDQ\GLDJUDPVJUDSKVRUURXJK
ZRUNLQJ
 

'RQRWXVHSDSHUFOLSVKLJKOLJKWHUVJOXHRUFRUUHFWLRQIOXLG  

$QVZHUVDOOTXHVWLRQV
 

 
7KHQXPEHURIPDUNVLVJLYHQLQEUDFNHWV>@DWWKHHQGRIHDFK
TXHVWLRQRUSDUWTXHVWLRQ

7RWDO 

trendyline


7KLVGRFXPHQWFRQVLVWVRI
7KLVGRFXPHQWFRQVLVWVRI
 SULQWHGSDJHV
ULQWHGSDJHV
7KLVGRFXPHQWFRQVLVWVRISULQWHGSDJHV

809 trendyline


 /HWKDO'RVH/'LVDPHDQVRIPHDVXULQJWR[LFLW\/'LVWKHDPRXQWRIDPDWHULDOZKLFKFDXVHV
WKH GHDWK RI  RI D JURXS RI WHVW DQLPDOV ,WVYDOXH LV H[SUHVVHG DV WKH PDVV RI D FKHPLFDO
DGPLQLVWHUHGSHUNJERG\PDVVRIDWHVWDQLPDO

/HWKDO &RQFHQWUDWLRQ /& LV DQRWKHU PHDQV RI PHDVXULQJ WR[LFLW\ 7KH YDOXH RI /&LV WKH
FRQFHQWUDWLRQRIDFKHPLFDOLQDLUWKDWNLOOVRIWKHWHVWDQLPDOVGXULQJWKHREVHUYDWLRQSHULRG

7KHWDEOHEHORZVKRZVWKHYDOXHVIRUWKH/'DQG/&DORQJZLWKWKHWR[LFLW\UDWLQJV

7R[LFLW\UDWLQJ &RPPRQO\XVHGWHUP /'2UDO PJNJ  /&,QKDODWLRQ SSP 

 ([WUHPHO\7R[LF RUOHVV RUOHVV


 +LJKO\7R[LF  

 0RGHUDWHO\7R[LF  

 6OLJKWO\7R[LF  


 3UDFWLFDOO\1RQWR[LF  

 5HODWLYHO\+DUPOHVV RUPRUH RUPRUH




  

 D   PRO RI D WR[LF FRPSRXQG &+12 ZDV IRXQG WR FDXVH GHDWK LQ  RIWKH WHVW
DQLPDOVZHLJKLQJNJ

&DOFXODWHWKH/'RI&+12DQGVWDWHLWVWR[LFLW\UDWLQJ

>@

   

 

1-&
trendyline
         >7XUQRYHU

810 trendyline


 E  3KRVSKLQH3+LVDJDVZLGHO\XVHGLQWKHVHPLFRQGXFWRULQGXVWULHVDVDGRSDQWDQGD
SUHFXUVRUIRUWKHGHSRVLWLRQRIFRPSRXQGVHPLFRQGXFWRUV

)RU VDIHW\ FRQVLGHUDWLRQV WKH SHUPLVVLEOH OLPLWV RI SKRVSKLQH PXVW QRW H[FHHG RI WKH

/&YDOXH
:KHQH[SUHVVLQJWKHFRQFHQWUDWLRQRIDVPDOOTXDQWLW\RIJDVSDUWVSHUPLOOLRQ SSP FDQ
EH XVHG 3SP LV XVXDOO\ XVHG IRU YROXPH RI JDVHV DQG LV H[SUHVVHG DV VKRZQ LQ WKH
HTXDWLRQEHORZ
YROXPHRIJDV
&RQFHQWUDWLRQRIJDV LQSSP   
YROXPHRIDLU
  L  *LYHQ WKDW WKH /& IRU SKRVSKLQH LV  PJ P  DW URRP WHPSHUDWXUH FRQYHUW WKH
/&WRSSPDQGGHWHUPLQHLWVWR[LFLW\UDWLQJ
>@
  

  LL  $VHPLFRQGXFWRUIDFWRU\UHOHDVHVJRISKRVSKLQHLQDGD\
8VLQJ \RXU DQVZHU LQ E L  GHWHUPLQH WKH PLQLPXP YROXPH RI WKH IDFWRU\ WKDW ZLOO
HQVXUHWKHYROXPHRISKRVSKLQHLVZLWKLQWKHSHUPLVVLEOHOLPLWVDWURRPWHPSHUDWXUH
>@

  >7RWDO@

trendyline
 

1-&          >7XUQRYHU

811 trendyline


 D  /LTXHIDFWLRQRIDLULVGRQHLQGXVWULDOO\E\VXUURXQGLQJSUHFRROHGKLJKSUHVVXUHDLULQDFRLO
VXUURXQGHGE\FROGZDWHU7KHDLUFRROVGRZQE\DODUJHDPRXQW ZKHQLWH[SDQGVLQWRD
UHJLRQRIORZSUHVVXUH7KLVEHKDYLRXULVQRWH[KLELWHGE\LGHDOJDVHV

  L  6WDWHWZRDVVXPSWLRQVRIWKHNLQHWLFWKHRU\RIJDVHV

   



>@

  LL  6XJJHVWZK\DLUFRROVZKHQLWH[SDQGV

   

>@

  LLL  &RQYHUVHO\JDVHVFDQEHOLTXHILHGDWURRPWHPSHUDWXUHE\FRPSUHVVLQJWKHPXQGHU
KLJKSUHVVXUHV([SODLQZK\DJDVOLTXHILHVZKHQFRPSUHVVHG

   

>@

 E  &ULWLFDOWHPSHUDWXUHLVWKHPD[LPXPWHPSHUDWXUHDWZKLFKDJDVFDQEHFRQYHUWHGLQWRD
OLTXLGE\DQLQFUHDVHLQSUHVVXUH:KHQWKHWHPSHUDWXUHLVDERYHWKHFULWLFDOWHPSHUDWXUH
WKHYDSRXUFDQQRWEHOLTXHILHGQRPDWWHUKRZPXFKSUHVVXUHLVDSSOLHG

  L  'UDZWKHGRWDQGFURVVGLDJUDPIRU12ZKHUH1LVWKHFHQWUDODWRP

   >@

trendyline


1-&          >7XUQRYHU

812 trendyline


  LL  $UUDQJH WKH JDVHV 12 ) DQG +2 LQ DQ LQFUHDVLQJ RUGHU RI WKHLU FULWLFDO
WHPSHUDWXUHV([SODLQ\RXUDQVZHU

  










>@

>7RWDO@

 

1-&
trendyline
         >7XUQRYHU

813 trendyline


 +\GURJHQ LV D EHWWHU VRXUFH RI IXHO FRPSDUHG WR IRVVLO IXHOV DV LW LV FOHDQ DQG UHQHZDEOH 7KH
FRQYHQWLRQDO PHWKRG RI REWDLQLQJ SXUH K\GURJHQ LV WR HOHFWURO\VH ZDWHU+RZHYHU PDQ\
LQGXVWULHVVWLOOSUHIHUIRVVLOIXHORUFRDODVWKHPDLQVRXUFHRIHQHUJ\2QHUHDVRQLVWKHKLJKFRVW
RIHOHFWURO\VLV

 D  6WDWHDQRWKHUUHDVRQZK\WKHXVHRIK\GURJHQDVDIXHOLVQRWSRSXODU


  >@

 E  7KH GLDJUDP EHORZ VKRZV DQ DOWHUQDWLYH PHWKRG RI SURGXFLQJ K\GURJHQ IURP RUJDQLF
PDWHULDOZKHUHHWKDQRLFDFLGLVFRQYHUWHGE\EDFWHULDWRFDUERQGLR[LGH

YROWPHWHU

+
&+&22+
; +
<
+ +

&+&22+
+
&+&22+

,RQH[FKDQJHPHPEUDQH
  L  &RPSOHWHWKHWDEOHEHORZ

(OHFWURGH 7\SHRIUHDFWLRQ 3RODULW\

 
; 

 
< 

>@

  LL  &RQVWUXFWDQHTXDWLRQIRUWKHUHDFWLRQDWHDFKHOHFWURGH

   (OHFWURGH;

trendyline

FWURGH
WUR <

(OHFWURGH<
>@
  
>7RWDO@
1-&          >7XUQRYHU

814 trendyline


 &KURPLXPLVDWUDQVLWLRQHOHPHQWZKLFKH[KLELWVPRUHWKDQRQHR[LGDWLRQVWDWHLQLWVFRPSRXQGV
PDQ\RIZKLFKDUHFRORXUHG

7KH IORZ FKDUW EHORZ VKRZV WKH UHDFWLRQ RI D SXUSOH FKURPLXP ,,,  FRPSRXQG ; ZKLFK KDV D
FU\VWDOOLQHVWUXFWXUH

 3XUSOHFU\VWDORI; 


ZDWHU


'DUNJUHHQVROXWLRQ 
$



3RUWLRQ 3RUWLRQ


1D2+ DT  
=Q V  $FLGLILHG.0Q2 DT 
GURSZLVH



*UH\JUHHQSSW %OXHVROXWLRQ 2UDQJHVROXWLRQ
% '  (

([FHVV
1D2+ DT   1D2+ DT 

'DUNJUHHQVROXWLRQ
&  <HOORZVROXWLRQ
)


 D  L  :ULWHWKHHOHFWURQLFFRQILJXUDWLRQRIWKHFKURPLXPLRQLQ$

>@

  LL  6XJJHVWDQH[SODQDWLRQIRU$EHLQJFRORXUHG


  


trendyline



>@

1-&          >7XUQRYHU

815 trendyline


 E  L  ([SODLQ ZK\ % GLVVROYHV LQ H[FHVV 1D2+ DT  WR IRUP & <RX VKRXOG XVH WKH LGHDV
EHKLQGWKH/H&KDWHUOLHUVSULQFLSOHDQGVROXELOLW\SURGXFWKVSLQ\RXUDQVZHU


  


>@

  LL  ,GHQWLI\WKHFKURPLXPFRQWDLQLQJVSHFLHVLQ'

>@

  LY  6WDWHWKHW\SHRIUHDFWLRQDQGZULWHDQHTXDWLRQIRUWKHFRQYHUVLRQRI(WR)


  
>@

 F  &RQVLGHUWKHWZRFKURPLXPFRPSOH[HV>&U&l 1+ @62DQG>&U) 1+ @&l

  L  'HVFULEHDFKHPLFDOWHVWZKLFKGRHVQRW LQYROYHVLOYHUQLWUDWHWKDWFRXOGGLVWLQJXLVK
EHWZHHQDTXHRXVVROXWLRQVRI>&U&l 1+ @62DQG>&U) 1+ @&l


  


>@

  LL  ([SODLQZK\WKHLRQLFUDGLXVRI&ULVPXFKVPDOOHUWKDQWKDWRI&D


  
>@

   >7RWDO@
 

1-&
trendyline
         >7XUQRYHU

816 trendyline


 3LSHULGLQHV DUH F\FOLF DPLQHV ZKLOH S\ULGLQHV DUH KHWHURF\FOLFRUJDQLF FRPSRXQGV VWUXFWXUDOO\
UHODWHGWREHQ]HQH

1 1
+
SLSHULGLQH S\ULGLQH 

 D  7KHVWUXFWXUHRIWULPHWK\OSLSHULGLQHLVVKRZQEHORZ

1
+ 
,WFDQEHV\QWKHVLVHGE\UHDFWLQJDPPRQLDZLWKDGLEURPRFRPSRXQG*

1+&+%UWULPHWK\OSLSHULGLQH+%U

    *

  L  6XJJHVWWKHVWUXFWXUHRIFRPSRXQG*

>@

  LL :KHQ FRPSRXQG * LV UHDFWHG ZLWK 1D2+ LQ HWKDQRO D PL[WXUH RI LVRPHULF DONHQHV
  ZLWKPROHFXODUIRUPXOD&+LVIRUPHG

6XJJHVWWKHVWUXFWXUHVRIDOOWKHLVRPHUV

<RXPD\XVHVNHOHWDOUHSUHVHQWDWLRQVLQ\RXUDQVZHUV

>@

trendyline


1-&          >7XUQRYHU

817 trendyline



LLL  &RPSRXQG*FDQEHFRQYHUWHGWRFRPSRXQG+E\ILUVWFRQYHUWLQJLWWRDGLROIROORZHG
  E\WZRPRUHVWHSVDVVKRZQ

, ,,
*
+2 +2 2 2
FRPSRXQG+ 
&RPSOHWHWKHWDEOHEHORZ

6WHS 5HDJHQWVDQG&RQGLWLRQV 7\SHRIUHDFWLRQ


,  


,,  


>@


 E  7KH WDEOH EHORZ VKRZV WKH SKE YDOXHV RI WKUHH PRQRSURWLF EDVHV DPPRQLD
HWK\OSLSHULGLQHDQGGLPHWK\S\ULGLQH


1+
1+  1 
DPPRQLD HWK\OSLSHULGLQH GLPHWK\S\ULGLQH

SKE   




  L  *LYHDQHTXDWLRQZKLFKUHSUHVHQWVWKHUHDFWLRQRIHWK\OSLSHULGLQHZLWKZDWHU

>@

  LL  ([SODLQZK\HWK\OSLSHULGLQHKDVDORZHUSKEYDOXHWKDQDPPRQLD





>@

1-&          >7XUQRYHU

818 trendyline



  $  FP VDPSOH RI - D VROXWLRQ FRQWDLQLQJ HWK\OSLSHULGLQH DQG GLPHWK\S\ULGLQH
ZDV WLWUDWHG DJDLQVW  PROGP+&l DT  LQ WKH SUHVHQFH RI WZR LQGLFDWRUV EURPRWK\PRO
EOXHDQGPHWK\ORUDQJH

,W ZDV IRXQG WKDW  FP RI +&l DT  ZHUH QHHGHG WR FKDQJH WKH FRORXU RI WKH ILUVW
LQGLFDWRUDQGDfurther 7.80 cm3ZHUHQHHGHGWRFKDQJHWKHFRORXURIWKHVHFRQGLQGLFDWRU

  LLL  8VHWKHGDWDWRFDOFXODWHWKHFRQFHQWUDWLRQRIHDFKRIWKHWZREDVHVLQ-

>@

  LY  &DOFXODWHWKHLQLWLDOS+RI-

>@

 

1-&          >7XUQRYHU

819 trendyline



  Y  +HQFHVNHWFKWKHVKDSHRIWKHS+FXUYHGXULQJWKLVWLWUDWLRQVKRZLQJWKHS+YDOXHV
DQGYROXPHVRI+&l DT DWVLJQLILFDQWSRLQWV

  >@

  >7RWDO@
 

1-&          >7XUQRYHU

820 trendyline



 7KH UHDFWLRQ VFKHPH EHORZ VKRZV WKH V\QWKHVLV RI FRPSRXQG ( VWDUWLQJ IURP DQ DONHQH DQG D
SKHQROGHULYDWLYH

>7KH&+2JURXSFDQEHFRQVLGHUHGWREHLQHUW@

2&+

&+&+ &+
+2 &+&+

2&+
,,
,

&RPSRXQG$ &RPSRXQG%

1+ &+ 

2&+
2+ &+
1
+2 &2&l &+
&RPSRXQG'
2&+
&RPSRXQG&

2 &+
2 + +
&+
+2 & 2 & & 1
&+
&+ +
2&+
&RPSRXQG(


 D  L  6WDWHUHDJHQWVDQGFRQGLWLRQVIRUVWHSV,DQG,,


6WHS,

HS,,
6WHS 

>@

1-&          >7XUQRYHU

821 trendyline



  LL  6XJJHVWWKHLGHQWLWLHVRIFRPSRXQGV$DQG%

&RPSRXQG$ &RPSRXQG%

 

>@

  LLL  'HVFULEH D PHFKDQLVP IRU WKH UHDFWLRQ WDNLQJ SODFH LQ VWHS ,, ,Q \RXU DQVZHUV
LQFOXGHDOOQHFHVVDU\FXUO\DUURZVORQHSDLUVDQGUHOHYDQWGLSROHV

   >@

 

 

 

 

 

 

 

 

  LY  6WDWH WKH W\SH RI VWHUHRLVRPHULVP H[KLELWHG E\ ' 'UDZ DSSURSULDWH VWUXFWXUHV WR
LOOXVWUDWH\RXUDQVZHU

>@

trendyline


1-&          >7XUQRYHU

822 trendyline



  Y  'UDZWKHRUJDQLFSURGXFWIRUPHGZKHQFRPSRXQG$UHDFWVZLWKFRPSRXQG'

>@

 E  &RPSRXQG)LVDQLVRPHURI(
2&+
2 + + &+
+2 & 2 & & 1

&+ + &+
2&+ FRPSRXQG) 
'HVFULEHDVLPSOHFKHPLFDOWHVWWRGLVWLQJXLVKEHWZHHQFRPSRXQGV(DQG)

  



>@

>7RWDO@





1-&
trendyline
         >7XUQRYHU

823 trendyline
Suggested Answers for 2016 SH2 Chemistry Paper 2
(P) (a) Preliminary Calculation
3
For titration, 25cm of sea water is used,
3 3
Amt of Cl in 25.0 cm = 25/1000 x 2.0 x 10
5
= 5.0 x 10 mol
+
= Amt of Ag
3
Assuming titre volume to be 25.00 cm ,
5
Required [AgNO 3 ] = (5.0 x 10 )/25 x 1000
3 3
= 2.0 x 10 mol dm
3
Hence, the 0.1 mol dm silver nitrate solution provided is unsuitable for use in the titration (i.e. too
concentrated)

3 3 3
Need to dilute the given 0.1 mol dm silver nitrate solution to 2.0 x 10 mol dm
Need to dilute it 50 times.
3 3
Preparation of a standard silver nitrate solution of 2.0 x 10 mol dm
3
1. Using a burette, transfer 5.00 cm of the given silver nitrate solution into a 250
3
cm volumetric flask. Top up to the mark using deionised water.
2. Cap the flask, invert it and shake to obtain a homogeneous solution.

Precipitation Titration
3 
1. Fill a burette with the newly-prepared 2.0 x 10 mol dm silver nitrate solution.
3 3
2. Pipette 25.0 cm of sea water into a 250 cm conical flask.
3
3. Add 1 cm of potassium chromate(VI) solution into the conical flask.
4. Titrate until brick-red precipitate first appears.
3
5. Repeat titration till results within 0.10cm are obtained.

Sample calculation of concentration of chloride ions in sea water


3 3
Let concentration of standard silver nitrate solution be Mmol dm , titre volume be y cm , volume
3
of sea water used for titration be 25.0 cm
+
Amt of Ag = y/1000 x M

= yM/1000 mol

= Amt of Cl

[Cl ] = yM/1000 25/1000
3
= yM/25 mol dm

trendyline
3
(b) The student obtained a titre volume of 20.60 cm from her experiment.
p

Determine point.
ne the concentration of chloride ions remaining in the solution at the end po

You mayy assume the following:


3
25.0 cm of sea water was used in the titration
titrat

824 trendyline
3 
1 cm of 1.00 mol dm potassium chromate(VI) solution, K 2 CrO 4 was used as the indicator
12
K sp value of Ag 2 CrO 4 is 1.1 10
+ 10
Ag (aq) + Cl (aq) AgCl(s) K sp value = 1.8 x 10
+ 2 12
2Ag (aq) + CrO 4 (aq) Ag 2 CrO 4 (s) K sp value =1.1 x 10
2
Amt of CrO 4 = 1/1000 x 1.00
3
= 1.0 x 10 mol
2 3
At endpoint, [CrO 4 ] = (1.0 x 10 )/ [(25.0+1+20.60)/1000]
3
= 0.02146 mol dm

IP of Ag 2 CrO 4 = K sp
+ 2 2 12
[Ag ] [CrO 4 ] = 1.1 x 10
+ 2 12
[Ag ] [0.02146] = 1.1 x 10
+ 6 3
[Ag ] = 7.16 x 10 mol dm

IP of AgCl = K sp
+ 10
[Ag ] [Cl ] = 1.8 x 10
6 10
[7.16 x 10 ] [Cl ] = 1.8 x 10
5 3
[Cl ] = 2.51 x 10 mol dm

(c) Subsequently, the student investigates the solubility of silver chloride in aqueous hydrochloric acid.

By considering the ions present in a solution of silver chloride in aqueous hydrochloric acid, predict and
explain how the solubility of silver chloride will be affected by the concentration of the acid.

Prediction: Solubility of AgCl decreases when concentration of HCl increases.


+
Explanation: Ag (aq) + Cl (aq) AgCl(s)

In the presence of HCl, there is common ion effect. [Cl ] increases, position of equilibrium shifts
right, solubility decreases.

(d) Identify the following variables in the investigation of solubility of silver chloride in hydrochloric acid.


Independent variable: concentration of HCl / amount of HCl / concentration of Cl

Dependent variable: mass of AgCl that dissolved / solubility of AgCl / amount of AgCl that
dissolved
3
(e) Calculate the solubility of silver chloride in a 2 mol dm hydrochloric acid solution.

+ 10
[Ag ] [Cl ] = 1.8 x 10

trendyline
e
10
[x] [x+2.0]
0]] = 1.8 x 10

Assume
e x <<< 2.0,
10
10
[x] [2.0] = 1.8 x 10

825 trendyline
11 3
x = 9.00 x 10 mol dm

[1]

[Total:12]

1 One means of measuring toxicity is using LD, which stands for "Lethal Dose". LD 50 is the amount of a material which
causes the death of 50% of a group of test animals. LD 50 value is expressed as the weight of a chemical
administered per kg body mass of a test animal.

Another means of measuring toxicity is using LC, which stands for "Lethal Concentration". The concentrations of the
chemical in air that kills 50% of the test animals during the observation period is the LC 50 value.

The table below shows the values for the LD 50 and LC 50 along with the toxicity ratings.

Toxicity Commonly used 1


LD 50 : Oral (mg kg ) LC 50 : Inhalation (ppm)
Rating term

1 Extremely Toxic 1 or less 10 or less

2 Highly Toxic 1 50 10 100

3 Moderately Toxic 50 500 100 1000

4 Slightly Toxic 500 5000 1000 10,000

Practically Non-
5 5000 15,000 10,000 100,000
toxic

6 Relatively Harmless 15,000 or more 100,000 >

4
(a) 9.90 10 mol of a toxic compound, C 4 H 5 NO, was found to cause death in 50% of test animals weighing 1
kg.

Calculate the LD 50 of the compound and state its toxicity rating. [2]
4
Mass of compound = 9.90 x 10 x (48 + 5 + 14 + 16)

= 0.08217 g
0.19206 10 3
Hence, LD 50 = = 82.17 mg/kg
1

82.2 mg/kg [1]

The compound is moderately toxic/3 [1]

trendyline
826 trendyline
(b) Phosphine is a gas that is widely used in the semi-conductor industry as a dopant and as a precursor for the
deposition of compound semiconductors.

1
For safety of operations, the permissible limits of phosphine must not exceed of its LC 50 value.
10
When expressing a small quantity of gas, parts per million (ppm) can be used to indicate it. Ppm is usually
used for volume of gases and is expressed in the equation below:
volume of gas
Concentration in ppm = 106
volume of air

3
(i) Given that the LC 50 for phosphine is 55 mg m at room temperature, convert the LC 50 to ppm and
determine its toxicity rating. [3]

55 10 3 3
Volume of PH 3 = x 24 dm
34.0
3 3
= 1.6176 x 10 x 24 dm
3
= 0.03882 dm [1]

0.03882 103 6
LC 50 = 1
x 10

= 38.82

38.8 ppm [1]

The toxicity rating is highly toxic/2 [1]

(ii) A semiconductor factory releases 36 g of phosphine in a day.

Using your value in (i), determine the minimum volume of the factory that will allow the volume of
phosphine to be within permissible limits at room temperature.
[2]

Allowed amount of PH 3 in ppm = 38.82 10

= 3.882 ppm [1]

Given amount of of PH 3 produced a day is 36g


6
3.882 =
3
x 10

36
24 6
3.882 =
34

x 10
6 3
Minimum volume of factory = 6.546 x 10 dm [1]

[Total:7]

2 (a)
water. The
trendyline
Liquefaction
n of air is done industrially by surrounding pre-cooled
on

This behaviour
viour is not exhibited by ideal gases.
pre cooled high-pressure
high ressure air in a coil surrounded by cold
pressure.
e air cools down by a large amount when it expands into a region of low pre

827 trendyline
(i) State two assumptions of the kinetic theory of gases.

A gas is composed of tiny particles that have negligible volumes compared to the volume
in which they move or the volume of the gas container.

There are no attractive or repulsive forces between the gas particles.[2]

(ii) Suggest why air cools when it expands.

For expansion of air, energy is needed to overcome the intermolecular forces of


attraction between the air molecules, which is absorbed from the surroundings.
[1]

(iii) Conversely, gases can be liquefied at room temperature by compressing them under high pressure.
Explain why a gas liquefy when compressed.

When compressed, the gas particles are close enough for the intermolecular forces of
attraction to be significant and liquification occurs. [1]

(b) Critical temperature is the maximum temperature at which a gas can be converted into a liquid by an increase
in pressure. When the temperature is above the critical temperature, the vapour cannot be liquefied no matter
how much pressure is applied.

(i) Draw the dot - and - cross diagram for the N 2 O, where N is the central atom.

N NO N NO[1]

(iii) Arrange the gases N 2 O, F 2 and H 2 O, in increasing order of their critical temperatures and explain
your answer.

__F 2 ___ < __N 2 O_ < __H 2 O__

The hydrogen bonds between water molecules is the strongest, followed by the weaker
permanent dipole-dipole interactions between N 2 O molecules and the temporary dipole-
induced dipole between the F 2 molecules being the weakest.

Hence, H 2 O can remain as a liquid even at high temperature due to strongest


intermolecular forces and hence it has the highest critical temperature or the stronger the
intermolecular forces, a higher temperature is needed to convert the liquid to gas and hence
the higher the temperature.

trendyline
[3]

[Total:8]

828 trendyline
3 Hydrogen is a better source of fuel compared to fossil fuels as it is clean and renewable. The conventional method
of obtaining pure hydrogen is to electrolyse water. However, many industries still prefer fossil fuel or coal as the
main source of energy. One reason is the high cost of electrolysis.

(a) State another reason why the use of hydrogen as a fuel is not popular.

Hydrogen exists as a gas and it is difficult/high cost to store or transport.


Or

Hydrogen is dangerous as it is explosive.[1]

(b) The diagram below shows an alternative method of producing hydrogen from organic material where
ethanoic acid is converted by bacteria to carbon dioxide.

voltmeter

+
H
CH3COOH +
X Y
H
+
H
+ H

CH3COO
+
H
CH3COOH

Ion exchange membrane


(i) Complete the table below.

Electrode Type of reaction Polarity

X Oxidation Negative

Y Reduction Positive

[1]

(ii)

trendyline
Construct an equation for
f the reaction at each electrod
electrode.

829 trendyline
+
Electrode X: CH 3 COOH + 2H 2 O 2CO 2 + 8H + 8e
+
Electrode Y: 2H + 2e H 2 [2]

[Total: 4]

4 Chromium is a transition element which exhibits more than one oxidation state in its compounds, many of which are
coloured.

The flow chart below shows the reaction of a purple chromium(III) compound, X, which has a crystalline structure.

Purple crystal of X

water

Dark green solution

Portion 1 Portion 2

NaOH(aq) Zn(s) Acidified KMnO4(aq)


dropwise

Grey-green ppt Blue Orange solution


solution

Excess
NaOH(aq) NaOH(aq)

Dark green
Yellow solution

(a) (i) Write the electronic configuration of the chromium ion in A.


2 2 6 2 6 3
1s 2s 2p 3s 3p 3d [1]

(ii) Suggest an explanation for A being coloured.

trendyline
3+ 3+
When ligand approach Cr , the 3d orbitals of C
Cr splits
p into 2 different
diff energy levels (d-d
splitting).
When an electron in the lower energy d orbital is excited to the hhigher energy d orbital, a
photon with wavelength in the visible region is absorbed (d-d
(d-d
d transition).
tra
trans

830 trendyline
The colour of A observed is the complementary colour.
[3]

(b) (i) Explain why B dissolves in excess NaOH(aq) to form C. You should use the ideas behind the Le
Chaterliers principle and solubility product, K sp , in your answer.
3
B is Cr(OH) 3 and C is [Cr(OH) 6 ] .

3
Cr(OH) 3 + 3OH [Cr(OH) 6 ]

3
When excess OH is added, equilibrium shifts to the right to form [Cr(OH) 6 ] . This
3+
decreases the concentration of Cr and hence causing the ionic product of Cr(OH) 3 to
fall below its K sp . Therefore, Cr(OH) 3 dissolves.

Or

3+
Cr(OH) 3 Cr (aq) + 3OH (aq) Equilibrium 1
3+ 3
Cr + 6OH [Cr(OH) 6 ] -- Equilibrium 2

3
When excess OH is added, position of equilibrium 2 shifts right to form [Cr(OH) 6 ] . The
3+
concentration of Cr decreases and the ionic product of Cr(OH) 3 falls below its K sp .

(ii) Identify the chromium-containing species in D.


2+ 2+
[Cr(H 2 O) 6 ] or Cr (aq)
[1]

(iv) State the type of reaction and write an equation for the conversion of E to F.

Acid base reaction or Neutralisation reaction

2 2
Cr 2 O 7 + 2OH 2CrO 4 + H2O [2]

(c)

trendyline
Consider
der the two chromium complexes, [CrCl(NH
[CrCl
[CrCl(NH
(N 3 ) 5 ]SO 4 and [CrF(N
[CrF(NH 3 ) 5 ]C
]Cll 2 .

831 trendyline
(i) Describe a chemical test, which does not involve silver nitrate, that could distinguish between
aqueous solutions of [CrCl(NH 3 ) 5 ]SO 4 and [CrF(NH 3 ) 5 ]Cl 2 .

Add Ba(NO 3 ) 2 (aq) or BaCl 2 (aq) or Ba(OH) 2 (aq) or Ca(NO 3 ) 2 (aq). [CrCl(NH 3 ) 5 ]SO 4 will
form a white precipitate and [CrF(NH 3 ) 5 ]Cl 2 will not form any white precipitate or the
solution will remain colourless. [2]

2+ 2+
(ii) Explain why the ionic radius of Cr is much smaller than that of Ca .
2+ 2+
Cr has higher nuclear charge (more protons) than Ca and the electrons in the d
orbitals are poorly shielding. Therefore, the nuclear attraction on valence electron is
2+ 2+
higher in Cr and electrons are attracted closer to the nucleus in Cr . [1]

[Total: 12]

5 Piperidines are cyclic amines while pyridines are heterocyclic organic compounds structurally related to benzene.

N N
H
piperidine pyridine

(a) The structure of 2,3,5trimethylpiperidine is shown below.

N
H
It can be synthesised by reacting ammonia with a dibromo compound G.

NH 3 + C 8 H 16 Br 2 2,3,5trimethylpiperidine + 2HBr
G

(i) Suggest the structure of compound G.

trendyline
B
ren
Br
n Br

832 trendyline
(ii) When compound G is reacted with NaOH in ethanol, a mixture of isomeric alkenes with molecular
formula C 8 H 14 is formed.

Suggest the structures of all the isomers.

You may use skeletal representations in your answers.

2,4-dimethylhexa-1,5-diene geometirc isomers of 2,4-dimethylhexa-1,4-diene

(iii) Compound G can be converted to compound H by first converting it to a diol, followed by two more
steps as shown.

I II
G
HO HO O O
compound H
Complete the table below.

Step Reagents and Conditions Type of reaction

aqueous alkaline iodine / I 2 , NaOH(aq) / I 2 (aq), Oxidation


I
NaOH(aq); warm (not heat)

concentrated sulfuric acid (trace amount); heat Condensation


II
under reflxu

(b) The table below shows the pK b values of three monoprotic bases, ammonia, 2-ethylpiperidine and 2,3-
dimethypyridine.

NH
NH 3 N
ammonia 2-ethylpiperidine 2,3-dimethypyridine

pK b 4.74 3.55 7.43

(i) Give an equation which represents the reaction of 2-ethylpiperidine with water.

trend
tr d NH
+ H2O
NH2+
+ OH

833 trendyline
(ii) Explain why 2-ethylpiperidine has a lower pK b value than ammonia.

o
2-ethylpiperidine: 2 aliphatic amine with 2 electron-donating alkylgroups, lone ep of
+
N is readily available to be donated to H

Ammonia does not have any electron-donating alkyl group.

3
A 10.0 cm sample of J, a solution containing 2-ethylpiperidine and 2,3-dimethypyridine, was titrated
against 0.50 mol dm HCl(aq) in the presence of two indicators, bromothymol blue and methyl orange.
3

3
It was found that 12.20 cm of HCl(aq) were needed to change the colour of the first indicator and a
3
further 7.80 cm were needed to change the colour of the second indicator.

(iii) Use the data to calculate the concentration of each of the two bases in J.

In titrating a mixture of bases, the stronger or strongest base will be neutralised first.

2-ethylpiperidine in J = HCl to reach 1st eq pt

12.20
= x 0.50 = 6.10 103 mol
1000
12.20
x 0.50
[2-ethylpiperidine] in J = 1000 = 0.610mol dm3
10.0
1000
nd
2,3-dimethypyridine in J = HCl to reach 2 eq pt

7.80
= x 0.50 = 3.90 103 mol
1000

7.80
x 0.50
[2,3-dimethypyridine] in J = 1000 = 0.390mol dm3
10.0
1000

(iv) Calculate the initial pH of J.

3.88
K b (2-ethylpiperidine) is = 10 ( 7600) x K b (2,3-dimethylpyridine), 2-ethylpiperidine is
a much stronger base)

initial pH of solution J = pH of 2-ethylpiperidine in solution

[OH] = ([base] x K b ) = (0.610 x 103.55)

= 1.31 102 mol dm3

trendyline
pOH = 1.88, pH = 12.1

(v) Hence, sketch the shape of the pH curve during this titration, showing the
t pH values and volumes
of HCl(aq)
HC at significant points
points.

834 trendyline
- initial pH = 12.1
st 3
- pH at 1 max buffer capacity = 10.45 (= 14 3.55), 6.10 cm HCl(aq)
nd 3
- pH at 2 max buffer capacity = 6.57 (= 14 7.43), 16.10cm HCl(aq)
st 3
- 1 eq pt 12.20 cm HCl(aq)
nd 3
- 2 eq pt 20.00 cm HCl(aq)
- correct shape (2-steps)

pH

12.1

10.45
>7

6.57

<7

6.10 12.20 16.10 20.00


3
volume of HCl(aq) / cm

6
trendyline
The reaction scheme
cheme below shows the synthesis of compound

[The CH 3 O group
oup can be considered to b
compoun E starting

be inert.]
nert ]
arting from an alkene and a phenol derivative.

835 trendyline
OCH3

CH3CH=CH2
HO CH2CH3

OCH3
II
I

Compound A Compound B

NH(CH3)2

O CH3
OH CH3
N
HO COCl CH3
Compound D
OCH3
Compound C

O CH3
O H H
CH3
HO C O C C N
CH3
CH3 H
OCH3
Compound E

(a) (i) State reagents and conditions for steps Iand II.

Step I: KMnO 4 , dil H 2 SO 4 , heat


Step II: Cl 2 (aq) or Br 2 (aq)

(ii) Suggest the identities of compounds A and B.

A Compound B

O CH3 CH3 H
H C C H
HO CO2H
OH Cl
OCH3

(iii) Describe a mechanism for the reaction taking place in step II. In your answers, include all
necessary curly arrows, lone pairs and relevant dipoles.

trendyline
Electrophilic Addition
mechanism:
Detailed mechanism

H H
C C CH3 H
H
CH3 H C C H Cl
Cl Cl
Cl

CH3 H 836 CH3 H trendyline

H C C H H C C H
H
slow
+

Penalty:

any missing arrow/ wrong direction of arrow/arrow pointing to wrong atoms (for 1st
and 2nd step)
missing partial charge
2+LQVWHDGRI+2DWWDFNLQQGVWHS
missing slow step in 1st step
missing lone pair on nucleophile

(iv) State the type of stereoisomerism exhibited by D. Draw appropriate structures to illustrate your
answer.

Optical Isomerism

OH OH

N C H H C N
CH3 CH3

(v) Draw the organic product formed when compound A reacts with compound D.

trendyline
837 trendyline
OCH3

HO CO2 CH3CH(OH)CH2NH(CH3)2

OCH3

OCH3
O
HO
O
OCH3 N
- .
(b) Compound F is an isomer of E.

OCH3
O H H CH3
HO C O C C N

CH3 H CH3
OCH3 compound F

Describe a simple chemical test to distinguish between compounds E and F.

Add aqueous bromine:

E: yellow brown / orange bromine decolourises

F: yellow brown / orange colour remains

Or

Add liquid bromine:

E: reddish brown bromine decolourises

F: reddish brown colour remains

trendyline
838 trendyline
1

NATIONALJUNIOR COLLEGE

H
SH2 PRELIMINARY EXAMINATION
Higher 2

CANDIDATE
NAME

SUBJECT REGISTRATION
CLASS NUMBER

CHEMISTRY 9647/03
Paper 3 Free response
Monday 29 Aug 2016
2 hour

READ THESE INSTRUCTIONS FIRST

Answer any four questions.

A Data Booklet is provided.

The use of an approved scientific calculated is expected, where appropriate.

You are reminded of the need for good English and clear presentation in your answers.

At the end of the examination, fasten all your work securely behind the cover page.

The number of marks is given in brackets [ ] at the end of each question or part
question.

trendyline
NJC/H2Chem/03/2016 [Turn over

839 trendyline
2

This paper consists of 11 printed pages.

Answer any four questions.

1 (a) Alums are salts formed when a monovalent cation with a large radius (e.g. K+
or NH 4 +) and a trivalent cation with a small radius (e.g. Al3+, Fe3+ or Cr3+)
combine with sulfate ions.

An ammonium iron alum has the formula (NH 4 ) a Fe(SO 4 ) b .12H 2 O.

To determine the chemical formula of the alum, two separate samples of


0.500 g of the alum were each dissolved in 100 cm3 of water.

An excess of NaOH(aq) was added to the first alum solution and the mixture
was boiled. The ammonia gas evolved exactly neutralised 5.2 cm3 of 0.200 mol
dm3 HCl(aq).

An excess of zinc was added to the second alum solution, which reduced Fe3+
to Fe2+, and the resulting solution was filtered. The filtrate collected required
10.4 cm3 of 0.0200 mol dm3acidified KMnO 4 (aq) for complete reaction.

(i) Calculate the amount of NH 4 + ions present in 0.500g of the alum. [1]

(ii) Write an equation for the reaction between the filtrate and acidified
KMnO 4.
Hence, calculate the amount of Fe3+ ions present in 0.500 g of the alum.
[2]

(iii) Using your answers in (i) and (ii), determine the values of a and b. [2]

(b) The Pauson-Khan reaction is a gas phase reaction between an alkyne, an


alkene and carbon monoxide to form a cyclic carbonyl compound, as shown
below.
O
Y
O Y
+ + catalyst
C
Y'
Y'
where Y and Y can be Cl or H.

nd
Cl and
trendyline
rganic
ganic compound A was produced from the Pauson-Khan
An organic
pound
ound has the following composition by mass; 51.5 % C, 13.7%
compound
d 4.3%
4 3% H.
H M r of A is 116.5.
116 5
Pauson-
Pauson-Khan reaction. The
13 O, 30.5%

NJC/H2Chem/03/2016 [Turn over

840 trendyline
3

Upon heating with ethanolic AgNO 3 , Agave a white precipitate.

(i) Determine the molecular formula ofA and draw its structural formula.
[3]

(ii) By quoting relevant data from the Data Booklet and given that the bond
HQHUJ\ RI &2 LV  N- PRO1, prove that the enthalpy change of
reaction in the Pauson-Khan reaction to form A LVN-PRO1.[2]

(iii) Predict the sign of the entropy change for the formation of Ain the above
reaction. Hence, deduce if the reaction is feasible at a high or low
temperature.
[2]

(iv) With the aid of a Boltzmann diagram, predict and explain the effect on
the rate of the Pauson-Khan reaction without a catalyst.
[3]

(c) Another possible product of the Pauson-Khan reaction is shown below:


O

CO2H

(i) Draw the product formed when B reacts with each of the following.

I. cold KMnO 4 (aq) in KOH(aq)


II. H 2 , Pt
III. LiAlH 4

[3]

(ii) Compound C is an isomer of B, and has the structure shown below:


O

CO2H

trendyline
C yl
NJC/H2Chem/03/2016 [Turn over

841 trendyline
4

Deduce which compound, B or C, is more acidic.


[2]

[Total: 20]

trendyline
NJC/H2Chem/03/2016 [Turn over

842 trendyline
5

2 Bovine serum albumin(BSA)is a serum albumin protein which is a main constituent of


cow milk. It is often used as a protein concentration standard in laboratory
experiments. The BSA molecule is a single chain of 607 amino acids, 66% of which
are incorporated into -pleated sheets. It is a globular protein and takes up a roughly
spherical shape in water.

Six of the most common amino acids in the BSA molecule are listed below.

Amino acid Formula of side chain


(R in RCH(NH 2 )CO 2 H)
Isoleucine -CH(CH 3 )CH 2 CH 3
Methionine -CH 2 CH 2 SCH 3
Tyrosine -CH 2 C 6 H 4 OH
Asparagine -CH 2 CONH 2
Lysine -CH 2 (CH 2 ) 3 NH 2
Asparate -CH 2 CO 2 H
Table 1

(a) (i) There are three pK a values associated with lysine: 2.17, 9.04, 12.48.

Make use of these pK a values to suggest the major species present in


aqueous solutions of lysine with the following pH values:

pH 1
pH 3
pH 10
pH 14
[4]

(ii) Suggest two positive tests (stating reagents, conditions and


observations) that could be used to confirm the identity of two amino
acids: asparagine and tyrosine.
[2]

(b) (i) Describe the conditions needed to hydrolyse a protein non-


enzymatically.
[1]
(ii) Briefly describe what is meant by the primary structure of proteins.
[1]

(iii) Describe how a polypeptide chain is held in the shape of a -pleated

trendyline
sheet.
[1]

(iv) With the aid of a diagram and using suitable amino acids from Table 1,
describe how hydrogen bondingding maintains the tertiary structure of

NJC/H2Chem/03/2016 [Turn over

843 trendyline
6

proteins.
[2]

(v) The following recipe illustrates how a lemon sponge cake can be made.

Steps:

1. Cream butter and sugar until light and fluffy.


2. Beat in eggs.
3. Sift over the flour, add lemon juice and rind.
4. Bake for 10 minutes at 180C.

Suggest how any of the two steps described above can cause
denaturation, making specific reference to suitable pairs of amino acids
present in BSA.
[4]

(c) When a beam of 31T + particles is passed through an electric field, the angle of
P

deflection is found to be +5.00o. You may assume that all the 31T + particles
P

travel at the same speed through an electric field of constant strength.


(i) Calculate the angles of deflection if 31T + is replaced with the following,
P

leaving your answers to 2 decimal places:

32
S2
56
Fe3+
[2]

The mass of particle D is 25 times that of 31T +.When a beam of D is passed


P

through the same electric field asthat used for 31T +, the angle of deflection is
P

found to be +0.60o.

(ii) Determine the overall charge on a particle of D.


[1]
(iii) A particle of D contains 27 protons. Deduce the number of electrons in
this particle and hence write its electronic configuration.
[1]

(iv) A nuclear reaction is a reaction in which there is a change inthe atomic


nucleus.
An experimental nuclear reactor uses 13C and tritium, 31T , as fuel. A

trendyline
nuclear reaction between these two atoms are described below (n is a
neutron).

13
C + 3T X + n + 4He
H

NJC/H2Chem/03/2016 [Turn over

844 trendyline
7

Suggest the identity of X with the aid of the Data Booklet.

[1]
[Total: 20]

3 (a) The kinetics of the hydrolysis of methyl ethanoate may be investigated by


determining the concentration of ethanoic acid produced.

CH 3 CO 2 CH 3 + H 2 O CH 3 CO 2 H + CH 3 OH

In a 1 dm3vessel, 0.350 mol of CH 3 CO 2 CH 3 was heated with dilute


hydrochloric acid which acted as a catalyst. The following results were
obtained.

Time / s [CH 3 CO 2 H] / mol dm3


0 0
340 0.105
680 0.185
1080 0.243
1440 0.278

(i) What is meant by the term order of reaction?


[1]

(ii) By drawing a suitable graph using the data given above, determine the
order of reaction with respect to methyl ethanoate.
[3]

(iii) Given that the hydrolysis reaction is first order with respect to
hydrochloric acid, write the rate equation for this reaction.
[1]

(b) Esters can also undergo reduction in the presence of a strong reducing agent
such as lithium aluminium hydride, LiAlH 4 . The reaction produces two alcohols.

RCO 2 R RCH 2 OH + ROH

Using this information, and that provided below, draw out the full mechanism

trendyline
for the
he
e formation of ethanol from methyl ethanoate.
ethanoate

ere is an initial nucleophilic attack by hydride ion, H, on the ester to


There
here
enerate
nerate a negatively charged ion.
generate io
ion.
The
he C=O bond reforms and the OR OR group is eliminated from
O f the ester,

NJC/H2Chem/03/2016 [Turn over

845 trendyline
8

forming an aldehyde.
A second hydride ion attacks the aldehyde to generate an alkoxide ion.
The alkoxide ion is then protonated to form the alcohol.

[3]

(c) Besides esters, nitriles can also undergo reduction to give aldehydes in the
presence of another strong reducing agent, diisobutyl aluminium hydride
(DIBAL).
RCN RCHO

(i) By considering the change in oxidation states of the reacting carbon,


suggest why the conversion of a nitrile to an aldehyde is a reduction
reaction.
[1]

CH2Br 4 steps
CH2CH(OH)CO2H

Compound E

(ii) Compound E can be synthesised using DIBAL as a reducing agent in


one of the steps above.

Suggest the reagents and conditions required for each step of synthesis
and identify all the intermediate compounds.
[5]

(d) A Wittig reagent can convert an aldehyde to an enone, a ketone with an


adjacent double bond.
O O
Ph3P R' CHO R'
R R
Wittig reagent enone
(wKHUH3KUHSUHVHQWV& 6 H 5 )

Wittig reagent F was reacted with an aldehyde G to give an enone H. H was


then reacted with hot acidified potassium manganate(VII) to give two different

trendyline
compounds,
ounds, J and K.
pounds,
H cann also be synthesised
synthesised by the following route.
synthesis

NJC/H2Chem/03/2016 [Turn over

846 trendyline
9

OH
ethanolic NaOH hot acidified K2Cr2O7
heat
L H
enone
Cl

(i) Suggest the identities of compounds H, J, Kand L. [4]

(ii) Suggest the identities of compounds F and G. [2]


[Total: 20]

4 (a) The question below is related to some compounds of Group II elements.


(i) Write an equation, with state symbols, for the thermal decomposition of
strontium nitrate.
[1]

(ii) Use the Data Booklet to explain how the thermal stability of zinc nitrate
might compare to strontium nitrate. [3]

(iii) The chemical properties of beryllium and its compounds show similarity
to those of aluminium. Beryllium form complexes of coordination number
4.

I. Aluminium chloride, AlCl 3 , dimerises to form Al 2 Cl 6 .


Draw the displayed formula of Al 2 Cl 6 , showing clearly the bond
angle about Al.
[1]

II. Explain why AlCl 3 dimerises. [1]

III. Explain, using a relevant equation, why a solution of beryllium


chloride is acidic. [2]

IV. Beryllium oxide, similar to aluminium oxide, is amphoteric.


Suggest the nature of bonding in beryllium oxide.
[1]

V. Write equations for the reaction of beryllium oxide with HCl(aq)


and NaOH(aq). [2]

(b) Capsaicin is a slightly acidic compound responsible for the burning sensation of
chilli peppers. Its molecular structure can be deduced by the following reaction
scheme.

trendyline
[Assumem that OCH 3 group is inert.]
ume inertt.]

NJC/H2Chem/03/2016 [Turn over

847 trendyline
10

boil with H+ NH2


C18H27NO 3 + C10H18O 2
Capsaicin N
HO

OCH 3
M

(i) When N is heated with concentrated acidified potassium


manganate(VII), P, C 6 H 10 O 4 , and Q, C 4 H 8 O, are formed.

P is formed from the reaction of Br(CH 2 ) 4 Br with hot ethanolic KCN


followed by hydrolysis. Q can be reduced to an alcohol, R, which is
optically active.

Deduce the structures of N, P, Q, R and capsaicin.


[5]

(ii) Draw the organic compound formed when M reacts with CH 3 CH 2 COCl.
[1]

(c) Peroxodisulfate(VI) ion, S 2 O 8 2-, is capable of oxidising tartaric acid,


(CH(OH)CO 2 H) 2 , to carbon dioxide and methanoic acid.
The Eo cell of the reaction is +0.99 V.

(CH(OH)CO 2 H) 2 + 3S 2 O 8  + 2H 2 O 2CO 2 + 2HCO 2 H + 6H+ + 6SO 4 

The reaction is very slow, even when temperature is increased.


However, the reaction rate can be increased by adding a suitable catalyst.

The half equation for the reduction of carbon dioxide to tartaric acid is

2CO 2 + 2HCO 2 H + 6H+ + 6e(CH(OH)CO 2 H) 2 + 2H 2 O

Using the Data Booklet, show how cobalt(III) ion can act as a homogenous
catalyst.
[3]

[Total: 20]

trendyline
NJC/H2Chem/03/2016 [Turn over

848 trendyline
11

5 (a) Chlorine is manufactured from brine (concentrated NaCl) by electrolysis using


inert electrodes at room temperature. The anode and cathode compartments
are separated by a diaphragm which is a permeable membrane.

BrineCl2(g)Gas S

Compound T

Diaphragm

trendyline
n
(i) Suggest
gg the identities of g
gas S and compound
p T. [1]

(ii) Using relevant Eodata ta from th


the Data Booklet, suggest
ggest a reason
re why the
electrolysis of dilute NaC (aq) produces mainly oxygen at
NaCl(aq) a the anode,
whereas the electrolysis of brinee produces
pr mainly chlorine gas.
g

NJC/H2Chem/03/2016 [Turn over

849 trendyline
12

[2]

(iii) Suggest the products that might be formed if the diaphragm is removed
and the solution is stirred. Write an equation for any reaction that occurs.
[1]

(b) Orchids can be made into jewellery that gleams with either a reddish tint of
copper or shiny gold. In copper-plating, orchids are coated with a thin layer of
graphite paste before placing them in a bath of aqueous copper(II) sulfate and
electroplating.

(i) Suggest a reason why orchids are first coated with graphite. [1]

(ii) To ensure high standards of electroplated orchids, the copper coating


must be at least 0.5 mm thick. Given that the total surface area of a
typical orchid is 10 cm2 and the operating current is 20 A, calculate the
time required to electroplate an orchid.
[Density of copper = 8.96 g cm3]
[3]

trendyline
NJC/H2Chem/03/2016 [Turn over

850 trendyline
13

(c) The use of Data Booklet is relevant to this question.

When iron is heated with bromine, FeBr 3 is produced. However, when heated
with iodine, FeI 2 is produced instead.

(i) Write the two equations for the formation of FeBr 3 .


[2]

(ii) Hence explain why when iron is heated with iodine, FeI 2 is produced but
notFeI3 . [2]

(iii) An unknown elementUis diatomic and is below iodine in Group VII of the
Periodic Table. The hydride of U, HU, and hydride of iodine, HI, differ in
their physical properties and reactivities.

(I) Suggest and explain how the polarities of H-U and H-I bonds
differ.

(II) Suggest and explain how the boiling points of HU and HI


differ.

(III) Suggest and explain how the acidities of HU and HI differ


when dissolved in water.

[4]

(d) Methane and steam react to give hydrogen and carbon monoxide as follows.

CH 4 (g) + H 2 O(g) 3 H 2 (g) + CO(g)

A mixture of 3.0 mol of CO and 8.0 mol of H 2 is heated in a closed container of


2 dm3 and allowed to reach dynamic equilibrium at 300 K. The amount of
hydrogen at equilibrium was found to be 6.5 mol.

(i) Determine the amounts of methane and steam at equilibrium.


[2]

(ii) Write the expression for the equilibrium constant, K c , for the above
equilibrium and calculate the K c at 300 K.
[2]

[Total: 20]

NJC/H2Chem/03/2016 [Turn over

851 trendyline
Suggested answers for 2016 SH2 Chemistry Paper 3
+ +
1 (a) Alums are salts formed when a monovalent cation with a large radius (e.g. K or NH 4 ) and a trivalent cation with a
3+ 3+ 3+
small radius (e.g. Al , Fe or Cr ) combine with sulfate ions.
An ammonium iron alum has the formula (NH 4 ) a Fe(SO 4 ) b .12H 2 O.
To determine the chemical formula of the alum, two separate samples of 0.5 g of the alum were each dissolved in 100
3
cm of water.
An excess of NaOH(aq) was added to the first alum solution and the mixture was boiled. The ammonia gas evolved
3 3
exactly neutralised 5.2 cm of0.200 mol dm HCl(aq).
3+ 2+
An excess of zinc was added to the second alum solution, which reduced Fe to Fe , and the resulting solution was
3 3
filtered. The filtrate collected required10.4 cm of 0.0200 mol dm acidified KMnO 4 (aq) for complete reaction.
+
(i) Calculate the amount of NH 4 ions present in 0.5 g of the alum. [1]

HCl + NH 3 NH 4 Cl
+ 5.2 3
Amount of NH 4 = x 0.200 = 1.04 x 10 mol [1]
1000

(ii) Write an equation for the reaction between the filtrate and acidified KMnO 4.
3+
Hence, calculate the amount of Fe ions present in 0.5 g of the alum. [2]
2+ 3+
[O] Fe Fe +e
+ 2+
[R] MnO 4 + 8H +5e Mn + 4H 2 O

2+ + 2+ 3+
Overall: 5Fe + MnO 4 + 8H Mn + 4H 2 O + 5Fe []
10.4 4
Amount of MnO 4 = x 0.0200 = 2.08 x 10 mol []
1000
3+ 4 3
Amount of Fe = 2.08 x 10 x5 = 1.04 x 10 mol [1]

(iii) Using your answers in (i) and (ii), determine the values of a and b. [2]
3+
There is 1Fe per formula unit of (NH 4 ) a Fe(SO 4 ) b .12H 2 O
3+ 1.04 10 3 1
= =
4+ 1.04 10 3 1

Hence, a =1 [1]
Comparing the charges of the ions:
 b) =0
b =2 [1]
or
0.5
M r of compound = = 480.77
1.04 10 3

Given than a= 1, 14.0 + 1.0 x 4 + 55.8 + (32.1 + 16.0 x 4) b + 12 x (16.0 + 2.0) = 480.77
b 

(b) The Pauson-Khan reaction is a gas phase reaction between an alkyne, an alkene and carbon monoxide to form a
cyclic carbonyl compound, as shown below.
Y O

O Y
+ + catalyst
C
Y'
Y'

trendyline
where Y and Y ca
can be Cl or H.
An organic compound
mpound A was produced from the Pauson
Pauson-Khan has the following composition
an reaction.The compound h
by mass; 51.5 % C, 13.7% O, 30.5% Cl and nd 4.3% H. M r of
o A is 116.5
116.5.
th ethanolic AgNO 3 , A gave
Upon heating with ve a white precipita
precipitate.

852 trendyline
(i) Determine the molecular formula of A and draw its structural formula. [3]
C O Cl H
% by mass 51.5 13.7 30.3 4.3
Relative amount / mol 4.29 0.856 0.853 4.3
Simplest ratio 5 1 1 5
[1] for table
Empirical formula of A: C 5 H 5 OCl
Let molecular formula of Abe (C 5 H 5 OCl) n
n(12.0 x 5 + 1.0 x 5 + 16.0 + 35.5) = 116.5
n =1
O

Molecular formula of A= C 5 H 5 OCl [1] Cl [1]

1
(ii) By quoting relevant data from the Data Booklet DQGJLYHQWKDWWKHERQGHQHUJ\RI&2LVN-PRO , prove
1
that the enthalpy change of reaction in the Pauson-Khan reaction to form A LVN-PRO . [2]

Bonds broken:1 &21 &&1 C=C


Bonds IRUPHG&&& && 2 [1]
+ = BE (bonds broken) BE(bonds formed)
1
= 1079 + 840 + 610 [  N-PRO [1]

(iii) Predict the sign of the entropy change for the formation of A in the above reaction. Hence, deduce if the
reaction is feasible at a high or low temperature. [2]

There is a decrease in the number of gaseous particles, hence there is a decrease in the degree
of disorderliness; entropy decreases or S is negative. [1]
Both + DQG 6 DUH of negative values; temperature must be low enough for 76 
+VXFKWKDW*. [1]

(iv) With the aid of a Boltzmann diagram, predict and explain the effect on the rate of the Pauson-Khan reaction
without a catalyst. [3]

Marking

trendyline
g points
- y-axis:
xis:
- start
roportio number of particles;
is proportion/
rt from zero (origin)
- peak
(orig
particles x-axis:

ak of the graph lies on the left half


- graph plateaus off towards the end
axis energy
x-axis: energ / kinetic energy

853 trendyline
- labelling of particles with energies above E a and E a
Total marks for graph:[1]; each point missing:[-]

Without a catalyst, the activation energy will be higher, thus there are fewer particles [] with
energy greater than or equal to the activation energy, resulting in lower frequency ofeffective
collisions[] and rate of the reactiondecreases.[]

(c) Another possible product of the Pauson-Khan reaction is shown below:


O

CO2H

B
(i) Draw the product formed when B reacts with each of the following.
I. cold KMnO 4 (aq) in KOH(aq)
II. H 2 , Pt
III. LiAlH 4
[3]
O
OH OH
CO2- K+

HO CO2H CH2OH

I. HO II. III.
[1]each

(ii) Compound C is an isomer of B, and has the structure shown below:


O

CO2H

C
Deduce which compound, B or C, is more acidic.[2]

C is a stronger acid than B. [1]


The delocalisation of lone pair of electrons from Oin CO 2 to the overlapped p orbitals in C=O
and C=C helps to disperse the negative charge on the conjugate base (carboxylate anion) of C,

O- , to a greater extent than that of B,making the anion of Cmore stable. [1]

[Total: 20]

2 Bovine serum albumi, BSAis a serum albumin protein which is a main constituent of cow milk. It is often used as a protein
concentration standard in lab experiments. The BSA molecule is a single chain of 607 amino acids, 66% of which are
incorporated into a -pleated sheet. It is a globular protein and takes up a roughly spherical shape in water.
Six of the most common amino acids in the BSA molecule are listed below.

trendyline
trendy
e dyyline
e
Amino acid Formula
Fo of side cha
chain
(R in RCH(NH 2 )CO 2 H H)
Isoleucine -CH(CH
CH(CH 3 )CHCH 2 CHH3
Methionine -CH
-CH
C 2 CH 2 SCH 3
Tyrosine -CH
-CH 2 C 6 H 4 OH
Asparagine -CH 2 CONH 2
Lysine
L i -CH
CH 2 CH 2 CH 2 CH 2 NH 2
Asparate -CH 2 COOH
3

854 trendyline
Table 1: Types of amino acid
(a) (i) There are three pK a values associated with lysine: 2.17, 9.04, 12.48.
Make use of these pK a values to suggest the major species present in solutions of lysine with the following pH values:

pH 1
pH 3
pH 10
pH 14
[4]
H H H
H
H 3N + C COOH H 3N + C COO- H2N C COO-
-
H2N C COO
(CH2)4 (CH2)4
(CH2)4
+ (CH2)4
+
NH3 NH3 NH2
+
NH3
pH 1 pH 3 pH10 pH14

(ii) Suggest two positive tests (stating reagents and observations) that would enable the amino acids asparagine and
tyrosine to confirm their identities. [2]

[1 each]
Test 1: Add NaOH(aq), warm
Observations: Tyrosine: no evolution of alkaline gas which turns red litmus paper blue;
Asparagine: alkaline gas evolves, turns red litmus paper blue.
Test 2a:Add neutral FeCl 3 (aq)
Observations: Tyrosine: purple complex forms; Asparagine: no purple complex forms.
Test 2b: Add Br 2 (aq)
Observations: Tyrosine: yellow / orange solution decolourised; Asparagine: yellow / orange solution
remains.

(b) (i) Describe the conditions needed to hydrolyse a protein non-enzymatically. [1]

HCl(aq)/H 2 SO 4 (aq); prolonged heating


or
NaOH(aq)/KOH(aq); prolonged heating

(ii) Briefly describe what is meant by the primary structure of proteins. [1]

Primary protein structure is a unique sequence of amino acids joined by peptide bonds in a polypeptide
chain.

(iii) Describe how a polypeptide chain is held in the shape of a -pleated sheet. [1]

The polypeptide backbone (polypeptide chain) is folded into -pleatedsheet.It is maintained through
hydrogen bonding found at regular intervals between N-H group of one peptide and C=Ogroup of
another peptide of the polypeptide backbone.

(iv) Using suitable amino acids in Table 1, describe hydrogen bonding interaction in the tertiary structure of proteins with
the aid of a diagram. [2]

855 trendyline
Tyrosine/Lysine/Asparate/Asparagine [1] [1]

(v) The following recipe illustrates how a lemon sponge cake can be made.
1. Cream butter and sugar until light and fluffy.
2. Beat in eggs.
3. Sift over the flour, add lemon juice and rind.
4. Bake for 10 minutes at 180C
Suggest how the above changes can cause denaturation, making specific reference to suitable pairs of amino acids
present in BSA. [2]

Beat in eggs:Mechanical agitation can break the weak interactions (van der Waals and Hydrogen
bonding) that hold the configuration of the protein together. (Isoleucine, Methionine, Tyrosine,
Asparagine) [2]

Add lemon juice:Destruction of ionic bonds occurs extensively at extreme pH conditions.


+
Adding acids: H + COO COOH
+
H from acids reacts ZLWK&22 on R groups of amino acid residues to formCOOH.Ionic bonds present
are broken. (Lysine and Asparate) [2]

Bake for 10 minutes at 180C: Energy supplied is sufficient to overcome the intermolecular
interactions/ionic bonds which hold the configuration of the protein together.(any pairs of amino acids)
[2]
+ o
(c) When a beam of 31T particles is passed through an electric field, the angle of deflection is found to be +5.00 . You may
+
P

assume that all the 31T particles travel at the same speed through an electric field of constant strength.
P

+
Calculate the angles of deflection if 31T is replaced with the following, leavingyour answers to 2 decimal places.
P

32 2-
(i) S [1]


Angle of deflection

2

1 5 = 0.94 [1]
Angle of deflection for 32S2= 32
3
56 3+
Fe [1]

Angle of deflection

3
Angle of deflection = + 561 5 = +0.80 [1]
3
+
The mass of particle Q is 25 times that of 31T .When a beam of Q is passed through the same electric field as in (a), the angle
P

o
of deflection is found to be +0.60 .
(ii) Determinethe overall charge on a particle of Q. [1]

trendyline
re
Q is positively charged


Angle of deflection
ction




+0.60 = 75
1 5
3
x 1]
= +3 [1]

856 trendyline
(iii) A particle ofQ contains 27 protons. Deduce the number of electrons in thisparticle and hence write its electronic
configuration. [1]

No of proton = 27
No of electron = 27-3 = 24
2 2 6 2 6 6
Electronic configuration: 1s 2s 2p 3s 3p 3d

(iv) A nuclear reaction is a reaction in which is a change to an atomic nucleus.


An experimental nuclear reactor use 13 C and Tritium, 31T, as fuel. A nuclear reaction between these two atoms are
described below (n is a neutron).
13
C + 31T ;Q 4He

Suggest the identities of X with the use of the Data Booklet. [1]

11
X: 5B or boron or B

3 (a) The kinetics of the hydrolysis of methyl ethanoate may be investigated by determining the concentration of ethanoic acid
produced.
CH 3 CO 2 CH 3 + H 2 O CH 3 CO 2 H + CH 3 OH
3
In a 1 dm mixture, 0.350 mol of CH 3 CO 2 CH 3 was heated with dilute hydrochloric acid which acts as the catalyst. The
following results were obtained.
3 3
Time / s [CH 3 CO 2 H] / mol dm [CH 3 CO 2 CH 3 ] / mol dm
0 0 0.350
340 0.105 0.245

680 0.185 0.165

1080 0.243 0.107

1440 0.278 0.072

(i) What is meant by the term order of reaction? [1]


m n
Rate = k [A] [B] where
m =order of reaction with respect to A
n =order of reaction with respect to B
m+n =overall order of reaction
or
The order of reaction is the power to which the concentration of a reactant is raised in an
experimentally determined rate equation.

(ii) By drawing a suitable graph using the data given above, determine the order of reaction with respect to methyl
ethanoate. [3]

Since CH 3 CO 2 CH 3 CH 3 CO 2 H, 0.350 mol of CH 3 CO 2 H must be formed with 100% yield as 0.350 mol
of CH 3 CO 2 CH 3 was used.
st
50% yield, [CH 3 CO 2 H] formed = 0.175 mol (1 half-life)
nd
75% yield, [CH 3 CO 2 H] formed = 0.263 mol (2 half-life)

trendyline
6

857 trendyline
Correctly plotted of all points [1]
Correctly labelled axes and 2 half-lives
Correctly identified 2 half-lives:
t (1) = t (2) [1]
order of reaction wrt methyl ethanoate is 1. [1]
(stating order is 1 wo correct reasoning will not score this mark)

(iii) Given that the hydrolysis reaction is first order with respect to hydrochloric acid, write down the rate equation for
this reaction. [1]
+
Rate = k[CH 3 CO 2 CH 3 ][H ]

0.400 [CH3CO2CH3] / mol dm3


0.375
0.350
0.325
0.300
0.275
0.250
0.225
0.200
0.175
0.150
0.125
0.100
0.075
0.050
0.025 t (1) = 630 s t (2) = 630 s
0.000
0 100 200 300 400 500 600 700 800 900 1000 1100 1200 1300
00
0 14
1400
00
0

trendyline
(b) Esters can also undergo
dergo reduction in the presence of a strong reducing agent such as lithium
ndergo lithiu aluminium hydride, LiAlH 4 .
The reaction produces
uces two alcohols
alcohols.
RCO
CO 2 R RCH
RC 2 OH + ROH
Using this information,
tion,
on, and that provided below, suggest a mechanism for the formation of et ethanol from methyl ethanoate.

There is an initialal nucleophilic attack by hydride ion, H , on the ester to generate a negat
tial negatively charged ion.
The C=O bond ndd reforms
f andd th OR group is eliminated from the ester, forming an ald
the OR aldehyde.
A second hydrideride ion attacks the aldehyde to generate e an alkoxide ion
ion.
The alkoxide ion is then protonated to form the alcohol.

858 trendyline
O O
+
O O
H
H
Mark Scheme:
O O
Penalise 1 m if wrong starting cpd given.
O
Penalise 0.5m for every following
H
mistakes:
O O Each missing/wrong arrows (up
to 2 m)
+ H Missing any partial charge on
H ester and aldehyde
H Missing lone pairs on hydride

O
OH
H+
H
H

[3]

(c) Besides esters, nitriles can also undergo reduction to give aldehyde in the presence of another strong reducing agent,
dissobutyl aluminium hydride (DIBAL).
RCN RCHO
(i) By considering the change in oxidation states of the reacting carbon, suggest why the conversion of a nitrile to an
aldehyde is a reduction reaction. [1]

The oxidation state of carbon changes from +3 in nitrile to +1 in aldehyde.

CH2Br 4 steps
CH2CH(OH)CO2H

Compound X
(ii) Compound X can be synthesised using DIBAL as a reducing agent in one of the steps above.
Suggest the reagents and conditions required for each step of synthesis and identify all the intermediate
compounds.
[5]
CH2Br CH2CN CH2CHO CH2CH(OH)CN CH2CH(OH)Co2H
ethanolic NaCN DIBAL HCN dil H2SO4
heat trace amount of NaCN heat

[1]: each intermediate product(total 3m)


[]: each reagent and condition (total 2m)

(d) A Wittig reagent can convert an aldehyde to an enone, a ketone with an adjacent double bond.
O O
Ph3P R' CHO R'
R R
wittig reagent enone
:KHUH3KUHSUHVHQWV&
VHQWV& 6 H 5 )

859 trendyline
Wittig reagent F was reacted with an aldehyde G to give an enone H. H was then reacted with hot acidified potassium
manganate (VII) to give two different compounds, J and K.
H can also be synthesised by the following route.
OH
ethanolic NaOH hot acidified K2Cr2O7
heat
L H
enone
Cl

(i) Suggest the identities of compounds H, J, K and L. [4]


H: J/K:
O OH

K/J: L:
O OH
HO

O
[1] each
(iii) Suggest the identities of compounds F and G. [2]
1 each
F: Ph 3 PCH 2 COCH(CH 3 ) 2 G: CH 3 CH 2 CHO [1] each

4 (a) The question below is related to some Group II compounds.


(i) Write an equation, with state symbols, for the thermal decomposition of strontium nitrate. [1]

Sr(NO 3 ) 2 V 6U2 V 12 2 (g) + O 2 (g) [1]


Or
2Sr(NO 3 ) 2 V 6U2 V 12 2 (g) +O 2 (g) [1]

(ii) Use the Data Booklet to explain how the thermal stability of zinc nitrate might compare to strontium nitrate.
[3]
From Data Booklet,
2+ 2+
ionic radius of Zn = 0.074 nm ; ionic radius of Sr = 0.113 nm []

2+ charge
Zn has a higer ratio / charge-density and hence a higher polarizing power. []
size
2+
Hence Zn will polarise/distort the electron cloud of NO 3 to a greater extent; NO bond is
weakened more []
Easier/requires less energy to overcome the N-O bond in zinc nitrate. []
Thus, Zn(NO 3 ) 2 is thermally less stable than Sr(NO 3 ) 2 . [1].

(iii) The chemical properties of beryllium and its compounds show similarity to that of aluminium. Beryllium
complexes have coordinate number 4.
I. Aluminium chloride, AlCl 3 , dimerises to form Al 2 Cl 6 . Draw the displayed formula of Al 2 Cl 6 , showing
clearly the bond angle about Al. [1]

trendyline
d
109.5

860 trendyline
II. Explain why AlCl 3 dimerises. [1]

Al is electron deficient/has one vacant p orbital/has not achieved octet and hence can
accept a lone pair from Cl.

III. Explain, using relevant equations, why a solution of beryllium chloride is acidic. [2]
2+ + +
[Be(H 2 O) 4 ] + H 2 O [Be(H 2 O) 3 OH] + H 3 O [1]

2+ charge
Be has high charge density (or ratio) and is able to polarise O-H bond of water,
size
+
weakening and breaking the OH bond to give H . [1]

IV. Beryllium oxide, similar to aluminium oxide, is amphoteric.


Suggest the nature of bonding in beryllium oxide. [1]

Beryllium oxide is ionic with covalent character. [1]

V. Write equations for the reaction of beryllium oxide with HCl(aq) and with NaOH(aq). [2]

BeO + 2HCl BeCl 2 + H 2 O [1]


2
BeO + 2OH + H 2 O [Be(OH) 4 ] [1]
or BeO + 2NaOH + H 2 O Na 2 [Be(OH) 4 ]

(b) The compound responsible for the hot taste of chilli peppers is capsaicin. Its molecular structure can be
deduced by the following reaction scheme. Capsaicin is slightly acidic.
boil with H+ NH2
C18H27NO 3 + C10H18O 2
Capsaicin N
HO

OCH 3
M
(i) When N is heated with concentrated acidified potassium manganate(VII), P, C 6 H 10 O 4 and Q, C 4 H 8 O, are
formed.
P is formed from the reaction of Br(CH 2 ) 4 Br with hot ethanolic KCN followed by hydrolysis. Q can be reduced
to an alcohol, R, which is optically active.
Deduce the structures of N, P, Q, R and capsaicin. [5]

P: HOOCCH 2 CH 2 CH 2 CH 2 CO 2 H Q: CH 3 COCH 2 CH 3 R: CH 3 CH(OH)CH 2 CH 3


CH3
HO

O CH3
N:
Capcaisin:
CH3
NH

HO O
CH3
OCH 3

trendyline
[1] each

10

861 trendyline
(ii) Assuming that OCH 3 is inert, draw the organic compound formed when M reacts with CH 3 CH 2 COCl.
O

O N
H
O
OCH3
[1]

(iii) Peroxodisulfate(VI) ion, S 2 O 8 , is capable of oxidising tartaric acid, (CH(OH)CO 2 H) 2 , to carbon dioxide and
o
methanoic acid. The E cell of the reaction is +0.99 V.
 + 
(CH(OH)CO 2 H) 2 + 3S 2 O 8 + 2H 2 O 2CO 2 + 2HCO 2 H + 6H + 6SO 4
The reaction is very slow, even when temperature is increased.
However, the reaction rate can be increased by adding a suitable catalyst.
The half equation for the reduction of carbon dioxide to tartaric acid is
+
2CO 2 + 2HCO 2 H + 6H + 6e (CH(OH)CO 2 H) 2 + 2H 2 O
Using the Data Booklet, show how cobalt(III) ion can act as a homogenous catalyst.
2 2 0
S 2 O 8 + 2e 2SO 4 E red = +2.01V

o o  2 o
Since E cell =E red (S 2 O 8 / SO 4 ) + E ox ((CH(OH)CO 2 H) 2 /HCO 2 H, CO 2 )
o
+0.99 =+ 2.01 + E ox ((CH(OH)CO 2 H) 2 /HCO 2 H, CO 2 )
o
E ox ((CH(OH)CO 2 H) 2 /HCO 2 H, CO 2  9
0
HenceE red (HCO 2 H, CO2 / (CH(OH)CO 2 H) 2 ) =+ 1.02 V [1]
3+
With Co catalyst,
Step 1
3+ 2+ o
Co +e Co E red = +1.82 V
+ o
(CH(OH)CO 2 H) 2 + 2H 2 O 2CO 2 + 2HCO 2 H + 6H + 6e E ox 9

3+ + 2+
(CH(OH)CO 2 H) 2 + 2H 2 O + 6Co 2CO 2 + 2HCO 2 H + 6H + 6Co
o
E cell = + 0.80 V > 0, reaction is feasible. [1]

Step 2
2+ 3+ o
Co Co +e E ox 9
2 2 o
S 2 O 8 + 2e 2 SO 4 E red = +2.01 V
2+ 2 3+ 2
2Co + S 2 O 8 2Co + 2 SO 4
o
E cell = + 0.19 V > 0, reaction is feasible. [1]
3+ 2+ 2+
Co oxidises tartaric acid to give CO 2 and HCOOH and itself is reduced to Co . Co then
2 2 3+ 3+
reduces S 2 O 8 to SO 4 and Co is regenerated. Hence Co acts as a homogeneous catalyst.

11

862 trendyline
5 (a) Chlorine is manufactured from brine, concentrated NaCl, by electrolysis using inert electrodes at room
temperature.The anode and cathode compartments are separated by a diaphragm which is a permeable membrane.
Brine Cl2(g) Gas S

Diaphragm

(i) Suggest the identities of gas S and compound T. [1]

S: Hydrogen / H 2 []T: NaOH []


o
(ii) Using relevant E data from the Data Booklet, suggest the reason why the electrolysis of dilute NaCl(aq)
produces mainly oxygen at the anode, whereas the electrolysis of brine produces mainly chlorine gas.[2]
+ o
2 H2O O 2 + 4 H + 4 e---(1) E ox = 1.23 V
o
2 Cl Cl 2 + 2 e---(2) E ox = 1.36 V []
of Cl, hence water is

When dilute NaCl is used, E ox H 2 O is more positive/less negative than E ox
preferentially oxidised to O 2 . []

When brine is used, concentration of Cl is high, eqm (2) shifts right and
E ox (Cl) is more positive/less negative than E ox of H 2 O [] (due the small initial difference
o

between theE ox values ) hence Clis preferentially oxidised to Cl 2 . []


o

(iii) Suggest what the products might be if the diaphragm is removed and the solution is stirred, by writing an
equation for any reaction that occurs. [1]

Cl 2 (g) + 2 NaOH (aq) NaCl (aq) + NaClO (aq) + H 2 O (l) OR


Cl 2 (g) + 2 OH DT &l (aq) + ClO (aq) + H 2 O (l)
No need for state symbols. [1] or [0]

(b) Orchids can be made into jewellery that gleams with either a reddish tint of copper or shiny gold. In copper-plating,
orchids are coated with a thin layer of graphite paste before placing them in a bath of aqueous copper(II) sulfate and
electroplating.

(i) Suggest a reason why orchids are first coated with graphite. [1]
To conduct electricity

(ii) To ensure high standards, the copper coating must be at least 0.5 mm thick. Given that the total surface area
2
of a typical orchid is 10 cm and the operating current is 20 A, calculate the time required to plate an orchid.
3

trendyline
[Density of copper = 8.96 g cm ] [3]

12

863 trendyline
0.5 3
Volume of Cu layer = 10 = 0.5 cm
10
Mass of Cu layer = 8.96 0.5 = 4.48 g
4.48
Amt of Cu in the layer = = 0.07055 mol [1]
63.5
Amt of electrons required = 2(0.07055) = 0.14110 mol [1]
Amt of charge required = 0.14110 96500 = 13616 C
13616
Time required to plate an orchid = = 680.81 = 681 s [1]
20
(c) The use of Data Booklet is relevant to this question.
When iron is heated with bromine, FeBr 3 is produced. However, when heated with iodine, FeI 2 is produced instead.
(i) Write the two equations for the formation of FeBr 3 . [2]

Fe + Br 2 )H%U 2 [1 or 0]
FeBr 2 + Br 2 )H%U 3 [1 or 0]

(ii) Hence explain why when iron is heated with iodine, FeI 2 is produced but not FeI 3 . [2]


I 2 + 2e I E = +0.54 V---(1)
2+
Fe Fe + 2e E ox = +0.44 V---(2)
2+ 3+
Fe Fe +e E ox = -0.77 V---(3)
(1) + (2): Fe + I 2 )H, 2

E cell = +0.54 + 0.44 = +0.98 V > 0,reaction is feasible [1]
(1) + (3): FeI 2 + I 2 )H, 3

E cell = +0.54 -0.77 = -0.23 V < 0, reaction is not feasible
2+ 3+
Iodine can only oxidise iron to Fe , not Fe . [1]

(iii) An unknown element, U, which exists as diatomic molecules, is said to be positioned below iodine in Group
VII. The hydride of U, HU, and of iodine, HI, differ in their physical properties and reactivities.
(I) Suggest and explain how the polarities of H-U and H-I bonds differ. [1]

H-U bond is less polar [] as U is less electronegative [].

(II) Suggest and explain how the boiling points of HU and HI differ. [2]

HU molecules have a larger electron cloud [] than HI molecules, hence a greater ease of
distortion of electron cloud [; temporary dipole-LQGXFHG GLSROH LQWHUDFWLRQV WGLG  DUH PRUH
readily induced.
More energy is required to RYHUFRPHWKHVWURQJHUWGLGLQWHUDFWLRQVEHWZHHQ+8PROHFXOHV>@

(III) Suggest and explain how the acidities of HU and HI differ when each is dissolved in water.
[1]
HU is a stronger acid []
HU bond is weaker than HI bond [].
OR

y
+
HU bond
nd is longer than HI
H I bond, hence
e is easier to break
br H U bonds to give H . []
H-U
(d) eam react to give hydrogen and carbon monoxide as follows.
Methane and steam
CH 4 (g)
g) + H 2 O(g)
(g) 3 H 2 (g)
g) + CO(g)
3
A mixture of 3.0
0 moll
m of CO and 8.0 molll of H 2 is heated in a closed container of 2 dm d and allowed to reach
dynamic equilibrium
um at 300 K. The amount of hydrogen at equilibrium was found to be 6.5 m
ium mol.
(i) e the amounts of methane and steam at equilibrium.
Determine equ [2]

13

864 trendyline
CH 4 (g) + H 2 O(g) 3H 2 (g) + CO(g)
Initialamt/ moll 0 0 8.0 3.0
1.5
Eqm amt/moll 0.5 0.5 6.5 3.0 =2.5
3
0.500 mol each for methane and steam

(ii) Write the expression for the equilibrium constant, K c , for the above equilibrium and calculate K c at 300 K. [2]

[ H 2 ]3 [CO]
kc
[CH 4 ][H 2 O]
3
6.5 2.5

2.0 2.0

2
0.5

2.0
687 mol 2 dm 6
Expression [1] answer with units [1]

trendyline
14

865 trendyline
H
NANYANG JUNIOR COLLEGE
JC 2 PRELIMINARY EXAMINATION
Higher 2

CHEMISTRY 9647/01
Paper 1 Multiple Choice 27September 2016
1 hour
Additional Materials: Multiple Choice Answer Sheet
Data Booklet

READ THESE INSTRUCTIONS FIRST

Write in soft pencil.


Do not use staples, paper clips, highlighters, glue or correction fluid.
Write your name, class and tutors name on the Answer Sheet in the spaces provided unless
this has been done for you.

There are forty questions on this paper. Answer all questions. For each question there are
four possible answers A, B, C and D.
Choose the one you consider correct and record you choice in soft pencil on the separate
Answer Sheet.

Read the instructions on the Answer Sheet very carefully.

Each correct answer will score one mark. A mark will not be deducted for a wrong answer.
Any rough working should be done in this booklet.

This document consists of 20 printed pages and 0 blank page.


[Turn over
2

Section A

For each question there are four possible answers, A, B, C and D. Choose the one
you consider to be correct.

1 In an experiment, lanthanum, 57La, was reacted with hydrogen to produce the


non-stoichiometric ionic compound LaH 2.9 . Assuming that the compound
contains H, La2+ and La3+, what is the percentage of La3+ present in LaH 2.9 ?

A 10%
B 20%
C 80%
D 90%

2 The two most common isotopes of titanium are 56


22Tiand
58
22Ti.

Which statement about the isotopes of titanium is correct?

A Both isotopes have more electrons than neutrons.


B One of the isotopes has more protons than the other.
C The electronic configuration of Ti2+ ion for both isotopes is the same.
56 58
D In the same electric field strength, 22Tiwill be deflected more than 22Ti.

3 Which compound does not have a co-ordinate bond?

A CO B CS 2 C NO 3  D NH 4 +

4 In which of the following pairs do the species have the same bond angle?

A H 2 O 2 and N 2 F 2 B PH 3 and SF 3 +
C POCl 3 and SO 3 2 D HCHO and BrF 3

H2 Chemistry 9647/01 NYJC J2/16 PX


3

5 Two glass vessels M and N are connected by a closed valve.

M contains helium at 20 qC at a pressure of 1 105 Pa. N has been evacuated,


and has three times the volume of M. In an experiment, the valve is opened and
the temperature of the whole apparatus is raised to 100qC.
What is the final pressure in the system?

A 3.18 104 Pa
B 4.24 104 Pa
C 1.25 105 Pa
D 5.09 105 Pa

6 Which value would be required to estimate the lattice energy for the hypothetical
ionic compound MgH?

A the electronic affinity of hydrogen


B the first ionisation energy of hydrogen
C the magnesiumhydrogen bond energy
D the standard enthalpy change of formation of MgH 2

H2 Chemistry 9647/01 NYJC J2/16 PX [Turn over


4

7 A student performed an experiment to investigate a hypothetical reaction.


2A + BoC
The graph of [A] against time(t) for the experiment is shown below.
[A]
0.0500

0.0250

0.0125

t / min
0 20 40
Given that the initial [B] is 2.0 mol dm3 and units for the rate constant is
mol1 dm3 min1, which of the following statementsis true?

A The reaction is elementary.


B The half-life of the reaction remains constant when a catalyst is added.
C The gradient of tangent at t = 0 min increases by four times when the initial
[B] doubles.
D The half-life of the reaction remains constant when [A] doubles but halved
when [B] doubles.

8 Glaubers salt, Na 2 SO 4 10H 2 O, was used by J. R. Glauber in the 17th century as


a medicinal agent. At 25 oC, the numerical value of the equilibrium constant, K p
for the loss of water of hydration from Glaubers salt is 4.08 10.
Na 2 SO 4 10H 2 O(s) Na 2 SO 4 (s) + 10H 2 O(g)
Which of the following statements can be correctly deduced about this
equilibrium reaction?

A The reaction reaches dynamic equilibrium when the forward and reverse rate
constants are equal.
B The numerical value of the vapour pressure of water at 25 oC in a closed
container holding a sample of Na 2 SO 4 10H 2 O(s) is 3.64 10.
C The ratio of the hydrated form to anhydrous form of the Glaubers salt
remains the same when nitrogen gas is added while keeping the total
pressure of the system constant.
D The numerical value of K p will increase when temperature is increased.

H2 Chemistry 9647/01 NYJC J2/16 PX


5

9 Highly toxic disulfur decafluoride decomposes by a free-radical process.


S 2 F 10 (g) SF 4 (g) + SF 6 (g)
In a study of the decomposition, S 2 F 10 was placed in a 2.0 dm3 flask and heated
to 100 oC. The equilibrium [S 2 F 10 ] was found to be 0.5 mol dm. More S 2 F 10
was then added and the new equilibrium [S 2 F 10 ] was 2.5 mol dm.
What is the amount of S 2 F 10 reacted in terms of the equilibrium constant, K c of
the decomposition reaction when more S 2 F 10 was added?

A (0.5K c )0.5  K c )0.5


B (2.5K c )0.5  K c )0.5
C (2K c )0.5  K c )0.5
D (10K c )0.5  K c )0.5

10 Phosphorus acid, H 3 PO 3 , has two acid dissociation values:


K a1 = 1.0 102 mol dm3 K a2 = 2.5 107 mol dm3
Which of the following statements is correct about H 3 PO 3 ?
I: H 3 PO 3 is a dibasic acid as one of the H atom is bonded to P atom.
II: The K b values of H 2 PO 3  and HPO 3 2are 4.0 1012 and 1.0
1012respectively.
III: K a2 is smaller than K a1 because it gets increasingly difficult to remove a H+
from a negatively charged ion.

A I only
B I&III
C II&III
D All statements are correct

H2 Chemistry 9647/01 NYJC J2/16 PX [Turn over


6

11 The diagram below shows the change of pH produced when 20.0 cm3 of
trimethylamine was titrated with 0.01 mol dm3 of HCl(aq) at 25 oC.

pH

11

Volume of acid/ cm3


25.00N

Which of the following statements is incorrect?

A The K b value of trimethylamine is 8.00 105.


B When 35.00 cm3 of acid is added, pH of the solution is 2.7.
C Phenolphthalein is not a suitable indicator for the above titration.
D When concentration of HCl is doubled, the end point volume of the titration
would be halved while the pH at equivalence point would remain the same.

12 The e.m.f of three different experiments was measured under standard


conditions and the results are shown below:
Experiment 1: W(s) + X2+(aq) o W2+(aq) + X(s) E cell > 0
+ 2+
Experiment 2: X(s) + 2Y (aq) o X (aq) + 2Y(s) E cell < 0
Experiment 3: Z(s) + W2+(aq) oZ2+(aq) + W(s) E cell > 0

Which of the following statements is correct?

A Z2+ is the strongest oxidising agent.


B If concentration of Y+(aq) is lowered in experiment 2, the E cell would be more
positive.
C The reaction of Z(s) + W2+(aq) oZ2+(aq) + W(s) is spontaneous.
D The reducing power decreases in the order Z>W>Y>X.

13 In a lead-acid battery reaction, the charging process is achieved by passing an


electric current through the cell. The overall reaction equation for the charging
process is given below.
2PbSO 4 + 2H 2 O o Pb + PbO 2 + 2H 2 SO 4
H2 Chemistry 9647/01 NYJC J2/16 PX
7

After charging, PbSO 4 on one plate is converted to PbO 2 . The other plate is
converted to Pb. When the two plates are connected, a current will flow, enabling
the discharging process to take place.

Which species is oxidised when the battery is discharged?

A Pb
B H2O
C PbO 2
D SO 4 2

14 The diagram represents an unsuccessful attempt to prepare and collect sulfur


dioxide.
dilute
hydrochloric acid

X Y

sodium
aqueous concentrated
sulfite,
heat NaOH sulfuric acid
Na2SO3
Which modification would make the experiment successful?

A omitting flask X entirely


B omitting flask Y entirely
C using dilute sulfuric acid instead of dilute hydrochloric acid
D collecting by upward delivery

H2 Chemistry 9647/01 NYJC J2/16 PX [Turn over


8

15 Which of the following statements for strontium or its compounds is incorrect?

A Strontium sulfate is sparingly soluble in water.


B Strontium hydroxide is dehydrated to the oxide on strong heating.
C Strontium reacts with cold water to form strontium oxide and hydrogen.
D Strontium carbonate decomposes at a higher temperature than calcium
carbonate.

16 Letters written on paper using aqueous ammonium thiocyanate are invisible until
turned blood red by brushing the paper with aqueous iron(III) chloride. If the
ammonium thiocyanate is first made alkaline, the letters are orange and less
clear.
Which of the following statements is correct?

A The colour changes are due to ligand exchange reactions only.


B Aqueous iron(III) chloride can exist as [Fe(OH)(H 2 O) 5 ] 2+ in solution.
C With aqueous ammonium thiocyanate, [Fe(SCN)(H 2 O) 5 ]3+ complex is formed.
D With alkaline ammonium thiocyanate, the letters are less clear because a
less stable complex is formed.

17 For the sequence hydrogen chloride, hydrogen bromide and hydrogen iodide,
there is a decrease in
I volatility
II pK a value
III thermal stability
IV reducing power

A I&IV
B II&III
C I, II&III
D All of the above

18 A mixture of 1 mol of chlorine and 1 mol of bromine was reacted completely with
iron(II) solution. The resulting mixture was subsequently treated with excess
AgNO 3 (aq). After standing for 5 minutes,excess dilute NH 3 (aq) was added to the
mixture and filtered to obtain filtrate F and residue G.
Which of the following statements is correct?

A If excess HNO 3 (aq) is added to filtrate F, a white precipitate of mass 143.5 g


will be obtained.
B Mass of residue G is 376 g.
C Filtrate F contains [Ag(NH 3 ) 2 ]+,Cl and Fe3+ ions.
D Residue G is made up of more than one precipitate.
H2 Chemistry 9647/01 NYJC J2/16 PX
9

19 Cracking is a thermal decomposition process by which large hydrocarbon


molecules are broken down by passing them over a heated catalyst at high
pressure. The products are smaller alkanes and alkenes.
The cracking of a single hydrocarbon molecule, C n H 2n+2 , produces two
hydrocarbon molecules only. Each hydrocarbon product contains the same
number of carbon atoms in one molecule. Each hydrocarbon product has non-
cyclic structural isomers.
What is the value of n?

A4 B6 C8 D9

20 Carvone is found in spearmint.

Carvone

How many V and S bonds are present in this molecule?

V S
A 11 6
B 13 6
C 25 3
D 25 6

21 Many different compounds have been used in aerosol sprays, refrigerators and in
making foamed plastics.
Which compound will cause the most ozone depletion?

A CCl 3 F
B CH 2 FCHClF
C CH 3 CH 2 CH 2 CH 3
D N2O

H2 Chemistry 9647/01 NYJC J2/16 PX [Turn over


10

22 The reactivity of methoxybenzene, C 6 H 5 OCH 3 is similar to that of phenol.

How many moles of aqueous nitric acid are required to react with one mole of
bis(4-methoxyphenyl)amine?
NH

H3CO OCH3
bis(4-methoxyphenyl)amine

A 2
B 3
C 4
D 5

23 Which of the following will not yield a final organic product containing deuterium?
(D = 2H)

compound reagents and conditions

NH2
A H3C DCl, D 2 O, heat under reflux
O

B CH 3 CH 2 CH 2 COCl D 2 O, room conditions

C CH 3 CH 2 CHO DCN, trace NaCN, 15 oC

OH
H3C CH 3 OD, conc D 2 SO 4 , heat under
D
reflux
O

H2 Chemistry 9647/01 NYJC J2/16 PX


11

24 Compound X is used to synthesize DEET, a common active ingredient in


mosquito repellent. A brick-red precipitate is observed when it reacts with
Fehlings reagent. 1 mol of Xundergoes the following reactions.

K2Cr2O7/H+ KMnO4/H+
V X Y
' '

Y reacts with HNO 3 under suitable conditions to produce only 1 possible organic
product. V reacts with Na 2 CO 3 to produce only 1 mol of CO 2 (g).

What could X be?


A B

CH2OH COOH

CHO CH2CHO

C D

CH2OH CH=CHCOOH

CH2CHO COCH3

H2 Chemistry 9647/01 NYJC J2/16 PX [Turn over


12

25 Which of the following statements about compound W is correct?

Cl

HO2C
O

O HN
O

O
compound W

A An aqueous solution of W is approximately neutral as there is 1 carboxylic


acid group and 1 amine group present in the structure.
B The product formed when W is reduced by LiAlH 4 will react with Na to
produce 2 moles of H 2 (g).
C The reduction of W by LiAlH 4 will cause the oxidation state of any carbon
LQYROYHGLQWKHUHGXFWLRQWRGHFUHDVHIURPWR
D W will require 3 moles of NaOH(aq) for complete reaction if the reaction is to
take place with heating.

H2 Chemistry 9647/01 NYJC J2/16 PX


13

26 The Hofmann elimination is a process where an amine undergoes treatment with


excess methyl iodide to form a tertiary amine intermediate followed by treatment
with silver oxide, water and heat to form an alkene.

CH3 H
CH3
CH3 excess CH3I CH3 Ag2O, H2O H3C
H H3C

+ -
heat
+
NH2 N (CH3)3 II
(CH3)3N

What is the structure of the alkene formed when a cyclic amine, piperidine,
undergoes the Hofmann elimination?

NH

Piperidine

A CH 2 =CHCH 2 CH 2 CH 2 N(CH 3 ) 2
B CH 2 =CHCH 2 CH 2 CH 3

C N CH3

27 Which of the following compounds can be used, in a 1-step conversion, to


produce the following cyclic structure?
O O

O N
H
A HO 2 CCH 2 NH 2 and HO 2 CCH 2 OH
B HO 2 CCH 2 CONHCH 2 OH
C ClOCCH 2 COCl and H 2 NCH 2 OH
D ClOCCH 2 OH and H 2 NCH 2 CO 2 H

H2 Chemistry 9647/01 NYJC J2/16 PX [Turn over


14

28 One gram of each of the following compounds was heated with NaOH(aq), and
then dilute HNO 3 (aq) and AgNO 3 (aq) were added.
Which compound will produce the largest mass of AgCl(s) in 30 seconds?

A B C D
O CH2Cl O

Cl
Cl Cl

Cl
Cl Cl
O CH2Cl

O
C 3 H 2 O 2 Cl 2 C 6 H 10 Cl 2 C 6 H 4 Cl 2 C 4 H 4 O 2 Cl 2
M r = 141 M r = 153 M r = 147 M r = 155

29 Consider the following four compounds.


1 ethanamide, CH 3 CONH 2

NH2
2 phenylamine,

3 piperidine, NH

H H
C C
4 pyridine, N H C N H
C C
H H

What is the relative order of decreasing basicity of these compounds?

A 3 2 1 4
B 3 2 4 1
C 3 4 2 1
D 3 4 1 2

H2 Chemistry 9647/01 NYJC J2/16 PX


15

30 Upon complete hydrolysis, 1 mol of polypeptide X gives the following amino acids.

amino acid structure Mr mass of amino acid / g

asparagine O 132 16.8

C NH2

H2C
NH2
C
H
COOH

proline NH 115 7.3

COOH

serine OH 105 13.3

H2C
NH2
C
H
COOH

What is the M r of X?

A 352
B 499
C 517
D 589

H2 Chemistry 9647/01 NYJC J2/16 PX [Turn over


16

Section B

For each of the questions in this section one or more of the three numbered
statements 1 to 3 may be correct.

Decide whether each of the statements is or is not correct (you may find it helpful to
put a tick against the statements which you consider to be correct).

The responses A to D should be selected on the basis of

A B C D
1, 2 and 3 1 and 2 2 and 3 1 only
are only are only are is
correct correct correct correct

No other combination of statements is used as a correct response.

31 The hexagonal form of boron nitride, h-BN, is analogous to graphite where it has
a planar hexagonal layered structure of alternating boron and nitrogen atoms.

Which properties are shown by h-BN?

1 Its N atoms are sp2 hybridised.


2 It can be used as a dry lubricant.
3 It is soluble in organic solvents but not soluble in water.

32 Which of the following has the same value as the standard enthalpy change of
formation of carbon monoxide?

1 'H
(CO 2 ) 'H
(CO)
2 'H
(CO) 'H
(graphite)
3 'H
(CO 2 )

H2 Chemistry 9647/01 NYJC J2/16 PX


17

The responses A to D should be selected on the basis of

A B C D
1, 2 and 3 1 and 2 2 and 3 1 only
are only are only are is
correct correct correct correct

No other combination of statements is used as a correct response.

33 A student investigated the reaction between iodide and peroxodisulfate.

2I(aq) + S 2 O 8 2(aq) oI 2 (aq) + 2SO 4 2(aq)

He first performed the experiment by adding Fe3+(aq) as catalyst. He then


performed a second experiment under the same condition but did a change at t 1 .
The graphs of [I 2 ] against time for both experiments are shown below.
[I2]

First experiment
Second experiment

time / min
t1
Which of the following statements are correct?

1 acidified VO 2 Cl(aq) can also be used as a catalyst.


2 Sodium cyanide could have been added at t 1 .
3 The Boltzmann distribution graph for the first experiment is as follow
Number of
Particles

Kinetic Energy
Ea (catalysed)

H2 Chemistry 9647/01 NYJC J2/16 PX [Turn over


18

The responses A to D should be selected on the basis of

A B C D
1, 2 and 3 1 and 2 2 and 3 1 only
are only are only are is
correct correct correct correct

No other combination of statements is used as a correct response.

34 Phosphorus is an element in the third period, Na to Ar, of the Periodic Table.


What is true for phosphorus and none of the other elements in this period?

1 Phosphorus is the only element in this period with exactly four atoms in its
molecules.
2 Phosphorus is the only element in this period which forms two acidic oxides.
3 Phosphorus is the only element in this period whose chlorides react with
water to form acidic solutions.

35 Which of the statements of the Group II elements (magnesium to barium) or their


compounds is correct?

1 Reactivity of Group II elements with oxygen increases down the group.


2 The volume of acidic gas evolved from the decomposition of 1 mol of Group
II nitrate is four times that of the neutral gas evolved.
3 Solubility of the Group II sulfates decreases down the group.

H2 Chemistry 9647/01 NYJC J2/16 PX


19

The responses A to D should be selected on the basis of

A B C D
1, 2 and 3 1 and 2 2 and 3 1 only
are only are only are is
correct correct correct correct

No other combination of statements is used as a correct response.

36 The use of the Data Booklet is relevant to this question.


Which of the following species would convert manganese(II) sulfate to
manganese(IV) oxide in acidic solution?

1 Pb(NO 3 ) 4
2 Na 2 S 2 O 8
3 CoCl 2

37 Which of these alwaysapplies to a nucleophile?

1 It has a lone pair of electrons.


2 It is negatively charged.
3 It attacks a double bond.

38 Long-chain alkanes are converted on an industrial scale into alkylsulfates for use
as detergents, e.g. sodium lauryl sulfate.

H H H O

H C C C O S ONa

H H H O
10

sodium lauryl sulfate

What deductions about the properties of this substance can be made from this
structure?

1 Part of the structure is polar and is hydrophilic.


2 The alkyl chain is soluble in oil droplets.
3 All the CCC bond angles are 109o.

H2 Chemistry 9647/01 NYJC J2/16 PX [Turn over


20

The responses A to D should be selected on the basis of

A B C D
1, 2 and 3 1 and 2 2 and 3 1 only
are only are only are is
correct correct correct correct

No other combination of statements is used as a correct response.

39 There is a range of reactions of the aldehyde group which have the pattern
OH
H3C H3C
C O + HX C
H X
H

of which the formation of a cyanohydrin (where X = CN) is one.

Which compounds could be obtained by such an addition to an aldehyde group,


followed by a dehydration?

1 CN

2 H
O
O OCH3

NH CH
N CH3
3

40 Which of the following reactions produce a carbocation as an intermediate


product and an organic molecule containing a chiral centre that is optically
inactive as the final product?

1 C 6 H 10 + HBro C 6 H 11 Br
2 CH 3 CH=CH 2 + Br 2 + NaNO 3 o CH 3 CH(ONO 2 )CH 2 Br + NaBr
3 CH 3 C(CH=CH 2 )(CH 2 CH 3 )Br + NaOH o CH 3 C(CH=CH 2 )(CH 2 CH 3 )OH +
NaBr

H2 Chemistry 9647/01 NYJC J2/16 PX


21

H2 Chemistry 9647/01 NYJC J2/16 PX [Turn over


H
NANYANG JUNIOR COLLEGE
JC 2 PRELIMINARY EXAMINATION
Higher 2

CANDIDATE
NAME
INDEX
CLASS 1 5 NO.
TUTOR

CHEMISTRY 9647/02
Paper 2 Structured 14 September 2016
2 hours
Candidates answer on the Question Paper

Additional Materials: Data Booklet

READ THESE INSTRUCTIONS FIRST


Write your name and class on all the work you hand in.
Write in dark blue or black pen on both sides of the paper.
You may use a soft pencil for any diagrams, graphs or rough working.
Do not use staples, paper clips, highlighters, glue or correction fluid.

Answer all questions in the spaces provided.


A Data Booklet is provided.

At the end of the examination, fasten all your work securely together.
The number of marks is given in brackets [ ] at the end of each question or part question.

For Examiners Use

1 P /12
2 / 15
3 / 6
4 / 13
5 / 6
6 / 20 /60

This document consists of 17 printed pages and 1 blank page.

H2 Chemistry 9647/02 NYJC J2/16 PX [Turn over


For
2 Examiner's
Use
Answer all questions in the spaces provided.

1 Planning (P)

Bakers yeast is a useful enzyme which can be used to catalyse the decomposition of hydrogen
peroxide.

H2O 2 o H2O + O 2

You are provided with

Set-up apparatus:
x 1 x retort stand with clamp
x 1 x 100 cm3 conical flask
x 1 x L-shape glass tube connector fitted with a rubber bung
x 1 x rubber tubing connection
x 1 x gas syringe (possible capacities of 10, 20 or 100 cm3)

Reagents and apparatus


x 100 cm3 of yeast suspension
x 100 cm3 of 3% (by weight) hydrogen peroxide
x Distilled water
x Stopwatch
x All other common laboratory equipment

When 8.0 cm3 of the yeast suspension, 4.0 cm3 of H 2 O 2 and 18.0 cm3 of distilled water is mixed,
10 cm3 of oxygen gas was produced in 90 s.

Use the above information and the reagents provided to design an experiment to

x prove first order reaction with respect to yeast by a graphical method


x prove first order reaction with respect to hydrogen peroxide by a graphical method

In your experiment, you should perform 4 other experimental runs (including the above run) to
measure the volume of oxygen gas produced at regular intervals. The volume of yeast suspension
used should be varied in all the runs while keeping volume of H 2 O 2 constant at 4.0 cm3 .

In your experiment, illustrate / describe the following:

x A labelled set-up for your experiment using the set-up apparatus provided
x Table of volumes that you will be using in your experiment
x Calculations to determine the maximum volume of oxygen gas produced in each
experimental run. In your calculations, assume that the density of H 2 O 2 is 1.00 g cm3 and
conditions are at r.t.p.
x Procedure for measuring the volume of oxygen gas produced at regular intervals and use
the results to find the initial rate for each run graphically
x Graphical analysis involving initial rates to deduce order with respect to yeast
x Graphical analysis involving half-lives to deduce order with respect to H 2 O 2 based on one
selected run

H2 Chemistry 9647/02 NYJC J2/16 PX


For
3 Examiner's
Use
Experimental setup:

....................................................................................................................................

....................................................................................................................................

....................................................................................................................................

....................................................................................................................................

....................................................................................................................................

....................................................................................................................................

....................................................................................................................................

....................................................................................................................................

....................................................................................................................................

....................................................................................................................................

....................................................................................................................................

....................................................................................................................................

....................................................................................................................................

H2 Chemistry 9647/02 NYJC J2/16 PX [Turn over


For
4 Examiner's
Use
....................................................................................................................................

....................................................................................................................................

....................................................................................................................................

....................................................................................................................................

....................................................................................................................................

....................................................................................................................................

....................................................................................................................................

....................................................................................................................................

....................................................................................................................................

....................................................................................................................................

....................................................................................................................................

....................................................................................................................................

....................................................................................................................................

....................................................................................................................................

....................................................................................................................................

....................................................................................................................................

....................................................................................................................................

....................................................................................................................................

....................................................................................................................................

....................................................................................................................................

....................................................................................................................................

....................................................................................................................................

....................................................................................................................................

....................................................................................................................................

....................................................................................................................................

....................................................................................................................................

....................................................................................................................................

[Total: 12]

H2 Chemistry 9647/02 NYJC J2/16 PX


For
5 Examiner's
Use
BLANK PAGE

H2 Chemistry 9647/02 NYJC J2/16 PX [Turn over


For
6 Examiner's
Use
2 A solder is an alloy of metals which is used to join other metal pieces together. A specialist
solder that can be used to join together pieces of aluminium is made from a mixture by
mass of 65% zinc, 20% aluminium and 15% copper.

An H[SHULPHQWDO SURFHGXUH FDQ FRQUP WKH FRPSRVLWLRQ RI D SRZGHUHG VDPSOH RI WKLV
solder, by adding reagents and then extracting from the mixture each of the following in
sequence;
(i) copper metal,
(ii) aluminium as aluminium hydroxide,
(iii) zinc as zinc hydroxide.

You are provided with


DVDPSOHRIWKLVVROGHUZLWKDSSUR[LPDWHPDVVJ
PROGP3 sulfuric acid,
PROGP3 ammonia.

No other reagents should be used. Standard laboratory equipment is available.

(a) &RPSOHWHWKHRZFKDUWEHORZWR show the order in which the reagents would be added to
the solder to allow you to extract and separate the components as copper metal, aluminium
hydroxide and zinc hydroxide in Step 1, Step 2 and Step 3 respectively.

Step 1 Step 2 Step 3


reagent(s) added reagent(s) added reagent(s) added

substance(s) present substance(s) present substance(s) present


in solution in solution in solution

substance(s) removed substance(s) removed substance(s) removed


by filtration (if any) by filtration (if any) by filtration (if any)

[3]

H2 Chemistry 9647/02 NYJC J2/16 PX


For
7 Examiner's
Use
(b) (i) For some of the steps in the procedure you would need to be careful to add an
appropriate quantity of a reagent.
For each step of your procedure, explain why particular quantities of reagent should
be chosen.

Step 1 ................................................................................................................

..........................................................................................................................

..........................................................................................................................

Step 2 ................................................................................................................

..........................................................................................................................

..........................................................................................................................

Step 3 ................................................................................................................

..........................................................................................................................

...................................................................................................................... [4]

(ii) Write the ionic equations for the reactions taking place in Step 2.

..........................................................................................................................

...................................................................................................................... [2]

(iii) State the observations for the reactions taking place in Step 1 and Step 3.

Step 1 ................................................................................................................

Step 3 ............................................................................................................ [2]

(c) $OXPLQLXPK\GUR[LGHDQG]LQFK\GUR[LGHWKDWKDYHEHHQH[WUDFWHGDUHGLIFXOWWRGU\VRLWLV
better to convert them to their oxides.
Describe how this could be done and how you would ensure that each hydroxide has been
completely converted into its oxide.

....................................................................................................................................

....................................................................................................................................

....................................................................................................................................

....................................................................................................................................

....................................................................................................................................

....................................................................................................................................

....................................................................................................................................

....................................................................................................................................

................................................................................................................................ [2]
H2 Chemistry 9647/02 NYJC J2/16 PX [Turn over
For
8 Examiner's
Use
(d) If the mass of aluminium oxide obtained was 1.50 g, calculate the mass of aluminium that
was present in the solder. [1]

(e) (YHQ LI WKH H[SHULPHQWDO GLIFXOWLHV RI H[WUDFWLQJ DOO RI WKH FRSSHU IURP WKH PL[WXUH ZHUH
RYHUFRPHLWZRXOGEHGLIFXOWWRREWDLQDQDFFXUDWHPDVVRIFRSSHUIURPWKLVH[SHULPHQW
Suggest why this is so.

....................................................................................................................................

....................................................................................................................................

................................................................................................................................ [1]

[Total: 15]

H2 Chemistry 9647/02 NYJC J2/16 PX


For
9 Examiner's
Use
3 The relationship pV=nRT can be derived from the laws of mechanics by assuming ideal
behaviour for gases.

(a) The graph below represents the relationship between pV and p for a real gas at three
different temperatures, T 1 , T 2 and T 3 .

pV T1 T2

T3

p
(i) Draw one line on the graph to show what the relationship should be for the same
amount of an ideal gas. [1]

(ii) With reference to the graph, state and explain which temperature, T 1 , T 2 or T 3 is the
lowest temperature.

.........................................................................................................................

.........................................................................................................................

.........................................................................................................................

.........................................................................................................................

..................................................................................................................... [2]

(b) A flask with a volume of 100 cm3 was first weighed with air filling the flask, and then with
another gas Y, filling the flask. The results, measured at 26 oC and 1.00 x 105 Pa are shown.

Mass of flask containing air = 47.930 g


Mass of flask containing Y = 47.989 g
Density of air = 0.00118 g cm3

Calculate the relative molecular mass, M r , of Y. [3]

[Total: 6]

H2 Chemistry 9647/02 NYJC J2/16 PX [Turn over


For
10 Examiner's
Use
4 PbCl 2 is a sparingly soluble salt.
PbCl 2 (s) Pb2+(aq) + 2Cl(aq) 'H > 0
A student investigated the solubility product and solubility of PbCl 2 under different
conditions. He first added 0.0100 g of solid PbCl 2 into 120 cm3 of an unknown
concentration of HCl(aq) at 25 oC and subsequently made four changes at t 1 , t 2 , t 3 and t 4 .

The graphs below show the mass of PbCl 2 dissolved and solubility product of PbCl 2
against
time.

Mass of PbCl2
dissolved

0 t1 t2 t3 t4
time

Ksp

1.70 105

0 t1 t2 t3 t4
time

H2 Chemistry 9647/02 NYJC J2/16 PX


For
11 Examiner's
Use
(a) Given that 0.00465 g of PbCl 2 remained undissolved after addition of 0.0100 g of PbCl 2
into HCl, deduce the unknown concentration of the HCl. [3]

(b) Determine the mass of PbCl 2 that can dissolve in 500 cm3 of water at 25 oC. [2]

H2 Chemistry 9647/02 NYJC J2/16 PX [Turn over


For
12 Examiner's
Use
(c) Some possible changes listed below were made by the student at t 1 , t 2 and t 3 .
1. Cool reaction mixture to 10 oC in water bath
2. Heat reaction mixture to 50 oC in water bath
3. Addition of AgNO 3 (aq)
4. Addition of PbCl 2 (s)
5. Addition of water

With reference to both graphs, suggest and explain the changes made at t 1 , t 2 and t 3 .

change made at t 1 ..........................................................................................................

explanation ...................................................................................................................

....................................................................................................................................

....................................................................................................................................

....................................................................................................................................

....................................................................................................................................

change made at t 2 .........................................................................................................

explanation ...................................................................................................................

....................................................................................................................................

....................................................................................................................................

....................................................................................................................................

....................................................................................................................................

change made at t 3 ..........................................................................................................

explanation ...................................................................................................................

....................................................................................................................................

....................................................................................................................................

....................................................................................................................................

................................................................................................................................[6]

(d) The student added concentrated HCl at t 4 . On both the graphs on Page 10, draw how the
two graphs would look like after t 4 . [2]

[Total: 13]

H2 Chemistry 9647/02 NYJC J2/16 PX


For
13 Examiner's
Use
6 Compound A (C 12 H 16 O 2 ) exhibits optical isomerism. It does not react with hot acidified
K 2 Cr 2 O 7 . 1 mole of A reacts with 1 mole of PCl 5 to form B. A reacts with cold dilute KMnO 4
to form C (C 12 H 18 O 4 ). Upon heating with acidified KMnO 4 , A gives D (C 9 H 10 O 4 ) and E. D
produces effervescence when aqueous Na 2 CO 3 is added. 1 mole of D reacts with 3 moles
of aqueous bromine. E gives F, a yellow precipitate with warm alkaline aqueous iodine.

Draw the structures of A, B, C, D, E and F.

A B

C D

E F

[Total: 6]

H2 Chemistry 9647/02 NYJC J2/16 PX [Turn over


For
14 Examiner's
Use
5 The Ritter reaction is an organic reaction used to convert a nitrile and a tertiary alcohol, in a
second order reaction, to form an amide using strong aqueous acid as the catalyst.

O R2
R2 + R3
 H (aq) C C
R1 CN R3 C OH
R1 N R4
R4
H
, ,
R1 R2 R3 and R4 are alkyl groups

The proposed mechanism is as follows:

R2 R2
R3 C OH  H C R3  H2O
+
step 1
R4 R4

H H
R2 R2 O R2
 H2O
R1 CN C R3 step 2 R1 C N C R3 step 3 R1 C N C R3
R4 R4 R4

step 4

H
O R2 O R2
R1 C N C R3  H+ step 5 R1 C N C R3
H R4 H R4

(a) Predict the structures of P, Q and T for each of the following Ritter reactions. [3]

+ O
H (aq)
+
N
H
P Q
R
(C 8 H 7 N) (C 6 H 12 O)

H2 Chemistry 9647/02 NYJC J2/16 PX


For
15 Examiner's
Use

+
HO CN H (aq)

(C 8 H 15 NO)

(b) Describe a simple chemical test to distinguish compound R and S, stating clearly the
observations.

Test .............................................................................................................................

Observation .................................................................................................................

....................................................................................................................................

................................................................................................................................ [2]

(c) Suggest the types of reaction occurring in steps 3 and 4.

Step 3 ...........................................................................................................................

Step 4 ....................................................................................................................... [2]

(d) Complete the mechanism of step 3 and 4 below with arrows, showing the lone pairs and
dipole charges. [3]

H O H H H
R2 O R2 O R2
R1 C N C R3 step 3 R1 C N C R3 step 4 R1 C N C R3
R4 R4 H R4

(e) Suggest which step of the proposed mechanism on Page 14 is the slowest step. Explain
your reasoning.

....................................................................................................................................

....................................................................................................................................

....................................................................................................................................

................................................................................................................................[1]

H2 Chemistry 9647/02 NYJC J2/16 PX [Turn over


For
16 Examiner's
Use
(f) Primary alcohols such as ethanol are not suitable reagents for the Ritter reaction.

(i) Draw a diagram to show the orbitals of the carbon in the carbocation in step 1 and
state the type of hybridisation involved. [1]

(ii) Suggest why tertiary alcohols are used in the Ritter reaction but not primary
alcohols.

.........................................................................................................................

.........................................................................................................................

.........................................................................................................................

.........................................................................................................................

.........................................................................................................................

..................................................................................................................... [2]

(iii) Suggest why phenylmethanol is a suitable reagent for the Ritter reaction even
though it is a primary alcohol.

OH

phenylmethanol

.........................................................................................................................

.........................................................................................................................

.........................................................................................................................

.........................................................................................................................

..................................................................................................................... [1]

H2 Chemistry 9647/02 NYJC J2/16 PX


For
17 Examiner's
Use
(g) This equation for the Ritter reaction is repeated from page 14.

O R2
R2 + R3
 H (aq) C C
R1 CN R3 C OH
R1 N R4
R4
H
, ,
R1 R2 R3 and R4 are alkyl groups

(i) Using suitable data from the Data Booklet, calculate the enthalpy change of reaction.
[2]

(ii) Suggest and explain if the Ritter reaction is spontaneous at high or low temperature.

.........................................................................................................................

.........................................................................................................................

.........................................................................................................................

.........................................................................................................................

.........................................................................................................................

.........................................................................................................................

.........................................................................................................................

.........................................................................................................................

..................................................................................................................... [3]

[Total: 20]

H2 Chemistry 9647/02 NYJC J2/16 PX [End[Turn over


of Paper]
H
NANYANG JUNIOR COLLEGE
JC 2 PRELIMINARY EXAMINATION
Higher 2

CHEMISTRY 9647/03
Paper 3 Free Response 21 September 2016
2 hours
Candidates answer Section A on the Question Paper

Additional Materials: Writing Paper


Data Booklet

READ THESE INSTRUCTIONS FIRST

Write your name and class on all the work you hand in.
Write in dark blue or black pen on both sides of the paper.
You may use a soft pencil for any diagrams, graphs or rough working.
Do not use staples, paper clips, highlighters, glue or correction fluid.

Answer any four questions.


A Data Booklet is provided.
You are reminded of the need for good English and clear presentation in your answers.

At the end of the examination, fasten all your work securely together.
The number of marks is given in brackets [ ] at the end of each question or part question.

This document consists of 13 printed pages and 1 blank page.

[Turn over
2
Answer any four questions.

1(a) Copper(I) sulfate, Cu 2 SO 4 , can be made from copper(I) oxide under non-aqueous
conditions. On adding this salt to water, it immediately undergoes a disproportionation
reaction.

(i) Suggest, with a reason, the colour of copper(I) sulfate. [2]

(ii) Using suitable data from the Data Booklet, explain why the disproportionation
reaction occurs, and write an equation for it. [3]

(b) Palladium(II) salts can form square planar complexes. Successive addition of ammonia and
hydrogen chloride to an aqueous palladium(II) salt produces, under different conditions,
three compounds with empirical formula PdN 2 H 6 Cl 2 . Two of these, A and B, are non-ionic,
with M r = 211. A has a dipole moment, whereas B has none. The third compound, C, is
ionic, having M r = 422, and contains palladium in both its cation and anion.

For each A, B and C, deduce a structure that fits the above data, explaining your reasons
fully. [6]

(c) Benzene ring is often represented as a structure that has a ring within the hexagon.
Alternatively, chemists have also represented the structure of benzene in the following
forms, known as resonance structures.

Benzene (two resonance forms)

The resonance relationship is indicated by the double headed arrow between them. The
only difference between resonance forms is the placement of the pi electrons and
non-bonding electrons.

Most aromatic compounds undergo electrophilic substitution. However aryl halides undergo
a limited number of substitution reactions with strong nucleophiles.

An example of a reaction is as follows:

Nu
A X A Nu + X

where A is an electron withdrawing group and X is a halogen

The mechanism of this reaction has two steps:


x addition of the nucleophile
x elimination of the halogen leaving group

H2 Chemistry 9647/03 NYJC J2/16 PX [Turn Over


3
Step 1 involves the addition of the nucleophile (Nu). The Nu attacks the carbon atom
bonded to a halogen, causing the pi bond to break. A resonance stabilised carbanion with a
QHZ&1XERQGLVIRUPHG7KHDURPDWLFULQJLVGHVWUR\HGLQWKLVVWHS

Nu
X X
Nu

A A

Step 2 involves the loss of the halogen X, reforming the aromatic ring.

Nu
X Nu
 X


A A

Two other resonance structures of the intermediate in Step 1 are shown below:

Nu Nu
X X
I II
  Z
A A

(i) Copy the above diagram and draw the resonance structure, Z. In your answer,
show any relevant charges, lone pairs of electrons and movement of electrons in
forming Z. [2]

The reaction below shows the synthesis of compound, D.

N(CH3)2 N(CH3)2
O 2N
O 2N
potassium metal +
+ HCl

Cl HO O

D
(ii) Suggest the role of potassium metal in the reaction. [1]

(iii) Use the information given above to draw out the full mechanism for the reaction that
forms D, labelling the slow and fast steps. In your answer, showing any relevant
charges, lone pair of electrons and movement of electrons. [3]

(d) Describe and explain the relative ease of hydrolysis of the following three
chloro-compounds. [3]
Cl COCl CH2Cl

[Total: 20]

H2 Chemistry 9647/03 NYJC J2/16 PX [Turn Over


4
2 Sulfuric acid, H 2 SO 4 , can behave as an acid, an oxidising agent or as a dehydrating agent
in various reactions.

(a) Draw a diagram to illustrate the shape of pure sulfuric acid and indicate the bond angle
about the sulfur atom. [2]

(b) The Contact Process is used for the manufacture of sulfuric acid. One of the reactions that
takes place is the following reversible reaction:

2SO 2 (g) + O 2 (g) 2SO 3 (g) 'H = 197 kJ mol1

Sulfur dioxide and oxygen in a 2:1 molar ratio at a total initial pressure of 3 atm is passed
over a catalyst in a fixed volume vessel at 400 oC. When equilibrium is established, the
percentage of sulfur trioxide in the mixture of gases is found to be 30%.

(i) Write an expression for the equilibrium constant, K p , of the reaction. [1]

(ii) Calculate the value of K p at 400 oC, stating its units. [3]

(iii) How would the percentage conversion of SO 2 into SO 3 be affected when the
pressure is raised? Explain. [2]

(c) Dilute sulfuric acid takes part in typical acid-base reactions and it can be used to distinguish
the following solids: MgO, BaO and SiO 2 .

State the observations, if any, to indicate the differences in their reaction when water is
added to each solid followed by dilute sulfuric acid. [4]

(d) Sulfur dioxide is a major pollutant from sulfuric acid plants. The SO 2 emitted into the
atmosphere is oxidised in the air, which then reacts with water to form sulfuric acid, hence
causing acid rain:

2SO 2 (g) + O 2 (g) + 2H 2 O(l) o 2H 2 SO 4 (l) 'H 1

Using the data below and data from (b), construct an energy cycle to calculate

(i) the enthalpy change of formation of SO 2 (g), and hence

(ii) the enthalpy change of reaction, 'H 1 for the above reaction.

Enthalpy change of formation of H 2 O(l) = 286 kJ mol1


Enthalpy change of formation of H 2 SO 4 (l) = 811 kJ mol1
Enthalpy change of formation of SO 3 (g) = 493 kJ mol1
[4]

H2 Chemistry 9647/03 NYJC J2/16 PX [Turn Over


5
(e) Alcohols react with concentrated sulfuric acid at high temperatures to form alkenes. A
common side reaction that can happen is the formation of ethers, which is also catalysed
by concentrated sulfuric acid.

R R R
conc H2SO4
2R C OH R C O C R + H2O
heat
R R R
R R R
conc H2SO4
2R C OH R C O C R + H2O
heat
R R R

The mechanism occurs via 3 steps:

Step 1:
An acid base reaction in which H+ from H 2 SO 4 protonates the oxygen atom in alcohol. This
step is very fast and reversible.

Step 2:
A second alcohol molecule functions as the nucleophile and attacks the product from
step 1. The CO bond is cleaved and a water molecule leaves the molecule. This creates
R
+
O
an oxonium ion R' H intermediate.

Step 3:
Another acid base reaction in which the proton in the oxonium ion is removed by a suitable
base (in this case a water molecule) to give the ether product. This step is very fast and
reversible.

(i) Draw the ether formed when cyclopentanol undergoes the above reaction.

OH

cyclopentanol
[1]

(ii) Draw out the full mechanism for the reaction between two cyclopentanol molecules
to form an ether. In your answer, show any relevant charges, lone pairs of electrons
and movement of electrons. [3]

[Total: 20]

H2 Chemistry 9647/03 NYJC J2/16 PX [Turn Over


6

H2 Chemistry 9647/03 NYJC J2/16 PX [Turn Over


7
3 Garlic contains many amino acids, minerals and enzymes. Garlic also contains at least 33
sulfur compounds like alliin, allicin and ajoene. The sulfur compounds are responsible for
both garlics pungent odour and many of its medical effects.

(a) When raw garlic is chopped or crushed, the enzyme allinase converts alliin into allicin,
which is responsible for the odour of fresh garlic.

O NH2 O

S allinase S
COOH S
alliin allicin

Allicin can also be biosynthesised by using serine and glutathione.

NH2 COOH O

HO NH
COOH H2N NH COOH

O
SH
serine glutathione

(i) Explain why raw chopped garlic has a stronger odour than when it is cooked. [1]

(ii) When one molecule of serine reacts with one molecule of glutathione, it is possible
to form two esters with different structural formulae.
Draw the structural formula of each of these esters. [2]

(iii) Draw the structural formulae of the products when glutathione is hydrolysed. [3]

Alliin has pK a values of 1.84 and 8.45.

(iv) Make use of these pK a values to suggest the major species present in solutions of
alliin with the following pH values. [3]
x pH 1
x pH 7
x pH 11

(v) Calculate the pH of 0.10 mol dm3 solution of alliin. [1]

(vi) With reference to the pK a values, identify the major species formed when 10 cm3 of
0.10 mol dm3 NaOH is added to 10 cm3 of 0.10 mol dm3 protonated alliin. Hence,
deduce whether the solution is acidic, neutral or alkaline. [2]

(vii) Sketch the pH-volume added curve you would expect to obtain when 30 cm3 of
0.10 mol dm3 NaOH is added to 10 cm3 of 0.10 mol dm3 protonated alliin.
Briefly describe how you have calculated the various key points on the curve. [4]

H2 Chemistry 9647/03 NYJC J2/16 PX [Turn Over


8
(b) Diallyl disulfide is one of the principal components of the distilled oil of garlic. It is a
yellowish liquid which is insoluble in water and has a strong garlic odour. It is produced
during the decomposition of allicin.

S S
S S

allicin diallyl disulfide


Diallyl disulfide can be produced industrially from sodium disulfide and allyl chloride at
temperatures of 40 60 C in an inert atmosphere.

40  60 oC
Cl S
Na2S2 + 2 S + 2NaCl
inert gas
allyl chloride

(i) Give the IUPAC name of allyl chloride. [1]

(ii) Explain, in thermodynamic terms, suggest why diallyl disulfide is insoluble in water.
[2]

(iii) State the type of reaction when diallyl disulfide is converted back to allicin. [1]

[Total: 20]

H2 Chemistry 9647/03 NYJC J2/16 PX [Turn Over


9
4 Alkynes are organic molecules which contain carbon-carbon triple bonds and are part of the
homologous series with formula of C n H Q i.e.

R1 C C R2

where R 1 and R 2 = H or alkyl or aryl groups

Alkynes exhibit similar chemical properties to alkenes.


e.g. addition reactions with electrophiles i.e. X 2 or HX to form alkenes

R1 R2
HX
R1 C C R2 C C

H X

e.g. oxidation by hot concentrated KMnO 4 to form mixture of carboxylic acids

KMnO 4
R1 C C R2 R1COOH + R2COOH

However, unlike alkenes, terminal alkynes are able to react with strong bases like sodium
amide.
 +
RC CH + NaNH2 RC C Na + NH3

(a) Ethyne, C 2 H 2 , is heated with excess sodium bromide and concentrated sulfuric acid to
produce a dihalide, C 2 H 4 Br 2 . The overall reaction may be considered to take place in two
stages, the first between inorganic reagents only and the second involving the organic
reagent.

(i) Write an equation for the first stage. [1]

(ii) Suggest a structure for the dihalide formed. [1]

(iii) When the concentrated sulfuric acid is added to the reaction mixture, cooling is
necessary to prevent the formation of inorganic by-products.
Write an equation to explain the formation of these inorganic by-products. [1]

(b) Compound A, is an enyne chloride (i.e. compounds that contains chloro, alkyne and alkene
functional groups).

Cl

Compound A

One mole of compound A reacts with two moles of Br 2 to produce a mixture of


4 stereoisomers. Draw structures of the stereoisomers formed. [3]

H2 Chemistry 9647/03 NYJC J2/16 PX [Turn Over


10
(c) Compound B, which is an isomer of Compound A and also an enyne chloride, is treated
with sodium amide, NaNH 2 followed by heating under reflux to form compound C, C 8 H 10 .
Compound C reacts with hot concentrated KMnO 4 to produce butane-1,4-dioic acid only.

(i) Explain the reaction with NaNH 2 . [1]

(ii) Hence, explain the formation of compound C. [1]

(iii) Suggest skeletal structures for compounds B and C. [2]

When a current of 1.0 A was passed through aqueous potassium maleate (KO 2 CCH=CHCO 2 K) for
15 minutes, it was found that 110 cm3 H 2 , measured at r.t.p, was collected at the cathode. The
following reaction took place.
2H 2 O + 2e o H 2 + 2OH
(d) State the relationship between the Faraday constant, F and the Avogadros constant, L. [1]

(e) Using the data above and the Data Booklet, calculate a value for Avogadros constant. [3]

(f) Ethyne and CO 2 gas were produced at the anode. In order to determine the stoichiometry
of the anode reaction, the volume of the gases collected at the anode was measured. The
anode gas was first passed through aqueous NaOH before being collected in a gas
syringe. The following data was collected:

x mass of bottle containing NaOH before experiment = 10.501 g


x mass of bottle containing NaOH after experiment = 10.904 g
x initial reading on syringe = 10.0 cm3
x final reading on syringe = 120.0 cm3

(i) State the oxidation state of carbon in ethyne. [1]

(ii) With the help of an equation, explain the purpose of passing the anode gas through
NaOH. [1]

(iii) Calculate the volume of CO 2 produced, assuming r.t.p conditions. [1]

(iv) Hence, suggest an ionic equation for the reaction that occurred at the anode. [1]

(g) When aqueous potassium maleate was acidified, maleic acid, HO 2 CCH=CHCO 2 H
(pK a1 = 1.90 and pK a2 = 6.07) was liberated. Fumaric acid (pK a1 = 3.03 and pK a2 = 4.44) is
a stereoisomer of maleic acid.
With a suitable illustration, suggest a reason why maleic acid has a lower pK a1 but higher
pK a2 than fumaric acid.
[2]

[Total: 20]

H2 Chemistry 9647/03 NYJC J2/16 PX [Turn Over


11
5(a) Dopamine is an organic compound of the catecholamine and phenethylamine families that
plays several important roles in the brain and body. Its name is derived from its chemical
structure: it is an amine synthesised by removing a carboxyl group from a molecule of
its precursor compound, L-DOPA.

Below is a synthetic route involving L-DOPA and dopamine:

O
HO HO NH2
OH
+ CO 2
NH2
HO HO
L-DOPA Dopamine

Reaction I Reaction III


Compound H

O
Product J
HO
Cl
HO N
NH2
HO
HO
Reaction II
anhydrous AlCl 3

Isomers F and G
C9H9O 3N

(i) State the reagents and conditions and any observations in Reaction I. [1]

(ii) Aluminium chloride is used as a catalyst in electrophilic substitution reactions. The


chlorination of benzene is represented by the following overall equation.

Cl
+ Cl 2 + HCl

The reaction occurs in several steps.


x The first step is the reaction between Cl 2 and AlCl 3 .
Cl 2 + AICl 3 Cl+ + AICI 4 
+
x The benzene is then attacked by the Cl cation in the second step.

The first step is the reaction between Cl 2 and AICI 3 .


Cl 2 + AICl 3 Cl+ + AICI 4 
+
The benzene ring is then attacked by the CI cation in the second step.
AICl 3 reacts in a similar way with acyl chlorides, producing a carbocation
that can then attack a benzene ring.

Predict the structures of isomers F and G in Reaction II. [2]


H2 Chemistry 9647/03 NYJC J2/16 PX [Turn Over
12

(iii) In Reaction III, dopamine was reacted with alkyl halide H to give the final product J.
Draw the displayed formula of H. [2]

H2 Chemistry 9647/03 NYJC J2/16 PX [Turn Over


13
(b) Dopamine is a bidentate ligand. When different volumes of 0.0030 mol dm3 of aqueous
Cr(III) and 0.0020 mol dm3 of alkaline dopamine solution were mixed, a complex R is
formed. Analysis of R shows that its formula is [Cr(C 8 H 9 NO 2 ) x (H 2 O) y ]z, where x, y and z
are integers.
To determine the stoichiometry of the complex ion formed, the colour intensities of these
different mixtures were measured using a colorimeter. The following absorption spectrum
was obtained.
Relative Absorbance

Volume of Cr3+ / cm3

10 9 8 7 6 5 4 3 2 1
Volume of alkaline dopamine / cm3

(i) Use the graph and the information given to determine the formula of complex R.
Show your workings clearly. [3]

(ii) The crystal field describes the breaking of orbital degeneracy in transition metal
complexes due to the presence of ligands. When the d-orbitals split into high energy
and low energy orbitals, the difference in enerJ\RIWKHWZROHYHOVLVGHQRWHGDV o .
7KH UHODWLRQVKLS EHWZHHQ o and colours of complexes can be described in the
equation below:
hc
'o =
O
where h is Plancks constant, c is the speed of light and O is the wavelength of light
absorbed
colour absorbed O / nm
violet 410
indigo 430
blue 480
blue-green 500
green 530
yellow 580
orange 610
red 680
*LYHQWKDW o for complex R is 4.125 x 1022 kJ and using relevant data from the Data
Booklet, calculate the wavelength of light. Deduce the colour of complex R. [2]
H2 Chemistry 9647/03 NYJC J2/16 PX [Turn Over
14
(c) Iodine is not very soluble in water, it is freely soluble in KI(aq), according to the following
equilibrium:
I(aq) + I 2 (s) I 3 (aq)

(i) Draw a fully labelled experimental set-up for a voltaic cell made up of a
Cr 2 O 7 2/ Cr3+ half-cell and a I 2 /I half-cell under standard conditions. Indicate clearly
the anode and cathode and show the flow of electrons. [3]

(ii) By using appropriate values from the Data Booklet, predict what, if anything, will
happen when a small amount of acidified vanadium(II) chloride is added to a
solution of I 2 (aq). the I 2 /I half-cell.
[3]

(d) Explain the following statements.

(i) BrF 3 is a covalent compound which exhibits electrical conductivity in liquid state
at room temperature. With the aid of an equation, suggest an explanation for its
electrical conductivity. [2]

(ii) SiCl 4 reacts violently in water but CCl 4 has no reaction with water. [1]

(iii) Compounds NeF 2 and NeF 4 do not exist but XeF 2 and XeF 4 exist.
[1]

[Total: 20]

H2 Chemistry 9647/03 NYJC J2/16 PX [Turn Over

[End of Paper]
NYJC 2016 Prelim H2 Chemistry Paper 1 Answers
1 D 6 A 11 D 16 B 21 A 26 A 31 B 36 B
2 C 7 D 12 C 17 C 22 B 27 D 32 B 37 D
3 B 8 B 13 A 18 D 23 D 28 A 33 B 38 A
4 B 9 D 14 A 19 C 24 C 29 C 34 D 39 C
5 A 10 B 15 C 20 C 25 B 30 C 35 A 40 C

trendyline
H
NANYANG JUNIOR COLLEGE
JC 2 PRELIMINARY EXAMINATION
Higher 2

CANDIDATE
NAME Teachers Mark Scheme
INDEX
CLASS 1 5 NO.
TUTOR

CHEMISTRY 9647/02
Paper 2 Structured 14 September 2016
2 hours
Candidates answer on the Question Paper

Additional Materials: Data Booklet

READ THESE INSTRUCTIONS FIRST


Write your name and class on all the work you hand in.
Write in dark blue or black pen on both sides of the paper.
You may use a soft pencil for any diagrams, graphs or rough working.
Do not use staples, paper clips, highlighters, glue or correction fluid.

Answer all questions in the spaces provided.


A Data Booklet is provided.

At the end of the examination, fasten all your work securely together.
The number of marks is given in brackets [ ] at the end of each question or part question.

For Examiners Use

1 P /12
2 / 15
3 / 6
4 / 13
5 / 6
6 / 20 /60

trendyline
y
This document consists of 17 printed pages and 1 blank page.

H2 Chemistry 9647/02 NYJC J2/16 PX [Turn over


For
2 Examiner's
Use
Answer all questions in the spaces provided.

1 Planning (P)

Bakers yeast is a useful enzyme which can be used to catalyse the decomposition of hydrogen
peroxide.

H2O 2 o H2O + O 2

You are provided with

Set-up apparatus:
x 1 x retort stand with clamp
x 1 x 100 cm3 conical flask
x 1 x L-shape glass tube connector fitted with a rubber bung
x 1 x rubber tubing connection
x 1 x gas syringe (possible capacities of 10, 20 or 100 cm3)

Reagents and apparatus


x 100 cm3 of yeast suspension
x 100 cm3 of 3% (by weight) hydrogen peroxide
x Distilled water
x Stopwatch
x All other common laboratory equipment

When 8.0 cm3 of the yeast suspension, 4.0 cm3 of H 2 O 2 and 18.0 cm3 of distilled water is mixed,
10 cm3 of oxygen gas was produced in 90 s.

Use the above information and the reagents provided to design an experiment to

x prove first order reaction with respect to yeast by a graphical method


x prove first order reaction with respect to hydrogen peroxide by a graphical method

In your experiment, you should perform 4 other experimental runs to measure the volume of
oxygen gas produced at regular intervals. The volume of yeast suspension used should be varied
in all the runs while keeping volume of H 2 O 2 constant at 4.0 cm3 .

In your experiment, illustrate / describe the following:

x A labelled set-up for your experiment using the set-up apparatus provided
x Table of volumes that you will be using in your experiment
x Calculations to determine the maximum volume of oxygen gas produced in each
experimental run. In your calculations, assume that the density of H 2 O 2 is 1.00 g cm3 and
conditions are at r.t.p.
x Procedure for measuring the volume of oxygen gas produced at regular intervals and use
the results to find the initial rate for each run graphically
x Graphical analysis involving initial rates to deduce order with respect to yeast
x Graphical analysis involving half-lives to deduce order with respect to H 2 O 2 based on one
selected run

trendyline
H2 Chemistry 9647/02 NYJC J2/16 PX
For
3 Examiner's
Use
Experimental setup:

1 mark 3 apparatus drawn and labelled correctly


1 mark all apparatus drawn, labelled and connected correctly
Table of volumes

Run Vol of yeast /cm3 Vol of H 2 O 2 /cm3 Vol of water /cm3 Total volume /cm3
1 4.0 4.00 22.0 30.0
2 6.0 4.00 20.0 30.0
3 8.0 4.00 18.0 30.0
4 10.0 4.00 16.0 30.0
5 12.0 4.00 14.0 30.0

1 mark volume of yeast is between 2 20 cm3, volume of H 2 O 2 is constant


1 mark volume of water added accordingly to keep total volume constant

Calculations to determine the maximum volume of oxygen gas produced in each


experimental run
Mass of 4.00 cm3 of H 2 O 2 = 1.00 x 4.00 = 4.00 g
Moles of H 2 O 2 = (4.00 x 0.03) / 34.0 = 0.00353 mol [1]
Moles of O 2 gas = (0.00353) = 0.00176 mol
Volume of O 2 gas = 0.00176 x 24000 = 42.4 cm3 [1]
Hence minimum size of syringe to be used is the 100 cm3 size if the reaction is allowed to
go to completion

trendyline
H2 Chemistry 9647/02 NYJC J2/16 PX [Turn over
For
4 Examiner's
Use

Procedure for measuring the volume of oxygen gas produced at fixed intervals
1. Record the initial reading on the syringe
2. Using a burette, measure 2.0 cm3 of the yeast suspension and add into the conical flask
3. Using a measuring cylinder, measure 24.0 cm3 of the distilled water and add into the
conical flask.
4. Using a burette, measure 4.00 cm3 of the H 2 O 2 into a small beaker.
5. Pour the H 2 O 2 into the conical flask, stopper the conical flask with the conical flask and
start the stop watch immediately. Swirl the conical flask a few times
6. Record the reading on the syringe every 10 seconds (or other regular time intervals, until 5
readings are collected i.e. 10 cm3 is produced.
7. Repeat steps 1 9 for the next 4 experimental runs, changing the volumes according to the
table given. For run 5 (or any other run), make sure to use the 100 cm3 syringe and allow
the reaction to go to completion.

1 mark 3/7 steps correctly written with appropriate apparatus used


1 mark all steps correctly written with appropriate apparatus used

Graphical analysis using the initial rates to deduce order with respect to yeast
For the five set of readings, plot the volume of O 2 produced against time. The tangent at
time = 0 is the initial rate (see graph below)

Vol of O2 Tangent = initial rate

Time

1 mark
Using the initial rates obtained, plot a graph of initial rates against volume of yeast used. If
order is 1, a straight line through the origin should be obtained.

rate

Volume

trendyline
of yeast

1 mark

H2 Chemistry 9647/02 NYJC J2/16 PX


For
5 Examiner's
Use

BLANK PAGE

Graphical analysis using a half-life method to deduce order with respect to H 2 O 2


using the selected run
Using the graph for run 5, find 2 consecutive half-lives. If order is one, the half-lives should
be constant i.e. t 1 = t 2 . [1]

Vol of
O2
42
(42)
(42)

Time

t1 t2
1 mark correct graph. X accepted instead of 42 but the graph must plateau.

[Total: 12]

H2 Chemistry 9647/02 NYJC J2/16 PX [Turn over


For
6 Examiner's
Use
2 A solder is an alloy of metals which is used to join other metal pieces together. A specialist
solder that can be used to join together pieces of aluminium is made from a mixture by
mass of 65% zinc, 20% aluminium and 15% copper.

An experimHQWDO SURFHGXUH FDQ FRQUP WKH FRPSRVLWLRQ RI D SRZGHUHG VDPSOH RI WKLV
solder, by adding reagents and then extracting from the mixture each of the following in
sequence;
(i) copper metal,
(ii) aluminium as aluminium hydroxide,
(iii) zinc as zinc hydroxide.

You are provided with


DVDPSOHRIWKLVVROGHUZLWKDSSUR[LPDWHPDVVJ
PROGP3 sulfuric acid,
PROGP3 ammonia.

No other reagents should be used. Standard laboratory equipment is available.

(a) &RPSOHWHWKHRZFKDUWEHORZWRVKRZWKe order in which the reagents would be added to


the solder to allow you to extract and separate the components as copper metal, aluminium
hydroxide and zinc hydroxide in Step 1, Step 2 and Step 3 respectively.

Step 1 Step 2 Step 3


reagent(s) added reagent(s) added reagent(s) added

H 2 SO 4 NH 3 H 2 SO 4

substance(s) present substance(s) present substance(s) present


in solution in solution in solution

ZnSO 4 Zn(NH 3 ) 4 2+ (NH 4 ) 2 SO 4 blank ok

Al 2 (SO 4 ) 3 (NH 4 ) 2 SO 4 blank ok

substance(s) removed substance(s) removed substance(s) removed


by filtration (if any) by filtration (if any) by filtration (if any)

Cu Al(OH) 3 Zn(OH) 2

[3]

6 points 3 mks.
trendyline
H2 Chemistry 9647/02 NYJC J2/16 PX
For
7 Examiner's
Use

(b) (i) For some of the steps in the procedure you would need to be careful to add an
appropriate quantity of a reagent.
For each step of your procedure, explain why particular quantities of reagent should
be chosen.

Step 1 excess H 2 SO 4 to completely dissolve Zn and Al [1] ..................................

..........................................................................................................................

..........................................................................................................................

Step 2 excess NH 3 to ppt Al(OH) 3 and Zn(OH) 2 and subsequently form Zn(NH 3 ) 4 2+

/dissolve Zn(OH) 2 [1] ...........................................................................................

..........................................................................................................................

Step 3 sufficient/enough H 2 SO 4 [1] neutralise/react with NH 3 [1] so that Zn(OH) 2


is .......................................................................................................................

re-ppt but not excess as that Zn(OH) 2 reacts/dissolves ..........................................

...................................................................................................................... [4]

(ii) Write the ionic equations for the reactions taking place in Step 2.

Al3+ + 3OH- Al(OH) 3 [1]...........................................................................................

[Zn(H 2 O) 6 ]2+ + 4NH 3 Zn(NH 3 ) 4 2+ [1] ................................................................... [2]

(iii) State the observations for the reactions taking place in Step 1 and Step 3.

Step 1 Pink solid in colourless solution. Effervescence produced. Gas evolved


extinguished lighted splint with a pop sound. ......................................................

Step 3 White ppt re-ppt/re-forms............................................................................ [2]

(c) $OXPLQLXPK\GUR[LGHDQG]LQFK\GUR[LGHWKDWKDYHEHHQH[WUDFWHGDUHGLIFXOWWRGU\VRLWLV
better to convert them to their oxides.
Describe how this could be done and how you would ensure that each hydroxide has been
completely converted into its oxide.

x Weigh the hydroxides and heat [1]


x Cool and re-weigh
x Repeat heating and cooling until constant mass is achieved [1]

....................................................................................................................................

....................................................................................................................................

....................................................................................................................................

trendyline
.....................................................
....................................................
....................................................................................................................................

................................................................................................................................ [2]

H2 Chemistry 9647/02 NYJC J2/16 PX [Turn over


For
8 Examiner's
Use
(d) If the mass of aluminium oxide obtained was 1.50 g, calculate the mass of aluminium that
was present in the solder. [1]

2Al Al 2 O 3
n(Al 2 O 3 ) = 1.50/102.0 = 0.0147 mol
n(Al) = 2(0.0147) = 0.0294 mol
m(Al) = 0.0294 x 27.0 = 0.794 g [1]

(e) (YHQ LI WKH H[SHULPHQWDO GLIFXOWLHV RI H[WUDFWLQJ DOO RI WKH FRSSHU IURP WKH PL[WXUH ZHUH
RYHUFRPHLWZRXOGEHGLIFXOWWRREWDLQDQDFFXUDWHPDVVRIFRSSHUIURPWKLVH[SHULPHQW
Suggest why this is so.

Mass/amount/percentage of copper is small ..........................................................................

....................................................................................................................................

................................................................................................................................ [1]

[Total: 15]

trendyline
H2 Chemistry 9647/02 NYJC J2/16 PX
For
9 Examiner's
Use
3 The relationship pV=nRT can be derived from the laws of mechanics by assuming ideal
behaviour for gases.

(a) The graph below represents the relationship between pV and p for a real gas at three
different temperatures, T 1 , T 2 and T 3 .

pV T1 T2

T3

[1]

p
(i) Draw one line on the graph to show what the relationship should be for the same
amount of an ideal gas. [1]

(ii) With reference to the graph, state and explain which temperature, T 1 , T 2 or T 3 is the
lowest temperature.

T 1 (because it shows greatest deviation) [1]...............................................................

With decrease in temperature, KE of particles is reduced, and intermolecular ...........

forces of attraction become more significant, and the gas behaves less ideally [1] ....

.........................................................................................................................

..................................................................................................................... [2]

(b) A flask with a volume of 100 cm3 was first weighed with air filling the flask, and then with
another gas Y, filling the flask. The results, measured at 26 oC and 1.00 x 105 Pa are shown.

Mass of flask containing air = 47.930 g


Mass of flask containing Y = 47.989 g
Density of air = 0.00118 g cm3

Calculate the relative molecular mass, M r , of Y. [3]

Mass of air = 100 x 0.00118 = 0.118 g


Mass of flask = 47.930 0.118 = 47.812 g
Mass of Y = 47.989 47.812 = 0.177 g [1]

m
pV nRT RT
Mr
m 0.177 u 8.31u 299
Mr RT [1]correct working
pV 1u 105 u 100 u 10 6

trendyline
44.0 [1]correct answer

[Total: 6]

H2 Chemistry 9647/02 NYJC J2/16 PX [Turn over


For
10 Examiner's
Use
4 PbCl 2 is a sparingly soluble salt.
PbCl 2 (s) Pb2+(aq) + 2Cl(aq) 'H > 0
A student investigated the solubility product and solubility of PbCl 2 under different
conditions. He first added 0.0100 g of solid PbCl 2 into 120 cm3 of an unknown
concentration of HCl(aq) at 25 oC and subsequently made four changes at t 1 , t 2 , t 3 and t 4 .

The graphs below show the mass of PbCl 2 dissolved and solubility product of PbCl 2
against
time.

Mass of PbCl2
dissolved [1]

0 t1 t2 t3 t4
time

Ksp
[1]

1.70 105

0 t1 t2 t3 t4
time

trendyline
H2 Chemistry 9647/02 NYJC J2/16 PX
For
11 Examiner's
Use
(a) Given that 0.00465 g of PbCl 2 remained undissolved after addition of 0.0100 g of PbCl 2
into HCl, deduce the unknown concentration of the HCl. [3]

Mass of PbCl 2 dissolved = 0.0100 0.00465 = 0.00535 g [1]

Solubility of PbCl 2 in HCl(aq), x = = 1.604 104 mol dm3 [1]

Let x be the unknown concentration of HCl(aq),


PbCl 2 (s) Pb2+(aq) + 2Cl(aq)
3
4
1.604 10 mol dm 2(1.604 104) mol dm3

HCl(aq) o H+(aq) + Cl(aq)


x mol dm3 x mol dm3 x mol dm3

At saturation, ionic product = K sp


K sp = [Pb2+][Cl]2
1.70 10 = (1.604 104)(2 1.604 104 + x)2
5

x = 0.3252
= 0.325 mol dm3 [1]

Alternative:
Assuming 2(1.604 104) is small,
Ksp = [Pb2+][Cl]2
1.70 105 = (1.604 104)(x)2
x = 0.3256
= 0.326 mol dm3

(b) Determine the mass of PbCl 2 that can dissolve in 500 cm3 of water at 25 oC. [2]

Let m be the mass of PbCl 2 dissolve in 500 cm3 of water.

PbCl 2 (s) Pb2+(aq) + 2Cl(aq)

Solubility of PbCl 2 in water = [1]

K sp remain unchanged at 25 oC.


K spp = [Pb2+][Cl]2
4( )3 = 1.70 105 mol2 dm6
m = 2.252 g = 2.25 g [1]

trendyline
H2 Chemistry 9647/02 NYJC J2/16 PX [Turn over
For
12 Examiner's
Use
(c) Some possible changes listed below were made by the student at t 1 , t 2 and t 3 .
1. Cool reaction mixture to 10 oC in water bath
2. Heat reaction mixture to 50 oC in water bath
3. Addition of AgNO 3 (aq)
4. Addition of PbCl 2 (s)
5. Addition of water

With reference to both graphs, suggest and explain the changes made at t 1 , t 2 and t 3 .
Addition of water or Addition of AgNO (aq) [1] 3
change made at t 1 ..........................................................................................................

explanation ...................................................................................................................

total volume increase when water is added to reaction mixture, hence [Pb2+] and [Cl]
....................................................................................................................................
The
decrease. According to Le Chateliers Principle, position of equilibrium shifts right to increase
the [Pb2+] and [Cl]. Thus, more PbCl2 dissolve. Ksp remain constant as Ksp is temperature
....................................................................................................................................
dependent [1]
or Addition of AgNO3(aq), presence of Ag+ caused precipitation of AgCl(s), hence [Cl]
....................................................................................................................................
decrease. According to Le Chateliers Principle, position of equilibrium shifts right to increase

[Cl ]. Thus, more PbCl2 dissolve and Ksp remain constant as Ksp is temperature dependent.
....................................................................................................................................

change made at t 2 Addition of PbCl 2 (s) [1] .............................................................................

explanation The solution is already saturated. Hence, as more PbCl 2 (s) is added, the mass

of PbCl 2 dissolve will not increase. K sp remain constant as K sp is temperature dependent. [1]

....................................................................................................................................

....................................................................................................................................

....................................................................................................................................

change made at t 3 Heat reaction mixture to 50 oC in water bath [1] ..............................

explanation According to Le Chateliers Principle, as temperature increase, the position of...

equilibrium shifts right to favour the forward endothermic reaction, to absorb heat. ...............

Hence, [Pb2+] and [Cl] increase. The mass of CaSO 4 dissolved and K sp increase. [1]...........

................................................................................................................................[6]

(d) The student added concentrated HCl at t 4 . On both the graphs on Page 10, draw how the
two graphs would look like after t 4 . [2]

PbCl 2 + 2Cl- PbCl 4 2

Formation of complex, PbCl 4 2, increases the solubility of PbCl 2 . Hence, mass of PbCl 2
dissolve increase. K sp remain constant as K sp is temperature dependent.

trendyline
[Total: 13]

H2 Chemistry 9647/02 NYJC J2/16 PX


For
13 Examiner's
Use
5 Compound A (C 12 H 16 O 2 ) exhibits optical isomerism. It does not react with hot acidified
K 2 Cr 2 O 7 . 1 mole of A reacts with 1 mole of PCl 5 to form B. A reacts with cold dilute KMnO 4
to form C (C 12 H 18 O 4 ). Upon heating with acidified KMnO 4 , A gives D (C 9 H 10 O 4 ) and E. D
produces effervescence when aqueous Na 2 CO 3 is added. 1 mole of D reacts with 3 moles
of aqueous bromine. E gives F, a yellow precipitate with warm alkaline aqueous iodine.

Draw the structures of A, B, C, D, E and F.

CH3 H CH3 CH3 H CH3

HO C C C CH3 Cl C C C CH3

OH [1] OH [1]

A B

CH3 H CH3 CH3 OH

HO C C C CH3 HO C C O
OH OH

OH [1] OH [1]

C D

CH3
CHI 3 [1]
O C CH3 [1]

E F

[Total: 6]

H2 Chemistry 9647/02 NYJC J2/16 PX [Turn over


For
14 Examiner's
Use
6 The Ritter reaction is an organic reaction used to convert a nitrile and a tertiary alcohol, in a
second order reaction, to form an amide using strong aqueous acid as the catalyst.

O R2
R2 + R3
 H (aq) C C
R1 CN R3 C OH
R1 N R4
R4
H
, ,
R1 R2 R3 and R4 are alkyl groups

The proposed mechanism is as follows:

R2 R2
R3 C OH  H C R3  H2O
+
step 1
R4 R4

H H
R2 R2 O R2
 H2O
R1 CN C R3 step 2 R1 C N C R3 step 3 R1 C N C R3
R4 R4 R4

step 4

H
O R2 O R2
R1 C N C R3  H+ step 5 R1 C N C R3
H R4 H R4

(a) Predict the structures of P, Q and T for each of the following Ritter reactions. [3]

CN

[1] OH [1]
+ O
H (aq)
+
N
H
Q R
P
(C 6 H 12 O)
(C 8 H 7 N)

trendyline
H2 Chemistry 9647/02 NYJC J2/16 PX
For
15 Examiner's
Use

O
H
+
HO CN H (aq) N

S [1]

(C 8 H 15 NO)

(b) Describe a simple chemical test to distinguish compound R and S, stating clearly the
observations.
(1) Na, room temp or (2) PCl5/SOCl2, room temp or (3) KMnO4, H2SO4(aq), heat [1]
Test .............................................................................................................................
(1) Compound S: effervescence of H2 gas evolved which extinguishes lighted
Observation .................................................................................................................
splint with a pop sound. Compound R: no effervescence of H2 observed.
(2) Compound S: white fumes of HCl evolved Compound R: no white fumes.
....................................................................................................................................
(3) Compound R: Purple KMnO4 decolourise, white ppt of Benzoic acid formed.
................................................................................................................................
Compound S: purple KMnO4 remained. [1] [2]

(c) Suggest the types of reaction occurring in steps 3 and 4.


Nucleophilic Addition [1]
Step 3 ...........................................................................................................................
Acid-base [1]
Step 4 ....................................................................................................................... [2]

(d) Complete the mechanism of step 3 and 4 below with arrows, showing the lone pairs and
dipole charges. [3]

H O H H H
R2 O R2 O
R2
R1 C N C R3 step 3 R1 C N C R3 step 4 R1 C N C R3
R4 R4 H R4

H 
G G
H O H H H
R2 O R2 O R2
R1 C N C R3 step 3 R1 C N C R3 step 4 R1 C N C R3
G G
R4 R4 H R4
x correct arrows for step 3
x correct arrows for step
p4

x show G and G in n both step


stepstrendyline
x lone pairs on H 2 O and N atom of imine

 PDUNV  PDUNV  PDUN  PDUNV

H2 Chemistry 9647/02 NYJC J2/16 PX [Turn over


For
16 Examiner's
Use
(e) Suggest which step of the proposed mechanism on Page 14 is the slowest step. Explain
your reasoning.

Step 1 is the slowest step as Ritter reaction is a second order reaction (as stated in the
question), involving one molecule of tertiary alcohol and a H+ ion. [1]

................................................................................................................................ [1]

(f) Primary alcohols such as ethanol are not suitable reagents for the Ritter reaction.

(i) Draw a diagram to show the orbitals of the carbon in the carbocation in step 1 and
state the type of hybridisation involved. [1]

sp2 [1]

(ii) Suggest why tertiary alcohols are used in the Ritter reaction but not primary
alcohols.
H R3
C C
R H R2 R4
primary carbocation tertiary carbocation

Tertiary alcohol forms a tertiary carbocation in step 1, which is more stable than a
primary carbocation from a primary alcohol. [1]

The three electron donating alkyl groups donate electrons to the carbocation to
disperse the positive charge (decrease the positive charge intensity) and stablise
the tertiary carbocation. [1]

..................................................................................................................... [2]

(iii) Suggest why phenylmethanol is a suitable reagent for the Ritter reaction even
though it is a primary alcohol.

OH Note:
The primary carbocation formed from
phenylmethanol in step 1 has an
unhybridised p-orbital which forms an
phenylmethanol
RYHUODSZLWKWKH-orbital of the carbons

trendyline
nd
n dyyline
d
in benzene.
H
C
H

H2 Chemistry 9647/02 NYJC J2/16 PX


For
17 Examiner's
Use

The positive charge on the carbocation delocalised into the benzene ring, therefore
stabilising the primary carbocation. [1]
or
Electron from the benzene ring delocalised to the carbocation, stabilising the
positive charge.

..................................................................................................................... [1]

(g) This equation for the Ritter reaction is repeated from page 14.

O R2
R2 + R3
 H (aq) C C
R1 CN R3 C OH
R1 N R4
R4
H
R1, R2, R3 and R4 are alkyl groups

(i) Using suitable data from the Data Booklet, calculate the enthalpy change of reaction.
[2]

From the Data Booklet,

%( &1   %( &1  


%( &2   BE (C=O) = 740
%( 2+   %( 1+  

'H r %( ERQGVEURNHQ %( ERQGVIRUPHG


= 890 + 360 + 460 (2305 + 740 + 390) [1] for correct substitution of values
N-PRO1 [1] for final answer and allow ecf

(ii) Suggest and explain if the Ritter reaction is spontaneous at high or low temperature.

 'G = '+7'S
x Low temperature
x 'S is negative
x as there is a decrease in entropy, whereby two reactant molecules
undergo Ritter reaction to form one product molecule.
x T'S is positive, 'G is negative only at low temperature

1 = 0 mark 2 = 1 mark 3 = 2 marks 4 = 3 marks

.........................................................................................................................

..................................................................................................................... [3]

[Total: 20]

trendyline
H2 Chemistry 9647/02 NYJC J2/16 PX [End[Turn over
of Paper]
H
NANYANG JUNIOR COLLEGE
JC 2 PRELIMINARY EXAMINATION
Higher 2

Teachers Mark Scheme


CHEMISTRY 9647/03
Paper 3 Free Response 21 September 2016
2 hours
Candidates answer Section A on the Question Paper

Additional Materials: Writing Paper


Data Booklet

READ THESE INSTRUCTIONS FIRST

Write your name and class on all the work you hand in.
Write in dark blue or black pen on both sides of the paper.
You may use a soft pencil for any diagrams, graphs or rough working.
Do not use staples, paper clips, highlighters, glue or correction fluid.

Answer any four questions.


A Data Booklet is provided.
You are reminded of the need for good English and clear presentation in your answers.

At the end of the examination, fasten all your work securely together.
The number of marks is given in brackets [ ] at the end of each question or part question.

Section B
Answer all questions on separate answer paper. For Examiners Use

Section A

Total

This document consists of 13 printed pages and 1 blank page.

trendyline
[[Turn over
2
Answer any four questions.

1(a) Copper(I) sulfate, Cu 2 SO 4 , can be made from copper(I) oxide under non-aqueous
conditions. On adding this salt to water, it immediately undergoes a disproportionation
reaction.

(i) Suggest, with a reason, the colour of copper(I) sulfate. [2]

Colourless. [1] Cu+ has an electronic configuration of [Ar] 3d10 or fully filled d
orbitals, hence d-d transition cannot take place. [1]

(ii) Using suitable data from the Data Booklet, explain why the disproportionation
reaction occurs, and write an equation for it. [3]

E (Cu2+/Cu+) = +0.15 V
E (Cu+/Cu) = +0.52 V
[1] for quoting E values

E cell = E (Cu+/Cu)  E (Cu2+/Cu+) = +0.52  (+0.15) = + 0.37 V > 0


Since E cell > 0, reaction is feasible. [1]

2Cu+ Cu2+ + Cu [1]

(b) Palladium(II) salts can form square planar complexes. Successive addition of ammonia and
hydrogen chloride to an aqueous palladium(II) salt produces, under different conditions,
three compounds with empirical formula PdN 2 H 6 Cl 2 . Two of these, A and B, are non-ionic,
with M r = 211. A has a dipole moment, whereas B has none. The third compound, C, is
ionic, having M r = 422, and contains palladium in both its cation and anion.

For each A, B and C, deduce a structure that fits the above data, explaining your reasons
fully. [6]

M r (PdN 2 H 6 Cl 2 ) = 106 + 2(14.0) + 6 +2(35.5) = 211


A and B have the molecular formula PdN 2 H 6 Cl 2. ()
C has the molecular formula (PdN 2 H 6 Cl 2 ) 2 Pd 2 N 4 H 12 Cl 4 . ()
Since palladium (II) salts form square planar complexes, it has a coordination number of
4.()

Structures of A and B:
Cl NH3 H3N Cl

Pd Pd
A: B:
Cl NH3
[1] Cl NH3 [1]
Since A has a dipole moment and B has no dipole moment, A is a polar molecule and B is
a non-polar molecule. ()

4() [2], 3(),2() [1]

C: [Pd(NH 3 ) 3 Cl]+ ; [Pd(NH 3 )Cl 3 ] OR


[Pd(NH 3 ) 4 ]2+; [PdCl 4 ]2
[1] each for correct cation and anionM r (PdN 2 H 6 Cl 2 ) = 106 + 2(14.0) + 6 +2(35.5) = 211

A and B have the molecular formula PdN 2 H 6 Cl 2. [1]


C has the molecular formula (PdN 2 H 6 Cl 2 ) 2 Pd 2 N 4 H 12 Cl 4 . [1]

trendyline
H2 Chemistry
y 9647/03 NYJC J2/16 PX
P [[Turn
T Over
3
Structures of A and B:

Cl NH3 H3N Cl

Pd Pd
A: B:
Cl NH3 [1] Cl NH3 [1]

C: [Pd(NH 3 ) 3 Cl]+ ; [Pd(NH 3 )Cl 3 ] OR


[Pd(NH 3 ) 4 ]2+; [PdCl 4 ]2
[1] each for correct cation and anion

(c) Benzene ring is often represented as a structure that has a ring within the hexagon.
Alternatively, chemists have also represented the structure of benzene in the following
forms, known as resonance structures.

Benzene (two resonance forms)

The resonance relationship is indicated by the double headed arrow between them. The
only difference between resonance forms is the placement of the pi electrons and
non-bonding electrons.

Most aromatic compounds undergo electrophilic substitution. However aryl halides undergo
a limited number of substitution reactions with strong nucleophiles.

An example of a reaction is as follows:

Nu
A X A Nu + X

where A is an electron withdrawing group and X is a halogen

The mechanism of this reaction has two steps:


x addition of the nucleophile
x elimination of the halogen leaving group

H2 Chemistry 9647/03 NYJC J2/16 PX [Turn Over

trendyline
4
Step 1 involves the addition of the nucleophile (Nu). The Nu attacks the carbon atom
bonded to a halogen, causing the pi bond to break. A resonance stabilised carbanion with a
QHZ&1XERQGLVIRUPHG7KHDURPDWLFULQJLVGHVWUR\HGLQWKLVVWHS

Nu
X X
Nu

A A

Step 2 involves the loss of the halogen X, reforming the aromatic ring.

Nu
X Nu
 X


A A

Two other resonance structures of the intermediate in Step 1 are shown below:

Nu Nu
X X
I II
  Z
A A

(i) Copy the above diagram and draw the resonance structure, Z. In your answer,
show any relevant charges, lone pairs of electrons and movement of electrons in
forming Z. [2]

Nu Nu Nu
X X

X
I II
 
A A A

[1] for correct structure of Z


[1] for correct arrow pushing

The reaction below shows the synthesis of compound, D.

N(CH3)2 N(CH3)2
O 2N
O 2N
potassium metal +
+ HCl

Cl HO O

D
(ii) Suggest the role of potassium metal in the reaction. [1]

It is used to generate a stronger nucleophile, C 6 H 5 CH(O)CH 2 CH 2 N(CH 3 ) 2. [1]


H2 Chemistry 9647/03 NYJC J2/16 PX [Turn Over

trendyline
5

(iii) Use the information given above to draw out the full mechanism for the reaction that
forms D, labelling the slow and fast steps. In your answer, showing any relevant
charges, lone pair of electrons and movement of electrons. [3]

N(CH3)2 N(CH3)2

+  + 1/2 H2
K + HO K O

N(CH3)2
N(CH3)2
O 2N 
O 2N
slow Cl
+
O
Cl 
O

N(CH3)2 N(CH3)2
O 2N  O 2N
fast 
Cl
+ Cl
O O

[1] for step 1 (accept OH as the nucleophile)


[1] for step 2
[1] for labelling slow and fast steps correctly

(d) Describe and explain the relative ease of hydrolysis of the following three
chloro-compounds. [3]
Cl COCl CH2Cl

The relative ease of hydrolysis of the three chloro-compounds in increasing order is as


follows:

Cl CH 2Cl COCl

< <

[1]

H2 Chemistry 9647/03 NYJC J2/16 PX [Turn Over

trendyline
6
Chlorobenzene is resistant to hydrolysis. The p orbital of chlorine overlaps with S electron
cloud of benzene ring hence strengthening the C-Cl bond and hence the bond does not
break easily. [1]
COCl

Hydrolysis of is the fastest. The acyl/C=O carbon atom is attached to 2 highly


CH 2 Cl

electronegative atoms, O and Cl while that in is only attached to one


electronegative atom (Cl).
COCl

Hence acyl/C=O carbon atom in is more electron deficient (i.e. carries a larger
SDUWLDOSRVLWLYHFKDUJH WKDQWKHFDUERQDWRPLQ& 6 H 5 CH 2 Cl.

Therefore is most susceptible to nucleophilic attacks. [1]


[Total: 20]

H2 Chemistry 9647/03 NYJC J2/16 PX [Turn Over

trendyline
7
2 Sulfuric acid, H 2 SO 4 , can behave as an acid, an oxidising agent or as a dehydrating agent
in various reactions.

(a) Draw a diagram to illustrate the shape of pure sulfuric acid and indicate the bond angle
about the sulfur atom. [2]

tetrahedral shape + Diagram [1]

S
O OH
OH
bond angle about S = 109.5O [1]

(b) The Contact Process is used for the manufacture of sulfuric acid. One of the reactions that
takes place is the following reversible reaction:

2SO 2 (g) + O 2 (g) 2SO 3 (g) 'H = 197 kJ mol1

Sulfur dioxide and oxygen in a 2:1 molar ratio at a total initial pressure of 3 atm is passed
over a catalyst in a fixed volume vessel at 400 oC. When equilibrium is established, the
percentage of sulfur trioxide in the mixture of gases is found to be 30%.

(i) Write an expression for the equilibrium constant, K p , of the reaction. [1]

P
2
SO3
KP = [1]
P P
2
SO2 O2

(ii) Calculate the value of K p at 400 oC, stating its units. [3]

2SO 2 (g) + O 2 (g) 2SO 3 (g)


Initial Partial Press /atm 2 1 0
Change in Press /atm -2x -x +2x
Final Partial Press /atm 2 - 2x 1-x 2x

Total Pressure, P T = 2 - 2x + (1 x) + 2x = 3- x [1]


2x
 SO 3 = 3 - x = 0.30

? x = 0.391 [1]

0.782
2

KP = 0.679 atm 1 correct answer and units [1]


1.218 (0.609)
2

(iii) How would the percentage conversion of SO 2 into SO 3 be affected when the
pressure is raised? Explain. [2]

 conversion to SO 3 will increase.


With increased pressure, by Le Chateliers Principle, position of equilibrium will shift
to the right so as to favour the side which produces the less number of moles of gas.

H2 Chemistry 9647/03 NYJC J2/16 PX [Turn Over

trendyline
8
(c) Dilute sulfuric acid takes part in typical acid-base reactions and it can be used to distinguish
the following solids: MgO, BaO and SiO 2 .

State the observations, if any, to indicate the differences in their reaction when water is
added to each solid followed by dilute sulfuric acid. [4]

Solid MgO is insoluble in water / sparingly soluble in water.


It is a basic oxide and dissolves in H 2 SO 4 (aq) to form a colorless solution.

Solid BaO dissolves in water to form colorless solution , upon adding H 2 SO 4 (aq) a white
ppt BaSO 4 is formed
SiO 2 does not dissolve in water or acid.

[2] for observations with water

[2] for observations with dil H 2 SO 4

(d) Sulfur dioxide is a major pollutant from sulfuric acid plants. The SO 2 emitted into the
atmosphere is oxidised in the air, which then reacts with water to form sulfuric acid, hence
causing acid rain:

2SO 2 (g) + O 2 (g) + 2H 2 O(l) o 2H 2 SO 4 (l) 'H 1

Using the data below and data from (b), construct an energy cycle to calculate

(i) the enthalpy change of formation of SO 2 (g), and hence


2 x 98.5
2SO2 (g) + O2 (g) o 2SO3 (g)

2 x -493

2S (s) + 3 O2 (g)

2 x 'Hf (SO2) = 2 x 493 (2 x 98.5)


= 789 kJmol-1
[1] for ans, [1] for working

(ii) the enthalpy change of reaction, 'H 1 for the above reaction.

Enthalpy change of formation of H 2 O(l) = 286 kJ mol1


Enthalpy change of formation of H 2 SO 4 (l) = 811 kJ mol1
Enthalpy change of formation of SO 3 (g) = 493 kJ mol1
[4]
'H rxn
2SO2 (g) + O2 (g) + 2H2O (l) o 2H2SO4 (g)

789 - 2 x -286 2 x -811

2S (s) + 4O2 (g) + 2H2 (g)

? 'Hrxn = (2 x 811) (789) (2 x 286)


= 261 kJmol-1
[1] for ans, [1] for working
H2 Chemistry 9647/03 NYJC J2/16 PX [Turn Over
9

H2 Chemistry 9647/03 NYJC J2/16 PX [Turn Over

trendyline
10
(e) Alcohols react with concentrated sulfuric acid at high temperatures to form alkenes. A
common side reaction that can happen is the formation of ethers, which is also catalysed
by concentrated sulfuric acid.

R R R
conc H2SO4
2R C OH R C O C R + H2O
heat
R R R
R R R
conc H2SO4
2R C OH R C O C R + H2O
heat
R R R

The mechanism occurs via 3 steps:

Step 1:
An acid base reaction in which H+ from H 2 SO 4 protonates the oxygen atom in alcohol. This
step is very fast and reversible.

Step 2:
A second alcohol molecule functions as the nucleophile and attacks the product from
step 1. The CO bond is cleaved and a water molecule leaves the molecule. This creates
R
+
O
an oxonium ion R' H intermediate.

Step 3:
Another acid base reaction in which the proton in the oxonium ion is removed by a suitable
base (in this case a water molecule) to give the ether product. This step is very fast and
reversible.

(i) Draw the ether formed when cyclopentanol undergoes the above reaction.

OH

cyclopentanol
[1]

O
[1]

(ii) Draw out the full mechanism for the reaction between two cyclopentanol molecules
to form an ether. In your answer, show any relevant charges, lone pairs of electrons
and movement of electrons. [3]

H2 Chemistry 9647/03 NYJC J2/16 PX [Turn Over

trendyline
11
+
:OH OH2
fast
Step 1:
+
H

H H
+ H
O +
:OH slow O
Step 2: + H2O
G

H fast
+
Step 3: O O + H3O+
H2O:

1 mk for each step (with arrows, lone pairs) total 3 mks

[Total: 20]

H2 Chemistry 9647/03 NYJC J2/16 PX [Turn Over

trendyline
12
3 Garlic contains many amino acids, minerals and enzymes. Garlic also contains at least 33
sulfur compounds like alliin, allicin and ajoene. The sulfur compounds are responsible for
both garlics pungent odour and many of its medical effects.

(a) When raw garlic is chopped or crushed, the enzyme allinase converts alliin into allicin,
which is responsible for the odour of fresh garlic.

O NH2 O

S allinase S
COOH S
alliin allicin

Allicin can also be biosynthesised by using serine and glutathione.

NH2 COOH O

HO NH
COOH H2N NH COOH

O
SH
serine glutathione

(i) Explain why raw chopped garlic has a stronger odour than when it is cooked. [1]

Enzyme allinase is denatured by heat. [1]

(ii) When one molecule of serine reacts with one molecule of glutathione, it is possible
to form two esters with different structural formulae.
Draw the structural formula of each of these esters. [2]

NH2

O
COOH
O C O

NH
H2N NH COOH

O
SH [1]

COOH O NH2

NH O
H2N NH C COOH

O O
SH [1]

H2 Chemistry 9647/03 NYJC J2/16 PX [Turn Over

trendyline
13
(iii) Draw the structural formulae of the products when glutathione is hydrolysed. [3]

COOH O

OH H2N
H2N OH H2N COOH

O
SH
[1] x 3

Alliin has pK a values of 1.84 and 8.45.

(iv) Make use of these pK a values to suggest the major species present in solutions of
alliin with the following pH values. [3]
x pH 1
x pH 7
x pH 11

pH 1
+
O NH3

S
COOH [1]

pH 7
+
O NH3

S -
COO [1]

pH 11
O NH2

S -
COO [1]

(v) Calculate the pH of 0.10 mol dm3 solution of alliin. [1]

[H+] = 10 1.84 u 0.10 = 0.03801 mol dm3


pH = lg 0.01188 = 1.42 [1]

(vi) With reference to the pK a values, identify the major species formed when 10 cm3 of
0.10 mol dm3 NaOH is added to 10 cm3 of 0.10 mol dm3 protonated alliin. Hence,
deduce whether the solution is acidic, neutral or alkaline. [2]

pKa = 8.45
+
O NH3 Ka = 108.45 = 3.54 109

S -
COO pKb = 14 1.84 = 12.16
Kb = 1012.16 = 6.918 1013
[1]
H2 Chemistry 9647/03 NYJC J2/16 PX [Turn Over

trendyline
14

Since K a (NH 3 +) > K b (COO), the solution will be acidic. [1]

(vii) Sketch the pH-volume added curve you would expect to obtain when 30 cm3 of
0.10 mol dm3 NaOH is added to 10 cm3 of 0.10 mol dm3 protonated alliin.
Briefly describe how you have calculated the various key points on the curve. [4]

At 5 cm3, 1st maximum buffering capacity point


pH = pK 1 = 1.84

At 10 cm3, 1st equivalence point, solution contains RCH(NH 3 )+COO which is an


acidic salt with K a = K 2 = 108.45
RCH(NH 3 )+COO + H 2 O RCH(NH 3 )+COOH + H 3 O+ K 2 = 108.45
10
u 0.10
+
[RCH(NH 3 ) COO ] = 1000 =0.05 mol dm3
20
1000
pH = lg 0.05 u 10 8.45 = 4.875 | 4.88

pK a1  pK a 2 1.84 + 8.45
OR pH= = 5.145 = 5.15
2 2

At 15 cm3, 2nd maximum buffering capacity point


pH = pK 2 = 8.45

At 20 cm3, 2nd equivalence point, solution contains RCH(NH 2 )COO which is a basic
salt with K b = 10(14  K2) = 10(14 8.45) = 105.55
10
u 0.10
[RCH(NH 2 )COO ] = 1000 = 0.03333 mol dm3
30
1000
pOH = lg 0.03333 u 10 5.55 = 3.513
pH = 14  3.513 = 10.48 | 10.5
At 30 cm3, solution contains 10 cm3 excess NaOH
[OH] = concentration of the 10 cm3 of excess NaOH in a total solution
10
u 0.10
= 1000 = 0.025 mol dm3
40
1000
pH = 14  pOH = 14  (lg 0.025) = 12.4

H2 Chemistry 9647/03 NYJC J2/16 PX [Turn Over

trendyline
15

pH

12.4

Equivalence point 2
10.5

Equivalence point 1

8.45

4.88 or 5.15
1.84
1.42

0
5 10 15 20 25 30 VNaOH/cm3
Key points to look out for
x Label the x and y-axis clearly
x Curve start at pH = 1.42 with a sharp increase before flattening off due to formation
of buffer
x Two neutralisation points at 10 cm3 and 20 cm3, labelling of pH values at equivalence
points is not needed
x Two buffer regions centred at pH values 1.84 and 8.45 (clearly labelled) respectively
x When V NaOH = 30 cm3, pH = 12.4, showed calculations on how this pH value is
obtained
Total of 4 marks
Deduct 1 mark for each missing bullet point

(b) Diallyl disulfide is one of the principal components of the distilled oil of garlic. It is a
yellowish liquid which is insoluble in water and has a strong garlic odour. It is produced
during the decomposition of allicin.

H2 Chemistry 9647/03 NYJC J2/16 PX [Turn Over

trendyline
16
O

S S
S S

allicin diallyl disulfide


Diallyl disulfide can be produced industrially from sodium disulfide and allyl chloride at
temperatures of 40 60 C in an inert atmosphere.

40  60 oC
Cl S
Na2S2 + 2 S + 2NaCl
inert gas
allyl chloride

(i) Give the IUPAC name of allyl chloride. [1]

3-chloroprop-1-ene [1]

(ii) Explain, in thermodynamic terms, suggest why diallyl disulfide is insoluble in water.
[2]
Diallyl disulfide is a non-polar molecule. The energy evolved in the formation of
dispersion forces between diallyl disulfide and water molecules is insufficient [1] to
overcome the energy required to break the dispersion forces between diallyl
disulfide molecules and hydrogen bonding between water molecules. [1]

(iii) State the type of reaction when diallyl disulfide is converted back to allicin. [1]

Oxidation [1]

[Total: 20]

H2 Chemistry 9647/03 NYJC J2/16 PX [Turn Over

trendyline
17
4 Alkynes are organic molecules which contain carbon-carbon triple bonds and are part of the
homologous series with formula of C n H Q i.e.

R1 C C R2

where R 1 and R 2 = H or alkyl or aryl groups

Alkynes exhibit similar chemical properties to alkenes.


e.g. addition reactions with electrophiles i.e. X 2 or HX to form alkenes

R1 R2
HX
R1 C C R2 C C

H X

e.g. oxidation by hot concentrated KMnO 4 to form mixture of carboxylic acids

KMnO 4
R1 C C R2 R1COOH + R2COOH

However, unlike alkenes, terminal alkynes are able to react with strong bases like sodium
amide.
 +
RC CH + NaNH2 RC C Na + NH3

(a) Ethyne, C 2 H 2 , is heated with excess sodium bromide and concentrated sulfuric acid to
produce a dihalide, C 2 H 4 Br 2 . The overall reaction may be considered to take place in two
stages, the first between inorganic reagents only and the second involving the organic
reagent.

(i) Write an equation for the first stage. [1]

NaBr + H 2 SO 4 oNaHSO 4 + HBr [1]

(ii) Suggest a structure for the dihalide formed. [1]

CH 3 CHBr 2 [1] follow markovnikov rule

(iii) When the concentrated sulfuric acid is added to the reaction mixture, cooling is
necessary to prevent the formation of inorganic by-products.
Write an equation to explain the formation of these inorganic by-products. [1]

2HBr + H 2 SO 4 oBr 2 + SO 2 + 2H 2 O [1]

(b) Compound A, is an enyne chloride (i.e. compounds that contains chloro, alkyne and alkene
functional groups).

Cl

H2 Chemistry 9647/03 NYJC J2/16 PX [Turn Over

trendyline
18
Compound A

One mole of compound A reacts with two moles of Br 2 to produce a mixture of


4 stereoisomers. Draw structures of the stereoisomers formed. [3]

CH2Cl CH2Cl
CH3 CH3
H3C CH3
Br Br
cis Br
CH3 CH3
Br
Br Br Br Br

CH2Cl CH2Cl
CH3 CH3
Br Br
Br Br
trans Br Br
CH3 CH3
H3C Br Br CH3

(c) Compound B, which is an isomer of Compound A and also an enyne chloride, is treated
with sodium amide, NaNH 2 followed by heating under reflux to form compound C, C 8 H 10 .
Compound C reacts with hot concentrated KMnO 4 to produce butane-1,4-dioic acid only.

(i) Explain the reaction with NaNH 2 . [1]

To deprotonate the terminal alkyne group


Or acid-base reaction with terminal alkyne. [1]

(ii) Hence, explain the formation of compound C. [1]

The deprotonated alkyne acts as a nucleophile which undergoes


intramolecular nucleophilic substitution to form Compound C [1]

(iii) Suggest skeletal structures for compounds B and C. [2]

Cl

B:

C:

H2 Chemistry 9647/03 NYJC J2/16 PX [Turn Over

trendyline
19
When a current of 1.0 A was passed through aqueous potassium maleate (KO 2 CCH=CHCO 2 K) for
15 minutes, it was found that 110 cm3 H 2 , measured at r.t.p, was collected at the cathode. The
following reaction took place.
2H 2 O + 2e o H 2 + 2OH
(d) State the relationship between the Faraday constant, F and the Avogadros constant, L. [1]

F = Le where e = charge on 1 electron [1]

(e) Using the data above and the Data Booklet, calculate a value for Avogadros constant. [3]

Q = 1.0 15 60 = 900 C [1]


n(H 2 ) = 110 / 24000 = 0.00458 mol
n(e) = 2 n(H 2 ) = 0.00917 mol [1]
F = Q / n(e) = 98182 = 98200 C
L = 98182 / 1.60 1019 = 6.14 1023 [1]

(f) Ethyne and CO 2 gas were produced at the anode. In order to determine the stoichiometry
of the anode reaction, the volume of the gases collected at the anode was measured. The
anode gas was first passed through aqueous NaOH before being collected in a gas
syringe. The following data was collected:

x mass of bottle containing NaOH before experiment = 10.501 g


x mass of bottle containing NaOH after experiment = 10.904 g
x initial reading on syringe = 10.0 cm3
x final reading on syringe = 120.0 cm3

(i) State the oxidation state of carbon in ethyne. [1]

1 [1]

(ii) With the help of an equation, explain the purpose of passing the anode gas through
NaOH. [1]

2NaOH + CO 2 oNa 2 CO 3 + H 2 O
Purpose of NaOH is to absorb CO 2 [1]

(iii) Calculate the volume of CO 2 produced, assuming r.t.p conditions. [1]

Change in mass of NaOH = mass of CO 2 produced = 0.403 g


n(CO 2 ) = 0.403 / 44.0 = 0.009159 mol
V(CO 2 ) = 24000 n(CO 2 ) = 220 cm3 [1]

(iv) Hence, suggest an ionic equation for the reaction that occurred at the anode. [1]

Volume of ethyne gas = 110 cm3



O 2 CCH=CHCO 2  o C 2 H 2 + 2CO 2 + 2e

(g) When aqueous potassium maleate was acidified, maleic acid, HO 2 CCH=CHCO 2 H
(pK a1 = 1.90 and pK a2 = 6.07) was liberated. Fumaric acid (pK a1 = 3.03 and pK a2 = 4.44) is
a stereoisomer of maleic acid.
With a suitable illustration, suggest a reason why maleic acid has a lower pK a1 but higher
pK a2 than fumaric acid.
[2]

H2 Chemistry 9647/03 NYJC J2/16 PX [Turn Over

trendyline
20

hydrogen bonding

G G
G OH O-

O O

[1]
Maleic acid is the cis-isomer while fumaric acid is the trans isomer. Hence the negative
carboxylate ion is stabilised by intramolecular hydrogen bonding. Hence maleic acid is stronger
acid than fumaric acid. [1]

[Total: 20]

H2 Chemistry 9647/03 NYJC J2/16 PX [Turn Over

trendyline
21
5(a) Dopamine is an organic compound of the catecholamine and phenethylamine families that
plays several important roles in the brain and body. Its name is derived from its chemical
structure: it is an amine synthesised by removing a carboxyl group from a molecule of
its precursor compound, L-DOPA.

Below is a synthetic route involving L-DOPA and dopamine:

O
HO HO NH2
OH
+ CO 2
NH2
HO HO
L-DOPA Dopamine

Reaction I Reaction III


Compound H

O Product J
HO
Cl
HO N
NH2
HO
HO
Reaction II
anhydrous AlCl 3

Isomers F and G
C9H9O 3N

(i) State the reagents and conditions and any observations in Reaction I. [1]

Reagents & conditions: PCl 5 or SOCl 2


Observation: White fumes of HCl
1 mark for both

(ii) Aluminium chloride is used as a catalyst in electrophilic substitution reactions. The


chlorination of benzene is represented by the following overall equation.

Cl
+ Cl 2 + HCl

The reaction occurs in several steps.


x The first step is the reaction between Cl 2 and AlCl 3 .
Cl 2 + AICl 3 Cl+ + AICI 4 
x The benzene is then attacked by the Cl+ cation in the second step.

The first step is the reaction between Cl 2 and AICI 3 .


Cl 2 + AICl 3 Cl+ + AICI 4 

H2 Chemistry 9647/03 NYJC J2/16 PX [Turn Over

trendyline
22
+
The benzene ring is then attacked by the CI cation in the second step.
AICl 3 reacts in a similar way with acyl chlorides, producing a carbocation
that can then attack a benzene ring.

Predict the structures of isomers F and G in Reaction II. [2]


OH
HO HO
NH2 NH2
HO
O O 1 mark each

(iii) In Reaction III, dopamine was reacted with alkyl halide H to give the final product J.
Draw the displayed formula of H. [2]

H H
H H H
Br C C Br
C C C
H H
H H H [12] correct RX & displayed formula

H2 Chemistry 9647/03 NYJC J2/16 PX [Turn Over

trendyline
23
(b) Dopamine is a bidentate ligand. When different volumes of 0.0030 mol dm3 of aqueous
Cr(III) and 0.0020 mol dm3 of alkaline dopamine solution were mixed, a complex R is
formed. Analysis of R shows that its formula is [Cr(C 8 H 9 NO 2 ) x (H 2 O) y ]z, where x, y and z
are integers.
To determine the stoichiometry of the complex ion formed, the colour intensities of these
different mixtures were measured using a colorimeter. The following absorption spectrum
was obtained.
Relative Absorbance

| | | | | | | | | | | | | | | | | | | |
Volume of Cr3+ / cm3
| | | | | | | | | | | | | | | | | | | | |
10 9 8 7 6 5 4 3 2 1
Volume of alkaline dopamine / cm3

(i) Use the graph and the information given to determine the formula of complex R.
Show your workings clearly. [3]

n(Cr3+) : n(dopamine)
2.5 7.5
= 0.0030 x : 0.0020 x
1000 1000
= 7.50 x 106 : 1.50 x 105
= 1:2 [1]
?x=2

(ii) The crystal field describes the breaking of orbital degeneracy in transition metal
complexes due to the presence of ligands. When the d-orbitals split into high energy
and low energy orbitals, the difference in energ\RIWKHWZROHYHOVLVGHQRWHGDV o .
7KH UHODWLRQVKLS EHWZHHQ o and colours of complexes can be described in the
equation below:
hc
'o =
O
where h is Plancks constant, c is the speed of light and O is the wavelength of light
absorbed
colour absorbed O / nm
violet 410
indigo 430
blue 480
blue-green 500
H2 Chemistry 9647/03 NYJC J2/16 PX [Turn Over

trendyline
24
green 530
yellow 580
orange 610
red 680
*LYHQWKDW o for complex R is 4.125 x 1022 kJ and using relevant data from the Data
Booklet, calculate the wavelength of light. Deduce the colour of complex R. [2]

hc
'o =
O
6.63 x 10-34 (3 x 108 )
  [1]
4.125 x 10-22 x 1000
= 4.82 x 10-7m or 482 nm

Since colour absorbed is blue (482 nm blue), the colour of complex R should be the
orange (complementary colour). [1]

(c) Iodine is not very soluble in water, it is freely soluble in KI(aq), according to the following
equilibrium:
I(aq) + I 2 (s) I 3 (aq)

(i) Draw a fully labelled experimental set-up for a voltaic cell made up of a
Cr 2 O 7 2/ Cr3+ half-cell and a I 2 /I half-cell under standard conditions. Indicate clearly
the anode and cathode and show the flow of electrons. [3]

Platinised Pt anode Platinised Pt cathode


() (+)

1 mol dm3 of
Cr2O72 & Cr3+
1 mol dm3 of H+
1 mol dm3 of I&
1 mol dm3 of I3

All 5 points 3 marks, 3-4 points 2 marks, 2 points 1 mark


x Cr 2 O 7 2/Cr3+ half cell (with acidified medium)
x I 3 /I half cell (with Pt electrode)
x Complete circuit (salt bridge), standard condition (1 mol dm3)
x Direction of electron flow
x Clear indication of anode, cathode

H2 Chemistry 9647/03 NYJC J2/16 PX [Turn Over

trendyline
25
(ii) By using appropriate values from the Data Booklet, predict what, if anything, will
happen when a small amount of acidified vanadium(II) chloride is added to a
solution of the I 2 (aq). /I half-cell.
[3]

I 2 + 2e- 2I (+0.54)


V3+ + 2e- V2+
VO2+ + 2H+ + e- V3+ + H 2 O
VO 2 + + 2H+ + e- VO2+ + H 2 O (+1.00)

I 2 (aq) + V2+(aq) o 2I(aq) + V3+(aq)


E cell = (+0.54) (0.26) = +0.80 V > 0 ?reaction is feasible. x

I 2 (aq) + 2V3+(aq) + 2H 2 O(l) o 2I(aq) + 2VO2+(aq) + 4H+(aq)


E cell = (+0.54) (+0.34) = +0.20 V > 0 ?reaction is feasible. x

I 2 (aq) + 2VO2+(aq) + 2H 2 O(l) o 2I(aq) + 2VO 2 +(aq) + 4H+(aq)


E cell = (+0.54) (+1.00) = 0.46 V < 0 ?reaction is not feasible. x

V2+ will be oxidised to V3+ and finally VO2+ while I 2 will be reduced to I. x

All 4 points 3 marks, 2-3 points 2 marks, 1 point 1 mark


A must to show calculation of E cell for all 3 equations.

(d) Explain the following statements.

(i) BrF 3 is a covalent compound which exhibits electrical conductivity in liquid state
at room temperature. With the aid of an equation, suggest an explanation for its
electrical conductivity. [2]

BrF 3 autoionisesI ionises in the liquid state to give BrF 2 + and BrF 4 ions
which are mobile, thus accounting for its electrical conductivity. [1]
2BrF 3 o BrF 2 + + BrF 4 [1]

(ii) SiCl 4 reacts violently in water but CCl 4 has no reaction with water. [1]

Water molecules could not form co-ordinate bonds with the central carbon
atom of CCl 4 because carbon is in period 2 and cannot accommodate lone pair
of electrons from H 2 O. [1]

(iii) Compounds NeF 2 and NeF 4 do not exist but XeF 2 and XeF 4 exist.
[1]

Xenon has larger radii; therefore the electron attraction to the nucleus is
weaker in comparison to the smaller noble gases. OR
Energy levels of the 5p orbital, 6s and 4f orbitals are very close and hence
energetically accessible. Only a small amount of energy is needed to excite the
paired electrons from the 5p orbitals to the 6s and 4f electrons where they can then
form covalent bonds with other atoms. [1]

[Total: 20]

H2 Chemistry 9647/03 NYJC J2/16 PX [Turn Over

trendyline
26

[End of Paper]

H2 Chemistry 9647/03 NYJC J2/16 PX [Turn Over

trendyline
PIONEER JUNIOR COLLEGE

JC2 PRELIMINARY EXAMINATION


HIGHER 2

CANDIDATE
NAME

CT INDEX
1 5 S
GROUP NUMBER

CHEMISTRY 9647/01
Paper 1 Multiple Choice 23 September 2016
1 hour
Additional Materials: Multiple Choice Answer Sheet
Data Booklet

READ THESE INSTRUCTIONS FIRST

Write in soft pencil.


Do not use staples, paper clips, highlighters, glue or correction fluid.
Write your name, Centre number and index number on the Answer Sheet in the spaces provided
unless this has been done for you.

There are forty questions on this paper. Answer all questions. For each question there are four
possible answers A, B, C and D.
Choose the one you consider correct and record your choice in soft pencil on the separate
Answer Sheet.

Read the instructions on the Answer Sheet very carefully.

Each correct answer will score one mark. A mark will not be deducted for a wrong answer.
Any rough working should be done in this booklet.

This document consists of 20 printed pages [Turn over


962 trendyline
2
Section A
For each question, there are four possible answers labelled A, B, C and D. Choose the
one you consider to be correct.

1 Which of the following quantities is equal to the Avogadro constant?

A the number of atoms in 24 dm3 of nitrogen gas at r.t.p. condition


B the number of molecules in 1 dm3 of oxygen at 273 K and 1 atm
C the number of electrons removed to form Ne+ ions from 6.02 x 10-23 mol of Ne
atoms
D the number of ions in 83.5 g of [Co(NH 3 ) 5 Cl]Cl 2

2 Which of the following species has no unpaired electrons?

A Fe B Ge2+ C Si D Ti3+

3 Why is the molecule of BCl 3 planar, whereas the molecule of PH 3 is pyramidal?

A The repulsion between chlorine atoms is greater than that between hydrogen
atoms.
B The covalent radius of chlorine is greater than that of hydrogen.
C The boron atom is BCl 3 has six electrons in its valence shell, whereas the
phosphorus atom in PH 3 has eight.
D The boron atom has no d-orbitals available for bonding.

4 In which pair do the isomers have identical boiling points?

A CH3 CH3
and
H C C H
Cl CH2Cl ClCH2 Cl

B H3C H H3C CH3


C C and C C
H CH3 H H

C CH 3 CH 2 CH 2 CH 2 OH and (CH 3 ) 2 CHCH 2 OH

D CH 3 (CH
CH
H 2 ) 4 CH 3 and (CH
C 3 ) 2 CHCH(CH
CH(C 3 ) 2

PJC 2016 9647/01/JC2/H2/MYE/2016 [Turn Over


963 trendyline
3

5 P
Which of the following shows a graph of against T for an ideal gas?

(P = pressure; = density; T = temperature in C).

A P B P

T T
C D
P P

T T

6 One mol of neon gas at temperature T 1 was added to another one mol of neon and
the temperature was increased to T 2 .
Which of the following diagrams correctly represents the Boltzmann distribution of
molecular speeds before and after the changes were made?

A B T2
T2 T1
proportion of
proportion of

molecules
molecules

T1

molecular speed molecular speed

C D
T2 T1
proportion of

of
molecules

molecules

T1 T2
proportion

trendyline
tr
trend
tren in
ine
mole
propo
pro

molecular speed molecular speed


sp

@PJC 2016 9647/01/JC2/H2/Preliminary Exam/2016 [Turn over


964 trendyline
4
7 Use of Data Booklet is relevant to this question.

Dinitrogen oxide burns in ethyne to produce water vapour, carbon dioxide and
nitrogen gas as the only products.

5N 2 O(g) + C 2 H 2 (g) H 2 O(g) + 2CO 2 (g) + 5N 2 (g)

Given that the N=N and N=O bond energies in dinitrogen oxide are +418 kJ mol1
and +686 kJ mol1 respectively, what is the enthalpy change of the reaction?

A +1670 kJ mol1
B +1710 kJ mol1
C 1670 kJ mol1
D 1710 kJ mol1

8 Quarter-life, t 1/4 , of a radioactive isotope is defined as the time taken for the sample
to decay to its original amount.

Potassium-argon dating is used to determine the age of a rock. 40K is a radioactive


isotope of potassium and it decays to 40Ar with a constant t 1/4 of 2.50 x 109 years. A
sample of moon rock was found to contain 6.25% of the original amount of 40K.

How old is the rock?

A 2.50 x 109 years B 5.00 x 109 years


C 7.50 x 109 years D 1.00 x 1010 years

9 During electrolysis, 0.785 g of chromium is deposited on the cathode when 8690 C


of electricity is passed into a chromium-containing electrolyte.

Which of the following could have been the electrolyte?

A CrCl 2 B CrCl 3 C K 2 CrO 3 D K 2 Cr 2 O 7

trendyline
@PJC 2016 9647/01/JC2/H2/Preliminary Exam/2016 [Turn over
965 trendyline
5
10 Use of the Data Booklet is relevant to this question.

The graph below shows the variation in electromotive force (emf) of the following
electrochemical cell with lg [A+(aq)] at 298 K.

Cu(s) | Cu2+(aq, 1 mol dm3) || A+(aq) | A(s)

emf / V
0.46

lg [A+(aq)] 0
7.80
Which of the following statements is incorrect?

A Cu(s) is the negative electrode.


B The direction of electron flow in the external circuit will be reversed when the
concentration of A+(aq) is 1.00 x 10-7 mol dm-3 .
C The standard electrode potential of the A+(aq) | A(s) half-cell is +0.80 V.
D The emf of the given cell under standard conditions will be +0.46 V.

11 Gas D dissociates on heating to set up the equilibrium below:

D(g) E(g) + F(g)

A quantity of D was heated at a constant pressure, p, at a certain temperature. The


1
equilibrium partial pressure of D was found to be p.
7

What is the equilibrium constant, K p , at this temperature?

A 6p B 9p C 36p D 6p
7 7 7

trendyline
@PJC 2016 9647/01/JC2/H2/Preliminary Exam/2016 [Turn over
966 trendyline
6
12 Paracetamol (pK a = 9.5) is a widely used over-the-counter pain reliever and fever
reducer. Its solubility in water is 12.78 mg cm-3 at 25 oC. (M r of Paracetamol =
151.0)

O O
-
NH OH NH O + H+

paracetamol

What is its pH in water at 25 oC?

A 1.07
B 5.29
C 6.79
D 8.71

13 The pH change when 0.0500 mol dm-3 HCl is added dropwise to 5.00 cm3 of
0.100 mol dm-3 CH 3 NH 2 (aq) is shown in the graph below.

At which point on the graph is pH= 14 - pK b , where K b is the base dissociation


constant of the weak base?

A

B

C
D

Volume of 0.0500 mol dm-3 HCl added / cm3

trendyline
d li
@PJC 2016 9647/01/JC2/H2/Preliminary Exam/2016 [Turn over
967 trendyline
7

14 Which of the following statements explains the observations that magnesium


hydroxide dissolves in ammonium chloride, but not in aqueous sodium chloride?

A The ammonium ion changes the solubility product of Mg(OH) 2 .


B NH 4 + ion is first formed, and then acts through a common ion effect.
C NH 4 Cl dissociates less fully than NaCl.
D The NH 4 + ion acts as an acid.

15 A saturated solution of Ca(OH) 2 is found to have a pH of 12.3 at 25 C.

Which of the following statements is incorrect?

A The K sp of Ca(OH) 2 is 4 10-6 mol3 dm-9.


B The solubility of Ca(OH) 2 would increase when temperature is raised to 35 C.
C The solubility of Ca(OH) 2 will decrease when solid Na 2 O is added.
D The pH of the solution would increase when Ca(NO 3 ) 2 is added

16 Use of the Data Booklet is relevant to this question.

Natural water in reservoirs often contains very finely divided solid particles of
between 1 and 100 nm in diameter which have negative charges on their surface.

One stage in the water purification process consists of adding salt solutions
containing high charge-density cations which will neutralise the negative charges
and cause the solid particles to join together and be precipitated out.

Which compound, in aqueous solution, would be the most effective in precipitating


the finely divided solid particles?

A AlCl 3
B MgCl 2
C SiCl 4
D PCl 3

trendyline
@PJC 2016 9647/01/JC2/H2/Preliminary Exam/2016 [Turn over
968 trendyline
8

17
white ppt,
NaOH(aq)
insoluble in excess
O2 water colourless
M white solid
solution
H2SO4(aq) white ppt

Based on the information above, what could element M be?

A aluminium
B calcium
C potassium
D silicon

18 HCl is stable to heat, but HI decomposes into its elements when heated.

What is the reason for this difference?

A HI is a stronger reducing agent than HCl.


B Cl Cl bond is stronger than II bond.
C HCl bond is stronger than HI bond.
D HI is more volatile than HCl.

19 Use of the Data Booklet is relevant to this question.

Which of the following solutions would result in a colour change when left to stand in
the atmosphere?

A an acidified solution of cobalt(II) nitrate


B a solution of potassium hexacyanoferrate(III)
C an acidified solution of tin(II) chloride
D an acidified solution of vanadium(II) sulfate

@PJC 2016 9647/01/JC2/H2/Preliminary Exam/2016 [Turn over


969 trendyline
9
20 Adding concentrated HCl(aq) to CuSO 4 (aq) causes the colour of the solution to
change from blue to green.

Which of the following row correctly shows the number of d-electrons and the
energy gap between the d-orbitals, before and after the reaction?

number of d-electrons energy gap between the d-orbitals


A changes changes
B changes remains the same
C remains the same changes
D remains the same remains the same

21 In the reaction of methane with chlorine in the presence of light, the following energy
level diagram (for single reaction step) is obtained.

energy

reactants

reaction pathway

To which of the following steps does this diagram apply?

A CH3 + CH3 C2H6

B CH4 + Cl CH3Cl + H

C CH3 + Cl2 CH3Cl + Cl

D Cl2 Cl + Cl

trendyline
@PJC 2016 9647/01/JC2/H2/Preliminary Exam/2016 [Turn over
970 trendyline
10
22 The cholesterol-lowering agents called statins, such as pravastatin, are among the
most widely prescribed drugs in the world.

HO O
OH OH O
C
OH
Pravastatin
CH3

How many chiral centres are present in the product when pravastatin was treated
with H 2 , Ni catalyst at 200 0C?

A 6 B 7 C 8 D 9

23 CH=CHCH3

CN

CH2CH3

compound P

Which of the following statements about compound P is incorrect?

A There is only one sp hybridised carbon atom in a molecule of P.


B A molecule of P contains four bonds.
C After P reacts with HCl(g), all but one carbon atom in the product formed are
sp3 hybridised.
D After P reacts with LiAlH 4 , all the carbon atoms in the product formed are sp3
hybridised.

trendyline
@PJC 2016 9647/01/JC2/H2/Preliminary Exam/2016 [Turn over
971 trendyline
11
24 Industrially, ethyl ethanoate may be prepared from cheap, low-grade ethanol as
follows:

Cu catalyst Cu catalyst
CH3CH2OH CH3CHO CH3CH(OH)OCH2CH3
- H2 + CH3CH2OH
Cu catalyst
-H2

CH3CO2CH2CH3

Which process is included in the above reaction sequence?

A disproportionation
B electrophilic addition
C nucleophilic addition
D reduction

25 Which process will not give a good yield of CH 3 CO(CH 2 ) 3 CO 2 H?

A
Cl KOH in ethanol hot KMnO4 (aq), H3O+
heat

B KOH(aq) hot KMnO4 (aq), H3O+


H2NCO(CH2)3CHClCH3
heat

C NaOH in ethanol hot KMnO4 (aq), H3O+


CH3CH(OH)(CH2)3CH2Cl
heat

D
hot KMnO4 (aq), H3O+

trendyline
@PJC 2016 9647/01/JC2/H2/Preliminary Exam/2016 [Turn over
972 trendyline
12
26 Which of the following reactions would not produce 1,3-dicarboxylic acid?

A CHO

+ Fehling's solution followed by acidification

CHO

B H

C=CH2CH3

+ hot acidified potassium manganate(VII)

CO 2CH3

C CN

+ aq H2SO4, heat

CO 2CH3

D CH(OH)CH3

+ warm, alkaline aqueous iodine followed by acidification

COCH3

trendyline
@PJC 2016 9647/01/JC2/H2/Preliminary Exam/2016 [Turn over
973 trendyline
13
27 When heated strongly, phthalic acid undergoes condensation reaction to form
phthalic anhydride.
O O
C C
OH
O + H2O
OH
C C
O O
phthalic acid phthalic anhydride

On reaction with water, the anhydride gives phthalic acid. What would be formed
when phthalic anhydride reacts with ethanol instead of water?

A O B O
C C
OH OCH2CH3

OCH2CH3 OCH2CH3

C O D O

C C
OCH2CH3 OCH2CH3
OCH2CH3
OH
C
C
O
O

28 Experiments are carried out on three compounds Q, R, and S.


CH3(CH2)16CO2 CH2

CH3(CH2)16CO2 CH
BrCH2CH2CH2Br
CH3(CH2)16CO2 CH2 CN

Q X R
Y SZ
A sample of 0.01 mol of each compound is heated under reflux with 50 cm3 of
1 mol dm-3 aqueous NaOH (in excess) until hydrolysis is completed and any
ammonia gas produced was expelled from the solution. The excess NaOH was
then titrated in each case and was found to require 20 cm3, 30 cm3 and 40 cm3 of
1 mol dm-3 HCl for neutralisation.

Which sequence of compounds matches these results?

trendyline
20 c 3
0 cm 30 cm3 0 cm3
40
A Q R S
B Q S R
C R S Q
D S R Q

@PJC 2016 9647/01/JC2/H2/Preliminary Exam/2016 [Turn over


974 trendyline
14
29 Why are amides, RCONH 2 , less basic than amines, RNH 2 ?

A Amides form zwitterion in which the nitrogen atom carries a positive charge.
B Amides have a resonance structure involving the movement of a pair of
electrons from the nitrogen atom to the oxygen atom.
C Electrons on the nitrogen atom move on the C-N bond giving it some double
bond character so that it is more difficult to break.
D The amide carbonyl group withdraws electrons from the NH 2 group to make
the hydrogen atoms more acidic.

30 Silk fibroin is a type of fibrous protein made up almost entirely of pleated sheets.
The main amino acids found in silk fibroin are alanine, glycine, serine and cysteine.
The structures of these amino acids are given below.

H2N CH COOH H2N CH COOH H2N CH COOH H2N CH COOH


H CH3 CH2OH CH2SH

glycine alanine serine cysteine

Which of the following statements about the type of interactions present in silk
fibroin is incorrect?

A The pleated sheets are held together by hydrogen bonding.


B There are van der Waals forces between glycine and alanine in the protein.
C There are disulfide linkages present in silk fibroin.
D Serine forms ionic bonds with another serine amino acid in the protein.

trendyline
@PJC 2016 9647/01/JC2/H2/Preliminary Exam/2016 [Turn over
975 trendyline
15
Section B
For each of the questions in this section, one or more of the three numbered statements
1 to 3 may be correct.
Decide whether each of the statements is or is not correct (you may find it helpful to put a
tick against the statements that you consider to be correct).

The responses A to D should be selected on the basis of

A B C D
1, 2 and 3 are 1 and 2 only are 2 and 3 only are 1 only is correct
correct correct correct

No other combination of statements is used as a correct response.

31 A 0.0200 mol sample of an oxochloride of T, TOCl y required 10.00 cm3 of


0.400 mol dm-3 acidified potassium manganate(VII) for oxidation to TO 3 - ions.

Which of the following statements are correct?

1 5 mol TOCl y react with 1 mol MnO 4 -.


2 The change in oxidation state of T in the reaction is +1.
3 The value of y in TOCl y is 2.

32 The ion U5+ has 38 electrons and 46 neutrons.

Which of the following statements are correct?

1 In an electric field, the ion U5+ will be deflected to the same extent as the
yttrium ion, 89Y5+.
2 The electronic configuration of U5+ is [Kr] 4d2.
3 The third ionisation energy of element U is significantly higher than its second
ionisation energy.

trendyline
@PJC 2016 9647/01/JC2/H2/Preliminary Exam/2016 [Turn over
976 trendyline
16
The responses A to D should be selected on the basis of

A B C D
1, 2 and 3 are 1 and 2 only are 2 and 3 only are 1 only is correct
correct correct correct

No other combination of statements is used as a correct response.

33 The graph shows the results of an investigation of the initial rate of hydrolysis of
casein by the enzyme trypsin. In the experiments, the initial concentration of casein
was varied but that of trypsin was kept constant.

initial
reaction
rate

0 [casein]

Which conclusions can be deduced from these results?

1 When [casein] is low, the rate is first order with respect to [casein].
2 When [casein] is high, the rate is dependent of [trypsin].
3 When [casein] is high, the rate is dependent of [casein].

34 Button cells are tiny cells used to power small electronic devices such as wrist
watches and calculators. One such cell consists of lithium electrode and a paste of
manganese(IV) oxide electrode dipped in an organic electrolyte. The e.m.f.
produced by the cell is 3.00 V. In discharging the cell, LiMnO 2 will be formed.

Which of the following statements about the Li-MnO 2 cell are correct?

1 Electrons flow from the lithium electrode to the manganese(IV) oxide electrode.
2 The equation for the reaction at the cathode is MnO 2 + e MnO 2 -.
3 Water is added to manganese(IV) oxide to form the paste.

trendyline
@PJC 2016 9647/01/JC2/H2/Preliminary Exam/2016 [Turn over
977 trendyline
17
The responses A to D should be selected on the basis of

A B C D
1, 2 and 3 are 1 and 2 only are 2 and 3 only are 1 only is correct
correct correct correct

No other combination of statements is used as a correct response.

35 Which of the following graphs correctly represents the variation in the specified
property of the elements in Period 3?

1 0.25

0.2
atomic radii/nm

0.15

0.1

0.05

0
11 12 13 14 15 16 17 18
atomic number

2 0.25

0.2
ionic radii/nm

0.15

0.1

0.05

0
11 12 13 14 15 16 17
atomic number

3
0.25
conductivity

0.2
atomic radii/nm

0.15
electrical

0.1

ttrendyline
rendyline
ren
rendy
dyl
dy
yliline
l nee
0.05

0
11 12 13 14 15 16 17 18
umb
atomic number

@PJC 2016 9647/01/JC2/H2/Preliminary Exam/2016 [Turn over


978 trendyline
18
The responses A to D should be selected on the basis of

A B C D
1, 2 and 3 are 1 and 2 only are 2 and 3 only are 1 only is correct
correct correct correct

No other combination of statements is used as a correct response.

36 For the reaction,

(CH 3 ) 3 CX + CH 3 CH 2 O-  &+ 3 ) 3 COCH 2 CH 3 + X- (where X = Cl, Br or I)

which of the following statements are likely to be correct?

1 The reaction involves an electrophilic attack by CH 3 CH 2 O-.


2 The rate determining step involves the formation of a carbocation.
3 The reaction will take place more rapidly when X is I than when X is Cl.

37 The reaction of phenylhydrazine is similar to that of 2,4-dinitrophenylhydrazine.

CHO + NHNH2 products

phenylhydrazine

Which changes in bonding occur in the reactants and products during the reaction
above?

Bond broken Bond formed


1 CH CC
2 NH OH
3 . C=O C=N

trendyline
@PJC 2016 9647/01/JC2/H2/Preliminary Exam/2016 [Turn over
979 trendyline
19
The responses A to D should be selected on the basis of

A B C D
1, 2 and 3 are 1 and 2 only are 2 and 3 only are 1 only is correct
correct correct correct

No other combination of statements is used as a correct response.

38 Dopamine is a neurotransmitter found in many animals, including vertebrates and


invertebrates.

CO2H
HO CH2CH
NH2
HO

Dopamine

Which of the following statements are correct?

1 1 mole of dopamine reacts with 3 moles of acyl chloride


2 1 mole of dopamine reacts with 1.5 moles of Br 2 (aq).
3 1 mole of dopamine reacts with 3 moles of Na 2 CO 3.

39 Compound V, with molecular formula C 6 H 12 , was reacted with acidified potassium


manganate(VII) and the resultant mixture obtained was colourless. The mixture was
thereafter distilled to form distillate W and residual solution X.

Distillate W is able to form a yellow precipitate with alkaline aqueous iodine.


Residual solution X reacts with sodium carbonate to form a colourless gas.

Which of the following statements are true?

1 Compound V will decolourise aqueous bromine.


2 Distillate W can form an orange precipitate with 2,4-dinitrophenylhydrazine.
3 Residual solution X can form white fumes with PCl 5 .

trendyline
@PJC 2016 9647/01/JC2/H2/Preliminary Exam/2016 [Turn over
980 trendyline
20
The responses A to D should be selected on the basis of

A B C D
1, 2 and 3 are 1 and 2 only are 2 and 3 only are 1 only is correct
correct correct correct

No other combination of statements is used as a correct response.

40 Benzoylglycine (hippuric acid) was first isolated from stallions urine.

CONHCH2CO 2H

Benzoylglycine

Which properties does this compound possess?

1 It can be hydrolysed to produce an amino acid.


2 It can be made by reacting benzoyl chloride with aminoethanoic acid.
3 It can be neutralised by reaction with cold aqueous sodium hydroxide.

End of Paper

trendyline
@PJC 2016 9647/01/JC2/H2/Preliminary Exam/2016 [Turn over
981 trendyline
H2 Chemistry Preliminary Examination Paper 1 Worked solution

1 D Option A is wrong: the number of atoms in 24 dm3 of nitrogen gas at r.t.p. condition
24 dm3 N 2 1 mol N 2 2 mol N 2 x L

Option B is wrong: the number of molecules in 1 dm3 of oxygen at 273 K and 1 atm
1 dm3 O 2 1/22.4 mol O 2 1/22.4 x L

Option C is wrong: the number of electrons removed to form Ne+ ions from 6.02 x 10-23 mol
of Ne atoms
No of electrons removed = (6.02 x 1023) x (6.02 x 10-23) = 6.022 electrons

Option D is correct: the number of ions in 83.5 g of [Co(NH 3 ) 5 Cl]Cl 2


Amount = 83.5/250.4 = 1/3 mol
1 mol [Co(NH 3 ) 5 Cl]Cl 2 3 mol ions
1/3 mol [Co(NH 3 ) 5 Cl]Cl 2 1 mol ions = L
2 B Fe : [Ar] 3d6 4s2 (one unpaired d-orbital)
Option A is wrong: 26
2
Option B is correct: 32 Ge : [Ar] 3d10 4s2 (no unpaired, all paired)

Option C is wrong: 14 Si : [Ne] 3s2 3p2 (two unpaired p-orbitals)

Option D is wrong: Ti3 : [Ar] 3d1 (one unpaired d-orbital)


22

3 C By VSEPR theory, BCl 3 has 3 bond pairs, and no lone pairs. This give rise to trigonal planar
structure.
PH 3 has 3 bond pairs and 1 lone pair, therefore it is trigonal pyramidal structure.
4 A The molecules in option A are a pair of enantiomers. Enantiomers are known to have
identical chemical and physical properties, with the exception of its ability to rotate plane
polarised light. Therefore the enantiomers will have identical boiling point.
5 B RT
Option A is wrong : P =
Mr
P
kT = k (toC + 273) (similar to y = mx + c)

Plot wrong as y-intercept is zero. Note temp axis is in C and y-intercept = 273k.
P
Option B is correct: kT = k (t + 273) (similar to y = mx + c)

trendyline
Option C is
s wrong:
w Not a linear plot of y = mx + c

Option D is
s wrong: Not a linear plot of y = mx + c

982 trendyline
6 C One mol of neon gas at temperature T 1 was added to another one mol of neon and the
temperature was increased to T 2 .
As such, the area under the curve should increase since there is an increase in the number
of moles of gases.
When temperature increases to T 2, the curve should tend to the right as the molecular speed
should increase at a higher temperature.
Option C is correct
7 C H = (5(BE(N=N) + BE(N=O)+2BE(C-H) + BE(CC)) (2BE(O-H) + 4BE(C=O) +
5(NN))
= (5(418 + 686)+2(410) + (840)) (2(460) + 4(740) + 5(994))
= - 1670 kJ mol-1
8 B
No. of quarter life Amount of 40K left
0 100%
1 25%
2 6.25%

Since two quarter life have passed, the time required would be 2.5 x 109 x 2 = 5.0 x 109
years
9 D 0.785 g Cr 0.0151 mol Cr
8690 C 0.09005 mol e
0.0151 mol Cr 0.09005 mol e
1 mol Cr 5.96 mol e 6 mol e

Crn+QH&U
n=6
Option A is wrong : OS of Cr = +2
Option B is wrong : OS of Cr = +3
Option C is wrong : OS of Cr = +4
Option D is correct: : OS of Cr = +6

10 B Cu(s) | Cu2+(aq, 1 mol dm3) || A+(aq) | A(s)


Option A is wrong as statement is correct : The Cu2+/Cu half-cell is the anode and Cu is the
negative electrode
Option B is correct as statement is incorrect : [A+(aq)] = 1.00 x 107 and log [A+(aq)] = -7.
From the graph, emf> 0 and the direction of electron flow will not be reversed.
Option C is wrong as statement is correct : E cell = E red + E ox +0.46 = E red + (-0.34) E red

trendyline
y
= + 0.8
0.80V
Option D iss wrong as statement is correct : At st
std [A+(aq)] = 1.00 mol dm-3
d conditions, [A
+
[A (aq)] = 0 emf = +0.46 V (from graph)

983 trendyline
11 B D(g) E(g) + F(g)
Initial pressure/atm p 0 0
Eqm pressure/atm1p3p3p 7 7 7
Note : Total pressure = p (constant)
3
( atm)2
Kp = 7 = 9p/7atm
1
( atm)
7
12 B Method 1: Calculation confirmation method
pK a = 9.5
K a = 10-9.5
[paracetamol] = [(12.78 x 10-3)/151] / (1 x 10-3) = 0.084 mol dm-3
[anion][H ]
Ka
[paracetamol]
[anion][H ]
109.5
[0.0846]
[ H ] 5.17 x106
pH lg[ H ] 5.29

Method 2: Elimination method smart guess


Paracetamol behaves as weak acid, which undergoes partial dissociation
hence pH must be near to 7
Most probable pH is 5.29

13 A pH = 14- pK b
pK b = 14- pH
pK b = pOH
i.e. max buffer capacity

Option A is correct
14 D NH 4 + is the conjugate acid of a weak base, NH 3 .

NH 4 + dissociates partially in water to produce H 3 O+ as follows:


NH 4 + + H 2 O NH 3 + H 3 O+

trendyline
yll
y
Mg(OH) 2 (s)) tthen undergoes an acid base reaction (neutralisation) with the H 3 O+ ions
acidbase
produced.
g(OH) 2 (s) + 2H 3 O+(aq)
Mg(OH) aq Mg2+(aq)
q) + 4H 2 O
O(l)

984 trendyline
15 D A saturated solution of Ca(OH) 2 is found to have a pH of 12.3 at 25 C.

Ca(OH) 2 Ca2+ + 2OH-.. eqm (1)

Option A is correct:K sp of Ca(OH) 2 = [Ca2+][OH-]2 = (1/2 x 10-1.7) (10-1.7)2 = 4 10-6 mol3 dm-9.

Option B is correct: The solubility of Ca(OH) 2 would increase when temperature is raised to
35 C.

Option C is correct: When solid Na 2 O is added, NaOH is formed, hence [OH-] increases. As
such, POE of eqm (1) above will shift to the left, hence [OH-] decrease. The solubility of
Ca(OH) 2 will decrease.

Option D is incorrect: When Ca(NO 3 ) 2 is added, [Ca2+] increases. As such, POE of eqm
(1) above will shift to the left, hence [OH-] decrease. The pH of the solution would decrease.

16 A Important information in question : high charge-density cations


Only AlCl 3 and MgCl 2 will dissolve in water to give cations, Al3+ and Mg2+
Hence, most effective high charge-density cation is Al3+
17 B All of the elements burns in presence of oxygen to give respective oxides. All the metal
oxides formed will be soluble in water but SiO 2 is insoluble in water.
Al 2 O 3 is amphoteric oxide which reacts with NaOH, ie soluble in excess NaOH through
formation of the following complex
Al 2 O 3 (s) + 2NaOH(aq) + 3H 2 O(l 1D>$l(OH) 4 ](aq)
Both K 2 O and CaO are basic oxide which do not react with NaOH, thus remains as white
ppt.
However, only CaO will react with H 2 SO 4 to give a sparingly soluble white ppt.
CaO(s) + H 2 SO 4 DT &D62 4 (s) + H 2 O(l)

18 C Thermal decomposition of hydrogen halides, HX, involves the breaking of the covalent
H-X bond.
General trend of Group VII halides: Going down Group VII, the valence np orbital of the
halogen atom is larger and more diffused. Thus, the overlap of orbitals between the 1s
orbital of H and the np orbital of halogen atom becomes less effective. Consequently, the
H-X bond is longer and weaker as indicated by the bond energies. Less energy is
required to break the H-X bond going down the group, i.e. thermal stability decreases.
Conclusion : HClbond is stronger than HI bond.

19 D Option A :E cell = -0.59V, reaction is not feasible.

Option B: hexacyanoferrate(III) cannot be further oxidised.

Option C: Sn is not a transition element, therefore while it can be oxidised from Sn2+ to Sn4+,
there is no color change.

Option D is
3+
to V which
correct:
s correct
h is violet.
y
corre : E cell = 0.97V, therefore reaction is feasible. V2+ is g
green, and it changes

985 trendyline
20 C [Cu(H 2 O) 6 ] 2+ + 6 Cl- [CuCl 4 ]2-+ 6H 2 O
Blue yellow

The oxidation state of Cu is +2 in both complexes; therefore the number of d electrons will
be the same.

Different ligand splits the field differently and thus accounting for the different colors. The
energy gap is going to be different.

Thus option C is the only correct answer.


21 D The graph shows that the reactants are of lower energy than the products. Hence, the best
possible answer will be option D as the reactant which is Cl 2 will be of lower energy than the
product which are the highly reactive radicals.
22 D When treated with H 2 , Ni cat at 200oC, the product will be

As shown in the diagram, there are 9 chiral carbons in the product.


23 D Option A:The CN in compound P has sp hybridised carbon atom. (Correct)
Option B7KHSLERQGVLQWKHWZRGRXEOHERQGVSLERQGVLQ&1. (Correct)
Option C: Reaction with HCl(g) will lead to all the alkenes in the compounds to undergo
electrophilic addition and the carbon will become a sp3 hybridised except for -&1(Correct)
Option D: LiAlH 4 will not be able to reduce the C=C, hence not all the carbons in the
product formed will not be sp3 hybridised. (Incorrect)

24 C Cu catalyst Cu catalyst
CH3CH2OH CH3CHO CH3CH(OH)OCH2CH3
- H2 + CH3CH2OH
Cu catalyst
-H2

CH3CO2CH2CH3
First step: oxidation

trendyline
Second step: p: nucleophilic addition
Third step: oxidation

986 trendyline
25 C

26 A The rest of the options will produce 1,3-dicarboxylic acid.

27 C Using the example of phthalic anhydride producing phthalic acid with water, ethanol should
follow the same reaction as water. The general formula of the reaction is shown in the
equation below.

C 6 H 4 (CO) 2 O (anhydride) + ROH C 6 H 4 (COOH)COOR [R CH 3 CH 2 -]


28 B Q 1 mol of Q requires 3 mol of NaOH for the hydrolysis process.
Amount of NaOH left after hydrolysis = (50/100 x 1) 0.03 = 0.02 mol
1 mol NaOH requires 1 mol of HCl for neutralisation
Vol of HCl required to neutralise the remaining NaOH = 0.02 / 1 = 0.02dm3= 20cm3

R 1 mol of R requires 1 mol of NaOH for the hydrolysis process.


Amount of NaOH left after hydrolysis = (50/100 x 1) 0.01= 0.04 mol
1 mol NaOH requires 1 mol of HCl for neutralisation
Vol of HCl required to neutralise the remaining NaOH = 0.04 / 1 = 0.04dm3 = 40cm3

S 1 mol of S requires 2 mol of NaOH for substitution of 2 Br.

trendyline
Amount
unt of NaOH left after hydrolysis = (50/100 x 1) 0.02 = 0.03 mol
ount
1 mol
ol NaOH requires 1 mol of HCl for neutralisation
quired to neutralise the remaining NaOH = 0.03 / 1 = 0.03dm3= 30cm3
Vol off HCl required

987 trendyline
29 B Option A: Amino acids form zwitterions not amides.

Option B: Basicity depends on the availability of lone pairs of electron on nitrogen for
coordination with proton. The lone pair of electrons on the N atom can be delocalised with
the electrons of the C=O double bond. This creates the resonance structure. This makes
the amides less basic than amines.

Option C: Need to concentrate on the availability of lone pairs of electrons on nitrogen, not
on how it is more difficult to break the C-N bond.

Option D: The powerful electron-withdrawing effect of the carbonyl group C=O reduces
the electron density on the N atom, making it less effective as a proton acceptor. Not make
the hydrogen atoms more acidic.
30 D Option A: True

Option B: Both glycine and alanine have non polar R groups, hence would have van der
Waals forces between them.

Option C: Cystein is a sulfur containing amino acid which is capable of forming disulfide
linkages between the cysteine molecules.

Option D: Serine forms hydrogen bonds with another serine amino acid in the protein as the
side group is an alcohol group.
31 A Option 1 is correct : 0.0200 mol T 0.00400 mol MnO 4 -
5 mol T 1 mol MnO 4 -

Option 2 is correct : 5 mol T 1 mol MnO 4 - 5 mol e


1 mol T 1 mol e
1 mol T lost1 mol e
Change in OS of T =+1

Option 3 is correct : TOCl y TO 3 -


+4 +5

y
OCl y = +4
OC
OS of T inTOCl
In TOCl y , +4 + ((-2)
2) + (y (-1))
( )) = 0
y=2

988 trendyline
89 5
32 A U
Option 1 is correct: The ion U5+ has the symbol 43 and thus the same charge by mass
ratio as yttrium ion,89Y5+. Both deflected to same extent.

Option 2 is correct:U is the technetium, T c . Electronic configuration of T c is[Kr] 4d5 5s2 and
that of U5+is[Kr] 4d2.

Option 3 is correct :U2+ (g) U3+ (g) + e (3rd IE)


[Kr] 4d5[Kr] 4d4
The energy required to remove the a mol of e from U2+ (g) will be significantly higher than
2nd IE as it involves the removal of electron from an inner quantum shell.

33 B Trypsin is an enzyme that catalyses the reaction. When the [substrate] (casein) is low,
increasing the concentration of casein will increase the rate of reaction as there are plenty of
active sites available on the catalyst, therefore its first order with respect to casein.

However, as the concentration of casein rise, the active sites in the enzyme will be
saturated, and therefore, increasing the concentration of casein will have no effect on the
rate of reaction, thus its a zero order reaction.
34 B Option 1 is correct:In discharging the cell, Li+ MnO 2 - will be formed. Li is oxidised to Li+ and
electrons released will flow to the MnO 2 electrode.

Option 2 is correct: At the MnO 2 electrode (cathode), MnO 2 gains electron and is reduced to
MnO 2 -.

Option 3 is wrong: Water cannot be used as it will react with Li, a Group I metal.
35 A Graph 1 is correct: (Explanation can be found in Pg 4 Periodicity notes)
The atomic radii of the elements decrease across period 3 gradually. Across a period,
there is an increase in the effective nuclear charge, due to increasing number of
protons in the nucleus and approximately constant shielding effect.
The radius of Ar is, in fact, the van der Waals radius between two atoms. Hence, largest
radius.

Graph 2 is correct: explanation will use actual element and its ion to minimise confusion (Pg
4 Periodicity notes)
The cationic radius decreases from atomic number 11 to 14 (which is actually Na+ to
Si4+) (Electronic configuration: 1s2 2s2 2p6).
- Nuclear charge increases from Na+ to Si4+ due to an increase in protons.
- Shielding effect remains approximately constant as these cations are isoelectronic.
- Due to the increase in nuclear charge, the valence electrons in these ions are held
more tightly to the nucleus, accounting for a smaller cationic radius.

trendyline
n radius decreases from Si4- to Cl- (Electronic
The anionic
onic configuration 1s2 2s2 2p6 3s2
ectronic configuration:
configurat
6
3p )
arly for the anions (Si4-, P3-, S2- and Cl
- Similarly
ila Cl-) are isoelectronic (constant shielding).
The decrease in anionic radii is also due to the increase in nucle
nuclea
nuclear charge.
Radii of anions > Radii of cations because se of
o an extra shell of electr
electrons in the anions.

989 trendyline
Graph 3 is correct: Explanation can be found in Pg 5 Periodicity notes
Na, Mg and Al are metals.
Conductivity increases from Na to Al due to an increase in the number of valence
electrons donated to the delocalised electron cloud by the metal atoms.
Si is a metalloid (having both properties of metals and non-metals). It has low
conductivity. It is a semi-conductor.
P, S and Cl are non-metals. The electrons are localised in covalent pairs and are unable
to move to conduct electricity. They have very low conductivities. They are non-
conductors (insulators).

Na Mg Al Si P4 S8 Cl 2
Metals Metalloid Non-Metals

36 C Option 1: The reaction involves nucleophilic attack not electrophilic attack.


Option 2: The RX compound is a tertiary RX hence it will be undergoing SN 1 mechanism
which involves the formation of a carbocation in the slow step.
Option 3: The reaction involves the cleaving of C-X bond so if the X is I, the reaction will take
place rapidly as the C-I bond is weaker than C-Cl bond.
37 C During the reaction, C=O bond from benzaldehyde and 2 N-H bonds from phenylhydrazine
will be broken. The products will have C=N bond and O-H (from water) being formed.
38 D Option 1: One mole of dopamine has 1 NH 2 group and 2 OH groups. Hence, one mole of
dopamine will react with 3 moles of RCOCl. ( the NH 2 group will form amide with RCOCl
and the OH group will form ester with RCOCl).
Option 2: Dopamine will undergo electrophilic substitution with 3 moles ofBr 2 (aq) wrt to the
two phenol groups.

Option 3: One mole of dopamine will react with one mole of Na 2 CO 3 as only the COOH
group in the compound will react.
39 A Distillate W forms a yellow ppt with alkaline aq iodine W has C=O group.
X reacts with Na 2 CO 3 X has COOH group.
Option 1: Given the formula of V, V should have unsaturated bonds in it so it should be able
to decolourise aq bromine.
Option 2: W is a carbonyl compound so it should be able to form orange ppt with 2,4-DNPH.
Option 3: X has COOH group so it should be able to react with PCl 5 .
40 A Option 1: It can be hydrolysed to form Benzoic acid and aminoethanoic acid.

trendyline
Option 2: Benzoyle oy cchloride
o de a and
d a aminoethanoic
oet a o c acid
ac d can
ca undergo
u nucleophilic
/condensation
condensatio to form benzoylglycine and HCl
substitution/condensation HC as the
t e products.
product
p odu
Option 3: Cold
old NaOH(aq) can react with
wi COOH
COOH group through acid
acid-base
bas reaction.
acid-bas

990 trendyline
PIONEER JUNIOR COLLEGE

JC2 PRELIMINARY EXAMINATION


HIGHER 2

CANDIDATE
NAME

CT INDEX
1 5 S
GROUP NUMBER

CHEMISTRY 9647/02
Paper 2 Structured 13 September 2016
2 hours
Additional Materials: Data Booklet

READ THESE INSTRUCTIONS FIRST

Write your class, index number and name on all work you hand in.
Write in dark blue or black pen on both sides of the paper.
You may use a soft pencil for any diagrams, graphs or rough workings.
Do not use staples, paper clips, highlighters, glue or correction fluid.

Answer all questions.

At the end of the examination, fasten all your work securely together.
The number of marks is given in brackets [ ] at the end of each question or part question.

FOR EXAMINERS USE


Paper 2
1 /12
2 /12
3 /14
4 /8
5 /9
6 /17
Penalty sf units

trendyline
y ne
Total
/72

This document consists of 17 printed pages.

PJC 2016 9647/02/JC2/H2/Preliminary Exam/2016 [Turn Over


991 trendyline
2
Answer all questions in the spaces provided.

1 Planning (P)

The formation of a cobalt(II) complex ion, from Co2+ and an unknown ligand(L-), is
accompanied by the appearance of a blue colouration.

Co2+ + 4L- >&R/ 4 ]2-

A machine, known as a spectrometer, is able to measure the amount of light that is


absorbed when a specific wavelength of visible light is shone through a few cm3 of the
solution. The amount of light absorbed is then expressed as an absorbance value, and
the absorbance value is proportional to the concentration of the cobalt(II) complex ion.

This technique can be used to confirm the formula of the cobalt(II) complex ion. A
series of solutions containing varying volume ratios of L- to Co2+, both of the same
concentration, while keeping the total volume of the Co2+ and L- constant, is prepared.
A spectrometer is used to measure the absorbance of the solutions. A graph of
absorbance value against volume of L- is then plotted.

Using the graph of absorbance value against volume of L-, the ratio of L- to Co2+ equal
to that in the cobalt(II) complex ion can be then determined from the maximum point.
This is known as the Jobs Method.

(a) The spectrometer is set to use the wavelength of the light that is absorbed most
strongly by the complex ion.

Colour Wavelengths (nm)


Red 620-750
Orange 590-620
Yellow 570-590
Green 495-590
Cyan 475-495
Blue 450-475
Violet 380-450

Suggest a wavelength in the visible spectrum from which a suitable wavelength of


light might be chosen. Explain your answer.

trendyline
.
.

[2]

PJC 2016 9647/02/JC2/H2/Preliminary Exam/2016 [Turn Over


992 trendyline
3
(b) Using the information given above, you are required to write a plan to confirm the
formula of the cobalt(II) complex ion as [CoL 4 ]2-.

You may assume that you are provided with:


solid cobalt(II) nitrate, Co(NO 3 ) 2 .6H 2 O;
solution of L-, of concentration 0.100 mol dm-3;
access to a spectrometer and instructions for its use;
graph paper;
the apparatus and chemicals normally found in a school or college laboratory.

Your plan should include details of:


the preparation of 250 cm3 of 0.100 mol dm-3 aqueous cobalt(II) nitrate;
the preparation of five 50 cm3 complex ion solutions containing varying volume
ratios of L- to Co2+;
how the absorbance values would be obtained;
a sketch of the graph of absorbance value against volume of L- that you would
expect to obtain;
how the calculation can be performed to determine the formula of [CoL 4 ]2-.

....

....

....

....

....

....

....

....

....

....

....

....

....

trendyline
....
...
..

....
...

....
...

....
PJC 2016 9647/02/JC2/H2/Preliminary Exam/2016 [Turn Over
993 trendyline
4

....

....

....

....

....

....

....

....

....

....

....

....

....

....

....

....

....

....

....

....

....

....

....

trendyline
....
...

....
...
...
[10]
[Total: 12]

PJC 2016 9647/02/JC2/H2/Preliminary Exam/2016 [Turn Over


994 trendyline
5
2 The mission of the Hydrogen and Fuel Cells Programme by the U.S. Department of
Energy (DoE) is to reduce petroleum use, greenhouse gas emissions, and air pollution
and to contribute to a more diverse and efficient energy infrastructure by enabling the
widespread commercialisation of hydrogen and fuel cell technologies.

One such technology involves creating an energy system concept that stores hydrogen
in the form of metal hydrides. Magnesium hydride (MgH 2 ) is one of the compounds
currently being researched on its viability on hydrogen storage. One way to release
hydrogen from metal hydrides is to react them with water, where a hydroxide is formed
as a side product.

The other way to obtain hydrogen is through the thermal decomposition of the metal
hydride MH 2 . When hydrogen is needed, these metal hydrides can be heated to
release molecular hydrogen.

MH 2 M + H2
This adsorption and desorption of hydrogen in metals serves like a hydrogen bank.
The rate of desorption of hydrogen is proportional on the strength of ionic bonds formed
between the metal and hydride ions.

Apart from MgH 2 , other Group II hydrides are also being studied and researched on the
same purpose. The table below shows some properties of the Group II hydrides.

MgH 2 CaH 2 SrH 2 BaH 2


Percentage of H / % by mass of H 7.7 4.8 2.2 1.4
Density / g cm-3 1.74 1.54 2.64 3.62
Melting point / & 650 842 777 727
H f / kJ mol-1 -114 -144 -119 -118

(a) (i) Write an equation for the reaction between solid MgH 2 and water.

..
[1]

(ii) Draw the dot-and-cross diagram for MgH 2 .

trendyline [1]

PJC 2016 9647/02/JC2/H2/Preliminary Exam/2016 [Turn Over


995 trendyline
6
(b) (i) Explain, with reasoning, the trend on the rate of desorption of hydrogen for
the Group II metal hydrides from CaH 2 to BaH 2.

..

..

..

..
[2]

(ii) Which metal hydride, CaH 2 or BaH 2, would you expect to decompose more
readily? Explain your answer by using relevant data from the above table.

..

..

..

..
[2]

(iii) Using relevant information from the Data Booklet and the information given
below, construct a Born-Haber cycle to calculate the lattice energy for
CaH 2 .
(The first electron affinity for hydrogen is -72.8 kJ mol-1 and the enthalpy
change of atomisation of calcium is +178 kJ mol-1)

trendyline
..

[3]

PJC 2016 9647/02/JC2/H2/Preliminary Exam/2016 [Turn Over


996 trendyline
7
(c) A car company is planning to develop cars that run on hydrogen fuel produced by
MgH 2 .

The following information is for a typical petrol-fuelled car.

Weight of vehicle / kg 1088


Fuel tank / L 42
Fuel consumption / km L-1 17
Energy consumption of petrol / kJ kg-1 46.4
Density of petrol / g cm-3 0.8 g

(i) Calculate the energy produced by a petrol-fuelled car running at full tank.

[1]

(ii) It was found that 1 kg of hydrogen produce 3 times more energy than petrol
with the same mass.

Assuming 100% efficiency, calculate the amount of magnesium metal


needed to store the hydrogen as the hydride to provide the same amount of
energy in the hydrogen-fuelled car. (1 L = 1000 cm3)

MgH 2 Mg + H 2

trendyline [2]
[Total: 12]

PJC 2016 9647/02/JC2/H2/Preliminary Exam/2016 [Turn Over


997 trendyline
8
3 (a) In the presence of excess oxygen, a liquid alcohol, compound A, C x H y O,
undergoes complete combustion to produce 3.19 dm3 of hot gases at a
temperature of 250 C at a pressure of 1 atm,

Upon cooling to room temperature, the volume contracted to 836 cm3. The gases
were then passed through aqueous sodium hydroxide and the final volume
remaining was 100 cm3.

(i) Fill in the following blanks for the equation for the complete combustion of
A.

C x H yO + O2 CO 2 + H2O
[1]

(ii) Using the ideal gas equation, find the amount of CO 2 and amount of H 2 O
that was produced respectively from the complete combustion of compound
A.

[3]

(iii) Using your answer to (a)(i) and (a)(ii), calculate the value of x and y and
hence, write down the empirical formula for A.

[1]

(ii) The actual number of moles of gases obtained from the combustion is
found to be higher than that calculated when using the ideal gas equation.

Account for the difference between the two results.

trendyline
.

.


[1]

PJC 2016 9647/02/JC2/H2/Preliminary Exam/2016 [Turn Over


998 trendyline
9
(b) Methane can be obtained when carbon monoxide is heated to 900 C with
hydrogen gas as shown in the following equation.

CO(g) + 3H 2 (g) CH 4 (g) + H 2 O(g)

(ii) Write an expression for the K c of the reaction, stating its units.

[1]
(iii) 1 mol of carbon monoxide was heated with 4 mol of hydrogen gas in a
10 dm3 vessel. At equilibrium, the vessel contained 0.387 mol of methane.

Calculate the K c for the reaction of carbon monoxide.

[2]

(iv) Suggest, with an explanation, how the position of equilibrium might alter
when 1 mol of helium was added into the vessel.

[1]

trendyline
PJC 2016 9647/02/JC2/H2/Preliminary Exam/2016 [Turn Over
999 trendyline
10
(c) Anhydrous aluminium sulfate is commonly used in organic reactions to remove
water. It decomposes at 600 C to produce an insoluble white solid and misty
fumes. The white solid dissolves in both aqueous sodium hydroxide and aqueous
hydrochloric acid while the misty fumes turn moist blue litmus red.

(i) Suggest the identities of the white solid and the misty fumes. Hence, write a
balanced equation for the decomposition of aluminium sulfate.

.
[2]

(ii) Write equations for the reactions of the white solid with aqueous sodium
hydroxide and hydrochloric acid.

.
[2]
[Total: 14]

4 Batteries for hearing aids comprise of zinc and carbon electrodes. These electrodes in
an electrolyte, maintained at a pH of 11, forms a galvanic cell in which zinc is oxidised
and oxygen from the air is reduced.

External circuit

Carbon

Rod
Electrolyte gel

Zinc Electrode

(a) Write a balanced equation for the overall chemical reaction and calculate the
E cell of this cell.

trendyline [2]

PJC 2016 9647/02/JC2/H2/Preliminary Exam/2016 [Turn Over


1000 trendyline
11
(b) Indicate, on the diagram on page 10, the polarity of both electrodes and the
direction of electron flow in the external circuit.
[2]

(c) Explain how the e.m.f. of the cell would change if the pH of the electrolyte
decreases.


[2]

(d) If a current of 4 x 10-5 A was drawn from the cell, calculate how long a zinc
electrode weighing 2 g will last before it needs to be replaced. Give your answer
correct to the nearest day.

[2]
[Total: 8]

trendyline
PJC 2016 9647/02/JC2/H2/Preliminary Exam/2016 [Turn Over
1001 trendyline
12
5 Insulin is a hormone made by the pancreas that allows your body to use glucose from
carbohydrates in the food that you eat for energy. Insulin helps keeps your blood sugar
level from getting too high or too low.

The insulin molecule consists of 51 amino acid residues, in two polypeptide chains.
H2N

Section X

H2N

CO2H

CO2H

Some information on the amino acids of insulin is given below.

amino acid Abbreviated Formula of side chain pI


name (R in RCH(NH 2 )CO 2 H)
arginine arg -(CH 2 ) 3 NHC(NH 2 )=NH 10.76
cysteine cys -CH 2 SH 5.07
glutamic acid glu -CH 2 CH 2 COOH 3.22
glycine gly -H 5.97

(a) Draw the structural formula of section X in insulin.

trendyline [1]

PJC 2016 9647/02/JC2/H2/Preliminary Exam/2016 [Turn Over


1002 trendyline
13
(b) The diagram shows the results of electrophoresis on a mixture of the amino acids
obtained from hydrolysis of section X at pH 6.0.

Draw the structure of the species responsible for the spots labelled R, S and T.

R S T

[3]

(c) The polypeptide chains of insulin coil to form short sections of -helix which
stabilises the secondary structure.

Describe with the aid of a sketch, how a polypeptide chain is held in the shape of
DQ-helix.

trendyline
[3]

PJC 2016 9647/02/JC2/H2/Preliminary Exam/2016 [Turn Over


1003 trendyline
14
(d) The three-dimensional structure of insulin is further stabilised by disulfide
linkages. Write an equation for the formation of such linkage and state the type of
reaction involved.

..................................................................................................................................

..................................................................................................................................
[2]
[Total: 9]

6 Lactones are cyclic esters. They are formed by intramolecular esterification of the
corresponding hydroxycarboxylic acids.

(a) (i) The reaction scheme below shows how lactone E is synthesised from
compound D.

Complete the reaction scheme by giving the structural formulae of the


intermediate organic products in the spaces provided and stating the
reagents and conditions for steps I, III, IV, V and VI.

O O

I II Br

Al2O3, 400oC

III

V IV
NC

VI

trr
ttrendyline
O

PJC 2016 9647/02/JC2/H2/Preliminary Exam/2016 [Turn Over


1004 trendyline
15

Step Reagents and conditions

III

IV

VI
[8]

(ii) Suggest a simple chemical test to distinguish compound D from E.

..

..

..

..
[2]

(iii) Name and outline the mechanism when compound D is treated with HCN in
the presence of aqueous NaOH at 10 oC.

trendyline [3]

PJC 2016 9647/02/JC2/H2/Preliminary Exam/2016 [Turn Over


1005 trendyline
16
(b) Lactones with three- or four-membered rings are very reactive compared to five-
or six-membered rings, making their isolation difficult.

Suggest why lactones with three- or four-membered rings are very reactive.

.
[1]

(c) Iodolactonisation reaction was first reported by M. J. Bougalt in 1904 and has
since become one of the most effective ways to synthesise lactones.

An example of the iodolactonisation is the reaction of 4-pentenoic acid as shown


below.

O I2, NaHCO3
I
OH O
O

Two simplified steps in the iodolactonisation mechanism are given below.

Step 1
O O
+ HCO3- + H2O + CO2
OH -
O

Step 2
O slow Intermediate fast I
+ I2 + I-
- F O
O O

(i) Suggest the role of HCO 3 - in step 1 of the mechanism.

..
[1]

trendyline
PJC 2016 9647/02/JC2/H2/Preliminary Exam/2016 [Turn Over
1006 trendyline
17
(ii) The type of reaction in Step 2 of the mechanism is electrophilic addition.

Draw the structure of intermediate F.

[1]

(iii) Suggest the structural formulae of the final organic product formed when
CH 2 =CHCH(CH 3 )CH 2 CH 2 COOH undergoes iodolactonisation in a similar
process as above.

[1]
[Total: 17]

End of Paper

trendyline
PJC 2016 9647/02/JC2/H2/Preliminary Exam/2016 [Turn Over
1007 trendyline
PIONEER JUNIOR COLLEGE

JC2 PRELIMINARY EXAMINATION


HIGHER 2

CANDIDATE
NAME

CT INDEX
1 5
GROUP NUMBER

CHEMISTRY 9647/02
Paper 2 Structured 13 September 2016
2 hours
Additional Materials: Data Booklet

READ THESE INSTRUCTIONS FIRST

Write your class, index number and name on all work you hand in.
Write in dark blue or black pen on both sides of the paper.
You may use a soft pencil for any diagrams, graphs or rough workings.
Do not use staples, paper clips, highlighters, glue or correction fluid.

Answer all questions.

At the end of the examination, fasten all your work securely together.
The number of marks is given in brackets [ ] at the end of each question or part question.

FOR EXAMINERS USE


Paper 2
1 / 12 5 / 9
2 / 12 6 / 17
3 / 14 Penalty sf units
4 / 8 Total / 72

trendyline
y
This document consists of 19 printed pages and 1 BLANK page.

1008 trendyline
2
Answer all questions in the spaces provided.

1 Planning (P)
The formation of a cobalt(II) complex ion, from Co2+ and an unknown ligand (L-), is
accompanied by the appearance of a blue colouration.

Co2+ + 4L- >&R/ 4 ]2-

A machine, known as a spectrometer, is able to measure the amount of light that is


absorbed when a specific wavelength of visible light is shone through a few cm3 of the
solution. The amount of light absorbed is then expressed as an absorbance value, and
the absorbance value is proportional to the concentration of the cobalt(II) complex ion.

This technique can be used to confirm the formula of the cobalt(II) complex ion. A
series of solutions containing varying volume ratios of L- to Co2+, both of the same
concentration, while keeping the total volume of the Co2+ and L- constant, is prepared.
A spectrometer is used to measure the absorbances of the solutions. A graph of
absorbance against volume of L- is then plotted.

Using the graph of absorbance against volume of L-, the ratio of L- to Co2+ equal to that
in the cobalt(II) complex ion can be then determined from the maximum point. This is
known as the Jobs Method.

(a) The spectrometer is set to use the wavelength of the light that is absorbed most
strongly by the complex ion.

Colour Wavelengths (nm)


Red 620-750
Orange 590-620
Yellow 570-590
Green 495-590
Cyan 475-495
Blue 450-475
Violet 380-450

Suggest a wavelength in the visible spectrum from which a suitable wavelength of


light might be chosen. Explain your answer.
[2]
Any value from 591 to 619 nm.

The observed colour of the cobalt complex ion is blue thus the colour
complementary to blue, orange, will be absorbed.

trendyline
PJC 2016 9647/02/JC2/H2/Preliminary Exam/2016 [Turn Over
1009 trendyline
3
(b) Using the information given above, you are required to write a plan to confirm the
formula of the cobalt(II) complex ion as [CoL 4 ]2-.

You may assume that you are provided with:


solid cobalt(II) nitrate, Co(NO 3 ) 2 .6H 2 O;
solution of L-, of concentration 0.100 mol dm-3;
access to a spectrometer and instructions for its use;
graph paper;
the apparatus and chemicals normally found in a school or college laboratory.

Your plan should include details of:

the preparation of 250 cm3 of 0.100 mol dm-3 aqueous cobalt(II) nitrate;
the preparation of five 50 cm3 complex ion solutions containing varying volume
ratios of L- to Co2+;
how the absorbance values would be obtained;
a sketch of the graph of absorbance against volume of L- that you would
expect to obtain;
how the calculation can be performed to determine the formula of [CoL 4 ]2-.
[10]
2+
Preparation of aqueous Co
M r of cobalt nitrate = 290.9
Amount of cobalt nitrate in 250 cm3 = (0.1 / 1000) x 250 = 0.0250 mol
Mass of cobalt nitrate required = 0.0250 x 290.9 = 7.27 g

Using a clean and dry weighing boat / bottle, weigh accurately 7.27 g of
solid cobalt nitrate with an electronic balance.
Transfer the solid into a 100 cm3 beaker.
Rinse the weighing boat with deionised water and transfer the washings
into the beaker.
Dissolve the solid with deionised water.
Transfer the solution into a 250 cm3 graduated flask with the aid of a filter
funnel and a glass rod.
Rinse the beaker with deionised water and transfer the washings into the
graduated flask.
Top the graduated flask to the mark with deionised water.
Cap the graduated flask and shake well to ensure the solution is well mixed.

Preparation of 4 solutions
Flask Vol of L- / cm3 Vol of Co2+ / cm3
1 25.00 25.00
2 35.00 15.00
3 40.00 10.00
4 42.00 8.00
5 (max 6:1 ratio) 45.00 5.00





trendyline
5 solutions
otal volume = 50 cm3
Total
Att least 2 volume smaller cm3 of L-
smaller or greater than 40 c
No volume of L- or Co2+ shouldld be 0 cm3.

PJC 2016 9647/02/JC2/H2/Preliminary Exam/2016 [Turn Over


1010 trendyline
4
Procedure
Fill a burette with the prepared Co2+ solution and another with the
L- solution.
Transfer 25 cm3 of Co2+ solution into a 100 cm3 conical flask, followed
by 25 cm3 of L- solution.
Swirl the solution to ensure it is homogeneous
Repeat steps 1 to 3 for with the stated volumes of solutions for Flask
2 to 4.
Set up the spectrometer to absorb yellow light.
Measure and record the absorbance of each of the 4 solutions.
Plot a graph of absorbance against volume of L- solution.

Calculations
Maximum of point = 40 cm3 of L-
Amount of L- = (40 / 1000) x 0.1 = 0.00400 mol
Amount of Co2+ = (10 / 1000) x 0.1 = 0.00100 mol
Ratio of L- to Co2+ = 4:1

Graph

[10]
[Total: 12]

PJC 2016 9647/02/JC2/H2/Preliminary Exam/2016 [Turn Over


1011 trendyline
5
2 The mission of the Hydrogen and Fuel Cells Programme by the U.S. Department of
Energy (DoE) is to reduce petroleum use, greenhouse gas emissions, and air pollution
and to contribute to a more diverse and efficient energy infrastructure by enabling the
widespread commercialisation of hydrogen and fuel cell technologies.

One such technology involves creating an energy system concept that stores hydrogen
in the form of metal hydrides. Magnesium hydride (MgH 2 ) is one of the compounds
currently being researched on its viability on hydrogen storage. One way to release
hydrogen from metal hydrides is to react them with water, where a hydroxide is formed
as a side product.

The other way to obtain hydrogen is through the thermal decomposition of the metal
hydride MH 2 . When hydrogen is needed, these metal hydrides can be heated to
release molecular hydrogen.

MH 2 M + H2

This adsorption and desorption of hydrogen in metals serves like a hydrogen bank.
The rate of desorption of hydrogen is proportional on the strength of ionic bonds formed
between the metal and hydride ions.

Apart from MgH 2 , other Group II hydrides are also being studied and researched on the
same purpose. The table below shows some properties of the Group II hydrides.

MgH 2 CaH 2 SrH 2 BaH 2


Percentage of H / % by mass of H 7.7 4.8 2.2 1.4
Density / g cm-3 1.74 1.54 2.64 3.62
Melting point / & 650 842 777 727
H f / kJ mol-1 -114 -144 -119 -118

(a) (i) Write an equation for the reaction between solid MgH 2 and water.
MgH 2 (s) + 2H 2 O(l) Mg(OH) 2 (s) + 2H 2 (g)
[1]

(ii) Draw the dot-and-cross diagram for MgH 2 .


xx 2+ -
x
x Mg xx 2 Hx
xx

[1]

(b) (i) Explain, with reasoning, the trend on the rate of desorption of hydrogen for
the Group II metal hydrides from CaH 2 to BaH 2.
q q
Strength of ionic bond
r r

trendyline
nd
dyyl
d
Radius

ylin
ne
ne a2+ < Srr2+ < Ba2+
adius of cation, r + : Ca
Strength
trength
rength of ionic bond : BaH
Rate
aH 2 < Sr
SrH 2 < CaH 2
ate of desorption of hydrogen from CaH 2 < SrH 2 < Ba
BaH 2 (fastest)
[2]

PJC 2016 9647/02/JC2/H2/Preliminary Exam/2016 [Turn Over


1012 trendyline
6
(ii) Which metal hydride, CaH 2 or BaH 2, would you expect to decompose more
readily? Explain your answer by using relevant data from the above table.
H f becomes less exothermic from CaH 2 to BaH 2
-H f becomes less endothermic from CaH 2 to BaH 2

Hence,
Ease of decomposition: CaH 2 < BaH 2
[2]

(iii) Using relevant information from the Data Booklet and the information given
below, construct a Born-Haber cycle to calculate the lattice energy for
CaH 2 .

(The first electron affinity for hydrogen is -72.8 kJ mol-1 and the enthalpy
change of atomisation of calcium is +178 kJ mol-1)

enthalpy
kJ mol-1

Ca2+(g) + 2e + 2H(g)
2 x 1st EA H Ca2+(g) + 2H-(g)

1st + 2nd IE Ca

Ca(g) + 2H(g) LE CaH2


BE (H-H) or
Ca(g) + H2(g) 2 x Hatom H

Ca(s) + H2(g) Hatom Ca


0
Hf (CaH2)
CaH2 (s)

H f o (CaH 2 ) = H at o (Ca)+ 2H at o (H)+ 1st IE(Ca)+2nd IE(Ca)+


2 x 1st EA(H) + H latt o (CaH 2 )

trendyline
tre
e d
en
-144 = 178 + 436 + 590 2(-72.8) + H latt o (CaH 2 )
2(-72.8)
90 + 1150 + 2

LE= 2.35 x 103 kJ mo


E= -2.35 mol-11

[3]

PJC 2016 9647/02/JC2/H2/Preliminary Exam/2016 [Turn Over


1013 trendyline
7
(c) A car company is planning to develop cars that run on hydrogen fuel produced by
MgH 2 .

The following information is for a typical petrol-fuelled car.

Weight of vehicle / kg 1088


Fuel tank / L 42
Fuel consumption / km L-1 17
Energy consumption of petrol / kJ kg-1 46.4
Density of petrol / g cm-3 0.8 g

(i) Calculate the energy produced by a petrol-fuelled car which is running at


full tank.
Mass of fuel at full tank = 42 x 1000 x 0.8 = 33600 g

Energy produced by the petrol at full tank = 33600/1000 x 46.4


= 1559.04 kJ
[1]

(ii) It was found that 1 kg of hydrogen produce 3 times more energy than petrol
with the same mass.

Assuming 100% efficiency, calculate the amount of magnesium metal


needed to store the hydrogen as the hydride to provide the same amount of
energy in the hydrogen-fuelled car. (1 L = 1000 cm3)

MgH 2 Mg + H 2

1 kg petrol produces 46.4 kJ of energy


1 kg H 2 produces (46.4 x 3) = 138.9 kJ of energy

Hence to produce 1559.04 kJ of energy,


mass of hydrogen required = 1559.04/138.9 =11.15 kg

or
Mass of H2 = 33600 / 3 = 11.2 kg

Mg + H 2 MgH 2
Number of mole of H 2 needed = 11.15 x 1000/2 = 5575 or 5600

0J+ 2
Number of mole of Mg needed = 5575

Mass of Mg metal needed = 5575 x 0.0243


= 135 kg or 136 kg

trendyline [2]
[Total: 12]

PJC 2016 9647/02/JC2/H2/Preliminary Exam/2016 [Turn Over


1014 trendyline
8
3 (a) In the presence of excess oxygen, a liquid alcohol, compound A, C x H y O,
undergoes complete combustion to produce 3.19 dm3 of hot gases at a
temperature of 250 C at a pressure of 1 atm,

Upon cooling to room temperature, the volume contracted to 836 cm3. The gases
were then passed through aqueous sodium hydroxide and the final volume
remaining was 100 cm3.

(i) Fill in the following blanks for the equation for the complete combustion of
A.

C x H yO + (x + y - 1)O 2 xCO 2 + (y/2)H 2 O


[1]

(ii) Using the ideal gas equation, find the amount of CO 2 and amount of H 2 O
that was produced respectively from the complete combustion of compound
A.

Amount of gases in hot gases = (101000 x 3.19 x 10-3) / (8.31 x 523)


= 0.0741 mol

Volume of CO 2 = 836 100 = 736 cm3


Amount of CO 2 = (101000 x 736 x 10-6) / (8.31 x 298) = 0.0300 mol

Volume of O 2 = 100 cm3


Amount of O 2 = (101000 x 100 x 10-6) / (8.31 x 298) = 0.00408 mol

Amount of H 2 O = 0.0741 0.0300 0.00408 = 0.0401 mol

[3]

(iii) Using your answer to (a)(i) and (a)(ii), solve for x and y in C x H y O and
hence determine the empirical formula of A.
Amount of C = 0.0300 mol
Amount of H = 0.0401 x 2 = 0.0802 mol

Empirical ratio (C:H) = 0.0300 : 0.0802


x = 3; y = 8

Empirical formula = C 3 H 8 O
[1]

(ii) The actual number of moles of gases obtained from the combustion is
found to be higher than that calculated when using the ideal gas equation.

trendyline
trendy
Account
ccount for the difference between the two results.

Significant
ignificant
gnificant intermolecul
intermolecular forces exists between the
th gas molecules,
hence
ence
nce thee gases deviates from ideality.

[1]

PJC 2016 9647/02/JC2/H2/Preliminary Exam/2016 [Turn Over


1015 trendyline
9
(b) Methane can be obtained when carbon monoxide is heated to 900 C with
hydrogen gas as shown in the following equation.
CO(g) + 3H 2 (g) CH 4 (g) + H 2 O(g)
(ii) Write an expression for the K c of the reaction, stating its units.
[CH4 ][H2 O]
Kc = 3
mol-2 dm6
[CO][H2 ]
[1]
(iii) 1 mol of carbon monoxide was heated with 4 mol of hydrogen gas in a
10 dm3 vessel. At equilibrium, the vessel contained 0.387 mol of methane.

Calculate the K c for the reaction of carbon monoxide.


CO 3H 2 CH 4 H2O
Initial mol 1 4 0 0
mol -0.387 -1.161 +0.387 +0.387
Eq mol 0.613 2.839 0.387 0.387
[ ] mol dm-3 0.0613 0.284 0.0387 0.0387

(0.0387 )(0.0387 )
Kc = = 1.07 mol-2 dm6
(0.0613 )(0.284 ) 3
[2]

(iv) Suggest, with an explanation, how the position of equilibrium might alter
when 1 mol of helium was added into the vessel.

When an inert gas is added, the total pressure increase but the partial
pressure of the reactants and products do not change. There
would not be any change to the position of equilibrium.

OR
When an inert gas is added, since the volume of the vessel remains
the same, the concentration of the reactants and products do not
change. There would not be any change to the position of equilibrium.
[1]

(c) Anhydrous aluminium sulfate is commonly used in organic reactions to remove


water. It decomposes at 600 C to produce only an insoluble white solid and misty
fumes. The white solid dissolves in both aqueous sodium hydroxide and aqueous
hydrochloric acid while the misty fumes turn moist blue litmus red.

(i) Suggest the identities of the white solid and the misty fumes. Hence, write a
balanced equation for the decomposition of aluminium sulfate.
White solid: Al 2 O 3 Misty fumes: SO 3
Al 2 (SO 4 ) 3 Al 2 O 3 + 3SO 3
[2]

trendyline
(ii) with aqueous sodium
Write equations for the reactions of the white solid w
hydroxide
ydroxide and hydrochloric acid.
All 2 O 3 + 6HCl
6HCl $lCl
$lCl 3 + 3H 2 O [1]
$
All 2 O 3 + 2NaOH + 3H 2 21D>$
21D>$l(OH)
1D>$l $l(OH)
H 4 ] [1]
1]
[2]
[Total: 14]

PJC 2016 9647/02/JC2/H2/Preliminary Exam/2016 [Turn Over


1016 trendyline
10
4 Batteries for hearing aids comprise of zinc and carbon electrodes. These electrodes in
an electrolyte, maintained at a pH of 11, forms a galvanic cell in which zinc is oxidised
and oxygen from the air is reduced.

External circuit +ve

Carbon

Rod
Electrolyte gel

-ve Zinc Electrode

(a) Write a balanced equation for the overall chemical reaction and calculate the
E cell of this cell.

2 Zn + O 2 + 2H 2 O 2 Zn2+ + 4OH- [or 2 Zn(OH) 2 (s)]

E o cell = +0.40 + (+0.76) = + 1.16 V


[2]

(b) Indicate, on the diagram above, the polarity of both electrodes and the direction of
electron flow in the external circuit.
[2]

(c) Explain how the e.m.f. of the cell would change if the pH of the electrolyte
decreases.
When pH decreases, [OH-] falls and so, position of equilibrium of the
reduction half-cell shifts to the right. Hence, E reduction becomes more
positive, making e.m.f. more positive.
[2]

(d) If a current of 4 x 10-5 A was drawn from the cell, calculate how long a zinc
electrode weighing 2 g will last before it needs to be replaced. Give your answer
correct to the nearest day.
2
Amount of Zn = = 0.03058 mol
65.4

Zn Zn2+ + 2 e

trendyline
tre yl
Amountt of electrons = 2 0.0305858 = 0.06116 mol
m
Q = 0.06116 96500 = 5902 C
Q 59
5902
t= = 5
1.476 108 s = 1708
= 1.47 708 days
day
I 4 10
1
[2]
[Total: 8]

PJC 2016 9647/02/JC2/H2/Preliminary Exam/2016 [Turn Over


1017 trendyline
11
5 Insulin is a hormone made by the pancreas that allows your body to use glucose from
carbohydrates in the food that you eat for energy. Insulin helps keeps your blood sugar
level from getting too high or too low.

The insulin molecule consists of 51 amino acid residues, in two polypeptide chains.
H2N

Section X

H2N

CO2H

CO2H

Some information on the amino acids of insulin is given below.

amino acid Abbreviated Formula of side chain pI


name (R in RCH(NH 2 )CO 2 H)
arginine arg -(CH 2 ) 3 NHC(NH 2 )=NH 10.76
cysteine cys -CH 2 SH 5.07
glutamic acid glu -CH 2 CH 2 COOH 3.22
glycine gly -H 5.97

(a) Draw the structural formula of section X in insulin.


Formula must start with N terminus, showing R groups, ending with C
terminus:
H H O H H O H H O
N C C N C C N C C
H H C H H C H

trendyline
dy
H C H H C H

H C H COO
COOH
N
HN
HN C H
NH2
[1]

PJC 2016 9647/02/JC2/H2/Preliminary Exam/2016 [Turn Over


1018 trendyline
12
(b) The diagram shows the results of electrophoresis on a mixture of the amino acids
obtained from hydrolysis of section X at pH 6.0.

Draw the structure of the species responsible for the spots labelled R, S and T.

R S T

+ - + - + -
H3N CH CO2 H3N CH CO 2 H3N CH CO 2
+
(CH 2)3NHC(NH 3) =NH
CH2 H

CH2
-
Accept protonation on
CO2 any basic amine group
of side chain. (more
than 1 protonation also
k allowed)
[3]

trendyline
PJC 2016 9647/02/JC2/H2/Preliminary Exam/2016 [Turn Over
1019 trendyline
13
(c) The polypeptide chains of insulin coil to form short sections of -helix which
stabilises the secondary structure.

Describe with the aid of a sketch, how a polypeptide chain is held in the shape of
DQ-helix.

[3]

(d) The three-dimensional structure of insulin is further stabilised by disulfide


linkages.

Write an equation for the formation of such linkage and state the type of reaction
involved.

Type of reaction: oxidation

-CH 2 S-H + H-SCH 2 - -CH 2 -S-S-CH 2 - + 2[H] (or H 2 )


OR
-CH 2 S-H + H-SCH 2 - + [O] (or O 2 ) -CH 2 S-SCH 2 - + H 2 O

trendyline [2]
[Total: 9]

PJC 2016 9647/02/JC2/H2/Preliminary Exam/2016 [Turn Over


1020 trendyline
14
6 Lactones are cyclic esters. They are formed by intramolecular esterification of the
corresponding hydroxycarboxylic acids.

(a) (i) The reaction scheme below shows how lactone E is synthesised from
compound D.
O

D
Complete the reaction scheme by giving the structural formulae of the
intermediate organic products in the spaces provided and stating the
reagents and conditions for steps I, III, IV, V and VI.

O O O

I II Br
Br
Al2O3, 400oC
HO
D

III

O
OH O

V IV
NC
HOOC HOOC

VI

trendyline
E

PJC 2016 9647/02/JC2/H2/Preliminary Exam/2016 [Turn Over


1021 trendyline
15

Step Reagents and conditions


I Br 2 (aq)

III NaCN/KCN in ethanol, heat

IV H 2 SO 4 (aq), heat

V NaBH 4 in methanol

VI concentrated H 2 SO 4 , heat

[8]

(ii) Suggest a simple chemical test to distinguish compound D from E.


Reagents and conditions: 2,4-dinitrophenylhydrazine
Observations:
For compound D, orange ppt is formed.
For compound E, no orange ppt is formed (or no observable change)

Or
Reagents and conditions: K 2 Cr 2 O 7 , H 2 SO 4 (aq), heat
Observations:
For compound D, orange solution remains (or no observable change)
For compound E, orange solution turned green.
[2]

(iii) Name and outline the mechanism when compound D is treated with HCN in
the presence of aqueous NaOH at 10 oC.
Name of mechanism: Nucleophilic Addition

1D2+ DT +&1 DT + 2 O(l) + Na+(aq) + CN-(aq)

O -
O
C CN
CN- C

slow

- OH
O H-CN CN
fast C
CN
C
+ CN-

ttrendyline
tr
re
e in
in
[3]

PJC 2016 9647/02/JC2/H2/Preliminary Exam/2016 [Turn Over


1022 trendyline
16
(b) Lactones with three- or four-membered rings are very reactive compared to five-
or six-membered rings, making their isolation difficult.

Suggest why lactones with three- or four-membered rings are very reactive.

Three- or four-membered rings have significant ring/angle strain.


[1]

(c) Iodolactonisation reaction was first reported by M. J. Bougalt in 1904 and has
since become one of the most effective ways to synthesise lactones.

An example of the iodolactonisation is the reaction of 4-pentenoic acid as shown


below.
O I2, NaHCO3
I
OH O
O

Two simplified steps in the iodolactonisation mechanism are given below.

Step 1
O O
+ HCO3- + H2O + CO2
OH -
O

Step 2
O slow Intermediate fast I
+ I2 + I-
- F O
O O
-
(i) Suggest the role of HCO 3 in step 1 of the mechanism.

It acts as a base to abstract H+ from the carboxylic acid.


[1]

(ii) The type of reaction in Step 2 of the mechanism is electrophilic addition.

Draw the structure of intermediate F.


+
I C O
H
-
O
[1]

(iii) Suggest the structural formulae of the final organic product formed when
CH 2 =CHCH(CH 3 )CH 2 CH 2 COOH undergoes iodolactonisation in a similar
process as above.
CH3

trendyline
ttr
re O O

End of Paper
[1]
[Total: 17]

PJC 2016 9647/02/JC2/H2/Preliminary Exam/2016 [Turn Over


1023 trendyline
PIONEER JUNIOR COLLEGE

JC2 PRELIMINARY EXAMINATION


HIGHER 2

CANDIDATE
NAME

CT INDEX
1 5 S
GROUP NUMBER

CHEMISTRY 9647/03
Paper 3 Free Response 15 September 2016
2 hours

Candidates answer on separate paper.


Additional Materials: Answer Paper
Graph Paper
Data Booklet
Cover Page

READ THESE INSTRUCTIONS FIRST

Write your class, index number and name on all work that you hand in.
Write in dark blue or black pen on both sides of the paper.
You may use a soft pencil for any diagrams, graphs or rough workings.
Do not use staples, paper clips, highlighters, glue or correction fluid.

Answer any four questions.


A Data Booklet is provided.
You are reminded of the need for good English and clear presentation in your answers.

The number of marks is given in brackets [ ] at the end of each question or part question.
At the end of the examination, fasten all your work securely together.

trendyline
y
This document consists of 12 printed pages and 1 BLANK Page.

1024 trendyline
2
Answer any four questions.
Begin each question on a fresh piece of writing paper.

1 (a) Linalyl acetate is a naturally occurring phytochemical found in many flowers. It is


one of the principal components of the essential oils of bergamot and lavender. It
has the following structure.

Linalyl acetate can be synthesised by the reaction of carboxylate anion with an


alkyl halide as shown below:

CH 3 CO 2 -Na+ + RC(CH 3 )(CH=CH 2 )I 1DI + RC(CH 3 )(CH=CH 2 )OCOCH 3

where R =
The kinetics of this reaction was determined by measuring the concentration of the
remaining RC(CH 3 )(CH=CH 2 )I with time.
The initial concentrations of RC(CH 3 )(CH=CH 2 )I and CH 3 CO 2 -Na+ were
0.0050 mol dm3 and 0.100 mol dm3 respectively. The following data was obtained.

Time/ min [RC(CH 3 )(CH=CH 2 )I] / mol dm-3


0 0.0050
15 0.0040
30 0.0032
45 0.0026
60 0.0021
75 0.0017

(i) Plot a suitable graph to show that the order of reaction with respect to
RC(CH 3 )(CH=CH 2 )I is one.
[3]

(ii) A new set of experiment was carried out and the initial concentrations of
RC(CH 3 )(CH=CH 2 )I and CH 3 CO 2 -Na+ was increased to 0.0100 mol dm-3,

trendyline
0.200 mol dm-3 respectively
and 0. respectively.
ly. When a similar
s graph was
wa plotted, it was
found
d that the gradient at each point doubled.
doubled

Deduce th order of reaction with respect to CH 3 CO 2 -Na+, and explain your


uce the
answer.
wer. [2]

PJC 2016 9647/03/JC2 Preliminary Exam/2016 [Turn Over


1025 trendyline
3

(iii) Construct a rate equation for the reaction between CH 3 CO 2 -Na+ and
RC(CH 3 )(CH=CH 2 )I, stating the units for the rate constant.
[1]

(iv) Using your answer in (a)(iii), suggest the mechanism for the reaction
between CH 3 CO 2 -Na+ and RC(CH 3 )(CH=CH 2 )I that leads to the formation of
the ester, RC(CH 3 )(CH=CH 2 )OCOCH 3 .
[3]

(b) An example of a homogeneous catalyst is Fe2+(aq), which is used in the oxidation


of sodium ethanedioate, Na 2 C 2 O 4 , with acidified potassium manganate(VII),
KMnO 4 . Ethanedioate is oxidised to carbon dioxide.

(i) What do you understand by the term homogeneous catalyst? [1]

(ii) Write an overall equation for the reaction of sodium ethanedioate with
acidified potassium manganate(VII). [1]

(iii) The Eo cell of the reaction between sodium ethanedioate with acidified
potassium manganate(VII) is +2.01 V. However, the reaction is found to
proceed at a very slow rate.

Explain why the reaction between sodium ethanedioate and acidified


potassium manganate(VII) is slow. [1]

(iv) By considering suitable Eo values from the Data Booklet and the data given
below, explain how Fe2+ functions as a catalyst for the reaction between
sodium ethanedioate with acidified potassium manganate(VII), writing
equations where appropriate.

2CO 2 (g) + 2e- C 2 O 4 2-(aq) E = - 0.49V


[3]

(v) In the absence of a Fe2+ catalyst, the rate of reaction between potassium
manganate(VII), and sodium ethanedioate is shown in the graph below.

Rate

0 time

trendyline
d li
Explain
ain the shape of the graph. [2]

(c) The rate off a chemical temperatures.


chemical reaction is usually increased at elevated tem

Explain, with the aid of a Boltzmann Distribution


D graph, why an increase in
temperature increases the rate of reaction. [3]

PJC 2016 9647/03/JC2 Preliminary Exam/2016 [Turn Over


1026 trendyline
4
[Total: 20]

2 (a) Ammonia and para-phenylenediamine, H 2 NC 6 H 4 NH 2 , are Bronsted-Lowry bases


that are used widely in permanent hair dyes. Para-phenylenediamine has the
structure shown below:

H2N NH2

para-phenylenediamine

Para-phenylenediamine, H 2 NC 6 H 4 NH 2 , can ionise in stages.

+ H+ +H+
+ + +
H2N NH2 H2N NH3 H3N NH3
pKb1 pKb2

The two pK b values associated with para-phenylenediamine are shown in the table
below.

Base formula pK b1 pK b2
ammonia NH 3 4.7 -
para-phenylenediamine H 2 NC 6 H 4 NH 2 7.7 11.0

(i) Explain what is meant by the term Bronsted-Lowry base. [1]

(ii) Suggest a reason why the pK b1 value of para-phenylenediamine is higher


than the pK b1 of ammonia. [1]

(iii) Suggest a reason why the pK b1 value of para-phenylenediamine is lower than


pK b2 . [1]

(iv) Explain what is meant by a buffer solution. [1]

(v) Assuming that a solution at pH 6.5 contains only [H 2 NC 6 H 4 NH 2 ] and


[ H 2 NC6 H 4 NH 3 ]
[H 2 NC 6 H 4 NH 3 +], calculate the ratio of in the solution.
[ H 2 NC6 H 4 NH 2 ]

(ignore the effect of pK b2 of para-phenylenediamine on the pH) [2]

(vi) Calculate the pH of solution containing equimolar amount of H 2 NC 6 H 4 NH 3 +

trendyline
and +H 3 NC 6 H 4 NH 3 + [2]

PJC 2016 9647/03/JC2 Preliminary Exam/2016 [Turn Over


1027 trendyline
5

(b) Ammonia is commonly used in qualitative analysis of halides in aqueous solutions,


such as detecting the presence of chloride and bromide ions in natural water
sources such as rivers, lakes and streams.

(i) 5 cm3 of 0.0100 mol dm-3 of silver nitrate is added to a 30 cm3 sample of river
water containing chloride ions.

What is the minimum concentration, in mol dm-3 , of chloride ions present in


the river water when the first trace of precipitate appears?

Given solubility product of the silver chloride is 2.0 x 10-10 mol2 dm-6.
[2]

(ii) To a test tube containing another river sample containing bromide ions,
describe what you would see when aqueous silver nitrate is added to the
sample, followed by excess aqueous ammonia to the resulting mixture. [1]

(iii) Explain, with aid of appropriate equation(s), the observations in (b)(ii).

You should use the concepts of Le Chateliers Principle and solubility


product, K sp , to explain your answer. [4]

(c) (i) When iodine reacts with sodium hydroxide, iodide and iodate(V) are formed.

Write a balanced ionic equation, with state symbols, for the formation of IO 3 -
when iodine reacts with sodium hydroxide. Name the type of reaction taking
place. [2]

(ii) When Group II iodates(V), M(IO 3 ) 2 , are heated at high temperature, it form
the metal oxide, iodine vapour and a colourless gas that relights glowing
splinter.

It was observed that Group II iodates(V) down the group have to be heated
more strongly before the iodine vapour appears. Explain why thermal stability
increases down the group for Group II iodate(V). [3]
[Total: 20]

trendyline
PJC 2016 9647/03/JC2 Preliminary Exam/2016 [Turn Over
1028 trendyline
6

3 (a) Liquid methyl t-butyl ether, MTBE, is a fuel additive that is used as an oxygenate to
raise the octane number in fuel. This will help the fuel to burn completely and
reduce air pollution.
O

MTBE

(i) What is meant by the term standard enthalpy change of formation of MTBE?
[1]

(ii) Use the following data and relevant data from Data Booklet to construct an
appropriate energy cycle, find the standard enthalpy change of formation of
MTBE.

Standard enthalpy change of atomisation of C(s) +715 kJ mol1


Standard enthalpy change of atomisation of MTBE(l) +6822 kJ mol1
[4]

(iii) Given that the standard enthalpy change of vaporisation of MTBE is


+30.4 kJ mol-1 and using relevant data from the Data Booklet and (a)(ii),
calculate the bond energy of C-O in MTBE. [2]

(iv) Suggest a reason for the difference in the C-O bond energy in (a)(iii) from
the value given in the Data Booklet. [1]

(b) MTBE can be produced from methanol and but-1-ene by the following equation.
The reaction has a standard entropy change of reaction of -281 J mol1 K1.

CH 3 OH(l) + C 4 H 8 (l) C 5 H 12 O(l) H


MTBE

The table below lists the H f (standard enthalpy change of formation) values for
some compounds.

compound H f / kJ mol-1
CH 3 OH(l) -239
C 4 H 8 (l) -0.4

(i)

(ii)
trendyline
Calculate
culate the
above
ve
HH f for the reaction in (b)
e and your answer to (a)(ii).

Determine
(a i)).
(a)(i

ermine the feasibility of the pro


but-1-ene at 298 K.
(b using the value
values from the table

production of MTBE from


fro methanol and
[2]
[1]

PJC 2016 9647/03/JC2 Preliminary Exam/2016 [Turn Over


1029 trendyline
7

(c) Methyl shift can occur in organic chemistry reactions in reactions involving
carbocation intermediates.

An example of a methyl shift in a reaction between an alcohol and HI is shown


below. In this reaction compound D is produced as the major product.

OH H I +
OH2

+
Step 1
-
+ I
+ H2O + I-
carbocation A

methyl shift

+
I + H2O I- + H2O

compound D carbocation B

(i) What is the role of HI in step 1? [1]

(ii) Suggest a reason why there is a methyl shift to produce carbocation B from
carbocation A. [1]

(iii)

The structure of compound E is OH .

Predict the major final product formed when compound E reacts with HBr
based on the methyl shift reaction given in (c). [1]

trendyline
PJC 2016 9647/03/JC2 Preliminary Exam/2016 [Turn Over
1030 trendyline
8

(iv) Compound D undergoes the following reactions.

step 1 CH3COCl

I NH2 NHCOCH3
D F

step 2

step 3

NH2

Suggest suitable reagents for steps 1, 2 and 3. [3]

(v) Suggest one simple chemical test which would enable you to distinguish
O

H2N

between compound F and .

You should state the reagents and conditions for the test and describe the
observations.
[3]
[Total: 20]

trendyline
PJC 2016 9647/03/JC2 Preliminary Exam/2016 [Turn Over
1031 trendyline
9

BLANK PAGE

trendyline
PJC 2016 9647/03/JC2 Preliminary Exam/2016 [Turn Over
1032 trendyline
10

4 (a) Compound G, C 10 H 15 NO 3 , is a natural stimulant made in the adrenal gland of the


kidney and is carried in the bloodstream and affects the autonomous nervous
system, which controls functions such as heart rate and dilation of the pupils.

G is soluble in dilute sodium hydroxide and dilute hydrochloric acid but does not
react with sodium carbonate.

G undergoes reaction with hot acidified potassium dichromate(VI) to form J. J


gives an orange precipitate with 2,4-dinitrophenylhydrazine but does not give a
silver mirror with Tollens reagent. Upon heating with concentrated H 2 SO 4, G forms
K, C 10 H 13 NO 2 . K does not react with PCl 5 . On heating with acidified potassium
manganate(VII), K forms three compounds, L, C 7 H 6 O 4 , M, C 2 H 4 O 2 , and an
ammonium salt, CH 3 NH 3 +.

Deduce the structures for each compound, G to M, and give an account of the
chemistry involved. [10]

(b) Like alcohols and halogenoalkanes, amines can be converted into alkenes by an
elimination reaction known as Hofmann elimination.

For example, 1-methyl-pentylamine is converted into hex1ene as shown.

Hofmann Elimination
+ side products
NH2
reaction

An interesting feature of the Hofmann elimination is that it gives a less substituted


alkene compared to other elimination reactions.

(i) Draw the likely structure of the starting amine if the following alkene N is the
main product formed when the amine undergoes a Hofmann elimination
reaction.

N
[1]

trendyline
PJC 2016 9647/03/JC2 Preliminary Exam/2016 [Turn Over
1033 trendyline
11

(ii) Suggest, in no more than 3 steps, a suitable synthetic pathway to convert N


into the following compounds.

For each step, state clearly the reagents and conditions, indicating the
structural formula of all intermediate compounds formed.

(I) OH
HO

[2]

(II)
O

[3]

(c) Amines can be formed by reductive amination of an aldehyde or ketone with


ammonia or amine in the presence of reducing agent.

A reductive amination to produce an amine P takes place in the pathway shown.

CH2Cl CH2Cl OH CH2Cl


CH2COCH3 NH3 CH2 C NH2 CH2 C NH
CH3 Step II CH3
Step I

Step III

CH2Cl
CH2 CH NH2
CH3

P (C10H14NCl)

trendyline
(i) g
Suggest yp of reaction that occurs in steps
the type p I and II. [2]

(ii) Suggest
gest the reagents and conditions for step III.
III
II. [1]

(iii) On heating,
eatin P forms Q,
Q, C 10 H 13 N. Suggest a structure for Q.
Q.
[1]
[Total: 20]

PJC 2016 9647/03/JC2 Preliminary Exam/2016 [Turn Over


1034 trendyline
12

5 (a) An electrochemical cell involving tartaric acid, (CH(OH)CO 2 H) 2 , has the following
cell diagram notation.

Pt(s)|(CH(OH)CO 2 H) 2 (aq) |CO 2 (g) |H+(aq) || Co3+(aq), Co2+(aq)|Pt(s)

(i) Given that the e.m.f of the cell is +0.80 V and using relevant data from the
Data Booklet, determine the standard electrode potential of the
CO 2 (g)|(CH(OH)CO 2 H) 2 (aq) half-cell. [2]

(ii) Draw a labelled diagram of the above electrochemical cell. [3]

(b) Fehlings solution is made by adding equal volumes of a blue solution A containing
aqueous copper(II) sulfate and a colourless solution B containing potassium
sodium tartrate and sodium hydroxide. In a strong base, an alpha-hydroxy group in
the tartrate ion, (CH(OH)CO 2 ) 2 , deprotonates as shown in the diagram below.

O O
-
- OH - O
O O
- O
- + H+
O OH
OH
O O
Deprotonated tartrate ion

The deprotonated tartrate ion acts as a bidentate ligand and binds to the Cu2+(aq)
using both oxygen atoms from the alpha-hydroxy groups. A square planar
bistartratocuprate(II) complex, [Cu(tart) 2 ]4, forms, and a deep blue solution is seen.

[tart = deprotonated tartrate ion]

(i) Draw the shape of [Cu(tart) 2 ]4 complex, indicating clearly the type of bonds
formed between the Cu2+ and the ligands. [2]

(ii) The open straight chain form of Dglucose, a reducing sugar, is shown.

OH OH O
HO
H
OH OH
Dglucose

State
e what would be observed when Dglucose
D glucose
u solution is warmed with a
few drops of Fehlings solution. Hence, or otherwise, write
w the balanced
equation
tion for the reaction described above.
ation above. [2]

(iii) Explain
ain
in why there is a need to form the [C (t t 2 ]44 comple
t [Cu(tart) complex in the Fehlings
solution
ti ttest.
t [1]

PJC 2016 9647/03/JC2 Preliminary Exam/2016 [Turn Over


1035 trendyline
13
(iv) Explain why the Fehlings solution is deep blue while potassium sodium
tartrate solution is colourless. [4]

(v) Stronger field ligands are known to give rise to a larger energy gap between
the two sets of d-orbitals in a transition metal complex.

Violet Blue Green Yellow Orange Red


400 nm 430 nm 480 nm 560 nm 590 nm 630 nm 750 nm
. Energy of light decreases

To test for proteins, Biuret reagent which also contains the [Cu(tart) 2 ]4
complex is used. In the presence of peptides, it undergoes ligand exchange
to form [Cu(peptide) 2 ]2+. The deep blue solution turns violet.

Using the above electromagnetic spectrum of visible light, state the relative
field strength of peptides and deprotonated tartrate ions. Explain your
answer. [3]

(c) Adolf Kolbe first published a method describing the preparation of an alkane from
the electrolysis of an aqueous solution of sodium salt of a monoprotic acid, using
inert electrodes.

In the Kolbe electrolysis of sodium propanoate, butane was formed at the anode in
the following reaction.
2CH 3 CH 2 CO 2 CH 3 CH 2 CH 2 CH 3 + 2CO 2 + 2e-

(i) Write the half equation for the reaction at the cathode during the electrolysis
of sodium propanoate. Hence write an equation for the overall reaction. [2]

(ii) A student tried to synthesise another hydrocarbon using an aqueous solution


of a sodium salt of a diprotic acid, succinic acid, (CH 2 CO 2 H) 2 . The succinate
ion, (CH 2 CO 2 ) 2 ,undergoes a similar reaction as propanoate ion at the
anode.
(CH 2 CO 2 ) 2 R + 2CO 2 + 2e-

Suggest an identity of the hydrocarbon R. [1]


[Total : 20]

End of Paper

trendyline
PJC 2016 9647/03/JC2 Preliminary Exam/2016 [Turn Over
1036 trendyline
PIONEER JUNIOR COLLEGE

JC2 PRELIMINARY EXAMINATION


HIGHER 2

CANDIDATE ANSWER SCHEME


NAME

CT INDEX
1 5
GROUP NUMBER

CHEMISTRY 9647/03
Paper 3 Free Response 15 September 2016
2 hours

Candidates answer on separate paper.


Additional Materials: Answer Paper
Data Booklet
Cover Page

READ THESE INSTRUCTIONS FIRST

Write your class, index number and name on all work that you hand in.
Write in dark blue or black pen on both sides of the paper.
You may use a soft pencil for any diagrams, graphs or rough workings.
Do not use staples, paper clips, highlighters, glue or correction fluid.

Answer any four questions.


A Data Booklet is provided.
You are reminded of the need for good English and clear presentation in your answers.

The number of marks is given in brackets [ ] at the end of each question or part question.
At the end of the examination, fasten all your work securely together.

trendyline
y This document consists of 22 printed pages.

1037 trendyline
2
Answer any four questions.
Begin each question on a fresh piece of writing paper.

1 (a) Linalyl acetate is a naturally occurring phytochemical found in many flowers. It is


one of the principal components of the essential oils of bergamot and lavender. It
has the following structure.

Linalyl acetate can be synthesised by the reaction of carboxylate anion with an


alkyl halide as shown below:

CH 3 COO-Na+ + RC(CH 3 )(CH=CH 2 )I 1DI + RC(CH 3 )(CH=CH 2 )OCOCH 3

where R =
The kinetics of this reaction was determined by measuring the concentration of the
remaining RC(CH 3 )(CH=CH 2 )I with time. The initial concentrations of
RC(CH 3 )(CH=CH 2 )I and CH 3 COO-Na+ were 0.0050 mol dm3 and 0.100 mol dm3
respectively . The following data was obtained.

Time/ min [RC(CH 3 )(CH=CH 2 )I] / mol dm-3


0 0.0050
15 0.0040
30 0.0032
45 0.0026
60 0.0021
75 0.0017

(i) Plot a suitable graph to show that the order of reaction with respect to
RC(CH 3 )(CH=CH 2 )I is one. [3]
constant t (at about 48.25 minutes) first order

trendyline
PJC 2016 9647/03/JC2 Preliminary Exam/2016
1038 trendyline
3
(ii) A new set of experiment was carried out and the initial concentrations of
RC(CH 3 )(CH=CH 2 )I and CH 3 CO 2 -Na+ was increased to 0.0100 mol dm-3,
and 0.200 mol dm-3 respectively. When a similar graph was plotted, it was
found that the gradient at each point doubled.

Deduce the order of reaction with respect to CH 3 CO 2 -Na+, and explain your
answer. [2]
- +
The reaction was zero order with respect to CH 3 COO Na .

Gradient of a concentration-time graph gives the rate of reaction.

The reaction is first order with respect to RC(CH 3 )(CH=CH 2 )I, and doubling
the concentration will double the rate. This implies that changing the
concentration of CH 3 CO 2 -Na+ has no effect on the rate of reaction.

(iii) Construct a rate equation for the reaction between CH 3 COO-Na+ and
RC(CH 3 )(CH=CH 2 )I, stating the units for the rate constant. [1]

Rate = k[RC(CH 3 )(CH=CH 2 )I]

Units of k = min-1

(iv) Using your answer in (iii), suggest the mechanism for the reaction between
CH 3 CO 2 -Na+ and RC(CH 3 )(CH=CH 2 )I that leads to the formation of the
ester, RC(CH 3 )(CH=CH 2 )OCOCH 3 .
[3]

Nucleophilic substitution (S N 1)
R
CH3
slow +
C I C R + I
CH3 CH2=CH2

CH2=CH2
CH3
CH2=CH2
+ R
C R CH3
-
CH3COO : fast
CH2=CH2 C

OOCCH3

trendyline
PJC 2016 9647/03/JC2 Preliminary Exam/2016
1039 trendyline
4
(b) An example of a homogeneous catalyst is Fe2+(aq), which is used in the oxidation
of sodium ethanedioate, Na 2 C 2 O 4 , with acidified potassium manganate(VII),
KMnO 4 . Ethanedioate is oxidised to carbon dioxide.

(i) What do you understand by the term homogeneous catalyst. [1]

A homogeneous catalyst is a catalyst that exists in the same phase as the


reactants.

(ii) Write an overall equation for the reaction of sodium ethanedioate with
acidified potassium manganate(VII). [1]

2MnO 4 - + 16H+ + 5C 2 O 4 2- --> 2Mn2+ + 8H 2 O + 10CO 2

(iii) The Eo cell of the reaction between sodium ethanedioate with acidified
potassium manganate(VII) is +2.01 V. However, the reaction is slow.

Explain why the reaction is slow. [1]

High activation energy due to the repulsion between two negatively charged
ions MnO 4 - and C 2 O 4 2- to react, the rate of reaction is very slow.

(iv) By considering suitable Eo values from the Data Booklet and the data given
below, explain how Fe2+ functions as a catalyst for the reaction between
sodium ethanedioate with acidified potassium manganate(VII), writing
equations where appropriate.

2CO 2 (g) + 2e- C 2 O 4 2-(aq) E = - 0.49V


[3]

Fe2+ can speed up the rate of reaction as it is able to attract the oppositely
charged ions in the following equations:

MnO 4 - + 8H+ + 5Fe2+ ------> 5Fe3+ + 4H 2 O + Mn2+ E = +0.75V

C 2 O 4 2- + 2Fe3+ --------> 2CO 2 + 2Fe2+ E = +1.26V

Since the E is positive for both reactions, they are energetically feasible.

trendyline
PJC 2016 9647/03/JC2 Preliminary Exam/2016
1040 trendyline
5
(iv) In the absence of a Fe2+ catalyst, the rate of reaction between potassium
manganate(VII), and sodium ethanedioate is shown in the graph below.

Rate

0 time
Explain the shape of the graph. [2]

The rate of reaction is slow at the beginning of the reaction.

As the reaction proceeds, [Mn2+] increases, which catalyses the reaction.


The rate of reaction increases.

The rate of reaction will eventually decrease as [reactants] decreases to very


low levels despite the increasing concentration of the Mn2+ catalyst.

(c) The rate of a chemical reaction is usually increased at elevated temperatures.

Explain, with the aid of a Boltzmann Distribution graph, why an increase in


temperature increases the rate of reaction. [3]
[Total: 20]

At a higher temperature, the average kinetic energy of the particles is higher.


The particles move faster and collide with each other more frequently.

In addition, the number of particles with energy E a increases exponentially,


resulting in a greater number of effective collisions per unit time. This gives
rise to a large increase in the rate of reaction.

trendyline
PJC 2016 9647/03/JC2 Preliminary Exam/2016
1041 trendyline
6
2 (a) Ammonia and para-phenylenediamine, H 2 NC 6 H 4 NH 2 , are Bronsted-Lowry bases
that are used widely in permanent hair dyes. Para-phenylenediamine has the
structure shown below:

H2N NH2

para-phenylenediamine

Para-phenylenediamine, H 2 NC 6 H 4 NH 2 , can ionise in stages.

+ H+ +H+
+ + +
H2N NH2 H2N NH3 H3N NH3
pKb1 pKb2

The two pK b values associated with para-phenylenediamine are shown in the table
below.

Base formula pK b1 pK b2
ammonia NH 3 4.7 -
para-phenylenediamine H 2 NC 6 H 4 NH 2 7.7 11.0

(i) Using ammonia, NH 3 , explain what is meant by the term Bronsted-Lowry


base.
[1]
Ammonia, NH 3 , has a lone pair of electrons which acts as a base to accept a
proton.

(ii) Suggest a reason why the pK b1 value of para-phenylenediamine is higher


than the pK b1 of ammonia. [1]

The lone pair of electrons on N of H 2 NC 6 H 4 NH 2 can be delocalized into the


bezene ring, making it less available to accept a proton, compared to NH 3 .
Hence pK b1 value of para-phenylenediamine is larger than the pK b1 of
ammonia.

(iii) Suggest a reason why the pK b1 value of para-phenylenediamine is lower


than pK b2 . [1]
More energy is required to accept another H+ by H 2 NC 6 H 4 NH 3 +, from H 2 O to
form H 3 NC 6 H 4 NH 3 2+, which is electrostatically unfavourable. Hence pK b1

trendyline
y
value
e of
o para-phenylenediamine e is lower than pK
pK b2.

OR
H 3 + can b
The lone pair of electrons on NH 2 of H 2 NC 6 H 4 NH be delocalized into
the bezene ring to a greater extent
extent,
tent, making it less available
avail to accept a
proton. Hence pK b1 value of para-phenylenediamine is lower than the pK b2 .

PJC 2016 9647/03/JC2 Preliminary Exam/2016


1042 trendyline
7
(iv) Explain what is meant by a buffer solution. [1]
A buffer solution is one which is able to resist a change in pH (i.e. maintain
an almost constant pH) upon the addition of a small amount of acid or alkali,
or on dilution.

(v) Assuming that a solution at pH 6.5 contains only [H 2 NC 6 H 4 NH 2 ] and

+
[ H 2 NC6 H 4 NH 3 ]
[H 2 NC 6 H 4 NH 3 ], calculate the ratio of in the solution.
[ H 2 NC6 H 4 NH 2 ]
[2]

pH + pOH = 14
pOH = 14 - 6.5 = 7.5

[ H 2 NC6 H 4 NH 3 ]
pOH pK1 lg
[ H 2 NC6 H 4 NH 2 ]

[ H 2 NC6 H 4 NH 3 ]
7.5 7.7 lg
[ H 2 NC6 H 4 NH 2 ]
[ H 2 NC6 H 4 NH 3 ]
0.63
[ H 2 NC6 H 4 NH 2 ]

(vi) Calculate the pH of solution containing equimolar amount of H 2 NC 6 H 4 NH 3 +


and +H 3 NC 6 H 4 NH 3 +.
[2]

when equimolar amount of H 2 NC 6 H 4 NH 3 + and H 3 NC 6 H 4 NH 3 2+ is formed,


[H 2 NC 6 H 4 NH 3 +] = [+H 3 NC 6 H 4 NH 3 +]

This is the maximum buffer capacity at second stage of neutralization.


Hence
[ H 3 NC6 H 4 NH 32 ]
pOH pK 2 lg
[ H 2 NC6 H 4 NH 3 ]
pOH = pK 2
pOH = 11
pH = 3

PJC 2016 9647/03/JC2 Preliminary Exam/2016


1043 trendyline
8
(b) Ammonia is commonly used in qualitative analysis of halides in aqueous solutions,
such as detecting the presence of chloride and bromide ions in natural water
sources such as rivers, lakes and streams.

(i) 5 cm3 of 0.0100 mol dm-3 of silver nitrate is added to a 30 cm3 sample of river
water containing chloride ions.

What is the minimum concentration, in mol dm-3 , of chloride ions present in


the river water when the first trace of precipitate appears?

Given solubility product of the silver chloride is 2.0 x 10-10 mol2 dm-6.
[2]
+
Ag (aq) + X DT $J; s)

The ppt is AgX.


[Ag+] at point of mixing = no. of moles of Ag+ before mixing total vol
= (0.0100 x 5/1000) 35/1000
= 1.428 10-3 mol dm-3

[Cl-] at point of mixing = no. of moles of X-before mixing total vol.


= ([Cl-] initial x 30/1000) 35/1000 mol dm-3

For precipitation to take place,


Ionic product (AgCl) Ksp(AgCl)

( [Ag+][Cl -] ) at point of mixing 2.0 x 10-10

30
[Cl ]initial
(1.428 103 ) ( 1000 )
35 2.0 x 10-10
1000

-
[Cl ] initial 1.63 x 10-7

Hence, minimum concentration is 1.63 x 10-7 mol dm-3

(ii) To a test tube containing another river sample containing bromide ions,
describe what you would see when aqueous silver nitrate is added to the
sample, followed by excess aqueous ammonia to the resulting mixture
[1]

Br-: cream ppt of AgBr observed; insoluble in excess NH 3 (aq)

trendyline
PJC 2016 9647/03/JC2 Preliminary Exam/2016
1044 trendyline
9
(iii) Explain, with aid of appropriate equation(s), the observations in (b)(ii).

You should use the concepts of Le Chateliers Principle and solubility


product, K sp , to explain your answer.
[4]

Ag+(aq) + Br-(aq) AgBr(s) ....(1)


Precipitate of AgBr is observed when I.P> K sp

Ag+(aq) + 2NH 3 (aq) [Ag(NH 3 ) 2 ]+(aq) .(2)


When excess NH 3 is added (high [NH 3 ]) to AgBr(s), the position of
equilibrium (2) shifts to the right in order to reduce the effect of excess
NH 3 (aq). OR complex [Ag(NH 3 ) 2 ]+ is formed

[Ag+] decreases hence equilibrium (1) shifts to the left.


but the ionic product of AgBr is still larger than K sp (AgBr),
AgBr remains insoluble.
OR
AgBr(s) + 2NH 3 (aq) [Ag(NH 3 ) 2 ]+(aq) + Br -(aq)

(c) (i) When iodine reacts with sodium hydroxide, iodide and iodate(V) are formed.

Write a balanced ionic equation, with state symbols, for the formation of IO 3 -
when iodine reacts with sodium hydroxide. Name the type of reaction taking
place. [2]

3I 2 (s) + 6OH-(aq) 5I-(aq) + IO 3 -(aq) + 3H 2 O(l)

Disproportionation

(ii) When Group II iodates(V), M(IO 3 ) 2 , are heated at high temperature, it form
the metal oxide, iodine vapour and a colourless gas that relights glowing
splinter.

It was observed that Group II iodates(V) down the group have to be heated
more strongly before the iodine vapour appears. Explain why thermal stability
increases down the group for Group II iodate(V). [3]
[Total: 20]

Cationic size of M2+ increases down Group II


resulting in decreasing charge density and/or hence decreasing
polarising power.
Thus the electron cloud of the IO 3 - is less distorted and thus less
weakening effect on the I-O bond in IO 3 -.

trendyline
More energy required to bring about decomposition of M(IO 3 ) 2 ,
re
resulting
esulting in higher temperature required for the purple iodine
io vapour to
be
e observed.
Thermal o M(IO 3 ) 2 increases down the group.
hermal stability of

PJC 2016 9647/03/JC2 Preliminary Exam/2016


1045 trendyline
10
3 (a) Methyl t-butyl ether, MTBE, is a fuel additive that is used an oxygenate to raise the
octane number in fuel. This will help the fuel to burn more completely and reduce
air pollution.
O

Methyl t-butyl ether, MTBE

(i) What is meant by the term standard enthalpy change of formation of


MTBE?
[1]
Standard enthalpy change of formation of MTBE is the heat change when
1 mole of a MTBE is formed from its constituent elements in their standard
states under standard conditions of 298 K and 1 atmospheric pressure.

(ii) Use the following data and relevant data from Data Booklet to construct an
appropriate energy cycle, find the standard enthalpy change of formation of
MTBE.

Standard enthalpy change of atomisation of C(s) +715 kJ mol1


Standard enthalpy change of atomisation of MTBE(l) +6822 kJ mol1
[4]
+I 07%(
5C(s) + 6H2(g) + 1/2 O2(g) C5H12O(l)

5 Hat(C ) %( 2 2
6BE(H2)
Hat(C5H12O)

5C(g) + 12H(g) + O(g)

By Hess Law,
H f (MTBE) = 5H at (C) + 6BE(H 2 ) + BE (O 2 ) H at (C 5 H 12 O)
H f (MTBE) = 5(715) + 6BE(H-H) + BE(O=O) - 6822
H f (MTBE) = 3575 + (6 x 436) + (496) - 6822
H f (MTBE) = -383 kJ mol-1

(iii) Given that the standard enthalpy change of vaporisation of MTBE is


+30.4 kJ mol-1 and using relevant data from the Data Booklet and (a)(ii),
calculate the bond energy of C-O in MTBE.

Bond energy of C 5 H 12 O involves breaking a total of:

trendyline
2(C-O), 3(C-C)
O), 3(C
O C) and 12(C-H)
12(C H) bonds.nd

H at (C 5 H 12 2  + vap (C 5 H 12 O) + 2BE(C-O)
2BE(C
2BE( -O) + 3BE(C-C)
3BE(C
3BE(C--C) + 12BE(C-H)
68222 = 30.4 + 2BE(C-O)
2BE(C
BE(C-O) O) + 3BE(C-C)
3BE(C--C) + 12BE(C-H)
3BE(C 12BE(C
12BE(C--H)
6791.6
1.6
6 = 2BE(C
2BE(C-O)
O) + (3 x 350) + (12 x 41 410)
6791.6
16 = 2BE(C-O) + 5970
BE(C-O) = +411 kJ mol-1

PJC 2016 9647/03/JC2 Preliminary Exam/2016


1046 trendyline
11
(iv) Suggest a reason for the difference in the C-O bond energy in (a)(iii) from
the value given in the Data Booklet. [1]

The bond energy values from the Data Booklet are average values. The
actual bond dissociation energy values depend on the specific chemical and
electronic environment that a bond is in.

(b) MTBE can be produced from methanol and but-1-ene by the following equation.
The reaction has a standard entropy change of reaction of -281 J mol1 K1.

CH 3 OH(l) + C 4 H 8 (l) C 5 H 12 O(l) H


MTBE

The table below lists the H f (standard enthalpy change of formation) values for
some compounds.
compound H f / kJ mol-1
CH 3 OH(l) -239
C 4 H 8 (l) -0.4

(i) Calculate the Ho for the reaction in (b) using the values from the table
above and your answer to (a)(ii).
[1]

H= P+ f (products) Q+ f (reactants)
= -383 (-239 0.4)
= -143.6 kJ mol1
= -144 kJ mol1

(ii) Determine the feasibility of the production of MTBE from methanol and
but-1-ene at 298 K. [2]

* = + T6
= -143.6 298(-281 x 103)
= -59.9 kJ mol1 < 0, reaction is energetically feasible at 298 K.
(ECF)

Final answer and statement on energetically feasible.

trendyline
PJC 2016 9647/03/JC2 Preliminary Exam/2016
1047 trendyline
12
(c) Methyl shift can occur in organic chemistry reactions in reactions involving
carbocation intermediates.

An example of a methyl shift in a reaction between an alcohol and HI is shown


below. In this reaction compound D is produced as the major product.

OH H I +
OH2

+
Step 1
-
+ I
+ H2O + I-
carbocation A

methyl shift

+
I + H2O I- + H2O

compound D carbocation B

(i) What is the role of HI in step 1?

acid

(ii) Suggest a reason for the methyl shift to produce carbocation B from
carbocation A. [1]

B is a secondary carbocation which is more stable than the A which is a


primary carbocation. Hence, a more stable carbocation B is formed as an
intermediate during the reaction.

(iii)

The structure of compound E is OH .

Predict the major final product formed when compound E reacts with HBr
based on the methyl shift reaction in (c). [1]
Br

trendyline
PJC 2016 9647/03/JC2 Preliminary Exam/2016
1048 trendyline
13
(iv) Compound D undergoes the following reactions.

step 1 CH3COCl

I NH2 NHCOCH3
D F

step 2

step 3

NH2

Suggest suitable reagents for steps 1, 2 and 3. [3]

step 1: excess NH 3 in ethanol and heat it in sealed tube.


step 2: NaCN in ethanol, heat
step 3: LiAlH 4 in dry ether / H 2 , Ni cat, 2000C.

(v) Suggest one simple chemical test which would enable you to distinguish
O

H2N

between compound F and .

You should state the reagents and conditions for the test and describe the
observations.

[Total: 20]
Reagents and conditions: NaOH(aq), heat
O

H2N

Observation: the gas liberated which is NH 3 turns moist red


litmus paper blue.

trendyline
trendyl
For F, m
moist
oist red litmus paper remains red

PJC 2016 9647/03/JC2 Preliminary Exam/2016


1049 trendyline
14
4 (a) Compound G, C 10 H 15 NO 3 , is a natural stimulant made in the adrenal gland of the
kidney and is carried in the bloodstream and affects the autonomous nervous
system, which controls functions such as heart rate, dilation of the pupils.

G is soluble in dilute sodium hydroxide and dilute hydrochloric acid but does not
react with sodium carbonate.

G undergoes reaction with hot acidified potassium dichromate(VI) to form J. J


gives an orange precipitate with 2,4-dinitrophenylhydrazine but does not give a
silver mirror with Tollens reagent. Upon heating with concentrated H 2 SO 4, G forms
K, C 10 H 13 NO 2 . K does not react with PCl 5 . On heating with acidified potassium
manganate(VII), K forms three compounds, L, C 7 H 6 O 4 , M, C 2 H 4 O 2 , and an
ammonium salt, CH 3 NH 3 +.

Deduce the structures for each compound, G to M, and give an account of the
chemistry involved.
[10]
G is soluble in dilute sodium hydroxide and dilute hydrochloric acid but does not
react with sodium carbonate. G contains basic/amine groups and phenol

G undergoes oxidation with acidified potassium dichromate(VI) to form J, which


undergoes condensation with 2, 4 - DNPH.

=> J is a ketone
=> G contains secondary alcohol

G undergoes elimination with concentrated H 2 SO 4 , heat to form K, C 10 H 14 NO 2


=> G is confirmed to be a secondary alcohol.
=> K is an alkene

K does not react with PCl 5


=> K is a has 2 phenol groups

K undergoes oxidation and acidic hydrolysis with acidified potassium manganate


to form L, M and CH 3 NH 3 +.
OH

CH 3 HO

C
O
O OH C

MI LH OH

OH
OH OH

HO
HO HO

trendyline
ttr
re dyyli e
CH 3
CH 3 CH 3
H
C CH 3
C CH 3
C CH
H3 C N
C N
C N H
H
O H
OH H
KG H H G JF
E
*The position of the substituent is not important in this question.
[1 for each of the compound]
PJC 2016 9647/03/JC2 Preliminary Exam/2016
1050 trendyline
15
(b) Like alcohols and halogenoalkanes, amines can be converted into alkenes by an
elimination reaction known as Hofmann elimination.

For example, 1-methyl-pentylamine is converted into hex1ene as shown.

Hofmann Elimination
+ side products
NH2
reaction

An interesting feature of the Hofmann elimination is that it gives a less substituted


alkene compared to other elimination reactions.

(i) Draw the likely structure of the starting amine if the following alkene N is the
main product formed when the amine undergoes a Hofmann elimination
reaction.

[1]

NH2

(ii) Suggest, in no more than 3 steps, a suitable synthetic pathway to convert N


into the following compounds.

For each step, state clearly the reagents and conditions, indicating the
structural formula of all intermediate compounds formed.
[5]

(I) OH
HO

trendyline
PJC 2016 9647/03/JC2 Preliminary Exam/2016
1051 trendyline
16
COOH

COOH
KMnO4, dil H2SO4
heat

LiAlH4 in dry ether,


r.t.p

CH2OH

= CH2OH
HO OH

(II)

OH O

Br2 (aq) KMnO4(aq),


Br Br
H2SO4(aq),
heat

NaOH in
ethanol

trendyline
PJC 2016 9647/03/JC2 Preliminary Exam/2016
1052 trendyline
17
(c) Amines can be formed by reductive amination of an aldehyde or ketone with
ammonia or amine in the presence of reducing agent.

A reductive amination to produce an amine P takes place in the pathway shown.

CH2Cl CH2Cl OH CH2Cl


CH2COCH3 NH3 CH2 C NH2 CH2 C NH
CH3 Step II CH3
Step I

Step III

CH2Cl
CH2 CH NH2
CH3

P (C10H14NCl)

(i) Suggest the type of reaction that occurs at steps I and II.
[2]
Step I Nucleophilic addition
Step II - Elimination

(ii) Suggest the reagents and conditions for step III.


[1]
H 2 in nickel catalyst, 200oC, LiAlH 4

(iii) On heating, P forms Q, C 10 H 13 N. Suggest a structure for Q.


[1]
H
N

[Total: 20]

trendyline
PJC 2016 9647/03/JC2 Preliminary Exam/2016
1053 trendyline
18
5 (a) An electrochemical cell involving tartaric acid, (CH(OH)CO 2 H) 2 , has the following
cell diagram notation.

Pt(s)|(CH(OH)CO 2 H) 2 (aq) |CO 2 (g) |H+(aq) || Co3+(aq), Co2+(aq)|Pt(s)

(i) Given that the e.m.f of the cell is +0.80 V and using relevant data from the
Data Booklet, determine the standard electrode potential of the
CO 2 (g)|(CH(OH)CO 2 H) 2 (aq) half-cell.

At the cathode, E red (Co3+(aq), Co2+(aq)) = +1.82 V

Since e.m.f. of cell = +0.80 V, and E cell = E red + E oxid


E oxid = E cell E red
= +0.80 (+1.82) = -1.02 V

At the anode, E red = +1.02 V

(ii) Draw a labelled diagram of the electrochemical cell.

electron flow
V <

platinum electrode CO2 gas at


salt bridge
25oC, 1 atm
1 mol dm3 Co3+(aq) 1 mol dm3 H+(aq)
1 mol dm3 Co2+(aq)) 1 mol dm3
(CH(OH)CO2H)2(aq)
platinum electrode

(b) Fehlings solution is made by adding equal volumes of a blue solution A containing
aqueous copper(II) sulfate and a colourless solution B containing potassium
sodium tartrate and sodium hydroxide. In a strong base, an alpha-hydroxy group in
the tartrate ion, (CH(OH)CO 2 ) 2 , deprotonates as shown in the diagram below.

O O
-
- OH - O
O O
- O
- + H+
O OH
OH
O O
Deprotonated tartrate ion

using both
trendyline
tonated tartrate ion acts as a bidentate ligand and binds to the Cu2+(aq)
The deprotonated
h oxygen atoms from tthe alpha
bistartratocuprate(II)
seen.
cuprate(
uprat III) compl
complex
alpha-
alpha-hydroxy
complex,, [Cu(tart)

[tart = deprotonated tartrate ion]


4
4
-hydroxy
ydroxy groups.
Cu(tart 2 ] , forms
groups A square planar
forms, and
a a deep blue solution is

PJC 2016 9647/03/JC2 Preliminary Exam/2016


1054 trendyline
19

(i) Draw the shape of [Cu(tart) 2 ]4 complex, indicating clearly the type of bonds
formed between the Cu2+ and the ligands.

(may want to include coordination number)


[2]
O O

-
OH O
-
.. -
O O
2+
Cu
- -
O O
-
O HO
..

O O

(ii) The open straight chain form of Dglucose, a reducing sugar, is shown.

OH OH O
HO
H
OH OH

Dglucose

State what would be observed when Dglucose solution is warmed with a


few drops of Fehlings solution.

Write the balanced equation for the reaction described above.


[2]
Observation: Brick-red/reddishbrown precipitate of Cu 2 O observed

Equation:
OH OH O OH OH O

HO
H
+ 2Cu 2+
+ 5OH -
Cu2O + HO O
- + 3H2O
OH OH OH OH

(iii) Explain why there is a need to form the [Cu(tart) 2 ]4 complex in the Fehlings
solution test.
[1]

The deprotonated tartrate ions, by complexing with Cu2+ prevents the


precipitation of Cu2+ as solid Cu(OH) 2 from the reaction of CuSO 4 and

t dyli
trendyline
NaOH.

The Cu22++ in the complex can then be reduced to Cu+ in the fform of copper(I)
oxide,
e, Cu 2 O.
2Cu2+ + H 2 O + 2
2e- Cu 2 O + 2
2H+

PJC 2016 9647/03/JC2 Preliminary Exam/2016


1055 trendyline
20
(iv) Explain why the Fehlings solution is deep blue while potassium sodium
tartrate solution is colourless.
[4]
In the presence of ligands, the partially filled 3d-orbitals of Cu2+ split into 2
sets of different energy levels.

The small difference in energy between the two sets of orbitals corresponds
to the energy of visible light in the electromagnetic spectrum

On absorbing energy from the visible light, an electron from a lower d-orbital
gets promoted to a vacant / half-filled orbital of higher energy.

The colour observed is complementary to that absorbed.

For a compound of Group I, the metal ion does not have electrons in the 3d-
orbitals and hence the d-d electron transitions would not occur. Hence, the
solution of this compound is colourless.
(v) Stronger field ligands are known to give rise to a larger energy gap between
the two sets of d-orbitals in a transition metal complex.

Violet Blue Green Yellow Orange Red


400 nm 430 nm 480 nm 560 nm 590 nm 630 nm 750 nm
. Energy of light decreases

To test for proteins, Biuret reagent which also contains the [Cu(tart) 2 ]4
complex is used. In the presence of peptides, it undergoes ligand exchange
to form [Cu(peptide) 2 ]2+. The deep blue solution turns violet.

Using the above electromagnetic spectrum of visible light, state the relative
ligand field strength of peptides and deprotonated tartrate ions. Explain your
answer.
[3]
Peptides are stronger field ligands than the deprotonated tartrate ions.

[Cu(tart) 2 ]4 appears blue because it absorbs orange light. [Cu(peptide) 2 ]2+


appears violet because it absorbs yellow light.

Yellow light has higher energy than orange light, the energy gap between
the d orbitals of [Cu(peptide) 2 ]2+ is larger than that of [Cu(tart) 2 ]4.

trendyline
PJC 2016 9647/03/JC2 Preliminary Exam/2016
1056 trendyline
21
(c) Adolf Kolbe first published a method describing the preparation of an alkane from
the electrolysis of an aqueous solution of sodium salt of a monoprotic acid, using
inert electrodes.

In the Kolbe electrolysis of sodium propanoate, butane was formed at the anode in
the following reaction.

2CH 3 CH 2 CO 2 CH 3 CH 2 CH 2 CH 3 + 2CO 2 + 2e-

(i) Write the half equation for the reaction at the cathode during the electrolysis
of sodium propanoate. Hence write an equation for the overall reaction.
[2]

2H 2 O + 2e H 2 + 2OH
Overall reaction:
2CH 3 CH 2 CO 2 + 2H 2 O CH 3 CH 2 CH 2 CH 3 + 2CO 2 + H 2 + 2OH

(ii) A student tried to synthesise another hydrocarbon using an aqueous


solution of a sodium salt of a diprotic acid, succinic acid, (CH 2 CO 2 H) 2 . The
succinate ion, (CH 2 CO 2 ) 2 ,undergoes a similar reaction as propanoate ion
at the anode.
(CH 2 CO 2 ) 2 R + 2CO 2 + 2e-

Suggest an identity of the hydrocarbon R.


[1]
CH 2 =CH 2 OR C 2 H 4 [1]

[Total : 20]

End of Paper

trendyline
PJC 2016 9647/03/JC2 Preliminary Exam/2016
1057 trendyline
1

RAFFLES INSTITUTION
2016 YEAR 6 PRELIMINARY EXAMINATION

Higher 2

CHEMISTRY 9647/01
Paper 1 Multiple Choice 26 September 2016
1 hour
Additional Materials: Multiple Choice Answer Sheet
Data Booklet

READ THESE INSTRUCTIONS FIRST

Do not open this question booklet until you are told to do so.

Write in soft pencil.


Do not use staples, paper clips, highlighters, glue or correction fluid.
Write your name, class and index number in the spaces provided on the Answer Sheet.

There are forty questions in this paper. Answer all questions.

For each question there are four possible answers A, B, C and D.


Choose the one you consider correct and record your choice in soft pencil on the separate
Answer Sheet.

Read the instructions on the Answer Sheet very carefully.

Each correct answer will score one mark. A mark will not be deducted for a wrong answer.

Any rough working should be done in the question booklet.

trendyline This
his document consists of 18 printed
rinted pages.
pages

Raffles Institution 2016 9647/01/S/16 [Turn Over

1058 trendyline
2

Section A
For each question, there are four possible answers, A, B, C, and D. Choose the one you
consider to be correct.

1 Which of the following statements best explains why helium has the highest first
ionisation energy among all the elements in the Periodic Table?

A It is unreactive.
B It is the least electronegative element.
C It has only one completely filled principal quantum shell.
D Its valence electron is poorly shielded and is very close to the nucleus.

2 When a beam of protons travelling at the same speed passes through an electric field
of constant strength, the protons, 1H+, are deflected through an angle of +12o.

Under identical conditions, which particle would be deflected through an angle of +8o?

2
A H+ B 3
He2+ C 6
Li2+ D 12
C3+

3 Use of the Data Booklet is relevant to this question.

Which particle has three unpaired electrons?

A V3+ B F2+ C Ni D S

4 FeSO4 reacts with K2Cr2O7 in acid solution according to the following equation.

14H+ + 6Fe2+ + Cr2O72 o 2Cr3+ + 6Fe3+ + 7H2O

A 20.0 cm3 sample of FeSO4 solution required 13.10 cm3 of 0.100 mol dm3 acidified
K2Cr2O7 for complete reaction.

What was the concentration of the FeSO4 solution?

A 0.0655 B 0.153 C 0.393 D 0.916

trendyline
Raffles Institution 2016 9647/01/S/16 [Turn Over

1059 trendyline
3

5 1,3,5-triazine is an aromatic compound and its structure resembles that of benzene.

1,3,5-triazine

Which statement about 1,3,5-triazine is correct?

A Its empirical formula is CN.


B A molecule of 1,3,5-triazine has six V bonds.
C The CNS bond is formed by 2sp22sp2 overlap.
D All the carbon-nitrogen bonds have equal bond lengths.

6 Which of the following shows the correct shape and bond angle of the molecule given?

molecule shape bond angle


A OF2 bent 117o
B BH3 trigonal pyramidal 120o
C XeF4 tetrahedral 109.5o
D SF6 octahedral 90o

7 At 25 oC and 101 kPa, a 1.00 g sample of dry air contains 78.09% nitrogen,
20.95% oxygen, 0.93% argon and 0.03% carbon dioxide by volume.

What is the average relative molecular mass of the air sample?

A 14.67 B 24.52 C 28.95 D 30.26

8 Which of the following statements about an ideal gas is correct?

A The total volume of the individual molecules is significant when compared to the
volume of the container which the gas occupies.

trendyline
B An ideal g
gas can liquefy
q y at low temperature
p and high
g pressure.
p
C The gas
as molecules are in constant, random motion.
motion
D The gas
as molecules have no mass.
mas

Raffles Institution 2016 9647/01/S/16 [Turn Over

1060 trendyline
4

9 Sodium carbonate reacts with ethanoic acid according to the equation below.

'Hr
Na2CO3(s) + 2CH3COOH(aq) o 2CH3COONa(aq) + H2O(l) + CO2(g)

In an experiment to determine the enthalpy change of reaction, 'Hr, 7.5 g of solid


sodium carbonate (Mr = 106) was added to 50 cm3 of excess aqueous ethanoic acid.
The temperature of the resultant solution was monitored at various time intervals and
the following graph was obtained.

time of mixing of reactants

Given that the specific heat capacity of the solution is 4.18 J cm3 K1, what is the
enthalpy change of reaction, 'Hr?

A +27.5 kJ mol1
B +29.5 kJ mol1
C +31.6 kJ mol1

trendyline
D +34.0 kJ mol1

Raffles Institution 2016 9647/01/S/16 [Turn Over

1061 trendyline
5

10 The DNA double helix consists of two DNA strands held together by hydrogen bonding.

When heated, the DNA double helix separates into two random-coil single strands.
When cooled, the random coils reform the double helix.

DNA double helix 2 random coils

Which of the following graphs corresponds to the forward process?

A B

'G / kJ mol1 'G / kJ mol1

0 T/K

0 T/K

C D

'G / kJ mol1 'G / kJ mol1

0 T/K 0 T/K

11 Use of the Data Booklet is relevant to this question.

1,10-phenanthroline (phen) is an organic compound that is used as a ligand in


coordination chemistry. It forms complexes with metal ions such as Fe3+ and Fe2+.

The redox equilibrium between [Fe(phen)3]3+ and [Fe(phen)3]2+ is shown below.

[Fe(phen)3]3+ + e [Fe(phen)3]2+ E{ = V

Aqueous chlorine oxidises [Fe(phen)3]2+ to [Fe(phen)3]3+, but both aqueous bromine

trendyline
and iodine do not.

What could ?
of?
d be the value of

A 0.88 B 1.18
1 18 C +0.88
0 88 D +1.18

Raffles Institution 2016 9647/01/S/16 [Turn Over

1062 trendyline
6

12 Use of the Data Booklet is relevant to this question.

An electrochemical cell is set up as shown:

T = 298 K

Pt salt bridge Cu

[Cr2O72] = [Cr3+] = [H+] = 0.1 mol dm


m3 [Cu2+] = 0.1 mol dm3

Which statement about the e.m.f. of the above cell is correct?

A The e.m.f. of the cell is +0.99 V.


B Addition of water to the Cr2O72/Cr3+ half-cell decreases the e.m.f. of the cell.
C Addition of H+(aq) to the Cu2+/Cu half-cell decreases the e.m.f. of the cell.
D Addition of excess NH3(aq) to the Cu2+/Cu half-cell decreases the e.m.f. of the cell.

13 The two-step mechanism for the reaction


2X(g) + Y(g) o X2Y(g)
is shown below.

Step 1: X(g) + Y(g) o XY(g) (slow)


Step 2: XY(g) + X(g) o X2Y(g) (fast)

Which of the following is the rate equation for the reaction?

A rate = k[X]2
B rate = k[X][Y]
C rate = k[XY][X]
D rate = k[X]2[Y]

trendyline
Raffles Institution 2016 9647/01/S/16 [Turn Over

1063 trendyline
7

14 Ethanoic acid dissociates in aqueous solution as follows, where is the degree of


dissociation.

CH3CO2H(aq) CH3CO2(aq) + H+(aq)


1 

Which of the following statements is correct?

2C
A The Ka, in terms of and initial concentration C, is .
1

B pH decreases with decreasing concentration of ethanoic acid.

C Ka increases with increasing concentration of ethanoic acid.

D does not change with the concentration of ethanoic acid.

15 The solubility product, Ksp, of magnesium hydroxide at 25 C is X.

The solubility of magnesium hydroxide at 25 C is S.

Which of the following statements is correct at 25 C?

A The pH of a saturated solution of magnesium hydroxide is 14 + lg(2X)1/3.

B The solubility of magnesium hydroxide in a solution of magnesium chloride is


larger than S.
C When solid sodium hydroxide is dissolved in a saturated solution of magnesium
hydroxide, Ksp of magnesium hydroxide becomes smaller than X.

D The Ksp of barium hydroxide is smaller than X.

16 Which property of the Period 3 elements, from Na to Cl, increases consistently across
the period?

A enthalpy change of atomisation


B number of unpaired electrons
C electronegativity
D atomic radius

trendyline
Raffles Institution 2016 9647/01/S/16 [Turn Over

1064 trendyline
8

17 Use of the Data Booklet is relevant to this question.


W, X, Y and Z are four consecutive elements in the third period of the Periodic Table.
The formula of the hydride formed by each element is given in the table below.

element W X Y Z

formula of hydride WH2 XH3 YH4 ZH3

Which of the following shows the correct trends when the second ionisation energies of
the elements are plotted against the highest oxidation numbers that the elements
exhibit in their oxides?

A B

C D

18 An azide ion, N3, is called a pseudohalide because it has some properties which are
similar to those of chloride ions.
Which of the following statements is not likely to be correct?

trendyline
A CH3CH
H2N3 reacts with hot, ethanolic KCN to give CH3CH2CN.
B N3 ions
s react with a Cu2+ ion
ns n to give a complex, [Cu(N3)4]22.
C HN3 may
ay be prepared by warming NaN
Na 3 with
wit concentrated H2SO4.
D AgN3 is soluble in water.
water

Raffles Institution 2016 9647/01/S/16 [Turn Over

1065 trendyline
9

19 Which chlorine-containing products are obtained when chlorine is bubbled into hot
concentrated NaOH?

A NaClO3 only
B NaClO only
C NaCl and NaClO3
D NaCl and NaClO

20 A Lewis acid is an electron-pair acceptor, while a Lewis base is an electron-pair donor.

1 C6H6 + Br+ o [C6H6Br]+


2 CH3CH2NH3+ + NH3 o CH3CH2NH2 + NH4+
3 CH3CH2+ + Br o CH3CH2Br

Which equations represent a Lewis acid-base reaction?

A 3 only
B 1 and 2
C 2 and 3
D 1, 2 and 3

21 What is the total number of isomers possible for C3H4?

A 1 B 3 C 5 D 7

trendyline
Raffles Institution 2016 9647/01/S/16 [Turn Over

1066 trendyline
10

22 The following reaction was found to be exothermic.

(CH3)3CBr + NaOH o (CH3)3COH + NaBr

The rate equation of the reaction is: rate = k[(CH3)3CBr].

Which reaction profile fits the given information?

A B

Energy Energy

(CH3)3CBr product
products

+ OH
(CH3)3CBr
products + OH

Progress of reaction Progress of reaction

C D

Energy Energy

(CH3)3CBr (CH3)3CBr

+ OH + OH

products products

Progress of reaction Progress of reaction

23 One gram of each of the following compounds was heated with NaOH(aq), and then
dilute HNO3(aq) and AgNO3(aq) were added.

CH3CH2Cl CH2=CHBr CH3COBr C6H5I

Which statement is correct?

A The reaction with C6H5I gave a pale yellow precipitate.

trendyline
B The reaction
eaction
ac with CH3COBr gave a precipitate that dissolved completely
comp in NH3(aq).
C The reaction
eaction with CH2=CHBr
CHBr gave 1.76 g of precipitate.
D The reaction
eaction
action with CH3CH2Cl gave the largest mass of precipitate.

Raffles Institution 2016 9647/01/S/16 [Turn Over

1067 trendyline
11

24 Sorbitol is a sugar alcohol with a sweet taste, and is used as a sugar substitute.

The structures of sorbitol and glucose are given below.

sorbitol glucose

A sample containing a mixture of sorbitol and glucose reacts completely with 0.2 mol of
hydrogen gas in the presence of nickel catalyst.

Another identical sample reacts with excess sodium metal to form 1.4 mol of hydrogen
gas at room temperature and pressure.

What is the sorbitol : glucose ratio in the sample?

A 3:2
B 3:1
C 6:1
D 7:1

25 Aspirin has the following structure.

If 2-hydroxybenzoic acid is used as the starting material for making aspirin, which set of
reagents and conditions will give the best yield?

A ethanoyl chloride at room temperature


B phosphorus pentachloride at room temperature
C heat with ethanol and a small amount of concentrated sulfuric acid

trendyline
D heat with
th ethanoic acid and a small amount of conc
concentrated sulfuri
sulfuric acid

Raffles Institution 2016 9647/01/S/16 [Turn Over

1068 trendyline
12

26 Alkylbenzenes can be synthesised from benzene by introducing an acyl group via


electrophilic substitution, followed by the reduction of the carbonyl group with zinc
amalgam and hydrochloric acid.

For example, ethylbenzene can be synthesised by the following route:

CH3COCl, AlCl3 Zn(Hg), HCl, heat

Which compound cannot be synthesised by the same method?

A B

C D

27 Cortisone is a hormone released by the adrenal gland in response to stress.

cortisone

When cortisone is reacted with excess lithium aluminium hydride, how many chiral
centres will be present in the product molecule?

A 7 B 8 C 9 D 10

trendyline
Raffles Institution 2016 9647/01/S/16 [Turn Over

1069 trendyline
13

28 Consider the following four compounds, which are structural isomers of one another.

1 CH2ClCHClCOOH
2 CHCl2CH2COOH
3 CHCl2COCH2OH
4 ClCH2OCOCH2Cl

Which sequence arranges the compounds in order of decreasing Ka?

A 1 2 3 4
B 2 1 3 4
C 4 3 1 2
D 4 3 2 1

29 The Hunsdiecker reaction is a reaction in which halogenoalkanes can be produced


from silver carboxylate salts, with the use of halogens. A proposed mechanism of the
reaction is shown below.

Which statement is not correct?

A Step 1 is favoured by the formation of a precipitate.


B Step 2 involves heterolytic fission.
C CO2 is also a product in step 3.
D Alkane RR is a possible side-product of the reaction.

30 In the tertiary structure of a water-soluble globular protein, it was found that the types of
amino acid residues present on the outer surface of the protein were different from
those present on the inside.

Which of the following amino acid residues is most likely found on the outer surface of
such proteins?

A B

C
ttrendyline
rre
e liine
n D

Raffles Institution 2016 9647/01/S/16 [Turn Over

1070 trendyline
14

Section B
For each of the following questions, one or more of the three numbered statements 1 to 3
may be correct.
Decide whether each of the statements is or is not correct (you may find it helpful to put a tick
against the statements that you consider to be correct).
The responses A to D should be selected on the basis of
A B C D
1, 2 and 3 1 and 2 2 and 3 1 only
are only are only are is
correct correct correct correct

No other combination of statements is used as a correct response.

31 In which pairs does compound 1 have a higher melting point than compound 2?

compound 1 compound 2
1 CH3CO2NH4 CH3CH2NH2
2 SiCl4 SiO2
3 Ni(CO)4 NiSO4

32 The Maxwell-Boltzmann distribution for gas M at a given temperature is shown below.

n = number of molecules
with a given energy

nb
nc
na

0 A B C molecular energy

Which statements are correct for the number of molecules with molecular energies
A, B and C?

1
2
3
trendyline
nc decreases
creases
reases when more gas M is added at the same temperature.
na increases
reases
eases when temperature is lowered.
Addition
on of catalyst at the same temperature
temperature

ratu has no effect on na, nb and nc.

Raffles Institution 2016 9647/01/S/16 [Turn Over

1071 trendyline
15

The responses A to D should be selected on the basis of


A B C D
1, 2 and 3 1 and 2 2 and 3 1 only
are only are only are is
correct correct correct correct

No other combination of statements is used as a correct response.

33 The stoichiometry of a catalysed reaction is shown by the equation below.

P(g) + Q(g)  R(g) + S(l)

Two experiments were carried out in which the rate of production of R was measured.
The results are shown in the diagram below.

Which changes in the conditions might explain the results shown?

1 A lower pressure was used in experiment 2.


2 A different catalyst was used in experiment 2.
3 Product S was continuously removed from the reaction vessel in experiment 2.

34 Use of the Data Booklet is relevant to this question.

Two catalysed reactions and their respective catalysts are shown below.

Reaction Equation Catalyst


I S2O82 + 2I o 2SO42 + I2 Fe3+
II 2MnO4 + 5C2O42 + 16H+ o 2Mn2+ + 10CO2 + 8H2O Mn2+

Which statements about the reactions and the catalysts are correct?

trendyline
1 Mn2+ iss an auto-catalyst in reaction II.
a auto-catalyst
2 Fe2+ could
ould replace Fe3++ as a homogeneous catalyst in reaction I.
3 Mn2+ could
ould replace Fe3++ as a homogeneous catalyst in reaction I.

Raffles Institution 2016 9647/01/S/16 [Turn Over

1072 trendyline
16

The responses A to D should be selected on the basis of


A B C D
1, 2 and 3 1 and 2 2 and 3 1 only
are only are only are is
correct correct correct correct

No other combination of statements is used as a correct response.

35 Use of the Data Booklet is relevant to this question.


Element 117 is one of the most recently discovered elements in the Periodic Table.
In June 2016, the International Union of Pure and Applied Chemistry announced that
the discoverers have proposed the name, Tennessine (117Ts), for this element.
Which properties are Tennessine most likely to have?

1 Ts is a dark coloured solid at room temperature and pressure.


2 The covalent radius of Ts is greater than 0.140 nm.
3 The 6d and 7s orbitals of a Ts atom are completely filled with electrons.

36 Which of the following reactions will form a coloured product?

heat
1 Ca(NO3 )2 (s)  o
heat
2 NaI(s)  H2SO4 (l)  o

3 2,4-DNPH

37 In which reactions is the product shown a possible product?

2 excess Br2(aq)
dark, rtp

3
endyl
Raffles Institution 2016 9647/01/S/16 [Turn Over

1073 trendyline
17

The responses A to D should be selected on the basis of


A B C D
1, 2 and 3 1 and 2 2 and 3 1 only
are only are only are is
correct correct correct correct

No other combination of statements is used as a correct response.

38 The reaction of methyl alcohols with alkaline aqueous iodine can be described by the
following scheme:

Which of the following types of reactions are present?

1 oxidation
2 acid-base reaction
3 nucleophilic addition

39 Lithium acetylide, HCC Li+, is a useful nucleophilic reagent in organic synthesis.


Lithium acetylide is reacted with compound X, followed by the addition of water.

X
Which of the following could be the products of the above reaction?

3 trendyline
tr
r
Raffles Institution 2016 9647/01/S/16 [Turn Over

1074 trendyline
18

The responses A to D should be selected on the basis of

A B C D
1, 2 and 3 1 and 2 2 and 3 1 only
are only are only are is
correct correct correct correct

No other combination of statements is used as a correct response.

40 Fibroin, a protein present in Spidermans web, consists of layers of anti-parallel beta-


pleated sheets. Its primary structure consists of the amino acid sequence shown below:

Man-made Kevlar can be stretched without snapping easily. Spiderman is impressed


by the tensile strength of the man-made Kevlar and intends to upgrade his spider silk
so that it will not snap midway through his swings.

The primary structure of Kevlar is shown below:

Which statements are correct?

1 Both fibroin and Kevlar have high tensile strength due to their ability to form
intermolecular hydrogen bonding between adjacent strands.

2 Kevlars additional strength is derived from instantaneous dipole induced dipole


interactions between the aromatic rings of adjacent strands.

3 Kevlar is less water-absorbent than fibroin due to the non-polar aromatic rings,
and thus could be used on rainy days.

trendyline END OF PAPER


APE

Raffles Institution 2016 9647/01/S/16 [Turn Over

1075 trendyline
1

RAFFLES INSTITUTION
2016 YEAR 6 PRELIMINARY EXAMINATION

Higher 2

CANDIDATE
NAME

CLASS INDEX NUMBER

CHEMISTRY 9647/02
Paper 2 Structured Questions 14 September 2016
2 hours
Candidates answer on the Question Paper.

Additional Materials: Data Booklet

READ THESE INSTRUCTIONS FIRST

Write your name, class and index number in the spaces provided at the top of this page.
Write in dark blue or black pen in the spaces provided.
You may use a soft pencil for any diagrams, graphs or rough working.
Do not use staples, paper clips, highlighters, glue or correction fluid.

Answer all questions.


A Data Booklet is provided. Do not write anything in it.
You are reminded of the need for good English and clear presentation in your answers.

The number of marks is given in brackets [ ] at the end of each question or part question.

For Examiners Use

1 / 12

2 / 13

3 / 13

4 / 18

5 / 16

ttrendyline
tren
ndy
dylin
line
Paper 1 Paper
aper 2 Paper 3 Planning
nn Pen
Penalty Total
Tota Grade

/ 40 / 60
6 / 80 / 12
1 / 100

This document consists of 24 printed pages.

Raffles Institution 2016 9647/02/S/16 [Turn Over


1076 trendyline
2

1 Planning (P) For


examiners
use
Copper plating is a method employed to coat an objects surface with a thin layer of
copper. The object to be plated is lowered into an electrolyte containing Cu2+ ions, which
are reduced on the surface of the object to form a thin layer of copper when an electric
current is passed through the set-up.
This set-up can also be used to determine the Faraday constant (F), which is the charge
carried by 1 mol of electrons, as described below.
A pre-weighed metal object and a pure copper rod are placed in a copper(II) sulfate
solution, and then connected to an electrical source. After a sufficient duration, the mass
of copper plated onto the metal object is determined by taking the difference between the
objects mass before and after plating.

The mass of copper plated is dependent on the amount of electrical charge flowing
through the circuit. The value of the Faraday constant may be determined graphically by
repeating the experiment several times using different amounts of electrical charge, and
plotting the collected data on a graph.

(a) The amount of electrical charge flowing through the circuit is dependent on time
and electrical current.
Show that the relationship between the mass of copper plated onto the metal object
and the time taken for the copper plating is given by:

63.5 I where
m t m = mass of copper plated in grams (g),
2F I = current of circuit in amperes (A),
t = time in seconds (s)

[2]

(b) The value of the Faraday constant (F) is widely accepted by chemists to be
96500 C mol1.
Using this value and given an electrical power source that supplies a constant
current of 0.500 A, calculate the maximum time (to the nearest second) for which
copper plating can be carried out so that the mass of copper plated onto the metal
object will not exceed 1.00 g.

trendyline maximum
aximum time = ......
max ............ [1]

Raffles Institution 2016 9647/02/S/16 [Turn Over


1077 trendyline
3

(c) By considering your answers to (a) and (b), write a plan for determining the value of For
the Faraday constant in a laboratory. examiners
use

You may assume that you are provided with:

x 0.0500 mol dm3 copper(II) sulfate solution,


x 5 pieces of pure copper, each weighing approximately 1.5 g,
x 5 pieces of metal objects to be plated, each weighing exactly 2.00 g,
x an electrical power source that supplies a constant current of 0.500 A,
x distilled water,
x filter paper,
x the equipment and materials normally found in a school or college laboratory.

Your plan should include the following:

x a clearly labelled diagram of the set-up,


x details of how you would vary a variable of this experiment to collect 5 sets of
data that will be used to calculate the Faraday constant,
x a sketch of the graph you would expect to obtain using the data collected and
how the graph can be used to calculate the Faraday constant,
x the measures you would take to ensure the accuracy of the experiment.

Diagram for experimental set-up:

..

..

..

..

trendyline
..

..
..
..

..
..
..

Raffles Institution 2016 9647/02/S/16 [Turn Over


1078 trendyline
4

For
examiners
.. use

..

..

..

..

..

..

..

..

..

..

..

..

..

..

..

..

..

..

trendyline [9]

[Total: 12]

Raffles Institution 2016 9647/02/S/16 [Turn Over


1079 trendyline
5

2 (a) Vitamin C refers to a group of compounds which includes ascorbic acid and its For
salts. examiners
use

ascorbic acid compound A

Compound A is the mirror image of ascorbic acid.

(i) Besides compound A, ascorbic acid has two other stereoisomers. Draw the
structures of these two isomers.

[2]

(ii) Pure ascorbic acid rotates plane-polarised light clockwise by 20 (or +20).
Compound A was mixed with ascorbic acid, in an unknown proportion, in a
beaker.
Plane-polarised light was shone through the beaker and it was found that
the light had rotated 10 anti-clockwise (or 10).
Calculate the percentage of ascorbic acid and compound A in the mixture.

trendyline
% ascorbic acid = .;; % compound A = . [2]

Raffles Institution 2016 9647/02/S/16 [Turn Over


1080 trendyline
6

(iii) Ascorbic acid is used as a food additive due to its solubility in water. Explain For
why ascorbic acid is soluble in water. examiners
use

..

..

..

..
[1]

(b) Ascorbic acid has antioxidant properties and it exists mainly as the ascorbate ion at
the physiological pH of 7.4.

An ascorbate ion reacts with a hydroxyl free radical in a 1:1 ratio to give product B
and a radical anion C as shown below:

(i) Explain what is meant by a free radical.

..

..
[1]

(ii) Suggest the identity of B.

..
[1]

trendyline
Raffles Institution 2016 9647/02/S/16 [Turn Over
1081 trendyline
7

(c) Asparagine and serine are important D-amino acids in the synthesis of proteins. For
examiners
use

asparagine serine

Suggest, with reagents and conditions, a simple chemical test to distinguish


between the two D-amino acids.

Reagents and conditions:

...

...

Observations:

...

...

...

...
[2]

Raffles Institution 2016 9647/02/S/16 [Turn Over


1082 trendyline
8

(d) Nitrogen mustard gas is commonly used in chemotherapy for cancer. For
examiners
It was suggested that the synthesis of nitrogen mustard can be carried out via the use
following pathway:

ICl in CCl4
compound D
step 1
step 2
ethene

where X = Cl or I

nitrogen mustard

(i) Suggest the structure for compound D.

[1]

(ii) State and explain whether X (in nitrogen mustard) is more likely to be Cl
or I.

..

..

..
[1]

(iii) Suggest the reagents and conditions for step 2.

..

..
[2]
[Total: 13]

trendyline
Raffles Institution 2016 9647/02/S/16 [Turn Over
1083 trendyline
9

3 Use of the Data Booklet is relevant to this question. For


examiners
(a) Haemocyanin is the primary oxygen transport molecule found in the blood stream use
of lobsters.
Unlike haemoglobin which contains iron, haemocyanin contains copper that binds
reversibly to oxygen molecules. Oxygenation of haemocyanin results in an
oxidation reaction, and a colour change as shown below:

Cu(I) Cu(II)
deoxygenated oxygenated
colourless blue

(i) Write the full electronic configuration of a Cu2+ ion.

[1]

(ii) By considering your answer to (a)(i), explain why oxygenated haemocyanin


is coloured.

..

..

..

..

..

..

..

..

..

..
[3]

(iii) Account for the absence of colour in deoxygenated haemocyanin.

..

..

..

trendyline
..

..

[1]

Raffles Institution 2016 9647/02/S/16 [Turn Over


1084 trendyline
10

(b) Bis(tartrate) complex of Cu2+, which is present in Fehlings solution, is used to test For
for aldehydes. examiners
use

(i) The reduction half-equation of bis(tartrate) complex of Cu2+ is given as:


2Cu2+(complex) + 2OH + 2e o Cu2O + H2O
Construct the half-equation for the oxidation of aldehyde (RCHO) under
alkaline conditions, and hence, construct the overall equation.

Oxidation half-equation:

..

Overall equation:

..
[2]

(ii) The bis(tartrate) complex of Cu2+ has the following structure:

It is synthesised by reacting aqueous Cu2+ ions with tartrate ions, C4H4O62,


in an alkaline medium as shown below:

2C4H4O62 + Cu2+ + 2OHo 2H2O

Draw the structure of the tartrate ion, C4H4O62.

trendyline [1]

Raffles Institution 2016 9647/02/S/16 [Turn Over


1085 trendyline
11

(c) Silver is a common metal used for jewellery. However, due to the increased For
amounts of hydrogen sulfide being released into the atmosphere by the examiners
use
combustion of fossil fuels, tarnishing of silver occurs relatively quickly, resulting in
the formation of black silver sulfide.

Tarnished silver has a layer of silver sulfide, Ag 2S, on its surface. It may be
restored easily by dipping it into warm water containing a piece of aluminium foil
and a pinch of sodium chloride.

warm water
containing NaCl
tarnished silver
jewellery aluminium foil

(i) The piece of aluminium acts as the anode which reduces the silver sulfide;
itself being oxidised to aluminium sulfide. Write the balanced equation for
this reaction.

..
[1]

(ii) A student calculated E{cell for this reaction using E{(Ag+/Ag) and E{(Al3+/Al).

However, the actual Ecell of this reaction is different from what he has
calculated. Besides not being performed under standard conditions,
suggest another reason to account for the difference.

..

..

..

..
[1]

(iii) Suggest a purpose for the addition of sodium chloride in this reaction.

..

..

..

..
[1]

Raffles Institution 2016 9647/02/S/16 [Turn Over


1086 trendyline
12

(iv) While carrying out this procedure, a faint aroma of rotten egg smell is given For
off. This is due to the further reaction of aluminium sulfide with water, examiners
use
producing aluminium hydroxide and hydrogen sulfide gas.
Al2S3 + 6H2O 2Al(OH)3 + 3H2S
17.1 cm3 of hydrogen sulfide gas was released at the end of the experiment,
which was conducted at room temperature and pressure.
Using the information given and your answer to (c)(i), calculate the
percentage of silver that was tarnished if the original jewellery contained
2.00 g of pure silver.

[2]

[Total: 13]

trendyline
Raffles Institution 2016 9647/02/S/16 [Turn Over
1087 trendyline
13

4 Phenols and compounds containing the phenol functional group have many uses. For
examiners
(a) One use of phenol is in studying the kinetics of the reaction between bromide and use
bromate(V) ions in acid solution.
The initial rate of this reaction can be studied by the clock method using phenol
and methyl orange. The equations for the reactions are as follows:

5Br + BrO3 + 6H+ 3Br2 + 3H2O reaction I

+ 3Br2 + 3HBr reaction II

Br2 + methyl orange (acid form) o bleached methyl orange + 2Br reaction III
pink colourless
Aqueous Br is added to acidified BrO3 solution containing a small and constant
amount of phenol solution and methyl orange indicator. The Br2 produced by
reaction I will immediately react in reaction II until all the phenol has been used up.
At that point, free Br2 will be present in the solution and bleach the methyl orange
reagent in reaction III. This causes the reaction mixture to turn from pink to
colourless.
To study the kinetics of reaction I, KBr(aq), KBrO3(aq) and H2SO4(aq), each with
an initial concentration of 0.100 mol dm3, are reacted with 0.00010 mol dm3 of
phenol solution.

(i) Explain briefly why bromination of phenol occurs readily.

..

..

..

..
[1]

(ii) It is noticed that the reaction mixture turns a little cloudy as the reaction
progresses. Give a reason for the observation of cloudiness in the reaction
mixture.

..

trendyline
..

..

..


[1]

Raffles Institution 2016 9647/02/S/16 [Turn Over


1088 trendyline
14

For
To find the order of reaction with respect to H+, five experiments are conducted. examiners
use
In all the experiments, 10 cm3 of KBr(aq), 10 cm3 of KBrO3(aq), 5 cm3 of phenol
solution and 2 drops of methyl orange indicator are used, with water added to keep
the total volume constant. The volume of H2SO4(aq) used and the time taken for the
mixture to turn from pink to colourless for the 5 experiments are shown in the table
below:

Time taken for


Experiment Volume of 1
colour to change
H2SO4(aq), V V2 / cm6 / s1
No. from pink to t
/ cm3
colourless, t / s
1 24.00 576 10.0 0.100
2 20.00 400 14.4 0.069
3 16.00 256 22.5 0.044
4 12.00 144 40.0 0.025
5 8.00 64 90.0 0.011

(iii) State the relationship between


x initial rate and t
x concentration of H+ and V.

..

..

..

..
[1]

(iv) To determine the order of reaction with respect to H+, use the data in the
table above to plot a straight line graph on the grid provided on page 15.
Label the axes clearly.

trendyline
Raffles Institution 2016 9647/02/S/16 [Turn Over
1089 trendyline
15

For
examiners
use

[3]

(v) With reference to the graph, state the order of reaction with respect to H+.
Briefly explain your answer.

..

..

trendyline
..

..

[2]

Raffles Institution 2016 9647/02/S/16 [Turn Over


1090 trendyline
16

(b) Bisphenol A, commonly known as BPA, has the structure shown below: For
examiners
use

It is one of the two reactants used to make polycarbonate plastic which is a


lightweight, high-performance polymer used in making containers, sports safety
equipment and medical devices.

When reacted with compound E, BPA undergoes condensation to form


polycarbonate plastic. The structure of the polycarbonate, showing two repeat units,
is shown below:

(i) Compound E contains carbon, oxygen and halogen only. It reacts with
aqueous silver nitrate to give a white precipitate. Draw the displayed formula
of compound E.

[1]

Bakelite, a polymer made from the condensation reaction of phenol and


methanal, has the structure shown below:

trendyline
ndyl e
Raffles Institution 2016 9647/02/S/16 [Turn Over
1091 trendyline
17

(ii) With reference to their structures, explain why Bakelite is more rigid than For
polycarbonate plastic. examiners
use

..

..

..

..

..

..
[1]

trendyline
Raffles Institution 2016 9647/02/S/16 [Turn Over
1092 trendyline
18

(c) Umbelliferone is a yellowish-white crystalline solid occurring naturally in carrots and For
is used as a sunscreen agent. It has the structure shown below: examiners
use

The following reaction scheme shows how umbelliferone may be synthesised from
compound F, which has a molecular formula of C8H8O3.
In the boxes provided, fill in the missing
x reagents and conditions,
x structures of compounds F and G.

compound F H2SO4(aq),
C8H8O3 heat under reflux

compound G

tre
trendyline
re dy in
umbelliferone
ne
i [5]

Raffles Institution 2016 9647/02/S/16 [Turn Over


1093 trendyline
19

(d) Thymol is a white crystalline solid with a pleasant odour and provides the distinctive For
strong flavour of the herb thyme. It has the structure shown below: examiners
use

thymol

(i) Carvacrol is a positional isomer of thymol.

When warmed with acidified aqueous potassium manganate(VII), both


thymol and carvacrol gave the same organic product.

Draw the structure of carvacrol.

[1]

trendyline
Raffles Institution 2016 9647/02/S/16 [Turn Over
1094 trendyline
20

(ii) Compound H has the structure shown below: For


examiners
use

compound H

To separate H from a mixture of H and thymol, the following method is used.


Using a separatory funnel, an ether solution of H and thymol is shaken with
an aqueous solution J, and the aqueous layer is separated from the organic
layer.

1. Add 2. Vigorous 3. Remove


aqueous shaking aqueous
solution J layer

ether solution of H aqueous solution J


and thymol

aqueous layer

If the organic layer contains only compound H after separation, suggest a


suitable reagent which can be used as aqueous solution J. Give a reason
for your answer.

..

..

..

..

..

trendyline
..

[2]

[Total: 18]

Raffles Institution 2016 9647/02/S/16 [Turn Over


1095 trendyline
21

5 The table below gives data about some physical properties of some Group I and II For
examiners
elements. use
property potassium calcium strontium barium radium
relative atomic mass 39.1 40.1 87.6 137
atomic radius (metallic) / nm 0.227 0.197 0.215 0.217 0.220
ionic radius (2+) / nm 0.099 0.113 0.135 0.140
3
density / g cm 0.890 1.55 2.60 3.50 5.50
1st ionisation energy / kJ mol1 418 590 548 502 509
2nd ionisation energy / kJ mol1 3070 1150 1060 966 979
solubility of sulfate in water
11.1 0.210 0.0130 0.000240
(at 20 C) / g per 100 cm3

(a) Explain why the atomic radius of barium is larger than that of strontium.

...

...

...

...

...

...
[2]

(b) Calcium and barium were separately added to cold water. Describe what you would
observe and write equations for the reactions that occur.

...

...

...

...

...

...

...

trendyline
...

...

[3]

Raffles Institution 2016 9647/02/S/16 [Turn Over


1096 trendyline
22

(c) When added to water, potassium floats on the water surface. Explain whether For
calcium floats or sinks when it is first added to water. examiners
use

...

...

...

...
[1]

(d) Determination of the relative atomic mass of an element depends on information


like the relative isotopic masses of the isotopes and the relative abundance of each
of the isotopes. Suggest why no value is quoted for the relative atomic mass of
radium in the table.

...

...

...

...
[1]

(e) Write the expression for the solubility product, Ksp, of SrSO4.

Using data given in the table on page 21, calculate the solubility product of SrSO4
and state its units.

trendyline [3]

Raffles Institution 2016 9647/02/S/16 [Turn Over


1097 trendyline
23

(f) The ionic radius of the sulfate ion is much larger than the ionic radii of Group II For
examiners
metal ions. This results in the magnitude of the lattice energy of Group II sulfates to use
decrease only very slightly down the group.

(i) State and explain the trend of the enthalpy change of hydration of Group II
sulfates down the group.

..

..

..

..

..

..
[1]

(ii) The relationship between lattice energy, the enthalpy change of hydration
and the enthalpy change of solution is given as follows:
'Hsol = 'Hhyd LE
Deduce the variation of the 'Hsol of Group II sulfates down the group and
hence explain the trend of the solubility of Group II sulfates shown in the
table on page 21.

..

..

..

..

..

..

..

..

..

..

trendyline
..

..

[2]

Raffles Institution 2016 9647/02/S/16 [Turn Over


1098 trendyline
24

(g) Barium forms barium peroxide, BaO2, on heating in oxygen. For


examiners
The reaction involves the following energy change: use

O2(g) + 2e o O22(g) 'H1

By using the following data and those given in the table on page 21, construct a
Born-Haber cycle for barium peroxide in the grid below and calculate 'H1.
enthalpy change of formation of barium peroxide 630 kJ mol1
lattice energy of barium peroxide 3443 kJ mol1
enthalpy change of atomisation of barium +176 kJ mol1

Energy

trendyline END OF PAPER


ER
[3]

[Total: 16]

Raffles Institution 2016 9647/02/S/16 [Turn Over


1099 trendyline
1

RAFFLES INSTITUTION
2016 YEAR 6 PRELIMINARY EXAMINATION

Higher 2

CHEMISTRY 9647/03
Paper 3 Free Response 16 September 2016
2 hours
Candidates answer on separate paper.

Additional Materials: Answer Paper


Data Booklet

READ THESE INSTRUCTIONS FIRST

Do not open this question booklet until you are told to do so.

Write your name, class and index number on all the work you hand in.
Write in dark blue or black pen on both sides of the paper.
You may use a soft pencil for any diagrams, graphs or rough working.
Do not use staples, paper clips, highlighters, glue or correction fluid.

Answer any four questions.


Begin each question on a fresh sheet of paper.
A Data Booklet is provided. Do not write anything in it.
You are reminded of the need for good English and clear presentation in your answers.

At the end of the examination, fasten all your work securely together.

The number of marks is given in brackets [ ] at the end of each question or part question.

trendyline This
his document consists of 12 printed
rinted pages.
pages

Raffles Institution 2016 9647/03/S/16 [Turn Over


1100 trendyline
2

Answer any four questions.


Begin each question on a fresh sheet of paper.

1 (a) Gallium trichloride, GaCl3, can be formed by direct combination of the elements.
It has a melting point of 78 qC.

Suggest the type of chemical bonding present in GaCl3 and explain why gallium
has a high tendency to form chlorides of this nature. [2]

(b) Gallium hydride, with empirical formula GaH3, can be prepared from GaCl3.
A 0.55 g sample of gaseous gallium hydride occupies a volume of 68 cm 3 at a
temperature of 243 K and a pressure of 1.12 x 105 Pa.

Determine the Mr of gallium hydride and hence its molecular formula. [3]

(c) GaH3 reacts with (CH3)3N at 178 K to give compound A, which has the following
composition by mass:

H, 11.0%; C, 37.8%; N, 14.7%; Ga, 36.5%.

A molecule of compound A has only one Ga atom.

(i) Determine the molecular formula of A. [1]

(ii) Draw a structure to show clearly


x the shape of A around the gallium atom and
x the types of bonds around the gallium atom. [2]

(d) Lithium aluminium hydride, LiAlH4, and lithium gallium hydride, LiGaH4, are both
donors of hydride, H. Thus, they are capable of reducing a large number of organic
molecules.

Suggest a possible reason why LiAlH4 is a stronger reducing agent than LiGaH4. [1]

(e) LiAlH4 can be used to reduce amides to amines as shown below:

1 LiAlH4 1

in dry ether
2 2
DEET is the active ingredient in many insect repellents. The following reaction
scheme involves the preparation and reaction of DEET.

ttrendyline
tr
r y e step 1

B
step
tep 2

DEET
T
LiA
step 3
AlH4 in dry ether

Raffles Institution 2016 9647/03/S/16 [Turn Over


1101 trendyline
3

(i) Suggest the structures of compound B and DEET. [2]

(ii) Suggest the organic reactant used in step 2. [1]

(f) Venturamide A is a cyclic peptide with anti-malarial activity. It has the following
structure.

Suggest reagents and conditions for the basic hydrolysis of Venturamide A and
draw the structures of the organic products.

Assume that the 2 rings, and , are inert in the reaction. [3]

(g) The identity of the side-chain and pKa values associated with aspartic acid, asp,
and alanine, ala, are as follows :

amino acid aspartic acid, asp alanine, ala


R CH2COOH CH3

amino acid aspartic acid, asp alanine, ala


pKa1 2.10 2.35
pKa2 3.86 9.87
pKa3 9.82

(i) Draw the structures of the major species present in a solution of the asp-ala
dipeptide at each of the following pH values at room temperature.

pH = 1
pH = 3.86
pH = 12 [3]

(ii) Draw the zwitterionic form of asp-ala and write an equation to show how it

trendyline
can remove small amounts of H+. [2]
[Total: 20]

Raffles Institution 2016 9647/03/S/16 [Turn Over


1102 trendyline
4

2 (a) Hydrazine, N2H4, is used as a rocket fuel.


It combusts in oxygen according to the equation:
N2H4(l) + O2(g) o N2(g) + 2H2O(l)
Using the following data, calculate the enthalpy change of combustion of hydrazine.

compound 'Hf / kJ mol1


N2H4(l) +51
H2O(l) 286
[2]

(b) The standard electrode potential for the O2/H2O half-cell was measured.

O2(g) + 4H+(aq) + 4e 2H2O(l) E{ = +1.23 V

Draw a labelled diagram to show how the standard electrode potential of the
O2/H2O half-cell was measured. Include in your diagram the direction of electron
flow. [3]

(c) (i) Hydrazine is increasingly used in fuel cells, and these direct hydrazine fuel
cells are a more practical and viable source of energy as compared to
hydrogen fuel cells.

wire connecting a pair of


graphite electrodes
N2H4(l) air (contains O2 gas)

N2 H2O

anode NaOH
H cathode

Write balanced half-equations, with state symbols, for the reactions taking
place at the anode and the cathode when the fuel cell, using aqueous sodium
hydroxide as electrolyte, is in operation. [2]

(ii) Given that E{cell for the reaction that takes place in the fuel cell is +1.56 V,
use values from the Data Booklet to calculate the standard electrode potential
for the N2/N2H4 half-cell. [1]

(iii) Suggest an advantage of using hydrazine as a fuel over hydrogen gas in a


fuel cell. [1]

trendyline
Raffles Institution 2016 9647/03/S/16 [Turn Over
1103 trendyline
5

(d) 2-chloronitrobenzene undergoes nucleophilic aromatic substitution when heated


with aqueous NaOH at 100 oC.
Cll
C

+ NaOH + NaC
NaCll reaction I

2-chloronitrobenzene

(i) Suggest what is unusual about reaction I. [1]

The mechanism for reaction I involves a negatively charged anion intermediate.


Three resonance structures of the anion intermediate are shown below:
OH OH
.. OH

CCl
l CCl
l CCl
l
.. ..

1 2 3

(ii) Explain why structure 3 is the most stable resonance structure. [1]

Compounds Q and R can undergo nucleophilic aromatic substitution that is similar


to reaction I in (d).
C
Cll F

compound Q compound R

(e) Despite the stronger CF bond in R as compared to the CCl bond in Q,


R is more reactive than Q. Suggest why this is so. [2]

(f) The structure of 2,4-dinitrophenylhydrazine is given below.

trendyline
yll
1 2

Suggest,
st, with reasons, how the basicity of the N atoms labelled N-1 and N-2 on
2,4-dinitrophenylhydrazine
nitrophenylhydrazine
trophenylhydrazine compare with each other. [1]

Raffles Institution 2016 9647/03/S/16 [Turn Over


1104 trendyline
6

(g) Nitronium tetrafluoroborate, NO2+BF4, is a nitronium salt which can be used as a


nitrating agent for aromatic compounds.

C6H6 + NO2+BF4 o C6H5NO2 + HF + BF3

p-anisidine may be prepared using the following route.


(Assume that the OCH3 group is unreactive.)

step 1 step 2
NO2BF4

p-anisidine

(i) Name the type of reaction and describe the mechanism for the reaction in
step 1. Show relevant lone pairs and charges, and use curly arrows to
indicate the movement of electrons. [3]

(ii) Suggest reagents and conditions for step 2. [1]

Fats and oils are sometimes oxidised to form aldehydes and ketones that cause a
rancid smell in food. p-anisidine is widely used for detecting the presence of such
aldehydes and ketones.

The reaction between an aldehyde and p-anisidine is similar to the reaction


between an aldehyde and 2,4-dinitrophenylhydrazine.

(iii) Name the type of reaction which RCHO, an aldehyde found in palm oil,
undergoes with p-anisidine. [1]

(iv) Draw the displayed formula of the product formed by the reaction between
RCHO and p-anisidine. [1]
[Total: 20]

trendyline
Raffles Institution 2016 9647/03/S/16 [Turn Over
1105 trendyline
7

3 (a) Isoprenol is a terpenoid that can be used to prepare other useful compounds.

Br2(aq)

isoprenol compound E

Name and draw a mechanism for the conversion of isoprenol to compound E. Show
relevant lone pairs and charges, and use curly arrows to indicate the movement of
electrons. [4]

(b) Limonene is a compound used in perfumery because of its strong orange scent.

F
(C10H20O4)

limonene
compound G

Compound F can be prepared from either limonene or compound G. One mole of F


reacts with excess PCl5 to produce 4 moles of gas.

(i) Suggest a structure for compound F. [1]

(ii) Suggest reagents and conditions for the preparation of compound F from
x limonene
x compound G [2]

(c) Compound W, C10H16, is a structural isomer of limonene. It is used in perfumery for


its sweet herbal scent.

W exhibits geometric isomerism. When W is reacted with hydrogen in the presence


of platinum catalyst, (CH3)2CH(CH2)3CH(CH3)CH2CH3 is formed.

When W is treated with hot acidified KMnO4(aq), it produces three organic


compounds X, Y and Z , each having the same number of carbon atoms.

Both compounds X and Z react with 2,4-dinitrophenylhydrazine to give an orange


precipitate.

Both compounds Y and Z react with excess magnesium to give an explosive gas.

Suggest
ound
trendyline
st structures for compounds X,
un W
compound W..
X, Y and
d Z,
Z, and draw the geometric
geom isomers of
[5]

Raffles Institution 2016 9647/03/S/16 [Turn Over


1106 trendyline
8

(d) Platinum(IV) chloride reacts with ammonia to form a platinum(IV) salt with the
formula, Pt(NH3)xCly. The cation in the salt is an octahedral complex.

When 0.100 mol of Pt(NH3)xCly is dissolved in excess aqueous AgNO3, 28.7 g of


white precipitate is formed.

Determine the formula of the cation present in Pt(NH3)xCly. [2]

(e) [Pt(NH3)2Cl2] is a square planar complex which displays geometric isomerism.

Cisplatin, the cis-isomer of [Pt(NH3)2Cl2], kills cancer cells by altering the


configuration of their DNA. The trans-isomer, transplatin, does not exhibit such anti-
cancer activity.

(i) Draw the structures for both cisplatin and transplatin. [1]

(ii) When cisplatin enters cancer cells, one of the chloride ligands in the complex
is first replaced by a water molecule.

Write a balanced equation for this reaction. [1]

(f) Carboplatin also exhibits anti-cancer activity, but it does not result in the severe
side-effects which cisplatin causes.

Carboplatin has a square planar structure around Pt and it can be made by


replacing the two chloride ligands in cisplatin with a bidentate
cyclobutanedicarboxylate ion.

cyclobutanedicarboxylate ion

(i) Explain what is meant by a bidentate ligand. [1]

(ii) Draw the structure of carboplatin. [1]

(g) Another method of killing cancer cells involves injecting concentrated ethanol
directly into the tumour.

Explain,

trendyline
n, in terms of bonding,
cancerr cells.
bonding, how ethanol may cause the denaturation
denatura
denaturat of proteins in
[2]
[Total: 20]

Raffles Institution 2016 9647/03/S/16 [Turn Over


1107 trendyline
9

Use of the Data Booklet is relevant to this question.

4 (a) Phosphorus pentachloride, PCl5, is a solid which melts at 162 C.

Describe how a pure solid sample of PCl5 can be made from its elements. [2]

(b) Phosphazenes are a group of P(V)/N(III) compounds with cyclic structures.

PCl5 reacts with NH4Cl to give phosphazene S, N3P3Cl6, and steamy white fumes
are evolved.

Write an equation for this reaction and suggest a structure for phosphazene S. [2]

(c) Some halogens also react readily with methane.

CH4(g) + X2(g) o CH3X(g) + HX(g)

(i) Use bond energy data from the Data Booklet to calculate the 'H{ of this
reaction where X is iodine, I. [2]

(ii) Hence, suggest why it is not possible to make iodomethane, CH3I, by this
reaction. [1]

(d) Halogenoalkanes can undergo homolytic fission in the upper atmosphere.

(i) Explain the term homolytic fission. [1]

(ii) Suggest the most likely organic radical that would be formed by the homolytic
fission of bromochloromethane, CH2BrCl. Explain your answer. [1]

(e) Methylene, :CH2, is a neutral carbon electrophile containing two unpaired


electrons.

Methylene has a few possible structures. In one of them, the centre carbon atom is
sp3 hybridised, with each unpaired electron occupying a hybrid orbital. The HCH
bond angle is 136.

Explain why

(i) methylene can act as an electrophile, [1]

(ii) the HCH bond angle is larger than a regular tetrahedral angle of 109.5. [1]

(f) Two moles of methylene combine to form one mole of ethene.

trendyline
A saturated hydrocarbon
y T,, C4H8, can be formed byy combining four moles of
methylene.
lene
lene.

Draw the
he structure of T.
T. [1]

Raffles Institution 2016 9647/03/S/16 [Turn Over


1108 trendyline
10

(g) Methylene undergoes further homolytic fission to form a CH fragment with three
unpaired electrons. In 1978, a symmetrical and saturated molecule, tetrahedrane
(CH)4, was made by a German chemist, by combining 4 CH fragments.

Draw the structure of tetrahedrane. [1]

(h) Ketenes are very reactive organic compounds. A ketene J can be prepared by
reacting methylene with carbon monoxide. J can also be prepared from ethanoyl
chloride by an elimination reaction in which HCl is lost.

(i) Use the information to deduce the structure of J, and hence, draw its
displayed formula. [1]

(ii) Identify the type of hybridisation of each carbon atom in J. [2]

(i) Diketene K is formed by dimerisation of ketene J.

diketene K

Ethyl acetoacetate is used in perfume to achieve a sweet smell similar to that of


green apples.

1 mole of diketene K reacts with 1 mole of ethanol to form 1 mole of ethyl


acetoacetate as the only product.

Heating 1 mole of ethyl acetoacetate with alkaline aqueous iodine produces 1 mole
of propanedioate ions and 2 moles of yellow precipitate.

(i) Determine the molecular formula of ethyl acetoacetate. [1]

(ii) Explain why one mole of ethyl acetoacetate forms two moles of yellow
precipitate after heating with alkaline iodine solution, and hence suggest the
structure of ethyl acetoacetate. [2]

(iii) Ethylamine reacts with diketene K in the same way as ethanol.

Deduce the structure of the product formed when diketene K reacts with
ethylamine. [1]
[Total: 20]

trendyline
Raffles Institution 2016 9647/03/S/16 [Turn Over
1109 trendyline
11

5 An oxocarbon is a chemical compound consisting of only carbon and oxygen, with the
most common being carbon monoxide (CO) and carbon dioxide (CO2).

(a) Carbon monoxide is one of the most widespread and dangerous industrial hazards.
The formation of carboxyhaemoglobin, Hb(CO)4, takes place much more readily
than the formation of oxyhaemoglobin, Hb(O2)4.

Hb(aq) + 4O2(aq) Hb(O2)4(aq)

Hb(aq) + 4CO(aq) Hb(CO)4(aq)

(i) Draw a dot-and-cross diagram for CO. [1]

(ii) With reference to haemoglobin, describe and explain what is meant by the
quaternary structure of proteins. You should state the type of bonding or
interaction involved. [2]

(iii) A treatment method for carbon monoxide poisoning is to provide the patient
with pure oxygen gas over a period of time.

With reference to the equations given above, explain why this is a suitable
treatment method. [2]

(b) The water-gas shift reaction is used to produce hydrogen gas industrially by
reacting carbon monoxide with water vapour.

CO(g) + H2O(g) CO2(g) + H2(g)

(i) Write an expression for the equilibrium constant, Kp. [1]

An evacuated tank was initially filled with 0.0197 atm of CO and 0.0394 atm of H2O
at 600 K. It was sealed and left to stand. At equilibrium, the partial pressure of CO 2
was found to be 0.0191 atm.

(ii) Calculate the value of Kp at 600 K. [2]

The table below shows how the value of Kp varies with temperature.

T/K 300 900 1500


value of Kp 1.170 x 105 1.978 0.2195

(iii) Suggest whether the forward reaction of the water-gas shift reaction is an
exothermic or an endothermic reaction. Explain your answer. [1]

trendyline
Raffles Institution 2016 9647/03/S/16 [Turn Over
1110 trendyline
12

The Gibbs free energy, 'G{ (in J mol1), and the equilibrium constant, Kp, of a
reaction are related by the following equation:

'G{ = RT ln Kp,

where R is the molar gas constant and T is the temperature in kelvins.

(iv) Using the given values of Kp, calculate the values of 'G{ at 300 K and 900 K.
Hence, determine the value of 'S{ for the water-gas shift reaction.
(Assume that both 'H{ and 'S{ are independent of temperature.) [3]

(v) Comment on the magnitude of the calculated value of 'S{. [1]

(c) Two organic acids which dissociate to form oxocarbon anions are shown below.

Name Deltic acid Squaric acid

Structure

pKa1 2.6 1.5


pKa2 6.0 3.4

(i) Sketch a clearly labelled titration curve to show how pH changes when a
10 cm3 solution of 0.100 mol dm3 deltic acid is titrated against 30 cm3 of
0.100 mol dm3 aqueous sodium hydroxide. [3]

(ii) Suggest a reason why squaric acid has a lower pKa1 value than deltic acid. [1]

(d) An aromatic compound X, C12H6O12, produces carbon dioxide when reacted with
aqueous sodium carbonate.

Heating X with concentrated sulfuric acid for several hours produces a neutral
oxocarbon Y, C12O9.

When Y is heated strongly with ammonia, a compound Z, C12H3O6N3, is formed.

When Z is heated strongly with dilute sulfuric acid, compound X is formed.

trendyline
Suggest
st structures for
f compounds
p X,
X, Y and Z. [3]
[Total: 20]

END OF PAPER
APE

Raffles Institution 2016 9647/03/S/16 [Turn Over


1111 trendyline
Raffles Institution
2016 Year 6 Preliminary Examination
Suggested Answers to H2 Chemistry Paper 1

1 2 3 4 5 6 7 8 9 10
D B B C D D C C B D
11 12 13 14 15 16 17 18 19 20
D B B A A C C D C D
21 22 23 24 25 26 27 28 29 30
B D D A A B C A B A
31 32 33 34 35 36 37 38 39 40
D C B A A B D A B A

trendyline
1112 trendyline
Raffles Institution
2016 Year 6 Preliminary Examination
Suggested Answers to H2 Chemistry Paper 2

Question 1
(a) Cu2+ + 2e o Cu
m 2m
To plate m g ( mol) of copper requires mol of electrons,
63.5 63.5
2m
i.e. to plate m g of copper requires F C of electrical charge.
63.5
2m
Since Q = It, it follows that F It .
63.5
63.5 I
m t (shown)
2F

63.5 u 0.500
(b) 1.00 t t 6079 s
2 u 96500
maximum t is 6079 s (or 6080, correct to 3 sf)

(c) Set-up diagram


+ 
electrical
power
source
pure copper
rod

object to be
0.0500 mol dm3 plated
CuSO4

pure copper rod + object to be plated + electrolyte


electrical power source connected in correct direction
(Object to be plated does not need to be fully submerged for the determination of
Faraday)

Procedure
1. Connect the metal object to be plated to the circuit as shown in the set-up above. (Metal
object should be the cathode, with the negative end of the battery connected to it.)
2. Turn on power source and start stopwatch at the same time. After some time, copper
should be deposited on the metal object.
3. Power source can be turned off after 1200 s.
4. Remove copper-plated object from solution and wash it with distilled water to remove

trendyline
traces of CuSO4.
5. Allow object
ct to dry on a piece of filter paper.
pape
6. Record the e mass of the plated object, and obtain the difference in ma mass by subtracting
2.00 g from
m this value.
7. Repeat stepsps 1 to 6 with durations of 2400 s, 3600 s, 4800 s, 6000 s.
eps
ph of mass of copper deposited against
8. Plot a graph gain time and obtain a line o
of best-fit.
63.5 u 0.500
9. Gradient of line = and Faraday can be calculated from this equation.
2uF
2016 Raffles Institution 1
1113 trendyline
Graph

Mass of
1.2
copper
deposited
1
(m) / g
0.8

0.6

0.4

0.2

0
00 5000
1000 10000
2000 15000
3000 20000
4000 25000
5000 30000 Time/s
6000 35000

Best-fit line passing through origin

Question 2
(a)(i)

(a)(ii) Let y be the % of ascorbic acid in the mixture.


(+20)(y/100) + (100y)(20/100) = 10
y = 25
Hence, the mixture contained 25% of ascorbic acid and 75% of compound A.

(a)(iii) Ascorbic acid contains 4 hydroxyl groups which can form intermolecular hydrogen bonding
with water, hence making it soluble.

(b)(i) atom / group of atoms with unpaired electron(s)

(b)(ii) H2O

(c) Reagents and conditions: aqueous sodium hydroxide, heat strongly


Observations: A gas that turned damp red litmus paper blue was evolved for asparagine

trendyline
endyyline
yl
but not for serine.
ine.
ne
OR Reagents and conditions: K2CrC 2O7(aq), H2SO4(aq),
aq), heat
heat
Observations: For serine, solution
solutio changed from orange to o green.
green. N No
o ccolour change was
observed for asparagine.
sparagine.
OR Reagents and conditions: KMnO4(aq), H2SO4(aq), q), heat
Observations: For
F serine,
i solution
l ti changed
h d ffrom purple
l ((or violet)
i l t) tto colourless.
l No colour
change was observed for asparagine.
2016 Raffles Institution 2
1114 trendyline
(d)(i) structure of compound D:
Cl I
(d)(ii) X is likely to be Cl as the CI bond is weaker / easier to break / has lower bond energy.

(d)(iii) Reagents: alcoholic ammonia or (limited) ammonia dissolved in ethanol


Condition: heat in sealed tube

Question 3
(a)(i) 1s2 2s2 2p6 3s2 3p6 3d9
In oxygenated haemocyanins, the presence of ligands causes the splitting of the five
originally degenerate 3d orbitals in Cu2+ into two sets of slightly different energy levels.
As these orbitals are partially filled, the electrons from the lower energy d-orbitals can
absorb energy corresponding to the visible light spectrum and get promoted to higher
energy d-orbitals (i.e. d-d transition).
The colour observed is the complement of the colour absorbed.

(a)(ii) Deoxygenated haemocyanin contains Cu+, which has a fully filled 3d subshell. Hence, an
electron from a lower energy d-orbital cannot be promoted to a higher energy d-orbital.
As no light in the visible light spectrum is absorbed (or all wavelengths in the visible light
spectrum are transmitted), deoxygenated haemocyanin is colourless.

(b)(i) [O]: RCHO + 3OH o RCOO + 2H2O + 2e


[R]: 2Cu2+(complex) + 2OH + 2e o Cu2O + H2O
Overall: 2Cu2+(complex) + 5OH + RCHO o Cu2O + 3H2O + RCOO

(b)(ii) tartrate ion

(c)(i) 2Al + 3Ag2S o 6Ag + Al2S3

(c)(ii) The redox reaction involves the reduction of Ag2S(s) to Ag and the oxidation of Al to Al2S3(s)
rather than the reduction of Ag+(aq) to Ag and the oxidation of Al to Al3+(aq).

(c)(iii) NaCl acts as the salt bridge in this redox reaction / facilitates electron transfer.

(c)(iv) 2Al + 3Ag2S o 6Ag + Al2S3


Al2S3 + 6H2O o 2Al(OH)3 + 3H2S
Amount of H2S given off = (17.1 / 24000) = 7.125 x 104 mol
3Ag2S Al2S3 3H2S 6Ag
Amount of tarnished silver = 7.125 x 104 x 2 = 1.425 x 103 mol
[(1.425 x 103 x 108)) / 2.00]] x 100% = 7.70%

trendyline
trendyl
n
% of tarnished silver = [(

Question 4
(a)(i) The lone pairr of electrons on O of OHOH delocalises into the benzene ring,
ring causing the ring
rin
to be more electron-rich and hence susceptible to electrophilic su substitution. Hence,
bromination off phenol occurs rapidly.
rapidly

2016 Raffles Institution 3


1115 trendyline
(a)(ii) 2,4,6-tribromophenol formed in reaction II is insoluble and appeared as a white precipitate,
thus causing the reaction mixture to appear a little cloudy.

1
(a)(iii) initial rate v and [H+] v VH+
t

(a)(iv) 1/t / s1
0.120

0.100

0.080

0.060

0.040

0.020

0.000 VH+2/ cm6


0 100 200 300 400 500 600 700

(a)(v) Order of reaction with respect to H+ is 2.


1
As rate v and [H+] v VH+, the straight line obtained, which passes through the origin, shows
t
1
that v (VH+)2 rate v [H+]2 reaction is second order with respect to H+.
t

(b)(i) displayed structure of compound E:

Cl Cl
(b)(ii) Multiple cross-links / covalent bonds between the polymer strands in Bakelite cause the
structure to be more rigid.

trendyline
2016 Raffles Institution 4
1116 trendyline
(c)

10 20 oC

compound F H2SO4(aq),
C8H8O3 heat under reflux

compound G

excess concentrated H2SO4,


170 oC

PCl3, PCl5 or SOCl2

(d)(i)

trendyline
rre
(d)(ii) Solution J is aqueous
queou NaOH (or KOH)
KOH).
Thymol reactss with NaOH(aq) to form an ionic salt,
alt, which is insoluble in the o
soluble in the aqueous layer due to the formation of ion-dipole interactions.
organic layer but is

2016 Raffles Institution 5


1117 trendyline
Question 5
(a) Since Ba has a greater number of filled principal quantum shells, its valence electrons are
located in a shell with a larger principal quantum number.
The valence electrons are further away from the nucleus, resulting in a larger radius.

(b) Small pieces of calcium react steadily with cold water producing bubbles of hydrogen and a
white precipitate of calcium hydroxide.
Ca(s) + 2H2O(l) o Ca(OH)2(s) + H2(g)

Small pieces of barium react vigorously with cold water producing bubbles of hydrogen and
a solution of barium hydroxide.
Ba(s) + 2H2O(l) o Ba(OH)2(aq) + H2(g)

(c) The density of calcium (1.55 g cm3) is greater than the density of water (1.00 g cm3), so
calcium sinks in water.

(d) Radium is radioactive, so some or all of the isotopes are unstable. Hence, the relative
abundance of some or all of the isotopes is difficult to determine.

(e) Ksp of SrSO4 = [Sr2+][SO42]


Solubility of sulfate in water in mol dm3
= (0.0130 / 183.7) x 1000 / 100
= 7.077 x 104 mol dm3
Ksp = (7.077 x 104)2 = 5.01 x 107 mol2 dm6

(f)(i) Cationic radii of Group II ions increase down the group, resulting in decreasing charge
density. Hence, the magnitude of hydration energy decreases significantly down the group.

(f)(ii) The magnitude of LE decreases only very slightly down the group, whereas the magnitude of
hydration energy decreases significantly down the group.

Hence, Hsol is less exothermic down the group,


and solubility of Group II sulfates decreases down the group.

(g)
Energy
Ba2+ (g) + O22 (g)

'H1

Ba2+ (g) + O2 (g) + 2e

1st IE + 2nd IE of Ba
LE of BaO2
Ba(g) + O2 (g)

'Ha [Ba (s)]

ttrendyline
dy i
Ba(s) + O2 (g)
0

'H
Hf [BaO
Ba 2 (s)]
B 2(s)
BaO (

By Hess Law,
'H1 = 1468 176 630 + 3443 = +1169 kJ mol1
2016 Raffles Institution 6
1118 trendyline
Raffles Institution
2016 Year 6 Preliminary Examination
Suggested Answers to H2 Chemistry Paper 3

1 (a) covalent bonding


Ga3+ has a high charge density and so it distorts the electron cloud of Cl to such an extent that
electron sharing becomes predominant.

(b) 0.55
(1.12 x 105)(68 x 106) = x 8.31 x 243
M
M = 146 g mol1
Mr = 146
molecular formula is Ga2H6

(c) (i) H C N Ga
% 11.0 37.8 14.7 36.5
% / Ar 11.0 3.15 1.05 0.524
ratio 21 6 2 1

molecular formula of A is GaN2C6H21

(ii) CH3
CH3
N CH3

H
H Ga
H

CH3 N CH
3
CH3
trigonal bipyramidal shape with N(CH3)3 groups in axial positions;
two NoGa dative bonds and three GaH covalent bonds

(d) AlH bond is more polar than the GaH bond, and hence, it is easier to break the AlH bond to
release the hydride.
OR Al is less electronegative/more electropositive than Ga, and so, there is a larger Gon H of
AlH. Hence, it is easier for the GH to gain an electron pair from AlH bond to form H.

(Note: Electronegativity values for H: 2.20, Al: 1.61, Ga: 1.81)

(e) (i)

O Cll
C O N

(ii)
B:

H N
ttrendyline
r dy DEET :

2016 Raffles Institution 1


1119 trendyline
1 (f) NaOH(aq), heat under reflux/prolonged heating/prolonged boiling

COO-Na+
Na+-OOC NH2
H2N S N
N
O
N S
NH2 COO Na - +

(g) (i) O
H3N
N COOH
H
COOH
pH = 1
O O
H 3N H3N
N COO- N COO-
H H
COOH COO-
pH = 3.86
O
H2N
N COO-
H
COO-
pH =12

(ii) O
H3N
N COO-
H
COOH
O O
H 3N H 3N
N COO- + N COOH
+H
H H
COOH COOH

trendyline
2016 Raffles Institution 2
1120 trendyline
2 (a) 'Hc = 2(286) (+51)
= 623 kJ mol1

(b)
e e T = 298 K
V

H2(g) salt bridge O2(g)


(1 atm) (1 atm)

Pt Pt

H+(aq) H+(aq)

[H+(aq)] = 1 mol dm3 [H+(aq)] = 1 mol dm3

(c) (i) Cathode: O2(g) + 2H2O(l) + 4e o 4OH(aq)


Anode: N2H4(l) + 4OH(aq) o N2(g) + 4H2O(l) + 4e

(ii) E{cell = +0.40 (y) = +1.56 V


y = 1.16 V
E{(N2/N2H4) = 1.16 V

(iii) As the fuel is liquid at room temperature, it can be handled and stored more easily than
hydrogen. OR Hydrazine has an e.m.f. of 1.56 V, which is higher than the 1.23 V for a
hydrogen fuel cell. OR Hydrogen yields explosive mixtures with air or oxygen, so it must be
handled with great care.

(d) (i) Due to partial double bond character of the C-Cl bond, it is very strong and nucleophilic
substitution does not usually take place. OR The benzene ring is electron-rich and does
not usually attract nucleophiles.
!
(ii) The electron-withdrawing NO2 group is bonded directly (OR closest to) to the carbon
with the negative charge, and thus, is able to disperse the negative charge most
effectively.
!
(e) F is more electronegative than Cl
and disperses the negative charge on the anion intermediate to a larger extent

(f)

1
2

tre
ren
N-1 is less basic than N-2. The lone pair of electrons on N-1 is delocalised into the
benzene ring, and is hence less available for donation to a proton.
2016 Raffles Institution 3
1121 trendyline
2 (g) (i) Electrophilic substitution

slow

BF4 + fast
BF3 HF

(ii) Sn, concentrated HCl, followed by NaOH(aq)

(iii) condensation OR nucleophilic addition elimination

(iv)

OR

3 (a) Electrophilic addition

slow

G
G+

..
fast
..

..
..

ttrendyline
tr
re
re
ennd
dyli
yliiin
ne fast
ast
H3O

2016 Raffles Institution 4


1122 trendyline
3 (b) (i)
ROH + PCl5 o RCl + POCl3 + HCl

1 mole of F has 4 moles of OH


groups, so 4 moles of HCl(g) will
be produced.

Compound F is

(ii) limonene to F: cold KMnO4(aq), NaOH(aq)


G to F: LiAlH4, dry ether

(c) O
HO OH
OH

O
O O
O
X Y Z

W
(E isomer) (Z isomer)

(d) Amount of AgCl precipitate formed = 28.7 / (108 + 35.5) = 0.200 mol
Amount of Cl in 0.100 mol of complex = 0.200 mol
Mole ratio of complex to chloride is 0.100 : 0.200 = 1 : 2
Since there are 2 Cl per cation, and the charge on the cation is 4+, the charge on the complex
cation is 2+.
As the coordination number is 6, there must be 6 2 = 4 NH3 per cation
The cation is [Pt(NH3)4Cl2]2+.

(e) (i)
Cl Cl

Cl Cl

cisplatin transplatin

(ii) [Pt(NH3)2Cl2] + H2O o [Pt(NH3)2Cl(H2O)]+ + Cl

(f) (i) A bidentate ligand is one which can form 2 co-ordinate bonds simultaneously with the

trendyline
central me
metal ion (or atom) / donatess 2 lone pairs of
o electrons to tthe central metal ion (or
atom).

2016 Raffles Institution 5


1123 trendyline
3 (f) (ii)

(g) Ethanol can disrupt the tertiary and quaternary structures of proteins as its OH group can
form hydrogen bonds with hydrophilic R groups such as C=O, CO2H, OH, NH2, NHR
groups.

The ethyl group of ethanol can disrupt tertiary and quaternary structures by forming van der
Waals forces of attraction with the non-polar hydrophobic R groups such as alkyl groups.
Hence, the protein rearranges itself and is denatured.

4 (a) P reacts with excess chlorine gas. Collect solid and recrystallise.

(b) 3PCl5 + 3NH4Cl o N3P3Cl6 +12HCl


Cl Cl

Cl Cl

Cl Cl

(c) (i) energy change for bonds broken = BE[CH] + BE[II] = 410 + 151 = 561 kJ mol1
energy change for bonds formed = {BE[CI] + BE[HI]} = (240 + 299) = 539 kJ mol1
'H = 561 539 = +22 kJ mol1
(ii) The overall reaction is endothermic. 'S | 0 as number of (moles of) gaseous molecules
before and after the reaction is the same. 'G = 'H T'S > 0 such that the reaction is not
energetically feasible OR no strong bonds / only weak bonds are formed OR high Ea

(d) (i) breaking of a bond to form (two) radicals / species with unpaired electrons
(ii) CH2Cl AND the C-Br bond is the weakest / needs the least energy to break

(e) (i) electron-deficient / able to accept (lone pair of) electrons


(ii) Electron-electron repulsion increases in the order:
unpaired-unpaired < unpaired bond pair < bond pair bond pair

(f)
cyclobutane

(g)

trr
2016 Raffles Institution 6
1124 trendyline
4 (h) (i)

(ii) sp (for carbonyl C) and sp2 (for C bonded to 2 H)

(i) (i) C6H10O3


(ii)
due to CH3COR and CH3CH2OH (after hydrolysis)
(iii) CH3COCH2CONHCH2CH3 (amide formation)

5 (a) (i)

x
C O x
xx
x
x
(ii) The quaternary structure of haemoglobin consists of four polypeptide chains, where two
D-subunits and two E-subunits are held together through R group interactions, namely
ionic attractions, van der Waals interactions, hydrogen bonds and disulfide linkages.

Each subunit is covalently bonded to a haem residue, which contains a central Fe2+
ion that forms dative bonds with 4 N atoms in a porphyrin ring and an N atom of a histidine.

(iii) Providing O2 forms Hb(O2)4(aq), which shifts the position of equilibrium of


Hb(aq) + 4O2(aq) Hb(O2)4(aq) to the right, removing Hb(aq) from the bloodstream.

By Le Chateliers Principle, the position of equilibrium of Hb(aq) + 4CO(aq)


Hb(CO)4(aq) will shift left to counteract the loss of Hb(aq), thus reducing the amount of
Hb(CO)4(aq) in the body.

(b) (i) pCO2 pH2


Kp
pCO pH2O

(ii)
CO (g) + H2O (g) CO2 (g) + H2 (g)

Initial pressure
0.0197 0.0394 0 0
/ atm
Change in
pressure / atm
0.0191 0.0191 +0.0191 +0.0191

Eqm pressure
0.0006 0.0203 0.0191 0.0191
/ atm

(0.0191)(0.0191)

trendyline
y e
Kp 30.0
.0006)(0.0203
.0006)(0.0203)
(0.0006)(0.0203)

(iii) The valuelue


ue of Kp decreases as temperature increase
increases, so increa
increasing the temperature
favours s the endothermic reverse reaction
reaction.
actio . Thus, the forward re
reaction of the water-gas
shift reaction
ti is
i an exothermic
th i reaction.
ti

2016 Raffles Institution 7


1125 trendyline
5 (b) (iv) 'G{ (300) RT ln Kp (300)
(8.31)(300) ln (117000)
29090 J mol1 (4 sf)

'G{ (900) RT ln Kp (900)


(8.31)(900) ln (1.978)
5101 J mol1 (4 sf)

'G{ (300) 'H{ (300)'S{ 29090 J mol1 ----- (1)


'G{ (900) 'H{ (900)'S{ 5101 J mol1 ----- (2)

(1) (2): 'S{ 40.0 J K1 mol1 (3 sf)

(v) The magnitude is small as there is no change in the number of (moles of) gaseous
molecules when reactants changed to products. Hence, there is little change in the number
of ways that energy can be distributed over the molecules.

(c) (i)

6.0

2.6

Sketch should include the following:


x labelled axes, with volume of NaOH up to 30.0 cm3, and correct shape of graph
x the 2 pKa values with the 2 corresponding volumes of NaOH
x volumes of NaOH required to reach the 2 equivalence points

(c) (ii) There are more electron withdrawing C=O groups in squaric acid than in deltic acid. As
such, the conjugate acid anion of squaric acid is more stable than that of deltic acid.
Hence, squaric acid is more acidic and has a lower pKa1 value than deltic acid.

(d)

trrendyline
ttrendyline
Xtrend
dyl e Y Z

2016 Raffles Institution 8


1126 trendyline
RIVER VALLEY HIGH SCHOOL
YEAR 6 PRELIMINARYEXAMINATIONII
CANDIDATE
NAME

CLASS 6

CENTRE INDEX
NUMBER S NUMBER

H2 CHEMISTRY 9647/01
Paper 1 Multiple Choice 23 September 2016
1 hour

Additional Materials: Multiple Choice Answer Sheet


Data Booklet

READ THESE INSTRUCTIONS FIRST

Do not use staples, paper clips, highlighters, glue or correction fluid.


Write your name, class, centre number and index number on the Answer Sheet in
the spaces provided.
There are forty questions on this paper. Answer all questions. For each question
there are four possible answers A, B, C and D.
Choose the one you consider correct and record your choice in soft pencil on the
separate Answer Sheet.

Read the instructions on the Answer Sheet very carefully.

Each correct answer will score one mark. A mark will not be deducted for a wrong
answer.

trendyline
Any rough working
king should be done in this booklet.
b

This document consists of 17 printed


ted pages and 3 blank page
pages.

1127 trendyline
2

Section A
For each question there are four possible answers, A, B, C and D. Choose the one
you consider to be correct.

1 For complete combustion of 20.0 g of heptane, what is the minimum volume of


oxygen gas required at s.t.p.?

A 0.2 dm3

B 2.2 dm3

C 49.3 dm3

D 52.8 dm3

2 Element X exists as 3 different isotopes and has a relative atomic mass of 66.6.
Which of the following compositions of isotopes is correct?

A 25% 65X, 50% 66X and 25% 67X

B 30% 65X, 60% 66X and 10% 67X

C 50% 65X, 20% 66X and 30% 67X

D 10% 65X, 20% 66X and 70% 67X

3 Which of the following does not contain a singly occupied orbital?

A N2+ B S C Br+ D Sc3+

4 Dinitrogen tetroxide, N 2 O 4 , has a simple covalent structure.


Which of the following correctly describes the bonding within the dinitrogen tetroxide
molecule?

bond bond dative bond

A 3 2 2

t dyli
trendyline
B 5 0 4

C 5 2 2

D 5 4 0

River Valley High School 9647/01/PRELIM II/16 [Turn over


2016 Preliminary Examination II
1128 trendyline
3

5 The Valence Shell Electron Pair Repulsion (VSEPR) theory is used to predict the
shapes of molecules.
Which shape is correctly predicted by VSEPR?

number of bonded
number of lone pairs
electron pairs around shape
around central atom
central atom

A 2 4 non-linear

B 3 2 T-shaped

C 4 1 square planar

D 3 2 see-saw

6 Equal masses of gaseous N 2 , NH 3 and N 2 O are injected into an evacuated


container to produce a total pressure of 3 atm.
How do the partial pressures of N 2 , NH 3 and N 2 O compare?

A p N2 = p NH3 = p N2O B p N2 <p NH3 <p N2O

C p NH3 <p N2 <p N2O D p N2O <p N2 <p NH3

7 Use of the Data Booklet is relevant to this question.


The enthalpy changes for three reactions are given below:

Na(s) Na(g) H = +107 kJ mol1

H+(aq) + OH(aq) H 2 O(l) H = 58 kJ mol1

2Na+(g) + 2H+(aq) + 2e 2Na+(aq) + H 2 (g) H = 1700 kJ mol1

What is the enthalpy change for the following reaction?


Na(s) + H 2 O(l) Na+(aq) + OH(aq) + H 2 (g)

A 191 kJ mol1

B 307 kJ mol1

trendyline
C 685 k mol1
5 kJ

D 1 kJ mol11
1041
41

River Valley High School 9647/01/PRELIM II/16 [Turn over


2016 Preliminary Examination II
1129 trendyline
4

8 The dissolution of ammonium nitrate is a chemical reaction commonly used in cold


packs that serve to relief pain in sports injury. A thin plastic membrane separates
powdered ammonium nitrate and water. When the cold pack is squeezed, the
membrane breaks and the ammonium nitrate dissolves in water.

NH 4 NO 3 (s) NH 4 NO 3 (aq)

What are the signs of H, S, G for the overall process?

H S G

A +

B +

C + +

D + + +

9 Hydrogen reacts with iodine according to the equation:

H 2 (g) + I 2 (g) 2HI(g)

P atm of hydrogen is allowed to react with Q atm of iodine at constant temperature.


At equilibrium, it is found that

A the partial pressure of hydrogen is greater than P.

B the total pressure of the system is (P + Q) atm.

C the total pressure of the system is greater than (P + Q) atm.

D the total pressure of the system is less than (P + Q) atm.

10 When 10 cm3 of 0.02 mol dm3 aqueous Ba(OH) 2 is added to separate solutions of
10 cm3 of 0.1 mol dm3 MgCl 2 and 10 cm3 of 0.1 mol dm3 CaCl 2 , what would be
observed?
(K sp of Mg(OH) 2 = 3.2 1011 mol3 dm9; K sp of Ca(OH) 2 = 1.35 105 mol3 dm9)

A Only Ca(OH) 2 would be precipitated.

trendyline
B Only Mg(OH)
M 2 would be precipitated.
ta

C Both Mg(OH) 2 and Ca(OH


Ca(OH) 2 would
ould be precipitate
precipitated.

D Neither
her Mg(OH)
M (OH) 2 nor Ca(OH)
C (OH) 2 would be precipitated
precipitated.

River Valley High School 9647/01/PRELIM II/16 [Turn over


2016 Preliminary Examination II
1130 trendyline
5

11 The auto-ionisation of water is represented by the following equation:

H 2 O(l) H+(aq) + OH(aq) H = +57 kJ mol1

Given that the value of the equilibrium constant, K w , is 1.0 1014 mol2 dm6 at
298 K, which of the following statements is true?

A Water is not a neutral liquid at a temperature lower than 298 K.

B When water is heated, the concentration of OH increases.

C The pH of water at temperatures higher than 298 K is greater than 7.

D The association of water molecules by hydrogen bonding increases as


temperature rises.

12 Consider the following reaction:


3X + Y X 3 Y
The mechanism involves the following steps:

2X X 2 (fast)

X2 + Y X2Y (slow)

X2Y + X X3Y (fast)

Based on the information, what is the rate equation for this reaction?

A Rate = k[X]2 B Rate = k[X 2 ][Y]

C Rate = k[X]2[Y] D Rate = [X]3[Y]

13 The decomposition of compound A is a first order reaction which proceeds


according to the equation:

A(s) 2B(s) + C(g)

A sample of A gave 80 cm3 of C on complete decomposition and it took


40 min for 40 cm3 of C to be evolved.
For another identical sample of A, how long would it take for 70 cm3 of C to be
evolved?

C
40 min

80 min
n
trendyline B

D
70 min

120 min

River Valley High School 9647/01/PRELIM II/16 [Turn over


2016 Preliminary Examination II
1131 trendyline
6

14 The Daniell Cell comprises two half-cells a copper electrode dipped in copper(II)
sulfate solution and a zinc electrode dipped in zinc sulfate solution linked by a salt
bridge.
Which of the following will increase the cell e.m.f. of the Daniell Cell?

A Adding solid copper(II) sulfate to the copper half-cell.

B Increasing the mass of copper electrode.

C Increasing the concentration of the zinc sulfate solution.

D Allow the system to reach equilibrium before measuring cell e.m.f.

15 Two separate electrolyses were performed as follows, under the same conditions of
temperature and pressure.

1. When aqueous hydrochloric acid was electrolysed for five minutes, 100 cm3
of hydrogen were collected from the cathode.
2. When aqueous sulfuric acid was electrolysed for five minutes, 200 cm3 of
oxygen were collected from the anode.
If the current used in electrolysis 1 was I, what was the current used in electrolysis
2?

A 4I B 2I C I D 0.5I

16 Which of the following is a trend across Period 3 of the Periodic Table?

A The radii of ions decrease.

B The first ionisation energy decreases.

C The melting points of elements decrease.

D The compounds of elements become increasingly covalent.

trendyline
River Valley High School 9647/01/PRELIM II/16 [Turn over
2016 Preliminary Examination II
1132 trendyline
7

17 Which of the following equations represents the reaction when calcium nitrate is
heated strongly?

A Ca(NO 3 ) 2 CaO + N 2 O 5

B Ca(NO 3 ) 2 Ca(NO 2 ) 2 + O 2

C 2Ca(NO 3 ) 2 2CaO + 4NO 2 + O 2

D 3Ca(NO 3 ) 2 Ca 3 N 2 + 4NO 2 + 5O 2

18 Use of the Data Booklet is relevant to this question.


Which of the following solutions of halide will not change the colour of acidified
potassium dichromate(VI) ?

A chloride B bromide C iodide D astatide

19 Which statement correctly defines a transition element?

A Transition elements exhibit more than one oxidation state in their compounds.

B Transition elements or their compounds are widely used as catalysts.

C Transition elements form many coloured compounds.

D Transition elementshave partially filled d orbitals.

20 Which of the following compounds cannot be obtained from propene in a single


reaction?

A CO 2 B CH 3 CH 2 CH 3

C CH 3 CH 2 CH 2 CN D CH 2 (OH)CH(OH)CH 3

trendyline
River Valley High School 9647/01/PRELIM II/16 [Turn over
2016 Preliminary Examination II
1133 trendyline
8

21 How many alkenes (including stereoisomers) can be formed when the following
alcohol is heated with concentrated sulfuric acid?

OH

A 0 B 2 C 3 D 4

22 Saccharin, an artificial sweetening agent, can be synthesised from methylbenzene


using the following reaction scheme:
CH3 CH3 O O CH3 O O

S S
ClSO3H Cl NH3 NH2

Step 1 Step 2

Step 3

O
NH
O
S

Saccharin
What type of reactions do steps 1 and 2 illustrate?

Step 1 Step 2

A Electrophilic substitution Condensation

ttrendyline
trendy
dyline
li
B Electrophilic
ect op c subst
substitution
tut o Nucleophilic
uc eop Addition

C Nucleophilic substitution Conde


Condensation

D Nucleophilic substitution Nucleophi


Nucleophilic
ucleoph Addition

River Valley High School 9647/01/PRELIM II/16 [Turn over


2016 Preliminary Examination II
1134 trendyline
9

23 Fluothane, CF 3 CHBrCl, is a volatile liquid widely used as an anaesthetic.


Which of the following statements about fluothane is not correct?

A Fluothane may cause the depletion of the ozone layer.

B Weak van der Waals forces hold molecules of fluothane together.

C Fluothane undergoes elimination when treated with hot ethanolic NaOH.

D When a sample of fluothane is heated with aqueous silver nitrate, a mixture of


white and cream precipitate is formed.

24 Eugenol is a common component of perfumes and essential oils. It is also used as


an antiseptic.
H3C O

HO CH2CH=CH2

eugenol
(You may treat the OCH 3 group attached to the benzene ring as inert.)

Which of the following statements about eugenol is correct?

A It reacts with 2,4-DNPH to give an orange precipitate.

B It gives white fumes when reacted with thionyl chloride.

C It decolourises hot acidified potassium manganate(VII).

D It reacts with 2 moles of Br 2 in a suitable solvent to form a product with 4


bromine atoms incorporated.

trendyline
River Valley High School 9647/01/PRELIM II/16 [Turn over
2016 Preliminary Examination II
1135 trendyline
10

25 Cyclohex-3-en-1-one has the following structure:

cyclohex-3-en-1-one
Which of the following compounds is formed when cyclohex-3-en-1-one is reacted
with NaBH 4 dissolved in an organic solvent?

A B
OH OH

C D
O O

26 Compound J gives the following observations with Tollens Reagent and


Na 2 CO 3 (aq).

Reagent Observation

Tollens Reagent Silver mirror observed

Na 2 CO 3 (aq) No effervescence observed

From the above observations, what could be a possible structure for J?

A (COOH) 2

B CH 3 COCHO

C CH 3 COCOOH

D CH 3 COCOCH 3

trendyline
River Valley High School 9647/01/PRELIM II/16 [Turn over
2016 Preliminary Examination II
1136 trendyline
11

27 In the study of organic reaction mechanisms, radioactive oxygen-18, 18O, is often


used to tag organic molecules. The radioactivity of the products can then be
detected to deduce which products contain 18O.
When 18O-tagged propan-1-ol is heated with ethanoic acid in the presence of a
small amount of concentrated sulfuric acid, which of the following set of products will
be obtained?

A CH 3 CH 2 CH 2 18OCOCH 3 and H 2 O

B CH 3 CH 2 CH 2 OCOCH 3 and H 2 18O

C CH 3 CH 2 CH 2 18OCOCH 3 and H 2 18O

D CH 3 CH 2 CH 2 OCOCH 3 and H 2 O

28 Which of the following correctly shows the compounds arranged in decreasing pK a


values?

A CH 2 FCOOH > CH 2 ClCOOH > CH 2 BrCOOH

B CH 3 COOH > C 6 H 5 COOH > C 6 H 5 OH

C CCl 3 COOH > CHCl 2 COOH > CH 2 ClCOOH

D C 2 H 5 OH > C 6 H 5 OH > C 6 H 5 COOH

trendyline
River Valley High School 9647/01/PRELIM II/16 [Turn over
2016 Preliminary Examination II
1137 trendyline
12

29 Valinomycin is a peptide-like molecule that disrupts plasma membrane formation in


bacteria.

valinomycin
Which of the following will not be produced when valinomycin is heated with dilute
H 2 SO 4 (aq) for a period of time?

A O B O

+
OH NH3
HO HO

C O D O

+
NH3 OH
HO HO

trendyline
River Valley High School 9647/01/PRELIM II/16 [Turn over
2016 Preliminary Examination II
1138 trendyline
13

30 Denaturation of proteins occurs due to the destruction of the tertiary structure where
various R group interactions are disrupted by external factors.
Which of the following correctly shows the R group interactions that will be affected
by the corresponding external factor?

R group interactions External factor

A disulfide bridges extreme pH changes

B ionic interactions addition of alcohol

C van der Waals forces addition of heavy metal ions

D hydrogen bonding extreme pH changes

trendyline
River Valley High School 9647/01/PRELIM II/16 [Turn over
2016 Preliminary Examination II
1139 trendyline
14

Section B
For each of the questions in this section, one or more of the three numbered
statements 1 to 3 may be correct.
Decide whether each of the statements is or is not correct (you may find it helpful to
put a tick against the statements that you consider to be correct).
The responses A to D should be selected on the basis of

A B C D

1, 2 and 3 are 1 and 2 only are 2 and 3 only are 1 only is


correct correct correct correct

No other combination of statements is used as a correct response.

31 Which statements about aluminium are correct?

1 It forms amphoteric oxides.

2 It forms an ionic compound with oxygen.

3 It forms an ionic compound with chlorine.

32 Between ethanol molecules, there exists

1 hydrogen bonds.

2 instantaneous dipole-induced dipole interactions.

3 covalent bonds.

33 For a first order reaction, which of the following graphs will be a straight line passing
through the origin?

1 Initial rate vs concentration of reactant

2 Initial rate vs time taken for complete reaction

3 Concentration of reactant vs time taken for complete reaction

River Valley High School 9647/01/PRELIM II/16 [Turn over


2016 Preliminary Examination II
1140 trendyline
15

A B C D

1, 2 and 3 are 1 and 2 only are 2 and 3 only are 1 only is


correct correct correct correct

34 Which statements correctly describe an electrolytic cell?

1 It is driven by an external battery.

2 The polarities of the cathode and anode are negative and positive respectively.

3 It is used to extract unreactive metals from the ores.

35 Which of the following statementsare true for astatine, the element below iodine in
Group VII of the Periodic Table?

1 Silver astatide is insoluble in concentrated aqueous ammonia.

2 Astatine oxidises potassium chloride to chlorine.

3 Hydrogen astatide is more stable to heat than hydrogen iodide.

36 When placed in a strong magnetic field, paramagnetic objects are attracted to the
field. Paramagnetism arises when an element has unpaired electrons.
Which of the following chemical species would you expect to exhibit
paramagnetism?

1 Cr

2 Mn

3 Cu+

trendyline
River Valley High School 9647/01/PRELIM II/16 [Turn over
2016 Preliminary Examination II
1141 trendyline
16

A B C D

1, 2 and 3 are 1 and 2 only are 2 and 3 only are 1 only is


correct correct correct correct

37 Deuterium, D, is the 2H isotope of hydrogen.

Which of the following reactions could give an organic compound having a


deuterium atom incorporated?
O
DCN
1
NaCN in D2O
H

NaOD in D2O
2 CN heat

DBr
3 OH
heat

38 In which of the following reactions will the inorganic reagent act as the nucleophile?
Br
HBr
1

NaCN
2
Br CN

O O
NH3
3
Cl NH2

trendyline
River Valley High School 9647/01/PRELIM II/16 [Turn over
2016 Preliminary Examination II
1142 trendyline
17

A B C D

1, 2 and 3 are 1 and 2 only are 2 and 3 only are 1 only is


correct correct correct correct

39 Compound X is a flavouring agent in food.


CHO

HO O

O
X
Which of the following reagents will cause a colour change when the reagent is
added to a sample of compound X?

1 aqueous bromine

2 neutral iron(III) chloride solution

3 hot acidified potassium dichromate(VI)

40 Which of the following descriptions about haemoglobin are correct?

1 Both O 2 and CO can bind to haemoglobin.

2 The quaternary structure of haemoglobin consists of two -helices and two


-pleated sheets.

3 A maximum of six oxygen molecules may be bound to each haemoglobin


molecule.

trendyline
- End off Paper -

River Valley High School 9647/01/PRELIM II/16 [Turn over


2016 Preliminary Examination II
1143 trendyline
18

BLANK PAGE

trendyline
River Valley High School 9647/01/PRELIM II/16 [Turn over
2016 Preliminary Examination II
1144 trendyline
19

BLANK PAGE

trendyline
River Valley High School 9647/01/PRELIM II/16 [Turn over
2016 Preliminary Examination II
1145 trendyline
20

BLANK PAGE

trendyline
River Valley High School 9647/01/PRELIM II/16 [Turn over
2016 Preliminary Examination II
1146 trendyline
1

RIVER VALLEY HIGH SCHOOL


YEAR 6 PRELIMINARYEXAMINATIONII
CANDIDATE
ANSWER KEY
NAME

CLASS 6

CENTRE INDEX
NUMBER S 3 0 4 4 NUMBER
0 0

H2 CHEMISTRY 9647/01
Paper 1 Multiple Choice 23 September 2016
1 hour

1. C 11. B 21. D 31. B

2. D 12. C 22. A 32. B

3. D 13. D 23. C 33. D

4. C 14. A 24. C 34. B

5. B 15. A 25. B 35. D

6. D 16. D 26. B 36. B

7. A 17. C 27. A 37. D

8. C 18. A 28. D 38. C

9. B 19. D 29. C 39. A

10. C 20. C 30. D 40. D

trendyline 9647/01
River Valley High School
2016 Preliminary Examinations II(Answer Key)

1147 trendyline
RIVER VALLEY HIGH SCHOOL
YEAR 6 PRELIMINARYEXAMINATIONII

CANDIDATE
NAME

CLASS 6

CENTRE INDEX
NUMBER S NUMBER

H2 CHEMISTRY 9647/02
Paper 2Structured Questions 13 September 2016
2 hours
Candidates answer on the Question Paper.
Additional Materials: Data Booklet

READ THESE INSTRUCTIONS FIRST

Write your name, class, centre number and index number on all the work you hand in.
Write in dark blue or black pen on both sides of paper.
You may use a soft pencil for any diagrams, graphs or rough working.
Do not use staples, paper clips, highlighters, glue or correction fluid.

Answer all questions in the space provided.


A Data Booklet is provided. Do NOT write anything on it.

The number of marks is given in brackets [] at the end of each question or part question.

For Examiners Use


Paper 2

Question Total
1 2 3 4 5 6
Number (Paper 2)

Marks
12 13 8 9 16 14 72

trre
trendyline
reen
ndylin
in
ne
Paper 1 Paper 3
Pa Total
40 80 192

This
his document consists of 18
18 printed pages and 2 blank pages.
page
pag

River Valley High School 9647/02/PRELIM II/16 [Turn over


2016 Preliminary ExaminationII

1148 trendyline
2

1 Planning (P)

Salicylic acid is known for its ability to ease aches and pains as well as reduce
fevers. Its analgesic and anti-inflammatory properties makes it one of the most
important medications needed in a basic health system, placing it on the World
Health Organisation Model List of Essential Medicines.
It can be synthesised via the alkaline hydrolysis of methyl salicylate, followed
by acidification.

COOCH3 COOH

OH OH

methyl salicylate salicylic acid


(M r = 152) (M r = 138)

Methyl salicylate is a liquid with a density of 1.174 g cm3.


Aqueous sodium hydroxide used for the hydrolysis has a concentration of
6.0 mol dm3.

Like most organic reactions, the yield of this reaction is less than 100%. Using
the procedure described below, a typical yield of pure salicylic acid is 60%.

Methyl salicylate is mixed with aqueous sodium hydroxide in a molar ratio of


1:2. The reaction is heated under reflux conditions for around 30 minutes to
form a di-sodium salicylate salt.
When the reaction mixture has cooled down, concentrated hydrochloric acid is
added until in excess to cause the crude salicylic acid to precipitate. This
process is highly exothermic.
The crude salicylic acid is then purified by recrystallisation from water.

(a) Using the information given above:

(i) write a balanced equation for the formation of the di-sodium


salicylate salt;

[1]

River Valley High School 9647/02/PRELIM II/16


2016 Preliminary ExaminationII

1149 trendyline
3
(ii) calculate the volumes of methyl salicylate and aqueous sodium
hydroxide you would use to prepare 10 g of pure salicylic acid,
showing your working.

[2]

(b) Write a plan for the preparation of 10 g of pure salicylic acid.


In your plan, you should:
draw a diagram of the assembled apparatus you would use when
heating the reaction mixture;
give a full description of the procedures, including the capacities
of the apparatus involved, you would use to prepare and purify
the salicylic acid;
explain how you would minimise a potential safety hazard.

River Valley High School 9647/02/PRELIM II/16 [Turn over


2016 Preliminary Examination II

1150 trendyline
4

trendyline

River Valley High School 9647/02/PRELIM II/16


2016 Preliminary ExaminationII

1151 trendyline
5



[9]

[Total: 12]

River Valley High School 9647/02/PRELIM II/16 [Turn over


2016 Preliminary Examination II

1152 trendyline
6

2 Sodium azide is a chemical found in car safety airbags. When the car
undergoes a head-on collision, a series of chemical reactions occur in the gas
generator chamber. The first reaction produces nitrogen gas to fill the airbag.
The equation for the reaction is shown below:
3
NaN 3 (s) Na(s) + N 2 (g) H< 0
2

(a) The volume of a fully inflated airbag is 60 dm3.


Calculate the mass of NaN 3 needed to fill such an airbag to an internal
pressure of 150 kPa at 300 C.

[2]

(b) (i) Given that S r = S(products) S(reactants) and the following


information, calculate S for the above reaction.

Molar entropy of NaN 3 (s) 70.5 J mol1 K1

Molar entropy of Na(s) 51.3 J mol1 K1

Molar entropy of N 2 (g) 191.6 J mol1 K1

[1]

(ii) Explain why the reaction is feasible at all temperatures.

trendyline


[1]

River Valley High School 9647/02/PRELIM II/16


2016 Preliminary ExaminationII

1153 trendyline
7
(iii) Suggest why the reaction in fact occurs only at temperatures
above 300 C.


[1]

(c) Sodium metal formed in the above reaction is very reactive and is made
to react with potassium nitrate present in the airbag, forming sodium
oxide as one of the products.

(i) Explain what is meant by the lattice energy of sodium oxide.


[1]

(ii) Using the information given below and relevant information from
the Data Booklet, construct an energy level diagram to calculate
the lattice energy of sodium oxide.
Enthalpy change of formation of sodium oxide = 416 kJ mol1
Enthalpy change of atomisation of sodium = +107 kJ mol1
Sum of 1st and 2nd electron affinity of oxygen = +702 kJ mol1

trendyline [3]

River Valley High School 9647/02/PRELIM II/16 [Turn over


2016 Preliminary Examination II

1154 trendyline
8

(iii) Given that the 1st electron affinity of oxygen is 142 kJ mol1,
calculate the 2nd electron affinity of oxygen and explain the
significance of its sign.


[2]

(d) A working airbag should become fully inflated within 0.04 s of the initial
collision. At this point, it is actually unsafe for the drivers body to hit the
inflated airbag due to the high internal pressure (150 kPa). The airbag is
hence designed to deflate immediately such that by the time the drivers
body hit the airbag, usually 2 s after the initial collision, the internal
pressure of the airbag would have dropped to 101 kPa, providing a
softer cushioning for the driver.
An airbag at a safety test has a depressurisation rate of 23 kPa s1.
Explain if this airbag is safe for use, showing relevant calculations.

...
[2]

[Total: 13]

trendyline
River Valley High School 9647/02/PRELIM II/16
2016 Preliminary ExaminationII

1155 trendyline
9

BLANK PAGE

trendyline
River Valley High School 9647/02/PRELIM II/16 [Turn over
2016 Preliminary Examination II

1156 trendyline
10

3 (a) What do you understand by the term standard electrode potential?


[2]

(b) The following cell was set up between a copper electrode and an
unknown metal electrode M2+(aq)/M(s). The standard cell potential was
found to be 0.76V, and the copper foil was connected to the positive
end of the voltmeter.

Direction of electron flow

salt bridge

1 mol dm3 M2+ 1 mol dm3


CuSO4
Unknown metal Copper foil
wire, M

(i) Use the Data Booklet to calculate the standard electrode potential
of the M2+(aq)/M(s) system.


[1]

trendyline
(ii)) Draw an arrow in the box
b above
ve to show the direction
direc of electron
voltmeter.
flow through the voltmeter [1]

River Valley High School 9647/02/PRELIM II/16


2016 Preliminary ExaminationII

1157 trendyline
11
(iii) Predict the outcomes of the following situations. Describe what
you will see and write ionic equations, with state symbols, for any
reactions that occur.

I A rod of metal M is dipped into a solution of 1 mol dm3 CuSO 4 .

[2]

II Dilute sulfuric acid is added into a beaker containing a powdered


sample of metal M.


[2]

[Total: 8]

River Valley High School 9647/02/PRELIM II/16 [Turn over


2016 Preliminary Examination II

1158 trendyline
12

4 The four most abundant salts in sea-water are as follows.

Salt kg per m3

Sodium chloride 27.5

Magnesium chloride 6.75

Magnesium sulfate 5.625

Calcium sulfate 1.80

Magnesium oxide is obtained from sea-water by the following steps.


Step 1:
controlled Step 2:
sea water addition of filtrate addition of Mg(OH)2

CO3 OH Step 3: heat

MgO

The relevant numerical values of the solubility products are given below.

Salt K sp

Sodium carbonate -

Calcium carbonate 5.0 109

Magnesium carbonate 1.0 105

Magnesium hydroxide 1.5 1011

Calcium hydroxide 7.9 106

(a) Explain why the addition of carbonate ions in Step 1 is necessary and
has to be controlled.

trendyline




[2]

River Valley High School 9647/02/PRELIM II/16


2016 Preliminary ExaminationII

1159 trendyline
13
(b) Suggest why the Ksp value of sodium carbonate is not provided.


[1]

(c) Using the reaction scheme above, Barny used 1 dm3 of sea water to
extract magnesium oxide.

(i) Calculate the concentration of Mg2+ in sea water.

[2]

(ii) Barny chose a pH of 9.5 to carry out the extraction of Mg(OH) 2 .


Calculate the maximum mass of magnesium oxide that can be
obtained at pH 9.5.
You may assume that negligible volumes of CO 3 2 and OHwere
used.

[3]

(iii) Suggest how Barny can modify his experiment to improve on the
yield of MgO?

trendyline



[1]

[Total: 9]

River Valley High School 9647/02/PRELIM II/16 [Turn over


2016 Preliminary Examination II

1160 trendyline
14

5 (a) In the periodic table, elements can exhibit either single or variable
oxidation states. One example of an element which can exhibit variable
oxidation state is manganese, Mn, while aluminium, Al is an example of
an element which cannot exhibit variable oxidation state.

(i) Draw a dot and cross diagram for aluminium chloride, AlCl 3 .

[1]

(ii) In the vapour state, the M r of aluminium chloride was found to be


267. However, when aluminium chloride is in the solid state, its
M r was found to be 133.5.
With the aid of suitable diagram, explain this behaviour exhibited
by aluminium chloride and the type of bonding involved.

trendyline





[3]

River Valley High School 9647/02/PRELIM II/16


2016 Preliminary ExaminationII

1161 trendyline
15
(b) Manganese is often found in minerals in combination with iron and is a
metal with important industrial metal alloy uses, particularly in stainless
steels. Manganese is found in various black minerals known
as pyrolusite. Pyrolusite consists mainly of manganese(IV) oxide.
Manganese(IV) oxideis the most common starting material for the
production of compounds of manganese in other oxidation states.
Manganese(IV) oxide undergoes a 2-step reaction to produce Y and Z.

Hot water
Oxidation in alkaline
/ alkaline medium
medium Brown black solid Y in a
MnO 2 MnO 4 2
I II purple solution
containingZ
(i) Define the term transition element.


[1]

(ii) Write the electronic configuration of Mn.

[1]

(iii) Explain why manganese can exhibit variable oxidation state.

[1]

(iv) The brown black solid Y contains 63.8% by mass of manganese


and 36.2% by mass oxygen.
Determine the empirical formula of Y.

trendyline [1]

River Valley High School 9647/02/PRELIM II/16 [Turn over


2016 Preliminary Examination II

1162 trendyline
16

(v) Suggest the identity of Z and state the type of reaction that
occurs at step II.
Hence, construct a balanced equation for the reaction.

[3]

(vi) With reference to your answer in (v), suggesthow bubbling


carbon dioxide gas into the hot solution of MnO 4 2 increases the
yield of Y and Z.


[2]

(c) Manganese(II) carbonate, MnCO 3 , undergoes thermal decomposition in


a similar wayto a Group II carbonate.
MnCO 3 decomposesat 200 C while CaCO 3 decomposes at 840 C.

(i) Write an equation, with state symbols, for the thermal


decomposition of MnCO 3 .

[1]

(ii) Explain why MnCO 3 decompose at a lower temperature than


CaCO 3 .


[2]

trendyline [Total: 16]

River Valley High School 9647/02/PRELIM II/16


2016 Preliminary ExaminationII

1163 trendyline
17

BLANK PAGE

trendyline
River Valley High School 9647/02/PRELIM II/16 [Turn over
2016 Preliminary Examination II

1164 trendyline
18

6 (a) 4-Amino-N-phenylbenzamide, the structure of which is drawn below, is


used in the treatment of epilepsy.
O H

H2N C N

4-amino-N-phenylbenzamide

(i) 4-Amino-N-phenylbenzamide is hydrolysed by warm aqueous


sodium hydroxide. Draw the displayed formulae of the two
hydrolysis products below.

AB [2]

(ii) Which of the two hydrolysis product A&B is a liquid?

. [1]

(iii) When the other product is carefully neutralised with an aqueous


mineral acid to pH 7, C is obtained.
Draw the structure of C.

[1]

(iv) Upon evaporation of all the solvent from the solution of C, a white
solid is obtained.

trendyline
Suggest a physical property of the solid.

. [1]

River Valley High School 9647/02/PRELIM II/16


2016 Preliminary ExaminationII

1165 trendyline
19
(b) State the reagents and conditions required for the conversion of
benzene into phenylamine in two steps. In your answer, identify the
structure of the intermediate.


[3]

(c) The following structure shows part of a protein molecule.


H H O H H O H H O

N C C N C C N C C

CH2 CH2 (CH2)4

OH CO2H NH2

(i) Using suitable diagrams, describe two interactions that the


protein fragment above can exhibit in its tertiary structure

trendyline [3]

River Valley High School 9647/02/PRELIM II/16 [Turn over


2016 Preliminary Examination II

1166 trendyline
20

(ii) In solution, amino acids exist as zwitterions. Choose one of the


amino acids that can be hydrolysed from the protein molecule
above to illustrate what is meant by this term.

[1]

(iii) Amino acids act as buffers in solution. By means of equations,


show how your chosen amino acid can act as a buffer when:

(I) dilute hydrochloric acid is added to its solution;

[1]

(II) dilute sodium hydroxide is added to its solution.

[1]

[Total: 14]

trendyline End of Paper


Pape

River Valley High School 9647/02/PRELIM II/16


2016 Preliminary ExaminationII

1167 trendyline
RIVER VALLEY HIGH SCHOOL
YEAR 6 PRELIMINARYEXAMINATIONII

CANDIDATE
NAME

CLASS 6

CENTRE INDEX
NUMBER S NUMBER

H2 CHEMISTRY 9647/02
Paper 2Structured Questions 13 September 2016
2 hours
Candidates answer on the Question Paper.
Additional Materials: Data Booklet

READ THESE INSTRUCTIONS FIRST

Write your name, class, centre number and index number on all the work you hand in.
Write in dark blue or black pen on both sides of paper.
You may use a soft pencil for any diagrams, graphs or rough working.
Do not use staples, paper clips, highlighters, glue or correction fluid.

Answer all questions in the space provided.


A Data Booklet is provided. Do NOT write anything on it.

The number of marks is given in brackets [] at the end of each question or part question.

For Examiners Use


Paper 2

Question Total
1 2 3 4 5 6
Number (Paper 2)

Marks
12 13 8 9 16 14 72

trre
trendyline
reen
ndylin
in
ine
ne
Paper 1 Paper 3
Pape Total
tal
40 80 192

River Valley High School 9647/02/PRELIM II/16 [Turn over


2016 Preliminary ExaminationII

1168 trendyline
2

1 (a) (i)

[1]

(ii) 10
Amount of salicylic acid in 10 g = = 0.0725 mol
138
0.0725
Theoretical amount of salicylic acid = 100 = 0.121 mol
60
Amount of methyl salicylate required = 0.121 mol
Mass of methyl salicylate required = 0.121 152 = 18.4 g
18.4
Volume of methyl salicylate required = = 15.7 cm3
1.174
Amount of sodium hydroxide required = 0.121 2 = 0.242 mol
0.242
Volume of sodium hydroxide required = 1000
6 .0
[2]
= 40.3 cm3

(b)

Water out

(Liebig)
condenser

Water in

(100 cm3) round bottom


flask containing reaction
mixture
Heating mantle

trendyline
y
River Valley High School 9647/02/PRELIM II/16
2016 Preliminary ExaminationII

1169 trendyline
3

1. Using a 25 cm3 measuring cylinder, measure 16 cm3 (accept up


to 20 cm3) of methyl salicylate and place it in a 100 cm3 round
bottom flask.
2. Using a 50 cm3 measuring cylinder, measure 41 cm3 (accept up
to 50 cm3) of aqueous sodium hydroxide and place it into the
100 cm3 round bottom flask. Add some boiling chips to the
mixture.
3. Set up the reflux set-up shown in the above diagram and heat
the mixture for 30 minutes.
4. Allow the reaction mixture to cool down to room temperature
and remove the boiling chips.
5. Place the round bottom flask in an ice bath.
6. Add concentrated hydrochloric acid to the reaction mixture
slowly, with stirring.
7. Test the acidity of the mixture with a blue litmus paper.
8. Filter the mixture and wash the residue (crude salicylic acid)
with a small amount of cold water.
9. Transfer the crude salicylic acid to a clean 100 cm3 conical
flask. Add a small volume of water and heat the mixture until all
the crude product dissolves.
10. Filter the hot solution, using a fluted filter paper, into a pre-
warmed clean and dry conical flask. Allow the solution to cool
down slowly for crystals to form.
11. When the solution is cooled to room temperature, submerge the
conical flask into an ice bathto allow more crystals to form.
12. Filter the mixture to obtain the pure salicylic acid crystals
and pat dry the crystals using filter paper.
[9]

[Total: 12]

2 (a) pV = nRT
(150 103)(60 103) = n(8.31)(300 + 273)
Amount of nitrogen gas = 1.89 mol
1.89
Amount of sodium azide = = 1.26 mol
3/2
Mass of sodium azide = 1.26 65.0 = 81.9 g [2]

(b) (i) 3
S = (51.3 + 191.6) 70.5 = +268.2 J mol1 K1
2 [1]

trendyline
yline
y
(ii)
i) H
H iss negative, S is positive, TS
TS
S iss negative.
negative.
Since
G
G = H
H T S
S,
TS, G
G is always negative regardless of
temperature
temperature. [1]

River Valley High School 9647/02/PRELIM II/16 [Turn over


2016 Preliminary Examination II

1170 trendyline
4

(iii) The activation energy of the reaction is very high. [1]

(c) (i) Lattice energy of sodium oxide is the enthalpy change


when one mole of solid sodium oxide is formed from
its constituent gaseous ions Na+ and O2. [1]

(ii)
Energy / kJ mol1

2Na+(g) + O2(g)

+702
2Na+(g) + 2e + O(g)

2(+494)
2Na(g) + O(g) LE (Na2O)

2(+107) +
(+496)
2Na(s) + O2(g)
0

416
Na2O(s)

LE (Na 2 O) = 416 2(+107) (+496) 2(+494) (+702)


= 2568 kJ mol1

[3]

(iii) 2nd electron affinity = +702 (142) = +844 kJ mol1


The positive sign of 2nd EA means that energy is required
to overcome the repulsion between the anion and the electron
to be added since both are negatively charged. [2]

(d) (150 101)


Safe depressurisation rate = = 25 kPa s1
2 0.04
Since the airbag being tested depressurises more slowly than the safe
depressurisation rate, it is not safe for use. [2]

[Total: 13]

trendyline
River Valley High School 9647/02/PRELIM II/16
2016 Preliminary ExaminationII

1171 trendyline
5

3 (a) What do you understand by the term standard electrode potential?



Standard electrode potential, E , of an electrode is the relative
potential of this electrode under standard conditionscompared with the
standard hydrogen electrode whose electrode potential is assigned as
0 V. The standard hydrogen electrode consists of H 2 (g) at 1 atm
bubbling over platinum electrode coated with finely divided platinum
which is dipped into 1 mol dm3 H+(aq) at 298 K. [2]

(b) The following cell was set up between a copper electrode and an
unknown metal electrode M2+(aq)/M(s). The standard cell potential was
found to be 0.76V, and the copper foil was connected to the positive
end of the voltmeter.

Direction of electron flow

salt bridge

1 mol dm M
3 2+
1 mol dm
3

CuSO4
Unknown metal
Copper foil
wire, M

(i) Use the Data Booklet to calculate the standard electrode


potential of the M2+(aq)/M(s) system.

Since copper is the positive electrode of the galvanic cell, it is


the cathode.
Standard electrode potential of the M2+(aq)/M(s)
= 0.34 0.76= 0.42V [1]

(ii) Draw an arrow in the box above to show the direction of electron
flow through the voltmeter.

[1]

trrendyline
(iii)
i)) Predict the outcomes of the following
followi situations. Describe what
you will see and write ionic equations,
equations, with state symbols, for
any reactions that occu
occur.

River Valley High School 9647/02/PRELIM II/16 [Turn over


2016 Preliminary Examination II

1172 trendyline
6

I A rod of metal M is dipped into a solution of 1 mol dm3 CuSO 4 .

Metal M dissolves in (blue) solution and pink solid of Cu is


formed. (Blue solution lightens in colour.)
M(s) + Cu2+(aq) Cu(s) + M2+(aq) [2]

II Dilute sulfuric acid is added into a beaker containing a


powdered sample of metal M.

Efferversence is observed; colourless odourless gas evolved


that extinguishes a lighted splint with a pop sound. Metal M
dissolves in (colourless) solution.
2H+(aq) + M(s) M2+(aq) + H 2 (g) [2]

[Total: 8]

4 The four most abundant salts in sea-water are as follows.

Salt kg per m3

Sodium chloride 27.5

Magnesium chloride 6.75

Magnesium sulfate 5.625

Calcium sulfate 1.80

Magnesium oxide is obtained from sea-water by the following steps.

Step 1: Step 2:
sea water filtrate addition of Mg(OH)2
controlled

addition of OH Step 3:

CO3 heat

MgO

The relevant numerical values of the solubility products are given below.

trendyline
y
River Valley High School 9647/02/PRELIM II/16
2016 Preliminary ExaminationII

1173 trendyline
7

Salt K sp

Sodium carbonate -

Calcium carbonate 5.0 109

Magnesium carbonate 1.0 105

Magnesium hydroxide 1.5 1011

Calcium hydroxide 7.9 106

(a) Explain why the addition of carbonate ions in Step 1 is necessary and
has to be controlled.

Both calcium carbonate and magnesium carbonate are sparingly


soluble salts. Carbonate ions are added to precipitate out calcium
carbonate first.
In Step 1, the addition of carbonate must be controlled such
that little/no magnesium carbonate is precipitated/magnesium ions
remain in the filtrate. [2]

(b) Suggest why the Ksp value of sodium carbonate is not provided.

Sodium carbonate is not a sparingly soluble salt/it is very solublein


water. [1]

(c) Using the reaction scheme above, Barny used 1 dm3 of sea water to
extract magnesium oxide.

(i) Calculate the concentration of Mg2+in sea water.

In 1 dm3 of sea water,


6.7510 3
Amount of MgCl 2 = (24.3 + 35.5 2)
10 3
= 7.08 102 mol
5.62510 3
Amount of MgSO 4 = (24.3 + 32.1 + 16.0 4)
10 3
= 4.67 102 mol
[Mg2+] = 0.118 mol dm3

trendyline
y [2]

River Valley High School 9647/02/PRELIM II/16 [Turn over


2016 Preliminary Examination II

1174 trendyline
8

(ii) Barny chose a pH of 9.5 to carry out the extraction of Mg(OH) 2 .


Calculate the maximum mass of magnesium oxide that can be
obtained at pH 9.5.
You may assume that negligible volumes of CO 3  and OH
were used.

At pH 9.5, [OH] =104.5 mol dm3


K sp = [Mg2+][OH]2
Max [Mg2+] after addition of OH(aq) = 1.5 1011 (104.5)2
= 0.0150mol dm3
Amount of Mg2+ precipitated = 0.118 0.0150 = 0.103 mol
Maximum mass of MgO = 0.103 (24.3 + 16.0) = 4.13 g
(or 4.15 g) [3]

(iii) Suggest how Barny can modify his experiment to improve on the
yield of MgO?

He can use a higher pH to lower the concentration of Mg2+(aq)


to increase his yield/He can lower the temperature to lower the
K sp / He can add NaOH(aq) directly to sea water in a controlled
manner. [1]

[Total: 9]

5 (a) In the periodic table, elements can exhibit either single or variable oxidation
states. One example of an element which can exhibit variable oxidation
state is manganese, Mn, while aluminium, Al is an example of an element
which cannot exhibit variable oxidation state.

(i) Draw a dot and cross diagram for aluminium chloride, AlCl 3 .


Cl

Al Cl
Cl


[1]

(ii) In the vapour state, the M r of aluminium chloride was found to be


267. However, when aluminium chloride is in the solid state, its M r
was found to be 133.5.

ttrendyline
y behaviour exhibited by
With the aid of suitable diagram, explain this behav
involved.
aluminium chloride and the type of bonding involved

River Valley High School 9647/02/PRELIM II/16


2016 Preliminary ExaminationII

1175 trendyline
9

Aluminium chloride molecules can form dimers (between


themselves) where one aluminium chloride molecule forms dative
covalent bonds with another Aluminium chloride molecule. Hence,
the observed M r of the dimer is 267 at vapour state.

Dative bond is formed. The lone pair on the Cl atom of AlCl 3 is


donated to the empty orbital of the electron-deficient Al in AlCl 3 to
form a dative (covalent) bond so that Al can attain a stable octet
configuration. [3]

(b) Manganese is often found in minerals in combination with iron. Manganese


is a metal with important industrial metal alloy uses, particularly in stainless
steels. Manganese is found in various black minerals known
as pyrolusite.Pyrolusite consists mainly of manganese(IV) oxide.
Manganese(IV) oxide is the most common starting material for the
production of compounds of manganese in other oxidation states.
Manganese(IV) oxide undergoes a 2-step reaction to produce Y and Z.
Hot water
Oxidation in alkaline
/ alkaline medium
medium Brown black solid Y in a
MnO 2 MnO 4 2 purple solution
I II
containingZ

(i) Define the termtransition element.

A transition element is a d-block element which forms at least one


stable simple ion in which there is a partially-filled d subshell. [1]

(ii) Write the electronic configuration of Mn.

Mn : 1s2 2s2 2p6 3s2 3p6 3d5 4s2 [1]

(iii) Explain why manganese can exhibit variable oxidation state.

3d and 4s electrons are of similar energies, hence a variable


number of 3d and 4s electrons can be involved in bonding (ionic or
covalent).

t
trendyline
y [1]

River Valley High School 9647/02/PRELIM II/16 [Turn over


2016 Preliminary Examination II

1176 trendyline
10

(iv) The brown black solid Y contains 63.8% by mass of manganese and
36.2% by mass oxygen.
Determine the empirical formula of Y.

element Mn O
mass ratio 63.8 36.2
63.8 36.2
mole ratio = 1.16 = 2.26
54.9 16.0
1.16 2.26
simplest ratio = 1.00 = 1.94
1.16 1.16
1 2

Empirical formula: MnO 2 .


[1]

(v) Suggest the identity of Z and state the type of reaction that occurs at
step II.
Hence, construct a balanced equation for the reaction.

Zis MnO 4
Step IIis a disproportionation/redoxreaction.
3MnO42+ 2H2O 2MnO4+ MnO2 + 4OH [3]

(vi) With reference to your answer in (v), suggest how bubbling carbon
dioxide gas into the hot solution of MnO 4 2increases the yield of Y
and Z.

CO 2 is acidic in nature. It can remove the OH.


Equilibrium position of 3MnO42+ 2H2O 2MnO4+ MnO2 +
4OHshifts to the right to replenish the OH, hence it helps in the
disproportionation. [2]

(c) Manganese(II) carbonate, MnCO 3 , undergoes thermal decomposition in a


similar way to a Group II carbonate.
MnCO 3 decomposes at 200 C while CaCO 3 decomposes at 840 C.

(i) Write an equation, with state symbols, for the thermal decomposition
of MnCO 3 .

MnCO 3 (s) MnO(s) + CO 2 (g)

ttrendyline
e dy
d li e
[1]

(ii) Explain why MnCO 3 decompose at a lower te


temperature than
CaCO 3 .

River Valley High School 9647/02/PRELIM II/16


2016 Preliminary ExaminationII

1177 trendyline
11

The ionic radius of Mn2+ is smaller, hence Mn2+ has a higher charge
density than Ca2+.
As a result, the ability of Mn2+ to polarise the large CO 3 2anion is
greater and the CO bonds are weakened to a larger extent. Hence
MnCO 3 decomposes at a lower temperature than CaCO 3. [2]

[Total: 16]

6 (a) 4-Amino-N-phenylbenzamide, the structure of which is drawn below, is


used in the treatment of epilepsy.
O H

H2N C N

4-amino-N-phenylbenzamide

(i) 4-Amino-N-phenylbenzamide is hydrolysed by warm aqueous [2]


sodium hydroxide. Draw the displayed formulae of the two
hydrolysis products below.

H O H
N C

H
-
O Na
+ N

AB

(ii) Which of the two hydrolysis product A&B is a liquid? [1]

(iii) When the other product is carefully neutralised with an aqueous [1]
mineral acid to pH 7, C is obtained.
Draw the structure of C.

ttrendyline
re
en
nd
nddyy
H
+
H N C
-
H O

River Valley High School 9647/02/PRELIM II/16 [Turn over


2016 Preliminary Examination II

1178 trendyline
12

(iv) Upon evaporation of all the solvent from the solution of C, a white [1]
solid is obtained.
Suggest a physical property of the solid.

High melting and boiling point / conduct electricity in molten or


aqueous state

(b) State the reagents and conditions required for the conversion of benzene [3]
into phenylamine in two steps. In your answer, identify the structure of the
intermediate.

Step 1: concentrated H 2 SO 4 , concentrated HNO 3 , 55C


Step 2: Sn, Concentrated HCl, heat, followed by NaOH(aq)
Intermediate: nitrobenzene

(c) The following structure shows part of a protein molecule.


H H O H H O H H O

N C C N C C N C C

CH2 CH2 (CH2)4

OH CO2H NH2

(i) Using suitable diagrams, describe two interactions that the protein [3]
fragment above can exhibit in its tertiary structure

Hydrogen bonding between serine and aspartic acid/serine and


lysine (reject aspartic acid and lysine)
Ionic interactions between aspartic acid and lysine
+
CH2COO H3N (CH2)4
(asp) (lys)
ionic interactions

E.g. of hydrogen bonding



(CH2)4 N H O CH2
H
H

(Lys) (Ser)

ttrendyline
hydrogen bonding

River Valley High School 9647/02/PRELIM II/16


2016 Preliminary ExaminationII

1179 trendyline
13

(ii) In solution, amino acids exist as zwitterions. Choose one of the [1]
amino acids that can be hydrolysed from the protein molecule above
to illustrate what is meant by this term.

H H H

- + - -
+
+
H3N C CO2 H3N C CO2 H3N C CO2

CH2 CH2 (CH2)4

OH CO2H NH2
OR OR [1]

(iii) Amino acids act as buffers in solution. By means of equations, show [2]
how your chosen amino acid can act as a buffer when
(I) dilute hydrochloric acid
(II) dilute sodium hydroxide
is added to its solution.
H H

+ - +
H3N C CO2 H3N C CO2H

CH2 CH2

I OH + H+ OH

H H

+ - -
H3N C CO2 H2N C CO2

CH2 CH2

II OH + OH  OH + H2O

[Total: 14]

trendyline
River Valley High School 9647/02/PRELIM II/16 [Turn over
2016 Preliminary Examination II

1180 trendyline
RIVER VALLEYHIGH SCHOOL
YEAR 6 PRELIMINARY EXAMINATION II

CANDIDATE
NAME

CLASS 6

CENTRE INDEX
NUMBER S NUMBER

H2 CHEMISTRY 9647/03
Paper 3 Free Response 21 September 2016
2 hours
Candidates answer on separate paper.

Additional Materials: Answer Paper


Cover Page
Data Booklet

READ THESE INSTRUCTIONS FIRST

Write your name, class, centre number and index number on all the work you hand in.
Write in dark blue or black pen on both sides of paper.
You may use a soft pencil for any diagrams, graphs or rough working.
Do not use staples, paper clips, highlighters, glue or correction fluid.

Answer any four questions.


Begin each question on a fresh sheet of paper.
A Data Booklet is provided. Do not write anything on it.
You are reminded of the need for good English and clear presentation in your answers.

The number of marks is given in brackets [ ] at the end of each question or part question.
At the end of the examination, fasten all your work securely together, with the cover page on
top.

trendyline
This
his document consists of
o 10 printed pages
page and 2 blank
ank pages.
page

River Valley High School 9647/03/PRELIM II/16 [Turn over


2016 Preliminary Examination II
1181 trendyline
2

Answer any four questions.

1 Many compounds of chlorine are manufactured from brine, NaCl(aq). The


electrolysis of brine produces Cl 2 (g)and NaOH(aq). In some industrial
electrolytic cells, these two substances are allowed to react further. The
products formed in this second reaction dependp on the operating conditions
used.

(a) Write balanced equations for the reaction between Cl 2 (g) and

(i) cold aqueous NaOH; [1]

(ii) hot aqueous NaOH. [1]

(b) Chlorine dioxide, ClO 2 , is used in the treatment of water. It is formed from
ClO 3 ions in an acidic solution.
ClO 3 + 2H+ + e ClO 2 + H 2 O

(i) Draw the dot-and-cross diagrams of ClO 2 and H 2 O 2 .


You may assume that there is no dative bond in either compound. [2]

(ii) Construct the overall equation for the reaction of ClO 3 ions with
H 2 O 2 in acidic solution. [1]

(iii) What is the role of H 2 O 2 in the reaction? [1]

(c) Chlorine can also be used to disinfect water. When chlorine is added to
water, it produceshypochlorous acid, HOCl. Hypochlorous acid is a weak
acid that dissociates into hypochlorite ions, OCl, according to the
following equation.
HOCl(aq) H+(aq) + OCl(aq)

Free chlorine refers to the total chlorine content in HOCl and OCl. The
dissociation curve below illustrates the ratio of hypochlorous acid to
hypochlorite at different pH values.

H
O
O
C
Cl
l

t
[Source: www.hach.com]
River Valley High School 9647/03/PRELIM II/16
2016 Preliminary Examination II
1182 trendyline
3

(i) Determine the pK a of hypochlorous acid. [1]

(ii) Hence, calculate the pH of a 0.0025 moldm3hypochlorous acid


solution. [2]

To determine whether the free chlorine in a sample of tap water meets the
regulatory limit (4mg Clper litre), OCl is quantitatively reduced to Cl by I
ions, which is in turn oxidised to I 2 . The I 2 is titrated with standard sodium
thiosulfate. The following reaction takes place during the titration.

I 2 + 2S 2 O 3  I + S 4 O 6 

(iii) Construct a balanced equation between OCl(aq) and acidified


KI(aq). [1]

(iv) When 2 dm3 of tap water was tested, 6.0 cm3 of 0.00455mol dm3
sodium thiosulfate was required to discharge the colour of iodine.
Calculate the concentration of OCl in the sample of water. [2]

(v) Suggest why the calculated concentration of OClhas the same


value as the concentration of free chlorine. [2]

(vi) Determine if the sample of tap water is safe for consumption. [2]

(d) Chlorine is also used in organic chemistry to produce the Lewis acid
catalyst, FeCl 3 ,for the reaction between methylbenzene and chlorine.

(i) Describe the mechanism of the above reaction. [3]

(ii) FeCl 3 reacts in a similar way with acyl chlorides. Predict the
structure of the product of the following reaction.
O

Cl

FeCl 3
C9H8O

[1]

[Total: 20]

trendyline
River Valley High School 9647/03/PRELIM II/16 [Turn over
2016 Preliminary Examination II
1183 trendyline
4

2 (a) Nitrous oxide or dinitrogen oxide, N 2 O, is commonly known as "laughing


gas" due to the euphoric effects of inhaling it.It is used in surgery and
dentistry for its anesthetic and analgesic effects. To produce N 2 O,
ammonium nitrate is decomposed at 170 C. Water is a by-product of this
reaction.

(i) Write an equation for the production of dinitrogen oxide from


ammonium nitrate. [1]

(ii) In the manufacturing of N 2 O gas, 1kg of N 2 O gas is produced for


every 2.1kg of ammonium nitrate used. Determine the percentage
yield of dinitrogen oxide. [2]

(b) At 1200K, in the presence of gold wire, dinitrogen oxide decomposes as


shown:
2N 2 O(g) 2N 2 (g) + O 2 (g).
To follow the rate of reaction, the change in concentration of a sample of
N 2 O is measure against time. The results are shown below:

Time, t / s Concentration of N 2 O / 103moldm3

0 2.50

1000 2.01

2000 1.62

3000 1.31

4000 1.05

5000 0.85

6000 0.68

7000 0.55

(i) What do you understand by the term half-life of N 2 O? [1]

(ii) Plot the above data on a graph paper.


Use the following scale:
2 cm to represent 1000 s on the x-axis; and

trendyline
2 cm to represent
nt to represen 25 103m
0.25
represent 0. moldm3 on the
y-axis.
y-axis. [2]

(iii) From your graph, deduce the order of the reaction


reactio with respect
toN 2 O. [2]

River Valley High School 9647/03/PRELIM II/16


2016 Preliminary Examination II
1184 trendyline
5

(iv) Calculate the rate constant for the reaction and state its units. [2]

(v) The gold wire acts as a heterogeneous catalyst in this reaction.

Explain the terms in italics and outline the mode of action of the
catalyst. [3]

(c) Alkenes react withcarbenes R 2 C: to yieldcyclopropanes.


One way to generate a substituted carbene is by reacting chloroform,
CHCl 3 , with a strong base.
The mechanism to generate a carbeneis described below.
1. Potassium hydroxide, KOH, removes the proton from CHCl 3 , leaving
behind the electron pair. An anionic intermediate is formed.
2. Cl is lost and a neutral dichlorocarbene is formed.

(i) Based on the description above, draw a mechanism to show the


generation of dichlorocarbene.
Show relevant lone pairs and use curly arrows to indicate the
movement of electron pairs. [3]

An example of a reaction between an alkene and a carbene is as shown:


Cl Cl
CH3CH2 H
C C
+ CHCl + KOH H H
3 + KCl + H O
2
C C C
H CH3 CH3CH2 CH3

(ii) Suggest the structure of the productformed when cyclohexene


reacts with chloroform in the presence of KOH. [1]

(iii) Suggest the role of carbene in its reaction with an alkene. Explain
your answer. [2]

(iv) Suggest the type of reaction when carbene reacts with an alkene. [1]

[Total: 20]

3
trendyline
Copper is a rare element, constituting only 6.8 103 percent of the Earths
River Valley High School 9647/03/PRELIM II/16 [Turn over
2016 Preliminary Examination II
1185 trendyline
6

crust by mass. However, it has a wide range of uses such as in alloys,


plumbing and in electrical cables. The common oxidation states of copper are
+2 and +1.

(a) Copper can be obtained by roasting a copper ore, CuFeS 2 , to give Cu 2 S,


which is further oxidised to form metallic copper. This impure copper can
be purified by electrolysis.

(i) Draw a diagram to illustrate the electrolytic cell used in the


purification of copper, using CuSO 4 (aq) as the electrolyte. [2]

(ii) If a current of 0.8 A was passed through the cell, calculate the time
required for 0.25 g of pure copper to be collected. [2]

(b) Anhydrous copper(II) sulfate, CuSO 4 (s), is a white powder. It dissolves in


water to form a pale blue solution.

(i) State the species responsible for the pale blue colour of the
solution. [1]

(ii) Explain why the solution is pale blue in colour. [3]

(iii) Describe the colour changes observed when the following solutions
are added to the pale blue solution, giving the formulae of all
relevant species.
I.Dilute NH 3 (aq), until in excess;
II.KI(aq). [4]

(c) When Na 2 CO 3 is added to a solution of CuSO 4 and the resultant mixture


is filtered, a green solid, cupric carbonate, is obtained. The formula of this
solid is Cu 2 (OH) a (CO 3 ) b .
0.10 mol of Cu 2 (OH) a (CO 3 ) b required 0.40 mol of hydrochloric acid for
complete reaction. The products of this reaction include copper(II)
chloride, carbon dioxide and water.

(i) Given that 2.4 dm3 of CO 2 was formed at room temperature and
pressure, determine the values of a and b. [2]

(ii) Hence, write an equationfor the reaction of cupric carbonate with


hydrochloric acid. [1]

(d)
trendyline
Cu 2 O, a brick red solid, is a compound where copper is in the oxidation
River Valley High School 9647/03/PRELIM II/16
2016 Preliminary Examination II
1186 trendyline
7

state of +1. It can be formed by reacting an alkaline solution of complexed


Cu2+ with a particular organic functional group.

(i) State the functional group that will produce Cu 2 O with the alkaline
solution of complexed Cu2+. [1]

(ii) Name the type of reaction undergone by the functional group in (i). [1]

(e) Organocopper compounds are useful reagents in organic synthesis. One


such class of compounds is the Gilman reagents, with the general formula
R 2 CuLi, where R represents an alkyl group.
Gilman reagents are effective nucleophiles used in conjugate addition
reactions. Conjugate addition involves theaddition of a nucleophile across
a C=C bond instead of across aC=O bond. This is illustrated in the
example below:
.
O 1 R2CuLi, ether O
.
2 HCl
+ RCu + LiCl
R

The Gilman reagent can be seen as providing the R nucleophile while a


mineral acid (e.g. HCl) provides the proton that is added to one of the
carbon atoms in the C=C bond.

The following synthesis involves a Gilman reagent for one of the steps:
OH O

.
1 (CH3CH2)2CuLi, ether
. NaBH4
2 HCl
A B
I II III

Suggest the reagent and conditions for Step I and the structures of
compounds A and B. [3]

[Total: 20]

4 (a) Copper(I)
per(II) chloride is used as a precursor of a fungicide and
per( an a catalyst for
a variety
riety of organic reactions.
reactions It is sparingly
sp water.
soluble in water

River Valley High School 9647/03/PRELIM II/16 [Turn over


2016 Preliminary Examination II
1187 trendyline
8

(i) Define the standard enthalpy change of hydration of an ion. [1]

(ii) Using the data given in the following table, draw an appropriate
energy cycle and calculate the standard enthalpy change of
solution for copper(I) chloride.

H hyd (Cu+) 593 kJ mol1

H hyd (Cl) 378 kJ mol1

Lattice Energy of CuCl 979 kJ mol1 [3]

(b) (i) Explain what is meant by the term entropy of a chemical system. [1]

Describe and explain how the entropy of the following systems will change
during the stated process. Assume the pressure of each system remains
at 1 atm throughout.

(ii) 1 mol of Cl 2 (g) at 298 K is heated to 500 K. [1]

(iii) 1 mol of Cl 2 (g) at 298 K is reacted with copper according to the


following equation.
Cu(s) + Cl 2 J &X&l 2 (s) [1]

(c) Copper(II) ions are pollutants found in water. One method of purification is
via precipitation of Cu(OH) 2 and CuS.

K sp (298K)

CuS 6.3 1036

Cu(OH) 2 2.0 1019

Using the above data, calculate:

(i) the minimum concentration of Cu2+ for a precipitate of CuS to form


in 10 ppm S2 solution;
(1 ppm = 1 mg dm3) [2]

(ii) the maximum pH of a solution containing 0.100 moldm3 Cu2+ ions. [3]

trendyline
(d) Manyy organic compounds also contain chlo
chlorine. An exam
example would be
1,2-dichlorocyclohexane,
dichlorocyclohexane,, which exhibits both geometrica
dichlorocyclohexane geometrical and optical
isomerism.
merism.
erism

(i) Explain why 1,2-dichlorocyclohexane


cyc can exhibit
exhib geometrical
isomerism. [1]

River Valley High School 9647/03/PRELIM II/16


2016 Preliminary Examination II
1188 trendyline
9

(ii) The effect of plane polarised light on 1,2-dichlorocyclohexane was


investigated. Three different types of 1,2-dichlorocyclohexane was
identified:
Molecule X rotated plane polarised light to the left
Molecule Y rotated plane polarised light to the right
Molecule Zhad no effect on plane polarised light

Suggest an explanation for these observations. [3]

(e) Compounds C and D both have the molecular formula C 7 H 7 Cl. In an


experiment, both compounds are separately heated under reflux for some
time with aqueous sodium hydroxide. The resulting solutions are cooled
and acidified with dilute nitric acid. When aqueous silver nitrate is added
subsequently, a white precipitate is formed with C while no precipitate is
formed with D.
Suggest the structures of C and D. Explain your reasoning for each
compound. [4]

[Total: 20]

5 (a) The electrical conductivities of some Period 3 elements are shown below.

ttrendyline
ren
ndyline
ndy li e
Element

Electrical
conductivity
at 298 K /
Na
a

2.1
Mg

2.3
3
All
A

3
3.5
P

Non-
Non
S

Non-conductors
-cond
Cl

River Valley High School 9647/03/PRELIM II/16 [Turn over


2016 Preliminary Examination II
1189 trendyline
10

107 S m1

(i) With reference to the data above, describe and explain the
difference in electrical conductivities of the elements above. [3]

(ii) Silicon carbide (SiC), also known as carborundum and moissanite,


is used in abrasive and cutting tools.
Suggest the structure and bonding of SiC. [2]

(b) Chlorides of Period 3 elements dissolve in water to give solutions of


varying pH.
Explain the following pH values and write the chemical equations for any
reactions that occur:

(i) AlCl 3 dissolves readily in water to form an acidic solution (pH = 3). [3]

(ii) SiCl 4 (l) dissolves in water to form a strongly acidic solution (pH =
[2]
2).

(c) Caffeic acid is an organic compound found in all plants as it is a key


intermediate in the biosynthesis of lignin, one of the principal components
of plant biomass.
Caffeic acid has the molecular formula C 9 H 8 O 4 . Caffeic acid reacts with
Br 2 (l) to give C 9 H 8 O 4 Br 2 . When treated with hot KMnO 4 and H 2 SO 4 ,
caffeic acid reacts to give F, C 7 H 6 O 4 , and a colourless gas that produces
a white solid with Ca(OH) 2 (aq). F reacts with PCl 5 to give G, C 7 H 5 O 3 Cl,
with the production of steamy white fumes. G reacts with water to form an
acidic solution. When added to neutral FeCl 3 (aq), G also forms a violet
colouration. G reacts with Br 2 (aq) to give H, C 7 H 3 O 4 Br 3 .
Use the information above to deduce the structuresof caffeic acid and
compounds F to H, explaining all the reactions involved. [10]

[Total: 20]

End of Paper

trendyline BLANK PAGE


PAG

River Valley High School 9647/03/PRELIM II/16


2016 Preliminary Examination II
1190 trendyline
11

BLANK PAGE

River Valley High School 9647/03/PRELIM II/16 [Turn over


2016 Preliminary Examination II
1191 trendyline
RIVER VALLEYHIGH SCHOOL
YEAR 6 PRELIMINARY EXAMINATION II

CANDIDATE
NAME

CLASS 6

CENTRE INDEX
NUMBER S NUMBER

H2 CHEMISTRY 9647/03
Paper 3 Free Response 21 September 2016
2 hours
Candidates answer on separate paper.

Additional Materials: Answer Paper


Cover Page
Data Booklet

READ THESE INSTRUCTIONS FIRST

Write your name, class, centre number and index number on all the work you hand in.
Write in dark blue or black pen on both sides of paper.
You may use a soft pencil for any diagrams, graphs or rough working.
Do not use staples, paper clips, highlighters, glue or correction fluid.

Answer any four questions.


Begin each question on a fresh sheet of paper.
A Data Booklet is provided. Do not write anything on it.
You are reminded of the need for good English and clear presentation in your answers.

The number of marks is given in brackets [ ] at the end of each question or part question.
At the end of the examination, fasten all your work securely together, with the cover page on
top.

trendyline
River Valley High School 9647/03/PRELIM II/16 [Turn over
2016 Preliminary Examination II
1192 trendyline
2

Answer any four questions.

1 Many compounds of chlorine are manufactured from brine, NaCl(aq). The


electrolysis of brine produces Cl 2 (g)and NaOH(aq). In some industrial
electrolytic cells, these two substances are allowed to react further. The
products formed in this second reaction depends on the operating conditions
used.

(a) Write balanced equations for the reaction between Cl 2 (g) and

(i) cold aqueous NaOH; [1]

Cl 2 + 2NaOH NaCl + NaClO + H 2 O

(ii) hot aqueous NaOH. [1]

3Cl 2 + 6NaOH 5NaCl + NaClO 3 + 3H 2 O

(b) Chlorine dioxide, ClO 2 , is used in the treatment of water. It is formed from
ClO 3 ions in an acidic solution.
ClO 3 + 2H+ + e ClO 2 + H 2 O

(i) Draw the dot-and-cross diagrams of ClO 2 and H 2 O 2 .


You may assume that there is no dative bond in either compound. [2]

. .. .
xx . . x xx x
x x .
xO x Cl xO x

. xx xx .
H xO
xx
xxO x H
xx

(ii) Construct the overall equation for the reaction of ClO 3 ions with
H 2 O 2 in acidic solution. [1]

2ClO 3 + 2H+ + H 2 O 2 2ClO 2 + 2H 2 O + O 2

(iii) What is the role of H 2 O 2 in the reaction? [1]

Reducing agent

t
River Valley High School 9647/03/PRELIM II/16
2016 Preliminary Examination II
1193 trendyline
3

(c) Chlorine can also be used to disinfect water. When chlorine is added to
water, it produces hypochlorous acid, HOCl. Hypochlorous acid is a weak
acid that dissociates into hypochlorite ions, OCl, according to the
following equation.
HOCl(aq) H+(aq) + OCl(aq)

Free chlorine refers to the total chlorine content in HOCl and OCl. The
dissociation curve below illustrates the ratio of hypochlorous acid to
hypochlorite at different pH values.

H O
O C
Cl l

[Source: www.hach.com]

(i) Determine the pK a of hypochlorous acid [1]

pH = pK a when [HA] = [A]. From the graph, pH = 7.5 when [OCl]=


[HOCl]. Therefore, pK a is 7.4 7.6.

(ii) Hence, calculate the pH of a 0.0025 mol dm3 hypochlorous acid


solution. [2]

pK a = 7.5, K a = 107.5 = 3.16 108 mol dm3


[H+] = (3.16 108 0.0025)1/2 = 8.89 106 mol dm3
S+ OJ 8.89 106) = 5.05

To determine whether the free chlorine in a sample of tap water meets the
regulatory limit (4 mg Clper litre), OCl is quantitatively reduced to Cl by
I ions, which is in turn oxidised to I2 . The I2 is titrated with standard
sodium thiosulfate. The following reaction takes place during the titration.

ttrendyline
rendyline
I 2 + 2S 2 O 3  II + S 4 O 6 

(iii) Constru
Construct
onstruct
ct a balanced equation between OCl(aq) and acidified
betwee OCl
KI(aq)
( [1]

2H+(aq) + 2I(aq) + OCl DT &l (aq) + I 2 (aq) + H 2 O(l)


River Valley High School 9647/03/PRELIM II/16 [Turn over
2016 Preliminary Examination II
1194 trendyline
4

(iv) When 2 dm3 of tap water was tested, 6.0 cm3 of 0.00455 mol dm3
sodium thiosulfate was required to discharge the colour of iodine.
Calculate the concentration of OCl in the sample of water. [2]

Amount of S 2 O 3  = 2.73 105 mol


Amount of iodine produced = 1.37 105 mol
Amount of OCl present = 1.37 105mol
Concentration = 6.83 104 mol dm6

(v) Suggest why the calculated concentration of OClhas the same


value as the concentration of free chlorine. [2]

In the equilibrium, HOCl(aq) H+(aq) + OCl(aq), as OCl is


reduced by I, [OCl] decreases and the equilibrium position shifts
right to favour the formation of more OCl until all the HOCl
completely dissociates.

(vi) Determine if the sample of tap water is safe for consumption. [2]

Mass concentration of free chlorine = 6.83 106 35.5 = 0.242


mg per litre.
The tap water is safe for consumption.

(d) Chlorine is also used in organic chemistry to produce the Lewis acid
catalyst, FeCl 3 ,for the reaction between methylbenzene and chlorine.

(i) Describe the mechanism of the above reaction. [3]

FeCl 3 + Cl 2 FeCl 4 + Cl+


Cl
H Cl

FeCl 4 + HCl + FeCl
+ +
Cl slow f ast 3

ttrendyline
rendyline
(ii) FeCl 3 reacts in a similarlar way with acyl chlorides.
chloride Predict the
structure of the product of the following reaction. [1]

River Valley High School 9647/03/PRELIM II/16


2016 Preliminary Examination II
1195 trendyline
5

Cl

FeCl 3
C9H8O

[Total: 20]

trendyline
River Valley High School 9647/03/PRELIM II/16 [Turn over
2016 Preliminary Examination II
1196 trendyline
6

2 (a) Nitrous oxide or dinitrogen oxide, N 2 O, is commonly known as "laughing


gas" due to the euphoric effects of inhaling it.It is used in surgery and
dentistry for its anesthetic and analgesic effects. To produce N 2 O,
ammonium nitrate is decomposed at 170 C. Water is a by-product of this
reaction.

(i) Write an equation for the production of dinitrogen oxide from


ammonium nitrate. [1]

NH 4 NO 3 V  H 2 O (g) + N 2 O (g)

(ii) In the manufacturing of N 2 O gas, 1kg of N 2 O gas is produced for


every 2.1kg of ammonium nitrate used. Determine the percentage
yield of dinitrogen oxide. [2]
2.1 1000
Amount of NH 4 NO 3 used = = 26.3 mol
14 2 4 16 3

Theoretical amt of N 2 O produced = 26.3 mol

Theoretical mass of N 2 O produced = 26.3 (14 2+16) = 1157g

1000
% yield = 100% = 86.4%
1157

(b) At 1200K, in the presence of gold wire, dinitrogen oxide decomposes as


shown:
2N 2 O(g) 2N 2 (g) + O 2 (g).
To follow the rate of reaction, the change in concentration of a sample of
N 2 O is measure against time. The results are shown below:

Time, t / s Concentration of N 2 O / 103 mol


dm3

0 2.50

1000 2.01

2000 1.62

3000 1.31

4000 1.05

ttrendyline
rendyline
5000 0.85

6000 0.68
.6

7000 0.55
0 55

River Valley High School 9647/03/PRELIM II/16


2016 Preliminary Examination II
1197 trendyline
7

(i) What do you understand by the term half-life of N 2 O? [1]

Half-life is time taken for the concentration of N 2 O to be halved.

(ii) Plot the above data on a graph paper.


Use the following scale:
2 cm to represent 1000 s on the x-axis; and
2 cm to represent to represent 0.25 103 mol dm3 on the
y-axis. [2]

3
Concentration of N2O / 103 mol

2.5
2
1.5
dm3

1
0.5
0
0 2000 4000 6000 8000

Time/s

(iii) From your graph, deduce the order of the reaction with respect to
N 2 O. [2]

When concentration decreases from 2.5 103 to 1.25 103, time


taken is 3300 s.
When concentration decreases from 1.25 103 to 0.7 103, time
taken is 3100 s. (to be shown on the graph)
Since half-life is approximately constant, the order of reaction with
respect to N 2 O is 1.
Show on graph that half time constant

(iv) Calculate the rate constant for the reaction and state its units. [2]

ln 2
Rate constant = = 2.17 x 104 s1

ttrendyline
t1 / 2

Correct
orrect answer
answe
Correct unit
units

(v) The gold wire acts as a heterogeneous catalyst in this reaction. [3]
River Valley High School 9647/03/PRELIM II/16 [Turn over
2016 Preliminary Examination II
1198 trendyline
8

Explain the terms in italics and outline the mode of action of the
catalyst.

A catalyst is a substance that provides an alternative pathway with


lower activation energy, remaining chemically unchanged at the
end of reaction. A heterogeneous catalyst is one that is in a
different phases as the reactants.
Adsorption
The gold wire provides active sites whereby N 2 O molecules may
be adsorbed.
The adsorption weakens NO bonds so that the N 2 O molecules
are more reactive.
Reaction
The N 2 O molecules on the gold surface are in close proximitand
the correct orientationso that they can readily react together.
Desorption

The products, N 2 and O 2 , formed diffuse away from the surface of


the catalystand the active sites become available again.

(c) Alkenes react with carbenes R 2 C: to yield cyclopropanes.


One way to generate a substituted carbene is by reacting chloroform,
CHCl 3 , with a strong base.
The mechanism to generate a carbene is described below.
1. Potassium hydroxide, KOH, removes the proton from CHCl 3 , leaving
behind the electron pair. An anionic intermediate is formed.
2. Cl is lost and a neutral dichlorocarbene is formed.

(i) Based on the description above, draw a mechanism to show the


generation of dichlorocarbene.
Show relevant lone pairs and use curly arrows to indicate the
movement of electron pairs. [3]

trendyline
River Valley High School 9647/03/PRELIM II/16
2016 Preliminary Examination II
1199 trendyline
9

HO
H

Cl C Cl KOH
Cl C Cl + H2O
Cl Cl

Cl
C + Cl

Cl

An example of a reaction between an alkene and a carbene is as shown:


Cl Cl
CH3CH2 H
C C
+ CHCl + KOH H H
+ KCl + H O
3
2
C C C
H CH3 CH3CH2 CH3

(ii) Suggest the structure of the productformed when cyclohexene


reacts with chloroform in the presence of KOH. [1]

Cl

Cl

(iii) Suggest the role of carbene in its reaction with an alkene. Explain
your answer. [2]

Carbene acts as an electrophile.


Carbon in carbene is electron deficient as it is bonded to two

ttrendyline
y
electronegative
g chlorine atoms/it has onlyy 6 valence electrons,
hence it acts as an electrophile.
electroph

(iv) Suggest the type of reaction when carbene reacts with an alkene. [1]

River Valley High School 9647/03/PRELIM II/16 [Turn over


2016 Preliminary Examination II
1200 trendyline
10

Addition

[Total: 20]

trendyline
River Valley High School 9647/03/PRELIM II/16
2016 Preliminary Examination II
1201 trendyline
11

3 Copper is a rare element, constituting only 6.8 103 percent of the Earths
crust by mass. However, it has a wide range of uses such as in alloys,
plumbing and in electrical cables. The common oxidation states of copper are
+2 and +1.

(a) Copper can be obtained by roasting a copper ore, CuFeS 2 , to give Cu 2 S,


which is further oxidised to form metallic copper. This impure copper can
be purified by electrolysis.

(i) Draw a diagram to illustrate the electrolytic cell used in the


purification of copper, using CuSO 4 (aq) as the electrolyte. [2]
battery connected
correctly

pure Cu
(cathode)

impure Cu
(anode)

dilute CuSO4

(ii) If a current of 0.8 A was passed through the cell, calculate the time
required for 0.25 g of pure copper to be collected. [2]

Amt of Cu = 0.25 / 63.5 = 3.94 103 mol


Cu2+(aq) + 2e &X V
Amt of e required = 2(3.94 103) = 7.88 103 mol
Quantity of charge required = nF = 7.88 103 96500 = 760 C
Time required = Q / I = 760 / 0.8 = 950 s (or 15.8 min)

(b) Anhydrous copper(II) sulfate, CuSO 4 (s), is a white powder. It dissolves in


water to form a pale blue solution.

(i) State the species responsible for the pale blue colour of the
solution. [1]

ttrendyline
rendyline
dyli
[Cu(H 2 O) 6 ]2+

(ii) Explain
xplain why the solution is pale blue in colour.
colour [3]

Cu2+
C 2
has a partially filled 3d subshell (electronic configuration
1s 2s 2p63s23p63d9). In the presence of H 2 O ligands in
2 2

River Valley High School 9647/03/PRELIM II/16 [Turn over


2016 Preliminary Examination II
1202 trendyline
12

[Cu(H 2 O) 6 ]2+, the 3d orbitals of Cu2+are split into 2 sets of orbitals


with different energies/become non-degenerate.The difference in
energies (E) between these 2 sets of non-degenerate 3d orbitals
is small and radiation from the visible region of the electromagnetic
spectrum is absorbedwhen an electron is promoted from a lower
energy d-orbital to another unfilled/partially-filled d orbital of higher
energy. The (blue)colour observed corresponds to the complement
of the (orange) lightabsorbed.

(iii) Describe the colour changes observed when the following solutions
are added to the pale blue solution, giving the formulae of all
relevant species.
I.Dilute NH 3 (aq), until in excess;
II.KI(aq). [4]

I. A (pale) blue precipitate of Cu(OH) 2 is formed, which is soluble


in excess NH 3 (aq) to form a dark blue
2+
solutionof [Cu(NH 3 ) 4 (H 2 O) 2 ] .
II. An off-white/cream precipitateof CuI(Accept: Cu 2 I 2 ) is
formed in a brown solutionof I 2 .

(c) When Na 2 CO 3 is added to a solution of CuSO 4 and the resultant mixture


is filtered, a green solid, cupric carbonate, is obtained. The formula of this
solid is Cu 2 (OH) a (CO 3 ) b .
0.10 mol of Cu 2 (OH) a (CO 3 ) b required 0.40 mol of hydrochloric acid for
complete reaction. The products of this reaction include copper(II)
chloride, carbon dioxide and water.

(i) Given that 2.4 dm3 of CO 2 was formed at room temperature and
pressure, determine the values of a and b. [2]

Cu 2 (OH) a (CO 3 ) b + (a+2b)H+ &X2+ + bCO 2 + (a+b)H 2 O


Amt of CO 2 formed = 2.4 / 24 = 0.10 mol
1 0.10
b = 1
b 0.10

ttrendyline
y
1 0.10
a+ 2b = 4 a
a = 2
a 2b 0.440

(ii) Hence, write an equationfor


Hence or the
t reaction of cupric ccarbonate with [1]

River Valley High School 9647/03/PRELIM II/16


2016 Preliminary Examination II
1203 trendyline
13

hydrochloric acid.

Cu 2 (OH) 2 CO 3 + 4HCl &XCl 2 + CO 2 + 3H 2 O

(d) Cu 2 O, a brick red solid, is a compound where copper is in the oxidation


state of +1. It can be formed by reacting an alkaline solution of complexed
Cu2+ with a particular organic functional group.

(i) State the functional group that will produce Cu 2 O with the alkaline
solution of complexed Cu2+. [1]

Aliphatic/Non-aromatic aldehyde

(ii) Name the type of reaction undergone by the functional group in (i). [1]

Oxidation

(e) Organocopper compounds are useful reagents in organic synthesis. One


such class of compounds is the Gilman reagents, with the general formula
R 2 CuLi, where R represents an alkyl group.
Gilman reagents are effective nucleophiles used in conjugate addition
reactions. Conjugate addition involves theaddition of a nucleophile across
a C=C bond instead of across aC=O bond. This is illustrated in the
example below:
.
O 1 R2CuLi, ether O
.
2 HCl
+ RCu + LiCl
R

The Gilman reagent can be seen as providing the R nucleophile while a


mineral acid (e.g. HCl) provides the proton that is added to one of the
carbon atoms in the C=C bond.

The following synthesis involves a Gilman reagent for one of the steps:

OH O

.
1 (CH3CH2)2CuLi, ether
. NaBH4
2 HCl
A B
I II III

trendyline
y
gest the reagent and conditions for Step I and the structures of
Suggest
compounds
pound A and B
pounds B..

Reagents and conditions for Step I:


[3]

River Valley High School 9647/03/PRELIM II/16 [Turn over


2016 Preliminary Examination II
1204 trendyline
14

acidified K 2 Cr 2 O 7 (aq), heat (under reflux)


Structures:

O OH

A B

[Total: 20]

trendyline
River Valley High School 9647/03/PRELIM II/16
2016 Preliminary Examination II
1205 trendyline
15

4 (a) Copper(I) chloride is used as a precursor of a fungicide and a catalyst for


a variety of organic reactions. It is sparingly soluble in water.

(i) Define the standard enthalpy change of hydration of an ion. [1]

The standard enthalpy change of hydration of an ion is the enthalpy


change when one mole of gaseous ions is hydrated under
standard conditions.

(ii) Using the data given in the following table, draw an appropriate
energy cycle and calculate the standard enthalpy change of
solution for copper(I) chloride.

H hyd (Cu+) 593 kJ mol1

H hyd (Cl) 378 kJ mol1

Lattice Energy of CuCl 979 kJ mol1 [3]

H soln (CuCl(s)) = H hyd (Cu+) + H hyd (Cl) LE(CuCl)


= (593) + (378) (979) = +8 kJ mol1
Hsoln(CuCl(s))
CuCl(s) Cu+(aq) + Cl(aq)

Hhyd(Cu+)
LE(CuCl) Hhyd(Cl)

Cu+(g) + Cl(g)

(b) (i) Explain what is meant by the term entropy of a chemical system. [1]

Entropy is a measure of the disorder in a system. (The more


disordered a system, the greater the number of ways for particles
and energy to be dispersed, therefore the larger its entropy.)

Describe and explain how the entropy of the following systems will change
during the stated process. Assume the pressure of each system remains
at 1 atm throughout.

(ii) 1 mol of Cl 2 (g) at 298 K is heated to 500 K. [1]

S> 0. There is an increase in disorder as the


Entropy increases/

ttrendyline
rendyline
rendy
d liine
particles have more kinetic energy
energ on average
averag and there
are more ways to distribute
distribu the larger number of quanta of
energy.
nergy.

(iii) 1 moll off Cl 2 (g) at 298 K is reacted with copper according


a
ac to the
ffollowing
ll i equation. ti [1]

River Valley High School 9647/03/PRELIM II/16 [Turn over


2016 Preliminary Examination II
1206 trendyline
16

Cu(s) + Cl 2 J &X&l 2 (s)

Entropy decreases/S< 0. There is an decrease in disorder as


the number of gaseous molecules decreases from 1 to 0.

(c) Copper(II) ions are pollutants found in water. One method of purification is
via precipitation of Cu(OH) 2 and CuS.

K sp (298 K)

CuS 6.3 1036

Cu(OH) 2 2.0 1019

Using the above data, calculate:

(i) the minimum concentration of Cu2+ for a precipitate of CuS to form


in 10 ppm S2 solution;
(1 ppm = 1 mg dm3) [2]

[S2] = (0.01 / 32.1) = 3.12 104 mol dm3


[Cu2+] = (6.3 1036) / (3.12 104) = 2.02 1032 mol dm3

(ii) the maximum pH of a solution containing 0.100 moldm3 Cu2+ ions. [3]

K sp (Cu(OH) 2 ) = [Cu2+][OH]2
2.0 1019 = (0.1)(x)2
[OH] = 1.41 109 mol dm3

pOH = lg(1.41 109) = 8.85


pH= 14 pOH = 14 8.85 = 5.15

(d) Many organic compounds also contain chlorine. An example would be


1,2-dichlorocyclohexane, which exhibits both geometrical and optical
isomerism.

(i) Explain why 1,2-dichlorocyclohexane can exhibit geometrical


isomerism. [1]

There is restricted rotation about the carbon-carbon bond in the


ring structure, resulting in cis- and trans-1,2-
dichlorocyclohexane[1].

ttrendylin
rendyylin
ne
e
Cll
C Cll
C Cl

Cl OR Cl Cl

River Valley High School 9647/03/PRELIM II/16


2016 Preliminary Examination II
1207 trendyline
17

cis isomer trans isomer

(ii) The effect of plane polarised light on 1,2-dichlorocyclohexane was


investigated. Three different types of 1,2-dichlorocyclohexane was
identified:
Molecule X rotated plane polarised light to the left
Molecule Y rotated plane polarised light to the right
Molecule Zhad no effect on plane polarised light

Suggest an explanation for these observations. [3]

Molecule Explanation

Cl
Molecules possess two
X chiral carbons and no plane
of symmetry. Hence, they
are mirror images of each
Cl
other that are non-
Cl superimposable (i.e.
enantiomers of each other)
Y Structure of X and Y can be
interchanged.
Cl

Cl

Although the molecule has


Cl OR two chiral carbons, there is a
Z plane of symmetry present.
Cl Hence, the mirror images are
superimposable.

Cl

Cl Cl

ttrendyline
X
nd
dy nne
e Cll
C

Y
Cll
C

River Valley High School 9647/03/PRELIM II/16 [Turn over


2016 Preliminary Examination II
1208 trendyline
18

(e) Compounds C and D both have the molecular formula C 7 H 7 Cl. In an


experiment, both compounds are separately heated under reflux for some
time with aqueous sodium hydroxide. The resulting solutions are cooled
and acidified with dilute nitric acid. When aqueous silver nitrate is added
subsequently, a white precipitate is formed with C while no precipitate is
formed with D.
Suggest the structures of C and D. Explain your reasoning for each
compound. [4]

C CH2Cl

C undergoes nucleophilic substitution when heated with


NaOH. Free chloride ion then combines with Ag+ to form white
precipitate of AgCl.

D Cl

Cl Cl

OR OR

CCl bond in D has partial double bond character due to


the delocalisation of a lone pair of electrons of chlorine into
the benzene ring. Hence, D does not undergo nucleophilic
substitution with NaOH.

[Total: 20]

5 (a) The electrical conductivities of some Period 3 elements are shown below.

Element Na Mg Al P S Cl

ttrendyline
ndy
d li
Electrical
conductivit
conductivity
at 298 K /
107 S m1
2.1 2.3 3
3.5 Non-
Non
Non-conductors
-cond

River Valley High School 9647/03/PRELIM II/16


2016 Preliminary Examination II
1209 trendyline
19

(i) With reference to the data above, describe and explain the
difference in electrical conductivities of the elements above. [3]

Na, Mg &Al
They are metals/have giant metallic structure.
Presence of delocalised valence electrons which act as mobile
charge carriers thus making them good conductors.
Electrical conductivity increases from Na (2.1 107 S m1) to Mg
(2.3 107 S m1) to Al (3.5 107 S m1)as the number of valence
electrons increases.

P 4 , S 8 &Cl 2
They are non-metals / have simple covalent structure.
Absence of mobile charge carriers thus they are non-conductors.

(ii) Silicon carbide (SiC), also known as carborundum and moissanite,


is used in abrasive and cutting tools.
Suggest the structure and bonding of SiC. [2]

Giant covalent structurewithstrong electrostatic forces of attraction


between the Si and C nuclei and the shared pair of electrons.

(b) Chlorides of Period 3 elements dissolve in water to give solutions of


varying pH.
Explain the following pH values and write the chemical equations for any
reactions that occur:

(i) AlCl 3 dissolves readily in water to form an acidic solution (pH = 3). [3]

Both hydration of ions and substantial hydrolysis of Al3+(aq) occurs.


Due to its high charge density, Al3+ is highly polarising
and weakens the OH bonds in the water molecules of the
complex, causing the OH bonds to break and hence release the
hydrogen ions.
AlCl 3 (s) + 6H 2 2 O >$l(H 2 O) 6 ]3+(aq) + 3Cl(aq)
[Al(H 2 O) 6 ]3+(aq) + H 2 O(l) [Al(H 2 O) 5 (OH)]2+(aq) + H 3 O+(aq)

(ii)

ttrendyline
rendyline
d li
SiCl 4 (l) dissolves in water to form a strongly acidic solution (pH =
SiC
2))

SiCl 4 undergoes hydrolysis in water. This is becau


because Si atom in
SiCl 4 has energetically accessible vacant 3d orbital for dative
[2]

bonding with water molecules.


River Valley High School 9647/03/PRELIM II/16 [Turn over
2016 Preliminary Examination II
1210 trendyline
20

SiCl 4 (l) + 2H 2 O(l) SiO 2 (s) + 4HCl(aq)

(c) Caffeic acid is an organic compound found in all plants as it is a key


intermediate in the biosynthesis of lignin, one of the principal components
of plant biomass.
Caffeic acid has the molecular formula C 9 H 8 O 4 . Caffeic acid reacts with
Br 2 (l) to give C 9 H 8 O 4 Br 2 . When treated with hot KMnO 4 and H 2 SO 4 ,
caffeic acid reacts to give F, C 7 H 6 O 4 , and a colourless gas that produces
a white solid with Ca(OH) 2 (aq). F reacts with PCl 5 to give G, C 7 H 5 O 3 Cl,
with the production of steamy white fumes. G reacts with water to form an
acidic solution. When added to neutral FeCl 3 (aq), G also forms a violet
colouration. G reacts with Br 2 (aq) to give H, C 7 H 3 O 4 Br 3 .
Use the information above to deduce the structuresof caffeic acid and
compounds F to H, explaining all the reactions involved. [10]

Caffeic acid &+UDWLR1:1which suggests presence of benzene ring.


Caffeic acid has a C=C double bond/has an alkene group as it
undergoes electrophilic addition with Br 2 .
Caffeic acid undergoes oxidation/oxidative cleavagewith hot KMnO 4 to
give carbon dioxide gas. Loss of 2 carbons as CO 2 suggests formation
of ethanedioic acid/HO 2 CCO 2 Hafter oxidative cleavage.
F contains a carboxylic functional group as it undergoes
(nucleophilic) substitution with PCl 5 to give G, an acid chloride that
hydrolyses in water, and fumes of HCl.
G contains phenol functional group as it forms a complex with FeCl 3 (aq).
G undergoes electrophilic substitutionwith Br 2 (aq). Substitution of 3 Br
atoms suggests 3 positions on benzene ring that are available for
substitution.

Caffeic acid O

HO
OH

HO

F O

tre
endyline
end
nd yl
HO
H
OH

HO

River Valley High School 9647/03/PRELIM II/16


2016 Preliminary Examination II
1211 trendyline
21

G O

HO
Cl

HO

H Br O

HO
OH

HO Br

Br

[Total: 20]

End of Paper

trendyline
River Valley High School 9647/03/PRELIM II/16 [Turn over
2016 Preliminary Examination II
1212 trendyline





67$1'5(:6-81,25&2//(*(


-&3UHOLPLQDU\([DPLQDWLRQV

&KHPLVWU\ 
WK
+LJKHU  6HSWHPEHU
3DSHU0XOWLSOH&KRLFH KRXU
$GGLWLRQDO0DWHULDOV0XOWLSOH&KRLFH$QVZHU6KHHW'DWD%RRNOHW


5($'7+(6(,16758&7,216),567

:ULWHLQVRIWSHQFLO'RQRWXVHVWDSOHVSDSHUFOLSVKLJKOLJKWHUVJOXHRUFRUUHFWLRQIOXLG

7KHUHDUHTXHVWLRQVRQWKLVSDSHU$QVZHUDOOTXHVWLRQV)RUHDFKTXHVWLRQWKHUHDUHIRXU
SRVVLEOHDQVZHUV$%&DQG'&KRRVHWKHRQH\RXFRQVLGHUFRUUHFWDQGUHFRUG\RXUFKRLFH
LQVRIWSHQFLORQWKHVHSDUDWH$QVZHU6KHHW

(DFK FRUUHFW DQVZHU ZLOO VFRUH RQH PDUN $ PDUN ZLOO QRW EH GHGXFWHG IRU D ZURQJ DQVZHU
$Q\URXJKZRUNLQJVKRXOGEHGRQHLQWKLVERRNOHW




   

   

trendyline
y
LVGRFXPHQWFRQVLVWVR 


7KLVGRFXPHQWFRQVLVWVRISULQWHGSDJHVLQFOXGLQJWKLVSDJH
KLVGRFXPHQWFRQVLVWVRI SULQWHGSDJHV
SULQWHGSDJHV LQFOXGLQJ
QFOXGLQJWKLVSDJ
KLVSDJ

>7XUQ2YHU
1213 trendyline





6HFWLRQ$
)RU HDFK TXHVWLRQ WKHUH DUH IRXU SRVVLEOH DQVZHUV $ % & DQG ' &KRRVH WKH RQH \RX
FRQVLGHUWREHFRUUHFW

 :KDWYROXPHRI&2ZLOOEHOLEHUDWHGZKHQ\FPRIPL[WXUHRIYROXPHRIPHWKDQH
DQGHWKHQHDUHFRPSOHWHO\EXUQWLQR[\JHQ"

 $ \FP

 % \
FP

 & \
FP

 ' \FP




 Use of the Data Booklet is relevant to this question.

(WKDQHGLRDWH LRQV &2 DUH R[LGLVHG E\ DFLGLILHG DTXHRXV SRWDVVLXP PDQJDQDWH 9,, 
WR JLYH FDUERQ GLR[LGH :KDW YROXPH RI  PRO GP SRWDVVLXP PDQJDQDWH 9,,  LV
UHTXLUHGWRR[LGLVHFRPSOHWHO\[PRORIWKHVDOW1D+&2+&2"

 $ FP

 % FP

 & FP

 ' FP

trendyline
>7XUQ2YHU
1214 trendyline





 :KLFKJUDSKFRUUHFWO\GHVFULEHVWKHEHKDYLRXURIIL[HGPDVVHVRIWKHLGHDOJDVHV;DQG
<ZKHUH;KDVDKLJKHUMUWKDQ<"

$ 39 &RQVWDQW3 % 39 &RQVWDQW3
< ;

<
;

7& 7.

   

& 39 &RQVWDQW7 ' 3 &RQVWDQW7

;
<
<

;

3 39



trendyline
>7XUQ2YHU
1215 trendyline





 7KHWDEOHEHORZVKRZVWKHLRQLVDWLRQHQHUJ\ LQN-PRO RIIRXUHOHPHQWVODEHOOHGAB


CDQGD

(OHPHQW VW,( QG,( UG,( WK,( WK,( WK,(

A      

B      

C      

D      


:KLFKHOHPHQWDERYHKDVQRpHOHFWURQVLQWKHYDOHQFHVKHOO"

 $ (OHPHQW$

 % (OHPHQW%

 & (OHPHQW&

 ' (OHPHQW'

trendyline


>7XUQ2YHU
1216 trendyline





 7K
KHGLDJUDPVKRZVWKHVWUXFWXUHRIISDUWRIDF
FU\VWDORILFH
H



:
:KLFKRIWKH IROORZLQJLVVLQFRUUHFW"
"

 $ $OOERQG
GDQJOHVVX HDFKR[\JHQDWRPDUH
XUURXQGLQJH

 % 7ZRHOH
HFWURQVIURP JHQDWRPDUHLQYROYHGLQIRUPLQJK\GURJHQE
PHDFKR[\J ERQGV

 & 7KHK\G
GURJHQERQ
QGVDUHZHD
DNHUWKDQWKH
H2+FRYD
DOHQWERQGVV

 ' 7KH RS
SHQ VWUXFWXUUH RI LFH FD
DXVHV LFH WR
R KDYH D KLJKHU YROXP
PH DQG ORZ
ZHU GHQVLW\
WKDQZD
DWHU

 :
:KLFKRIWKH KHUPLFHQWKDOS\FKDQJH"
IROORZLQJKDVDQH[RWK

 $
%D J 
%D J  H 

 %
0J6 VV 0J J
J 6 J 

 & 1 J
J 1 J 

 '
) J H )  J 

trendyline


>7XUQ2YHU
1217 trendyline





 pV
7KHYDOXHRI LVSORWWHGDJDLQVWpDWWKHVDPHWHPSHUDWXUHIRUWKUHHJDVHV1H1+
RT
DQG&+ZKHUHpLVWKHSUHVVXUHDQG9LVWKHYROXPHRIWKHJDV

*DV 

*DV 

:KLFKRIWKHIROORZLQJFRUUHFWO\LGHQWLILHVWKHJDVHV"




  *DV*DV*DV

 $ 1+&+1H

 % 1+1H&+

 & 1H&+1+

 ' &+1H1+

 trendyline
>7XUQ2YHU
1218 trendyline





 :KLFKJUDSKZRXOGFRQILUPWKDWWKHUDWHRIGHFRPSRVLWLRQRIDPPRQLXPFKORULGHLVDILUVW
RUGHUUHDFWLRQ"

$ %

 >1+&l@
>1+&l@



 
  WLPH  WLPH


& '

 
UDWHRI UDWHRI
GHFRPSRVLWLRQ  GHFRPSRVLWLRQ 
RI1+&l RI1+&l
 

 
 
 
 >1+&l@  >1+&l@
 

trendyline


>7XUQ2YHU
1219 trendyline





 ,QDQHQ]\PHFDWDO\VHGUHDFWLRQWKHUDWHLVDIIHFWHGE\WKHFRQFHQWUDWLRQRIVXEVWUDWHDV
VKRZQLQWKHJUDSKEHORZ

7KHPHFKDQLVPIRUWKHUHDFWLRQLVDVVKRZQ

HQ]\PHVXEVWUDWHHQ]\PHVXEVWUDWHFRPSOH[ IDVW 

HQ]\PHVXEVWUDWHFRPSOH[HQ]\PHSURGXFW VORZ 

:KLFKRIWKHIROORZLQJLVLQFRUUHFW"


 $ :KHQ>VXEVWUDWH@LVORZWKHUHDFWLRQLVILUVWRUGHUZLWKUHVSHFWWRWKHVXEVWUDWH

 % :KHQ>VXEVWUDWH@LVKLJKWKHUHDFWLRQLV]HURRUGHUZLWKUHVSHFWWRWKHVXEVWUDWH

 & 7KHUDWHHTXDWLRQLVJLYHQDVUDWH N>HQ]\PHVXEVWUDWHFRPSOH[@

 ' 7KHUHDFWLRQRFFXUVLQDWZRVWHSUHDFWLRQ

trendyline
>7XUQ2YHU
1220 trendyline





 7KHHTXDWLRQIRUWKH+DEHU3URFHVVLVVKRZQEHORZ

:KLFKRIWKHIROORZLQJVWDWHPHQWVUHJDUGLQJWKH+DEHU3URFHVVLVFRUUHFW"

 $ 7KH XVH RI )H FDWDO\VW KHOSV WR ORZHU WKH DFWLYDWLRQ HQHUJ\ RI WKH IRUZDUG UHDFWLRQ
PRUHWKDQWKDWRIWKHEDFNZDUGUHDFWLRQ

 % 7KHXVHRIDKLJKHUSUHVVXUHLQFUHDVHVWKHHTXLOLEULXPFRQVWDQW

 & 7KHUDWHFRQVWDQWRIWKHEDFNZDUGUHDFWLRQZLOOGHFUHDVHPRUHWKDQWKHUDWHFRQVWDQW
RIWKHIRUZDUGUHDFWLRQZKHQWHPSHUDWXUHLVGHFUHDVHG

 ' 7KHUDWHFRQVWDQWRIWKHEDFNZDUGUHDFWLRQZLOOGHFUHDVHOHVVWKDQWKHUDWHFRQVWDQW
RIWKHIRUZDUGUHDFWLRQZKHQWHPSHUDWXUHLVLQFUHDVHG

 $VWXGHQWFDUULHGRXWDWLWUDWLRQLQYROYLQJPROGP%D 2+ DQGPROGP+&22+


:KLFKRIWKHIROORZLQJLVDVXLWDEOHLQGLFDWRUWRLQGLFDWHWKHHQGSRLQWRIWKLVWLWUDWLRQ"

 $ 3KHQROSKWKDOHLQ S+UDQJH 

 % 0HWK\OUHG S+UDQJH 

 & 0HWK\ORUDQJH S+UDQJH 

trendyline
 ' HL
HLVQRVXLWDEOHLQGLFDWRU
7KHUHLVQRVXLWDEOHLQGLFDWRU

>7XUQ2YHU
1221 trendyline





 0DJQHVLXP DUVHQDWH 0J $V2  LV D VSDULQJO\ VROXEOH VDOW FRPPRQO\ XVHG WR PDNH
LQVHFWLFLGHV,WGLVVRFLDWHVLQZDWHUDFFRUGLQJWRWKHHTXDWLRQEHORZ

,IWKHVROXELOLW\SURGXFWLVSZKDWLVWKHYDOXHRI>$V2 DT @DWHTXLOLEULXP"


 $ S 

 %  
S 


 &  
S 


 '  
S


trendyline
>7XUQ2YHU
1222 trendyline





 Use of the Data Booklet is relevant to this question.

$QHOHFWURFKHPLFDOFHOOLVVHWXSDVVKRZQ

3W &X

)H62 DT  6DOW
)H 62  DT  EULGJH
&X62 DT 


:KLFKRIWKHIROORZLQJFRUUHFWO\GHVFULEHVWKHFHOO"

 $ EFHOOLV9ZKHQVRGLXPF\DQLGHLVDGGHGWRWKH)H)HKDOIFHOO

 % &XLVWKHSRVLWLYHHOHFWURGHZKHQVRGLXPK\GUR[LGHLVDGGHGWRWKH)H)HKDOIFHOO

 & 7KHUHDFWLRQLVOHVVVSRQWDQHRXVZKHQH[FHVVDTXHRXVDPPRQLDLVDGGHGWR&X&X
KDOIFHOO

 ' 7KH FDWLRQV LQ WKH VDOW EULGJH ZLOO PRYH RYHU WR &X&X KDOIFHOO WR SUHYHQW
DFFXPXODWLRQRISRVLWLYHFKDUJHV

trendyline


>7XUQ2YHU
1223 trendyline





 7KHSURSHUWLHVRIHDFK3HULRGHOHPHQWUHVHPEOHWKRVHRIWKH3HULRGHOHPHQWGLUHFWO\
DERYHLW

:KLFK3HULRGHOHPHQWVIRUPR[LGHVWKDWGLVVROYHLQZDWHUWRJLYHDQDFLGVROXWLRQ"

 $ $VDQG6H

 % *DDQG*H

 & *DDQG6H

 ' &DDQG6H

 (OHPHQWV:;<DQG=DUHLQ3HULRGRIWKH3HULRGLF7DEOH

 :KDVWKHKLJKHVWPHOWLQJSRLQWDPRQJDOOWKH3HULRGHOHPHQWV
 7KHFKORULGHVRI<DQG=DUHDFLGLFZKLOHWKHFKORULGHRI;LVQHXWUDO
 7KHR[LGHRI<LVEDVLFZKLOHWKHR[LGHRI=LVDPSKRWHULF

:KLFKRIWKHIROORZLQJVWDWHPHQWVLVFRUUHFW"


 $ 7KHDWRPLFUDGLXVLQFUHDVHVLQWKHIROORZLQJRUGHU:=<;

 % 7KHILUVWLRQLVDWLRQHQHUJ\GHFUHDVHVLQWKHIROORZLQJRUGHU;!<!=!:

 & 7KHS+RIWKHFKORULGHVGHFUHDVHVLQWKHIROORZLQJRUGHU:!=!<!;

 ' 7KHHOHFWULFDOFRQGXFWLYLW\LQFUHDVHVLQWKHIROORZLQJRUGHU:=<;

trendyline


>7XUQ2YHU
1224 trendyline





 :KDWFDQEHVHHQZKHQDSLHFHRIPDJQHVLXPULEERQLVSODFHGLQFROGZDWHU"

 $ $YLJRURXVHIIHUYHVFHQFHRFFXUV

 % %XEEOHVRIJDVIRUPVORZO\RQWKHPDJQHVLXPULEERQ

 & 7KHPDJQHVLXPULEERQIORDWVRQWKHVXUIDFHRIWKHZDWHUDQGUHDFWVTXLFNO\

 ' 7KHPDJQHVLXPULEERQJORZVDQGDZKLWHVROLGLVSURGXFHG

 $TXHRXV FKORULQH LV DGGHG WR DTXHRXV VRGLXP EURPLGH DQG WKH PL[WXUH LV VKDNHQ ZLWK
WHWUDFKORURPHWKDQHDQRUJDQLFVROYHQW:KDWREVHUYDWLRQZRXOGEHPDGH"

 $ $FRORXUOHVVOD\HUIRUPVRQWRSRIDUHGGLVKEURZQOD\HU

 % 7KHVROXWLRQLQWKHWHVWWXEHWXUQVRUDQJH

 & $EURZQVROLGLVVHHQLQDFRORXUOHVVVROXWLRQ

 ' $\HOORZLVKJUHHQJDVLVSURGXFHG

trendyline


>7XUQ2YHU
1225 trendyline





 $ FRPSRXQG RI FKURPLXP KDV WKH JHQHUDO IRUPXOD RI &U&l+2 ,W LV REVHUYHG WKDW 
PROHRIWKLVFRPSRXQGUHDFWVZLWKPROHVRIDTXHRXV$J12$OVRWKLVFRPSRXQGJLYHV
WKHIROORZLQJDEVRUSWLRQVSHFWUXP

&RORXU :DYHOHQJWK QP 
9LROHW 
%OXH 
*UHHQ 
<HOORZ 
2UDQJH 
5HG 

:KLFK RI WKH IROORZLQJ VKRZV WKH FRUUHFW VWUXFWXUH RI WKH FKURPLXP FRPSRXQG DQG LWV
FRORXU"

  6WUXFWXUH &RORXU

$ >&U &l  +2 @&l+2 9LROHW

% >&U +2 @&l *UHHQ

& >&U &l +2 @&l+2 2UDQJH

trendyline
' &U l  +2 @+2
&U &
>&U &l 2UDQJHYLROHW
2UDQJHYLROH
2UDQJHYLROHW


>7XUQ2YHU
1226 trendyline





 &RPSRXQG;&+2

 FDQEHR[LGLVHGE\VRGLXPGLFKURPDWH 9, 
 UHDFWVZLWK1D%+

:KDWFRXOGEH;"


 $ &+&22&+2+

 % &+&+ 2+ &22+

 & &+2&2&+2+

 ' +2&+&+ 2+ &+2

 :KLFKWUDQVIRUPDWLRQKDVDVHWRIFRQGLWLRQVWKDWLVFRUUHFW"

$

%

&

'




trendyline
>7XUQ2YHU
1227 trendyline





 &RPSRXQG:KDVWKHIROORZLQJVWUXFWXUH

:KDWUHDFWVFRPSOHWHO\ZLWKPROHRI:"


 $ PRORI1D V 

 % PRORI+%U J 

 & PRORI1D2+ DT 

 ' PRORIGLQLWURSKHQ\OK\GUD]LQH

 :KLFKOLTXLGZKHQVKDNHQZLWK$J12 DT ZLOOJLYHDSUHFLSLWDWHLQWKHVKRUWHVWWLPH"


 $ WHWUDFKORURPHWKDQH

 % SURSDQR\OFKORULGH

 & FKORUREXWDQH

 ' FKORURSURSHQH

trendyline
>7XUQ2YHU
1228 trendyline





 )RU WKH IROORZLQJ SDLUV RI RUJDQLF FRPSRXQGV ZKLFK UHDJHQW ZLOO EH DEOH WR GLVWLQJXLVK
WKHP"


    5HDJHQW

$ &+&+&+ &+ &+&+ &l J 

% &+&2&+, &+&+&+2 ,LQ1D2+ DT 

& &+&+&+&l &+&+&+%U 1D2+ DT 

' &+&+&+2 +&22+ 7ROOHQVUHDJHQW




 1\ORQLVDW\SHRISRO\DPLGHWKDWLVXVHGIRUFRQVWUXFWLRQPDWHULDOVGXHWRLWVKLJKWHQVLOH
VWUHQJWKDQGUHVLVWDQFH$VHFWLRQRIQ\ORQPROHFXOHLVVKRZQEHORZ

$VDPSOHRIQ\ORQZDVVXEMHFWHGWRDVHULHVRIFKHPLFDOUHDFWLRQVDQG&RPSRXQGV34
DQG5ZHUHREWDLQHGDWHDFKVWHSUHVSHFWLYHO\

 5HDJHQW 2UJDQLF&RPSRXQG
6WHS +RWFRQFHQWUDWHG1D2+IROORZHGE\FDUHIXO 3
DFLGLILFDWLRQ
6WHS /L$O+LQGU\HWKHU 4
6WHS H[FHVVFRQFHQWUDWHG+62KHDW 5


:KLFKRIWKHIROORZLQJVWDWHPHQWVLVWUXH"


 $ &RPSRXQG3ZLOOUHDFWZLWK'13+WRSURGXFHRUDQJHSUHFLSLWDWH

 % &RPSRXQG4JLYHVRIIZKLWHIXPHVZLWK3&l


&

' trendyline
6WHSLVDQDGGLWLRQUHDFWLRQ
LVDQDGGLWLRQUHDFWLRQ
LVDQDGGLWLRQUHDFWLRQ

&RPSRXQG3FDQXQGHUJRLQWUDPROHFXODUQXFOHRSKLOLFVXEVWLWXWLRQWRIRUPDQDPLGH
SRXQG 3FDQXQGHUJRLQWUDPROHFXODUQXFOHRSKLOLFVXEVWLWXWLRQ
FDQXQGHUJRLQWUDPROHFXODUQXFOHRSKLOLFVXEVWLWXWLR

>7XUQ2YHU
1229 trendyline





 :KLFK RI WKH IROORZLQJ LV WKH PDMRU LQWHUPHGLDWH IRU WKH UHDFWLRQ RI PHWK\OF\FORKH[HQH
DQGEURPLQHLQWKHSUHVHQFHRIFRQFHQWUDWHGPDJQHVLXPK\GURJHQVXOIDWH0J 262+ "

$ %

 

& '

 

trendyline 

>7XUQ2YHU
1230 trendyline





 7KH R[LGDWLRQ RI DOFRKRO : XVLQJ KRW DFLGLILHG SRWDVVLXP PDQJDQDWH 9,,  SURGXFHG D
FRPSRXQGRIIRUPXOD&+2:KLFKRIWKHIROORZLQJUHDFWLRQVZLOOSURGXFHWKHEHVW\LHOGRI
DOFRKRO:"

 $ 5HDFWLQJFKORURSURSDQHZLWKKRWHWKDQROLF1D2+

 % 5HDFWLQJSURSDQRLFDFLGZLWK/L$l+

 & 5HDFWLQJFKORURSURSDQHZLWKKRWDTXHRXV1D2+

 ' 5HDFWLQJSURSHQHZLWKFROGZDWHU

 $PR[LFLOOLQLVDQDQWLELRWLFXVHIXOIRUWKHWUHDWPHQWRIEDFWHULDOLQIHFWLRQVVXFKDVPLGGOH
HDULQIHFWLRQV

$PR[LFLOOLQ


+RZ PDQ\ PROHV RI 1D2+ DT  ZLOO UHDFW ZLWK RQH PROH RI DPR[LFLOOLQ ZKHQ LW LV KHDWHG
XQGHUUHIOX[ZLWKDQH[FHVVRI1D2+ DT "


 $ 

 % 


&

'


 trendyline
>7XUQ2YHU
1231 trendyline





 %RODQGLROLVDEDQQHGV\QWKHWLFDQDEROLFVWHURLG

%RODQGLRO


:KLFKRIWKHIROORZLQJVWDWHPHQWVLVQRWWUXH"


 $ %RODQGLROKDVVWHUHRLVRPHUVZKHQUHDFWHGZLWKOLTXLGEURPLQH

 % 3DVVLQJKRWDOXPLQLXPR[LGHRYHUERODQGLROSURGXFHVDFRPSRXQGZLWKWKUHHGRXEOH
ERQGV

 & 2UDQJHSRWDVVLXPGLFKURPDWH 9, WXUQVJUHHQZKHQ%RODQGLROLVKHDWHGZLWKLW

 ' %RODQGLROSURGXFHVRQHPROHRIK\GURJHQJDVZKHQUHDFWHGZLWKVRGLXPPHWDO

 :KHQDONDQH-&+LVUHDFWHGZLWKEURPLQHXQGHUXOWUDYLROHWOLJKWRQO\WKUHHLVRPHULF
PRQREURPRFRPSRXQGV./DQG0DUHSURGXFHG2QO\FRPSRXQG/LVFKLUDO:KDWDUH
WKHVWUXFWXUHVRI-DQG/"


  - /

 $ &+ &&+&+ &+ &&+%U&+

 % &+ &+&+ &+  &+ &+ &+%U &+ &+ 

trendyline
 & &+ &+&+&+&+ &+ &+&+%
&+&+%U&+&+

 ' &+
+&+
+&+&+&+&+ &
&+
+&+&+%U&
&+%U&+&+&+

>7XUQ2YHU
1232 trendyline





 7KH &XUWLXV UHDUUDQJHPHQW FDQ EH XVHG WR SURGXFH FDUEDPDWHV IURP DF\O D]LGHV $
JHQHUDOH[DPSOHRIWKH&XUWLXVUHDUUDQJHPHQWLVVKRZQEHORZ

:KLFKSDLURIFRPSRXQGVFDQEHXVHGWRV\QWKHVLVHHWK\OFDUEDPDWH&+&+2&21+"


 $

 %

 &

 '

>7XUQ2YHU
1233 trendyline





6HFWLRQ%

)RUHDFKRIWKHTXHVWLRQVLQWKLVVHFWLRQRQHRUPRUHRIWKHWKUHHQXPEHUHGVWDWHPHQWVWR
PD\EHFRUUHFW'HFLGHZKHWKHUHDFKRIWKHVWDWHPHQWVLVRULVQRWFRUUHFW \RXPD\ILQGLW
KHOSIXOWRSXWDWLFNDJDLQVWWKHVWDWHPHQWVWKDW\RXFRQVLGHUWREHFRUUHFW 
7KHUHVSRQVHV$WR'VKRXOGEHVHOHFWHGRQWKHEDVLVRI

$ % & '
DQGDUH DQGRQO\DUH DQGRQO\DUH RQO\LVFRUUHFW
FRUUHFW FRUUHFW FRUUHFW

1RRWKHUFRPELQDWLRQRIVWDWHPHQWVLVXVHGDVDFRUUHFWUHVSRQVH


 :KLFKVWDWHPHQWVUHJDUGLQJWKHIROORZLQJUHDFWLRQVDUHWUXH"

L 12%U +%U12+2

LL 1D+1D+


LLL &U2 2+ &U2 +2



  ,QUHDFWLRQL12UDGLFDOLVSURGXFHG

  1D+SURGXFHGLQUHDFWLRQLLFDQDFWDVDQR[LGLVLQJDJHQW

  5HDFWLRQLLLLVDUHGR[UHDFWLRQDVWKHQXPEHURIR[\JHQDWRPVKDVGHFUHDVHG

 :KLFK RI WKH IROORZLQJ VHWV RI FRPSRXQGV FRQVLVWV RI D JLDQW LRQLF VWUXFWXUH D JLDQW
FRYDOHQWVWUXFWXUHDQGDVLPSOHFRYDOHQWVWUXFWXUH"

  $l)JUDSKLWH%H&l

  $l%U326L

trendyline
  $l2%)
% 6L&ll

>7XUQ2YHU
1234 trendyline





 =LQFUHDFWVUHDGLO\ZLWKGLOXWHK\GUREURPLFDFLGWRIRUP]LQFEURPLGHDQGK\GURJHQ

=Q V +%U DT =Q%U DT + J 

7KH VWDQGDUG HQWKDOS\ FKDQJH IRU WKLV UHDFWLRQ FDQ EH PHDVXUHG LQ WKH ODERUDWRU\:KDW
IXUWKHU LQIRUPDWLRQ LV QHHGHG LQ RUGHU WR FDOFXODWH WKH VWDQGDUG HQWKDOS\ FKDQJH RI
IRUPDWLRQRIDTXHRXV]LQFEURPLGH"


  HIIRU+%U DT 

  VWDQGQGHOHFWURQDIILQLW\RI=Q

  HIIRU+ J 

 Use of the Data Booklet is relevant to this question.

:KHQ FKURPLXP LV KHDWHG VHSDUDWHO\ ZLWK FKORULQH EURPLQH DQG LRGLQH ZKLFK RI WKH
IROORZLQJFDQEHREWDLQHGDVWKHILQDOSURGXFW"

  &U,

  &U&l

  &U%U

trendyline
>7XUQ2YHU
1235 trendyline





 7KHIROORZLQJVKRZVDUHDFWLRQVFKHPHLQYROYLQJDTXHRXV&X62

7KHVWUXFWXUHRIHGWDLRQLVDVVKRZQ

:KLFKRIWKHIROORZLQJVWDWHPHQWVUHJDUGLQJWKHUHDFWLRQVFKHPHDUHFRUUHFW"

  6LVSRVLWLYHIRUVWHS,,,

  7KHGRUELWDOVRI&XLQVSHFLHV4DUHGHJHQHUDWH

  6WHSV9DQG9,LQYROYHOLJDQGH[FKDQJH

trendyline


>7XUQ2YHU
1236 trendyline





 /DZVRQHLVDG\HZKLFKFDQEHH[WUDFWHGIURPWKHKHQQDSODQW/DZVRQLDLQHUPLV

/DZVRQH


:KLFKRIWKHIROORZLQJUHDJHQWZLOOUHDFWZLWK/DZVRQH"

  1+2+

  
>$J 1+ @ 
  &RQFHQWUDWHG+12

 $QDPLQHPD\EHGHULYHGIURPDQDPLGHLQDUHDFWLRQNQRZQDV+RIIPDQQUHDUUDQJHPHQW
3DUWRIWKHPHFKDQLVPLVDVVKRZQ

:KLFKRIWKHIROORZLQJVWDWHPHQWVLVOLNHO\WREHWUXH"


  1D2+LVXVHGDVDEDVHLQ6WHSVDQG

trendyline
  %ULVEHKDYLQJDVDQHOHFWURSKLOHLQ6WHS
VEHKDYLQJDVDQHOHFWURSKLOHLQ6WHS
EHKDYLQJDVDQHOHFWURSKLOHLQ

  R[LGDWLRQVWDWHRI% LQ6WHSLV
7KHR[LGDWLRQVWDWHRI%U
R[LGDWLRQVWDWHRI%U Q6WHSLV

>7XUQ2YHU
1237 trendyline





 7KH DFWLYH LQJUHGLHQW LQ DQWLGLDUUKRHD PHGLFLQHV LV ORSHUDPLGH 7KH V\QWKHVLV RI
ORSHUDPLGHLVVKRZQEHORZ

2
2 %U %U
+%U 62&l
2+ &l
, ,,
2 2

,,, 1+ &+ 
+2 2
&l 1 +2
1 &l 1+ %U
1
,9
/RSHUDPLGH 2 

:KDWW\SHRIUHDFWLRQLVVKRZQE\DWOHDVWRQHRIWKHVWHSVLQWKHDERYHURXWH"


  +\GURO\VLV

  1XFOHRSKLOLFVXEVWLWXWLRQ

  (OHFWURSKLOLFVXEVWLWXWLRQ



trendyline
>7XUQ2YHU
1238 trendyline





 6RPHRIWKHDPLQRDFLGVIRXQGLQVR\SURWHLQDUHOLVWHGLQWKHWDEOHEHORZ

$PLQRDFLG 5JURXS

&\VWHLQH &+6+

7KUHRQLQH &+ 2+ &+

*OXWDPLFDFLG &+&+&22+

9DOLQH &+ &+ 

:KLFKRIWKHIROORZLQJVWDWHPHQWVDUHWUXH"


  $GGLQJDUHGXFLQJDJHQWZLOOGLVUXSWWKHGLVXOILGHERQGVDQGGHQDWXUHWKHSURWHLQ

  7KH5JURXSVRIJOXWDPLFDFLGDQGWKUHRQLQHZLOOLQWHUDFWZLWKHDFKRWKHUWKURXJK
K\GURJHQERQGV

  $GGLQJ1D2+ DT ZLOOGHQDWXUHWKHSURWHLQ

 :KLFK RI WKH IROORZLQJ SDLUV RI FRPSRXQGV ZRXOG SURGXFH DQ DPLGH ZKHQ UHDFWHG
WRJHWKHU"

  &+&+& &+ 1+DQG&+&2&l

 



  &+&+ &+ 1+DQG&+&22+

(1' 2) 3$3(5
$3
(1'2)3$3(5

>7XUQ2YHU
1239 trendyline



6$-&+&+(035(/,03$3(5 :RUNHGVROXWLRQV 
         
& % $ ' % ' & & & &
         
$ ' % $ $ % $ % ' %
         
% % % % % & % $ $ &
         
' ' ' & ' ' % % $ '

 5DWLRRIPHWKDQHHWKHQH  

/HWWKHWRWDOYROXPHRIPL[WXUHEH\FP

 
LHIUDFWLRQRIPHWKDQHLQPL[WXUH  IUDFWLRQRIHWKHQHLQPL[WXUH  
 

 &+2&2+2 &+2&2+2

9ROXPHFP     
y y y [ y 
   

  y
+HQFHWRWDOYROXPHRI&2  y y   FP
  


 )URP0Q2 DT &2 DT + DT 0Q DT &2 J +2 l 

5HDFWLQJPROHUDWLRRI0Q2&2   

PRORI1D+&2+&2JLYHVPRORI&2

[PRORI1D+&2+&2JLYHV[PRORI&2

)URP 


QRRIPROHVRI0Q2UHDFWHGZLWK[PROHVRI&2  [[ PRO

trendyline
d




0Q2VROXWLRQUHTXLUHG
.0Q2
+HQFHYROXPHRI.0Q2 ROXWLRQUHTXLUHG   GP FP





1240 trendyline


 pV Q57

m
pV  57
Mr

m
2SWLRQ$: 6KDSHRIJUDSKLVFRUUHFW 5LVWKHJUDGLHQW6LQFH;KDVDKLJKHU0U
Mr
LWVJUDGLHQWLVOHVVVWHHS &RUUHFW 

2SWLRQ%6KDSHRIJUDSKLVFRUUHFW)URPDERYHVLQFH;KDVDKLJKHU0ULWV
JUDGLHQWVKRXOGEHOHVVVWHHS+HQFHJUDSKLVZURQJ

2SWLRQ&6KDSHRIJUDSKLVZURQJ7KHJUDSKVVKRXOGEHDKRUL]RQWDOOLQH6LQFH7
LVFRQVWDQWS9LVFRQVWDQWIRUHDFKFRPSRXQG

2SWLRQ'6KDSHRIJUDSKLVZURQJ7KHJUDSKVVKRXOGEHDYHUWLFDOOLQH6LQFH7LV
FRQVWDQWS9LVFRQVWDQWIRUHDFKFRPSRXQG

   


 (OHPHQW A EHORQJV WR *URXS 9 7KLV LV EHFDXVH WKH ILUVW ELJ MXPS LQ LRQLVDWLRQ
HQHUJ\RFFXUVEHWZHHQWKHWKDQGWK,(LQGLFDWLQJWKDWWKHWKHOHFWURQLVUHPRYHG
IURPWKHLQQHUTXDQWXPVKHOO9DOHQFHFRQILJXUDWLRQQVQS

(OHPHQW B EHORQJV WR *URXS ,9 7KLV LV EHFDXVH WKH ILUVW ELJ MXPS LQ LRQLVDWLRQ
HQHUJ\RFFXUVEHWZHHQWKHWKDQGWK,(LQGLFDWLQJWKDWWKHWKHOHFWURQLVUHPRYHG
IURPWKHLQQHUTXDQWXPVKHOO9DOHQFHFRQILJXUDWLRQQVQS

(OHPHQW CPD\ EHORQJV WR *URXS 9,,, 7KLV LV EHFDXVH WKHUH LV QR QRWLFHDEOH ELJ
MXPSEHWZHHQWKHLRQLVDWLRQHQHUJLHV 

(OHPHQW D EHORQJV WR *URXS ,, 7KLV LV EHFDXVH WKH ILUVW ELJ MXPS LQ LRQLVDWLRQ
HQHUJ\ RFFXUV EHWZHHQ WKH QG DQG UG ,( LQGLFDWLQJ WKDW WKH WK HOHFWURQ LV
UHPRYHG IURP WKH LQQHU TXDQWXP VKHOO +HQFH LWV YDOHQFH HOHFWURQLF
FRQILJXUDWLRQLVQVDQGKDVQRpHOHFWURQVLQWKHYDOHQFHVKHOO

 2SWLRQ $ LV FRUUHFW DV WKH  K\GURJHQ ERQGV DQG  2+ FRYDOHQW ERQGV ZRXOG IRUP D
ERQGDQJOHRI

2SWLRQ % LV ZURQJ DV WZR ORQH SDLUV IURP HDFK R[\JHQ DWRP DUH LQYROYHG LQ K\GURJHQ
ERQGLQJ

2SWLRQ&LVFRUUHFWDVLQWHUPROHFXODUIRUFHVRIDWWUDFWLRQDUHZHDNHUWKDQVWURQJFRYDOHQW
ERQGV


trendyline
FWDQGWKLVLVWKHH[SODQDWLRQZK\LFHIORDWV RQZDWHU
FWDQGWKLVLVWKHH[SODQDWLRQZK\LFHIORDWV
2SWLRQ'LVFRUUHFWDQGWKLVLVWKHH[SODQDWLRQZK\LFHIORDWVRQZDWHU

1241 trendyline


 2SWLRQ$LVZURQJDVVWLRQLVDWLRQHQHUJ\LVHQGRWKHUPLF

2SWLRQ % LV ZURQJ DV ODWWLFH GLVVRFLDWLRQ HQHUJ\ RU RSSRVLWH RI ODWWLFH HQHUJ\  LV
HQGRWKHUPLF

2SWLRQ&LVZURQJDVERQGEUHDNLQJ EUHDNLQJWKH11ERQG LVHQGRWKHUPLF

2SWLRQ ' LV FRUUHFW DV VW HOHFWURQ DIILQLW\ UHOHDVHV KHDW GXH WR HOHFWURVWDWLF IRUFHV RI
DWWUDFWLRQEHWZHHQWKHQXFOHXVRI)DQGHOHFWURQ

 1+GHYLDWHWKHPRVWIURPLGHDOJDVEHFDXVHLWKDVVWURQJHUK\GURJHQERQGLQJEHWZHHQ
WKHPROHFXOHVWKDQLQGXFHGGLSROHLQGXFHGGLSROHLQWHUDFWLRQVZKLFKH[LVWLQ&+DQG1H


/DUJH DQG KHDY\ JDV SDUWLFOHV ZLWK VLJQLILFDQW HOHFWURQ FORXG VL]H GHYLDWH PRUH WKDQ
VPDOOHUJDVSDUWLFOHV 1HYV&+&+KDVDODUJHUHOHFWURQFORXGVL]H 7KHJUHDWHUWKH
HOHFWURQFORXGVL]HWKHVWURQJHUWKHYDQGHU:DDOVIRUFHVRIDWWUDFWLRQEHWZHHQWKHJDV
SDUWLFOHV7KXV1HKDVWKHOHDVWGHYLDWLRQ
+HQFHJDVLV1HJDVLV&+JDVLV1+



 2SWLRQ$KDVWKHZURQJVKDSHIRUDILUVWRUGHUUHDFWLRQ

2SWLRQ%LVZURQJDVWKHFRQVWDQWKDOIOLIHLVQRWVWDWHG+HQFHLWGRHVQRWFRQILUPZKHWKHU
LWLVDILUVWRUGHURUVHFRQGRUGHUUHDFWLRQ

2SWLRQ&LVFRUUHFW5DWHLVGLUHFWO\SURSRUWLRQDOWRFRQFHQWUDWLRQRI1+&l.

2SWLRQ'LVZURQJDVUDWHVKRXOGLQFUHDVHZKHQFRQFHQWUDWLRQRI1+&l LVKLJK

 

 

 (Q]\PHVXEVWUDWH FRPSOH[ LV DQ LQWHUPHGLDWH +HQFH LW FDQQRW DSSHDU LQ WKH UDWH
HTXDWLRQ7KHUDWHHTXDWLRQVKRXOGEHUDWH N>HQ]\PH@>VXEVWUDWH@

 2SWLRQ$LVZURQJDVDFDWDO\VWZLOOORZHUWKHDFWLYDWLRQHQHUJ\RIERWKIRUZDUGDQGEDFNZDUGUHDFWLRQVE\
WKHVDPHPDJQLWXGH

2SWLRQ%LVZURQJDVHTXLOLEULXPFRQVWDQWLVRQO\DIIHFWHGE\DFKDQJHLQWHPSHUDWXUH

trendyline
FWDVZKHQWKHWHPSHUDWXUHGHFUHDVHVWKHU FRQVWDQWVRI
FWDVZKHQWKHWHPSHUDWXUHGHFUHDVHVWKHUDWH
2SWLRQ&LVFRUUHFWDVZKHQWKHWHPSHUDWXUHGHFUHDVHVWKHUDWHFRQVWDQWVRIERWKIRUZDUGDQGEDFNZDUG
FUHDVH
UHDVH 7KH SRVLWLRQ RI HTXLOLEULXP ZLOO VKLIW
UHDFWLRQV ZLOO GHFUHDVH VKLIW WR WKH ULJKW WR UHOHDVH
U KHDW 6LQFH IRUZDUG
UHG
HG WKH UDWH FRQVWDQW RI WKH EDFNZDUG UHDFWLRQ ZLOO GHFUH
UHDFWLRQ LV IDYRXUHG GHFUHDVH PRUH WKDQ WKDW IRU WKH
IRUZDUGUHDFWLRQ

2SWLRQ'LVZURQJDVWKHUDWHFRQVWDQWVRIERWKIRUZDUGDQGEDFNZDUGUHDFWLRQVVKRXOGLQFUHDVHZKHQ

1242 trendyline


WHPSHUDWXUHLQFUHDVHV

 2SWLRQ $ LV FRUUHFW DV WKLV LV D ZHDN DFLGVWURQJ EDVH WLWUDWLRQ ZKLFK ZLOO UHVXOW LQ WKH IRUPDWLRQ RI
FDUER[\ODWHVDOWWKDWXQGHUJRHVEDVLFK\GURO\VLVWRJLYHDS+!DWHTXLYDOHQFHSRLQW

 6 >0J@>$V2@

/HWWKHVROXELOLW\RI0J $V2 EH[PROGP

6  [  [ [ 6

[  6 

>$V2@ [  6   6 



2SWLRQ $ LV ZURQJ DV ZKHQ 1D&1 LV DGGHG WR WKH )H)H KDOI FHOO WKH KDOI HTXDWLRQ WR ORRN DW LV DV
JLYHQEHORZ

7KHUHIRUHEFHOOLV   9

2SWLRQ%LVFRUUHFW

6LQFH&X&XKDVDPRUHSRVLWLYHEYDOXHLWZLOOEHWKHFDWKRGHZKHUHUHGXFWLRQWDNHVSODFH7KHUHIRUH
&XZLOOEHWKHSRVLWLYHHOHFWURGH

2SWLRQ&LVZURQJ

trendyline


EFHOOEHIRUHDGGLQJH[FHVVDT1+
QJH[FHVVDT1+      9
9

EFHOODIWHUDGGLQJH[FHVVDT1+
H[FHVV DT 1+     
H[FHVVDT1+     9 PRUHVSRQWDQ
9 PRUHVSRQWDQHRXV 
9 PRUHVSRQWD

2SWLRQ ' LV ZURQJ 6LQFH &X&X LV WKH DQRGH WKLV PHDQV WKDW &X ZLOO EH IRUPHG DQG WR PDLQWDLQ

1243 trendyline


HOHFWULFDOQHXWUDOLW\DQLRQVLQWKHVDOWEULGJHZLOOPRYHRYHULQVWHDG


 SHULRGHOHPHQWV0J$l6L36
SHULRGHOHPHQWV&D*D*H$V6H

0J2GLVVROYHVLQZDWHUWRIRUPDONDOLQHVROXWLRQ
$l2DQG6L2GRQRWGLVVROYHLQZDWHUKHQFHWKHVROXWLRQUHPDLQVQHXWUDO
32 %RWK R[LGHV RI SKRVSKRURXV UHDFW YLJRURXVO\ 32 V +2 +32 DT 
ZLWKZDWHUWRIRUPDFLGLFVROXWLRQV
32 32 V +2 +32 DT 

+32LVDZHDNGLEDVLFDFLG2QO\RIWKH
+DWRPVDUHDFLGLF

62 62GLVVROYHVUHDGLO\LQZDWHUWRIRUPDQDFLGLF 62 J +2 +62 DT 


VROXWLRQRIVXOIXURXVDFLG
62 62 J +2 +62 DT 
62 UHDFWV H[RWKHUPLFDOO\ ZLWK ZDWHU WR IRUP
VXOIXULF DFLG ZKLFK LV D VWURQJO\ DFLGLF
VROXWLRQ


  :KDVWKHKLJKHVWPHOWLQJSRLQWDPRQJDOOWKH3HULRGHOHPHQWV+HQFH:LV6L
 7KHFKORULGHRI;LVQHXWUDO+HQFH;LV1D
 7KHR[LGHRI=LVDPSKRWHULF+HQFH=LV$l.
 7KHFKORULGHRI<LVDFLGLFDQGWKHR[LGHRI<LVEDVLF+HQFH<LV0J

2SWLRQ$LVFRUUHFW7KHDWRPLFUDGLXVLQFUHDVHVLQWKHIROORZLQJRUGHU6L$l0J1D
ZKLFKLV:=<;
2SWLRQ%LVZURQJ7KHILUVWLRQLVDWLRQHQHUJ\VKRXOGGHFUHDVHLQWKHIROORZLQJRUGHU6L!
0J!$l!1DZKLFKLV:!<!=!; )URPWKHGDWDERRNOHW 
2SWLRQ&LVZURQJ7KHS+RIWKHFKORULGHVVKRXOGGHFUHDVHLQWKHIROORZLQJRUGHU1D!
0J!$l!6LZKLFKLV;!<!=!:
2SWLRQ'LVZURQJ7KHHOHFWULFDOFRQGXFWLYLW\VKRXOGLQFUHDVHLQWKHIROORZLQJRUGHU6L
1D0J$lZKLFKLV:;<=)URP1DWR$lQXPEHURIGHORFDOLVHGYDOHQFHHOHFWURQV
LQFUHDVHV IURP  WR  WKHUHIRUH WKHUH LV DQ LQFUHDVH LQ HOHFWULFDO FRQGXFWLYLW\ RI WKHVH
PHWDOV 6L LV D PHWDOORLG semimetal  WKXV LW LV QRW D JRRG FRQGXFWRU LW LV D VHPL

trendyline
ZFRQGXFWLYLW\ 
FRQGXFWRUZLWKORZFRQGXFWLYLW\ 


1244 trendyline


 5HDFWLRQVRI*URXS,,HOHPHQWVZLWKZDWHU

 (RUHG9 

0 DT H0 5HDFWLRQZLWKZDWHU


V 

5HDFWVVORZO\ZLWKFROGZDWHUEXWUHDGLO\ZLWKVWHDPWR
IRUPK\GURJHQDQGK\GUR[LGHV
0J V +2 J 0J2 V + J 
Mg  UHDFWVYLJRURXVO\ZLWKVWHDP

0J V +2 0J 2+  V + J 

UHDFWVVORZO\ZLWKFROGZDWHU



 &KORULQHLVPRUHUHDFWLYHWKDQEURPLQH+HQFHWKHDTFKORULQHZRXOGGLVSODFHWKHEURPLGH
LRQ WR IRUP EURPLQH DQG FKORULGH LRQ %URPLQH LV PRUH VROXEOH LQ RUJDQLF VROYHQW WKDQ
ZDWHU VLQFH WKH HQHUJ\ UHOHDVHG IURP WKH YDQ GHU :DDOV IRUFHV EHWZHHQ EURPLQH DQG
WHWUDFKORURPHWKDQH LV PRUH WKDQ WKDW UHOHDVHG IURP WKH YDQ GHU :DDOV IRUFHV EHWZHHQ
EURPLQHDQGZDWHU EURPLQHZRXOGGLVVROYHLQWKHRUJDQLFVROYHQWWRIRUPDUHGGLVKEURZQ
OD\HU7KHFKORULGHLRQZLOOEHLQWKHDTXHRXVOD\HU7KHUHZLOOEHWZRLPPLVFLEOHOD\HUV

 6LQFHYLVLEOHOLJKWLQWKHQPUHJLRQLVWKHOHDVWDEVRUEHGWKLVPHDQVWKDWLWLVUHIOHFWHGDQGVHHQDV

WKHFRORXURIWKHFRPSOH[ZKLFKLVJUHHQ

PRORI$J&lDUHIRUPHGZKLFKPHDQVWKDWWKHUHDUHPRORIIUHH&l7KLVVXJJHVWVWKDWQRQHRIWKH&l
 DUHGDWLYHERQGHGWR&U


 

 6LQFH &RPSRXQG ; FDQ EH R[LGLVHG LW PXVW KDYH SULPDU\ DOFRKRO RU VHFRQGDU\ DOFRKRO RU
DOGHK\GH 1D%+ RQO\ UHGXFHV FDUERQ\O FRPSRXQG +HQFH ; PXVW FRQWDLQ DOGHK\GH RU
NHWRQH

2SWLRQ$LVZURQJHVWHUDQGSULPDU\DOFRKRO QRFDUERQ\OFRPSRXQGSUHVHQW 

trendyline
2SWLRQ%LVZURQJVHFRQGDU\DOFRKRODQGFDUER[\OLFDFLG QRFDUERQ\OFRPSRXQGSUHVHQW 
JVHFRQGDU\DOFRKRODQGFDUER[\OLFDFLG QRFDUERQ\OFRPSRX
JVHFRQGDU\DOFRKRODQGFDUER[\OLFDFLG QRFD \ S

JHVWHUDQGSULPDU\DOFRKRO QRFDUERQ\OFRPSRXQGSUHVHQW
HVWHUDQGSULPDU\DOFRKRO QRFDUERQ\OFRPSRXQGSUHVHQW
2SWLRQ&LVZURQJHVWHUDQGSULPDU\DOFRKRO QRFDUERQ\OFRPSRXQGSUHVHQW 

W L O K O G O K O G OG K G W
2SWLRQ'LVFRUUHFWSULPDU\DOFRKROVHFRQGDU\DOFRKRODQGDOGHK\GHSUHVHQW


1245 trendyline


 2SWLRQ$LVZURQJDV)H&lPXVWEHDQK\GURXVDVLWXQGHUJRHVK\GURO\VLVLQWKHSUHVHQFH
RIZDWHU+HQFH)H&lLVGHVWUR\HGDQGFDQQRWDFWDVFDWDO\VW

2SWLRQ%LVFRUUHFWDV+%U J LVSURGXFHGZKLFKUHDFWVZLWK2+ QXFOHRSKLOLFVXEVWLWXWLRQ 


DQG& & HOHFWURSKLOLFDGGLWLRQ 

2SWLRQ&LVZURQJDV/L$l+GRHVQRWUHGXFHDONHQHV

2SWLRQ'LVZURQJDVWKHFRQGLWLRQVJLYHQDUHPHDQWIRUQXFOHRSKLOLFDGGLWLRQRIFDUERQ\O
FRPSRXQGVQRWQXFOHRSKLOLFVXEVWLWXWLRQRIFKORURDONDQHV

 2SWLRQ$LVZURQJ2QO\PROHVRI1DLVUHTXLUHGWRUHDFWZLWK2+

2SWLRQ%LVFRUUHFWPROHVRI+%UUHDFWVZLWKPROHVRI& &DQGPROHVRI2+

2SWLRQ&LVZURQJ7KHUHLVRQO\RQHHVWHUWKDWFDQXQGHUJREDVLFK\GURO\VLV

2SWLRQ'LVZURQJ7KHUHDUHRQO\WZRFDUERQ\OJURXSV NHWRQHV SUHVHQW

 2QO\&+&+&2&l FDQUHDFWZLWKZDWHUWRSURGXFH&+&+&22+DQG+&l&lUHDFWVZLWK


$JWRIRUP$J&l


 2SWLRQ$&lLVQRWDJRRGGLVWLQJXLVKLQJWHVWIRUDONDQHVDQGDONHQHV

2SWLRQ%<HOORZSUHFLSLWDWHLVVHHQIRU&+&2&+,EXWQRWIRU&+&+&+2

2SWLRQ & 7R GLVWLQJXLVK KDORJHQRDONDQHV WKH XQNQRZQV PXVW ILUVW EH KHDWHG ZLWK
1D2+ DT IROORZHGE\FRROLQJDQGDFLGLILFDWLRQZLWK+12 DT /DVWO\$J12 DT PXVWEH
DGGHGIRUWKHVLOYHUKDOLGHSSWWREHIRUPHG

2SWLRQ'7ROOHQVUHJHQWZLOOR[LGLVHERWKFRPSRXQGVDQGEHUHGXFHGWR$JDQGVLOYHU
PLUURULVVHHQ

trendyline
1246 trendyline



2SWLRQ$&RPSRXQG3LV DQGGRHVQRWUHDFWZLWK'13+

2SWLRQ % &RPSRXQG 4 KDV D SULPDU\ DOFRKRO IURP UHGXFWLRQ RI &22+ DQG FDQ UHDFW
ZLWK3&l

2SWLRQ&6WHSLVHOLPLQDWLRQ (OLPLQDWLRQRIDOFRKROWRIRUPDONHQH 

2SWLRQ'$QDFLGFKORULGHLVUHTXLUHGWRIRUPDQDPLGH


2QO\ SRODULVHG FDQ SURYLGH WKH HOHFWURSKLOH WR DWWDFN & & LQ WKH ILUVW VWHS WR
IRUPWKHLQWHUPHGLDWH(OHFWURSKLOH%UZLOOEHDGGHGWR&ZLWKPRUH+2WKHUQHJDWLYHO\
FKDUJHGVSHFLHVDQGZDWHUZLOOEHDWWUDFWHGWRFDUERFDWLRQLQWKHQH[WVWHS

(OHFWURSKLOLF$GGLWLRQ

 

 

trendyline
1247 trendyline


 2[LGDWLRQRIDOFRKROZLWK.0Q2ZLOO\LHOGHLWKHUFDUER[\OLFDFLG LILWVSULPDU\DOFRKRO RU


NHWRQH LILWVVHFRQGDU\DOFRKRO 6LQFHWKHFRPSRXQGRQO\KDVR[\JHQDWRPLWVKRXOG
EHDNHWRQH7KHVWUXFWXUDOIRUPXODRIWKHFRPSRXQGLV

7KXVVWUXFWXUDOIRUPXODRIDOFRKRO:LV

3URSDQRO DVVHFRQGDU\DOFRKROXQGHUJRHVR[LGDWLRQWRJLYHNHWRQH

2SWLRQ$(OLPLQDWLRQUHDFWLRQWR\LHOGDONHQH :URQJ 

2SWLRQ%5HGXFWLRQRIDFLGWR\LHOGSULPDU\DOFRKRO :URQJ 

2SWLRQ&1XFOHRSKLOLFVXEVWLWXWLRQWR\LHOGSURSDQRODVHFRQGDU\DOFRKRO &RUUHFW 

2SWLRQ'(OHFWURSKLOLFDGGLWLRQZRXOGUHTXLUHFRQF+62DW&IROORZHGE\ERLOLQJZLWK
+2 :URQJ 


 

3KHQRODQGFDUER[\OLFDFLGJURXSVZLOOXQGHUJRQHXWUDOLVDWLRQ

DPLGHJURXSVZLOOXQGHUJRDONDOLQHK\GURO\VLV

7KXVPROHVRI1D2+DUHUHTXLUHG

trendyline


1248 trendyline




2SWLRQ$)DOVH7KHUHDUHVWHUHRLVRPHUV 

2SWLRQ%(OLPLQDWLRQRI+2SURGXFHVPRUH& &7RWDORI& &

2SWLRQ&%RWKDOFRKROFDQEHR[LGLVHG

2SWLRQ'2+JURXSVUHDFWVZLWK1DWRIRUPPROHRI+

 2SWLRQ$LVFRUUHFW

$OWKRXJK2SWLRQ'JLYHVLVRPHUVRIWKHPKDYHFKLUDOFDUERQV

trendyline


1249 trendyline


 6WUXFWXUDOIRUPXODRIHWK\OFDUEDPDWH

)URPWKHSDWWHUQ

5 +

5 &+&+

7KXVWKHDF\OD]LGHDQGDOFRKROXVHGDUH

 2SWLRQLLVFRUUHFWDVWKHUHLVDORQHHOHFWURQRQ1
2SWLRQLLLVZURQJDV1D+LVDUHGXFLQJDJHQWMXVWOLNH1D%+DQG/L$l+
2SWLRQLLLLVZURQJDVWKHR[LGDWLRQQXPEHURI&UUHPDLQVDW,WLVDQDFLGEDVHUHDFWLRQ
LQVWHDG

 2SWLRQ  LV ZURQJ DV $l%U DQG 32 DUH ERWK VLPSOH FRYDOHQW VWUXFWXUHV 6L LV JLDQW
FRYDOHQWVWUXFWXUH7KHUHLVQRJLDQWLRQLFVWUXFWXUH
2SWLRQ  LV ZURQJ DV $l2 KDV JLDQW LRQLF VWUXFWXUH %) DQG 6L&l DUH ERWK VLPSOH
FRYDOHQWPROHFXOHV7KHUHLVQRJLDQWFRYDOHQWVWUXFWXUH


=Q V +%U DT =Q%U DT + J 

 HIIRU+%U DT  HIIRU=Q%U DT 

=Q V %U l + J 

1RWHHIIRUHOHPHQWVLQVWDQGDUGVWDWHV HJ=Q V  + J LV]HUR7KXVLWLVQRWQHHGHG

1250 trendyline


 

&l&U&l&UEFHOO      9 VSRQWDQHRXV 

&l&U&l&UEFHOO      9 VSRQWDQHRXV 

%U&U%U&UEFHOO      9 VSRQWDQHRXV 

%U&U%U&UEFHOO      9 VSRQWDQHRXV 

,&U,&UEFHOO      9 VSRQWDQHRXV 

,&U,&UEFHOO      9 VSRQWDQHRXV 

7KHUHIRUH&U&l&U%UDQG&U,ZLOOEHWKHILQDOSURGXFWVREWDLQHG

 2SWLRQLVFRUUHFW/LJDQGH[FKDQJHZLWKWKHKH[DGHQGDWHHGWDLRQUHVXOWVLQDQLQFUHDVHLQQXPEHURI
VSHFLHVIURPWRDVVKRZQEHORZ

>&X 1+  +2 @HGWD >&X HGWD @1++2

2SWLRQ  LV ZURQJ DV WKH G RUELWDOV LQ VSHFLHV ' >&X&l@ DUH QRQGHJHQHUDWH VLQFH WKHUH DUH OLJDQGV
ERQGHGWR&XZKLFKVSOLWWKHGRUELWDOVLQWRGLIIHUHQWHQHUJ\OHYHOV

2SWLRQLVZURQJDVVWHSV9DQG9,LQYROYHUHGR[UHDFWLRQ&RORXUFKDQJHLVGXHWR&XEHLQJUHGXFHG
WR&X

 2SWLRQ  &RQGHQVDWLRQ WDNHV SODFH EHWZHHQ 1+2+ DQG & 2 LQ /DZVRQH

trendyline
d


2SWLRQV 7ROOHQVUHDJHQWDQGFRQF+12
OOHQVUHDJHQWDQGFRQF+12
HQVUHDJHQWDQGFRQF+
+ GRQRWUHDFWZLWK/DZVRQH7K
GRQRWUHDFWZLWK/DZVRQH7KHUHDUHQR
WRUHDFWZLWK7ROOHQVUHDJHQW(OHFWURSKLOLFV
RUHDFWZLWK7ROOHQVUHDJHQW(OHFWURSKLOLFVXEVWLWXWLRQRIEHQ
E WLW WL IE
DOGHK\GHJURXSVWRUHDFWZLWK7ROOHQVUHDJHQW(OHFWURSKLOLFVXEVWLWXWLRQRIEHQ]HQHULQJ
UHTXLUHVFRQF+12ZLWKFRQF+62FDWDO\VWDW&


1251 trendyline


 2SWLRQ  LV WUXH ,Q SUHVHQFH RI 1D2+ 1+ LV EHKDYLQJ DV D %URQVWHG DFLG DV LW KDV
GRQDWHGD++HQFH1D2+LVDFWLQJDVDEDVH

2SWLRQLVWUXHDV%UFDQEHSRODULVHGLQWRSURGXFH 

2SWLRQ  LV ZURQJ DV R[LGDWLRQ VWDWH RI %U LV  5HDVRQ $V 1 LV PRUH HOHFWURQHJDWLYH
WKDQ %U 1ZLOO GUDZ WKH HOHFWURQVLQ WKHFRYDOHQW ERQG EHWZHHQ 1 DQG %U WRZDUGV LWVHOI
KHQFH%UKDVHOHFWURQVDURXQGLW6LQFH%ULVVXSSRVHGWRKDYHYDOHQFHHOHFWURQVEXWLW
QRZKDVRQO\WKHR[LGDWLRQVWDWHRI%ULV

 2SWLRQ+\GURO\VLVWRRNSODFHLQVWHSWRFOHDYHWKHHVWHUERQG

2SWLRQ1XFOHRSKLOLFVXEVWLWXWLRQRFFXUVLQVWHSV,,WR,9

2SWLRQ1RHOHFWURSKLOLFVXEVWLWXWLRQWRRNSODFH

 2SWLRQ  7UXH 'LVXOILGH ERQGV DUH IRUPHG YLD R[LGDWLRQ $GGLQJ D UHGXFLQJ DJHQW ZLOO
UHYHUVHWKHUHDFWLRQGLVUXSWLQJWKHGLVXOILGHERQGV

2SWLRQ7UXH

2SWLRQ  7UXH DV JOXWDPLF DFLG FDQ UHDFW ZLWK 1D2+ WR IRUP FDUER[\ODWH LRQV +HQFH
K\GURJHQERQGLQJFDQEHGLVUXSWHG

 2SWLRQLVFRUUHFW$PLQH$FLGFKORULGH $PLGH


2SWLRQLVZURQJ7HUWLDU\DPLQHVFDQQRWXQGHUJRFRQGHQVDWLRQZLWKDFLGFKORULGHVWR
IRUPDPLGHVDVLWGRHVQRWKDYHD+DWRPWREHUHPRYHG+HQFHQRDPLGHZLOOEHIRUPHG
2SWLRQLVZURQJ$PLQHFDUER[\OLFDFLG 6DOW7KLVLVDQHXWUDOLVDWLRQUHDFWLRQVR
DPLGHZLOOQRWEHSURGXFHG


(1'2)3$3(5

trendyline
e
1252 trendyline
1

Name: Class:

ST ANDREWS JUNIOR COLLEGE

JC 2 Preliminary Exam
Chemistry 9647/02
Higher 2 31August2016
Paper 2 2 hours

Candidates answer in the spaces provided on the question paper.


Additional Materials: Data Booklet
READ THESE INSTRUCTIONS FIRST
Write in dark blue or black pen.
You may use a soft pencil for any diagrams, graphs or rough working.
Do not use staples, paper clips, highlighters, glue or correction fluid.
Answer all questions.
You are reminded of the need for good English and clear presentation in your answers.
The number of marks is given in brackets [ ] at the end of each question or part question.
For Examiners Use:
Question Marks
1
2
3
4
5

t endy
tre
trendyline
dy
6
7
Total (72 marks)
ks)
s)

This document consists of27printed pages including this page.

[Turn Over
1253 trendyline
2

BLANK PAGE

trendyline
[Turn Over
1254 trendyline
3

1 A student found a bottle of solid benzoic acid in the school laboratory. She was tasked to
determine the standard enthalpy change of neutralisation of benzoic acid. The standard
enthalpy change of neutralisation is when one mole of water is formed between an acid and
a base.
H+ (aq) + OH (aq) H 2 O (l)

You may assume you are provided with the following:


FA 1, aqueous sodium hydroxide, NaOH
Solid benzoic acid
Polystyrene (styrofoam) cups
Apparatus normally found in a school laboratory

Before carrying out the experiment, an aqueous solution of 2.00 mol dm-3 benzoic acid
(FA 2) is first prepared. Subsequently, in separate experiments, different volumes of FA 2
and FA 1 are mixed while keeping the total volume of the reaction mixture constant. In each
H[SHULPHQWWKHWHPSHUDWXUHULVH7LVWREHGHWHUPLQHG$JUDSKRI7DJDLQst volume of
FA 1 used is then plotted.

Data from the graph can then be used to determine:


the 7 max ,
the concentration of FA 1,
the enthalpy change of neutralisation between FA 1 and FA 2.

It is given that 4.18 J is required to raise the temperature of 1 cm3 of any solution by
1 oC.

(a) Write an equation to represent the standard enthalpy change of neutralisation


between sodium hydroxide and benzoic acid.

trendyline
.

[1]

[Turn Over
1255 trendyline
4

1 (b) (i) Write a detailed plan to prepare 2.00 mol dm-3 solution of FA 2.

..

..

..

..

..

..

.....

..

..

..

..

[3]

(ii) It is predicted that the maximum temperature change for the neutralisation
would occur when the volume of FA 1 mixed is between 25 cm3 and 30 cm3.The
total volume of any mixture should be kept constant at 50 cm3.

Write a detailed plan on how you could determine the temperature changes for
the series of reactions between FA 1 and FA 2.
Your plan should include details of:
all essential experimental details,
appropriate volumes of solutions to be used,

trendyline
a tabulation of the experimental data to be collec
collected (including volumes
used)
sed),,
used),
ow these measurements can be used to obtain the temperature change,
how
T
T.

[Turn Over
1256 trendyline
5

1 (b) (ii)
..

..

..

..

..

..

.....

..

..

..

..

..

trendyline [5]

[Turn Over
1257 trendyline
6

1 (b) (iii) The following plots were plotted on a grid after another similar experiment was
conducted. Draw suitable graphs through the plotted points.
[1]

trendyline
[Turn Over
1258 trendyline
7

1 (c) By using the graph in (b)(iii), calculate


the concentration of FA 1, given that the total volume of the mixture is 50 cm3.
the enthalpy change of neutralisation for the reaction between FA 1 and FA 2.

[2]
[Total: 12]

trendyline
[Turn Over
1259 trendyline
8

2 (a) Chromium carbonyl, alsoknown as chromium hexacarbonyl, is a chemical compound with


the formula Cr(CO) 6 . Cr(CO) 6 is zerocovalent, meaning that Cr has an oxidation state of
0. Draw the structure of the complex and state the shape and bond angle around the Cr
atom in the complex.

[2]
(b) Chromium hexacarbonyl undergoes the following ligand replacement reaction.

Cr(CO) 6 + PR 3 Cr(CO) 5 PR 3 + CO

Two separate experiments were carried out to study the rate of this reaction.
In the first experiment, the ligand PR 3 was in a large excess and [Cr(CO) 6 ] was
measured against time. The results are shown on the graph below.

y
[Turn Over
1260 trendyline
9

2 (b) In the second experiment, Cr(CO) 6 was in a large excess, and [PR 3 ] was measured
against time. The following results were obtained.

time / s [PR 3 ] / moldm3

0 0.0100

120 0.0076

200 0.0060

360 0.0028

(i) On the graph, plot the data given in the table.


[1]
(ii) Use the graphs to determine the order of reaction with respect to Cr(CO) 6 and
PR 3 .In each case explain how you arrived at your answer.

Cr(CO) 6

PR 3

[2]
(iii) Write the rate equation for the reaction, and calculate a value for the rate constant,
stating the units of the rate constant.

trendyline [3]

[Turn Over
1261 trendyline
10

2 (b) (iv) Two possible mechanisms for this reaction are given below. State themechanism
which is consistent with the rate equation you have written in (b)(iii) and explain
your answer.

Mechanism A Cr(CO) 6 Cr(CO) 5 + CO slow


Cr(CO) 5 + PR 3 Cr(CO) 5 PR 3 fast

Mechanism B Cr(CO) 6 + PR 3 [OC- - -Cr(CO) 4 - - -PR 3 ] Cr(CO) 5 PR 3 +


CO
(transition state)

[2]
[Total: 10]

[Turn Over
1262 trendyline
11

3 This question refers to compounds of calcium and barium.

(a) Oil paints contain traces of red Ca2+ and green Ba2+ions which give them their
characteristic red and green colours respectively.Oil painting is typically done on canvas.
During the manufacturing process of canvas, hydrogen orthoperiodateions, HIO 6 4, is left
on the canvas.

Orthoperiodate salts are formed when orthoperiodate ions, IO 6 5-,reacts with the Ca2+ and
Ba2+ions in the paint, causing the decolourisation of the red and green colour of the paint.
The equations below show the formation of IO 6 5- and its salts.

HIO 6 4 (aq) H+ (aq) + IO 6 5 (aq) K a = 4.04 x 10-6mol dm-3

5 M2+ (aq) + 2 IO 6 5 (aq) M 5 (IO 6 ) 2 (s) where M2+isCa2+ or Ba2+

Relevant K sp values are given in the table below :

Salt K sp
Ca 5 (IO 6 ) 2 4.0 x 10-9
Ba 5 (IO 6 ) 2 1.6 x 10-15

The painting was stored in a display cabinet at a low pH environment.Over the weekend,
the electrical supply of the display cabinet was disrupted which resulted in an increase of
the environments pH. It was found that the green portions of the painting were
decolourised due to the formation of solid barium orthoperiodate, Ba 5 (IO 6 ) 2 .

trendyline
[Turn Over
1263 trendyline
12

3 (a) Explain qualitatively why the green colouredareas of the painting were decolourised but
not the red coloured areas when the pH increased.

..

..

..

..

..

..

..

..

..

..

[3]

trendyline
[Turn Over
1264 trendyline
13

3 (b) G, H and S are related by the following equation.


G = H - TS
The Ellingham diagram below shows how G changes between0K and 1800Kfor the
following reaction.
CaO (s) + H 2 O (l) Ca(OH) 2 (s)

For the range of temperatures in the graph above, it can be assumed that the enthalpy
change and the entropy change of the reaction remain approximately constant.

(i) Predict the sign of S for the reaction, showing your reasoning.

trendyline
.

[2]

[Turn Over
1265 trendyline
14

3 (b) (ii) Using the graph, calculate S.

[1]
(iii) Using the graph, determine H.

.
[1]

(iv) Determine the temperature above which the reaction become non-spontaneous.

.
[1]
[Total: 8]

trendyline
[Turn Over
1266 trendyline
15

4 The table below gives data about some physical properties of the elements calcium, copper
and chromium.

Physical Property Calcium Copper Chromium

Relative atomic mass 40.1 63.5 52.0

Atomic radius / nm 0.197 0.128 0.117

Density/ gcm-3 1.54 8.92 7.20

First ionisation energy/ kJ mol-1 590 745 653

(a) (i) Explain why the first ionisation energy of copper and chromium are both higher
than that of calcium.

[2]

(ii) Use relevant data from the table to explain qualitatively why the densities of copper
and chromium are significantly greater than that of calcium.

trendyline


.

[1]

[Turn Over
1267 trendyline
16

4 (b) Chromium (III) nitrate is more acidic than chromium (II) nitrate. Explain with the aid of a
relevant equation.

..

..

..

[2]

(c) When a particular copper ore was reduced, an alloy was produced which was composed
mainly of copper, but with gold and chromium impurities. It contained no other metal. In
order to purify it, the alloy was made the anode of an electrolytic cell, with a pure copper
cathode and aqueous copper (II) sulfate as the electrolyte.

Explain, with reference to E values, what happens to the gold and chromium impurities
during this purification process. The electrode reaction for the standard Au3+/Au half cell
is Au3+ (aq) + 3e Au (s) E= +1.50 V

..

..

..

..

..

..

..

..

trendyline
..

..

[3]

[Turn Over
1268 trendyline
17

4 (d) A current of 1.8 A was passed through the cell described in (c) for17 minutes, and the
electrodes removed, washed, dried and weighed. It was found that some mass was lost
from the anode.

After filtering it off and drying it, the deposit below the anode weighed 0.085g. On adding
an excess of hydroxide ions to the electrolyte, a grey-green precipitate was formed. Its
mass was 0.304g. (You may assume that negligible amount of copper ions remained in
the electrolyte.)

(i) Calculate the expected increase in mass of the cathode.

[2]

(ii) Identify the grey-green precipitate.

grey-green precipitate: .

[1]

(iii) It was found that the mass of copper removed from the alloy was 0.466g. Calculate
the masses of gold and chromium removed from the alloy. Hence calculate the total
mass lost by the anode.

trendyline [Total: 14]


[3]

[Turn Over
1269 trendyline
18

5 (a) A student conducted an experiment to identify the trend in thermal stability of hydrogen
halides. The reaction occurred according to the following equation:

2HX(g) H 2 (g) + X 2 (g)

The approximate K c values for the above equilibrium at 500oC is shown in the table
below.
Kc
Temperature /oC
HCl HBr HI
500 1013 109 105

(i) Using the Data Booklet, explain the trend in thermal stability of hydrogen halides
down the group.

[2]

(ii) Hence, deduce if thetrend in (a)(i) agrees with the K c data given above.

trendyline


.

[1]

[Turn Over
1270 trendyline
19

5 (b) A student conducted another experiment to distinguish between samples of sodium


chlorideand phosphoruspentachloride. It was found that red onion water can be used as
an indicator. The table below shows the colour of the indicator at various pH.

pH Colour

Less than 7 Red


7 Violet

(i) To each unknown sample of chlorides, he added a few drops of red onion water.
He recorded the colour of the indicator. Predict the identities of the chlorides.

Colour of red onion water Identity of chloride

Red

Violet

[1]

(ii) With the aid of an equation, describe the action of water on


phosphoruspentachloride.

[2]
[Total: 6]

trendyline
e
[Turn Over
1271 trendyline
20

6 Paracetamol is an effective analgesic discovered by Joseph von Mering in 1893. Its structure
is as shown.

HO NH

(a) Explain why all the carbon-carbon bonds in benzene are of the same length.

.......

...

[1]

(b) (i) Paracetamol may be synthesised from phenol as shown.

Step 1 Step 2 Step 3 HO NH


HO A B

State the reagents and conditions for steps 1 to 3. Draw the structure of the
intermediate product B.

trendyline
[Turn Over
1272 trendyline
21

Reagents and conditions:

Step 1:
....

Step 2:
....

Step 3:

Structure of B

....

[4]
6 (b) (ii) Based on Step 3 of the given reaction scheme, draw the structure of a possible
side-product.

[1]

trendyline
[Turn Over
1273 trendyline
22

(iii) Phenylephrine is a decongestant which may be taken with paracetamol when a


person is suffering from cold. The structure of phenylephrine is as shown.
OH
H
HO N

State and explain how the basicity of phenylephrinemight compare with that of
paracetamol.

[2]

6 (b) (iv) Suggest a simple chemical test to distinguish between paracetamol and
compound A.

CONH2

HO

Compound A

trendyline
[Turn Over
1274 trendyline
23

[2]

trendyline
[Turn Over
1275 trendyline
24

6 (c) (i) Hoechst and Celanese discovered a simpler synthetic route for paracetamol as
shown.
O O
OH OH OH OH
O CF3COOH,
SOCl 2,
HF catalyst NH2OH Amberlyst 15

Step 1 Step 2 N Step 3 HN O


O
OH

State the types of reaction for steps 1 and 2.

Step 1:
....

Step 2:
....

[2]

(ii) Step 3is known as the Beckmann arrangement, where an oxime is converted into
an amide.

N OH R
R O
HN
R R
Oxime Amide

N
Draw the structure of the product formed when OH undergoes the
Beckmann arrangement.

trendyline [1]

[Turn Over
1276 trendyline
25

[Total: 13]

trendyline
[Turn Over
1277 trendyline
26

7 Coffee beans contain chlorogenic acid, which is an antioxidant and an important biosynthetic
intermediate. The structure of chlorogenic acid is as shown.

OH

OH

O O
OH
O

OH
HO HO

(a) State the number of moles of hot NaOH (aq) that will react with 1 mole of chlorogenic
acid.

.....

[1]

trendyline
[Turn Over
1278 trendyline
27

7 (b) On heating with dilute acid, chlorogenic acid produces two compounds, X and Y.

HO O OH
OH
O
OH
OH
HO HO
OH
Compound X Compound Y

When compound Y reacts with hot excess concentrated sulfuric acid, a product with a
molecular formula of C 7 H 6 O 3 is formed. A student claims that the product formed has the
following structural formula but the teacher disagrees.

OH
HO

Draw the correct structure of the product and explain what is wrong with the students
answer.

Structure of product

.....

[2]

[Turn Over
1279 trendyline
28

7 (c) HO O

OH
OH

Compound Z

(i) Compound Z reacts with Br 2 (l) at room temperature and in the dark. Describe the
mechanism showing curly arrows, charges, dipoles and any relevant lone pairs.

trendyline [3]

[Turn Over
1280 trendyline
29

7 (c) (ii) The product of(c)(i) exists as a mixture of 4 stereoisomers. State the type of
isomerism exhibited by the product and draw all the stereoisomers.

...

[3]
[Total: 9]

trendyline End of Paper


ape

[Turn Over
1281 trendyline



1DPH  &ODVV 


67$1'5(:6-81,25&2//(*(


-&3UHOLPLQDU\([DP 787256&23< 
&KHPLVWU\ 
+LJKHU $XJXVW
3DSHU KRXUV

&DQGLGDWHVDQVZHULQWKHVSDFHVSURYLGHGRQWKHTXHVWLRQSDSHU
$GGLWLRQDO0DWHULDOV'DWD%RRNOHW
5($'7+(6(,16758&7,216),567
:ULWHLQGDUNEOXHRUEODFNSHQ
<RXPD\XVHDVRIWSHQFLOIRUDQ\GLDJUDPVJUDSKVRUURXJKZRUNLQJ
'RQRWXVHVWDSOHVSDSHUFOLSVKLJKOLJKWHUVJOXHRUFRUUHFWLRQIOXLG
$QVZHUDOOTXHVWLRQV
<RXDUHUHPLQGHGRIWKHQHHGIRUJRRG(QJOLVKDQGFOHDUSUHVHQWDWLRQLQ\RXUDQVZHUV
7KHQXPEHURIPDUNVLVJLYHQLQEUDFNHWV>@DWWKHHQGRIHDFKTXHVWLRQRUSDUWTXHVWLRQ
)RU([DPLQHUV8VH
4XHVWLRQ 0DUNV
 
 
 
 
 
 
 

t endy
tre
trendyline
dy
7RWDO PDUNV 
NV 
V 



FXPHQWFRQVLVWVR 
FXPHQWFRQVLVWVRI SULQWHGSDJHV LQFOXGLQJWKLVSDJH DQG
LQWHGSDJHV LQFOXGLQJWKLVSDJH DQG
EODQ
7KLVGRFXPHQWFRQVLVWVRISULQWHGSDJHV LQFOXGLQJWKLVSDJH DQGEODQNSDJH

>7XUQ2YHU

1282 trendyline


 $ VWXGHQW IRXQG D ERWWOH RI VROLG EHQ]RLF DFLG LQ WKH VFKRRO ODERUDWRU\ 6KH ZDV WDVNHG WR
GHWHUPLQH WKH VWDQGDUG HQWKDOS\ FKDQJH RI QHXWUDOLVDWLRQ RI EHQ]RLF DFLG 7KH VWDQGDUG
HQWKDOS\FKDQJHRIQHXWUDOLVDWLRQLVZKHQRQHPROHRIZDWHULVIRUPHGEHWZHHQDQDFLGDQG
DEDVH
+ DT 2+ DT +2 l 

<RXPD\DVVXPH\RXDUHSURYLGHGZLWKWKHIROORZLQJ
 )$DTXHRXVVRGLXPK\GUR[LGH1D2+
 6ROLGEHQ]RLFDFLG
 3RO\VW\UHQH VW\URIRDP FXSV
 $SSDUDWXVQRUPDOO\IRXQGLQDVFKRROODERUDWRU\

%HIRUH FDUU\LQJ RXW WKH H[SHULPHQW DQ DTXHRXV VROXWLRQ RI  PRO GP EHQ]RLF DFLG
)$  LV ILUVWSUHSDUHG6XEVHTXHQWO\LQVHSDUDWHH[SHULPHQWVGLIIHUHQWYROXPHV RI)$ 
DQG)$DUHPL[HGZKLOHNHHSLQJWKHWRWDOYROXPHRIWKHUHDFWLRQPL[WXUHFRQVWDQW,QHDFK
H[SHULPHQWWKHWHPSHUDWXUHULVH7LVWREHGHWHUPLQHG$JUDSKRI7DJDLQVWYROXPHRI
)$XVHGLVWKHQSORWWHG

'DWDIURPWKHJUDSKFDQWKHQEHXVHGWRGHWHUPLQH
 WKH7PD[
 WKHFRQFHQWUDWLRQRI)$
 WKHHQWKDOS\FKDQJHRIQHXWUDOLVDWLRQEHWZHHQ)$DQG)$

,W LV JLYHQ WKDW  - LV UHTXLUHG WR UDLVH WKH WHPSHUDWXUH RI  FP RI DQ\ VROXWLRQ E\
R&
 D  :ULWHDQHTXDWLRQWRUHSUHVHQWWKHVWDQGDUGHQWKDOS\FKDQJHRIQHXWUDOLVDWLRQEHWZHHQ
VRGLXPK\GUR[LGHDQGEHQ]RLFDFLG
>@
 
  &+&22+ DT 1D2+ DT &+&22 1D DT +2 l 

 E  L  :ULWHDGHWDLOHGSODQWRSUHSDUHPROGPVROXWLRQRI)$

t nd li
trendyline
>@
  RSUHSDUHFPRIPROGPEHQ]RLFDFLG
7RSUHSDUHFP EHQ]RLFDFLG )$ 
EHQ]RLFDFLG )$
)$
0RODUPDVVRI& JPR 
RODUPDVVRI&+2 JPRO

>7XUQ2YHU

1283 trendyline


 $PRXQWSUHVHQWLQFPRIPROGPVROXWLRQ J
  8VH D ZHLJKLQJ EDODQFH WR PHDVXUH DFFXUDWHO\  J PLQLPXP  J RU
PD[LPXP  J  RI VROLG &+2LQ D FOHDQ DQG GU\   ZHLJKLQJ ERWWOH  5HFRUG WKH
PDVVRIWKHVROLGDQGWKHZHLJKLQJERWWOH
 'LVVROYHWKHVROLG LQDFPFPEHDNHUZLWKFPRIZDWHU
  7UDQVIHU WKH VROXWLRQ DQG ZDVKLQJV   LQWR D  FP JUDGXDWHG IODVN YROXPHWULF
IODVNDQG PDNHXSWRWKHPDUNGURSZLVHZLWKGHLRQL]HGGLVWLOOHGZDWHU
 6KDNHWKHVROXWLRQWRREWDLQDKRPRJHQHRXVVROXWLRQDQGODEHOLW)$
 5HZHLJKWKHHPSWLHGZHLJKLQJERWWOH$OWHUQDWLYHZDVKWKHZHLJKLQJERWWOHLQWKH
SURFHGXUH

LIVWXGHQWFDOFXODWHJEXWXVHDQDOWHUQDWLYHYDOXHLJQRUH

   LL  ,WLVSUHGLFWHGWKDWWKHPD[LPXPWHPSHUDWXUHFKDQJHIRUWKHQHXWUDOLVDWLRQZRXOG
RFFXUZKHQWKHYROXPHRI)$PL[HGLVEHWZHHQFPDQGFP7KHWRWDO
YROXPHRIDQ\PL[WXUHVKRXOGEHNHSWFRQVWDQWDWFP

:ULWHDGHWDLOHGSODQRQKRZ\RXFRXOGGHWHUPLQHWKHWHPSHUDWXUHFKDQJHVIRU
WKHVHULHVRIUHDFWLRQVEHWZHHQ)$DQG)$
<RXUSODQVKRXOGLQFOXGHGHWDLOVRI
 DOOHVVHQWLDOH[SHULPHQWDOGHWDLOV
 DSSURSULDWHYROXPHVRIVROXWLRQVWREHXVHG
 D WDEXODWLRQ RI WKH H[SHULPHQWDO GDWD WR EH FROOHFWHG LQFOXGLQJ YROXPHV
XVHG 
 KRZ WKHVH PHDVXUHPHQWV FDQ EH XVHG WR REWDLQ WKH WHPSHUDWXUH FKDQJH
7
 3URFHGXUH
 3ODFHDGU\SRO\VW\UHQHFXSLQWRDFPEHDNHU
 8VHDFPFPPHDVXULQJF\OLQGHUEXUHWWH WRWUDQVIHUFPRI)$ LQWRWKH
VW\URIRDPFXSODEHOOHG)$
  8VH D  FP  FPPHDVXULQJ F\OLQGHUEXUHWWH   WR WUDQVIHU  FP RI )$  LQWR

trendyline
\
DQRWKHUVW\URIRDPFXSODEHOOHG)$
  8VH D WKHUPRPHWHU   WR PHDVXUH WKH WHPSHUDWXUH RI WKH )$  DQG )$  VROXWLRQ
WKH )$
HO\5HFRUGDVT
HO\5HFRUGDV
VHSDUDWHO\5HFRUGDVT FA and
TFA1 F 2UHVSHFWLYHO\
nd TFA2 HVSHFWLYHO\

>7XUQ2YHU

1284 trendyline


 7UDQVIHUWKHFRQWHQWVRI)$FXSWRWKH)$FXS
 6WLUWKHPL[WXUHJHQWO\XVLQJWKHWKHUPRPHWHUSURYLGHG
 5HFRUGWKHPD[LPXPKLJKHVWWHPSHUDWXUHUHDFKHG 5HFRUGDVTmax:DVKDQGGU\
ERWKWKH)$DQG)$VW\URIRDPFXSV
 5HSHDWVWHSVWRIRU0L[WXUHVWR

7DEXODWLRQ
9RORI 9RORI ,QLWLDO ,QLWLDO $YHUDJH +LJKHVW
7
)$ )$ WHPSHUDWXUH WHPSHUDWXUH WHPSHUDWXUH WHPS R
  R R R R
&
FP  FP  RI)$ & RI)$ &  & &

      

      

      

      

      

















t e dy e
trendyline




>7XUQ2YHU

1285 trendyline


 E  LLL  7KHIROORZLQJSORWVZHUHSORWWHGRQDJULGDIWHUDQRWKHUVLPLODUH[SHULPHQWZDV
 FRQGXFWHG'UDZVXLWDEOHJUDSKVWKURXJKWKHSORWWHGSRLQWV
>@
  














 F  %\XVLQJWKHJUDSKLQ E LLL FDOFXODWH
 WKHFRQFHQWUDWLRQRI)$JLYHQWKDWWKHWRWDOYROXPHRIWKHPL[WXUHLVFP
 WKHHQWKDOS\FKDQJHRIQHXWUDOLVDWLRQIRUWKHUHDFWLRQEHWZHHQ)$DQG)$
>@
  9ROXPHRI)$ 1D2+ XVHGZKHQWHPSHUDWXUHFKDQJHLVPD[LPXP
WRFP

1RRIPROHVRIEHQ]RLFDFLGXVHG
  [
PROH

trendyline

D2+EHQ]RLFDFLG
1D2+EHQ]RLFDFLG
RRIPROHVRI1D2+ )$
RRIPROHVRI1D2+ )$ XVHG
XVHG PR
1RRIPROHVRI1D2+ )$ XVHG PROH
PROH

>7XUQ2YHU

1286 trendyline



&RQFHQWUDWLRQRIVRGLXPK\GUR[LGHLQ)$

PROGP
PROGP

)URPJUDSKPD[LPXP7 R&

TZKHQ7LVKLJKHVW

-
1RRIPROHVRIZDWHUIRUPHG 

(QWKDOS\FKDQJHRIQHXWUDOLVDWLRQEHWZHHQ1D2+DQGEHQ]RLFDFLG

N-PRO
  >7RWDO@
 

trendyline >7XUQ2YHU

1287 trendyline


 D  &KURPLXP FDUERQ\O DOVR NQRZQ DV FKURPLXP KH[DFDUERQ\O LV D FKHPLFDO FRPSRXQG
ZLWK WKH IRUPXOD &U &2  &U &2  LV ]HURFRYDOHQW PHDQLQJ WKDW &U KDV DQ R[LGDWLRQ
VWDWHRI'UDZWKHVWUXFWXUHRIWKHFRPSOH[DQGVWDWHWKHVKDSHDQGERQGDQJOHDURXQG
WKH&UDWRPLQWKHFRPSOH[
   
   


6KDSH2FWDKHGUDO
%RQG$QJOHR
   >@
 E  &KURPLXPKH[DFDUERQ\OXQGHUJRHVWKHIROORZLQJOLJDQGUHSODFHPHQWUHDFWLRQ

&U &2 35&U &2 35&2


trendyline >7XUQ2YHU

1288 trendyline


  7ZRVHSDUDWHH[SHULPHQWVZHUHFDUULHGRXWWRVWXG\WKHUDWHRIWKLVUHDFWLRQ
,QWKHILUVWH[SHULPHQWWKHOLJDQG35ZDVLQDODUJHH[FHVVDQG>&U &2 @ZDVPHDVXUHG
DJDLQVWWLPH7KHUHVXOWVDUHVKRZQRQWKHJUDSKEHORZ



,Q WKH VHFRQG H[SHULPHQW &U &2  ZDV LQ D ODUJH H[FHVV DQG >35@ ZDV PHDVXUHG
DJDLQVWWLPH7KHIROORZLQJUHVXOWVZHUHREWDLQHG


WLPHV >35@PROGP

 

 

 

 


  L  2QWKHJUDSKSORWWKHGDWDJLYHQLQWKHWDEOH
   

>7XUQ2YHU

1289 trendyline


  






 QG
 W
VWW

   >@
  LL  8VHWKHJUDSKVWRGHWHUPLQHWKHRUGHURIUHDFWLRQZLWKUHVSHFWWR&U &2 DQG35
,QHDFKFDVHH[SODLQKRZ\RXDUULYHGDW\RXUDQVZHU
   
   &U &2 
$WOHDVWVHWVRIKDOIOLIHFOHDUO\ODEHOOHGRQWKHJUDSKIRUH[SHULPHQW
W V
WKHRUGHURIUHDFWLRQZLWKUHVSHFWWR>&U &2 @LVRQH

35
EHFDXVH E  35 JUDSK LV D VWUDLJKW OLQH ZLWK D FRQVWDQW JUDGLHQW RU OLQH VKRZV D
FRQVWDQWUDWHRIUHDFWLRQRUQRFKDQJHLQUDWHRUVKRZVDOLQHDUGHFUHDVHWKHRUGHU
RIUHDFWLRQZLWKUHVSHFWWR>35@LV]HUR
   >@
   
  LLL  :ULWHWKHUDWHHTXDWLRQIRUWKHUHDFWLRQDQGFDOFXODWHDYDOXHIRUWKHUDWHFRQVWDQW

trendyline
VWDWLQJWKHXQLWVRIWKHUDWHFRQVWDQ
VWDWLQJWKHXQLWVRIWKHUDWHFRQVWDQW
   

>7XUQ2YHU

1290 trendyline


   UDWH N>&U &2 @
WV
N OQW  V
25DOWHUQDWLYHPHWKRGXVLQJLQLWLDOUDWHPHWKRG
,QLWLDO5DWH0HWKRG
*UDGLHQW      
7DNHPRGXOXVDQGLJQRUHWKHQHJDWLYHVLJQKHQFHUDWHLV
   >@
   

  LY  7ZR SRVVLEOH PHFKDQLVPV IRU WKLV UHDFWLRQ DUH JLYHQ EHORZ 6WDWH WKHPHFKDQLVP
ZKLFK LV FRQVLVWHQW ZLWK WKH UDWH HTXDWLRQ \RX KDYH ZULWWHQ LQ E LLL  DQG H[SODLQ
\RXUDQVZHU

0HFKDQLVP$&U &2 &U &2 &2VORZ
&U &2 35&U &2 35IDVW

0HFKDQLVP%&U &2 35>2&&U &2 35@&U &2 35&2
 WUDQVLWLRQVWDWH 

   
   PHFKDQLVP$LVFRQVLVWHQW
EHFDXVHLWKDVDVORZVWHS 6WHS LQYROYLQJRQHPROHPROHFXOHRI&U &2 
  >@
  >7RWDO@
 

>7XUQ2YHU

1291 trendyline


 7KLVTXHVWLRQUHIHUVWRFRPSRXQGVRIFDOFLXPDQGEDULXP

 D  2LO SDLQWV FRQWDLQ WUDFHV RI UHG &D DQG JUHHQ %D LRQV ZKLFK JLYH WKHP WKHLU
FKDUDFWHULVWLF UHG DQG JUHHQ FRORXUV UHVSHFWLYHO\ 2LO SDLQWLQJ LV W\SLFDOO\ GRQH RQ
FDQYDV 'XULQJ WKH PDQXIDFWXULQJ SURFHVV RI FDQYDV K\GURJHQ RUWKRSHULRGDWH LRQV
+,2LVOHIWRQWKHFDQYDV

2UWKRSHULRGDWH VDOWV DUH IRUPHG ZKHQ RUWKRSHULRGDWH LRQV ,2 UHDFWV ZLWK WKH &D
DQG%DLRQVLQWKHSDLQWFDXVLQJWKHGHFRORXULVDWLRQRIWKHUHGDQGJUHHQFRORXURIWKH
SDLQW7KHHTXDWLRQVEHORZVKRZWKHIRUPDWLRQRI,2DQGLWVVDOWV

+,2 DT  + DT ,2 DT KD [PROGP


0 DT ,2 DT 0 ,2  V ZKHUH0LV&DRU%D



5HOHYDQWKVSYDOXHVDUHJLYHQLQWKHWDEOHEHORZ

6DOW KVS
&D ,2  [

%D ,2  [



7KHSDLQWLQJZDVVWRUHGLQDGLVSOD\FDELQHWDWDORZS+HQYLURQPHQW2YHUWKHZHHNHQG
WKHHOHFWULFDOVXSSO\RIWKHGLVSOD\FDELQHWZDVGLVUXSWHGZKLFKUHVXOWHGLQDQLQFUHDVHRI
WKH HQYLURQPHQWV S+ ,W ZDV IRXQG WKDW WKH JUHHQ SRUWLRQV RI WKH SDLQWLQJ ZHUH
GHFRORXULVHGGXHWRWKHIRUPDWLRQRIVROLGEDULXPRUWKRSHULRGDWH%D ,2 

([SODLQTXDOLWDWLYHO\ZK\WKHJUHHQFRORXUHGDUHDVRIWKHSDLQWLQJZHUHGHFRORXULVHGEXW
QRWWKHUHGFRORXUHGDUHDVZKHQWKHS+LQFUHDVHG
  

trendyline >7XUQ2YHU

1292 trendyline


  :KHQS+LQFUHDVHV>+@GHFUHDVHVUHVXOWLQJLQWKHHTXLOLEULXPVKLIWLQJULJKWWRSURGXFH
+LRQVE\/&37KXVWKHFRQFHQWUDWLRQRI ,2LRQVLVKLJKDVWKHSRVLWLRQRIHTXLOLEULXP
VKLIWVWRZDUGVWKHULJKW+HQFH,3RI%D ,2 LVKLJKHUWKDQWKDWRILWV.VSDQGKHQFHWKH
SUHFLSLWDWH RI EDULXP RUWKRSHULRGDWH %D ,2 IRUPV DQG WKH JUHHQ FRORXUHG DUHD
FRQWDLQLQJEDULXPLRQVZHUHGHFRORXULVHG

+RZHYHU&D ,2 KDVKLJKHU.VSYDOXHDQGKHQFHGRQRWSUHFLSLWDWHVHDVLO\HYHQZLWK
WKLVLQFUHDVHLQFRQFHQWUDWLRQVRI ,2LRQVWKHUHVXOWLQJLRQLFSURGXFWRI&DLRQVDQG
,2LRQVVWLOOGRHVQRWH[FHHGVWKH.VSKHQFHWKHSUHFLSLWDWHRI&D ,2 LVQRWIRUPHG
DQGWKHUHGFRORXUHGDUHDFRQWDLQLQJFDOFLXPLRQVLVQRWEHLQJDIIHFWHG
   >@


trendyline >7XUQ2YHU

1293 trendyline


 E  *+DQG6DUHUHODWHGE\WKHIROORZLQJHTXDWLRQ
* +76
7KH(OOLQJKDPGLDJUDPEHORZVKRZVKRZ*FKDQJHVEHWZHHQ.DQG.IRUWKH
IROORZLQJUHDFWLRQ
&D2 V +2 l &D 2+  V 


)RUWKHUDQJHRIWHPSHUDWXUHVLQWKHJUDSKDERYHLWFDQEHDVVXPHGWKDWWKHHQWKDOS\
FKDQJHDQGWKHHQWURS\FKDQJHRIWKHUHDFWLRQUHPDLQDSSUR[LPDWHO\FRQVWDQW

  
  L  3UHGLFWWKHVLJQRI6IRUWKHUHDFWLRQVKRZLQJ\RXUUHDVRQLQJ
   
   7KH 6LV QHJDWLYH DV WKHUH LV D GHFUHDVH LQ WKH QXPEHU RI OLTXLG PROHFXOHV DQG
KHQFH WKH V\VWHP EHFRPHV OHVV GLVRUGHUHG  WKHUH DUH OHVVHU ZD\V RU DUUDQJLQJ
WKHSDUWLFOHVLQWKHV\VWHP
OR Based on the equation: G = H - TS; since the graph is a plot of the G
against T, rearranging the equation will give G = (-S)T + H

y
which correspond to y= ((m) x + C
Hence the gradient of the graph (m) is ((-S). 

>7XUQ2YHU

1294 trendyline


   >@
  LL  8VLQJWKHJUDSKFDOFXODWH6
   
   P    
    

6 N-PRO.

Based on the equation: G = H - TS; since the graph is a plot of the G against
T, rearranging the equation will give G = (-S)T + H
which correspond to y= (m) x + C
Hence the gradient of the graph (m) is (-S).
   >@
  LLL  8VLQJWKHJUDSKGHWHUPLQH+
   

   + N-PRO 
   >@
  LY  'HWHUPLQHWKHWHPSHUDWXUHDERYHZKLFKWKHUHDFWLRQEHFRPHQRQVSRQWDQHRXV
   
   .
  >@
  >7RWDO@
  


 

trendyline >7XUQ2YHU

1295 trendyline


 7KH WDEOH EHORZ JLYHV GDWD DERXW VRPH SK\VLFDO SURSHUWLHV RI WKH HOHPHQWV FDOFLXP FRSSHU
DQGFKURPLXP


3K\VLFDO3URSHUW\ &DOFLXP &RSSHU &KURPLXP

5HODWLYHDWRPLFPDVV   

$WRPLFUDGLXVQP   

'HQVLW\JFP   

)LUVW,RQLVDWLRQ(QHUJ\N-PRO   




 D   L  ([SODLQ ZK\ WKH ILUVW LRQLVDWLRQ HQHUJ\ RI FRSSHU DQG FKURPLXP DUH ERWK KLJKHU
WKDQWKDWRIFDOFLXP
   
   &RSSHU DQG FKURPLXP KDYH PRUH SURWRQV DQG KHQFH KLJKHU QXFOHDU FKDUJH WKDQ
FDOFLXP
 7KH DGGLWLRQDO HOHFWURQV RI WKH GEORFN PHWDOV DUH LQ WKH LQQHU G VXEVKHOO ZKLFK
SURYLGHUHODWLYHO\SRRUVKLHOGLQJIRUWKHRXWHUPRVWYDOHQFHVHOHFWURQV
 +HQFH WKH RXWHUPRVW  YDOHQFH  V HOHFWURQV RI WKH FRSSHU DQG FKURPLXP DUH
DWWUDFWHGPRUHVWURQJO\WRWKHQXFOHXVUHVXOWLQJLQKLJKHULRQL]DWLRQHQHUJ\
   >@
   
  LL  8VHUHOHYDQWGDWDIURPWKHWDEOHWRH[SODLQTXDOLWDWLYHO\ZK\WKHGHQVLWLHVRIFRSSHU
DQGFKURPLXPDUHVLJQLILFDQWO\JUHDWHUWKDQWKDWRIFDOFLXP
   

 
&RSSHUDQGFKURPLXPKDYHODUJHUDWRPLFPDVVHV   WKDQ&D  DQGKDYH

VPDOOHU DWRPLF UDGLL 7KLV OHDGV WR VPDOOHU YROXPH 6LQFH GHQVLW\   KHQFH WKH

GHQVLWLHVRIFRSSHUDQGFKURPLXPDUHVLJQLILFDQWO\JUHDWHUWKDQWKDWRIFDOFLXP

  >@
 E  &KURPLXP ,,, QLWUDWHLVPRUHDFLGLFWKDQFKURPLXP ,, QLWUDWH([SODLQZLWKWKHDLGRID

trendyline
QW
QWHTXDWLRQ
UHOHYDQWHTXDWLRQ

>7XUQ2YHU

1296 trendyline


$ VROXWLRQ RI &U LV PRUH DFLGLF GXH WR WKH KLJKHU FKDUJH GHQVLW\ RI WKH
 
FDWLRQ>&U +2 @ DT  >&U 2+ +2 @ DT + DT 

  >@
  

 F  :KHQDSDUWLFXODUFRSSHURUHZDVUHGXFHGDQDOOR\ZDVSURGXFHGZKLFKZDVFRPSRVHG
PDLQO\RIFRSSHUEXWZLWKJROGDQGFKURPLXPLPSXULWLHV,WFRQWDLQHGQRRWKHUPHWDO,Q
RUGHUWRSXULI\LWWKHDOOR\ZDVPDGHWKHDQRGHRIDQHOHFWURO\WLFFHOOZLWKDSXUHFRSSHU
FDWKRGHDQGDTXHRXVFRSSHU ,, VXOIDWHDVWKHHOHFWURO\WH

([SODLQZLWKUHIHUHQFHWR( YDOXHVZKDWKDSSHQVWRWKHJROGDQGFKURPLXPLPSXULWLHV
GXULQJWKLVSXULILFDWLRQSURFHVV7KHHOHFWURGHUHDFWLRQIRUWKHVWDQGDUG$X$XKDOIFHOO
LV$X DT H $X V ( 9
  &X DT H &X V ( 9

&U DT H &U V  ( 9RU
&U DT H &U V ( 9LJQRUHDOORWKHUHTQV
4XRWLQJRI(YDOXHVIURPGDWDERRNOHW
 ,PSXULWLHVZLWK(PRUHSRVLWLYHWKDQ&XOLNH$XZLOOIDOORIIDVDQRGHVOXGJH
 :KHQ DQ HOHFWULF FXUUHQW LV DSSOLHG FRSSHU DW WKH DQRGH   WRJHWKHU ZLWK &U
ZKLFKDUHPRUHHDVLO\R[LGL]HGWKDQ&XGXHWRWKHLUPRUHQHJDWLYH(YDOXHVDUH
R[LGL]HGWRWKHLULRQV&XDQG&UWKHQPLJUDWHWRWKHFDWKRGH
 $W WKH FDWKRGH RQO\ &X LRQV DUH UHGXFHG WR &X GXH WR LWV PRUH SRVLWLYH ( 
&UUHPDLQDVLRQVLQWKHVROXWLRQDVWKH\GRQRWUHGXFHWRPHWDOVHDVLO\
  >@
  
 

trendyline >7XUQ2YHU

1297 trendyline


 G  $FXUUHQWRI $ZDV SDVVHGWKURXJK WKHFHOO GHVFULEHGLQ F  IRU PLQXWHVDQG WKH
HOHFWURGHVUHPRYHGZDVKHGGULHGDQGZHLJKHG,WZDVIRXQGWKDWVRPHPDVVZDVORVW
IURPWKHDQRGH

$IWHUILOWHULQJLWRIIDQGGU\LQJLWWKHGHSRVLWEHORZWKHDQRGHZHLJKHGJ2QDGGLQJ
DQ H[FHVV RI K\GUR[LGH LRQV WR WKH HOHFWURO\WH D JUH\JUHHQ SUHFLSLWDWH ZDV IRUPHG ,WV
PDVVZDVJ <RXPD\DVVXPHWKDWQHJOLJLEOHDPRXQWRIFRSSHULRQVUHPDLQHGLQ
WKHHOHFWURO\WH 

  L  &DOFXODWHWKHH[SHFWHGLQFUHDVHLQPDVVRIWKHFDWKRGH
   4 ,W
[[
&
$PWRI&XGHSRVLWHG  [  [PRO
0DVVRI&XGHSRVLWHG H[SHFWHGLQFUHDVHLQPDVVRIFDWKRGH
[PRO[ J
   >@
   
  LL  ,GHQWLI\WKHJUH\JUHHQSUHFLSLWDWH
   
JUH\JUHHQSUHFLSLWDWHFKURPLXP ,,, K\GUR[LGH25&U 2+ 
   >@
   
  LLL  ,WZDVIRXQGWKDWWKHPDVVRIFRSSHUUHPRYHGIURPWKHDOOR\ZDVJ&DOFXODWH
WKHPDVVHVRIJROGDQGFKURPLXPUHPRYHGIURPWKHDOOR\+HQFHFDOFXODWHWKHWRWDO
PDVVORVWE\WKHDQRGH
   0DVVRIJROGUHPRYHG J
$PWRI&U 2+   [  PRO
0DVVRI&UUHPRYHG [ J
$FWXDOPDVVORVWE\DQRGH  J
   >@

t d li
trendyline
   >7RWDO@
 

>7XUQ2YHU

1298 trendyline


 D  $VWXGHQWFRQGXFWHGDQH[SHULPHQWWRLGHQWLI\WKHWUHQGLQWKHUPDOVWDELOLW\RIK\GURJHQ
KDOLGHV7KHUHDFWLRQRFFXUUHGDFFRUGLQJWRWKHIROORZLQJHTXDWLRQ

+; J  + J ; J 

7KHDSSUR[LPDWHKFYDOXHVIRUWKHDERYHHTXLOLEULXPDWR&LVVKRZQLQWKHWDEOHEHORZ

KF
7HPSHUDWXUHR&
+&l +%U +,
   


  L  8VLQJ WKH 'DWD %RRNOHW H[SODLQ WKH WUHQG LQ WKHUPDO VWDELOLW\ RI K\GURJHQ KDOLGHV
GRZQWKHJURXS
   
    +&l +%U +,
%RQGHQHUJ\N-PRO   

4XRWLQJRIERQGHQHUJ\YDOXHVIURPGDWDERRNOHW
 'RZQ WKH JURXS WKH VL]H RI WKH KDORJHQ LQFUHDVHV WKHUHIRUH WKH RUELWDOV
RYHUODSEHFRPHVOHVVHIIHFWLYH
 WKHERQGVWUHQJWKEHFRPHVZHDNHUVR+;ERQGLVPRUHHDVLO\EURNHQ
 +HQFHWKHWKHUPDOVWDELOLW\RIWKHK\GURJHQKDOLGHVGHFUHDVHVGRZQWKHJURXS
   >@
   
  LL  +HQFHGHGXFHLIWKHWUHQGLQ D L DJUHHVZLWKWKHKFGDWDJLYHQDERYH
   
   $V.FEHFRPHVPRUHSRVLWLYHLQFUHDVHVWKHUHDUHPRUHSURGXFWV
+HQFHWKLVDJUHHVZLWK D L 
   >@
   

trendyline >7XUQ2YHU

1299 trendyline


 E  $ VWXGHQW FRQGXFWHG DQRWKHU H[SHULPHQW WR GLVWLQJXLVK EHWZHHQ VDPSOHV RI VRGLXP
FKORULGHDQGSKRVSKRUXVSHQWDFKORULGH,WZDVIRXQGWKDWUHGRQLRQZDWHUFDQEHXVHGDV
DQLQGLFDWRU7KHWDEOHEHORZVKRZVWKHFRORXURIWKHLQGLFDWRUDWYDULRXVS+

S+ &RORXU

/HVVWKDQ 5HG
 9LROHW


   
  L  77RHDFKXQNQRZQVDPSOHRIFKORULGHVKHDGGHGDIHZGURSVRIUHGRQLRQZDWHU
+HUHFRUGHGWKHFRORXURIWKHLQGLFDWRU3UHGLFWWKHLGHQWLWLHVRIWKHFKORULGHV


  

&RORXURIUHGRQLRQZDWHU ,GHQWLW\RIFKORULGH

5HG 3KRVSKRUXVSHQWDFKORULGH

9LROHW 6RGLXPFKORULGH

   >@
   
  LL  :LWK WKH DLG RI DQ HTXDWLRQ GHVFULEH WKH DFWLRQ RI ZDWHU RQ SKRVSKRUXV
SHQWDFKORULGH
   3&lXQGHUJRK\GURO\VLVUHDGLO\LQZDWHUWRIRUPDVWURQJO\DFLGLFVROXWLRQ
,QH[FHVVZDWHU3&l V +2 l +32 DT +&l DT 

   >@
   >7RWDO@


trendyline
 

>7XUQ2YHU

1300 trendyline


 3DUDFHWDPROLVDQHIIHFWLYHDQDOJHVLFGLVFRYHUHGE\-RVHSKYRQ0HULQJLQ,WVVWUXFWXUH
LVDVVKRZQ


 D  ([SODLQZK\DOOWKHFDUERQFDUERQERQGVLQEHQ]HQHDUHRIWKHVDPHOHQJWK
  
 
,QEHQ]HQHULQJWKHSLHOHFWURQVSLERQGVDUHGHORFDOL]HGRYHUWKH&DQGKHQFHWKH
&&ERQGOHQJWKDUHWKHVDPH

  >@
  
 E  L  3DUDFHWDPROPD\EHV\QWKHVLVHGIURPSKHQRODVVKRZQ



6WDWH WKH UHDJHQWV DQG FRQGLWLRQV IRU VWHSV  WR  'UDZ WKH VWUXFWXUH RI WKH
LQWHUPHGLDWHSURGXFW%
   
   >@
   5HDJHQWVDQGFRQGLWLRQV
6WHS+12 DT URRPWHPSHUDWXUH
6WHS6QFRQF+&lKHDWIROORZHGE\ FDUHIXOQHXWUDOLVDWLRQZLWK 1D2+if first
part of step 2 is wrong, then no mark for second part even if second part is correct.
6WHS&+&2&lURRPWHPSHUDWXUH

6WUXFWXUHRI%

trendyline

   

>7XUQ2YHU

1301 trendyline


  LL  %DVHGRQ 6WHS RI WKHJLYHQUHDFWLRQVFKHPHGUDZ WKHVWUXFWXUH RIDSRVVLEOH
VLGHSURGXFW
   

RU 

   >@
   
  LLL  3KHQ\OHSKULQH LV D GHFRQJHVWDQW ZKLFK PD\ EH WDNHQ ZLWK SDUDFHWDPRO ZKHQ D
SHUVRQLVVXIIHULQJIURPFROG7KHVWUXFWXUHRISKHQ\OHSKULQHLVDVVKRZQ


6WDWH DQG H[SODLQ KRZ WKH EDVLFLW\ RI SKHQ\OHSKULQHPLJKW FRPSDUH ZLWK WKDW RI
SDUDFHWDPRO

   3KHQ\OHSKULQHLVPRUHEDVLFWKDQSDUDFHWDPRO
3DUDFHWDPRO KDV DQ DPLGH JURXS LQ ZKLFK WKH ORQH SDLU RI HOHFWURQV RQ 1 LV
GHORFDOLVHGRYHU2&1ERQGLQWRWKHEHQ]HQHULQJDQGWKHUHIRUHLVXQDYDLODEOH
WRDFFHSW+
)RU SKHQ\OHSKULQH LW KDV D SULPDU\ DPLQH 7KH  HOHFWURQ GRQDWLQJ 5 JURXSV
PDNHVWKHORQHSDLURIHOHFWURQVRQ1PRUHDYDLODEOHWRDFFHSW+
   >@
   

trendyline >7XUQ2YHU

1302 trendyline


  LY  6XJJHVW D VLPSOH FKHPLFDO WHVW WR GLVWLQJXLVK EHWZHHQ SDUDFHWDPRO DQG
FRPSRXQG$




&RPSRXQG$

   $GG1D2+ DT WRHDFKRIWKHFRPSRXQGVHSDUDWHO\DQGKHDW,QVHUWDJODVVURG
GLSSHGZLWK FRQFHQWUDWHG+&lLQWRHDFKWHVWWXEH)RUFRPSRXQG$DFRORXUOHVV
SXQJHQWJDVLVHYROYHGDQGIRUPVZKLWHIXPH
)RUSDUDFHWDPROWKHUHLVQRHYROXWLRQRIFRORXUOHVVSXQJHQWJDVDQGKHQFHQR
ZKLWHIXPHVLVVHHQ

25

$GG1D2+ DT WRHDFKRIWKHFRPSRXQGVHSDUDWHO\DQGKHDW
)RUFRPSRXQG$DFRORXUOHVVSXQJHQWJDVLVHYROYHGDQGWXUQVPRLVWUHGOLWPXV
SDSHUEOXH
)RU SDUDFHWDPRO WKHUH LV QR HYROXWLRQ RI FRORXUOHVV SXQJHQW JDV 7KH PRLVW
OLWPXVSDSHUUHPDLQVUHG

   >@
   

trendyline >7XUQ2YHU

1303 trendyline


 F  L  +RHFKVW DQG &HODQHVH GLVFRYHUHG D VLPSOHU V\QWKHWLF URXWH IRU SDUDFHWDPRO DV
VKRZQ


6WDWHWKHW\SHVRIUHDFWLRQIRUVWHSVDQG
   >@
  
6WHS(OHFWURSKLOLFVXEVWLWXWLRQ

6WHS&RQGHQVDWLRQ$GGLWLRQHOLPLQDWLRQ

   














trendyline >7XUQ2YHU

1304 trendyline


  LL  6WHSLVNQRZQDVWKH%HFNPDQQDUUDQJHPHQWZKHUHDQR[LPHLVFRQYHUWHGLQWR
DQDPLGH




'UDZWKHVWUXFWXUHRIWKHSURGXFWIRUPHGZKHQ XQGHUJRHVWKH
%HFNPDQQDUUDQJHPHQW
  



   >@
   >7RWDO@
 &RIIHHEHDQVFRQWDLQFKORURJHQLF DFLGZKLFKLV DQDQWLR[LGDQW DQGDQLPSRUWDQWELRV\QWKHWLF
LQWHUPHGLDWH7KHVWUXFWXUHRIFKORURJHQLFDFLGLVDVVKRZQ


tre
endyline
endy 


>7XUQ2YHU

1305 trendyline


 D  6WDWH WKH QXPEHU RI PROHV RI KRW 1D2+ DT  WKDW ZLOO UHDFW ZLWK  PROH RI FKORURJHQLF
DFLG

  >@
  PRO
  
 E  2QKHDWLQJZLWKGLOXWHDFLGFKORURJHQLFDFLGSURGXFHVWZRFRPSRXQGV;DQG<

+2 2 2+
2+
2
2+
2+
+2 +2
2+
&RPSRXQG ; &RPSRXQG < 
:KHQ FRPSRXQG < UHDFWV ZLWK KRW H[FHVV FRQFHQWUDWHG VXOIXULF DFLG D SURGXFW ZLWK D
PROHFXODUIRUPXODRI&+2LVIRUPHG$VWXGHQWFODLPVWKDWWKHSURGXFWIRUPHGKDVWKH
IROORZLQJVWUXFWXUDOIRUPXODEXWWKHWHDFKHUGLVDJUHHV




'UDZ WKH FRUUHFW VWUXFWXUH RI WKH SURGXFW DQG H[SODLQ ZKDW LV ZURQJ ZLWK WKH VWXGHQWV
DQVZHU
  

>7XUQ2YHU

1306 trendyline


  7KH SURGXFW IRUPHG LV XQVWDEOH DV WKHUH LV ULQJ VWUDLQ 2U VS K\EULGL]HG & VKRXOG KDYH
OLQHDUVKDSHEXWWKLVLVQRWSRVVLEOHLQDULQJVWUXFWXUHDQGKHQFHLWLVXQVWDEOH


  >@
  
 F 


  L  &RPSRXQG=UHDFWVZLWK%U l DWURRPWHPSHUDWXUHDQGLQWKHGDUN'HVFULEHWKH
PHFKDQLVPVKRZLQJFXUO\DUURZVFKDUJHVGLSROHVDQGDQ\UHOHYDQWORQHSDLUV
   (OHFWURSKLOLFDGGLWLRQ

trendyline
   >@
 

>7XUQ2YHU

1307 trendyline


   
  LL  7KH SURGXFW RI F L  H[LVWV DV D PL[WXUH RI  VWHUHRLVRPHUV 6WDWH WKH W\SH RI
LVRPHULVPH[KLELWHGE\WKHSURGXFWDQGGUDZDOOWKHVWHUHRLVRPHUV
   
2SWLFDOLVRPHULVP


   >@
   >7RWDO@

(QGRI3DSHU

trendyline >7XUQ2YHU

1308 trendyline
1

Name: Class:

ST ANDREWS JUNIOR COLLEGE

JC2 Preliminary Examinations


Chemistry 9647/03

Higher 2 16 September 2016

Paper 3 2 hours

Candidates answer on separate paper.

Additional Materials: Writing paper, Data Booklet

READ THESE INSTRUCTIONS FIRST

Write your name and civics group on all the work you hand in.
Write in dark blue or black pen.
You may use a soft pencil for any diagrams, graphs or rough working.
Do not use staples, paper clips, highlighters, glue or correction fluid.

Answer any four questions.


You are reminded of the need for good English and clear presentation in your answers.

At the end of the examination, fasten all your work securely together.
The number of marks is given in brackets [ ] at the end of each question or part question.

This document consists of 14printed pages including this page.

trendyline [Turn over

1309 trendyline
2

1 Propenoic acid, also commonly known as acrylic acid, is used in the formation of many
polymers.
(a) Acrylic acid can be synthesised from 2-chloroethanol in the laboratory by the following
route.
NaCN
in ethanol A
HO
step 1 step 2 step 3
Cl CN CO2H
acrylic acid

(i) Suggest the structure for intermediate A. [1]


(ii) Suggest reagents and conditions for step 2 and for step 3. [2]
(iii) What type of reaction is occurring in step 2? [1]

(b) Acrylic acid is manufactured industrially from the reaction of propene and oxygen gas.
(i) Write a balanced equation for this reaction. [1]
(ii) Propenal was isolated as an intermediate product in the reaction.
Draw the structure of propenal. [1]

(c) Polyacrylate is used in environmentally-friendly detergents to remove calcium ions and


magnesium ions in water.
In the manufacture of polyacrylate, acrylic acid was first converted into acrylic chloride,
followed by reaction with an alcohol to form a monomer. Polyacrylate is formed when the
monomer undergoes polymerisation.
SOCl 2
pyridine CH3OH
CO2H COCl CO2CH3
acrylic chloride
polymerisation

H CO2CH3

C C

H H n
o acr a
p ly yl t e

trendyline
(i) e a balanced equation for the conversion of acrylic
Write ac acid into a
acrylic chloride. [1]
(ii) gest
est the type of reaction occurring in the fo
Suggest formation of the monomer
formation m from
c chloride.
acrylic [1]
(iii) gest how polyacrylate can remove
Suggest ove magnesium ions from water through

1310 trendyline
3

complex formation. [1]

1 (d) In the kinetic studies of polymerisation, transition element-containing catalysts were used.
A copper(I)-containing catalyst is shown below.
nBu

N N

Cu

N N

nBu

Cu(I)-nBuPCA catalyst
(i) What is the co-ordination number of copper in the Cu(I)-nBuPCA catalyst? [1]
(ii) What is the electronic configuration of copper in Cu(I)-nBuPCA? [1]
(iii) Suggest whether copper in Cu(I)-nBuPCA catalyst can be classified as a
transition element. [1]

(e) Cu(I) undergoes disproportionation to form Cu and Cu(II).


(i) Use relevant data from the Data Booklet to predict the spontaneity of this
disproportionation reaction. [2]
(ii) Describe and explain what you would see when aqueous ammonia is added
slowly to an aqueous solution of copper(II) chloride, until the aqueous ammonia
is in excess. Write equations for any reactions that occur. [4]
(iii) Suggest an explanation for the difference in colour between [Cu(H 2 O) 6 ]2+ and
[CuCl 4 ]2. [2]
[Total: 20]

trendyline
1311 trendyline
4

[Turn Over
2 In a Mixed Claisen Condensation Reaction, an ester and a NHWRQHUHDFWWRIRUPD-diketone.
.
O 1 CH3CH2ONa, ethanol O O
O . +
2 H
+ + CH CH OH
3 2
OCH2CH3

-
diketone

(a) (i) 'UDZ WKH -diketone formed when 2,2-dimethylcyclohexanone reacts with
ethylethanoate in the Mixed Claisen Condensation Reaction.
O
O

CH3CH2O

2,2-dimethylcyclohexanone ethylethanoate
[1]

(ii) In not more than 3 steps, propose the synthesis of 2,2-dimethylcyclohexanone


from the following starting material.
Cl

[5]

(b) Due to the presence of two ketone IXQFWLRQDO JURXSV -diketones are a highly
valuable substrate in several organic chemistry syntheses.
One of the many examples is selective reduction, in which only one of the ketone
reduced.
O O O OH

selective reduction

- - -
4 hydroxypentan 2 one

Draw the mechanism for the reaction between 4-hydroxypentan-2-one and HCN.
[3]

trendyline
1312 trendyline
5

2 (c) -diketones exist in equilibrium with a tautomeric form, known as an enol.


This is known as keto-enol tautomerisation. The keto-enol tautomerisation of
pentan-2,4-dione is shown below.
O O O OH

H<0
-
keto enol tautomerisation

keto enol

(i) Write an expression for K c for the keto-enol tautomerisation of pentan-2,4-dione. [1]
(ii) In the case of pentan-2,4-dione, 76% of the mixture exists as an enol at room
temperature and pressure. Calculate a value for K c . [1]
(iii) Sketch a graph showing how the rates of the forward and reverse reactions
change from the time pure keto is added to the time the reaction reaches
equilibrium. Label your two lines clearly. [2]

(d) The keto-enol tautomerisation is catalysed by the presence of an inorganic base, such as
NaOH.
O O
O O
+ OH +H O
2

keto

O OH
+ OH

enol

(i) State how the presence of NaOH would affect the K c , as calculated in (c)(ii). [1]

trendyline
(ii) he
e same diagram in (c)(iii), show
On the how how the ra forwa and reverse
rates of the forward
ions change for the base catalysed keto-
reactions keto-enol tautomerisation.
keto-enol tautomerisation
Labell your lines clearly. [1]
(iii) ce, explain the effect of NaOH on the reaction.
Hence, [2]

1313 trendyline
6

[Turn Over
2 (e) The enol form is more stable than the keto form due to two reasons
Reason I: Intramolecular hydrogen bonding
Reason II: Position of p orbitals

O OH

1 3
2
enol

(i) Draw an appropriate diagram to show how reason I contributes to the stability [1]
of the enol form.
(ii) State the hybridisation of the carbon atoms at positions 1, 2 and 3.
Hence, explain how reason II contributes to the stability of the enol form. [2]
[Total: 20]

trendyline
1314 trendyline
7

3 (a) Antiseptics and disinfectants are used to kill bacteria. The difference between
antiseptics and disinfectants is that antiseptics are used on living skin and
disinfectants are used on surfaces of objects.
Calcium hypochlorite, Ca(OCl) 2 , is often used to sanitise public swimming pools. It is
produced by treating calcium hydroxide with chlorine gas.
2Ca(OH) 2 + 2Cl 2 Ca(OCl) 2 + CaCl 2 + 2H 2 O
(i) Write an equation for the decomposition of calcium hydroxide. [1]
(ii) Deduce whether calcium hypochlorite would decompose at a higher or
lower temperature than calcium hydroxide. Explain your reasoning. [2]
(iii) In order to test the percentage purity of the calcium hypochlorite granules,
1 g of the calcium hypochlorite granules above was dissolved in water
and the solution was then reacted with excess iodide in an acidic medium.
OCl + 2H+ + 2I Cl + H 2 O+ I2
The resulting solution was then titrated with 0.500 mol dm3 sodium
thiosulfate. 22.5 cm3 of thiosulfate was required to reach the end-point.
Calculate the percentage purity of the granules. [2]
(iv) Commercially, calcium hypochlorite is available as granules. Using your
answer in (a)(iii),calculate the volume of chlorine gas (measured at room
temperature and pressure) required to manufacture 1 kg of calcium [2]
hypochlorite granules.

trendyline
1315 trendyline
8

[Turn Over
3 (b) Chloroxylenol is commonly used in antibacterial soaps due to its disinfecting
and sanitising properties. It is also responsible for the distinctive odour of
Dettol.
An isomer of chloroxylenol is compound X.
Cl

OH

CH2CH2Cl

OH

chloroxylenol compound X

When hot ethanolic silver nitrate was added to chloroxylenol and compound X
separately, the observations were different. State what you would observe for
each compound and explain the difference in observations. [3]

(c) Compounds A and B are structural isomers of chloroxylenol. Both A and Bgive
a violet colouration with neutral FeCl 3 .
A rotates plane-polarised light, but B does not.When B is treated with hot
alcoholic sodium hydroxide, a compound of molecular formula C 8 H 8 O is
formed.
When A is treated carefully with hot aqueous sodium hydroxide, compound C,
C 8 H 10 O 2 , is formed.
C reacts with hot concentrated potassium dichromate(VI) to form compound D.
C reacts with hot concentrated potassium manganate(VII) to form compound
Eand carbon dioxide gas.
D gives an orange precipitate with 2,4-dinitrophenylhydrazine, but E does not.
Suggest structures for A to E, and explain the observations described above. [10]

[Total: 20]

1316 trendyline
9

trendyline
1317 trendyline
10

4 The table below summarises the different types of acids found in various types of beverages.

Beverage Acid Formula pK a1 pK a2 pK a3


Beer Carbonic H 2 CO 3 6.37 10.3
Fruit Juices Malic HO 2 CCH 2 CH(OH)CO 2 H 3.46 5.10
Soda Phosphoric H 3 PO 4 2.12 7.21 12.3

(a) (i) Explain why the pK a1 of malic acid is so much lower than the pK a1 of
carbonic acid. [2]
(ii) Explain why the pK a3 of phosphoric acid is so much higher than the pK a2 of
malic acid. [1]

(b) (i) 50.0 cm3 of 0.5 mol dm3 aqueous H 3 PO 3 was mixed with 30cm3 of excess
aqueous NaOH. It was found that the temperature of the solution rose by
8.4C after mixing. Calculate the enthalpy change of reaction per mole of
H 3 PO 3 (aq).

Given: Specific heat capacity of water = 4.18 J g1 K1;


Density of water = 1.0 g cm3. [2]
(ii) Hence, given that the enthalpy change of neutralisation of HCl (aq) by
1D2+ DT LV kJ mol1, suggest the basicity of H 3 PO 3 .
[1]

(c) A medical student wanted to investigate the buffer solution in fluids for
intravenous injection. He needed to prepare a buffer of pH 7.2. Of the pK a values
given in the table above, suggest a conjugate acidbase pair which is most
suitable as the buffer solution. [1]

trendyline [Turn Over

1318 trendyline
11

4 (d) A student titrated 20 cm3 of the buffer sample in (c) containing an acid and its salt, with
0.1 mol dm3 NaOH(aq) and obtained the titration curve below. The maximum buffer
capacity is obtained when 20 cm3 of NaOH is added.
The equivalence point of the titration is obtained when 60 cm3 of aqueous sodium
hydroxide is added.
[You may use HA and A to represent the species identified in (c) to answer part (d)].

pH

7.2

Vol of NaOH / cm3


20 60

(i) Explain why the pH rises only slightly at the start of the titration when a
small amount of sodium hydroxide is added. [1]
(ii) Determine the total amount of acid in the buffer. Hence, show that the ratio
of concentrations of the [HA] and [A] at the start of the titration is 3:1. [2]
(iii) Calculate the K a of the acid at the start of the titration. [1]

(e) An electrochemical cell is composed of two halfcells, namely the AgBrAg and
Ag+Ag half-cells at 298K.

AgBr(s) + e Ag(s) + Br (aq) E = +0.088 V

(i) Using the data booklet and the half-equation of AgBrAg above, write the
overall equation for the cell reaction and calculate the E cell . [2]

trendyline
1319 trendyline
12

4 (e) (ii) The Nernst equation is used to calculate the voltage, E cell , generated
under nonstandard conditions.

RT 1
E cell = E cell nF
ln ( )
Ksp

where n is the number of moles of electrons transferred per mole of


equation, R is the molar gas constant, F is the Faraday constant and K sp is
the solubility product of AgBr.
When E cell = 0, the system has achieved equilibrium.
Decide on a suitable value of n and hence, use your answer in(e)(i) and
the Nernst equation to calculate the solubility product of AgBr at 298 K,
[3]
stating its units.
(iii) Predict how the voltage of the cell, E cell , will change when NH 3 (aq) is
added to the Ag+Ag cell. [2]
(iv) Given that silver arsenate, Ag 3 AsO 4 has a K sp of 1.40x1022, show by
calculation, if AgBr or Ag 3 AsO 4 is the more soluble salt. [2]
[Total: 20]

trendyline [Turn Over

1320 trendyline
13

5 About 10% of the mass of a chicken egg is made up of proteins. The main protein in egg whites
is ovalbumin.
Three of the amino acids obtained from the complete hydrolysis of ovalbumin and their
isoelectric points are given in the table below.
Amino acid Formula of side chain Isoelectric point
Aspartic acid (asp) CH 2 CO 2 H 2.77
Lysine (lys) (CH 2 ) 4 NH 2 9.74
Tyrosine (tyr) 5.66

CH2 OH

(a) State how ovalbumin could be hydrolysed in the laboratory to form the amino acids in [1]
the table above.

(b) (i) Define the term isoelectric point. [1]


(ii) Electrophoresis is an electrolytic method used to separate amino acids based
on their isoelectric points. A simplified diagram of an electrophoresis set-up is
shown below. Depending on the pH of the buffer solution used, the amino
acids will move differently. The positions of the amino acids are ascertained by
spraying the solution with ninhydrin solution. The colourless amino acid spots
will turn blue.

cathode
Mixture of amino anode
acids

Buffer solution

Using data from the table above, explain how electrophoresis can be used to
separate and identify the constituent amino acids in a mixture of aspartic acid,

trendyline
lysine
e and tyrosine in a buffer solution of pH 8. You should
ne shoul include the
structural
ctural formulae of the three amino acids at pH 8 in your answ
answer. [3]

1321 trendyline
14

5 (c) With reference to protein structure, explain why egg white coagulates when cooked. [2]

(d) Besides reacting with amino acids, ninhydrin is also commonly used in fingerprint
analysis in criminal investigations. Another method to analyse fingerprints is called
cyanoacrylate (Super Glue) fuming. One common cyanoacrylate used for this
purpose is ethyl cyanoacrylate. The structures of ninhydrin and cyanoacrylate are
given below.

O
O
OH

OH O

CN

O
ninhydrin ethyl cyanoacrylate

Suggest a chemical test that could be used to distinguish ninhydrin and ethyl
cyanoacrylate from each other. You should state what you would observe for each
compound in the test. [2]

(e) Besides being rich in protein, eggs are also consumed widely as it contains vitamins
and minerals needed on a daily basis. An example of a key nutrient that can be
obtained from eating eggs is phosphorus, which is as important as calcium in building
strong bones in our body.
(i) Describe the reaction of phosphorus(III) oxide, P 4 O 6 , with water. Include the pH
value of the resulting solution, and write an equation for the reaction that occurs. [2]
(ii) Write an equation to illustrate the acid-base nature of phosphorus(V) oxide,
P 4 O 10 . [1]

trendyline [Turn Over

1322 trendyline
15

5 (f) Eggs are also rich in iodine which is a key component of hormones in the thyroid
gland. Iodine deficiency results when we do not consume enough iodine in our diet.
In order to combat iodine deficiency, table salt is mixed with various salts of iodine in
the production of iodised salts.
Potassium iodate, KIO 3 , is often used in iodised salts. With the aid of an equation,
describe how potassium iodate can be prepared using iodine as a starting material. [2]

(g) Describe what you would see if concentrated sulfuric acid is added to separate
samples of solids KBr and KI. Suggest an explanation for the differences in reaction,
and write equations for the reactions that occurred. [4]

(h) Iodine has many isotopes. Only two of its isotopes are used in medicine, such as
radiotherapy and imaging. In order to identify the mass of these two isotopes, they
were first ionised to form singly-charged ions of +1 charge and passed through an
electric field of constant strength. Their angles of deflection, along with a reference
sample of helium nucleus, 42He, were recorded in the table below.
Angle of deflection
Isotope X 1.07
Isotope Y 1.14
Heliumnucleus, 42He 70.1

Calculate the mass of each isotope, giving your answer to the nearest whole number.
[2]
[Total: 20]

~ END OF PAPER ~

trendyline
1323 trendyline
1

Name: Class:

ST ANDREWS JUNIOR COLLEGE

JC2 Preliminary Examinations


Chemistry 9647/03

Higher 2 16 September 2016

Paper 3 2 hours

Candidates answer on separate paper.

Additional Materials: Writing paper, Data Booklet

READ THESE INSTRUCTIONS FIRST

Write your name and civics group on all the work you hand in.
Write in dark blue or black pen.
You may use a soft pencil for any diagrams, graphs or rough working.
Do not use staples, paper clips, highlighters, glue or correction fluid.

Answer any four questions.


You are reminded of the need for good English and clear presentation in your answers.

At the end of the examination, fasten all your work securely together.
The number of marks is given in brackets [ ] at the end of each question or part question.

This document consists of 14printed pages including this page.

trendyline [Turn over

1324 trendyline
2

1 Propenoic acid, also commonly known as acrylic acid, is used in the formation of many
polymers.
(a) Acrylic acid can be synthesised from 2-chloroethanol in the laboratory by the following
route.
NaCN
in ethanol A
HO
step 1 step 2 step 3
Cl CN CO2H
acrylic acid

(i) Suggest the structure for intermediate A. [1]


HO
CN

(ii) Suggest reagents and conditions for step 2 and for step 3. [2]
Step 2: excess conc. H 2 SO 4 , 170OC
Step 3: H 2 SO 4 (aq), heat
(iii) What type of reaction is occurring in step 2? [1]
Elimination

(b) Acrylic acid is manufactured industrially from the reaction of propene and oxygen gas.
(i) Write a balanced equation for this reaction. [1]
CH 3 CH=CH 2 + 3/2O 2 CH 2 =CHCO 2 H + H 2 O
(ii) Propenal was isolated as an intermediate product in the reaction. Draw the
structure of propenal. [1]
O

(c) Polyacrylate is used in environmentally-friendly detergents to remove calcium ions and


magnesium ions in water.
In the manufacture of polyacrylate, acrylic acid was first converted into acrylic chloride,
followed by reaction with an alcohol to form a monomer. Polyacrylate is formed when the
monomer undergoes polymerisation.

trendyline
1325 trendyline
3

SOCl 2
pyridine CH3OH
CO2H COCl CO2CH3
acrylic chloride
polymerisation

H CO2CH3

C C

H H n
p oly acr ylate

(i) Write a balanced equation for the conversion of acrylic acid into acrylic chloride. [1]
+ SOCl 2 + SO2 + HCl
CO2H COCl
(ii) Suggest the type of reaction occurring in the formation of the monomer from
acrylic chloride. [1]
Nucleophilic substitution/Condensation
(iii) Suggest how polyacrylate can remove magnesium ions from water through
complex formation. [1]
The lone pair of electrons on O of the ester functional group / ligandcan
form dative bond/coordinate bonds with empty orbitals of the cation.

(d) In the kinetic studies of polymerisation, transition element-containing catalysts were used.
A copper(I)-containing catalyst is shown below.
nBu

N N

Cu

N N

nBu

Cu(I)-nBuPCA catalyst
(i) What is the co-ordination number of copper in the Cu(I)-nBuPCA catalyst? [1]
4

trendyline
(ii) C I)-nBuPCA?
Whatt is the electronic configuration of copper in Cu( [1]
1s22s
s22p
2 63s23p63d10
(iii) est whether copper in Cu(I))-nBuPCA
gest
Suggest -nBuPCA
uPCA catalyst can be classified
c as a
[1]

1326 trendyline
4

transition element.
No. Cu+ is not a transition element as it does not have incompletely filled
3d orbitals / does not have partially filled 3d orbitals / has completely filled
3d orbitals.

(e) Cu(I) undergoes disproportionation to form Cu and Cu(II).


(i) Use relevant data from the Data Booklet to predict the spontaneity of this
disproportionation reaction. [2]
Cu+ + e Cu E = +0.52V
Cu2+ + e Cu+ E = +0.15V

E cell = (+0.52) (+0.15) = +0.37V


E cell>0, spontaneous
(ii) Describe and explain what you would see when aqueous ammonia is added
slowly to an aqueous solution of copper(II) chloride, until the aqueous ammonia
is in excess. Write equations for any reactions that occur. [4]
When NH 3 is added slowly, blue ppt is formed.
NH 3 + H 2 O NH 4 + + OH
[Cu(H 2 O) 6 ]2+ + 2OH Cu(OH) 2 + 6H 2 O --- Eqm 1
Blue solution blue ppt

When NH 3 is added in excess, ligand exchange occurs.


[Cu(H 2 O) 6 ]2+ + 4NH 3 [Cu(NH 3 ) 4 (H 2 O) 2 ]2+ + 4H 2 O
deep blue solution

The formation of [Cu(NH 3 ) 4 (H 2 O) 2 ]2+ decreases the concentration of


[Cu(H 2 O) 6 ]2+ and shifts the position of Eqm 1 to left. Hence, the blue ppt
dissolves to form a deep blue solution.
(iii) Suggest an explanation for the difference in colour between [Cu(H 2 O) 6 ]2+ and
[CuCl 4 ]2. [2]
The different ligands, H 2 O and Cl, result in a different energy gap.
Different amount of energy/wavelength of light is absorbed to promote the

trendyline
trendy
tron
ron from the lower energy group to the hi
electron higher energy gro
group.
Lightt refl
ref ected/not absorbed is different.
reflected/not different.
[Total: 20]

1327 trendyline
5

2 In a Mixed Claisen &RQGHQVDWLRQ5HDFWLRQDQHVWHUDQGDNHWRQHUHDFWWRIRUPD-diketone.


.
O 1 CH3CH2ONa, ethanol O O
O . +
2 H
+ + CH CH OH
3 2
OCH2CH3

-
diketone

(a) (i) 'UDZ WKH -diketone formed when 2,2-dimethylcyclohexanone reacts with
ethylethanoate in the Mixed Claisen Condensation Reaction.
O
O

CH3CH2O

2,2-dimethylcyclohexanone ethylethanoate
[1]

O O

(ii) In not more than 3 steps, propose the synthesis of 2,2-dimethylcyclohexanone


from the following starting material.
Cl

[5]
OH O

Cl

. KMnO4/
ethanolic NaOH 1 conc H2SO4 at 0OC
heat . o n water K2Cr2O7
2 b ili g H2SO4 (aq)
heat
[1] [1] [1] [1] [1]

(b) 'XH WR WKH SUHVHQFH RI WZR NHWRQH IXQFWLRQDO JURXSV -diketones are a highly
valuable substrate in several organic chemistry syntheses.

trendyline
y examples
One of the many p is selective reduction, in which onlyy one of the ketone
reduced.

1328 trendyline
6

O O O OH

selective reduction

- - -
4 hydroxypentan 2 one [3]
Draw the mechanism for the reaction between 4-hydroxypentan-2-one and HCN.
Nucleophilic addition

HCN H+ + CN

NaCN Na+ + CN
OR
NaOH + HCN NaCN + H 2 O

O O OH
=

R

O OH
O H
CN
CN
slow + CN


R
CN CN
R R

(c) -diketones exist in equilibrium with a tautomeric form, known as an enol.


This is known as keto-enol tautomerisation. The keto-enol tautomerisation of
pentan-2,4-dione is shown below.
O O O OH

H<0
-
keto enol tautomerisation

keto enol

(i) Write an expression for K c for the keto-enol tautomerisation of pentan-2,4-dione. [1]
Kc = [enol]/[keto]

trendyline
(ii) e case
In the c pentan-2,4-dione, 76% of the mixt
of pentan-2,4-dione, mixture exists as an enol at room
erature and pressure. Calculate a value for K c .
temperature [1]

Kc = 0.76/0.24

1329 trendyline
7

= 3.17 (3sf)
(iii) Sketch a graph showing how the rates of the forward and reverse reactions
change from the time pure keto is added to the time the reaction reaches
equilibrium. Label your two lines clearly. [2]

rate

forward

reverse

time

teqm

(d) The keto-enol tautomerisation is catalysed by the presence of an inorganic base, such as
NaOH.

trendyline
1330 trendyline
8

O O
O O
+ OH +H O
2

keto

O OH
+ OH

enol

(i) State how the presence of NaOH would affect the K c , as calculated in (c)(ii). [1]
No effect.
(ii) On the same diagram in (c)(iii), show how the rates of the forward and reverse
reactions change for the base catalysed keto-enol tautomerisation.
Label your lines clearly. [1]

rate

forward,
catalysed

forward

reverse
reverse,
catalysed

teqm teqm time


(catalysed)

trendyline
1331 trendyline
9

(iii) Hence, explain the effect of NaOH on the reaction. [2]


The catalyst NaOH provides an alternative pathway of lower activation
energy, which increases the number of particles having energy more than
or equals to the activation energy. Hence, the frequency of effective
collision increasesandboth the rate of the forward and reverse reaction
increases / equilibrium reached faster.

(e) The enol form is more stable than the keto form due to two reasons
Reason I: Intramolecular hydrogen bonding
Reason II: Position of p orbitals

O OH

1 3
2
enol

(i) Draw an appropriate diagram to show how reason I contributes to the stability [1]
of the enol form.

H
hydrogen bond

O O

(ii) State the hybridisation of the carbon atoms at positions 1, 2 and 3.


Hence, explain how reason II contributes to the stability of the enol form. [2]
sp2
The carbons 1, 2 and 3 have an unhybridised p orbital each. The adjacent
p orbitals allow the GHORFDOLVDWLRQRIHOHFWURQV.
[Total: 20]

trendyline
1332 trendyline
10

3 (a) Antiseptics and disinfectants are used to kill bacteria. The difference between
antiseptics and disinfectants is that antiseptics are used on living skin and
disinfectants are used on surfaces of objects.
Calcium hypochlorite, Ca(OCl) 2 , is often used to sanitise public swimming pools. It is
produced by treating calcium hydroxide with chlorine gas.
2Ca(OH) 2 + 2Cl 2 Ca(OCl) 2 + CaCl 2 + 2H 2 O
(i) Write an equation for the decomposition of calcium hydroxide. [1]
Ca(OH) 2 CaO + H 2 O
(ii) Deduce whether calcium hypochlorite would decompose at a higher or
lower temperature than calcium hydroxide. Explain your reasoning. [2]
Hypochlorite ion has a larger electron cloud, hence it is more easily
polarised. The O-Cl bond is weakened to a greater extent thus less
energy is required to decompose calcium hypochlorite and it decomposes
at a lower temperature.
(iii) In order to test the percentage purity of the calcium hypochlorite granules,
1 g of the calcium hypochlorite granules above was dissolved in water
and the solution was then reacted with excess iodide in an acidic medium.
OCl + 2H+ + 2I Cl + H 2 O+ I2
The resulting solution was then titrated with 0.500 mol dm3 sodium
thiosulfate. 22.5 cm3 of thiosulfate was required to reach the end-point.
Calculate the percentage purity of the granules. [2]
I 2 + 2S 2 O 3  2I + S 4 O 6 
Amount of thiosulfate = 0.0225 x 0.5
= 0.01125 mol
Amount of I2 = 0.01125 / 2 = 0.005625 mol
OCl + 2H+ + 2I Cl + H 2 O+ I2
Amount of hypochlorite = 0.005625 mol
Amount of calcium hypochorite, Ca(OCl) 2 = 0.005625 / 2
= 0.0028125 mol
Mass of calcium hypochlorite = 0.0028125 x (40.1 + 32 + 71)
= 0.40246 g
Percentage purity = (0.40246 / 1) x 100% = 40.2 %

trendyline
(iv) Commercially,
ommercially, calcium hypochlorite
e is available as granules. Using
Us your

1333 trendyline
11

answer in (a)(iii),calculate the volume of chlorine gas (measured at room


temperature and pressure) required to manufacture 1 kg of calcium [2]
hypochlorite granules.
Amount of calcium hypochlorite = 1000 / (40.1 + 32 + 71) = 6.988 mol
Theoretical amount of calcium hypochlorite to manufacture
= (100/40.2) x 6.988
= 17.38 mol
Amount of chlorine gas required = 17.38 x 2 = 34.76 mol
Volume of chlorine gas required = 34.76 x 24 = 834 dm3

(b) Chloroxylenol is commonly used in antibacterial soaps due to its disinfecting


and sanitising properties. It is also responsible for the distinctive odour of
Dettol.
An isomer of chloroxylenol is compound X.
Cl

OH

CH2CH2Cl

OH

chloroxylenol compound X

When hot ethanolic silver nitrate was added to chloroxylenol and compound X
separately, the observations were different. State what you would observe for
each compound and explain the difference in observations. [3]
No ppt for chloroxylenol,whiteppt for compound X
Lone pair of electrons on chlorine is delocalised into benzene ring, giving rise
to partial double bond character to C-Cl in chloroxylenol, thus more energy is
required to break C-Cl bond / C-Cl bond is stronger / harder to break, hence it

trendyline
yli e
is not broken. Hence,, no ppt
pp seen.
C-Cl bond
nd in compound X is broken to form ree Cl / white ppt ofAgC
orm free offAgClis
ofAg
formed.

1334 trendyline
12

(c) Compounds A and B are structural isomers of chloroxylenol. Both A and Bgive
a violet colouration with neutral FeCl 3 .
A rotates plane-polarised light, but B does not.When B is treated with hot
alcoholic sodium hydroxide, a compound of molecular formula C 8 H 8 O is
formed.
When A is treated carefully with hot aqueous sodium hydroxide, compound C,
C 8 H 10 O 2 , is formed.
C reacts with hot concentrated potassium dichromate(VI) to form compound D.
C reacts with hot concentrated potassium manganate(VII) to form compound
Eand carbon dioxide gas.
D gives an orange precipitate with 2,4-dinitrophenylhydrazine, but E does not.
Suggest structures for A to E, and explain the observations described above. [10]

[Total: 20]

A and B gives violet colouration with neutral phenol present


FeCl 3 .

A rotates plane polarised light, but B does not. A has a chiral centre
Or indicate * on the chiral carbon of A correctly
OH Cl

C
*
CH3
H

When A is treated with hot aq NaOH, C, Nucleophilic substitutionof halogenoalkane /


C 8 H 10 O 2 , is formed. chloroalkaneto form alcohol.
-Cl in A is replaced by -OH in C.
OH OH
CH3
C
H

C reacts with hot conc


trendyline
dy
yll
onc K 2 Cr 2 O 7 to form D. Oxid
C

Oxidation of 2 alcohol to form D, a ketone.

1335 trendyline
13

OH O

C reacts with hot conc KMnO 4 to form Eand Oxidationof benzylic H to form E, with a benzoic
carbon dioxide gas. OH

CO2H

E
acidfunctional group
D gives an orange ppt with 2,4-DNPH, but E Condensation
does not. D is aketone
B forms C 8 H 8 O when treated with hot alcoholic Elimination of HCl.
NaOH B is a 1Ohalogenoalkane
OH

CH2CH2Cl

trendyline
1336 trendyline
14

4 The table below summarises the different types of acids found in various types of beverages.

Beverage Acid Formula pK a1 pK a2 pK a3


Beer Carbonic H 2 CO 3 6.37 10.3
Fruit Juices Malic HO 2 CCH 2 CH(OH)CO 2 H 3.46 5.10
Soda Phosphoric H 3 PO 4 2.12 7.21 12.3

(a) (i) Explain why the pK a1 of malic acid is so much lower than the pK a1 of
carbonic acid. [2]
HO 2 CCH 2 CH(OH)CO 2 has an electron-withdrawing group (-OH or CO 2 H)
which disperses the negative charge on the COOincreasing the stability
of anion compared to HCO 3 -, hence malic acid is more acidic.

OR

Malic acid has a more stable anion, HO 2 CCH 2 CH(OH)CO 2 , which is


stabilised by intramolecular hydrogen bonding and this helps to disperse
the negative charge. HCO 3 cannot be stabilised this way, hence malic
acid is more acidic.

(ii) Explain why the pK a3 of phosphoric acid is so much higher than the pK a2 of
malic acid. [1]
It is harder to lose a H+ from a doubly charged(or more negatively-
charged) ion, HPO 4 2 than from singly charged anion,

HO 2 CCH 2 CH(OH)CO 2 .

(b) (i) 50.0 cm3 of 0.5 mol dm3 aqueous H 3 PO 3 was mixed with 30cm3 of excess
aqueous NaOH. It was found that the temperature of the solution rose by
8.4C after mixing. Calculate the enthalpy change of reaction per mole of
H 3 PO 3 (aq).

Given: Specific heat capacity of water = 4.18 J g1 K1;

trendyline
m3.
Density of water = 1.0 g cm [2]
2
No. of moles of H 3 PO 3 used
ed = 2.5 10 mol
Q = mc
mcT = 80 4.18 8.4 = 2808.96J
mcT

1337 trendyline
15

Enthalpy change RIUHDFWLRQ N-PRO1


(ii) Hence, given that the enthalpy change of neutralisation of HCl (aq) by
1D2+ DT LV kJ mol1, suggest the basicity of H 3 PO 3 .
[1]
2 or dibasic

(c) A medical student wanted to investigate the buffer solution in fluids for
intravenous injection. He needed to prepare a buffer of pH 7.2. Of the pK a values
given in the table above, suggest a conjugate acidbase pair which is most
suitable as the buffer solution. [1]
2
H 2 PO 4 , HPO 4

(d) A student titrated 20 cm3of the buffer sample in (c) containing an acid and its salt, with
0.1 mol dm3 NaOH(aq) and obtained the titration curve below. The maximum buffer
capacity is obtained when 20 cm3 of NaOH is added.
The equivalence point of the titration is obtained when 60 cm3 of aqueous sodium
hydroxide is added.
[You may use HA and A to represent the species identified in (c) to answer part (d)].

pH

7.2

Vol of NaOH / cm3


20 60

(i) Explain why the pH rises only slightly at the start of the titration when a small
amount of sodium hydroxide is added. [1]

trendyline
tren li
mall amount of NaOH is added, th
When a small e large
the e res
reservoir of HA will remove it,
ng
g fairly
maintaining irly constant pH.
pH.
(ii) Determine
rmine
mine the total amount of acid in the buffer. Hence, show
sshow that
tha the ratio
oncentrations of the [HA] and [A
of concentrations A] at the start of the titration is 3:1. [2]

1338 trendyline
16

Total no. of moles of HA in buffer = no. of moles of NaOH required to reach


equivalence point = 0.1 x 60/1000 = 6 x 103 moles

Let original amount of A-in bufferbe y mol.


When 20cm3 of NaOH is added, MBC occurs.
Amount of Aat MBC = (2 x 103 +y) mol
Note: HA + OHA+ H 2 O

At MBC, [HA] = [A]


Hence, amount of HA = amount of A (since volume is the same).
Since a further 40cm3 of NaOH is required to neutralise all HA at MBC,
amount of HA at MBC = 4 x 103 mol = amount of A at MBC.
=>(2 x 103 +y) = 4 x 103
y = 4 x 103 2 x 103 = 2 x 103
No. of moles of HA:A = 6 x 103 : 2 x 103 = 3:1
(iii) Calculate the K a of the acid at the start of the titration. [1]
pH = pK a + lg [salt]/[acid]
7.2 = lgK a + lg (1/3)
K a = 2.10 x 108 mol dm3

(e) An electrochemical cell is composed of two halfcells, namely the AgBrAg and
Ag+Ag halfcells at 298K.

AgBr(s) + e Ag(s) + Br (aq) E = +0.088 V

(i) Using the data booklet and the half-equation of AgBrAg above, write the
overall equation for the cell reaction and calculate the E cell . [2]

AgBr + e Ag+ + Br
Ag+ + e Ag

Overall equation: Ag+ +Br AgBr

trendyline
en
E cell = 0.80
80
0 0.088
088 = +0.712
0.712 V

(ii) The Nernst equation is used to calculate the voltage


voltage, E cellll , genera
generated under

1339 trendyline
17

nonstandard conditions.

RT 1
E cell = E cell nF
ln ( )
Ksp

where n is the number of moles of electrons transferred per mole of


equation, R is the molar gas constant, F is the Faraday constant and K sp is
the solubility product of AgBr.
When E cell = 0, the system has achieved equilibrium.
Decide on a suitable value of n and hence, use your answer in(e)(i) and the
Nernst equation to calculate the solubility product of AgBr at 298 K, stating
[3]
its units.
n = 1;
At equilibrium, E cell = 0 V
0 = E cell (RT/nF) ln(1/K sp )
Therefore, 0 = +0.712 (8.31)(298)/(1)(96500) ln (1/K sp )
K sp = 8.92 x 1013 mol2 dm6

(iii) Predict how the voltage of the cell, E cell , will change when NH 3 (aq) is
added to the Ag+Ag cell. [2]
[Ag+] will decreasedue to [Ag(NH 3 ) 2 ]+ formed:
Ag+ + 2NH 3 [Ag(NH 3 ) 2 ]+ + Br
OR
Aqueous NH 3 will react with Ag+ to form a soluble complex, [Ag(NH 3 ) 2 ]+.

Ag+ + e Ag
Position of equilibrium will shift left,resulting in less positive E cathode .
Since E cell =E cathode E anode , Voltage/ E cell will be less positive and E cell
will be less positive.

(iv) Given that silver arsenate, Ag 3 AsO 4 has a K sp of 1.40x1022, show by


calculation, if AgBr or Ag 3 AsO 4 is the more soluble salt. [2]
Let x be solubility of AgBr.

trendyline
tre
e
K sp = x2
9.4 x 107 mol dm-3
x = 9.44

1340 trendyline
18

Let y be solubility of Ag 3 AsO 4


K sp = y(3y)3 = 27y4
y= 1.51 x 106mol dm-3
Since Ag 3 AsO 4 has a higher solubility, Ag 3 AsO 4 is more soluble.
[Total: 20]

5 About 10% of the mass of a chicken egg is made up of proteins. The main protein in egg
whites is ovalbumin.
Three of the amino acids obtained from the complete hydrolysis of ovalbumin and their
isoelectric points are given in the table below.
Amino acid Formula of side chain Isoelectric point
Aspartic acid (asp) CH 2 CO 2 H 2.77
Lysine (lys) (CH 2 ) 4 NH 2 9.74
Tyrosine (tyr) 5.66

CH2 OH

(a) State how ovalbumin could be hydrolysed in the laboratory to form the amino acids [1]
in the table above.
Aqueous acid / alkali (must specify),
heat for several hours / prolonged heating
Or
Conc HCl (6 mol dm3), heat for 24 hours

(b) (i) Define the term isoelectric point. [1]


Isoelectric point is defined as the pH at which the amino acid carries no net
electrical charge.
(ii) Electrophoresis is an electrolytic method used to separate amino acids based
on their isoelectric points. A simplified diagram of an electrophoresis set-up is
shown below. Depending on the pH of the buffer solution used, the amino
acids will move differently. The positions of the amino acids are ascertained by

trendyline
spraying
g the solution with ninhydrin solution. The colourless amino
ng amin acid spots
will turn
rn
n blue
blue.

cathode
1341 trendyline
19

Mixture of amino anode


acids

Buffer solution

Using data from the table above, explain how electrophoresis can be used to
separate and identify the constituent amino acids in a mixture of aspartic acid,
lysine and tyrosine in a buffer solution of pH 8. You should include the
structural formulae of the three amino acids at pH 8 in your answer. [3]
At pH 8:
COO

H2N

COO

COOH

H2N

H3N (CH2)4NH3
COO
O
aspartic acid lysine tyrosine
Lysine has an overall positive charge, hence it will move towards the
cathode.
Aspartic acid and tyrosine both have an overall negative charge, hence both
will move towards the anode.
Since tyrosine has a larger mass (or heavier), it will move slower than
aspartic acid, hence it is closer to the middle. (Or vice versa)

(c) With reference to protein structure, explain why egg white coagulates when cooked. [2]
Heatwill disrupt hydrogen bonds and van der Waals forces, causing the secondary
and tertiary(and quaternary) structures to be destroyed / denatured. Hence, the

trendyline
egg white coagulates.

(d) Besides reacting


acting
cting with
h amino acids, ninhydrin is also commonly used in fingerprint
f

1342 trendyline
20

analysis in criminal investigations. Another method to analyse fingerprints is called


cyanoacrylate (Super Glue) fuming. One common cyanoacrylate used for this
purpose is ethyl cyanoacrylate. The structures of ninhydrin and cyanoacrylate are
given below.

O
O
OH

OH O

CN

O
ninhydrin ethyl cyanoacrylate

Suggest a chemical test that could be used to distinguish ninhydrin and ethyl
cyanoacrylate from each other. You should state what you would observe for each
compound in the test. [2]
Add 2,4-DNPH to each compound separately. Orange ppt formed for ninhydrin but
no ppt formed for ethyl cyanoacrylate.
OR
Add cold KMnO 4 and H 2 SO 4 (aq) to each compound separately. Purple solution
decolourised for ethyl cyanoacrylate but remains purple for ninhydrin.
OR
Add cold KMnO 4 and NaOH (aq) to each compound separately. Purple solution
decolourised for ethyl cyanoacrylate and brown ppt formed. But solution remains
purple for ninhydrin.
OR
Add Br 2 (aq) to each compound separately. Orange solution decolourised for ethyl
cyanoacrylate but remains orange for ninhydrin.

(e) Besides being rich in protein, eggs are also consumed widely as it contains vitamins
and minerals needed on a daily basis. An example of a key nutrient that can be
obtained from eating eggs is phosphorus, which is as important as calcium in
building strong bones in our body.

trendyline
(i) ibe the reaction of phosphorus(
Describe phosphorus III
II ) oxide,
xide, P 4 O 6 , with water.
water Include the
wate
ue of the resulting solution, and write an equation for the reaction that [2]
lue
pH value
s.
occurs.

1343 trendyline
21

P 4 O 6 reacts vigorously (or readily or violently) with water.


pH = 3
P 4 O 6 + 6H 2 O 4H 3 PO 3
(ii) Write an equation to illustrate the acid-base nature of phosphorus(V) oxide,
P 4 O 10 . [1]
P 4 O 10 + 12NaOH 4Na 3 PO 4 + 6H 2 O

(f) Eggs are also rich in iodine which is a key component of hormones in the thyroid
gland. Iodine deficiency results when we do not consume enough iodine in our diet.
In order to combat iodine deficiency, table salt is mixed with various salts of iodine in
the production of iodised salts.
Potassium iodate, KIO 3 , is often used in iodised salts. With the aid of an equation,
describe how potassium iodate can be prepared using iodine as a starting material. [2]
Add iodine to KOH
3I 2 + 6KOH KIO 3 + 5KI + 3H 2 O

(g) Describe what you would see if concentrated sulfuric acid is added to separate
samples of solids KBr and KI. Suggest an explanation for the differences in
reaction, and write equations for the reactions that occurred. [4]
KX + H 2 SO 4 HX + KHSO 4
2HX + H 2 SO 4 SO 2 + X 2 + 2H 2 O
X = Br and I
White fumes are seen.
Reddish-brown fumes are seen for KBrwhile violet vapouris observed for KI.
8HI + H 2 SO 4 H 2 S + 4I 2 + 4H 2 O
Iodide ion is more easily oxidisedthan bromide ion OR iodide ion is a better reducing
agentthan bromide ion, hence further reaction will take place for KI.

trendyline
(h) Iodine has many isotopes. Only two of its isotopes are used in medicine
medicine, such as
pyy and imaging. In order to identify the mass of these two isot
radiotherapy isotopes, they

1344 trendyline
22

were first ionised to form singly-charged ions of +1 charge and passed through an
electric field of constant strength. Their angles of deflection, along with a reference
sample of helium nucleus, 42He, were recorded in the table below.
Angle of deflection
Isotope X 1.07
Isotope Y 1.14
Heliumnucleus, 42He 70.1

Calculate the mass of each isotope, giving your answer to the nearest whole
number. [2]

2
= 70.1
4
1
= 1.07

2
( ) 70.1
4
1 =
( ) 1.07

x = 131
1
= 1.14

2
( ) 70.1
4
1 =
( ) 1.14

y = 123

[Total: 20]

trendyline
1345 trendyline
1

SERANGOON JUNIOR COLLEGE


General Certificate of Education Advanced Level
Higher 2

Candidate Name

Class

CHEMISTRY 9647/01
JC2 PreliminaryExamination 23 September2016
Paper 1 Multiple Choice 1 hour
Additional Materials: Data Booklet
Optical Mark Sheet (OMS)

READ THESE INSTRUCTIONS FIRST

On the separate multiple choice OMS given, write your name, subject title and class in the
spaces provided.
Shade correctly your FIN/NRIC number.

There are 40 questions in this paper. Answer all questions.


For each question there are four possible answers A, B, C and D.
Choose the one you consider correct and record your choice using a soft pencil on the
separate OMS.
Each correct answer will score one mark.
A mark will not be deducted for a wrong answer.

You are advised to fill in the OMS as you go along; no additional time will be given for the
transfer of answers once the examination has ended.

Any rough working should be done in this question paper.

trendyline
This document consists of18printed
of18
of1 printed pages.

Turn Over]
1346 trendyline
2

Answer all questions


1 The relative abundances of the isotopes of a sample of titanium are shown in the table
below.
Relative Isotopic Mass 46 47 48 49 50
Relative Abundance 11.2 10.1 100.0 7.3 7.0

What is the relative atomic mass of titanium in this sample?

A 48.00
B 47.92
C 47.90
D 47.89

2 To identify an oxide of nitrogen, 0.10 mol of the oxide is mixed with an excess of
hydrogen and passed over a catalyst at a suitable temperature.

H 2 (g)
N x O yxNH 3 + yH 2 O

The water produced weighs 7.20 g. The ammonia produced is neutralised by 200 cm3
of 1.0 mol dm-3 HCl.

What is the formula of the oxide of nitrogen?

A N2O
B NO
C NO 2
D N2O4

3 50 cm3 of a 0.10 mol dm-3 solution of a metallic salt was found to react exactly with
25.0 cm3 of 0.10 mol dm-3 aqueous sodium sulfite. In this reaction, the sulfite ion is
oxidised as follows.
2- 2- +
SO 3 (aq) + H 2 O(l)SO 4 (aq) + 2H (aq) + 2e

What is the new oxidation number of the metal in the salt if its original oxidation number
was +3?

A +1
B +2

trendyline
C +4
D +5

Turn Over]
1347 trendyline
3

4 Of the following, which is the strongest oxidising agent?

A O2+
B O2
C O2
D O 2 2

5 Paramagnetism refers to the magnetic state of an atom with one or more unpaired
electrons. The greater the number of unpaired electrons in an atom, the greater the
paramagnetism.

Which atom has the greatest paramagnetism?

A Oxygen
B Chlorine
C Scandium
D Arsenic

6 Which of the following is true of both real and ideal gases?

A Molecules can be liquefied.


B Molecules occupy a finite volume.
C Molecules are in constant random motion.
D Molecules behave identically at high pressure and low temperature.

7 The Valence Shell Electron Pair Repulsion Theory (VSEPR) is used to predict the
shapes of molecules.

Which shape is correctly predicted by VSEPR?

number of regions number of lone pairs shape


of electron density
A 3 1 Tetrahedral
B 3 1 Trigonal pyramidal
C 5 1 See-saw
D 5 1 Square pyramidal

8 trendyline
nitude
tude of the lattice energy of calcium oxide and calcium carbonate is
The magnitude
ol1 and 2814 kJmol1 respectively.
3414 kJmol respectivelyy The
T enthalpy change of decomposition
d

Turn Over]
1348 trendyline
4

of calcium carbonate is +176 kJmol1.

Using the information provided, calculate the enthalpy change of the following
reaction.
CO 3 2(g) O2(g) + CO 2 (g)

A - 424 kJ mol1
B +424 kJmol1
C - 776 kJmol1
D +776 kJmol1

9 Primary, secondary and tertiary amine have similarly shaped molecules.

What is the predominant intermolecular force of attraction in methylamine,


dimethylamine and trimethylamine?

methylamine dimethylamine trimethylamine


A hydrogen bonds hydrogen bonds hydrogen bonds
B hydrogen bonds permanent dipole Instantaneous dipole
permanent dipole induced dipole
C permanent dipole permanent dipole permanent dipole
permanent dipole permanent dipole permanent dipole
D hydrogen bonds hydrogen bonds permanent dipole
permanent dipole

10 The integrated form of first-order rate law is as shown.

ln A = kt + lnA o

where A is the activity at time t,


A o is the initial activity,
k is the rate constant,
t is the time taken.

What is the half-life for a first order reaction if 68% of a substance is reacted within
66s?

A 33 s
B 40 s
C 49 s
D 119 s

11 h equilibrium is K c = K p ?
For which

Turn Over]
1349 trendyline
5

A C(s) + H 2 O(l) CO(g) + H 2 (g)

B H 2 (g) + I 2 (g) 2HI(g)

C N 2 (g) + 3H 2 (g) 2NH 3 (g)

D 2NO 2 (g) N 2 O 4 (g)

12 The ionic product of water, K w , at two different temperatures is shown below,

K w / mol2 dm6 Temperature / K


1.00 x 1014 298
1.44 x 1014 303

Which statement is correct?

A Self-ionisation of water is an exothermic process.


B At 303 K, [H+] = 0.72 x 1014 mol dm3
C [H ][OH ]
Kw =
[H2O]
D At 303 K, pH < 7

13 Calculate the pH of the resulting solution when 10 cm3 of potassium hydroxide with
7 3
concentration at 7.5 x 10 mol dm is mixed with an equal volume of
8 
hydrogen bromide with concentration at 8.5 x 10 mol dm .

A 4.18
B 6.36
C 7.64
D 9.82

14 Aqueous hydrochloric acid was electrolysed for ten minutes, 200 cm3 of gasA was
collected at the anode.
The same current was then applied to concentrated sodium chloride solution in
another experiment and 400 cm3 of a gasB is collected at the cathode.

Which of these statements is correct?

A GasB is pale yellow.


B The time taken for the second electrolysis is also ten minutes.

trendyline
C ne gas was collected initiallyy in the first electrolysis system.
Chlorine
D Mercuryry
y electrodes can be used for the second electrolysis system if gas B is to
lected.
ected
be collected.
15 he
e following properties could be predicted for strontium or itits compounds?
Which of the

Turn Over]
1350 trendyline
6

A It does not burn in air.


B It forms a soluble sulfate.
C It reacts with cold water, liberating hydrogen.
D It forms a water-soluble carbonate which does not decompose on heating.

16 Equal volumes of chlorine gas were bubbled into hot and cold sodium hydroxide
separately until reactions were complete.

Which of the following statements is incorrect about the above reactions?

A Disproportionation reactions will occur.

B Greenish-yellow chlorine gas decolourises in both reactions.

C When aqueous silver nitrate is added to the resulting solutions, white precipitate
is formed.

D Sodium chlorate(VII) and sodium chlorate(I) are formed in the reactions


respectively.

17 Aqueous chlorine is added to aqueous sodium bromide and the mixture is shaken
with an equal volume of trichloromethane.

Which observation will be made?

A The solution in the test-tube turns colourless.


B The solution in the test-tube turns orange.
C A colourless layer forms on top of a purple layer.
D A colourless layer forms on top of an orange layer.

18 A reaction scheme starting from aqueous copper(II) sulfate solution is shown below.
Both G and H are copper-containing species.

excess
NH3(aq) NH3(aq) Na4edta(aq) 2-
CuSO4(aq) G H [Cu(edta)] (aq)
I II III

Which of the following statements is correct?

A Reaction III is a redox reaction.


B NH 3 acts as a ligand in reactionI.

trendyline
C H is a deep blue solution containing [Cu(NH 3 ) 4 (H
H 2 O) 2 ]SO 4 .
D The entropy
ntropy of the system reactionIIIoccurs.
syste decreases when reaction
reactionIII
II occurs.
ccurs

Turn Over]
1351 trendyline
7

19 Stigmasterol is an unsaturated plant sterol occurring in the plant fats of soybean.

How many stereoisomers does stigmasterol have?

A 29
B 210
C 211
D 212

20. Four carbon-carbon bonds are labelled in the diagram.

x
CH

y
z

Which bonds are made up of a sp2-sp2 overlap?

A x only

B x and y only

C w, x and y only

D w, y and z only

trendyline
Turn Over]
1352 trendyline
8

21. When heated with chlorine, the hydrocarbon 2,2,3,4-tetramethylpentane undergoes


free radical substitution. In a propagation step, the free radical J is formed by the
loss of one hydrogen atom.

CH3

H3C CH3
+ Cl J + HCl
H3C
CH3 CH3

How many different forms of J are theoretically possible?

A 3

B 4

C 5

D 6

22 Saccharin was an artificial sweetener used in some soft drinks. It was manufactured
from methylbenzene by a series of reactions.

CH3 CH3 CH3

ClSO3H SO2Cl NH3 SO2NH2

I II

Which of the following shows the correct reaction type for steps I and II?

I II
A Electrophilic substitution Electrophilic addition
B Electrophilic substitution Nucleophilic substitution
C Nucleophilic substitution Nucleophilic substitution
D Nucleophilic substitution Electrophilic substitution

trendyline
Turn Over]
1353 trendyline
9

23 Which of the following cannot be used to distinguish between the following compounds?

O O HO CHO

A Hot acidified K 2 Cr 2 O 7

B Neutral iron(III) chloride

C Diammine silver complex

D Phenylhydrazine

24 The reaction scheme below shows the synthesis of compound S.

Cl Br

CH3 NaOH(aq) K2Cr2O7/H2SO4(aq)


S
heat heat

Which of the following can be S?

A Cl O C OH O
O
CH3 CH3 C
C

OH
OH
C

B OH O D O Cl O

CH3 C C
HO OH

OH
C

trendyline
Turn Over]
1354 trendyline
10

25 The following synthetic route consists of three steps.

CH3 COOH

NO 2
Br

Which sequence of steps would give the highest yield?

Step 1 Step 2 Step 3

A KMnO 4 , H 2 SO 4 , heat conc HNO 3 , conc H 2 SO 4 , Br 2 , Fe, r.t.p


heat

B conc HNO 3 , conc H 2 SO 4 , Br 2 , Fe, r.t.p KMnO 4 , H 2 SO 4 , heat


heat

C Br 2 , AlBr 3 , r.t.p conc HNO 3 , dilute H 2 SO 4 , KMnO 4 , H 2 SO 4 , heat


heat

D Br 2 , Fe, r.t.p KMnO 4 , H 2 SO 4 , heat conc HNO 3 , conc H 2 SO 4 ,


heat

trendyline
Turn Over]
1355 trendyline
11

26 Ten grams of each of the following was heated with NaOH(aq) for a prolong period of
time. Subsequently, dilute HNO 3 (aq) and AgNO 3 (aq) were added.

Which compound will produce the greatest mass of silver halide precipitate?

A Br C Br

(M r = 134.9) Br(M r = 235.8)

B CH 3 COCl D Cl
(M r = 78.5)

Cl
(M r = 153.0)

27 After the reduction using tin and concentrated acid of nitrobenzene to phenylamine, an
excess of sodium hydroxide is added.

What is the purpose of the sodium hydroxide?

A to dry the product

B to liberate the phenylamine

C to neutralise the excess acid

D to lower the boiling point for subsequent distillation

28 Which of the following shows the correct order of basicity?

High pK b Low pK b
A
CH2 NH2 CH2 NH2 NH2 CONH2

B
CONH2 CH2 NH2 NH2 CH2 NH2

C
CONH2 NH2 CH2 NH2 CH2 NH2

D
t end
trendyline
dyyl ne
NH2ne ONH2
CO
CONH H2 NH2
CH CH
C H2 NH2

Turn Over]
1356 trendyline
12

29 Aspartame is a dipeptide derivatives used as an artificial sweetener. Its general


usefulness is restricted because it loses its sweetness after hydrolysis.

H2NCHCONHCHCO2CH3

CH2 CH2
- +
CO2 Na

Which product would be formed after prolong acid hydrolysis.

A + +
H3NCHCO2H H3NCHCO2CH3
CH2
- + CH2
CO2 Na
and

B +
H3NCHCO2H H2NCHCO2CH3
CH2
CH2
COOH and

C + +
H3NCHCO2H H3NCHCO2H
CH2
CH2
COOH
and andCH 3 OH

D +
H3NCHCO2H CH2CO2H
CH2
CH2
COOH and andCH 3 OH

Turn Over]
1357 trendyline
13

30 Electrophoresis is a technique of separating and identifying amino acids. A solution of


amino acids is absorbed into paper that is moistened with a buffer solution and
stretched between two electrodes. Positively charged species move towards the
cathode, negatively charged species towards the anode.

With a buffer at pH 5, which amino acid will move most readily towards the cathode?

A O C NH2 O

HO OH O OH
NH2 NH2

B O O D O

HO OH OH
NH2 NH2

trendyline
Turn Over]
1358 trendyline
14

For questions 31 40, one or more of the numbered statements 1 to 3 may be correct.
Decide whether each of the statements is or is not correct. The responses A to D should be
selected on the basis of

A B C D
1, 2 and 3 1 and 2 only 2 and 3 only 1 only
are correct are correct are correct is correct

No other combination of statements is to be used as correct response.

31 The graph below shows the first twelve ionisation energies for element T.

ionisation
energy

number of electrons removed

Which of the following statements are true?

1 It is in Group Iof the Periodic Table.

2 It forms an oxide which dissolves readily with acid.

3 It is in the third period (Na to Ar) of the Periodic Table.

32 The following chemical reactions are listed below.

Combustion of ethandioic acid


Evaporation of water
Atomisation of magnesium
Photolysis of chlorine

Which of the following statements are correct?

1
2
3 trendyline
S is positive for all reactions
reactions.
G is negative for all reactions.
H is positive
ositive for all reactions
reactions.

Turn Over]
1359 trendyline
15

33 A reaction in which a product acts as a catalyst is said to be autocatalytic.

2MnO 4 -(aq) + 5C 2 O 4 2-(aq) + 16H+(aq) 2Mn2+(aq) + 10CO 2 (g) + 8H 2 O(l)

In the oxidation of ethanedioate ions by manganate(VII) ions, the reaction is


accelerated by the Mn2+ ions produced during the reaction.

Graph of [MnO4-] against time

[MnO4-]/mol dm-3

(1) (1) Initially the reaction is slow.

(2) But the reaction rate increases as


(2) Mn2+ ions catalyse the reacton.

(3) The reaction rate falls the concentrations


of reactants decrease.

(3)

0 time/s

Which of the following statements can be deduced from the graph above?

1 Initial rate of reaction is the fastest at (1).

2 Reaction rate increases at (2) as Mn2+ is generated.

3 Reaction rate decreases at (3) as the concentration of the reactants


decrease.

34 The graph below shows how the percentage of reactant U(g) that remained in an
equilbrium mixture varies with 1/T at pressures of 1.5atm and 2 atm.

2 atm

%U (g) at 1.5 atm


equilibrium

trendyline
Which of the
e following statement can be deduced from this information?
he

1 The forward endothermic.


orward reaction is endothermic

2 The equation for


for the above reaction could beU(g)
beU
beU(g) V(g)) + W
g) V(g) W(g).
(g).
(g)

Turn Over]
1360 trendyline
16

3 The equilibrium constant, K p , increases as pressure increases in the system.

A B C D
1, 2 and 3 1 and 2 only 2 and 3 only 1 only
are correct are correct are correct is correct

35 A student was investigating the possibility of an electrochemical cell using Cu2+/Cu+


and Fe3+/Fe2+ half-cells.

Pt electrode Pt electrode
Salt Bridge

1 mol dm3 Cu+ (aq)


1 mol dm3 Fe2+ (aq)
3 2+
1 ld C ( ) 3 3+
1 ld F ( )
Which statements are true of the above set up?

1 Eocell = +0.62 V.
2 Copper and iron electrodes cannot be used instead.
3 When excess sodium hydroxide is added to the Fe2+/Fe3+ half-cell, the polarity of
the electrodes are reversed.

36 Barium sulfate is less soluble than magnesium sulfate.

Which of these factors are needed to be considered in order to explain this


observation?

1 H hyd of barium ion and magnesium ion.


2 H f of barium sulfate and magnesium sulfate.
3 Atomic radii of barium and magnesium.

37 Which observation about bromine or its compounds is correct?

1 When aqueous lead(II) nitrate is added to aqueous sodium bromide, a cream


precipitate is observed.

trendyline
2 When sodium bromide is treated with concentrated sulfuric acid, a gas that turns
moist blue litmus paper red is evolved
evolved.

3 Silver bromide is soluble in both dilute


ute and concentrated ammonia solution.

Turn Over]
1361 trendyline
17

A B C D
1, 2 and 3 1 and 2 only 2 and 3 only 1 only
are correct are correct are correct is correct

38 The catalytic converter is part of the exhaust system of modern cars.


Which reactions occur in the catalytic converter?
1 2CO + O 2 &2 2

2 12&21 2 + 2CO 2

CxHy + (2x+ y ) 12xCO 2 + y H 2 O+ (x+ ) N 2


y
3
2 2 4

39 Halogenoalkanes react with aqueous alkali. One mechanism of this reaction has the
reaction pathway diagram shown below.
Energy
X


Y
R-Hal +OH

R-OH + Hal

Reaction Pathway
Which statements are true? ( indicates a partial bond)

1 The reaction is a type of nucleophilic substitution.

2 X is C Hal
-

trre
e
3 Y is HO C H
Hal

Turn Over]
1362 trendyline
18

40 Which of the following phenomena involves denaturation of proteins?


1 Heating of egg white.
2 Production of bean-curd from soy milk.
3 Dissolving Penicillin in a test tube of hot acid.

Penicillin

END OF PAPER 1

trendyline
Turn Over]
1363 trendyline



6(5$1*221-81,25&2//(*(
*HQHUDO&HUWLILFDWHRI(GXFDWLRQ$GYDQFHG/HYHO
+LJKHU

&DQGLGDWH1DPH

&ODVV

&+(0,675< 
-&3UHOLPLQDU\([DPLQDWLRQ 6HS
3DSHU0XOWLSOH&KRLFH KRXU
$GGLWLRQDO0DWHULDOV 'DWD%RRNOHW
2SWLFDO0DUN6KHHW 206 

5($'7+(6(,16758&7,216),567

2QWKHVHSDUDWHPXOWLSOHFKRLFH206JLYHQZULWH\RXUQDPHVXEMHFWWLWOHDQGFODVVLQWKH
VSDFHVSURYLGHG
6KDGHFRUUHFWO\\RXU),115,&QXPEHU

7KHUHDUHTXHVWLRQVLQWKLVSDSHU$QVZHUDOOTXHVWLRQV
)RUHDFKTXHVWLRQWKHUHDUHIRXUSRVVLEOHDQVZHUV$%&DQG'
&KRRVH WKH RQH \RX FRQVLGHU FRUUHFW DQG UHFRUG \RXU FKRLFH XVLQJ D VRIW SHQFLO RQ WKH
VHSDUDWH206
(DFKFRUUHFWDQVZHUZLOOVFRUHRQHPDUN
$PDUNZLOOQRWEHGHGXFWHGIRUDZURQJDQVZHU

<RXDUHDGYLVHGWRILOOLQWKH206DV\RXJRDORQJQRDGGLWLRQDOWLPHZLOOEHJLYHQIRUWKH
WUDQVIHURIDQVZHUVRQFHWKHH[DPLQDWLRQKDVHQGHG

$Q\URXJKZRUNLQJVKRXOGEHGRQHLQWKLVTXHVWLRQSDSHU

<
 %  %  &  &
 '  '  %  '
 %  &  '  &
 $  %  $  %
 '  &  '  $
 &  '  '  '
 &  '  %  %
 '  &  &  $
 '  %  &  %
 %  %  '  %

trendyline
7KLVGRFXPHQWFRQVLVWVRISULQWHGSDJHV
7KLVGRFXPHQWFRQVLVWVRI
LVGRFXPHQWFRQVLVWVRI
I SULQWHGSDJHV

1364 trendyline



6XJJHVWHG:RUNHG6ROXWLRQIRU0&4V
 7KHUHODWLYHDEXQGDQFHVRIWKHLVRWRSHVRIDVDPSOHRIWLWDQLXPDUHVKRZQLQWKHWDEOH
 EHORZ
5HODWLYH,VRWRSLF0DVV     
5HODWLYH$EXQGDQFH     

:KDWLVWKHUHODWLYHDWRPLFPDVVRIWLWDQLXPLQWKLVVDPSOH"

 $ 
 % 
 & 
 ' 
$QVZHU%

         


5HODWLYHDWRPLFPDVV  
    
    
 


   


 7R LGHQWLI\ DQ R[LGH RI QLWURJHQ  PRO RI WKH R[LGH LV PL[HG ZLWK DQ H[FHVV RI
K\GURJHQDQGSDVVHGRYHUDFDWDO\VWDWDVXLWDEOHWHPSHUDWXUH

+ J 
1[2\[1+\+2

7KHZDWHUSURGXFHGZHLJKVJ7KHDPPRQLDSURGXFHGLVQHXWUDOLVHGE\FP
RIPROGP+&l

:KDWLVWKHIRUPXODRIWKHR[LGHRIQLWURJHQ"

 $ 12
 % 12
 & 12
 ' 12
$QVZHU' $VVXPLQJ DOO JDVHV DQG XVLQJ $YRJDGURV
 /DZ
$PWRI+2   PRO &RPSDULQJPROHUDWLRRI1[2\ZLWK1+

  
+HQFH[ 
$PWRI1+    PRO [ 



trend
dyline
dyylli
&RPSDULQJPROHUDWLRRI1[2\ZLWK+2
 
+HQFH\
+HQFH\ 
\ 

)RUPXODLV1
RUPXODLV12


1365 trendyline



  FP RI D  PRO GP VROXWLRQ RI D PHWDOOLF VDOW ZDV IRXQG WR UHDFW H[DFWO\ ZLWK
  FP RI  PRO GP DTXHRXV VRGLXP VXOILWH ,Q WKLV UHDFWLRQ WKH VXOILWH LRQ LV
R[LGLVHGDVIROORZV

  
62 DT +2 l 62 DT + DT H

:KDWLVWKHQHZR[LGDWLRQQXPEHURIWKHPHWDOLQWKHVDOWLILWVRULJLQDOR[LGDWLRQQXPEHU
ZDV"

 $ 
 % 
 & 
 ' 
$QVZHU%

$PRXQWRIVXOSKLWHLRQV   PRO



$PRXQWRIPHWDOOLFVDOW    PRO

/HW[EHWKHQHZR[LGDWLRQQRRIPHWDOLQVDOW

>5@0 [ H0[

6LQFHPROHVRIHOHFWURQVJDLQHG PROHVRIHOHFWURQVORVWLQDUHGR[UHDFWLRQ

 x 

 
[ 


 2IWKHIROORZLQJZKLFKLVWKHVWURQJHVWR[LGLVLQJDJHQW"

 $ 2
 % 2
 & 2
 ' 2
$QVZHU$

6WURQJHVWR[LGLVLQJDJHQWPHDQVLWZLOOXQGHUJRUHGXFWLRQHDVLO\DQGDVVXFKZLOOJDLQ
HOHFWURQV

2LVSRVLWLYHO\FKDUJHGDQGKHQFHLVDVWURQJHUHOHFWURQDFFHSWRUWKDQWKHRWKHUVSHFLHV


trendyline


1366 trendyline



 3DUDPDJQHWLVP UHIHUV WR WKH PDJQHWLF VWDWH RI DQ DWRP ZLWK RQH RU PRUH XQSDLUHG
HOHFWURQV7KHJUHDWHUWKHQXPEHURIXQSDLUHGHOHFWURQVLQDQDWRPWKHJUHDWHUWKH
SDUDPDJQHWLVP

:KLFKDWRPKDVWKHJUHDWHVWSDUDPDJQHWLVP"

 $ 2[\JHQ
 % &KORULQH
 & 6FDQGLXP
 ' $UVHQLF
$QVZHU'

2VVSXQSDLUHGHOHFWURQVLQWKHSRUELWDO

&O>1H@VSXQSDLUHGHOHFWURQLQWKHSRUELWDO

6F>$U@GVXQSDLUHGHOHFWURQLQWKHGRUELWDO

$V>$U@GVSXQSDLUHGHOHFWURQVLQWKHSRUELWDO

 :KLFKRIWKHIROORZLQJLVWUXHRIERWKUHDODQGLGHDOJDVHV"

$ 0ROHFXOHVFDQEHOLTXHILHG
% 0ROHFXOHVRFFXS\DILQLWHYROXPH
& 0ROHFXOHVDUHLQFRQVWDQWUDQGRPPRWLRQ
' 0ROHFXOHVEHKDYHLGHQWLFDOO\DWKLJKSUHVVXUHDQGORZWHPSHUDWXUH
$QVZHU&
0ROHFXOHVFDQEHOLTXHILHG 1RWWUXHLGHDOJDVGRQRWH[HUWDWWUDFWLYHIRUFHVRQRQH
DQRWKHU 

0ROHFXOHVRFFXS\DILQLWHYROXPH 1RWWUXHWKHVL]HRIWKHLGHDOJDVPROHFXOHLV
DVVXPHGWREHQHJOLJLEOHFRPSDUHGWRWKHYROXPHRIWKHFRQWDLQHULWRFFXSLHV 

0ROHFXOHVDUHLQFRQVWDQWUDQGRPPRWLRQ 7UXH 

0ROHFXOHVEHKDYHLGHQWLFDOO\DWKLJKSUHVVXUHDQGORZWHPSHUDWXUH 1RWWUXHUHDOJDVHV
EHKDYHPRVWLGHDOO\DWKLJKWHPSHUDWXUHDQGORZSUHVVXUH 


trendyline
y
7KH9DOHQFH6KHOO(OHFWURQ3DLU5HSXOVLRQ7KHRU\ 96(35 LVXVHGWRSUHGLFWWKH

1367 trendyline



VKDSHVRIPROHFXOHV

:KLFKVKDSHLVFRUUHFWO\SUHGLFWHGE\96(35"

 QXPEHURIUHJLRQV QXPEHURIORQHSDLUV VKDSH
RIHOHFWURQGHQVLW\
$   7HWUDKHGUDO
%   7ULJRQDOS\UDPLGDO
&   6HHVDZ
'   6TXDUHS\UDPLGDO
$QVZHU&
UHJLRQVRIHOHFWURQGHQVLW\FRQVLVWLQJRIORQHSDLUVHHVDZVKDSH
)
)
6
)
)RUH[DPSOH ) KDVDVHHVDZVKDSH

 7KH PDJQLWXGH RI WKH ODWWLFH HQHUJ\ RI FDOFLXP R[LGH DQG FDOFLXP FDUERQDWH LV
N-PRODQGN-PROUHVSHFWLYHO\7KHHQWKDOS\FKDQJHRIGHFRPSRVLWLRQ
RIFDOFLXPFDUERQDWHLVN-PRO

8VLQJ WKH LQIRUPDWLRQ SURYLGHG FDOFXODWH WKH HQWKDOS\ FKDQJH RI WKH IROORZLQJ
UHDFWLRQ
&2 J 2 J &2 J 

$ N-PRO
% N-PRO
& N-PRO
' N-PRO
$QVZHU'

+UHDFWLRQ
&D J &2 J 2 J &2 J &D J

/( &D&2 
/( &D2 

+GHFRPSRVLWLRQ 
&D&2 V &D2 V &2 J
 

%\+HVV/DZ 
+UHDFWLRQ/( &D2  /( &D&2 +GHFRPSRVLWLRQRI&D&2
+UHDFWLRQ /( &D&2 +GHFRPSRVLWLRQRI&D&2/( &D2 
+UHDFWLRQ  N-PRO

2U

ttrendyline
e dy
dyli e
8VLQJDOJHEUDLFPHWKRG
PH
 &D J 2 J
 J &D2 V 
&D2 V 
 

 &D  J &22 J &D&2
J &D&2 V 
V 
 &D&2 V &D2 V &2
&D2 V &2 J 
J 

2EMHFWLYH&2 J 2 J &2 J """

1368 trendyline



 UHYHUVH&D2 V &D J 2 J 


 &D J &2 J &D&2 V 
 &D&2 V &D2 V &2 J 

$LPDFKLHYHG&2 J 2 J &2 J 

 3ULPDU\VHFRQGDU\DQGWHUWLDU\DPLQHKDYHVLPLODUO\VKDSHGPROHFXOHV
:KDW LV WKH SUHGRPLQDQW LQWHUPROHFXODU IRUFH RI DWWUDFWLRQ LQ PHWK\ODPLQH
GLPHWK\ODPLQHDQGWULPHWK\ODPLQH"
 PHWK\ODPLQH GLPHWK\ODPLQH WULPHWK\ODPLQH
$ K\GURJHQERQGV K\GURJHQERQGV K\GURJHQERQGV
% K\GURJHQERQGV SHUPDQHQW GLSROH  ,QVWDQWDQHRXV GLSROH 
SHUPDQHQWGLSROH LQGXFHGGLSROH
& SHUPDQHQW GLSROH  SHUPDQHQW GLSROH  SHUPDQHQW GLSROH 
SHUPDQHQWGLSROH SHUPDQHQWGLSROH SHUPDQHQWGLSROH
' K\GURJHQERQGV K\GURJHQERQGV SHUPDQHQW GLSROH 
SHUPDQHQWGLSROH
$QVZHU'
1 1
+ &+ 1
+ + &+ + & &+
&+  &+ 
IURPOHIWWRULJKW WKHVWURQJHVWLQWHUPROHFXODUIRUFHLQPHWK\ODPLQHGLPHWK\ODPLQHDQG
WULPHWK\ODPLQHDUHK\GURJHQERQGVK\GURJHQERQGVDQGSHUPDQHQWGLSROHSHUPDQHQW
GLSROHUHVSHFWLYHO\7ULPHWK\ODPLQHGRHVQRWKDYHK\GURJHQERQGVEHFDXVHWKHUHDUHQR
K\GURJHQDWRPVGLUHFWO\ERQGHGWRWKHQLWURJHQDWRP

 7KHLQWHJUDWHGIRUPRIILUVWRUGHUUDWHODZLVDVVKRZQ
OQ$ NWOQ$R
ZKHUH$LVWKHDFWLYLW\DWWLPHW
$RLVWKHLQLWLDODFWLYLW\
NLVWKHUDWHFRQVWDQW
WLVWKHWLPHWDNHQ
:KDW LV WKH KDOIOLIH IRU D ILUVW RUGHU UHDFWLRQ LI  RI D VXEVWDQFH LV UHDFWHG ZLWKLQ
V"
$ V
% V
& V
' V
$QVZHU%
UHDFWHGPHDQVUHPDLQ
OQ N  OQ
N V
OQ OQ
8VLQJW  W  W VHFRQG
N 

t d li
trendyline
25
&I&R   Q
   Q
Q RQHKDOIOLIH
IOLIH  V

1369 trendyline



 )RUZKLFKHTXLOLEULXPLVKc Kp"



$ & V +2 l  &2 J + J 

% + J , J  +, J 

& 1 J + J  1+ J 

' 12 J  12 J 
$QVZHU%
:KHQWKHUHDUHHTXDOQXPEHURIPROHVRQHDFKVLGHRIWKHVWRLFKLRPHWULFHTXDWLRQ
Kc Kp

FRQVLGHULQJLGHDOJDVHVS9 Q57ZKLFKLPSOLHV

Q S
&RQFHQWUDWLRQRIDJDV   
9 57

&RQVLGHUWKHUHYHUVLEOHJDVHRXVV\VWHP D$ J E% J F& J G' J 


SF F S' G
>&@F >'@G S& F S' G
57 57 57 DE  FG KS 57 DE  FG 
KF D E  S   D
S $ S% E
>$@ >%@ $ D S% E

57 57

 7KHLRQLFSURGXFWRIZDWHU.ZDWWZRGLIIHUHQWWHPSHUDWXUHVLVVKRZQEHORZ

.ZPROGP 7HPSHUDWXUH.
[ 

[  

:KLFKVWDWHPHQWLVFRUUHFW"

$ 6HOILRQLVDWLRQRIZDWHULVDQH[RWKHUPLFSURFHVV
% $W.>+@ [PROGP
>+ @>2+ @
& .Z  
>+2@
' $W.S+
$QVZHU'
+2 +2+LVDQHQGRWKHUPLFSURFHVV

$W.Z >+@>2+@[ [ ZKHUH[LVWKH>+@ 


[ [PROGP
WKDWPHDQV>+@ [PROGP
S+   


t d li
trendyline
KHS+RIWKHUHVXOWLQJVROXWLRQZKHQFPRISRWDVVLXPK
WKHS+RIWKHUHVXOWLQJVROXWLRQZKHQFP
&DOFXODWHWKHS+RIWKHUHVXOWLQJVROXWLRQZKHQFP
LRQ
RQ DW
FRQFHQWUDWLRQ DW 

 [[  PRO



PRO GP  LV
RISRWDVVLXPK\GUR[LGHZLWK
RISRWDVVLXPK
V PL[HG ZLWK DQ HTXDO

EURPLGH ZLWK FRQFHQWUDWLRQ DW   [  PRO
K\GURJHQEURPLGHZLWKFRQFHQWUDWLRQDW[ PROGP

GP 
HTXD
HTX YROXPH RI

1370 trendyline




$ 
% 
& 
' 
$QVZHU&
.2++%U.%U+2

$PWRI.2+ [PRO$PWRI+%U [PRO


6LQFH+%ULVWKHOLPLWLQJUHDJHQWDQG.%ULVDQHXWUDOVDOWWKHUHPDLQLQJ.2+ZLOODFFRXQW
IRUWKHS+7KXVWKHS+RIWKHUHVXOWLQJVROXWLRQPXVWEHJUHDWHUWKDQ1RWHWKDWWKHUHLV
QREXIIHUVROXWLRQIRUVWURQJDFLGYVVWURQJEDVHUHDFWLRQ

$PWRI.2+OHIW [PRO
>2+@ [ [
7RWDO>2+@ [[ 
GLVVRFLDWLRQRIZDWHUPXVWEHFRQVLGHUHGDVWKHFRQFHQWUDWLRQRIWKHEDVHDQGDFLGLV
YHU\ORZ 

S2+ 
S+ 

 $TXHRXV K\GURFKORULF DFLG ZDV HOHFWURO\VHG IRU WHQ PLQXWHV  FP RI JDV$ ZDV
FROOHFWHGDWWKHDQRGH
7KHVDPHFXUUHQWZDVWKHQDSSOLHGWRFRQFHQWUDWHGVRGLXPFKORULGHVROXWLRQLQDQRWKHU
H[SHULPHQWDQGFPRIDJDV%LVFROOHFWHGDWWKHFDWKRGH

:KLFKRIWKHVHVWDWHPHQWVLVFRUUHFW" 

$ *DV%LVSDOH\HOORZ
% 7KHWLPHWDNHQIRUWKHVHFRQGHOHFWURO\VLVLVDOVRWHQPLQXWHV
& &KORULQHJDVZDVFROOHFWHGLQLWLDOO\LQWKHILUVWHOHFWURO\VLVV\VWHP
0HUFXU\HOHFWURGHVFDQEHXVHGIRUWKHVHFRQGHOHFWURO\VLVV\VWHPLIJDV%LVWR
'
EHFROOHFWHG
$QVZHU%
2SWLRQ$LVLQFRUUHFWEHFDXVHWKHJDVFROOHFWHGDWWKHFDWKRGHLV+
2SWLRQ&LVZURQJEHFDXVHWKHJDVFROOHFWHGDWWKHDQRGHLV2
2SWLRQ'LVZURQJEHFDXVHPHUFXU\HOHFWURGHVZLOOFDXVHVRGLXPWREHVHOHFWLYHO\
GLVFKDUJHGDWWKHFDWKRGHRIWKHVHFRQGHOHFWURO\VLV

2SWLRQ%LVFRUUHFWDQGWKHIROORZLQJVKRZVWKHFDOFXODWLRQ
)URP([SWDWWKHDQRGH )URP([SW$WWKHFDWKRGH
+22+H +2H+2+
 , &RQVLGHULQJVDPHFXUUHQWXVHGLQ([SW
VROYLQJ, $  WLPH
  VROYLQJWLPH PLQ

t dyli
trendyline
 

 KHIROORZLQJSURSHUWLHVFRXOGEHSUHGLFWHGIRUVWURQW
HIROORZLQJSURSHUWLHVFRXOGEHSUHGLFWHGIRUVWU LXPRULWV
:KLFKRIWKHIROORZLQJSURSHUWLHVFRXOGEHSUHGLFWHGIRUVWURQWLXPRULWVFRPSRXQGV"
VQRWEXUQLQDLU
QRW EXUQ LQ DLU
$ ,WGRHVQRWEXUQLQDLU
PV D VROXEOH VXOIDWH
% ,WIRUPVDVROXEOHVXOIDWH

1371 trendyline



 & ,WUHDFWVZLWKFROGZDWHUOLEHUDWLQJK\GURJHQ
' ,WIRUPVDZDWHUVROXEOHFDUERQDWHZKLFKGRHVQRWGHFRPSRVHRQKHDWLQJ
$QVZHU&
6WDWHPHQW&LVFRUUHFW6UUHDFWVYLJRURXVO\ZLWKFROGZDWHU
6U V +2 6U 2+  DT + J 

6UEXUQVYHU\IDVWLQDLUWRSURGXFHDZKLWHR[LGH6U V 2 J 6U2 V 


6U62LVQRWVROXEOHLQZDWHUDVVROXELOLW\RI*URXS,,VXOIDWHGHFUHDVHVGRZQWKH*URXS
6U&2LVQRWDZDWHUVROXEOHFRPSRXQGDQGLWVUHDVRQLQJLVVLPLODUWRLWVVXOIDWH


 (TXDO YROXPHV RI FKORULQH JDV ZHUH EXEEOHG LQWR KRW DQG FROG VRGLXP K\GUR[LGH
VHSDUDWHO\XQWLOUHDFWLRQVZHUHFRPSOHWH

:KLFKRIWKHIROORZLQJVWDWHPHQWVLVLQFRUUHFWDERXWWKHDERYHUHDFWLRQV"

$ 'LVSURSRUWLRQDWLRQUHDFWLRQVZLOORFFXU

% *UHHQLVK\HOORZFKORULQHJDVGHFRORXULVHVLQERWKUHDFWLRQV

& :KHQDTXHRXVVLOYHUQLWUDWHLVDGGHGWRWKHUHVXOWLQJVROXWLRQVZKLWHSUHFLSLWDWHLV
IRUPHG

' 6RGLXP FKORUDWH 9,,  DQG VRGLXP FKORUDWH ,  DUH IRUPHG LQ WKH UHDFWLRQV
UHVSHFWLYHO\

$QVZHU'

+RW&O2+&O&O2 FKORUDWH 9 +2


&ROG&O2+&O&O2 FKORUDWH , +2

7KHUHDFWLRQVLQYROYHGDUHGLVSURSRUWLRQDWLRQUHDFWLRQDQGWKHJUHHQLVK\HOORZFKORULQHJDV
ZLOOGHFRORXULVHGDVLWZDVUHDFWHGDZD\:KLWHSSWRIVLOYHUFKORULGHZLOOEHJHQHUDWHGZKHQ
$J12LVDGGHG


trendyline
TXDO YROXPH RI WULFKORURPHWKDQH
ZLWKDQHTXDOYROXPHRIWULFKORURPHWKDQH
P
$TXHRXV FKORULQH LV DGGHG WR DTXHRXV VRGLXP EURPLGH DQG WKH PL[WXUH LV VKDNHQ

1372 trendyline




:KLFKREVHUYDWLRQZLOOEHPDGH"

$ 7KHVROXWLRQLQWKHWHVWWXEHWXUQVFRORXUOHVV
% 7KHVROXWLRQLQWKHWHVWWXEHWXUQVRUDQJH
& $FRORXUOHVVOD\HUIRUPVRQWRSRIDSXUSOHOD\HU
' $FRORXUOHVVOD\HUIRUPVRQWRSRIDQRUDQJHOD\HU
$QVZHU'
&KORULQHLVDVWURQJR[LGLVLQJDJHQWDQGWKHEURPLGHLRQZLOOEHR[LGLVHWREURPLQH
&O%U&O%U

:KHQWULFKORURPHWKDQH DQRUJDQLFVROYHQW LVDGGHGWKH%UZLOOGLVVROYHLQWKLVRUJDQLF


OD\HUIRUPLQJDQRUDQJHOD\HU7KHDTXHRXVOD\HUPD\EHSDOH\HOORZ LIFRQFHQWUDWLRQRI
EURPLQHIRUPHGLVKLJK RUFRORXUOHVV LIFRQFHQWUDWLRQRIEURPLQHLVORZ 

2SWLRQ&LVRXWDVLRGLQH ZKLFKLVSXUSOH LVQRWIRUPHGLQWKLVUHDFWLRQ

2SWLRQ$DQG%DUHRXWDVZDWHUDQGWULFKORURPHWKDQHH[LVWDVLPPLVFLEOHOLTXLG

 $UHDFWLRQVFKHPHVWDUWLQJIURPDTXHRXVFRSSHU ,, VXOIDWHVROXWLRQLVVKRZQEHORZ


%RWK*DQG+DUHFRSSHUFRQWDLQLQJVSHFLHV

H[FHVV
1+ DT 1+ DT 1DHGWD DT
&X62 DT * + >&X HGWD @ DT
, ,, ,,, 

:KLFKRIWKHIROORZLQJVWDWHPHQWVLVFRUUHFW"
$ 5HDFWLRQ,,,LVDUHGR[UHDFWLRQ
% 1+DFWVDVDOLJDQGLQUHDFWLRQ,
& +LVDGHHSEOXHVROXWLRQFRQWDLQLQJ>&X 1+  +2 @62
' 7KHHQWURS\RIWKHV\VWHPGHFUHDVHVZKHQUHDFWLRQ,,,RFFXUV
$QVZHU&

5HDFWLRQ,,,LVOLJDQGH[FKDQJHUHDFWLRQKHQFHQRUHGR[RFFXUV$OVRVDPHQRRIERQGV
DQGEURNHQDQGIRUPHGKHQFHQRFKDQJHLQHQWURS\

5HDFWLRQ,LVSUHFLSLWDWLRQUHDFWLRQ1++22+1+
&X2+&X 2+  LGHQWLW\RI* 

*&X 2+ LVVROXEOHLQH[FHVV1+WRIRUP+>&X 1+  +2 @62DGHHSEOXH


VROXWLRQ

:KHQ1DHGWDZDVDGGHGOLJDQGH[FKDQJHUHDFWLRQRFFXUV
>&X 1+  +2 @('7$>&X ('7$ @1++2
(QWURS\RIWKHV\VWHPLQFUHDVHVGXHWRWKHUHOHDVHRI1+DQGZDWHUPROHFXOHV

trendyline
1373 trendyline



 6WLJPDVWHUROLVDQXQVDWXUDWHGSODQWVWHURORFFXUULQJLQWKHSODQWIDWVRIVR\EHDQ




+RZPDQ\VWHUHRLVRPHUVGRHVVWLJPDVWHUROKDYH"

 $ 
 % 
 & 
 ' 
$QVZHU%

7KHUH DUH  FKLUDO FDUERQ FHQWUHV DQG  GRXEOH ERQG ZKLFK FDQ H[KLELW JHRPHWULFDO
LVRPHULVP+HQFHWKHWRWDOQXPEHURIVWHUHRLVRPHUVLV 


y


 )RXUFDUERQFDUERQERQGVDUHODEHOOHGLQWKHGLDJUDP
RQFDUERQ ERQGV DUH ODEHOOHG LQ WKH
KH GLDJUDP
G

1374 trendyline




[
&+

\
]


Z


:KLFKERQGVDUHPDGHXSRIDVSVSRYHUODS"

 $ [RQO\

 % [DQG\RQO\

 & Z[DQG\RQO\

 ' Z\DQG]RQO\

$QVZHU%

%RQG[DQG\FRPSULVHVRIVSVSRYHUODS

%RQGZFRPSULVHVRIVSVSRYHUODS

%RQG]FRPSULVHVRIVSVSRYHUODS

trendyline


WHGZLWKFKORULQHWKHK\GURFDUERQWHWUDPHWK\OS
GZLWKFKORULQHWKHK\GURFDUERQ HQWDQH
W
 :KHQKHDWHGZLWKFKORULQHWKHK\GURFDUERQWHWUDPHWK\OSHQWDQHXQGHUJRHV
IUHHUDGLFDOVXEVWLWXWLRQ,QDSURSDJDWLRQVWHSWKHIUHHUDGLFDO-LVIRUPHGE\WKH

1375 trendyline



ORVVRIRQHK\GURJHQDWRP

&+

+& &+
&l-+&l
+&
&+ &+


+RZPDQ\GLIIHUHQWIRUPVRI-DUHWKHRUHWLFDOO\SRVVLEOH"

 $ 

 % 

 & 

 ' 

$QVZHU&

&+ &+ &+


+& +
&+ + &+ 

7KHUHDUHSRVVLEOHVLWHV DVLQGLFDWHGE\WKHVKDSHV WRORVHRQHK\GURJHQWRIRUP


GLIIHUHQWIUHHUDGLFDO

trendyline


 Saccharin ZDV DQ DUWLILFLDO VZHHWHQHU XVHG LQ VRPH VRIW GULQNV ,W ZDV
Z PDQXIDFWXUHG
 IURPPHWK\OEHQ]HQHE\DVHULHVRIUHDFWLRQV
\OEHQ]HQHE\DVHULHVRIUHDFWLRQV
EHQ]HQHE\DVHULHVRIUHDFWLRQV

1376 trendyline



&+ &+ &+

&l62+ 62&l 1+ 621+

, ,,

:KLFKRIWKHIROORZLQJVKRZVWKHFRUUHFWUHDFWLRQW\SHIRUVWHSV,DQG,," 

,,,
 $ (OHFWURSKLOLFVXEVWLWXWLRQ(OHFWURSKLOLFDGGLWLRQ
 % (OHFWURSKLOLFVXEVWLWXWLRQ1XFOHRSKLOLFVXEVWLWXWLRQ
 & 1XFOHRSKLOLFVXEVWLWXWLRQ1XFOHRSKLOLFVXEVWLWXWLRQ
 ' 1XFOHRSKLOLFVXEVWLWXWLRQ(OHFWURSKLOLFVXEVWLWXWLRQ
$QVZHU%
7KHILUVWUHDFWLRQLQYROYHVWKH62&lEHLQJVXEVWLWXWHGLQWRWKHEHQ]HQHULQJ
7KHVHFRQGUHDFWLRQLQYROYHVWKH1+QXFOHRSKLOLFVXEVWLWXWLRQZLWKWKH562&l

 :KLFKRIWKHIROORZLQJFDQQRWEHXVHGWRGLVWLQJXLVKEHWZHHQWKHIROORZLQJFRPSRXQGV"

2 2 +2 &+2


 $ +RWDFLGLILHG.&U2

 % 1HXWUDOLURQ ,,, FKORULGH

 & 'LDPPLQHVLOYHUFRPSOH[

 ' 3KHQ\OK\GUD]LQH

$QVZHU'
+2 &+2
2SWLRQ$FDQEHXVHG XQGHUJRR[LGDWLRQZLWK&U2WRIRUP 

2SWLRQ%LVSRVVLEOHDV ZLOOIRUPDYLROHWFRPSOH[

2SWLRQ&7ROOHQVUHDJHQWFDQEHXVHGWRLGHQWLI\ $VLOYHUPLUURUZLOOEHIRUPHG
XSRQKHDWLQJ

t d li
trendyline
2SWLRQ ' FDQQRW EH XVHG DV ERWK FRPSRXQGV KDYH FDUERQ\O FRPSRXQG ZKLFK FDQ IRUP
RUDQJHSSWZLWKSKHQ\OK\GUD]LQH
SKHQ\OK\GUD]LQH

1377 trendyline



 7KHUHDFWLRQVFKHPHEHORZVKRZVWKHV\QWKHVLVRIFRPSRXQG6

&l %U

&+ 1D2+ DT .&U2+62 DT


6
KHDW KHDW



:KLFKRIWKHIROORZLQJFDQEH6"

 $ &l 2

&+

 % 2+ 2

&+

 & 2+ 2
2
&+ &
&

2+
2+
&

2 

 ' 2 &l 2

& &
+2 2+

2+
&

2 

$QVZHU$
7KHUHDFWLRQVFKHPHIRUWKLVTXHVWLRQLVVKRZQEHORZ
&l %U &l 2+ &l 2
KHDW
&+ &+ &U2 &+
1D2+ DT
QXFOHRSKLOLF R[LGDWLRQ
VXEVWLWXWLRQ 
6WUXFWXUH6


trendyline


 LQJV\QWKHWLFURXWHFRQVLVWVRIWKUHHVWHSV
UHHV
7KHIROORZLQJV\QWKHWLFURXWHFRQVLVWVRIWKUHHVWHSV

1378 trendyline



&+ &22+

12
%U 

:KLFKVHTXHQFHRIVWHSVZRXOGJLYHWKHKLJKHVW\LHOG"

6WHS 6WHS 6WHS

 $ .0Q2+62KHDW FRQF+12FRQF+62 %U)HUWS


KHDW

 % FRQF+12FRQF+62 %U)HUWS .0Q2+62KHDW


KHDW

 & %U$l%UUWS FRQF+12GLOXWH+62 .0Q2+62KHDW


KHDW


 ' %U)HUWS .0Q2+62KHDW FRQF+12FRQF+62


KHDW

$QVZHU'

&+ &+ &22+ &22+

%U)HUWS .0Q2 FRQF+12


+62KHDW FRQF+62 12 
%U KHDW %U
%U 

3OHDVH QRWH WKDW FRQFHQWUDWHG +12 DQG FRQFHQWUDWHG +62 PXVW EH XVHG IRU
HOHFWURSKLOLFVXEVWLWXWLRQRIEHQ]HQHULQJ

trendyline


 V RI HDFK RI WKH IROORZLQJ ZDV KHDWHG IRU D SURORQJ SH
7HQ JUDPV SHU
SHULRG RI WLPH ZLWK
6XEVHTXHQWO\ GLOXWH +12 DT
1D2+ DT 6XEVHTXHQWO\GLOXWH+12 DT DQG$J12
DQG $J12 DT
DT ZHUHDGGHG
ZHUH DGGHG

1379 trendyline




:KLFKFRPSRXQGZLOOSURGXFHWKHJUHDWHVWPDVVRI$J%U V "

%U


&
$
 %U
Mr   Mr  


 
&+&2&l
% '
Mr   
Mr  
$QVZHU'
8SRQ KHDWLQJ ZLWK 1D2+ DT  QXFOHRSKLOLF VXEVWLWXWLRQ ZLOO RFFXU DQG DOO SRVVLEOH
KDORJHQRDONDQHVDQGDF\OKDOLGHVZLOOXQGHUJRQXFOHRSKLOLFVXEVWLWXWLRQ

)RU2SWLRQ$
$PRXQWRIRUJDQLFFSG  PRO
$PWRI%UVXEVWLWXWHG PRO
0DVVRI$J%USSWHG [   J

)RU2SWLRQ%
$PRXQWRIRUJDQLFFSG  PRO
$PWRI&OVXEVWLWXWHG PRO
0DVVRI$J&OSSWHG [   J

)RU2SWLRQ&
1RQXFOHRSKLOLFVXEVWLWXWLRQWRRNSODFH

)RU2SWLRQ'
$PRXQWRIRUJDQLFFSG  PRO
$PWRI&OVXEVWLWXWHG [ PRO
0DVVRI$J&OSSWHG [   J



trendyline


 HUHGXFWLRQXVLQJWLQDQGFRQFHQWUDWHGDFLGRIQLWUREHQ
UHGXFWLRQXVLQJWLQDQGFRQFHQWUDWHGDFLGRIQLWUREHQ]HQHWR
]HQHWR
$IWHUWKHUHGXFWLRQXVLQJWLQDQGFRQFHQWUDWHGDFLGRIQLWUREHQ]HQHWRSKHQ\ODPLQH
 HVVRIVRGLXPK\GUR[LGHLVDGGHG
DQH[FHVVRIVRGLXPK\GUR[LGHLVDGGHG

1380 trendyline



:KDWLVWKHSXUSRVHRIWKHVRGLXPK\GUR[LGH"

 $ WRGU\WKHSURGXFW

 % WROLEHUDWHWKHSKHQ\ODPLQH

 & WRQHXWUDOLVHWKHH[FHVVDFLG

 ' WRORZHUWKHERLOLQJSRLQWIRUVXEVHTXHQWGLVWLOODWLRQ

$QVZHU%

7KHPDLQUROHRI1D2+LVQRWWRQHXWUDOLVHWKHFRQFHQWUDWHGDFLGEXWUDWKHUWRUHDFWZLWKWKH
FRQMXJDWHDFLGWRIRUPWKHSKHQ\ODPLQH


trendyline


 KHIROORZLQJVKRZVWKHFRUUHFWRUGHURIEDVLFLW\"
HIROORZLQJVKRZVWKHFRUUHFWRUGHURIEDVLFLW\"
:KLFKRIWKHIROORZLQJVKRZVWKHFRUUHFWRUGHURIEDVLFLW\"


1381 trendyline



:HDNHVWEDVH 6WURQJHVWEDVH /RZ


 
KLJKpKb  pKb 


 $



 %



 &



 '


$QVZHU&
:HDNHVWEDVH6WURQJHVWEDVH

&21+ 1+
   
7KLVLVDQDPLGHDQG 7KHORQHSDLURI 7KHHOHFWURQ 7KHHOHFWURQ
LWLVQHXWUDO HOHFWURQRQWKH GLUHFWLQJFDXVHE\ GLUHFWLQJHIIHFW
QLWURJHQDWRPFDQ FDXVHE\WKH
GHORFDOLVHGLQWRWKH
EHQ]HQHULQJPDNLQJ WKH 
 JURXS
LWOHVVDYDLODEOHIRU
LVWKHJUHDWHVW
GDWLYHERQGZLWKWKH
7KLVFDXVHWKH
K\GURJHQSURWRQV
 ORQHSDLURI
 HOHFWURQVWREH
 PRUHDYDLODEOHIRU
GDWLYHERQGZLWK
+UHVXOWLQJLQD
VWURQJHUEDVH
)RU\RXUNQRZOHGJH3DSHU4 G 

,WLVNQRZWKDW .E [ LVOHVVEDVLFWKDQ&+1+ .E 


[ 
7KHSUHVHQFHRIWKHEHQ]HQHULQJDIIHFWWKHHOHFWURQGRQDWLQJHIIHFWRIWKH5JURXS


trendyline
y


 H LV D GLSHSWLGH GHULYDWLYHV XVHG DV DQ DUWLILFLDO VZHHWHQ


$VSDUWDPH VZHHWHQ
VZHHWHQHU ,WV JHQHUDO
VLVUHVWULFWHGEHFDXVHLWORVHVLWVVZHHWQHVVDIWHUK
LVUHVWULFWHGEHFDXVHLWORVHVLWVVZHHWQHVVDIWH \GURO\VLV
\GURO\VLV
XVHIXOQHVVLVUHVWULFWHGEHFDXVHLWORVHVLWVVZHHWQHVVDIWHUK\GURO\VLV


1382 trendyline





:KLFKSURGXFWZRXOGEHIRUPHGDIWHUSURORQJDFLGK\GURO\VLV

 $ &

DQG
DQG DQG&+2+

 % '

DQG DQG DQG&+2+

$QVZHU&
2
+1&+&21+&+& 2 &+
&+
&+
&
1D2 2

 
+1&+&2+ +1&+&2+ &+2+
&+
&+
&22+ 

7KHGLDJUDPDERYHVKRZVWKHDFLGLFK\GURO\VLVUHDFWLRQRIWKHDPLGHDQGHVWHUJURXS'R
QRWHWKDWVLQFHDFLGLVSUHVHQWDOOEDVLFJURXSV WKH1+WKH&221D ZLOOEHQHXWUDOLVHG

trendyline


RUHVLVLVDWHFKQLTXHRIVHSDUDWLQJDQGLGHQWLI\LQJDPLQRDFLGV
HVLVLVDWHFKQLTXHRIVHSDUDWLQJDQGLGHQWLI\LQJDPLQRDFLG
 (OHFWURSKRUHVLVLVDWHFKQLTXHRIVHSDUDWLQJDQGLGHQWLI\LQJDPLQRDFLGV$VROXWLRQRI
GV LV
DPLQR DFLGV LV DEVRUEHG
DEVRUEHG LQWR
LQWR SDSHU
SDSHU WKDW
WKDW LV
LV PRLVWHQHG
PRLVWHQHG ZLWK
ZLWK D
D EXI
IIH VROXWLRQ DQG
EXIIHU

1383 trendyline



VWUHWFKHG EHWZHHQ WZR HOHFWURGHV 3RVLWLYHO\ FKDUJHG VSHFLHV PRYH WRZDUGV WKH
FDWKRGHQHJDWLYHO\FKDUJHGVSHFLHVWRZDUGVWKHDQRGH

:LWKDEXIIHUDWS+ZKLFKDPLQRDFLGZLOOPRYHPRVWUHDGLO\WRZDUGVWKHFDWKRGH"

 $ &

 

 % '

 

$QVZHU'

)RU$DQG%VLQFHLWV5JURXSDUHDFLGLFWKHLULVRHOHFWULFSRLQWZLOOEHOHVVHUWKDQ7KXV
LQEXIIHUDWS+WKH\ZLOOEHLQDQDONDOLQHHQYLURQPHQWDQGZLOOWKXVEHKDYHDVDQDFLG
JHQHUDWLQJWKHDQLRQDQGWKXVPLJUDWLQJWRWKHDQRGH

2SWLRQ$ZLOOJHQHUDWH 

2SWLRQ%ZLOOJHQHUDWH 

)RU&DQG'WKHLU5JURXSDUHQHXWUDOWKXVWKHLULVRHOHFWULFSRLQWUHVLGHDURXQG7KXVLQ
EXIIHU DW S+  WKH\ ZLOO EH LQ DQ DFLGLF HQYLURQPHQW DQG WKXV ZLOO EHKDYH DV DQ DONDOLQH
JHQHUDWLQJWKHFDWLRQDQGPLJUDWLQJWRWKHFDWKRGH

2SWLRQ&ZLOOJHQHUDWH 

2SWLRQ'ZLOOJHQHUDWH 

6LQFH'LVKDVDORZHUMrDVFRPSDUHGWR&LWZLOOPLJUDWHPRUHUHDGLO\WRWRZDUGVWKH

ttrendyline
d li
FDWKRGH


)RU TXHVWLRQV    VWDWHPHQWV 


 RQH RU PRUH RI WKH QXPEHUHG VWDWHPHQWV
 WR 
 WR  PD\ EH FRUUHFW
'HFLGHZKHWKHUHDFKRIWKHVWDWHPHQWVLVRULVQRWFRUUHFW7KHUHVSRQVHV$WR'VKRXOGEH
HDFKRIWKHVWDWHPHQWVLVRULVQRWFRUUHFW7K
DFKRIWKHVWDWHPHQWVLVRULVQRWF HUHVSRQVHV $
VHOHFWHGRQWKHEDVLVRI
E L I

1384 trendyline




$ % & '
DQG DQGRQO\ DQGRQO\ RQO\
DUHFRUUHFW DUHFRUUHFW DUHFRUUHFW LVFRUUHFW

1RRWKHUFRPELQDWLRQRIVWDWHPHQWVLVWREHXVHGDVFRUUHFWUHVSRQVH

 7KHJUDSKEHORZVKRZVWKHILUVWWZHOYHLRQLVDWLRQHQHUJLHVIRUHOHPHQW7


LRQLVDWLRQ
HQHUJ\

QXPEHURIHOHFWURQVUHPRYHG


:KLFKRIWKHIROORZLQJVWDWHPHQWVDUHWUXH"

 ,WLVLQ*URXS,RIWKH3HULRGLF7DEOH

 ,WIRUPVDQR[LGHZKLFKGLVVROYHVUHDGLO\ZLWKDFLG

 ,WLVLQWKHWKLUGSHULRG 1DWR$U RIWKH3HULRGLF7DEOH

$QVZHU&
,WLVLQJURXS,,,RIWKH3HULRGLF7DEOHDVWKHILUVWFRQVHFXWLYHHOHFWURQVUHTXLUHGORZHU
HQHUJ\:LWKWKLVLGHD2SWLRQLVGHILQLWHO\ZURQJDQGGHSOR\LQJ0&4VNLOO\RXVKRXOGSLFN
2SWLRQ& DQG DVWKHFRUUHFWUHVSRQG

%HLQJLQJURXS,,,DQGKDYLQJPRUHWKDQHOHFWURQVLWFRXOGEHLQWKHWKLUGSHULRG
7KHJURXS,,,HOHPHQWFRXOGEHDOXPLQLXPZKRVHR[LGH $O2 FDQGLVVROYHUHDGLO\LQDFLG

trendyline
1385 trendyline



 7KHIROORZLQJFKHPLFDOUHDFWLRQVDUHOLVWHGEHORZ
 &RPEXVWLRQRIHWKDQGLRLFDFLG&+2 l 2 J &2 J +2 l 
 (YDSRUDWLRQRIZDWHU+2 l +2 J 
 $WRPLVDWLRQRIPDJQHVLXP0J V 0J J 
 3KRWRO\VLVRIFKORULQH&l J &l J 

:KLFKRIWKHIROORZLQJVWDWHPHQWVDUHFRUUHFW"
 6LVSRVLWLYHIRUDOOUHDFWLRQV
 *LVQHJDWLYHIRUDOOUHDFWLRQV
 +LVSRVLWLYHIRUDOOUHDFWLRQV
$QVZHU'

 + 6 *
&RPEXVWLRQRI $OZD\VQHJDWLYH 3RVLWLYH 6LQFH+LV
HWKDQGLRLFDFLG ([RWKHUPLF  7KHUHLVPRUH H[RWKHUPLFDQG6
)RUFRPEXVWLRQ PROHVRISURGXFW LVSRVLWLYH*ZLOO
HQHUJ\LVDOZD\V WKDQUHDFWDQW DOZD\VEHQHJDWLYH
UHOHDVHG DWDOOWHPSHUDWXUHV

(YDSRUDWLRQRI $OZD\VSRVLWLYH 3RVLWLYH * +76
ZDWHU (QGRWKHUPLF  'HSHQGHQWRQ
(QHUJ\QHHGWREH WHPSHUDWXUH
DEVRUEHGWR 6LQFH+DQG6LV
RYHUFRPHWKH SRVLWLYH*ZLOO
LQWHUPROHFXODU+ RQO\EHQHJDWLYHLI
ERQGLQJLQZDWHU WHPSHUDWXUH
$WRPLVDWLRQRI $OZD\VSRVLWLYH 3RVLWLYH LQFUHDVHV
PDJQHVLXP (QGRWKHUPLF  
(QHUJ\QHHGWREH 
DEVRUEHGWRFKDQJH
WKHPDJQHVLXP
VROLGLQWR
PDJQHVLXP
JDVHRXVDWRPV
3KRWRO\VLVRI $OZD\VSRVLWLYH 3RVLWLYH
FKORULQH (QGRWKHUPLF 
(QHUJ\LVDEVRUEHG
ZKHQERQGVDUH
EURNHQ

trendyline
1386 trendyline



 $UHDFWLRQLQZKLFKDSURGXFWDFWVDVDFDWDO\VWLVVDLGWREHDXWRFDWDO\WLF

0Q2 DT &2 DT + DT 0Q DT &2 J +2 l 

,Q WKH R[LGDWLRQ RI HWKDQHGLRDWH LRQV E\ PDQJDQDWH 9,,  LRQV WKH UHDFWLRQ LV
DFFHOHUDWHGE\WKH0QLRQVSURGXFHGGXULQJWKHUHDFWLRQ

*UDSKRI>0Q2@DJDLQVWWLPH

>0Q2@PROGP

  ,QLWLDOO\WKHUHDFWLRQLVVORZ

 %XWWKHUHDFWLRQUDWHLQFUHDVHVDV
 0QLRQVFDWDO\VHWKHUHDFWRQ

 7KHUHDFWLRQUDWHIDOOVWKHFRQFHQWUDWLRQV
RIUHDFWDQWVGHFUHDVH

  WLPHV

:KLFKRIWKHIROORZLQJVWDWHPHQWVFDQEHGHGXFHGIURPWKHJUDSKDERYH"

 ,QLWLDOUDWHRIUHDFWLRQLVWKHIDVWHVWDW  

 5HDFWLRQUDWHLQFUHDVHVDW  DV0QLVJHQHUDWHG

 5HDFWLRQ UDWH GHFUHDVHV DW   DV WKH FRQFHQWUDWLRQ RI WKH UHDFWDQWV
 GHFUHDVH
 $QVZHU&
)URPWKHJUDSKWKHJUDGLHQWDW  LVQRWWKHVWHHSHVW,WLVRQO\DWUHJLRQ  WKDWWKH
UHDFWLRQUDWHLQFUHDVHVGXHWRWKHDXWRJHQHUDWLRQRIFDWDO\VW0Q$VWKHUHDFWLRQ
SURJUHVVHVWKHFRQFHQWUDWLRQRIUHDFWDQWVGHFUHDVHDQGWKDWVZKHQUHDFWLRQUDWH
VWDUWVWRGZLQGOHGRZQJUDGXDOO\

trendyline
1387 trendyline



 7KHJUDSKEHORZVKRZVKRZWKHSHUFHQWDJHRIUHDFWDQW8 J WKDWUHPDLQHGLQDQ


HTXLOEULXPPL[WXUHYDULHVZLWK7DWSUHVVXUHVRIDWPDQGDWP


DWP +LJK3UHVVXUH 

8 J DW DWP


HTXLOLEULXP


+LJKWHPS/RZWHPS

:KLFKRIWKHIROORZLQJVWDWHPHQWFDQEHGHGXFHGIURPWKLVLQIRUPDWLRQ"

 7KHIRUZDUGUHDFWLRQLVHQGRWKHUPLF

 7KHHTXDWLRQIRUWKHDERYHUHDFWLRQFRXOGEH8 J 9 J : J 
 
 7KHHTXLOLEULXPFRQVWDQWKSLQFUHDVHVDVSUHVVXUHLQFUHDVHVLQWKHV\VWHP

 $QVZHU%

7KHILUVWFKDOOHQJHLVWRGHDOZLWKWKH7D[LV2QHZRUGRIDGYLFHLVWRODEHOKLJK
WHPSHUDWXUHDQGORZWHPSHUDWXUHRQWKHTXHVWLRQ7KLVZLOOPDNHWKLQJVHDVLHU
'RQRWHWKDW8LVD5($&7$17

8VLQJ DQ\ FXUYH DV WHPSHUDWXUH LQFUHDVHV ZKLOH SUHVVXUH LV FRQVWDQW  8
GHFUHDVHV WKLV PHDQV WKHUH ZLOO EH OHVV UHDFWDQW 8 DQG IRUZDUG UHDFWLRQ LV
IDYRXUHG $V LQFUHDVH LQ WHPSHUDWXUH E\ /H &KDWHOLHUV 3ULQFLSOH DOZD\V IDYRXU
HQGRWKHUPLFUHDFWLRQIRUZDUGUHDFWLRQLVHQGRWKHUPLF

)URPWKHH[SUHVVLRQJLYHQLQRSWLRQ8 J 9 J : J 
PROPRO

&RQVLGHULQJWKHWZRFXUYHVDWVHSDUDWHSUHVVXUHVDQGDWFRQVWDQWWHPSHUDWXUH

8 J DW DWP


HTXLOLEULXP




t d li
trendyline
$QLQFUHDVHLQSUHVVXUHFDXVHWKH8WRLQFUHDVH7KLVPHDQVWKHHTXLOLEULXP
HLQSUHVVXUHFDXVHWKH8WRLQFUHDVH7KL
SRVLWLRQPXVWVKLIWOHIW$VVXFKWKHH[SUHVVLRQLQRSWLRQPDWFKHVWKLVFODLP
XVWVKLIWOHIW$VVXFKWKHH[SUHVVLRQLQRSWLRQPDWFKHVWK

2SWLRQLVZURQJDVK
ZURQJDVKpLVRQO\WHPSHUDWXUHGHSHQGHQW
LVRQO\WHPSHUDWXUHGHSHQGHQW

1388 trendyline



 $VWXGHQWZDVLQYHVWLJDWLQJWKHSRVVLELOLW\RIDQHOHFWURFKHPLFDOFHOOXVLQJ&X&X
DQG)H)HKDOIFHOOV

9

3WHOHFWURGH 3WHOHFWURGH
6DOW%ULGJH

PROGP&X DT 
PROGP)H DT 
 O G  &  
  O G  ) 
:KLFKVWDWHPHQWVDUHWUXHRIWKHDERYHVHWXS"

 ERFHOO  9
 &RSSHUDQGLURQHOHFWURGHVFDQQRWEHXVHGLQVWHDG
 :KHQH[FHVVVRGLXPK\GUR[LGHLVDGGHGWRWKH)H)HKDOIFHOOWKHSRODULW\RI
WKHHOHFWURGHVDUHUHYHUVHG

$QVZHU$ DQGRQO\ 

2SWLRQLVWUXH
)URP'DWD%RRNOHW
Eo &X&X  9Eo )H)H  9
Eocell  9

2SWLRQLVWUXH,IFRSSHUDQGLURQHOHFWURGHDUHXVHG2WKHUUHDFWLRQZLOORFFXUDV
HTXLOLEULXPFDQEHHVWDEOLVKHGEHWZHHQWKHPHWDODQGWKHLULRQV

2SWLRQLVWUXH
:KHQ2+LVDGGHGWRWKHLURQKDOIFHOOWKHKDOIHTXDWLRQEHFRPHV
)H 2+ H)H 2+ 2+ZLWKDQERYDOXHRI
$VFRPSDUHGWREo )H)H  9

7KXV WKH LURQ FRQWDLQLQJ FRPSRXQG KDOIFHOO ZRXOG QRZ XQGHUJR R[LGDWLRQ DQG WKH
&X&XKDOIFHOOZRXOGQRZXQGHUJRUHGXFWLRQ+HQFHWKHSRODULW\RIWKHHOHFWURGHVDUH
UHYHUVHG

 %DULXPVXOIDWHLVOHVVVROXEOHWKDQPDJQHVLXPVXOIDWH

:KLFK RI WKHVH IDFWRUV DUH QHHGHG WR EH FRQVLGHUHG LQ RUGHU WR H[SODLQ WKLV
REVHUYDWLRQ"

 +K\GRIEDULXPLRQDQGPDJQHVLXPLRQ
 +IRIEDULXPVXOIDWHDQGPDJQHVLXPVXOIDWH

ttrendyline
dyli
 $WRPLFUDGLLRIEDULXPDQGPDJQHVLXP
$QVZHU' RQO\ 
QO\ 
+VROXWLRQ  +K\GUDWLRQRIFDWLRQ
DWLRQRIFDWLRQ+K\GUDWLRQRIDQLRQ +ODWWLFHHQHUJ\
ODWWLFHHQHUJ\
7KHPRUHVROXEOHWKHVXOIDWHWKHPRUHH[RWKHUPLFWKH+
HWKHVXOIDWHWKHPRUHH[RWKHUPLFWKH+VROXWLRQ ROXWLRQ
1RWHWKDWIRUERWK+
K +K\GUDWLRQDQG+ODWWLFHHQHUJ\
FH HQHUJ\ WKHVHWHUPVDUHGHSHQGHQWRQWKHLRQLFUDGLL
WKHVHWHUPVDUHGHSHQGHQWRQW
QRWWKHDWRPLFUDGLL

1389 trendyline



2SWLRQLVQRWIHDVLEOHDVHQWKDOS\FKDQJHRIIRUPDWLRQLVQRWUHTXLUHG

 :KLFKREVHUYDWLRQDERXWEURPLQHRULWVFRPSRXQGVLVFRUUHFW"

 :KHQ DTXHRXV OHDG ,,  QLWUDWH LV DGGHG WR DTXHRXV VRGLXP EURPLGH D FUHDP
SUHFLSLWDWHLVREVHUYHG

 :KHQVRGLXPEURPLGHLVWUHDWHGZLWKFRQFHQWUDWHGVXOIXULFDFLGDJDVWKDWWXUQV
PRLVWEOXHOLWPXVSDSHUUHGLVHYROYHG

 6LOYHUEURPLGHLVVROXEOHLQERWKGLOXWHDQGFRQFHQWUDWHGDPPRQLDVROXWLRQ

$QVZHU% DQGRQO\ 

2SWLRQ3E DT %U DT 3E%U V 


3E%U V LVDFUHDPSSW VDPHFRORXUDV$J%USSW 

2SWLRQ1D%U+62+%U1D+62
+%U+6262LVDFLGLFDQGWXUQVPRLVWEOXHOLWPXVSDSHUUHG%U+2

2SWLRQ$J%ULVRQO\VROXEOHLQFRQFHQWUDWHGDPPRQLD

 7KHFDWDO\WLFFRQYHUWHULVSDUWRIWKHH[KDXVWV\VWHPRIPRGHUQFDUV
:KLFKUHDFWLRQVRFFXULQWKHFDWDO\WLFFRQYHUWHU"
  &22&2

  12&21&2

  \
&[+\  [ \ 12[&2 \ +2 [ 1
  

$QVZHU$ DQG 

&DWDO\WLFFRQYHUWHUIXQFWLRQWRFDWDO\VHWKH

 2[LGDWLRQRIFDUERQPRQR[LGHWRFDUERQGLR[LGH
 5HGXFWLRQ RI QLWURJHQ R[LGH WR QLWURJHQ DQG R[\JHQ JDV E\ FDUERQ
PRQR[LGH
 &RPSOHWH FRPEXVWLRQRI K\GURFDUERQV E\ R[\JHQ RU QLWURJHQ
R[LGH5HGXFWLRQRIQLWURJHQR[LGHWRQLWURJHQDQGR[\JHQJDVE\FDUERQ
PRQR[LGH

trendyline
1390 trendyline



 +DORJHQRDONDQHVUHDFWZLWKDTXHRXVDONDOL2QHPHFKDQLVPRIWKLVUHDFWLRQKDVWKH
UHDFWLRQSDWKZD\GLDJUDPVKRZQEHORZ

(QHUJ\
 ;





 <

 5+DO2+


 52++DO


 5HDFWLRQ3DWKZD\
:KLFKVWDWHPHQWVDUHWUXH" LQGLFDWHVDSDUWLDOERQG 

  7KHUHDFWLRQLVDW\SHRIQXFOHRSKLOLFVXEVWLWXWLRQ

 
 ;LV & +DO
  


 
 <LV +2 & +DO


$QVZHU%  RQO\ 
<LVDFDUERFDWLRQLQWHUPHGLDWHQRWDSHQWDYDOHQWWUDQVLWLRQVWDWH

trendyline
1391 trendyline



 :KLFKRIWKHIROORZLQJSKHQRPHQDLQYROYHVGHQDWXUDWLRQRISURWHLQV"
  +HDWLQJRIHJJZKLWH
  3URGXFWLRQRIEHDQFXUGIURPVR\PLON
  'LVVROYLQJ3HQLFLOOLQLQDWHVWWXEHRIKRWDFLG


3HQLFLOOLQ

 $QVZHU% DQGRQO\ 

2SWLRQLQYROYHVGHQDWXUDWLRQ
+HDWLQJGXULQJFRRNLQJFDXVHVWKHDOEXPLQ SURWHLQ LQHJJZKLWHWRGHQDWXUH
+HDWLQJ ZLOO GLVUXSW WKHZHDN 9DQ GHU :DDOV IRUFHV DQG WR D OHVVHU H[WHQW
K\GURJHQ  ERQGV  KROGLQJ WKH TXDWHUQDU\ WHUWLDU\ DQG VHFRQGDU\ VWUXFWXUHV
UHVXOWLQJLQDPRUHGLVRUGHUHGDUUDQJHPHQW


2SWLRQLQYROYHVGHQDWXUDWLRQ
([WUDFW IURP  + 3DSHU  4 'RIX EHDQ FXUG  LV PDGH E\ FRDJXODWLQJ VR\
PLONDQGWKHQSUHVVLQJWKHFXUGVEHWZHHQILQHFORWKWRH[WUXGHPRVWRIWKHPRLVWXUH
&RDJXODWLQJ DJHQWV WKDW KDYH EHHQ XVHG LQFOXGH FHUWDLQ VDOWV DFLG RU HQ]\PHV
&RDJXODWLRQLVGXHWRWKHGHQDWXUDWLRQRIWKHSURWHLQLQWKHVR\PLON

6LPLODU SURFHVV LQ IRRG SUHSDUDWLRQ LQFOXGH FKHHVH PDNLQJ \RJXUW PDNLQJ DQG
EXWWHU PDNLQJ 0DNLQJ RI 0DUJDULQH LV QRW GHQDWXUDWLRQ EXW DGGLWLRQ RI K\GURJHQ
LQWRSRO\XQVDWXUDWHGDONHQHV 

2SWLRQGRHVQRWLQYROYHGHQDWXUDWLRQDVLWLVPHUHO\DQDFLGLFK\GURO\VLVSURFHVV

(1'2)3$3(5

trendyline
1392 trendyline



   6(5$1*221-81,25&2//(*(
*HQHUDO&HUWLILFDWHRI(GXFDWLRQ$GYDQFHG/HYHO
+LJKHU

&$1','$7(
1$0(

&/$66     


&+(0,675<       
-&3UHOLPLQDU\([DPLQDWLRQ   6HSWHPEHU
3DSHU6WUXFWXUHG4XHVWLRQV 63$  KU
&DQGLGDWHVDQVZHURQWKH4XHVWLRQ3DSHU

$GGLWLRQDO0DWHULDOV'DWD%RRNOHW      
   
5($'7+(6(,16758&7,216),567

:ULWH\RXUQDPHDQGFODVVRQDOOWKHZRUN\RXKDQGLQ
:ULWHLQGDUNEOXHRUEODFNSHQRQERWKVLGHVRIWKHSDSHU
<RXPD\XVHDVRIWSHQFLOIRUDQ\GLDJUDPVJUDSKVRUURXJKZRUN
'RQRWXVHVWDSOHVSDSHUFOLSVKLJKOLJKWHUVJOXHRUFRUUHFWLRQIOXLG

$QVZHUDOOTXHVWLRQV
$'DWD%RRNOHWLVSURYLGHG

$WWKHHQGRIWKHH[DPLQDWLRQIDVWHQDOO\RXUZRUNVHFXUHO\WRJHWKHU
7KH QXPEHU RI PDUNV LV JLYHQ LQ WKH EUDFNHWV >   @ DW WKH HQG RI HDFK TXHVWLRQ RU SDUW
TXHVWLRQV

 )RU([DPLQHUV8VH )RU([DPLQHUV8VH
 3   3  
 
 3   
   
 3   
   
 *5$1'   
 727$/  
   
   
  


trendyline
tren
n ine e
*5$'(  727$/ 

 3


 

7KLVGRFXPHQWFRQVLVWVRISULQWHGSDJHV
7KL G W L W I  L W G

7XUQRYHU@
1393 trendyline



 3  :KHQ D VSDULQJO\ VROXEOH VDOW FDOFLXP K\GUR[LGH &D 2+  LV DGGHG WR ZDWHU DQ
HTXLOLEULXPLVHVWDEOLVKHGEHWZHHQWKHXQGLVVROYHGVDOWDQGDVDWXUDWHGVROXWLRQRIWKH
VDOW


&D 2+  V &D DT 2+ DT 

7KH VROXELOLW\ SURGXFW DW ODERUDWRU\ WHPSHUDWXUH FDQ EH IRXQG E\ GHWHUPLQLQJ WKH
HTXLOLEULXP FRQFHQWUDWLRQ RI WKH K\GUR[LGH LRQV LQ D VDWXUDWHG VROXWLRQ RI FDOFLXP
K\GUR[LGH 7KLV HTXLOLEULXP FRQFHQWUDWLRQ FDQ EH IRXQG E\ WLWUDWLRQ ZLWK D VWDQGDUG
VROXWLRQRIK\GURFKORULFDFLG

7KH QXPHULFDO YDOXH RI WKH VROXELOLW\ SURGXFW RI FDOFLXP K\GUR[LGH LV DSSUR[LPDWHO\
[

 D  :ULWH DQ H[SUHVVLRQ IRU WKH VROXELOLW\ SURGXFW RI FDOFLXP K\GUR[LGH VWDWLQJ LWV
XQLWV



  


>@
 

trendyline
7XUQRYHU@
1394 trendyline



 <RX DUH UHTXLUHG WR ZULWH D SODQ WR GHWHUPLQH WKH VROXELOLW\ SURGXFW RI FDOFLXP
K\GUR[LGHDWODERUDWRU\WHPSHUDWXUH

<RXPD\DVVXPHWKDW\RXDUHSURYLGHGZLWK

 6ROLG&D 2+ 
 PROGPRIVWRFNVROXWLRQRI+&l
 GHLRQLVHGZDWHU
 LQGLFDWRUVQRUPDOO\IRXQGLQDVFKRROODERUDWRU\
 HTXLSPHQWQRUPDOO\IRXQGLQDVFKRROODERUDWRU\

 E  L  &DOFXODWHWKHPLQLPXPPDVVRI&D 2+ WKDWQHHGVWREHZHLJKHGLQRUGHU
WR REWDLQ  FP RI D VDWXUDWHG VROXWLRQ RI &D 2+  DW ODERUDWRU\
WHPSHUDWXUH

   















>@

  LL  $VVXPLQJ WKDW DSSUR[LPDWHO\  FP RI +&l ZDV UHTXLUHG WR UHDFW ZLWK
 FP RI VDWXUDWHG VROXWLRQ RI &D 2+  FDOFXODWH DQ DSSURSULDWH
FRQFHQWUDWLRQ RI +&l UHTXLUHG +HQFH FDOFXODWH WKH YROXPH RI WKH JLYHQ
VROXWLRQRI+&lUHTXLUHGWRSUHSDUHFPRIWKLVVROXWLRQ



   










trendyline





>@

7XUQRYHU@
1395 trendyline




  LLL  8VLQJWKHLQIRUPDWLRQJLYHQDERYHDQGWKHDQVZHUVLQE L DQGE LL \RXDUH
UHTXLUHG WR ZULWH D SODQ WR GHWHUPLQH WKH VROXELOLW\ SURGXFW RI FDOFLXP
K\GUR[LGH

<RX DUH DOVR UHTXLUHG WR H[SODLQ KRZ WKH GDWD \RX REWDLQ IURP WKLV
H[SHULPHQW PD\ EH XVHG WR GHWHUPLQH WKH VROXELOLW\ SURGXFW RI FDOFLXP
K\GUR[LGH

<RXUSODQVKRXOGLQFOXGH
 WKH SUHSDUDWLRQ RI  FP RI WKH VDWXUDWHG VROXWLRQ RI FDOFLXP
K\GUR[LGH
 WKHSUHSDUDWLRQRIFPRIVWDQGDUGVROXWLRQRIK\GURFKORULFDFLG
IURPWKHVWRFNVROXWLRQRIK\GURFKORULFDFLG
 WLWUDWLRQSURFHGXUH
 KRZ WKH UHVXOWV REWDLQHG FDQ EH XVHG WR GHWHUPLQH WKH VROXELOLW\
SURGXFWRIFDOFLXPK\GUR[LGH

  





















trendyline










7XUQRYHU@
1396 trendyline





































trendyline







>@

7XUQRYHU@
1397 trendyline



   >7RWDO@
 7KHPDQXIDFWXUHRIDPPRQLDIURPQLWURJHQDQGK\GURJHQWDNHVSODFHLQWZRVWDJHV

)LUVW6WDJH7KHPDQXIDFWXUHRIK\GURJHQIURPPHWKDQH
6HFRQG6WDJH7KHV\QWKHVLVRIDPPRQLD WKH+DEHU3URFHVV 

7KH)LUVW6WDJHRFFXUVYLDDWZRVWHSSURFHVV

 Steam Reforming
 Shift Reaction

%RWKVWHSVSURGXFHK\GURJHQJDVWKDWZLOOEHXVHGLQWKH6HFRQG6WDJHIRUWKHSURGXFWLRQ
RIDPPRQLD

 D  Steam Reforming LV D UHDFWLRQ WKDW FRQYHUWV PHWKDQH DQG VWHDP WR D PL[WXUH RI
FDUERQPRQR[LGHDQGK\GURJHQ7KHUHDUHWZRSRVVLEOHURXWHVIRULW

Primary Steam Reforming:6WHDPLVVXSSOLHGWRVWDUWWKHUHDFWLRQ
&+ J +2 J &2 J + J HR N-PROeqm  

Secondary Steam Reforming:6RPHK\GURJHQLVEXUQWWRIRUPVWHDPZKLFKWKHQUHDFWV
ZLWKPHWKDQHWRJHQHUDWHPRUHK\GURJHQ
+ J 2 J +2 J HR N-PRO
&+ J +2 J &2 J + J HR N-PRO


  L  6WDWH LI WKH RYHUDOO SURFHVV RI Secondary Steam Reforming LV DQ H[RWKHUPLF RU
HQGRWKHPLFSURFHVV

   
>@

  LL  6XJJHVW DQG H[SODLQ ZKLFK VWHDP UHIRUPLQJ SURFHVV Primary RU Secondary  LV
IDYRXUHGE\KLJKWHPSHUDWXUH

   







>@


trendyline







7XUQRYHU@
1398 trendyline




  LLL  7KH PHWKDQH JDV XVHG XVXDOO\ FRQWDLQV VRPH RUJDQLF VXOIXU FRPSRXQGV
56+DQGK\GURJHQVXOILGH+6ERWKRIZKLFKPXVWEHUHPRYHGRULWZLOOSRLVRQ
WKH FDWDO\VW +HQFH WKH PHWKDQH PL[WXUH QHHGV WR EH SDVVHG WKURXJK D
desulfurisationunitILUVW

 7KHVHRUJDQLFVXOIXUFRPSRXQGVDUHILUVWFRQYHUWHGLQWRK\GURJHQVXOILGH
DQGDK\GURFDUERQE\UHGXFWLRQZLWKK\GURJHQJDV

 7KH K\GURJHQ VXOILGH LV UHDFWHG ZLWK ]LQF R[LGH =Q2  WR SURGXFH ]LQF
VXOILGH =Q6 DVRQHRIWKHSURGXFWV

&RQVWUXFWWZREDODQFHGFKHPLFDOHTXDWLRQVVKRZLQJWKHUHDFWLRQVWKDWRFFXUUHG
LQWKHdesulfurisationunitXVLQJ56+WRUHSUHVHQWRUJDQLFVXOIXUFRPSRXQGVDQG
5+WRUHSUHVHQWK\GURFDUERQ

   

>@

 E  ,Q WKHShift Reaction WKH FDUERQ PRQR[LGH SURGXFHG GXULQJ WKH Steam Reforming
SURFHVVZDVFRQYHUWHGWRFDUERQGLR[LGHDQGK\GURJHQJDVShift ReactionLVGRQHYLD
DWZRVWHSSURFHVV

 High Temperature Shift ReactionZLWKLURQ ,,, R[LGHFDWDO\VW

 Low Temperature Shift Reaction ZLWK WKHUPDOO\ XQVWDEOH FRSSHU FDWDO\VW DW
.7KHFDUERQPRQR[LGHFRQFHQWUDWLRQLVIXUWKHUUHGXFHGWR

7KHUHDFWLRQLQYROYHGLQWKHShift Reaction LVLOOXVWDWHGLQWKHIROORZLQJHTXDWLRQ

&2 J +2 J &2 J + J HR N-PROeqm  



  L  6WDWHWKHIXOOHOHFWURQLFFRQILJXUDWLRQRILURQ ,,, LRQDQGFRSSHUPHWDO

   )HV

&XV
>@

  LL  :LWK UHIHUHQFH WR WKH Shift Reaction VWDWH DQG H[SODLQ LI LURQ ,,,  R[LGH DQG
FRSSHULVIXQFWLRQLQJDVKRPRJHQHRXVRUKHWHURJHQRXVFDWDO\VW
   




trendyline







>@
 

7XUQRYHU@
1399 trendyline



 F  7UDFHV RI FDUERQ GLR[LGH SURGXFHG LQ WKH Shift ReactionLV VXEVHTXHQWO\ UHPRYHG E\
SDVVLQJ LW ZLWK K\GURJHQ JDV RYHU D QLFNHO FDWDO\VW DW  . D SURFHVV NQRZQ DV
Methanation

&2 J + J &+ J +2 J 


8VLQJ WKH HQWKDOS\ FKDQJH YDOXHV RI eqm(1) DQG eqm(2) GHWHUPLQH WKH HQWKDOS\
FKDQJHIRUWKLVMethanationSURFHVV

  




















>@
 

trendyline
7XUQRYHU@
1400 trendyline



 G  7KHPDLQSURFHVVRIWKH+DEHU3URFHVVRFFXUVLQDIL[HGEHGUHDFWRU

1 J + J 1+ J HR N-PRO

7KH SURSRUWLRQ RI DPPRQLD LQ WKH HTXLOLEULXP PL[WXUH LQFUHDVHV ZLWK LQFUHDVLQJ
SUHVVXUH DQG ZLWK GHFUHDVLQJ WHPSHUDWXUH 4XDQWLWDWLYH GDWD DUH JLYHQ LQWKH
WDEOHEHORZ

3UHVVXUHDWP 3HUFHQWDJHRIDPPRQLDSUHVHQWDW
HTXLOLEULXPDWDUDQJHRI
WHPSHUDWXUH
 . . .
   
   
   
   
   
   

   
L  7RREWDLQDUHDVRQDEOH\LHOGZLWKIDYRXUDEOHUDWHKLJKSUHVVXUHVPRGHUDWH
WHPSHUDWXUHV DQG D FDWDO\VW DUH XVHG 8VLQJ WKH GDWD VHOHFW WKH
WHPSHUDWXUHDQGSUHVVXUHZKLFKZLOOUHVXOWLQWKHKLJKHVW\LHOGRIDPPRQLD

   7HPSHUDWXUH       3UHVVXUH 
>@

  LL  +HQFHXVLQJWKHGDWD\RXKDYHVHOHFWHGLQ G L GHWHUPLQHWKHYROXPHRI
DPPRQLDJDVQLWURJHQJDVDQGK\GURJHQJDVDWHTXLOLEULXPZKHQFPRI
QLWURJHQJDVLVUHDFWHGZLWKFPRIK\GURJHQJDVLQWKH+DEHU3URFHVV
/HDYH\RXUDQVZHUWRWZRGHFLPDOSODFHV

   

















trendyline



>@

   >7RWDO@

7XUQRYHU@
1401 trendyline



 D  $OO\O LVRWKLRF\DQDWH $,7&  LV UHVSRQVLEOH IRU WKH SXQJHQW WDVWH RI ZDVDEL ZKLOH
FDSVDLFLQLVDQDFWLYHFRPSRQHQWRIFKLOOLSHSSHUDQGLVIRXQGLQVSLFHGSURGXFWV
OLNHFXUU\


$,7&




&DSVDLFLQ


  L  8VLQJ\RXUFRQFHSWRI96(35VWDWHWKHVKDSHDERXWWKHFDUERQODEHOOHG
DQGRQ$,7&

   &DUERQ&DUERQ>@


  LL  6WDWHWKHW\SHRIK\EULGLVDWLRQRIFDUERQVLQGLFDWHGLQ$,7&

  


>@

  LLL 
%HVLGHVWKHHWKHUJURXS VWDWHWKHRWKHUIXQFWLRQDOJURXSVWKDWDUH
SUHVHQWLQ&DSVDLFLQ

   >@

  LY  ,JQRULQJ WKH HIIHFW RI WKH 1 & 6 JURXS ZULWH DQ HTXDWLRQ VKRZLQJ KRZ
$,7&UHDFWVZLWKOLTXLG ,%U+HQFHZLWKUHIHUHQFHWRWKHData Booklet DQG
JLYHQ WKDW WKH ERQG HQHUJ\ RI ,%U WR EH  N- PROGHWHUPLQH WKH
HQWKDOS\FKDQJHRIUHDFWLRQZKHQ$,7&UHDFWVZLWKOLTXLG,%U

   
 
 
 
 

trendyline
 
 
 
 
>@

7XUQRYHU@
1402 trendyline



  Y  8VLQJ \RXU NQRZOHGJH LQ FKHPLFDO ERQGLQJ VXJJHVW ZK\ WKH EXUQLQJ
VHQVDWLRQRIFDSVDLFLQFDQQRWEHZDVKHGDZD\ZLWKZDWHU

   




>@

  YL  'UDZ WKH VWUXFWXUDO IRUPXOD RI WKH SURGXFWV IRUPHG ZKHQ FDSVDLFLQ LV
UHDFWHGZLWKKRWDTXHRXVSRWDVVLXPK\GUR[LGH

><RXDUHWRLJQRUHWKHHIIHFWRIWKH JURXS@















   >@

   >7RWDO@


 

trendyline
7XUQRYHU@
1403 trendyline



 D  3KRVJHQH LV D FRPSRXQG ZLWK WKH IRUPXOD &2&l ,W LV QRW WR EH FRQIXVHG ZLWK
FREDOW ,, FKORULGHZLWKWKHIRUPXOD&R&l

3KRVJHQHLVSURGXFHGLQGXVWULDOO\ZLWKFDUERQPRQR[LGHDQGFKORULQHJDV

&2 J &l J &2&l J HR N-PRO

7KH Kp RI WKLV SURFHVV LV DWP DW  . $ PL[WXUH FRQWDLQV  PRO RI FDUERQ
PRQR[LGH DQG  PRO RI FKORULQH LQLWLDOO\ ZDV UHDFWHG DQG DOORZ WR UHDFK
HTXLOLEULXP7KHHTXLOLEULXPPL[WXUHLVIRXQGWRFRQWDLQPRORISKRVJHQH

  L  'UDZWKH/HZLVVWUXFWXUHRISKRVJHQH

   



>@

  LL  :ULWHDQH[SUHVVLRQIRUKSIRUWKHHTXLOLEULXPEHWZHHQ&2&lDQGSKRVJHQH
DQG KHQFH XVLQJ WKH LQIRUPDWLRQ SURYLGHG FDOFXODWH WKH WRWDO SUHVVXUH RI WKH
V\VWHPDWHTXLOLEULXP

   
















>@

  LLL  6XJJHVWLIFKORULQHRUSKRVJHQHZLOOGHYLDWHPRUHIURPLGHDOJDVEHKDYLRXU

  



trendyline







7XUQRYHU@
1404 trendyline






>@

 E &U\VWDOV RI K\GUDWHG FREDOW ,,  FKORULGH &R&l+2 ORVH ZDWHU ZKHQ WKH\ DUH
 KHDWHGIRUPLQJDQK\GURXVFREDOW ,, FKORULGH&R&l

&R&l+2 V &R&l V +2 l 

  L  7KHVWDQGDUGHQWKDOS\FKDQJHIRUWKHUHDFWLRQ HLVN-PRO*LYHQ
WKDWWKHFDOFXODWLRQIRUHQWURS\FKDQJHWREHVLPLODUWRWKDWRIHQWKDOS\FKDQJH
XVH WKH IROORZLQJ HQWURS\ GDWD WR GHWHUPLQH WKH VWDQGDUG HQWURS\ FKDQJH RI
WKHUHDFWLRQDW.*LYH\RXUDQVZHUWRIRXUVLJQLILFDQWILJXUHV

&RPSRXQG S
-PRO.
&R&l+2 V  
&R&l V  
+2 l  
+2 J  


  



>@

  LL  +HQFHH[SODLQE\XVLQJFDOFXODWLRQZKHWKHUK\GUDWHGFREDOW ,, FKORULGHFDQ
EHVWRUHGDW.ZLWKRXWGHFRPSRVLWLRQ

   









>@

 F  :LWKWKHDLGRIDFKHPLFDOHTXDWLRQH[SODLQZK\DVROXWLRQRIFREDOW ,, FKORULGHKDV

trendyline
DS+RI

 




7XUQRYHU@
1405 trendyline
















>@
  >7RWDO@

 *URXS 9,,K\GULGHVDUH FRORXUOHVV JDVHV DW URRP WHPSHUDWXUH 7KH ILJXUHV EHORZ VKRZV
WKHUHODWLYHHQWKDOS\FKDQJHRIYDSRULVDWLRQRIWKHK\GULGHV







(QWKDO

S\

FKDQJ

HRI

YDSRUL

VDWLRQ




 ,&,l,,,


 D  :LWK UHIHUHQFH WR )LJXUH  H[SODLQ WKH DEQRUPDOO\ KLJK HQWKDOS\ FKDQJH RI
YDSRULVDWLRQRIK\GURJHQIOXRULGH

 


trendyline








7XUQRYHU@
1406 trendyline









>@




















 E 7KH WDEOH EHORZ VKRZV WKH pKaRI WKH UHVSHFWLYH FRPSRXQGV ZKHQ WKH\ DUH
 GLVVROYHGLQZDWHU

&RPSRXQGV pKa  &RPSRXQGV pKa
   +\GURJHQEURPLGH 
+\GURJHQFKORULGH   +\GURJHQLRGLGH 

0HWKDQRLFDFLG   :DWHU 
 
L  8VLQJ UHOHYDQW LQIRUPDWLRQ IURP WKH Data Booklet UDQN WKH DFLG VWUHQJWK RI
DTXHRXV VROXWLRQ FRQWDLQLQJ +&l +%U DQG +, LQ LQFUHDVLQJ RUGHU ([SODLQ
\RXUDQVZHU

  




trendyline







7XUQRYHU@
1407 trendyline






>@


  LL  $FFRXQWIRUWKHUHODWLYHDFLGLWLHVRIPHWKDQRLFDFLGDQGZDWHU

   





>@

   
   















 F  8QOLNHRWKHU*URXS 9,,K\GULGHVK\GURJHQIOXRULGH+)EHKDYHVDVDZHDNDFLGLQ
ZDWHU

$VROXWLRQRI+)ZDVWLWUDWHGDJDLQVWDTXHRXVSRWDVVLXPK\GUR[LGHXQWLORIWKH
+)SUHVHQWZDVQHXWUDOLVHG

  L  :ULWHWKHDFLGGLVVRFLDWLRQFRQVWDQWH[SUHVVLRQKDIRU+)

  

>@


trendyline
  LL  *LYHQWKDWWKHS+RIWKHUHVXOWLQJVROXWLRQGHVFULEHGDERYHLVFDOFXODWHK
LYHQWKDWWKHS+RIWKHUHVXOWLQJVROXWLRQGHVFULEHGDERYHLV
YHQWKDWWKHS+RIWKHUHVXOWLQJVROXWLRQGHVF  D
RI+)
I+)
I+)

   



7XUQRYHU@
1408 trendyline















>@























 G 2QH RI WKH PRVW FRPPRQ ZD\V WR PDNH K\GURJHQ FKORULGH LV WR UHDFW
 VRGLXP FKORULGH ZLWK FRQFHQWUDWHG VXOIXULF DFLG +62 RU FRQFHQWUDWHG
SKRVSKRULF 9 DFLG+32%RWKUHDFWLRQVRFFXUVLPLODUO\UHOHDVLQJZKLWHIXPHVRI
JDV

  L  &RQVWUXFW D EDODQFHG HTXDWLRQ LQYROYLQJ VRGLXP FKORULGH DQG FRQFHQWUDWHG
SKRVSKRULF 9 DFLG

  

>@


trendyline
  LL  ,Q WKH UHDFWLRQ ZLWK VRGLXP EURPLGH FRQFHQWUDWHG VXOIXULF DFLG SURGXFHV D
GGLVKEURZQVROXWLRQZKLOHFRQFHQWUDWHGSKRVSKR 9 DFLG
HGGLVKEURZQVROXWLRQZKLOHFRQFHQWUDWHGSKRVSKRULF
UHGGLVKEURZQVROXWLRQZKLOHFRQFHQWUDWHGSKRVSKRULF DFLGSURGXFHVZKLWH
PHV
IXPHV

XJJHVWDUHDVRQIRUWKHREVHUYDWLRQVDERYH
JJHVWDUHDVRQIRUWKHREVHUYDWLRQVDERYH
6XJJHVWDUHDVRQIRUWKHREVHUYDWLRQVDERYH


7XUQRYHU@
1409 trendyline



   









>@

   >7RWDO@


 

trendyline
7XUQRYHU@
1410 trendyline



 D  7KH FKHPLFDO VWUXFWXUH DQG VRPH UHDFWLRQV LQYROYLQJ FRPSRXQG 1 DUH VKRZQ
EHORZ

&XPHQH

SURFHVV





, ,,



1 3 4


 %U l 


 5

  L  6WDWHWKHUHDJHQWVDQGFRQGLWLRQVLQYROYHGLQ6WHSV,DQG,,

6WHS,

6WHS,,
>@

  LL  'UDZWKHVWUXFWXUHRIRUJDQLFSURGXFW5

   










trendyline





>@

7XUQRYHU@
1411 trendyline



  LLL  3URSRVHDFKHPLFDOWHVWWRGLVWLQJXLVKEHWZHHQ3DQG1LQZKLFKDSRVLWLYH
WHVW LV REVHUYHG IRU 3 RQO\ :ULWH D EDODQFHG FKHPLFDO HTXDWLRQ IRU WKH
UHDFWLRQWKDWKDVRFFXUUHG

   7HVW


2EVHUYDWLRQ


(TXDWLRQ






>@

 E  ,QWKHFXPHQHSURFHVVRQHPROHRIEHQ]HQHUHDFWVZLWKRQHPROHRISURSHQHLQ
WKH SUHVHQFH RI R[\JHQ IURP DLU WR IRUP SKHQRO DQG RUJDQLF VXEVWDQFH 7
6XEVWDQFH7GRHVQRWJLYHDQ\SRVLWLYHREVHUYDWLRQZLWKGLDPPLQHVLOYHUFRPSOH[
RUSKRVSKRUXVSHQWDFKORULGH

  L  )URPWKHLQIRUPDWLRQSURYLGHGLGHQWLI\VXEVWDQFH7

   



>@

  LL  +HQFHZLWKDQDLGRIDFKHPLFDOHTXDWLRQVKRZKRZWKHIXQFWLRQDOJURXSLQ
VXEVWDQFH7FDQEHSRVLWLYHO\LGHQWLILHG

   2EVHUYDWLRQ

(TXDWLRQ










>@

trendyline

   >7RWDO@


(1'2)3$3(5
(1'2)3$3(5
2)3$3(

7XUQRYHU@
1412 trendyline
6HUDQJRRQ-XQLRU&ROOHJH

-&+&+(0,675<  3DSHU6XJJHVWHG6ROXWLRQV

 3  :KHQ D VSDULQJO\ VROXEOH VDOW FDOFLXP K\GUR[LGH &D 2+  LV DGGHG WR ZDWHU DQ
HTXLOLEULXPLVHVWDEOLVKHGEHWZHHQWKHXQGLVVROYHGVDOWDQGDVDWXUDWHGVROXWLRQRIWKH
VDOW


&D 2+  V &D DT 2+ DT 

7KH VROXELOLW\ SURGXFW DW ODERUDWRU\ WHPSHUDWXUH FDQ EH IRXQG E\ GHWHUPLQLQJ WKH
HTXLOLEULXP FRQFHQWUDWLRQ RI WKH K\GUR[LGH LRQV LQ D VDWXUDWHG VROXWLRQ RI FDOFLXP
K\GUR[LGH 7KLV HTXLOLEULXP FRQFHQWUDWLRQ FDQ EH IRXQG E\ WLWUDWLRQ ZLWK D VWDQGDUG
VROXWLRQRIK\GURFKORULFDFLG

7KH QXPHULFDO YDOXH RI WKH VROXELOLW\ SURGXFW RI FDOFLXP K\GUR[LGH LV DSSUR[LPDWHO\
[

 D  :ULWH DQ H[SUHVVLRQ IRU WKH VROXELOLW\ SURGXFW RI FDOFLXP K\GUR[LGH VWDWLQJ LWV
XQLWV
>@


  KVS >&D@>2+@XQLWVPROGP

 <RX DUH UHTXLUHG WR ZULWH D SODQ WR GHWHUPLQH WKH VROXELOLW\ SURGXFW RI FDOFLXP
K\GUR[LGHDWODERUDWRU\WHPSHUDWXUH

<RXPD\DVVXPHWKDW\RXDUHSURYLGHGZLWK

 6ROLG&D 2+ 
 PROGPRIVWRFNVROXWLRQRI+&l
 GHLRQLVHGZDWHU
 LQGLFDWRUVQRUPDOO\IRXQGLQDVFKRROODERUDWRU\
 HTXLSPHQWQRUPDOO\IRXQGLQDVFKRROODERUDWRU\

 E  L  &DOFXODWHWKHPLQLPXPPDVVRI&D 2+ WKDWQHHGVWREHZHLJKHGLQRUGHU
WR REWDLQ  FP RI D VDWXUDWHG VROXWLRQ RI &D 2+  DW ODERUDWRU\
WHPSHUDWXUH
>@

   .VS[ 
/HWWKHVROXELOLW\EH[PROGP
[ [  [
[ [
[ PROGP

VROXELOLW\LQJGP [ [[  JGP

0LQPDVVRI&D 2+    J


trendyline
y
  LL  $VVXPLQJ WKDW DSSUR[LPDWHO\  FP RI +&
+&l
ll ZDV UHTXLU
UHTXLUHG WR UHDFW ZLWK
 FP RI VDWXUDWHG VROXWLRQ RI &D 2+  FDOFXODWH DQ DSSURSULDWH
+&ll UHTXLUHG +HQFH FDOFXODWH WKH YROXP
FRQFHQWUDWLRQ RI +
+& YROXPH RI WKH JLYHQ
VROXWLRQRI+&lUHTXLUHGWRSUHSDUHFP
VROXWLRQRI+&l
VROXWLRQRI+& lUHTXLUHGWRSUHSDUHFPRIWKLVVROXWLRQ
>@


1413 trendyline
   &D 2+ +&l &D&l+2

$PRXQWRI&D 2+    PRO 

$PRXQWRI+&O    PRO 

&RQFHQWUDWLRQRI+&l UHTXLUHG   PROGP 


9RORIWKHJLYHQVROXWLRQQHHGHG 


 GP FP 

  LLL  8VLQJWKHLQIRUPDWLRQJLYHQDERYHDQGWKHDQVZHUVLQE L DQGE LL \RXDUH
UHTXLUHG WR ZULWH D SODQ WR GHWHUPLQH WKH VROXELOLW\ SURGXFW RI FDOFLXP
K\GUR[LGH

<RX DUH DOVR UHTXLUHG WR H[SODLQ KRZ WKH GDWD \RX REWDLQ IURP WKLV
H[SHULPHQW PD\ EH XVHG WR GHWHUPLQH WKH VROXELOLW\ SURGXFW RI FDOFLXP
K\GUR[LGH

<RXUSODQVKRXOGLQFOXGH
 WKH SUHSDUDWLRQ RI  FP RI WKH VDWXUDWHG VROXWLRQ RI FDOFLXP
K\GUR[LGH
 WKHSUHSDUDWLRQRIFPRIVWDQGDUGVROXWLRQRIK\GURFKORULFDFLG
IURPWKHVWRFNVROXWLRQRIK\GURFKORULFDFLG
 WLWUDWLRQSURFHGXUH
 KRZ WKH UHVXOWV REWDLQHG FDQ EH XVHG WR GHWHUPLQH WKH VROXELOLW\
SURGXFWRIFDOFLXPK\GUR[LGH
>@
   3UHSDUDWLRQRIVDWXUDWHGVROXWLRQRIFDOFLXPK\GUR[LGH
 :HLJKDSSUR[LPDWHO\JRIVROLG
 8VLQJ D PHDVXULQJ F\OLQGHU SODFH  FP RI GHLRQLVHG ZDWHU
LQWRDGU\FRQLFDOIODVNEHDNHU
 $GGVROLGWRZDWHUDQGVWLUZLWKDJODVVURGWRGLVVROYHWKHVROLG
 :HLJKDQGDGGPRUHVROLGWRWKHZDWHUXQWLOH[FHVVVROLGUHPDLQV
QRPRUHVROLGGLVVROYHV
 $OORZ WKH PL[WXUH WR VWDQG IRU D SHULRG RI WLPH HJ  PLQV  WR
HVWDEOLVKHTXLOLEULXP
 )LOWHUWKHPL[WXUHWRREWDLQWKHILOWUDWH

   3UHSDUDWLRQRIK\GURFKORULFDFLGIRUPWKHVROXWLRQ
 )LOO D EXUHWWH ZLWK WKH VWRFN VROXWLRQ RI K\GURFKORULF DFLG 'UDLQ
 FP RI WKH K\GURFKORULF DFLG IURP WKH EXUHWWH LQWR D  FP
YROXPHWULF IODVN7RSXSWKHYROXPHWULFIODVNXS WR WKH PDUN ZLWK
GHLRQLVHGZDWHU$GGWKHODVWIHZGURSVRIGHLRQLVHGZDWHUXVLQJD
GURSSHUWLOOWKHERWWRPRIWKHPHQLVFXVFRLQFLGHVZLWKWKHPDUN
 6WRSSHUWKHYROXPHWULFIODVNILUPO\DQGVKDNHLWWRHQVXUHFRPSOHWH
PL[LQJ7KLVLVWRHQVXUHWKDWDKRPRJHQHRXVVROXWLRQLVREWDLQHG

ttrendyline
d li


1414 trendyline
   7LWUDWLRQSURFHGXUH
 )LOODFOHDQEXUHWWHZLWKWKHK\GURFKORULFDFLGVROXWLRQ
 3LSHWWHFPRIFDOFLXPK\GUR[LGHLQWRDFOHDQFRQLFDOIODVN
 $GGRUGURSVRIPHWK\ORUDQJHLQGLFDWRULQWRWKHFRQLFDOIODVNDQG
WLWUDWH WKLV VROXWLRQ ZLWK WKH K\GURFKORULF DFLG LQ WKH EXUHWWH ZLWK
FRQVWDQWVZLUOLQJDQGDGGGURSZLVHDVWKHWLWUDWLRQLVQHDUWKHHQG
SRLQW
 6WRSWKHWLWUDWLRQLPPHGLDWHO\ZKHQWKHILUVWGURSRIK\GURFKORULFDFLG
DGGHGWXUQVWKHVROXWLRQIURP\HOORZWRRUDQJH
 5HSHDWWKHWLWUDWLRQXQWLOWZRDFFXUDWHWLWUH UHDGLQJVIDOOZLWKLQ
FPRIHDFKRWKHUDUHREWDLQHG

   5HVXOWV
$VVXPHWKDWWKHYROXPHREWDLQHGLV9FP

 9
$PRXQWRI&D 2+ LQFP  [ PRO 
 
&RQFHQWUDWLRQRIVDWXUDWHG&D 2+  
 9  
  [  ]PROGP 

 
>&D @ ]PROGP 
>2+ @ ]PROGP 
. VS ] ]  ] PRO GP  

   >7RWDO@

trendyline
1415 trendyline
 7KHPDQXIDFWXUHRIDPPRQLDIURPQLWURJHQDQGK\GURJHQWDNHVSODFHLQWZRVWDJHV

)LUVW6WDJH7KHPDQXIDFWXUHRIK\GURJHQIURPPHWKDQH
6HFRQG6WDJH7KHV\QWKHVLVRIDPPRQLD WKH+DEHU3URFHVV 

7KH)LUVW6WDJHRFFXUVYLDDWZRVWHSSURFHVV

 Steam Reforming
 Shift Reaction

%RWKVWHSVSURGXFHK\GURJHQJDVWKDWZLOOEHXVHGLQWKH6HFRQG6WDJHIRUWKH
SURGXFWLRQRIDPPRQLD

D  Steam ReformingLVDUHDFWLRQWKDWFRQYHUWVPHWKDQHDQGVWHDPWRDPL[WXUHRI
FDUERQPRQR[LGHDQGK\GURJHQ7KHUHDUHWZRSRVVLEOHURXWHVIRULW

Primary Steam Reforming:6WHDPLVVXSSOLHGWRVWDUWWKHUHDFWLRQ
&+ J +2 J &2 J + J HR N-PROeqm  

Secondary Steam Reforming:6RPHK\GURJHQLVEXUQWWRIRUPVWHDPZKLFKWKHQ
UHDFWVZLWKPHWKDQHWRJHQHUDWHPRUHK\GURJHQ
+ J 2 J +2 J HR N-PRO
&+ J +2 J &2 J + J HR N-PRO

L  6WDWHLIWKHRYHUDOOSURFHVVRISecondary Steam ReformingLVDQH[RWKHUPLF
RUHQGRWKHPLFSURFHVV
>@
  2YHUDOOLVH[RWKHUPLF
LL  6XJJHVWZKLFKVWHDPUHIRUPLQJSURFHVV PrimaryRUSecondary LVIDYRXUHG
E\KLJKWHPSHUDWXUH
>@
  3ULPDU\VWHDPUHIRUPLQJLVIDYRXUHGE\KLJKWHPSHUDWXUH

)RU SULPDU\ VWHDP UHIRUPLQJ SURFHVV LV HQGRWKHUPLF %\ /H &KDWHOLHUV
3ULQFLSOH /&3 KLJKWHPSHUDWXUHIDYRXUHGWKHHQGRWKHUPLFUHDFWLRQZKLFK
LVWKHIRUZDUGUHDFWLRQDVH[FHVVKHDWHQHUJ\LVEHLQJDEVRUEHG
LLL  7KH PHWKDQH JDV XVHG XVXDOO\ FRQWDLQV VRPH RUJDQLF VXOIXU FRPSRXQGV
56+DQG K\GURJHQVXOILGH+6ERWKRIZKLFKPXVWEHUHPRYHGRULWZLOO
SRLVRQWKHFDWDO\VW+HQFHWKHPHWKDQHPL[WXUHQHHGVWREHSDVVHGWKURXJK
DdesulfurisationunitILUVW

 7KHVH RUJDQLF VXOIXU FRPSRXQGV DUH ILUVW FRQYHUWHG LQWR K\GURJHQ
VXOILGHDQGDK\GURFDUERQE\UHGXFWLRQZLWKK\GURJHQJDV

 7KH K\GURJHQ VXOILGH LV UHDFWHG ZLWK ]LQF R[LGH =Q2  WR
SURGXFH]LQFVXOILGH =Q6 DVRQHRIWKHSURGXFWV

&RQVWUXFW WZR EDODQFHG FKHPLFDO HTXDWLRQV VKRZLQJ WKH UHDFWLRQV WKDW

ttren
trendyli
trendyline
ndyline
yli
RFFXUUHG
FF LQ WKH desulfurisationunit
nu  XVLQJ
desulfurisationunit J 56+
5 WR UHSUHVHQW RUJDQLF VXOIXU
FRPSRXQGVDQG5+WRUHSUHVHQWK\GURFDUERQ
FRPSRXQGVDQG5+WRUHSUHVHQWK\GURFDUERQ
>@
  56++
6+ 5+
5++6 6

=Q2+6=Q6+2

1416 trendyline
E  ,QWKH Shift ReactionWKHFDUERQPRQR[LGHSURGXFHGGXULQJWKH Steam Reforming
SURFHVV ZDV FRQYHUWHG WR FDUERQ GLR[LGH DQG K\GURJHQ JDVShift Reaction LV
GRQHYLDDWZRVWHSSURFHVV

 High Temperature Shift ReactionZLWKLURQ ,,, R[LGHFDWDO\VW

 Low Temperature Shift ReactionZLWKWKHUPDOO\XQVWDEOHFRSSHUFDWDO\VWDW
.7KHFDUERQPRQR[LGHFRQFHQWUDWLRQLVIXUWKHUUHGXFHGWR

7KHUHDFWLRQLQYROYHGLQWKHShift Reaction LVLOOXVWDWHGLQWKHIROORZLQJHTXDWLRQ

&2 J +2 J &2 J + J HR N-PROeqm  



L  6WDWHWKHIXOOHOHFWURQLFFRQILJXUDWLRQRILURQ ,,, LRQDQGFRSSHUPHWDO
>@
  )HVVSVSG&XVVSVSGV

LL  :LWKUHIHUHQFHWRWKHShift ReactionVWDWHDQGH[SODLQLILURQ ,,, R[LGHDQG
FRSSHULVIXQFWLRQLQJDVKRPRJHQHRXVRUKHWHURJHQRXVFDWDO\VW>@
  7KH\DUHKHWHURJHQRXVFDWDO\VWV

7KHDYDLODELOLW\RISDUWLDOO\ILOOHGGRUELWDOVDOORZUHDFWDQWPROHFXOHVWR
EHDGVRUEHGRQWRWKHFDWDO\VWVXUIDFHYLDYDQGHU:DDOVLQWHUDFWLRQV

F  7UDFHVRIFDUERQGLR[LGHSURGXFHGLQWKHShift ReactionLVVXEVHTXHQWO\UHPRYHG
E\SDVVLQJLWZLWKK\GURJHQJDVRYHUDQLFNHOFDWDO\VWDW.DSURFHVVNQRZQ
DVMethanation

&2 J + J &+ J +2 J 

8VLQJ WKH HQWKDOS\ FKDQJH YDOXHV RI eqm(1) DQG eqm(2) RI WKLV TXHVWLRQ
GHWHUPLQHWKHHQWKDOS\FKDQJHIRUWKLVMethanationSURFHVV
>@
 8VLQJDOJHEUDLFPHWKRG
 &+ J +2 J &2 J + J HR N-PRO
 &2 J +2 J &2 J + J HR N-PRO

5HYHUVH  5HYHUVH  
&2 J + J &+ J +2 J HR N-PRO
&2 J + J &2 J +2 J HR N-PRO

2YHUDOO&2 J + J &+ J +2 J HR N-PRO
25
8VLQJHQHUJ\F\FOH
H
&2 J + J &+ J +2



t dyli
trendyl
trendyline

&2 J +
&2 J +2 J + J  J 

%\+HVV/DZV/DZ
H   N- PR 
  N-PRO

1417 trendyline
G  7KHPDLQSURFHVVRIWKH+DEHU3URFHVVRFFXUVLQDIL[HGEHGUHDFWRU
1 J + J 1+ J HR N-PRO
7KH SURSRUWLRQ RI DPPRQLD LQ WKH HTXLOLEULXP PL[WXUH LQFUHDVHV ZLWK LQFUHDVLQJ
SUHVVXUH DQG ZLWK IDOOLQJ WHPSHUDWXUH 4XDQWLWDWLYH GDWD DUH JLYHQ WKH WDEOH
EHORZ

3UHVVXUHDWP 3HUFHQWDJHDPPRQLDSUHVHQWDW
HTXLOLEULXPDWDUDQJHRIWHPSHUDWXUH
 . . .
   
   
   
   
   
   

   
L  7RREWDLQDUHDVRQDEOH\LHOGZLWKIDYRXUDEOHUDWHKLJKSUHVVXUHVPRGHUDWH
WHPSHUDWXUHV DQG D FDWDO\VW DUH XVHG 8VLQJ WKH GDWD VHOHFW WKH
WHPSHUDWXUHDQGSUHVVXUHZKLFKZLOOUHVXOWLQWKHKLJKHVW\LHOGRIDPPRQLD
>@
  7HPSHUDWXUH.
3UHVVXUHDWP
LL  +HQFHXVLQJWKHGDWD\RXKDYHVHOHFWHGLQ D L GHWHUPLQHWKHYROXPHRI
DPPRQLDJDVQLWURJHQJDVDQGK\GURJHQJDVDWHTXLOLEULXPZKHQFPRI
QLWURJHQJDVLVUHDFWHGZLWKFPRIK\GURJHQJDVLQWKH+DEHU3URFHVV
/HDYH\RXUDQVZHUWRWZRGHFLPDOSODFHV
>@
  /HW[EHWKHYROXPHRI1+DWHTXLOLEULXP
 1 J + J 1+ J 
,QLWLDO   
&KDQJH [ [ [
(TP [ [ [

7RWDOYROXPHRIJDVDWHTP [[[
 [
)URP D L 
6LQFHDW.DQGDWPSHUFHQWDJHDPPRQLDSUHVHQWLV

  

 +HQFHDWHTP
 [  [ 9ROXPHRI1+ FP
[ [ 9ROXPHRI1  FP
[  9ROXPHRI+    FP

>7RWDO@

trendyline
1418 trendyline
 D  $OO\OLVRWKLRF\DQDWH $,7& LVUHVSRQVLEOHIRUWKHSXQJHQWWDVWHRIZDVDELZKLOH
FDSVDLFLQ LV DQ DFWLYH FRPSRQHQW RI FKLOOL SHSSHU DQG LV IRXQG LQ VSLFHG
SURGXFWVOLNHFXUU\


$,7&





&DSVDLFLQ

 L  8VLQJ\RXUFRQFHSWRI96(35VWDWHWKHVKDSHDERXWWKHFDUERQODEHOOHG
DQGRQ$,7&
>@
  &DUERQ7ULJRQDOSODQDU
&DUERQ7HWUDKHGUDO
 LL  6XJJHVWWKHW\SHRIK\EULGLVDWLRQIRUDOOFDUERQVSUHVHQWLQ$,7&
>@
  VS


VSVSVS
 LLL 
%HVLGHV WKH HWKHU JURXS  VWDWH WKH RWKHU IXQFWLRQDO JURXSV WKDW
DUHSUHVHQWLQ&DSVDLFLQ
>@
  $ONHQHVHFRQGDU\DPLGHSKHQRO
 LY  ,JQRULQJWKHHIIHFWRIWKH1 & 6JURXSZULWHDQHTXDWLRQVKRZLQJKRZ
$,7& UHDFWV ZLWK OLTXLG ,%U +HQFH ZLWK UHIHUHQFH WR WKH Data
BookletDQG JLYHQ WKDW WKH ERQG HQHUJ\ RI ,%U WR EH  N- PRO

GHWHUPLQHWKHHQWKDOS\FKDQJHRIUHDFWLRQZKHQ$,7&UHDFWVZLWKOLTXLG
,%U
>@
 

,%U 


+ %( & & %( ,%U %( &, %( &%U %( && 
 

ttrendyline
rendyline
e dy
dyli e
 N-PRO

 Y  ZK WKH EXUQLQJ
8VLQJ \RXU NQRZOHGJH LQ FKHPLFDO ERQGLQJ VXJJHVW ZK\
VHQVDWLRQRIFDSVDLFLQFDQQRWEHZDVKHGDZD\ZLWKZDWHU
VHQVDWLRQRIFDSVDLFLQFDQQRWEHZDVKHGDZD\ZLWKZDWHU
>@
  &DSVDLFLQLVDVLPSOHPROHFXODUVWUXFWXUH7KHUHLVQRIDYRXUDEOHVROXWH
UVW
VROYHQW LQWHUDFWLRQEHWZHHQ FDSVDLFLQ DQG ZDWHU +HQFH WKH EXUQLQJ

1419 trendyline
HIIHFWFRXOGQRWEHZDVKHGDZD\ZLWKZDWHU
 YL  'UDZ WKH VWUXFWXUDO IRUPXOD RI WKH SURGXFWV IRUPHG ZKHQ FDSVDLFLQ LV
UHDFWHGZLWKKRWDTXHRXVSRWDVVLXPK\GUR[LGH


><RXDUHWRLJQRUHWKHHIIHFWRIWKH JURXS@
>@
  




  >7RWDO@

trendyline
1420 trendyline
 D  3KRVJHQH LV WKH FKHPLFDO FRPSRXQG ZLWK WKH IRUPXOD &2&l ,W LV QRW WR EH
FRQIXVHGZLWKFREDOW ,, FKORULGHZLWKWKHIRUPXOD&R&l

3KRVJHQHLVSURGXFHGLQGXVWULDOO\ZLWKFDUERQGLR[LGHDQGFKORULQHJDV

&2 J &l J &2&l J HR N-PRO

7KH Kp RI WKLV SURFHVV LV DWP DW  . $ PL[WXUH FRQWDLQV  PRO RI
FDUERQPRQR[LGHDQGPRORIFKORULQHLQLWLDOO\ZDVUHDFWHGDQGDOORZWRUHDFK
HTXLOLEULXP7KHHTXLOLEULXPPL[WXUHLVIRXQGWRFRQWDLQPRORISKRVJHQH

 L  'UDZWKH/HZLVVWUXFWXUHRISKRVJHQH
>@
 



 LL  :ULWH DQ H[SUHVVLRQ IRU KS IRU WKH HTXLOLEULXP EHWZHHQ &2 &l DQG
SKRVJHQHDQGKHQFHXVLQJWKHLQIRUPDWLRQSURYLGHGFDOFXODWHWKHWRWDO
SUHVVXUHRIWKHV\VWHPDWHTXLOLEULXP
>@
 
.S  


 &2 J &l J &2&l J 
,QLWLDOQRRI   
PROHV
&KDQJHLQQR   
RIPROHV
(TPQRRI   
PROHV

7RWDOQRRIPROHVRIJDVHVDWHTP  

.S 
P COCl

P P
CO Cl


  
  PT PT PT 
  

 37 37;37 
 37 37 
 37
37 DWP
 LLL  6XJJHVW LI FKORULQH RU SKRVJHQH ZLOO GHYLDWH PRUH IURP LGHDO JDV

ttrendyli
trendyline
rendyline
y
EHKDYLRXU
>@
  3KRVJHQHJDVGHYLDWHVPRUHIURPLGHDOLW\
KRVJHQHJDVGHYLDWHVPRUHIURPLGHDOLW\
7KHSHUPDQHQWGLSROHSHUPDQHQWGLSROHLQWHUDFWLRQEHWZHHQ
7KHSHUPDQHQWGLSROHSHUPDQHQWGLSROHLQWHUDFWLRQEHWZ
SKRVJHQHPROHFXOHVLVVWURQJHUDVFRPSDUHGWRWKHLQVWDQWDQHRXV
SKRVJHQHPROHFXOHVLVVWURQJHUDVFRPSDUHGWRWKHLQVW
GLSROHLQGXFHGGLSROHEHWZHHQFKORULQHPROHFXOHV
HQ

1421 trendyline
E  &U\VWDOVRIK\GUDWHGFREDOW ,, FKORULGH&R&l+2ORVHZDWHUZKHQWKH\DUH
KHDWHGIRUPLQJDQK\GURXVFREDOW ,, FKORULGH&R&l

&R&l+2 V &R&l V +2 l 

 L  7KHVWDQGDUGHQWKDOS\FKDQJHIRUWKHUHDFWLRQ HLVN-PRO
*LYHQ WKDW WKH FDOFXODWLRQ IRU HQWURS\ FKDQJH WR EH VLPLODU WR WKDW RI
HQWKDOS\ FKDQJH XVH WKH IROORZLQJ HQWURS\ GDWD WR GHWHUPLQH WKH
VWDQGDUGHQWURS\FKDQJHRIWKHUHDFWLRQDW.*LYH\RXUDQVZHUWR
IRXUVLJQLILFDQWILJXUHV

&RPSRXQG S
-PRO.
&R&l+2 V  
&R&l V  
+2 l  
+2 J  

>@
  S   
 -PRO. WRVI 

 LL  +HQFH H[SODLQ E\ XVLQJ FDOFXODWLRQ ZKHWKHU K\GUDWHG FREDOW ,, 
FKORULGHFDQEHVWRUHGDW.ZLWKRXWGHFRPSRVLWLRQ
>@
  G H7S
[  
 
 
-PRO

<HVLWFDQEHVWRUHGZLWKRXWGHFRPSRVLWLRQDVGLVSRVLWLYH
F  :LWK WKH DLG RI D FKHPLFDO HTXDWLRQ H[SODLQ ZK\ D VROXWLRQ RI FREDOW ,, 
FKORULGHKDVDS+RI
>@
 )<,IRU&RFKDUJHLRQLFUDGLXVQP 
)<,IRU$OFKDUJHLRQLFUDGLXVQP 
)<,IRU0JFKDUJHLRQLFUDGLXVQP 

&R LRQKDVKLJK FKDUJH GHQVLW\RUKDVKLJK SRODULVLQJSRZHUZKLFKFDQ
GLVWRUWWKHHOHFWURQFORXGRIWKHZDWHUOLJDQG

>&R +2 @>&R +2  2+ @+

7KXVVROXWLRQLVDFLGLF
 >7RWDO@

1422 trendyline
 *URXS 9,, K\GULGHV DUH FRORXUOHVV JDVHV DW URRP WHPSHUDWXUH 7KH ILJXUHV EHORZ
VKRZVWKHUHODWLYHHQWKDOS\FKDQJHRIYDSRULVDWLRQRIWKHK\GULGHV











+YDS









 D  :LWK UHIHUHQFH WR )LJXUH  H[SODLQ WKH DEQRUPDOO\ KLJK HQWKDOS\ FKDQJH RI
YDSRULVDWLRQRIK\GURJHQIOXRULGH
>@
 +)PROHFXOHVDUHKHOGWRJHWKHUE\VWURQJHULQWHUPROHFXODUK\GURJHQERQGVDV
FRPSDUHG WR WKH RWKHU +; PROHFXOHV KHOG E\ ZHDNHU SHUPDQHQW GLSROH
SHUPDQHQWGLSROHLQWHUDFWLRQV

0RUH HQHUJ\LVQHHGHGWRRYHUFRPHVWURQJHULQWHUPROHFXODUK\GURJHQERQGVLQ
+)H[SODLQLQJLWVDEQRUPDOO\KLJKHQWKDOS\FKDQJHRIYDSRULVDWLRQ

E  7KH WDEOH EHORZ VKRZV WKH pKaRI WKH UHVSHFWLYH FRPSRXQGV ZKHQ WKH\ DUH
GLVVROYHGLQZDWHU

&RPSRXQGV pKa  &RPSRXQGV pKa
   +\GURJHQEURPLGH 
+\GURJHQFKORULGH   +\GURJHQLRGLGH 
0HWKDQRLFDFLG   :DWHU 


L  8VLQJUHOHYDQWLQIRUPDWLRQIURPWKHData BookletUDQNWKHDFLGVWUHQJWKRI
DTXHRXVVROXWLRQFRQWDLQLQJ+&l+%UDQG+,LQLQFUHDVLQJRUGHU([SODLQ
\RXUDQVZHU
>@
  )URPData Booklet, %RQGHQHUJ\+&l  >+%U  !+,  
(DVHRIFOHDYDJHRI+;ERQG+&l<+%U+,
2UHQHUJ\QHHGHGWREUHDN+;ERQG+&l>+%U!+,
(DVHRIUHOHDVHRI+LRQ+&l+%U+,

trendyline
trendyline
dyli
$FLGLW\+&l
FL +%U+,




LL  $FFRXQW IRU WKH UHODWLYH DFLGLWLHV RII PHWKDQRLF
P DFLG DQG ZDWHU
$FFRXQWIRUWKHUHODWLYHDFLGLWLHVRIPHWKDQRLFDFLGDQGZDWHU

1423 trendyline
>@
  0HWKDQRLF DFLG LV DVWURQJHU DFLGWKDQ ZDWHUDV LWV FRQMXJDWH EDVH
FDUER[\ODWHDQLRQ 5&22 LVUHVRQDQFHVWDELOLVHGE\WKHGHORFDOLVDWLRQ
RIWKHQHJDWLYHFKDUJHRYHUWKH&DWRPDQGERWKR[\JHQDWRPV

$V VXFK PHWKDQRLF DFLG LV OHVV OLNHO\ WR DFFHSW D SURWRQ IURP +; WKDQ
ZDWHU

F  8QOLNHRWKHU*URXS 9,,K\GULGHVK\GURJHQIOXRULGH+)EHKDYHVDVDZHDNDFLG
LQZDWHU

$ VROXWLRQ RI +) ZDV WLWUDWHG DJDLQVW DTXHRXVSRWDVVLXP K\GUR[LGH XQWLO  RI
WKH+)SUHVHQWZDVQHXWUDOLVHG

L  :ULWHWKHDFLGGLVVRFLDWLRQFRQVWDQWH[SUHVVLRQKDIRU+)
>@
 

KD  


LL  *LYHQ WKDW WKH S+ RI WKH UHVXOWLQJ VROXWLRQ GHVFULEHG DERYH LV 
FDOFXODWHKDRI+)
>@
  S+ ORJ>+@
>+@  [PROGP

/HW[EHWKHLQLWLDO>+)@
 +)+)
,QLWLDOPROGP [  
&KDQJHPROGP [ [ [
(TPPROGP [ [ [


 
KD    

[PROGP

$OWHUQDWLYHVROXWLRQ

S+ S.DOJ^>VDOW@>DFLG@`
 S.DOJ^>)@>+)@`
 S.DOJ  
.D [PROGP







t dyli
trendyline





G  2QH RI I WKH
WK PRVW W FRPPRQ ZD\V WR W PDNH N K\GURJHQ
K G KO LG LV WR UHDFW
FKORULGH

1424 trendyline
VRGLXP FKORULGH ZLWK FRQFHQWUDWHG VXOIXULF DFLG +62 RU FRQFHQWUDWHG
SKRVSKRULF 9 DFLG+32%RWKUHDFWLRQVRFFXUVLPLODUO\UHOHDVLQJZKLWHIXPHV
RIJDV
 L  &RQVWUXFWDEDODQFHGHTXDWLRQLQYROYLQJVRGLXPFKORULGHDQGFRQFHQWUDWHG
SKRVSKRULF 9 DFLG
>@
  1D&l+32+&l1D+32

 LL  ,Q UHDFWLRQ ZLWK VRGLXP EURPLGH FRQFHQWUDWHG VXOIXULF DFLG SURGXFHV
UHGGLVKEURZQ VROXWLRQ ZKLOH FRQFHQWUDWHG SKRVSKRULF 9  DFLG SURGXFHV
ZKLWHIXPHV

6XJJHVWDUHDVRQIRUWKHREVHUYDWLRQVDERYH
>@
  &RQFHQWUDWHGVXOSKXULFDFLGLVDEOHWRR[LGLVH%UWRUHGGLVKEURZQ%UZKLOH
FRQFHQWUDWHG SKRVSKRULF 9  DFLG FDQQRW 7KXV FRQFHQWUDWHG
SKRVSKRULF 9 DFLGLVDZHDNHUR[LGLVLQJDJHQW

  >7RWDO@

trendyline
1425 trendyline
 D  7KH FKHPLFDO VWUXFWXUH DQG VRPH UHDFWLRQV LQYROYLQJ FRPSRXQG 1 DUH VKRZQ
EHORZ

&XPHQH
SURFHVV







, ,,



1 3 4


 %U l 


 5

L  6WDWHWKHUHDJHQWVDQGFRQGLWLRQVLQYROYHGLQ6WHSV,DQG,,

6WHS,
6WHS,,
>@
  6WHS,/L$l+LQGU\HWKHUUWS
6WHS,,&+&2&lUWS

LL  'UDZWKHVWUXFWXUHRIRUJDQLFSURGXFW5
>@
 

t dyli
trendyline







1426 trendyline
LLL  3URSRVHDFKHPLFDOWHVWWRGLVWLQJXLVKEHWZHHQ3DQG1LQZKLFKDSRVLWLYH
WHVW LV REVHUYHG IRU 3 RQO\ :ULWH D EDODQFHG FKHPLFDO HTXDWLRQ IRU WKH
UHDFWLRQWKDWKDVRFFXUUHG
>@
  7HVW$GG.&U2LQGLOXWH+62KHDW
2EVHUYDWLRQ2UDQJH.&U2WXUQVJUHHQ

*KMnO4 is not suitable as ferulic acid may be [O] as well due to presence of
C=C and benzylic H
*distillation with K2Cr2O7to form aldehyde is not preferredwhen carrying out
simple chemical tests

(TXDWLRQ

>2@ +2




E  ,QWKHFXPHQHSURFHVVRQHPROHRIEHQ]HQHUHDFWVZLWKRQHPROHRISURSHQHLQ
WKH SUHVHQFH RI R[\JHQ IURP DLU WR IRUP SKHQRO DQG RUJDQLF VXEVWDQFH 7
6XEVWDQFH 7 GRHV QRW JLYH DQ\ SRVLWLYH REVHUYDWLRQ ZLWK GLDPPLQH VLOYHU
FRPSOH[

 L  )URPWKHLQIRUPDWLRQSURYLGHGLGHQWLI\VXEVWDQFH7
>@
 


7

 LL  +HQFHZLWKDQDLGRIDFKHPLFDOHTXDWLRQVXJJHVWKRZWKHIXQFWLRQDOJURXS
LQVXEVWDQFH7FDQEHSRVLWLYHO\LGHQWLILHG
>@
  RUDQJHSSWZLOOEHREVHUYHG



K

ttrrendyline
trendyline
reendyline
d li


  >7RWDO@


(1'
(1'

1427 trendyline


6(5$1*221-81,25&2//(*(
*HQHUDO&HUWLILFDWHRI(GXFDWLRQ$GYDQFHG/HYHO
+LJKHU

&$1','$7(1$0(

&/$66

&+(0,675<
-&3UHOLPLQDU\([DPLQDWLRQ6HSWHPEHU
3DSHU)UHH5HVSRQVH KRXUV
&DQGLGDWHVDQVZHURQVHSDUDWHSDSHU

$GGLWLRQDO0DWHULDOV 'DWD%RRNOHW
:ULWLQJ3DSHUV

5($'7+(6(,16758&7,216),567

:ULWH\RXUQDPHDQGFODVVRQDOOWKHZRUN\RXKDQGLQ
:ULWHLQGDUNEOXHRUEODFNSHQRQERWKVLGHVRIWKHSDSHU
<RXPD\XVHDVRIWSHQFLOIRUDQ\GLDJUDPVJUDSKVRUURXJKZRUNLQJ
'RQRWXVHVWDSOHVSDSHUFOLSVKLJKOLJKWHUVJOXHRUFRUUHFWLRQIOXLG

$QVZHUDQ\IRXUTXHVWLRQV
$'DWD%RRNOHWLVSURYLGHG
7KHXVHRIDQDSSURYHGVFLHQWLILFFDOFXODWRULVH[SHFWHGZKHUHDSSURSULDWH
<RXDUHUHPLQGHGRIWKHQHHGIRUJRRG(QJOLVKDQGFOHDUSUHVHQWDWLRQLQ\RXUDQVZHUV

7KHQXPEHURIPDUNVLVJLYHQLQWKHEUDFNHWV>@DWWKHHQGRIHDFKTXHVWLRQRUSDUW
TXHVWLRQ
$W WKH HQG RI WKH H[DPLQDWLRQ IDVWHQ DOO \RXU ZRUN VHFXUHO\ WRJHWKHU ZLWK WKH FRYHU SDJH
SURYLGHG

trendyline7KLVGRFXPHQWFRQVLVWVRISULQWHGSDJHV
7KLV GRFXPHQW FRQVLVWV RI  SULQWHG SDJHV

7XUQRYHU@
1428 trendyline



 6RGLXP ELVXOILWH 1D+62  LV D XVHIXO FRPSRXQG WR WUHDW FKHPLFDO ZDVWH DIWHU RUJDQLF
V\QWKHVLV,WUHDFWVZLWKPHWKDQDOFRQYHUWLQJLWLQWRDQRQWR[LFFRPSRXQGZKLFKFDQEH
GLVSRVHGVDIHO\7KHRYHUDOOHTXDWLRQLVVKRZQEHORZ
+62+&2+&+ 2+ 62



 D  'UDZWKHGRWDQGFURVVGLDJUDPRIWKHELVXOILWHLRQ+62 >@

 E  7KH LQLWLDO UDWH RI WKLV UHDFWLRQ FDQ EH VWXGLHG E\ WKH FORFN PHWKRG XVLQJ
SKHQROSKWKDOHLQDVDVXLWDEOHLQGLFDWRU7KHVXGGHQDSSHDUDQFHRIWKHSLQNFRORXU
LQGLFDWHVWKHWLPHWRVWRSWKHVWRSZDWFK

$ VHULHV RI H[SHULPHQWV ZDV FDUULHG RXW XVLQJ GLIIHUHQW FRQFHQWUDWLRQV RI +62
DQG+&2+7KHIROORZLQJUHVXOWVZHUHREWDLQHG

H[SHULPHQWQXPEHU >+62@ >+&2+@ WLPHIRUWKH
PROGP PROGP DSSHDUDQFHRIWKHSLQN
FRORXUV
   
   
   
   


  L  :KDWLVWKHVLPSOHUHODWLRQVKLSEHWZHHQWKHWLPHWDNHQIRUWKHSLQNFRORXUWR
DSSHDUDQGWKHLQLWLDOUDWHRIUHDFWLRQ">@

  LL  &DOFXODWH WKH LQLWLDO UDWH IRU HDFK RI WKHVH IRXU H[SHULPHQWV DQG XVH WKH
UHVXOWV WR GHGXFH WKH RUGHU RI UHDFWLRQ ZLWK UHVSHFW WR WKH WZR UHDFWDQWV
([SODLQ\RXUUHDVRQLQJ>@

  LLL  8VLQJ\RXUDQVZHULQ E LL ZULWHWKHUDWHHTXDWLRQIRUWKHUHDFWLRQVWDWLQJ
WKHXQLWVRIWKHUDWHFRQVWDQW>@

  LY  6NHWFKWKHJUDSKRIFRQFHQWUDWLRQDJDLQVWWLPHIRUHDFKUHDFWDQW>@

 F  .LQHWLFVWXGLHVVXJJHVWVWKDWWKHPHFKDQLVPLQYROYHVWKHIROORZLQJWZRVWHSV

6WHS%LVXOILWHUHDFWVZLWKZDWHUYLDDQDFLGEDVHUHDFWLRQIRUPLQJVXOILWH62

6WHS  7KH UHVXOWLQJ VXOILWH UHDFWV ZLWK PHWKDQDO WR SURGXFH WKH QRQWR[LF
FRPSRXQG&+ 2+ 62DQGK\GUR[LGHLRQ

  L  :ULWHDEDODQFHGHTXDWLRQHDFKIRUVWHSDQGVWHS>@

  LL  6XJJHVW ZK\ SKHQROSKWKDOHLQ LV D VXLWDEOH LQGLFDWRU WR GHWHUPLQH WKH
FRPSOHWLRQRIWKHUHDFWLRQ>@
 

trendyline
7XUQRYHU@
1429 trendyline



 G  $OGRO FRQGHQVDWLRQV DUH LPSRUWDQW LQ RUJDQLF V\QWKHVLV DV D JRRG ZD\ WR IRUP
FDUERQFDUERQERQGV




ZKHUH555DQG5FDQEHDON\OJURXSRUK\GURJHQDWRP 

7KH IROORZLQJ UHDFWLRQ VHTXHQFH LOOXVWUDWH KRZ 0,%. D VROYHQW XVHG RQ SDLQWV
FRXOGEHV\QWKHVLVHGIURPFDUERQ\OFRPSRXQG%,WLVNQRZQWKDW0,%.GRHVQRW
XQGHUJRDWWDFNE\HOHFWURSKLOH






  L  8VLQJWKHLQIRUPDWLRQSURYLGHGGUDZWKHGLVSOD\HGIRUPXODFDUERQ\O%>@

  LL  )RU HDFK RI WKH IROORZLQJ UHDFWLRQV GHVFULEH WKH PHFKDQLVP VKRZLQJ FXUO\
DUURZV GLSROHV DQG DQ\ UHOHYDQW ORQH SDLUV<RX DUH WR VWDWH WKH FRQGLWLRQ
UHTXLUHGIRUUHDFWLRQ

5HDFWLRQ,QWHUPHGLDWH&ZLWKGU\K\GURJHQEURPLGHJDV
5HDFWLRQ,QWHUPHGLDWH&ZLWKDTXHRXVK\GURJHQF\DQLGH

+HQFH VXJJHVW ZLWK H[SODQDWLRQ LI WKH SURGXFWV IRUPHG IURP WKH WZR
UHDFWLRQVFRXOGURWDWHSODQHSRODULVHGOLJKW
>@

  LLL  'HGXFHWKHVWUXFWXUHRI0,%.
>@
   
   >7RWDO@


trendyline


7XUQRYHU@
1430 trendyline



 D  7KLRSKHQRO LV DQ RUJDQRVXOIXU FRPSRXQG ZLWK WKH IRUPXOD &+6+ 7KH FKHPLFDO
VWUXFWXUH RI WKLRSKHQRO LV DQDORJRXV WR SKHQRO H[FHSW WKDW WKH R[\JHQ DWRP LQ WKH
K\GUR[\OJURXSERQGHGWRWKHDURPDWLFULQJLVUHSODFHGE\DVXOIXUDWRP

6+



7KLRSKHQRO

 L  6XJJHVW LI WKH &6+ ERQG DQJOH LQ WKLRSKHQRO ZLOO EH ODUJHU WKDQ WKH &2+
ERQGDQJOHLQSKHQRO
>@
 LL  ([SODLQZK\WKLRSKHQROKDVDORZHUpKDYDOXHWKDQSKHQRO
>@

 LLL  7KH UHDFWLRQ VFKHPH EHORZ VKRZV KRZ RQH WKLRSKHQRO GHULYDWLYH FRXOG EH
V\QWKHVLVHG LQWR DQRWKHU RUJDQLF FRPSRXQG LQ IRXU VWHSV XVLQJ LQRUJDQLF
UHDJHQWV 'UDZ WKH VWUXFWXUH RI WKH LQWHUPHGLDWH FRPSRXQGV DQG VWDWH WKH
UHDJHQWVDQGFRQGLWLRQVIRUHDFKVWHSDQG

2+

VWHS VWHS VWHS DQK\GURXV


)H&O
6+ &1
+6 
>@

E  7KHXVHRIData BookletLVUHOHYDQWLQWKLVTXHVWLRQ

,Q DQ H[SHULPHQW WR GHWHUPLQH WKH HQWKDOS\ FKDQJH RI FRPEXVWLRQ RI WKLRSKHQRO
 FP RI WKLRSKHQRO ZLWK GHQVLW\ RI  J FP ZDV EXUQW DV IXHO WR EULQJ D
FDORULPHWHUFRQWDLQLQJFPRIZDWHUDW&WRERLO

 L  'HILQHVWDQGDUGHQWKDOS\FKDQJHRIFRPEXVWLRQ
>@

 LL  &DOFXODWHWKHHQWKDOS\FKDQJHRIFRPEXVWLRQRIWKLRSKHQROHcJLYHQWKDWWKH
SURFHVVLVHIILFLHQW
>@


trendyline
7XUQRYHU@
1431 trendyline



F  7KHXVHRIData BookletLVUHOHYDQWLQWKLVTXHVWLRQ

,QQXFOHDUPDJQHWLFUHVRQDQFH 105 VSHFWURVFRS\WKHSURWRQFKHPLFDOVKLIWFDQEH
XVHGWRGLDJQRVHWKHVWUXFWXUHRIDQRUJDQLFPROHFXOH

&RPSRXQG'KDVDPROHFXODUIRUPXODRI&+2:KHQKHDWHGZLWKK\GURFKORULFDFLG
FRPSRXQG(DQG)DUHSURGXFHG

&RPSRXQG(KDVDSURWRQFKHPLFDOVKLIWRISSP:KHQPROHRI(LVZDUPHGZLWK
DONDOLQH DTXHRXV LRGLQH  PROH RI \HOORZ SUHFLSLWDWHV DUH IRUPHG DORQJ ZLWK
FRPSRXQG*:KLWHIXPHVZHUHDOVRREVHUYHGZKHQ(LVUHDFWHGZLWKWKLRQ\OFKORULGH

&RPSRXQG)&+[2URWDWHVSODQHSRODULVHGOLJKWDQGKDVDSURWRQFKHPLFDOVKLIWRI
SSP &RPSRXQG ) KRZHYHU GR QRW H[KLELW FLVWUDQV LVRPHULVP DQG ZLOO QRW
GHFRORXULVHR[LGLVLQJDJHQWV

 L  6XJJHVWWKHVWUXFWXUHVRI'WR)DQGH[SODLQWKHREVHUYDWLRQVGHVFULEHGDERYH
>@

 LL  &RQVWUXFWDEDODQFHGFKHPLFDOHTXDWLRQRIWKHUHDFWLRQEHWZHHQ(DQGDONDOLQH
DTXHRXVLRGLQH
>@

  >7RWDO@


 

7XUQRYHU@
1432 trendyline



 D  +DHPRJORELQLVWKHLURQFRQWDLQLQJR[\JHQWUDQVSRUWSURWHLQIRXQGLQWKHUHGEORRG
FHOOV

,QJHQHUDOKDHPRJORELQFDQEHVDWXUDWHGZLWKR[\JHQPROHFXOHV R[\KDHPRJORELQ 
RUGHVDWXUDWHGZLWKR[\JHQPROHFXOHV GHR[\KDHPRJORELQ 

7KHDEVRUSWLRQ VSHFWUDRI R[\KDHPRJORELQ DQG GHR[\KDHPRJORELQ GLIIHU 7KH
R[\KDHPRJORELQ KDV VLJQLILFDQWO\ ORZHU DEVRUSWLRQ RI WKH QPZDYHOHQJWKDV
FRPSDUHG ZLWK GHR[\KDHPRJORELQ ZKLFK LV YDOXHG DW QP ZDYHOHQJWK 7KLV
GLIIHUHQFHLVXVHGIRUWKHPHDVXUHPHQWRIWKHDPRXQWRIR[\JHQLQDSDWLHQW
VEORRG
E\DQLQVWUXPHQWFDOOHGDSXOVHR[LPHWHU

  L  :LWK UHIHUHQFH WR WKH KDHPRJORELQ PROHFXOH GHVFULEH DQG H[SODLQ ZKDW LV
PHDQW E\ WKH WHUP quaternary structure RI SURWHLQV ,Q \RXU DQVZHU \RX
VKRXOGVWDWHWKHW\SHRIERQGLQJRULQWHUDFWLRQLQYROYHG
>@

  LL  +DHPRJORELQ LV W\SLFDOO\ DQ DOO DOSKDSURWHLQ FRQVLVWLQJ RI IRXU VXEXQLWV
+RZHYHU VRPH RI WKH FRQQHFWLQJ ORRSV EHWZHHQ WKH KHOLFHV FDQVRPHWLPHV
H[LVWLQDYHU\VKRUWEHWDVWUDQGHGFRQIRUPDWLRQ

:LWK WKH DLG RI D GLDJUDP GHVFULEH KRZ D SRO\SHSWLGH FKDLQ LV KHOG LQ WKH
VKDSHRIDEHWDVWUDQGHGFRQIRUPDWLRQ
>@

  LLL  8VLQJ WKH LQIRUPDWLRQ IURP WKH TXHVWLRQ H[SODLQ LI R[\KDHPRJORELQ RU
GHR[\KDHPJORELQFRQWDLQWKHODUJHUHQHUJ\JDS(EHWZHHQLWVGRUELWDO
>@






















t dyli
trendyline





 E  &DUERQ PRQR[LGH PDLQO\ FDXVHV DGYHUVH HIIHFWV LQ KXPDQV E\ FRPELQLQJ

7XUQRYHU@
1433 trendyline



ZLWKKDHPRJORELQLQWKHEORRG

+XPDQcytochrome c oxidaseLVFRPSRVHGRIVHYHUDOVXEXQLWV6RPHRIWKHDPLQR
DFLGVIRXQGLQcytochrome c oxidaseDUHOLVWHGEHORZ

$PLQRDFLG )RUPXODRIVLGHFKDLQ
  5LQ5&+ 1+ &2+

YDOLQH YDO  &+ &+ 
DVSDUWLFDFLG DVS &+&22+
O\VLQH O\V  &+&+&+&+1+
F\VWHLQH F\V  &+6+
DVSDUDJLQH DVS  &+&21+
WKUHRQLQH WKU  &+ 2+ &+
 
  L  ([SODLQKRZFDUERQPRQR[LGHDIIHFWVWKHQRUPDOIXQFWLRQLQJRIKDHPRJORELQ
>@

  LL  8VLQJ WKH GLSHSWLGH VHJPHQW RI O\VWKULQcytochrome c oxidase JLYH WKH
VWUXFWXUDO IRUPXODH RI WKH SURGXFWV REWDLQHG ZKHQ KRW K\GURFKORULF DFLG LV
DGGHG
>@

  LLL  Cytochrome c oxidaseFDQ XQGHUJR GHQDWXUDWLRQ GXH WR YDULRXV IDFWRUV
UHVXOWLQJLQORVVRILWVELRORJLFDOIXQFWLRQ

,GHQWLI\ WKH VLWH ZKHUHE\ WKH 5 JURXS LQWHUDFWLRQVDUH EURNHQ XQGHU WKH
IROORZLQJ FRQGLWLRQV DQG H[SODLQ KRZ WKHVH LQWHUDFWLRQV DUH DIIHFWHG LQ HDFK
FDVHV

    7KHDGGLWLRQRIDUHGXFLQJDJHQW
    7KHDGGLWLRQRIDEDVH
>@

 F  ,QF\DQLGHSRLVRQLQJWKHHQ]\PHcytochrome c oxidase ZKLFKDLGLQUHVSLUDWLRQLQ
KXPDQDUHWKRXJKWWREHDGYHUVHO\DIIHFWHG

7RWUHDWWKLVFDVHVRISRLVRQLQJWKH8QLWHG6WDWHVWDQGDUGF\DQLGHDQWLGRWHNLWZDV
GHYHORSHGDQGLWFRPSULVHVRIDWKUHHVWHSSURFHVV

6WHS,QKDOHVPDOOGRVDJHRIDP\OQLWULWH
6WHS,QWUDYHQRXVRIVRGLXPQLWULWH1D12
6WHS,QWUDYHQRXVRIVRGLXPWKLRVXOIDWH

  L  6RGLXP QLWULWH LV SDUW RI WKH F\DQLGH DQWLGRWH NLW ,W GHFRPSRVHV WR IRUP DQ
DFLGLFEURZQJDVDVROLGUHVLGXHDQGDIUHHUDGLFDO

:KHQWKHUHVLGXDOVROLGZDVSODFHGLQZDWHULWIRUPVDVROXWLRQRIS+

trendyline

8VLQJ
VLQJ WKH LQIRUPDWLRQ SURYLGHG FRQVWUXFW D EDODQFHG FKHPLFDO
FKHP
FKHPL HTXDWLRQ IRU
WKHGHFRPSRVLWLRQRIVRGLXPQLWULWH>@
HGHFRPSRVLWLRQRIVRGLXPQLWULWH>
HGHFRPSRVLWLRQRIVRGLXPQLWULWH>@

  LL  7KLRVXOIDWH
LRVXOIDWH UHDFWV GLIIHUHQWO\ ZLWK FKORULQH DQG LRGLQH 8VLQJ
KLRVXOIDWH 8VLQJ HTXDWLRQV RQO\
VXJJHVWDQH[SODQDWLRQIRUWKLVGLIIHUHQFH>@
XJJHVWDQH[SODQDWLRQIRUWKLVGLIIHUHQFH>@
HUH

7XUQRYHU@
1434 trendyline



   
 G  7KH 0DLOODUG UHDFWLRQ LV D FKHPLFDO UHDFWLRQ EHWZHHQ DPLQR DFLG DQG UHGXFLQJ
VXJDU WKDW JLYHV EURZQHG IRRG LWV GHVLUDEOH IODYRXU 3DQIULHG GXPSOLQJV IUHQFK
IULHV DQG WRDVWHG PDUVKPDOORZV XQGHUJR WKLV UHDFWLRQ ZKHQ KHDWHG DW KLJK
WHPSHUDWXUH

)LJXUH7KH0DLOODUGUHDFWLRQ

$PLQRDFLG




$FU\ODPLGH &+12 D SRVVLEOH KXPDQ FDUFLQRJHQ FDQ EH JHQHUDWHG DV D
E\SURGXFWRI0DLOODUGUHDFWLRQEHWZHHQUHGXFLQJVXJDUVDQGDPLQRDFLG

  L  8VLQJWKHWDEOHIURP E VWDWHWKHDPLQRDFLGWKDWLVUHTXLUHGLQWKHV\QWKHVLV
RIDFU\ODPLGH
>@

  LL  6XJJHVWWKHJDVWKDWZDVSURGXFHGLQ6WHS,,DQGSURSRVHKRZWKLVJDVFDQ
EHLGHQWLILHG
>@

  LLL  6XJJHVWWKHVWUXFWXUDOIRUPXODRIDFU\ODPLGH
>@

  LY  3URSRVH DQ LVRPHU RI DFU\ODPLGH ZKLFK GRHV QRW HYROYH DPPRQLD JDV RQ
KHDWLQJ ZLWK DTXHRXV VRGLXP K\GUR[LGH +HQFH VWDWH WKH W\SH RI LVRPHULVP
LQYROYHG
>@

trendyline

   >7RWDO@


7XUQRYHU@
1435 trendyline



 D  /LPHVWRQH LV PDLQO\ FRPSRVHG RI FDOFLWH &D&2 DQG LV XVXDOO\ ZKLWH RU
WUDQVSDUHQW VKLPPHULQJ FU\VWDOV GHSHQGLQJ RQ KRZ LW LV IRUPHG ,W WDNHV RQ D
GLIIHUHQWFRORXUZKHQH[SRVHGWRGLIIHUHQWLPSXULWLHVVXFKDVVLGHULWH)H&2

  L  8VLQJLQIRUPDWLRQIURPWKHData BookletVXJJHVWLIFDOFLWHRUVLGHULWHKDVD
ORZHUGHFRPSRVLWLRQWHPSHUDWXUH
>@

  LL  'RORPLWL]DWLRQLVDSURFHVVE\ZKLFKGRORPLWH&D0J &2 LVIRUPHGZKHQ
PDJQHVLXPLRQVUHSODFHVFDOFLXPLRQVLQFDOFLWH

:KHQJRIDQLPSXUHVDPSOHRIGRORPLWHZDVFRPSOHWHO\GLVVROYHGLQ
H[FHVV K\GURFKORULF DFLG WKH FDUERQ GLR[LGH HYROYHG ZDV EXEEHG LQWR
DTXHRXV FDOFLXP K\GUR[LGH ZKHUH  J RI ZKLWH LQVROXEOH VROLG ZDV
LVRODWHG&DOFXODWHWKHSHUFHQWDJHSXULW\RIWKHGRORPLWH
>@

 E  6WDODJPLWHV DQG VWDODFWLWHV LQ OLPHVWRQH FDYHV DUH FUHDWHG ZKHQ WKH FDOFLWH LV
GLVVROYHGLQUDLQZDWHUDQGVXEVHTXHQWO\SUHFLSLWDWHGZKHQWKH ZDWHUGULSVRIIWKH
FHLOLQJRIWKHFDYHWRWKHFDYHIORRU

'LVVROXWLRQRIFDOFLWHRFFXUVPDLQO\YLDWKHIROORZLQJHTXLOLEULXP

&D&2 V &D DT &2 DT   

+RZHYHULWLVDOVRDIIHFWHGE\WKHDPRXQWRIDWPRVSKHULFFDUERQGLR[LGHSUHVHQW
DQGLWFDQEHUHSUHVHQWHGYLDWKHVHIRXUDGGLWLRQDOHTXLOLEULD

&2 DT + DT +&2 DT   


+&2 DT + DT +&2 DT   
+&2 DT +2 l &2 DT   
&2 DT &2 J   

  L  &DOFXODWH WKH HQWKDOS\ FKDQJH RI VROXWLRQ RI FDOFLWH XVLQJ WKH LQIRUPDWLRQ
JLYHQEHORZ
 +RIN-PRO
&D&2 V  

&D DT  

&2 DT  
>@

  LL  ([SODLQ KRZ WKH HQWKDOS\ FKDQJH RI VROXWLRQ RI FDOFLWH ZRXOG GLIIHU IURP

trendyline
VLGHULWH
VLGHULWH
G
>@




7XUQRYHU@
1436 trendyline





  LLL  *LYHQ WKDW WKH HQWKDOS\ FKDQJH RI DWRPLVDWLRQ RI FDOFLXP LV  N- PRO
DQG WKH ODWWLFH HQHUJ\ RI FDOFLWH LV  N- PRO FDOFXODWH WKH HQWKDOS\
FKDQJH RI IRUPDWLRQ RI JDVHRXV FDUERQDWH LRQV XVLQJ DQ HQHUJ\ OHYHO
GLDJUDPDQGRWKHUUHOHYDQWLQIRUPDWLRQIURPWKHData Booklet
>@
  LY  :LWK UHIHUHQFH WR WKH HTXLOLEULXPV JLYHQ VXJJHVW ZKDW ZRXOG KDSSHQ WR WKH
FDOFLWHIRUPDWLRQVLQOLPHVWRQHFDYHVZKHQWKHDFLGLW\RIUDLQDQGULYHUZDWHU
LQFUHDVHVGXHWRSROOXWDQWV
>@

F  /LPHVFDOH ZKLFK LV SULPDULO\ FDOFLXP FDUERQDWH LV WKH RIIZKLWH GHSRVLW WKDW LV
FRPPRQO\ IRXQG LQ NHWWOHV ,Q DGGLWLRQ WR EHLQJ XQVLJKWO\ DQG KDUG WR FOHDQ
OLPHVFDOHLPSDLUVWKHRSHUDWLRQRIPHGLFDOHTXLSPHQW

(WKDQRLFDFLGDQGJO\FROLFDFLGFDQVHUYHDVJRRGGHVFDOLQJDJHQWDQGDUHXVHGWR
UHPRYHWKHOLPHVFDOH
2
2+
+2 
*O\FROLFDFLG

%RWKRUJDQLFDFLGVFDQEHWUHDWHGXVLQJWKHIROORZLQJVWHSVWRUHFRYHUDFDUERQ\O
FRQWDLQLQJFRPSRXQGIURPOLPHVFDOH

6WHS  +HDW WKH OLPHVFDOH ZLWK WKH RUJDQLF DFLG WR HQVXUH DOO WKH OLPHVFDOH KDV
UHDFWHG

6WHS(YDSRUDWHWKHVDPSOHDQGFROOHFWWKHVDOW

6WHS+HDWWKHVDOWLQDWHVWWXEHDQGDOORZWRFRROWRURRPWHPSHUDWXUH&DOFLXP
FDUERQDWHZLOOEHIRUPHGWRJHWKHUZLWKDOLTXLGFDUERQ\OFRQWDLQLQJFRPSRXQG

6WHS6HSDUDWHWKHFDOFLXPFDUERQDWHUHVLGXHIURPWKHOLTXLGFDUERQ\OFRQWDLQLQJ
FRPSRXQGXVLQJYDFXXPILOWUDWLRQ

 L  3URSRVH FKHPLFDO WHVW WR GLIIHUHQWLDWH HWKDQRLF DFLG DQG JO\FROLF DFLGLQ WKH
VFKRRO ODERUDWRU\ <RX DUH WR VWDWH FOHDUO\ WKH UHDJHQWV DQG FRQGLWLRQ XVHG
DQGWKHH[SHFWHGREVHUYDWLRQWREHPDGH
>@


trendyline
y
 LL  :ULWH D EDODQFHG HTXDWLRQIRU WKH UHDFWLRQ RI OLPHVFDOH DQG HWKDQRLF DFLG
,QFOXGHVWDWHV\PEROVLQ\RXUDQVZHU
>@



7XUQRYHU@
1437 trendyline




 LLL  :LWKUHIHUHQFHWRVWUXFWXUHDQGERQGLQJVXJJHVWZK\LWLVSRVVLEOHWRLVRODWH
WKHVDOWYLDHYDSRUDWLRQ
>@

 LY  :LWK UHIHUHQFH WR 6WHS  VXJJHVW WKH VWUXFWXUDO IRUPXODH RI WKH RUJDQLF
SURGXFWVIRUPHG ZKHQ FDOFLXP HWKDQRDWH DQG FDOFLXP JO\FRODWH LV KHDWHG LQ
VHSDUDWHWHVWWXEHV
>@

  >7RWDO@


trendyline


7XUQRYHU@
1438 trendyline



 D  7KHIROORZLQJWDEOHVKRZVVRPHFRPPRQR[LGDWLRQQXPEHUVRIVHOHFWHGPHWDOV

0HWDOV 2[LGDWLRQ1XPEHUV

6F   

7L   

9   

)H   

&X   

=Q   

7DEOH

  L  )URP7DEOHVWDWHDOOWKHLRQVWKDWDUHFRORUHG
>@

  LL  8VLQJRQHRIWKHLRQVVWDWHGLQ D L H[SODLQZK\LWLVFRORUHG
>@

  LLL  :KHQ GLOXWH DTXHRXV DPPRQLD LV DGGHG WR D VROXWLRQ WKDW FRQWDLQV
FRSSHU ,, LRQVDSDOHEOXHSUHFLSLWDWHLVREWDLQHG7KLVSUHFLSLWDWHGLVVROYHV
WRIRUPDGHHSEOXHVROXWLRQZKHQH[FHVVDPPRQLDLVDGGHG

*LYHDQH[SODQDWLRQIRUWKHVHREVHUYDWLRQVXVLQJUHOHYDQWHTXDWLRQV
>@

 E  ,URQ E\ PDVV LV WKH PRVW FRPPRQ HOHPHQW RQ(DUWK IRUPLQJ PXFK RI (DUWK
V
RXWHUDQGLQQHU FRUH ,WIRUPV FRPSRXQGV PDLQO\ LQ WKH  DQG  R[LGDWLRQ VWDWHV
,URQDQGLWVFRPSRXQGVDUHZLGHO\XVHGDVFDWDO\VW

  L  6XJJHVWZK\LURQKDVWKHDELOLW\WRH[LVWLQYDULDEOHR[LGDWLRQVWDWHV
>@

  LL  +\GURJHQSHUR[LGHGHFRPSRVHDFFRUGLQJWRWKHIROORZLQJHTXDWLRQ

+2 DT +2 l 2 J 

,URQ ,,, LRQVFDQEHXVHGWRFDWDO\VHWKLVGHFRPSRVLWLRQ8VLQJUHOHYDQWGDWD


IURPWKHData BookletVXJJHVWDPHFKDQLVPIRUWKLVFDWDO\VLVFDOFXODWLQJWKH
(YDOXHVIRUHDFKVWHS
>@


trendyline







7XUQRYHU@
1439 trendyline





  LLL  7KHIROORZLQJGLDJUDPVKRZVWKHUHDFWLRQVRIVRPHLURQFRPSRXQGV

6WHS, )H DT 
 <HOORZVROXWLRQ
&l JUHHQ


 0J 6WHS,,


 >)H &1 @ DT 
JDV-


 )H


 'HHSEOXH
 SUHFLSLWDWH.


   1DPHWKHW\SHRIUHDFWLRQLQ6WHS,,DQGLGHQWLI\JDV-DQGSUHFLSLWDWH.
>@
 

trendyline
7XUQRYHU@
1440 trendyline



 F  0ROWHQ FDUERQDWH IXHO FHOOV 0&)&V  DUH KLJKWHPSHUDWXUH IXHO FHOOV WKDW XVH
DQHOHFWURO\WHFRPSRVHG RI D PROWHQ FDUERQDWH VDOW PL[WXUH VXVSHQGHG LQ D SRURXV
FKHPLFDOO\LQHUWFHUDPLFPDWUL[

7KHIROORZLQJGLDJUDPVKRZD0&)&

+ 2

:DWHUDQG
KHDWRXW &DUERQGLR[LGHLQ
&2

(OHFWURGH (OHFWURGH
0 (OHFWURO\WH 1

&2


$WWKHHOHFWURGH1FDUERQGLR[LGHLVUHDFWHGZLWKR[\JHQWRIRUPFDUERQDWHDVWKH
RQO\SURGXFW7KHFDUERQDWHLRQVPRYHWRZDUGVHOHFWURGH0DQGUHDFWZLWKK\GURJHQ
WRIRUPFDUERQGLR[LGHDQGZDWHU

  L  :ULWHKDOIHTXDWLRQVIRUWKHUHDFWLRQVWDNLQJSODFHDWWKHHOHFWURGHVRIWKHIXHO
FHOODQGKHQFHFRQVWUXFWWKHRYHUDOOHTXDWLRQIRUWKHIXHOFHOOUHDFWLRQ
>@

 LL  6WDWHWKHGLUHFWLRQRIWKHHOHFWURQIORZZLWKUHVSHFWWRWKHHOHFWURGH
>@

  LLL  %HVLGHV EHLQJ KLJK LQ HIILFLHQF\ DQG KLJK RSHUDWLQJ FRVW VXJJHVW RQH
DGYDQWDJHDQGGLVDGYDQWDJHRIWKHPROWHQFDUERQDWHIXHOFHOO
>@

   >7RWDO@

trendyline
(1'2)3$3(5
(1'2)3

7XUQRYHU@
1441 trendyline



%/$1.


7XUQRYHU@
1442 trendyline



%/$1.


trendyline
7XUQRYHU@
1443 trendyline
1

CHEMISTRY 9647/01

Paper 1 Multiple Choice 19th September 2016


1 hour
Additional materials: Multiple Choice Answer Sheet
Data Booklet

READ THESE INSTRUCTIONS FIRST

Write in soft pencil.


Do not use staples, paper clips, glue or correction fluid.

Write yourname
andCivics Group

Write and shade


your CG number
followed by
last 3 digits of NRIC

There are forty questions on this paper. Answer all questions. For each question there are four
possible answers A, B, C and D.
Choose the one you consider correct and record your choice in soft pencil on the separate
Answer Sheet.

Read the instructions on the Answer Sheet very carefully.

Each correct answer will score one mark. A mark will not be deducted for a wrong answer.
Any rough working should be done in this booklet.
The use of an approved scientific calculator is expected, where appropriate.

trendyline
This document consistss of 18 printed
nted p
pages.

2016 TJC H2 Chemistry Preliminary Examination [Turn Over


1444 trendyline
2

Section A
Part 1
For each question there are four possible answers, A, B, C and D. Choose the one you
consider to be correct.

1 During the bacterial leaching of low grade copper ores, acidified water is sprayed onto the ore
chalcopyrite, CuFeS 2 . Bacteria then convert the insoluble ore into a solution according to the
equation shown below:
4CuFeS 2 + 17O 2 + 4H+ 4Cu2+ + 4Fe3+ + 8SO 4 2- + 2H 2 O
No change occurs in the oxidation state of copper.
What changes in oxidation state occur for the iron and the sulfur in this reaction?

Change in oxidation state


Fe S
A +1 +8
B +1 -8
C -1 +6
D -1 -6

2 When cobalt metal is reacted with a solution containing cobalt(III) ions, only cobalt(II) ions are
formed. How many moles of Co and Co3+(aq) would result in a mixture containing both
cobalt(II) and cobalt(III) ions in the mole ratio of 3:1 after the reaction had taken place?

Moles of Co Moles of Co3+


A 1 2
B 1 3
C 1 5
D 2 3

3 Sodium hydrogen carbonate, NaHCO 3 , can be prepared from sodium sulfate by a three-step
process.
Na 2 SO 4 (s) + 4C(s) Na 2 S(s) + 4CO(g)
Na 2 S(s) + CaCO 3 (s) CaS(s) + Na 2 CO 3 (s)
Na 2 CO 3 (s) + H 2 O(l) + CO 2 (g) 2NaHCO 3 (s)

trendyline
sodium hydrogen carbonate (M r = 84.
What is the mass of sodiu 84.0),
84 0), to the ne
n
nearest kg, that could be
formed from 100 kg of the sodium sulfate (M r = 142
142.1),
142.1), assuming a 90% yyield in each step?

A 43 B 86 C 106 D 118 kg

2016 TJC H2 Chemistry Preliminary Examination [Turn Over


1445 trendyline
3

4 The following are flight paths of charged particles when accelerated in an electric field.

+
RS

ST

TU

Which of the following correctly identifies S, T and U?


S T U
15
A O+ 14
C+ 14
N+
15 14
B O C+ 28
Si+
14 16
C 1 O2+ 28
Si2+
14 14
D 1 C+ 28
Si4+

5 Lecithinis used as an emulsifying and stabilising agent in the food, pharmaceutical and
cosmetic industries.
R
O
O
O
O O P O
O O CH3
+
R N
H3C CH3
lecithin
=
where R long hydrophobic carbon chains

Which of the following statements about lecithin is correct?

A It can form van der Waals forces with other non-polar molecules.
B It has a low boiling point.
C It cannot rotate plane of polarised light.
D It can form hydrogen bonds between its own molecules.

6 Mixtures of argon and another gas are commonly used during welding. One such gaseous
mixture has a density of 1.82 g dm3 at s.t.p.

trendyline
e the other gas in this mixture? [Density of argon
What could be ar = 1.78 g dm3 at s.t.p.]
A neon B oxygen
xyge
oxygen
C fluorine D carbon
bon dioxide
carbon diox

2016 TJC H2 Chemistry Preliminary Examination [Turn Over


1446 trendyline
4

7 Which line in the table gives the signs of H, S and G for the melting of ice at room
temperature and pressure?

H S G
A +
B +
C + + +
D + +

8 Which reaction represents standard enthalpy change at 298 K?

A P 4 (l) 4P(g)
B H 2 (g) + Br 2 (g) 2HBr(g)
C HF(aq) + NaOH(aq) NaF(aq) + H 2 O(l)
D CH 4 (g) + 2O 2 (g) CO 2 (g) + 2H 2 O(g)

9 In the construction of heart pacemakers, it is possible to use a tiny magnesium electrode


which creates an electrical cell with the inhaled oxygen. The relevant half-cells are as
follows:
Mg2+ + 2e- Mg E= -2.38V
O 2 + 2H+ + 2e-H 2 O E= +1.23V
V

Under standard conditions, the cell e.m.f. would be +3.61V, but in the body a potential of
+3.25V is more usual.
What is the best explanation for this lower e.m.f.?

A the low concentration of Mg2+ ions surrounding the magnesium electrode


B the pH of between 7 and 8 of the body fluid surrounding the electrodes
C the high resistance of the body fluids surrounding the electrodes
D the small size of the magnesium electrode

10 Consider the following equilibrium system:


2AlCl 3 (g) Al 2 Cl 6 (g)
Which of the following statements will cause the position of the above equilibrium to shift to
the left?

trendyline
A decreasing
g the volume of the vessel
ng
B increasing
g the temperature
C pumping inert
nert gas into the vessel at constant volume
D pumping AlCl 3 gas into the vessel

2016 TJC H2 Chemistry Preliminary Examination [Turn Over


1447 trendyline
5

11 Which of the following statements is true about the following energy profile for a catalysed
reaction?

Energy F

E3

E1

E2

Extent of reaction

A F is the intermediate formed.


B The enthalpy change of the reaction is E 2 E 3 .
C The reaction is catalysed by a heterogenous catalyst.
D The second step of the reaction is the rate determining step.

12 Ethyl ethanoate undergoes a slow acid-catalysed hydrolysis in water where the concentration
of acid in the solution remains constant.
H+
CH 3 CO 2 CH 2 CH 3 + H 2 O CH 3 CO 2 H + CH 3 CH 2 OH

The rate equation is found to be


rate = k[CH 3 CO 2 CH 2 CH 3 ][H+]

In an experiment, when 1.0 mol dm-3HCl was reacted with 0.10 mol dm-3 ethyl ethanoate, the
half-life was found to be 42 min.

In a second experiment, 2.0 mol dm-3HCl was reacted with 0.20 mol dm-3 ethyl ethanoate.
What is the half-life of the second experiment?

A 10.5 min B 21 min C 42 min D 63 min

13 Ethanol has a pK a of 15.5 at room temperature. Sodium ethoxide can be produced by


reacting ethanol with sodium metal, and Solution P is 0.1mol dm-3 sodium ethoxide.
Which of the following statements is incorrect?

trendyline
A s a weak acid.
Ethanol is
B Ethoxide is a strong base
base.
C The pH off Solution P is
s 13.
D The type of reaction between ethanol and sod
sodium is acid -base
acid-base

2016 TJC H2 Chemistry Preliminary Examination [Turn Over


1448 trendyline
6

14 Which of the following graphs shows the correct trend in the physical property of the
period 3 elements?
A B

C D

15 Which of the following elements has an oxide with a giant structure and a chloride which is
readily hydrolysed?
A sodium B carbon
C silicon D phosphorus

16 Two experiments are carried out with anhydrous potassium chloride and observations 1
and 2 are made at the end of each experiment.
Experiment 1: Concentrated sulfuric acid is added to potassium chloride and the fumes
produced are bubbled into aqueous potassium bromide solution observation 1.
Experiment 2: Potassium chloride is dissolved in concentrated aqueous ammonia and the
resulting solution is then added to aqueous silver nitrate observation 2.
What are the observations1 and2?
observation1 observation2
A colourless solution colourless solution
B colourless solution white precipitate
preci
C brown solution
own soluti colourless
olourless solution
s
D own solution
brown soluti white precipitate
prec
preci

2016 TJC H2 Chemistry Preliminary Examination [Turn Over


1449 trendyline
7

17 Which property of the Group II elements (magnesium to barium) and their compounds
increases with increasing proton number?
A the magnitude of the enthalpy change of hydration of the metal ion
B the solubility of the sulfate
C the stability of the carbonate to heat
D the tendency to form complex ions

18 Letters written on paper using aqueous ammonium thiocyanate are invisible until turned
blood red by brushing the paper with aqueous iron(III) chloride. If the ammonium
thiocyanate is first made alkaline, the letters turned orange brown and are less clear.
Which of the following substances, when formed on the paper in these reactions, best
explains these observations?

with aqueous ammonium thiocyanate with alkaline aqueous


ammonium thiocyanate
A [Fe(H 2 O) 6 ]3+ Fe(OH) 3
B [Fe(H 2 O) 5 (SCN)]2+ [Fe(H 2 O) 6 ]3+
C [Fe(H 2 O) 5 (SCN)]2+ Fe(OH) 3
D [Fe(H 2 O) 5 (SCN)]2+ [Fe(OH) 4 ]-

19 An energy profile diagram for a single reaction step is shown below.

To which of the following steps in the reaction of ethane with bromine in the presence of
light does this diagram apply?
A CH 3 CH 2 + CH 3 CH 2 CH 3 CH 2 CH 2 CH 3
B CH 3 CH 2 + Br 2 CH 3 CH 2 Br + Br
C CH 3 CH 2 + Br CH 3 CH 2 Br

trendyline
D Br 2 Br
B
Br+
+ Br

2016 TJC H2 Chemistry Preliminary Examination [Turn Over


1450 trendyline
8

20 The oxo reaction is an important industrial process in which an alkene combines directly
with carbon monoxide and hydrogen under suitable conditions. The reaction with ethene is
shown below.
catalyst
CH 2 =CH 2 + CO + H 2 CH 3 CH 2 CHO
hightemperature
and pressure

Which of the following structural formulae correctly represents the product of the oxo
reaction starting with but-2-ene?

A CH 3 CH 2 CH(CH 3 )CHO
B CH 3 CH 2 CH 2 CH 2 CHO
C CH 3 CH 2 COCH 2 CH 3
D (CH 3 ) 2 CHCH 2 CHO

21 Compounds X and Y are isomers with the molecular formula C 3 H 6 O 2 .


O O
C O CH
CH3 3
CH3CH2 C OH

Compound XCompound Y
Which reagent can be used to distinguish between X and Y?

A sodium metal
B 2,4-dinitrophenylhydrazine
C aqueous alkaline iodine
D aqueous sodium hydroxide

22 The compound hex-3-en-1-ol has a strong leafy smell of newly cut grass and is used in
perfumery.
CH 3 CH 2 CH=CHCH 2 CH 2 OH
hex-3-en-1-ol

What are the organic products when hex-3-en-1-ol is treated with hot acidic K 2 Cr 2 O 7 ,
followed by hydrogen gas in the presence of platinum?

A CH 3 CH 2 CH 2 CH 2 CH 2 CH 2 OH
B CH 3 CH 2 CO 2 H and HO 2 CCH 2 CO 2 H

trendyline
C CH 3 CH
H 2 CH
C 2 OH and HOCH 2 CH 2 CH 2 OH
D CH 3 CH
H 2 CH 2 CH 2 CH
C 2 CO 2 H

2016 TJC H2 Chemistry Preliminary Examination [Turn Over


1451 trendyline
9

23 Which of the follow reaction produces an organic species that contains deuterium atom?
NaOD(aq)
A CH3CH2Br
heat
MnO4-, NaOD(aq)
B CH3CH2OH
heat
conc D2SO4
C CH3CH2OH
heat
conc D2SO4
D CH3CO2H + CH3OH
heat

24 Compound Z, C 4 H 6 O 2 , undergoes the following reactions.


It gives a red precipitate with Fehlings solution.
It gives white fumes with PCl 5 .
Upon reaction with hydrogen gas and nickel catalyst, an optically active product
is obtained.
What is compound Z?

O O OH
O
H C CH CH C H
A 2 2 B

H OH O OH
O
C C C H H C C H
C CH2 D C

H H CH 2

trendyline
2016 TJC H2 Chemistry Preliminary Examination [Turn Over
1452 trendyline
10

25 Psilocin is the pharmacologically active agent in the body after ingestion of some species
of psychedelic mushrooms.
H
OH
CH2 CH2 N
CH3

N
H
psilocin
Which bond formed by the following orbitals overlap is not present in psilocin?

A sp3 sp3 overlap between two C atoms


B sp2 sp2 overlap between two C atoms
C sp3 sp2 overlap between C and N atoms respectively
D sp3 sp3 overlap between C and N atoms respectively

26 How many stereoisomers are possible for the following compound?

A 23 B 27
C 28 D 210

trendyline
2016 TJC H2 Chemistry Preliminary Examination [Turn Over
1453 trendyline
11

27 What is the structure of the compound formed when compound W was heated with
aqueous sodium hydroxide, followed by acidification?
Br Br

NC CONHCH 3

trendyline
2016 TJC H2 Chemistry Preliminary Examination [Turn Over
1454 trendyline
12

28 Which one of the following shows the given molecules arranged in order of decreasing pK b
values?

A Cl
, Cl ,

B Cl
Cl , ,

C Cl
, Cl ,

D Cl
, , Cl

29 A polypeptide is subjected to hydrolysis.


The following was found:
The enzyme trypsin hydrolyses a peptide bond at the carboxylic end of arginine (arg) to
give 2 tripeptides and gly
The enzyme chemotrypsin hydrolyses a peptide bond at the carboxylic end of
tyrosine (tyr) to give 2 dipeptides and a tripeptide
The chemical cyanogen bromide cleaves the peptide bond at the carboxylic end of
methionine (met) to give a tetrapeptide and a tripeptide

What is the primary structure of the polypeptide?


A protyr-arg-met-tyr-arg-gly
B glyarg-tyr-met-arg-tyr-pro
C protyr-arg-tyr-met-arg-gly
D glyarg-met-tyr-arg-tyr-pro

trendyline
2016 TJC H2 Chemistry Preliminary Examination [Turn Over
1455 trendyline
13

30 Carnosine LVDGLSHSWLGHIRUPHGIURP-alanine and histidine, and has been documented in


recent research as having the ability to prevent many of the detrimental effects of aging.

carnosine

The isoelectric point of carnosine is 6.83. Which of the following is the predominant species
present in an aqueous solution at physiological pH 7.4?

trendyline
2016 TJC H2 Chemistry Preliminary Examination [Turn Over
1456 trendyline
14

Part 2
For each of the questions in this section, one or more of the three numbered statements 1 to
3may be correct.
Decide whether each of the statements is or is not correct. (You may find it helpful to put a tick
against the statements which you consider to be correct.)
The responses A to D should be selected on the basis of
A B C D
1,2 and 3 are 1 and 2 only 2 and 3 only 1 only is
correct are correct are correct correct
No other combination of statements is used as a correct response.

31 Boron nitride, BN, exists in two possible forms, hexagonal boron nitride and cubic boron
nitride. The structure of hexagonal boron nitride and cubic boron nitride are similar to
graphite and diamond respectively.

nitrogen atom

boron atom

hexagonal boron nitride cubic boron nitride

Which of the following statements are correct?

1 There is dative bonding in cubic boron nitride.


2 The layers in hexagonal boron nitride are held together by van der Waals forces.
3 The boron-nitrogen bond in cubic boron nitride is shorter than that in hexagonal
boron nitride.

32 The standard enthalpy changes of combustion of carbon are as follows:


C(graphite) = 393.1 kJ mol-1
C(diamond) = 395.0 kJ mol-1
Which of the following deductions can be made from the data above?

trendyline
1 e is less stable than diamond.
Graphite
2 e has a lower energy content than diamond
Graphite diamon
diamond..
3 nversion
version of diamond to graphite is exothermic.
The conversion exothermic

2016 TJC H2 Chemistry Preliminary Examination [Turn Over


1457 trendyline
15

The responses A to D should be selected on the basis of


A B C D
1,2 and 3 are 1 and 2 only 2 and 3 only 1 only is
correct are correct are correct correct
No other combination of statements is used as a correct response.

33 The circuit shown in the diagram was set up.


switch

brass electrode (an alloy 0.05 mol dm-3sulfuric acid platinum electrode
of copper and zinc

A steady current of 1.00 A was passes through the electrolyte for 10 minutes at room
conditions.
Which statements regarding the electrolysis are correct?

1 Zinc dissolves preferentially at the anode.


2 Effervescence is observed at the cathode.
3 pH of the solution increases.

34 Consider the following equilibrium system:


Fe 3 O 4 (s) + CO(g) CO 2 (g) + 3FeO(s) + YH
Which of the following statements are correct?

1 Increasing the temperature causes the position of equilibrium to shift to the right.
2 The equilibrium constant, K C , for this reaction has no units.
3 Adding FeOcauses the position of equilibrium to shift to the left.

35 Which of the following shows a general decrease for Period 3 chlorides from MgCl 2 to PCl 5 ?

trendyline
1 pH of the resulting solution when dissolve in water
2 m oxidation states of the elements in the chlorides
maximum chloride
3 electrical conductivity in solid state

2016 TJC H2 Chemistry Preliminary Examination [Turn Over


1458 trendyline
16

The responses A to D should be selected on the basis of


A B C D
1,2 and 3 are 1 and 2 only 2 and 3 only 1 only is
correct are correct are correct correct
No other combination of statements is used as a correct response.

36 The following data refer to copper as a typical transition element and to calcium as an s-
block element.
For which property are the data under the correct element?
property copper calcium
1 density/g cm-3 8.92 1.54
2 melting point/C 1085 842
3 electrical conductivity/ relative units 9.6 85

37 Consider the following equilibria shown together with the magnitude of their respective
equilibrium constants:
[Ni(H 2 O) 6 ]2+ + 6NH 3 [Ni(NH 3 ) 6 ]2+ + 6H 2 O
K stab = [[Ni(NH 3 ) 6 ]2+] = 2.0 x 108
[[Ni(H 2 O) 6 ]2+][NH 3 ]6

[Ni(H 2 O) 6 ]2+ + 3en [Ni(en) 3 ]2+ + 6H 2 O


K stab = [[Ni(en) 3 ]2+] = 4.0 x 1018
[[Ni(H 2 O) 6 ]2+][en]3
whereen represents ethylenediamine, a bidentate ligand.
Which of the following statements are correct?

1 Ammonia is a stronger ligand than water.


2 Ethylenediamine is a stronger ligand than ammonia.
3 The equilibrium constant, K stab , for the following reaction is equal to K stab / K stab :
[Ni(NH 3 ) 6 ]2+ + 3en [Ni(en) 3 ]2+ + 6NH 3

trendyline
2016 TJC H2 Chemistry Preliminary Examination [Turn Over
1459 trendyline
17

The responses A to D should be selected on the basis of


A B C D
1,2 and 3 are 1 and 2 only 2 and 3 only 1 only is
correct are correct are correct correct
No other combination of statements is used as a correct response.

38 Epoxy resins are polymers which are used in adhesives. One monomer used in their
manufacture has the following structure:

CH 2
CH CH 2
C l

This monomer is manufactured from propene in three stages:

CH CH
Cl2 (g) CH CH C
Stage I : CH2 3 CH2 2 l

Cl2 (aq) CH CH C
CH CH C CH2 2 l
Stage II : CH2 2 l
Cl OH

CH CH C OH- (aq) CH CH CH C
CH2 2 l 2 2 l
Stage III :
Cl OH
O

Which statements are correct for the above synthesis?

1 Stage I involves free radical substitution.


2 Stage II involves electrophilic addition.
3 Stage III involves nucleophilic addition.

39 Ovalbumin is the predominant protein in egg white.


A perfect poached egg, with firm white surrounding a velvety yolk, can be made by the
addition of vinegar into boiling water, before tipping the egg in and leaving it to cook for 1-2
minutes.
The addition of vinegar allows the egg white to coagulate faster around the yolk, instead of
dispersing in the water. Which of the following explains this observation?
1 H+ in vinegar denatures ovalbumin by protonating the R groups of amino acid
residues.

trendyline
2 H+ in vinegar
negar
egar denatures ovalbumin by protonating the N atom of the p
peptide linkages.
3 H+ in vinegar
negar denatures ovalbumin by breaking it down to its constitu
constituent amino acids.

2016 TJC H2 Chemistry Preliminary Examination [Turn Over


1460 trendyline
18

The responses A to D should be selected on the basis of


A B C D
1,2 and 3 are 1 and 2 only 2 and 3 only 1 only is
correct are correct are correct correct
No other combination of statements is used as a correct response.

40 Which of the following reagents can be used to distinguish between the two compounds
shown below?

1 aqueous bromine
2 acidified KMnO 4 , reflux
3 CH 3 Br with heating, followed by the addition of AgNO 3

trendyline
2016 TJC H2 Chemistry Preliminary Examination [Turn Over
1461 trendyline
3UHOLPLQDU\([DPLQDWLRQ
+&KHPLVWU\0&46ROXWLRQ

         
$ % % ' $ ' ' & % %
         
' % ' & & $ & & ' $
         
$ ' $ & & % % $ $ '
         
% & $ % ' % $ % ' &

 

trendyline
1462 trendyline
$ &
2EVHUYDWLRQ+62&l +&l+62
$ &KDUJH RI*URXS ,, PHWDO LRQV LV WKH VDPH EXW
+&l EXEEOHG LQWR %U QR YLVLEOH UHDFWLRQ LV WKHLRQLFUDGLXVLQFUHDVHVGRZQWKHJURXS+HQFH
REVHUYHGDQGVROXWLRQUHPDLQHGFRORXUOHVV ZLWKGHFUHDVLQJFKDUJHGHQVLW\WKHPDJQLWXGHRI
2EVHUYDWLRQ$GGLWLRQRI$JLQWRVROXWLRQ +K\GGHFUHDVHVGRZQWKHJURXS
FRQWDLQLQJ&lDQG1+ DT JLYHVDFRORXUOHVV
% 6ROXELOLW\ RI *URXS ,, VXOIDWH GHFUHDVHV GRZQ
VROXWLRQFRQWDLQLQJ>$J 1+ @DV$J&lLVVROXEOH
WKHJURXS
LQDT1+
& 'RZQ WKH JURXS DV WKH FKDUJH GHQVLW\ RI WKH
PHWDO LRQ GHFUHDVHV LWV SRODULVDWLRQ SRZHU
GHFUHDVHVWKXVWKHUHLVOHVVZHDNHQLQJRIWKH&
2ERQGUHVXOWLQJLQJUHDWHUVWDELOLW\
'7HQGHQF\WRIRUPFRPSOH[HVVKRXOGEH
GHFUHDVLQJDVWKHFKDUJHGHQVLW\RIWKHPHWDOLRQ
GHFUHDVHVGRZQWKHJURXS

& '
(QHUJ\ OHYHO GLDJUDP LQGLFDWHV LQ WKH VLQJOH
%ORRG UHG VHHQ LV GXH WR WKH FRPSOH[ IRUPHG LQ
UHDFWLRQ VWHS WKH UHDFWDQW V  XQGHUJR ERQG
WKHIROORZLQJUHDFWLRQ
EUHDNLQJRQO\LHUHDFWDQW V DEVRUEHQHUJ\7KHUH

>)H +2 @ 6&1
 LVQRERQGIRUPDWLRQWDNLQJSODFH+HQFHWUXHIRU

+2>)H +2  6&1 @ %ORRGUHG RSWLRQ'RQO\,QRSWLRQV$DQG&ERQGIRUPDWLRQ
 RQO\ RFFXUV ,Q RSWLRQ % WKHUH LV ERQG EUHDNLQJ
,QWKHSUHVHQFHRIDQDONDOLUHGEURZQSUHFLSLWDWH IROORZHGE\ERQGIRUPDWLRQLQWKHUHDFWLRQ
RILURQ ,,, K\GUR[LGHLVIRUPHGDVZHOOWKXV
GLOXWLQJWKHEORRGUHGFRORXUPDNLQJWKHOHWWHUV
DSSHDURUDQJHEURZQ
>)H +2 @2+ 
+2>)H 2+  +2 @ V UHGEURZQSSW
$ $
2[RUHDFWLRQ '13+ DQG ,1D2+ DT  GRHVQW UHDFW ZLWK ;
DQG<;LVDQHVWHUQRWDFDUERQ\OFRPSRXQG
< UHDFWV ZLWK 1D2+ DT  EXW LW GRHVQW JLYH
REVHUYDEOHUHVXOW
<UHDFWVZLWK1DWRJLYHHIIHUYHVFHQFHRI+


6LPLODUO\IRUEXWHQH


' $
&+&+&+ &+&+&+2+ 1XFOHRSKLOLFVXEVWLWXWLRQJLYHV&+&+2'
&U2+ % R[LGDWLRQWRJLYH&+&2
 & (OLPLQDWLRQWRJLYH&+ &+

trendyline
trend
dyline
&+&+&+ &+&+&2 2+ ' &RQGHQVDWLRQWRJLYH&+
HQVD &2&+
ZKLFKJRHVWKURXJKK\GURJHQDWLRQWRIRUP
K\GURJHQDWLRQWRIRUP
K\GURJHQDWLRQWRIRUP 
&+&+&+&+&+&2 2+
+


1463 trendyline
&
=LVDQDOGHK\GHDQGFRQWDLQV2+JURXS8SRQ
UHGXFWLRQE\+=IRUPVDSURGXFWWKDWFRQWDLQV &+
FKLUDOFDUERQDQGQRSODQHRIV\PPHWU\
$ 'RHVQRWFRQWDLQ2+ + 
% ,VQRWDQDOGHK\GH ' 3URGXFWIURPUHGXFWLRQ
& &RQWDLQV2+DQGDOGHK\GH3URGXFWRI &+ 2+ &+ &+ &+2+GRHVQRWKDYHFKLUDO
UHGXFWLRQUHDFWLRQLVDFKLUDOFRPSRXQG FDUERQ

& %
0D[LPXPQXPEHURIVWHUHRLVRPHUV QP 
7KHUHDUHFKLUDOFDUERQVLQFRPSRXQG7KHUH
LVQWDQ\& &WKDWFDQGLVSOD\FLVWUDQV
LVRPHULVP



$VVKRZQLQWKHGLDJUDPDERYHWKHUHLVQRVS & 

DWRPRYHUODSSLQJZLWKVS1DWRP

%
%U %U

DT1D2+KHDW
1& &21+&+ 

 DFLGLILFDWLRQ




$ $
7KHORZHUWKHS.E, WKHVWURQJHUWKHEDVH 7U\SVLQWUHDWPHQW
+HQFHPROHFXOHVVKRXOGEHDUUDQJHGIURP   DUJ   DUJ JO\
ZHDNHVWEDVHWRWKHVWURQJHVWEDVH 
 &KHPRWU\SVLQWUHDWPHQW
 W\U DUJ W\U  DUJ JO\
 W\U DUJ  W\U DUJ JO\
LVWKHOHDVWEDVLFGXHWRWKHFORVH 
SUR[LPLW\RIWKHHOHFWURQZLWKGUDZLQJ&lDWRP &\DQRJHQEURPLGHWUHDWPHQW
ZKLFKGHFUHDVHVWKHHOHFWURQGHQVLW\RQWKH1  W\U DUJ W\U PHW DUJ JO\ y
DWRP  W\U DUJ PHW W\U DUJ JO\

trend
dyline
d
 3ULPDU\VWUXFWXUH
3ULPDU\VWUXFWXUH
VWUXF
SURW\UDUJPHWW\UDUJJO\
SUR \UDUJPHWW\UDUJJO\

LVWKHPRVWEDVLFGXHWRWKHSUHVHQFHRIWZR
FGXHWRWKHSUHVHQFHRIWZ
FGXHWRWKHSUHVHQFHRIWZR 1RWH$PXFKIDVWHUZD\ZRXOGEHWRDQDO\VHWKH
1RWH$PXFKIDVWHUZD\ZRX
RWH$PXFKIDVWHUZD\ZRX
HOHFWURQUHOHDVHJURXSVDQGWKHDEVHQFHRIDQ\
XSVDQGWKHDEVHQFHRIDQ\
SV DQG WKH DEVHQFH RI DQ\ RSWLRQVJLYHQLQVWHDGRIVROYLQJLQWKHIRUZDUG
RSWLRQVJLYHQLQVWHDGRIVROY
WLRQVJLYHQLQVWHDGRIVRO
HOHFWURQZLWKGUDZLQJJURXSV
JURXSV GLUHFWLRQ
GLUHFWLRQ
LUH

1464 trendyline
'
*
*HQHUDOUXOH HV
7
7DNLQJWKHQ QHXWUDOIRUPRIWKHGLSHHSWLGHLQWRD
DFFRXQW
,IIS+!SI&
&22+ZLOOE EHGHSURWRQ QDWHGUHVXOOWLQJLQDQHHJDWLYHO\FKD
DUJHGGLSHSSWLGH
,IIS+SI1
1+ZLOOEHSURWRQDWHGUHVXOWLQJLQ QDSRVLWLYHO\FKDUJHGGLSHSWLGH
,IIS+ SIWKHGLSHSWLGH
HLVHOHFWULFD
DOO\QHXWUDO
S+!S,R RIFDUQRVLQHH S,  GHSURWR RQDWLRQ

%
H JUDSKLWH WKHUH
/LNH W DUH H[WHQVLYH YD
DQ GHU :DD
DOV
HV RI DWWUDFWLRQ EH
IRUFH HWZHHQ WKH H OD\HUV LQ
KH[DDJRQDOERUR RQQLWULGH
2SWLRQLVLQFRRUUHFW7KHEERURQQLWURJJHQERQGLQQ
KH[D DJRQDOERURRQQLWULGHLVVKRUWHUDQGGVWURQJHU
WKDQQWKDWLQFXE
ELFERURQQLWWULGHGXHWR
RWKHSLERQ
QG
SUHVVHQW(DFK%%DWRPXVHVVDOOLWVWKUH
HHYDOHQFH
HOHF
FWURQVWRIRUPFRYDOHQWERQGVZLWK KWKUHH
JKERXULQJQLWURJHQDWRP
QHLJ PV(DFKQLLWURJHQDWRP P
Q WKH FXELFF VWUXFWXUH HDFK % DWRP
,Q D LV ERQ
QGHG VWLOOKDVDORQHSDLURIHOHFWWURQVZKLFKKLWXVHVWR
FRYDOHQWO\ WR
R  1 DWRPV LQ D WHWUD
DKHGUDO PDQQQHU IRUP
PDGDWLYHSLERQGZLWKDQDGMDFHQ QWERURQ
Z
ZKHUHRQHR RIWKHERQGVVLVDGDWLYH HERQG ZKH HUH1 DWRPP
OR
RQHSDLURIHHOHFWURQLVG
GRQDWHGWR% %DWRP 

& $$
(]QQ=Q 9 (&X&X 99
)RUDQRGHR[LG GDWLRQVHOHFW(WKDWLV
VPRUH
QHJD DWLYH+HQFH=QLVSUHHIHUHQWLDOO\G
GLVFKDUJHG
LVFRUUHFW
)RUFDWKRGHUH HGXFWLRQVHHOHFW(WKDWWLVPRUH
SRVLWLYH
(]QQ=Q 9 (++ 9
+HQFH+HYROYYHG
+ DDT H + J HIIH
HUYHVFHQFHREVHUYHG 
LVFR
RUUHFW
>+@GHFUHDVHVS+RIVROXWLRQLQFUHDV VHVLV
FRUUHHFW
 

%
6
6WDWHPHQW LVFRUUHFW,QFUHDVLQJWKHWHPSHUD DWXUHIDYRXUVWKHHQGR RWKHUPLF IRUZDUG UHDFFWLRQDQGE\\
/H&KDWHOLHUVSULQFLSOHWKHSRVLWLR
RQRIHTXLOLEULXPZLOOVKLIWWRWKHULJ
JKW
6
6WDWHPHQW LVFRUUHFW6LQFH)H2 V DQG)H H2 V DUHER RWKLQVROLGVWDWHVWKH\\DUHQRWSD
DUWRIWKH.&
H[SUHVVLRQ.& >&2 J J @>&2 J @@7KHUHIRUH.&KDVQRX XQLWV
6
6WDWHPHQW HFW$GGLQJDVROLGUHDFFWDQWRUSUR
LVQRWFRUUH RGXFWGRHVQ QRWDIIHFWWK
KHSRVLWLRQR
RI
HTXLOLEULXP

' %
%
 S+RIDTX XHRXVVROXWLRQRIFKORULG GHVGHFUHDVVHV
2SWLRQ  'XH WR LWV JUHDDWHU QXFOHDUU FKDUJH &X
&

trendyline
d
IURP 0J&l WR 3&l 
KDV
KDV D KLJKHU
KLJKH DWRPLF PD DVV DQG VP PDOOHU DWRPPLF
 PD[LPXP P R[LGDWLRQ
RQ
Q VWDWHV RI WKH
W HOHPHQQWV LQ
YR PH+HQFH
YROX H+HQFHLWVGHQVLW
LWVGHQVLW\\\LVKLJKHUWK
LV
LV KDQ&D
WKHFKORULLGHVLQFUHDV
HDVVVHVIURP0J
HD HVIURP0J
HVIURP0 J&l
J &lWR3&
WR3&l
 $OODUHLQVVXODWRUVLQW
LQ
QWWKHVROLGVW
WWKHVROLGVWD
HVROLGVWD
DWHIURP0J
D WHIURP0J
WHIURP0 J&l
J &l 2SWL 0HWD
SW RQ  0HW
0H DOOLF
D OOLF ERQG IR
IR
RU
U &X LV VWWURQJHU VLQFFH
U
WR3&l WK
ERWK
ERW K G DQG V HOH HOHFFFWURQV
WUR DUH GHORFDOLVH HG
FRP
FR
RP
PSDUHGWRWK
SDUHGWRWKKHVHOHFWURQV RQVRQO\IR RU&D+HQFFH
PRUH HHQHUJ\LVVQHHGHGGX XULQJPHOWLQJDFFRXQWLQ QJ

1465 trendyline
IRULWVKLJKHUPSW
2SWLRQ(OHFWULFDOFRQGXFWLYLW\IRU&XVKRXOGEH
KLJKHUVLQFHLWKDVDJUHDWHUQXPEHURI
GHORFDOLVHGHOHFWURQV ERWKGDQGV WRFRQGXFW
HOHFWULFLW\FRPSDUHGWRWKHVHOHFWURQVRQO\IRU
&D

$ .VWDE BB>>1L 1+ @@BB
6WDWHPHQWLVFRUUHFW7KHPDJQLWXGHRIWKH >>1L +2 @@>1+@

HTXLOLEULXPFRQVWDQWLVODUJH !! ZKLFK
LPSOLHVWKHSRVLWLRQRIHTXLOLEULXPOLHVIDUWRWKH .VWDE BB>>1L HQ @@BBB
ULJKW >>1L +2 @@>HQ@
 
6WDWHPHQWLVFRUUHFW6LQFH.LVODUJHULQ .VWDE >>1L HQ @@>1+@
PDJQLWXGHWKDQ.LWLPSOLHVWKDWHQKDVDKLJKHU >>1L 1+ @@>HQ@

WHQGHQF\WRUHSODFHWKHZDWHUOLJDQGVWKDQ
DPPRQLD BB>>1L HQ @@BB[>>1L +2 @@>1+@
 >>1L +2 @@>HQ@>>1L 1+ @@
6WDWHPHQWLVFRUUHFW 
 .VWDE.VWDE
 
%
6WDWHPHQWVDQGDUHFRUUHFW
5HDFWDQWLQVWDJH,,,LVVDWXUDWHGLWFDQQRWXQGHUJRDGGLWLRQUHDFWLRQ6WDJH,,,LVHOLPLQDWLRQ

' &
2SWLRQLVFRUUHFW3URWRQDWLRQRI5JURXSVRIWKH 2SWLRQ  LVLQFRUUHFW %RWK FRPSRXQGV UHDFW ZLWK
UHVLGXHVLQWKHWHUWLDU\VWUXFWXUHGLVUXSWVH[LVWLQJ DTXHRXVEURPLQHWRJLYHDZKLWHSSW
LRQLFLQWHUDFWLRQVDQGK\GURJHQERQGVEHWZHHQ
2SWLRQ  LV FRUUHFW 7KH DON\O VLGH FKDLQ RQ WKH
WKHPZKLFKUHVXOWVLQGHQDWXUDWLRQ
EHQ]HQH ULQJ ZLOO XQGHUJR R[LGDWLRQ DV ORQJ DV
2SWLRQLVLQFRUUHFW7KHDPLGHIXQFWLRQDOJURXS
WKHUHLVD+DWRPSUHVHQWDWWKHEHQ]\OLFSRVLWLRQ
LQWKHSHSWLGHOLQNDJHVDUHQHXWUDODQGZLOOQRW
3XUSOHDFLGLILHG.0Q2ZLOOEHGHFRORXULVHG
UHDFWZLWK+
2SWLRQLVLQFRUUHFW3ULPDU\VWUXFWXUHLVRQO\ 2SWLRQLVFRUUHFW7KH1+RIWKHSKHQ\ODPLQH
DIIHFWHGGXULQJK\GURO\VLV&RPSOHWHK\GURO\VLV ZLOOXQGHUJRQXFOHRSKLOLFVXEVWLWXWLRQZLWK&+%U
ZLOORQO\RFFXUZKHQRYDOEXPLQLVKHDWHGIRUD 7KH%UOHDYLQJJURXSZLOOIRUPDFUHDPSSWZLWK
SURORQJHGSHULRGRIWLPHXQGHUKLJKO\DFLGLF $J12

FRQGLWLRQVHJPROGP +&l DT KHDWIRUD
IHZKRXUV






trendyline
1466 trendyline
1

PRELIMINARY EXAMINATIONS
s HIGHER 2

CANDIDATE
NAME

CIVICS
GROUP /

CENTER INDEX
S
NUMBER NUMBER

CHEMISTRY 9647/02
Paper 2 Structured Questions 29th August 2016
2 hours
Candidates answer on the Question Paper.
Additional Materials: Data Booklet

READ THESE INSTRUCTIONS FIRST


Write your Civics Group, centre number, index number and name on all the work you
hand in.
Write in dark blue or black pen.
You may use anHB pencil for any diagrams or graphs.
Do not use staples, paper clips, glue or correction fluid.

Answer all questions. For Examiners Use


The use of an approved scientific calculator is expected, 1 / 12
where appropriate.
A Data Booklet is provided. 2 / 15
3 / 12
At the end of the examination, fasten all your work securely
together. 4 /9
The number of marks is given in brackets [ ] at the end of 5 /24
each question or part question.
Paper 1 / 40
Paper 3 / 80
Total

trendyline s document consists of 21


This
Answer all
A ll the
2 printed pages.
th questions
ti

2016 TJC H2 Chemistry Preliminary Exam [Turn over


1467 trendyline
2

1 Planning (P)

Potentiometric titration is a technique similar to direct titration of a redox reaction and can
be used to determine the concentration of a reducing ion such as Fe2+ without the use of an
indicator. It involves measurement of E cell at intervals as anoxidising titrantis added. The
titration is carried out on one side of a cell whose other half is a reference electrode which
can be a standard hydrogen electrode or a calomel electrode.

Consider the titration of Fe2+ with standard Ce4+, the course of which is monitored
potentiometrically as shown in Figure 1 below. The reference electrode used is the calomel
electrode shown in Figure 2.

Figure 1: Set-up for potentiometric


Figure 2: A calomel titration
electrode

Titration reaction: Fe2+ + Ce4+ Fe3+ +

To follow the course of the reaction, a pair of electrodes is inserted into the reaction mixture.
At the calomel reference electrode,
Reference half reaction is:Hg 2 Cl 2 (s) + 2e 2Hg(l) + 2Cl- E = + 0.24 V

At the suitable electrode, there are two reactions that will come to equilibrium:

Fe3+ + e Fe2+ E = + 0.77 V


Ce4+ + e Ce3+ E = + 1.70 V

As the Ce4+ titrant is added, Fe2+ is oxidised to Fe3+and the cell potential is controlled by the
ratio of reduced and oxidised iron according to the Nerst equation which causes the
potential to rise as more iron becomes oxidised.
[Fe2 ]
E(Fe3+/Fe2+) 0.24.(1)

trendyline
y
E cell = - 0.05916log
[Fee3 ]

valence point is reached, the Fe2+ will have been totally co


When the equivalence consumed, and the
en be controlled by the concentration ratio of Ce33+/ C
potential will then Ce4+. Thus beyond the
C 4+/Ce3+ cell with excess Ce4+.
end-point the potential rises rapidly to that of a Ce

2016 TJC H2 Chemistry Preliminary Exam [Turn over


1468 trendyline
3

[Ce3 ]
E cell = E(Ce4+/Ce3+) - 0.05916 log 0.24..(2)
[Ce 4 ]
A graph of E cell against volume of titrant added can be drawn and the resulting titration curve
looks much like the familiar acid-base titration curve. The end point is found not by
measuring a particular cell voltage, but by finding what volume of titrant gives the steepest
part of the curve.

Solution FA 1 was prepared by dissolving 0.960 g of iron tablets in 100 cm3 of 1 mol dm-3
HClO 4 acid. By titration method using KMnO 4 , the percentage of iron(II) ions in the iron
tablets used to make FA 1 was found to be 12.5%. A student wishes to verify the
percentage of iron(II) ions present in FA 1 by potentiometric titration using standard 0.100
mol dm-3 Ce4+ solution.
(a) Suggest a suitable electrode for the potentiometric titration between Fe2+ and Ce4+
solution.

Suitable indicator electrode:

[1]

(b) Perform a pre-calculation to estimate the end-point volume for the potentiometric
titration of 100 cm3 of FA 1 with 0.100 mol dm-3Ce4+ solution.

[1]

trendyline
trendylin
dyline
(c) Using allll the
t above information including
uding the estimated
estim end-point
end-point volume, you are
required to writea plan to verify e percentage of Fe2+ present inFA
verifythe inFA1by potentiometric
titration.

You may assume that you are provided with the following:
followin

2016 TJC H2 Chemistry Preliminary Exam [Turn over


1469 trendyline
4

100 cm3 of FA 1 solution prepared by dissolving 0.960 g of iron tablets in 1 mol dm-3
HClO 4 acid.
1.25 mol dm-3 Ce4+ solution
a calomel reference electrode
a suitable electrode
a voltmeter
the apparatus normally found in a school laboratory.

Your plan should include details of

the preparation of 0.100 mol dm-3 Ce4+solution


how the electrochemical cell set-up in Figure 1 is assembled
how you would recognize that the end-point has been reached
a table for the recording of readings needed for the plotting of titration curve,
showing suitable volumes of titrant added
the calculation to verifythe percentage of Fe2+ in FA 1 in terms of theend-point
volume, V o cm3 of Ce4+

trendyline
2016 TJC H2 Chemistry Preliminary Exam [Turn over
1470 trendyline
5

[6]

trendyline
tre
(d) Use the Nerst equation (1)) and (2) to calculate theE ce
cell values when

(i) e3+] = [Fe2+]


[Fe

2016 TJC H2 Chemistry Preliminary Exam [Turn over


1471 trendyline
6

E cell :

(ii) [Ce3+] = [Ce4+]

E cell :

[1]

(e) If the end-point volume is V o cm3 of Ce4+, sketch the graph you would expect to obtain.
Show clearly in your sketch, the titration volumes in terms of V o when you will be
obtaining the E cell values calculated in (d)(i) and (d)(ii).

[3]

[Total:12]

trendyline
Gases A and B react as shown in the equation given.
given

2016 TJC H2 Chemistry Preliminary Exam [Turn over


1472 trendyline
7

GasesA, B and D were injected into a vessel at 25oC and allowed to reach dynamic
equilibrium. The graph below (not drawn to scale) shows how the concentration of the four
gases varies with time.
concentration / moldm-3

3.0

A and B
0.25

0.1 C
0
t0 t1 t2 t3 t4 time

(a) (i) Explain what is meant by dynamic equilibrium.

[1]

(ii) Write an expression for K c for this equilibrium, stating the units.

[1]

(iii)

trendyline
Calculate
alculate the value for K c at time,
time t 1 .

2016 TJC H2 Chemistry Preliminary Exam [Turn over


1473 trendyline
8

[2]

(b) (i) State Le Chateliers principle.

[1]

At time, t 2 , the volume of the system was reduced which lead to an increase in
concentration of the gases. This causes the position of the equilibrium to shift.

(ii) Suggest and explain what happens to the system between time t 3 to t 4 .

[2]

(iii) Sketch on the same axes how the concentration of the four gases will change when
more gas C was introduced at time, t 4 .

trendyline
[2]

(iv) State
tate explain the effect of the presence of a catalyst on the equilibrium
ate and explain
position
sition and the value of the equilibrium constant.
osition constant

2016 TJC H2 Chemistry Preliminary Exam [Turn over


1474 trendyline
9

[2]

(c) The standard Gibbs free energy change of reaction, Go in J mol-1, is related to the
equilibrium constant by the following equation.
Go = RT ln (0.02478K c )

(i) Using your answer in (a)(iii), calculate Goin kJ mol-1.

[1]

(ii) Explain the significance of the sign of your answer in (c)(i).

[1]

(iii) Using your answer in (c)(i) and any other information on page 7, calculate So.

[2]

3 (a) trendyline
1- chloropropane
opropane can be synthesized by the following reaction at roo
[Total: 15]

room temperature.

2016 TJC H2 Chemistry Preliminary Exam [Turn over


1475 trendyline
10

uv light
CH 3 CH 2 CH 3 (g) + Cl 2 (g) CH 3 CH 2 CH 2 Cl(g) + HCl(g) -------- Reaction I

With reference to the Data BookletFDOFXODWHH r of Reaction I.

[1]

(b) An alternate method of synthesis of 1-chloropropane is as follows:


CH 3 CH 2 CH 2 OH(l) + PCl 5 (s) CH 3 CH 2 CH 2 Cl(l) + POCl 3 (l) + HCl(g) ----Reaction II

(i) Suggest one reason why Reaction II is a better method to produce 1-


chloropropane than Reaction I.

[1]

(ii) Suggest whether the standard change in entropy of Reaction II is likely to be


positive or negative, explaining your reasoning.

trendyline
[2]

2016 TJC H2 Chemistry Preliminary Exam [Turn over


1476 trendyline
11

(c) 1-chloropropane is an important reactant in the synthesis of propylbenzene:


C 6 H 6 (l) + CH 3 CH 2 CH 2 Cl (l) C 6 H 5 CH 2 CH 2 CH 3 (l) + HCl(g) ----- Reaction III

(i) Define standard enthalpy change of formation.

[1]

(ii) Using the data listed below,


H f o/kJ mol-1
CH 3 CH 2 CH 2 Cl(l)161.3
HCl(l) 167.2
C 6 H 6 ( l) +49.0
C 6 H 5 CH 2 CH 2 CH 3 (l)38.4

enthalpy change of vapourisation of HCl is +74.9 kJ mol-1

FRQVWUXFWDQHQHUJ\F\FOHWRFDOFXODWHH r of Reaction III.

trendyline [2]

2016 TJC H2 Chemistry Preliminary Exam [Turn over


1477 trendyline
12

(d) Propylbenzene is a fuel additive and solvent.It undergoes the following chemical
reactions.

ICl,
AlCl3

PCl5

(i) State the reagents and conditions for the following steps:

Step 1 :

Step 2 :

[2]

(ii) Draw the structural formula of the organic compounds X, Y and Z.

X: Y:

trendyline
trendyline
2016 TJC H2 Chemistry Preliminary Exam [Turn over
1478 trendyline
13

Z:

[3]

[Total:12]

trendyline
2016 TJC H2 Chemistry Preliminary Exam [Turn over
1479 trendyline
14

4 When propanone vapour is passed over a heated filament, it breaks down into methane and
a reactive substance called ketene.
O
||
1
CH 3 C CH 3 CH 4 + CH 2 = 2C = O
ketene

(a) (i) Draw the hybrid orbitals around each of the 2 carbon atoms, 1C and 2C in ketene
molecule. State the type of hybridisation for each carbon atom.

Carbon atom 1C Carbon atom 2C

Hybridisation: .... Hybridisation: .

[2]

(ii) Draw the displayed formula for the ketene molecule and on it show suggested
values of the bond angles around 1C and 2C atoms.

[1]

trendyline
2016 TJC H2 Chemistry Preliminary Exam [Turn over
1480 trendyline
15

Ketene is a highly reactive compound. It readily undergoes the reactions shown below.
CH 2 = C = O + HCl CH 3 C = O
|
Cl

CH 2 = C = O + H 2 O CH 3 C = O
|
OH

(b) Name and describe the mechanism for the reaction between ketene and water,
showing all charges and using curly arrows to show the movement of electron pairs.

[3]

trendyline
2016 TJC H2 Chemistry Preliminary Exam [Turn over
1481 trendyline
16

(c) Ketene reacts with ethanol to form a sweet smelling compoundP, C 4 H 8 O 2 , which
reacts with LiAlH 4 to give Q.Identify P and Q. Name the type of reaction for the
transformation of P to Q.

P Q

Type of reaction from P to Q:


[3]

[Total: 9]

trendyline
2016 TJC H2 Chemistry Preliminary Exam [Turn over
1482 trendyline
17

5 Ethanedioic acid, H 2 C 2 O 4 , reacts with sodium hydroxide to form sodium ethanedioate,


Na 2 C 2 O 4 . One of the main applications of ethanedioate ions is in rust-removal, as
ethanedioate ions form water-soluble derivatives with iron(III) ions.

(a) An electrochemical cell is set up using Cu2+/Cu and CO 2 /C 2 O 4 2- half-cells. The


electrode in CO 2 /C 2 O 4 2- half-cell is the negative electrode and the cell potential is
+0.77V.

(i) Using relevant data from the Data Booklet, determine the electrode potential of
the CO 2 /C 2 O 4 2- half-cell.

[2]

(ii) A small quantity of solid potassium iodide is added to the Cu2+/Cu half-cell. Write
an equation and state the observations for the reaction that will occur in this half-
cell. Hence, predict and explain the effect on the e.m.f. of the electrochemical
cell.

[3]

trendyline
2016 TJC H2 Chemistry Preliminary Exam [Turn over
1483 trendyline
18

(b) The K a values for the step-wise dissociation of ethanedioic acid are given below.

HO 2 C-CO 2 H HO 2 C-CO 2 - + H+ K a1 = 5.6 x 10-2 mol dm-3


HO 2 C-CO 2 --O 2 C-CO 2 - + H+ K a2 = 5.4 x 10-5 mol dm-3

Calculate the pH of 0.10 mol dm-3 of sodium ethanedioate salt, Na 2 C 2 O 4 .

[2]

(c) Hydrated salt of magnesium ethanedioate has the formula MgC 2 O 4 .nH 2 O. When
6.18 g of hydrated magnesium ethanedioate is heated strongly, a white solid W
and three gases X, Y and Z are generated. Gases X, Y and Z are passed through
anhydrous calcium chloride, followed by aqueous sodium hydroxide. The flasks
containing anhydrous calcium chloride and aqueous sodium hydroxide had an
increase in mass of 1.50 g and 1.83 g respectively. The residual gas Z occupied 1
dm3 at room temperature and pressure. Gas Z is poisonous and can be heated
with oxygen to give gas Y.

(i) Suggest the identities of W, X, Y and Z.

W:

X:

Y:

Z:

trendyline
[2]

2016 TJC H2 Chemistry Preliminary Exam [Turn over


1484 trendyline
19

(ii) Hence, determine the value of n, showing your working clearly.

[3]

(iii) Ethanedioate ions form insoluble precipitates with many metal ions, one of
which is calcium ethanedioate, a primary constituent of kidney stones.

Calcium ethanedioate decomposes in the same manner as magnesium


ethanedioate. Using relevant data from the Data Booklet, deduce whether
calcium ethanedioate would decompose at a higher or lower temperature
than magnesium ethanedioate. Explain your reasoning.

trendyline [3]

2016 TJC H2 Chemistry Preliminary Exam [Turn over


1485 trendyline
20

(d) A tetradentate ligand, ethylenediamine-N,N-diacetate (EDDA) ion, can be


synthesised from ethanedioic acid through a series of steps. Ethanedioic acid is
first converted to ethane-1,2-diamine through a 3-step synthesis. Steps 4 to 7
represent the mechanism of Skrecker synthesis, which can be used to synthesise
amino acids.

E: F:
step 1 step 2
HO 2 CCO 2 H
ethanedioic acid

step 3

H 2 NCH 2 CH 2 NH 2
ethane-1,2-diamine

step 4

OH H H OH
H C N CH2CH2 N C H
H H

G: step 5

step 6 H C N CH2CH2 N C H
HCN
H H

step 7
OH-(aq), heat

CO2- H H CO2-

trendyline
e
H C N CH2CH2 N C H
H H
EDDA

2016 TJC H2 Chemistry Preliminary Exam [Turn over


1486 trendyline
21

(i) Propose the 3-step synthesis for the formation of ethane-1,2-diamine from
ethanedioic acid, naming the reagents and conditions used in each step.
Draw the structures of the intermediate compoundsE and F in the spaces
provided in the synthetic route.

Step 1:

Step 2:

Step 3:
[5]

(ii) Carbonyl compound and amine react in the following way.

O OH H

C
C + R"NH2 R N R"
R R'
R'

Step 4 of the synthetic route in (d) involves the reaction of ethane-1,2-


diamine with 2 moles of a carbonyl compound. Identify the carbonyl
compound.

[1]

(iii) Draw the structure of compound G in the space provided. [1]

(iv) State the types of reaction in step 5 and step 7.

Step 5:

ep 7:
Step
[2]
[Total: 24]

2016 TJC H2 Chemistry Preliminary Exam [Turn over


1487 trendyline
1

PRELIMINARY EXAMINATIONS
s HIGHER 2

CANDIDATE
NAME

CIVICS
GROUP /

CENTER INDEX
S
NUMBER NUMBER

CHEMISTRY 9647/02
Paper 2 Structured Questions 29th August 2016
2 hours
Candidates answer on the Question Paper.
Additional Materials: Data Booklet

READ THESE INSTRUCTIONS FIRST


Write your Civics Group, centre number, index number and name on all the work you
hand in.
Write in dark blue or black pen.
You may use anHB pencil for any diagrams or graphs.
Do not use staples, paper clips, glue or correction fluid.

Answer all questions. For Examiners Use


The use of an approved scientific calculator is expected, 1 / 12
where appropriate.
A Data Booklet is provided. 2 / 15
3 / 12
At the end of the examination, fasten all your work securely
together. 4 /9
The number of marks is given in brackets [ ] at the end of 5 /24
each question or part question.
Paper 1 / 40
Paper 3 / 80
Total

trendyline rinted pages.


This document consists of 21 printed

2016 TJC H2 Chemistry Preliminary Exam [Turn over


1488 trendyline
2

Answer all the questions

1 Planning (P)

Potentiometric titration is a technique similar to direct titration of a redox reaction and can
be used to determine the concentration of a reducing ion such as Fe2+ without the use of an
indicator. It involves measurement of E cell at intervals as anoxidising titrantis added. The
titration is carried out on one side of a cell whose other half is a reference electrode which
can be a standard hydrogen electrode or a calomel electrode.

Consider the titration of Fe2+ with standard Ce4+, the course of which is monitored
potentiometrically as shown in Figure 1 below. The reference electrode used is the calomel
electrode shown in Figure 2.

Figure 1: Set-up for potentiometric


Figure 2: A calomel titration
electrode

Titration reaction: Fe2+ + Ce4+ Fe3+ +

To follow the course of the reaction, a pair of electrodes is inserted into the reaction mixture.
At the calomel reference electrode,
Reference half reaction is:Hg 2 Cl 2 (s) + 2e 2Hg(l) + 2Cl- E = + 0.24 V

At the suitable electrode, there are two reactions that will come to equilibrium:

Fe3+ + e Fe2+ E = + 0.77 V


Ce4+ + e Ce3+ E = + 1.70 V

As the Ce4+ titrant is added, Fe2+ is oxidised to Fe3+and the cell potential is controlled by the
ratio of reduced and oxidised iron according to the Nerst equation which causes the

t dyli
trendyline
potential to rise
e as more iron becomes oxidised.
oxidise
[Fe2 ]
E cell = E(Fee3+/Fe
Fe2+) - 0.0591
0.05916log
log 0.24.(1)
[Fe3 ]

e2+ will have been totally consumed, and the


When the equivalence point is reached, the Fe

2016 TJC H2 Chemistry Preliminary Exam [Turn over


1489 trendyline
3

potential will then be controlled by the concentration ratio of Ce3+/ Ce4+. Thus beyond the
end-point the potential rises rapidly to that of a Ce4+/Ce3+ cell with excess Ce4+.
[Ce3 ]
E cell = E(Ce4+/Ce3+) - 0.05916 log 0.24..(2)
[Ce 4 ]
A graph of E cell against volume of titrant added can be drawn and the resulting titration curve
looks much like the familiar acid-base titration curve. The end point is found not by
measuring a particular cell voltage, but by finding what volume of titrant gives the steepest
part of the curve.

Solution FA 1 was prepared by dissolving 0.960 g of iron tablets in 100 cm3 of 1 mol dm-3
HClO 4 acid. By titration method using KMnO 4 , the percentage of iron(II) ions in the iron
tablets used to make FA 1 was found to be 12.5%. A student wishes to verify the
percentage of iron(II) ions present in FA 1 by potentiometric titration using standard 0.100
mol dm-3 Ce4+ solution.
(a) Suggest a suitable electrode for the potentiometric titration between Fe2+ and Ce4+
solution.

Suitable indicator electrode: Pt [1]

(b) Perform a pre-calculation to estimate the end-point volume for the potentiometric titration
of 100 cm3 of FA 1 with 0.100 mol dm-3Ce4+ solution.

No of moles of Fe2+ in 100 cm3 of FA 1 = (0.125 x 0.960) 55.8 = 0.00215 mol

Vol of Ce4+required to reach the end-point = (0.00215 0.100) dm3 = 21.50 cm3[1]

(c) Using all the above information including the estimated end-point volume, you are
required to writea plan to verifythe percentage of Fe2+ present inFA1by potentiometric
titration.

You may assume that you are provided with the following:

100 cm3 of FA 1 solution prepared by dissolving 0.960 g of iron tablets in 1 mol dm-3
HClO 4 acid.
1.25 mol dm-3 Ce4+ solution
a calomel reference electrode
a suitable electrode
a voltmeter
the apparatus normally found in a school laboratory.

t dy
trendyl
trendyline
Your plan
n sshould include details off

eparation of 0.100 mol dm-3 Ce


the preparation e44+solution
how thee electrochemical
ectrochemical cell set-up in Figure 1 is assembled
how you would ld recognize
i that
th t the end-point
d point ha
has b
been reached
a table for the recording of readings needed for the plotting of titration curve,

2016 TJC H2 Chemistry Preliminary Exam [Turn over


1490 trendyline
4

showing suitable volumes of titrant added


the calculation to verifythe percentage of Fe2+ in FA 1 in terms of theend-point
volume, V o cm3 of Ce4+

Preparation of the standard 0.100 mol dm-3 Ce4+solution


1. Using a burette, transfer 20.00 cm3of 1.25 mol dm-3 Ce4+ solution directly into
a 250 cm3 standard flask.
2. Make up to the mark with distilled water. Stopper and shake well to obtain a
homogeneous solution.
1mcorrect volume for dilution using burette/pipette for measurement.
1m- using a standard & logical procedure in the preparation
Procedure to record cell potential in the potentiometric titration
1. Transfer the given 100.0 cm3 of FA 1 solution into a 500 ml beaker.
2. Fill a burette with the prepared 0.100 mol dm-3Ce4+ solution.
3. Assemble the apparatus as shown in Figure 1.
4. Titrate FA 1 solution with Ce4+solution . Read and record the cell voltage
registered on the voltmeter as each aliquot of Ce4+solution is introduced
from the burette.Carry out a more careful titration by adding titrant in small
increments as the volume gets close to the equivalence point.
5. The end-point volume is reached when the E cell value switches from cell
potential based on Fe3+/Fe2+ to one based on Ce4+/Ce3+.
Table of readings

Volume of Ce4+ solution added / cm3 E cell / V


5.00
10.00
15.00
17.00
19.00
21.00
23.00
25.00
30.00
35.00
40.00
45.00

1m Procedure on the assembly of set-up as in figure 1 and titrating carefully


by adding titrant in small increments nearing the end-point
1m -how to recognize that the end-point had been passed
1m table of readingswith volume exceeding 21.50 cm3 of Ce4+solution

Calculationof % of Fe2+ in FA 1 in terms of end-point volume, V o cm3of Ce4+soluton

No of moles of Ce4+ used = 0.100V o 1000 mol = 10-4 V o

trendyline
y
ol of Fe2++ present
= no of moles re in 100 cm3 of FA 1
Mass of Fe22+ present in 0.960 g of iron tablets = 1
10-44 V o x 55.8 = 5.58 x 10-3V o g
Hence % of Fe2+ in FA 1 = (5.58 x 10-3 V o 0.960) x 100 = 0.581V o % [6]

(d) Use the Nerst equation (1) and (2) to calculate


l theE cell values when

2016 TJC H2 Chemistry Preliminary Exam [Turn over


1491 trendyline
5

(i) [Fe3+] = [Fe2+] and


(ii) [Ce3+] = [Ce4+].

(i) E cell = +0.53 V (ii) E cell = +1.46V [1]

(e) If the end-point volume is V o cm3 of Ce4+, sketch the graph you would expect to obtain.
Show clearly in your sketch, the titration volumes in terms of V o when you will be
obtaining the E cell values calculated in (d)(i) and (d)(ii).

Shape of titration with axes clearly labelled.


E cell at X = +0.53 V at V o when [Fe2+] = [Fe3+]
E cell at Y = +1.46V at 2V o when [Ce3+] = [Ce4+][3]

[Total:12]

trendyline
2016 TJC H2 Chemistry Preliminary Exam [Turn over
1492 trendyline
6

2 Gases A and B react as shown in the equation given.

GasesA, B and D were injected into a vessel at 25oC and allowed to reach dynamic
equilibrium. The graph below (not drawn to scale) shows how the concentration of the four
gases varies with time.

concentration / moldm-3

3.0

A and B
0.25

0.1 C

t0 t1 t2 t3 t4 time

(a) (i) Explain what is meant by dynamic equilibrium.

When a reversible reaction reaches a state of dynamic equilibrium, reaction


continues to occur but the rates of the forward and backward
reactions areequal such that there is no net change in the concentrations of the
reactants and products.
[1]

(ii) Write an expression for K c for this equilibrium, stating the units.

K c = [C][D]2/[A][B] mol dm-3

[1]

trendyline
2016 TJC H2 Chemistry Preliminary Exam [Turn over
1493 trendyline
7

(iii) Calculate the value for K c at time, t 1 .

Initial conc. / mol dm-3 0.25 0.25 0 3.0

Change in conc. / mol dm-3 0.1 0.1 + 0.1 + 0.2

Equil. conc. / mol dm-3 0.15 0.15 0.13.2

K c = 0.1 x 3.22 / 0.15 x 0.15 = 45.5mol dm-3

[2]

(b) (i) State Le Chateliers principle.

If a system at dynamic equilibrium is subjected to a change which disturbs the


equilibrium, the system will respond in such a way so as to counteract the effect of
the change so as to re-establish the equilibrium.

[1]

At time, t 2 , the volume of the system was reduced which lead to an increase in
concentration of the gases. This causes the position of the equilibrium to shift.

(ii) Suggest and explain what happens to the system between time t 3 to t 4 .

Position of equilibrium shifts to the left as concentration of A and B increases


gradually and concentration of C and D decreases gradually.

Since the backwardreaction is exothermic, hence the temperature of the system


must have beendecreasedat time, t 3 .

[2]

(iii) Sketch on the same axes how the concentration of the four gases will change when
more gas C was introduced at time, t 4 .

Sharp increase in [C] followed by decrease in [C], decrease in [D] and increase
in [A]&[B]
Decrease in [D] is twice the increase in [A], [B] and [C]. Equilibrium [C] should
not be lower than the initial [C] at t 4 .

[2]

(iv)

trendyline
osition
y
tate and explain the effect of the presence of a catalyst on the
State
sition and the value of the equilibrium constant.
position

equ
constant

A catalyst have no effect on the equilibrium


th equilibrium

position and the vvalue of K c.

2016 TJC H2 Chemistry Preliminary Exam [Turn over


1494 trendyline
8

The presence of catalystincreases the rate of the forward and backward reaction
to the same extent. (K C = k f / k b K c constant.)

[2]

(c) The standard Gibbs free energy change of reaction, Go in J mol-1, is related to the
equilibrium constant by the following equation.
Go = RT ln (0.02478K c )

(i) Using your answer in (a)(iii), calculate Goin kJ mol-1.

8.31 x 298 x ln (0.02478 x 45.5)


Go = = 0.297kJ mol-1
1000

[1]

(ii) Explain the significance of the sign of your answer in (c)(i).

Since Go< 0, the reaction is spontaneous at room temperature.

[1]

(iii) Using your answer in (c)(i) and any other information on page 7, calculate So.

Go = Ho TSo

So = [90 (0.297)] / 298 = +303 J mol-1K-1

Note: sign must be shown

[2]
[Total: 15]

3 (a) 1- chloropropane can be synthesized by the following reaction at room temperature.

uv light
CH 3 CH 2 CH 3 (g) + Cl 2 (g) CH 3 CH 2 CH 2 Cl(g) + HCl(g) -------- Reaction I

With reference to the Data BookletFDOFXODWHH r of Reaction I.

Bonds broken: 1 C-H bond, 1 Cl-Cl bond


Bonds formed: 1 C-Cl bond, 1 H-Cl bond

trendyline


+
+r
+ = +(410+244) (340+431)
43
J mol-11
= 117 kJ
[1]

2016 TJC H2 Chemistry Preliminary Exam [Turn over


1495 trendyline
9

(b) An alternate method of synthesis of 1-chloropropane is as follows:


CH 3 CH 2 CH 2 OH(l) + PCl 5 (s) CH 3 CH 2 CH 2 Cl(l) + POCl 3 (l) + HCl(g) ----Reaction
II

(i) Suggest one reason why Reaction II is a better method to produce


1-chloropropane than Reaction I.

Reaction I can undergo further free-radical substitution to give a mixture of


halogenoalkanes thus the yield is much lower compared to Reaction II.
[1]
(ii) Suggest whether the standard change in entropy of Reaction II is likely to be
positive or negative, explaining your reasoning.
So is likely to be positive.

This is because in the reaction, gaseous molecules are formed& highly
disordered.There are more ways to arrange the molecules and distribute
the energy.
[2]
(c) 1-chloropropane is an important reactant in the synthesis of propylbenzene:
C 6 H 6 (l) + CH 3 CH 2 CH 2 Cl (l) C 6 H 5 CH 2 CH 2 CH 3 (l) + HCl(g) ----- Reaction III

(i) Define standard enthalpy change of formation.

The standard enthalpy change of formation of a substance, H f o, is the


enthalpy change when one mole of the substance is formed from its constituent
elementsin their standard states at 298 K and 1 atm.

[1]

(ii) Using the data listed below,


H f o/kJ mol-1
CH 3 CH 2 CH 2 Cl(l)161.3
HCl(l) 167.2
C 6 H 6 ( l) +49.0
C 6 H 5 CH 2 CH 2 CH 3 (l)38.4

enthalpy change of vapourisation of HCl is +74.9 kJ mol-1

FRQVWUXFWDQHQHUJ\F\FOHWRFDOFXODWHH r of Reaction III.

trendyline
2016 TJC H2 Chemistry Preliminary Exam [Turn over
1496 trendyline
10

C6H6(l) + CH3CH2CH2Cl (l) C6H5CH2CH2CH3(l) + HCl(g)


Hor

+49.0 + (-161.3)

- 38.4 + (- 167.2) + 74.9

9C(s) + 13/2H2(g) + Cl2(g) C6H5CH2CH2CH3(l) + HCl(l)

FRUUHFWHQHUJ\F\FOH state symbols and coefficients)


By Hess Law, +49.0 + (-161.3) + Ho r = -38.4 + (- 167.2) + 74.9
 Ho r = - 18.4 kJ mol-1

[2]

(d) Propylbenzene is a fuel additive and solvent.It undergoes the following chemical
reactions.

ICl,
AlCl3

PCl5

(i) State the reagents and conditions for the following steps:

Step 1 : Concentrated H 2 SO 4 , concentrated HNO 3 , UHIOX[0C

Step 2 : Heat under reflux with Sn, concentrated HCl, followed by


NaOH (aq)

trendyline
[2]

(ii) Draw
raw
aw the structural formula of the organic
organi compounds X, Y and Z.

2016 TJC H2 Chemistry Preliminary Exam [Turn over


1497 trendyline
11

X: or Y:

Z:

[3]

[Total:12]

4 When propanone vapour is passed over a heated filament, it breaks down into methane and
a reactive substance called ketene.
O
||
1
CH 3 C CH 3 CH 4 + CH 2 = 2C = O
ketene

(a) (i) Draw the hybrid orbitals around each of the 2 carbon atoms, 1C and 2C in ketene
molecule. State the type of hybridization for each carbon atom.

Carbon atom 1C Carbon atom 2C

Hybridisation: sp2 Hybridisation: sp

trendyline
[2]

2016 TJC H2 Chemistry Preliminary Exam [Turn over


1498 trendyline
12

(ii) Draw the displayed formula for the ketene molecule and on it show suggested
values of the bond angles around 1C and 2C atoms.

[1]

Ketene is a highly reactive compound. It readily undergoes the reactions shown below.
CH 2 = C = O + HCl CH 3 C = O
|
Cl

CH 2 = C = O + H 2 O CH 3 C = O
|
OH

(b) Name and describe the mechanism for the reaction between ketene and water,
showing all charges and using curly arrows to show the movement of electron pairs.

Name of mechanism: Electrophilic Addition

Step 1:

Step 2:

[3]

(c) Ketene reacts with ethanol to form a sweet smelling compoundP, C 4 H 8 O 2 , which

trendyline
t LiAlH 4 to give Q.Identify
reacts with y P and Q. Na
Name the type o
of reaction for the
transformation
mation of P to Q
Q..

2016 TJC H2 Chemistry Preliminary Exam [Turn over


1499 trendyline
13

P Q

Type of reaction from P to Q: Reduction


[3]

[Total: 9]

5 Ethanedioic acid, H 2 C 2 O 4 , reacts with sodium hydroxide to form sodium ethanedioate,


Na 2 C 2 O 4 . One of the main application of ethanedioate ions is in rust-removal, as
ethanedioate forms water-soluble derivatives with iron(III) ions.
(a) An electrochemical cell is set up using Cu2+/Cu and CO 2 /C 2 O 4 2- half-cells. The
electrode in CO 2 /C 2 O 4 2- half-cell is the negative electrode and the cell potential is
+0.77V.
(i) Using relevant data from the Data Booklet, determine the electrode potential of
the CO 2 /C 2 O 4 2- half-cell.
E Cu2+/Cu = +0.34V = E red
E cell = E red - E ox
+0.77 = +0.34 - E CO2/C2O42-

E CO2/C2O42- = -0.43V
[2]
2+
(ii) A small quantity of solid potassium iodide is added to the Cu /Cu half-cell. Write
an equation and state the observations for the reaction that will occur in this half-
cell. Hence, predict and explain the effect on the e.m.f. of the electrochemical
cell.
2Cu2+(aq) + 4I- DT &XI(s) + I 2 (aq)
White ppt in brown solution/ brown ppt observed in the Cu2+/Cu half-cell.
Since CuI is formed, the [Cu2+(aq)] will decrease. E Cu2+/Cu (E red ) will become
more negative and the e.m.f. of the electrochemical cell will be less positive/
decrease.
[3]

trendyline
e
(b) The K a va
values for the step
step-wise
-wise dissociation
ciation of ethanedioic
ethan acid are given
g below.

O 2 C-
HO C-CO
CO 2 H CO 2 - + H+
HO 2 C-CO 1 -2 mol dm
K a1 = 5.6 x 10 d -3

2016 TJC H2 Chemistry Preliminary Exam [Turn over


1500 trendyline
14

HO 2 C-CO 2 --O 2 C-CO 2 - + H+ K a2 = 5.4 x 10-5 mol dm-3

Calculate the pH of 0.10 mol dm-3 of sodium ethanedioate salt, Na 2 C 2 O 4 .


-
O 2 C-CO 2 - + H 2 O HO 2 C-CO 2 - + OH-
[OH-@ . b F [-14/(5.4x10-5) = 4.30 x 10-6 mol
pH = 14 lg(4.30 x 10-6) = 8.63
[2]

(c) Hydrated salt of magnesium ethanedioate has the formula MgC 2 O 4 .nH 2 O. When 6.18
g of hydrated magnesium ethanedioate is heated strongly, a white solid W and three
gases X, Y and Z are generated. Gases X, Y and Z are passed through anhydrous
calcium chloride, followed by aqueous sodium hydroxide. The flasks containing
anhydrous calcium chloride and aqueous sodium hydroxide had an increase in mass
of 1.50 g and 1.83 g respectively. The residual gas Z occupied 1 dm3 at room
temperature and pressure. Gas Z is poisonous and can be heated with oxygen to give
gas Y.

(i) Suggest the identities of W, X, Y and Z.


W: MgO, X: H 2 O
Y: CO 2 , Z: CO
Any 2 correct = 1 mark
[2]
(ii) Hence, determine the value of n, showing your working clearly.
No. of moles of CO = 1/24 = 0.0417 mol
Mass of CO = 0.0417 x (12.0 + 16.0) = 1.17 g
Mass of MgO = 6.18 - 1.50 - 1.83 - 1.17 = 1.68 g
No. of moles of MgO = 1.68/ (24.3 + 16.0) = 0.0417 mol
No. of moles of H 2 O = 1.50/ 18.0 = 0.0833 mol
n = 0.0833/ 0.0417 = 2
[3]

(iii) Ethanedioate ions form insoluble precipitates with many metal ions, one of which
is calcium ethanedioate, a primary constituent of kidney stones.

Calcium ethanedioate decomposes in the same manner as magnesium


ethanedioate. Using relevant data from the Data Booklet, deduce whether
calcium ethanedioate would decompose at a higher or lower temperature than
magnesium ethanedioate. Explain your reasoning.

Ionic radius of Ca2+ = 0.099 nm, Mg2+ = 0.065 nm

trendyline
tren y
Ca2+ is larger than Mg2+, hence
ence possesses
possesse smaller charge charg density / lower
power.. Electron cloud of C 2 O 4 2-- is distorted to a smaller extent and
polarising power
the C-O
C O bond is weaken to a smaller extent.
extent

Calcium ethanedioate has


Calc as a higher decomposition
composition tempera
temperature.
[3]

2016 TJC H2 Chemistry Preliminary Exam [Turn over


1501 trendyline
15

trendyline
2016 TJC H2 Chemistry Preliminary Exam [Turn over
1502 trendyline
16

(d) A tetradentate ligand, ethylenediamine-N,N-diacetate (EDDA) ion, can be synthesised


from ethanedioic acid through a series of steps. Ethanedioic acid is first converted to
ethane-1,2-diamine through a 3-step synthesis. Steps 4 to 7 represent the mechanism
of Skrecker synthesis, which can be used to synthesise amino acids.

trendyline
(i) Propose
ropose the 3-step synthesis forr the
t formation of ethane-1,2-diamine
ethane- from

2016 TJC H2 Chemistry Preliminary Exam [Turn over


1503 trendyline
17

ethanedioic acid, naming the reagents and conditions used in each step. Draw
the structures of the intermediate compoundsE and F in the spaces provided in
the synthetic route.

Step 1: Anhydrous LiAlH 4 , dry ether as solvent, room temperature

Step 2: PCl 5 (s), cold

Step 3: Excess conc NH 3 in ethanol, heat in sealed tube

Alternative Answer:

Step 1: PCl 5 (s), cold

E: (COCl) 2

Step 2: NH 3 (g), room temperature

F: (CONH 2 ) 2

Step 3: Anhydrous LiAlH 4 , dry ether as solvent, room temperature


[5]

(ii) Carbonyl compound and amine react in the following way.

O OH H

C
C + R"NH2 R N R"
R R'
R'

Step 4 of the synthetic route in (d) involves the reaction of ethane-1,2-diamine


with 2 moles of a carbonyl compound. Identify the carbonyl compound.

CH 2 O
[1]

(iii) Draw the structure of compound G in the space provided. [1]

(iv) State the types of reaction in step 5 and step 7.

ep 5:
Step Elimination

Step
ep 7:
7 Alkaline
li h hydrolysis
d l i
[2]

2016 TJC H2 Chemistry Preliminary Exam [Turn over


1504 trendyline
18

[Total: 24]

trendyline
2016 TJC H2 Chemistry Preliminary Exam [Turn over
1505 trendyline
1

CHEMISTRY 9647/03
Paper 3 Free Response 13th September 2016
2 hours
Candidates answer on separate paper.

Additional materials: Answer paper


Graph Paper
Data Booklet

READ THESE INSTRUCTIONS FIRST


Write your name, Civics Group, Centre number and Index number in the spaces provided on the
cover page and on all the work you hand in.
Write in dark blue or black pen on both sides of the paper.
You may use a HB pencil for any diagrams, graphs or rough working.
Do not use staples, paper clips, glue or correction fluid.

Answer any four questions.


A Data Booklet is provided.
The use of an approved scientific calculator is expected, where appropriate.
You are reminded of the need for good English and clear presentation in your answers.

At the end of the examination, fasten all your work securely together.
The number of marks is given in brackets [ ] at the end of each question or part question.

trendyline
This document consists of 15 printed pages.
pages

2016TJC H2 Chemistry Preliminary Exam [Turn Over

1506 trendyline
2

Answer any four questions

1 Ethanedioic acid is a substance found in many plant foods. Cabbage is among the plant foods
with high ethanedioic acid content. However, its anion, C 2 O 4 2- can bind to iron to form iron(II)
ethanedioate, which renders much of the iron in cabbage unusable by the body.

(a) 50.0 cm3 sample of iron(II) ethanediaote, FeC 2 O 4 was extracted from 300 g of cabbage,
diluted in water and the solution made up to 250 cm3. A 25.0 cm3 portion of this solution
was acidified and required 26.90 cm3 of 0.0100 mol dm-3 potassium manganate(VII) for
oxidation of iron(II) to iron(III) and ethanedioate ions to carbon dioxide.

(i) State the change in oxidation number for manganese and carbon in the reaction.
[1]

(ii) Write down all the relevant ion-electron halfequations and hence the overall redox
equation for the reaction between potassium manganate(VII) and iron(II)
ethanedioate. [2]

(iii) Calculate the concentration, in mol dm-3, of iron(II) ethanedioate in the original
sample.[2]

(iv) Calculate the number of moles of iron in each gram of cabbage. [1]

(b) Ethanedioic acid dissociates in water according to the following equation.

HOOC-COOH + H 2 O HOOC-COO- + H 3 O+

The table below compares the K a values of three organic acids.

Formula Ka

Ethanoic acid CH 3 COOH 1.74 x 10-5

Ethanedioic acid HO 2 CCO 2 H 6.46 x 10-2

Oxoethanoic acid (CHO)COOH 4.79 x 10-4

With reference to the K a values, comment on the order of acidity of the three organic

trendyline
acids. [2]

2016TJC H2 Chemistry Preliminary Exam [Turn Over

1507 trendyline
3

(c) Compound A,with molecular formula C 5 H 8 O, decolourises aqueous bromine and reacts
with PCl 5 giving off white fumes. Upon refluxing A with acidified potassium
manganate(VII), a symmetrical product B, C 5 H 6 O 5 , is formed. B does not give a red
precipitate with Fehlings solution but an orange precipitate is observed with 2,4-
dinitrophenylhydrazine. 1 mole of B also reacts with 1 mole of Na 2 CO 3 with
effervescence observed. B reacts with NaBH 4 to form C. Deduce the structures of
compounds A, B and C, explaining the chemistry of the reactions involved.
[8]

(d) (i) Describe what you see when separate samples of sodium and sulfur are burned in
excess air. Write equations for the reactions that occur. [2]

(ii) The products resulting from the reactions in (d)(i) both react with water. Write
equations for these two reactions and describe the effect of the resulting solutions
on Universal Indicator solution. [2]

[Total: 20]

trendyline
y
2016TJC H2 Chemistry Preliminary Exam [Turn Over

1508 trendyline
4

2 Thallium is a Group III element discovered in 1861, and its toxicity was quickly noted.
Thallium isotopes and compounds have useful applications, sometimes despite their toxic
nature.

(a) Thallium forms compounds similar to aluminium, and thallium(III) chloride and
thallium(III) fluoride have very different structures and bonding.

(i) Draw and state the shape of thallium(III) chloride, TlCl 3 .


[1]

(ii) In gaseous phase, thallium(III) chloride is able to form a compound with a M r of


621. Draw a dot-and-cross diagram of this compound, and state the Cl-Tl-Cl bond
angle.
[2]

(iii) Explain, in terms of structure and bonding, why thallium(III) fluoride has a high
melting point of 550 oC.[2]

(iv) Thallium is also able to form thallium(I) chloride, which is crystalline in nature.

In the crystal lattice of sodium and potassium chlorides, the co-ordination number
of each ion is 6. In the crystal lattice of thallium(I) chloride, TlCl, the co-ordination
number has a different value.

Suggest an explanation for the co-ordination number in the TlCl lattice being

trendyline
different from those in NaCland KCl.[1]
[ ]

2016TJC H2 Chemistry Preliminary Exam [Turn Over

1509 trendyline
5

(b) Thallium(III) chloride, TlCl 3 , reacts with hot water to produce thallium(III) oxide
precipitate and hydrochloric acid.

(i) Write a balanced equation for the above reaction.[1]

(ii) In an experiment, a 15 g impure sample of thallium(III) chloride (M r = 310.5) was


mixed with hot water, and filtered. Water was then added to the filtrate to make a
standard solution in a 250 cm3 volumetric flask. A 25.0 cm3 aliquot was titrated
against 0.5 mol dm-3 aqueous sodium hydroxide, requiring a titre of 22.00 cm3.
Determine the percentage purity of the sample.
You may assume that the impurities are insoluble in water and are inert.
[2]

(c) Thallium(I) sulfate(M r = 504.1) was often employed as a rodenticide and is highly
poisonous. The lethal dosage is 16 ppm of the rats mass. Assuming a typical rat has a
mass of 230 g, determine the number of moles of thallium(I)sulfate needed to kill a rat.
[2]

(d) Thallium(III) ethanoate, Tl(CH 3 CO 2 ) 3 , is able to react with an alkene through addition.
When cyclohexene reacts with thallium(III) ethanoate, a compound with the molecular
formula C 10 H 16 O 4 is formed. Suggest the structural formula of this compound, and state
the number of stereoisomers that this compound has.[2]

201
(e) Tl is an isotope of thallium that is often used in medical diagnostics due to its
radioactive decay by electron capture with emission of gamma rays. The electronic
configuration of the Tl atom is [Xe]4f145d106s26p1.
Electron capture is a process in which a proton-rich nucleus of an atom absorbs one
electron from the first or second quantum shell to change one proton into a neutron.
201
(i) Suggest the species formed, including the nucleon number, when Tl undergoes
electron capture.[1]

(ii) The species formed immediately after electron capture is known to be in excited
state, before returning to ground state by emitting gamma rays.
Name the orbital where the excited electron is most likely located in, and explain
why high energy gamma rays are emitted when the species return to ground

trendyline
state.
ate.
te
[2]]

2016TJC H2 Chemistry Preliminary Exam [Turn Over

1510 trendyline
6

(iii) A patient is typically injected with 0.02 g of thallium for heart diagnosis. The
following graph shows the radioactive decay (through electron capture) of a 0.02
g sample of 201Tl dissolved in 1 dm3 of inert solvent.
[201Tl] / g dm-3
0.02

0.01

0
100 200 300 Time / h

Using the graph, deduce the order of reaction for the radioactive decay of 201Tl.[2]

(iv) Hence, write the rate equation for this reaction and determine the rate constant,
stating its units.[2]

[Total: 20 m]

trendyline
2016TJC H2 Chemistry Preliminary Exam [Turn Over

1511 trendyline
7

3 The halogens and organic halides are useful laboratory reagents and have many applications.

(a) State and explain the difference in the reactions of chlorine and iodine with sodium
thiosulfate respectively. Write balanced equations for the reactions. [3]

(b) Deduce which gas, fluorine or chlorine, would behave more ideally at low pressure.
Explain your answer.
[1]

Organic halides are a group of compounds comprising alkyl halides, ethenylhalides and aryl
halides.

Alkyl halides are used as solvents for relatively non-polar compounds.They are also used as the
starting materials for the synthesis of many compounds.

(c) Alcohols react with hydrogen halides to produce alkyl halides.

(i) Suggest the type of reaction that occur. [1]

(ii) 2-iodopropane is formed from propan-2-ol in the presence of sodium iodide


andconcentrated phosphoric acid.

Explain, with the aid of an equation, why concentrated sulfuric acid is not used.[2]

(d) 2-chloro-2-methylpropane undergoes alkaline hydrolysis to form 2-methylpropan-2-ol.

(i) Name and describe the mechanism of the hydrolysis of 2-chloro-2-methylpropane.


[3]

(ii) With the aid of the Data Booklet, explain what will happen to the rate of reaction if 2-
bromo-2-methylpropane was used instead. [2]

trendyline
2016TJC H2 Chemistry Preliminary Exam [Turn Over

1512 trendyline
8

(e) Ethenylchloride is an important industrial chemical mainly used to produce the


polymer,PVC.

Ethynereacts with anhydrous hydrogen chloride gas over a mercuric chloride catalyst to
give ethenyl chloride.

ethyne ethenyl chloride

(i) With the aid of the Data Booklet, determine the enthalpy change of the reaction.
[2]

(ii) Explain why ethenyl chloride does not undergo alkaline hydrolysis readily. [1]

(f) 1-iodopropane can be converted to butylamine via a two-stage synthesis.Butylamineis


then used as an ingredient in the manufacture of pesticide.

(i) Suggest reagents and conditions for the two stages. Draw the structure of the
intermediate compound. [3]

(ii) Suggest a chemical test to distinguish between 1-iodopropane and


1-chloropropane. [2]

[Total: 20]

trendyline
2016TJC H2 Chemistry Preliminary Exam [Turn Over

1513 trendyline
9

4 Co-Cr alloys are most commonly used to make artificial joints including knee and hip joints
due to high wear-resistance and biocompatibility. However in 2010, faulty Co-Cr alloy hip
joints manufactured by DePuy was removed from market due to leakage of toxic chromium
and cobalt into the body muscles and blood stream.

(a) State the electronic configurations of

(i) an atom of chromium,


[1]

(ii) the Co3+ion. [1]

(b) Why are transition metal complexes coloured? [3]

trendyline
2016TJC H2 Chemistry Preliminary Exam [Turn Over

1514 trendyline
10

(c) The colour of the complexes can be measured by a spectrometer. The amount of light
absorbed is expressed as an absorbance value. The higher the absorbance, the more
of a particular wavelength of light is being absorbed.
The spectrum below shows the major absorption peaks for [CrL 6 ]3+(aq) and[CrJ 6 ]3+(aq).

Relative
absorbance

[CrJ6]3+(aq)

[CrL6]3+(aq)

300 400 500 600 700 800


Higher energy Lower energy
Wavelength/nm

colour Wavelength/nm

Violet 400-430

Blue 430-490

Green 490-570

Yellow 570-590

Orange 590-620

(i)
trendyline
rend
dyline
li e red

What are the coloursof [CrL 6 ]3+


3
(aq)
aq) and
620-750
620--750
620 5

[CrJ 6 ]3+
a [CrJ 3
(aq)? [2]

2016TJC H2 Chemistry Preliminary Exam [Turn Over

1515 trendyline
11

(ii) What deduction can be made from the spectra about the size of the d-orbital
splitting in the two complexes? [2]

Cobalt forms many complexes with ligands such as H 2 O, NH 3 and . The most
common oxidation states of cobalt are +2 and +3.
Cations containing cobalt in these two oxidation states have significantly different colours
even when the ligands are the same.

(d) When H 2 O 2 is mixed with pink Co2+(aq) solution, no significant colour change is
observed.
When Co2+(aq) is mixed with excess NH 3 (aq) a slight colour change occurs and cationQ
is formed.
When cationQ is mixed with H 2 O 2 , a reddish brown solution is observed.
E data for the cobalt complex ions and hydrogen peroxide are given below.
[CoL 6 ]3+ + e- [CoL 6 ]2+E = +1.82V for L = H 2 Oand E = +0.11V for L = NH 3
H 2 O 2 + 2e- 2OH- E = +0.87V

Suggest an explanation for the above observations, giving equations where appropriate
and identify the cationQ.[5]

(e) Explain why the neutral complex formed between a Co2+ ion and two
ions is soluble in cyclohexane.
[1]

The direct oxidation of methanoic acid in a fuel cell, DFAFC, shown below represents
potentially the most efficient method of obtaining useful energy from a renewable fuel.

trendyline
2016TJC H2 Chemistry Preliminary Exam [Turn Over

1516 trendyline
12

(f) The electrons pass around the external circuit to the cathode.
The protons produced migrate across the proton exchange membrane, PEM, to the
cathode, where they react with oxygen from air, producing water.

Write the equations for the anode and cathode reactions.Hence construct the equation
for the overall reaction. [2]

(g) One method for the construction of DFAFC, involves electroplating a layer of platinum
onto the surface of the proton exchange membrane, PEM. The electrolyte for this
process is a solution containing Pt(NH 3 ) 4 2+ and Cl- ions. The PEM is the cathode in this
electrolytic cell.

(i) Suggest the half-equation for the cathode reaction that deposits platinum on the
PEM. [1]

(ii) In one such preparation, a PEM with a surface area of 25 cm2 was immersed in
an electrolyte bath and a current of 0.0875 A was passed for 95 minutes.
Calculate the mass of platinum deposited onto the surface of PEM. [2]

[Total: 20]

5 (a) Myoglobin and haemoglobin are globular proteins that serve to bind and deliver oxygen.
The former serves as an intracellular storage site for oxygen in the muscle tissues, while
the latter transports oxygen in the bloodstream.
Each myoglobin molecule contains one haem group.
In deoxygenated myoglobin, a water ligand is weakly bound to the Fe2+ of the haem group
as shown:

haem group

y
2016TJC H2 Chemistry Preliminary Exam [Turn Over

1517 trendyline
13

(i) Briefly describe how myoglobin maintains its three-dimensional shape.


[2]

(ii) In the presence of the haem group, the polypeptide chain coalesces into a
myoglobin molecule.
With the aid of the equation,
* + 76
and by considering the chemical interactions involved, explain why this is a
spontaneous process at low temperatures.
[2]

(iii) Deoxygenated myoglobin is responsible for the dull purplish red muscle colour,
observed in the depth of the muscle. When meat is freshly cut, colour change occurs
in two stages:
Stage 1: Bright red colour when muscle surface is exposed to air.
Stage 2: With time, the bright red colour slowly changes to an unattractive, dull
brown colour associated with stale and spoiled meat.
With reference to the structure of the haem group and the reaction that occurs in
each stage, explain the colour changes involved.
[2]

(b) One of the isolated compounds that is primarily responsible for the umami (savory) flavor
is glutamate, which is a salt formed from glutamic acid that is commonly found in meat.

glutamate

t dyli
trendyline
It is thought
ought that glutamate stimulate a specific receptor site located
the tongue,
locate in the taste buds on
gue, which is essentially a protein molecule. A simplified d
site is shown
hown below:
diagram of the receptor

2016TJC H2 Chemistry Preliminary Exam [Turn Over

1518 trendyline
14

OH +
H3N

H2 N

(i) State two different types of interactions that can be formed between glutamate and
the receptor protein. Illustrate your answer with labelled diagram(s).
[2]

(ii) To enhance the flavor of food, many retailers in the food industry use monosodium
glutamate (MSG) as a food additive, which can be easily synthesized in the
laboratory.
Given that laboratory synthesis gives a racemic mixture, explain why 50% of the
glutamate synthesized in the laboratory is unable to bind precisely with the correct
orientation to the receptor protein. [1]

The structure of glutamic acid is shown below:

t d li
trendyline
glutamic
ic acid

10 cm3 of the fully protonated glutamic acid is titrated with 20 cm3 o


of 1.00 moldm3 sodium
hydroxide solution. Its titration curve is show
shown below:

2016TJC H2 Chemistry Preliminary Exam [Turn Over

1519 trendyline
15

pH

Vol of NaOH / cm3

(iii) Show that the concentration of glutamic acid is 1.0 moldm3 at A.


[1]

(iv) Hence, calculate the pH of the solution at A.


[1]

(v) Identify the specie(s) present at B.


With the help of an equation, explain how the solution at B can resist a change in pH
when a small amount of base is added. [2]

(vi) In a separate experiment, the solution at B was reacted with 10.0 cm3 of
0.05 moldm3 sodium hydroxide.
Calculate the pH of the mixture after the reaction.
You may find it useful to represent the fully protonated glutamic acid as HA.
[2]

(c) Compound P, C 9 H 11 NO 2 , rotates the plane ofpolarised light and reacts with the following
reagents:
aqueous bromine to give compound Q, C 9 H 9 NO 2 Br 2
sodium carbonate to give a salt
PCl 5 to give a 5-membered cyclic compound R
Deduce

t dyli
trendyline
e the structural formulae of compounds P, Q and
nd R.[5]
R.[5

[Total: 20]

2016TJC H2 Chemistry Preliminary Exam [Turn Over

1520 trendyline
16

trendyline
2016TJC H2 Chemistry Preliminary Exam [Turn Over

1521 trendyline
1

CHEMISTRY 9647/03
Paper 3 Free Response 13th September 2016
2 hours
Candidates answer on separate paper.

Additional materials: Answer paper


Graph Paper
Data Booklet

READ THESE INSTRUCTIONS FIRST


Write your name, Civics Group, Centre number and Index number in the spaces provided on the
cover page and on all the work you hand in.
Write in dark blue or black pen on both sides of the paper.
You may use a HB pencil for any diagrams, graphs or rough working.
Do not use staples, paper clips, glue or correction fluid.

Answer any four questions.


A Data Booklet is provided.
The use of an approved scientific calculator is expected, where appropriate.
You are reminded of the need for good English and clear presentation in your answers.

At the end of the examination, fasten all your work securely together.
The number of marks is given in brackets [ ] at the end of each question or part question.

trendyline
This document consists of 15 printed pages.
pages

2016TJC H2 Chemistry Preliminary Exam [Turn Over

1522 trendyline
2

Answer any four questions

1 Ethanedioic acid is a substance found in many plant foods. Cabbage is among the plant foods
with high ethanedioic acid content. However, its anion, C 2 O 4 2- can bind to iron to form iron (II)
ethanedioate, which renders much of the iron in cabbage unusable by the body.

(a) 50.0 cm3 sample of iron(II) ethanediaote, FeC 2 O 4 was extracted from 300 g of cabbage,
diluted in water and the solution made up to 250 cm3. A 25.0 cm3 portion of this solution
was acidified and required 26.90 cm3 of 0.0100 mol dm-3 potassium manganate(VII) for
oxidation of iron(II) to iron(III) and ethanedioate ions to carbon dioxide.

(i) State the change in oxidation number for manganese and carbon in the reaction.
[1]

Mn: From +7 to +2 [Or decrease by 5]


C: From +3 to +4 [Or increase by 1]

(ii) Write down all the relevant ion-electron halfequations and hence the overall redox
equation for the reaction between potassium manganate(VII) and iron(II)
ethanedioate. [2]

MnO 4 - + 8H+ + 5e- Mn2+ + 4H 2 O


C 2 O 4 2- 2CO 2 + 2e-
Fe2+ Fe3+ + e-
3MnO 4 - + 24H+ + 5C 2 O 4 2- + 5Fe2+ 3Mn2+ + 5Fe3+ + 12H 2 O + 10CO 2
(iii) Calculate the concentration, in mol dm-3, of iron(II) ethanedioate in the original
sample [2]
5
Number of moles of FeC 2 O 4 in 25.0 cm3 = x 2.69 x 10-4
3
= 4.48 x 10-4mol

Number of moles of FeC 2 O 4 in 250 cm3= 4.48 x 10-4 x 250/25.0


= 4.48 x 10-3 mol

4.48 x 10 -3
Concentration of FeC 2 O 4 in the original sample =
50 x 10 -3
= 0.0896 mol dm-3

(iv) Calculate the number of moles of iron in each gram of cabbage. [1]

4.48 x 10 -3
No. of moles of iron per gram of spinach =
300
= 1.49 x 10-5mol/g

trendyline
2016TJC H2 Chemistry Preliminary Exam [Turn Over

1523 trendyline
3

(b) Ethanedioic acid dissociates in water according to the following equation.

HOOC-COOH + H 2 O HOOC-COO- + H 3 O+

The table below compares the K a values of three organic acids.

Formula Ka

Ethanoic acid CH 3 COOH 1.74 x 10-5

Ethanedioic acid HO 2 CCO 2 H 6.46 x 10-2

Oxoethanoic acid (CHO)COOH 4.79 x 10-4

With reference to the K a values, comment on the order of acidity of the three organic
acids. [2]

The bigger the K a , the stronger the acid. Acid strength is dependent on the
stability of the anion formed when the acid ionises. The more stable the anion,
the stronger is the acid.
Strength of acid: Ethanedioic acid >Oxoethanoic acid >Ethanoic acid

Oxoethanoic acid is a stronger acid than ethanoic acid as -CHO group is


electron withdrawing. The negative charge on the anion is dispersed, thereby
stabilizing the ion compared to ethanoate ion.

Ethanedioic acid is the strongest acid as stabilisation of the monoanion


byintramolecular hydrogen bonding with the unionised COOH groupresults in
the highest K a value compared with the other two acids.

(c) Compound A,with molecular formula C 5 H 8 O, decolourises aqueous bromine and reacts
with PCl 5 giving off white fumes. Upon refluxing A with acidified potassium
manganate(VII), a symmetrical product B, C 5 H 6 O 5 , is formed. B does not give a red
precipitate with Fehlings solution but an orange precipitate is observed with 2,4-
dinitrophenylhydrazine. 1 mole of B also reacts with 1 mole of Na 2 CO 3 with
effervescence observed. B reacts with NaBH 4 to form C. Deduce the structures of
compounds A, B and C, explaining the chemistry of the reactions involved.
[8]

A decolourises aqueous bromine C=C present


A reacts with PCl 5 giving off white fumes of HCl -OH present
A undergoes oxidation with KMnO 4 where the OH group is oxidised and C=C
in a ring undergoes total bond cleavage to form B
OR A undergoes oxidation with KMnO 4 to give a symmetrical product B with
no loss in C atoms C=C is in a ring and A is a cyclic compound

trendyline
y n
B does
oes not give a red precipitate with Fehlin
Fehlings solution bbut form an orange
precipitate -dinitrophenylhydrazine B is a ket
2,4-dinitrophenylhydrazine
cipitate with 2,4
2,4- ketone and A is a 2o
keto
alcohol
ohol
h
1 mole O 3 B has 2 COOH
ole of B also reacts with 1 mole of Na 2 CO CO groups.
eacts with NaBH 4 to form C onl
B reacts only
o nly the carbonyl group iin B is reduced but
the carboxylic
b li acid
id groups remaini unaffected.
ff d

2016TJC H2 Chemistry Preliminary Exam [Turn Over

1524 trendyline
4

OH

A:

O O
HO C CH2 C CH2 COOH
B:

C:
( 9 marking points: Maximum 8)

(d) (i) Describe what you see when separate samples of sodium and sulfur are burned in
excess air. Write equations for the reactions that occur. [2]

Sodium burns readily with a brilliant yellow flame in air or oxygen to form
white sodium oxide, Na2 O.
4Na(s) + O 2(g) 2Na 2 O(s)

Sulfur burns slowly with a blue flame on heating in air or oxygen to form
colourless sulfur dioxide, SO 2. (Note: SO 3 is not formed)

S(s) + O 2 (g) SO 2 (g) or S 8 + 8O 2 8SO 2

Both equations correct


Both observations correct

(ii) The products resulting from the reactions in (d)(i) both react with water. Write
equations for these two reactions and describe the effect of the resulting solutions
on Universal Indicator solution. [2]
Na 2O(s) + H 2 O(l) 2NaOH(aq)
Universal Indicator turns violet.
SO 2 (g) + H 2O(l) H 2 SO 3(aq)
Universal Indicator turns red.

[Total: 20]

trendyline
2016TJC H2 Chemistry Preliminary Exam [Turn Over

1525 trendyline
5

2 Thallium is a Group III element discovered in 1861, and its toxicity was quickly noted.
Thallium isotopes and compounds have useful applications, sometimes despite their toxic
nature.

(a) Thallium forms compounds similar to aluminium, and thallium(III) chloride and
thallium(III) fluoride have very different structures and bonding.

(i) Draw and state the shape of thallium(III) chloride, TlCl 3 .


[1]

Cl

Tl
Cl Cl

Trigonal planar

(ii) In gaseous phase, thallium(III) chloride is able to form a compound with a M r of


621. Draw a dot-and-cross diagram of this compound, and state the Cl-Tl-Cl bond
angle.
[2]
Based on the M r , the compound is Tl 2 Cl 6 .

Cl Cl Cl
x x x
Tl
x x
Tlx
Cl

Cl
Cl
109.5o

(iii) Explain, in terms of structure and bonding, why thallium(III) fluoride has a high
melting point of 550 oC.[2]

Thallium(III) fluoride exists as a giant ionic lattice structure and


a large amount of energy is needed to break strong ionic bonds between
oppositely charged ions.

trendyline
2016TJC H2 Chemistry Preliminary Exam [Turn Over

1526 trendyline
6

(iv) Thallium is also able to form thallium(I) chloride, which is crystalline in nature.

In the crystal lattice of sodium and potassium chlorides, the co-ordination number
of each ion is 6. In the crystal lattice of thallium(I) chloride, TlCl, the co-ordination
number has a different value.

Suggest an explanation for the co-ordination number in the TlCl lattice being
different from those in NaCl and KCl..[1]

The size of the Tl+cations is larger than the Group Ications such as Na+ and
K+. More chloride ions can be packed around the larger Tl+cations.

(b) Thallium(III) chloride, TlCl 3 , reacts with hot water to produce thallium(III) oxide
precipitate and hydrochloric acid.

(i) Write a balanced equation for the above reaction.[1]

2TlCl 3 + 3H 2 O Tl 2 O 3 + 6HCl

(ii) In an experiment, a 15 g impure sample of thallium(III) chloride (M r = 310.5) was


mixed with hot water, and filtered. Water was then added to the filtrate to make a
standard solution in a 250 cm3 volumetric flask. A 25.0 cm3 aliquot was titrated
against 0.5 mol dm-3 aqueous sodium hydroxide, requiring a titre of 22.00 cm3.
Determine the percentage purity of the sample.
You may assume that the impurities are insoluble in water and are inert.
[2]

2TlCl
TllCl 3 +&l
o of moles of NaO
No NaOH = 0.022 x 0.5 = 0.0110 mo
mol
o of moles of HC
No HCll produced
oduced from the sample = 0.0110 x 10
1 = 0.110 mol
o of moles of TlCl 3 in sample = 0.110
No 01110 / 3 = 0
0.0367
0367 mol

2016TJC H2 Chemistry Preliminary Exam [Turn Over

1527 trendyline
7

Mass of TlCl 3 in sample = 310.5 x 0.0367 = 11.4 g


Percentage purity of TlCl 3 in sample = 11.4 / 15 x 100% = 76.0%

(c) Thallium(I) sulfate(M r = 504.1) was often employed as a rodenticide and is highly
poisonous. The lethal dosage is 16 ppm of the rats mass. Assuming a typical rat has a
mass of 230 g, determine the number of moles of thallium(I)sulfate needed to kill a rat.
[2]

Mass of thallium(I) sulfate needed for lethal dosage = 230 x 16 / 106 = 0.00368 g
No of moles of thallium(I) sulfate needed = 0.00368 / 504.1= 7.30 x 10-6mol

(d) Thallium(III) ethanoate, Tl(CH 3 CO 2 ) 3 , is able to react with an alkene through addition.
When cyclohexene reacts with thallium(III) ethanoate, a compound with the molecular
formula C 10 H 16 O 4 is formed. Suggest the structural formula of this compound, and state
the number of stereoisomers that this compound has.[2]

CH3COO OCOCH3
This compound has a cis- and a trans-isomer, and the trans-isomer is optically
active with two enantiomers. Therefore this compound will have 3 stereoisomers.
201
(e) Tl is an isotope of thallium that is often used in medical diagnostics due to its
radioactive decay by electron capture with emission of gamma rays. The electronic
configuration of the Tl atom is [Xe]4f145d106s26p1.
Electron capture is a process in which a proton-rich nucleus of an atom absorbs one
electron from the first or second quantum shell to change one proton into a neutron.
201
(i) Suggest the species formed, including the nucleon number, when Tl undergoes
electron capture.[1]

201Hg

(ii) The species formed immediately after electron capture is known to be in excited
state, before returning to ground state by emitting gamma rays.
Name the orbital where the excited electron is most likely located in, and explain
why high energy gamma rays are emitted when the species return to ground
state.
[2]

6p. (can accept any orbital of higher energy than 6p)


When the electron capture occurs, an inner shell (1 or 2) is now short of an

trendyline
tren d li
dyline
p electron is at a high
electron. The 6p g energy
gy level,, and needs to release a
large
rge
ge amount of energy
energ to replace the absorbed electron in shell 1 or 2 as
e atom returns to ground state.
the

2016TJC H2 Chemistry Preliminary Exam [Turn Over

1528 trendyline
8

(iii) A patient is typically injected with 0.02 g of thallium for heart diagnosis. The
following graph shows the radioactive decay (through electron capture) of a 0.02
g sample of 201Tl dissolved in 1 dm3 of inert solvent.

[201Tl] / g dm-3
0.02

0.01

0
100 200 300 Time / h

Using the graph, deduce the order of reaction for the radioactive decay of 201Tl.[2]

From the graph, (t ) 1 = (t ) 2 = 75 h. Since t is constant,


order of reaction is 1.

(iv) Hence, write the rate equation for this reaction and determine the rate constant,
stating its units.[2]

rate = k[201Tl]
k = ln2 / t = ln2 / 75 h = 0.00924 h-1

[Total: 20 m]

trendyline
2016TJC H2 Chemistry Preliminary Exam [Turn Over

1529 trendyline
9

3 The halogens and organic halides are useful laboratory reagents and have many applications.

(a) State and explain the difference in the reactions of chlorine and iodine with sodium
thiosulfate respectively. Write balanced equations for the reactions. [3]

Cl 2 + e 2Cl- E = +1.36 V
I2 + e 2I- E = +0.54 V
Oxidising power of the halogens decreases down the group as can be seen from
the less positive E value, hence iodine can only oxidise thiosulfate to tetrathionate.
4Cl 2 + S 2 O 3 2- + 5H 2 O 8Cl- + 2SO 4 2- + 10H+
I 2 + 2S 2 O 3 2- S 4 O 6 2- + 2I -

(b) Deduce which gas, fluorine or chlorine, would behave more ideally at low pressure.
Explain your answer.
[1]

Fluorine gas will behave more ideally. The electron cloud size of chlorine is larger
than fluorine, hence the van der Waals forces of attraction is stronger.

Organic halides are a group of compounds comprising alkyl halides, ethenylhalides and aryl
halides.

Alkyl halides are used as solvents for relatively non-polar compounds. They are also used as
the starting materials for the synthesis of many compounds.

(c) Alcohols react with hydrogen halides to produce alkyl halides.

(i) Suggest the type of reaction that occur. [1]

Nucleophilic substitution
(ii) 2-iodopropane is formed from propan-2-ol in the presence of sodium iodide
andconcentrated phosphoric acid.

Explain, with the aid of an equation, why concentrated sulfuric acid is not used. [2]

The hydrogen iodide produced will be readily oxidised by concentrated


sulfuric acid to iodine hence will not be availble to react with the alcohol.
8HI + H 2 SO 4 4I 2 + H 2 S + 4H 2 O
(d)
2-chloro-2-methylpropane undergoes alkaline hydrolysis to form 2-methylpropan-2-ol.

(i) Name and describe the mechanism of the hydrolysis of 2-chloro-2-methylpropane.


[3]

trendyline
d
Nucleophilic
p substitution S N 1

2016TJC H2 Chemistry Preliminary Exam [Turn Over

1530 trendyline
10

(ii) With the aid of the Data Booklet, explain what will happen to the rate of reaction if 2-
bromo-2-methylpropane was used instead. [2]

BE(C-Cl) = 340 kJ mol-1


BE(C-Br) = 280 kJ mol-1
Since bond energy of C-Cl is higher than that of C-Br, the bond strength of
C-Cl is greater than that of C-Br. Hence the reactivity of the alkyl halides
decreases in the order RBr>RCl.The rate of reaction will increase

(e) Ethenylchloride is an important industrial chemical mainly used to produce the


polymer,PVC.

Ethynereacts with anhydrous hydrogen chloride gas over a mercuric chloride catalyst to
give ethenyl chloride.

ethyne ethenyl chloride

(i) With the aid of the Data Booklet, determine the enthalpy change of the reaction.
[2]

H r = enthalpy change for bond break + enthalpy change for bond form

= BE(H-Cl %( &&  [BE(C-Cl) + BE(C-H) + BE(C=C)]
= 431 + 840 (340 + 410 + 610) = -89 kJ mol-1

(ii) Explain why ethenyl chloride does not undergo alkaline hydrolysis readily. [1]

The lone pair of electrons on chlorine delocalised across the double bond
resulting in partial double bond character in CCl bond, hence a stronger CCl
bond which is harder to break.

trendyline
OR
The carbon-carbon double
The nucleophile is unlikely to attack the carbon d bond as it is
electron
lectron rich.

(f) 1-iodopropane
ropane can be converted to butylamine
buttylamine
yla via a two-stage synthesis.Butylamine is
two stage sy

2016TJC H2 Chemistry Preliminary Exam [Turn Over

1531 trendyline
11

then used as an ingredient in the manufacture of pesticide.

(i) Suggest reagents and conditions for the two stages. Draw the structure of the
intermediate compound. [3]

(ii) Suggest a chemical test to distinguish between 1-iodopropane and 1-chloropropane.


[2]

Procedure:
Step: (1) Boil RX with NaOH(aq).
(2) Acidify with dilute HNO 3 .
(3) Add AgNO 3 (aq).
OR
Boil with silver(I) ethanoate in alcohol.

White precipitate (AgCl) observed for 1-chloropropane.


Yellow precipitate (AgI) observed for 1-iodopropane.

[Total: 20]

4 trendyline
y
Co-Cr alloys are most commonly used to make artificial joints including knee and hip joints
due to high wear-resistance and biocompatibility.
bilit However in 2010, faulty Co-Cr alloy hip

2016TJC H2 Chemistry Preliminary Exam [Turn Over

1532 trendyline
12

joints manufactured by DePuy was removed from market due to leakage of toxic chromium
and cobalt into the body muscles and blood stream.

(a) State the electronic configurations of

(i) an atom of chromium,


[1]

1s22s22p63s23p63d54s1

(ii) the Co3+ion. [1]

1s22s22p63s23p63d6

(b) Why are transition metal complexes coloured? [3]

Transition metal complexes are often coloureddue to the presence of partially


filled d-orbitals in the metal ion.
In complexes, the presence of ligands causes a splitting of the energy of d
orbitals into 2 groups ZLWKDQHQHUJ\JDS(EHWZHHQWKHP
When a d-electron from lower energy group is promoted to the higher energy
group (d-d transition), radiation in the visible region of the electromagnetic
VSHFWUXPFRUUHVSRQGLQJWR(LVDEVRUEHG
Light energy that is not absorbed will be seen as the colour of the complex.

trendyline
2016TJC H2 Chemistry Preliminary Exam [Turn Over

1533 trendyline
13

(c) The colour of the complexes can be measured by a spectrometer. The amount of light
absorbed is expressed as an absorbance value. The higher the absorbance, the more
of a particular wavelength of light is being absorbed.
The spectrum below shows the major absorption peaks for [CrL 6 ]3+(aq) and[CrJ 6 ]3+(aq).

Relative
absorbance

[CrJ6]3+(aq)

[CrL6]3+(aq)

300 400 500 600 700 800


Higher energy Lower energy
Wavelength/nm

The wavelengths associated with each colour of light are given below.

colour Wavelength/nm

Violet 400-430

Blue 430-490

Green 490-570

Yellow 570-590

trendyline
rend
dyline
yli e Orange

red
590-620
590--620
590 2

620-750
620
0-750
5

2016TJC H2 Chemistry Preliminary Exam [Turn Over

1534 trendyline
14

(i) What are the coloursof [CrL 6 ]3+(aq) and [CrJ 6 ]3+(aq)? [2]

[CrL 6 ]3+(aq) : yellow/orange


[CrJ 6 ]3+(aq) : green/blue

(ii) What deduction can be made from the spectra about the size of the d-orbital
splitting in the two complexes? [2]

The d-orbital splitting for [CrL 6 ]3+(aq) is larger than [CrJ 6 ]3+(aq)
because electrons in lower d-subshell absorbs energy in shorter
wavelength and higher energy to excite to higher d-VXEVKHOO(LV
larger.

Cobalt forms many complexes with ligands such as H 2 O, NH 3 and . The most
common oxidation states of cobalt are +2 and +3.
Cations containing cobalt in these two oxidation states have significantly different colours
even when the ligands are the same.

(d) When H 2 O 2 is mixed with pink Co2+(aq) solution, no significant colour change is
observed.
When Co2+(aq) is mixed with excess NH 3 (aq) a slight colour change occurs and cationQ
is formed.
When cationQ is mixed with H 2 O 2 , a reddish brown solution is observed.
E data for the cobalt complex ions and hydrogen peroxide are given below.
[CoL 6 ]3+ + e- [CoL 6 ]2+E = +1.82V for L = H 2 Oand E = +0.11V for L = NH 3
H 2 O 2 + 2e- 2OH-E = +0.87V

Suggest an explanation for the above observations, giving equations where appropriate
and identify the cationQ.[5]
E Co3+/Co2+ =+1.82V
E H2O2/OH- =+0.87V
E H2O2/H2O is less positive than E Co3+/Co2+ , H 2 O 2 is unable to oxidiseCo(aq)2+ to
Co(aq)3+ and there is no ligand exchange reaction taking place. Hence, no
colour change.
When excess NH 3 is added, a ligand exchange reaction occurs. NH 3 replaces
H 2 O, ligands, resulting in a slight colour change.
[Co(H 2 O) 6 ]2+ + 6NH 3 [Co(NH 3 ) 6 ]2++ 6H 2 O
[explanation & equation: 1m]
Cation Q is [Co(NH 3 ) 6 ]2+.

E[ Co(NH3)6] 3+ /[Co(NH3)6] 2+ =+0.11 V

e dyy
E H2O2/OH-
H- =+0.87V
OH
E cell = +0.87
+0.87-(+0.11)
(+0.11) = +0.76 V > 0

When H 2 O 2 is added to Q,
Q
H 2 O 2 oxidises Co2+ in
n [Co(NH 3 ) 6 ]2+
2
tto
oCCo3+ in [Co(NH
[Co( 3 ) 6 ]3+, res
re
resulting in a reddish
brownwn solution observed
observed.

2016TJC H2 Chemistry Preliminary Exam [Turn Over

1535 trendyline
15

H 2 O 2 + 2[Co(NH 3 ) 6 ]2+ 2OH- + 2[Co(NH 3 ) 6 ]3+


(e) Explain why the neutral complex formed between a Co2+ ion and two

ions is soluble in cyclohexane. [1]

It is a simple covalent molecule. The weak van der Waals forces of


attraction formed between Co-complex and cyclohexane
molecules liberate sufficient energy to overcome van der Waalsforces of
attraction between Co-complex molecules and those between cyclohexane
molecules.
For your information:

The direct oxidation of methanoic acid in a fuel cell, DFAFC, shown below represents
potentially the most efficient method of obtaining useful energy from a renewable fuel.

(f) The electrons pass around the external circuit to the cathode.
The protons produced migrate across the proton exchange membrane, PEM, to the
cathode, where they react with oxygen from air, producing water.

Write the equations for the anode and cathode reactions.Hence construct the equation
for the overall reaction. [2]

HCO 2 H CO 2 + 2H+ + 2e- [Anode]


O 2 + 4H+ + 4e- 2H 2 O [cathode]
2HCO 2 H + O 2 2CO 2 + 2H 2 O [overall reaction]

(g) One method for the construction of DFAFC, involves electroplating a layer of platinum

trendyline
onto thee surface of the proton exchange membrane,membr PEM. The electrolyte for this
Cl- ions. The PEM iis the cathode in this
processs is a solution containing Pt(NH 3 ) 4 2+ and Cl
electrolytic
ytic cell.

(i) Suggest the ha


half -equation
equation for the cathode reaction that depo
half-equation deposits platinum on the
PEM. [1]

2016TJC H2 Chemistry Preliminary Exam [Turn Over

1536 trendyline
16

Pt(NH 3 ) 4 2+ + 2e- Pt + 4NH 3

(ii) In one such preparation, a PEM with a surface area of 25 cm2 was immersed in
an electrolyte bath and a current of 0.0875 A was passed for 95 minutes.
Calculate the mass of platinum deposited onto the surface of PEM. [2]

Q = It = 0.0875 x 95 x 60 = 498.75 C
No of mol of electrons = 498.75/96500 = 5.17 x 10-3mol
No of mol of Pt deposited = 5.17 x 10-3/2 = 2.58 x 10-3mol
mass of Pt deposited = 2.58 x 10-3 x 195 = 0.504 g

[working : 1m & answer:1m]


[Total: 20]

5 (a) Myoglobin and haemoglobin are globular proteins that serve to bind and deliver oxygen.
The former serves as an intracellular storage site for oxygen in the muscle tissues, while
the latter transports oxygen in the bloodstream.
Each myoglobin molecule contains one haem group.
In deoxygenated myoglobin, a water ligand is weakly bound to the Fe2+ of the haem group
as shown:

haem group

Myoglobin Haem group in


deoxygenated myoglobin

(i) Briefly describe how myoglobin maintains its three-dimensional shape.


[2]

trendyli
trendyline
ndyline
dyli
 It is maintained by the interactions
nteractions between
be the R-groups of the amino
acids,
cids, such as hydrogen bonds between polar group groups (OH, NH 2 ), ionic
bondsbetween
ondsbetween oppositely charged group CO 2 & +N
groups (CO -
NH 3 ), van der Waals
forcesof
rcesof attractio
attraction between non
non-polar
olar side chains (e.g: alkyl group)
and disulfide linkagesbetween SH groups.

2016TJC H2 Chemistry Preliminary Exam [Turn Over

1537 trendyline
17

(ii) In the presence of the haem group, the polypeptide chain coalesces into a
myoglobin molecule.
With the aid of the equation,
* + 76
and by considering the chemical interactions involved, explain why this is a
spontaneous process at low temperatures.
[2]

The bending and folding of the primary structure into a tertiary structure
is exothermic(negative H) due to the formation of various interactions
between the R-groups of the constituent amino acids.
The process is accompanied by a decrease in entropy (negative S) as the
formation of a more ordered myoglobin molecule decreases the degree of
disorder because there are now fewer ways in which the molecule and energy
can be distributed.
Hence G is negative at low temperatures.

(iii) Deoxygenated myoglobin is responsible for the dull purplish red muscle colour,
observed in the depth of the muscle. When meat is freshly cut, colour change occurs
in two stages:
Stage 1: Bright red colour when muscle surface is exposed to air.
Stage 2: With time, the bright red colour slowly changes to an unattractive, dull
brown colour associated with stale and spoiled meat.
With reference to the structure of the haem group and the reaction that occurs in
each stage, explain the colour changes involved.
[2]

The colour of meat depends on the ligand attached to the 6th coordinate site in
the haem group, and the oxidation stage of the iron centre.
:KHQ WKH PHDW LV H[SRVHG WR R[\JHQ IURP WKH DLU OLJDQG H[FKDQJH
(oxygenation) occurs to form oxymyoglobin. O 2 replaces H 2 O ligand and is
dative bonded to the haem group, which gives the bright red colour observed.
:KHQ PHDW KDV EHHQ VWRUHG IRU WRR ORQJ WKH )H2+ in the haem group
undergoes oxidation to form Fe3+, which gives rise to the dull brown colour.

(b) One of the isolated compounds that is primarily responsible for the umami (savory) flavor
is glutamate, which is a salt formed from glutamic acid that is commonly found in meat.

trendyline
dyline
y glutamate
utam

2016TJC H2 Chemistry Preliminary Exam [Turn Over

1538 trendyline
18

It is thought that glutamate stimulate a specific receptor site located in the taste buds on
the tongue, which is essentially a protein molecule. A simplified diagram of the receptor
site is shown below:

OH +
H3N

H2 N

(i) State two different types of interactions that can be formed between glutamate and
the receptor protein. Illustrate your answer with labelled diagram(s).
[2]

Any two illustrated

(ii) To enhance the flavor of food, many retailers in the food industry use monosodium
glutamate (MSG) as a food additive, which can be easily synthesized in the
laboratory.
Given that laboratory synthesis gives a racemic mixture, explain why 50% of the
glutamate synthesized in the laboratory is unable to bind precisely with the correct

ttrendyline
e dy e
orientation
ientation to the receptor protei
protein. [1]

Only
Only one mirror image form of glutamate
glutamat synthesized provides the desired
biological
ological effect as it has the correct orientation to bind to the protein
receptors. The other form is inactive.
ctiv

2016TJC H2 Chemistry Preliminary Exam [Turn Over

1539 trendyline
19

The structure of glutamic acid is shown below:

glutamic acid
10 cm3 of the fully protonated glutamic acid is titrated with 20 cm3 of 1.00 moldm3 sodium
hydroxide solution. Its titration curve is shown below:

pH

Vol of NaOH / cm3

(iii) Show that the concentration of glutamic acid is 1.0 moldm3 at A.


[1]

n(NaOH) used to reach 1st endpoint = 0.010 mol


n(glutamic acid) = 0.010 mol
[glutamic acid] = 0.010/0.0100= 1.0 moldm3

(iv) Hence, calculate the pH of the solution at A.


[1]

trendyline
pH
dy
H+] = = . . = 0.0804 mol dm3
[H
pH at A = log
og 110 (0.0804) = 1.
1.10

2016TJC H2 Chemistry Preliminary Exam [Turn Over

1540 trendyline
20

(v) Identify the specie(s) present at B.


With the help of an equation, explain how the solution at B can resist a change in pH
when a small amount of base is added. [2]


and

When a small amount of alkali is added,

+ OH+ H 2O

Negligible change in pH as OH ions are removed.

(vi) In a separate experiment, the solution at B was reacted with 10.0 cm3 of
0.05 moldm3 sodium hydroxide.
Calculate the pH of the mixture after the reaction.
You may find it useful to represent the fully protonated glutamic acid as HA.
[2]

HA + OH- A- + H 2O
At point B, nHA = n A- = 0.010 / 2= 0.005 mol
Number of moles NaOH added = 0.05 x 10/1000 = 5.00 x 10-4mol
Number of moles HA at equilibrium = 0.005 5.00 x 10-4mol = 0.0045 mol

Number of moles A at equilibrium = 0.005 + 5.00 x 10-4mol = 0.0055 mol
pH = pKa + lg [salt]/[acid]
= 2.19 + lg {(0.0055/0.025) / (0.0045/0.025)} = 2.28

(c) Compound P, C 9 H 11 NO 2 , rotates plane polarised light and reacts with the following
reagents:
aqueous bromine to give compound Q, C 9 H 9 NO 2 Br 2
sodium carbonate to give a salt
PCl 5 to give a 5-membered cyclic compound R
Deduce the structural formulae of compounds P, Q and R.[5]

P rotates plane polarised light P contains a chiral carbon; has no plane of

trendyline
trendyline
symmetry
try and a non-superimposable
ble mirror imag
image.
P reacts
cts with aq Br 2 to
o give compound Q, C 9 H 9 NO 2 Br 2
P isphenylamine
henylamine or pheno
phenol
one of the 2, 4, 6 positions relative
ve to first substituent on benzene is occupied

2016TJC H2 Chemistry Preliminary Exam [Turn Over

1541 trendyline
21

with another substituent


P reacts with sodium carbonate to give a salt
P contains a COOH group.
(Hence, P cant be phenol as molecular formula only contains 2 O atoms)
OH in COOH reacts with PCl 5 to give COCl . Formation of 5-membered ring
NH 2 group in P attacks the electron deficient carbonyl C in the COClgroup.
NH 2 group &the other substituent on benzene ring are ortho to each other.
[4 marking points, max 2 marks]

P: R:

Q:

[1m for each structure]


[Total: 20]

trendyline
2016TJC H2 Chemistry Preliminary Exam [Turn Over

1542 trendyline
Tampines Junior College
2016 Preliminary Examination
H2 Chemistry Paper 1

trendyline
1543 trendyline
2

Section A

For each question, there are four possible answers, A, B, C and D. Choose the one you
consider to be correct.

1 Use of the Data Booklet is relevant to this question.

How many atoms are present in 1 cm3 of ethane gas under room conditions?

(8 x 24000) (8 x 6.02 x1023 )


A 23 B
(6.02 x 10 ) (22400)

(8 x 6.02 x1023 )
C D 6.02 x 1023 x 30
(24000)

2 Sodium percarbonate, x(Na2CO3).y(H2O2) is a chemical that is used in cleaning


powders. When dissolved in water, it releases hydrogen peroxide and sodium carbonate,
both useful cleaning agents.

A sample of 10.0 cm3 of 0.100 mol dm-3 sodium percarbonate requires 20.0 cm3 of
0.100 mol dm-3 of acidified KI before the end point of titration is reached.

Another identical sample releases 48.0 cm3 of carbon dioxide at room conditions on
reaction with excess acid.

What is the value of x and y respectively?

A 2 and 1 B 3 and 2
C 2 and 3 D 3 and 1

3 Which of the following molecules is non polar?

A AsBr3
B C2Cl4
C CH3CBr3
D H2S

4 Which of the following pairs does the first molecule have a larger bond angle than the
second molecule?

A PCl3, PBr3
B H2S, H2O
C NH3, BF3
D SnCl2, OC
OCl
Cl2

TPJC 9647/1/JC2/Prelim/16
1544 trendyline
3

5 Which statement is not true about the structure of silicon dioxide, SiO2?

A The bond angle about Si is 109.5o.


B The bond angle about O is 180o.
C SiO2 has a giant molecular structure.
D SiO2 is insoluble in water.

6 Use of the Data Booklet is relevant to this question.

Which one of the following species has the same number of neutrons and protons, and
more electrons than protons?

37
A Cl B 48
Ti4+ C 79
Br+ D 32
S2-

7 Which statement regarding isotopes is incorrect?

A They have the same atomic number.


B They have the same neutron number.
C They have different mass number.
D They have different physical properties.

8 Use of the Data Booklet is relevant to this question.

The standard enthalpy change of formation of C3H8(g) is 104 kJ mol1.

3C(s) + 4H2(g) 3C(g) + 8H(g)

C3H8(g)

What is the standard enthalpy change of atomisation of carbon, C(s)?

A 711 kJ mol-1 B +711 kJ mol-1


C 129 kJ mol-1 D +129 kJ mol-1

trendyline
TPJC 9647/1/JC2/Prelim/16
1545 [Turn Over
trendyline
4

9 Hydrogen gas and fluorine gas react to form hydrogen fluoride as shown in the equation
below:
H2(g) + F2(g) 2HF(l) Ho = 542 kJ mol1

The standard entropy change of this reaction is 160 J K1 mol1.

Which statement is correct?

A At room temperature, the reaction is not spontaneous.


B An addition of catalyst will increase the yield of the product.
C There is an increase in the degree of disorder.
D The products are more stable than the reactants.

10 For the reaction 3A(aq) + 2B(aq) C(aq) at room temperature, the rate equation is
Rate = k [H+][B]
Which statement is incorrect?

A The reaction involves more than one step.


B H+ is a catalyst in the above reaction.
C When the concentration of A is doubled at 10 oC, the rate of reaction decreases as
compared to room temperature.
D When the concentration of H+ and B is doubled, the amount of C formed is increased
four-fold.

11 The value of Kp of the reaction D2(g) + 2E2(g)  2DE2(g) is 3. Under the same reaction
conditions, what is the value of Kp for the reaction DE2(g) D2(g) + E2(g)?


A
B 3 C 1.5 D 3

12 What is the pH of the solution when 60 cm3 of 0.15 mol dm3 of CH3COOH and 80 cm3 of
0.10 mol dm3 CH3COONa+ are mixed at 25 oC?
(Ka of CH3COOH = 1.78 x 10-5 mol dm-3)

A 4.70 B 4.75 C 9.20 D 9.25

13 A 1.50 A current passed through a solution of osmium chloride, resulting in the reduction
of the metal ion. After 20 minutes, 0.888 g of osmium has been deposited. What is the

trendyline
oxidation state of osmium in osmium chloride?

A +1 B +2
+2 C +3
+3 D +4
+4

TPJC 9647/1/JC2/Prelim/16
1546 trendyline
5

14 Phosphorus trichloride and chlorine react according to the following equation:

PCl3(g) + Cl2(g) PCl5(g) 'Ho = +202 kJ mol1

A mixture of PCl3 and Cl2 was placed in a syringe and equilibrium was achieved. The
following graph was obtained. What could have happened at time t?

[PCl3]

t
A A catalyst was added to the mixture.
B The plunger was compressed by 5 cm3.
C The syringe was dropped in ice cold water.
D Cl2 was removed from the mixture.

15 On strong heating, SrSO4 decomposes into SrO and SO3. The compound Sr(NO3)2
decomposes at a much higher temperature than SrSO4. Which statement best explains
the lower thermal stability of SrSO4?

A SrSO4 has a more exothermic lattice energy than Sr(NO3)2.


B NO2 is a smaller molecule than SO3.
C SO42 is more easily polarised than NO3.
D The charge on SO42 is greater than that on NO3.

16 Which property of the elements in the Period 3 (sodium to sulfur) decreases numerically
across the Period?

A pH of its oxides in water


B Ionic radius

trendyline
C Melting po
points of its oxides
D 1st ionisation
ation
tion energy

TPJC 9647/1/JC2/Prelim/16
1547 [Turn Over
trendyline
6

17 Radium is the last element in Group II in the Periodic Table.

Which statement is not true for radium?

A Its oxide dissolves in water to form a basic solution.


B It is the most reactive element in the group.
C The melting point of its chloride is lower to that of FrCl.
D It reacts with water to form hydrogen gas.

18 The hexaaquairon(III) complex hydrolyses as shown.

[Fe(H2O)6]3+ [Fe(H2O)5OH]2+ + H+

Which of the following statements is true?

A There is a colour change from yellow to green.


B The hydrolysis is favoured by low pH values.
C The electronic configuration of Fe3+ changed from [Ar]3d5 to [Ar]3d6.
D [Fe(H2O)6]2+ is less acidic in solution as compared to [Fe(H2O)6]3+.

19 Vanadium(III) chloride reacts with carbon monoxide, CO, to form a compound in which
the coordination number of the vanadium complex is 6. A formula unit of one such
compound contains a cation and only one chloride ion.

What is the formula of this compound?

A V(CO)5Cl B V(CO)5Cl2
C V(CO)4Cl3 D V(CO)3Cl3

20 In which reaction is the inorganic reagent acting as an electrophile?

Cl

A + Cl2 + HCl

Cl OH
B + KOH + KCl

+ Br2 Br
+ HBr

ttrendyline
end
C

O
O
OH

D + HCN
N CN

TPJC 9647/1/JC2/Prelim/16
1548 trendyline
7

21 Dopamine is a compound present in the body that functions as a neurotransmitter and a


precursor for other substances including adrenaline. It is also synthesized as a drug for
treatment of some neurodegenerative disease such as Parkinsons disease.

NH2
OH

O
HO
OH

Dopamine

Which statement about dopamine is not true?

A It is optically active.
B It can form a zwitterion in aqueous solution.
C It forms an orange precipitate with 2,4DNPH.
D One mole of dopamine forms an anion with 3 charge when reacted with 3 moles of
NaOH.

22 Codeine is an opioid pain medication. It is used to treat mild to severe pain in patients.

H3CO

O H
N
H

HO

Codeine

(The COC and OCH3 groups do not undergo any reaction.)

Four students made claims about the reactions of codeine.

F 1 mol of codeine reacts with 2 mol of H2(g).

G 1 mol of codeine reacts with 1 mol of PCl5(s).

H 1 mol of codeine reacts with 2 mol of CH3CO2H(l) under suitable conditions.

trendyline
I 1 mol of codeine reacts with 1 mol of dry HCl(g).

Which pair of students claims is correct?

A F and G B G and
nd H
C H and I D F and I

TPJC 9647/1/JC2/Prelim/16
1549 [Turn Over
trendyline
8

23 Which reaction scheme is unable to produce the intended final product?

COOH COCl COOCH2CH3


D
PCl5 CH3CH2OH

24 An organic compound has the following properties:


x It decolourises hot, acidified KMnO4(aq).
x It gives a yellow precipitate with aqueous alkaline iodine.
x It forms a silver mirror with Tollens solution.
Which compound could give these results?

A CH3COOCH2CH2COCH3
B CH3CH(OH)CH2CH2CHO
C HCOCH2CH2CH2CHO
D CH3CH2C(CH3)(OH)(CH2)2COCH3

25
1,2ethanediamine, can be synthesised from ethanol through a
series of reactions.

Which set of reagents, used in sequential order, would be the most suitable for this
synthesis?

A PCl5, Cl2 in UV, aqueous NH3

trendyline
B Excess concentrated
o HNO3, LiAlH4
C CH3NH2, aqueous NH3
D Concentrated
trated
rated H2SO4, liquid Br2, ethanolic NH3

TPJC 9647/1/JC2/Prelim/16
1550 trendyline
9

26 Vitamin D exists in 2 forms, D2 and D3. The structure of vitamin D2 is shown below.

HO

Vitamin D2

How many stereoisomers does vitamin D2 have?

A 32 B 64 C 128 D 256

27 Which statement on protein is correct?

A Denaturation breaks down all the structures of the protein.

B Some examples of Rgroup interactions are ionic bonds, covalent bonds and
hydrogen bonds.

C Hydrogen bond in the secondary structure is formed between the O of the C=O
group and the H on the carbon.

D Heavy metal ions affect the hydrogen bonds in the proteins which will then lead to
denaturation.

28 Which reagent and conditions can be used to distinguish between the two compounds
below?
OH
H2N

OH

trendyline
OH
OH OH
A Aqueouss NaOH
NaO
B Acidified
d aqueous KMnO
KMn 4, heat
C Aqueous bromine

TPJC 9647/1/JC2/Prelim/16
1551 [Turn Over
trendyline
10

D Acidified aqueous K2Cr2O7, heat


29 Treatment of compound J with hot alcoholic KOH produces a mixture of compounds.
Which of the following compounds could be produced in this reaction?

Br
OH

Br

Compound J

Br
OH

B
A

C D

30 Which one of the following shows the given ions arranged in order of increasing basicity?

A CH3O-, CH3COO-, CH2(Cl)COO-


B CH3CH(Cl)CH2COO-, CH3CH2COO-, C6H5O-
C CH3CH(Cl)CH2COO-, CH3CH(Cl)COO-, CH3CH(F)COO-

COO- COO- COO-

, , Br

trendyline
TPJC 9647/1/JC2/Prelim/16
1552 trendyline
11

Section B

For each question in this part, one or more of the three numbered statements 1 to 3 may be
correct.

Decide whether each of the statements is or is not correct (you may find it helpful to put a
tick against the statement that you consider to be correct).

The responses A to D should be selected on the basis of

A B C D
1, 2 and 3 1 and 2 only 2 and 3 only 1 only
are correct are correct are correct is correct

No other combination of statements can be used as a correct response.

31 Which of the following diagrams correctly describes the behaviour of a fixed mass of an ideal
gas?

trendyline
TPJC 9647/1/JC2/Prelim/16
1553 [Turn Over
trendyline
12

The responses A to D should be selected on the basis of

A B C D
1, 2 and 3 1 and 2 only 2 and 3 only 1 only
are correct are correct are correct is correct

No other combination of statements can be used as a correct response.

32 The graph shows the first fifteen ionisation energies for element L.

What can be deduced about the element L from the graph?

1 It is in Period 3 of the Periodic Table.

2 It is in Group IV of the Periodic Table.

3 The oxide of L can react with hot concentrated NaOH.

33 The rate equation for the reaction between compounds M and N to give one product is
shown below.

Rate = k [M]2 [N]p

The expression for the unit of rate constant, k, is given as molq dmr s-1.

Which sets of values of p, q and r fit the information?

p q r
1 0 -1 +3
2 1 -2 +6
3 2 -3 +12

34 Use of the Data Booklet is relevant to this question.

Based on its position in the Periodic Table, which properties will element Te (atomic
number 52) have?

trendyline
1 Its oxide
e can react with water readily to form a strongly
stron acidic solution.
solutio

2 It will form
orm an oxide with formula T
Tee2O3.

3 It will form
orm
rm a chloride with formula T
TeCl
eCl5.

TPJC 9647/1/JC2/Prelim/16
1554 trendyline
13

The responses A to D should be selected on the basis of

A B C D
1, 2 and 3 1 and 2 only 2 and 3 only 1 only
are correct are correct are correct is correct

No other combination of statements can be used as a correct response.

35 In a fuel cell in which hydrazine (H2NNH2) comes into contact with fresh air on the
surface of a catalyst, the reactions taking place at the two electrodes are as follows.

Anode: H2NNH2 + 4OH 4H2O + N2 + 4e

Cathode: O2 + 2H2O + 4e 4OH

Which of the following statements are correct?

1 Nitrogen is oxidised at the anode.

2 Oxygen is reduced at the cathode.

3 The electrode potential of the cell becomes less positive as the concentration of
hydrazine increases.

36 Which of the following molecules are optically inactive?

3
Cl

trendyline
TPJC 9647/1/JC2/Prelim/16
1555 [Turn Over
trendyline
14

The responses A to D should be selected on the basis of

A B C D
1, 2 and 3 1 and 2 only 2 and 3 only 1 only
are correct are correct are correct is correct

No other combination of statements can be used as a correct response.

37 Myrcene can be extracted from sweet basil.

Mycrene

Which of the following options show the correct major product formed based on the
given reaction conditions?

Reaction Conditions Product

Cold, alkaline
1
KMnO4(aq)

2 Dry HBr(g)

tren
ndyline
n y in
Br2(aq)

TPJC 9647/1/JC2/Prelim/16
1556 trendyline
15

The responses A to D should be selected on the basis of

A B C D
1, 2 and 3 1 and 2 only 2 and 3 only 1 only
are correct are correct are correct is correct

No other combination of statements can be used as a correct response.

38 Which compounds give only one organic product when reacted with hot, acidified
KMnO4(aq)? Ignore optical isomers.

39 Dexamethasone could be administered to women at the risk of pre-term birth to reduce


the chances of complications in babies such as respiratory distress syndrome,
intraventricular haemorrhage and pre-natal death.

Dexamethasone

Which reagents will react with dexamethasone?

trendyline
1 Fehlings
s solution

2 2, 4DNPH
NPH
P

3 Red phosphorus
osphorus and bromine

TPJC 9647/1/JC2/Prelim/16
1557 [Turn Over
trendyline
16

The responses A to D should be selected on the basis of

A B C D
1, 2 and 3 1 and 2 only 2 and 3 only 1 only
are correct are correct are correct is correct

No other combination of statements can be used as a correct response.

40 Which of the following compounds would react with NaBH4?

--End of Paper--

trendyline
TPJC 9647/1/JC2/Prelim/16
1558 trendyline
Tampines Junior College 2016 Preliminary Examination
H2 Chemistry Paper 1 Worked Solution

1 2 3 4 5 6 7 8 9 10
C A B D B D B B D D
11 12 13 14 15 16 17 18 19 20
A A D B C A C D C A
21 22 23 24 25 26 27 28 29 30
C B C B D C B D A B
31 32 33 34 35 36 37 38 39 40
A C B D B D A D C B

Section A

1 Use of the Data Booklet is relevant to this question.

How many atoms are present in 1 cm3 of ethane gas under room conditions?

(8 x 24000) (8 x 6.02 x1023 )


A B
(6.02 x 1023 ) (22400)

(8 x 6.02 x1023 )
C D 6.02 x 1023 x 30
(24000)

Ans: C

1 cm3 of ethane is equals to mol of ethane molecule. There are 8 atoms per

ethane molecule, so the total amount of atoms is . The number of atoms

would then be x 6.02 x1023
2 Sodium percarbonate, x(Na2CO3).y(H2O2) is a chemical that is used in cleaning
powders. When dissolved in water, it releases hydrogen peroxide and sodium
carbonate, both useful cleaning agents.

A sample of 10.0 cm3 of 0.100 mol dm-3 sodium percarbonate requires 20.0 cm3 of
0.100 mol dm-3 of acidified KI before the end point of titration is reached.

Another identical sample releases 48.0 cm3 of carbon dioxide at room conditions on
reaction with excess acid.

What is the value of x and y respectively?

A 2 and 1 B 3 and 2
3 and 1
C 2 and 3 D

trendyline
Ans: A
Amount of sodium percarbonate = 10/1000 x 0.10 = 0.001 mol
m
As 1 mole of sodium percarbonate will produce y mole of H2O2 and x mole of
Na2CO3
Amount of H2O2 = 0.001y
001y mol
mo
Amount of Na2CO3 = 0.001x
0 001x mol

1559 trendyline
3

H2O2 + 2H+ + 2I- I2 + 2H2O


Amount of KI reacted = 20/1000 x 0.100 = 0.002 mol
Since KI: H2O2 is 2:1,
Amount of H2O2 = 0.001 mol, thus y = 1

Na2CO3 + 2H+ H2O + CO2 + 2Na+


Amount of carbon dioxide released = 48.0/24000 = 0.002 mol
Since Na2CO3: CO2 is 1:1
Amount of Na2CO3 = 0.002 mol, thus x =2
3 Which of the following molecules is non polar?

A AsBr3
B C2Cl4
C CH3CBr3
D H2S

Ans: B
AsBr3 is trigonal pyramidal in shape, there is a net dipole so it is polar. C2Cl4 is
trigonal planar in shape about each C. There is no net dipole as the dipole
moments cancels out each other. The shape about each C in CH3CBr3 is
tetrahedral but since the atoms connected to atoms with different
electronegativity, it is polar. H2S is bent in shape and thus there is a net dipole
moment.

4 Which of the following pairs does the first molecule have a larger bond angle than the
second molecule?

A PCl3, PBr3
B H2S, H2O
C NH3, BF3
D SnCl2, OCl2

Ans: D
PCl3, PBr3 both are trigonal pyramidal. As Cl is more electronegative than Br,
thus the bond pairs of electrons are further away from the central atom. There is
lesser repulsion between the bond pairs of electrons and thus bond angle is
smaller in PCl3.

H2S, H2O both are bent. As S is less electronegative than O, thus the bond pairs
of electrons are further away from the central atom. There is lesser repulsion
between the bond pairs of electrons and thus bond angle is smaller in H2S.

trendyline
NH3, BF3 NH
H3 is trigonal pyramidal and B
BF
F3 is
s trigonal planar. The
Th bond angles
are 107o and
nd 120o respectively.

SnCl2, OCl2 both are bent. SnC


SnCl
Cl2 has one lone pair of electrons wh
while OCl2has 2.
pulsion between the lone pair lon
Greater repulsion lone pair and lone pair b
bond pair of
electrons; therefore the bond angle in OCl2 will be smaller.

TPJC 9647/1/JC2/Prelim/16
1560 [Turn Over
trendyline
4

5 Which statement is not true about the structure of silicon dioxide, SiO2?

A The bond angle about Si is 109.5o.


B The bond angle about O is 180o.
C SiO2 has a giant molecular structure.
D SiO2 is insoluble in water.

Ans: B
The structure of SiO2 is as such

Black dot Si; Red dot - O

Si has 4 O atoms tetrahedrally attached to it and thus the bond angle is 109.5o. The
structure is similar to that of diamond (giant molecular) and is insoluble in water.
We can see that the bond angle about O is not 180o as it is bent due to the 2 lone
pair of electrons on O.

6 Use of the Data Booklet is relevant to this question.

Which one of the following species has the same number of neutrons and protons, and
more electrons than protons?

37
A Cl B 48
Ti4+ C 79
Br+ D 32
S2-

Ans: D
neutrons protons electrons
37
Cl 20 17 18
48 4+
Ti 26 22 18
79
Br+ 44 35 34
32 2-
S 16 16 18

trendyline
TPJC 9647/1/JC2/Prelim/16
1561 trendyline
5

7 Which statement regarding isotopes is incorrect?

A They have the same atomic number.


B They have the same neutron number.
C They have different mass number.
D They have different physical properties.

Ans: B
Isotopes are atoms with the same number of protons but that have a different
number of neutrons. Thus they cannot have the same neutron number.

8 Use of the Data Booklet is relevant to this question.

The standard enthalpy change of formation of C3H8(g) is 104 kJ mol1.

3C(s) + 4H2(g) 3C(g) + 8H(g)

C3H8(g)

What is the standard enthalpy change of atomisation of carbon, C(s)?

A 711 kJ mol-1 B +711 kJ mol-1


C 129 kJ mol-1 D +129 kJ mol-1

Ans: B 3x + 4(436)
3C(s) + 4H2(g) 3C(g) + 8H(g)

-104 2(350) + 8(410)

C3H8(g)
Besides 104 kJ mol-1, the other values are obtained from the Data Booklet. Using
Hess law and solving for x, we get option B.

trendyline
TPJC 9647/1/JC2/Prelim/16
1562 [Turn Over
trendyline
6

9 Hydrogen gas and fluorine gas react to form hydrogen fluoride as shown in the equation
below:
H2(g) + F2(g) 2HF(l) Ho = 542 kJ mol1

The standard entropy change of this reaction is 160 J K1 mol1.

Which statement is correct?

A At room temperature, the reaction is not spontaneous.


B An addition of catalyst will increase the yield of the product.
C There is an increase in the degree of disorder.
D The products are more stable than the reactants.
Ans: D
G = H TS. The value of G at room temperature = 542 (298)(160/1000)
= 494 kJ mol-1, since G < 0, it is spontaneous at room temperature. Adding a
catalyst only speeds up the rate of the reaction but does not increase the reaction
yield. The reaction has a decrease in the degree of disorder as 'S is negative (or
there is a decrease in gaseous molecules from 2 to 0). Only option D is correct as
the reaction is exothermic, the products contains less energy (more stable) than
the reactants.
10 For the reaction 3A(aq) + 2B(aq) C(aq) at room temperature, the rate equation is
Rate = k [H+][B]
Which statement is incorrect?

A The reaction involves more than one step.


B H+ is a catalyst in the above reaction.
C When the concentration of A is doubled at 10 oC, the rate of reaction decreases as
compared to room temperature.
D When the concentration of H+ and B is doubled, the amount of C formed is increased
four-fold.

Ans: D
The rate equation does not involve all the reactants, this can only mean that the
reaction is a multi-step reaction. A catalyst doesnt take part in the reaction and
should not be present in either the reactants or products. Option C is correct
because a decrease in temperature and doubling of the concentration of A (not
part of rate equation hence does not affect rate), the rate of reaction will decrease.
Option D is wrong because when both the species involved in the rate equation is
doubled, the rate increases 4-fold, not the amount of C formed.

trendyline
TPJC 9647/1/JC2/Prelim/16
1563 trendyline
7

11 The value of Kp of the reaction D2(g) + 2E2(g)  2DE2(g) is 3. Under the same reaction
conditions, what is the value of Kp for the reaction DE2(g) D2(g) + E2(g)?


A
B 3 C 1.5 D 3

Ans: A

D2(g) + 2E2(g) 2DE2(g), Kp = = 3, = . Taking the inverse
 

on both sides, = Kp = (Kp is the Kp of the 2nd equation)

12 What is the pH of the solution when 60 cm3 of 0.15 mol dm3 of CH3COOH and 80 cm3 of
0.10 mol dm3 CH3COONa+ are mixed at 25 oC?
(Ka of CH3COOH = 1.78 x 10-5 mol dm-3)

A 4.70 B 4.75 C 9.20 D 9.25

Ans: A
The above system is a buffer. We need to use pH = pKa + lg [salt]/[acid]. The new
total volume is 140 cm3 (60 + 80). Hence the new concentrations of the acid and
salt are 60/140 x 0.15 = 0.0643 mol dm-3 and 80/140 x 0.10 = 0.0571 mol dm-3
respectively. Hence pH = -lg (1.78 x 10-5) + lg (0.0571 / 0.0643) = 4.70

13 A 1.50 A current passed through a solution of osmium chloride, resulting in the reduction
of the metal ion. After 20 minutes, 0.888 g of osmium has been deposited. What is the
oxidation state of osmium in osmium chloride?

A +1 B +2 C +3 D +4
Ans: D

Q = It = 1.50 x 20 x 60 = 1800 C

Q = nF, n = 1800/96500 = 0.0187 mol

Amount of Os = 0.888/190 = 0.00467 mol

One mol of Os received 0.0187/0.00467 = 4 mol of electrons. Thus the oxidation


state of Os in osmium chloride is +4.

trendyline
TPJC 9647/1/JC2/Prelim/16
1564 [Turn Over
trendyline
8

14 Phosphorus trichloride and chlorine react according to the following equation:

PCl3(g) + Cl2(g) PCl5(g) 'Ho = +202 kJ mol1

A mixture of PCl3 and Cl2 was placed in a syringe and equilibrium was achieved. The
following graph was obtained. What could have happened at time t?

[PCl3]

t
A A catalyst was added to the mixture.
B The plunger was compressed by 5 cm3.
C The syringe was dropped in ice cold water.
D Cl2 was removed from the mixture.

Ans: B
At time t, the concentration of PCl3 increased rapidly, only option B is correct as
when the plunger was compressed, the volume decreased rapidly and hence the
concentration would increase rapidly. Option A is wrong as a catalyst does not
affect equilibrium position and hence change the concentrations of the reactants.
Options C would cause the equilibrium to shift to the right and thus decrease the
concentration of PCl3. For option D, although the equilibrium would shift to the
left, the increase would be gradual, not rapidly.
15 On strong heating, SrSO4 decomposes into SrO and SO3. The compound Sr(NO3)2
decomposes at a much higher temperature than SrSO4. Which statement best explains
the lower thermal stability of SrSO4?

A SrSO4 has a more exothermic lattice energy than Sr(NO3)2.


B NO2 is a smaller molecule than SO3.
C SO42 is more easily polarised than NO3.
D The charge on SO42 is greater than that on NO3.
Ans: C
nion is larger and more easily polarised, the electron cl
When the anion cloud is easier to
distort and
d thus the compound is easier to decompose
decompos as the co covalent bonds in
the anion are weakened to a greater extent.
extent
ent.
t The ease of decompo
decomposition explains
why the thermal stability of SrSO4 is low.

TPJC 9647/1/JC2/Prelim/16
1565 trendyline
9

16 Which property of the elements in the Period 3 (sodium to sulfur) decreases numerically
across the Period?

A pH of its oxides in water


B Ionic radius
C Melting points of its oxides
D 1st ionisation energy

Ans: A
The oxides changes from basic to acidic as we move across the period. Thus the
pH decreases across the period.

17 Radium is the last element in Group II in the Periodic Table.

Which statement is not true for radium?

A Its oxide dissolves in water to form a basic solution.


B It is the most reactive element in the group.
C The melting point of its chloride is lower to that of FrCl.
D It reacts with water to form hydrogen gas.
Ans: C
Melting point of ionic solids depends on their lattice energies. For RaCl and FrCl,
Ra ions has a higher charge (2+) and smaller size than Fr ions (1+), thus RaCl
would have a higher melting point.

18 The hexaaquairon(III) complex hydrolyses as shown.

[Fe(H2O)6]3+ [Fe(H2O)5OH]2+ + H+

Which of the following statements is true?

A There is a colour change from yellow to green.


B The hydrolysis is favoured by low pH values.
C The electronic configuration of Fe3+ changed from [Ar]3d5 to [Ar]3d6.
D [Fe(H2O)6]2+ is less acidic in solution as compared to [Fe(H2O)6]3+.

Ans: D
Fe3+ will polarise the O-H bond in water (or undergo cation hydrolysis) more than

trendyline
Fe2+ due too the higher charge. Thus H+ will bee generate
generated in a sm
smaller extent for
Fe2+ in water
er and hence
hence less acidic.

TPJC 9647/1/JC2/Prelim/16
1566 [Turn Over
trendyline
10

19 Vanadium(III) chloride reacts with carbon monoxide, CO, to form a compound in which
the coordination number of the vanadium complex is 6. A formula unit of one such
compound contains a cation and only one chloride ion.

What is the formula of this compound?

A V(CO)5Cl B V(CO)5Cl2
C V(CO)4Cl3 D V(CO)3Cl3

Ans: C
Since the compound only contains one chloride ion as the counter ion, the
Vanadium complex must have a 1+ charge. Taking into account that V is in a +3
oxidation state, the only correct option that allows for the complex to have 1+
charge and have 6 coordinate/dative bonds is C. (the complex is [V(CO)4Cl2]+)

20 In which reaction is the inorganic reagent acting as an electrophile?

Cl

A + Cl2 + HCl

Cl OH
B + KOH + KCl

+ Br2 Br
+ HBr
C

O
OH

D + HCN CN

Ans: A
The chlorine molecule in option A will be acting as the electrophile (benzene
undergoes electrophilic substitution). Other options, the inorganic reagents are
acting as the nucleophile (B and D) or radical (C).

trendyline
TPJC 9647/1/JC2/Prelim/16
1567 trendyline
11

21 Dopamine is a compound present in the body that functions as a neurotransmitter and a


precursor for other substances including adrenaline. It is also synthesized as a drug for
treatment of some neurodegenerative disease such as Parkinsons disease.

NH2
OH

O
HO
OH

Dopamine

Which statement about dopamine is not true?

A It is optically active.
B It can form a zwitterion in aqueous solution.
C It forms an orange precipitate with 2,4DNPH.
D One mole of dopamine forms an anion with 3 charge when reacted with 3 moles of
NaOH.

Ans: C
Option C is not correct as there is no carbonyl group (aldehyde or ketone) in
dopamine. Thus there is no reaction with 2,4 DNPH to form an orange
precipitate.

trendyline
TPJC 9647/1/JC2/Prelim/16
1568 [Turn Over
trendyline
12

22 Codeine is an opioid pain medication. It is used to treat mild to severe pain in patients.

H3CO

O H
N
H

HO

Codeine

(The COC and OCH3 groups do not undergo any reaction.)

Four students made claims about the reactions of codeine.

F 1 mol of codeine reacts with 2 mol of H2(g).

G 1 mol of codeine reacts with 1 mol of PCl5(s).

H 1 mol of codeine reacts with 2 mol of CH3CO2H(l) under suitable conditions.

I 1 mol of codeine reacts with 1 mol of dry HCl(g).

Which pair of students claims is correct?

A F and G B G and H
C H and I D F and I
Ans: B
Student F is incorrect as only the C=C would undergo electrophilic addition with
H2(g). Student G is correct as PCl5 undergoes nucleophilic substitution with R-OH.
Student H is correct as there are 2 functional groups that react with CH3COOH (-
OH (condensation) and NR3(acid-base neutralisation)). Student I is incorrect as
there is more than one functional group that reacts with the dry HCl(g)
( nucleophilic substitution with R-OH and electrophilic addition on C=C)

trendyline
TPJC 9647/1/JC2/Prelim/16
1569 trendyline
13

23 Which reaction scheme is unable to produce the intended final product?

COOH COCl COOCH2CH3


D
PCl5 CH3CH2OH

Ans: C
The first step is wrong as the dehydration step requires excess concentrated
sulfuric acid at 170oC rather than ethanolic KOH. So reaction C cannot take place
at all.

24 An organic compound has the following properties:


x It decolourises hot, acidified KMnO4(aq).
x It gives a yellow precipitate with aqueous alkaline iodine.
x It forms a silver mirror with Tollens solution.
Which compound could give these results?

A CH3COOCH2CH2COCH3
B CH3CH(OH)CH2CH2CHO
C HCOCH2CH2CH2CHO
D CH3CH2C(CH3)(OH)(CH2)2COCH3
Ans: B
x It decolourises hot, acidified KMnO4(aq) contains groups that can be oxidised
e.g. primary/secondary alcohols, aldehydes
x It gives a yellow precipitate with aqueous alkaline iodine contains the
C(CH3)(OH)H group (cannot be ketone as the first test indicates that it can be

trendyline
oxidised)
x It forms a silver mirror with Tollens solution aldehydes present.
Only B fulfils
fils
ls all the above criteria.
criteri

TPJC 9647/1/JC2/Prelim/16
1570 [Turn Over
trendyline
14

25
1,2ethanediamine, can be synthesised from ethanol through a
series of reactions.

Which set of reagents, used in sequential order, would be the most suitable for this
synthesis?

A PCl5, Cl2 in UV, aqueous NH3


B Excess concentrated HNO3, LiAlH4
C CH3NH2, aqueous NH3
D Concentrated H2SO4, liquid Br2, ethanolic NH3

Ans: D
Br NH2
OH Br H2N

26 Vitamin D exists in 2 forms, D2 and D3. The structure of vitamin D2 is shown below.

HO

Vitamin D2

How many stereoisomers does vitamin D2 have?

A 32 B 64 C 128 D 256

Ans: C
The chiral carbons are marked with an asterisk. There is a total of 6 chiral
carbons. And there is also one C=C bond (circled in the diagram below) that can
exhibit cis-trans isomerism. Thus the total number of stereoisomers is 26+1 = 128.

trendyline
TPJC 9647/1/JC2/Prelim/16
1571 trendyline
15

27 Which statement on protein is correct?

A Denaturation breaks down all the structures of the protein.

B Some examples of Rgroup interactions are ionic bonds, covalent bonds and
hydrogen bonds.

C Hydrogen bond in the secondary structure is formed between the O of the C=O
group and the H on the carbon.

D Heavy metal ions affect the hydrogen bonds in the proteins which will then lead to
denaturation.

Ans: B
The R-group interactions are hydrogen bonding, ionic bonding, Van der Waals
interactions and disulfide linkages. The covalent bonds mentioned is the disulfide
linkages.

28 Which reagent and conditions can be used to distinguish between the two compounds
below?
OH
H2N

OH OH
OH OH
A Aqueous NaOH
B Acidified aqueous KMnO4, heat
C Aqueous bromine
D Acidified aqueous K2Cr2O7, heat

Ans: D
K2Cr2O7 is a weaker oxidising agent and is capable of oxidising the 2o alcohol in
the molecule on the right where a colour change from orange to green would be
observed.

KMnO4 cannot be used as the both compounds will react to give benzoic acid.

trendyline
TPJC 9647/1/JC2/Prelim/16
1572 [Turn Over
trendyline
16

29 Treatment of compound J with hot alcoholic KOH produces a mixture of compounds.


Which of the following compounds could be produced in this reaction?

Br
OH

Br

Compound J

Br
OH

B
A

C D

Ans: A
J undergoes dehydrohalogenation (elimination) with hot alcoholic KOH. (remove
HBr). Only option A can be formed. For B, C and D, the OH should still be on the
molecule.

30 Which one of the following shows the given ions arranged in order of increasing
basicity?

A CH3O-, CH3COO-, CH2(Cl)COO-


B CH3CH(Cl)CH2COO-, CH3CH2COO-, C6H5O-
C CH3CH(Cl)CH2COO-, CH3CH(Cl)COO-, CH3CH(F)COO-

COO- COO- COO-

, , Br

Ans: B

trendyline
aker an acid, the stronger the conjugate base. So in optio
The weaker

with electron
ctron group, carboxylic acid, phenol)
tron withdrawing group, phenol
option B, the acid
that the salts were derived from becomes progressive weaker. (carboxylic
(c acid

TPJC 9647/1/JC2/Prelim/16
1573 trendyline
17

Section B

For each question in this part, one or more of the three numbered statements 1 to 3 may be
correct.

Decide whether each of the statements is or is not correct (you may find it helpful to put a
tick against the statement that you consider to be correct).

The responses A to D should be selected on the basis of

A B C D
1, 2 and 3 1 and 2 only 2 and 3 only 1 only
are correct are correct are correct is correct

No other combination of statements can be used as a correct response.

31 Which of the following diagrams correctly describes the behaviour of a fixed mass of an ideal
gas?

Ans: A

trendyline
All 3 options
ns are correct. Using pV
we plot a graph
pV = nRT as

we plot p against 1/T, y = 1/x type curve


curve.
as the equation. For 1
raph of p against V, y = 1/x type curve.
curve For 2
e. For
F 3 3, p
pV
1,, we have p = nRT/V, so if
2, we have rreciprocal graph when
V = nRT which is a constant for any
values of V, thus we get y = c type line, where c is a constant.

TPJC 9647/1/JC2/Prelim/16
1574 [Turn Over
trendyline
18

32 The graph shows the first fifteen ionisation energies for element L.

What can be deduced about the element L from the graph?

1 It is in Period 3 of the Periodic Table.

2 It is in Group IV of the Periodic Table.

3 The oxide of L can react with hot concentrated NaOH.

Ans: C
There is a big jump in the ionisation energy between the 4th and 5th electron. This
means that there were 4 valence electrons in the outermost shell (Group IV).
Being in Group IV, the oxides will be acidic and thus react with a base.

33 The rate equation for the reaction between compounds M and N to give one product is
shown below.

Rate = k [M]2 [N]p

The expression for the unit of rate constant, k, is given as molq dmr s-1.

Which sets of values of p, q and r fit the information?

p q r
1 0 -1 +3
2 1 -2 +6
3 2 -3 +12

Ans: B

Rate = k [M]2 [N]p


k = rate/ [M]2 [N]p

the units of k can be worked out as shown.

ttrendyline


Simplifying
g the equation and comparing with the given units of k, we get
q = p 1 and
nd r = 3 + 3p. Looking at the options, only 1 and 2 gi
give the correct
values of each of the unknowns.

TPJC 9647/1/JC2/Prelim/16
1575 trendyline
19

34 Use of the Data Booklet is relevant to this question.

Based on its position in the Periodic Table, which properties will element Te (atomic
number 52) have?
1 Its oxide can react with water readily to form a strongly acidic solution.

2 It will form an oxide with formula Te2O3.

3 It will form a chloride with formula TeCl5.

Ans: D
Te is in group VI, same as O and S. As such, we would expect the properties to
have some similarities to them. Statement 1 only is correct as with S, Te would
react with water to form an acidic oxide in solution. Similar to S, Te would form
oxides with formula of TeO2 or TeO3. Its chloride formula is TeCl4 or TeCl6.

35 In a fuel cell in which hydrazine (H2NNH2) comes into contact with fresh air on the
surface of a catalyst, the reactions taking place at the two electrodes are as follows.

Anode: H2NNH2 + 4OH 4H2O + N2 + 4e

Cathode: O2 + 2H2O + 4e 4OH

Which of the following statements are correct?

1 Nitrogen is oxidised at the anode.

2 Oxygen is reduced at the cathode.

3 The electrode potential of the cell becomes less positive as the concentration of
hydrazine increases.

Ans: B

Both statements 1 and 2 are correct. The oxidation number of N changes from -2
to 0 at the anode (oxidation). The oxidation number of O changes from 0 to -2 at
the cathode (reduction). Statement 3 is wrong because the potential should
increase (become more positive) when the concentration of hydrazine increases.

trendyline
TPJC 9647/1/JC2/Prelim/16
1576 [Turn Over
trendyline
20

36 Which of the following molecules are optically inactive?

3
Cl

Ans: D
A compound with chiral carbons isx optically inactive if the molecule is
symmetrical. For the 1st compound, there is such a mirror plane, which cancels
out the optical activity of each of the individual chiral C.

trendyline
TPJC 9647/1/JC2/Prelim/16
1577 trendyline
21

37 Myrcene can be extracted from sweet basil.

Mycrene

Which of the following options show the correct major product formed based on the
given reaction conditions?

Reaction Conditions Product

Cold, alkaline
1
KMnO4(aq)

2 Dry HBr(g)

3 Br2(aq)

trendyline
Ans: A
The firstt reaction is a mild oxidation reaction; an alkene
a function group
functional
becomes s a diol. The second reaction is an electrophilic addition reaction,
n of H and Br across the alkene double bond (major produ
addition product shown),
and the 3rdd reaction is another electrophilic addition reaction of OH
O and Br
across the C=C (major product shown))

TPJC 9647/1/JC2/Prelim/16
1578 [Turn Over
trendyline
22

38 Which compounds give only one organic product when reacted with hot, acidified
KMnO4(aq)? Ignore optical isomers.

Ans: D

After undergoing oxidation with hot, acidified KMnO4(aq), the products formed
O

OH
OH

for is O . For , O and


O OH

. For compound 2, the compound formed is HO O , which


decomposes to form carbon dioxide and water. Hence only compound 1 gives 1
organic compound after reaction.

trendyline
TPJC 9647/1/JC2/Prelim/16
1579 trendyline
23

39 Dexamethasone could be administered to women at the risk of pre-term birth to reduce


the chances of complications in babies such as respiratory distress syndrome,
intraventricular haemorrhage and pre-natal death.

Dexamethasone

Which reagents will react with dexamethasone?


1 Fehlings solution

2 2, 4DNPH

3 Red phosphorus and bromine

Ans: C

Fehlings solution reacts with aldehyde functional groups only, which is not
present in dexamethasone. 2,4DNPH reacts with carbonyl functional groups
while red phosphorus and bromine (forms PBr3) reacts with alcohol functional
groups via nucleophilic substitution reaction to form RBr.

trendyline
TPJC 9647/1/JC2/Prelim/16
1580 [Turn Over
trendyline
24

40 Which of the following compounds would react with NaBH4?

Ans: B
NaBH4 reduces aldehydes and ketones only. Functional groups in the 3rd compound
(carboxylic acid and alkene) do not undergo reduction with NaBH4. (They can only be
reduced by LiAlH4 and H2/Pd respectively)

--End of Paper--

trendyline
TPJC 9647/1/JC2/Prelim/16
1581 trendyline
2

Tampines Junior College 2016 Preliminary Examination


H2 Chemistry Paper 2
1 Planning (P)

The solubility of a salt is determined by the amount of salt that can dissolve in a given volume
of water at a specific temperature. If the solubility is less than 104 mol dm3 of water, the salt
is said to be sparingly soluble.

(a) Barium carbonate is a sparingly soluble salt. Write the Ksp expression for barium
carbonate, stating its units.

.[1]

(b) Solubility can also be defined as the mass of a substance that can dissolve in
100 cm3 of water at a specific temperature. In this experiment, you are tasked to write a
detailed plan for an experiment on how you could determine the solubility of barium
carbonate, in grams per 100 cm3 of water at 25 oC.

You are provided with the following:


x Solid barium carbonate
x Deionised water
x Evaporating dish
x Oven
x Filter Paper
x Water bath
x And other apparatus normally found in a school laboratory

No other chemicals may be used in your plan.

Your plan should contain the following:


x determining the minimum mass of barium carbonate to be used
x appropriate procedure to measure a suitable mass of barium carbonate
x appropriate procedure and apparatus used for the dissolution process
x determining the mass of barium carbonate dissolved
x appropriate table to record measured data
x simple calculations on how the solubility can be determined

........

........

........

........

trendyline


........


........

........

TPJC 9647/2/JC2/Prelim/16
1582 trendyline
3

........

........

........

........

........

........

........

........

........

........

........

........

........

........

........

........

........

........

........

........

........

........

........

........

........

trendyline


........



........

........

TPJC 9647/2/JC2/Prelim/16 [Turn Over


1583 trendyline
4

........

........

........

........

........

........

........

........

........

........

........

........

........

........

........

........

.[9]

(c) Describe one major source of error for the experiment and suggest an improvement
which could significantly increase the accuracy of the experiment.

........

........

........

........

........

trendyline
....[2]

[Total: 12]

TPJC 9647/2/JC2/Prelim/16
1584 trendyline
5

2 Ammonia, NH3, is a colourless gas with a sharp and distinct odour. Made by both humans
and nature, ammonia serves as a starting material for the production of many commercially
important nitrogen compounds.

(a) (i) Ammonia can be produced by heating ammonium chloride with calcium hydroxide.
The reaction also produces a white solid. Write a balanced equation for this
reaction.

...[1]

(ii) One such synthesis in (a)(i) produced 200 dm3 of ammonia at 400 oC and
50 kPa. Calculate the mass of ammonium chloride that was used in the reaction.

[2]

(iii) Under high temperature, ammonia behaves as if there are no intermolecular


forces of attraction between molecules. Explain why this is so.

...[1]

(b) The Haber process uses atmospheric nitrogen and hydrogen to produce ammonia gas
in the presence of an iron catalyst. The equilibrium to represent the Haber process is as
follows:

'H < 0

trendyline
3H2(g) + N2(g) 2NH3(g)

(i) State
ate
e the
e reaction conditions used for the Haber process.



...[1]

TPJC 9647/2/JC2/Prelim/16 [Turn Over


1585 trendyline
6

(ii) Explain for your chosen temperature in (b)(i).

...[2]

(iii) The iron catalyst used in the Haber process behaves as a heterogeneous catalyst.
Explain what is meant by the term heterogeneous catalyst.

[1]

(iv) Explain which feature of iron allows it to act as an appropriate catalyst for the
Haber process.

...[1]

(c) When an equimolar mixture of gaseous ammonia and gaseous hydrogen bromide at an
initial total pressure of 4 atm were injected into a chamber that was maintained at
450 oC, white crystals of ammonium bromide were formed as shown in the following
equilibrium:

NH3(g) + HBr(g) NH4Br(s) 'H > 0

It was found that the value of the equilibrium constant for the forward reaction,
Kp, at 450 oC was 10.0.

(i) Write down an expression for Kp.

trendyline [1]

TPJC 9647/2/JC2/Prelim/16
1586 trendyline
7

(ii) Calculate the value of the equilibrium constant for the reverse reaction,
Kp, at 450 oC, stating its units.

[1]

(iii) Calculate the pressure exerted by hydrogen bromide at equilibrium.

[3]

(iv) State and explain how the partial pressure of the gases and the value of Kp would
change if the temperature of the chamber was 600 oC.

trendyline


.. [2]

[Total: 16]

TPJC 9647/2/JC2/Prelim/16 [Turn Over


1587 trendyline
8

3 (a) Draw a dot and cross diagram to show the bonding in a phosphate(V) ion, PO43.

[1]
(b) State the expected bond angle in a phosphate(V) ion.

...[1]

(c) (i) The following information involving potassium phosphate, K3PO4 is shown below.

Enthalpy change of hydration for K+(g) 320 kJ mol1


Enthalpy change of hydration for PO43(g) 290 kJ mol1
Enthalpy change of solution for K3PO4(s) +23.0 kJ mol1

Write an equation to represent the enthalpy change of solution for potassium


phosphate.

.[1]

(ii) Using the enthalpy values given in (c)(i), draw an appropriate energy level
diagram and hence, determine the lattice energy of potassium phosphate.

[3]

(iii) State and explain if the numerical value of the lattice energy of calcium phosphate
is higher or lower than the lattice energy of potassium phosphate.
.

trendyline
.


.[1]

[Total: 7]

TPJC 9647/2/JC2/Prelim/16
1588 trendyline
9

4 (a) The properties of Period 3 elements show a periodic trend. These elements occur in
nature and have at least one stable isotope.

(i) Sketch a graph to show the trend of atomic size across Period 3 from sodium to
chlorine. Explain the trend observed.

[3]

(ii) State and explain the difference between the first ionisation energies of sulfur and
phosphorus.

trendyline

[2]

TPJC 9647/2/JC2/Prelim/16 [Turn Over


1589 trendyline
10

(b) Chlorate(V) ions, ClO3, are powerful oxidisers which are commonly used in
pyrotechnics and fertilizers.

In an experiment, it was found that 31.0 cm3 of ClO3 reacts exactly with 25.8 cm3 of
0.030 mol dm3 of lead(II) ions to form chlorine gas and lead(IV) ions.

Write an overall equation for the reaction and hence, calculate the concentration of
ClO3 required.

[3]

(c) Chlorate(V) ions can also be formed by reacting chlorine gas with NaOH(aq). State the
reaction conditions required and write an equation for the transformation.

..

..

..[2]

[Total: 10]

trendyline
TPJC 9647/2/JC2/Prelim/16
1590 trendyline
11

5 FeCl2(aq) underwent a series of reactions as shown below.

(a) Write down the electronic configuration for the Fe2+ ion.

..[1]

(b) State the colour observed for FeCl2(aq).

..[1]

(c) State the identities of A D.

A:

B:

C:

D: [4]

(d) The Kstab of C is less than the Kstab of D. Account briefly for the difference in Kstab values.

.....

.....

......[1]

[Total: 7]

trendyline
TPJC 9647/2/JC2/Prelim/16 [Turn Over
1591 trendyline
12

6 A reaction scheme involving aromatic molecules is shown below.

(a) State the reagents and conditions for steps 1-4 of the reaction scheme above.

Step 1:

Step 2:

Step 3:

Step 4: .......[4]

(b) Draw the structure of the other possible isomer that can be obtained from step 1.

trendyline [1]

TPJC 9647/2/JC2/Prelim/16
1592 trendyline
13

(c) When compound F is placed in water, an acidic solution of pH 2 is formed. Write an


equation to account for the observation.

[1]

(d) Draw the structure of the organic products formed when compound G reacts with
hot NaOH(aq).

[2]

(e) Suggest a simple chemical test which would allow you to distinguish between
compound G and compound H.

trendyline




...[2]

[Total: 10]

TPJC 9647/2/JC2/Prelim/16 [Turn Over


1593 trendyline
14

7 The structure of silk, a natural protein fibre, have been the subject of much research in recent
years. The structure of silk fibre is composed of protein chains, which mainly consists of
glycine, alanine and serine amino acids.

Amino Acid Structure


Glycine (gly) NH2CH2COOH
Alanine (ala) NH2CH(CH3)COOH
Serine (ser) NH2CH(CH2OH)COOH

(a) (i) Draw the structural formula of the tripeptide ala-gly-ser.

[1]

(ii) Suggest how many other tripeptides can be formed by the three amino acids in
(a)(i) (each amino acid can only be used once).

.[1]

(iii) With the aid of a diagram, suggest how the tertiary structure can be stabilised with
a polypeptide chain consisting only of these three amino acids.

[2]

(iv) Hence explain if a change in pH would denature the tertiary structure described in
(a)(iii).

trendyline


[1]

TPJC 9647/2/JC2/Prelim/16
1594 trendyline
15

(b) (i) Amino acids can act as a buffer in water. Define the term buffer.

[1]

(ii) Write equations to represent how a solution of serine can behave as a buffer when
a small amount of acid and base is separately added.

[2]

(c) Serine can also react with chloroethane under suitable conditions in a 1:1 mole ratio.
Draw the structure of the organic compound produced and state the type of reaction
involved.

[2]

[Total: 10]

---End of Paper---

TPJC 9647/2/JC2/Prelim/16 [Turn Over


1595 trendyline
1
For
Tampines Junior College 2016 Preliminary Examination Examiners
H2 Chemistry Paper 2 Solution Use

1 Planning (P)

The solubility of a salt is determined by the amount of salt that can dissolve in a given
volume of water at a specific temperature. If the solubility is less than 104 mol dm3 of
water, the salt is said to be sparingly soluble.

(a) Barium carbonate is a sparingly soluble salt. Write the Ksp expression for barium
carbonate, stating its units.
[1]
2+ 2 2 6
Ksp = [Ba ] [CO3 ] mol dm

(b) Solubility can also be defined as the mass of a substance that can dissolve in
100 cm3 of water at a specific temperature. In this experiment, you are tasked to write
a detailed plan for an experiment on how you could determine the solubility of barium
carbonate, in grams per 100 cm3 of water at 25 oC.

You are provided with the following:


x Solid barium carbonate
x Deionised water
x Evaporating dish
x Oven
x Filter Paper
x Water bath
x And other apparatus normally found in a school laboratory

No other chemicals may be used in your plan.

Your plan should contain the following:


x determining the minimum mass of barium carbonate to be used
x appropriate procedure to measure a suitable mass of barium carbonate
x appropriate procedure and apparatus used for the dissolution process
x determining the mass of barium carbonate dissolved
x appropriate table to record measured data
x simple calculations on how the solubility can be determined

[9]
3 -4 -3
1. Solubility of sparingly soluble salt in 100 cm = 10 x 10 = 10 mol per
100cm3
Minimum mass of BaCO3 to be used = 10-3 x 197 = 0.197 g
2. Using an electronic balance, weigh about 1.00 1.50 g of BaCO3 into the

trendyline
ne
weighing bottle.
3. Use a 50 cm3 burette to measure 100 .00 cm3 of deionised water and
100.00
transfer
ansfer the water into a clean, dry beaker.
ransfer
4. Place into a water bath maintained at 25 oC [1 for step 3&4]
lace the beaker in
5. Add the barium carbonate into the water and stir the solu
solution with the
glass rod until the solid does not dissolve anymore.

2016 JC2 H2 Chem/ SA2 Exam/ Paper 2 [Turn Over


1596 trendyline
6. Weigh the mass of a piece of filter paper. Filter the mixture using the
filter paper.
7. Transfer the filter paper + residue onto an evaporating dish and place the
dish into an oven to dry.
8. (After 10 minutes, take the evaporating dish out of the oven and let it
cool down in a desiccator.)
9. Using an electronic balance, weigh the residue with the filter paper.
10. Repeat steps 7-9 until a constant mass is obtained.
11. Repeat steps 1 10 with another similar sample.

Table
Experiment 1 Experiment 2
Mass of empty weighing m1 m'1
bottle
Mass of weighing bottle + m2 m'2
barium carbonate
Mass of barium carbonate m3 = m2 m 1 m'3 = m'2
before dissolution m'1

Mass of filter paper m4 m'4


Mass of filter paper + m5 m'5
excess barium carbonate
residue
Mass of excess barium m6 = m5 m 4 m'6 = m'5
carbonate residue m'4

Mass of barium carbonate m7 = m3 m 6 m'7 = m'3 -


dissolved m'6

Calculations:
Solubility of barium carbonate = (m7 + m7) / 2 = M g per 100 cm3

(c) Describe one major source of error for the experiment and suggest an improvement
which could significantly increase the accuracy of the experiment.
[2]
Heat loss to surrounding can lead to the water baths temperature being
inconsistent.
We can improve this by using an electric water bath which can help us to
maintain a consistent temperature.
or
Ensure consistency when stirring to allow maximum amount of barium
carbonate before filtering.
We can improve by using a magnetic stirrer.

trendyline
or
ature of the reaction should remain at 25 oC durin
Temperature during
durin the filtration
process.
s.
We can conduct the experiment in a laboratory with a thermosta
thermostat.

[Total: 12]

1597 trendyline
3
For
2 Ammonia, NH3, is a colourless gas with a sharp and distinct odour. Made by both humans Examiners
and nature, ammonia serves as a starting material for the production of many commercially Use
important nitrogen compounds.

(a) (i) Ammonia can be produced by heating ammonium chloride with calcium
hydroxide. The reaction also produces a white solid. Write a balanced equation
for this reaction.
[1]
2NH4Cl + Ca(OH)2 2NH3 + CaCl2 + 2H2O

(ii) One such synthesis in (a)(i) produced 200 dm3 of ammonia at 400 oC and
50 kPa. Calculate the mass of ammonium chloride that was used in the
reaction.
[2]
In order to solve for the amount of ammonia gas,
PV = nRT
(50 x 103) x (200 x 10 3) = n x 8.31 x (400+273)
n = 1.788 mol

Since 2NH4Cl : 2NH3 (allow ecf)


Amount of NH4Cl = 1.788 mol
Mass of NH4Cl = 1.788 x (14 + 4 + 35.5)
= 95.7 g

(iii) Under high temperature, ammonia behaves as if there are no intermolecular


forces of attraction between molecules. Explain why this is so.
[1]
At a higher temperature, particles have greater (kinetic) energy. Thus with
sufficient kinetic energy, particles can overcome the existing
intermolecular attractions (or hydrogen bonding).

(b) The Haber process uses atmospheric nitrogen and hydrogen to produce ammonia
gas in the presence of an iron catalyst. The equilibrium to represent the Haber
process is as follows:

3H2(g) + N2(g) 2NH3(g) 'H < 0

(i) State the reaction conditions used for the Haber process.
[1]
o
Conditions: 200 atm, 450 500 C, (finely divided iron)
(ii) Explain for your chosen temperature in (b)(i).
[2]
According to Le Chateliers Principle, the use of a low temperature will

ttrendyline
trend
dyli
favour
vour
our the exothermic reaction,
reaction, hence equil
equilibrium will shift to the right (or
forward
rward reaction is favoured),
favoured), and result in a high yield of N
NH3.
nce, a moderate temperature of 450oC is chosen so tthat the rate of
Hence,
production
oductio
ducti n of NH3 will
w notno be too slow.
slow

2016 JC2 H2 Chem/ SA2 Exam/ Paper 2 [Turn Over


1598 trendyline
(iii) The iron catalyst used in the Haber process behaves as a heterogeneous
catalyst. Explain what is meant by the term heterogeneous catalyst.

[1]

Catalyst and reactants are in different states/phases.


(iv) Explain which feature of iron allows it to act as an appropriate catalyst for the
Haber process.

[1]

In Fe, there are available 3d and 4s electrons and empty d orbitals which
allows for the ready adsorption (through the formation of bonds) of
reactant molecules onto the catalyst surface.

(c) When an equimolar mixture of gaseous ammonia and gaseous hydrogen bromide at
an initial total pressure of 4 atm were injected into a chamber that was maintained at
450 oC, white crystals of ammonium bromide were formed as shown in the following
equilibrium:

NH3(g) + HBr(g) NH4Br(s) 'H > 0

It was found that the value of the equilibrium constant for the forward reaction, Kp, at
450 oC was 10.0.
(i) Write down an expression for Kp.
[1]
Kp = 1
(PNH3)(PHBr)

(ii) Calculate the value of the equilibrium constant for the reverse reaction, Kp, at
450 oC, stating its units.
[1]
Kp = 1 / 10.0 = 0.100 atm2

(iii) Calculate the pressure exerted by hydrogen bromide at equilibrium.


[3]
Let x be the partial pressure of NH3 reacted.
NH3 HBr
Initial partial pressure /atm 2 2
Change partial pressure /atm x x
Equilibrium partial pressure /atm 2x 2x

Kp = 1
(PNH3)(PHBr)

10.0 = 1 / (2 x)2

trendyline x))2
0.1 = (2 x
Solving
Solving for x,x,

x = 1.68 atm or 2.32


32 atm
a (rejected as x cannot
cann be > 2)

Equilibrium pressure of HBr = 2 1.68 = 0.32 atm

1599 trendyline
5
For
(iv) State and explain how the partial pressure of the gases and the value of Kp Examiners
would change if the temperature of the chamber was 600 oC. Use
[2]
When temperature increases, according to Le Chateliers Principle, the
system would respond by decreasing the temperature system, favouring
the endothermic reaction.

Hence favours the forward reaction, equilibrium shifts right, partial


pressure of the gases decreases and the value of Kp increases.

[Total: 16]

3 (a) Draw a dot and cross diagram to show the bonding in a phosphate(V) ion, PO43.
[1]

(b) State the expected bond angle in a phosphate(V) ion.


[1]
Angle: 109.5o
(c) (i) The following information involving potassium phosphate, K3PO4 is shown
below.

Enthalpy change of hydration for K+(g) 320 kJ mol1


Enthalpy change of hydration for PO43(g) 290 kJ mol1
Enthalpy change of solution for K3PO4(s) +23.0 kJ mol1

Write an equation to represent the enthalpy change of solution for potassium


phosphate.
[1]
K3PO4(s) 3K+(aq) + PO43-(aq)
(ii) Using the enthalpy values given in (c)(i), draw an appropriate energy level
diagram and hence, determine the lattice energy of potassium phosphate.
[3]

2016 JC2 H2 Chem/ SA2 Exam/ Paper 2 [Turn Over


1600 trendyline
Lattice Energy = (3 x 320) + ( 290) 23 = 1273 kJ mol1
1270 kJ mol1 (3.s.f)

(iii) State and explain if the numerical value of the lattice energy of calcium
phosphate is higher or lower than the lattice energy of potassium phosphate.
The numerical value of lattice energy of calcium phosphate would be
higher. Since the phosphate anion is the same for both compound,
calcium ions has a higher charge of 2+ and smaller radius as compared to
that of potassium ions 1+ charge and larger radius.
[Total: 7]

4 (a) The properties of Period 3 elements show a periodic trend. These elements occur in
nature and have at least one stable isotope.

(i) Sketch a graph to show the trend of atomic size across Period 3 from sodium to
chlorine. Explain the trend observed.
[3]

trendyline
(Period 3 Elements/ Proton Number)

Labelling
b lli off atomic
t i radius
di on y axis,
i and d th
the respective
ti elements on the
x axis. (Atomic radius values are not required). Graph shows a decreasing

1601 trendyline
7
For
trend. Examiners
Use
Across the Period, nuclear charge increases as the number of protons
increase and shielding effect remains similar as electrons are being added
to the same quantum shell.

Hence, valence electrons are more strongly attracted to the nucleus.


Atomic radius decreases across the period.

(ii) State and explain the difference between the first ionisation energies of sulfur
and phosphorus.
[2]
Inter-electronic repulsion occurs between the paired electrons in the 3p
orbital in sulfur.

Therefore, less energy is required to remove a (paired) 3p electron from


sulfur than (an unpaired electron) from phosphorous, hence sulfur has a
lower 1st ionisation energy than phosphorus.

(b) Chlorate(V) ions, ClO3, are powerful oxidisers which are commonly used in
pyrotechnics and fertilizers.

In an experiment, it was found that 31.0 cm3 of ClO3 reacts exactly with 25.8 cm3 of
0.030 mol dm3 of lead(II) ions to form chlorine gas and lead(IV) ions respectively.

Write an overall equation for the reaction and hence, calculate the concentration of
ClO3 required.
[3]
2+ 4+
Oxidation: Pb Pb + 2e
Reduction: 2ClO3 + 12H+ + 10e Cl2 + 6H2O [1 for reduction half equation]

Overall Equation: 2ClO3 + 12H+ + 5Pb2+ o Cl2 + 6H2O + 5Pb4+

Amount of lead (II) ions = 25.8/1000 x 0.030 = 7.74 x 104 mol

2ClO3 5Pb2+

Amount of ClO3 = (2/5) x 7.74 x 104 = 3.096 x 104 mol

Concentration of ClO3 = 3.096 x 104 / 31 x 103 = 9.99 x 103 mol dm3


(3 s.f. with units)

(c) Chlorate(V) ions can also be formed by reacting chlorine gas with NaOH(aq). State
the reaction conditions required and write an equation for the transformation.

trendyline
[2]
Reactionn conditions: Hot

Hot: 3Cl2((aq)) + 6NaOH(aq)


6N OH( o 5NaCl(aq) + NaClO3(aq)) + 3H2O
O(l)

[Total:10]

2016 JC2 H2 Chem/ SA2 Exam/ Paper 2 [Turn Over


1602 trendyline
5 FeCl2(aq) underwent a series of reactions as shown below.

(a) Write down the electronic configuration for the Fe2+ ion.
[1]
2 2 6 2 6 6
1s 2s 2p 3s 3p 3d
(b) State the colour observed for FeCl2(aq).
[1]
FeCl2(aq) is a green solution.
(c) State the identities of A D.
[4]
A: Fe(OH)2
B: Fe(OH)3
C: [Fe(CN)6]3
D: [Fe(edta)]
(d) The Kstab of C is less than the Kstab of D. Account briefly for the difference in Kstab
values.
[1]
4-
D has one polydentate/hexadentate ligand(or edta ) is generally more stable(or
stronger) than C which consist of 6 monodentate ligands (or CN-).
[Total: 7]

trendyline
1603 trendyline
9
For
6 A reaction scheme involving aromatic molecules is shown below. Examiners
Use

(a) State the reagents and conditions for steps 1-4 of the reaction scheme above.
[4]
Step 1: Add dilute HNO3, rtp
Step 2: Add acidified KMnO4(aq), heat
Step 3: Add PCl3 or PCl5 or SOCl2, rtp
Step 4: Add phenol, rtp

(b) Draw the structure of the other possible isomer that can be obtained from step 1.
[1]

(c) When compound F is placed in water, an acidic solution of pH 2 is formed. Write an


equation to account for the observation.

[1]

tre
trendyline
rendylin
in
ne
ne
2016 JC2 H2 Chem/ SA2 Exam/ Paper 2 [Turn Over
1604 trendyline
(d) Draw the structure of the organic products formed when compound G reacts with
hot NaOH(aq).
[2]

(e) Suggest a simple chemical test which would allow you to distinguish between
compound G and compound H.
[2]

Neutral FeCl3. A purple solution would be observed for Compound G but no


purple solution would be observed for Compound H.

or Br2(aq). The reddish brown solution of Br2(aq) will decolourize in Compound


G but will not decolourize in Compound H.

or Na(s) effervescence will be observed in Compound G but there will not be


effervescence in Compound H.
[Total: 10]

trendyline
1605 trendyline
11
For
7 The structure of silk, a natural protein fibre, have been the subject of much research in Examiners
recent years. The structure of silk fibre is composed of protein chains, which mainly Use
consists of glycine, alanine and serine amino acids.

Amino Acid Structure


Glycine (gly) NH2CH2COOH
Alanine (ala) NH2CH(CH3)COOH
Serine (ser) NH2CH(CH2OH)COOH

(a) (i) Draw the structural formula of the tripeptide ala-gly-ser.


[1]

(ii) Suggest how many other tripeptides can be formed by the three amino acids in
(a)(i) (each amino acid can only be used once).
[1]
5
(iii) With the aid of a diagram, suggest how the tertiary structure can be stabilised
with a polypeptide chain consisting only of these three amino acids.
[2]

or van der Waals between appropriate Rgroups


or hydrogen bonding between 2 ser Rgroups
(iv) Hence explain if a change in pH would denature the tertiary structure described
in (a)(iii).
[1]

ttren
endyline
i
No it would not as a change in pH would only disrupt io ionic interactions
between
tween the R
Rgroups.
Rgroups
groups..
or
No as there are no ionic
ionic interactions between the R
R
Rgroups.
groups

2016 JC2 H2 Chem/ SA2 Exam/ Paper 2 [Turn Over


1606 trendyline
(b) (i) Amino acids can act as a buffer in water. Define the term buffer.
[1]
A buffer solution is a solution that is able to resist drastic change in pH
when a small amount of acid or base is added.

(ii) Write equations to represent how a solution of serine can behave as a buffer
when a small amount of acid and base is separately added.
[2]
+ + +
NH3 CH(CH2OH)COO + H NH3 CH(CH2OH)COOH
NH3+CH(CH2OH)COO + OH NH2CH(CH2OH)COO- + H2O

(c) Serine can also react with chloroethane under suitable condition in a 1:1 mole ratio.
Draw the structure of the organic compound produced and state the type of reaction
involved.
[2]
SN2 Nucleophilic Substitution

[Total: 10]

---End of Paper---

trendyline
1607 trendyline
Tampines Junior College 2016 Preliminary Examination
H2 Chemistry Paper 3

1 (a) (i) Write an equation to show how phenylamine acts as a weak base in
water. [1]

(ii) The base dissociation constant, Kb, for phenylamine is


3.98 x 1010 mol dm3 at 25 oC. Calculate the pH of a buffer solution
made up of phenylamine and its salt in a 3:1 ratio. [2]

(b) Sunset Yellow is used as an aqueous food colourant in sweets, orange


squash and jams. It is synthesised from 4-aminobenzenesulfonic acid.

4-aminobenzenesulfonic acid

(i) Explain why 4-aminobenzenesulfonic acid is less basic than


phenylamine. [2]

(ii) The structure of Sunset Yellow is shown below.

Sunset Yellow

The sodium sulfonate groups (SO3Na+) have little effect on the


colour of Sunset Yellow. Hence, suggest a reason why the sodium
sulfonate groups are needed in the molecule. [1]

(c) Tetralin can undergo mono-bromination by reacting with bromine in the


presence of anhydrous FeBr3.

(i) trendyline
trendylin
d Tetralin

Suggest a reason why the FeBr3 used


Sugge sed is anhydrous.
anhydro [1]

1608 trendyline
2

(ii) Name and describe the mechanism for this reaction, including curly
arrows showing the movement of electrons, and all charges. [3]

(iii) Explain why the bromination of phenylamine does not require the
use of anhydrous FeBr3. [1]

(d) Another possible catalyst for the bromination reaction is the use of
transition elements.

(i) Define what is meant by the term transition element. [1]

(ii) A white precipitate is formed initially when aqueous ammonia is


gradually added to aqueous copper(I) nitrate.

Write an equation for this reaction. [1]

(iii) However, the white precipitate in (d)(ii) dissolves when an excess of


aqueous ammonia is added, resulting in a colourless solution.

Explain why the final solution is colourless. [1]

(iv) When excess aqueous ammonia is added to a solution containing


copper(II) nitrate, a deep blue solution was formed.

Explain for the deep blue colouration and state the colour of light
absorbed by the solution. [4]

(v) Suggest the series of colour change and formula of the complex
formed when excess concentrated HCl is gradually added to a
solution of copper(II) nitrate. [2]

[Total: 20]

trendyline
TPJC 9647/3/JC2/Prelim/16

1609 trendyline
3

2 (a) (i) Name and describe the mechanism of the hydrolysis reaction of
CH3CH2Cl using NaOH(aq). In your answer, show any relevant curly
arrows charges, dipoles or lone pairs of electrons you consider to be
important in this mechanism. [3]

(ii) Write the rate equation for the above reaction. [1]

(iii) State, if any, how the rate constant is affected with an increase in the
concentration of CH3CH2Cl. [1]

(iv) CH3CH2Cl can be prepared by reacting ethanol with SOCl2. Write an


equation for this reaction and suggest an advantage of using SOCl2
as a reagent for halogenation. [2]

(b) (i) Hydrolysis of halogenoalkanes releases halide ions that can be


precipitated as silver halides upon addition of AgNO3(aq). The
halogenoalkanes can be distinguished by the different colours of the
silver halides.

When NH3(aq) is added in excess, the white precipitate of AgCl(s)


dissolves to give a colourless solution. Explain the dissolution of
AgCl(s) precipitate in excess NH3(aq) with the aid of relevant
equations. [3]

(ii) Halide ions can also be distinguished by the addition of acidified


K2Cr2O7(aq).

Use relevant Eo values in the Data Booklet to predict the reaction, if


any, when acidified K2Cr2O7(aq) is added to Cl(aq) and I(aq)
separately. In each case, calculate Ecell and write a balanced
equation for any reaction that occurs. [3]

(c) Silver chloride is sparingly soluble in water at room temperature.

Ag+(aq) + Cl(aq) AgCl(s)

Predict the signs of 'Hppt and 'Sppt for the precipitation reaction. Give a
reason for each of your prediction. [3]

trendyline
TPJC 9647/3/JC2/Prelim/16 [Turn Over

1610 trendyline
4

(d) (i) When a precipitate is formed, 'Gppt, in J mol-1, is given by the


following expression.
'Gppt = 2.303RTlogKsp

The Ksp of silver fluoride, AgF(s), is 1.01 mol2 dm6 at room


temperature.

Use the above 'Gppt expression to determine whether AgF(s) is


soluble in water at room temperature. [2]

(ii) Hence, predict and explain how the solubility of AgF in water would
change when a solution of NaF is added. [2]

[Total: 20]

trendyline
TPJC 9647/3/JC2/Prelim/16

1611 trendyline
5

3 Cosmetic chemistry has been responsible for inventing a range of different


cosmetic products. Over the years, it is estimated that the cosmetic industry
makes 42.8 billion dollars per annum, and a large part of the industry depends
on the research and development behind the chemistry of cosmetics.

Benzaldehyde imparts a bitter almond flavour and aroma. It is commonly used


to produce almond-flavoured food and certain cosmetic products. Benzaldehyde
can be synthesised from cinnamaldehyde obtained from the oil of cinnamon.

(a) (i) Devise a three-step synthesis, starting from cinnamaldehyde, for the
manufacture of benzaldehyde. Give the reagents and conditions
required and the structure of the intermediates involved.

[5]

(ii) Suggest one simple chemical test which would enable you to
distinguish between cinnamaldehyde and benzaldehyde. State the
observations for each compound. [2]

(iii) Benzaldehyde is considered to be soluble in water. By considering


relevant interactions, explain the solubility of benzaldehyde in water. [2]

(b) Ethanol, a common solvent used in cosmetic products due to its volatility,
can undergo oxidation to form ethanoic acid as shown in the half-equation
below:

CH3COOH + 4H+ + 4e CH3CH2OH + H2O E = 0.06 V

The cell notation of an electrochemical cell involving ethanol is given below

Pt(s) | CH3CH2OH(aq), CH3COOH(aq) || MnO4(aq), Mn2+(aq) | Pt(s)

(i) Draw a fully labelled diagram of the electrochemical cell, indicating

trendyline
the flow..
h direction of electron flow [3]

(ii) With reference to the


t Data Booklet
Booklet,, write
write
e an overall equation for the
reaction
eaction which take
takes place in the cell.
cell [2]

TPJC 9647/3/JC2/Prelim/16 [Turn Over

1612 trendyline
6

(iii) Hence, calculate the Ecell for the reaction. [1]

(iv) Describe and explain the effect on Ecell when the concentration of
hydrogen ions is increased in the Mn2+(aq), MnO4(aq) | Pt(s) half
cell. [2]

(c) In the cosmetic jewellery industry, electroplating is the application of a


metal coating to a conducting surface (e.g. pendant) by an electrochemical
process.

One common electrolytic cell for the plating of silver consists of a silver
electrode connected to the anode and the article to be plated connected to
the cathode. The electrolyte used is silver(I) sulfate solution.

(i) Calculate the time taken, in hours, for 2.00 g of silver to be plated
when a current of 0.250 A flows through the electrolyte. [2]

(ii) Suggest and explain whether the electroplating process would be


accelerated if an electrolyte with a higher concentration is used. [1]

[Total: 20]

TPJC 9647/3/JC2/Prelim/16

1613 trendyline
7

4 Shikimic acid, C7H10O5, is a monobasic carboxylic acid used in the production of


Tamiflu, an antiviral medical that is used to treat children with Influenza A and
Influenza B. Shikimic acid can be extracted from the seeds of the sweetgum
fruit.

(a) When the extract of 0.300 g of sweetgum fruit seeds was titrated with
1.00 g dm3 aqueous sodium hydroxide, 16.45 cm3 of aqueous sodium
hydroxide was required to neutralise the Shikimic acid present.

Calculate the percentage by mass of Shikimic acid extracted from the


sweetgum fruit seeds.

You may assume that Shikimic acid is the only compound in the
sweetgum fruit seeds that reacts with sodium hydroxide. [3]

(b) Shikimic acid can undergo condensation with ethanol using concentrated
sulfuric acid as a catalyst to form an ester and water. The enthalpy
change for the forward reaction is 20.0 kJ mol1 and the activation
energy is +50.0 kJ mol1.

(i) Draw a fully labelled reaction pathway diagram for this reaction. [2]

(ii) Determine the activation energy of the reverse reaction. [1]

(iii) On the diagram you have drawn in (b)(i), draw the reaction
pathway diagram for the condensation reaction carried out in the
absence of sulfuric acid. Label the graph for the uncatalysed
reaction as (b)(iii). [1]

(iv) Describe, with the aid of a labelled Boltzmann distribution diagram,


the effect of adding concentrated sulfuric acid on the rate of the
condensation reaction. [4]

trendyline
TPJC 9647/3/JC2/Prelim/16 [Turn Over

1614 trendyline
8

(c) The following information were obtained from tests that were conducted
to deduce the structure of Shikimic acid.

x Shikimic acid is a hexacyclic monobasic carboxylic acid with the


molecular formula, C7H10O5.
x Shikimic acid does not produce an orange precipitate with Bradys
reagent.
x On treatment of Shikimic acid with H2(g) and nickel catalyst at
150 oC, compound A was formed.
x 1 mole of Shikimic acid reacts exactly with 4 moles of PCl5 to give
compound B.
x Shikimic acid decolourises hot acidified potassium manganate(VII)
solution to give a single compound C. The structure of compound C is
shown in the diagram below.

Compound C

(i) Deduce the structures of Shikimic acid, and compounds A and B. [7]

(ii) State the number of possible stereoisomers of Shikimic acid. [1]

(iii) Draw the structure of the organic product formed when compound
C reacts with NaBH4. [1]

[Total: 20]

trendyline
TPJC 9647/3/JC2/Prelim/16

1615 trendyline
9

5 (a) Describe the action of water on separate samples of magnesium and


barium, writing equations including state symbols for any reaction that
occurs. [4]

(b) (i) Calcium hydroxide is sparingly soluble in water and has a solubility
product with a numerical value of 5.50 x 106.

Write an expression for the solubility product, Ksp, of calcium


hydroxide and calculate the solubility of calcium hydroxide in water. [2]

(ii) A student tried to precipitate Ca(OH)2 by mixing 0.20 mol dm3 of


Ca2+(aq) with an equal volume of 0.20 mol dm3 NH3(aq).

Show through calculation why precipitation of Ca(OH)2 will not occur.


(Kb of NH3 = 1.80 x 105 mol dm3) [2]

(iii) Hence, deduce the minimum concentration of Ca2+(aq) that the


student must use that will result in precipitation of Ca(OH)2. [2]

(c) Draw the structural formula of the organic products formed when
compound D reacts with the following reagents.

Compound D

x Alkaline KMnO4(aq), heat


x NaOH(aq)
x Dry HCl(g) [3]

(d) (i) Compound D contains two functional groups that exhibit acidic
properties. Identify the two functional groups. [1]

(ii) Based on the two functional groups identified in (d)(i), state and
explain which of these two groups has a smaller pKa value. [4]

(iii)

trendyline
d li
dy
Suggest a common reagent that
strength of the functional
illustrate
llustra the
he reactio
reaction.
th t can
n be used to illustrate tthe acid
functiona group stated in
n (d)(
(d)(ii).. Write
te an equa
equation to
[2]

[Total: 20]
TPJC 9647/3/JC2/Prelim/16 [Turn Over

1616 trendyline
Tampines Junior College 20126 Preliminary Examination
H2 Chemistry Paper 3 Solutions

1 (a) (i) Write an equation to show how phenylamine acts as a weak base in
water. [1]
C6H5NH2(aq) + H2O(l) C6H5NH3+(aq) + OH(aq)

(ii) The dissociation constant, Kb, for phenylamine is


10 3 o
3.98 x 10 mol dm at 25 C. Calculate the pH of a buffer solution
made up of phenylamine and its salt in a 3:1 ratio. [2]
pOH = pKb + lg [salt]/[base]
pOH = -lg(3.98 x 10-10) + lg (1/3)
= 9.400 + (-0.477)
= 8.923

pH = 14 8.923 = 5.08

(b) Sunset Yellow is used as an aqueous food colourant in sweets, orange


squash and jams. It is synthesised from 4-aminobenzenesulfonic acid.

4-aminobenzenesulfonic acid

(i) Explain why 4-aminobenzenesulfonic acid is less basic than


phenylamine. [2]
SO3H is an electron-withdrawing group, hence it increases the
delocalisation of the lone pair of electrons on N atom into the
benzene ring.

The lone pair of electrons on N atom in 4-aminobenzenesulfonic


acid is thus less available to accept a proton, making it a
weaker base than phenylamine.

(ii) The structure of Sunset Yellow is shown below.

t dyli
trendyline
colour of Sunset
Sunset Yellow

The sodium sulfonate groups (SO3Na a+) have little effect on the
nset Yellow. Suggest a reason
re why the sodium susulfonate [1]

1617 trendyline
2

groups are needed in the molecule.


The sodium sulfonate groups increases the solubility of Sunset
Yellow in water through ion-dipole interactions.

(c) Tetralin can undergo bromination by reacting with bromine in the presence
of anhydrous FeBr3.

Tetralin
(i) Suggest a reason why the FeBr3 used is anhydrous. [1]
To prevent FeBr3 from undergoing hydrolysis/reaction with
water making it unable to act as a Lewis acid.

(ii) Name and describe the mechanism for this reaction, including curly
arrows showing the movement of electrons, and all charges. [3]
Mechanism is electrophilic substitution

FeBr3 + Br2 [FeBr4] + Br+ (single arrow accepted)

(2, 3, 4 & 5 position is accepted)

(iii) Explain why the bromination of phenylamine does not require the
use of anhydrous FeBr3. [1]
S
The lone pair of electrons on the N atom delocalises into the (S

t dyli
trendyline
tren
system/electron
sy
system /electron cloud of the)
he benzenene ring,
ri
ring, hence incr
increasing
the electron density ofo the benzene ring (or more electro
electron rich
or more activated),
activated making phenylamine more reactive to towards
electrophilic substitution.
substitution

TPJC 9647/3/JC2/Prelim/16

1618 trendyline
3

(d) (i) Another possible catalyst for the bromination reaction is the use of
transition elements.

Define what is meant by the term transition elements. [1]


A transition element is a d-block element which forms at least
one stable ion with a partially filled d subshell.
(ii) A white precipitate is formed initially when aqueous ammonia is
gradually added to aqueous copper(I) nitrate.

Write an equation for this reaction. [1]


[Cu(H2O)6]+(aq) + OH(aq) CuOH(s) + 6H2O(l)
white precipitate

Or
Cu+(aq) + OH(aq) CuOH(s)
white precipitate
(iii) However, the white precipitate in (d)(ii) dissolves when an excess of
aqueous ammonia is added, resulting in a colourless solution.

Explain why the final solution is colourless. [1]


Cu+ has a completely filled d subshell / does not have a vacant
d-orbital such that do od electronic transition cannot occur.
(iv) When excess aqueous ammonia is added to solution containing
copper(II) nitrate, a deep blue colour arises.

Explain for the deep blue colouration and state the colour of light
absorbed by the solution. [4]
In transition element complexes, the central Cu2+ ion has
partially filled d orbital.

In the presence of ligands, the d orbitals split into two groups of


different energy levels, with an energy gap (E) between them.

Electrons in a lower energy d-orbital can jump/promote to a


higher energy level by absorbing light energy. This process is
known as dood electronic transition.

The energy absorbed corresponds to the energy gap between


the lower and higher energy d-orbitals. The colour observed is

trendyline
y
the complementary
p y of the colours absorbed.

Blue colour is observed as orange colour is absorbed


absorbed.

(v) Suggest
S
Sugg est the series of colour change and formula of the complex
c [2]

TPJC 9647/3/JC2/Prelim/16 [Turn Over

1619 trendyline
4

formed when excess concentrated HCl is gradually added to a


solution of copper(II) nitrate.
It turns from blue to green (yellow-green) to yellow.
Complex formed: [CuCl4]2-
[20 marks]

trendyline
TPJC 9647/3/JC2/Prelim/16

1620 trendyline
5

2 (a) (i) Name and describe the mechanism of the hydrolysis reaction of
CH3CH2Cl using NaOH(aq). In your answer show any relevant curly
arrows charges, dipoles or lone pairs of electrons you consider to be
important in this mechanism. [3]
Mechanism is nucleophilic substitution, SN2

(ii) Write the rate equation for the above reaction. [1]
Rate = k [CH3CH2Cl] [OH]

(iii) State how the rate constant is affected with an increase in the
concentration of CH3CH2Cl.
No effect (rate constant is only affected by temperature and
catalyst)
[1]
(iv) CH3CH2Cl can be prepared by reacting ethanol with SOCl2. Write an
equation for this reaction and suggest an advantage of using SOCl2
as a reagent for halogenation. [2]
CH3CH2OH + SOCl2 CH3CH2Cl + SO2 + HCl

This method has the advantage of forming only gaseous side-


products which are easily removed from the system.

(b) (i) Hydrolysis of halogenoalkanes releases halide ions that can be


precipitated as silver halide upon addition of AgNO3(aq). The
halogenoalkanes can be distinguished by the different colours of the
silver halides.

When NH3(aq) is added in excess, the white precipitate of AgCl


dissolves to give a colourless solution. Describe the dissolution of
AgCl precipitate in excess NH3(aq) with the aid of relevant
equations. [3]
AgCl(s) Ag+(aq) + Cl(aq) (1)

trendyline
y
Ag+ (aq) + 2NH3 (aq) [Ag(NH
NH3)2]+ (aq) (2)

According to Le Chateliers Principle, addition


a dition of excess
NH3(aq)/ [NH3] increases shifts equilibrium (2)
2) to the right which
+
causes [Ag (aq)] to decrease.

TPJC 9647/3/JC2/Prelim/16 [Turn Over

1621 trendyline
6

This results in the position of equilibrium (1) to be shifted to the


right, hence AgCl dissolves.
Or
This causes the ionic product of AgCl < Ksp of AgCl, hence AgCl
dissolves.

(ii) Halide ions can also be distinguished by the addition of acidified


K2Cr2O7(aq).

Using relevant Eo values in the Data Booklet to predict the reaction, if


any, when acidified K2Cr2O7(aq) is added to Cl(aq) and I(aq)
separately. In each case, calculate Ecell and write a balanced
equation for any reaction that occurs. [3]
Cr2O72 + 6Cl + 14H+ 2Cr3+ + 3Cl2 + 7H2O (for reference)
Ecell = +1.33 (+1.36) = 0.03 V < 0 (reaction is not feasible)

Cr2O72 + 6I + 14H+ 2Cr3+ + 3I2 + 7H2O


Ecell = +1.33 (+0.54) = +0.79 V > 0 (reaction is feasible)

(c) Silver chloride is sparingly soluble in water at room temperature.

Ag+(aq) + Cl(aq) AgCl(s)

Predict the signs of 'Hppt and 'Sppt for the precipitation reaction. Give a
reason for each of your prediction. [3]
Both 'Hppt and 'Sppt would be negative.

'Sppt is negative as the system becomes less disordered/less varied


ways of arranging particles due to a phase/physical state change
from (aq) to (s).

'Hppt is negative because more heat is released during the formation


of strong ionic bonds between Ag+ ions and Cl ions in AgCl(s) but
less heat is absorbed to overcome the weaker ion-dipole interactions
between Ag+ ions and Cl ions with water molecules.

(d) (i) When a precipitate is formed, 'Gppt, in J mol-1, is given by the

trendyline
trendyline
following expression.
fo
'Gppt= 2.303RTlogK
Ksp

The Ksp of silver fluoride,


fluoride AgF, is 1.01 m 2 dm
1 01 mol m6 a
at room [2]

TPJC 9647/3/JC2/Prelim/16

1622 trendyline
7

temperature.

Use the above expression of 'Gppt to determine whether AgF is


soluble in water at room temperature.
'Gppt = 2.303 x8.31 x298 x log(1.01)
= +24.6 J mol1 (sign + units + 3 s.f.)

A positive value of 'Gppt indicates that the precipitation


process is not spontaneous/not feasible/will not occur. Hence,
AgF is soluble in water at room temperature.

(ii) Hence, predict and explain how the solubility of AgF would change
when a solution of NaF is added. [2]
The presence of F ions from the dissolved NaF increases [F] /
due to common ion effect.

By Le Chateliers principle, this causes the position of


equilibrium of AgF(s) Ag+(aq) + F(aq) to shift to the left,
resulting in a lower solubility of AgF in NaF solution than in
water.

[20 marks]

trendyline
TPJC 9647/3/JC2/Prelim/16 [Turn Over

1623 trendyline
8

3 Cosmetic chemistry has been responsible for inventing a range of different


cosmetic products. Over the years, it is estimated that the cosmetic industry
makes $42.8 billion dollars per annum, and a large part of the industry depends
on the research and development behind the chemistry of cosmetics.

Benzaldehyde imparts a bitter almond flavour and aroma. It is commonly used


to produce almond-flavoured food and certain cosmetic products. Benzaldehyde
can be synthesised from cinnamaldehyde obtained from the oil of cinnamon.

(a) (i) Devise a three-step synthesis, starting from cinnamaldehyde, for the
manufacture of benzaldehyde. Give the reagents and conditions
required and the structure of the intermediates involved.

[5]
Step 1) Acidified KMnO4(aq), heat/reflux

Step 2) LiAlH4 in dry ether

Step 3) Acidified K2Cr2O7 (aq), heat with immediate distillation

Intermediate 1:

Intermediate 2:

(ii) Suggest one simple chemical test which would enable you to

trendyline
yyline
distinguish
d st gu s between
bet ee cinnamaldehyde
c a a de yde anda d benzaldehyde.
be a de yde Sta
State any
observations you would make with each compound
compound.. [2]
To each of the two compounds, add Fehlings solution.

Cinnamaldehyde
i ld h d will give a reddish
ddish brown precipitate
precipitate.

TPJC 9647/3/JC2/Prelim/16

1624 trendyline
9

Benzaldehyde will not give a reddish brown precipitate.

To each of the two compounds, add Br2(aq) / Br2(l) in CCl4.

Cinnamaldehyde will decolourize reddish brown Br2 solution.


Benzaldehyde will not decolourize reddish brown Br2 solution.

(iii)Benzaldehyde is considered to be soluble in water. By considering


relevant interactions, explain the solubility of benzaldehyde in water. [2]
Benzaldehyde is soluble in water because the energy released
by the formation of hydrogen bonds between molecules of
benzaldehyde and water [1] is sufficient to overcome the
permanent dipole-permanent dipole interactions between
benzaldehyde molecules and the hydrogen bonds between
water molecules.
(b) Ethanol, a common solvent used in cosmetic products due to its volatility,
can undergo oxidation to form ethanoic acid as shown in the half equation
below:

CH3COOH + 4H+ + 4e CH3CH2OH + H2O E = 0.06 V

The cell notation of an electrochemical cell involving ethanol is given below

Pt(s) | CH3CH2OH(aq), CH3COOH(aq) || MnO4-(aq), Mn2+(aq) | Pt(s)

(i) Draw a fully labelled diagram of the electrochemical cell, indicating


the flow of electrons. [3]

e-
Temperature 298 K
Pressure 1 atm

Salt bridge
Pt(s) electrode
Pt(s) electrode

CH3CH2OH (aq) MnO4- (aq)


1 mol dm-3 1 mol dm-3

trendyline
dyli
CH3COOH (aq)
C Mn2+ (aq)
(a
1 mol dm-3 dm-3
1 mol d

(ii) With reference to the Data Booklet,


ook , write an overall balanced
Booklet ba [2]

TPJC 9647/3/JC2/Prelim/16 [Turn Over

1625 trendyline
10

equation for the reaction which takes place in the cell.


From the Data Booklet,
MnO4- + 8H+ + 5e- Mn2+ + 4H2O E = + 1.52 V

Oxidation: CH3CH2OH + H2O CH3COOH + 4H+ + 4e (x5)


Reduction: MnO4- + 8H+ + 5e- Mn2+ + 4H2O (x4)

Overall equation:
5CH3CH2OH + 4MnO4- + 12H+ 5CH3COOH + 4Mn2+ + 11H2O
(iii) Hence, calculate the Ecell for the reaction. [1]
Ecell = Ered - Eoxi
= + 1.52 ( 0.06)
= + 1.58 V
(iv) Describe and explain the effect on the cell potential when the
concentration of hydrogen ions is increased in the
2+ -
Mn (aq), MnO4 (aq) | Pt half cell. [2]
MnO4- + 8H+ + 5e- Mn2+ + 4H2O

When [H+] increase, system attempts to decrease [H+]. By Le


Chateliers Principle, the position of equilibrium shifts right
favouring reduction process. E MnO4-/Mn2+ will be more positive
(than +1.52 V).

Ecell will be more positive [1] (than +1.58 V)


(c) In the cosmetic jewellery industry, electroplating is the application of a
metal coating to a conducting surface (e.g. pendant) by an electrochemical
process.

One of the common electrolytic cell consists of a silver electrode


connected to the anode and the article to be plated connected to the
cathode. The electrolyte used is silver(I) sulfate solution.

(i) Calculate the time taken, in hours, for 2 g of silver to be plated when
a current of 0.250 A flows through the electrolyte. [2]
Ag+(aq) + e- Ag(s)

Amount of silver to be plated = 2/108


= 1.852 x 10-2 mol

t
trend y
e- Ag
Amount of electrons, n = 1.85 0--22 mol
1.852 x 10 mo

Quantity of charge, Q = nF
0--22 x 96500
= 1.852 x 10

TPJC 9647/3/JC2/Prelim/16

1626 trendyline
11

= 1787 C

Time, t = 1787/0.250
= 7148 s
= 7148/3600
= 1.99 hr

(ii) Suggest and explain if the electroplating process would be


accelerated if an electrolyte with a higher concentration is used. [1]
No as the electroplating process is dependent on the time and
current of the electrolytic cell.
Or
Concentration of the electrolyte does not affect the amount of
electrons discharged in the solution.
[Total: 20]

trendyline
TPJC 9647/3/JC2/Prelim/16 [Turn Over

1627 trendyline
12

4 Shikimic acid, C7H10O5, is a monobasic carboxylic acid used in the production of


Tamiflu, an antiviral medical that is used to treat children with influenza A and
influenza B symptoms. Shikimic acid can be extracted from the seeds of the
sweetgum fruit.

(a) When the extract of 0.300 g of sweetgum fruit seeds was titrated with
1.00 g dm-3 aqueous sodium hydroxide, the titration required 16.45 cm3 of
aqueous sodium hydroxide to neutralise the shikimic acid present.

Calculate the percentage by mass of shikimic acid extracted from the


sweetgum fruit seeds.

You may assume that shikimic acid is the only active ingredient in the
sweetgum fruit seeds that reacts with sodium hydroxide. [3]
RCOOH + NaOH RCOO-Na+ + H2O
Mr of Shikimic acid = 174, Mr of NaOH = 40
Concentration of sodium hydroxide= 1.00 / 40 = 0.0250 mol dm-3

Amount of NaOH used in titration = 0.0250 x 16.45/1000 = 4.11 x 10-


4
mol
Since RCOOH NaOH
Amount of RCOOH used in titration = 4.11 x 10-4 mol

Mass of RCOOH used in titration = 4.11 x 10-4 x 174 = 0.0716 g


% mass of shikimic acid = 0.0716 / 0.30 x 100% = 23.9%
(b) Shikimic acid can undergo condensation with ethanol using concentrated
sulfuric acid as a catalyst to form an ester and water. The enthalpy
change for the forward reaction is 20.0 kJ mol-1 and the activation
energy is +50.0 kJ mol-1.

(i) Draw a fully labelled reaction pathway diagram for this catalysed
reaction. [2]

trendyline
TPJC 9647/3/JC2/Prelim/16

1628 trendyline
13

bi) Catalysed reaction

+50.0

(ii) Determine the activation energy of the reverse catalysed reaction. [1]
Activation energy = 50.0 + 20.0 = +70.0 kJ mol-1

(iii) On the same diagram in (b)(i), draw the reaction pathway diagram
for the condensation reaction in the absence of sulfuric acid. Label
the graph as for this uncatalysed reaction as (b)(iii) clearly. [1]

(iv) Describe, with the aid of a labelled Boltzmann distribution diagram,


the effect of concentrated sulfuric acid on the condensation
reaction. [4]

Fraction of
molecules
Number of molecules with
energy t Ea (uncatalysed)
Number of molecules with
energy t Ea (catalysed)

0
energy E
Ea Ea
A catalyst provides an alternative pathway of lower activation
energy.

trendyline
ttrendylin
dyline
y
As a result, there is a greater number of molecules having
energy greater than or equal to activation energy
an increase in the number/frequency
energy. This leads
numberr equenc of effective col
and therefore increases the rate
ate of reaction
reaction..
co
le to
collisions

TPJC 9647/3/JC2/Prelim/16 [Turn Over

1629 trendyline
14

(c) The following information were obtained from tests that were conducted
to deduce the structure of shikimic acid.

x Shikimic acid is a hexacyclic monobasic carboxylic acid with the


molecular formula, C7H10O5.
x Shikimic acid does not produce an orange precipitate with Bradys
reagent.
x On treatment of shikimic acid with H2(g) and platinum catalyst,
compound A was formed.
x 1 mole of shikimic acid reacts exactly with 4 moles of PCl5 to give
compound B.
x Shikimic acid decolourises hot acidified potassium manganate(VII)
solution to give a single compound C. The structure of compound C is
shown in the diagram below.

Compound C

(i) Deduce the structures of shikimic acid and compounds A and B. [7]

Shikimic acid A B

Or

ttrendyline
Shikimic
kimic acid A B

TPJC 9647/3/JC2/Prelim/16

1630 trendyline
15

Evidence Deduction
Shikimic acid does not Shikimic acid does not
produce an orange contain an aldehyde or a
precipitate with Bradys ketone (carbonyl) functional
reagent. group.

On treatment of shikimic Shikimic acid contains


acid with H2(g) and platinum alkene which undergoes
catalyst, compound A was (catalytic) addition or
formed. reduction to form alkane in
A.

1 mole of shikimic acid Shikimic acid contains 4


reacts exactly with 4 moles alcohol or carboxylic acid
of PCl5 to give compound B. which undergoes
nucleophilic substitution to
give chloroalkane or acyl
chloride in B.

Since shikimic acid is


monobasic, it contains 1
carboxylic acid and 3
alcohol functional groups
Shikimic acid decolourises Shikimic acid contains 2o
hot acidified potassium alcohol and alkene which
manganate(VII) solution to undergoes oxidation to give
give a single compound C. ketone and carboxylic acid
respectively.
(ii) State the number of possible stereoisomers of shikimic acid. [1]
Number of isomers = 23 = 8
(iii) Draw the structure of the organic product when compound C reacts
with NaBH4. [1]

t d li
[Total 20]

TPJC 9647/3/JC2/Prelim/16 [Turn Over

1631 trendyline
16

5 (a) Describe the action of water on separate samples of magnesium and


barium, writing equations including state symbols for any reaction that
occurs. [4]
Mg has no reaction with cold water. It only reacts with steam.

Mg(s) + H2O(g) MgO(s) + H2(g)

Ba reacts violently/vigorously with cold water.

Ba(s) + 2H2O(l) Ba(OH)2(aq) + H2(g)

(b) (i) Calcium hydroxide is sparingly soluble in water and has a solubility
product with a numerical value of 5.50 x 106.

Write an expression for the solubility product, Ksp, of calcium


hydroxide and calculate the solubility of calcium hydroxide in water. [2]
Let the solubility of Ca(OH)2 be y mol dm3.
Ca(OH)2(s) Ca2+(aq) + 2OH(aq)
y y 2y

Ksp = [Ca2+] [OH]2

5.50 x 106 = (y) x (2y)2


= 4y3

y = 1.11 x 102 mol dm3

(ii) A student tried to precipitate Ca(OH)2 by mixing 0.20 mol dm3


Ca2+(aq) with equal volume of 0.20 mol dm3 NH3(aq).

Show through calculation why precipitation of Ca(OH)2 will not occur.


(Kb of NH3 = 1.80 x 105 mol dm3) [2]
NH3(aq) + H2O(l) NH4+(aq) + OH(aq)

Since dissociation of NH3(aq) is negligible,


and [OH] = [NH4+]

[NH4+] [OH]
Kb = [NH3]

t dyli
trendyline
[OH]2
1.80 x 105 = (0.20)

[OH] = 1.90
1.9 x 103 mol
ol dm3

TPJC 9647/3/JC2/Prelim/16

1632 trendyline
17

Ionic product of Ca(OH)2 = [Ca2+] [OH]2

 2
=(
)( )

= 9.00 x 108 mol3 dm9 < Ksp Ca(OH)2, no precipitation


[1]
(iii) Hence, deduce the minimum concentration of Ca2+(aq) that the
student must use that will result in precipitation. [2]
2+ 2
[Ca ] [OH ] = Ksp Ca(OH)2
[Ca2+] (1.90 x 103)2 = 5.50 x 106
[Ca2+] = 1.52 mol dm3

Minimum [Ca2+] needed for precipitation to occur


= 2 x 1.52 = 3.04 mol dm3
(c) Draw the structural formula of the organic products formed when
compound D reacts with the following reagents.

Compound D
x Alkaline KMnO4(aq), heat [1]

ttrendyline
d li
x NaOH(aq) [1]

TPJC 9647/3/JC2/Prelim/16 [Turn Over

1633 trendyline
18

x Dry HCl(g) [1]

(d) (i) Compound D contains two acidic functional groups that exhibit acidic
properties. Identify the two functional groups. [1]
Phenol and carboxylic acid
(ii) Based on the two functional groups identified in (d)(i), state and
explain which of these two groups has a smaller pKa value. [4]
The carboxylic acid group has a lower pKa.

The negative charge on the carboxylate ion is dispersed


through delocalisation of electrons within the RCOO S
system.

The negative charge of the phenoxide ion is dispersed through


delocalisation of electrons within the benzene S system/ p
orbital of O overlaps with the S system of benzene ring

However, the dispersal of negative charge is more effective in


the carboxylate ion than the phenoxide ion. This results in the
carboxylate ion to be more stable and hence, a greater extent of
dissociation by the carboxylic acid functional group and thus a
stronger acid, lower pKa.

t dyli
trendyline
(iii) Suggest a common reagent that
Su th can be e used
use to illustrate the acid
strength of the functional group stated in (d)(
(d)(ii).. Write an equ
equation to
illustrate
llustra the reaction.
reactio [2]
Reagent and condition: Na2CO3

TPJC 9647/3/JC2/Prelim/16

1634 trendyline
19

[20 marks]

trendyline
TPJC 9647/3/JC2/Prelim/16 [Turn Over

1635 trendyline
VICTORIAJUNIOR COLLEGE
JC 2 PRELIMINARY EXAMINATIONS
Higher 2

CHEMISTRY 9647/01
Paper 1Multiple Choice 22 September 2016

1 hour

Additional Materials: Multiple Choice Answer Sheet


Data Booklet

READ THESE INSTRUCTIONS FIRST

Write in soft pencil.


Do not use staples, paper clips, highlighters, glue or correction fluid.
Write your NRIC/FIN number, name and CT group on the Answer Sheet.

There are fortyquestions. Answer all questions. For each question there are four possible
answers A, B, C and D.
Choose the one you consider correct and record your choices in soft pencil on the separate
Answer Sheet.

Read the instructions on the Answer Sheet very carefully.

Each correct answer will score one mark. A mark will not be deducted for a wrong answer.
Any rough working should be done in this booklet.

trendyline
y
This document consists of 16 printed pagesand 0 blank page.

VJC 2016 9647/01/PRELIM/16


1636 [Turn over trendyline
2

Section A

For each question there are four possible answers, A, B, C, and D. Choose the one you
consider to be correct.

1 Use of the Data Booklet is relevant to this question.

The most common ionmolecule reaction in gas clouds of the Universe is as shown.

H 2 (g) + H 2 +(g) H(g) + H 3 +(g)

What could be the composition of an H 3 + ion?

protons neutrons electrons


A 2 1 1
B 2 1 2
C 3 0 1
D 3 0 2

2 Two glass vessels M and N are connected by a closed valve.

M N

M contains helium at 20 oC at a pressure of 1 x 105 Pa. N has been evacuated, and has
three times the volume of M. In an experiment, the valve is opened and the temperature
of the whole apparatus is raised to 100 oC.

What is the final pressure in the system?

A 3.18 104 Pa
B 4.24 104 Pa
C 1.25 105 Pa
D 5.09 105 Pa

3 The shell of a chickens egg makes up 5% of the mass of an average egg. An average
egg has a mass of 50 g.

trendyline
Assume the
e egg shell is pure calcium carb
he carbonate.

How many neutralise 50 cm3 of 2.0


y complete chickens egg shells would be needed to neutrali
mol dm3 ethanoic
thanoic acid?

A 1 B 2 C 3 D 4

VJC 2016 9647/01/PRELIM/16


1637 trendyline
3

4 Aluminium carbide, Al 4 C 3 , reacts readily with aqueous sodium hydroxide. The two
products of the reaction are NaAlO 2 and a hydrocarbon. Water molecules are also
involved as reactants.

What is the formula of the hydrocarbon?

A CH 4 B C2H6 C C3H8 D C 6 H 12

5 Which statement about the ammonium ion, NH 4 +, is correct?

A All bond angles are 107o.


B Ammonium ions are formed when ammonia behaves as an acid.
C Ammonium ions are unreactive when heated with NaOH(aq).
D The bonds are all of the same length.

6 2methylpropan1ol and butan1ol are structural isomers. 2methylpropan1ol has


a lower boiling point.

Which statement explains why the boiling point of 2methylpropan1ol is lower than
that of butan1ol?

A 2methylpropan1ol cannot form hydrogen bonds.


B 2methylpropan1ol has weaker covalent bonds than butan1ol.
C 2methylpropan1ol has weaker dispersion forces than butan1ol.
D 2methylpropan1ol molecules have more surface area than butan1ol
molecules.

7 The Gibbs free energy change, G eq , is related to the equilibrium constant, K eq , by the
following equation.

G eq = 2.303 RT lg K eq
where R is the gas constant and T is the temperature in Kelvin.

Which statement about the following equilibrium is correct?

CO 2 (g) CO 2 (aq)

A Increasing the pressure at constant temperature makes G eq more negative.


B Adding a catalyst makes G eq more negative.
C The dissolution of CO 2 in water is spontaneous for all values of K eq .
D At constant temperature, a shift in position of equilibrium to the right results in the
same value of G eq .

trendyline
VJC 2016 9647/01/PRELIM/16
1638 [Turn over trendyline
4

8 Metaldehyde, (CH 3 CHO) 4 , is used as a solid fuel for camping stoves. The equation for
the complete combustion of metaldehyde is shown.

(CH 3 CHO) 4 (s) + 10O 2 (g) 8CO 2 (g) + 8H 2 O(l)

H{ c = standard enthalpy change of combustion.

Which expression will give a correct value for the enthalpy change of formation of
metaldehyde?

A H{ c metaldehyde (8H{ c carbon + 8H{ c hydrogen)


B H{ c metaldehyde (8H{ c carbon + 16H{ c hydrogen)
C (8H{ c carbon + 8H{ c hydrogen) H{ c metaldehyde
D (8H{ c carbon + 16H{ c hydrogen) H{ c metaldehyde

9 Use of the Data Booklet is relevant to this question.

In research laboratories, the standard calomel reference electrode and the AgCl/Ag
reference electrodes are often used in place of the standard hydrogen electrode (SHE).
Their standard reduction potentials with respect to the SHE are shown below.

Calomel electrode: Hg 2 Cl 2 + e Hg + Cl Eo = +0.27 V


AgCl/Ag electrode: AgCl + e Ag + Cl Eo = +0.22 V

A halfcell containing Au+/Au has a reduction potential of +1.56 V with respect to a


calomel electrode.

Which of the following statements is correct?

A The reduction potential of the Au+/Au halfcell with respect to the SHE is +1.83 V.
B The reduction potential of the Au+/Au halfcell with respect to the AgCl/Ag
electrode is +1.78 V.
C Ag has a higher tendency to be oxidised than H 2 .
D Hg 2 Cl 2 has a lower tendency to be reduced than AgCl.

10 What is the volume of 0.100 mol dm3 NaOH that should be added to 20 cm3 of
0.100 mol dm3 CH 3 CO 2 H (K a = 1.8 x 105 mol dm3) to form a buffer of pH 5.5?

A 15.0 cm3 B 16.0 cm3 C 17.0 cm3 D 18.0 cm3

11 H 2 (g) and I 2 (g) react according to the following equilibrium.

H 2 (g) + I 2 (g) 2HI(g)

trendyline
When 0.10 m of H 2 (g) and 0.10 mol of I2 (g) are heated in a seale
0 mol sealed vessel at 600 K,
0.06 mol of HI was present at equilibrium
equilibrium.

e K c value for the reaction at 600 K?


What is the

A 0.184 B 0.360 C 0.735 D 1.36

VJC 2016 9647/01/PRELIM/16


1639 trendyline
5

12 After the Fukushima nuclear disaster in 2011, a radioactive isotope of iodine, 131I, was
detected at water purification plants in Japan. Radioactive decay is a first order reaction,
and the halflife of radioactive 131I is 8 days.

What is the mass of 131I left after 40 days if the initial mass of 131I was 15 g?

15 15 15 15
A g B 5g C g D g
2 5 2 28 28

13 Which of the following shows the properties of a Group II element? [ = ohm]

melting electrical
point / oC density / g cm 3
conductivity / 1 m1
A 98 0.97 2.4 107
B 113 2.07 7.7 107
C 649 1.74 2.2 107
D 1744 11.3 6.0 107

14 X, Y and Z are elements in the third period of the Periodic Table. The oxide of X is
amphoteric, the oxide of Y is basic and the oxide of Z is acidic.

What is the order of increasing ionic radius for these three elements?

A X, Y, Z B X, Z, Y C Y, X, Z D Y, Z, X

15 Sulfur is an element in the third period, Na to Ar, of the Periodic Table.

Which is the correct statement for sulfur?

A Sulfur is the only element in this period which forms two acidic oxides.
B Sulfur has the highest first ionisation energy of the elements in this period.
C Sulfur is the only element in this period which can form compounds with variable
oxidation states.
D Sulfur is the only element in this period which can exist, at room temperature and
pressure, as a simple molecule containing eight atoms.

16 Which anions containing chlorine are formed when chlorine is passed into hot aqueous
potassium hydroxide?

A Cl and ClO

trendyline
B Cl and
nd ClO 3
d ClO
C Cl and ClO 4
nd ClO
Cl
D ClO and ClO
C 3

VJC 2016 9647/01/PRELIM/16


1640 [Turn over trendyline
6

17 Four different complexes can be obtained by reacting aqueous cobalt(III) chloride with
ammonia under various conditions. Different proportions of chloride are precipitated
when each complex is treated with aqueous silver nitrate.

empirical number of moles of AgCl


formula colour of solid precipitated per mole of complex

I CoCl 3 (NH 3 ) 6 orange 3


II CoCl 3 (NH 3 ) 5 violet 2
III CoCl 3 (NH 3 ) 4 violet 1
IV CoCl 3 (NH 3 ) 4 green 1

Which of the following statements is correct regarding the four different complexes?

A Only complexes III and IV have the coordination number of 6.


B The cations in all four complexes have the same charge.
C Ligands of complex I consists of both chloride and ammonia.
D Complexes III and IV are stereoisomers.

18 In 1869 Ladenburg suggested a structure for benzene, C 6 H 6 , in which one hydrogen


atom is attached to each carbon atom.

Ladenburg structure

A compound C 6 H 4 Br 2 could be formed with the same carbon skeleton as the


Ladenburg structure.

How many structural isomers would this compound have?

A 2 B 3 C 5 D 6

19 Samples of the gases CH 3 CH 2 Cl and Cl 2 are mixed together and irradiated with light.

Which of the following is not likely to be formed in the reaction?

A CH 3 CHCl 2
B CHCl 2 CH 2 Cl

trendyline
C CH 3 CH
H 2 CH 2 CH 3
D CH 2 C
ClCH(Cl)CH(Cl)CH
lCH(C
CH(Cll)CH(C
)CH(Cll)CH 3

VJC 2016 9647/01/PRELIM/16


1641 trendyline
7

20 Compound U gives a positive test with Tollens reagent and can be oxidised with
acidified K 2 Cr 2 O 7 to form 3methylbutanoic acid.

Compound U can be reduced by NaBH 4 to form compound V which reacts with


ethanoyl chloride to form compound W.

What is the structure of compound W?

A CH 3 CH(CH 3 )CH 2 CH 2 CO 2 CH 3
B CH 3 CH 2 CH 2 CH(OCOCH 3 )CH 3
C CH 3 CH(CH 3 )CH 2 CH 2 OCOCH 3
D CH 3 CH 2 CH(CH 3 )CH 2 OCOCH 3

21 The Wittig reaction offers a highly versatile method to synthesise a wide variety of
alkenes. The mechanism for the reaction is summarised as follows.

Ph3P O PPh3
+ X PHPh3 1) NaH O PPh3
+
X CH R I HC R C C R
O II C C R
2) R'
R' R' C R'
,
(Ph denotes a phenyl group C6H5 )

Which of the following correctly represents the type of reaction occurring in stages I and
II?

stage I stage II
A nucleophilic substitution elimination
B nucleophilic substitution condensation
C nucleophilic addition elimination
D nucleophilic addition condensation

22 Which of the following can be used to distinguish between the two compounds?

H O
C HO O O
O O C

A H 2 and Pd
B Br 2 (aq)

trendyline
C g reagent
warm Fehlings g
D hot acidified K 2 Cr 2 O 7

VJC 2016 9647/01/PRELIM/16


1642 [Turn over trendyline
8

23 The presence of a halogen in an organic compound may be detected by warming the


organic compound with aqueous silver nitrate.

Which compound would be the quickest to produce a precipitate?

A B C D
F F Cl Cl

F Cl Cl Br

I F Cl F

24 Warfarin (M r = 308) is an oral anticoagulant that is used to prevent heart attacks,


strokes and blood clots. The structural formula of warfarin is given below.

O O

O
OH

Which of the following statements about warfarin is correct?

A It has a total of 4 stereoisomers.


B It reacts with CH 3 COCl to form white fumes.
C 1 mol of warfarin reacts with 1 mol of alcoholic potassium cyanide in the presence
of heat.
D 1 mol of warfarin reacts with 1 mol of hydrogen gas in the presence of nickel
catalyst and heat.

25 The reaction conditions for four different transformations are given below.

Which transformation has a set of reagents or conditions that is not correct?


+
CH3CH2O Na (alc)
A CH3CHBrCH3 CH3CH=CH2
reflux

OH Na(s) O Br
B Br
Br heat

O O O O
CH3NH2
C C CH3 C C CH3
C N N
HO OH
H H

trendyline
re
e
H2
D CH3CN
CN H3C
CH
C H2NH2
CH
Ptt
P

VJC 2016 9647/01/PRELIM/16


1643 trendyline
9

26 Compound R is formed from compound Q as shown below.

PCl5
compound Q N
O
compound R

Which structure could be Q?

CO2H
A
N
H
CO2H

B
N
H
CO2H

C
N
H
CO2H
D
N
H

27 The reaction of compound E (M r = 152) with hot acidified potassium manganate(VII)


yields three products, F, G and H.

F can be converted to G with the use of alkaline aqueous iodine followed by


acidification with aqueous sulfuric acid.

Which of the following shows the structure of compound E?

A O

B O

C OH

te
trendyline
OH
D

VJC 2016 9647/01/PRELIM/16


1644 [Turn over trendyline
10

28 Compound J is a pharmaceutical drug.


OH
CH3CONH

H2N CO2CH2CH3
J

What is the species formed when compound J is heated with HCl(aq)?

OH
CH3CONH
A
H3N CO2CH2CH3
OH
H3N
B
H3N CO2H
OH
H3N
C
H3N CO2CH2CH3
OH
CH3CONH
D
H3N CO2H

29 Which synthetic route will lead to the successful synthesis of the following product?
CO2H

CH2CH2CH3

CHO LiAlH4 CH2OH K2Cr2O7 CO2H

A in dry ether H2SO4(aq)


CH2CH=CH2 CH2CH2CH3 heat CH2CH2CH3
CHO CH2OH CO2H
LiAlH4 KMnO4
B in dry ether H2SO4(aq)
CH2CH=CH2 CH2CH2CH3 heat CH2CH2CH3
CHO CH2OH CO2H
H2 KMnO4
C

D
H2C
CH CH
t dy i e
trendyline
H=CH
H2
HO
CHO
C
Ptt
P

H2

Pt
H2C
CH
C H2C
CH H3
CH
CH2O
CH H
OH
H2SO4(aq)
at
hea

K2Cr2O7

H2SO4(aq)
CH2CH2CH3
CO2H

CH2CH=CH2 CH2CH2CH3 heat CH2CH2CH3


VJC 2016 9647/01/PRELIM/16
1645 trendyline
11

30 Soy sauce is produced by the fermentation of soy beans by the mould Aspergillus
oryzae. The distinctive salty taste of the sauce is due to salts of glutamic acid formed
during fermentation.

NH2

HO2C CO2H
glutamic acid

The pHvolume curve obtained when 30 cm3 of NaOH is added to 10 cm3 of the
protonated form of glutamic acid of the same concentration is given below.

pH

9.5

4.1

2.1

0 10 20 30 VNaOH added / cm3

Which of the following is the major species present at point S?

A B
+ +
NH3 NH3
-
HO2C CO2H HO2C CO2

C D
+
NH3 NH2
- - - -
O2C CO2 O2C CO2

trendyline
VJC 2016 9647/01/PRELIM/16
1646 [Turn over trendyline
12

Section B

For each of the questions in this section, one or more of the three numbered statements 1 to
3 may be correct.

Decide whether each of the statements is or is not correct (you may find it helpful to put a tick
against the statements that you consider to be correct).

The responses A to D should be selected on the basis of

A B C D
1, 2 and 3 1 and 2 2 and 3 1 only
are only are only are is
correct correct correct correct

No other combination of statements is used as a correct response.

31 Hydrogen reacts with gaseous bromine to form hydrogen bromide

H 2 (g)+ Br 2 (g)  2HBr(g)

and with gaseous iodine to form hydrogen iodide.

H 2 (g)+ I 2 (g)  2HI(g)

For the first reaction, the rate equation is

k1 [H2 ][Br2 ]1.5


rate =
[Br2 ]+k2 [HBr]

For the second reaction, the rate equation is

rate = k[H 2 ][I2 ]

What can be deduced from this information only?

1 For the hydrogen / bromine reaction, the formation of HBr slows down the rate of
the forward reaction.
2 Only the hydrogen / iodine reaction could be a single step reaction.
3 The mechanism of the hydrogen / bromine reaction involves free radicals.

trendyline
VJC 2016 9647/01/PRELIM/16
1647 trendyline
13

32 Use of the Data Booklet is relevant to this question.

Given the enthalpy changes of the following reactions.

Mg(s) + Cl 2 (g) MgCl 2 (s) H = 642 kJ mol1


Mg(s) Mg2+(g) H = +2334 kJ mol1
Cl 2 (g) + 2e 2Cl(g) H = 454 kJ mol1

Which of the following statements are correct?

1 The enthalpy change of formation of MgCl 2 (s) is 642 kJ mol1.


2 The enthalpy change of atomisation of Mg(s) is +148 kJ mol1.
3 The electron affinity of Cl(g) is 698 kJ mol1.

33 Use of the Data Booklet is relevant to this question.

The Direct Borohydride Fuel Cell (DBFC) is a type of alkaline fuel cell that uses NaBH 4
as the fuel and O 2 as the oxidant. A DBFC was set up under standard conditions
according to the diagram below.

The electrode potential of the NaBH 4 /NaBO 2 halfcell is given below.

NaBO 2 + 6H 2 O + 8e NaBH 4 + 8OH Eo = 1.24 V

Which of the following statements regarding the DBFC are correct?

1 The voltmeter will register a reading of +1.64 V.


2 Electrode X is the anode and electrode Y is the cathode in the DBFC.
3 The BO 2 ion produced at electrode X has a bond angle of 180o.

trendyline
VJC 2016 9647/01/PRELIM/16
1648 [Turn over trendyline
14

The responses A to D should be selected on the basis of

A B C D
1, 2 and 3 1 and 2 2 and 3 1 only
are only are only are is
correct correct correct correct

No other combination of statements is used as a correct response.

34 The K sp values of three metal hydroxides are given in the table below.

metal hydroxide K sp
Ca(OH) 2 4.7 x 10 mol3 dm9
6

Sn(OH) 2 5.4 x 1027 mol3 dm9


Cr(OH) 3 6.7 x 1031 mol4 dm12

Which of the following statements are correct?

1 The concentration of OH in a saturated solution of Ca(OH) 2 is 0.021 mol dm3.


2 Ca(OH) 2 (s) will be precipitated if equal volumes of 0.02 mol dm3 NaOH(aq) and
0.02 mol dm3 Ca(NO 3 ) 2 (aq) are mixed together.
3 Cr(OH) 3 has the lowest solubility amongst the three metal hydroxides.

35 Which of the following properties generally increase down Group II?

1 thermal stability of the iodate(V) salt, M(IO 3 ) 2


2 magnitude of the lattice energy of the oxide, MO
3 charge density of the M2+ ion

36 Which of the following statements regarding the metal complex below are correct?

Cl CH3
O
O N
M CH3
N Cl
H2
N

trendyline
1 The oxidation number of M in the complex is +3.
2 The coordination
oordination number of M in the
he complex is 6.
6
3 The complex
omplex contains a tetradentate ligand.
ligan

VJC 2016 9647/01/PRELIM/16


1649 trendyline
15

37 Compound L has a chiral centre and forms a zwitterion.

Which could be the structure of L?

HO
CH3
1 H2N C CO2H
H
H NH2

2 HO C C H
H CO2H
H H NH2

3 C C C H
H H CO2H

38 In which of the following pairs are the members, I and II,


stereoisomers of each other, and
the overall dipole moment of I is larger than that of II?

I II
O2N Br O2N H
1 C C C C
H H H Br
CH3 H CH3 Br
2 C C C C
Br Cl H Cl
Br Br
Br C Br C
C H C H
3
C Br C Cl
H H
Cl Br

39 Which of the following statements regarding compound T are correct?

CH3
HN CH3 OH
C N CONHCH2CH2CO2H
H H
compound T

1 CompoundTreacts
poundT
pound Treacts with excess aqueous bromine to give a product
p with five
mine
ne atoms.
bromine
2 ol of compound T reacts with 3 mol of ethanoyl chloride.
1 mol
3 ol of compound T reacts with 3 mol
1 mol ol of
o cold dilute hydrochloric acid.

VJC 2016 9647/01/PRELIM/16


1650 [Turn over trendyline
16

The responses A to D should be selected on the basis of

A B C D
1, 2 and 3 1 and 2 2 and 3 1 only
are only are only are is
correct correct correct correct

No other combination of statements is used as a correct response.

40 The diagram shows a molecule of a compound used as a flame retardant.

Br
Br
Br

Br Br
Br

Which statements about this structure are correct?

1 Each brominated C atom is chiral.


2 The molecular formula is C 12 H 20 Br 6 .
3 The CCC bond angles are all 120o.

trendyline
VJC 2016 9647/01/PRELIM/16
1651 trendyline
17

Level of difficulty Percentage No of questions


Easy 22.5 9
Moderate 55 22
Difficult 22.5 9

MCQ Questions for H2


Qn Topic Remarks Answer
1 Atomic Structure (p, n, e) E D
2 Gases (calculation) M A
3 Atoms, Molecules, Stoichiometry (calculation) M B
4 Atoms, Molecules, Stoichiometry (equation) M A
5 Bonding (, bond length) & IE I (acidbase properties) M D
6 Bonding (IMF) M C
7 Entropy T D
8 Enthalpy (H f , H c ) M C
9 Eo (electric cells) T B
10 IE I (buffer calculation) M C
11 Chemical Eqm (K c calculation) E C
12 Kinetics I (t 1/2 calculation) E B
13 Group II (physical properties) E C
14 Periodic Table (ionic radius) M A
15 Periodic Table (properties of S) M D
16 Group VII (Cl 2 + hot OH reaction) E B
17 Transition Metals (ligand / coordination no.) T D
18 Organic (structural isomerism) M A
19 Organic (FRS mechanism) M C
20 Organic (RCHO, ROH reactions) M C
21 Organic (types of mechanism) M A
22 Organic (distinguishing test: RCHO, ester, phenol) M D
23 Organic (RX: AgNO 3 reaction) E A
24 Organic (C=C, ester, RCO, ROH reactions) M B
25 Organic (RX, C=C, RCO 2 H reactions) T C
26 Organic (RX, RCO 2 H, amine reactions) T B
27 Organic (C=C, ROH, RCO reactions) T A
28 Organic (hydrolysis reaction: amide, amine, ester) M B
29 Organic (phenol, C=C, amine reactions) M D
30 Organic (amino acid titration curve) E C
31 Kinetics II (mechanism, rate equation) M B
32 H calculation M B
33 Eo (fuel cell) + Bonding (bond ) T A
34 IE II (K sp calculation) T D
35 Group II (physical/chemical properties) E D
36 Transition Metals (reaction) M A
37 Organic (amino acid) E A
38 Organic (stereoisomers, dipole moment) M D
39 Organic (phenol, RCO 2 H, amine, amide reactions) T B
40 Organic (chiral C, MF, ) M D

Qn
No trendyline
trendylin
y ne Modified/New MCQ Questions for H1
Topic Remarks Answer

VJC 2016 9647/01/PRELIM/16


1652 [Turn over trendyline
Victoria Junior College Option A: Both 2methylpropan1ol and butan
2016JC2 H2 Chemistry Prelim Paper 1 1ol can form intermolecular hydrogen bonds due
Suggested Answers to the presence of OH in the molecules.
Option B: Both molecules have strong covalent
No. Ans No. Ans No. Ans No. Ans bonds between their atoms.
1 D 11 C 21 A 31 B Options C and D: 2methylpropan1ol has
weaker dispersion forces than butan1ol as 2
2 A 12 B 22 D 32 B
methylpropan1olis more spherical and has less
3 B 13 C 23 A 33 A (not more) surface area for electron interaction
4 A 14 A 24 B 34 D than butan1ol.
5 D 15 D 25 C 35 D
7 D
6 C 16 B 26 B 36 A
Options A, B and D: K eq is unaffected by
7 D 17 D 27 A 37 A increasing the pressure, adding a catalyst or a
8 C 18 B 28 B 38 D shift in position of equilibrium. K eq is only affected
9 A 19 C 29 D 39 B by a change in temperature. Hence G eq has the
10 C 20 C 30 C 40 D same value at constant temperature.
Option C: When K eq < 1, lg K eq will be negative
1 D and G eq will be positive according to the
+
In H 3 , equationG eq =2.303 RT lg K eq , i.e. the
p = 3, n = 0, e = 3 1 = 2 (one less e than p) dissolution of CO 2 in water is nonspontaneous
when K eq < 1.
2 D
No. of moles of helium remains the same at the 8 C
P1 V1 P2 V2 (CH 3 CHO) 4 (s) + 10 O 2 (g) 8CO 2 (g) + 8H 2 O(l)
two temperatures. Using = ,
T1 T2 H{ c metaldehyde
1 105 V1 P2 (V1 + 3V1) = (8H{ f CO 2 + 8H{ f H 2 O) H{ f metaldehyde
=
20 + 273 100 + 273
4
Hence H{ f metaldehyde
P 2 = 3.18 10 Pa = (8H{ f CO 2 + 8H{ f H 2 O) H{ c metaldehyde
= (8H{ c carbon + 8H{ c hydrogen) H{ c metaldehyde
3 B
Mass of CaCO 3 in one chickens egg shell 9 A
5
= 50 = 2.5 g
100
No. of moles of CaCO 3 in one chickens egg shell
= 2.5 100.1= 0.025 mol
No. of moles of ethanoic acid, CH 3 CO 2 H used
3
= 2.0 50 10 = 0.10 mol
2+
2CH 3 CO 2 H + CaCO 3 Ca (CH 3 CO 2 ) 2 + H 2 O + CO 2
No. of moles of CaCO 3 needed
= 0.10 2= 0.050 mol
No. of complete chickens egg shells needed
= 0.050 0.025= 2
o +
Option A: E (Au /Au) wrt the SHE = +1.56 +
4 A (+0.27) = +1.83 V(refer to diagram above)
Equation: o +
Option B: E (Au /Au) wrt the AgCl/Ag electrode =
Al 4 C 3 + 4NaOH + 4H 2 O 4NaAlO 2 + 3CH 4 +1.56 + (0.27 0.22) = +1.61 V (refer to diagram
Hydrocarbon is CH 4 . above)
Option C: The reduction potential of AgCl/Aghalf
5 D +
cell (+0.22 V) is more positive than that of H /H 2
+
H +
NH4 is tetrahedral. The bond halfcell (0.00V). Hence Ag has a lower tendency
o
angles are 109 and the bonds to be oxidised than H 2 .
N are all of the same length. Option D: The reduction potential of
H H +
NH4 ions are formed when NH3 Hg 2 Cl 2 /Hghalfcell (+0.27V) is more positive than
H behaves as a base, i.e. a proton that of AgCl/Ag halfcell (+0.22V). Hence Hg 2 Cl 2
acceptor. has a higher tendency to be reduced than AgCl.

trendyline
+
On heating, NH 4 + OH NH 3 + H 2 O
10 C
1
3
6 C Let volume of 0.100 mol dm NaOH added be V
3
CH3 c .
cm
+
CH3 C CH2OH CH3CH
CH H2CH2C
CH H2OH
CH H NaOH
OH + CH 3 COO 2 H CH 3 CO 2 Na + H 2 O
No. of moles of NaOH added
add (limiting reagent)
4
H = 0.100 V 1000 = V 10 mol
- - - - -
2 methylpropan 1 ol butan 1 ol = no. of moles of CH 3 CO 2

1
1653 trendyline
No. of moles of CH 3 CO 2 H left
4
= (0.100 20 1000) (V 10 ) 16 B
4
= (20 V) 10 mol In hot KOH(aq),Cl 2 undergoes disproportionation
+
H [CH3 CO2 ]
- reaction as shown below.
Ka =
3Cl 2 + 6OH 5Cl + ClO 3 + 3H 2 O
[CH3 CO2 H]
5 (105.5 )(V 104 )
1.8 x 10 = 4
17 D
20V 10
3 Formula of complex
V = 17.0 cm I [Co(NH 3 ) 6 ]Cl 3
II [Co(NH 3 ) 5 Cl]Cl 2
11 C III [Co(NH 3 ) 4 Cl 2 ]Cl
H 2 (g) + I 2 (g) 2HI(g) IV [Co(NH 3 ) 4 Cl 2 ]Cl
Initial / mol 0.10 0.10 0 Option A: All complexes, I to IV, have
Change / mol 0.03 0.03 +0.06 coordination number of 6.
Eqm / mol 0.07 0.07 0.06 Option B: Charge of the cation depends on how

many Cl ligand it has. Only cations in complexes
[HI]2 (0.06)2
Kc = = = 0.735 III and IV have the same charge of +1. Cation in
[I2 ][H2 ] (0.07)2
complex I has a charge of +3 and cation in
complex II has a charge of +2.
12 B
Option C: Complex I only consists of ammonia

ligands since all 3 moles of Cl are precipitated by
AgNO 3 per mole of the complex.
Option D: ComplexesIII and IV have the same
structural formula but the ligands have different
40 days is equivalent to 5 halflives; hence the
131 15 arrangement in space as they have different
mass of I left would be 5 g. colours.
2

13 C 18 B
Option A shows the properties of a Group I 3 structural isomers are shown below.
CBr CBr CH
element.
CH CH CH CBr CH CBr
Option B shows the properties of a nonmetal
with low electrical conductivity.
CH CH CH
Option C shows the properties of a Group
CH CBr CH CH CH CBr
IIelement with relatively high melting point and
has high electrical conductivity. Melting point is
higher for Group IIelements than Group I 19 C
Option A: CH 3 CHCl 2 can be formed from one of
elements due to stronger metallic bonds.
the propagation steps:
Option Dshows the properties of a transition metal
o
with very high melting point> 1000 C and very CH CHCl + Cl2
3 CH CHCl2 + Cl
3
high density. Option B: CHCl 2 CH 2 Cl can be formed from
CH 3 CHCl 2 in the propagation step:
14 A
CH CHCl2 + Cl
3 CH CHCl + HCl
From the acidbase nature of the oxides,it can be 2 2

deduced that X is in Group III,Yis in Group I or CH2CHCl2 + Cl2 CH2ClCHCl2 + Cl


GroupII, and Z is a nonmetal. Since ionic radius
of metal ion decreases across the period due to Option C: CH3CH2 is not produced from
increasing nuclear charge, ion ofX is smaller than CH 3 CH 2 Cl as the CCl bond is not broken.Hence
ion ofY. Nonmetal ion of the same period has a CH 3 CH 2 CH 2 CH 3 is not likely to be formed.
larger ionic radius than a metal ion as it gains Option D: CH 2 ClCH(Cl)CH(Cl)CH 3 can be formed
electrons to form an additional shell of electrons. from a termination step:
Hence, ion ofZ has the largest ionic radius. CH2ClCH(Cl) + CH(Cl)CH 3
CH2ClCH(Cl)CH(Cl)CH3
15 D
Option A: Phosphorus also forms two

trendyline
acidicoxides, P 4 O 6 and P 4 O 10 . 20 C
Option B: Ar has
st
s tthe highest 1 ionisation sitive test with Tollens reagent U contains
Positive
energy.Option C: Phosphorus and chlorine can CHO group.Since U is oxidised by acidified
alsoform compounds ds with variable oxidation
oxidatio K 2 Cr 2 O 7 to
o CH 3 CH(CH 3 )CH 2 CO 2 H, U is

states, e.g. PCl 3 andd PCl
PCl 5 , Cll and ClO
CO . C 3 CH(CH 3 )CH
CH H 2 CHO.
CHO.Ucan
CH Uc be reduced by
Option D:Only sulfur ulfur exists
i t as S 8 simple NaBHBH 4 tto form
fformV,
V, CHH 3 CH(
CH(CH 3 )CH 2 CH 2 OH, which
CH
molecules. reacts with CH 3 COCl to form an esterW,
CH 3 CH(CH 3 )CH 2 CH 2 OCOCH 3 .

2
1654 trendyline
24 B
Option A: Warfarin has 2 stereoisomers as it has
21 A only one chiral centre. It has no geometric
Nucleophilic substitution of X by :PPh 3 in stage I. isomers as the C=C bond is found in the ring
structure.
X PPh3
Ph3P X C R OptionB: The OH group reacts with CH 3 COCl to
IHC R form an ester and white fumes of HCl.
R'
R' OptionC: Warfarin does not react with KCN(alc) in
Elimination by removal of atom or groups of atoms the presence of heat as there is no alkyl halide
from adjacent carbon atoms in stage IIto form a (i.e. RX) present. The ketone functional group
multiple bond. present in Warfarin can react with HCN, NaCN
catalyst at room temperature, not KCN(alc).
O PPh3
O PPh3 + OptionD: 1 mol of warfarin reacts with 2 mol of H 2
C C R gas in the presence of nickel catalyst and heat as
II C C R
R' R'
both the ketone and the alkene functional groups
are reduced.
[In condensation reaction, two molecules react
and become covalently bonded with the 25 C
concurrent loss of a small molecule, e.g. H 2 O, Option A:CH 3 CHBrCH 3 undergoes elimination
HCl.] +
reaction with CH 3 CH 2 O Na (alc), reflux to form
CH 3 CH=CH 2 .
22 D OptionB:
Option A: The benzaldehyde is reduced by H 2 , Pd.
However, there is no visible observation to -
OH Na O Na
+
distinguish between the two compounds. Br B-r +
Option B: Phenol cannot react with Br 2 (aq) in this
Br Br
question as it has substituents at the 2,4and 6
positions w.r.t. the phenol functional group.
Option C: The benzaldehyde cannot react with O
Fehlings reagent. It can only react with Tollens Br
reagent.
Option D: Orange acidified K 2 Cr 2 O 7 turns green OptionC:Amine does not react with carboxylic
for the following molecule. acid to form amide. N.B.Amine reacts with acyl
O O O O chloride to form amide.
C C OptionD:CN is reduced by H 2 , Pt toCH 2 NH 2 .
H O H OH
acid
hydrolysis OH 26 B
CO2H COCl
oxidation
O O PCl 5
C N
HO OH N
N
C O H H O
OH compound Q compound R
Orange acidified K 2 Cr 2 O 7 remainsorange for the
following molecule as there is no oxidation taking
27 A
place.
OptionA:
O O OH
HO O O HO OH + +
C O O O O
OH OH
acid O H F
C G
hydrolysis CH3 OH - O O+ -
OH + 3I2 + 5OH CHI3 + 3I + 4H2O
- -
O O O O
F
23 A +
H (aq)
Size of atom: F<Cl<Br<I
O O

trendyline
y e
Strength of bond: CF> CCl> CBr > CI
The larger the size e of the halogen atom, the OH OH
O
longer and weakerr the C CX
C X bond (where X= G
F,Cl,Br,I) which is more
ore easily broken. Hence C
C OptionB:
Optio
OptionB:
OH
Ibond is most easily ly broken and the I ions
sily ons +O +
O O
+ O O
formed will react with Ag (aq) to form a yellow O OH OH
OH
precipitate of AgI.
None of the products react with alkaline aqueous
iodine to form any of the other product.
3
1655 trendyline
OptionC: case for the hydrogen / bromine reaction. Hence,
OH O O the slow rate determining step involves 1
O
+
OH O OH molecule of H 2 and 1 molecule ofI 2 , which could
OH
O be a single step reaction for the hydrogen / iodine
Only two products are formed. reaction.
OptionD:
HO OH
OH O O
OH + O + Option3: Incorrect
O O
O O There is no evidence from the rate equation that
OH OH the mechanism of the hydrogen / bromine
reactioninvolves free radicals.
OH O
- -
O + 3I + 6OH O + CHI + 3I + 5H O 32 B (1 and 2 only are correct)
O 2 3 2
O O O Option1: Correct
-
OH CH3
-
O O From Mg(s) + Cl 2 (g) MgCl 2 (s), the enthalpy
1
+
change of formation of MgCl 2 is 642 kJ mol , i.e.
H (aq) the energy change when 1 mol of MgCl 2 is
formed from its constituent elements in their
OH
O
standard states, Mg(s) and Cl 2 (g).
O O Option2: Correct
2+
From Mg(s) Mg (g),
OH OH st nd
+2334 = H atom (Mg) + 1 I.E. + 2 I.E.
H atom (Mg) = +2334 736 1450 = +148 kJ
28 B mol
1

Both the amide and ester functional groups will Option3: Incorrect
undergo acid hydrolysis. Amine will undergo acid
From Cl 2 (g) + 2e 2Cl (g),
base reaction with HCl(aq). st
454 = B.E.(ClCl) +2(1 E.A.)
st 1
1 E.A. = (454 244) 2 = 349 kJ mol
29 D
Options A and B: LiAlH 4 in dry ether is unable to 33 A (1, 2 and 3 are correct)
reduce alkene. Option1: Correct
OptionC: KMnO 4 with H 2 SO 4 (aq), heat will cause
NaBO 2 + 6H 2 O + 8e NaBH 4 + 8OH

E
o
9
both CH 2 OH and CH 2 CH 2 CH 3 side chains on
O 2 + 2H 2 O + 4e 4OH

E = +0.40 V
o

the benzene ring to be oxidised to CO 2 H as both o


E cell = 0.40   9
side chains have a benzylic hydrogen. Option 2: Correct
OptionD: Both aldehyde and alkene functional NaBH 4 is oxidised to NaBO 2 at the anode X while
groups will be reduced by H 2 , Pt. Only the primary
O 2 is reduced to OH at the cathode Y.
alcohol will be oxidised by K 2 Cr 2 O 7 with Option3: Correct
H 2 SO 4 (aq), heat to form CO 2 H. 180o
O B O
30 C The BO 2 anion has 2 bond pairs
Point S corresponds to the second equivalence and 0 lone pair around the central B atom, giving
+ o
it a linear shape with a bond angle of 180 .
NH3
34 D (1 only is correct)
HO2C CO2H Option1: Correct
point for the titration of with 2+
NaOH. Ca(OH) 2 (s) Ca (aq) + 2OH (aq)
3
Relative acid strength: CO 2 H > alkyl Let the solubility of Ca(OH) 2 be y mol dm .
2+ 2 6
CO 2 H >NH 3 .
+ [Ca ][OH ] = 4.7 x 10
2 6
Thus the species present at point S is (y)(2y) = 4.7 x 10
3
+ y = 0.0106 mol dm
3
NH3 [OH ] = 2y = 0.021 mol dm
Option2: Incorrect
O2C CO2 When equal volumes of solutions are mixed
.
together, dilution occurs and their concentrations
2+
31 B (1 and 2 only are correct) are halved. Hence, new [Ca ] = 0.01 mol

trendyline
3 3
Option1: Correct dm and nd new [OH ] = 0.01 mol dm .
2+ 2 2
Since [HBr] is in the denominator of the rate Ionic
onic product = [Ca
[ ][OH ] = (0.01)(0.01)
6
6 6
6 3 9
equation, an increase se in its concentration will
ase w = 1.0 1 < 4.7 x 10 mo
.0 10 mol dm
lower the value of rate.
ate.
e. Since ionic product < <KK sp (Ca(OH) 2 ), precipitation
Option2:Correct occur.
will not occu
For the hydrogen / iodine reaction
reaction, the given rate Option3: Incorrect
equation follows the molecularity of the given Sn(OH) 2 has the lowest solubility amongst the
three metal hydroxides.
equation(H 2 (g)+ I 2 (g)  2HI(g)) but this is not the
4
1656 trendyline
metal Solubility / mol the C=C bond in molecule I, the overall dipole
K sp 
hydroxide dm moment is larger than that in molecule II.
6
4.7 10 Option 2: Incorrect
Ca(OH) 2 3 9 0.0106
mol dm Molecules I and IIare structural isomers of each
27
5.4 10  other and are not stereoisomers.
Sn(OH) 2 3 9 1.11 x 10
mol dm Option 3: Incorrect
31
6.7 10  Molecules I and IIare enantiomers, hence they
Cr(OH) 3 4 12 1.26 x 10
mol dm are stereoisomers.The overall dipole moment of
35 D (1 only is correct) the two molecules are identical.
Option1: Correct
Down the group, cationic radius increases and 39 B (1 and 2 only are correct)
2+
charge density of M decreases. Polarising Option 1: Correct
2+
power of M decreases and the electron cloud of CH3
iodate(V) ion is less distorted. Thermal stability of HN OH excess
CH3
M(IO 3 ) 2 increases. Br2(aq)
Option2: Incorrect C N CONHCH2CH2CO2H
q+ q- H H
Since L.E. , magnitude of L.E. decreases CH3
r+ + r-
down the group with increasing cationic radius, r + . HN Br Br OH
CH3
Option3: Incorrect
2+ Br C N CONHCH2CH2CO2H
Charge density of M ion decreases down the
group as ionic radius increases while the charge Br H H Br
remains the same. Option 2:Correct
CH3
36 A (1, 2 and 3 are correct) HN OH
Option1: Correct CH3 + 3CH COCl
3

Overall charge of complex is neutral with two Cl C N CONHCH2CH2CO2H

ligands and one RCO 2 ligand.Hence the H H
oxidation number of M is +3. CH3
Option2: Correct O
C N CH3
OCOCH3
Coordination number is the number of dative
bonds around M which is equal to 6 in the CH3 C N CONHCH2CH2CO2H
complex. H C
Option3: Correct + 3HCl
O CH3
Besides the two Cl ligands, the other four dative
bonds are from the same tetradentate ligand. Option 3: Incorrect
Only 2 mol of cold HCl would react with 1 mol of T.
CH3
37 A (1, 2 and 3 are correct)
Option1: Correct HN OH
CH3
HO
CH3 C N CONHCH2CH2CO2H + 2HCl
* H H
H3N C CO2
Cl CH
3
H H
N OH
Option2: Correct H CH3 Cl
H NH3 C N CONHCH2CH2CO2H
* HH H
HO C C H
H CO2 40 D (1 only is correct)
Option 1:Correct Br
Option3: Correct
H H NH3
Option 2:Incorrect Br

The molecular formula
*
C C C H is C 12 H 18 Br 6 . Br

Option 3:Incorrect
H H CO2 The CCC bond

trendyline
angles
gles a
o
are all 109.5 as Br Br
38 D (1 only is correct)
t) every C at atomis
om sp
3
Br
Option 1: Correct hybridised.
hybridi
hybr i sed.
Molecule I is the cis molecu IIIis th
s isomer and molecule the
trans isomer, hence they are stereoisomers.
stereoisomers Both
NO 2 and Br are electronwithdrawing
lectronwithdrawing in nature.
Since these two groups are on the same side of

5
1657 trendyline
VICTORIA JUNIOR COLLEGE
JC 2 PRELIMINARY EXAMINATIONS
Higher 2

CANDIDATE
NAME ...

CT GROUP ..

CHEMISTRY 9647/02
Paper 2Structured 13 September 2016

Candidates answer on the Question Paper. 2 hours

Additional Materials: Data Booklet

READ THESE INSTRUCTIONS FIRST

Write your name and CT group on all the work you hand in.
Write in dark blue or black pen on both sides of the paper.
You may use a soft pencil for any diagrams, graphs or rough working.
Do not use staples, paper clips, highlighters, glue or correction fluid.

Answer all questions.


A Data Booklet is provided.
The number of marks is given in brackets [ ] at the end of each question or part question.

For Examiners Use

trendyline
ne Total

This document consists of 16printed pages and 0 blank page.

1658 trendyline
VJC 2016 9647/02/PRELIM/16[Turn over
2
Answer all the questions in the space provided.

1 Planning (P)

When potassium manganate(VII) reacts with sodium ethanedioate, a redox reaction occurs as
shown below:

2MnO 4 + 5C 2 O 4 2 + 16H+ 2Mn2+ + 10CO 2 + 8H 2 O reaction 1

A product that is generated during the course of the reaction and helps to speed up the
reaction rate is known as an autocatalyst. The autocatalyst for the above reaction is Mn2+.

The kinetics of this reaction can be investigated by determining the concentration of MnO 4
over the course of the reaction. Fixed aliquots (portions) of the reaction mixture are withdrawn
at regular time intervals and added to an excess of potassium iodide.

The MnO 4 in the aliquots reacts with excess iodide as shown below.

2MnO 4 + 10I + 16H+ 2Mn2+ + 5I2 + 8H 2 O reaction 2

The amount of iodine formed can then be determined by titration with the sodium thiosulfate
solution as shown below.

I 2 + 2S 2 O 3 2 2I + S 4 O 6 2 reaction 3

The volume of sodium thiosulfate used is proportional to the concentration of MnO 4 .

(a) In this experiment, both sodium ethanedioate and sulfuric acid are used in large excess.
Explain the purpose of using a large excess of sodium ethanedioate and sulfuric acid.


[1]

(b) Using the information given above, you are required to write a plan for the determination
of the concentration of MnO 4 at regular timing intervals.

You may assume that you are provided with the following:
0.0500 moldm3 potassium manganate(VII), KMnO 4
0.500 moldm3 sodium ethanedioate, Na 2 C 2 O 4
1.00 moldm3 sulfuric acid, H 2 SO 4
0.200 moldm3 potassium iodide, KI
solid Na 2 S 2 O 3
starch indicator
stopwatch
apparatus and chemicals normally found in a school or college laboratory.

Your plan should include:


justification for
o the volume of each reactants to be used in reaction 1 so as to allow a

trendyline
minimum
nimum
m of 6 aliquots of the e reaction mix withdrawn and the
mixture to be with
ncentration of MnO 4 in each aliquot to be determined;
concentration determined
o the intended concentration of the sodium thiosulfate, Na 2 S 2 O 3 , to be used;
brief, but specific, details of the apparatus you would use, bearing
bear in mind the
levels of precision they offer;
an outline
tli off h
how th lt would
the results ld b bt
b i d
be obtained;
a sketch of the graph you would expect to obtain.


1659 trendyline
VJC 2016 9647/02/PRELIM/16
3

trendyline

1660 trendyline
VJC 2016 9647/02/PRELIM/16[Turn over
4

trendyline [10]

(c) The order of reaction with respect to MnO 4 can be determined by initial rate method.
Use your graph in (b), suggest how the initial rate can be determined.
1661 trendyline
VJC 2016 9647/02/PRELIM/16
5

....[1]

[Total: 12]

2 (a) Proline (pro) is one of the naturally occurring amino acids. It can be synthesised
from 4hydroxybutanal in 4 steps.

step I step II OH step III step IV O


HO CHO E HO C H F
OH
CO2H
4 hydroxybutanal NH
proline
(i) Suggest the structures of Eand F.

E F

[2]

(ii) Suggest the reagents and conditions required for the following steps.
stepI: .
stepIII: ..
stepIV: ...[3]

(b) The two pK a values associated with proline are2.0 and 10.5.

Make use of these pK a values to suggest the major species present in the solution
of proline with the following pH values.

pH 1 pH 7 pH 1 pH pH
12 7 pH 12

(c)
tren
trendyline
ndylin
ndylin
ne
ne
The structures ofamino acids, valine (val) and cys are shown below.
[3]

1662 trendyline
VJC 2016 9647/02/PRELIM/16[Turn over
6
O O

OH HS OH
NH2 NH2
valine cysteine

(i) Draw the structure of valprocys peptide.

[2]

(ii) State the possible R group interactions that valprocys may have with
another valprocys peptide.

[2]

[Total: 12]

3 (a) A student used the following setup in an attempt to produce chlorine gas using
potassium chloride solution.

graphite graphite
salt bridge graphite electrode B electrode A
Zn electrode

1.00 mol dm3 1.00 mol dm3 200 cm3 of 1.0 mol dm3
Zn2+ (aq) Co2+ (aq), Co3+ (aq) KCl (aq)
half-cell 1 half-cell 2

(i) Using relevant data from the Data Booklet, predict the type of redox reaction
that occurs at each halfcell.State the polarity at the graphite electrodes A
and B.

Halfcell 1:
Halfcell 2:

trendyline
Graphite electrode A:
A:
:
Graphite electrode B: [2]

(ii) 20 cm3of hydrogen gas was collected on one of the electrodes after 10
minutes at room temperature and pressure. Assuming 20% of the hydrogen
1663 trendyline
VJC 2016 9647/02/PRELIM/16
7

gas was lost during collection, calculate the current produced by the battery
setup.

[2]

(iii) With reference to relevant data from the Data Booklet, explain the absence
of chlorine being discharged on the other electrode.

.. [1]

(b) 20 cm3 of a gas mixture containing gaseous ethanol, carbon monoxide and excess
oxygen was burned completely. There was a contraction of 1 cm3. When the
product mixture was passed through sodium hydroxide, there was a further
contraction of 6 cm3. All gas volumes are measured at 400 K and 1 atm.

Determine the molar composition of the ethanol and carbon monoxide in this gas
mixture.

trendyline [3]

[Total: 8]

4 Magnesium and beryllium are Group II elements but beryllium behaves differently from
that of magnesium. There is said to be 1664
a diagonal relationship between beryllium and
trendyline
VJC 2016 9647/02/PRELIM/16[Turn over
8
aluminium as they show similar chemical behaviour due to their similarities in
electronegativity and charge density.

(a) (i) When a few drops of water are added to solid beryllium chloride, steamy
white fumes are evolved and a white solid remains, which is insoluble in
water.

Write a balanced equation for this reaction.

.. [1]

(ii) When a large amount of water is added to solid beryllium chloride, a clear,
weakly acidic solution is obtained. Explain.

.. [1]

(iii) At 750 oC, the relative molecular mass of gaseous beryllium chloride
corresponds to the formula BeCl 2 . At 550 oC, gaseous beryllium chloride
exists as a mixture of BeCl 2 and Y (relative molecular mass of Y is 160).
Determine the molar composition of gaseous beryllium chloride at 550 oC
which has a relative molecular mass of 100. Draw a diagram to illustrate the
nature of the bonding in Y and indicate the value of the bond angle about Be.

[2]

(iv) When aluminium chloride, AlCl 3 , is mixed with ammonia, compound Z is


formed and the reaction is exothermic. The diagram below shows the
relationship between the heat evolved per mole of the mixture and the mole
fraction of AlCl 3 .
Heat evolved /
J mol1

0 0.5 1.0
1
Mole fraction of AlCl
AlC
A Cl3

Draw a diagram to show the bonding in a molecule of compound Z.


1665 trendyline
VJC 2016 9647/02/PRELIM/16
9

[1]

(v) Sketch a labelled diagram similar to (a)(iv) that you would expect to obtain
when AlCl 3 is replaced by BeCl 2 .

Heat evolved/
J mol1

0
1.0
Mole fraction of BeCl2 [1]

(vi) Write a balanced equation of a chlorination reaction in Organic Chemistry


that uses anhydrous BeCl 2 as a catalyst.

[1]

(b) Magnalium is an alloy of aluminium and magnesium which is used in boat-building.


The diagram below shows some reactions of magnalium.

sample of HCl(aq) colourless excessNaOH(aq) white


magnalium solution ppt

filter

HCl(aq)
added
white dropwise colourless white
solid, filtrate, residue,
D C B

Identify B,C and D.

trendyline
B: ..
.
.. C:
C: .. D: .. [3]

(c) Grignard reagents, RMgX, can be prepared by the reaction of magnesium with
halogenoalkane, RX, using dry ether as the solvent.
1666 trendyline
VJC 2016 9647/02/PRELIM/16[Turn over
10
The mechanism of the reaction involves two steps.

Step 1: It involves the transfer of one electron from Mg to the halogen X followed
by the homolytic fission of the carbonhalogen bond. This forms MgX,
which is a radical.

R X + Mg R + Mg X

Step 2: MgX radical couples with the alkyl radical formed in step 1.

R + Mg X RMgX

(i) Draw curve arrows to show the movement of electrons in step 1 of the
mechanism given above.

[1]

(ii) Suggest which step in the mechanism is the ratedetermining step. Explain
your answer.

.. [1]

Grignard reagents can be used to prepare alcohols from carbonyl compounds as


shown by the reaction of CH 3 CH 2 MgBr with ethanal below:

O step I OMgBr step II OH


CH3CH2MgBr + CH3 C CH3 C H CH3 C H + Mg(OH)Br
H CH2CH3 CH2CH3

(iii) What type of reaction takes place in steps IandII?


stepI: ..
stepII: ....[2]

(iv)
trendyline
The variation of the rate constantant k with temperature for the reaction of
CH 3 CH 2 MgBr with ethanal is given below:
k E 1 1
ln 2 a ( )
k1 R T2 T1
1667 trendyline
VJC 2016 9647/02/PRELIM/16
11

whereE a denotes the activation energy in J mol1, R is the gas constant, and
T is the temperature in Kelvin.
Given that at 25 oC, the rate constant and activation energy of the reaction is
9.16 x 103mol1dm3 s1 and 102 kJ mol1 respectively.
Explain, with calculations, the effect of temperature change on the rate of the
reaction when the temperature is raised by 5 oC.

.. [2]

(v) Draw the structural formula of the Grignard reagent that will react with
butan2one to form 3methylpentan3ol in a similar twostep reaction.

[1]

(vi) Suggest why CH 3 CO(CH 2 ) 3 CH 2 Br and magnesium cannot be used to


prepare the Grignard reagent CH 3 CO(CH 2 ) 3 CH 2 MgBr.

...[1]

(vii) Ethanal is oxidised to ethanoic acid by acidified potassium dichromate.


Using data from the Data Booklet and the data below, calculate Eo cell of the
reaction.
Eo(CH 3 CO 2 H|CH 3 CHO) = +0.92 V
Hence, explain why the reaction takes place only in the presence of heat.

.. [2]

trendyline
[Total: 20]

5 (a) The relationship pV = nRT can be derived


de from the laws of mechanics by
assuming ideal behaviour for gases.
The graph represents the relationship between pV and p for a real gas at three
different temperatures, T 1 , T 2 and T 3 .
1668 trendyline
VJC 2016 9647/02/PRELIM/16[Turn over
12
pV

(i) Draw one line on the graph to show what the relationship should be for the
same amount of an ideal gas and state which of T 1 , T 2 or T 3 is the lowest
temperature.

Lowest Temperature: .. [1]

(ii) State and explain the effect of pressure on the extent to which a gas
deviates from ideal behaviour.

.. [1]

(b) A flask with a volume of 100 cm3 was first weighed with air filling the flask.The
same flask was then weighed withonly gas, Y, filling the flask. The results,
measured at 26oC and 1.00 105 Pa, are shown.

Mass of flask containing air = 47.930 g


Mass of flask containing Y = 47.989 g
Density of air = 0.00118 g cm3

Calculate the relative molecular mass, M r , of Y.

trendyline
[2]

(c) In recent
ent years there has been worldwide interest in the possible
cent possib extraction of
shale gas (a form of natural gas) as an important energy source.

One of the problems associated with using shale gas is its variable composition.
Table 1 shows the percentage composition of shale gas from four different
sources J, K, L and M.
1669 trendyline
VJC 2016 9647/02/PRELIM/16
13

Table 1
source CH 4 C2Hx C3Hy CO 2 N2
J 80.3 8.1 2.3 1.4 7.9
K 82.1 14.0 3.5 0.1 0.3
L 88.0 0.8 0.7 10.4 0.1
M 77.5 4.0 0.9 3.3 14.3
In the formula above, x and y are variables.

(i) Draw the structures of three possible compounds with the formula C 3 H y .

[1]

(ii) Which source of shale gas, J, K, L or M, will provide the most energy when
burned? Explain your answer.

.. [1]

(iii) Suggest one method by which carbon dioxide can be removed from shale
gas.

.. [1]

Table 2 shows a comparison of the relative amounts of pollutants produced when


shale gas, fuel oil and coal are burned to produce the same energy.

Table 2
air pollutant shale gas fuel oil coal
CO 2 117 164 208
CO 0.040 0.033 0.208
NO 2 0.092 0.548 0.457
SO 2 0.001 1.12 2.59
particulates 0.007 0.84 2.74

(iv) Suggest why shale gas produces the smallest amount of CO 2 .

.. [1]

(v)
trendyline
Explain which of the three fuels, shale gas, fuel oil or coal,
contributor
ontributor to acid rain.
rain


coa is the largest

.. [1]
1670 trendyline
VJC 2016 9647/02/PRELIM/16[Turn over
14
(d) Use the following data and data from the Data Booklet to construct a labelled
energy level diagram to determine the lattice energy of Fe 2 O 3 .

enthalpy change of 4Fe(s) + 3O 2 (g) 2Fe 2 O 3 (s) 1644 kJ mol1


enthalpy change of atomisation of Fe(s) +414 kJ mol1
electron affinity of the oxygen atom 141 kJ mol1
electron affinity of the O ion +844 kJ mol1

Energy

[4]

[Total: 13]

6 (a)

trendyline
Coffee
e beans contain chlorogenic acid.

1671 trendyline
VJC 2016 9647/02/PRELIM/16
15

OH

O O OH
O OH

HO OH
HO
chlorogenic acid

(i) How many moles of H 2 (g) will be evolved when 1 mol of chlorogenic acid
reacts with an excess of sodium metal?

.. [1]

On heating with dilute aqueous acid, chlorogenic acid produces two compounds D
and E.
OH
+ OH OH
chlorogenic H (aq) O
+
acid heat HO
HO OH O OH
OH
D E
conc. H2SO4 excess
heat Br2(aq)

F G
C7H6O3

When compound D is heated with concentrated H 2 SO 4 , compound F, C 7 H 6 O 3 , is


formed.
Compound F does not decolourise cold dilute acidified KMnO 4 but reacts with
Br 2 (aq).
When compound E is treated with an excess of Br 2 (aq), compound G is produced.
(ii) If the test with cold dilute acidified KMnO 4 had been positive, which
functional group would this have shown to be present in F?
.. [1]

(iii) Suggest the structures for compounds F and G, and draw them in the
relevant boxes below.

trendyline
y C7H6O3
F G
[2]

(b) Allyl alcohol, CH 2 =CHCH 2 OH, is a colourless liquid which is soluble in water.
Crotyl alcohol, CH 3 CH=CHCH 2 OH,1672
is a colourless liquid. trendyline
VJC 2016 9647/02/PRELIM/16[Turn over
16
(i) Describe how you would distinguish between allyl alcohol and crotyl alcohol.
The compounds may be distinguished by a preliminary chemical reaction
followed by a subsequent testing of the reaction products. Include clearly the
reagents, conditions and observations for each compound.



.. [2]

(ii) Allyl alcohol may be converted into propanal by using a ruthenium(IV)


catalyst in water.
ruthenium(IV)
CH 2 =CHCH 2 OH CH 3 CH 2 CHO
State the type of reaction and explain your answer.

[1]

[Total: 7]

trendyline
1673 trendyline
VJC 2016 9647/02/PRELIM/16
2016 H2 Chemistry Prelim Exam Paper 2 Suggested Answers

1 Planning (P)

When potassium manganate(VII) reacts with sodium ethanedioate, a redox reaction occurs as
shown below:

2MnO 4 + 5C 2 O 4 2 + 16H+ 2Mn2+ + 10CO 2 + 8H 2 O reaction 1

A product that is generated during the course of the reaction and helps to speed up the
reaction rate is known as an autocatalyst. The autocatalyst for the above reaction is Mn2+.

The kinetics of this reaction can be investigated by determining the concentration of MnO 4
over the course of the reaction. Fixed aliquots (portions) of the reaction mixture are withdrawn
at regular time intervals and added to an excess of potassium iodide.

TheMnO 4 in the aliquots reacts with excess iodide as shown below.

2MnO 4 + 10I + 16H+ 2Mn2+ + 5I2 + 8H 2 O reaction 2

The amount of iodine formed can then be determined by titration with the sodium thiosulfate
solution as shown below.
I 2 + 2S 2 O 3 2 2I + S 4 O 6 2 reaction 3

The volume of sodium thiosulfate used is proportional to the concentration of MnO 4 .

(a) In this experiment, both sodium ethanedioate and sulfuric acid are used in large excess.
Explain the purpose of using a large excess of sodium ethanedioate and sulfuric acid.

This is to ensure that the concentration of sodium ethanedioate and sulfuric


acid remains almost constant throughout the reaction / concentration do not
affect rate. Hence, the order of reaction with respect to MnO 4 can be determined.

(b) Using the information given above, you are required to write a plan for the determination
of the concentration of MnO 4 at regular timing intervals.

You may assume that you are provided with the following:
0.0500 moldm3 potassium manganate(VII), KMnO 4
0.500 moldm3 sodium ethanedioate, Na 2 C 2 O 4
1.00 moldm3 sulfuric acid, H 2 SO 4
0.200 moldm3 potassium iodide, KI
solid Na 2 S 2 O 3
starch indicator
stopwatch
apparatus and chemicals normally found in a school or college laboratory.

Your plan should include:


justification for
o the volume of each reactants to be used in reaction 1 so as to allow a
minimum of 6 aliquots of the reaction mixture to be withdrawn and the

trendyline
concentration
ncentration
e of MnO 4 in each aliquot
liquot to be determined;
determined
det ;
o the intended concentration of the sodium thiosulfate, Na N 2 S 2 O 3 , to be used;
brief, but specific, details of the apparatus you would use, bearing
bear in mind the
levels of precision they offer;
an outline
tline
ine of how the results would be obtained;
obtained
a sketchch of the graph you would expectt to obtain
obtain.
[10]

1674 trendyline
VJC 2016 9647/02/PRELIM/16[Turn over
2
Justification
Assuming 10 cm3 aliquot is withdrawn for each titration, minimum volume
required for reaction 1 is 60 cm3.
Volume of MnO 4 used = 25.0 cm3
Volume of Na 2 C 2 O 4 used = 50.0 cm3
Volume of H 2 SO 4 used = 25.0 cm3
Total volume used for reaction 1 = 100 cm3
for all the volumes used.

Calculation for concentration of Na 2 S 2 O 3 to be used


25.0
Amount of MnO 4 present in reaction 1 =0.0500 x
1000
= 1.25 x 103mol

10.0
Amount of MnO 4 present in 10 cm3 aliquot =1.25 x 103 x
100
4
=1.25 x 10 mol

5
Amount of I 2 produced in reaction 2 =1.25 x 104 x
2
=3.13 x 104mol

Amount of S 2 O 3 2 required in reaction 3 =3.125 x 104 x 2


=6.25 x 104mol

Assuming a titre volume of 25.00 cm3 (acceptable range 1030 cm3)


25.00
[S 2 O 3 2] used for titration =6.25 x 104
1000
=0.0250 moldm3

To prepare a standard solution of 100 cm3 Na 2 S 2 O 3


100
Mass of Na 2 S 2 O 3 used =0.0250 x x (158.2)
1000
=0.396 g

Preparation of Na 2 S 2 O 3 standard solution


1. Weigh accurately about 0.315 g of Na 2 S 2 O 3 into a beaker.
2. Add distilled water to the beaker to dissolve Na 2 S 2 O 3 and transfer the
solution quantitatively into a 100 ml standard flask.
3. Add distilled H 2 O to the mark and shake the flask to give a homogenous
solution.

Preparation of reaction mixture


1. Using a 100 cm3measuring cylinder, add 50.0 cm3 of Na 2 C 2 O 4 and 25 cm3
H 2 SO 4 into a conical flask.
2. Add 25 cm3 MnO 4 to a 50 cm3measuring cylinder.
3. Pour MnO 4 to the reaction mixture in the conical flask and start the
stopwatch immediately.

ttrendyline
end
ndyyline
l
Ensure thorough g mixing g by
y swirling
g the flask.
(marking
king point for starting stopwatch is awarded only if MnO Mn 4 is added as
st rre
the last agent.)
reagent.)
4. out 1 min, pipette 10.0 cm3 of the reaction mixture into a conical
At about
bout
ontaining 10 cm3 of KI solution
flask containing solution.. Label conical flask 1.
solut
5. Repeatat step 4 five more times at aboutout two minute intervals, ttransferring the
aliquots into 5 separate conical flasks. Label conical flask 2 to 6.

1675 trendyline
VJC 2016 9647/02/PRELIM/16
3

Titration with sodium thiosulfate


1. Fill a 50 cm3 burette with Na 2 S 2 O 3 .
2. Titrate the iodine in conical flask 1 against Na 2 S 2 O 3 .
3. When the colour of the solution turns pale yellow, add about 1 cm3
of starchindicator. The solution will turn blueblack.
4. Continue the titration and the endpoint is reached when the blue
black colour just disappears.
3. Repeat step 2 to 4 for conical flask 2 to 6.

Shape of graph expected

Volume of S2O32- /
cm3

time / min

[12, max 10]

(c) The order of reaction with respect to MnO 4 can be determined by initial rate method.
Use your graph, suggest how the initial rate can be determined.
As shown in the graph in (b), draw the tangent and then determine the gradient
which gives the initial rate of reaction.
[1]
[Total: 12]

2 (a) Proline (pro) is one of the naturally occurring amino acids. It can be synthesised
from 4hydroxybutanal in 4 steps.
step I step II OH step III step IV O
HO CHO E HO C H F
OH
CO2H
4 hydroxybutanal NH
proline
(i) Suggest the structures of Eand F.
E F

OH Cl
HO C H Cl C H
CN CO2H

[2]

(ii) Suggest the reagents and conditions required for the following
followin steps.
I: HCN
Step I: CN with KCN catalyst, room temperature
Step III: concHCl,
concHCl ZnCl 2 , heat
heat,
Step IV: NH 3 (alc) heat in a sealed tube
[3]

1676 trendyline
VJC 2016 9647/02/PRELIM/16[Turn over
4

(b) The two pK a values associated with proline are2.0 and 10.5.
Make use of these pK a values to suggest the major species present in the solution
of proline with the following pH values.
pH 1 pH 7 pH 12
O O O
- -
OH O O
+ +
NH2 NH2 NH

[3]

(c) The structures ofamino acids, valine (val) and cys are shown below.

O O

OH HS OH
NH2 NH2
valine cysteine

(i) Draw the structure of valprocys peptide.

SH
O
N OH
H2N N
H
O O

[2]

(ii) State the possible R group interactions that valprocys may have with
another valprocys peptide.

Disulfide bond formed between the CH 2 SH in the cys groups.


van der waals forces formed between the nonpolar CH(CH 3 ) 2 groups
in the valine or the nonpolar CH 2 CH 2 CH 2 in the proline.
[2]
[Total: 12]

3 (a) A student used the following setup in an attempt to produce chlorine gas using
potassium chloride solution.

graphite graphite
salt bridge graphite electrode B electrode A
Zn electrode

ttrendyline
dy ne
1.00 mol dm3
Zn2++ ((aq)

half-cell
-cell 1
1.00 mol dm3
Co2++ (aq), Co3++ ((aq)
ha -cell 2
half-cell
1 mol dm3
200 cm3 of 1.0
KCl (aq)

1677 trendyline
VJC 2016 9647/02/PRELIM/16
5

(i) Using relevant data from the Data Booklet, predict the type of redox reaction
that occurs at each halfcell. State the polarity at the graphite electrodes A
and B.
Halfcell 1: oxidation
Halfcell 2: reduction
Graphite electrode A: negative (reduction occurs)
Graphite electrode B: positive (oxidation occurs)
[2]

(ii) 20 cm3of hydrogen gas was collected on one of the electrodes after 10
minutes at room temperature and pressure. Assuming 20% of the hydrogen
gas was lost during collection, calculate the current produced by the battery
setup.
Graphite electrode A: 2H 2 O + 2e H 2 + 2OH
Amount of hydrogen gas collected = 20 / 24000 = 8.33 x 104mol

Actual amount of hydrogen produced= (8.33 x 104) x= 1.04 x 103mol
Number of moles of electrons transferred = 2 x (1.04 x 103)
= 2.08 x 103mol
Q = It
2.08 x 103 x (96500) = I x (10 x 60)
Current, I = 0.335 A
[2]

(iii) With reference to relevant data from the Data Booklet, explain the absence
of chlorine being discharged on the other electrode.
O 2 + 4H+ + 4e 2H 2 O Eo = +1.23 V
Cl 2 + 2e 2Cl Eo = +1.36 V
At the anode, Eo(O 2 |H 2 O) is less positive than Eo(Cl 2 |Cl), so H 2 O will
undergo oxidation preferentially to give O 2 .
[1]

(b) 20 cm3 of a gas mixture containing gaseous ethanol, carbon monoxide and excess
oxygen was burned completely. There was a contraction of 1 cm3. When the
product mixture was passed through sodium hydroxide, there was a further
contraction of 6 cm3. All gas volumes are measured at 400 K and 1 atm.

Determine the molar composition of the ethanol and carbon monoxide in this gas
mixture.
Let the volume of C 2 H 5 OH and CO measured at 1 atm be x cm3and y cm3,
respectively.

C 2 H 5 OH + 3O 2 2CO 2 + 3H 2 O
c/cm3 x 3x +2x +3x
Total change in volume = 2x + 3x x 3x = x

2CO + O 2 2CO 2
c/cm3 y 1/2y +y
Total change in volume = y y y = y
Hence x y = 1

trendyline
The further
rther contraction is due to the removal of CO 2 by NaOH,
urther
Hence,e, 2x + y = 6
x = 1, y =4

Molarr composition of ethanol = 1/20 x 100% = 5%


Molarr composition of CO = 4/20 x 100%
0% = 20%

[3]
[Total: 8]
1678 trendyline
VJC 2016 9647/02/PRELIM/16[Turn over
6
4 Magnesium and beryllium are Group II elements but beryllium behaves differently from
that of magnesium. There is said to be a diagonal relationship between beryllium and
aluminium as they show similar chemical behaviour due to their similarities in
electronegativity and charge density.

(a) (i) When a few drops of water are added to solid beryllium chloride, steamy
white fumes are evolved and a white solid remains, which is insoluble in
water.
Write a balanced equation for this reaction.
BeCl 2 (s) + H 2 O(l) BeO(s) + 2HCl(g)
OR BeCl 2 (s) + 2H 2 O(l) Be(OH) 2 (s) + 2HCl(g)
[1]

(ii) When a large amount of water is added to solid beryllium chloride, a clear,
weakly acidic solution is obtained. Explain.
Due to the high charge density of Be2+, BeCl 2 undergoes hydration
and hydrolysis/ Be2+ (aq) pulls electrons away from one of the
surrounding water molecules, hence weakening the OH bond which
results in the release of a proton and give rise to an acidic solution.
[1]

(iii) At 750 oC, the relative molecular mass of gaseous beryllium chloride
corresponds to the formula BeCl 2 . At 550 oC, gaseous beryllium chloride
exists as a mixture of BeCl 2 and Y (relative molecular mass of Y is 160).

Determine the molar composition of gaseous beryllium chloride at 550 oC


which has a relative molecular mass of 100. Draw a diagram to illustrate the
nature of the bonding in Y and indicate the value of the bond angle about Be.
100 = 80x + 160(1x)
x = 0.75 and 1x = 0.25
Composition is 75% BeCl 2 and 25% Y

Cl
Cl Be Be Cl
Cl
Trigonal planar with respect to Be, hence bond angle is 120o
[2]

(iv) When aluminium chloride, AlCl 3 , is mixed with ammonia, compound Z is


formed and the reaction is exothermic. The diagram below shows the
relationship between the heat evolved per mole of the mixture and the mole
fraction of AlCl 3 .

Heat evolved /
J mol1

0 0.5 1.0
Mole fraction of AlCl3

tre
re
Draw a diagram to show the bonding in a mo
molecule of compound
compo Z.
Diagram of Compound Z :
Cl
Cl H
Cll A
C All N H
Cll
C H
[1]

1679 trendyline
VJC 2016 9647/02/PRELIM/16
7

(v) Sketch a labelled diagram similar to (a)(iv) that you would expect to obtain
when AlCl 3 is replaced by BeCl 2 .

Heat evolved/
J mol1

0 0.33 1.0
Mole fraction of BeCl2

[1]

(vi) Write a balanced equation of a chlorination reaction in Organic Chemistry


that uses anhydrous BeCl 2 as a catalyst.
anhydrous BeCl2 Cl
+ Cl
2 room temp + HCl

[1]

(b) Magnalium is an alloy of aluminium and magnesium which is used in boat-building.


The diagram below shows some reactions of magnalium.

sample of HCl(aq) colourless excess NaOH(aq) white


magnalium solution ppt

filter

HCl(aq)
added
white dropwise colourless white
solid, filtrate, residue,
D C B

Identify B,C and D.

B : Mg(OH) 2
C : Al(OH) 4
D :Al(OH) 3
[3]

trendyline
1680 trendyline
VJC 2016 9647/02/PRELIM/16[Turn over
8
(c) Grignard reagents, RMgX, can be prepared by the reaction of magnesium with
halogenoalkane, RX, using dry ether as the solvent.

The mechanism of the reaction involves two steps.

Step 1: It involves the transfer of one electron from Mg to the halogen X followed
by the homolytic fission of the carbonhalogen bond. This forms MgX,
which is a radical.

R X + Mg R + Mg X

Step 2: MgX radical couples with the alkyl radical formed in step 1.

R + Mg X RMgX

(i) Draw curve arrows to show the movement of electrons in step 1 of the
mechanism given above.
R X Mg R + Mg X
[1]

(ii) Suggest which step in the mechanism is the ratedetermining step. Explain
your answer.
Step 1 is the ratedetermining step
Step 1 has higher activation energy as it involves breaking of covalent
bond.

[1]

Grignard reagents can be used to prepare alcohols from carbonyl compounds as


shown by the reaction of CH 3 CH 2 MgBr with ethanal below:

O step I OMgBr step II OH


CH3CH2MgBr + CH3 C CH3 C H CH3 C H + Mg(OH)Br
H CH2CH3 CH2CH3

(iii) What type of reaction takes place in steps IandII?


stepI : Nucleophilic addition
stepII : Hydrolysis or Acidbase reaction
[2]

trendyline
1681 trendyline
VJC 2016 9647/02/PRELIM/16
9

(iv) The variation of the rate constant k with temperature for the reaction of
CH 3 CH 2 MgBr with ethanal is given below:

k2 Ea 1 1
ln ( )
k1 R T2 T1

whereE a denotes the activation energy in J mol1, R is the gas constant, and
T is the temperature in Kelvin.

Given that at 25 0C, the rate constant and activation energy of the reaction is
9.16 x 103mol1dm3 s1 and 102 kJ mol1 respectively.

Explain, with calculations, the effect of temperature change on the rate of the
reaction when the temperature is raised by 5 oC.

k2 102000 1 1
ln 3
= ( )
9.16 10 8.31 303 298

k 2 = 1.81 x 102mol1 dm3 s1


A rise in temperature results in a larger rate constant. Since the
concentrations of the reactants are kept constant, a larger rate
constant indicates a faster reaction. (or rate constant doubles, hence
rate doubles)
[2]

(v) Draw the structural formula of the Grignard reagent that will react with
butan2one to form 3methylpentan3ol in a similar twostep reaction.
CH 3 CH 2 MgBr
[1]

(vi) Suggest why CH 3 CO(CH 2 ) 3 CH 2 Br and magnesium cannot be used to


prepare the Grignard reagent CH 3 CO(CH 2 ) 3 CH 2 MgBr.
CH 3 CO(CH 2 ) 3 CH 2 MgBr cannot be prepared as it will react further with
the ketone group in the same molecule.
[1]

(vii) Ethanal is oxidised to ethanoic acid by acidified potassium dichromate.


Using data from the Data Booklet and the data below, calculate Eo cell of the
reaction.

Eo(CH 3 CO 2 H|CH 3 CHO) = +0.92 V

Hence, explain why the reaction takes place only in the presence of heat.
Eo cell = 1.33 0.92
= 0.41 V
Eo cell > 0, hence reaction is thermodynamically feasible.
Heat is needed to overcome the high activation energy needed
tobreak strong covalent bonds.
(or reaction is kinetically not feasible in the absence of heat as it

trendyline
d li
involves
nv breaking of strong
g covalent bonds)
bonds
bon )
[2]
[Total: 20]

1682 trendyline
VJC 2016 9647/02/PRELIM/16[Turn over
10
5 (a) The relationship pV = nRT can be derived from the laws of mechanics by
assuming ideal behaviour for gases.

The graph represents the relationship between pV and p for a real gas at three
different temperatures, T 1 , T 2 and T 3 .

pV

(i) Draw one line on the graph to show what the relationship should be for the
same amount of an ideal gas and state which of T 1 , T 2 or T 3 is the lowest
temperature.
T 1 is the lowest temperature.
[1]

(ii) State and explain the effect of pressure on the extent to which a gas
deviates from ideal behaviour.
A gas deviates from ideal behaviour at high pressure as the molecular
volume is significant / no longer negligible OR the intermolecular
forces of attraction are significant.
[1]

(b) A flask with a volume of 100 cm3 was first weighed with air filling the flask.The
same flask was then weighed withonly gas, Y, filling the flask. The results,
measured at 26oC and 1.00 105 Pa, are shown.

Mass of flask containing air = 47.930 g


Mass of flask containing Y = 47.989 g
Density of air = 0.00118 g cm3

Calculate the relative molecular mass, M r , of Y.

Mass of air = 0.00118 100 = 0.118 g


Mass of flask = 47.930 0.118 = 47.812 g
Mass of Y = 47.989 47.812 = 0.177 g
0.177
1.00 105 100 106 = 8.31 (26 + 273)
Mr
M r of Y = 44.0
[2]

trendyline
1683 trendyline
VJC 2016 9647/02/PRELIM/16
11

(c) In recent years there has been worldwide interest in the possible extraction of
shale gas (a form of natural gas) as an important energy source.

One of the problems associated with using shale gas is its variable composition.
Table 1 shows the percentage composition of shale gas from four different
sources J, K, L and M.

Table 1

source CH 4 C2Hx C3Hy CO 2 N2


J 80.3 8.1 2.3 1.4 7.9
K 82.1 14.0 3.5 0.1 0.3
L 88.0 0.8 0.7 10.4 0.1
M 77.5 4.0 0.9 3.3 14.3
In the formula above, x and y are variables.

(i) Draw the structures of three possible compounds with the formula C 3 H y .

CHand CH 2 CCH 2 [Any 3 of 4.]


CH 3 CH 2 CH 3 , CH 3 CH=CH 2 , CH 3 C
[1]

(ii) Which source of shale gas, J, K, L or M, will provide the most energy when
burned? Explain your answer.

Source K
Souce K has the highest percentage composition of hydrocarbons
which will provide the most energy when burned.
[1]

(iii) Suggest one method by which carbon dioxide can be removed from shale
gas.

Carbon dioxide can be removed from shale gas by passing it through


aqueous sodium hydroxide.
[1]

Table 2 shows a comparison of the relative amounts of pollutants produced when


shale gas, fuel oil and coal are burned to produce the same amount of energy.

Table 2

air pollutant shale gas fuel oil coal


CO 2 117 164 208
CO 0.040 0.033 0.208
NO 2 0.092 0.548 0.457
SO 2 0.001 1.12 2.59
particulates 0.007 0.84 2.74

(iv) Suggest why shale gas produces the smallest amount of CO 2 .


Shale gas contains small hydrocarbon moleculeswith low C:H

ttrendyline
ratiounlike
ratio
at unlike fuel oil and coal
al which contain
cont more carb
carbon atoms, and
thus produces the smallest amount of CO 2 .
[1]

(v) Explain
xplain which of the three fuels, shale gas, fuel oil or coal,
ccoa is the largest
t ib t to
contributor t acid
id rain.
i
coal
Coal produced the largest relative amounts of CO 2 , NO 2 and SO 2 when
burned to produce the same amount of energy.
1684 [1]
trendyline
VJC 2016 9647/02/PRELIM/16[Turn over
12

(d) Use the following data and data from the Data Booklet to construct a labelled
energy level diagram to determine the lattice energy of Fe 2 O 3 .

enthalpy change of 4Fe(s) + 3O 2 (g) 2Fe 2 O 3 (s) 1644 kJ mol1


enthalpy change of atomisation of Fe(s) +414 kJ mol1
electron affinity of the oxygen atom 141 kJ mol1
electron affinity of the O ion +844 kJ mol1

Energy

2Fe3+(g) + 3O2(g)

3(844) = 2532
2Fe3+(g) + 3O(g) + 6e

3/2(496) = 744 3(141) = 423


2Fe3+(g) + 3O(g) + 3e
2Fe3+(g) + 3/2 O 2 (g) + 6e

2(762 + 1560 + 2960)


= 10564

LE

2Fe(g) + 3/2 O 2 (g)

2(414) = 828

2Fe(s) + 3/2 O 2 (g)


0
1644/2 = 822

Fe 2 O 3 (s)

By Hess law,
822 = 828 + 10564 + 744 423 + 2532 + LE (ecf)
LE = 15067 1.51 x 104 kJ mol1
[5, max 4]
[4]
[Total: 13]

6 (a) Coffee beans contain chlorogenic acid.

OH

O O OH
O OH

trendyline
d
HO OH
HO
HO
chlorogenic
hlorogenic acid
ac

(i) How many moles of H 2 (g) will be e


evolved when 1 mol of cchlorogenic acid
reacts with an excess of sodium metal?
3 mol
[1]
1685 trendyline
VJC 2016 9647/02/PRELIM/16
13

On heating with dilute aqueous acid, chlorogenic acid produces two compounds D
and E.

OH
+ OH OH
chlorogenic H (aq) O
+
acid heat HO
HO OH O OH
OH
D E
.
conc H2SO4 excess
heat Br2(aq)

F G
C7H6O3

When compound D is heated with concentrated H 2 SO 4 , compound F, C 7 H 6 O 3 , is


formed.

Compound F does not decolourise cold dilute acidified KMnO 4 but reacts with
Br 2 (aq).

When compound E is treated with an excess of Br 2 (aq), compound G is produced.

(ii) If the test with cold dilute acidified KMnO 4 had been positive, which
functional group would this have shown to be present in F?

C=C bond OR alkene functional group


[1]

(iii) Suggest the structures for compounds F and G, and draw them in the
relevant boxes below.

OH OH OH Br
O OH
HO2C
Br
Br OH
OR Br
HO2C OH
C7H6O3
F G
[2]

(b) Allyl alcohol, CH 2 =CHCH 2 OH, is a colourless liquid which is soluble in water.

Crotyl alcohol, CH 3 CH=CHCH 2 OH, is a colourless liquid which is used as a


solvent.

(i) Describe how you would distinguish between allyl alcohol and crotyl alcohol.

trendyline
p
The compounds mayy be distinguished
g byy a preliminary
p y chemical reaction
followed by a subsequent testing of the reaction products. In
Include clearly the
reagents, conditions and observations for each compound.

Step I: KMnO 4 , NaOH(aq),


Na
NaOH(aq), cold
col
Step II: I 2 , NaOH(aq), warm
Crotyl alcohol gives yellow ppt.
Allyl alcohol does not give yellow ppt.
[2]
1686 trendyline
VJC 2016 9647/02/PRELIM/16[Turn over
14

(ii) Allyl alcohol may be converted into propanal by using a ruthenium(IV)


catalyst in water.

ruthenium(IV)
CH 2 =CHCH 2 OH CH 3 CH 2 CHO

State the type of reaction and explain your answer.

It is a disproportionation reaction where the C=C bond is reduced and


the primary alcohol is oxidised.
[1]

[Total: 7]

trendyline
1687 trendyline
VJC 2016 9647/02/PRELIM/16

 
9,&725,$-8
81,25&2
2//(*(
-&35(/,0,1$5<(
(;$0,1$
$7,216
+LJKH
HU




&+(0,6
675< 


3DSHU)UUHH5HVSRQ
QVH 6HSWH
HPEHU
 
&DQGLGDWHVVDQVZHURQ
QVHSDUDWHSDSHU KRXUUV

$GGLWLRQDO0DWHULDOV $QVZHUU3DSHU
'DWD%R
RRNOHW



5($'7+
+(6(,167
758&7,21
16),567

:ULWH\RXUQDPHDQG&7JURXSR RQDOOWKHZRRUN\RXKDQ QGLQ
DUNEOXHRUE
:ULWHLQGD EODFNSHQRQERWKVLGHVVRIWKHSDS SHU
<RXPD\X XVHDVRIWSHHQFLOIRUDQ\\GLDJUDPVJUDSKVRUURXJKZRUNNLQJ
'RQRWXVHHVWDSOHVSD DSHUFOLSVK
KLJKOLJKWHUVVJOXHRUFR
RUUHFWLRQIOXLLG

$QVZHUDQ Q\IRXUTXHVVWLRQV
$'DWD%RR RNOHWLVSURYYLGHG
<RXDUHUHHPLQGHGRIWWKHQHHGIRUJRRG(QJOLVKDQGFOH HDUH[SODQDWWLRQLQ\RXUDQVZHUV

7KHQXPEH HURIPDUNVLVJLYHQLQEUDFNHWV>@DWWKHHQGGRIHDFKTXHVWLRQRUSSDUWTXHVWLR
RQ



t endyylin
trendyline
tre nee




 7KLV
KLVVGRFXPHQW
VGRFXPHQWFRQVLVWVRIISULQWHG
ISULQWHG
HGSDJHVDQG
EODQNSDJH
H 



1688 trendyline

9-&35(/,0 XUQRYHU
>7X


 D  $WKLJKWHPSHUDWXUHV12UHDFWVZLWK+WRSURGXFHQLWURXVR[LGH12DJUHHQKRXVHJDV

12 J + J 12 J +2 J 

7RVWXG\WKHNLQHWLFVRIWKLVUHDFWLRQDW&LQLWLDOUDWHVIRUWKHIRUPDWLRQRI12ZHUH
PHDVXUHGXVLQJYDULRXVLQLWLDOSDUWLDOSUHVVXUHVRI12DQG+

,QLWLDOSUHVVXUHN3D ,QLWLDOUDWHRI
([SHULPHQW SURGXFWLRQRI12
P12  P+  N3DV
   

   

   

L  'HGXFHWKHUDWHHTXDWLRQLQWHUPVRISDUWLDOSUHVVXUH    >@

LL  8VLQJWKHUDWHHTXDWLRQDQGWKHLQIRUPDWLRQLQWKHWDEOHDERYHFDOFXODWHDYDOXHIRU
WKHUDWHFRQVWDQWJLYLQJLWVXQLWV       >@

LLL  +HQFHFDOFXODWHWKHLQLWLDOUDWHDWZKLFK12UHDFWVZKHQN3DRI12DQGN3D
RI+DUHPL[HGDW&       >@

LY  :ULWH WKH UDWH HTXDWLRQ IRU WKH UHDFWLRQ ZKHQ 12 LV XVHG LQ ODUJH H[FHVV +HQFH
GHWHUPLQH WKH WLPH HODSVHG WR UHGXFH WKH SDUWLDO SUHVVXUH RI + WR KDOI RI LWV LQLWLDO
YDOXHLIN3DRI12DQGN3DRI+DUHPL[HGDW&   >@

Y  7ZRPHFKDQLVPV$DQG%DUHSURSRVHGEHORZ

$ 12 12

   12+ +212


% 12+ +12

   +12 12+2

:KLFKPHFKDQLVPLVFRQVLVWHQWZLWKWKHUDWHHTXDWLRQREWDLQHGLQ D L ([SODLQ\RXU
DQVZHU           >@


E  3RO\QLWURJHQ FRPSRXQGV KDYH JUHDW SRWHQWLDO IRU EHLQJ XVHG DV KLJK HQHUJ\ GHQVLW\
PDWHULDOV 7KH\ DUH WKHUPRG\QDPLFDOO\ XQVWDEOH +XJH DPRXQWV RI HQHUJ\ DUH UHOHDVHG
IURPWKHLUGHFRPSRVLWLRQRUUHDFWLRQVOHDGLQJWRPRUHVWDEOHSURGXFWV

L  7KHSRO\QLWURJHQVSHFLHV1ZDVILUVWLVRODWHGLQ2QHRILWVUHVRQDQFHIRUPVLV
VKRZQEHORZ

trendyline
d i


WKHERQGDQJOHDQGPROHFXODUVKDSHDURXQGQLWURJHQODEHOO
KHERQGDQJOHDQGPROHFXODUVKDSHDURXQGQLWURJHQODEHOO
R DS
S RX LW HG
6WDWHWKHERQGDQJOHDQGPROHFXODUVKDSHDURXQGQLWURJHQODEHOOHGa  >@

ROLG >1@>$V)@ ZDV DFKLHYHG
LL  7KH V\QWKHVLV RI D ZKLWH LRQLF VROLG DFK E\ UHDFWLQJ
>1)@>$V)@ZLWKK\GUD]RLFDFLG+1LQOLTXLG+)DWR&
:ULWHDEDODQFHGHTXDWLRQIRUWKLVUHDFWLRQ      >@

1689 trendyline
9-&35(/,0


LLL  >1)@>$V)@ ZDV ILUVW SUHSDUHG E\ UHDFWLQJ 1) ZLWK WKH VWURQJ /HZLV DFLG $V)
1)KDVWKHIROORZLQJVWUXFWXUHDQGH[KLELWVFLVWUDQVLVRPHULVP



'UDZ WKH FLVWUDQV LVRPHUV RI 1) LQFOXGLQJ DOO ORQH SDLUV DQG VXJJHVW DQ
DSSURSULDWHK\EULGLVDWLRQIRUHDFKQLWURJHQDWRPLQ1)    >@

 
F  1LWURJHQLVIRXQGLQDOORUJDQLVPVSULPDULO\LQDPLQRDFLGVZKLFKPDGHXSSURWHLQV

,VROHXFLQH1+&+ &+ &+ &+&+ &2+LVDQDPLQRDFLGZKHUHWKHDPLQRJURXSDQG
FDUER[\OLFDFLGJURXSDUHVHSDUDWHGE\RQHFDUERQ,WLVDFRQVWLWXHQWDPLQRDFLGLQPDQ\
SURWHLQPROHFXOHV,VROHXFLQHFDQH[LVWDVGLIIHUHQWVWHUHRLVRPHUV

L  6WDWH WKH UHDJHQWV DQG FRQGLWLRQV QHHGHG WR EUHDN SURWHLQ GRZQ LQWR LWV FRQVWLWXHQW
DPLQRDFLGV          >@

LL  'HILQHWKHWHUPstereoisomers       >@

LLL  7KHUH DUH IRXU RSWLFDO LVRPHUV RI LVROHXFLQH 'UDZ WKH VWUXFWXUHV RI WKHVH IRXU
VWHUHRLVRPHUVRILVROHXFLQHLQGLFDWLQJFOHDUO\WKHFKLUDOFHQWUHV   >@


G  DPLQRPHWK\OEXW\QH &+ &&1+ &+  LV XVHG DV WKH VWDUWLQJ PDWHULDO WR PDNH
RWKHUFKHPLFDOV
'UDZDODEHOOHGGLDJUDPWRVKRZWKHRUELWDOVWKDWIRUP& &LQDQDON\QH  >@

>7RWDO@


 

trendyline
1690 trendyline
9-&35(/,0>7XUQRYHU


 D  6W\UHQH SKHQ\OHWKHQH  LV D XVHIXO RUJDQLF LQWHUPHGLDWH XVHG IRU WKH V\QWKHVLV RI PDQ\
SURGXFWV)RULQVWDQFHVW\UHQHLVDEOHWRXQGHUJRWKHUHDFWLRQVKRZQEHORZ




L  6WDWHWKHUHDJHQWVDQGFRQGLWLRQVUHTXLUHGIRUUHDFWLRQV,,,DQG,,,  >@

LL  %\FRQVLGHULQJWKHVXEVWLWXHQW V DWWDFKHGWRWKHFDUERQ\OJURXSJLYHWZRUHDVRQVIRU
WKHORZHUUHDFWLYLW\RISKHQ\OHWKDQRQHWRZDUGVQXFOHRSKLOHVFRPSDUHGWRSURSDQRQH
&+&2&+          >@

LLL  6XJJHVW D VLPSOH FKHPLFDO WHVW WR GLVWLQJXLVK EHWZHHQ LRGRHWK\O EHQ]HQH DQG
LRGREHQ]HQH&+,6WDWHWKHREVHUYDWLRQVIRUHDFKFRPSRXQG   >@
     
LY  7KH VROXELOLW\ RI LRGRHWK\O EHQ]HQH LQ ZDWHU ZDV IRXQG WR EH  PJGP ([SODLQ
ZK\ LRGRHWK\O EHQ]HQHKDVVXFKDORZVROXELOLW\LQZDWHU    >@


E  ,RGRHWK\O EHQ]HQHH[LVWVDVDSDLURIHQDQWLRPHUV$DQG%

$Q RSWLFDOO\ SXUH VDPSOH FRQWDLQLQJ RQO\ LVRPHU $ GLVVROYHG LQ KH[DQH XQGHUZHQW D
QXFOHRSKLOLF VXEVWLWXWLRQ UHDFWLRQ ZLWK D VROXWLRQ FRQWDLQLQJ UDGLRDFWLYH LRGLGH LRQV ,
$IWHUWKHUHDFWLRQRIWKH LRGRHWK\O EHQ]HQHLVLQFRUSRUDWHGZLWKUDGLRDFWLYHLRGLGH

$Q LQYHVWLJDWLRQ LQWR WKH NLQHWLFV RI WKH UHDFWLRQ UHYHDOHG WKDW WKH UDWH RI UHDFWLRQ ZDV
LQGHSHQGHQWRIWKHFRQFHQWUDWLRQRI,

L  'HVFULEH WKH PHFKDQLVP IRU WKH UHDFWLRQ RI LRGRHWK\O EHQ]HQH ZLWK , ,Q \RXU
DQVZHU FOHDUO\ ODEHO WKH , DQG LQFOXGH DQ\ UHOHYDQW FKDUJHV FXUO\ DUURZV DQG
GLSROHVRUORQHSDLUVRIHOHFWURQVWKDW\RXFRQVLGHULPSRUWDQWLQWKLVPHFKDQLVP >@

LL  &DOFXODWH WKH SHUFHQWDJH RI LRGRHWK\O EHQ]HQH WKDW H[LVWV DV HQDQWLRPHU
%LQFRUSRUDWHGZLWK,LQWKHSURGXFWPL[WXUH      >@


F  7KHWHUPWULLRGLGHUHIHUVWRPROHFXOHVWKDWFRQWDLQWKUHHLRGLQHDWRPVLQWKHLUPROHFXODU
IRUPXODVXFKDVQLWURJHQWULLRGLGH1,DQGDOXPLQLXPWULLRGLGH$l,

$l, KDV D VLPSOH PROHFXODU VWUXFWXUH ,Q WKH JDV SKDVH LW GLPHULVHV UHDGLO\ WR IRUP WKH
GLPHU$l,7KHWZRVSHFLHVDUHUHODWHGE\WKHHTXLOLEULXPVKRZQEHORZ

$l, J $l, J    H!

L  $  J VDPSOH RI $l2, ZDV DOORZHG WR YDSRXULVH LQ D  GP YHVVHO DW  R&
&DOFXODWHWKHLQLWLDOSUHVVXUHRI$l2,       >@

trendyline

$l2, ZDV  ILQG WWKH KS YDOXH IRU WKH
LL  *LYHQ WKDW WKH GHJUHH RI GLVVRFLDWLRQ  RI $l
ULXPDWR& 
EULXPDW
HTXLOLEULXPDW        >@





1691 trendyline
9-&35(/,0


LLL  ([SODLQWKHHIIHFWRQWKHGHJUHHRIGLVVRFLDWLRQRI$l2,IRUHDFKRIWKHFKDQJHVEHORZ

 7KHUHDFWLRQZDVFDUULHGRXWDWR& 
 $ VDPSOH RI $U J  ZDV LQWURGXFHG LQWR D GP YHVVHO FRQWDLQLQJ $l, J DQG
$l, J DWR&        >@


G  7KHWDEOHEHORZVKRZVWKHERLOLQJSRLQWVRI3&lDQG3)


&RPSRXQG %RLOLQJ3RLQWR&

3&l 
3) 

([SODLQZK\3&lKDVDKLJKHUERLOLQJSRLQWWKDQ3)     >@

    >7RWDO@


 

trendyline
1692 trendyline
9-&35(/,0>7XUQRYHU


 D  /HDG ,, QLWUDWHGHFRPSRVHVRQKHDWLQJLQWKHVDPHZD\DVPDJQHVLXPQLWUDWH%\XVLQJ
UHOHYDQWGDWDIURPWKHData BookletSUHGLFWDQGH[SODLQZKLFKRIWKHWZRQLWUDWHVOHDG ,, 
QLWUDWHRUPDJQHVLXPQLWUDWHZRXOGKDYHDKLJKHUGHFRPSRVLWLRQWHPSHUDWXUH  >@


E  7KHKDORJHQVDUHUHDFWLYHHOHPHQWVDQGDUHLQYROYHGLQPDQ\UHDFWLRQV

L  :KHQ FKORULQH LV EXEEOHG WKURXJK FROG VRGLXP K\GUR[LGH VROXWLRQ IROORZHG E\ DQ
H[FHVVRIDFLGLILHGVLOYHUQLWUDWHVROXWLRQRQO\  RIWKHFKORULQHZKLFKKDVGLVVROYHG
LVSUHFLSLWDWHGDVVLOYHUFKORULGH:KHQWKHVRGLXPK\GUR[LGHVROXWLRQLVKRWXSWR   
RIWKHFKORULQHFDQEHSUHFLSLWDWHG([SODLQWKHREVHUYDWLRQVZLWKWKHDLGRIUHOHYDQW
HTXDWLRQV          >@

LL  :KHQ FKURPLXP LV PDGH WR UHDFW ZLWK FKORULQH WKH PDLQ SURGXFW RI WKH UHDFWLRQ LV
&U&l +RZHYHU ZKHQ FKURPLXP LV PDGH WR VHSDUDWHO\ UHDFW ZLWK RWKHU KDORJHQV
VXFK DV IOXRULQH DQG LRGLQH WKH PDLQ SURGXFWV DUH &U) DQG &U, %\ UHIHUHQFH WR
UHOHYDQW ER YDOXHV ZKDW FDQ \RX FRQFOXGH DERXW WKH UHODWLYH R[LGLVLQJ DELOLW\ RI WKH
KDORJHQV"          >@

7KH KDORJHQV DOVR IRUP PDQ\ LQWHUKDORJHQ FRPSRXQGV LQ ZKLFK WZR GLIIHUHQW KDORJHQV
DUHFRPELQHG,QWHUKDORJHQFRPSRXQGVOLNH%U&lKDYHVLPLODUSURSHUWLHVWRWKHKDORJHQV

LLL  6XJJHVW DQ HTXDWLRQ IRU WKH UHDFWLRQ WKDW RFFXUV EHWZHHQ %U&l DQG DTXHRXV .,
            >@

LY  %U&lUHDFWVZLWKDONHQHVIDVWHUWKDQWKHSXUHKDORJHQVVXFKDV&lDQG%U6KRZWKH
VORZVWHSRIWKHUHDFWLRQEHWZHHQ%U&lDQGSURSHQH    >@
  

F  $OLWKLXPLRQEDWWHU\LVDUHFKDUJHDEOHEDWWHU\,QDIXOO\FKDUJHGEDWWHU\OLWKLXPLRQVRFFXS\WKH
WLQ\VSDFHVLQJUDSKLWHWRIRUP/L&'XULQJGLVFKDUJHOLWKLXPLRQVDUHSURGXFHGZKLFKPLJUDWH
WKURXJK WKH HOHFWURO\WH ZKLOH DW WKH VDPH WLPH JUDSKLWH LV EHLQJ UHJHQHUDWHG ,Q WKH RWKHU
HOHFWURGHOLWKLXPLRQVUHDFWZLWK&R2WRIRUP/L&R2


trendyline


HTXDWLRQVIRUWKHHOHFWURGHUHDFWLRQVRFFXUULQJDWWKHDQRGH
L  :ULWHHTXDWLRQVIRUWKHHOHFWURGHUHDFWLRQVRFFXUULQJDWWKHDQRGHDQGFDWKRGHGXULQJ
GLVFKDUJH
DUJH
UJH          >@

LL  +HQFHZULWHDQHTXDWLRQIRUWKHRYHUDOOUHDFWLRQWKDWRFFXUVGXULQJGLVFKDUJH
H ZULWH DQ HTXDWLRQ IRU WKH RYHUDOOO UHDFWLRQ
UH WKDW RFFXUV GXULQJ >@

LLL  %\GHWHUPLQLQJWKHR[LGDWLRQQXPEHUVRIWKHUHOHYDQWHOHPHQWVLGHQWLI\WKHHOHPHQWV
WKDWKDYHEHHQR[LGLVHGDQGUHGXFHG       >@
1693 trendyline
9-&35(/,0



G  L  :LWK UHIHUHQFH WR WKH Data Booklet GUDZ D IXOO\ ODEHOOHG GLDJUDP VKRZLQJ KRZ \RX
FRXOG PHDVXUH WKH VWDQGDUG HOHFWURGH SRWHQWLDO IRU WKH $J$J V\VWHP ,QGLFDWH WKH
GLUHFWLRQRIHOHFWURQIORZ        >@

LL  7KHHOHFWURGHSRWHQWLDOEDQGWKHFRQFHQWUDWLRQRIVLOYHULRQVLQVROXWLRQXQGHUQRQ
VWDQGDUGFRQGLWLRQVDUHUHODWHGE\WKHIROORZLQJHTXDWLRQ


E OJ 
> $J DT @ 



7KH VROXELOLW\ SURGXFW FRQVWDQW RI $J&l LV KVS     PRO GP 8VLQJ WKH
HTXDWLRQJLYHQDERYHFDOFXODWHWKHHOHFWURGHSRWHQWLDODW.RIDKDOIFHOOIRUPHG
E\

,  D$JHOHFWURGHLPPHUVHGLQDVDWXUDWHGVROXWLRQRI$J&l   

,,  D$JHOHFWURGHLPPHUVHGLQDPROGP VROXWLRQRI.&lFRQWDLQLQJVRPH$J&l
SUHFLSLWDWH          
           >@

LLL  $FHOOLVPDGHE\FRQQHFWLQJWKHKDOIFHOOLQ G L WRDVWDQGDUG&X&XKDOIFHOO%\
UHIHUHQFHWRUHOHYDQWGDWDIURPWKH'DWD%RRNOHWFDOFXODWHWKHFHOOHPIDQGZULWHDQ
HTXDWLRQ ZLWKVWDWHV\PEROV IRUWKHUHDFWLRQWKDWKDVRFFXUUHG   >@

>7RWDO@


 

trendyline
1694 trendyline
9-&35(/,0>7XUQRYHU


 D  &RPSRXQG*KDVWKHVWUXFWXUHVKRZQ



*LYHWKHVWUXFWXUHVRIWKHRUJDQLFFRPSRXQGVIRUPHGE\FRPSRXQG*XQGHUWKHIROORZLQJ
FRQGLWLRQV

L  1D2+ DT UHIOX[         >@

LL  ([FHVV&+&2&lURRPWHPSHUDWXUH       >@


E  /DFWLFDFLGFDQEHXVHGLQIRRGSURGXFWVDVS+UHJXODWRUVRUDVDIRRGSUHVHUYDWLYH,WFDQ
DOVREHXVHGDVDIODYRXULQJDJHQW

L  6XJJHVWUHDJHQWVDQGFRQGLWLRQVQHHGHGWRFRQYHUWSURSHQHLQWRK\GUR[\SURSDQRLF
DFLG FRPPRQO\NQRZQDVODFWLFDFLG LQQRPRUHWKDQVWHSV*LYHWKHVWUXFWXUHVRI
WKHLQWHUPHGLDWHV



            >@

LL  :KHQ ODFWLF DFLG LV UHIOX[HG ZLWK FRQFHQWUDWHG VXOIXULF DFLG FRPSRXQG + ZLWK WKH
PROHFXODU IRUPXOD RI &+2 LV IRUPHG ,W GRHV QRW UHDFW ZLWK VRGLXP *LYH WKH
VWUXFWXUDOIRUPXODRI+         >@

LLL  'HVFULEHDQGH[SODLQWKHUHODWLYHDFLGLW\RIWKHIROORZLQJWKUHHFRPSRXQGV  


 >@


F  ([SHULPHQW  LV FRQGXFWHG LQ ZKLFK WKH FRQGXFWLYLW\ RI  FPRI %D 2+  VROXWLRQ LV
PRQLWRUHGDVLWLVWLWUDWHGZLWKPROGPRIDTXHRXV+627KHGDWDFROOHFWHGIURP
WKHH[SHULPHQWLVSORWWHGLQWKHJUDSKEHORZ
&RQGXFWLYLW\

trendyline
endylin
ndylin
dyl
dyl
&RQ

   
   
9ROXPHRI+62 DGGHGFP



1695 trendyline
9-&35(/,0


L  7KH FRQGXFWLYLW\ RI WKH %D 2+  VROXWLRQ GHFUHDVHV DV WKH YROXPH RI PRO GP
+62LVDGGHGIURPFPWRFP

,GHQWLI\WKHFKHPLFDOVSHFLHVWKDWHQDEOHWKHVROXWLRQWRFRQGXFWHOHFWULFLW\DVWKHILUVW
FPRI+62 DT LVDGGHG+HQFHH[SODLQZK\WKHFRQGXFWLYLW\GHFUHDVHV >@

LL  8VLQJWKHLQIRUPDWLRQLQWKHJUDSKFDOFXODWHWKHLQLWLDOFRQFHQWUDWLRQRI%D 2+  >@
         

G  ,Q([SHULPHQWDQRWKHUFPVDPSOHRIPROGPDTXHRXV%D 2+ LVPL[HGZLWK
 FP RI  PRO GP RI HWKDQRLF DFLG WR IRUP VROXWLRQ $ %DULXP HWKDQRDWH
%D &+&2 LVWKHSURGXFWRIWKHUHDFWLRQ7KHDFLGGLVVRFLDWLRQFRQVWDQWKDRIHWKDQRLF
DFLGLVPROGP&DOFXODWHWKHS+RIWKHVROXWLRQ$    >@
  

H  +DOI RI WKH VROXWLRQ $ IRUPHG LQ G  LV DGGHG WR EHDNHU ; ZKHUH WKHUH LV D OD\HU RI
FRORXUOHVVRLODVVKRZQLQWKHILJXUHEHORZ


2LO

6ROXWLRQ$


 %HDNHU;

7KH LQGLFDWRU +,Q LV D ZHDN DFLG ZLWK D SKD YDOXH RI  ,W UHDFWV ZLWK ZDWHU DV
UHSUHVHQWHGLQWKHHTXDWLRQEHORZ

+,Q DT +2 O ,Q DT +2 DT 
 \HOORZ  EOXH

$VPDOODPRXQWRILQGLFDWRU+,QLVDGGHGLQWREHDNHU;7KHPL[WXUHVDUHVWLUUHGZHOODQG
WKHWZROD\HUVDUHDOORZHGWRVHSDUDWH

$VVXPHWKHS+RIVROXWLRQ$LVXQDIIHFWHGE\WKHSUHVHQFHRIFRORXUOHVVRLO

L  6WDWH WKH FRORXU REVHUYHG LQ WKH RLO OD\HU RI EHDNHU ; ([SODLQ WKH REVHUYDWLRQ LQ
WHUPVRIDFLGEDVHHTXLOLEULXPDQGLQWHUPROHFXODULQWHUDFWLRQV   >@

LL  $ IHZ GURSV RI DTXHRXV VRGLXP K\GUR[LGH DQG K\GURFKORULF DFLG DUH DGGHG WR WKH
EHDNHU;VXFFHVVLYHO\DQGVWLUUHGZHOO([SODLQZLWKWKHDLGRIHTXDWLRQ V ZK\WKHUHLV
QRFKDQJHLQWKHFRORXURIWKHRLOOD\HUDIWHUHDFKDGGLWLRQ   >@

>7RWDO@
 

1696 trendyline
9-&35(/,0>7XUQRYHU


 D  )HUURFHQH )H &+  LV DQ RUDQJH VROLG ,Q WKLV FRPSOH[ &+ LV WKH OLJDQG DQG LW
GRQDWHVHOHFWURQVIURPWKHULQJWRWKHYDFDQWGRUELWDORI)H7KHVWUXFWXUHRIIHUURFHQH
LVJLYHQEHORZ

)HUURFHQHFDQXQGHUJRDVHULHVRIGLIIHUHQWUHDFWLRQVDQGVRPHDUHDQDORJXHVRIRUJDQLF
UHDFWLRQV7ZRUHDFWLRQVRIIHUURFHQHDUHVKRZQEHORZ
 

 ,

 ,,


L  6WDWHWKHW\SHVRIUHDFWLRQIRUUHDFWLRQV,DQG,,     >@

LL  6WDWHWKHR[LGDWLRQQXPEHURI)HLQIHUURFHQHDQGKHQFHLWVHOHFWURQLFFRQILJXUDWLRQ
            >@

LLL  6XJJHVWZK\)HUURFHQHLVDFRORXUHGFRPSOH[     >@

LY  7KHFRPSOHPHQWDU\FRORXUVDUHLOOXVWUDWHGZLWKWKHIROORZLQJFRORXUZKHHO



JUHHQ \HOORZ

EOXH RUDQJH


 SXUSOH UHG



$TXHRXV)HLRQLVJUHHQLQFRORXU6XJJHVWDQGH[SODLQLIZDWHUFDXVHVDODUJHUVSOLW
EHWZHHQWKHWZRJURXSVRIGRUELWDOVDVFRPSDUHGWR&+   >@

Y  %\FRQVLGHULQJUHDFWLRQ,,VXJJHVWLIER )H &+ _)H &+  LVPRUHSRVLWLYHRUOHVV
SRVLWLYHWKDQER )H_)H ([SODLQ\RXUDQVZHU     >@

YL  7KHUHDFWLRQEHWZHHQ62DQG,LVVORZLQWKHDEVHQFHRIFDWDO\VW)HUURFHQHFDQ
EHDVXLWDEOHFDWDO\VWIRUWKLVUHDFWLRQEHWZHHQ62DQG,


trendyline
([SODLQZLWKWKHDLGRIHTXDWLRQVKRZLWZRUNV<RXPD\XVH)H &
QZLWKWKHDLGRIHTXDWLRQVKRZLWZRUNV<R
QZLWKWKHDLGRIHTXDWLRQVKRZLWZRUNV<RXPD\XVH)H + WRUHSUHVHQW
IHUURFHQH
HQH
HQH          >@

YLL %\ FRQVLGHULQJ
QVLGHULQJ \RXU DQVZHUV WR
RQVLGHULQJ Y  DQG
WR Y DQ YL
YL  UDQJH RI YDOXHV IRU
 VXJJHVW D UD
ER )H & &+ _)H &+          >@




1697 trendyline
9-&35(/,0


E  (SKHGULQH &+12 LV D GUXJ XVHG WR SUHYHQW ORZ EORRG SUHVVXUH GXULQJ VSLQDO
DQDHVWKHVLD,WLVRSWLFDOO\DFWLYHDQGKDVFKLUDOFHQWUHV

:KHQ(SKHGULQHLVKHDWHGZLWKDFLGLILHGSRWDVVLXPPDQJDQDWH 9,, LWJLYHVEHQ]RLFDFLG
&+&2+ DV RQH RI WKH SURGXFWV :KHQ LW LV KHDWHG ZLWK DFLGLILHG SRWDVVLXP
GLFKURPDWH 9,  LW JLYHV 3 3 JLYHV DQ RUDQJH SUHFLSLWDWH ZKHQ WUHDWHG ZLWK 
GLQLWURSKHQ\OK\GUD]LQHEXWLWKDVQRUHDFWLRQZLWK7ROOHQVUHDJHQW

:KHQ (SKHGULQH LV KHDWHG ZLWK HTXLPRODU &+&l LW JLYHV 4 &+12 DV WKH PDMRU
SURGXFW :KHQ LW LV KHDWHG ZLWK H[FHVV &+&l LW JLYHV 5 &+12&l DV WKH PDMRU
SURGXFW

'HGXFHWKHVWUXFWXUHVRI(SKHGULQH34DQG5([SODLQ\RXUDQVZHU   >@


F  'UDZ D ODEHOOHG GLDJUDP WR VKRZ KRZ EHQ]DOGHK\GH FDQ EH V\QWKHVLVHG IURP
SKHQ\OPHWKDQROLQDODERUDWRU\
        

        >@

>7RWDO@


trendyline
1698 trendyline
9-&35(/,0>7XUQRYHU
+&KHPLVWU\3UHOLP([DP3DSHU6XJJHVWHG$QVZHUV

 D  $WKLJKWHPSHUDWXUHV12UHDFWVZLWK+WRSURGXFHQLWURXVR[LGH12DJUHHQKRXVHJDV

12 J + J 12 J +2 J 

7RVWXG\WKHNLQHWLFVRIWKLVUHDFWLRQDW&LQLWLDOUDWHVIRUWKHIRUPDWLRQRI12ZHUH
PHDVXUHGXVLQJYDULRXVLQLWLDOSDUWLDOSUHVVXUHVRI12DQG+

,QLWLDOSUHVVXUHN3D ,QLWLDOUDWHRI
([SHULPHQW SURGXFWLRQRI12
P12  P+  N3DV
   

   

   

L  'HGXFHWKHUDWHHTXDWLRQLQWHUPVRISDUWLDOSUHVVXUH    >@

&RPSDULQJH[SWVDQG
5DWHLQFUHDVHVE\WLPHVZKHQWKHP+ LQFUHDVHVE\WLPHV
2UGHURIUHDFWLRQZLWKUHVSHFWWR+LV
&RPSDULQJH[SWVDQG
5DWHLQFUHDVHVE\WLPHVZKHQP12 LQFUHDVHVE\WLPHV
2UGHURIUHDFWLRQZLWKUHVSHFWWR12LV
 +HQFHUDWH k P12 P+ 

LL  8VLQJWKHUDWHHTXDWLRQDQGWKHLQIRUPDWLRQLQWKHWDEOHDERYHFDOFXODWHDYDOXHIRU
WKHUDWHFRQVWDQWJLYLQJLWVXQLWV       >@

8VLQJGDWDIURPH[SW
 k    
k  
 XQLW  N3D V

LLL  +HQFHFDOFXODWHWKHLQLWLDOUDWHDWZKLFK12UHDFWVZKHQN3DRI12DQGN3D
RI+DUHPL[HGDW&       >@

UDWH  N3DV HFI 
UDWHRIGLVDSSHDUDQFHRI12  N3DV

LY  :ULWH WKH UDWH HTXDWLRQ IRU WKH UHDFWLRQ ZKHQ 12 LV XVHG LQ ODUJH H[FHVV +HQFH
GHWHUPLQH WKH WLPH HODSVHG WR UHGXFH WKH SDUWLDO SUHVVXUH RI + WR KDOI RI LWV LQLWLDO
YDOXHLIN3DRI12DQGN3DRI+DUHPL[HGDW&  
            >@

 5DWH kP+ 
6LQFHPHP12 !!P+ 
FH
k kP
kkP12  

    HFI 

FI
 
W OQk
OQk
V




1699 trendyline
9-&35(/,0 >7XUQRYHU


Y  7ZRPHFKDQLVPV$DQG%DUHSURSRVHGEHORZ
$ 12 12

   12+ +212


% 12+ +12

   +12 12+2

:KLFKPHFKDQLVPLVFRQVLVWHQWZLWKWKHUDWHHTXDWLRQREWDLQHGLQ D L ([SODLQ\RXU
DQVZHU           >@

%
7KHPROHFXODULW\RIWKHUHDFWDQWVLQWKHVORZVWHSLVZLWKUHVSHFWWR12DQGZLWK
UHVSHFW WR + ,W LV FRQVLVWHQW ZLWK WKH H[SHULPHQWDOO\GHWHUPLQHG RUGHU RI UHDFWLRQ
ZLWKUHVSHFWWRHDFKUHDFWDQW


E  3RO\QLWURJHQ FRPSRXQGV KDYH JUHDW SRWHQWLDO IRU EHLQJ XVHG DV KLJK HQHUJ\ GHQVLW\
PDWHULDOV 7KH\ DUH WKHUPRG\QDPLFDOO\ XQVWDEOH +XJH DPRXQWV RI HQHUJ\ DUH UHOHDVHG
IURPWKHLUGHFRPSRVLWLRQRUUHDFWLRQVOHDGLQJWRPRUHVWDEOHSURGXFWV

L  7KHSRO\QLWURJHQVSHFLHV1ZDVILUVWLVRODWHGLQ2QHRILWVUHVRQDQFHIRUPVLV
VKRZQEHORZ



6WDWHWKHERQGDQJOHDQGPROHFXODUVKDSHDURXQGQLWURJHQODEHOOHGa  >@

%RQGDQJOHDQ\YDOXHEHWZHHQRDQGR
6KDSHEHQW

LL  7KH V\QWKHVLV RI D ZKLWH LRQLF VROLG >1@>$V)@ ZDV DFKLHYHG E\ UHDFWLQJ
>1)@>$V)@ZLWKK\GUD]RLFDFLG+1LQOLTXLG+)DWR&

:ULWHDEDODQFHGHTXDWLRQIRUWKLVUHDFWLRQ      >@

 >1)@>$V)@+1>1@>$V)@+)

LLL  >1)@>$V)@ ZDV ILUVW SUHSDUHG E\ UHDFWLQJ 1) ZLWK WKH VWURQJ /HZLV DFLG $V)
1)KDVWKHIROORZLQJVWUXFWXUHDQGH[KLELWVFLVWUDQVLVRPHULVP



'UDZ WKH FLVWUDQV LVRPHUV RI 1) LQFOXGLQJ DOO ORQH SDLUV DQG VXJJHVW DQ
DSSURSULDWHK\EULGLVDWLRQIRUHDFKQLWURJHQDWRPLQ1)   
  >@






t nd
trendyline
d
HSDLUVVKRZQ
$OOORQHSDLUVVKRZQ
$OOVSFRUUHFWO\ODEHOOHG




 
1700 trendyline
9-&35(/,0


F  1LWURJHQLVIRXQGLQDOORUJDQLVPVSULPDULO\LQDPLQRDFLGVZKLFKPDGHXSSURWHLQV

,VROHXFLQH1+&+ &+ &+ &+&+ &2+LVDQDPLQRDFLGZKHUHWKHDPLQRJURXSDQG
FDUER[\OLFDFLGJURXSDUHVHSDUDWHGE\RQHFDUERQ,WLVDFRQVWLWXHQWDPLQRDFLGLQPDQ\
SURWHLQPROHFXOHV,VROHXFLQHFDQH[LVWDVGLIIHUHQWVWHUHRLVRPHUV
L  6WDWH WKH UHDJHQWV DQG FRQGLWLRQV QHHGHG WR EUHDN SURWHLQ GRZQ LQWR LWV FRQVWLWXHQW
DPLQRDFLGV          >@

+62 DT RU1D2+ DT 
+HDW

LL  'HILQHWKHWHUPstereoisomers       >@

 0ROHFXOHVZLWKVDPHVWUXFWXUDOIRUPXODEXWGLIIHUHQWDUUDQJHPHQWRIDWRPVLQVSDFH

LLL  7KHUH DUH IRXU RSWLFDO LVRPHUV RI LVROHXFLQH 'UDZ WKH VWUXFWXUHV RI WKHVH IRXU
VWHUHRLVRPHUVRILVROHXFLQHLQGLFDWLQJFOHDUO\WKHFKLUDOFHQWUHV   >@



  
7ZRFKLUDOFHQWUHVFRUUHFWO\ODEHOOHGLQRQHVWUXFWXUH


G  DPLQRPHWK\OEXW\QH &+ &&1+ &+  LV XVHG DV WKH VWDUWLQJ PDWHULDO WR PDNH
RWKHUFKHPLFDOV
'UDZDODEHOOHGGLDJUDPWRVKRZWKHRUELWDOVWKDWIRUP& &LQDQDON\QH  >@

 



  

>7RWDO@



trendyline









1701 trendyline
9-&35(/,0>7XUQRYHU


 D  6W\UHQH SKHQ\OHWKHQH  LV D XVHIXO RUJDQLF LQWHUPHGLDWH XVHG IRU WKH V\QWKHVLV RI PDQ\
SURGXFWV)RULQVWDQFHVW\UHQHLVDEOHWRXQGHUJRWKHUHDFWLRQVKRZQEHORZ



L  6WDWHWKHUHDJHQWVDQGFRQGLWLRQVUHTXLUHGIRUUHDFWLRQV,,,DQG,,,  >@

  , +, J URRPWHPSHUDWXUH
    LJQRUHVWDWHV\PEROXQOHVVLWVJLYHQDVDTXHRXVVWDWH 
  ,, 1D2+ DT KHDW
  ,,, .&U2 DT +62 DT KHDW

LL  %\FRQVLGHULQJWKHVXEVWLWXHQW V DWWDFKHGWRWKHFDUERQ\OJURXSJLYHWZRUHDVRQVIRU
WKHORZHUUHDFWLYLW\RISKHQ\OHWKDQRQHWRZDUGVQXFOHRSKLOHVFRPSDUHGWRSURSDQRQH
&+&2&+          >@

6WHULF KLQGUDQFH RI WKH EXON\SKHQ\O JURXS PDNHV LW KDUGHU IRU WKH QXFOHRSKLOH WR
DSSURDFK
7KHSRUELWDOVRIWKHFDUERQ\OFDUERQRYHUODSZLWKWKHRUELWDOVRIWKHEHQ]HQHULQJ
25(OHFWURQVIURPWKHEHQ]HQHULQJGHORFDOLVHWRWKHFDUERQ\OFDUERQPDNLQJLWOHVV
HOHFWURQGHILFLHQWDQGKHQFHOHVVVXVFHSWLEOHWRQXFOHRSKLOLFDWWDFN

LLL  6XJJHVW D VLPSOH FKHPLFDO WHVW WR GLVWLQJXLVK EHWZHHQ LRGRHWK\O EHQ]HQH DQG
LRGREHQ]HQH&+,6WDWHWKHREVHUYDWLRQVIRUHDFKFRPSRXQG   >@

1D2+ DT KHDW
IROORZHGE\H[FHVV+12 DT DQG$J12 DT 
,RGRHWK\O EHQ]HQHIRUPV\HOORZSUHFLSLWDWH
,RGREHQ]HQHGRHVQRWIRUP\HOORZSUHFLSLWDWH
      
LY  7KH VROXELOLW\ RI LRGRHWK\O EHQ]HQH LQ ZDWHU ZDV IRXQG WR EH  PJGP ([SODLQ
ZK\ LRGRHWK\O EHQ]HQHKDVVXFKDORZVROXELOLW\LQZDWHU    >@

7KH SUHGRPLQDQW LQWHUDFWLRQV EHWZHHQ LRGRHWK\O EHQ]HQH PROHFXOHV DUH GLVSHUVLRQ
IRUFHV
:HDN GLSROHLQGXFHG GLSROH LQWHUDFWLRQV IRUPHG EHWZHHQ LRGRHWK\O EHQ]HQH DQG
ZDWHUUHOHDVHLQVXIILFLHQWHQHUJ\WRRYHUFRPHWKHVWURQJK\GURJHQERQGLQJLQZDWHU


E  ,RGRHWK\O EHQ]HQHH[LVWVDVDSDLURIHQDQWLRPHUV$DQG%

$Q RSWLFDOO\ SXUH VDPSOH FRQWDLQLQJ RQO\ LVRPHU $ GLVVROYHG LQ KH[DQH XQGHUZHQW D
QXFOHRSKLOLF VXEVWLWXWLRQ UHDFWLRQ ZLWK D VROXWLRQ FRQWDLQLQJ UDGLRDFWLYH LRGLGH LRQV ,
$IWHUWKHUHDFWLRQRIWKH LRGRHWK\O EHQ]HQHLVLQFRUSRUDWHGZLWKUDGLRDFWLYHLRGLGH

$Q LQYHVWLJDWLRQ LQWR WKH NLQHWLFV RI WKH UHDFWLRQ UHYHDOHG WKDW WKH UDWH RI UHDFWLRQ ZDV

trendyline

HQWRIWKHFRQFHQWUDWLRQRI
QW
LQGHSHQGHQWRIWKHFRQFHQWUDWLRQRI , 

L  'HVFULEHLEH LRGRHWK\O EHQ]HQH ZLWK , ,Q \RXU
EH WKH PHFKDQLVP IRU WKH UHDFWLRQ RI LRGRHWK\O EHQ]HQH
DQVZHU WKH 
U FOHDUO\ ODEHO WKH
HU WK 
, DQG LQFOXGH DQ\ UHOHYDQW FKDUJHV
FKDUJHV FXUO\ DUURZV DQG
VRUORQHSDLUVRIHOHFWURQVWKDW\RXFRQVLGHULPSRUWDQW
RUORQHSDLUVRIHOHFWURQVWKDW\RXFRQVLGHULPSRUWD LQWKLV WK
GLSROHVRUORQHSDLUVRIHOHFWURQVWKDW\RXFRQVLGHULPSRUWDQWLQWKLVPHFKDQLVP >@


1702 trendyline
9-&35(/,0





LL  &DOFXODWH WKH SHUFHQWDJH RI LRGRHWK\O EHQ]HQH WKDW H[LVWV DV HQDQWLRPHU %
LQFRUSRUDWHGZLWK,LQWKHSURGXFWPL[WXUH      >@

6LQFHRI$UHDFWHGRI$UHPDLQHGXQUHDFWHG
 SHUFHQWDJHRIHQDQWLRPHU% 
      


F  7KHWHUPWULLRGLGHUHIHUVWRPROHFXOHVWKDWFRQWDLQWKUHHLRGLQHDWRPVLQWKHLUPROHFXODU
IRUPXODVXFKDVQLWURJHQWULLRGLGH1,DQGDOXPLQLXPWULLRGLGH$l,

$l, KDV D VLPSOH PROHFXODU VWUXFWXUH ,Q WKH JDV SKDVH LW GLPHULVHV UHDGLO\ WR IRUP WKH
GLPHU$l,7KHWZRVSHFLHVDUHUHODWHGE\WKHHTXLOLEULXPVKRZQEHORZ

$l, J $l, J   H!

L  $  J VDPSOH RI $l2, ZDV DOORZHG WR YDSRXULVH LQ D  GP YHVVHO DW  R&
&DOFXODWHWKHLQLWLDOSUHVVXUHRI$l2,       >@

Q57
3$l ,   
9

 

 


3D

LL  *LYHQ WKDW WKH GHJUHH RI GLVVRFLDWLRQ  RI $l2, ZDV  ILQG WKH KS YDOXH IRU WKH
HTXLOLEULXPDWR&         >@
       
 3$l , DIWHUGLVVRFLDWLRQ  [ HFI 
3D
 3l,  [[ HFI 
   3D
3$l,  

 KS   
3$l , 
 3D


trendyline
QWKHHIIHFWRQWKHGHJUHHRIGLVVRFLDWLRQRI$ll,IRUHDFKRIW
QWKHHIIHFWRQWKHGHJUHHRIGLVVRFLDWLRQRI$
LLL  ([SODLQWKHHIIHFWRQWKHGHJUHHRIGLVVRFLDWLRQRI$l IRUHDFKRIWKHFKDQJHVEHORZ

 7KHUHDFWLRQZDVFDUULHGRXWDWR&  
 $ VDPSOH RI $U J  ZDV LQWURGXFHG LQWR D GP YHVVHO FRQWDLQLQJ
FR $l, J DQG
R
$ll, J DW &        >@



1703 trendyline
9-&35(/,0>7XUQRYHU


 :KHQWKHWHPSHUDWXUHLVGHFUHDVHGWKHEDFNZDUGH[RWKHUPLFUHDFWLRQLVIDYRXUHG
 +HQFH WKH SRVLWLRQ RI HTXLOLEULXPZLOO VKLIW WR WKH OHIW DQG GHJUHH RI GLVVRFLDWLRQ
GHFUHDVHV


:KLOHWRWDOSUHVVXUHLQWKHYHVVHOLQFUHDVHVDVQXPEHURIPROHVRIJDVHVLQFUHDVHV
ZLWKLQMHFWLRQRILQHUW$U J 
 PROHIUDFWLRQDQGSDUWLDOSUHVVXUHRIWKH$l, J DQG$l, J UHPDLQFRQVWDQW
 +HQFHWKHUHLVQRVKLIWLQWKHSRVLWLRQRIHTXLOLEULXPDQGQRFKDQJHLQWKHGHJUHHRI
GLVVRFLDWLRQ


G  7KHWDEOHEHORZVKRZVWKHERLOLQJSRLQWVRI3&lDQG3)


&RPSRXQG %RLOLQJ3RLQWR&

3&l 
3) 

([SODLQZK\3&lKDVDKLJKHUERLOLQJSRLQWWKDQ3)     >@

 3&lPROHFXOHVDUHKHOGE\GLSROHGLSROHLQWHUDFWLRQV
 3)PROHFXOHVDUHKHOGE\GLVSHUVLRQIRUFHV
 +HQFHPRUHHQHUJ\LVQHHGHGWRRYHUFRPH
 WKHVWURQJHUGLSROHGLSROHLQWHUDFWLRQVLQ3&l
   
>7RWDO@


 D  /HDG ,, QLWUDWHGHFRPSRVHVRQKHDWLQJLQWKHVDPHZD\DVPDJQHVLXPQLWUDWH%\XVLQJ
UHOHYDQWGDWDIURPWKHData BookletSUHGLFWDQGH[SODLQZKLFKRIWKHWZRQLWUDWHVOHDG ,, 
QLWUDWHRUPDJQHVLXPQLWUDWHZRXOGKDYHDKLJKHUGHFRPSRVLWLRQWHPSHUDWXUH  >@

 3E 12 KDVWKHKLJKHUGHFRPSRVLWLRQWHPSHUDWXUH
 %RWK0JDQG3EKDYHWKHVDPHLRQLFFKDUJH  EXWLRQLFUDGLXVRI3ELVODUJHUWKDQ
WKDWRI0J
 &KDUJHGHQVLW\RI3ELVORZHU RUSRODULVLQJSRZHURI3ELVORZHU 
 (OHFWURQFORXGRI12LVGLVWRUWHGWRDVPDOOHUH[WHQWE\3E
 3E 12 KDVDKLJKHUWKHUPDOVWDELOLW\DQGWKXVDKLJKHUGHFRPSRVLWLRQWHPSHUDWXUH


E  7KHKDORJHQVDUHUHDFWLYHHOHPHQWVDQGDUHLQYROYHGLQPDQ\UHDFWLRQV

L  :KHQ FKORULQH LV EXEEOHG WKURXJK FROG VRGLXP K\GUR[LGH VROXWLRQ IROORZHG E\ DQ
H[FHVVRIDFLGLILHGVLOYHUQLWUDWHVROXWLRQRQO\  RIWKHFKORULQHZKLFKKDVGLVVROYHG
LVSUHFLSLWDWHGDVVLOYHUFKORULGH:KHQWKHVRGLXPK\GUR[LGHVROXWLRQLVKRWXSWR   
RIWKHFKORULQHFDQEHSUHFLSLWDWHG([SODLQWKHREVHUYDWLRQVZLWKWKHDLGRIUHOHYDQW

trendyline
HTXDWLRQV
RQ
RQV          >@

WDWLRQ$J DT &
3UHFLSLWDWLRQ$J
SLWDWLRQ$J DT &ll DT
 DT $J&l V 
DT $J&
$J&ll V 
:LWKFROG1D2+ DT 
FROG1D2+ DT 
ROG1D2+ DT 
  1D2+ DT 1D&
&l J 1D2+ DT 1D&l DT 1D&l2 DT +
J 1D2+ DT  &l DT 1D&
T 1D&ll2 DT +2 O 
2
 UDWLR RI &ll&l2
0RODUUDWLRRI&l &l2 

  RIWKHFKORULQHLVSUHFLSLWDWHGDV$J&l

 
 1704 trendyline
9-&35(/,0


 :LWKKRW1D2+ DT 
 &l J 1D2+ DT 1D&l DT 1D&l2 DT +2 O 
  0RODUUDWLRRI&l&l2 
 RIWKHFKORULQHLVSUHFLSLWDWHGDV$J&l


LL  :KHQ FKURPLXP LV PDGH WR UHDFW ZLWK FKORULQH WKH PDLQ SURGXFW RI WKH UHDFWLRQ LV
&U&l +RZHYHU ZKHQ FKURPLXP LV PDGH WR VHSDUDWHO\ UHDFW ZLWK RWKHU KDORJHQV
VXFK DV IOXRULQH DQG LRGLQH WKH PDLQ SURGXFWV DUH &U) DQG &U, %\ UHIHUHQFH WR
UHOHYDQW ER YDOXHV ZKDW FDQ \RX FRQFOXGH DERXW WKH UHODWLYH R[LGLVLQJ DELOLW\ RI WKH
KDORJHQV">@

  &RUUHFWERGDWDIRU)&lDQG,
 )H) ER 9
 &l2H&lER 9
 ,H, ER 9

 2[LGLVLQJSRZHU)!&l!,DVER ;; LVOHVVSRVLWLYHGRZQWKHJURXS
 &RUUHFW21RI&ULQDOOWKUHHFRPSRXQGV
)LVWKHVWURQJHVWR[LGLVLQJDJHQWDQGR[LGLVHV&UWR&U)ZKHUHWKHR[LGDWLRQVWDWHRI&U
LQ&U)LV
&l LV OHVVR[LGLVLQJWKDQ) DQGR[LGLVHV&U WR &U&l ZKHUHWKHR[LGDWLRQVWDWHRI &U LQ
&U&lLV
,RGLQHLVWKHZHDNHVWR[LGLVLQJDJHQWDQGR[LGLVHV&UWR&U,ZKHUHWKHR[LGDWLRQVWDWHRI
&ULQ&U,LV

7KH KDORJHQV DOVR IRUP PDQ\ LQWHUKDORJHQ FRPSRXQGV LQ ZKLFK WZR GLIIHUHQW KDORJHQV
DUHFRPELQHG,QWHUKDORJHQFRPSRXQGVOLNH%U&lKDYHVLPLODUSURSHUWLHVWRWKHKDORJHQV

LLL  6XJJHVW DQ HTXDWLRQ IRU WKH UHDFWLRQ WKDW RFFXUV EHWZHHQ %U&l DQG DTXHRXV .,
            >@
 %U&l.,.&l.%U,

LY  %U&lUHDFWVZLWKDONHQHVIDVWHUWKDQWKHSXUHKDORJHQVVXFKDV&lDQG%U6KRZWKH
VORZVWHSRIWKHUHDFWLRQEHWZHHQ%U&lDQGSURSHQH    >@
  




F  $OLWKLXPLRQEDWWHU\LVDUHFKDUJHDEOHEDWWHU\,QDIXOO\FKDUJHGEDWWHU\OLWKLXPLRQVRFFXS\WKH
WLQ\VSDFHVLQJUDSKLWHWRIRUP/L&'XULQJGLVFKDUJHOLWKLXPLRQVDUHSURGXFHGZKLFKPLJUDWH
WKURXJK WKH HOHFWURO\WH ZKLOH DW WKH VDPH WLPH JUDSKLWH LV EHLQJ UHJHQHUDWHG ,Q WKH RWKHU
HOHFWURGHOLWKLXPLRQVUHDFWZLWK&R2WRIRUP/L&R2



trendyline
1705 trendyline
9-&35(/,0>7XUQRYHU





L  :ULWHHTXDWLRQVIRUWKHHOHFWURGHUHDFWLRQVRFFXUULQJDWWKHDQRGHDQGFDWKRGHGXULQJ
GLVFKDUJH          >@
 
  $QRGH/L&&/L H
  &DWKRGH&R2/LH/L&R2

LL  +HQFHZULWHDQHTXDWLRQIRUWKHRYHUDOOUHDFWLRQWKDWRFFXUVGXULQJGLVFKDUJH >@

 /L&&R2&/L&R2

LLL  %\GHWHUPLQLQJWKHR[LGDWLRQQXPEHUVRIWKHUHOHYDQWHOHPHQWVLGHQWLI\WKHHOHPHQWV
WKDWKDYHEHHQR[LGLVHGDQGUHGXFHG       >@

 &LVR[LGLVHGDV21LQFUHDVHGIURPLQ/L&WRLQ&
 &RLVUHGXFHGDV21GHFUHDVHGIURPLQ&R2WRLQ/L&R2

G  L  :LWK UHIHUHQFH WR WKH Data Booklet GUDZ D IXOO\ ODEHOOHG GLDJUDP VKRZLQJ KRZ \RX
FRXOG PHDVXUH WKH VWDQGDUG HOHFWURGH SRWHQWLDO IRU WKH $J$J V\VWHP ,QGLFDWH WKH
GLUHFWLRQRIHOHFWURQIORZ        >@
 

 
 KLJKUHVLVWDQFHYROWPHWHU

 ! 9 !
 
H VDOWEULGJH H

 + J  $J V 
 DWP&



3W $TXHRXVVLOYHUQLWUDWH
 'LOXWHVXOIXULFDFLG
 >$J@ PROGP
>+@ PROGP


trendyline









1706 trendyline
9-&35(/,0


LL  7KHHOHFWURGHSRWHQWLDOEDQGWKHFRQFHQWUDWLRQRIVLOYHULRQVLQVROXWLRQXQGHUQRQ
VWDQGDUGFRQGLWLRQVDUHUHODWHGE\WKHIROORZLQJHTXDWLRQ


E OJ 
> $J DT @ 



7KH VROXELOLW\ SURGXFW FRQVWDQW RI $J&l LV KVS     PRO GP 8VLQJ WKH
HTXDWLRQJLYHQDERYHFDOFXODWHWKHHOHFWURGHSRWHQWLDODW.RIDKDOIFHOOIRUPHG
E\

,  D$JHOHFWURGHLPPHUVHGLQDVDWXUDWHGVROXWLRQRI$J&l   
    
 >$J@  [  
  [PROGP

  E ORJ
[  
  9

,,  D$JHOHFWURGHLPPHUVHGLQDPROGPVROXWLRQRI.&lFRQWDLQLQJVRPH$J&l
SUHFLSLWDWH         

[
  >$J@  

  [PROGP

E ORJ 
[  
  9
>@

LLL  $FHOOLVPDGHE\FRQQHFWLQJWKHKDOIFHOOLQ G L WRDVWDQGDUG&X&XKDOIFHOO%\
UHIHUHQFHWRUHOHYDQWGDWDIURPWKH'DWD%RRNOHWFDOFXODWHWKHFHOOHPIDQGZULWHDQ
HTXDWLRQ ZLWKVWDWHV\PEROV IRUWKHUHDFWLRQWKDWKDVRFFXUUHG   >@
 &XH&X ER 9
 EFHOO   
 9
 (TQ&X V $J DT &X DT $J V 

>7RWDO@


 D  &RPSRXQG*KDVWKHVWUXFWXUHVKRZQ

trendyline


WUXFWXUHVRIWKHRUJDQLFFRPSRXQGVIRUPHGE\FRPSRXQG
SR GV *
*LYHWKHVWUXFWXUHVRIWKHRUJDQLFFRPSRXQGVIRUPHGE\FRPSRXQG*XQGHUWKHIROORZLQJ

FRQGLWLRQV





1707 trendyline
9-&35(/,0>7XUQRYHU


L  1D2+ DT UHIOX[         >@





LL  ([FHVV&+&2&lURRPWHPSHUDWXUH       >@


 



E  /DFWLFDFLGFDQEHXVHGLQIRRGSURGXFWVDVS+UHJXODWRUVRUDVDIRRGSUHVHUYDWLYH,WFDQ
DOVREHXVHGDVDIODYRXULQJDJHQW

L  6XJJHVWUHDJHQWVDQGFRQGLWLRQVQHHGHGWRFRQYHUWSURSHQHLQWRK\GUR[\SURSDQRLF
DFLG FRPPRQO\NQRZQDVODFWLFDFLG LQQRPRUHWKDQVWHSV*LYHWKHVWUXFWXUHVRI
WKHLQWHUPHGLDWHV



     >@



 6WHS,&ROG.0Q21D2+ DT 
 6WHS,,.&U2+62 DT UHIOX[
 6WHS,,,1D%+ DOF UHIOX[
         
    
LL  :KHQ ODFWLF DFLG LV UHIOX[HG ZLWK FRQFHQWUDWHG VXOIXULF DFLG FRPSRXQG + ZLWK WKH
PROHFXODU IRUPXOD RI &+2 LV IRUPHG ,W GRHV QRW UHDFW ZLWK VRGLXP *LYH WKH
VWUXFWXUDOIRUPXODRI+         >@








trendyline
en y e
en
LLL  'HVFULEHDQGH[SODLQWKHUHODWLYHDFLGLW\RIWKHIROORZLQJWKUHHFRPSRXQGV
LEHDQGH[SODLQWKHUHODWLYHDFLGLW\RIWKHIROORZLQJWKUH
EHDQGH[SODLQWKHUHODWLYHDFLGLW\RIWKHIROOR HFRPS  >@





1708 trendyline
9-&35(/,0


&RPSRXQG$LVWKHPRVWDFLGLFDQGODFWLFDFLGLVWKHOHDVWDFLGLF
$ FRQWDLQV WKH DF\O FKORULGH ZKLFK K\GURO\VHV UHDGLO\ LQ ZDWHU WR SURGXFH D VWURQJ
DFLG+&l.
3UHVHQFHRI&l DWRPLQ%H[HUWVDQHOHFWURQZLWKGUDZLQJLQGXFWLYHHIIHFW
ZKLFKSURGXFHVDPRUHVWDEOHDQLRQWKDQWKHDQLRQIRUPHGIURPODFWLFDFLG



F  ([SHULPHQW  LV FRQGXFWHG LQ ZKLFK WKH FRQGXFWLYLW\ RI  FP RI %D 2+  VROXWLRQ LV
PRQLWRUHGDVLWLVWLWUDWHGZLWKPROGPRIDTXHRXV+627KHGDWDFROOHFWHGIURP
WKHH[SHULPHQWLVSORWWHGLQWKHJUDSKEHORZ

&RQGXFWLYLW\

      
9ROXPHRI+62 DGGHGFP


L  7KH FRQGXFWLYLW\ RI WKH %D 2+  VROXWLRQ GHFUHDVHV DV WKH YROXPH RI PRO GP
+62LVDGGHGIURPFPWRFP

,GHQWLI\WKHFKHPLFDOVSHFLHVWKDWHQDEOHWKHVROXWLRQWRFRQGXFWHOHFWULFLW\DVWKHILUVW
FPRI+62 DT LVDGGHG+HQFHH[SODLQZK\WKHFRQGXFWLYLW\GHFUHDVHV >@

6SHFLHV%D DT DQG2+ DT 

$VWKHWLWUDWLRQSURFHHGVEHIRUHWKHHTXLYDOHQFHSRLQW
2+ DT LVUHPRYHGE\QHXWUDOLVDWLRQWRIRUP+2 O 
%D DT LVUHPRYHGE\SUHFLSLWDWLRQWRIRUP%D62 V 
7KXVWKHUHDUHOHVVPRELOHFKDUJHFDUULHUVDQGFRQGXFWLYLW\GHFUHDVHV

LL  8VLQJWKHLQIRUPDWLRQLQWKHJUDSKFDOFXODWHWKHLQLWLDOFRQFHQWUDWLRQRI%D 2+  >@
         
%D 2+ +62%D62+2
Q%D 2+  Q+ 62 
1RRIPROHVRI%D 2+  
PRO
&RQFHQWUDWLRQRI%D 2+  
PROGP


G  ,Q([SHULPHQWDQRWKHUFPVDPSOHRIPROGPDTXHRXV%D 2+ LVPL[HGZLWK
 FP RI  PRO GP RI HWKDQRLF DFLG WR IRUP VROXWLRQ $ %DULXP HWKDQRDWH

trendyline
tren
%D &+&2 2 LVWKHSURGXFWRIWKHUHDFWLRQ7KHDFLGGLVVRFLDWLRQFRQVW
LVWKHSURGXFWRIWKHUHDFWLRQ7KHDFLGGLVVRFLDWLRQFRQVWDQWK
RQ7KHDFLGG DRIHWKDQRLF
PROGP&DOFXODWHWKHS+RIWKHVROXWLRQ
DFLGLV &DOFXODWHWKHS+RIWKHVROXWLRQ$
&DOFXODWHWKHS+RIWKHVROXWLRQ$
$   
            >@
  
KD >+@>&++&2@>&+
>&+&2+@@

%D 2+ &+&2+%D &+&2 +2

1HZ>&+&2+@   [
1709 trendyline
9-&35(/,0>7XUQRYHU


PROGP
1HZ>&+&2@ [[
PROGP
>+@ 
>+@ [PROGP
S+ OJ [ 




H  +DOI RI WKH VROXWLRQ $ IRUPHG LQ G  LV DGGHG WR EHDNHU ; ZKHUH WKHUH LV D OD\HU RI
FRORXUOHVVRLODVVKRZQLQWKHILJXUHEHORZ


2LO

6ROXWLRQ$


 %HDNHU;

7KH LQGLFDWRU +,Q LV D ZHDN DFLG ZLWK D SKD YDOXH RI  ,W UHDFWV ZLWK ZDWHU DV
UHSUHVHQWHGLQWKHHTXDWLRQEHORZ

+,Q DT +2 O ,Q DT +2 DT 
 \HOORZ  EOXH

$VPDOODPRXQWRILQGLFDWRU+,QLVDGGHGLQWREHDNHU;7KHPL[WXUHVDUHVWLUUHGZHOODQG
WKHWZROD\HUVDUHDOORZHGWRVHSDUDWH

$VVXPHWKHS+RIVROXWLRQ$LVXQDIIHFWHGE\WKHSUHVHQFHRIFRORXUOHVVRLO

L  6WDWH WKH FRORXU REVHUYHG LQ WKH RLO OD\HU RI EHDNHU ; ([SODLQ WKH REVHUYDWLRQ LQ
WHUPVRIDFLGEDVHHTXLOLEULXPDQGLQWHUPROHFXODULQWHUDFWLRQV  
 >@

7KHRLOOD\HULV\HOORZLQFRORXU
6ROXWLRQ$LVRIS+DFLGIRUPRILQGLFDWRU+,Q DT SUHGRPLQDWHVVLQFHS+S.D
6LQFH+,QLVDQHXWUDO XQFKDUJHG PROHFXOHVRPHRILWFDQGLVVROYHEHWWHULQWKHRLO
OD\HU EHFDXVH RI GLVSHUVLRQ IRUFHV RU GLSROHLQGXFHG GLSROH RU GLSROHGLSROH
LQWHUDFWLRQV ZLWKWKHRLOPROHFXOHV

LL  $ IHZ GURSV RI DTXHRXV VRGLXP K\GUR[LGH DQG K\GURFKORULF DFLG DUH DGGHG WR WKH
EHDNHU;VXFFHVVLYHO\DQGVWLUUHGZHOO([SODLQZLWKWKHDLGRIHTXDWLRQ V ZK\WKHUHLV
QRFKDQJHLQWKHFRORXURIWKHRLOOD\HUDIWHUHDFKDGGLWLRQ  >@

trendyline

GLWLRQRI1D2+ODUJHUHVHUYRLURI& &2+UHPRYHVWKH2+DGGHG
2QDGGLWLRQRI1D2+ODUJHUHVHUYRLURI&+
GLWLRQRI1D2+ODUJHUHVHUYRLURI&+
&+&2 2+2+ 2+ &+
&+&2 & +
 22
GLWLRQRI+&llODUJHUHVHUYRLURI&+&2
GLWLRQRI+&
2QDGGLWLRQRI+&lODUJHUHVHUYRLURI&+ & UHPRYHVWKH+DGG
DGGHG

&+&2 2 + &+&2+ +
>+@UHPDLQVXQFKDQJHGWKH+,QIRUPVWLOOSUHGRPLQDWHVDQGLVVROXEOHLQWKHRLOOD\HU

>7RWDO@
 
1710 trendyline
9-&35(/,0


 D  )HUURFHQH )H &+  LV DQ RUDQJH VROLG ,Q WKLV FRPSOH[ &+ LV WKH OLJDQG DQG LW
GRQDWHVHOHFWURQVIURPWKHULQJWRWKHYDFDQWGRUELWDORI)H7KHVWUXFWXUHRIIHUURFHQH
LVJLYHQEHORZ

)HUURFHQHFDQXQGHUJRDVHULHVRIGLIIHUHQWUHDFWLRQVDQGVRPHDUHDQDORJXHVRIRUJDQLF
UHDFWLRQV7ZRUHDFWLRQVRIIHUURFHQHDUHVKRZQEHORZ
 

 ,

 ,,


L  6WDWHWKHW\SHVRIUHDFWLRQIRUUHDFWLRQV,DQG,,     >@

 ,HOHFWURSKLOLFVXEVWLWXWLRQ
 ,,UHGR[
 
LL  6WDWHWKHR[LGDWLRQQXPEHURI)HLQIHUURFHQHDQGKHQFHLWVHOHFWURQLFFRQILJXUDWLRQ
            >@

 2[LGDWLRQ1XPEHURI)H
 (OHFWURQLFFRQILJXUDWLRQVVSVSG
 
LLL  6XJJHVWZK\)HUURFHQHLVDFRORXUHGFRPSOH[     >@

 ,Q WKH SUHVHQFH RI OLJDQGV G RUELWDOV RI LURQ DUH VSOLW LQWR  JURXSVZLWK VOLJKWO\
GLIIHUHQWHQHUJ\ GGVSOLWWLQJ 
 $GHOHFWURQIURPWKHORZHUHQHUJ\JURXSLVSURPRWHGWRWKHKLJKHUHQHUJ\JURXS G
GWUDQVLWLRQ 
 E\DEVRUELQJHQHUJ\IURPWKHYLVLEOHUHJLRQRIWKHHOHFWURPDJQHWLFVSHFWUXP
 &RORXUREVHUYHGLVFRPSOHPHQWDU\WRWKHFRORXUDEVRUEHG

LY  7KHFRPSOHPHQWDU\FRORXUVDUHLOOXVWUDWHGZLWKWKHIROORZLQJFRORXUZKHHO


 JUHHQ \HOORZ


 EOXH RUDQJH


trendyline
ndyl
d
 UHG
SXUSOH



$TXHRXV)HXV )HLRQLVJUHHQLQFRORXUVXJJHVWDQGH[SODLQLIZDWHUF
RXV)H LRQLVJUHHQLQFRORXUVXJJHVWDQGH[SODLQLIZDWHUFDXVHVDODUJHUVSOLW
RQLVJUHHQLQFRORXUVXJJHVWDQGH[SODLQLIZDWHUF

EHWZHHQWKHWZRJURXSVRIGRUELWDOVDVFRPSDUHGWR&
HQWKHWZRJURXSVRIGRUELWDOVDVFRPSDUHGWR&
DVF +    >@



1711 trendyline
9-&35(/,0>7XUQRYHU


 &+FDXVHVDODUJHUGRUELWDOVSOLW
 :LWKZDWHUDVWKHOLJDQGHOHFWURPDJQHWLFZDYHVZLWKHQHUJ\FRUUHVSRQGLQJWRUHGLV
DEVRUEHGZKLFKKDVDORZHUHQHUJ\WKDQEOXHZKLFKLVDEVRUEHGZKHQ&+LVWKH
OLJDQG

Y  %\FRQVLGHULQJUHDFWLRQ,,VXJJHVWLIER )H &+ _)H &+  LVPRUHSRVLWLYHRUOHVV
SRVLWLYHWKDQER )H_)H ([SODLQ\RXUDQVZHU     >@

ER )H &+ _)H &+  LVOHVVSRVLWLYHWKDQER )H_)H 
 ,QUHDFWLRQ,,IHUURFHQHKDVEHHQR[LGLVHGE\)HWKXVER )H &+ _)H &+  KDV
DOHVVSRVLWLYHERYDOXHWKDQER )H_)H VLQFHERFHOO ERUHGXFHGERR[LGLVHG!

YL  7KHUHDFWLRQEHWZHHQ62DQG,LVVORZLQWKHDEVHQFHRIFDWDO\VW)HUURFHQHFDQ
EHDVXLWDEOHFDWDO\VWIRUWKLVUHDFWLRQEHWZHHQ62DQG,

([SODLQZLWKWKHDLGRIHTXDWLRQVKRZLWZRUNV<RXPD\XVH)H &+ WRUHSUHVHQW
IHUURFHQH          >@

  )H &+ 62)H &+ 62
  )H &+ ,)H &+ ,
  :LWKRXW WKH FDWDO\VW WKH UHDFWLRQ KDV D KLJK DFWLYDWLRQ HQHUJ\ GXH WR FROOLVLRQV
EHWZHHQLRQVRIWKHVDPHFKDUJH
  ,QWKHSUHVHQFHRIFDWDO\VWFROOLVLRQVEHWZHHQLRQVRIWKHVDPHFKDUJHDUHDYRLGHG
KHQFHLWORZHUVWKHDFWLYDWLRQHQHUJ\

YLL %\ FRQVLGHULQJ \RXU DQVZHUV WR Y  DQG YL  VXJJHVW D UDQJH RI YDOXHV IRU
ER )H &+ _)H &+          >@

 9ER )H &+ _)H &+  9


E  (SKHGULQH &+12 LV D GUXJ XVHG WR SUHYHQW ORZ EORRG SUHVVXUH GXULQJ VSLQDO
DQDHVWKHVLD,WLVRSWLFDOO\DFWLYHDQGKDVFKLUDOFHQWUHV

:KHQ(SKHGULQHLVKHDWHGZLWKDFLGLILHGSRWDVVLXPPDQJDQDWH 9,, LWJLYHVEHQ]RLFDFLG
&+&2+ DV RQH RI WKH SURGXFWV :KHQ LW LV KHDWHG ZLWK DFLGLILHG SRWDVVLXP
GLFKURPDWH 9,  LW JLYHV 3 3 JLYHV DQ RUDQJH SUHFLSLWDWH ZKHQ WUHDWHG ZLWK 
GLQLWURSKHQ\OK\GUD]LQHEXWLWKDVQRUHDFWLRQZLWK7ROOHQVUHDJHQW

:KHQ (SKHGULQH LV KHDWHG ZLWK HTXLPRODU &+&l LW JLYHV 4 &+12 DV WKH PDMRU
SURGXFW :KHQ LW LV KHDWHG ZLWK H[FHVV &+&l LW JLYHV 5 &+12&l DV WKH PDMRU
SURGXFW

'HGXFHWKHVWUXFWXUHVRI(SKHGULQH34DQG5([SODLQ\RXUDQVZHU   >@

 (SKHGULQHKDVFKLUDOFHQWUHVLWKDVWZRFDUERQDWRPVDWWDFKHGWRIRXUGLIIHUHQWJURXSV
 (SKHGULQH XQGHUJRHV VLGHFKDLQ R[LGDWLRQ RI EHQ]HQH ZLWK KRW DFLGLILHG SRWDVVLXP
PDQJDQDWH WR JLYH EHQ]RLF DFLG LW FRQWDLQV PRQRVXEVWLWXWHG EHQ]HQH ZLWK D EHQ]\OLF
K\GURJHQ

t ndyy e
 3XQGHUJRHVFRQGHQVDWLRQZLWKGLQLWURSKHQ\OK\GUD]LQHLWVXJJHVWVLWFRQWDLQVHLWKHU
DOGHK\GHRUNHWRQHIXQFWLRQDOJURXSV6LQFHLWGRHVQRWUHDFWZLWK7ROOHQVUHDJHQWLWPXVW
RUNHWRQH
U IXQFWLRQDOJURXSV6LQFHLWGRHVQRWUHDFWZLWK7ROOH
LQFHLWGRHVQRW
EHDNHWRQH H
 +HQFH(SKHGULQHFRQWDLQVDVHFRQGDU\DOFRKROVLQFHLWFDQEHR[LGLVHGE\GLFKURPDWHWR
SKHGULQHFRQWDLQVDVHFRQGDU\DOFRKRO
KHGULQHFRQWDLQVDVHFRQGDU\DOFRKROVLQFHLWFDQEHR[LGLVH LQFHLWFDQEHR[LGLVH
IRUPDNHWRQH RQH
QH
 (SKHGULQHXQGHUJRHVQXFOHRSKLOLFVXEVWLWXWLRQZLWK&+
XQGHUJRHVQXFOHRSKLOLFVXEVWLWXWLRQ WLRQZLWK&+&lWRIRUP
&lWRIRUP4,WFRQWDLQVDQDPLQH
l 4,W
JURXS
 :KHQ (SKHGULQH LV UHDFWHG ZLWK H[FHVV &+&l WZR PHWK\O JURXSV KDYH EHHQ
LQFRUSRUDWHGLQ5DVFRQILUPHGE\WKHFKDQJHLQWKHPROHFXODUIRUPXODKHQFHLWFRQWDLQV
VHFRQGDU\DPLQHJURXS
1712 trendyline
9-&35(/,0



 
         


F  'UDZ D ODEHOOHG GLDJUDP WR VKRZ KRZ EHQ]DOGHK\GH FDQ EH V\QWKHVLVHG IURP
SKHQ\OPHWKDQROLQDODERUDWRU\
        

        >@

          
WKHUPRPHWHU SRVLWLRQHGRSSRVLWHWKH   
       
HQWUDQFHRIFRQGHQVHU      
            
ZDWHURXW
            
            
            
            
         
 FRQGHQVHU   
            
.&U2+62
          
  
SKHQ\OPHWKDQRO          
       ZDWHULQ    
      
DQWLEXPSLQJ      
      JUDQXOHV
      
KHDWLQJ
    PDQWOH
   
SXUHEHQ]DOGHK\GH    
            
             
             >7RWDO@

trendyline
1713 trendyline
9-&35(/,0>7XUQRYHU
2

Section A
For each question there are four possible answers, A, B, C, and D. Choose the one you
consider to be correct and shade your choice on the answer sheet provided.

1 For indoor air quality safety standards, the maximum safe tolerance level of carbon
monoxide gas is 6 104 m3 of carbon monoxide in 1 m3 of air. Given that a typical room
measures4 m by 4 m by 3 m, calculate the number of molecules of carbon monoxide
present in the room at this tolerance level at room temperature and pressure?

A 1.13 1024

B 7.22 1023

C 1.51 1022

D 7.22 1020

2 Soluble mercury compounds such as mercury(II) nitrate are highly toxic. One removal
method involves the treatment of wastewater contaminated with mercury(II) nitrate with
sodium sulfide to produce solid mercury(II) sulfide and sodium nitrate solutions.

Calculate the mass of mercury(II) sulfide formed when 0.020 dm3 of 0.100 mol dm3
sodium sulfide reacts with 0.050 dm3 of 0.010 mol dm3 mercury(II) nitrate?

A 0.117 g B 0.133 g C 0.280 g D 0.466 g

3 Gaseous particle X hasa proton number n, and a charge of +1.


Gaseous particle Y has a proton number (n+1), and is isoelectronic with X.

Which statement about X and Y is correct?

A When placed in an electric field, the angle of deflection for X is the same as that of
Y.

B X requires more energy than Y when a further electron is removed from each
particle.

C X releases more energy than Y when an electron is added to particle.

D X has a larger radius than Y.

trendyline
9647 / YJC / 2016 / JC2 Preliminary Examination / Paper 1

1714 trendyline
3

4 Covalent bonds are formed by orbital overlap. The shape of unsaturated hydrocarbon
molecules can be explained in terms of hybridisation of orbitals.

Which bond is not present in the molecule shown below?

H C C H
C C
H
C H
H
H

A bond formed by 2sp3 2sp2 overlap.

B bond formed by 2sp3-2sp overlap.

C bond formed by 1s-2sp overlap.

D bond formed by 2p-2p overlap.

5 Which statement about graphite is not correct?

A The carbon-carbon bonds in graphite are shorter than those in diamond.

B It can act as a good lubricant due to the weak forces of attraction between the layers
of atoms.

C Carbon to carbon distances between the planes of hexagonal rings are smaller than
those within the planes.

D It acts as a good conductor of electricity in the direction parallel to the planes


containing hexagonal rings of carbon.

trendyline
9647 / YJC / 2016 / JC2 Preliminary Examination / Paper 1

1715 trendyline
4

6
Gas T

Gas U

Which of the following could be the identities of the gases?

Gas T Gas U

A H 2 (g) at 298 K CO 2 (g) at 298 K

B CH 4 (g) at 298 K NH 3 (g) at 298 K

C O 2 (g) at 500 K O 2 (g) at 298 K

D N 2 (g) at 298 K N 2 (g) at 500 K

7 The polymerisation of propene to form polypropene occurs readily at room temperatures.

What will be the signs of G, H and S for the polymerisation reaction at


roomtemperature?

G H S

A +

B + +

C +

ttrendyline
d li
D

9647 / YJC / 2016 / JC2 Preliminary Examination / Paper 1

1716 trendyline
5

8 The rate of the redox reaction between hydrochloric acid and Mg

Mg + 2HCl MgCl 2 + H 2

can be followed by measuring the time taken for the same volume of hydrogen to be
produced from a range of hydrochloric acid concentrations.

To find the order with respect to hydrochloric acid, which would be the most suitable graph
to plot using the data?

A [HCl] against time

1
B [HCl] against
time

C Volume H 2 against time

1
D Volume H 2 against
time

9 The numerical value of the solubility product of nickel(II) carbonate is 6.6 109 while that
of silver carbonate is 2.1 1011 at 25 C.

Which of the following statements is true?

A Addition of silver nitrate increases the solubility of silver carbonate.

B The solubility of silver carbonate is higher than the solubility of nickel(II) carbonate.

C Addition of nitric acid to a solution containing nickel(II) carbonate increases the


solubility product of nickel(II) carbonate.

D Nickel(II) carbonate precipitates first when sodium carbonate is added to a solution


containing equal concentrations of nickel(II) and silver ions.

10 What is the pH of 10 cm3 of 0.05 moldm3 sodium benzoate?

[K a of benzoic acid = 6 105moldm3]

A 6.46

B 7.46

C 8.46

trendyline
D 9.46

9647 / YJC / 2016 / JC2 Preliminary Examination / Paper 1

1717 trendyline
6

11 A sample of 20.0 cm3 of 0.20 moldm3 iron(II) sulfate is titrated against 0.05 moldm3
potassium manganate(VII) in the presence of excess fluoride ions. It is found that
20.0 cm3 of the manganate(VII) solution is required to reach the end-point.

What is the oxidation number of manganese at end-point?

A +3

B +4

C +5

D +6

12 Anodisation is a process to increase corrosion resistance and surface hardness in


aluminium objects.
A possible set up for the process is shown as below:

Which of the following statements about the anodisation of aluminium is true?

A Aluminium dissolves at the anode.

B Hydrogen gas is liberated at the anode.

C Water is oxidised at the anode to form oxygen gas.

D Replacing the electrolyte with NaOH(aq) will cause the reaction to cease.

13 Which of the following elements forms an oxide with a giant structure and a chloride which
is readily hydrolysed?

trendyline
A on
Silicon

B um
m
Barium

C um
m
Sodium

D Phosphorus
9647 / YJC / 2016 / JC2 Preliminary Examination / Paper 1

1718 trendyline
7

14 An element in Period 3 has the following properties:

- Its first ionisation energy is larger than both the elements before and after it across the
period.
- It conducts electricity.
- It reacts slowly with cold water but react readily with steam to give a gas.

Which statement about this element is not correct?

A It has a high melting and boiling point.

B It forms an oxide that produces a solution that is acidic when added to water.

C It forms a chloride that produces a solution that isslightly acidic when added to water.

D Its oxidation state in compounds usually follows the group number it belongs to in the
Periodic Table.

15 The following report appeared in a newspaper article.

Drums of bromine broke open after a vehicle crash on the motorway. Traffic was diverted
aspurple gaseous bromine drifted over the road (it is denser than air), causing irritation to
drivers eyes. Firemen sprayed water over the scene of the accident, dissolving the
bromineand washing it away.

What is wrong with the report?

A Bromine does not dissolve in water.

B Bromine does not vapourise readily.

C Bromine is less dense than air.

D Bromine is not purple.

16 When crystalline potassium chromate(VI) was dissolved in water, a yellow solution Q was
formed. Addition of dilute sulfuric acid to Q gave an orange solution R. When hydrogen
sulfide was bubbled through solution R, there was a color change in the solution and yellow
sulfur was produced.

Which process did not occur in the above experiment?

A Ligand exchange

B Redox reaction

17
C

D trendyline
ipitation
Precipitation

-base
base
Acid-base

Chlorine compounds show oxidation states ranging from 1 to +7.

9647 / YJC / 2016 / JC2 Preliminary Examination / Paper 1

1719 trendyline
8

What are the reagents and conditions necessary for the oxidation of chlorine gas into
acompound containing chlorine in the +5 oxidation state?

A Cold dilute NaOH

B Hot concentrated NaOH

C Concentrated H 2 SO 4 at room temperature

D AgNO 3 (aq) followed by NH 3 (aq) at room temperature

18 Which statement correctly defines a transition element?

A Transition elements form many coloured compounds.

B Transition elements or their compounds are widely used as catalysts.

C Transition elements form one or more stable ions with partially filled d-orbitals.

D Transition elements exhibit more than one oxidation state in their compounds.

19 How many stereoisomers does a molecule of Nandrolone has?


O

O C

O
Nandrolone

A 8 B 16 C 32 D 64

20
trendyline
The structure of compound Z is as seen below:

9647 / YJC / 2016 / JC2 Preliminary Examination / Paper 1

1720 trendyline
9

OH

Compound Z

It is completely reacted with hot concentrated H 2 SO 4 to form compound Y. What is the total
number of isomers of Y?

A 2 B 3 C 4 D 5

21 2methylpropanoic acid can be synthesised from 1chloropropane through a series of


reactions.

Which set of reagents, used in sequential order, would be the most suitable for this
synthesis?

A PCl 5 , acidified KMnO 4

B ethanolic KCN, dilute HCl

C aqueous KOH, HCl, ethanolic KCN, dilute HCl

D ethanolic KOH, HBr, ethanolic KCN, dilute H 2 SO 4

22 Which statement about ethanal and propanone is not correct?

A Both give a positive tri-iodomethane test.

B Both react with 2,4-dinitrophenylhydrazine reagent.

C Both may be prepared by the oxidation of an alcohol.

D Both react with hot acidified sodium dichromate(VI).

23
trendyline
When sodium optically
dium iodide in propanone iss added to an optical ly active sample of

9647 / YJC / 2016 / JC2 Preliminary Examination / Paper 1

1721 trendyline
10

2-bromobutane, a sodium bromide precipitate is formed after 13 minutes upon heating.

NaI + CH 3 CHBrCH 2 CH 3 CH 3 CHICH 2 CH 3 + NaBr(s)

The experiment was repeated several times and the rate equation was found to be

Rate = k[CH 3 CHBrCH 2 CH 3 ][NaI]

Which of the following statements is not correct?

A The mechanism involves a reactive intermediate.

B The organic product sample obtained rotates the plane of polarised light.

C A similar experiment, using 1-bromobutane, will produce a precipitate in less than 13


minutes.

D A similar experiment, using 2-chlorobutane, will take more than 13 minutes to


produce a precipitate.

24 Which reagent can be used to distinguish between compounds R and S under suitable
conditions?
CH3 O
HO C C
H O CH2 Br
HO C C
CH3 H

CHO
R S

A Acidified potassium dichromate (VI)

B Alkaline copper(II) solution

C Sodium metal

D Phosphorus pentachloride

25 The two-stage reaction sequence given shows a possible mechanism for the reaction

9647 / YJC / 2016 / JC2 Preliminary Examination / Paper 1

1722 trendyline
11

between phenoxide ion and ethanoyl chloride.

CH3 CH3 CH3

PhO C O PhO C O PhO C O

Cl Cl Cl

where Ph = phenyl

How should the overall reaction be classified?

A Electrophilic addition

B Nucleophilic addition

C Electrophilic substitution

D Nucleophilic substitution

26 Cyanohydrins can be made from carbonyl compounds by generating CN ions from HCN in
the presence of a weak base.

1 1
R R OH
--
CN
C O + HCN C
R2 R2 CN

In a similar reaction, CH 2 COOCH 3 ions are generated from CH 3 COOCH 3 by strong bases.

Which compound can be made from an aldehyde and CH 3 COOCH 3 ?

A CH 3 CH(OH)COOCH 3

B CH 3 COOCH 2 CH(OH) CH 3

C CH 3 CH 2 CH(OH)CH 2 COOCH 3

D (CH 3 ) 2 C(OH)CH 2 COOCH 3

27
trendyline
Two common drugs administered when a patient experiences headache or fever are

9647 / YJC / 2016 / JC2 Preliminary Examination / Paper 1

1723 trendyline
12

paracetamol and aspirin.

COOH

O O
HO NH O

paracetamol aspirin

A solution of the two drugs was hydrolysed using hot aqueous sodium hydroxide.
Which organic product would be produced?

A B
COO -
O
-O N
H O
O

C D
O
-O NH2

28 Acetaminophen is a drug used in headache remedies. It has the following structure:

Acetaminophen

Which of the following reagents reacts with Acetaminophen?

A Sodium carbonate

B Cold sodium hydroxide

C Alkaline aqueous iodine

D 2,4-dinitrophenylhydrazine

29
trendyline
uence shows the correct order off increasing
Which sequence in p
pKK b in an aqueo
aqueous solution of equal

9647 / YJC / 2016 / JC2 Preliminary Examination / Paper 1

1724 trendyline
13

concentration?

A C 2 H 5 CONH 2 < C 6 H 5 NH 2 < C 2 H 5 NH 2 < C 2 H 5 NH 3 +

B C 2 H 5 NH 2 < C 6 H 5 NH 2 < C 2 H 5 NH 3 +< C 2 H 5 CONH 2

C C 2 H 5 NH 2 < C 6 H 5 NH 2 < C 2 H 5 CONH 2 < C 2 H 5 NH 3 +

D C 2 H 5 NH 3 +< C 2 H 5 CONH 2 < C 2 H 5 NH 2 < C 6 H 5 NH 2

30 There are three pK a values associated with glutamic acid: 2.1, 4.1 and 9.5.

The pH-volume curve obtained when 30 cm3 of NaOH is added to 10 cm3 of the protonated
form of glutamic acid of the same concentration is given below.

Which of the following is the major species present at point S?

A B

trendyline
C D

Section B
9647 / YJC / 2016 / JC2 Preliminary Examination / Paper 1

1725 trendyline
14

For each of the questions in this section, one or more of the three numbered
statements 1 to 3 may be correct.

Decide whether each of the statements is or is not correct (you may find it helpful to
pick a tick against the statements that you consider to be correct).

The responses A to D should be selected on the basis of

A B C D
1, 2 and 3 are 1 and 2 only 2 and 3 only 1 only is
correct are correct are correct correct

No other combination of statements is used as a correct response.

31 The radius and charge of each of the six ions are shown below.

ion J+ L+ M2+ X Y Z2
radius / nm 0.14 0.18 0.15 0.14 0.18 0.15

The ionic solids JX, LY and MZ are of the same lattice type. Which of the following
statements are correct?

1 The melting point increases in the order LY < JX < MZ.

2 The numerical value of hydration energy of Xis smaller than that of Z2.

3 The solution containing M2+ ions is more acidic than the solution containing J+ ions.

32 Use of the Data Booklet is relevant to this question.

Which of the following will increase the cell potential?

1 Adding iodine crystals into the anodic half cell.

33
2

3
trendyline
Adding
ng solid silver nitrate into the cathodichalf cell.

Increasing
ce

aq)/Br(aq)
asing the concentration of Br 2 (aq) in the Br 2 (aq)/Br
easing

From the position of the elements present in


n the
aq) half cell.

th Periodic Table and the physical properties

9647 / YJC / 2016 / JC2 Preliminary Examination / Paper 1

1726 trendyline
15

of the compounds, which compounds are covalent?

1 (CH 3 ) 2 SiCl 2 b.p. 70C

2 GeCl 4 b.p. 86C

3 AlBr 3 b.p. 265C

34 Use of the Data Booklet is relevant to this question.

The oxides of titanium, iron and nickel are used as catalysts in the industries.

Which properties are titanium, iron and nickel likely to have in common?

1 similar ionic radii

2 high melting points

3 variable oxidation states

35 Use of the Data Booklet is relevant to this question.

With reference to the two half equations below, which statements are correct?

[Fe(CN) 6 ]3 + e [Fe(CN) 6 ]4

Fe3+ + e Fe2+

1 [Fe(CN) 6 ]3is more stable than Fe3+

2 [Fe(CN) 6 ]4 is a stronger reducing agent than Fe2+

3 Both [Fe(CN) 6 ]3 and Fe3+ can oxidize MnO 4 2 to MnO 4

36 Propene is found to react with chlorine in the presence of aqueous sodium bromide and
sodium nitrate. Which of the following are not possible products of the reaction?

1 CH3 CH CH2

Br Br

2 CH3 CH CH2

ONO 2 Cl

trendyline
re
3 CH3 CH CH2

O
OH Cl

nses A to D should be selected on


The responses n the
th basis of

9647 / YJC / 2016 / JC2 Preliminary Examination / Paper 1

1727 trendyline
16

A B C D
1, 2 and 3 1 and 2 only 2 and 3 only 1 only is
are correct are correct are correct correct

No other combination of statements is used as a correct response.

37 Which of the following reactions will not give a good yield of the desired product?

38 The three compounds E, F and G have the following structures?

HO
COOCH3
N
N
N
N CH CH2 N
OH O
E F G

Which statements about E, F and G are correct?

1 E and G have the same empirical formula.

2 E and F are isomers

3 The Mr of F is exactly twice that of G

9647 / YJC / 2016 / JC2 Preliminary Examination / Paper 1

1728 trendyline
17

39 The energy profile for the following reaction is shown below. [D = 2H]

H H

C6H5 C Cl + OH C6H5 C OH + Cl
D D

energy
X

reactants

products

Reaction pathway
Which conclusions can be drawn?

1 The product has no effect on the rotation of plane polarised light.

2 The rate of reaction can be increased by increasing concentration of OH.

3 The structure of the transition state at point X is

C6H5
HO C Cl
H D

40 Which groups within an amino acid are able to form a cross-chain link to stabilise the
tertiary structure of a protein?

1 CH 2 CONH 2

2 NH2
H CH2 CH2 CH2 NH C
NH

3 H CH2

ttrendyline
tr
re N
H

~ END OF PAPER ~
9647 / YJC / 2016 / JC2 Preliminary Examination / Paper 1

1729 trendyline
2

Section A
For each question there are four possible answers, A, B, C, and D. Choose the one you
consider to be correct and shade your choice on the answer sheet provided.

1 For indoor air quality safety standards, the maximum safe tolerance level of carbon
monoxide gas is 6 104 m3 of carbon monoxide in 1 m3 of air. Given that a typical room
measures
4 m by 4 m by 3 m, calculate the number of molecules of carbon monoxide present in the
room at this tolerance level at room temperature and pressure?

A 1.13 1024

B 7.22 1023

C 1.51 1022

D 7.22 1020

Answer: B

For 1 m3 of air, 6 x 104of CO is allowed


For 4 4 3 = 48 m3 of air, volume of CO = 48 x 6 x 104= 0.0288 m3 = 28.8 dm3
Number of moles of CO = 28.8 / 24 = 1.20 mol
Number of molecules
= 1.20 (6.02 1023)
= 7.22 1023

2 Soluble mercury compounds such as mercury(II) nitrate are highly toxic. One removal
method involves the treatment of wastewater contaminated with mercury(II) nitrate with
sodium sulfide to produce solid mercury(II) sulfide and sodium nitrate solutions.

Calculate the mass of mercury(II) sulfide formed when 0.020 dm3 of 0.100 mol dm3
sodium sulfide reacts with 0.050 dm3 of 0.010 mol dm3 mercury(II) nitrate?

A 0.117 g B 0.133 g C 0.280 g D 0.466 g

Answer: A

Balanced equation:
Hg(NO 3 ) 2 (aq) + Na 2 S(aq) HgS(s) + 2NaNO 3 (aq)

No. of moles of Na 2 S
= 0.020 0.100 = 2.00 103mol
No. of moles of Hg(NO 3 ) 2
= 0.050 0.010 = 5.00 104 mol

2.00 103mol of Na 2 S will require 2.00 103 mol of Hg(NO 3 ) 2 for complete reaction.

trendyline
Since 5.00 0 104 mol of Hg(NO 3 ) 2 is useded (lesser than 2.00
2 103 need
needed), thus Hg(NO 3 ) 2
is limiting.
No. of moles es of HgS = 5.00 10 044mol
mo
Mass of HgS gS
S
= (5.00 10 044) (201
1 + 32.1)
32 1) = 0
0.11655
11655 g
= 0.117 g

9647 / YJC / 2016 / JC2 Preliminary Examination / Paper 1

1730 trendyline
3

3 Gaseous particle X hasa proton number n, and a charge of +1.


Gaseous particle Y has a proton number (n+1), and is isoelectronic with X.

Which statement about X and Y is correct?

A When placed in an electric field, the angle of deflection for X is the same as that of
Y.

B X requires more energy than Y when a further electron is removed from each
particle.

C X releases more energy than Y when an electron is added to particle.

D X has a larger radius than Y.

Answer: D

Particle X
Number of protons n
Number of electrons n-1
Thus Xhas a charge of +1

Particle Y
Number of protons n+1
Number of electrons n-1
Thus Y has a charge of 2

A is incorrect.

Angle of deflection

Y has double the charge of X DVFRPSDUHGWR KRZHYHUWKHDWRPLFPDVVRIY is


not double that of X (X and Y are consecutive elements as deduced from their proton
numbers). Thus their charge/mass ratios will not be the same.

B is incorrect. As X has one less proton than Y, it has a smaller nuclear charge. As both
particles are isoelectronic (same number of inner shell electrons), shielding effect will be
the similar. Effective nuclear of X will be smaller. The valence electrons of X will be less
strongly attracted to the nucleus. Thus less energy is needed to remove the outermost
electron.

C is incorrect. X releases lesser energy than Y when an electron is added. The attractive
force between the incoming electron and the nucleus is weaker than that for Y as X has
smaller effective nuclear charge.

D is correct.Same explanation as Option B. X has a smaller nuclear charge (fewer protons


than Y) and the same shielding effect as Y (same number of inner shell electrons).
Effective nuclear of X will be smaller. The valence electrons of X will be less strongly
attracted to the nucleus. Thus the radius of X is larger than that of Y.

t d li
trendyline
9647 / YJC / 2016 / JC2 Preliminary Examination / Paper 1

1731 trendyline
4

4 Covalent bonds are formed by orbital overlap. The shape of unsaturated hydrocarbon
molecules can be explained in terms of hybridisation of orbitals.

Which bond is not present in the molecule shown below?

H
C C H
H
C C
H C
H
H

A bond formed by 2sp3 2sp2 overlap.

B bond formed by 2sp3-2sp overlap.

C bond formed by 1s-2sp overlap.

D bond formed by 2p-2p overlap.

Answer: B

4 5
H
C C H
H 2 3
C C
1
H C
H
H

A is correct. There is a ERQGIRUPHGE\VS3 2sp2 overlap present between C1-C2.

B is incorrect. There isnt any ERQGIRUPHGE\VS3-2spoverlap present.

C is correct. There is a ERQGIRUPHGE\V-2sp overlap present between hydrogen atom


and C5.

D is correct. There is a bond formed by 2p-2p overlap present between C2-C3 and C4-
C5.

trendyline
9647 / YJC / 2016 / JC2 Preliminary Examination / Paper 1

1732 trendyline
5

5 Which statement about graphite is not correct?

A The carbon-carbon bonds in graphite are shorter than those in diamond.

B It can act as a good lubricant due to the weak forces of attraction between the layers
of atoms.

C Carbon to carbon distances between the planes of hexagonal rings are smaller than
those within the planes.

D It acts as a good conductor of electricity in the direction parallel to the planes


containing hexagonal rings of carbon.

Answer: C

A is true. The carbon-carbon bonds in graphite are formed by sp2-sp2 overlap whereas
carbon-carbon bonds in diamond are formed by sp3-sp3 overlap. sp2-sp2 overlap is more
effective than sp3-sp3 overlap due to the greater s-character, thus the C-C bond length in
graphite is shorter.

B is true. The weak id-id forces between the layers allow the layers to slide past one
another, thus graphite is a good lubricant.

C is false. Carbon to carbon distances between the planes of hexagonal rings (weak id-id
interactions) are longer than those within the planes (strong C-C covalent bonds).

D is true. Within each layer, each carbon atom is covalently bonded to three other carbon
atoms using 3 of the 4 valence electrons. The presence of delocalised electrons between
the layers accounts for graphite being a good conductor when a potential difference is
applied along the layer. It does not conduct across the layer.

trendyline
9647 / YJC / 2016 / JC2 Preliminary Examination / Paper 1

1733 trendyline
6

6
Gas T

Gas U

Which of the following could be the identities of the gases?

Gas T Gas U

A H 2 (g) at 298 K CO 2 (g) at 298 K

B CH 4 (g) at 298 K NH 3 (g) at 298 K

C O 2 (g) at 500 K O 2 (g) at 298 K

D N 2 (g) at 298 K N 2 (g) at 500 K

Answer: D

From graph, Gas T deviates more from ideality than Gas U.

A is incorrect. Deviation of CO 2 from ideal gas behaviour is larger than that of H 2 . CO 2 has
a greater number of electrons than O 2 , thus the electron cloud is more polarisable,
resulting in stronger instantaneous dipole-induced dipole interactions between CO 2
molecules.

B is incorrect. Deviation of NH 3 from ideal gas behaviour is larger than that of CH 4 . There
is stronger hydrogen bonding between NH 3 molecules than instantaneous dipole-induced
dipole interactions between CH 4 molecules.

C is incorrect. Deviation of O 2 (g) at 298 K from ideal gas behaviour is larger than that at
500 K. At higher temperatures, particles in a gas possess large kinetic energy and are able
to overcome intermolecular forces of attraction such that these attractions can be
considered to be insignificant and negligible.

D is correct. Deviation of N 2 (g) at 500 K from ideal gas behaviour is smaller than that at
298 K. At higher temperatures, particles in a gas possess large kinetic energy and are able
to overcome intermolecular forces of attraction such that these attractions can be

trendyline
consideredd to be insignificant and negligible.
negligib

9647 / YJC / 2016 / JC2 Preliminary Examination / Paper 1

1734 trendyline
7

7 The polymerisation of propene to form polypropene occurs readily at room temperatures.

What will be the signs of G, H and S for the polymerisation reaction at


roomtemperature?

G H S

A +

B + +

C +

Answer: D

4XHVWLRQVWDWHVWKDWWKHUHDFWLRQLVVSRQWDQHRXVDWURRPWHPSHUDWXUHWKXV*LVQHJDWLYH

The overall reaction involves the breaking of a C-&ERQGDQGWKHIRUPDWLRQRI&-&


ERQG6LQFHWKHERQGLVVWURQJHUWKDQWKHERQGWKHHQHUJ\UHOHDVHGIURPWKH
IRUPDWLRQRIWKHERQGLVJUHDWHUWKDQWKHHQHUJ\DEVRUEHGIRUWKHEUHDNLQJRIWKHERQd.
Thus H is negative.

7KHVLJQIRU6LVQHJDWLYHDVWKHUHLVDGHFUHDVHLQWKHQXPEHURIPROHFXOHVWKXVWKHUH
are fewer ways of arranging the molecules, level of disorderliness decreases.

trendyline
8 The rate off the redox reaction between hydrochloric acid
ac and Mg

Mg + 2HCl MgCl
MgCl 2 + H 2

can be followed
llowed
owed by measuring the time taken for the same volume of hydrogen to be
produced from a range of hydrochloric acid concentrations
co
concentrations.

9647 / YJC / 2016 / JC2 Preliminary Examination / Paper 1

1735 trendyline
8

To find the order with respect to hydrochloric acid, which would be the most suitable graph
to plot using the data?

A [HCl] against time

1
B [HCl] against
time

C Volume H 2 against time

1
D Volume H 2 against
time

Answer: B

A is incorrect.Using graph of [HCl] against time, the order ofreaction with respect to
hydrochloric acid cannot be found but only the instantaneous rate of reaction can be found
by finding drawing tangent and calculate the gradient to the tangent at different time.

B is correct.The rate equation proposed is Rate = k[HCl]n. Since question has mentioned
that time taken for the same volume of hydrogen to be produced, 1/time will thus be
deemed as rate.If order of reaction with respect to hydrochloric acid is 1 (n=1), the
appropriate graph, rate = [HCl] against 1/time (also deemed as rate) will be a straight line
with positive gradient and the gradient = k.

C is incorrect.As the question mentioned that the data collected is time taken to produce
the same volume when different concentrations of hydrochloric acid, the graph of volume of
hydrogen against time will be the same even different concentrations of hydrochloric acid is
used.

D is incorrect.As the question mentioned that the data collected is time taken to produce
the same volume when different concentrations of hydrochloric acid, the graph of volume of
hydrogen against 1/time will be the same despite that different concentrations of
hydrochloric acid are used.

9 The numerical value of the solubility product of nickel(II) carbonate is 6.6 109 while that
of silver carbonate is 2.1 1011 at 25 C.

Which of the following statements is true?

A Addition of silver nitrate increases the solubility of silver carbonate.

B The solubility of silver carbonate is higher than the solubility of nickel(II) carbonate.

C Addition of nitric acid to a solution containing nickel(II) carbonate increases the

trendyline
y
solubility product of nickel(II) carbonate.

D Nickel(II)
el((II
el II) carbonate precipitates first when sodium carbonate is added to a solution
containing
in g equal concentrations of nickel(II)
ainin nickel(
nickel(II
II) and silver ions.

Answer: B

9647 / YJC / 2016 / JC2 Preliminary Examination / Paper 1

1736 trendyline
9

A is incorrect.
Ag 2 CO 3 (s) 2Ag+(aq) + CO 3 2-(aq) ---- (1)
AgNO 3 (s) Ag+(aq) + NO 3 -(aq)
Addition of silver nitrate increases [Ag+] in eqm 1, the position of eqm 1 shifts LHS,
the solubility should decrease.

B is correct
2
K sp [Ag ] 2 [CO 3 ] 2.1 10 11
K sp (2s) 2 (s)

2.1 10 11
s3 1.73 10 4 moldm 3
4
2
K sp [Ni 2 ][CO 3 ] 6.6 10 9

s 6.6 10 9 8.14 10 5 moldm 3


From the above calculation it is seen that the solubility of silver carbonate is higher
than that of nickel(II) carbonate.

C is incorrect.Solubility product is only affected by temperature. Therefore addition


of nitric acid cannot change the magnitude of solubility product.

D is incorrect.
Assuming [Ni2+] = [Ag+] = 0.100 moldm-3
2
K sp [Ni2 ][CO3 ] 6.6 10 9
2 6.6 10 9
[CO3 ] 6.6 10 8 moldm 3
0.100
K [Ag ] 2 [CO 2 ] 2.1 10 11
sp 3
2.1 10 11
[CO 2 ] 2.1 10 9 moldm 3
3
0.100 2
Since lower [CO 3 2-] needed to see first trace of silver carbonate is lower than that
of nickel (II) carbonate, silver carbonate should be precipitated out first.

10 What is the pH of 10 cm3 of 0.05 moldm3 sodium benzoate?

[K a of benzoic acid = 6 105moldm3]

A 6.46

B 7.46

C 8.46

D 9.46

Answer: C

trendyline
y
Let the saltt B be the benzoate salt.
s

B + H 2 O BH
H + OH

9647 / YJC / 2016 / JC2 Preliminary Examination / Paper 1

1737 trendyline
10

Kb K w
Ka
14
1 10
6 10 6
1.6667 10 10 moldm 3
[OH ][BH]
Kb 1.6667 10 10
[B ]
[OH ] 2
1.6667 10 10
[B ]
[OH ] 1.6667 10 10 0.05
2.8868 10 6 moldm 3
pOH log(2.8868 10 6 ) 5.5395
pH 14 5.5395 8.46

11 A sample of 20.0 cm3 of 0.20 moldm3 iron(II) sulfate is titrated against 0.05 moldm3
potassium manganate(VII) in the presence of excess fluoride ions. It is found that 20.0
cm3 of the manganate(VII) solution is required to reach the end-point.

What is the oxidation number of manganese at end-point?

A +3

B +4

C +5

D +6

Answer: A

MnO 4 - + 8H+ + (7-n)e Mnn+ + 4H 2 O


Fe2+ Fe3+ + e
20
no of mol of electrons gained by Mn (7 - n) 0.05 0.00100 mol
1000
20
no of mol of electrons lost by Fe 0.20 0.00400 mol
1000
no of mol of eletctrons lost by Fe no of mol of electrons gained by Mn
(7 - n) 4
n3
Thus the new oxidation state of Mn is +3.

Alternative method:
0.00400 mol of Fe2+ lost 0.00400 mol of electron to Mn
0.00100 mol of MnO 4 - gained 0.00400 mol of electron from Fe2+
1 mol of MnO 4 - gained 0.004/0.001 = 4 mol of electron from Fe2+

trendyline
all oxidation state of Mn in MnO 4 - is +7 and tthe Mn gained 4 electrons, thus the
The original
new oxidation
tion state is +3
+3.

12 Anodisation
n is a process to increase corrosion resistance and surface h
hardness in
aluminium objects.

9647 / YJC / 2016 / JC2 Preliminary Examination / Paper 1

1738 trendyline
11

A possible set up for the process is shown as below:

Which of the following statements about the anodisation of aluminium is true?

A Aluminium dissolves at the anode.

B Hydrogen gas is liberated at the anode.

C Water is oxidised at the anode to form oxygen gas.

D Replacing the electrolyte with NaOH(aq) will cause the reaction to cease.

Answer: C

A is not true. In this electrolytic process, water is oxidised in preference to aluminium.


Therefore, aluminium is not oxidised hence it is not dissolved.

B is not true. At anode, water from electrolyte is oxidised to oxygen gas.


2H 2 O O 2 + 4H+ + 4e

C is true. The purpose of the process is to allow water from the electrolyte to be oxidised
to form oxygen gas.
2H 2 O O 2 + 4H+ + 4e
Oxygen liberated then reacts with the aluminium object at the anode to form the protective
layer of aluminium oxide.
4Al + 3O 2 2Al 2 O 3

D is not true. During the process, hydroxide ions from electrolyte can be oxidised to form
oxygen gas at the anode. Therefore, the process does not stop.
4OH- 2H 2 O + O 2 + 4e

13 Which of the following elements forms an oxide with a giant structure and a chloride which
is readily hydrolysed?

C
trendyline
on
Silicon

um
m
Barium

Sodium

9647 / YJC / 2016 / JC2 Preliminary Examination / Paper 1

1739 trendyline
12

D Phosphorus

Answer: A

A is correct. The oxide of silicon, SiO 2 , exists as a giant covalent structure while the
chloride of silicon, SiCl 4 , hydrolyses in water to form SiO 2 and HCl.

B is incorrect. Barium oxide has a giant ionic lattice structure and barium chloride does not
undergo hydrolysis.

C is incorrect. Similar explanation to B.

D is incorrect. Phosphorus pentoxide has a simple molecular structure although


phosphorus pentachloride undergoes hydrolysis readily.

Structure of P 4 O 10

14 An element in Period 3 has the following properties:

- Its first ionisation energy is larger than both the elements before and after it across the
period.
- It conducts electricity.
- It reacts slowly with cold water but react readily with steam to give a gas.

Which statement about this element is not correct?

A It has a high melting and boiling point.

B It forms an oxide that produces a solution that is acidic when added to water.

C It forms a chloride that produces a solution that isslightly acidic when added to water.

trendyline
D xidation
idation state in compounds usually follows tthe group number
Its oxidation numbe it belongs to in the
odic Table.
Periodic Tabl

Answer: B

The element
ent is magnesium
magnesium.

9647 / YJC / 2016 / JC2 Preliminary Examination / Paper 1

1740 trendyline
13

A is correct. The strong metallic bonding in magnesium results in its high melting and
boiling point.

B is incorrect. Magnesium oxide is basic in nature and does not produce H+.

C is correct.When dissolved in water, hydrated Mg2+ is formed. Due to its relatively high
charge density, Mg2+ partially hydrolyses in water.

[Mg(H 2 O) 6 ]2+ [Mg(H 2 O) 5 (OH)]+ + H+

D is correct. It has an oxidation state of +2.

15 The following report appeared in a newspaper article.

Drums of bromine broke open after a vehicle crash on the motorway. Traffic was diverted
aspurple gaseous bromine drifted over the road (it is denser than air), causing irritation to
drivers eyes. Firemen sprayed water over the scene of the accident, dissolving the
bromineand washing it away.

What is wrong with the report?

A Bromine does not dissolve in water.

B Bromine does not vapourise readily.

C Bromine is less dense than air.

D Bromine is not purple.

Answer: D

A is correct. Bromine dissolves in water to form an orange solution.

B is correct. Bromine has a relatively high boiling point due to its big and polarisable
electron cloud and hence stronger instantaneous dipoleinduced dipole forces.

C is correct. Bromine has a molecular weight of 160 grams per mole, which is denser than
air, which has a molecular weight of 28 grams per mole (mainly nitrogen gas).

D is incorrect. Bromine exists as a red-brown gas.

16 When crystalline potassium chromate(VI) was dissolved in water, a yellow solution Q was
formed. Addition of dilute sulfuric acid to Q gave an orange solution R. When hydrogen
sulfide was bubbled through solution R, there was a color change in the solution and
yellow sulfur was produced.

A
trendyline
Which process
ocess did not occur in the above experiment

Ligand
nd
d exchange
h
experiment?

9647 / YJC / 2016 / JC2 Preliminary Examination / Paper 1

1741 trendyline
14

B Redox reaction

C Precipitation

D Acid-base

Answer: A

K 2 CrO 4 dissolves in water to give Q, CrO 4 2-.


Q undergoes acid-base reaction to give R, Cr 2 O 7 2-. R undergoes redox reaction with H 2 S
where sulfur was precipitated.

17 Chlorine compounds show oxidation states ranging from 1 to +7.

What are the reagents and conditions necessary for the oxidation of chlorine gas into
acompound containing chlorine in the +5 oxidation state?

A Cold dilute NaOH

B Hot concentrated NaOH

C Concentrated H 2 SO 4 at room temperature

D AgNO 3 (aq) followed by NH 3 (aq) at room temperature

Answer: B

A is incorrect. The compound formed is NaOCl and the oxidation state of chlorine in the
compound is +1.

B is correct. The compound formed is NaClO 3 and the oxidation state of chlorine in the
compound is +5.

C is incorrect. No reaction occurs between chlorine gas and sulfuric acid.

D is incorrect. Cl 2 + H 2 O HClO + HCl


Ag+ + ClAgCl
AgCl + 2NH 3 [Ag(NH 3 ) 2 ]+Cl
Cl in product does not have oxidation state of +5.

18 Which statement correctly defines a transition element?

A Transition elements form many coloured compounds.

trendyline
B sition
ition elements or their compounds are wide
Transition widely used as catalysts.
cataly

C sition elements form one or more stable ions with partially fill
Transition filled d-orbitals.

D sition elements exhibit more than one oxidation state in their compounds.
Transition

Answer: C
9647 / YJC / 2016 / JC2 Preliminary Examination / Paper 1

1742 trendyline
15

Atransition element is defined as a d-block element which forms one or more stable ions
with partially filled d-orbitals.

19 How many stereoisomers does a molecule of Nandrolone has?


O

O C

O
Nandrolone

A 8 B 16 C 32 D 64

Answer: D

A chiral carbon has four different groups attached to it. Nandrolone has 6 chiral carbons.
Double bond within a ring cannot exhibit cis-trans isomerism.

Total number of stereoisomers = 26 = 64

20
trendyline
The structure
ure
re of compound Z is as seen below:

9647 / YJC / 2016 / JC2 Preliminary Examination / Paper 1

1743 trendyline
16

OH

Compound Z

It is completely reacted with hot concentrated H 2 SO 4 to form compound Y. What is the total
number of isomers of Y?

A 2 B 3 C 4 D 5

Answer: C

This is an elimination reaction. The OH group and a H atom on a adjacent C atom are
removed to form an alkene.

similar -H bonded to C=C, no cis-trans similar -CH3 bonded to C=C, no cis-trans


CH3
H H
C

CH3

OH

trans-isomer cis-isomer

21 2methylpropanoic acid can be synthesised from 1chloropropane through a series of


reactions.

Which set of reagents, used in sequential order, would be the most suitable for this
synthesis?

A PCl 5 , acidified KMnO 4

B ethanolic KCN, dilute HCl

trendyline
C aqueous
ou KOH, HCl,
eous HCl, ethanolic KCN,
N, dilute HCl
HC
Cl

D ethanolic
olic KOH, HBr, ethanolic KCN, dilute H 2 SO 4
nolic

Answer: D

9647 / YJC / 2016 / JC2 Preliminary Examination / Paper 1

1744 trendyline
17

Cl alcoholic KOH

reflux HBr(g), room temp


1-chloropropane
Br

dil H2SO4 , heat


CN
alcoholic KCN, reflux
COOH
2-methylpropanoic acid

22 Which statement about ethanal and propanone is not correct?

A Both give a positive tri-iodomethane test.

B Both react with 2,4-dinitrophenylhydrazine reagent.

C Both may be prepared by the oxidation of an alcohol.

D Both react with hot acidified sodium dichromate(VI).

Answer: D

A is correct. Both compounds contain H C CH3 structure and will produce a yellow ppt,
CHI 3 with aqueous alkaline iodine.
O
B is correct. Both compounds contain H C H functional group and will will produce an
orange ppt with 2,4-DNPH.

C is correct. Ethanal is an aldehyde that can be prepared by the oxidation of a primary


alcohol. Propanone is a ketone that can be prepared by the oxidation of a secondary
alcohol.

D is incorrect. Cr 2 O 7 2can only oxidise ethanal but not propanone. Aldehydes can be
further oxidised to carboxylic acids but ketones cannot be further oxidised.

23 When sodium iodide in propanone is added to an optically active sample of


2-bromobutane, a sodium bromide precipitate is formed after 13 minutes upon heating.

trendyline
NaI
NaI + CH 3 CHBrCH H 3 CH 3 CH
HBrC 2 CH CHICH
CH 2 CH 3 + NaBr(s)
NaBr(
NaBr(s

The experiment
ment was repeated several times and the rate equation was found to be
iment

Rate = k[CH 3 CHBrCH 2 CH 3 ][Na


][NaI]
[NaI]

9647 / YJC / 2016 / JC2 Preliminary Examination / Paper 1

1745 trendyline
18

Which of the following statements is not correct?

A The mechanism involves a reactive intermediate.

B The organic product sample obtained rotates the plane of polarised light.

C A similar experiment, using 1-bromobutane, will produce a precipitate in less than 13


minutes.

D A similar experiment, using 2-chlorobutane, will take more than 13 minutes to


produce a precipitate.

Answer: A

A is incorrect. The rate equation shows that both reactants are involved in the rate-
determining step. Hence, it is S N 2 mechanism where no intermediate is formed.

B is correct. The reactant is chiral and all its molecules undergo an inversion of
configuration to remain as a chiral compound. The product will display optical activity.

For example, the starting optically active sample contains 40% (+) and 60% (-) isomers.
After S N 2 reaction, the product sample obtained will contain 60% (+) and 40% (-) isomers
which is still optically active and rotate the plane of polarised light.

C2 H5 C2 H5 C2 H5

- - - *C
I C Br I C Br I + Br-
Cl
H H
CH3 H CH3 CH3

C is correct. 1-bromobutane is a primary halogenoalkane while 2-bromobutane is a


secondary halogenoalkane. With a primary halogenoalkane, the attacking nucleophile
experiences less steric hindrance when attacking the C+and reacts more readily and the
precipitate is produced faster.

D is correct. The CCl bond is shorter than the CBr bond and has higher bond energy.
More energy is needed to break the bond and the rate of reaction of 2-chlorobutane will be
slower than that of 2-bromobutane.

24 Which reagent can be used to distinguish between compounds R and S under suitable
conditions?

trendyline
9647 / YJC / 2016 / JC2 Preliminary Examination / Paper 1

1746 trendyline
19

CH3 O
HO C C
H O CH2 Br
HO C C
CH3 H

CHO
R S

A Acidified potassium dichromate (VI)

B Alkaline copper(II) solution

C Sodium metal

D Phosphorus pentachloride

Answer: B

A cannot be used for the distinguishing test as it will oxidise the secondary alcohol and
aldehyde in R as well as the aldehyde in S. Orange acidified K 2 Cr 2 O 7 will turn green.

B can be used for the distinguishing test. R is an aliphatic aldehyde and reacts to form a
brick red precipitate of Cu 2 O. S is an aromatic aldehyde and will not be able to reduce
alkaline copper(II) solution.

C cannot be used for thedistinguishing test. Na will react with -OH groups in both R and S
and give off H 2 gas.

D cannot be used for thedistinguishing test. PCl 5 will react with -OH groups in both R and S
to produce white fume of HCl.

25 The two-stage reaction sequence given shows a possible mechanism for the reaction
between phenoxide ion and ethanoyl chloride.

CH3 CH3 CH3

PhO C O PhO C O PhO C O

Cl Cl Cl

where Ph = phenyl

How should the overall reaction be classified?

A trophilic addition
Electrophilic additio

B eophilic addition
Nucleophilic

C trophilic substitution
Electrophilic

9647 / YJC / 2016 / JC2 Preliminary Examination / Paper 1

1747 trendyline
20

D Nucleophilic substitution

Answer: D

In the first step, PhO behaves as a nucleophile by attacking the attacking the C+ that is
attached to O and Cl atoms.

At the end of the reaction, the Cl has left the molecule after being substituted.

26 Cyanohydrins can be made from carbonyl compounds by generating CN ions from HCN in
the presence of a weak base.

1 1
R R OH
CN--
C O + HCN C
R2 R2 CN


In a similar reaction, CH 2 COOCH 3 ions are generated from CH 3 COOCH 3 by strong bases.

Which compound can be made from an aldehyde and CH 3 COOCH 3 ?

A CH 3 CH(OH)COOCH 3

B CH 3 COOCH 2 CH(OH) CH 3

C CH 3 CH 2 CH(OH)CH 2 COOCH 3

D (CH 3 ) 2 C(OH)CH 2 COOCH 3

Answer: C

A is incorrect. The nucleophile used is COOCH 3 .

B is incorrect. The nucleophile used is CH 3 COOCH 2 .

C is correct. The nucleophile used is CH 2 COOCH 3 and it reacts with an aldehyde


CH 3 CH 2 CHO.

D is incorrect. The nucleophile used is CH 2 COOCH 3 but it reacts with a ketone
CH 3 COCH 3

27 Two common drugs administered when a patient experiences headache or fever are
paracetamol and aspirin.

COOH

trre
ttrendyline
re yl
ylilin
y
O
O
HO NH
O

paracetamol

9647 / YJC / 2016 / JC2 Preliminary Examination / Paper 1

1748 trendyline
21

aspirin

A solution of the two drugs was hydrolysed using hot aqueous sodium hydroxide.
Which organic product would be produced?

A B
COO -
O

O N
H
O
O

C D
O
O NH2

Answer: D

Alkaline hydrolysis of the amide bond in paracetamol produces the following:

O
HO NH O- NH2 + O
O-

Alkaline hydrolysis of the ester bond in aspirin produces the following:

COOH COO -
O-
O O- + O
O

28 Acetaminophen is a drug used in headache remedies. It has the following structure:

Acetaminophen

trendyline
Which of the
he following reagents reacts with Acetaminophen
Acetaminophen??

A um carbonate
Sodium

B Cold sodium hydroxide

9647 / YJC / 2016 / JC2 Preliminary Examination / Paper 1

1749 trendyline
22

C Alkaline aqueous iodine

D 2,4-dinitrophenylhydrazine

Answer: B

A has no reaction. There is no COOH present to react with the sodium carbonate.

B will react with the acidic phenol group in acetaminophen in a neutralisation reaction.

C has no reaction. The H


C CH3 structure is present but no C or H is directly bonded to
C=O for the reaction to occur.

D has no reaction. The aldehyde and ketone functional groups are not present.

29 Which sequence shows the correct order of increasing pK b in an aqueous solution of equal
concentration?

A C 2 H 5 CONH 2 < C 6 H 5 NH 2 < C 2 H 5 NH 2 < C 2 H 5 NH 3 +

B C 2 H 5 NH 2 < C 6 H 5 NH 2 < C 2 H 5 NH 3 +< C 2 H 5 CONH 2

C C 2 H 5 NH 2 < C 6 H 5 NH 2 < C 2 H 5 CONH 2 < C 2 H 5 NH 3 +

D C 2 H 5 NH 3 +< C 2 H 5 CONH 2 < C 2 H 5 NH 2 < C 6 H 5 NH 2

Answer: C

Increasing pK b means becoming less basic.

The electron donating C 2 H 5 group in C 2 H 5 NH 2 makes the lone pair of electrons on N atom
most available for donation to a proton and therefore the strongest base.

In C 6 H 5 NH 2 , the lone pair of electrons on N atom is delocalised into the benzene ring,
making it less available for donation to a proton. Phenylamine is therefore a weaker base

trendyline
than aliphatic
atic amines..
tic amines

In C 2 H 5 CONH
ONH
N 2 , the lone pair of electrons on N atom is delocalised over
ove the carbonyl C=O
and hence e not available for donation to a proton.
proton

H 3 + , th
In C 2 H 5 NH there are no lone
l pair
i off electrons
l t on the
th N atom
t ffor d
donation to a proton.-

9647 / YJC / 2016 / JC2 Preliminary Examination / Paper 1

1750 trendyline
23

NH 3 + is an acidic group with a proton for donation.

30
There are three pK a values associated with glutamic acid: 2.1, 4.1 and 9.5.

The pH-volume curve obtained when 30 cm3 of NaOH is added to 10 cm3 of the protonated
form of glutamic acid of the same concentration is given below.

trendyline
9647 / YJC / 2016 / JC2 Preliminary Examination / Paper 1

1751 trendyline
24

Which of the following is the major species present at point S?

A B

C D

Answer: C

The protonated form of glutamic acid and its associated pK a values are:

-COOH with pKa of 2.1 is the most acidic. -NH 3 + is an electron withdrawing group and
disperses the negative charge of -COO- of its conjugate base and stabilises it. -NH 3 + is
closer to -COOH with pKa of 2.1 than -COOH with pKa of 4.1. Hence, the dissociation of
the carboxylic acid with pKa of 2.1 occurs to a larger extent.

trendyline
carboxylic acid (-COO

Adding NaOH
(-
(-COO -
((--NH 3 +) as the co
c acid is more acidic than protonated amine (-NH
Carboxylic conjugate base of the
OO ) is resonance stabilised. Hence, the dissociation of the carboxylic
N 3 +.
acid occurss to a larger extent than -NH
-NH

aOH would result in three equivalence


enc points being obtained a
as shown in the

9647 / YJC / 2016 / JC2 Preliminary Examination / Paper 1

1752 trendyline
25

graph.

The 1st equivalence point is achieved when 10 cm3 of NaOH was added:

+ OH + H2O

At point S, the 2nd equivalence point is achieved when 20 cm3 of NaOH was added:

+ OH + H2O

The 3rd equivalence point is achieved when 30 cm3 of NaOH was added:

+ OH + H2O

trendyline
9647 / YJC / 2016 / JC2 Preliminary Examination / Paper 1

1753 trendyline
26

Section B
For each of the questions in this section, one or more of the three numbered
statements 1 to 3 may be correct.

Decide whether each of the statements is or is not correct (you may find it helpful to
pick a tick against the statements that you consider to be correct).

The responses A to D should be selected on the basis of

A B C D
1, 2 and 3 are 1 and 2 only 2 and 3 only 1 only is
correct are correct are correct correct

No other combination of statements is used as a correct response.

31 The radius and charge of each of the six ions are shown below.

ion J+ L+ M2+ X Y Z2
radius / nm 0.14 0.18 0.15 0.14 0.18 0.15

The ionic solids JX, LY and MZ are of the same lattice type. Which of the following
statements are correct?

1 The melting point increases in the order LY < JX < MZ.

2 The numerical value of hydration energy of Xis smaller than that of Z2.

3 The solution containing M2+ ions is more acidic than the solution containing J+ ions.

Answer: A

Option 1 is correct. JX, LY and MZ are ionic solids, thus their melting points are
determined by the strength of electrostatic forces of attraction between the oppositely
charged ions. The larger the magnitude of the lattice energy, the higher the melting point.
+
Lattice energy +
+
Lattice energy for JX, LY and MZ are: 3.57, 2.78 and 13.3 respectively. Thus melting point
increases in the order LY < JX < MZ.

Option 2 is correct. Numerical value of hydration energy is dependent on the charge



density of the ion . Charge density of X is 1/0.14 = 7.14 while that of Z is 2/0.15 =

13.3.
Thus the numerical value of hydration energy of Xis smaller than that of Z2.

Option 3 is correct. Charge density of M2+ is 2 / 0.15 = 13.3 while that of J+ is

trendyline
y
1 / 0.14 = 7.14
7 .Hence, M2+ has a higher
7.14.Hence, her charge density
de and thus a higher polarising
power.M2+draw
drawss electron density from the datively bonded
bonded water molecule
mo
mol towards itself
more and weaken the O-H O H bond to a greater extent.This
exten
extent.This
Th gives rise to a more acidic
solution.

9647 / YJC / 2016 / JC2 Preliminary Examination / Paper 1

1754 trendyline
27

32 Use of the Data Booklet is relevant to this question.

Which of the following will increase the cell potential?

1 Adding iodine crystals into the anodic half cell.

2 Adding solid silver nitrate into the cathodichalf cell.

3 Increasing the concentration of Br 2 (aq) in the Br 2 (aq)/Br(aq) half cell.

Answer: C

Option 1 is incorrect. Adding iodine crystals remove thiosulfate ions through redox
reaction. [S 2 O 3 2-] decreases while [S 4 O 6 2-] will increase at the anode.
I 2 + 2S 2 O 3 2- S 4 O 6 2- + 2I-
The equilibrium at anode will be disturbed. According to Le Chateliers principle, the
position of equilibrium shift right to replenish the S 2 O 3 2- ions lost and to use up the S 4 O 6 2-
ions which were added from the above redox reaction. This makes E oxd (anode) to be more
positive.
S 4 O 6 2- + 2e 2S 2 O 3 2-
E cell becomes less positive as the E red (cathode) of same magnitude subtracts a larger
positive E oxd (anode) .

Option 2 is correct. Solid silver nitrate removes Br- ions in the cathode. This reduces [Br-].
Ag+(aq) + Br-(aq) AgBr(s)
The equilibrium at cathode will be disturbed. According to Le Chateliers principle, the
position of equilibrium will shift to right to replenish bromide ions which are precipitated
out. This makes E red (cathode) to be more positive.
Br 2 + 2e 2Br-
E cell = E red (cathode) - E oxd (anode)
E cell becomes more positive as the more positive E red (cathode) subtracts positive E oxd (anode)
which has the same magnitude.

trendyline
Option 3 iss correct.
c At the cathode,
Br 2 + 2e 2Br-
Increasing [Br [B 2 ] shifts the equilibrium at cathode towards
owards right. Le Chateliers
Chat principle
predicts that at position of equilibrium shifts to the right to use up the brom
bromine added. This
makes E red ed (cathode) to be more positive.
E cell = E red (cathode) - E oxd (anode)

9647 / YJC / 2016 / JC2 Preliminary Examination / Paper 1

1755 trendyline
28

E cell becomes more positive as the more positive E red (cathode) subtracts positive E oxd (anode)
which has the same magnitude.

33 From the position of the elements present in the Periodic Table and the physical properties
of the compounds, which compounds are covalent?

1 (CH 3 ) 2 SiCl 2 b.p. 70C

2 GeCl 4 b.p. 86C

3 AlBr 3 b.p. 265C

Answer: A

(CH 3 ) 2 SiCl 2 is covalent. This can be inferred from its low boiling point and that it resembles
(CH 3 ) 2 CCl 2 since both Si and C are in the same group.

GeCl 4 is covalent. Similar reasoning as above since both Ge and Si are in the same group.

AlBr 3 is covalent as its boiling point is relatively low.Al3+ has a high charge density and
polarises the electron cloud of Cl such that AlCl 3 is covalent. Since Bris more polarisable
due to larger electron cloud than Cl, AlBr 3 is likely to be covalent.

34 Use of the Data Booklet is relevant to this question.

The oxides of titanium, iron and nickel are used as catalysts in the industries.

Which properties are titanium, iron and nickel likely to have in common?

1 similar ionic radii

2 high melting points

3 variable oxidation states

Answer: A

Titanium, iron and nickel belong to the first row of the d-block transition elements.

They have similar ionic radii due to the increase in shielding effect which is largely
cancelled out by the increase in nuclear charge. The effective nuclear charge remains
relatively constant across the first row of the d-block elements.

They have high melting points due to the strong metallic bonding, as both their 4s and 3d
electrons have similar energy and can be delocalised.

They have variable oxidation states as both the 3d and4s orbitals are similar in energy. The
4s and some (or all) of the 3d electrons can be removed without requiring too much

trendyline
energy.

9647 / YJC / 2016 / JC2 Preliminary Examination / Paper 1

1756 trendyline
29

35 Use of the Data Booklet is relevant to this question.

With reference to the two half equations below, which statements are correct?

[Fe(CN) 6 ]3 + e [Fe(CN) 6 ]4

Fe3+ + e Fe2+

1 [Fe(CN) 6 ]3is more stable than Fe3+

2 [Fe(CN) 6 ]4 is a stronger reducing agent than Fe2+

3 Both [Fe(CN) 6 ]3 and Fe3+ can oxidize MnO 4 2 to MnO 4

Answer: B

[Fe(CN) 6 ]3 + e [Fe(CN) 6 ]4 E = + 0.36V

Fe3+ + e Fe2+ E = + 0.77V

Option 1 is correct.From the reduction potential values, it can be inferred that CN


stabilises the higher oxidation state of Fe (Fe3+) to a greater extent than H 2 O.

Option 2 is correct.The reduction potential of [Fe(CN) 6 ]3 is less positive than that of Fe3+,
hence [Fe(CN) 6 ]3 is a weaker oxidising agent than Fe3+.Thus [Fe(CN) 6 ]4 is a stronger
reducing agent than Fe2+.

Option 3 is incorrect. Reduction potential of MnO 4 /MnO 4 2 is +0.56V, hence the cell
potential for the reaction of [Fe(CN) 6 ]3 and MnO 4 is 0.20V. Since the cell potential is
negative, the reaction is non-spontaneous.

36 Propene is found to react with chlorine in the presence of aqueous sodium bromide and
sodium nitrate. Which of the following are not possible products of the reaction?

1 CH3 CH CH2

Br Br

2 CH3 CH CH2

ONO 2 Cl

3 CH3 CH CH2

Answer: D trendyline
OH

The reaction proceeds


d via
Cl

i electrophilic
l t hili addition,
dditi
iti during
d i which
hi h a carbocation
b intermediate is

9647 / YJC / 2016 / JC2 Preliminary Examination / Paper 1

1757 trendyline
30

formed.

H H Cl



slow ++
H C C C H + Cl Cl H C C C H

H H H H H H

The carbocation reacts with the different nucleophiles present, giving rise to only these
possible products.

37 Which of the following reactions will not give a good yield of the desired product?

trendyline
9647 / YJC / 2016 / JC2 Preliminary Examination / Paper 1

1758 trendyline
31

Answer: B

Both reactions 1 and 2 will not give a good yield of the desired product.

Reaction 1 is free radical substitution which can lead to multiple substitution due to the
numerous CH bonds present.

COOH

In reaction 2, COOHwill be formed by the oxidation of the two side chains of the
benzene structure.

38 The three compounds E, F and G have the following structures?

HO
COOCH3
N
N
N
N CH CH2
N
OH O
E F G

Which statements about E, F and G are correct?

1 E and G have the same empirical formula.

2 E and F are isomers

3 The Mr of F is exactly twice that of G

Answer: A

Option 1 is correct.
The molecular formua of E is C 12 H 10 N 2 O 2
The molecular formua of G is C 6 H 5 NO
Therefore, the empirical formula of E and G are the same, C 6 H 5 NO

Option 2 is correct.
The molecular formua of E is C 12 H 10 N 2 O 2
The molecular formua of F is C 12 H 10 N 2 O 2
Therefore, E and F have the same molecular formula but different structural formula. They

trendyline
rs.
are isomers.

Option 3 iss correct.


cular
ular formua of F iss C 12 H 10 N 2 O 2
The molecular
The molecular
cular
ular formua of G is C 6 H 5 NO
Since F hasas twice the number of atoms of each ea element in G, the M r of F is exactly twice

9647 / YJC / 2016 / JC2 Preliminary Examination / Paper 1

1759 trendyline
32

that of G.

trendyline
9647 / YJC / 2016 / JC2 Preliminary Examination / Paper 1

1760 trendyline
33

39 The energy profile for the following reaction is shown below. [D = 2H]

H H

C6H5 C Cl + OH C6H5 C OH + Cl
D D

energy
X

reactants

products

Reaction pathway
Which conclusions can be drawn?

1 The product has no effect on the rotation of plane polarised light.

2 The rate of reaction can be increased by increasing concentration of OH.

3 The structure of the transition state at point X is

C6H5
HO C Cl
H D

Answer: D

Option 1 is correct.
The graphs shows 2-steps in the mechanism. Hence, it is S N 1 mechanism.

H H

slow -
C6H5 C Cl C6H5 C+ + Cl

D D

H H
fast
C+ + C6H5 C OH

trendyline
C6H5 OH
D D

The carbocation
cation
ation intermediate formed is planar, allowing OHO to approac
approach from either side
of the plane,
e, forming equimolar amount of both enantiomers. Therefore,
Therefor the product does
Therefore
not displayy any optical activity due to the rotation
tat of polarised light canc
cancelling off each other

9647 / YJC / 2016 / JC2 Preliminary Examination / Paper 1

1761 trendyline
34

due to being equal but opposite in direction of rotation.

Option 2 is not correct.


From the slow step of the mechanism, rate=k[C 6 H 5 CH(Cl)D]
Therefore, the rate of reaction is independent of [OH].

Option 3 is not correct.

C6H5
HO C Cl
H D
is the transition state from a S N 2 mechanism, and not S N 1.

40 Which groups within an amino acid are able to form a cross-chain link to stabilise the
tertiary structure of a protein?

1 CH 2 CONH 2

2 NH2
H CH2 CH2 CH2 NH C
NH

3 H CH2

N
H

Answer: A

All of the above shown are R groups of -amino acids which are used to stabilise the
tertiary structure of a protein.

trendyline
9647 / YJC / 2016 / JC2 Preliminary Examination / Paper 1

1762 trendyline
2

Answer all the questions. For


Examiners
Use
1 Planning (P)

The selective partial hydrogenation of alkynes, ZKLFK FRQWDLQ && WULSOH ERQGV, to cis-
alkenes represents an importance class of chemical transformations that have found
extensive uses such as in bioactive molecules, in lubricants as well as in the syntheses of
organic intermediates.

The partial hydrogenation of 5-decyne forms cis-dec-5-ene and the reaction is as shown :

H3C CH3

CH 3 CH 2 CH 2 CH 2 C&CH 2 CH 2 CH 2 CH 3 + H 2  H 1

The enthalpy change for hydrogenation of 5-decyne, H 1 cannot be measured directly in


the laboratory. You are required to plan an experiment to find the enthalpy change for
hydrogenation of 5-decyne, H 1 via Hesss Law.

The enthalpy change for the combustion of hydrogen is286 kJ mol1. 5-decyne and cis-
dec-5-ene are both liquids at standard conditions.

(a) Based on the given information and the equation above, state two enthalpy changes
that are necessary to calculate H 1 .

[1]

(b) Using the information given above and your answer in (a), you are required to write a
plan to determine a value for the enthalpy change for hydrogenation of 5-decyne, H 1 .

You may assume that you are provided with

liquid5-decyne
liquid cis-dec-5-ene
copper calorimeter
two spirit lamps with a 5 cm-wick each
deionised water
a lighter
thermometer
apparatus normally found in a school or college laboratory

Your plan

trendyline
n should contain the following:
a diagram
iagram
appropriate
set-up
agram of the experimental set
set--up
opriate quantities of chemicals and solutions
propriate
all essential
ti l experimental
i t l details
d t il
solution

9647 / YJC / 2016 / JC2 Preliminary Examination / Paper 2

1763 trendyline
3

Diagram of the experimental set-up For


Examiners
Use

Procedure:

trendyline

9647 / YJC / 2016 / JC2 Preliminary Examination / Paper 2


[Turn over
1764 trendyline
4
For
Examiners
Use

[7]

(c) List the measurements that have to be tabulated and recorded to calculate the value
of enthalpy change for hydrogenation of5-decyne, H 1 .In your answer, you need to
show how these measurements are used to obtain the value ofH 1 .

You may assume the specific heat capacity of water is 4.18 J g1K1 and density of
water is 1g cm3.

Tables used to tabulate and record experimental data:

trendyline
9647 / YJC / 2016 / JC2 Preliminary Examination / Paper 2

1765 trendyline
5

Treatment of results: For


Examiners
Use

[4]

[Total: 12]

trendyline
9647 / YJC / 2016 / JC2 Preliminary Examination / Paper 2
[Turn over
1766 trendyline
6

2 Eighty-five percent of all pharmaceutical agents and vitamins involve chlorine chemistry; For
Examiners
many drugs require chlorine, fluorine, or bromine to be effective. Use

Data about thehalides are given below.

Percentage of HX Boiling point Dipole K a ( HOX ) at


pK a
Halide (X) decomposed at of HX / oC moment of 298K
(HX)
2000oC CX/ D
F +3 6 x 105 19.5 1.82

Cl 7 4 x 101 84.2 1.08 2.9 x 108

Br 9 4 67.1 0.82 2.4 x 109

I 10 30 35.1 0.44

(a) Hydrogen halides decompose at high temperature according to the equation:

2HX(g) H 2 (g) + X 2 (g)

Userelevant data from the Data Booklet, explain the trend indicated by
percentagedecomposition of HX.

...

... [3]

trendyline
9647 / YJC / 2016 / JC2 Preliminary Examination / Paper 2

1767 trendyline
7

(b) The boiling point of HF is higher than the rest of the hydrogen halides despite it being For
Examiners
the smallest molecule. By means of a diagram, indicate the interaction between two Use
molecules of HF that leads to this significant difference.

[2]

(c) When dissolved in water, the hydrogen halides show different pK a values. Write
balanced equations to represent the extent of ionisation of HF and HI in water.

HF : .

HI :
[2]

(d) (i) Draw a diagram to illustrate the shapeof oxoacid, HOCl, indicating clearly the
bond angle.

[2]

(ii) Using the information from the table, state which of the two acids, HOCl or HOBr
is stronger?

. [1]

(iii) Hypoiodous acid has the formula HOI. Predict and explain whether HOI is a
stronger or weaker acid than the acid you identified in (d)(ii).

..

trendyline


..


..

..[2]

9647 / YJC / 2016 / JC2 Preliminary Examination / Paper 2


[Turn over
1768 trendyline
8

(iv) Some solid NaOCl solution is dissolved in a solution of HOClat 298 K. The pH of For
Examiners
this mixture is determined to be 6.48. Calculate the ratio of OCl to HOCl in this Use
mixture.

[2]

(e) The halides can react with concentrated sulfuric acid.

(i) When concentrated sulfuric acid was warmed with sodium chloride, white fumes
of hydrogen chloride was evolved. Write a balanced equation for the reaction.

..[1]

(ii) The method described in e(i)cannot be used to prepare hydrogen iodide.


Describe what you would observe when sodium iodide is heated with
concentrated sulfuric acid. Explain the chemistry behind it, and give any relevant
equations.

..

..

..

..

trendyline
..

.

[5]

[Total: 20]
9647 / YJC / 2016 / JC2 Preliminary Examination / Paper 2

1769 trendyline
9

3 (a) AlCl 3 is a compound of interest to scientists due to its industrial applications.For For
Examiners
example, AlCl 3 can react with molten aluminium to form AlCl(g). This is a useful Use
procedure as the AlCl formed can be used to recover useful metalloids from their
compounds.

Typically, the reaction has to be carried out in the presence of argon instead of air.

(i) The reaction has to be carried out in an atmosphere of argon. Suggest a reason
for the need of argon.

..[1]

Consider the reaction betweenAlCl 3 (g) and molten aluminium that was carried out at
1573K :

2Al(l) + AlCl 3 (g)3 AlCl(g) reaction 1

In the reaction vessel, 0.200 mol of AlCl 3 (g) was reacted with 0.400 mol of molten Al.
When the reaction system achieved equilibrium with a total pressure of 1.50atm, it
was determined that 0.200 mol of molten Al remained in the vessel.

(ii) Write an expression for the equilibrium constant, K p for reaction 1and hence
calculate its value at 1573K.

[3]

9647 / YJC / 2016 / JC2 Preliminary Examination / Paper 2


[Turn over
1770 trendyline
10

The same reaction vessel was heated to 1700K rapidly in a way suchthat no reaction For
Examiners
took place during the short interval of heating. The reaction system was allowed to re- Use
establish equilibrium. It was found that the partial pressure of AlCl 3 (g) was 0.390 atm
when the equilibrium was re-established.

(iii) Using the information from a(ii), determine the partial pressures of AlCl 3 (g) and
AlCl(g) at the instant when the reaction was heated to 1700K, before the
equilibrium was re-established.You can assume that the gases behave ideally
and the volume of vessel is unchanged.

[2]

(iv) Using the calculated values from a(iii), calculate K p of the reaction at 1700K.
Hence, explain whether reaction 1 is endothermic or exothermic.

trendyline [4]

9647 / YJC / 2016 / JC2 Preliminary Examination / Paper 2

1771 trendyline
11

(v) The rate equation for reaction between Al and AlCl 3 is shown below. For
Examiners
Use
rate = k[AlCl 3 ][Al]

Suggest which of the two mechanisms shown is a possible mechanism for the
reaction.

Mechanism 1:
Al + AlCl 3  AlCl 2 + AlCl(slow)
Al+AlCl 2  2 AlCl(fast)

Mechanism 2:
AlCl 3  AlCl+ Cl 2 (slow)
2 Al +Cl 2  2 AlCl(fast)

..

..[1]

(b) Anhydrous AlCl 3 is often used as a catalyst in the Friedel-Crafts alkylation of benzene.

AlCl 3
C 6 H 6 + CH 3 CH 2 CH 2 Cl C 6 H 5 CH 2 CH 2 CH 3 + HCl reaction 2

(i) Explain why AlCl 3 can act as a catalyst in Friedel-Crafts alkylation.

...

..[1]

(ii) With the aid of an equation, explain the need to have anhydrous condition when
AlCl 3 is used for reaction 2.

..

..

..[2]

(iii) Use information from theData Booklet, calculate the enthalpy change of reaction
for reaction 2.

trendyline [2]

9647 / YJC / 2016 / JC2 Preliminary Examination / Paper 2


[Turn over
1772 trendyline
12

(iv) Describe the mechanism for the formation of C 6 H 5 CH 2 CH 2 CH 3 in reaction 2, For


Examiners
showing clearly the movement of electrons and partial charges. Use

[3]

One key limitation of the alkylation reaction is that a mixture of products tends to form
despite using only one type of halogenoalkane.For primary halogenoalkane, this is
due to the occurrence of a process known as hydride shift which results in
rearrangement of primary carbocation intermediates as shown :

H H H
hydride shift + 
CH 3 CH 2 && CH 3 CH 2 &&+
 
H H H H

(v) Explain why hydride shift occurs for primary halogenoalkane.

..

..

..

..[3]
.

[Total: 22]

9647 / YJC / 2016 / JC2 Preliminary Examination / Paper 2

1773 trendyline
13

4 (a) Cocaine, C 17 H 21 NO 4 , was first used as a local anaesthetic. It is also a powerful For
Examiners
stimulant. Its structure is as shown: Use

O
H3CO C*
O
NCH3
O

cocaine

(i) State the functional groups, other than phenyl ring, present in cocaine.

..[1]

(ii) State the hybridisation of the carbon atom labelled with the asterisk (*).

..[1]

(b) StudentXwas asked to suggest a suitable synthetic route to prepare cocaine from
methylbenzene and compound A.
O
HO C

NCH3
HO

compound A

Student X suggested the following steps:

Methylbenzene is oxidised with hot, acidified KMnO 4 to give


Step 1
benzoic acid.
Benzoic acid is converted into benzoyl chloride by reacting the
Step 2
resultant solution from step 1 with phosphorus pentachloride.
Step 3 Benzoyl chloride is reacted with compound A at room temperature.
The resultant compound from step 3 is reacted with methanol in the
Step 4
presence of concentrated sulfuric acid at room conditions.

trendyline
9647 / YJC / 2016 / JC2 Preliminary Examination / Paper 2
[Turn over
1774 trendyline
14

Explain why steps 2 and 4 of the synthetic route will not work. For
Examiners
Use

..

[2]

(c) Cocaine is sold in its protonated hydrochloride salt, known ascocaine hydrochloride.
Suggest a possible reason for this.

[1]

(d) Smoking cocaine is more stimulating than inhaling the salt as it is absorbed quickly by
the capillaries in the lung tissues. The salt is converted back to cocaine before
smoking.

(i) Suggest a suitable reagent to convert cocaine hydrochloride back into cocaine.

..[1]

(ii) Suggest a suitable solvent to extract cocaineand separate it from its salt.

..[1]

[Total: 7]

trendyline
9647 / YJC / 2016 / JC2 Preliminary Examination / Paper 2

1775 trendyline
15

5 An industrial method for the production of ethanol, C 2 H 5 OH, is outlined in the following flow For
Examiners
diagram. Use

ethene steam

reaction vessel

cooler

ethanol, ethene and water

ethene separatorA

dilute ethanol

separatorB water

concentrated ethanol

(a) (i) Unreacted ethene is removed in separator A. Suggest how the separated
ethene could be used to increase the efficiency of the overall process of the
manufacture of ethanol.

.....

.[1]

(ii) Name the process that takes place in separator B.

.....[1]

trendyline
(b) In the reaction
ction vessel, ethanol is produced in an exothermic
action exoth reaction.

(i) State the reagents and conditions


e th onditions for the industrial
industria preparation
reparation of ethanol from
ethene.
ne.
e.

...[2]

9647 / YJC / 2016 / JC2 Preliminary Examination / Paper 2


[Turn over
1776 trendyline
16

(ii) If 1.64 kg of ethanol is produced from 10.0 kg of ethene, calculate the percentage For
Examiners
yield of ethanol. Use

[2]

(c) Ethene is used to synthesise N-ethylpropanamide as shown in the following reaction


scheme:

CH 2 CH 2 CH 3 CH 3 CH 3 CH 2 Cl Q R CH 3 CH 2 COCl
step 1step 2

P N-ethylpropanamide

(i) Give the structural formulae of compounds P, Q and R.

Compound P:

Compound Q:

Compound R:

[3]

(ii) State the reagents and conditions for steps 1 and 2.

Step1: ..

Step2: .

trendyline
[2]

[Total: 11]

End of Paper

9647 / YJC / 2016 / JC2 Preliminary Examination / Paper 2

1777 trendyline


$QVZHUDOOWKHTXHVWLRQV )RU
([DPLQHUV
  8VH
 3ODQQLQJ 3 
 
 7KH VHOHFWLYH SDUWLDO K\GURJHQDWLRQ RI DON\QHV ZKLFK FRQWDLQ && WULSOH ERQGV WR FLV
DONHQHV UHSUHVHQWV DQ LPSRUWDQFH FODVV RI FKHPLFDO WUDQVIRUPDWLRQV WKDW KDYH IRXQG
H[WHQVLYHXVHVVXFKDVLQELRDFWLYHPROHFXOHVLQOXEULFDQWVDVZHOODVLQWKHV\QWKHVHVRI
RUJDQLFLQWHUPHGLDWHV

7KHSDUWLDOK\GURJHQDWLRQRIGHF\QHIRUPVFLVGHFHQHDQGWKHUHDFWLRQLVDVVKRZQ


&+&+&+&+&&&+&+&+&++ H1



7KH HQWKDOS\ FKDQJH IRU K\GURJHQDWLRQ RI GHF\QH H1 FDQQRW EH PHDVXUHG GLUHFWO\ LQ
WKH ODERUDWRU\ <RX DUH UHTXLUHG WR SODQ DQ H[SHULPHQW WR ILQG WKH HQWKDOS\ FKDQJH IRU
K\GURJHQDWLRQRIGHF\QHH1YLD+HVVV/DZ

7KH HQWKDOS\ FKDQJH IRU WKH FRPEXVWLRQ RI K\GURJHQ LV N- PRO GHF\QH DQG FLV
GHFHQHDUHERWKOLTXLGVDWVWDQGDUGFRQGLWLRQV

 
 D  %DVHGRQWKHJLYHQLQIRUPDWLRQDQGWKHHTXDWLRQDERYHVWDWHWZRHQWKDOS\FKDQJHV
WKDWDUHQHFHVVDU\WRFDOFXODWHH1

(QWKDOS\ FKDQJH IRU FRPEXVWLRQ RI GHF\QH DQG HQWKDOS\ FKDQJH IRU
FRPEXVWLRQRIFLVGHFHQH

  
 E 8VLQJWKHLQIRUPDWLRQJLYHQDERYHDQG\RXUDQVZHULQ D \RXDUHUHTXLUHGWRZULWHD
SODQWRGHWHUPLQHDYDOXHIRUWKHHQWKDOS\FKDQJHIRUK\GURJHQDWLRQRIGHF\QHH1

<RXPD\DVVXPHWKDW\RXDUHSURYLGHGZLWK

   OLTXLGGHF\QH
 OLTXLGFLVGHFHQH
 FRSSHUFDORULPHWHU
 WZRVSLULWODPSVZLWKDFPZLFNHDFK
 GHLRQLVHGZDWHU
 DOLJKWHU
 WKHUPRPHWHU
 DSSDUDWXVQRUPDOO\IRXQGLQDVFKRRORUFROOHJHODERUDWRU\

trendyline

QVKRXOGFRQWDLQWKHIROORZLQJ
<RXUSODQVKRXOGFRQWDLQWKHIROORZLQJ
 DGLDJUDPRIWKHH[SHULPHQWDOVHWXS
LDJUDPRIWKHH[SHULPHQWDOVHWXS
DJUDPRIWKHH[SHULPHQWDOVHWXS
 DSSURSULDWHTXDQWLWLHVRIFKHPLFDOVDQGVROXWLRQV
SURSULDWHTXDQWLWLHVRIFKHPLFDOVDQGVROXWLRQV
URSULDWHTXDQWLWLHVRIFKHPLFDOVDQGVROXWLRQ
 DOOHVVHQWLDOH[SHULPHQWDOGHWDLOV
HVVHQWLDO H[SHULPHQWDO GHWDLOV

<-&-&3UHOLPLQDU\([DPLQDWLRQ3DSHU

1778 trendyline


'LDJUDPR HWXS
RIWKHH[SHUULPHQWDOVH )RU
([DPLQHUV
8VH

GHF\QH

3URFHGXUH H
 8VLLQJDF FPPHDV VXULQJF\OLQGHUPHD DVXUHFPRIZD DWHULQWR
WKH
H FRSSHU FDORULPH HWHU SURYLGHG 3ODF FH WKH WK KHUPRPHWWHU DQG
UHF
FRUGWKHLQ QLWLDOWHPS SHUDWXUHRIIZDWHULQWKHFDORULPHWHU
 $GG G GHF\QQH WR WKH VSLULW
V ODPS XQWLO LW LV DERXW KDOI ILOOHG :HLJK
XVLQJDZHLJ JKLQJEDODQFHDQGUH HFRUGWKHWRWDOPDV VV
 3OD
DFHWKHVSLULWODPSX XQGHUWKHFDORULPHWHUDQGOLJKWWKHZLF FNRIWKH
ODP
PS
 6WLUU WKH ZDWH KH WKHUPRPHWHU XQWWLO WKHUH LV
HU ZLWK WK V D ULVH RI
R DERXW

 &
& ([WLQJX XLVK WKH IODPH DQG UHFRUG WK KH ILQDO WH
HPSHUDWXUH H RI WKH
ZDWWHU
 5HZZHLJKWKHVSLULWODP PSDQGFRQ QWHQWDQGUHFRUGWKHILQDOPD DVV
 5HSSHDWWKHZ ZKROHSURF FHGXUHZLWWKFLVGHF FHQH

trendyline
<
<-&-&3UHOLPLQDU\(
([DPLQDWLRQ3D
DSHU
>7XUQR
RYHU
1779 trendyline


 F  /LVWWKHPHDVXUHPHQWVWKDWKDYHWREHWDEXODWHGDQGUHFRUGHGWRFDOFXODWHWKHYDOXH )RU
([DPLQHUV
RI HQWKDOS\ FKDQJH IRU K\GURJHQDWLRQ RIGHF\QH H1,Q \RXU DQVZHU \RX QHHG WR 8VH
VKRZKRZWKHVHPHDVXUHPHQWVDUHXVHGWRREWDLQWKHYDOXHRIH1

<RX PD\ DVVXPH WKH VSHFLILF KHDW FDSDFLW\ RI ZDWHU LV  - J. DQG GHQVLW\ RI
ZDWHULVJFP

7DEOHVXVHGWRWDEXODWHDQGUHFRUGH[SHULPHQWDOGDWD

0DVVRIVSLULWODPSDQGGHF\QHEHIRUHFRPEXVWLRQJ $
0DVVRIVSLULWODPSDQGGHF\QHDIWHUFRPEXVWLRQJ %

0DVVRIVSLULWODPSDQGFLVGHFHQHEHIRUHFRPEXVWLRQJ &
0DVVRIVSLULWODPSDQGFLVGHFHQHDIWHUFRPEXVWLRQJ '

7HPSHUDWXUH RI ZDWHU LQ FDORULPHWHU IRU  H[SHULPHQW LQYROYLQJ (
GHF\QH EHIRUHFRPEXVWLRQ&
7HPSHUDWXUH RI ZDWHU LQ FDORULPHWHU IRU H[SHULPHQW LQYROYLQJ )
GHF\QH DIWHUFRPEXVWLRQ&

7HPSHUDWXUH RI ZDWHU LQ FDORULPHWHU IRU H[SHULPHQW LQYROYLQJ *
FLVGHFHQH EHIRUHFRPEXVWLRQ&
7HPSHUDWXUH RI ZDWHU LQ FDORULPHWHU IRU H[SHULPHQW LQYROYLQJ +
FLVGHFHQH DIWHUFRPEXVWLRQ&




7UHDWPHQWRIUHVXOWV

0DVVRIGHF\QHFRPEXVWHG  $% J
$PRXQWRIGHF\QHFRPEXVWHGQ  $% PRODUPDVVRIGHF\QH ;PRO

5LVHLQWHPSHUDWXUH7  )( .

9ROXPHRIZDWHUXVHG FP
'HQVLW\RIZDWHU JFP
0DVVRIZDWHU J

$VVXPH QR KHDW ORVV WR VXUURXQGLQJV DQG QHJOLJLEOH KHDW FDSDFLW\ RI FRSSHU
FDORULPHWHU


trendyline
+HDWHYROYHGIURPFRPEXVWLRQT
YHGIURPFRPEXVWLRQT KHDWDEVRUEHGE\ZDWHU
ROYHGIURPFRPEXVWLRQT KHDWDEVRUEHG PF7
PF

    7 -
   7 -





<-&-&3UHOLPLQDU\([DPLQDWLRQ3DSHU

1780 trendyline


(QWKDOS\FKDQJHIRUFRPEXVWLRQRIGHF\QH  TQ N-PRO )RU


([DPLQHUV
 8VH
5HSHDWVDPHVWHSVIRUFLVGHFHQH



&+&&&+&+&+&++ H1




+F + 

 
  2
  2 2
 
 

+F GHF\QH  &2+2

+F FLVGHF
%\ HQH  +HVVVODZ

+1 +F GHF\QH +F + 
+F FLVGHFHQH 

+1 +F GHF\QH +F FLVGHFHQH 

  >7RWDO@
  
  
  
  
  
  
  
  
  
  
  
  
  
  
  
  
  
  

trendyline
  
  
  
  
  
  

<-&-&3UHOLPLQDU\([DPLQDWLRQ3DSHU

>7XUQRYHU
1781 trendyline


   )RU
([DPLQHUV
 (LJKW\ILYHSHUFHQWRIDOOSKDUPDFHXWLFDODJHQWVDQGYLWDPLQVLQYROYHFKORULQHFKHPLVWU\ 8VH
PDQ\GUXJVUHTXLUHFKORULQHIOXRULQHRUEURPLQHWREHHIIHFWLYH

'DWDDERXWWKHKDOLGHVDUHJLYHQEHORZ

3HUFHQWDJHRI+; %RLOLQJSRLQW 'LSROH .D +2; DW
+DOLGH ;  S.D +;  GHFRPSRVHGDW RI+;R& PRPHQWRI .
R& &;'
)  [   

&l  [   [

%U     [

,     




 D  +\GURJHQKDOLGHVGHFRPSRVHDWKLJKWHPSHUDWXUHDFFRUGLQJWRWKHHTXDWLRQ

+; J + J ; J 

8VHUHOHYDQW GDWD IURP WKH Data Booklet H[SODLQ WKH WUHQG LQGLFDWHG E\
SHUFHQWDJHGHFRPSRVLWLRQRI+;


%RQGHQHUJLHVRI+;LQN-PRO LQGHFUHDVLQJRUGHU
+)+&l+%U+,
VKRZVGHFUHDVLQJERQGVWUHQJWKIURP+)WR+,

&RYDOHQW ERQG VWUHQJWK GHFUHDVHV IURP +) WR +, LV GXH WR LQFUHDVLQJ DWRPLF
UDGLXVRI)WR,OHDGLQJWROHVVHIIHFWLYHRYHUODSRIRUELWDOVIURP)WR,ZLWK+

+HQFHWKHUPDOVWDELOLW\GHFUHDVHVLQWKHRUGHU+)+&l+%U+,DQGSHUFHQWDJH
GHFRPSRVLWLRQLQFUHDVHVIURP+)WR+,DVLQGLFDWHGLQWKHWDEOH
   
 E  7KHERLOLQJSRLQWRI+)LVKLJKHUWKDQWKHUHVWRIWKHK\GURJHQKDOLGHVGHVSLWHLWEHLQJ
WKH VPDOOHVW PROHFXOH %\ PHDQV RI D GLDJUDP LQGLFDWH WKH LQWHUDFWLRQ EHWZHHQ WZR
PROHFXOHVRI+)WKDWOHDGVWRWKLVVLJQLILFDQWGLIIHUHQFH

    



+\GURJHQERQG


trendyline







<-&-&3UHOLPLQDU\([DPLQDWLRQ3DSHU

1782 trendyline


 )RU
([DPLQHUV
 F  :KHQ GLVVROYHG LQ ZDWHU WKH K\GURJHQ KDOLGHV VKRZ GLIIHUHQW S.D YDOXHV :ULWH 8VH
 EDODQFHGHTXDWLRQVWRUHSUHVHQWWKHH[WHQWRILRQLVDWLRQRI+)DQG+,LQZDWHU

+)+)+2)+2PXVWEHUHYHUVLEOHDUURZ

+,+,+2,+2RU+,,+
  
 G  L  'UDZ D GLDJUDP WR LOOXVWUDWH WKH VKDSHRI R[RDFLG +2&l LQGLFDWLQJ FOHDUO\ WKH
ERQGDQJOH

 


 VKDSHEHQW  ERQGDQJOHR

  LL  8VLQJWKHLQIRUPDWLRQIURPWKHWDEOHVWDWHZKLFKRIWKHWZRDFLGV+2&lRU+2%U
LVVWURQJHU"

+2&lLVDVWURQJHUDFLGDVLWV.DYDOXHLVODUJHUWKDQWKDWRI+2%U

  LLL  +\SRLRGRXV DFLG KDV WKH IRUPXOD +2, 3UHGLFW DQG H[SODLQ ZKHWKHU +2, LV D
VWURQJHURUZHDNHUDFLGWKDQWKHDFLG\RXLGHQWLILHGLQ G LL 
   
+2, LV D ZHDNHU DFLG 7KH FRQMXJDWH EDVH 2&l LV PRUH VWDEOH WKDQ 2,
7KLV LV GXH WR &lEHLQJ PRUH HOHFWURQHJDWLYH KHQFH PRUH HOHFWURQ
ZLWKGUDZLQJ DQG GLVSHUVHV WKH QHJDWLYH FKDUJH RQ R[\JHQ WR D ODUJHU
H[WHQWVWDELOLVLQJ2&l+HQFH+2&lLRQLVHVPRUHWKDQ+2,

25

+2,LVDZHDNHUDFLG7KHFRQMXJDWHEDVH2,LVOHVVVWDEOHWKDQ2&l7KLV
LV GXH WR ,EHLQJ OHVV HOHFWURQHJDWLYH KHQFH OHVV HOHFWURQ ZLWKGUDZLQJ
GLVSHUVHV WKH QHJDWLYH FKDUJH RQ R[\JHQ WR D VPDOOHU H[WHQW +HQFH
+2,LRQLVHVOHVVWKDQ+2&l

  LY  6RPHVROLG1D2&lVROXWLRQLVGLVVROYHGLQDVROXWLRQRI+2&lDW.7KHS+RI
WKLVPL[WXUHLVGHWHUPLQHGWREH&DOFXODWHWKHUDWLRRI2&lWR+2&lLQWKLV
PL[WXUH

8VLQJS+ S.DOJ >2&l@>+2&l@
 OJ [ OJ >2&l@>+2&l@

trendyline
>2&

>2&ll@>+2&l@
l 







<-&-&3UHOLPLQDU\([DPLQDWLRQ3DSHU

>7XUQRYHU
1783 trendyline


 )RU
([DPLQHUV
 H  7KHKDOLGHVFDQUHDFWZLWKFRQFHQWUDWHGVXOIXULFDFLG 8VH
   
  L  :KHQFRQFHQWUDWHGVXOIXULFDFLGZDVZDUPHGZLWKVRGLXPFKORULGHZKLWHIXPHV
RIK\GURJHQFKORULGHZDVHYROYHG:ULWHDEDODQFHGHTXDWLRQIRUWKHUHDFWLRQ

1D&l+621D+62+&l
   
  LL  7KH PHWKRG GHVFULEHG LQ H L FDQQRW EH XVHG WR SUHSDUH K\GURJHQ LRGLGH
'HVFULEH ZKDW \RX ZRXOG REVHUYH ZKHQ VRGLXP LRGLGH LV KHDWHG ZLWK
FRQFHQWUDWHGVXOIXULFDFLG([SODLQWKHFKHPLVWU\EHKLQGLWDQGJLYHDQ\UHOHYDQW
HTXDWLRQV

1D,+621D+62+,
 
+,+62,+6+2   

%ODFNVROLGDQGYLROHWIXPHVRI,VHHQDQGSXQJHQW RUURWWHQHJJVPHOO 
+6HYROYHG

+,LVDVWURQJHUUHGXFLQJDJHQWWKDQ+&l
   
+,LVR[LGL]HGWRLRGLQHKHQFHSXUH+,FDQQRWEHREWDLQHG
   
>7RWDO@
  
 D  $l&l LV D FRPSRXQG RI LQWHUHVW WR VFLHQWLVWV GXH WR LWV LQGXVWULDO DSSOLFDWLRQV)RU
H[DPSOH $l&l FDQ UHDFW ZLWK PROWHQ DOXPLQLXP WR IRUP $l&l J  7KLV LV D XVHIXO
SURFHGXUH DV WKH $l&l IRUPHG FDQ EH XVHG WR UHFRYHU XVHIXO PHWDOORLGV IURP WKHLU
FRPSRXQGV

7\SLFDOO\WKHUHDFWLRQKDVWREHFDUULHGRXWLQWKHSUHVHQFHRIDUJRQLQVWHDGRIDLU

  L  7KHUHDFWLRQKDVWREHFDUULHGRXWLQDQDWPRVSKHUHRIDUJRQ6XJJHVWDUHDVRQ
IRUWKHQHHGRIDUJRQ
   
7RSUHYHQWR[\JHQLQDLUIURPUHDFWLQJZLWKPROWHQDOXPLQLXP

  &RQVLGHUWKHUHDFWLRQEHWZHHQ$l&l J DQGPROWHQDOXPLQLXPWKDWZDVFDUULHGRXWDW
.

$l O $l&l J $l&l J UHDFWLRQ




trendyline
DF
DFWLRQYHVVHOPRORI$ l&l J ZDVUHDFWHGZLWK
,QWKHUHDFWLRQYHVVHOPRORI$l&l J ZDVUHDFWHGZLWKPRORIPROWHQ$l
VUHD
:KHQ WKH
H UHDFWLRQ V\VWHP DFKLHYHG HTXLOLEULXP ZLWK D WRWDO SUHVVXUH
SUHVVX RI DWP LW
SUHVVX
ZDVGHWHUPLQHGWKDWPRORIPROWHQ$lUHPDLQHGLQWKHYHVVHO
HUPLQHGWKDWPRORIPROWHQ$
UPLQHGWKDWPRORIPROWHQ$llUHPDLQHGLQWKHYHVVHO
UHPDLQHGLQWKHYHVVHO



<-&-&3UHOLPLQDU\([DPLQDWLRQ3DSHU

1784 trendyline


 )RU
([DPLQHUV
  LL  :ULWH DQ H[SUHVVLRQ IRU WKH HTXLOLEULXP FRQVWDQW .SIRU UHDFWLRQ DQG KHQFH 8VH
FDOFXODWHLWVYDOXHDW.

$l O $l&l J $l&l J 



QLPRO

QFPRO[

QHPRO

7RWDOQXPEHURIPROHVRIJDVHVDWHTXLOLEULXP  PRO

.S  3$l&l  3$lCl   ;  ;  DWP




  
 7KHVDPHUHDFWLRQYHVVHOZDVKHDWHGWR.UDSLGO\LQDZD\VXFKWKDWQRUHDFWLRQ
 WRRNSODFHGXULQJWKHVKRUWLQWHUYDORIKHDWLQJ7KHUHDFWLRQV\VWHPZDVDOORZHGWRUH
HVWDEOLVKHTXLOLEULXP,WZDVIRXQGWKDWWKHSDUWLDOSUHVVXUHRI$l&l J ZDVDWP
ZKHQWKHHTXLOLEULXPZDVUHHVWDEOLVKHG
   
  LLL  8VLQJWKHLQIRUPDWLRQIURPD LL GHWHUPLQHWKHSDUWLDOSUHVVXUHVRI$l&l J DQG
$l&l J  DW WKH LQVWDQW ZKHQ WKH UHDFWLRQ ZDV KHDWHG WR . EHIRUH WKH
HTXLOLEULXP ZDV UHHVWDEOLVKHG<RX FDQ DVVXPH WKDW WKH JDVHV EHKDYH LGHDOO\
DQGWKHYROXPHRIYHVVHOLVXQFKDQJHG

39 Q57VLQFH95DQGQDUHFRQVWDQW37





7KXV






3$l&l DWP  




ttrendyline




3$l&l3
&ll3 DWP  



<-&-&3UHOLPLQDU\([DPLQDWLRQ3DSHU

>7XUQRYHU
1785 trendyline


 )RU
([DPLQHUV
  LY  8VLQJ WKH FDOFXODWHG YDOXHV IURP D LLL  FDOFXODWH .S RI WKH UHDFWLRQ DW . 8VH
+HQFHH[SODLQZKHWKHUUHDFWLRQLVHQGRWKHUPLFRUH[RWKHUPLF
   
$l O $l&l J $l&l J 

3LDWP

3FDWP

3HDWP

.S    DWP



.SYDOXHLVODUJHUDW.FRPSDUHGWR.VKRZLQJIRUZDUGUHDFWLRQLV
IDYRXUHG %\ /H &KDWHOLHUV 3ULQFLSOH UHDFWLRQ  HTXLOLEULXP SRVLWLRQ KDV
VKLIWHGWRWKHULJKWWRUHPRYHH[FHVVKHDW

+HQFHUHDFWLRQLVHQGRWKHUPLF
   
  Y  7KHUDWHHTXDWLRQIRUUHDFWLRQEHWZHHQ$lDQG$l&lLVVKRZQEHORZ

UDWH N>$l&l@>$l@

6XJJHVW ZKLFK RI WKH WZR PHFKDQLVPV VKRZQ LV D SRVVLEOH PHFKDQLVP IRU WKH
UHDFWLRQ

0HFKDQLVP
$l$l&l$l&l2$l&l VORZ 
$l$l&l2 $l&l IDVW 

0HFKDQLVP
$l&l$l&l&l VORZ 
$l&l$l&l IDVW 

0HFKDQLVP  LV WKH SRVVLEOH PHFKDQLVP ,W WDOOLHV ZLWK WKH UDWH HTXDWLRQ
ZKHUHRQHPROHRI$lUHDFWVZLWKPROHRI$l&lLQWKHVORZVWHS

 E  $QK\GURXV$l&lLVRIWHQXVHGDVDFDWDO\VWLQWKH)ULHGHO&UDIWVDON\ODWLRQRIEHQ]HQH

$l&l3

trendyline
d
&+&+&+&+&l&
l +&+
& &+&+
++&
+&lUHDFWLRQ
 l
l
   
  L  ([SODLQZK\$l&l
SODLQZK\$
ODLQZK\$ll&lFDQDFWDVDFDWDO\VWLQ)ULHGHO&UDIWVDON\ODWLR
FDQDFWDVDFDWDO\VWLQ)ULHGHO&UDIWVDON\ODWLRQ
   
 $l KDV
KDV HPSW\
HPSW\ S
S RUELWDO
RUELWDO RU
RU HPSW\
HPSW\
PSW S S RUELWDO
RUELWDO  WR
WR DFFHSW
DFFHSW D ORQH SDLU RI
HOHFWURQVZKHQJHQHUDWLQJHOHFWURSKLOH
<-&-&3UHOLPLQDU\([DPLQDWLRQ3DSHU

1786 trendyline


 )RU
([DPLQHUV
  LL  :LWKWKHDLGRIDQHTXDWLRQH[SODLQWKHQHHGWRKDYHDQK\GURXVFRQGLWLRQZKHQ 8VH
$l&lLVXVHGIRUUHDFWLRQ

>$l +2 @+2>$l +2 2+@+2


$l&l ZLOO GLVVROYH RU XQGHUJRHV K\GUDWLRQ  LQ ZDWHU WR IRUP >$l +2 @ 
ZKLFKWKHQXQGHUJRHVK\GURO\VLVLQZDWHU

  LLL  8VHLQIRUPDWLRQIURPWKHData Booklet, FDOFXODWHWKHHQWKDOS\FKDQJHRIUHDFWLRQ
IRUUHDFWLRQ

+ >%( &+ %( &&l @>%( +&l %( && @
      N-PRO



   
  LY  'HVFULEH WKH PHFKDQLVP IRU WKH IRUPDWLRQ RI &+&+&+&+ LQ UHDFWLRQ 
VKRZLQJFOHDUO\WKHPRYHPHQWRIHOHFWURQVDQGSDUWLDOFKDUJHV

(OHFWURSKLOLFVXEVWLWXWLRQ

6WHS*HQHUDWLRQRIVWURQJHOHFWURSKLOH
IDVW
&+&+&+&l$l&l &+&+&+$l&l

6WHS(OHFWURSKLOLFDWWDFNRQULQJWRIRUPDFDUERFDWLRQLQWHUPHGLDWH

 + VORZ

&+&+&+   



6WHS5HVWRULQJWKHEHQ]HQHULQJDQGUHJHQHUDWLRQRIFDWDO\VW


IDVW


  $l&l     +&l$l&l
 


trendyline





<-&-&3UHOLPLQDU\([DPLQDWLRQ3DSHU

>7XUQRYHU
1787 trendyline


 )RU
([DPLQHUV
  2QHNH\OLPLWDWLRQRIWKHDON\ODWLRQUHDFWLRQLVWKDWDPL[WXUHRISURGXFWVWHQGVWRIRUP 8VH
GHVSLWH XVLQJ RQO\ RQH W\SH RI KDORJHQRDONDQH)RU SULPDU\ KDORJHQRDONDQH WKLV LV
GXH WR WKH RFFXUUHQFH RI D SURFHVV NQRZQ DV K\GULGH VKLIW ZKLFK UHVXOWV LQ
UHDUUDQJHPHQWRISULPDU\FDUERFDWLRQLQWHUPHGLDWHVDVVKRZQ

+++
K\GULGHVKLIW
&+&+&&&+&+&&+

++++

  Y  ([SODLQZK\K\GULGHVKLIWRFFXUVIRUSULPDU\KDORJHQRDONDQH

+\GULGH VKLIW OHDGV WR D PRUH VWDEOH VHFRQGDU\ FDUERFDWLRQDV WKHUH DUH
PRUHHOHFWURQGRQDWLQJDON\OJURXSVGLUHFWO\ERQGHGWRSRVLWLYHO\FKDUJHG
FDUERQ+HQFHWKHSRVLWLYHFKDUJHLQWKHFDUERFDWLRQLVGLVSHUVHG

>7RWDO@
  
 D  &RFDLQH &+12 ZDV ILUVW XVHG DV D ORFDO DQDHVWKHWLF  ,W LV DOVR D SRZHUIXO
VWLPXODQW,WVVWUXFWXUHLVDVVKRZQ

 2

+&2 &

2
 1&+

2



FRFDLQH

  L  6WDWHWKHIXQFWLRQDOJURXSVRWKHUWKDQSKHQ\OULQJSUHVHQWLQFRFDLQH

(VWHUDQGWHUWLDU\DPLQH
   
  LL  6WDWHWKHK\EULGLVDWLRQRIWKHFDUERQDWRPODEHOOHGZLWKWKHDVWHULVN 

VS




trendyline







<-&-&3UHOLPLQDU\([DPLQDWLRQ3DSHU

1788 trendyline


 )RU
([DPLQHUV
 E  6WXGHQW;ZDV DVNHG WR VXJJHVW D VXLWDEOH V\QWKHWLF URXWH WR SUHSDUH FRFDLQH IURP 8VH
PHWK\OEHQ]HQHDQGFRPSRXQG$ 
 2


+2 &
 1&+

 +2

FRPSRXQG$

6WXGHQW;VXJJHVWHGWKHIROORZLQJVWHSV

0HWK\OEHQ]HQH LV R[LGLVHG ZLWK KRW DFLGLILHG .0Q2 WR JLYH
6WHS
EHQ]RLFDFLG
%HQ]RLF DFLG LV FRQYHUWHG LQWR EHQ]R\O FKORULGH E\ UHDFWLQJ WKH
6WHS
UHVXOWDQWVROXWLRQIURPVWHSZLWKSKRVSKRUXVSHQWDFKORULGH
6WHS %HQ]R\OFKORULGHLVUHDFWHGZLWKFRPSRXQG$DWURRPWHPSHUDWXUH
7KHUHVXOWDQWFRPSRXQGIURPVWHSLVUHDFWHGZLWKPHWKDQROLQWKH
6WHS
SUHVHQFHRIFRQFHQWUDWHGVXOIXULFDFLGDWURRPFRQGLWLRQV


  
([SODLQZK\VWHSVDQGRIWKHV\QWKHWLFURXWHZLOOQRWZRUN

,Q VWHS  SKRVSKRUXV SHQWDFKORULGH ZLOO EH K\GURO\VHG LQ WKH SUHVHQFH RI
DTXHRXVVROXWLRQ RUSKRVSKRUXVSHQWDFKORULGHZLOOUHDFWZLWKZDWHU 

+HDWLVUHTXLUHGIRUVWHS

 F  &RFDLQHLVVROGLQLWVSURWRQDWHGK\GURFKORULGHVDOWNQRZQDVFRFDLQHK\GURFKORULGH
 6XJJHVWDSRVVLEOHUHDVRQIRUWKLV

,WKDVDKLJKHUPHOWLQJSRLQWDQGLVPRUHVWDEOH

 G  6PRNLQJFRFDLQHLVPRUHVWLPXODWLQJWKDQLQKDOLQJWKHVDOWDVLWLVDEVRUEHGTXLFNO\E\
WKH FDSLOODULHV LQ WKH OXQJ WLVVXHV  7KH VDOW LV FRQYHUWHG EDFN WR FRFDLQH EHIRUH
VPRNLQJ

  L  6XJJHVWDVXLWDEOHUHDJHQWWRFRQYHUWFRFDLQHK\GURFKORULGHEDFNLQWRFRFDLQH

1D2+ DT 

trendyline

  LL  6XJJHVWDVXLWDEOHVROYHQWWRH[WUDFWFRFDLQHDQGVHSDUDWHLWIURPLWVVDOW
JJHVWDVXLWDEOHVROYHQWWRH[WUDFWFRFDLQHDQGVHSDUDWHLWIUR
JJHVWDVXLWDEOHVROYHQWWRH[WUDFWFRFDLQHDQGVHSDUDWHLWIURP

(WKHU RUDQ\RWKHURUJDQLFVROYHQWOHVVSRODUWKDQZDWHU 
KHU RUDQ\RWKHURUJDQLFVROYHQWOHVVSRODUWKDQZDWHU
HU RUDQ\RWKHURUJDQLFVROYHQWOHVVSRODUWKDQZDWHU
  
  >7RWDO@

<-&-&3UHOLPLQDU\([DPLQDWLRQ3DSHU

>7XUQRYHU
1789 trendyline


   )RU
 $QLQGXVWULDOPHWKRGIRUWKHSURGXFWLRQRIHWKDQRO&+2+LVRXWOLQHGLQWKHIROORZLQJIORZ ([DPLQHUV
8VH
GLDJUDP

HWKHQH VWHDP



 UHDFWLRQYHVVHO


 FRROHU




HWKDQROHWKHQHDQGZDWHU



 HWKHQH VHSDUDWRU$


 GLOXWHHWKDQRO



 VHSDUDWRU% ZDWHU



 FRQFHQWUDWHGHWKDQRO


 D  L  8QUHDFWHG HWKHQH LV UHPRYHG LQ VHSDUDWRU $ 6XJJHVW KRZ WKH VHSDUDWHG
HWKHQH FRXOG EH XVHG WR LQFUHDVH WKH HIILFLHQF\ RI WKH RYHUDOO SURFHVV RI WKH
PDQXIDFWXUHRIHWKDQRO

7KH VHSDUDWHG HWKHQH FDQ EH UHWXUQHG EDFN WR WKH UHDFWLRQ YHVVHO WR
SURGXFHPRUHHWKDQRO

  LL  1DPHWKHSURFHVVWKDWWDNHVSODFHLQVHSDUDWRU%

)UDFWLRQDOGLVWLOODWLRQ RUGLVWLOODWLRQ 


trendyline







<-&-&3UHOLPLQDU\([DPLQDWLRQ3DSHU

1790 trendyline


 )RU
([DPLQHUV
 E  ,QWKHUHDFWLRQYHVVHOHWKDQROLVSURGXFHGLQDQH[RWKHUPLFUHDFWLRQ 8VH

  L  6WDWH WKH UHDJHQWV DQG FRQGLWLRQV IRU WKH LQGXVWULDO SUHSDUDWLRQ RI HWKDQRO IURP
HWKHQH

+2DQGFRQFHQWUDWHG+32
&DWP
   
  LL  ,INJRIHWKDQROLVSURGXFHGIURPNJRIHWKHQHFDOFXODWHWKHSHUFHQWDJH
\LHOGRIHWKDQRO


&+ &+ J +2 J &+&+2+ O 




7KHRUHWLFDO\LHOGRI   [J

NJ
NJ


3HUFHQWDJH\LHOG   


 F  (WKHQHLVXVHGWRV\QWKHVLVH1HWK\OSURSDQDPLGHDVVKRZQLQWKHIROORZLQJUHDFWLRQ
VFKHPH

&+&+ &+&+ &+&+&l 4 5 &+&+&2&l
VWHSVWHS


3 1HWK\OSURSDQDPLGH


  L  *LYHWKHVWUXFWXUDOIRUPXODHRIFRPSRXQGV34DQG5

3&+&+1+
4&+&+&1
5&+&+&22+

  LL  6WDWHWKHUHDJHQWVDQGFRQGLWLRQVIRUVWHSVDQG

VWHSHWKDQROLF.&1KHDW
VWHSGLOXWHVXOIXULFDFLGKHDW
  

trendyline
  >7RWDO@

(QGRI3DSHU
(QGRI3DSHU

<-&-&3UHOLPLQDU\([DPLQDWLRQ3DSHU

>7XUQRYHU
1791 trendyline


$QVZHUDQ\IRXUTXHVWLRQV
 
 3KRVSKRUXVZDVILUVWGLVFRYHUHGE\WKH*HUPDQDOFKHPLVW+HQQLJ%UDQGWVH[SHULPHQWZLWK
KXPDQ ZDVWH XULQH 3KRVSKRUXV RFFXUV LQ GLIIHUHQW DOORSWURSLF IRUPV ZLWK WKH WZR FRPPRQ
RQHVEHLQJZKLWHSKRVSKRUXVDQGUHGSKRVSKRUXV

 D  'HSHQGLQJ LI R[\JHQ LV LQ H[FHVV SKRVSKRUXV UHDFWV ZLWK R[\JHQ WR IRUP WZR GLIIHUHQW
R[LGHVSKRVSKRUXV ,,, R[LGHDQGSKRVSKRUXV 9 R[LGH

  L  :ULWH WKH HTXDWLRQV IRU WKH UHDFWLRQV RI SKRVSKRUXV ,,,  R[LGH DQG SKRVSKRUXV 9 
R[LGHZLWKZDWHUVHSDUDWHO\LQFOXGLQJVWDWHV\PEROV>@

  LL  'HVFULEH WKH REVHUYDWLRQ ZKHQ WKH UHVXOWLQJ VROXWLRQV IURP L  DUH DGGHG WRVROLG
DOXPLQLXPR[LGH>@

 E  1LWURJHQ ZKLFK LV LQ WKH VDPH JURXS DV SKRVSKRUXV IRUPV DQ LPSRUWDQW
PROHFXOHDPPRQLD1+1+LVXVHGWRPDNHIHUWLOLVHUVDVZHOODVH[SORVLYHV

  L  :KHQ DTXHRXV DPPRQLD LV JUDGXDOO\ DGGHG WR DQ DTXHRXV VROXWLRQ FRQWDLQLQJ
&X DT  VHYHUDO REVHUYDWLRQV DUH VHHQ 'HVFULEH DQG H[SODLQ WKHVH REVHUYDWLRQV
DQGZULWHHTXDWLRQVZKHUHDSSURSULDWH>@
   
  LL  :KHQ EODFN FRSSHU ,,  R[LGH LV VWLUUHG ZLWK OLTXLG DPPRQLD LW GLVVROYHV WR JLYH D
FRORXUHG VROXWLRQ 'XULQJ WKLV UHDFWLRQ WKH R[LGH LRQ LV DFWLQJ DV D %URQVWHG/RZU\
EDVH6XJJHVWDQHTXDWLRQIRUWKLVUHDFWLRQDQGVXJJHVWWKHFRORXURIWKHVROXWLRQ
           >@
   
 F  $ JURXS RI UHVHDUFKHUVKDYH UHFHQWO\ GHYHORSHG DQ HIILFLHQW ZD\ RI SURGXFLQJ K\GURJHQ
IURPXULQHDQLWURJHQFRQWDLQLQJFRPSRXQGDVVKRZQLQWKHGLDJUDPEHORZ






1    +
DQRGH DONDOLQHXUHDHOHFWURO\WLF FDWKRGH
 FHOO
XUHD 

8UHD&2 1+ LQDTXHRXV.2+ZKLFKDFWVDVWKHHOHFWURO\WHLVR[LGL]HGDWWKHDQRGH
WRIRUPQLWURJHQDQGFDUERQGLR[LGH$WWKHFDWKRGHZDWHULVUHGXFHGWRK\GURJHQJDV


trendyline







<-&-&3UHOLPLQDU\([DPLQDWLRQ3DSHU

1792 trendyline


7KHRYHUDOOHTXDWLRQIRUWKHXUHDHOHFWURO\WHFHOOLV

&2 1+  DT +2 O 1 J + J &2 J 

  L  :ULWH WKH KDOI HTXDWLRQ ZLWK VWDWH V\PEROV RFFXUULQJ DW HDFK HOHFWURGH LQ WKLV XUHD
HOHFWURO\WLFFHOO>@
   
  LL  %\ FRQVLGHULQJ ZK\ &2 LV QRW OLEHUDWHG DW WKH DQRGH GHGXFH WKH E\SURGXFW
SURGXFHGDWWKHDQRGH>@
   
 G  1LWURJHQ LV DOVR FRPPRQO\ IRXQG LQ RUJDQLF FRPSRXQGV ' &+21 LV D QHXWUDO
FRPSRXQG ZLWK D EHQ]HQH ULQJ 8SRQ UHDFWLRQ ZLWK KRW VRGLXP K\GUR[LGH ( DQG ) DUH
IRUPHG 2Q DFLGLILFDWLRQ ( IRUPV &+2ZKLFK JLYHV HIIHUYHVFHQFH ZLWK VRGLXP
FDUERQDWH

2QH PROH RI ) &+12 UHDFWV ZLWK RQH PROH RI DTXHRXV VXOIXULF DFLG DW URRP
WHPSHUDWXUH2QHPROHRI)LVDOVRDEOHWRGHFRORXULVHWKUHHPROHVRIDTXHRXVEURPLQH
DQGJLYHVDZKLWHSSW*

:KHQDFLGLILHGRUDQJHSRWDVVLXPGLFKURPDWH 9, LVDGGHGWR)FRPSRXQG+&+12LV
IRUPHGDQGWKHVROXWLRQWXUQVJUHHQ$\HOORZSSWLVDOVRIRUPHGZKHQ)DQG+LVUHDFWHG
ZLWKDONDOLQHDTXHRXVLRGLQHVHSDUDWHO\

6XJJHVW WKH VWUXFWXUHV IRU FRPSRXQGV'()* DQG + DQG H[SODLQ WKH REVHUYDWLRQV
GHVFULEHG DERYH  <RX GR QRW QHHG WR FRQVLGHU WKH SRVLWLRQV RI WKH VXEVWLWXHQWV RQ WKH
EHQ]HQHULQJ>@
   
   >7RWDO@
   















trendyline







<-&-&3UHOLPLQDU\([DPLQDWLRQ3DSHU

1793 trendyline


 D  :LWK WKH DLG RI DQ HTXDWLRQ GHILQH WKH WHUP standard enthalpy changeof formation RI
FDOFLXPFDUERQDWH&D&2>@
  
 E  7KH HQWKDOS\ FKDQJH RI WKH UHDFWLRQ + IRU WKH UHDFWLRQ VKRZQ EHORZ LV GHWHUPLQHG WR
EHN-PRO

&D V +&l DT &D&l DT + J   + N-PRO

,Q D VLPLODU H[SHULPHQW VROLG FDOFLXP FDUERQDWH UHDFWHG ZLWK DQ H[FHVV RI K\GURFKORULF
DFLGDQGWKHHQWKDOS\FKDQJHRIUHDFWLRQZDVIRXQGWREHN-SHUPROHRIFDOFLXP
FDUERQDWH

&D&2 V +&l DT &D&l DT &2 J +2 l + N-PRO

7KHYDOXHVRIHQWKDOS\FKDQJHRIIRUPDWLRQRIZDWHUDQGHQWKDOS\FKDQJHRIFRPEXVWLRQ
RIFDUERQDUHSURYLGHG
+IR +2  N-PRO
+FR &  N-PRO

8VLQJ WKH LQIRUPDWLRQ JLYHQ FDOFXODWH WKH VWDQGDUG HQWKDOS\ FKDQJH RI IRUPDWLRQ RI
FDOFLXPFDUERQDWH>@
   
 F  0DJQHVLXP FDUERQDWH GHFRPSRVHV RQ KHDWLQJ DFFRUGLQJ WR WKH HTXDWLRQ VKRZQ EHORZ
7KH YDOXHV RI WKH VWDQGDUG HQWKDOS\ FKDQJH DQG VWDQGDUG HQWURS\ FKDQJH RI WKH
GHFRPSRVLWLRQDUHSURYLGHG
0J&2 V 0J2 V &2 J 

+ N-PRO6 -PRO.

  L  :K\GRHVWKHHQWURS\LQFUHDVHZKHQPDJQHVLXPFDUERQDWHGHFRPSRVHV">@
   

  LL  &DOFXODWH WKH VWDQGDUG *LEEV IUHH HQHUJ\ FKDQJH *  IRU WKH GHFRPSRVLWLRQ RI
PDJQHVLXPFDUERQDWH+HQFHFRPPHQWRQWKHVSRQWDQHLW\RIWKHGHFRPSRVLWLRQRI
PDJQHVLXPFDUERQDWHDWURRPWHPSHUDWXUH>@
   
  LLL  &RPSDUH WKH HDVH RI WKHUPDO GHFRPSRVLWLRQ RI PDJQHVLXP FDUERQDWH ZLWK EDULXP
FDUERQDWH   >@
   
 G  'HVFULEH WKH UHDFWLRQ RI 0J DQG &D ZLWK ZDWHU JLYLQJ DSSURSULDWH HTXDWLRQV ZLWK VWDWH
V\PEROV>@
  
  

trendyline
  
  
  
  
  
  

<-&-&3UHOLPLQDU\([DPLQDWLRQ3DSHU

1794 trendyline


 H  0J UHDFWV ZLWK KDORJHQRDONDQHV 5; LQ GU\ HWKHU IRUPLQJ *ULJQDUG UHDJHQWV ZKLFK DUH
LPSRUWDQWLQWHUPHGLDWHVXVHGWRSURGXFHPDQ\RUJDQLFFRPSRXQGV

GU\HWKHU
5;0J50J;
   
   *ULJQDUGUHDJHQW

50J;LVFDOOHGDQRUJDQRPHWDOOLFFRPSRXQGZKHUH5LVDQDON\OJURXSDQG;LVFKORULQH
EURPLQH RU LRGLQH5 LQ 50J; EHKDYHV OLNH DQ DQLRQ 5 DQG LV VWURQJO\ EDVLF DQG
QXFOHRSKLOLF


+ &
2QUHDFWLRQZLWKFDUERQ\OFRPSRXQGV50J;IRUPVYDULRXVFODVVHVRIDOFRKROV
+




G
U
\

H
W
K
H
U

S
U
L
P
D
U

\
D
O
F
R
K
R
O


5
0
J
;
&
2

2
+
P
H
W
K
D
Q
D
O





+
2
+

5


+ &
+
D
O
G
H
K
\
G

H
R
W
K
H
U





G
U
\

H
W
K
H
U

V
H
F
R
Q
G
D
U
\

D
O
F
R
K
R
O


5
0
J
;
&
2

2
+
W
K
D
Q

P
H
W
K
D
Q
D
O





+
2
5

5


5 & 5




G
U

\
H
W
K
H
U

W
H
U
W
L
D
U
\

D
O
F
R
K
R
O


5
0
J
;
&
2

2
+
N
H
W
R
Q
H





+
2
5

5



   
  L  'UDZWKHVWUXFWXUHRIWKHRUJDQLFSURGXFWVIRUPHGZKHQWKHIROORZLQJFRPSRXQGVDUH
UHDFWHG
2 &


&
+



0
J
%
U

D
Q
G



>@
   
  LL  *LYHWKHGLVSOD\HGIRUPXODRIWKHEURPRDONDQHZKLFKFDQEHXVHGWRIRUPD*ULJQDUG
UHDJHQWWKDWZRXOGUHDFWZLWKEXWDQDOWRJLYHPHWK\OKH[DQRO>@
   
  LLL  :KHQ50J%ULVUHDFWHGZLWKHWKDQR\OEURPLGHPROHVRI50J%UDUHUHTXLUHGSHU
PROH RI HWKDQR\O EURPLGH DQG WKH SURGXFW IRUPHG LV &+&52+([SODLQ ZK\ RQH
PROH RI HWKDQR\O EURPLGH UHDFWV ZLWK WZR PROHV RI 50J%U ZKLOH RQH PROH RI
DOGHK\GHRUNHWRQHUHDFWVZLWKRQHPROHRI50J%U>@
   
   


trendyline






<-&-&3UHOLPLQDU\([DPLQDWLRQ3DSHU

1795 trendyline


 I  .HWRQHVDQGDOGHK\GHVUHDFWZLWK+&1YLDQXFOHRSKLOLFDGGLWLRQPHFKDQLVPVLPLODUWRWKH
UHDFWLRQZLWK*ULJQDUGUHDJHQWV7KHUHDFWLRQVFKHPHEHORZLVRQHVXFKH[DPSOH

2 2+
+&1 VWHS
+& & &+
$ +& & &+
DT 1D&1
FROG
&+1+

H[FHVV&+&2&l

%

  L  'UDZWKHVWUXFWXUHVRIFRPSRXQGV$DQG%>@
   
  LL  6WDWHWKHUHDJHQWDQGFRQGLWLRQIRUVWHS>@
   
   >7RWDO@
   




























trendyline








<-&-&3UHOLPLQDU\([DPLQDWLRQ3DSHU

1796 trendyline


 7UDQVSRUWSURWHLQVVHUYHWKHIXQFWLRQRIPRYLQJRWKHUPDWHULDOVZLWKLQDQRUJDQLVP7KH\DUH
YLWDO WR WKH JURZWK DQG OLIH RI DOO OLYLQJ WKLQJV +DHPRJORELQ LV DQ LURQFRQWDLQLQJ R[\JHQ
WUDQVSRUWSURWHLQIRXQGLQWKHUHGEORRGFHOOVRIDOOYHUWHEUDWHV

 D  6WDWHWKHW\SHRIERQGLQJRULQWHUDFWLRQVLQYROYHGLQWKHprimarysecondarytertiaryDQG
quaternaryVWUXFWXUHVRISURWHLQVOLNHKDHPRJORELQ>@
   
 E  7KHIROORZLQJDPLQRDFLGVDUHVRPHRIWKHDPLQRDFLGVSUHVHQWLQWKHSULPDU\VHTXHQFH
RIKDHPRJORELQ


)RUPXODRIVLGHFKDLQ
 $PLQR$FLGV ,VRHOHFWULF3RLQW
5LQ5&+ 1+ &2+ 

 /\VLQH  &+ 1+ 
 *OXWDPLFDFLG &+&+&22+ 
$VSDUWLFDFLG &+&22+ 
  
$Q HOHFWURSKRUHVLV H[SHULPHQW LV UDQ RQ D VROXWLRQ FRQWDLQLQJ WKH DERYH WKUHH DPLQR
DFLGV DW S+  7KH UHODWLYH SRVLWLRQV RI WKH DPLQR DFLGV DUH VKRZQ LQ WKH GLDJUDP
EHORZ








 $ &

$QRGH % 2ULJLQDOSRVLWLRQ


RIVDPSOH
 DQRGH FDWKRGH

6XJJHVWWKHLGHQWLWLHVRIDPLQRDFLGV$%DQG&  >@ 
   
 
 
 
 
 
 

trendyline
 
 
 
 
 
 F  +DHPRJORELQ LV DEOH WR H[KLELW SHUR[LGDVHOLNH DFWLYLW\ WR FDWDO\VH WKH R[LGDWLRQ RI

<-&-&3UHOLPLQDU\([DPLQDWLRQ3DSHU

1797 trendyline


FRORXUOHVVSKHQROSKWKDOHLQLQWRLWVEULJKWSLQNFRQMXJDWHEDVH7KLVUHDFWLRQLVRIWHQXVHG
WRWHVWIRUWKHSUHVHQFHRIEORRGDWDFULPHVFHQH

3KHQROSKWKDOHLQ LV D weak acid ZKLFK GLVVRFLDWHV LQ ZDWHU DFFRUGLQJ WR WKH IROORZLQJ
HTXDWLRQ

+2 2+ +2 2

+2 +2
2
2

2 2

3KHQROSKWKDOHLQ   &RQMXJDWHEDVHRISKHQROSKWKDOHLQ
FRORXUOHVV      SLQN

7KHKDRISKHQROSKWKDOHLQLV[PROGP

'LVWLQFW FRORXU FKDQJH IRU SKHQROSKWKDOHLQ RFFXUV ZKHQ KDOI RI WKH LQGLFDWRU LV
SKHQROSKWKDOHLQDQGWKHRWKHUKDOILVLWVFRQMXJDWHEDVH

  L  &DOFXODWHWKHS+DWZKLFKSKHQROSKWKDOHLQVKRZVGLVWLQFWFRORXUFKDQJH>@
   
  LL  7KHFKDQJHRIFRORXUIRUSKHQROSKWKDOHLQRFFXUVRYHUDOLPLWHGUDQJHRIS+DQGIDOOV
ZLWKLQ RIWKHSKDYDOXH

,QDQH[SHULPHQWDVROXWLRQRIDZHDNDFLGK\GUR[\ODPPRQLXPFKORULGH1+2+&l
ZDVWLWUDWHGDJDLQVWDVROXWLRQRIVRGLXPK\GUR[LGHDVVKRZQE\WKHHTXDWLRQEHORZ

1+2+&l1D2+1+2+1D&l+2

:LWK WKH DLG RI DQ HTXDWLRQ DQG \RXU DQVZHU LQ L  H[SODLQ LI SKHQROSKWKDOHLQ LV D
VXLWDEOHLQGLFDWRUIRUWKLVWLWUDWLRQ>@
   










trendyline





 G  7KH UHDFWLRQ NLQHWLFV
NL WL RI
I R[\JHQ ELQGLQJ
EL GL WR
W KDHPRJORELQ
K O EL LQL KXPDQ
K UHG EORRG FHOO

<-&-&3UHOLPLQDU\([DPLQDWLRQ3DSHU

1798 trendyline


VXVSHQVLRQ LV VWXGLHG 7KLV LV GRQH E\ PHDVXULQJ WKH UDWHV RI XSWDNH RI R[\JHQ E\
KDHPRJORELQDWGLIIHUHQWSDUWLDOSUHVVXUHVRIR[\JHQ

7KHJUDSKRILQLWLDOUDWHDJDLQVWSDUWLDOSUHVVXUHRIR[\JHQZDVSORWWHGEHORZ

*UDSKRI,QLWLDO5DWH[ 1PVYVpO2 / 1P



,QLWLDOUDWH[1P V








      
pO2 / 1P



  L  8VLQJWKHJUDSKDERYHVWDWHWKHRUGHURIUHDFWLRQZLWKUHVSHFWWRR[\JHQ>@
   
  LL  7KHFRQFHQWUDWLRQRIKDHPRJORELQZDVKDOYHGDQGDQHZVHULHVRIH[SHULPHQWVZDV
FRQGXFWHGDWWKHVDPHWHPSHUDWXUH:KHQDVLPLODUJUDSKZDVSORWWHGWKHJUDGLHQWDW
HDFKSRLQWZDVKDOIWKDWREWDLQHGIURPWKHDERYHJUDSK

'HGXFHWKHRUGHURIUHDFWLRQZLWKUHVSHFWWRKDHPRJORELQ"([SODLQ\RXUDQVZHU>@
   
  LLL 8VLQJ\RXUDQVZHUVWR L DQG LL FRQVWUXFWWKHUDWHHTXDWLRQIRUWKHUHDFWLRQEHWZHHQ
KDHPRJORELQDQGR[\JHQ>@
   
  LY 2[\JHQ GLVVROYHV LQ EORRG LQ WKH OXQJV DQG LWV FRQFHQWUDWLRQ XQGHU FRPPRQ
FRQGLWLRQV LV NHSW DW  [  PRO GP E\ EUHDWKLQJ 7KH FRQFHQWUDWLRQ RI
KDHPRJORELQLQWKHEORRGRIOXQJFDSLOODULHVLVDOVRFRQVWDQWDQGLVHTXDOWR[
PROGP

&DOFXODWHWKHUDWHFRQVWDQWLQFOXGLQJWKHXQLWJLYHQWKDWWKHUDWHRIR[\KDHPRJORELQ
IRUPDWLRQLV;PROGPVDW&>@
   
   
   
   
   

trendyline
   
   
   
   
   
 H  $QRWKHU W\SH RI SURWHLQ WKDW WUDQVSRUWV R[\JHQ WKURXJKRXW WKH ERGLHV RI LQYHUWHEUDWHV LV

<-&-&3UHOLPLQDU\([DPLQDWLRQ3DSHU

1799 trendyline


haemocyanin ZKLFK FRQWDLQV WZR FRSSHU DWRPV WKDW UHYHUVLEO\ ELQG D VLQJOH R[\JHQ
PROHFXOH

  L  Haemocyanin H[KLELWVFDWDO\WLFDFWLYLW\WRFDWDO\VHWKHK\GUR[\ODWLRQRIPRQRSKHQROV
WRGLSKHQROV

8VLQJDQDSSURSULDWHVNHWFKRIWKH0D[ZHOO%ROW]PDQQGLVWULEXWLRQFXUYHH[SODLQKRZ
WKHSUHVHQFHRIhaemocyaninLQFUHDVHVWKHUDWHRIK\GUR[\ODWLRQRIPRQRSKHQROV>@
   
  LL  3DUWLDO denaturation DFWXDOO\ LPSURYHV haemocyaninV FDWDO\WLF DELOLW\ E\ SURYLGLQJ
JUHDWHUDFFHVVWRWKHDFWLYHVLWH
 
1 :KDWLVPHDQWE\WKHWHUPdenaturation"
2 6XJJHVW KRZ D ZHDN DFLG PLJKW LQWHUDFW ZLWK haemocyanin WR EULQJ DERXW
GHQDWXUDWLRQ
   >@
   
 I  Cereulide LV DQRWKHU H[DPSOH RI D WUDQVSRUW SURWHLQ $V D QDWXUDO GRGHFDGHSVLSHSWLGH
LRQRSKRUH cereulide DFWV DV SRWDVVLXP WUDQVSRUWHUV ZKHUH SRWDVVLXP . SOD\V D YLWDO
UROHLQEDFWHULDOSK\VLRORJ\


Cereulide

  L  $SDUW IURP WKH SHSWLGH JURXS QDPH DQRWKHU IXQFWLRQDO JURXS WKDW LV SUHVHQW LQ WKH
PROHFXOH">@
   
  :KHQ WUHDWHG ZLWK D SHSWLGDVH HQ]\PH WKH SHSWLGH ERQGV LQ WKH ULQJ DUH K\GURO\VHG
2QO\WZRGLIIHUHQWSURGXFWVDUHIRUPHG

  LL  2XW RI WKH SURGXFWV IRUPHG WZR RI WKH DPLQR DFLG UHVLGXHV DUH SUHVHQW PRUH WKDQ
RQFHLQWKHSURWHLQ'UDZWKHVWUXFWXUDOIRUPXODRIWKHVHWZRDPLQRDFLGUHVLGXHV>@

trendyline

   >7RWDO@
   


 D  0DQJDQHVHLVDWUDQVLWLRQHOHPHQWWKDWFDQH[LVWLQVHYHUDOR[LGDWLRQVWDWHV

<-&-&3UHOLPLQDU\([DPLQDWLRQ3DSHU

1800 trendyline



0DQJDQHVHVSHFLHV 0Q 0Q2 0Q2
&RORXU LQDTXHRXVPHGLXP  SDOHSLQN JUHHQ SXUSOH


   
L  :ULWHWKHIXOOHOHFWURQLFFRQILJXUDWLRQVRI0QLQ0QDQG0Q2DQGH[SODLQZK\0Q
FDQH[KLELWYDULDEOHR[LGDWLRQVWDWHV>@

  LL  ([SODLQZK\WUDQVLWLRQHOHPHQWVIRUPFRPSRXQGVWKDWDUHFRORXUHG>@

 E  $VWXGHQWZDQWHGWRLQYHVWLJDWHWKHUDWHRIUHDFWLRQEHWZHHQ0Q2DQG&2LRQVLQDQ
DFLGLFPHGLXP7KHUHDFWLRQLVVKRZQLQWKHHTXDWLRQEHORZ

0Q2&2+0Q&2+2

7KH UDWH RI UHDFWLRQ ZDV IROORZHG E\ PL[LQJ WKH IROORZLQJ WZR VROXWLRQV DQG PHDVXULQJ
WKHFRQFHQWUDWLRQRIUHPDLQLQJ0Q2LRQVDWIL[HGWLPHLQWHUYDOV

   
6ROXWLRQ$  FP  RI  PRO GP  HWKDQHGLRLF DFLG  FP  RI  PRO GP 
VXOIXULFDFLGFPRIPROGPFRSSHU ,, VXOIDWHDQGFP
RIZDWHU

 
6ROXWLRQ% FP RIPROGP SRWDVVLXPPDQJDQDWH 9,, 

7KHIROORZLQJUHVXOWVZHUHREWDLQHG

7LPHV >0Q2@PROGP
 
 
 
 
 
 
 

  L  %\ PHDQV RI D JUDSKLFDO PHWKRG GHWHUPLQH WKH UDWH HTXDWLRQ XQGHU WKH FRQGLWLRQV
ZKHUHE\WKHUHDFWLRQZDVFRQGXFWHG>@

  LL  7KHUDWHRIFRQVXPSWLRQRI0Q2 DT DWDSDUWLFXODUWLPHLVPROGPV
8VLQJWKHHTXDWLRQIRUWKHUHDFWLRQEHWZHHQ0Q2DQG&2FDOFXODWHWKHUDWHRI

trendyline
PSWLRQRIHWKDQHGLRDWHLRQVDWWKDWSDUWLFXODU
PSWLRQRIHWKDQHGLRDWHLRQVDWWKDWSDUWLFXODUWLPH">@
FRQVXPSWLRQRIHWKDQHGLRDWHLRQVDWWKDWSDUWLFXODUWLPH">@

  LVWKHUROHRIFRSSHU ,, VXOIDWHLQWKHUHDFWLRQ">@
VWKHUROHRIFRSSHU ,, VXOIDWHLQWKHUHDFWLRQ">
LLL  :KDWLVWKHUROHRIFRSSHU ,, VXOIDWHLQWKHUHDFWLRQ">@
   

 F  3RWDVVLXPPDQJDQDWH 9,, LVFRPPRQO\XVHGDVDQR[LGLVLQJDJHQWLQRUJDQLFFKHPLVWU\

<-&-&3UHOLPLQDU\([DPLQDWLRQ3DSHU

1801 trendyline



$FHWRODFWLFDFLG&+2LVDSUHFXUVRULQWKHELRV\QWKHVLVRIWKHEUDQFKHGFKDLQDPLQR
DFLGVYDOLQHDQGOHXFLQH$FHWRODFWLFDFLGFDQEHV\QWKHVLVHGIURPFKORUREXWHQHE\
WKHIROORZLQJUHDFWLRQV

&O 2+
VWHS  &O VWHS  4 VWHS 
&1
&+2
&O &O 1D2+ DT +&1
KHDW 2+
3

DFLGLILHG .0Q2 VWHS 


KHDW

2 2

2+
2+

$FHWRODFWLF DFLG 

  L  6WDWHWKHUHDJHQWDQGFRQGLWLRQVXVHGLQVWHS DQGRXWOLQHWKHPHFKDQLVPIRUWKLV
UHDFWLRQ,Q\RXUDQVZHU\RXVKRXOGLQFOXGHFXUO\DUURZVVKRZLQJWKHPRYHPHQWRI
HOHFWURQVDQGDQ\UHOHYDQWFKDUJHVDQGGLSROHV>@

  LL  &RPSRXQG3REWDLQHGIURPVWHSGRHVQRWURWDWHWKHSODQHRISRODULVHGOLJKWGHVSLWH
LWFRQWDLQLQJDFKLUDOFDUERQ8VHWKHPHFKDQLVPZKLFK\RXKDYHSURSRVHGLQ L WR
H[SODLQWKLVREVHUYDWLRQ>@

  LLL  6XJJHVWWKHVWUXFWXUHRIFRPSRXQG4>@

  LY  'UDZWKHRUJDQLFSURGXFWVIRUPHGZKHQDFHWRODFWLFDFLGLVZDUPHGZLWKLRGLQHLQWKH
SUHVHQFHRIDTXHRXVSRWDVVLXPK\GUR[LGH>@

   >7RWDO@













trendyline





 D  WL K K E LG O W L KL O G E N
1HZDSSOLFDWLRQVVXFKDVK\EULGHOHFWULFYHKLFOHVDQGSRZHUEDFNXSUHTXLUHUHFKDUJHDEOH

<-&-&3UHOLPLQDU\([DPLQDWLRQ3DSHU

1802 trendyline


EDWWHULHVWKDWFRPELQHKLJKHQHUJ\GHQVLW\ZLWKKLJKFKDUJHDQGGLVFKDUJHUDWHFDSDELOLW\
2QHVXFKFXUUHQWEDWWHU\HOHFWURGHPDWHULDOLQXVHGLV/L&R2

7KHIROORZLQJFHOOGLDJUDPQRWDWLRQLVDVLPSOLILHGLOOXVWUDWLRQRIWKHHOHFWURFKHPLFDOFHOO

/L_/L__/L&R2_/L&R2_3W

,QWKLVFHOO/LLVR[LGLVHGWR/LZKLOH&R2LVUHGXFHGWR/L&R26WDWHWKHR[LGDWLRQVWDWH
RI&RLQ/L&R2DQGKHQFHZULWHWKHHOHFWURQLFFRQILJXUDWLRQRI&RLQ/L&R2>@
  
 E  ,Q UHFHQW \HDUV RUJDQLF UDGLFDO SRO\PHUV KDYH EHHQ GHYHORSHG DV DOWHUQDWLYHV WR
LQRUJDQLF PDWHULDOV EHFDXVH RI WKHLU OLJKW ZHLJKW IOH[LELOLW\ DQG WKHLU HQYLURQPHQWDOO\
IULHQGO\ IHDWXUHV  :LWK WKH LQFUHDVH LQ GHPDQG IRU WKLQQHU PRELOH SKRQHV DQG WDEOHW
FRPSXWHUV WKHUH LV D VXUJH WR LQYHQW ILOPOLNH RUJDQLFGHULYHG HOHFWURGHDFWLYH EDWWHU\
PDWHULDOVWRSRZHUVXFKGHYLFHV

2QHVXFKPDWHULDOLV7(03212 UDGLFDOZKLFKLVFDSDEOHRIUHSHDWHGVWDEOHR[LGDWLRQ
UHGXFWLRQ EHWZHHQ 1 2 UDGLFDOV DQG 1 2 FDWLRQV ,WV YROWDJH ZKHQ SDLUHG ZLWK D /L/L
KDOIFHOOLVDEOHWRUHDFK9ZKLFKLVFRPSDUDEOHWRDFRPPRQOLWKLXPLRQEDWWHU\

7KHIROORZLQJLVDVLPSOLILHGLOOXVWUDWLRQRIWKHHOHFWURFKHPLFDOFHOOVHWXS



  L  7KH IROORZLQJ KDOIHTXDWLRQ UHSUHVHQWV WKH UHDFWLRQ ZKLFK RFFXUV DW WKH RUJDQLF
GHULYHGHOHFWURGH

trendyline


:LWK WKH
KH XVH RI WKH
WKH Data Booklet
Booklett DQG WKH LQIRUPDWLRQ DERYH
DE YH ZULWH WKH RYHUDOO
Q DERYH
FHG
HG HTXDWLRQ IRU WKH HOHFWURFKHPLFDO UHDFWLRQ DQG
EDODQFHG QG FDOFXODWH
FDOFX WKH VWDQGDUG
RGHSRWHQWLDOIRU
GH SRWHQWLDO IR 7(03212 UDGLFDOKDOIFHOO>@
HOHFWURGHSRWHQWLDOIRU7(03212UDGLFDOKDOIFHOO>@
7(032 12UDGLFDOKDOIFHOO>@
  I WK GL O O W E G O W G LW
LL  ,QRUGHUIRUWKHUDGLFDOSRO\PHUWREHXVHGDVDQHOHFWURGHLWPXVWQRWEHVROXEOHLQ

<-&-&3UHOLPLQDU\([DPLQDWLRQ3DSHU

1803 trendyline


WKH HOHFWURO\WH 7R LPSURYH WKH XVDELOLW\ RI 7(03212 UDGLFDO WKH PDWHULDO FDQ EH
FRQYHUWHGWR370$ZKLFKKDVWKHIROORZLQJVWUXFWXUH


370$

   ([SODLQKRZWKHVWUXFWXUHRI370$ZLOOKHOSWRUHGXFHLWVVROXELOLW\LQSRODUVROYHQWV
OLNHZDWHU>@
   
  LLL  :DWHU FDQQRW EH XVHG DV WKH VROYHQW IRU /L _ /L KDOI FHOO :ULWH DQ HTXDWLRQ WR
LOOXVWUDWHWKHUHDFWLRQZKHQDQDTXHRXVPHGLXPLVXVHGDWWKH/L_/LKDOIFHOOZKLFK
KLQGHUVWKHZRUNLQJRIWKHEDWWHU\>@
   
  LY  7KH V\QWKHVLV RI 370$ UHTXLUHG WKH PRQRVXEVWLWXWLRQ RI WKH 7(03212 UDGLFDO
ZLWKFKORULQHDVVKRZQLQWKHUHDFWLRQEHORZ


2 1  &l 2 1 &O
 +&l

7(03212UDGLFDO

2XWOLQHWKHPHFKDQLVPIRUWKLVUHDFWLRQ>@
   
  Y  6XJJHVWWKHV\QWKHVLVURXWHIRUWKHIRUPDWLRQRI370$IURP7(03212UDGLFDO>@
   
 F  :KHQ WKH RUJDQLF UDGLFDO EDWWHU\ LV UHFKDUJHG OLWKLXP LRQ LV EHLQJ FRQYHUWHG EDFN WR
OLWKLXP PHWDO *LYHQ WKDW WKH FKDUJLQJ SURFHVV LV  HIILFLHQW KRZ PDQ\ KRXUV GRHV LW
WDNHWRUHVWRUHJRIOLWKLXPPHWDOXVLQJDFXUUHQWRI$">@
  
   >7RWDO@
  

(QGRI3DSHU
QGRI3DSHU

<-&-&3UHOLPLQDU\([DPLQDWLRQ3DSHU

1804 trendyline


$QVZHUDQ\IRXUTXHVWLRQV
 
 3KRVSKRUXVZDVILUVWGLVFRYHUHGE\WKH*HUPDQDOFKHPLVW+HQQLJ%UDQGWVH[SHULPHQWZLWK
KXPDQ ZDVWH XULQH 3KRVSKRUXV RFFXUV LQ GLIIHUHQW DOORSWURSLF IRUPV ZLWK WKH WZR FRPPRQ
RQHVEHLQJZKLWHSKRVSKRUXVDQGUHGSKRVSKRUXV

 D  'HSHQGLQJ LI R[\JHQ LV LQ H[FHVV SKRVSKRUXV UHDFWV ZLWK R[\JHQ WR IRUP WZR GLIIHUHQW
R[LGHVSKRVSKRUXV ,,, R[LGHDQGSKRVSKRUXV 9 R[LGH

  L  :ULWH WKH HTXDWLRQV IRU WKH UHDFWLRQV RI SKRVSKRUXV ,,,  R[LGH DQG SKRVSKRUXV 9 
R[LGHZLWKZDWHUVHSDUDWHO\LQFOXGLQJVWDWHV\PEROV>@

   32 V +2 O +32 DT 
32 V +2 O +32 DT 


  LL  'HVFULEH WKH REVHUYDWLRQ ZKHQ WKH UHVXOWLQJ VROXWLRQV IURP L  DUH DGGHG WRVROLG
DOXPLQLXPR[LGH>@

   $OXPLQLXPR[LGHGLVVROYHVLQWKHDFLGLFVROXWLRQWRIRUPDFRORXUOHVVVROXWLRQ
           

 E  1LWURJHQ ZKLFK LV LQ WKH VDPH JURXS DV SKRVSKRUXV IRUPV DQ LPSRUWDQW
PROHFXOHDPPRQLD1+1+LVXVHGWRPDNHIHUWLOLVHUVDVZHOODVH[SORVLYHV

  L  :KHQ DTXHRXV DPPRQLD LV JUDGXDOO\ DGGHG WR DQ DTXHRXV VROXWLRQ FRQWDLQLQJ
&X DT  VHYHUDO REVHUYDWLRQV DUH VHHQ 'HVFULEH DQG H[SODLQ WKHVH REVHUYDWLRQV
DQGZULWHHTXDWLRQVZKHUHDSSURSULDWH>@
   
  
1+ DT +2 O 1+ DT 2+ DT 

&X DT 2+ DT &X 2+  V   



1+ DT  XQGHUJRHV K\GURO\VLV LQ ZDWHU WR IRUP 2+ :KHQ D VPDOO DPRXQW RI
1+LVDGGHGDEOXHSUHFLSLWDWHRI&X 2+ LVIRUPHG 


>&X +2 @ DT 1+ DT >&X 1+  +2 @ DT +2 O 

$GGLWLRQRIH[FHVVDPPRQLDFDXVHVWKHIRUPDWLRQRIWKHPRUHVWDEOHGHHSEOXH
>&X 1+  +2 @FRPSOH[LRQWKURXJKDOLJDQGH[FKDQJHUHDFWLRQ

7KHUHDFWLRQORZHUVWKHFRQFHQWUDWLRQRI&X DT FDXVLQJ&X 2+ WRGLVVROYH
 

trendyline
ne

EULXPSRVLWLRQ  VKLIWVWRWKHOHIWDQGWKHLRQLFSURGXFW
EULXPSRVLWLRQ  VKLIWVWRWKHOHIWDQGWKHLRQLFSURGXFW VS
(TXLOLEULXPSRVLWLRQ  VKLIWVWRWKHOHIWDQGWKHLRQLFSURGXFWK




<-&-&3UHOLPLQDU\([DPLQDWLRQ3DSHU

1805 trendyline


  LL  :KHQ EODFN FRSSHU ,,  R[LGH LV VWLUUHG ZLWK OLTXLG DPPRQLD LW GLVVROYHV WR JLYH D
FRORXUHG VROXWLRQ 'XULQJ WKLV UHDFWLRQ WKH R[LGH LRQ LV DFWLQJ DV D %URQVWHG/RZU\
EDVH6XJJHVWDQHTXDWLRQIRUWKLVUHDFWLRQDQGVXJJHVWWKHFRORXURIWKHVROXWLRQ
           >@
   
   2Q DGGLWLRQ RI 1+ O  D GDUN EOXH VROXWLRQ LV REVHUYHG DV D FRPSOH[
>&X 1+ @LVIRUPHG

&X21+>&X 1+ @2+1+

 F  $ JURXS RI UHVHDUFKHUVKDYH UHFHQWO\ GHYHORSHG DQ HIILFLHQW ZD\ RI SURGXFLQJ K\GURJHQ
IURPXULQHDQLWURJHQFRQWDLQLQJFRPSRXQGDVVKRZQLQWKHGLDJUDPEHORZ






1    +
DQRGH DONDOLQHXUHDHOHFWURO\WLF FDWKRGH
 FHOO
XUHD 

8UHD&2 1+ LQDTXHRXV.2+ZKLFKDFWVDVWKHHOHFWURO\WHLVR[LGL]HGDWWKHDQRGH
WRIRUPQLWURJHQDQGFDUERQGLR[LGH$WWKHFDWKRGHZDWHULVUHGXFHGWRK\GURJHQJDV

7KHRYHUDOOHTXDWLRQIRUWKHXUHDHOHFWURO\WHFHOOLV

&2 1+  DT +2 O 1 J + J &2 J 

  L  :ULWH WKH KDOI HTXDWLRQ ZLWK VWDWH V\PEROV RFFXUULQJ DW HDFK HOHFWURGH LQ WKLV XUHD
HOHFWURO\WLFFHOO>@
   
   $QRGH&2 1+  DT 2+ DT 1 J +2 O &2 J H

&DWKRGH+2 O H+ J 2+ DT 

  LL  %\ FRQVLGHULQJ ZK\ &2 LV QRW OLEHUDWHG DW WKH DQRGH GHGXFH WKH E\SURGXFW
SURGXFHGDWWKHDQRGH>@
   
   .&2 RU.+&2 LVSURGXFHGDVDFLGLF&2UHDFWVZLWKDONDOLQH.2+

 G  1LWURJHQ LV DOVR FRPPRQO\ IRXQG LQ RUJDQLF FRPSRXQGV ' &+21 LV D QHXWUDO

trendyline
FRPSRXQG ZLWK D EHQ]HQH ULQJ 8SRQ UHDFWLRQ ZLWK KR KRW VRGLXP K\GUR[LGH
K\GUR ( DQG ) DUH
IRUPHG 2Q DFLGLILFDWLRQ
DFLGLILFDWLRQ (
( IRUPV &+2ZKLFK JLYHV HIIHUYHVFH
HIIHUYHVFHQFH ZLWK VRGLXP
FDUERQDWH

2QH PROH RI RI )
) &+122 UHDFWV
UHDFWV ZLWK
ZLWK RQH
RQH PROH
PROH RI
RI DTXHRXV
DTXHRXV VXOI
VXOIXULF DFLG DW URRP
WHPSHUDWXUH2QHPROHRI)LVDOVRDEOHWRGHFRORXULVHWKUHHPROHVRIDTXHRXVEURPLQH

<-&-&3UHOLPLQDU\([DPLQDWLRQ3DSHU

1806 trendyline


DQGJLYHVDZKLWHSSW*

:KHQDFLGLILHGRUDQJHSRWDVVLXPGLFKURPDWH 9, LVDGGHGWR)FRPSRXQG+&+12LV
IRUPHGDQGWKHVROXWLRQWXUQVJUHHQ$\HOORZSSWLVDOVRIRUPHGZKHQ)DQG+LVUHDFWHG
ZLWKDONDOLQHDTXHRXVLRGLQHVHSDUDWHO\

6XJJHVW WKH VWUXFWXUHV IRU FRPSRXQGV'()* DQG + DQG H[SODLQ WKH REVHUYDWLRQV
GHVFULEHG DERYH  <RX GR QRW QHHG WR FRQVLGHU WKH SRVLWLRQV RI WKH VXEVWLWXHQWV RQ WKH
EHQ]HQHULQJ>@
   
  6WDWHPHQW 7\SHRIUHDFWLRQ 'HGXFWLRQ
'UHDFWVZLWKKRWVRGLXP 'FRXOGEHHVWHURU
K\GURO\VLV
K\GUR[LGHWRIRUP(DQG) DPLGH
2QDFLGLILFDWLRQ(IRUPV
 ( LV DFLG VDOW IRUPLQJ
&+2ZKLFKJLYHV
QHXWUDOLVDWLRQ &+&22+
HIIHUYHVFHQFHZLWKVRGLXP
FDUERQDWH

)LVEDVLFDVLWUHDFWV
ZLWK DT +62 ) FRXOG
2QHPROHRI)&+12
EHSKHQ\ODPLQH
UHDFWVZLWKRQHPROHRI
QHXWUDOLVDWLRQ 
VXOIXULFDFLGDWURRP
 6LQFH )  +62   
WHPSHUDWXUH
)KDV1+

)FRQWDLQVULQJ
DFWLYDWLQJHOHFWURQ
GRQDWLQJ1+JURXSV
DWWDFKHGWREHQ]HQH
2QHPROHRI)LVDEOHWR
ULQJ
GHFRORXULVHWKUHHPROHVRI HOHFWURSKLOLF

DTXHRXVEURPLQHDQGJLYH VXEVWLWXWLRQ
7KLVVKRZVWKDW
DZKLWHSSW*
SRVLWLRQVLQWKH
EHQ]HQHDUH
VXEVWLWXWHG


$FLGLILHGRUDQJHSRWDVVLXP )FRQWDLQVD
GLFKURPDWH 9, LVDGGHGWR VHFRQGDU\DOFRKRO
)FRPSRXQG+&+12LV R[LGDWLRQ JURXSZKLFKLV
IRUPHGDQGWKHVROXWLRQ R[LGLVHGWRDNHWRQH
WXUQVJUHHQ LQ+

$\HOORZSSWLVDOVRIRUPHG + KDV  &+&2 DQG )
ZKHQ)DQG+LVUHDFWHG KDV&+&+ 2+ 
R[LGDWLRQ
ZLWKDONDOLQHDTXHRXV 
LRGLQHVHSDUDWHO\ <HOORZSSWLV&+,


trendyline






<-&-&3UHOLPLQDU\([DPLQDWLRQ3DSHU

1807 trendyline


(&+&221D


1+

1+

+2 &

)  &+ 


1+

1+

2 &

+  &+ 


1+

%U 1+

+2 & %U

*  &+ %U   



1+&2&+

1+&2&+

+2 &







'


 


trendyline&+ 

>7RWDO@

<-&-&3UHOLPLQDU\([DPLQDWLRQ3DSHU

1808 trendyline


 D  :LWK WKH DLG RI DQ HTXDWLRQ GHILQH WKH WHUP standard enthalpy changeof formation RI
FDOFLXPFDUERQDWH&D&2>@
  
  6WDQGDUGHQWKDOS\RIIRUPDWLRQRI&D&2LVWKHHQHUJ\UHOHDVHGRUUHTXLUHGZKHQ
PROHRI&D&2LVIRUPHGIURPLWVFRQVWLWXHQWHOHPHQWVXQGHUVWDQGDUGFRQGLWLRQVRI
.DQGDWP
&D V & V 2 J &D&2 V 

 E  7KH HQWKDOS\ FKDQJH RI WKH UHDFWLRQ + IRU WKH UHDFWLRQ VKRZQ EHORZ LV GHWHUPLQHG WR
EHN-PRO

&D V +&l DT &D&l DT + J   + N-PRO

,Q D VLPLODU H[SHULPHQW VROLG FDOFLXP FDUERQDWH UHDFWHG ZLWK DQ H[FHVV RI K\GURFKORULF
DFLGDQGWKHHQWKDOS\FKDQJHRIUHDFWLRQZDVIRXQGWREHN-SHUPROHRIFDOFLXP
FDUERQDWH

&D&2 V +&l DT &D&l DT &2 J +2 l + N-PRO

7KHYDOXHVRIHQWKDOS\FKDQJHRIIRUPDWLRQRIZDWHUDQGHQWKDOS\FKDQJHRIFRPEXVWLRQ
RIFDUERQDUHSURYLGHG
+IR +2  N-PRO
+FR &  N-PRO

8VLQJ WKH LQIRUPDWLRQ JLYHQ FDOFXODWH WKH VWDQGDUG HQWKDOS\ FKDQJH RI IRUPDWLRQ RI
FDOFLXPFDUERQDWH>@
   
  &D V +&l DT &D&l DT + J   N-PRO

&D&2 V +&l DT &D&l DT &2 J +2 O   N-PRO

 UHYHUVH  
&D V &2 J +2 O &D&2 V + J 
+U  N-PRO  

 >+IR &D&2 @>+IR &2  +IR +2 @
 +IR &D&2 >    @
+IR &D&2  N-PRO  

25

trendyline
<-&-&3UHOLPLQDU\([DPLQDWLRQ3DSHU

1809 trendyline
 




&\ \FOH

++IR &D&2    

N-P
 PRO   

 F  0D DJQHVLXP FDUERQDWH
F G
GHFRPSRVH QJ DFFRUGLQ
V RQ KHDWLQ QJ WR WKH HT TXDWLRQ VKR RZQ EHORZ
7KKH YDOXHV RI WKH VWDQGDUG HQWK KDOS\ FKDQJH DQG VWD DQGDUG HQWWURS\ FKDQJH RI WKH
GHHFRPSRVLWLRRQDUHSURYLG GHG
0J&2 V 0J2 VV &2 J J 

++  NN-PRO6

6 -PRO.

  L  :K\GRH HVWKHHQWURS S\LQFUHDVH
HZKHQPDJ JQHVLXPFDUUERQDWHGHFFRPSRVHV">>@
   
   7KH HQWURS\ LQFUH HDVHV GXH WR LQFUHDV VH LQ WKH QXPEHU
Q RII JDVHRXV SDUWLFOHV
UHVXOWLQJ
J LQ PRUH GLVRUGHUOLQ QHVV LQ WKHH V\VWHP DQG
D PRUH Z ZD\V RI GLVWULEXWLQJ
WKHHQHUUJ\DUUDQJLLQJWKHSDUUWLFOHV

  LL  &DOFXODWH H WKH VWDQG GDUG *LEEV IUHH HQHUJJ\ FKDQJH * IRU WWKH GHFRPS SRVLWLRQ RI
PDJQHVLXXPFDUERQD DWH+HQFHFRPPHQW RQWKHVSRQWDQHLW\RI WKHGHFRPSRVLWLRQRI
PDJQHVLXXPFDUERQD DWHDWURRPWHPSHUDWXUUH>@
   
   * +76

 
 [
[ 

 
 N-PRRO

6LQFH *  LV SRRVLWLYH WKH
H GHFRPS
SRVLWLRQ LV RQWDQHRXV DW URRP
V QRW VSR
WHPSHUDWXUH







<-&-&
&3UHOLPLQDU\([DPLQDWLRQ3
3DSHU

1810 trendyline


  LLL  &RPSDUH WKH HDVH RI WKHUPDO GHFRPSRVLWLRQ RI PDJQHVLXP FDUERQDWH ZLWK EDULXP
FDUERQDWH   >@
   
   0J&2GHFRPSRVHVDWDORZHUWHPSHUDWXUH RUPRUHHDVLO\ WKDQ%D&2
7KLV LV EHFDXVH 0J LRQ EHLQJ VPDOOHU DQG KDV KLJKHU FKDUJH GHQVLW\ DQG
SRODULVLQJSRZHUWKDQ%D

0JLVDEOHWRSRODULVHWKHODUJHHOHFWURQFORXGRIWKHFDUERQDWHLRQWRDODUJHU
H[WHQW DQG ZHDNHQLQJ &2 ERQG PRUH UHVXOWLQJ LQ ORZHU WKHUPDO VWDELOLW\ LQ
0J&2

 G  'HVFULEH WKH UHDFWLRQ RI 0J DQG &D ZLWK ZDWHU JLYLQJ DSSURSULDWH HTXDWLRQV ZLWK VWDWH
V\PEROV>@
  
  0J UHDFWV ZLWK VORZO\ ZLWK KRW ZDWHU HIIHUYHVFHQFH REVHUYHG DQG D ZKLWH SSW LV
IRUPHG

0J V +2 O 0J2 V + J 

&D UHDFWV UDSLGO\ ZLWK FROG ZDWHU HIIHUYHVFHQFH REVHUYHG DQG DFRORXUOHVV
VROXWLRQLVIRUPHG 

&D V +2 O &D 2+  DT + J  
  
 H  0J UHDFWV ZLWK KDORJHQRDONDQHV 5; LQ GU\ HWKHU IRUPLQJ *ULJQDUG UHDJHQWV ZKLFK DUH
LPSRUWDQWLQWHUPHGLDWHVXVHGWRSURGXFHPDQ\RUJDQLFFRPSRXQGV

GU\HWKHU
5;0J50J;
   
   *ULJQDUGUHDJHQW

50J;LVFDOOHGDQRUJDQRPHWDOOLFFRPSRXQGZKHUH5LVDQDON\OJURXSDQG;LVFKORULQH
EURPLQH RU LRGLQH5 LQ 50J; EHKDYHV OLNH DQ DQLRQ 5 DQG LV VWURQJO\ EDVLF DQG
QXFOHRSKLOLF

+ &

2QUHDFWLRQZLWKFDUERQ\OFRPSRXQGV50J;IRUPVYDULRXVFODVVHVRIDOFRKROV
+




G
U
\

H
W
K
H
U

S
U
L
P
D
U
\

D
O
F
R
K
R
O


5
0
J
;
&
2

2
+
P
H
W
K
D
Q
D
O





+
2
+

5


+ &
+
D
O
G
H
K
\
G
H

R
W
K
H
U





G
U

\
H
W
K
H
U

V
H
F
R
Q
G
D
U
\

D
O
F
R
K
R
O


5
0
J
;
&
2

2
+
W
K
D
Q

P
H
W
K
D
Q
D
O





+
2

trendyline
e dyl n
5

5


5 & 5

555




UU U
UUUG
GG
 \
\\
H
HH
W
K
U
UUH
HH

W
H WW
HH
U
W
UU
L
D
U
\

D
O
F
R
K
R
O


5 55
000
J
;
;;
&
&&
2
22

55

2
++
+
NN
N
HH
WW H
W
RR R
QQ Q
HH H





++
+
2
22
5

5



<-&-&3UHOLPLQDU\([DPLQDWLRQ3DSHU

1811 trendyline
 

  L  'UDZWKHHVWUXFWXUHR
RIWKHRUJDQ
QLFSURGXFWV
VIRUPHGZK
KHQWKHIROOR
RZLQJFRPS
SRXQGVDUH
UHDFWHG

2 &


&
+



0
J
%
U

D
Q
G


>@
   
  


   
  LL  *LYHWKHGLVSOD\HGIIRUPXODRIWWKHEURPRDOONDQHZKLFK KFDQEHXVHGWRIRUPD
D*ULJQDUG
UHDJHQWWWKDWZRXOGUUHDFWZLWKE RO>@
EXWDQDOWRJLLYHPHWK\\OKH[DQ
   
  


   
  LLLL  :KHQ50J%ULVUHD DFWHGZLWKHHWKDQR\OEUURPLGHPPROHVRI500J%UDUHUH HTXLUHGSHU
PROH RI HWKDQR\O EURPLGH
E DQGG WKH SURGX
XFW IRUPHG LV &+&52+([SODLQ Q ZK\ RQH
PROH RI HWKDQR\O EURPLGH
E UH
HDFWV ZLWK WZR PROHV RI 50J%UU ZKLOH RQHH PROH RI
DOGHK\GHHRUNHWRQHUHDFWVZLWKRQHPROHR RI50J%U> @
   
   2QH PR ROH RI50J J%U UHDFWV V ZLWK RQH PROH RIR &+&2% %U YLD QXFOHRSKLOLF
VXEVWLWX
XWLRQPHFKD DQLVPWRIRRUPRQHPROHRIWKHN NHWRQH&+&25
7KH RQHH PROH RI &+
& &25 IR RUPHG WKHQ Q UHDFWV ZLWK
Z DQRWKHU PROH RI 50J%U WR
IRUP&++&52+

&+&2% %U50J% %U&+&2 25

&+&25 550J%U&+&2 2+5

([SODQD
DWLRQE\ZULLWLQJWKHHTXDWLRQVDUHDFFHSWWDEOH







trendyline







<-&-&
&3UHOLPLQDU\([DPLQDWLRQ3
3DSHU

1812 trendyline


 I  .HWRQHVDQGDOGHK\GHVUHDFWZLWK+&1YLDQXFOHRSKLOLFDGGLWLRQPHFKDQLVPVLPLODUWRWKH
UHDFWLRQZLWK*ULJQDUGUHDJHQWV7KHUHDFWLRQVFKHPHEHORZLVRQHVXFKH[DPSOH

2 2+
+&1 VWHS
+& & &+
$ +& & &+
DT 1D&1
FROG
&+1+

H[FHVV&+&2&l

%

  L  'UDZWKHVWUXFWXUHVRIFRPSRXQGV$DQG%>@
   

$ % 


  LL  6WDWHWKHUHDJHQWDQGFRQGLWLRQIRUVWHS>@
   
+3WRU
+1LKHDWRU
/L$O+LQGU\HWKHU

   >7RWDO@
   


















<-&-&3UHOLPLQDU\([DPLQDWLRQ3DSHU

1813 trendyline


 7UDQVSRUWSURWHLQVVHUYHWKHIXQFWLRQRIPRYLQJRWKHUPDWHULDOVZLWKLQDQRUJDQLVP7KH\DUH
YLWDO WR WKH JURZWK DQG OLIH RI DOO OLYLQJ WKLQJV +DHPRJORELQ LV DQ LURQFRQWDLQLQJ R[\JHQ
WUDQVSRUWSURWHLQIRXQGLQWKHUHGEORRGFHOOVRIDOOYHUWHEUDWHV

 D  6WDWHWKHW\SHRIERQGLQJRULQWHUDFWLRQVLQYROYHGLQWKHprimarysecondarytertiaryDQG
quaternaryVWUXFWXUHVRISURWHLQVOLNHKDHPRJORELQ>@
  
  3ULPDU\6WUXFWXUHFRYDOHQWERQGV RUDPLGHRUSHSWLGHOLQNDJH 
 
6HFRQGDU\6WUXFWXUHK\GURJHQERQGLQJ

7HUWLDU\ 6WUXFWXUH  9DQ GHU :DDOV IRUFHV LRQLF ERQG K\GURJHQ ERQGLQJ DQG
GLVXOILGHOLQNDJH

4XDWHUQDU\ 6WUXFWXUH  9DQ GHU :DDOV IRUFHV LRQLF ERQG K\GURJHQ ERQGLQJ DQG
GLVXOILGHOLQNDJH        
  
 E  7KHIROORZLQJDPLQRDFLGVDUHVRPHRIWKHDPLQRDFLGVSUHVHQWLQWKHSULPDU\VHTXHQFH
RIKDHPRJORELQ


)RUPXODRIVLGHFKDLQ
 $PLQR$FLGV ,VRHOHFWULF3RLQW
5LQ5&+ 1+ &2+ 

 /\VLQH  &+ 1+ 
 *OXWDPLFDFLG &+&+&22+ 
$VSDUWLFDFLG &+&22+ 
  
$Q HOHFWURSKRUHVLV H[SHULPHQW LV UDQ RQ D VROXWLRQ FRQWDLQLQJ WKH DERYH WKUHH DPLQR
DFLGV DW S+  7KH UHODWLYH SRVLWLRQV RI WKH DPLQR DFLGV DUH VKRZQ LQ WKH GLDJUDP
EHORZ








 $ &

$QRGH % 2ULJLQDOSRVLWLRQ


RIVDPSOH

tre e
 DQRGH
RG FDWKRGH

6XJJHVWWKHLGHQWLWLHVRIDPLQRDFLGV$%DQG&
HLGHQWLWLHVRIDPLQRDFLGV$%
HLGHQWLWLHVRIDPLQRDFLGV %DQG
QG&&  >@ 
  $$VSDUWLFDFLG%*OXWDPLFDFLG&/\VLQH
FDFLG%*OXWDPLFDFLG&/\VL
FDFLG%*OXWDPLFDFLG&/\VLQH

 F  +DHPRJORELQ LV DEOH WR H[KLELW SHUR[LGDVHOLNH DFWLYLW\ WR FDWDO\VH WKH R[LGDWLRQ RI

<-&-&3UHOLPLQDU\([DPLQDWLRQ3DSHU

1814 trendyline


FRORXUOHVVSKHQROSKWKDOHLQLQWRLWVEULJKWSLQNFRQMXJDWHEDVH7KLVUHDFWLRQLVRIWHQXVHG
WRWHVWIRUWKHSUHVHQFHRIEORRGDWDFULPHVFHQH

3KHQROSKWKDOHLQ LV D weak acid ZKLFK GLVVRFLDWHV LQ ZDWHU DFFRUGLQJ WR WKH IROORZLQJ
HTXDWLRQ

+2 2+ +2 2

+2 +2
2
2

2 2

3KHQROSKWKDOHLQ   &RQMXJDWHEDVHRISKHQROSKWKDOHLQ
FRORXUOHVV      SLQN

7KHKDRISKHQROSKWKDOHLQLV[PROGP

'LVWLQFW FRORXU FKDQJH IRU SKHQROSKWKDOHLQ RFFXUV ZKHQ KDOI RI WKH LQGLFDWRU LV
SKHQROSKWKDOHLQDQGWKHRWKHUKDOILVLWVFRQMXJDWHEDVH

  L  &DOFXODWHWKHS+DWZKLFKSKHQROSKWKDOHLQVKRZVGLVWLQFWFRORXUFKDQJH>@
   
   S+ OJ [  
   
  LL  7KHFKDQJHRIFRORXUIRUSKHQROSKWKDOHLQRFFXUVRYHUDOLPLWHGUDQJHRIS+DQGIDOOV
ZLWKLQ RIWKHSKDYDOXH

,QDQH[SHULPHQWDVROXWLRQRIDZHDNDFLGK\GUR[\ODPPRQLXPFKORULGH1+2+&l
ZDVWLWUDWHGDJDLQVWDVROXWLRQRIVRGLXPK\GUR[LGHDVVKRZQE\WKHHTXDWLRQEHORZ

1+2+&l1D2+1+2+1D&l+2

:LWK WKH DLG RI DQ HTXDWLRQ DQG \RXU DQVZHU LQ L  H[SODLQ LI SKHQROSKWKDOHLQ LV D
VXLWDEOHLQGLFDWRUIRUWKLVWLWUDWLRQ>@
   
   1+2++21+2+2+

1+2+XQGHUJRHVK\GURO\VLVWRSURGXFH2+KHQFHLWVS+!



3KHQROSKWKDOHLQ LV VXLWDEOH DV WKHS+ WUDQVLWLRQ UDQJH RI WKH LQGLFDWRU  

trendyline
HV W W H DS G S F D JH R H W H H HTX D H FH SR
 OLHVZLWKLQWKHUDSLGS+FKDQJHRYHUWKHHTXLYDOHQFHSRLQW S+! . 
   
   
   
 G  7KH UHDFWLRQRQ NLQHWLFV RI R[\JHQ ELQGLQJ WR
WR KDHPRJORELQ LQ KXPDQ
KXPD UHG EORRG FHOO
VXVSHQVLRQ LV VWXGLHG 7KLV LV GRQH E\ PHDVXULQJ WKH UDWHV RI XSWDNH RI R[\JHQ E\

<-&-&3UHOLPLQDU\([DPLQDWLRQ3DSHU

1815 trendyline


KDHPRJORELQDWGLIIHUHQWSDUWLDOSUHVVXUHVRIR[\JHQ

7KHJUDSKRILQLWLDOUDWHDJDLQVWSDUWLDOSUHVVXUHRIR[\JHQZDVSORWWHGEHORZ

*UDSKRI,QLWLDO5DWH[ 1PVYVpO2 / 1P



,QLWLDOUDWH[1P V








      
pO2 / 1P



  L  8VLQJWKHJUDSKDERYHVWDWHWKHRUGHURIUHDFWLRQZLWKUHVSHFWWRR[\JHQ>@
   
   2UGHURIUHDFWLRQZLWKUHVSHFWWR2LV
   
  LL  7KHFRQFHQWUDWLRQRIKDHPRJORELQZDVKDOYHGDQGDQHZVHULHVRIH[SHULPHQWVZDV
FRQGXFWHGDWWKHVDPHWHPSHUDWXUH:KHQDVLPLODUJUDSKZDVSORWWHGWKHJUDGLHQWDW
HDFKSRLQWZDVKDOIWKDWREWDLQHGIURPWKHDERYHJUDSK

'HGXFHWKHRUGHURIUHDFWLRQZLWKUHVSHFWWRKDHPRJORELQ"([SODLQ\RXUDQVZHU>@
   
   2UGHURIUHDFWLRQZLWKUHVSHFWWRKDHPRJORELQLV

:KHQFRQFHQWUDWLRQRIKDHPRJORELQZDVKDOYHGFKDQJHLQWKHJUDGLHQWRIWKH
JUDSKLVKDOYHGDQGUDWHRIUHDFWLRQZDVDOVRKDOYHG
   
  LLL 8VLQJ\RXUDQVZHUVWR L DQG LL FRQVWUXFWWKHUDWHHTXDWLRQIRUWKHUHDFWLRQEHWZHHQ
KDHPRJORELQDQGR[\JHQ>@
   
   5DWH k>KDHPRJORELQ@>2@
   
  LY 2[\JHQ GLVVROYHV LQ EORRG LQ WKH OXQJV DQG LWV FRQFHQWUDWLRQ XQGHU FRPPRQ
FRQGLWLRQV LV NHSW DW  [  PRO GP E\ EUHDWKLQJ 7KH FRQFHQWUDWLRQ RI
KDHPRJORELQLQWKHEORRGRIOXQJFDSLOODULHVLVDOVRFRQVWDQWDQGLVHTXDOWR[
PROGP

&DOFXODWHWKHUDWHFRQVWDQWLQFOXGLQJWKHXQLWJLYHQWKDWWKHUDWHRIR[\KDHPRJORELQ
DWHWKHUDWHFRQVWDQWLQFOXGLQJWKHXQLWJLYHQWKDWW UDWHR
DWHWKHUDWHFRQVWDQWLQFOXGLQJWKHXQLWJLYHQWKDWWKH UDWH

 
 

IRUPDWLRQLV;
RQLV; PROGP
PROGP V DW&>@
   
   5DWH k>KDHPRJORELQ@>2
k
k>KDHPRJORELQ@>2 @
@

<-&-&3UHOLPLQDU\([DPLQDWLRQ3DSHU

1816 trendyline


[ k [  [ 

k [GPPROV
   
 H  $QRWKHU W\SH RI SURWHLQ WKDW WUDQVSRUWV R[\JHQ WKURXJKRXW WKH ERGLHV RI LQYHUWHEUDWHV LV
haemocyanin ZKLFK FRQWDLQV WZR FRSSHU DWRPV WKDW UHYHUVLEO\ ELQG D VLQJOH R[\JHQ
PROHFXOH

  L  Haemocyanin H[KLELWVFDWDO\WLFDFWLYLW\WRFDWDO\VHWKHK\GUR[\ODWLRQRIPRQRSKHQROV
WRGLSKHQROV

8VLQJDQDSSURSULDWHVNHWFKRIWKH0D[ZHOO%ROW]PDQQGLVWULEXWLRQFXUYHH[SODLQKRZ
WKHSUHVHQFHRIhaemocyaninLQFUHDVHVWKHUDWHRIK\GUR[\ODWLRQRIPRQRSKHQROV>@
   
  

>@

(DLVORZHUHGGXHWRDQDOWHUQDWLYHUHDFWLRQSDWKZD\7KHSURSRUWLRQRIUHDFWDQWV
ZLWKHQHUJ\JUHDWHUWKDQRUHTXDOVWKHDFWLYDWLRQHQHUJ\LQFUHDVHV


)UHTXHQF\RIHIIHFWLYHFROOLVLRQVLQFUHDVHVUDWHFRQVWDQWYDOXHLQFUHDVHVDQG
KHQFHUDWHRIUHDFWLRQLQFUHDVHV
   
  LL  3DUWLDO denaturation DFWXDOO\ LPSURYHV haemocyaninV FDWDO\WLF DELOLW\ E\ SURYLGLQJ
JUHDWHUDFFHVVWRWKHDFWLYHVLWH
 
1 :KDWLVPHDQWE\WKHWHUPdenaturation"
2 6XJJHVW KRZ D ZHDN DFLG PLJKW LQWHUDFW ZLWK haemocyanin WR EULQJ DERXW
GHQDWXUDWLRQ
   >@

trendyline
trend
dyline
yline
   
   QDWXUDWLRQ UHIHUV WR WKH EUHDNLQJ RI ZHDN ERQGV KROGLQ
1 'HQDWXUDWLRQ KROGLQJ WKH VHFRQGDU\
WHUWLDU\DQGTXDWHUQDU\VWUXFWXUHEXWQRWWKHFRYDOHQWERQGVZLWKLQWKHSULPDU\
\DQGTXDWHUQDU\VWUXFWXUH
DQGTXDWHUQDU\VWUXFWX EXWQRWWKHFRYDOHQWERQGVZ
EXWQRWWKHFRYDOHQWERQGVZ
VWUXFWXUH
XUH
UH


<-&-&3UHOLPLQDU\([DPLQDWLRQ3DSHU

1817 trendyline
 

2 3URWRQDWHV WKH DQLRQLF JURXS J 22 IRXQG LQ VLGH F
&2 FKDLQ IRU H[DPSOH
JOXWDPLF DFLG >@ DQG
D GLVUXSSWV LRQLF ER
RQG EHWZH HHQ WKH FKD DUJHG 5 JUURXSV  RI
&22DQ QG1+
   
 I  Ceereulide LV DQRWKHU H[[DPSOH RI DD WUDQVSRUW SURWHLQ $VV D QDWXUDOO GRGHFDGH
HSVLSHSWLGH
LRQ
QRSKRUH ce KHUH SRWDVVVLXP . SOD\V D YLWDO
ereulide DFWWV DV SRWDVVVLXP WUDQVSRUWHUV ZK
UROHLQEDFWHULDOSK\VLRORJ\


Cereu
ulide

  L  $SDUW IURP P WKH SHSWLGH JURXS QDPH DQRWWKHU IXQFWLR RQDO JURXS WWKDW LV SUHV
VHQW LQ WKH
PROHFXOH" ">@
   
   (VWHU
   
  :
:KHQ WUHDWHG G ZLWK D SH
HSWLGDVH HQ
Q]\PH WKH SHSWLGH ER RQGV LQ WKH ULQJ DUH K\GURO\VHG
K
2Q QO\WZRGLIIH HUHQWSURGXFFWVDUHIRUPPHG

  LL  2XW RI WKH
H SURGXFWV IRUPHG WZZR RI WKH DP
PLQR DFLG UHVLGXHV
U DUH
H SUHVHQW PRUH
P WKDQ
RQFHLQWK KHSURWHLQ''UDZWKHVWWUXFWXUDOIRUPXODRIWKHVHWZRDPLQ QRDFLGUHVLLGXHV>@
   
  

 


  


trendyline
   
   >7RWDO@
   
   
 D  0DDQJDQHVHLV
H LVVVDWUDQVLWLR
D WUDQVLWLR
RQHOHPHQWW
RQ HOHPHQW WWKDWFDQH[L
KDW FDQ H[
H[LLLVWLQVHYHUD
VW LQ VHYHUD
DOR[LGDWLRQ
D O R[LGDWLRQ VWD
VWDWHV


<-&-&
&3UHOLPLQDU\([DPLQDWLRQ3
3DSHU

1818 trendyline


0DQJDQHVHVSHFLHV 0Q 0Q2 0Q2


&RORXU LQDTXHRXVPHGLXP  SDOHSLQN JUHHQ SXUSOH


   
L  :ULWHWKHIXOOHOHFWURQLFFRQILJXUDWLRQVRI0QLQ0QDQG0Q2DQGH[SODLQZK\0Q
FDQH[KLELWYDULDEOHR[LGDWLRQVWDWHV>@

   
   7KHHOHFWURQLFFRQILJXUDWLRQRI0QLQ0QLVVVSVSG
7KHHOHFWURQLFFRQILJXUDWLRQRI0QLQ0Q2LVVVSVS


7KH HQHUJ\ GLIIHUHQFH EHWZHHQ WKH G DQG V VXEVKHOOV LV UHODWLYHO\ VPDOO
+HQFHDYDULDEOHQXPEHURIHOHFWURQVFDQEHUHPRYHG RUDYDULDEOHQXPEHURI
HOHFWURQVFDQEHXVHGIRUERQGIRUPDWLRQ 
   
  LL  ([SODLQZK\WUDQVLWLRQHOHPHQWVIRUPFRPSRXQGVWKDWDUHFRORXUHG>@

   
   ,QWKHSUHVHQFHRIOLJDQGVWKHGHJHQHUDWHGRUELWDOVDUHVSOLWLQWRWZRGLIIHUHQW
HQHUJ\OHYHOV6LQFHWKHGVXEVKHOOLVSDUWLDOO\ILOOHGWKHHOHFWURQVLQWKHORZHU
HQHUJ\GRUELWDOVFDQDEVRUEDSKRWRQRIHQHUJ\IURPWKHYLVLEOHOLJKWUDQJHWR
SURPRWH WR WKH KLJKHU HQHUJ\ G RUELWDOV 7KH REVHUYHG FRORXU LV
FRPSOHPHQWDU\WRWKHZDYHOHQJWKDEVRUEHG
   
 E  $VWXGHQWZDQWHGWRLQYHVWLJDWHWKHUDWHRIUHDFWLRQEHWZHHQ0Q2DQG&2LRQVLQDQ
DFLGLFPHGLXP7KHUHDFWLRQLVVKRZQLQWKHHTXDWLRQEHORZ

0Q2&2+0Q&2+2

7KH UDWH RI UHDFWLRQ ZDV IROORZHG E\ PL[LQJ WKH IROORZLQJ WZR VROXWLRQV DQG PHDVXULQJ
WKHFRQFHQWUDWLRQRIUHPDLQLQJ0Q2LRQVDWIL[HGWLPHLQWHUYDOV

   
6ROXWLRQ$  FP  RI  PRO GP  HWKDQHGLRLF DFLG  FP  RI  PRO GP 
VXOIXULFDFLGFPRIPROGPFRSSHU ,, VXOIDWHDQGFP
RIZDWHU

 
6ROXWLRQ% FP RIPROGP SRWDVVLXPPDQJDQDWH 9,, 






trendyline




7KHIROORZLQJUHVXOWVZHUHREWDLQHG
QJUHVXOWVZHUHREWDLQHG
OW EW L


<-&-&3UHOLPLQDU\([DPLQDWLRQ3DSHU

1819 trendyline


7LPHV >0Q2@PROGP
 
 
 
 
 
 


   
  L  %\ PHDQV RI D JUDSKLFDO PHWKRG GHWHUPLQH WKH UDWH HTXDWLRQ XQGHU WKH FRQGLWLRQV
ZKHUHE\WKHUHDFWLRQZDVFRQGXFWHG>@

   3ORWDJUDSKRI>0Q2@DJDLQVWWLPH

'HWHUPLQHKDOIOLIHIURPWKHJUDSKDQGVKRZWKDWWKH\KDYHWKHVDPHYDOXHRU
DSSUR[LPDWHO\WKHVDPH WV 5HIHUWRJUDSKDWWKHODVWSDJH

5DWH N>0Q2@
   
  LL  7KHUDWHRIFRQVXPSWLRQRI0Q2 DT DWDSDUWLFXODUWLPHLVPROGP V
8VLQJ WKH HTXDWLRQ IRU WKH UHDFWLRQ EHWZHHQ 0Q2DQG &2 FDOFXODWH WKH UDWH RI
FRQVXPSWLRQRIHWKDQHGLRDWHLRQVDWWKDWSDUWLFXODUWLPH">@

   5DWHRIFRQVXPSWLRQRI&2 [5DWHRIFRQVXPSWLRQRI0Q2
 [PROGPV
   
  LLL :KDWLVWKHUROHRIFRSSHU ,, VXOIDWHLQWKHUHDFWLRQ">@
   
   +RPRJHQRXVFDWDO\VW

   
   
   
   
   
   
   
   
   
   
   
   
   
   
 PDQJDQDWH 9,, LVFRPPRQO\XVHGDVDQR[LGLVLQJDJHQW
HGD LQR
F  3RWDVVLXPPDQJDQDWH 9,, LVFRPPRQO\XVHGDVDQR[LGLVLQJDJHQWLQRUJDQLFFKHPLVWU\


<-&-&3UHOLPLQDU\([DPLQDWLRQ3DSHU

1820 trendyline


$FHWRODFWLFDFLG&+2LVDSUHFXUVRULQWKHELRV\QWKHVLVRIWKHEUDQFKHGFKDLQDPLQR
DFLGVYDOLQHDQGOHXFLQH$FHWRODFWLFDFLGFDQEHV\QWKHVLVHGIURPFKORUREXWHQHE\
WKHIROORZLQJUHDFWLRQV




  L  6WDWH WKH UHDJHQW DQG FRQGLWLRQV XVHG LQ VWHS  DQG RXWOLQH WKH PHFKDQLVP IRU WKLV
UHDFWLRQ ,Q \RXU DQVZHU \RX VKRXOG LQFOXGH FXUO\ DUURZV VKRZLQJ WKH PRYHPHQW RI
HOHFWURQVDQGDQ\UHOHYDQWFKDUJHVDQGGLSROHV>@

   &l J RU&lLQ&&lURRPWHPSHUDWXUH

(OHFWURSKLOLFDGGLWLRQ

VORZ
  &l &l  &l

&l
&l
&l

&l
&l

 &l &l
&l
&l

1RWH
 7KHW\SHRIPHFKDQLVPUHDFWLRQPXVWEHVWDWHG
 7KH PHFKDQLVP DUURZV PXVW EH VKRZQ LQ WKH HTXDWLRQ WR LOOXVWUDWH WKH
PRYHPHQWRIHOHFWURQV
 /RQHSDLURQWKHFKORULGHLRQPXVWEHFOHDUO\VKRZQ
QHSDLURQWKHFKORULGHLRQPXVWEHFOHDUO\VKRZQ
HSDLURQWKHFKORULGHLRQPXVWEHFOHDUO\
 6ORZ
RZ
Z VWHS DQG WKH SDUWLDO FKDUJHV RQ FKORULQH PXVW EH LQGLFDWHG LQ WKH
FKDQLVP
FKDQLVP
PHFKDQLVP

  RXQG 3 REWDLQHG IURP VWHS  GRHVV QRW
Q URWDWH WKH SODQH RI SROD
LL  &RPSRXQG3REWDLQHGIURPVWHSGRHVQRWURWDWHWKHSODQHRISRODULVHGOLJKWGHVSLWH
LWFRQWDLQLQJDFKLUDOFDUERQ8VHWKHPHFKDQLVPZKLFK\RXKDYHSURSRVHGLQ L WR
<-&-&3UHOLPLQDU\([DPLQDWLRQ3DSHU

1821 trendyline


H[SODLQWKLVREVHUYDWLRQ>@

   7KH JHRPHWU\ DERXW WKH SRVLWLYHO\ FKDUJHG FDUERQ DWRP RI WKH FDUERFDWLRQ LV
WULJRQDOSODQDU RUSODQDU 




 &l
  &l
 &l



+HQFH WKH QXFOHRSKLOH KDV HTXDO FKDQFHV RI DWWDFNLQJ WKH SRVLWLYHO\ FKDUJHG
FDUERQDWRPIURPERWKDERYHDQGEHORZWKHSODQHWRSURGXFHHTXDODPRXQWRI
WKHHQDQWLRPHUVUHVXOWLQJDQUDFHPLFPL[WXUH

7KH RSWLFDO DFWLYLW\ RI WKH WZR HQDQWLRPHUV FDQFHO RXW HDFK RWKHU FRPSOHWHO\
7KXVLWLVQRWRSWLFDOO\DFWLYH
   
  LLL 6XJJHVWWKHVWUXFWXUHRIFRPSRXQG4>@

   2+

2
 
   
  LY 'UDZWKHRUJDQLFSURGXFWVIRUPHGZKHQDFHWRODFWLFDFLGLVZDUPHGZLWKLRGLQHLQWKH
SUHVHQFHRIDTXHRXVSRWDVVLXPK\GUR[LGH>@

   2 &+

   
. 2 & & & 2.

2+ 2   &+, 
   
   >7RWDO@







trendyline




 D  1HZDSSOLFDWLRQVVXFKDVK\EULGHOHFWULFYHKLFOHVDQGSRZHUEDFNXSUHTXLUHUHFKDUJHDEOH
FDWLRQVVXFKDVK\EULGHOHFWULFYHKLFOHVDQGSRZHUEDFNXSUH
KLFO
EDWWHULHVWKDWFRPELQHKLJKHQHUJ\GHQVLW\ZLWKKLJKFKDUJHDQGGLVFKDUJHUDWHFDSDELOLW\

<-&-&3UHOLPLQDU\([DPLQDWLRQ3DSHU

1822 trendyline


2QHVXFKFXUUHQWEDWWHU\HOHFWURGHPDWHULDOLQXVHGLV/L&R2

7KHIROORZLQJFHOOGLDJUDPQRWDWLRQLVDVLPSOLILHGLOOXVWUDWLRQRIWKHHOHFWURFKHPLFDOFHOO

/L_/L__/L&R2_/L&R2_3W

,QWKLVFHOO/LLVR[LGLVHGWR/LZKLOH&R2LVUHGXFHGWR/L&R26WDWHWKHR[LGDWLRQVWDWH
RI&RLQ/L&R2DQGKHQFHZULWHWKHHOHFWURQLFFRQILJXUDWLRQRI&RLQ/L&R2>@
  
  2[LGDWLRQVWDWHRI&RLV 
VVSVSG 
   
 E  ,Q UHFHQW \HDUV RUJDQLF UDGLFDO SRO\PHUV KDYH EHHQ GHYHORSHG DV DOWHUQDWLYHV WR
LQRUJDQLF PDWHULDOV EHFDXVH RI WKHLU OLJKW ZHLJKW IOH[LELOLW\ DQG WKHLU HQYLURQPHQWDOO\
IULHQGO\ IHDWXUHV  :LWK WKH LQFUHDVH LQ GHPDQG IRU WKLQQHU PRELOH SKRQHV DQG WDEOHW
FRPSXWHUV WKHUH LV D VXUJH WR LQYHQW ILOPOLNH RUJDQLFGHULYHG HOHFWURGHDFWLYH EDWWHU\
PDWHULDOVWRSRZHUVXFKGHYLFHV

2QHVXFKPDWHULDOLV7(03212 UDGLFDOZKLFKLVFDSDEOHRIUHSHDWHGVWDEOHR[LGDWLRQ
UHGXFWLRQ EHWZHHQ 1 2 UDGLFDOV DQG 1 2 FDWLRQV ,WV YROWDJH ZKHQ SDLUHG ZLWK D /L/L
KDOIFHOOLVDEOHWRUHDFK9ZKLFKLVFRPSDUDEOHWRDFRPPRQOLWKLXPLRQEDWWHU\

7KHIROORZLQJLVDVLPSOLILHGLOOXVWUDWLRQRIWKHHOHFWURFKHPLFDOFHOOVHWXS



  L  7KH IROORZLQJ KDOIHTXDWLRQ UHSUHVHQWV WKH UHDFWLRQ ZKLFK RFFXUV DW WKH RUJDQLF
GHULYHGHOHFWURGH

trendyline
HXVHRIWKH
I
HTXDWLRQ IRU
I WK D
:LWKWKHXVHRIWKHData
Data Booklet
WK HOHFWURFKHPLFDO
WKH O W K L O UHDFWLRQWL
W

kl tDQGWKHLQIRUPDWLRQDERYHZULWHW
DQGWKHLQIRUPDWLRQDERYHZULWH
G FDOFXODWH
DQG
LW
O O W WKH


t BookletDQGWKHLQIRUPDWLRQDERYHZULWHWKHRYHUDOOEDODQFHG
B
WK VWDQGDUG HOHFWURGH

<-&-&3UHOLPLQDU\([DPLQDWLRQ3DSHU

1823 trendyline


SRWHQWLDOIRU7(03212UDGLFDOKDOIFHOO  >@
   
   )URP'DWD%RRNOHW

/LH/LER 9

2YHUDOOHTXDWLRQ



ERFHOO ERUHGXFWLRQERR[LGDWLRQ
 ERUHGXFWLRQ  
ERUHGXFWLRQ 9

  LL  ,QRUGHUIRUWKHUDGLFDOSRO\PHUWREHXVHGDVDQHOHFWURGHLWPXVWQRWEHVROXEOHLQ
WKH HOHFWURO\WH 7R LPSURYH WKH XVDELOLW\ RI 7(03212 UDGLFDO WKH PDWHULDO FDQ EH
FRQYHUWHGWR370$ZKLFKKDVWKHIROORZLQJVWUXFWXUH


370$

   ([SODLQ KRZ WKH VWUXFWXUH RI 370$ ZLOO KHOS WR UHGXFH LWV VROXELOLW\ LQ SRODU VROYHQWV
OLNHZDWHU>@
   
   :LWKWKHLQFUHDVHLQFDUERQVLGHFKDLQVWKHHOHFWURQFORXGVL]HLQFUHDVHVDQG
EHFRPHV PRUH SRODULVDEOH  UHVXOWLQJ LQ DQ LQFUHDVH LQ LQVWDQWDQHRXV GLSROH
LQGXFHGGLSROH LGLG LQWHUDFWLRQVLQ370$FRPSDUHGWR7(03212UDGLFDO

+HQFHWKHIRUPDWLRQRIK\GURJHQERQGLQJEHWZHHQZDWHUDQG370$ZLOOQRWEH
H[RWKHUPLFHQRXJKWRRYHUFRPHWKHVWURQJHULGLGLQWHUDFWLRQEHWZHHQ370$DV
ZHOO DV K\GURJHQ ERQGLQJ EHWZHHQ ZDWHU PROHFXOHV WR GLVVROYH LW +HQFH

trendyline
\
VROXELOLW\LVUHGXFHG



DWLYHDQVZHU
L
$OWHUQDWLYHDQVZHU


<-&-&3UHOLPLQDU\([DPLQDWLRQ3DSHU

1824 trendyline


:LWK WKH LQFUHDVH LQ FDUERQ VLGH FKDLQV WKH LQGXFHG GLSROHSHUPDQHQW GLSROH
LQWHUDFWLRQ RU LQVWDQWDQHRXV GLSROHLQGXFHG GLSROH LQWHUDFWLRQ  EHWZHHQ WKH
FDUERQVLGHFKDLQDQGZDWHUEHFRPHVWKHSUHGRPLQDQWLQWHUDFWLRQ

,QVXIILFLHQW HQHUJ\ LV UHOHDVHG WR RYHUFRPH WKH K\GURJHQ ERQGLQJ EHWZHHQ
ZDWHUPROHFXOHVDVZHOODVWKHVWURQJHULGLGLQWHUDFWLRQEHWZHHQ370$

LLL :DWHUFDQQRWEHXVHGDVWKHVROYHQWIRU/L_/LKDOIFHOO:ULWHDQHTXDWLRQWRLOOXVWUDWH
WKH UHDFWLRQ ZKHQ DQ DTXHRXV PHGLXP LV XVHG DW WKH /L_ /L KDOI FHOO ZKLFK KLQGHUV
WKHZRUNLQJRIWKHEDWWHU\>@

/LZLOOUHDFWZLWKZDWHUWRIRUP/L2+

/L+2/L2++

LY 7KHV\QWKHVLVRI370$UHTXLUHGWKHPRQRVXEVWLWXWLRQRIWKH7(03212UDGLFDOZLWK
FKORULQHDVVKRZQLQWKHUHDFWLRQEHORZ

2
 1  &l 2 1 &O
 +&l

7(03212UDGLFDO

2XWOLQHWKHPHFKDQLVPIRUWKLVUHDFWLRQ>@

 )UHHUDGLFDOVXEVWLWXWLRQ

,QLWLDWLRQ

3URSDJDWLRQ

ttrendyline
ren
nddyliline 

<-&-&3UHOLPLQDU\([DPLQDWLRQ3DSHU

1825 trendyline


7HUPLQDWLRQ $Q\ 

>@

Y 6XJJHVWWKHV\QWKHVLVURXWHIRUWKHIRUPDWLRQRI370$IURP7(03212UDGLFDO>@

F :KHQ WKH RUJDQLF UDGLFDO EDWWHU\ LV UHFKDUJHG OLWKLXP LRQ LV EHLQJ FRQYHUWHG EDFN WR
OLWKLXP PHWDO *LYHQ WKDW WKH FKDUJLQJ SURFHVV LV  HIILFLHQW KRZ PDQ\ KRXUV GRHV LW
WDNHWRUHVWRUHJRIOLWKLXPPHWDOXVLQJDFXUUHQWRI$">@

0DVVRI/LWREHUHVWRUHG [ J

1RRIPROHRI/L  PRO

2U
trendyline
[ &
4 [


&

 V K


7LPH  K

<-&-&3UHOLPLQDU\([DPLQDWLRQ3DSHU

1826 trendyline



0DVVRI/LWREHUHVWRUHG [ J







   >7RWDO@
  

trendyline
<-&-&3UHOLPLQDU\([DPLQDWLRQ3DSHU

1827 trendyline











trendyline


(QGRI3DSHU

<-&-&3UHOLPLQDU\([DPLQDWLRQ3DSHU

1828 trendyline

You might also like